CHAPTER 1 CHECK ALL THE APPLY. Choose all options that best completes the statement or answers the question. 1) Cell theory is one of the foundations of biology. What are the tenets of the cell theory? Check all that apply. A) All organisms aremade up of more than one cell. B) All cells have theability to move. C) Cells carry geneticmaterial passed to daughter cells during cellular division. D) Cells arise fromother cells through the process of cell division. E) Organisms are formed through spontaneous generation. F) All living organisms consist of cells.
2) Darwin's theory of evolution is supported by many modern pieces of evidence. Check all that apply. A) New measurements of the age of the earth B) An understanding of the mechanism of heredity C) Human population growth D) Comparative studies of animal structures E) Similarities in DNA of related species
MULTIPLE CHOICE - Choose the one alternative that best completes the statement or answers the question. 3) Science is subdivided into specific areas of study, called disciplines. These divisions are artificial but are helpful to narrow the massive scope of scientific knowledge to a manageable amount. Given the brief definitionof each listed discipline, which is likely to provide the best answer to a question about how fluid dynamics affect blood pressure in mammals? A) Biochemistry—study of chemical reactions needed for life function, usually at the cellular level B) Bioinformatics—use of technology to study and store biological data C) Biophysics—study of biological processes through physics D) Biology—study of life
Version 1
1
4) Based on hierarchical levels of biological organization, which represents the broadest level? A) Endocrine system B) Three-toed sloths C) School of piranhas D) Amazon Basin E) Jaguars, giant anteaters, macaws, capybaras
5) Experiments are carried out to test a hypothesis by changing one variable at a time and including an unchanged variable termeda/an __________. A) experimental variable B) altered variable C) control D) stable variable
6) The method of reasoning that uses construction of general principles by careful examination of many specific cases is called __________. A) deductive reasoning B) theoretical reasoning C) hypothetical reasoning D) inductive reasoning E) experimental reasoning
7) Dr. Ratard was trying to determine the cause of a mysterious epidemic affecting fish in the gulf of New Mexico. His proposal that the deaths were caused by an organism called a protist is considered a/an __________.
Version 1
2
A) experiment B) hypothesis C) conclusion D) theory E) data set
8) After Darwin concluded his voyage on the Beagle, he proposed that the process of natural selection was a mechanism for __________. A) artificial selection B) evolution C) sexual selection D) speciation E) overpopulation of finches on the Galapagos Islands
9)
A scientific theory is __________.
A) a suggested explanation that accounts for observations B) a way to organize how we think about a problem C) a concept that is supported by experimental evidence that explains the facts in an area of study D) a way to understand a complex system by reducing it to its working parts
10)
Although a computer is not alive, it shares which fundamental property of life? A) Energy utilization B) Cellular organization C) Homeostasis D) Reproduction E) Heredity
Version 1
3
11) A yellow jacket, an insect in the order Hymenoptera, stung you. Then a wasp, an insect in Hymenoptera, stung you as well. A hornet, an insect in Hymenoptera, stung you last. It is an observable pattern that all insects in this order must have stingers. The type of reasoning this represents is __________. A) inductive reasoning B) deductive reasoning C) reductionism D) comparative reasoning
12)
Darwin hypothesized that Galapagos finches _________. A) had beaks that were adapted to allow them to eat local foods B) had migrated from South America at least 14 times to account for the variation in
species C) were hybrids of South American and African finches D) looked the same but had learned different behaviors that allowed them to eat different foods
13) A suggested explanation that might be true and is subject to testing by further observations is a/an __________. A) experiment B) generality C) hypothesis D) scientific principle E) theory
14) Based on the literature, you hypothesize that students in traditional biology lectures will have the same grades as students in online biology lectures. You decide to test your hypothesis by comparing grades of students in traditional and online biology lectures over a semester. As a result of the experiment, you observe that the grades in the traditional lectures and the grades in the online lecturesare not significantly different. What do these observations allow you to do?
Version 1
4
A) Reject the hypothesis B) Accept the hypothesis without further question C) Develop a scientific theory D) Reject the nullhypotheses
15) Your microwave will notturn on, and you speculate that a circuit breaker in the house has been tripped. In scientific terminology, the steps you would take to figure this out would be described as _________. A) forming conclusions from the results of experiments B) developing an observation based on a hypothesis C) developing a hypothesis based on an observation D) testing a prediction generated from a hypothesis
16) A student poses the question: How does the presence of dissolved salt affect the freezing point of water? To answer this question, the student set up two conditions. In the first condition, the student added salt to water in a container and referred to this condition as the variable. In the second condition, the student did not add any salt to water in a second container and referred to this condition as the control. The student took both containers and attempted to freeze the water at various temperatures to assess the freezing point. Would this be a valid experiment? A) Yes, because there is more than one variable. B) Yes, because there is one variable and a control C) No, because there is not more than one variable D) No, because there is only one variable and a control
17) The proposal that one type of organism can change gradually into another type over a long period of time is known as __________.
Version 1
5
A) evolution B) natural history C) preconception D) preservation
18) Darwin's ideas on evolutionwere advanced for his time. His approach to science and natural selection were supported by which main tenant? A) Various organismsand their structures resulted from a spontaneous action. B) Species wereunchangeable over the course of time. C) The world is fixedand constant. D) Operation of natural laws produces constant change and improvement.
19) Besides Darwin, the theory of evolution by means of natural selection was also independently proposed by __________. A) Alfred Wallace B) Charles Lyell C) Thomas Malthus D) Karl Popper E) Peter Raven
20) The term that Darwin used to describethe concept that organisms with superior strength, behavior, or other attributes are more likely to survive andpass their traits to the next generation than those that are not so well endowed is called __________. A) biological diversity B) geometric progression C) natural selection D) superior beings E) survival of modifications
Version 1
6
21) A key contribution to Darwin's thinking was the concept of limits put on the geometric growth of populations by nature, originally proposed by __________. A) Charles Lyell B) Thomas Malthus C) Karl Popper D) Peter Raven E) Russel Wallace
22)
Darwin's book in which he described his views on evolution is __________. A) Principles of Geology B) On the Principle of Population C) On the Origin of Species D) Survival of the Fittest
23) Recent discoveries of microscopic fossils have extended the known history of life to about __________. A) 3.5 billionyears ago B) 2 billion years ago C) 4.5 billionyears ago D) 1 billion years ago
24) In California, a species of salamanders were geographically separated over time. The group that lived in southern California relied heavily on large gold blotches on their skin that helped to camouflage them from predators. The group that lived along the coast adopted a color pattern that mimicked a poisonous, colorful newt common to that area. Instead of being camouflaged, these salamanders advertised their colors. What type of selection process has occurred over time?
Version 1
7
A) Artificial selection B) Natural selection C) Experimental selection D) Theoretical selection
25) The same basic array of bones is modified to give rise to the wing of a bat and the fin of a porpoise. Such anatomical structures are called __________. A) analogous B) uniform C) homologous D) inherited E) evolutionary modifications
26)
Structures that have similar functions are said to be __________. A) homologous B) analogous C) inherited D) uniform E) evolved
27)
Which is the closest relative to a whale? A) A shark B) A hippopotamus C) A goldfish D) An eagle
28) Differences in domesticated animalsover relatively shortperiods of time most likely occur through __________. Version 1
8
A) natural selection B) adaptation C) evolution D) experimental selection E) artificial selection
29) If you were to design a long-term research study to determine why there are no human births in Lapland during the months of August, September, and October, you would need to also examine a comparison population of humans in which births took place every month. The primary reason for including a comparison population within the design of this experiment would be to __________. A) accumulate more facts that could be reported to other scientists B) test the effects of more than one variable at the same time C) prove that there are no births in Lapland during August, September, and October D) act as a control that would ensure that the results obtained are due to a difference in only one variable
30) Essay on the Principle of Population, written by Thomas Malthus in 1798, influenced Darwin's thoughts as he struggled to understand what mechanisms could be at work to produce evolution. Malthus proposed that populations of animals and plants, including humans, __________. A) increased arithmetically in numbers while the nutrients available only increased geometrically B) increased geometrically in numbers while the nutrients available only increased arithmetically C) decreased arithmetically in numbers while the nutrients available increased geometrically D) increased geometrically in numbers while the nutrients available increased arithmetically E) evolved from mainland to islands, thus explaining why the island flora and fauna resembled the mainland species so closely
Version 1
9
31) A student set up an experiment to test if plants give off water vapor. Fifty pea plants, growing in pots, were covered with individual glass containers and left overnight. The next morning, the inside of each lid was covered in droplets of water. The lab student concluded that plants generally give off water vapor. What critique would you make of the experimental design? A) There was nocontrol so the water could have come from other sources such as air in the jaror the soil. B) There was not alarge enough sample of pea plants used to get adequate data. C) The student did nothave a clearly stated hypothesis before beginning the experiment. D) The experiment wasnot precise, meaning it was not reproducible.
32) It is known that many trees lose their leaves in response to decreasing day length. As a result, you think thatGingko trees may also lose their leaves in response to decreasing day length. This statement is an example of __________. A) deductive reasoning B) an experiment C) a hypothesis D) a theory
33) Multiple independent experiments have demonstrated that phytochrome helps trigger seasonal change responses in plants such as changing color and loosening of leaves. Plants have the ability to respond to seasonal changes in their surroundings. This statement is an example of __________. A) deductive reasoning B) an experiment C) a hypothesis D) inductive reasoning E) a theory
34) Plants are raised under artificial lights turned off and on by an electric clock. Some are given long periods of light, others short periods. This is an example of __________.
Version 1
10
A) deductive reasoning B) an experiment C) a hypothesis D) inductive reasoning E) a theory
35) Both walnut and Gingko trees lose their leavesin the fall when day length starts decreasing. Based on these observations, one may conclude that many tree species will lose their leaves in the fall in response to decreasing day length. This statement is an example of __________. A) deductive reasoning B) an experiment C) inductive reasoning D) a theory
36) Gingko trees are known to lose their leaves at a certain time each year throughout the United States. Based on this information, Gingko trees in Chinamust behave the same way. These statements are an example of __________. A) deductive reasoning B) an experiment C) inductive reasoning D) a theory
37)
The primary difference between fungi and animals is that fungi __________. A) digest their food externally B) are photosynthetic C) are multicellular D) have nuclei
Version 1
11
38) Wings of birds and butterflies have similar functions, but different evolutionary origins. They are __________. A) homologous structures B) physiological structures C) phylogenetic structures D) analogous structures
39) An alien from another planet landed on earth. He is fascinated by cars and is determined to figure out how they work. He decides to disassemble one of them and examine each part independently. He removes one of the tires and proceeds to learn all he can about the tire. He then removes one of the headlights and proceeds to learn all he can about the headlight. What type of approach is this alien taking to learn about the car? A) Reductionism B) Deductive reasoning C) Inductive reasoning D) Emergent properties
40) While you are riding the ski lift up to the top of the mountain on a very cold day, you start to shiver involuntarily. You know that the shivering is your bodyʹs attempt to help regulate your body temperature and is an example of which type of mechanism? A) Energy utilization B) Sensitivity C) Homeostasis D) Evolutionary adaptation
41) You have been assigned to address a problem of overpopulation of species X in a nearby county. One of the members of your team suggests introducing species Y, which is a natural predator of species X, but not normally found in the area. After some discussion, you go ahead and introduce species Y. What aspects of the hierarchical organization may be affected within a period of a several years? Version 1
12
A) Population, species, community B) Population, community C) Population, species, community, biosphere D) Organism, population, species
42) You have been assigned to analyze some extraterrestrial material recently collected from Mars. After examining a sample using a microscope, you jump up excitedly and shout to your colleagues that you have confirmed the existence of life on Mars. One of your colleagues takes a look at your sample and says she sees a single-celled "body" with little internal structure. Assuming that life on Mars can be classified into similar domains and kingdoms as Earth, to which domain or kingdom does your "blob" most likely belong? A) Eukarya B) Fungi C) Protista D) Archaea E) Animalia
43) Why was the determination of the actual sequence of the human genome considered to be descriptive science? A) It involved hypothesis-driven research. B) It did not involve hypothesis-driven research. C) It involved deductive reasoning. D) It did not involve deductive reasoning.
44) You look outside and realize that your grass needs to be mowed. You pick up the container of gasoline and see that you have approximately a third of a gallon left. You hypothesize that this amount will be enough to mow your entire lawn. Unfortunately, half way through mowing your lawn you run out of gasoline. You grumble and think to yourself that the next time you mow the lawn, you hypothesize that you will need to have at least two-thirds of a gallon of gasoline available. How did the results of your lawn-mowing experience influence the validity of your new hypothesis for future gasoline needs?
Version 1
13
A) Your prediction of future gas needs is based on experimental data and therefore increases the validity of your hypothesis. B) The hypothesis was invalidated by your experimental evidence. C) Your hypothesis was supported by trial and error. One more trial added to your data set. D) Your prediction proved that your hypothesis is correct.
45)
A plant and animal would be most similar at which level of organization? A) Cell B) Organelle C) Tissue D) Organ
46)
How does peer review influence the development of scientific theories?
A) Peer review allows other scientists to know what is current in their field. B) Careful evaluation of research results by other scientists ensures that only solid and legitimate research results are published, and helps prevent faulty research or false claims from being viewed as scientific fact. C) Peer review increases competition among scientists and thus increases the quality of the published work. D) Peer review makes it extremely difficult for work to be published other than earthshattering scientific theories.
47) Amanda was studying turtles based on DNA analysis. Under the current classification scheme, which of the following turtle species are thought to be most closely related? (1) Graptemys ouachitensis (2) Trachemys scripta (3) Apalone spinifera (4) Graptemys kohni
Version 1
14
A) 1 and 3 due to inductive reasoning B) 1 and 3 due to deductive reasoning C) 2 and 3 due to inductive reasoning D) 1 and 4 due toinductive reasoning E) 2 and 3 due to deductive reasoning F) 1 and 4 due to deductive reasoning
48) Marceau is studying small single-celled organisms that lack nuclei. These organisms can be broadly classified into thedomain __________. A) Bacteria B) Protista C) Animalia D) Fungi
49) Bacterial cells are placed into a 250 mL liquid growth medium in a closed laboratory flask. According to Malthusian theory, they will reproduce exponentially and then __________. A) continue reproducing geometrically as long as there are no limitations on food supply B) continue reproducing arithmetically as long as there are no limitations on food supply C) continue reproducing geometrically until the food supply is used up, then they will cease to grow D) continue reproducing arithmetically until the food supply is used up, then they will cease to grow
50) A dental student wants to test if fluoride is an effective additive against tooth decay. The student studies tooth decay in a population of people who live in neighborhoods supplied with fluoridated water. This student would like to ask whether access to fluoridated water prevents tooth decay. What would be an effective control group to ask this question?
Version 1
15
A) Individuals with access to fluoridated water B) Individuals with access to differing amounts of fluoride in the water C) Individuals who have fluoride added to their toothpaste but not their water D) Individuals with access to water with no fluoride added
51)
Which statement is true about the development of the theory of evolution?
A) Evolution from a common ancestor was proposed 100 years before Darwin's Origin of Species. B) Darwin was thefirst to propose evolution. C) Initialexperiments performed by Darwin did not support his hypothesis. D) In Darwin's timemost people believed that different organisms evolved from a common ancestor,but they did not know how.
52) Viruses defy characterization as living organisms even though they possess nucleic acids and proteins. The reason whyviruses are not considered to be alive is because they __________. A) lack genetic material B) cannot form spontaneously C) form spontaneously D) cannot reproduce without a host cell
53) Osmometer cells in the brain sense an increase in the salt concentration of plasma. This information is sent to the hypothalamus, which notifies the pituitary gland to release the hormone, ADH. ADH causes the kidney to save water, which lowers the salt concentration of the plasma. What characteristic of life does this overall pathway represent? A) Cellularorganization B) Sensitivity C) Energyutilization D) Evolutionaryadaptation E) Homeostasis
Version 1
16
54) A chemical imbalance in the blood can cause the heart to stop pumping blood, which will have a detrimental effect on other organs. This observation can be attributed to __________. A) reductionism B) emergent properties C) equilibrium state D) evolutionary conservation
55) Dr. Edward Jenner realized that cows have a disease called cowpox, which is like a disease that infects humans called smallpox. Jenner noticed that milkmaids whose hands were infected with cowpox were not contracting smallpox. Jenner infected a child with the pus from a cowpox blister and found that the child did not contract smallpox. Which statement represents a hypothesis supported by evidence? A) The cowpox virus prevented the smallpox virus from entering the child's immune system. B) The smallpox virus was so similar to the cowpox virus that the child's immune system ignored the smallpox virus. C) The cowpox infection will prevent the child from being infected by the smallpox virus. D) The cowpox infection will have no effect on the child's immunity to the smallpox virus.
56) A beautiful wood desk you may do your homework on was once a living tree, but after being cut down its tissues died. Now, it only exhibits which of the following propertiesof life? A) Growth, development, and reproduction B) Homeostasis C) Energy utilization D) Organization
Version 1
17
SECTION BREAK. Answer all the part questions. 57) As part of your research project, you travel to an island to learn more about the habitats and relationships of spiders, centipedes, and insects. The published research suggests the following relationships: Flies—prey for spiders Spiders—prey for centipedes and predators of flies Centipedes—predators of flies and spiders You and your assistant plot out five different areas of the island and count the numbers of spiders, centipedes, and insects living in each plot. Your results show the following: Plot
Insects 1 2 3 4 5
Spiders 300 226 147 739 79
Centipedes 25 17 15 78 13
4 10 21 0 93
57.1) Because you are studying the relationships between these three animal groups, the most plausibleexplanation for the high number of spiders in plot 4 is __________. A) there are too many flies overall B) there are no centipedes to prey onthe spiders and there are abundant flies upon which to feed C) the spiders preyed onthe centipedes and ignored the flies D) the flies and spiders worked together to eliminatethe centipedes
57.2)
The plots that were staked out on the island were part of the __________.
A) applied research B) basic research C) constructed model D) experimental design
57.3) Based in the information provided, the best explanation for the low numbers of spiders and flies in plot 5 is __________. Version 1
18
A) centipedes are actively consuming flies and spiders B) there were not enough flies to support a large centipede population C) centipedes prefer spiders to flies D) there were not enough spiders to catch and consume all the flies
57.4)
Thehypothesis that closely matches the data provided is __________.
A) herbivorous insects survive best onislands where spiders and centipedes live B) herbivorous insects feed on spiders and centipedes C) herbivorous insects and spider populations are decreased by centipedes D) spiders are the top predators on all islands
58)
58.1) The common ancestor that produced the most evolutionary recent derived characters is __________.
Version 1
19
A) 12 B) 9 C) 8 D) 6
58.2) The species that have had proportionally the most time to diverge are __________. A) R and D B) F and Z C) A and Z D) F and R E) F
58.3) If two different species of fish fossils were found in two different layers of sedimentary rock, what might one infer about the specimens? A) They died at the same time. B) The two species are unrelated. C) The species in the higher layer evolved from the species in the lower layer. D) The species in lower layer died first. E) The species in the higher layer died first.
59) Phil is conducting a seed germination experiment. He places threegroups of lettuce seeds in a 34º Celsius incubator with adequate moisture. One set of seeds is placed in adark area with no light source. A second set is placed under artificial light, and a third set of seeds is placed in direct sunlight. This experiment is intended to test Phil's hypothesis that light is necessary for lettuce seed germination.
59.1)
Version 1
Based on the experimental design, which variable was the control?
20
A) Seeds in the dark B) Type of light C) Germination rate D) Temperature E) Moisture
59.2)
Based on the experimental design, which variable was the dependent variable?
A) Seeds in the dark B) Type of light C) Germination rate D) Temperature E) Moisture
60) Luke went to a pediatrician when he was 6 months old. The pediatrician consulted a graph and concluded that Luke was in the 97 th percentile for height, weight and length. The pediatrician predicted that Luke would be tall when he reached adulthood.
60.1) What type of reasoning did the pediatrician use to generate her predictionabout Luke’s future growth in height? A) Inductive reasoning B) Deductive reasoning C) Applied theory D) Reductionism
60.2) What type of logic is being used when the pediatrician uses the graph to make conclusions about Luke’s progress?
Version 1
21
A) Inductive reasoning B) Applied theory C) Reductionism D) Deductive reasoning
61) Turtle hatchling survivorship rate is low in many turtle species due to predation. Amanda researched the predatory rate on a species of turtleeggs in the Red River. The eggs were harvested from trapped turtles, and the egg's cloaca film (reproductive discharge)was either washed off or left on once gathered. Research suggests that predators use the female's cloaca scent to locate the eggs. The eggs were only handled when wearing gloves and then reburied along islands where the turtles were trapped. The nests were monitored by cameras and manually on foot, and data on nest predation was recorded.
61.1)
Based on the experimental design, what is the dependent variable?
A) Number of hatchlings B) Cloaca film on eggs C) Eggs without cloaca film D) Time eggs spent in ground
61.2)
Based on the experimental design, what is the independent variable?
A) Number of hatchlings B) No cloaca filmon eggs C) Inducing egg laying D) Time eggs spent in ground
61.3)
Version 1
Based on the experimental design, what is the control?
22
A) Hatchling survival rate B) Cloaca scent on eggs C) No cloaca scent on eggs D) Time eggs spent in ground
Version 1
23
Answer Key Test name: Chap 01_3e 1) [C, D, F] 2) [A, B, E] 3) C
Version 1
24
Clarify Question ● What is the key concept addressed by the question? ● The question is about the differences between different divisions of biological research. ● What type of thinking is required? ● This question is asking for you to apply your understanding to identify which areas of research would focus on the movement of fluids. ● What key words does the question contain and what do they mean? ● Fluid dynamics—the physics of how fluids (liquids) move Gather Content ● What do you know about different divisions of biology? What other information is related to the question? ● Bioinformatics is the study of biological data like sequences using computers. ● Biology is a very broad area of research covering all areas of the study of life. ● Biochemistry is the study of biological molecules smaller than a cell. ● Biophysics is the study of how biological molecules move using the laws of physics. Choose Answer ● Given what you now know, what information is most likely to produce the correct answer? ● Biology is a very broad heading, and not the best answer. The best answer is biophysics because the movement of fluids are well described using the laws of physics. Reflect on Process ● Did your problem-solving process lead you to the correct answer? If not, where did the process break down or lead you astray? How can Version 1
25
you revise your approach to produce a more desirable result? ● If you got the correct answer, great job! If you got an incorrect answer, where did the process break down? Did you think that fluid dynamics were part of a cell or represented by large data sets? Did you not know that the movement of fluids was an area of study in physics?
4) D 5) C 6) D 7) B 8) B 9) C 10) A 11) A
Version 1
26
Clarify Question ● What is the key concept addressed by the question? question asks about different types reasoning.
● The
● What type of thinking is required? ● You are being asked to apply definitions of different types of reasoning to a specific example. ● What key words does the question contain and what do they mean? ● Order Hymenoptera—a specific taxanomic group of insects Gather Content ● What do you know about different types of reasoning? What other information is related to the question? ● Inductive reasoning uses data to create a hypothesis. ● Deductive reasoning draws conclusions from existing observations or definitions. ● Reductionism is breaking down something into its component parts to study them individually. ● All three insects that stung you (ouch!) in this example are in the same insect order, and they stung you in sequence. Choose Answer ● Given what you now know, what information is most likely to produce the correct answer? ● Because you were making the observations yourself by being stung, this is inductive reasoning. This is not reductionist as you are not breaking down a larger concept into component parts. Reflect on Process ● Did your problem-solving process lead you to the correct answer? If not, where did the process break down or lead you astray? How can you revise your approach to produce a more desirable result? ● If Version 1
27
you got the correct answer, great job! If you got an incorrect answer, where did the process break down? Did you confuse the definitions of deductive and inductive reasoning? Did you think that a larger concept was being broken into smaller components?
12) A 13) C 14) D
Version 1
28
Clarify Question ● What is the key concept addressed by the question? question is about hypotheses.
● The
● What type of thinking is required? ● This question is asking to apply your knowledge of a hypothesis to interpret the results of an experiment. ● What key words does the question contain and what do they mean? ● Null hypothesis—the opposite of your hypothesis ● You hypothesize that students in traditional biology lectures will have the same grades as students in online biology lectures Gather Content ● What do you know about hypotheses? What other information is related to the question? ● Can you accept or reject a hypothesis?In this case the null hypothesiswould be that the scores of the two groups of students will be different. ● Do you accept a hypothesis or reject a null hypothesis? Your results support your hypothesis, so you do not reject yourhypothesis. Choose Answer ● Given what you now know, what information is most likely to produce the correct answer? ● The correct answer is to reject the null hypothesis which is the opposite ofyour hypothesis, specifically,that the scores of the two groups of students will be different. Reflect on Process ● Did your problem-solving process lead you to the correct answer? If not, where did the process break down or lead you astray? How can you revise your approach to produce a more desirable result? ● If you got the correct answer, great job! If you got an incorrect answer, Version 1
29
where did the process break down? Did you think that you could accept your hypothesis instead of rejecting the null hypothesis?
15) C 16) B
Version 1
30
Clarify Question ● What is the key concept addressed by the question? question asks about the design of an experiment.
● The
● What type of thinking is required? ● You are being asked to analyze the validity of an experimental design. ● What key words does the question contain and what do they mean? ● Variable—factor that may influence a process being studied ● Control—experiment with no change in the variable of interest Gather Content ● What do you know about the design of an experiment? What other information is related to the question? ● In an experiment, you need a control as a base line and then change a single independent variable to see if affects the dependent variable. ● Consider the entire experimental design the student took to test his question, specifically thinking about the multiple conditions the student set up. Choose Answer ● Given what you now know, what information is most likely to produce the correct answer? ● The experiment does have a single variable, the amount of salt added.It also has a control which is a sample with no salt added. ● This would be a valid experiment because it has a control and only one variable was changed. Reflect on Process ● Did your problem-solving process lead you to the correct answer? If not, where did the process break down or lead you astray? How can you revise your approach to produce a more desirable result? ● If Version 1
31
you got the correct answer, great job! If you got an incorrect answer, where did the process break down? Did you think that one variable and a control was not part of a valid experimental design?Did you understand that changing more than one variable makes it difficult to interpret the results of an experiment?
17) A 18) D 19) A 20) C 21) B 22) C 23) A 24) B
Version 1
32
Clarify Question ● What is the key concept addressed by the question? question is about different forms of selection.
● The
● What type of thinking is required? ● This question is asking you to apply the definition of different forms of selection to a specific example. ● What key words does the question contain and what do they mean? ● Geographic separation—members of those two populations could no longer interbreed which can drive the evolution of new species Gather Content ● What do you know about selection? What other information is related to the question? ● Selection works on the phenotype of an organism, with beneficial traits being more likely to be passed on to the next generation. ● Which types of selection make sense? 1.Artificial selection requires humans to be involved to make the selection. 2.Theoretical and experimental selection are part of an experiment. The selection in this example did not involve humans or any experiments. Choose Answer ● Given what you now know, what information is most likely to produce the correct answer? ● This is an example of natural selection due to geographic separation. Once the two groups of salamanders became separated they could no longer interbreed and developed different adaptations to their local environments. Reflect on Process Version 1
33
● Did your problem-solving process lead you to the correct answer? If not, where did the process break down or lead you astray? How can you revise your approach to produce a more desirable result? ● If you got the correct answer, great job! If you got an incorrect answer, where did the process break down? Did you think that natural selection would not occur with geographical separation, or that humans were involved in the speciation in some way?
25) C 26) B 27) B 28) E 29) D 30) B 31) A
Version 1
34
Clarify Question ● What is the key concept addressed by the question? question is about designing an experiment. ● What type of thinking is required? analyze the design of an experiment.
● The
● This question is asking to
● What key words does the question contain and what do they mean? ● The hypothesis is that plants give off water vapor ● 50 pots each containing plants were covered with lids ● The next day each pot contained water vapor Gather Content ● What do you know about experimental design? What other information is related to the question? ● The weakness of this experiment is that it does not contain a control group, so you cannot conclude that the water came from the plants or from the soil or other components of the experiment. ● What are other explanations are there for these results? 50 plants is a large sample size, there was a hypothesis and the experiment was carried out correctly. So you can eliminate these as answers. Choose Answer ● Given what you now know, what information is most likely to produce the correct answer? ● The best answer is that there was not an appropriate control group. Half of the pots should have been prepared with no plants in them to be sure the water vapor was not coming from the soil or pots. Reflect on Process ● Did your problem-solving process lead you to the correct answer? If not, where did the process break down or lead you astray? How can Version 1
35
you revise your approach to produce a more desirable result? ● If you got the correct answer, great job! If you got an incorrect answer, where did the process break down? Did you not realize that all of the pots contained a plant? While the hypothesis was not clearly stated, you could infer it from the first sentence of the problem. Did you think that 50 plants was too small a sample?
32) C 33) E 34) B 35) C 36) A 37) A 38) D 39) A
Version 1
36
Clarify Question ● What is the key concept addressed by the question? question is about different types reasoning.
● The
● What type of thinking is required? ● This question is asking you to apply the definitions of different types of reasoning to a specific example. ● What key words does the question contain and what do they mean? ● Y is a predator of X whichmeans that Y eats X ● Inductive reasoning—uses data to create a hypothesis ● Deductive reasoning—draws conclusions from existing observations or definitions ● Reductionism—breaking down something into its component parts to study them individually Gather Content ● What do you know about different types of reasoning? What other information is related to the question? ● The key concept here is that the alien is breaking down the car to figure out how each part works independently. ● What are other types of reasoning? Is a hypothesis being tested or data analyzed? If not then this is not an example of inductive or deductive reasoning. Choose Answer ● Given what you now know, what information is most likely to produce the correct answer? ● Without data or a hypothesis and by breaking down the object being studied a reductionist method is being used. Reflect on Process Version 1
37
● Did your problem-solving process lead you to the correct answer? If not, where did the process break down or lead you astray? How can you revise your approach to produce a more desirable result? ● If you got the correct answer, great job! If you got an incorrect answer, where did the process break down? Did you forget the definitions of deductive and inductive reasoning? Did you not realize the that taking something apart to figure out how it works is reductionism?
40) C 41) A
Version 1
38
Clarify Question ● What is the key concept addressed by the question? ● The question is about the impact of a new species on the hierarchical organization of living systems. ● What type of thinking is required? ● To apply the definition of levels of organization to determine how they are affected by addition of a new species. ● What key words does the question contain and what do they mean? ● Y is a predator of X whichmeans that Y eats X ● Population—members of a species in a local area ● Species—all members of a species ● Community—multiple species interacting with each other ● Biosphere—both living and non-living components Gather Content ● What do you know about hierarchical organization? What other information is related to the question? ● Hierarchical refers to one level building on the level below it, like a pyramid. So if one level is affected it can impact the levels above it. ● If a predator is added which levels of structure would be affected? A predator would impact the prey species at the population level. Levels above this would also be affected up to the level of community. Choose Answer ● Given what you now know, what information is most likely to produce the correct answer? ● The best answer that the population of X will be affected, which in turn affects the species and the community that it belongs to. Reflect on Process Version 1
39
● Did your problem-solving process lead you to the correct answer? If not, where did the process break down or lead you astray? How can you revise your approach to produce a more desirable result? ● If you got the correct answer, great job!If you got an incorrect answer, where did you get stuck? A decrease in the number of species X would affect the hierarchical levels above it up to the community level. Increased predation will not have an impact at the level of an individual organism or the biosphere.
42) D
Version 1
40
Clarify Question ● What is the key concept addressed by the question? ● The question asks you to determining the domain of a new organism. ● What type of thinking is required? ● You are being asked to apply the criteria of different domains to specific examples. ● What key words does the question contain and what do they mean? ● Single celled—entire organism consists of only one cell ● Internal structure—refers to organelles like the nucleus Gather Content ● What do you know about different domains? What other information is related to the question? ● Protista have single celled members, but are not a monophyletic taxanomic group (= not a domain). ● Eukarya (a domain) and fungi (members of domain Eukarya) have single celled members, but have internal structures. ● Animals (members of domain Eukarya) are all multicellular and have internal structures. ● Archaea (a domain) are like bacteria and are single celled and do not have internal structure. Choose Answer ● Given what you now know, what information is most likely to produce the correct answer? ● All members of the domain Archaea are single celled and do not have internal structure, so this is the best answer. Reflect on Process ● Did your problem-solving process lead you to the correct answer? If not, where did the process break down or lead you astray? How can you revise your approach to produce a more desirable result? ● If Version 1
41
you got the correct answer, great job! If you got an incorrect answer, where did you get stuck? While three of the domains listed have single celled organisms, only one lacks internal structures.
43) B 44) A
Version 1
42
Clarify Question ● What is the key concept addressed by the question? ● The question asks about interpreting the results of an experiment. ● What type of thinking is required? ● You are being asked to apply your knowledge of interpreting the results of an experiment. ● What key words does the question contain and what do they mean? ● Hypothesis—suggested explanation that might be true and is subject to testing by further observations Gather Content ● What do you know about interpreting the results of an experiment? What other information is related to the question? ● If only half of the lawn was mowed with 1/3 gallon of gas, then it would be logical that 2/3 of a gallon would be needed to mow the whole lawn. ● You have data from an experiment:how much gas you had and the fact that the mower ran out of gas. Choose Answer ● Given what you now know, what information is most likely to produce the correct answer? ● While your results support your hypothesis, they do not prove the hypothesis. Other variables could also influence the amount of fuel necessary to mow your entire lawn. Reflect on Process ● Did your problem-solving process lead you to the correct answer? If not, where did the process break down or lead you astray? How can you revise your approach to produce a more desirable result? ● If you got the correct answer, great job! If you got an incorrect answer, where did the process break down? Did you think that you could prove a hypothesis?Did you understand that positive results can support a Version 1
43
hypothesis but not prove it?
45) B 46) B 47) D
Version 1
44
Clarify Question ● What is the key concept addressed by the question? ● The question is about the differences between inductive and deductive reasoning. ● What type of thinking is required? ● This question is asking for you to apply the definition of inductive and deductive reasoning to specific examples. ● What key words does the question contain and what do they mean? ● Studying turtles by DNA analysis—this means sequencing the DNA from individuals and comparing the similarity of the sequences, more closely related the species have more similar DNA sequences ● Genus and species names are given—losely related species will have the same genus Gather Content ● What do you know about relatedness based on species names? What other information is related to the question? ● There are two parts of a species name and those with the same genus are the most closely related. ● In which examples are general principles being used and in which are specific results being considered? In inductive reasoning there are data to analyze to create a hypothesis.In deductive reasoning you make conclusions or predictions from existing data or observations. Choose Answer ● Given what you now know, what information is most likely to produce the correct answer? ● Data (DNA sequences) are being analyzed to draw conclusions, so this is inductive reasoning.The two Version 1
45
turtle species with the same genus should have the most similarity in their DNA sequences. Reflect on Process ● Did your problem-solving process lead you to the correct answer? If not, where did the process break down or lead you astray? How can you revise your approach to produce a more desirable result? ● If you got the correct answer, great job! If you got an incorrect answer, where did you get stuck? Did you not realize that the two species of turtles from the same genus would be closely related? Did you not recognize that analyzing DNA sequences to draw a conclusion was inductive reasoning?
48) A 49) C 50) D
Version 1
46
Clarify Question ● What is the key concept addressed by the question? question is about designing an experiment.
● The
● What type of thinking is required? ● This question is asking you to apply you knowledge to identify an appropriate control for an experiment. ● What key words does the question contain and what do they mean? ● The hypothesis is to test the effectiveness of fluoride in the water on preventing tooth decay Gather Content ● What do you know about experimental design? What other information is related to the question? ● The neighborhood being tested already has fluoride in the water.If you want to see if fluoride is effective in preventing tooth decay you need a control group with no fluoride to compare it with. ● What are other explanations are there for these results? Adding any fluoride to the water would not be a good control group. Choose Answer ● Given what you now know, what information is most likely to produce the correct answer? ● The best control would be a group of individuals with no fluoride in their water. Reflect on Process ● Did your problem-solving process lead you to the correct answer? If not, where did the process break down or lead you astray? How can you revise your approach to produce a more desirable result? ● If you got the correct answer, great job! If you got an incorrect answer, where did you get stuck? Did you realize that fluoride is your Version 1
47
independent variable, this is the variable the researcher can control? Did you realize that adding any fluoride to the water of the control group would make it more difficult to test your hypothesis?
51) A 52) D 53) E 54) B 55) C
Version 1
48
Clarify Question ● What is the key concept addressed by the question? question is about identifying a hypothesis.
● The
● What type of thinking is required? ● This question is asking you to analyze statements describing the hypothesis being tested in an experiment. ● What key words does the question contain and what do they mean? ● Cowpox—a virus carried by cows that does not cause disease in humans ● Smallpox—a virus related to cowpox, but does cause infection in humans Gather Content ● What do you know about a hypothesis? What other information is related to the question? ● A hypothesis is tested in an experiment.In this case you want to identify a hypothesis that supports the observed results. ● The cowpox infection seems to be protecting the child from smallpox. This would require the smallpox virus being recognized by the child's immune system. Choose Answer ● Given what you now know, what information is most likely to produce the correct answer? ● The best hypothesis to support the data is that exposure to the cowpox virus protected the child from the smallpox virus. Reflect on Process ● Did your problem-solving process lead you to the correct answer? If not, where did the process break down or lead you astray? How can Version 1
49
you revise your approach to produce a more desirable result? ● If you got the correct answer, great job! If you got an incorrect answer, where did you get stuck?Did you realize that vaccinating a child with cowpox pus would protect the child from smallpox? Were you confused about the definition of a hypothesis?
56) D
Version 1
50
Clarify Question ● What is the key concept addressed by the question? question is about identifying different properties of life.
● The
● What type of thinking is required? ● You are being asked to apply the definitions of the properties of life. ● What key words does the question contain and what do they mean? ● Wood—tissue from a living tree Gather Content ● What do you know about the properties of life? What other information is related to the question? ● Growth, development, and reproduction—requires use of energy ● Homeostasis—maintenance of relatively constant internal conditions ● Energy utilization—harvesting and use of energy either from solar or consumed nutrients ● Organization—requires energy to establish, but can be maintained after death ● Which properties would be lost after an organism dies? Homeostasis, growth,and energy utilization all require that an organism be alive. Choose Answer ● Given what you now know, what information is most likely to produce the correct answer? ● While organization requires energy to become established, it can remain after the death of an organism. Reflect on Process ● Did your problem-solving process lead you to the correct answer? If not, where did the process break down or lead you astray? How can Version 1
51
you revise your approach to produce a more desirable result? ● If you got the correct answer, great job! If you got an incorrect answer, where did you get stuck? A tree still has structure after it dies from its cell walls. The other answers all require the organism to still be alive.
57) Section Break 57.1) B
Version 1
52
Clarify Question ● What is the key concept addressed by the question? ● The question is about a food web, specifically why flies and spiders are more abundant. ● What type of thinking is required? ● This question is asking to evaluate explanations about a food web in relation to observationand reasoning. ● What key words does the question contain and what do they mean? ● Flies—prey for spiders ● Spiders—prey for centipedes and predators of flies ● Centipedes—predators of flies and spiders Gather Content ● What do you know about food webs? What other information is related to the question? ● In a food web an organism will often be prey for some species and a predator of others. As a result the abundance of prey and predators can both influence the numbers of a specific species. ● Which food webs make sense? Both spiders and flies are increased in number, while centipedes are not present in plot 4. 1.If flies are at the bottom of the food web they will not consume other species, but serve as a food supply for them. 2.Spiders are in the middle of the food web and their numbers are influenced by the levels of prey (flies) and predators (centipedes). 3.Centipedes are the top predators in this food web and their numbers can be affected by the abundance of prey (flies and spiders).
Version 1
53
Choose Answer ● Given what you now know, what information is most likely to produce the correct answer? ● The best answer is that there are a lot of flies (prey) and few predators (centipedes) which leads to an abundance of spiders. Reflect on Process ● Did your problem-solving process lead you to the correct answer? If not, where did the process break down or lead you astray? How can you revise your approach to produce a more desirable result? ● If you got the correct answer, great job! If you got an incorrect answer, where did the process break down? Did you think that flies were not at the bottom of the food web, or centipedes were not at the top of the food web?
57.2) D 57.3) A
Version 1
54
Clarify Question ● What is the key concept addressed by the question? ● The question is about a food web, specifically why flies and spiders are less abundant. ● What type of thinking is required? ● This question is asking to evaluate explanations about a food web in relation to observationand reasoning. ● What key words does the question contain and what do they mean? ● Flies—prey for spiders ● Spiders—prey for centipedes and predators of flies ● Centipedes—predators of flies and spiders Gather Content ● What do you know about food webs? What other information is related to the question? ● Which food webs make sense? Both spiders and flies are decreased in number, while centipedes are increased in plot 5. 1.If centipedes preferred spiders over flies then the numbers of flies would be more similar to those in plots 14. 2.The centipede population is larger than in the other plots, so there must have been enough flies to support them. 3.If there were not enough spiders to consume all of the flies then the number of flies would be increased. Choose Answer ● Given what you now know, what information is most likely to produce the correct answer? ● Which food webs would be consistent with these results?The best answer is that the abundant centipedes ate a lot of flies and spiders, reducing their number in Version 1
55
plot 5. Reflect on Process ● Did your problem-solving process lead you to the correct answer? If not, where did the process break down or lead you astray? How can you revise your approach to produce a more desirable result? ● If you got the correct answer, great job! If you got an incorrect answer, where did the process break down? Did you think that flies were not at the bottom of the food web, or centipedes were not at the top of the food web?
57.4) C
Version 1
56
Clarify Question ● What is the key concept addressed by the question? ● The question asks about interpreting the results of an experiment on a food web. ● What type of thinking is required? ● This question is asking you to evaluate explanations about a food web as related to observation and reasoning. ● What key words does the question contain and what do they mean? ● Flies—prey for spiders ● Spiders—prey for centipedes and predators of flies ● Centipedes—predators of flies and spiders Gather Content ● What do you know about food webs? What other information is related to the question? ● In a food web an organism will often be prey for some species and a predator of others.As a result the abundance of prey and predators can both influence the numbers of a specific species.Centipedes are predators of flies and spiders. ● It is the interaction of the organisms that you need to match up to correctly identify the matching hypothesis. Choose Answer ● Given what you now know, what information is most likely to produce the correct answer? ● When centipedes increase in number, flies and spiders decrease. When centipedes decrease in number, flies and spiders increase. ● The best explanation of these results is that centipedes eat flies and spiders, so their numbers move inversely with each other. Reflect on Process Version 1
57
● Did your problem-solving process lead you to the correct answer? If not, where did the process break down or lead you astray? How can you revise your approach to produce a more desirable result? ● If you got the correct answer, great job! If you got an incorrect answer, where did the process break down? Did you realize that centipedes eat spiders and flies?Did you understand that when a predator number increases the prey decrease?
58) Section Break 58.1) D
Version 1
58
Clarify Question ● What is the key concept addressed by the question? question is about interpreting phylogenetic trees.
● The
● What type of thinking is required? ● This question is asking for you to apply the principles of a phylogenetic tree to infer relatedness. ● What key words does the question contain and what do they mean? ● Evolutionary recent derived characters—traits that related species have and appeared more recently than other shared traits Gather Content ● What do you know about relatedness based on a phylogenetic tree? What other information is related to the question? ● The Y axis measures time, the longer the line, the more time since the living species at the end of either branch shared a common ancestor. The nodes or branchpoints refer to recent common ancestors. Choose Answer ● Given what you now know, what information is most likely to produce the correct answer? ● R and D have the most recent common ancestor and this ancestor is indicated by branchpoint 6.The other pairs of species all have more distant common ancestors based on the lengths of the lines. Reflect on Process ● Did your problem-solving process lead you to the correct answer? If not, where did the process break down or lead you astray? How can you revise your approach to produce a more desirable result? ● If you got the correct answer, great job! If Version 1
59
you got an incorrect answer, where did you get stuck? Did you realize that the Y axis (vertical) represents time and the numbers at the branch points represent common ancestors?
58.2) B
Version 1
60
Clarify Question ● What is the key concept addressed by the question? question is about interpreting phylogenetic trees.
● The
● What type of thinking is required? ● This question is asking for you to apply the principles of a phylogenetic tree to infer the most distant common ancestor. ● What key words does the question contain and what do they mean? ● Time to diverge—the most time elapsed since the species had a common ancestor Gather Content ● What do you know about relatedness based on a phylogenetic tree? What other information is related to the question? ● The Y-axis reflects the amount of time since two species had a common ancestor. The common ancestors are indicated by the branch points. ● Which lines connecting two species are the longest? R and D have a common ancestor at branch point 6, this is the shortest of the links between two species. So these species had the shortest time to diverge. Choose Answer ● Given what you now know, what information is most likely to produce the correct answer? ● Using the logic above the two species with the longest time to diverge are F and Z with a common ancestor at branch point 12. Reflect on Process ● Did your problem-solving process lead you to the correct answer? If not, where did the process break down or lead you astray? How can you revise your approach to produce a more Version 1
61
desirable result? ● If you got the correct answer, great job!If you got an incorrect answer, where did you get stuck? Did you realize that the Y axis (vertical) represents time and the numbers at the branch points represent common ancestors?
58.3) D
Version 1
62
Clarify Question ● What is the key concept addressed by the question? question is about the formation of fossils.
● The
● What type of thinking is required? ● This question is asking you to apply the principles of fossil formation to interpret data. ● What key words does the question contain and what do they mean? ● Fossils—preserved organisms from geological timescale in the past Gather Content ● What do you know about relatedness based on location in a rock layer? What other information is related to the question? ● Sedimentary rock forms when sediment (silt and sand) drops to the bottom of a lake or ocean, covering whatever is on the bottom of the lake or ocean. In general when fossils are found in different layers, the older fossils are found at the bottom. ● In sedimentary rock the upper layers are newer. Whether or not the two different species are related does not impact what layer they are found in, and you also cannot assume that one evolved from the other. Choose Answer ● Given what you now know, what information is most likely to produce the correct answer? ● Sediments form at the bottom of a lake and accumulate over time.Thus the older fossils are in the older layer and these organisms died before those in the upper layer. Reflect on Process ● Did your problem-solving process lead you to the correct answer? If not, where did the process break down or lead you Version 1
63
astray? How can you revise your approach to produce a more desirable result? ● If you got the correct answer, great job! If you got an incorrect answer, where did you get stuck? Did you recall that as sedimentary rock forms it traps dead animals that can then become fossils?
59) Section Break 59.1) A
Version 1
64
Clarify Question ● What is the key concept addressed by the question? question is about designing an experiment.
● The
● What type of thinking is required? ● This question is asking you to apply your knowledge of experimental design to identify an appropriate control for an experiment. ● What key words does the question contain and what do they mean? ● The hypothesis is that light is necessary for seed germination Gather Content ● What do you know about experimental design? What other information is related to the question? ● Seeds were placed in three conditions, in the dark, artificial light and direct sunlight. ● Which of the three groups is the control? If you are testing to see if light is necessary for germination then the two conditions exposing seeds to light are not good control groups. Choose Answer ● Given what you now know, what information is most likely to produce the correct answer? ● The seeds in the dark are the control group. The other two groups are receiving some kind of light, and this is the hypothesis that is being tested, so these seeds are in the test or experimental group. Reflect on Process ● Did your problem-solving process lead you to the correct answer? If not, where did the process break down or lead you astray? How can you revise your approach to produce a more desirable result? ● If you got the correct answer, great job! If Version 1
65
you got an incorrect answer, where did you get stuck? A control group is what you are comparing your test groups to. Were you confused that the hypothesis being tested was that light was required for germination?
59.2) C
Version 1
66
Clarify Question ● What is the key concept addressed by the question? question is about designing an experiment.
● The
● What type of thinking is required? ● This question asks you to apply the definition of dependent variable to identify it in an experiment. ● What key words does the question contain and what do they mean? ● Dependent variable—the variable that the researchers are measuring Gather Content ● What do you know about dependent variables? What other information is related to the question? ● Dependent variables are what the researcher is measuring after making changes in the independent variable. ● Moisture and temperature were kept constant and were controlled variables. ● The amount of light was the independent variable the researcher changed. Choose Answer ● Given what you now know, what information is most likely to produce the correct answer? ● The variable being measured was the rate of seed germination, so this is the dependent variable. The amount of light was being changed by the experimenter, so light is the independent variable. Reflect on Process ● Did your problem-solving process lead you to the correct answer? If not, where did the process break down or lead you Version 1
67
astray? How can you revise your approach to produce a more desirable result? ● If you got the correct answer, great job! If you got an incorrect answer, where did you get stuck? Did you recall that the dependent variable is dependent on changes in the independent variable? In this example the amount of germination (dependent variable) was affected by the amount of light (independent variable).
60) Section Break 60.1) A
Version 1
68
Clarify Question ● What is the key concept addressed by the question? question is about different types reasoning.
● The
● What type of thinking is required? ● Applying the definitions of different types of reasoning to a specific example. ● What key words does the question contain and what do they mean? ● Applied theory—using a theory to solve a problem Gather Content ● What do you know about different types of reasoning? What other information is related to the question? ● Data was being analyzed and used to make a prediction. ● Deductive reasoning uses a definition or theory to draw conclusions. ● Reductionism breaks an object into pieces to study how each piece works independently. Choose Answer ● Given what you now know, what information is most likely to produce the correct answer? ● Inductive reasoning uses data to make a prediction, so this is the correct answer.The doctor was measuring Luke’s current height and using it to predict his future growth. Reflect on Process ● Did your problem-solving process lead you to the correct answer? If not, where did the process break down or lead you astray? How can you revise your approach to produce a more desirable result? ● If you got the correct answer, great job! If you got an incorrect answer, where did you get stuck? Did you Version 1
69
forget that treating a prediction or hypothesis from data is inductive reasoning? Or did you forget that deductive reasoning is creating a hypothesis from existing data or definitions?
60.2) D
Version 1
70
Clarify Question ● What is the key concept addressed by the question? question is about different types reasoning.
● The
● What type of thinking is required? ● Applying the definitions of different types of reasoning to a specific example. ● What key words does the question contain and what do they mean? ● Applied theory—using a theory to solve a problem Gather Content ● What do you know about different types of reasoning? What other information is related to the question? ● A definition or theory is used to draw conclusions in deductive reasoning. ● Inductive reasoning analyzes data to make a prediction. ● Reductionism breaks an object into pieces to study how each piece works independently. Choose Answer ● Given what you now know, what information is most likely to produce the correct answer? ● As an existing graph of percentile weight and length was used this is deductive reasoning. Based on Luke’s current height the doctor could make a prediction about Luke’s height in the future. Reflect on Process ● Did your problem-solving process lead you to the correct answer? If not, where did the process break down or lead you astray? How can you revise your approach to produce a more desirable result? ● If you got the correct answer, great job! If you got an incorrect answer, where did you get stuck? Did you forget that using a chart or table to make a prediction is deductive Version 1
71
reasoning?
61) Section Break 61.1) A
Version 1
72
Clarify Question ● What is the key concept addressed by the question? question is about designing an experiment.
● The
● What type of thinking is required? ● This question is asking you to apply the definition of the dependent variable to identify it in an experiment. ● What key words does the question contain and what do they mean? ● Cloaca—thin film on eggs, predators may use its scent to locate the eggs ● Dependent variable—what the researcher is measuring to see if it changes when the independent variable is changed Gather Content ● What do you know about dependent variables? What other information is related to the question? ● The dependent variable is the variable the researcher measures. ● The independent variable is the variable that the researcher changes, in this case the presence or absence of cloaca. ● Control variables are held constant, in this case the time spent in the ground. Choose Answer ● Given what you now know, what information is most likely to produce the correct answer? ● The dependent variable is the number of hatchlings, this is what the researchers are measuring to see if the presence of cloaca affects predation of the eggs. Reflect on Process ● Did your problem-solving process lead you to the correct answer? If not, where did the process break down or lead you Version 1
73
astray? How can you revise your approach to produce a more desirable result? ● If you got the correct answer, great job!If you got an incorrect answer, where did you get stuck? Did you recall that the dependent variable is dependent on changes in the independent variable? In this case the number of hatchlings is dependent on the presence or absence of cloaca.
61.2) B
Version 1
74
Clarify Question ● What is the key concept addressed by the question? question is about designing an experiment.
● The
● What type of thinking is required? ● This question is asking you to apply the definition of the independent variable to identify it in an experiment. ● What key words does the question contain and what do they mean? ● The eggs were harvested from trapped turtles and the egg's cloaca film (reproductive discharge) was either washed off or left on once gathered. Gather Content ● What do you know about controls? What other information is related to the question? ● Controls are what the researcher compares their test samples to. ● The dependent variable is the variable that the researcher measures, in this case the number of hatchlings. ● Control variables are held constant, in this case the time spent in the ground. Choose Answer ● Given what you now know, what information is most likely to produce the correct answer? ● The independent variable is the presence of cloaca affects predation of the eggs. Reflect on Process ● Did your problem-solving process lead you to the correct answer? If not, where did the process break down or lead you astray? How can you revise your approach to produce a more desirable result? ● If you got the correct answer, great job!If you Version 1
75
got an incorrect answer, where did you get stuck? Did you recall that the dependent variable is dependent on changes in the independent variable? In this case the number of hatchlings is dependent on the presence or absence of cloaca.
61.3) B
Version 1
76
Clarify Question ● What is the key concept addressed by the question? question is about designing an experiment.
● The
● What type of thinking is required? ● This question is asking you to apply the definition of a control group identify it in an experiment. ● What key words does the question contain and what do they mean? ● Control group—the group in which the researcher keeps the independent variable constant Gather Content ● What do you know about control groups? What other information is related to the question? ● The control group is what you compare the test group to by changing the independent variable. ● The independent variable is the variable that the researcher changes, in this case the presence or absence of cloaca. ● The dependent variable is the variable that the researcher measures, in this case the number of hatchlings. Choose Answer ● Given what you now know, what information is most likely to produce the correct answer? ● The control group are the turtle eggs that did not have their cloaca wiped off. The rate of hatching can then be compared to the rate of hatching in eggs with their cloaca wiped off. Reflect on Process ● Did your problem-solving process lead you to the correct answer? If not, where did the process break down or lead you Version 1
77
astray? How can you revise your approach to produce a more desirable result? ● If you got the correct answer, great job! If you got an incorrect answer, where did you get stuck? Did you confuse the control with the dependent and independent variables? Which eggs were not altered by the researcher in the experiment?
Version 1
78
CHAPTER 2 MULTIPLE CHOICE - Choose the one alternative that best completes the statement or answers the question. 1) Matter iscomposed of __________. A) atoms B) energy C) mass D) molecules
2) All atoms possess the ability to do work. The term that is defined as the ability to do work is __________. A) matter B) energy C) molecules D) space
3)
The number of protons in a given atom is equal to its __________. A) atomic number B) mass C) neutron number D) molecular number
4) Isotopes that are unstable and decay when their nucleus breaks up into elements with lower atomic numbers, emitting significant amounts of energy in the process, are called __________.
Version 1
1
A) energetic B) ionic C) radioactive D) isometric
5)
Atoms containing a specific number of protons are called __________. A) minerals B) elements C) metals D) molecules
6) Which chemical group forms hydrogen bonds with water and is most likely to explain why sugars dissolve well in water? A) -O-H B) -N-H C) -C-H D) -C-C
7) The negative logarithm of the hydrogen ion concentration in the solution is referred to as __________. A) pH B) atomic mass C) OH – concentration D) electronegativity E) specific heat
8) Bicarbonate ions in the blood can absorb hydrogen ions, keeping the pH balanced. Therefore bicarbonate is acting as a __________ in the blood. Version 1
2
A) buffer B) acid C) base D) alkaline
9)
Atomic nuclei contain protons and __________. A) isomers B) ions C) moles D) neutrons
10)
Carbon-12, carbon-13 and carbon-14 are examples of __________. A) ions B) isotopes C) isomers D) molecules
11) Organisms are composed of molecules, which are collections of smaller units, termed __________. A) monomers B) atoms C) electrons D) polymers E) ions
12)
Negatively charged subatomic particles that have almost no mass are called __________.
Version 1
3
A) electrons B) protons C) neutrons D) ions E) polymers
13) Atoms of a single element that possess different numbers of neutrons are called __________. A) polymers B) ions C) monomers D) isomers E) isotopes
14)
Cl + e
–
→ Cl
–
is an example of a __________ reaction.
A) oxidation B) reduction C) polymerization D) ionization
15) When atoms gain or lose electrons, they can become negatively or positively charged. These negatively or positively charged atoms are known as __________. A) isotopes B) ions C) isomers D) unstable atoms
16)
When two atoms share a pair of electrons, the bonding is referred to as __________.
Version 1
4
A) ionic B) covalent C) unstable D) hydrogen
17) Water molecules are polar with ends that exhibit partial positive and negative charges. These opposite charges allow water molecules to attract each other through __________. A) ionic bonds B) covalent bonds C) hydrogen bonds D) peptide bonds
18)
An atom has 20 electrons and 20 neutrons. What is the mass of this atom? A) 10 B) 20 C) 40 D) 80
19) Sue was monitoring the oil spill into the Gulf of Mexico from an oil tanker. From her observations, she noted that the oil was moving as large patches on the water. It did not appear as though the oil was dissolving into the water. Why did the oil not dissolve into the water? A) Hydrophobic interactions B) Surface tension C) Sea water acts as a solvent D) Water forms hydration shells E) Water has a high heat of vaporization
20)
The atomic number of an element is equal to the number of __________.
Version 1
5
A) protons only B) neutrons only C) protons plus electrons D) protons plus neutrons E) neutrons plus electrons
21) Oxygen has an atomic mass of 16 and an atomic number of 8. How many neutrons are present? A) 24 B) 8 C) 16 D) 4
22) The pH of your small intestines is around 7.5 and the pH of your large intestine can be 5.5. As substances travel from the small intestines to the large intestine, what would happen to the H+ ion concentration? A) It decreases 100-fold. B) It increases by 100-fold. C) It increases 10-fold. D) It increases 2-fold. E) It decreases 10-fold.
23) Oxygen-16 is abundant andhas 8 protons and 8 neutrons. Oxygen-18 is a/an __________ ofoxygen-16. A) ion B) isotope C) isomer D) dimer
Version 1
6
24) Which element's isotope is commonly used to determine when biological samples, such as fossils, were formed? A) Oxygen B) Hydrogen C) Carbon D) Nitrogen E) Sulfur
25) Atoms in which the number of electrons does not equal the number of protons are known as __________. A) valences B) ions C) isotopes D) isomers
26) The area around a nucleus where an electron is most likely to be found is the __________. A) electrical space B) energy level C) polar space D) orbital
27)
Regardless of its shape, a given orbital may contain no more than __________. A) one electron B) four electrons C) eight electrons D) two electrons
Version 1
7
28) All atoms tend to fill their outer energy levels with the maximum number of electrons, usually eight. Depending on whether atoms satisfy the octet rule will predict __________. A) the chemical behavior of the atoms B) whether they will be found in nature C) whether they will dissolve in water D) their radioactive energy
29) Mendeleev found that when he arranged the known elements according to their atomic mass, the entries in the table exhibited a pattern of chemical properties that repeated itself in groups of eight elements. This led to the generalization now known as __________. A) an atomic model B) valance electrons C) the periodic table D) the octet rule
30) Sodium has 11 electrons arranged in three energy levels. In order to become stable, sodium forms an ion with __________. A) no charge B) –1 charge C) –8 charge D) +1 charge E) +8 charge
31) In the crystal matrix of ordinary salt, the sodium and chlorine are held together by __________.
Version 1
8
A) peptide bonds B) covalent bonds C) ionic bonds D) hydrogen bonds E) nonpolar bonds
32) Two carbon atoms joined to each other by the sharing of two pairs of electrons form a/an __________. A) single bond B) ionic bond C) hydrogen bond D) double covalent bond
33) In a chemical analysis of an animaltissue sample, which element would be in the leastquantity? A) Carbon B) Hydrogen C) Nitrogen D) Oxygen E) Iodine
34) Life is thought to have evolved from complex molecules formed by the interaction of smaller molecules in oceans and the atmosphere. The substance which brought these molecules together to interact is __________. A) hydrogen B) acids C) water D) buffers E) salts
Version 1
9
35) Because oxygen is more electronegative than hydrogen, the water molecule is __________. A) hydrophobic B) hydrophilic C) nonpolar D) ionic E) polar
36) Water molecules are attracted to each other due tothe opposite charges created by partial charge separations within the molecules. These attractions are called __________. A) peptide bonds B) covalent bonds C) ionic bonds D) hydrogen bonds E) double bonds
37) An oxygen atom in water has two covalent bonds with hydrogen atoms and two unpaired electrons. How manyhydrogen bonds can awater molecule form? A) One B) Two C) Three D) Four E) Five
38) Nitrogen has a higher electronegativity than hydrogen. As a result you would expect that ammonia (NH 3) molecules can form __________ with each other.
Version 1
10
A) hydrogen bonds B) hydrophilicbonds C) ionic bonds D) covalent bonds E) cohesive bonds
39) When water ionizes, it produces equal amounts of hydrogen and hydroxide ions thatcan reassociate with each other. The pH of water is __________. A) three B) four C) five D) six E) seven
40) A scientist conducts a procedure that causes nitrogen atoms to gain neutrons. The resulting atoms will be __________. A) ions of nitrogen B) positively charged C) negatively charged D) isotopes of nitrogen E) new elements with higher atomic numbers
41) The half-life of carbon-14 is approximately 5,700 years. Using this information scientists have been able to determine the age of some artifacts left by humans. A scientist wants to know approximately how old a piece of wood was that she collected on the floor in an old cave that had recently been discovered. Her wood sample contained 2 grams of carbon-14. If the age of the wood was determined to be 22,800 years old, how much carbon-14 originally existed in this piece of wood?
Version 1
11
A) 32 grams B) 16 grams C) 12 grams D) 8 grams E) 4 grams
42) Plants transport water to their leaves through thexylem when water evaporates from the leaves. The evaporating water pulls other water molecules up the xylem through __________. A) hydrogen bonds B) ionic bonds C) covalent bonds D) hydrophobic interactions
43) Water is most dense and thus heaviest at 4°C. At 0°C, ice forms and can float on liquid water. Suppose ice were most dense at 0°C. What would happen in a lake at this temperature? A) The ice wouldcover the surface of the aquatic system and would never melt. B) The ice wouldcover the bottom of the aquatic system and would build up in layers overtime. C) Ice would not formbecause solids are always less dense than liquids. D) The cold temperatures and the subsequent ice formation would prevent hydrogen bonds from forming between the water molecules, thus causing the existing ice crystals to become disassociated from each other.
44) Your dog becomes ill and you rush him to the veterinarian's office. A technician draws blood your dog. After a few minutes the lab results are given to the vet, who immediately begins to fill a syringe with an unknownfluid. You inquire about the fluid, and the vet informs you the fluid is necessary to manage your dog's metabolic acidosis. Based on the information provided, what is acidosis, and what is the likely effect of the injection?
Version 1
12
A) Acidosis means thatyour dog's blood pH has dropped from its normal level, and an injection ofsaline is required to reverse the condition. B) Acidosis means thatyour dog's blood pH has increased from its normal level, and an injection ofsaline is required to reverse the condition. C) Acidosis means thatyour dog's blood pH has decreased from its normal level, and an injection ofbuffering solutionis required to reverse the condition. D) Acidosis means thatyour dog's blood pH has increased from its normal level, and an injection ofbuffering solutionis required to reverse the condition.
45) As you and a friend are entering a chemistry laboratory at your university, you see a sign that states: DANGER—RADIOACTIVE ISOTOPES IN USE. Your friend is an accounting major and has not had any science courses yet. She asks you what a radioactive isotope is and you respond correctly with __________. A) radioactive isotopes are atoms that are unstable and as a result emit energy in a process called radioactive decay B) radioactive isotopes are atoms that are stable and as a result emit energy in a process called radioactive decay C) radioactive isotopes are atoms that are stable and as a result only emit energy if they are exposed to higher temperatures D) radioactive isotopes are atoms that are unstable but unless actively disturbed by some chemical process will remain intact and pose no problems
46)
An effective way to increase the rate of the reaction A+B→ C would be to __________. A) add more C B) decrease the temperature C) add more A or B D) remove the catalyst
47)
The two nitrogen atoms in nitrogen gas (N
Version 1
2) share six electrons forming a __________.
13
A) triple covalentbond B) double covalentbond C) single covalentbond D) hydrogen bond E) double bond
48) Capillary action is one of the forces that aids water's upward movement in plants. The narrower the diameter of the tube, the farther the water column will rise. Capillary action is a result of water molecules __________. A) storing heat and thus moving faster because of heat of vaporization B) producing sufficient surface tension to overcome the pull of gravity C) having a strong cohesive force and attaching to the surrounding vessel walls D) having an adhesive force, which allows them to attach to the vessel walls E) being associated with hydrophobic molecules, which can result in upward movement
49)
Which atomic particle has no charge and is located in the nucleus? A) Electron B) Ion C) Neutron D) Proton E) Isotope
50)
The subatomic particle with a positive charge is __________. A) an electron B) an ion C) a neutron D) a proton E) an isotope
Version 1
14
51)
The smallest subatomic particle of those listed below is the __________. A) electron B) ion C) isotope D) neutron E) proton
52) Anatom that is negatively charged because it has accepted an electron is a/an __________. A) isomer B) ion C) isotope D) monomer
53) One mole of a substance contains 6.02 x 10 23 molecules. The atomic number of lithium (Li) is 3 and the atomic mass is approximately 7. If you have three moles of Li, how many grams does it weigh? A) 9 B) 21 C) 7 x 6.02 x 10 23 D) 3 x 6.02 x 10 23
54) After taking your biology exam, you return to your car only to find that you had left the lights on and now the car battery is dead. Your friend offers to jump-start your car, but when you go to hook up the jumper cables you find that the battery terminals are covered with corrosion due to battery acid condensation. Based on your knowledge, what substance could be used to clean the corrosion?
Version 1
15
A) Coffee (pH of 5) B) Water (pH of 7) C) Vinegar (pH of 3) D) Baking soda (pH of 9)
55)
The amino acid glycine (C
3NO
2H
6) is a/an __________.
A) organicmolecule B) inorganicmolecule C) element D) vitamin
56) Consider the following electronegativity values: Boron (B) = 1.8 Carbon (C) = 2.5 Chlorine (Cl) = 3.2 Selenium (Se) = 2.6 Which of the following bonds is the most polar? A) B-Cl B) C-Cl C) Se-Cl D) It cannot be determined from the information provided.
57) The reaction (H2 + F2 → 2HF) is an example of a redox reaction. In reality, two half reactions are occurring. The half reaction (H2 → 2H+ + 2e-) is a/an __________. A) reduction reaction B) oxidation reaction C) redox reaction D) potential energy reaction
Version 1
16
58) The electronic configuration of the noble gas neon, which has an atomic number of 10, can be written as follows: 1s 22s 22p 6. What is the electronic configuration of the noble gas argon, which has an atomic number of 18? A) 1s 22s 23p 64s 25p 6 B) 1s 22s 62p 23s 63p 2 C) 1s 22s 22p 63s 23p 6 D) 1s 22s 83p 8
59) You identifyan enzyme involved in a cellular reaction. How does the enzyme affect the reaction equilibrium between reactants and products and the time needed to reach equilibrium? A) It alters the reaction equilibrium and shortens the time needed to reach equilibrium. B) The reaction equilibrium is unaffected, but it shortens the time needed to reach equilibrium. C) It alters the reaction equilibrium and lengthens the time needed to reach equilibrium. D) The reaction equilibrium is unaffected, but it lengthens the time needed to reach equilibrium.
60) You walk down into your basement to find that the carpeting on the floor is damp. Concerned, you look around for large puddles of water or broken pipes, but find none. In fact, only the basement floor and carpeting is damp. You realize that water must have wicked into the carpet from the floor by __________. A) adhesion and cohesion B) adhesion and solubility C) adhesion, cohesion,and solubility D) cohesion and solubility
61) You recently discovered a new element and find that this particular element has multiple energy levels and one electron in its outer energy level. What would you expect will happen to an atom of this element if placed in water?
Version 1
17
A) It will lose an electron forming a positive ion. B) It will lose anelectron forming a negative ion. C) It will gain an electron forming a positive ion. D) It will gain anelectron forming a negative ion.
62) Sulfur is found directly below oxygen on the periodic table.What type of bond would you predict S-H would form? A) Polar covalent B) Ionic C) Hydrogen bond D) Nonpolar covalent bond
63)
Why is it necessary to take special safety precautions when using radioactivity? A) Radioactive substances will ionize cells. B) Radioactive substances have the potential to cause damage to living cells. C) Radioactive substances decay. D) Radioactive substances will perforate plasma membranes.
64) The high heat of vaporizationof water helps you to feel cooler when you sweat because the transition of water from a liquid to a gas requires a __________ of energy to break hydrogen bonds. The energy is __________ from heat produced by your body, thus helping to lower the surface temperature of your body. A) release;released B) release;obtained C) input;obtained D) input; released
Version 1
18
65) Salt is often used to melt ice on roads during the winter because it lowers the freezing/melting point of water. When salt dissolves in water, individual Na + and Cl – ions break away from the salt lattice and become surrounded by water molecules.Why would this cause ice to melt? A) Hydrogen bondsare broken, and the salt ions interfere with interactions between H and O. As aresult, it is more difficult for water molecules to bond and form ice. B) Hydrogen bondsare formed, and the salt ions bond with O. As a result, it is more difficultyfor water molecules to bond and form ice. C) Hydrogen bonds are broken, and the salt ions bond with O and H respectively. As a result, it is more difficult for water molecules to bond and form ice.
66) A chemist adds a chemical to pure water and there is a 100-fold increase in the concentration of hydrogen ions. What is the best approximation of the new pH value? A) 0 B) 5 C) 7 D) 9 E) 14
67) The electronegativity of nitrogen (N) is 3.0, while the electronegativity of hydrogen (H) is 2.1. Knowing this, consider how the electrons will be shared in ammonia (NH 3). What do you predict about the polarity of ammonia? A) Each H atom has a partial positive charge B) The N atom has a partial positive charge C) Each H atom has a partial negative charge D) The N atom has a strong positive charge
68) Magnesium chloride (MgCl 2)is a salt formed with ionic bonds between one magnesium ion and two chloride ions. Magnesium has two electrons in its outer shell and chlorine has seven electrons in its outer shell. How are the electrons transferred between these atoms?
Version 1
19
A) Both magnesium and chlorine are reduced B) Both magnesium and chlorine are oxidized C) Chlorine is oxidized and magnesium is reduced D) Magnesium is oxidized and chlorine is reduced
69) The carbonic acid and bicarbonate buffer in blood is extremely important to help maintain homeostasis. What would happen to the pH of the blood if bicarbonate were removed? A) Removing bicarbonate from the blood would increase the pH. B) Removing bicarbonate from the blood would decrease the pH. C) Removing bicarbonate from the blood would not affectthe pH.
70) The common basilisk lizard will run across water on its hind legs in an erect position when startled by predators. This lizard has large feet and flaps of skin along its toes. What properties of water allow this lizard to walk on water? A) Hydrogen bondsabsorb heat when they break and release heat when they form. This helps tominimize temperature changes. B) The surface tension created by hydrogen bonds is greater than the weight of the lizard initially. C) Polar molecules areattracted to ions and polar compounds, making these compounds soluble. D) Hydrogen bonds hold water moleculestogether; manyhydrogen bonds must be broken for water to evaporate.
71) Chlorine (Cl) has 7 electrons in its outer shell and potassium (K) has 1 electron in its outer shell. How is the bond in Cl 2 different from the bond in KCl? A) Cl 2 is covalent and KCl is ionic. B) Cl 2 and KCl are both ionic. C) Cl 2 is ionic and KCl is covalent.
Version 1
20
72) Carbon has four valence electrons and oxygen has six. Carbon dioxide would contain __________. A) one single covalent bond B) four hydrogen bonds C) two double covalent bonds D) two single covalent bonds
73) If water were nonpolar it would not form hydrogen bonds. At normal room temperatures this nonpolar water would be __________. A) a gas B) a liquid C) a solid
74) Proteins are three dimensional molecules made of strands of amino acids (imagine a ball of string). There are 20 different amino acids used in proteins found in living organisms. Some of these amino acidsare polar and others are nonpolar.Where would a series of nonpolar amino acids most likely be located in a protein that is found in the cytosol of an animal cell? A) On the surface of the protein B) In the interior of the protein C) At the very top of the protein D) At the very bottom of the protein
75) According to most car mechanics, plain water is the best coolant to use in an engine provided the engine is not being exposed to freezing temperatures. If the car is subject to freezing temperatures, then a mixture of water and ethylene glycol (antifreeze) is recommended, but it does not cool as efficiently as plain water. Why would ethylene glycol reduce the cooling efficiency of water?
Version 1
21
A) Ethylene glycolhas a lower heat capacity than water. B) Ethylene glycolhas a higher heat capacity than water. C) Hydrogen bonds in water allow high levels of heat absorption anda large increase in temperature. D) Ethylene glycolraises the freezing point of water.
76) Dennis had a history of heart disease in his family and was reducing his intake of saturated fats. Saturated means each carbon atom is bonded to as many hydrogen atoms as it can accept. If a carbon were bonded to two carbons, how many hydrogens could it accept? A) Zero B) One C) Two D) Three E) Four
77) ___________ are an important component of antibody recognition, where a substantial amount of atoms are in close proximity between the antibody and the antigen. A) Hydrogen bonds B) Covalent bonds C) Hydrophobic interactions D) Ionic bonds E) Van der Waals attractions
Version 1
22
Answer Key Test name: Chap 02_3e 1) A 2) B 3) A 4) C 5) B 6) A 7) A 8) A 9) D 10) B 11) B 12) A 13) E 14) B 15) B 16) B 17) C 18) C
Version 1
23
Clarify Question ● What is the key concept addressed by the question? question asks about calculating the mass of an atom.
● The
● What type of thinking is required? ● You are being asked to apply the definition of atomic mass to calculate an atomic mass. ● What key words does the question contain and what do they mean? ● Atomic mass—sum of the masses of protons and neutrons in an atom (electron mass is negligable) Gather Content ● What do you know about atomic mass? What other information is related to the question? ● The massof a proton anda neutron are both (approximately) 1. Choose Answer ● Given what you now know, what information is most likely to produce the correct answer? ● 20 protons + 20 neutrons = an atomic mass of 40. Reflect on Process ● Did your problem-solving process lead you to the correct answer? If not, where did the process break down or lead you astray? How can you revise your approach to produce a more desirable result? ● If you got the correct answer, great job! If you got an incorrect answer, where did the process break down? Did you confuse atomic number (the number of protons alone) with the atomic mass (the sum of protons and neutrons)?
19) A Version 1
24
Clarify Question ● What is the key concept addressed by the question? question asks why oil and water don’t mix.
● The
● What type of thinking is required? ● You are being asked to apply your knowledge of the properties of water and oil to explain why the oil does not dissolve in water. ● What key words does the question contain and what do they mean? ● Dissolve—one compound mixes completely with a solvent like water Gather Content ● What do you know about water as a solvent? What other information is related to the question? ● Water is a polar molecule, this means that other polar molecules or ions will dissolve in it.These water loving molecules are called hydrophilic. ● Oil is a non-polar molecule and will dissolve better in a non-polar solvent. These molecules are called hydrophobic, or water fearing. ● Water is a solvent with a high heat of vaporization and surface tension. Water also forms surface tension. However, none of these properties explain why oil does not dissolve in water. Choose Answer ● Given what you now know, what information is most likely to produce the correct answer? ● The best answer is that oil is hydrophobic, meaning it does not interact well with water. This is because water is a polar molecule and oil is non-polar. Water has O-H bonds which do not share electrons equally giving the O a partial negative charge and the H a partial positive charge. In contrast oil is made up of C-H bonds and the electrons are shared more equally leading Version 1
25
to no partial charges. Because oil does not have these charges it is not attracted to the charges in water and does not dissolve well in water. Reflect on Process ● Did your problem-solving process lead you to the correct answer? If not, where did the process break down or lead you astray? How can you revise your approach to produce a more desirable result? ● If you got the correct answer, great job! If you got an incorrect answer, where did the process break down? Did you know what the word hydrophobic meant?Did you have difficulty remembering which properties of water make it a good solvent for other polar molecules?
20) A 21) B
Version 1
26
Clarify Question ● What is the key concept addressed by the question? ● The question asks you to calculate the number of neutrons from the atomic mass and number of an atom. ● What type of thinking is required? ● You are being asked to apply your knowledge of how atomic mass and atomic number are calculated. ● What key words does the question contain and what do they mean? ● Atomic mass—number of neutrons plus protons in an atom ● Atomic number—number of protons in an atom Gather Content ● What do you know about the number of neutrons in an atom? What other information is related to the question? ● Atomic mass contains both the neutrons and protons in an atom. Choose Answer ● Given what you now know, what information is most likely to produce the correct answer? ● To calculate the number of neutrons, simply take Atomic mass (P + N) minus Atomic number (P). ● For this question about oxygen: 16 – 8 = 8 neutrons Reflect on Process ● Did your problem-solving process lead you to the correct answer? If not, where did the process break down or lead you astray? How can you revise your approach to produce a more desirable result? ● If you got the correct answer, great job! If you got an incorrect answer, where did the process break down? Did you know the definitions of how atomic mass and atomic numbers? Did you have difficulty figuring out how to calculate the number of neutrons by subtracting atomic mass Version 1
27
from atomic number?
22) B
Version 1
28
Clarify Question ● What is the key concept addressed by the question? question asks about relating pH to H+ concentration.
● The
● What type of thinking is required? ● You are being asked to apply the definition of pH to a specific example. ● What key words does the question contain and what do they mean? ● pH—this is the –log[H+] Gather Content ● What do you know about the link between H+ concentration and pH? What other information is related to the question? ● Because the relationship between H+ concentration and pH is a log scale, a change in pH of 1 is a change in H+ concentration of 10-fold, and because the relationship is inverse, an increase in pH of 1 is a decrease in H+ concentration by 10-fold. ● In moving from the small intestines to the large intestines the pH goes from 7.5 to 5.5, which means the substance is moving to a more acidic environment. This means that the H+ concentration will increase. Choose Answer ● Given what you now know, what information is most likely to produce the correct answer? ● Because the pH drops by 2 pH units, the H+ concentration increases by 10x10 or 102 or 100-fold. Reflect on Process ● Did your problem-solving process lead you to the correct answer? If not, where did the process break down or lead you astray? How can you revise your approach to produce a more desirable result? ● If you got the correct answer, great job! If you got an incorrect answer, where did the process break down? Did you recall that pH is a log scale? Version 1
29
Did you recall that pH has a negative relationship to H+ concentration, so an increase in pH leads to a decrease in H+ concentration?
23) B 24) C 25) B 26) D 27) D 28) A 29) D 30) D
Version 1
30
Clarify Question ● What is the key concept addressed by the question? ● The question asks about predicting if an atom will form an ion. ● What type of thinking is required? ● You are being asked to apply your knowledge of the structure of an atom to predict ion formation. ● What key words does the question contain and what do they mean? ● Energy levels—the first level holds 2 electrons and each higher level holds 8 electrons Gather Content ● What do you know about structure of atoms? What other information is related to the question? ● If you make a diagram of the structure of a sodium atom there are 2 electrons in the first shell, 8 in the second shell and 1 in the outer (valence) shell. ● If there is one valence electron then the atom could lose that electron or gain 7 more to have a full outer shell. Choose Answer ● Given what you now know, what information is most likely to produce the correct answer? ● The atom will lose its single valence electron, losing a -1 charge carried by the electron.This will give the Na ion a +1 charge. Reflect on Process ● Did your problem-solving process lead you to the correct answer? If not, where did the process break down or lead you astray? How can you revise your approach to produce a more desirable result? ● If you got the correct answer, great job! If you got an incorrect answer, where did the process break down? Did you have trouble drawing the Version 1
31
structure of a sodium ion? Did you recognize that the sodium atom would lose an electron to have a stable outer shell?
31) C 32) D 33) E 34) C 35) E 36) D 37) D
Version 1
32
Clarify Question ● What is the key concept addressed by the question? ● The question asks about the formation of hydrogen bonds in water. ● What type of thinking is required? ● You are being asked to apply your knowledge of the structure of water to predict the formation of hydrogen bonds. ● What key words does the question contain and what do they mean? ● Hydrogen bonds—weak ionic interactions between a partial positive charge on one atom and partial negative charge on another atom ● Water has the structure H-O-H Gather Content ● What do you know about hydrogen bonds? What other information is related to the question? ● Oxygen has a stronger attraction for electrons than hydrogen and ends up with two partial negative charges. ● Hydrogen has a weaker attraction for electrons than oxygen and ends up with a partial positive charge. ● Hydrogen bonds will form between the H atom on one water molecule and the O atom on a second water molecule. Choose Answer ● Given what you now know, what information is most likely to produce the correct answer? ● Each H atom can form one hydrogen bond with an O atom on another water molecule. ● Each O atom can form hydrogen bonds with two H atoms on other water molecules. ● Water can form hydrogen bonds with 4 water molecules.Two bonds will be through the H atoms and two through the O atom. Version 1
33
Reflect on Process ● Did your problem-solving process lead you to the correct answer? If not, where did the process break down or lead you astray? How can you revise your approach to produce a more desirable result? ● If you got the correct answer, great job! If you got an incorrect answer, where did the process break down? Could you draw the structure of a water molecule? Could you identify which atoms would be involved in forming a hydrogen bond?
38) A
Version 1
34
Clarify Question ● What is the key concept addressed by the question? question asks about electronegativity.
● The
● What type of thinking is required? ● You are being asked to apply your knowledge of electronegativity to predict what types of bonds a molecule could form. ● What key words does the question contain and what do they mean? ● Electronegativity—pull of an atom on an electron Gather Content ● What do you know about electronegativity? What other information is related to the question? ● Nitrogen has a higher electronegativity than hydrogen. This means that the nitrogen atoms will have a partial negative charge and the hydrogen atoms will have a partial positive charge. ● The partial charges on N and H atoms on adjacent NH3 molecules will be attracted to each other. ● These are not covalent bonds which share electrons. These are partial charges, so they are not ionic bonds. Because these are charged they are hydrophilic and not hydrophobic. Choose Answer ● Given what you now know, what information is most likely to produce the correct answer? ● The best answer is that the attraction through partial charges is a hydrogen bond between the N on one NH3 and the H on another. Reflect on Process ● Did your problem-solving process lead you to the correct answer? If not, where did the process break down or lead you astray? How can Version 1
35
you revise your approach to produce a more desirable result? ● If you got the correct answer, great job! If you got an incorrect answer, where did the process break down? Could you draw the structure of an NH3 molecule? Did you realize how hydrogen bonds form?
39) E 40) D 41) A
Version 1
36
Clarify Question ● What is the key concept addressed by the question? question asks about radioactive decay. ● What type of thinking is required? analyze data on radioactive decay.
● The
● You are being asked to
● What key words does the question contain and what do they mean? ● Half-life—amount of time in which half of a dose of radioactivity in a sample will be lost to decay Gather Content ● What do you know about half life? What other information is related to the question? ● The first step is to figure out how many half-lives have occurred. 22,800 ÷ 5,700 = 4. In 4 half-lives the amount of radiation will have been cut in half 4 times. ● In 4 half-lives the amount of radiation will have been cut in half 4 times. ● x2x2x2 = 24 = 16, which means that the total amount of radiation will be decreased 16-fold. Choose Answer ● Given what you now know, what information is most likely to produce the correct answer? ● If 2 grams of carbon-14 remain now and this represents the amount left after a 16-fold decrease, the original amount would be 2 grams x 16 = 32 grams. Reflect on Process ● Did your problem-solving process lead you to the correct answer? If not, where did the process break down or lead you astray? How can you revise your approach to produce a more desirable result? ● If you got the correct answer, great job! If you got an incorrect answer, Version 1
37
where did the process break down? Did you recall that the amount of sample could be cut in half with each decay? One common misconception is to think that 4 rounds of decay would decrease the radioactivity 4-fold. Each round cuts the amount of radioactivity in half, so this leads to a 16-fold decrease.
42) A
Version 1
38
Clarify Question ● What is the key concept addressed by the question? ● The question asks about attractive interactions between water molecules. ● What type of thinking is required? ● You are being asked to apply your knowledge about the properties of water. ● What key words does the question contain and what do they mean? ● Ionic bonds—a full positive charge on one molecule attracted to a full negative charge on another ● Hydrophobic interactions—hydrophobic groups repel water and clump together ● Hydrogen bonds—a partial negative charge on one atom is attracted to a partial positive charge on a hydrogen atom ● Covalent bonds—sharing a pair of electrons between two atoms Gather Content ● What do you know about the properties of water? What other information is related to the question? ● Water is a polar molecule with partial negative charges on oxygen atoms and partial positive charges on hydrogen atoms. ● Covalent bonds form between atoms within a molecule, the question refers to interactions between water molecules, so these cannot be covalent. ● Ionic bonds involve full charges, water has partial charges. ● Hydrophobic interactions are found in non-polar molecules, water is polar. Choose Answer ● Given what you now know, what information is most likely to produce the correct answer? ● The best answer is that hydrogen Version 1
39
bonds form between water molecules. As one water molecule evaporates it pulls the next water molecule up, moving water through the xylem. Reflect on Process ● Did your problem-solving process lead you to the correct answer? If not, where did the process break down or lead you astray? How can you revise your approach to produce a more desirable result? ● If you got the correct answer, great job! If you got an incorrect answer, where did the process break down? Did you recall the structure of water and types of bonds it forms?
43) B
Version 1
40
Clarify Question ● What is the key concept addressed by the question? ● The question asks about the properties of liquid water and ice. ● What type of thinking is required? ● You are being asked to apply your knowledge of the relative densities of liquid water and ice to what happens in a lake in the winter. ● What key words does the question contain and what do they mean? ● Density—measure of mass/volume Gather Content ● What do you know about ice? What other information is related to the question? ● Because ice has more rigid structure than liquid water it occupies more volume.Thus the same mass of liquid water and ice vary in their density. Because liquid water occupies less volume it has a larger density than ice. This is why ice floats on top of liquid water. ● With many molecules solids are denser than liquids. The question asks you to predict what would happen if ice was denser than liquid water. Choose Answer ● Given what you now know, what information is most likely to produce the correct answer? ● If ice were denser than water it would sink to the bottom of a lake during the winter. Over time the ice would fill in the lake from the bottom up, freezing everything in the lake. Reflect on Process ● Did your problem-solving process lead you to the correct answer? If not, where did the process break down or lead you astray? How can you revise your approach to produce a more desirable result? ● If you got the correct answer, great job! If you got an incorrect answer, Version 1
41
where did the process break down? Did you realize that if ice was denser than liquid water it would sink? The density would not affect the ability to form or break hydrogen bonds.
44) C
Version 1
42
Clarify Question ● What is the key concept addressed by the question? question asks about the treatment of acidosis.
● The
● What type of thinking is required? ● You are being asked to apply your knowledge of acids to explain how acidosis is treated. ● What key words does the question contain and what do they mean? ● Acidosis—this means that the blood is more acidic ● Buffers—these are used to adjust the pH of a solution ● Saline—a salt solution with the same osmolarity as blood Gather Content ● What do you know about acids and buffers? What other information is related to the question? ● Acids will lower the pH of a solution. ● Buffers can be used to neutralize acids. ● Saline is a salt solution and will not affect pH. Choose Answer ● Given what you now know, what information is most likely to produce the correct answer? ● If your dog has acidosis then the pH of his blood is too low. A buffer will be used to neutralize the acid and raise the pH back to a healthy level. Reflect on Process ● Did your problem-solving process lead you to the correct answer? If not, where did the process break down or lead you astray? How can you revise your approach to produce a more desirable result? ● If you got the correct answer, great job! If you got an incorrect answer, where did the process break down? Did you recall that acids will lower pH?Did you recall that buffers will neutralize a pH? Version 1
43
45) A 46) C
Version 1
44
Clarify Question ● What is the key concept addressed by the question? ● The question asks about factors that affect the rate of a reaction. ● What type of thinking is required? ● You are being asked to apply your knowledge of factors that affect the rate of a reaction. ● What key words does the question contain and what do they mean? ● Rate of reaction—the speed at which chemical bonds are breaking or forming in the reaction Gather Content ● What do you know about factors that affect the rate of a reaction? What other information is related to the question? ● If you want to increase the rate of a reaction, more of the reactants are added to increase the probability that they will interact and form a product.Enzymes catalyze reactions, making them go faster.Temperature can also increase the rate of a reaction. Choose Answer ● Given what you now know, what information is most likely to produce the correct answer? ● C is a product of the reaction, so increasing its concentration would slow down the reaction.Similarly removing an enzyme catalyst or decreasing the temperature would decrease the rate of the reaction. ● Increasing the concentration of the reactants, A and B, would drive the reaction forward and increase the rate. Reflect on Process ● Did your problem-solving process lead you to the correct answer? If not, where did the process break down or lead you astray? How can you revise your approach to produce a more desirable result? ● If Version 1
45
you got the correct answer, great job! If you got an incorrect answer, where did the process break down? Did you think that adding more of the product would increase the rate of the reaction?Did you realize that removing an enzyme catalyst or decreasing the temperature would slow down a reaction?
47) A
Version 1
46
Clarify Question ● What is the key concept addressed by the question? question asks about the formation of covalent bonds.
● The
● What type of thinking is required? ● You are being asked to apply your knowledge of covalent bonds to predict how many bonds are present in N3. ● What key words does the question contain and what do they mean? ● Covalent bonds—two atoms share a pair of electrons to form a covalent bond Gather Content ● What do you know about covalent bonds? What other information is related to the question? ● Two atoms share a pair of electrons to form a covalent bond. ● Atoms can form more than one covalent bond. ● If the two N atoms are sharing 6 electrons, and there are 2 electrons shared in one covalent bond, then there are 6 ÷ 2 = 3 covalent bonds. Choose Answer ● Given what you now know, what information is most likely to produce the correct answer? ● There are 3 covalent bonds between the two N atoms in N3. Reflect on Process ● Did your problem-solving process lead you to the correct answer? If not, where did the process break down or lead you astray? How can you revise your approach to produce a more desirable result? ● If you got the correct answer, great job! If you got an incorrect answer, where did the process break down? Did you realize that a covalent bond Version 1
47
contains 2 shared electrons?Did you realize that two atoms can have more than one covalent bond?
48) D 49) C 50) D 51) A 52) B 53) B
Version 1
48
Clarify Question ● What is the key concept addressed by the question? ● The question asks about calculating the mass of 3 moles of lithium. ● What type of thinking is required? ● You are being asked to apply the atomic mass of a compound to calculate the mass of 3 moles. ● What key words does the question contain and what do they mean? ● Atomic mass—mass (grams) in one mole of an element ● Atomic number—number of protons in an atom ● 6.02 x 1023 atoms/mole—Avagadro's number, and is universal for each atom Gather Content ● What do you know about converting from moles to mass? What other information is related to the question? ● If you have 3 moles of Li you want to multiply that by a factor with the units grams/mole so the moles cancel and you are left with grams. ● Atomic number has the units protons/atom and won't be useful in calculating grams. ● Avagadro’s number has the units atoms/mole and won’t be useful in calculating grams. Choose Answer ● Given what you now know, what information is most likely to produce the correct answer? ● Atomic mass has the units grams/mole. ● 3 moles of Li x 7 grams/mole = 21 grams of Li Reflect on Process ● Did your problem-solving process lead you to the correct answer? If not, where did the process break down or lead you astray? How can Version 1
49
you revise your approach to produce a more desirable result? ● If you got the correct answer, great job! If you got an incorrect answer, where did the process break down? Did you recall the units for atomic mass, atomic number, and Avagadro’s number?Were you able to set up the calculation properly?
54) D
Version 1
50
Clarify Question ● What is the key concept addressed by the question? question asks about pH.
● The
● What type of thinking is required? ● You are being asked to apply your knowledge of pH to neutralize an acid. ● What key words does the question contain and what do they mean? ● Acids have a pH less than 7 ● Bases have a pH greater than 7 Gather Content ● What do you know about acids? What other information is related to the question? ● Acids have a pH less than 7, to neutralize the acid you want to use a base with a pH greater than 7. ● Water is neutral (pH 7) and will not neutralize an acid. Choose Answer ● Given what you now know, what information is most likely to produce the correct answer? ● The only base on the list of answers is baking soda (pH 9). Adding a base with a high pH will neutralize an acid with a low pH. Reflect on Process ● Did your problem-solving process lead you to the correct answer? If not, where did the process break down or lead you astray? How can you revise your approach to produce a more desirable result? ● If you got the correct answer, great job! If you got an incorrect answer, where did the process break down? Did you recall that acids have a low pH? Did you realize that a base with a high pH would neutralize an acid?
Version 1
51
55) A 56) A
Version 1
52
Clarify Question ● What is the key concept addressed by the question? ● The question asks about the link between electronegativity and polar bonds. ● What type of thinking is required? ● You are being asked to analyze the relative electronegativities of two atoms in a bond to predict which would be the most polar. ● What key words does the question contain and what do they mean? ● Electronegativity—attraction of an atom for an electron ● Polar bond—covalent bond in which the electrons are not shared equally Gather Content ● What do you know about polar bonds? What other information is related to the question? ● The greater the difference in electronegativity between two atoms in a covalent bond the more polar the bond will be. ● To calculate the relative polarity of the bonds, simply subtract the electronegativities of each atom in the bond. Choose Answer ● Given what you now know, what information is most likely to produce the correct answer? ● Se-Cl; 3.2-2.6 = 0.6 ● C-Cl; 3.2-2.5 = 0.7 ● B-Cl; 3.2-1.8 = 1.4 ● The largest difference is between B and Cl, so the B-Cl bond will be the most polar with Cl pulling the electrons away from B. Reflect on Process ● Did your problem-solving process lead you to the correct answer? If not, where did the process break down or lead you astray? How can Version 1
53
you revise your approach to produce a more desirable result? ● If you got the correct answer, great job! If you got an incorrect answer, where did the process break down? Did you know what electronegativity and polar bonds were? Did you know to compare the difference in electronegativity between the two atoms to determine the polarity of the bond?
57) B
Version 1
54
Clarify Question ● What is the key concept addressed by the question? question asks about redox reactions.
● The
● What type of thinking is required? ● You are being asked to apply the definition of reactions that lose an electron. ● What key words does the question contain and what do they mean? ● Redox reaction—contains an oxidation and a reduction reaction ● Oxidation—loss of an electron ● Reduction—gain of an electron (the charge of the atom or molecule gaining the electron is reduced) Gather Content ● What do you know about redox reactions? What other information is related to the question? ● In a redox reaction electron(s) are transferred from one atom or molecule to another. This is made up of an oxidation half reaction where the electrons are lost by one reactant, and a reduction half reaction where the electrons are gained by the second reactant. ● In the half reaction H2 → 2H+ + 2e-, H2 is losing 2 electrons. Choose Answer ● Given what you now know, what information is most likely to produce the correct answer? ● Because H2 is losing 2 electrons this is an oxidation reaction. ● This would be balanced by the reduction half reaction, F2 + 2e- → 2F-. ● Finally the 2H+ + 2F-→ 2HF. ● Putting this all together you get the reaction: H2+ F2→ 2HF Version 1
55
Reflect on Process ● Did your problem-solving process lead you to the correct answer? If not, where did the process break down or lead you astray? How can you revise your approach to produce a more desirable result? ● If you got the correct answer, great job! If you got an incorrect answer, where did the process break down? Did you have trouble recognizing that an oxidation half reaction was being shown? Did the mention of a redox reaction confuse the problem for you?
58) C
Version 1
56
Clarify Question ● What is the key concept addressed by the question? ● The question asks about the configuration of electrons in an atom. ● What type of thinking is required? ● You are being asked to apply your knowledge of electron configuration to create one for Argon. ● What key words does the question contain and what do they mean? ● Electron configuration—occupancy of electrons in different shells ● Atomic number—number of protons in an atom (and the number of electrons in a neutral atom) Gather Content ● What do you know about electron configuration? What other information is related to the question? ● The first shell contains 2 electrons in an "s" orbital. A full first shell is represented 1s2 . ● The next shells each contain 8 electrons, 2 electrons in an "s" orbital and 6 electrons in a "p" orbital. A full second shell is represented 2s22p6. ● Argon has an atomic number 18 and Neon is 10. Thus Argon has 8 more electrons than Neon, which adds one more shell. Choose Answer ● Given what you now know, what information is most likely to produce the correct answer? ● Adding a third shell to Neon will add 8 more electrons, 2 electrons in an "s" orbital and 6 electrons in a "p" orbital. ● Following the same numbering as with Neon this would add 2s22p6. ● The complete electron configuration for Argon will be Version 1
57
1s22s22p63s23p6. Reflect on Process ● Did your problem-solving process lead you to the correct answer? If not, where did the process break down or lead you astray? How can you revise your approach to produce a more desirable result? ● If you got the correct answer, great job! If you got an incorrect answer, where did the process break down? Did you have trouble understanding what was being represented in the numbering system of an electron configuration? Did you realize that the 8 electrons in shells 2 and 3 were split between "s" and "p" orbitals.
59) B 60) A
Version 1
58
Clarify Question ● What is the key concept addressed by the question? question asks about properties of water.
● The
● What type of thinking is required? ● You are being asked to apply your understanding of properties of water to explain an observation. ● What key words does the question contain and what do they mean? ● Wicked—this means the water moved upwards, in this case from the floor to the carpet Gather Content ● What do you know about properties of water? What other information is related to the question? ● Water is a polar molecule that forms hydrogen bonds with other water molecules and polar compounds. This allows water to move upwards by wicking. ● Adhesion—water sticking to other molecules through hydrogen bonds. ● Cohesion—water molecules sticking to each other. ● Solubility—substances dissolving in water. Choose Answer ● Given what you now know, what information is most likely to produce the correct answer? ● Both adhesion and cohesion will allow water molecules to bind to polar molecules in the carpet and wick upwards from the floor. Reflect on Process ● Did your problem-solving process lead you to the correct answer? If not, where did the process break down or lead you astray? How can you revise your approach to produce a more desirable result? ● If Version 1
59
you got the correct answer, great job! If you got an incorrect answer, where did the process break down? Did you recall the definitions of adhesion, cohesion, and solubility?
61) A
Version 1
60
Clarify Question ● What is the key concept addressed by the question? question asks about the formation of ions.
● The
● What type of thinking is required? ● You are being asked to apply your knowledge of electron energy levels to the formation of ions. ● What key words does the question contain and what do they mean? ● Outer energy level—the outermost level of electrons based on the total number of electrons present in an atom Gather Content ● What do you know about ions? What other information is related to the question? ● To be most stable, an atom may gain or lose electrons to end up with a full outer shell of electrons as an atom is most stable when it’s outer energy level is full. ● This new atom has one extra electron in its outer shell. If it gains an electron it will still have a partially full shell (2 electrons), if it loses an electron it will have a full outer shell (8 electrons). Choose Answer ● Given what you now know, what information is most likely to produce the correct answer? ● Because an electron has a -1 charge, losing the electron to achieve a full outer shell will give the atom a +1 charge, forming a positive ion. Reflect on Process ● Did your problem-solving process lead you to the correct answer? If not, where did the process break down or lead you astray? How can you revise your approach to produce a more desirable result? ● If you got the correct answer, great job! If you got an incorrect answer, where did the process break down? Did you realize that the atom would Version 1
61
be most stable if it lost an electron? Did you recognize that losing an electron would lead to the formation of a positively charged ion?
62) A
Version 1
62
Clarify Question ● What is the key concept addressed by the question? ● The question asks about the properties of atoms based on their location in the periodic table. ● What type of thinking is required? ● You are being asked to apply your knowledge of the properties of atoms based on their location in the periodic table. ● What key words does the question contain and what do they mean? ● Periodic table—organizational chart of the elements that is arraigned based on their chemical properties Gather Content ● What do you know about the properties of atoms based on their location in the periodic table? What other information is related to the question? ● When one atom like oxygen has a higher attraction for electrons (electronegativity) than another atom like hydrogen sharing the electrons in a covalent bond, the electrons spend more time closer to the atom with the higher electronegativity. This causes a partial positive charge on the atom with a lower electronegativity and a partial negative charge on the atom with a higher electronegativity.This causes the covalent bond to have polarity. ● In an O-H bond, oxygen has a partial negative charge and hydrogen a partial positive charge.S is right below O on the periodic table, so it will have a similar electronegativity. Choose Answer ● Given what you now know, what information is most likely to produce the correct answer? ● A hydrogen bond is not a covalent bond between two ions, but relies on the attraction between two polar Version 1
63
covalent bonds.Ionic interactions occur when two ions are attracted to each other. A covalent bond in which the electrons are shared equally is a non-polar covalent bond. ● An S-H bond will be similar to an O-H bond and will be a polar covalent bond. Reflect on Process ● Did your problem-solving process lead you to the correct answer? If not, where did the process break down or lead you astray? How can you revise your approach to produce a more desirable result? ● If you got the correct answer, great job! If you got an incorrect answer, where did the process break down? Did you recall that S would have similar properties to O because of its location on the periodic table?Did you understand that and O-H bond would be a polar covalent bond?
63) B 64) C
Version 1
64
Clarify Question ● What is the key concept addressed by the question? question asks about evaporation of water.
● The
● What type of thinking is required? ● You are being asked to apply your knowledge of evaporation of water. ● What key words does the question contain and what do they mean? ● Heat of vaporization—amount of energy required to change 1 g of a substance from a liquid to a gas Gather Content ● What do you know about evaporation of water? What other information is related to the question? ● Water molecules are attracted to each other by hydrogen bonds, with the partial negative charge on one water molecule being attracted to the partial positive charge on the hydrogen in a neighboring water molecule.To overcome this attraction heat is needed to break these hydrogen bonds and allow a water molecule to move from being a liquid to being a gas. Choose Answer ● Given what you now know, what information is most likely to produce the correct answer? ● Because of hydrogen bonding between water molecules, more heat is required to convert the water into a gas.This means that more heat is released from your body as you sweat. ● Based on this logic, more heat is input into water and is obtained from your body leading to cooling. Reflect on Process ● Did your problem-solving process lead you to the correct answer? If not, where did the process break down or lead you astray? How can Version 1
65
you revise your approach to produce a more desirable result? ● If you got the correct answer, great job! If you got an incorrect answer, where did the process break down? Did you think that when water goes from a liquid to a gas it would release heat?Did you think that heat would be delivered to your body as your sweat evaporated from your skin?
65) A
Version 1
66
Clarify Question ● What is the key concept addressed by the question? ● The question asks about how salt would prevent ice from forming. ● What type of thinking is required? ● You are being asked to apply your knowledge of the properties of water to explain how salt melts ice. ● What key words does the question contain and what do they mean? ● Freezing point—temperature below which water molecules hydrogen bond to each other in a solid lattice commonly known as ice Gather Content ● What do you know about ice? What other information is related to the question? ● Ice forms because the water molecules are in a 3dimensional lattice with each water molecule hydrogen bonded to 4 other water molecules. ● When salt dissolves in water the NaCl dissociates into Na+ and Clions.Each of these ions interact with water molecules. The Na+ will be attracted to the partial negative charge on O in water and the Cl- ions will be attracted to the partial positive charge on H in water. Choose Answer ● Given what you now know, what information is most likely to produce the correct answer? ● Because the Na+ and Cl- ions interact with water molecules through the partial charges on water, these interfere with the ability of water molecules to hydrogen bond with each other. ● Because water molecules can no longer form hydrogen bonds with each other, they cannot form a lattice and ice does not form. Reflect on Process Version 1
67
● Did your problem-solving process lead you to the correct answer? If not, where did the process break down or lead you astray? How can you revise your approach to produce a more desirable result? ● If you got the correct answer, great job! If you got an incorrect answer, where did the process break down? Did you realize that ice forms because of hydrogen bonding between water molecules? Did you understand that salt ions interfere with this hydrogen bonding, reducing water’s ability to form ice?
66) B
Version 1
68
Clarify Question ● What is the key concept addressed by the question? ● The question asks about the relationship between pH and hydrogen ion concentration. ● What type of thinking is required? ● You are being asked to apply your knowledge of pH to predict what would happen to water when the hydrogen ion concentration is increased. ● What key words does the question contain and what do they mean? ● pH—the –log[H+] Gather Content ● What do you know about pH? What other information is related to the question? ● Because pH is the –log[H+] as the H+ concentration goes up the pH goes down. ● Because it is a log scale, each 10-fold change in H+ concentration is a change in pH of 1. ● In this case a 100-fold increase in H+ concentration would lead to a decrease in pH of 2. Choose Answer ● Given what you now know, what information is most likely to produce the correct answer? ● Pure water has a pH of 7 and the chemical dropped the pH by 2, so the new pH will be about 5. Reflect on Process ● Did your problem-solving process lead you to the correct answer? If not, where did the process break down or lead you astray? How can you revise your approach to produce a more desirable result? ● If you got the correct answer, great job! If you got an incorrect answer, where did the process break down? Did you have trouble using the Version 1
69
equation of pH? Did you recall that this was a negative log relationship between pH and H+ concentration, so the pH decreased? Did you recall that the pH of water is 7?
67) A
Version 1
70
Clarify Question ● What is the key concept addressed by the question? question asks about electronegativity.
● The
● What type of thinking is required? ● You are being asked to apply your knowledge of electronegativity to predict the effect on the partial charge of an atom. ● What key words does the question contain and what do they mean? ● Electronegativity—attraction of an atom for an electron ● Partial charges—charge of an atom that is the result of unequal sharing of electrons ● Electronegativity—attraction of an atom for an electron Gather Content ● What do you know about electronegativity? What other information is related to the question? ● In a polar covalent bond, the atom with the stronger attraction will have a partial negative charge and the atom with the weaker attraction will have a partial positive charge. ● The electronegativity of N is 3.0 and that of H is 2.1. ● N has a stronger attraction for the electrons in its covalent bond with H. Choose Answer ● Given what you now know, what information is most likely to produce the correct answer? ● Based on the same logic, each H atom will have a partial positive charge as the electrons are pulled towards N. Reflect on Process ● Did your problem-solving process lead you to the correct answer? If not, where did the process break down or lead you astray? How can Version 1
71
you revise your approach to produce a more desirable result? ● If you got the correct answer, great job! If you got an incorrect answer, where did the process break down? Did you recognize that the higher electronegativity on N meant it had more attraction for electrons than H?Did you realize that a stronger pull for electrons gave N a partial negative charge, while this led to a partial positive charge on each H?
68) D
Version 1
72
Clarify Question ● What is the key concept addressed by the question? question asks about reduction and oxidations.
● The
● What type of thinking is required? ● You are being asked to apply your knowledge to the formation of ions through reduction and oxidation. ● What key words does the question contain and what do they mean? ● Outer shell of electrons—the outermost level of electrons based on the total number of electrons present in an atom, atoms are most stable with a full outer shell of electrons Gather Content ● What do you know about oxidations and reductions? What other information is related to the question? ● If an atom loses an electron it becomes oxidized and forms a positive charge. If an atom gains an electron it becomes reduced and forms a negative charge. ● Mg has 2 extra electrons in its outer shell, so it will be most stable losing two electrons. ● Cl has 7 electrons in its outer shell, so it will be most stable gaining one electron. Choose Answer ● Given what you now know, what information is most likely to produce the correct answer? ● Mg will lose two electrons, forming Mg2+ and be oxidized in the process. ● Cl will gain one electron, forming Cl- and be reduced in the process. Reflect on Process ● Did your problem-solving process lead you to the correct answer? Version 1
73
If not, where did the process break down or lead you astray? How can you revise your approach to produce a more desirable result? ● If you got the correct answer, great job! If you got an incorrect answer, where did the process break down? Did you realize that Mg would lose electrons and Cl would gain them? Did you remember that oxidations lose electrons and reductions gain electrons?
69) B
Version 1
74
Clarify Question ● What is the key concept addressed by the question? question asks about buffers.
● The
● What type of thinking is required? ● You are being asked to apply your knowledge of buffers to predict what would happen if a base was removed. ● What key words does the question contain and what do they mean? ● Carbonic acid—acid of the buffer ● Bicarbonate—base of the buffer Gather Content ● What do you know about buffers? What other information is related to the question? ● Buffers contain an acid and its complementary base. Acids lower the pH of a solution and bases raise the pH. ● If the bicarbonate was removed from the solution all that would be left is the carbonic acid. Choose Answer ● Given what you now know, what information is most likely to produce the correct answer? ● Acids lower the pH of a solution, so the pH of blood would drop. Reflect on Process ● Did your problem-solving process lead you to the correct answer? If not, where did the process break down or lead you astray? How can you revise your approach to produce a more desirable result? ● If you got the correct answer, great job! If you got an incorrect answer, where did the process break down? Did you realize that bicarbonate was the base in the problem? One clue was that carbonic acid had to be the Version 1
75
acid, based on its name.Did you recall that acids lower the pH and bases raise the pH?
70) B 71) A
Version 1
76
Clarify Question ● What is the key concept addressed by the question? ● The question asks about differences between covalent and ionic bonds. ● What type of thinking is required? ● You are being asked to apply your knowledge of differences between covalent and ionic bonds. ● What key words does the question contain and what do they mean? ● Cl2—molecule of 2 chlorine atoms ● KCl—molecule of 1 potassium atom and 1 chlorine atom Gather Content ● What do you know about differences between covalent and ionic bonds? What other information is related to the question? ● In an ionic bond, atoms gain or lose electrons to form a stable outer shell forming either negative or positive ions respectively. In a covalent bond, two atoms share a pair of electrons filling their outer shells. ● The name carbon dioxide indicates that there are one carbon atom and two oxygen atoms in this molecule. Choose Answer ● Given what you now know, what information is most likely to produce the correct answer? ● Cl has 7 electrons in its outer shell.If it gains one electron it will form a Cl- ion.K has 1 electron in its outer shell, so if it loses one electron it will form a K+ ion.The Cl- and K+ ions will be attracted to each other through an ionic bond. ● In Cl, 6 of the electrons in the outer shell form pairs, leaving one unpaired electron.If two Cl atoms share their unpaired electrons, that gives each atom a total of 8 electrons in their outer shell and forms a covalent bond. Reflect on Process Version 1
77
● Did your problem-solving process lead you to the correct answer? If not, where did the process break down or lead you astray? How can you revise your approach to produce a more desirable result? ● If you got the correct answer, great job! If you got an incorrect answer, where did the process break down? Did you confuse covalent and ionic bonds?Did you understand that covalent bonds were formed by sharing electrons while ionic bonds were formed when atoms gained or lost electrons?
72) C
Version 1
78
Clarify Question ● What is the key concept addressed by the question? question asks about the formation of covalent bonds.
● The
● What type of thinking is required? ● You are being asked to apply your knowledge of the structure of atoms to predict how many covalent bonds carbon dioxide could form. ● What key words does the question contain and what do they mean? ● Valence electrons—electrons in the outer shell of an atom Gather Content ● What do you know about covalent bonds? What other information is related to the question? ● Typically an atom is most stable if it has a full outer shell.This can be accomplished by gaining, losing, or sharing electrons. When two electrons are shared between two atoms this forms a covalent bond. ● The name carbon dioxide indicates that there are one carbon atom and two oxygen atoms in this molecule. ● Carbon has 4 valence electrons, so it needs to share 4 more electrons to get a full outer shell. This means it can form 4 covalent bonds. ● Oxygen has 6 valence electrons, so it needs to share 2 more electrons to get a full outer shell. This means each oxygen atom can form 2 covalent bonds. Choose Answer ● Given what you now know, what information is most likely to produce the correct answer? ● The carbon atom can form a double bond with each of the oxygen atoms. That will give the carbon atom 4 bonds and each oxygen atom 2 bonds needed to fill their outer shells. Version 1
79
The structure O = C = O would thus contain two double covalent bonds. Reflect on Process ● Did your problem-solving process lead you to the correct answer? If not, where did the process break down or lead you astray? How can you revise your approach to produce a more desirable result? ● If you got the correct answer, great job! If you got an incorrect answer, where did the process break down? Did you have trouble figuring out how many bonds carbon and oxygen each needed to form to be stable? Were you confused on whether all of the bonds were between C and O or did you try to make some O-O bonds?
73) A
Version 1
80
Clarify Question ● What is the key concept addressed by the question? ● The question asks about the properties of liquid water and ice. ● What type of thinking is required? ● You are being asked to apply your knowledge of the vaporization of water. ● What key words does the question contain and what do they mean? ● Nonpolar—matchingaffinity for shared electrons within a molecule Gather Content ● What do you know about the vaporization of water? What other information is related to the question? ● Hydrogen bonds between polar water molecules cause attractions that make it harder for a single water molecule to move from liquid to a gas.Other molecules like carbon dioxide (CO2) and oxygen (O2) are about the same size as water, but cannot form hydrogen bonds and these molecules are gasses at room temperature. Choose Answer ● Given what you now know, what information is most likely to produce the correct answer? ● If water was nonpolar, the molecules would not be attracted to each other and water would be a gas instead of a liquid at room temperature. Reflect on Process ● Did your problem-solving process lead you to the correct answer? If not, where did the process break down or lead you astray? How can you revise your approach to produce a more desirable result? ● If you got the correct answer, great job! If you got an incorrect answer, where did the process break down? Did you recall that hydrogen bonds Version 1
81
cause water molecules to be attracted to each other and form a liquid?Did you think that a lack of hydrogen bonds would make water become a solid instead of a gas?
74) B
Version 1
82
Clarify Question ● What is the key concept addressed by the question? question asks about nonpolar amino acids.
● The
● What type of thinking is required? ● You are being asked to apply your knowledge of polarity to predict how this would affect the structure of a protein. ● What key words does the question contain and what do they mean? ● Polar—molecules in which electrons are not shared equally between atoms, leading to partial charges on the atoms ● Nonpolar—molecules in which electrons are shared equally between atoms, leading to no partial charges Gather Content ● What do you know about nonpolar molecules? What other information is related to the question? ● Nonpolar molecules do not interact well with water, which is a polar molecule. These nonpolar molecules actually interfere with the formation of hydrogen bonds between water molecules. ● The inside of an animal cell contains cytoplasm which is made up of a lot of water. Thus the inside of the cell is a polar environment. ● Nonpolar amino acids in the inside of a cell are repelled by the polar water molecules. Choose Answer ● Given what you now know, what information is most likely to produce the correct answer? ● The repulsion of non-polar amino acids by water will cause them to cluster in the center of the protein where they are shielded from water. This is why most proteins are ball shaped, with the non-polar or hydrophobic amino acids in the center of Version 1
83
the ball. Reflect on Process ● Did your problem-solving process lead you to the correct answer? If not, where did the process break down or lead you astray? How can you revise your approach to produce a more desirable result? ● If you got the correct answer, great job! If you got an incorrect answer, where did the process break down? Did you understand the difference between polar and non-polar amino acids? Did you realize that the inside of the cell would be a polar environment? Did you know that non-polar groups will cluster near each other to be shielded from water?
75) A
Version 1
84
Clarify Question ● What is the key concept addressed by the question? question asks about the ability of water to absorb heat.
● The
● What type of thinking is required? ● You are being asked to apply your understanding of the properties of water to explain the effect of adding antifreeze. ● What key words does the question contain and what do they mean? ● Cooling efficiency—measure of how well a solution can absorb heat Gather Content ● What do you know about the properties of water? What other information is related to the question? ● Water has a very high heat capacity. This means that it can absorb a lot of heat. ● Adding ethylene glycol decreases the cooling efficiency of the water in the radiator. This is done to lower the freezing point of water, which keeps the radiator from freezing solid in the winter. Choose Answer ● Given what you now know, what information is most likely to produce the correct answer? ● Adding ethylene glycol to the radiator decreases the amount of heat that the water can absorb because it has a lower heat capacity than water. Reflect on Process ● Did your problem-solving process lead you to the correct answer? If not, where did the process break down or lead you astray? How can you revise your approach to produce a more desirable result? ● If you got the correct answer, great job! If you got an incorrect answer, where did the process break down? Did you realize that the property of Version 1
85
water relevant to cooling efficiency was its heat capacity? Did you recognize that the question was not asking about changes in the freezing point?
76) C
Version 1
86
Clarify Question ● What is the key concept addressed by the question? question asks about forming covalent bonds.
● The
● What type of thinking is required? ● You are being asked to apply your knowledge of the structure of an atom to predict how many bonds it can form. ● What key words does the question contain and what do they mean? ● Saturated fat—fats with a backbone of C-C bonds with as many C-H bonds as possible Gather Content ● What do you know about covalent bonds? What other information is related to the question? ● Carbon has four valence electrons, and needs to share an additional four electrons to have a full outer shell. This means it needs to form four covalent bonds. ● In a saturated fat there is a backbone of C atoms attached by covalent bonds giving a structure like: -C-C-C● These covalent bonds provide two additional electrons, leaving a need for two more for carbon to have a full outer shell. Choose Answer ● Given what you now know, what information is most likely to produce the correct answer? ● To obtain the remaining two electrons, each C atom bonded to two other C atoms forms covalent bonds with two H atoms. Reflect on Process ● Did your problem-solving process lead you to the correct answer? If not, where did the process break down or lead you astray? How can you revise your approach to produce a more desirable result? ● If Version 1
87
you got the correct answer, great job! If you got an incorrect answer, where did the process break down? Did you understand what was meant by a saturated fat? Could you figure out how many total covalent bonds each C atom would form?
77) E
Version 1
88
CHAPTER 3 MULTIPLE CHOICE - Choose the one alternative that best completes the statement or answers the question. 1) Organic molecules have a carbon backbone and __________ such as -OH and -NH 2 that can form hydrogen bonds. A) R groups B) extensiongroups C) functionalgroups D) chemical groups
2) A modified form of cellulose found in the exoskeletons of insects and crustaceans is known as __________. A) chitin B) starch C) amylose D) glycogen
3)
These biological compounds are nonpolar and insoluble in water __________. A) carbohydrates B) lipids C) proteins D) nucleic acids
4)
A DNA molecule containing 32% thymine (T) would contain how much cytosine (C)? A) 18% B) 32% C) 36% D) 64%
Version 1
1
5) The covalent bonds connecting monomer units in sugars can be formed by the removal of a water molecule. This reaction is referred to as __________. A) a hydrolysis reaction B) an oxidation-reductionreaction C) a dehydration reaction D) a condensation reaction
6) High temperatures can cause proteins in a cell to denature. This can be reversed by heat shock proteins that function as __________. A) chaperones B) catalysts C) ribosomes D) proteasomes E) polymerases
7) Proteins digested by trypsin in the stomachare broken down to amino acidsin __________. A) anabolic reactions B) hydrolysis reactions C) denaturation reactions D) dehydration reactions
8) Relatively small organic molecules with a central carbon atom which is bonded to a carboxyl group, an amino group, a carbon containing group, and a hydrogen atom are called __________.
Version 1
2
A) amino acids B) fatty acids C) enzymes D) peptides E) nucleotides
9) Macromolecules that are used by most organisms to store hereditary information are called __________. A) transfer RNA molecules B) messenger RNA molecules C) proteins D) ribosomal RNAmolecules E) DNA molecules
10)
DNA and RNAcontain functional units known as __________. A) peptides B) enzymes C) amino acids D) nucleotides E) fatty acids
11)
Carbohydrates are polymers formed of structural units called __________. A) amino acids B) fatty acids C) nucleotides D) glycerols E) monosaccharides
Version 1
3
12)
Proteins are polymers formed of structural units called __________. A) fatty acids B) amino acids C) nucleotides D) phosphate groups E) monosaccharides
13)
Lipids are the only class of macromolecules that contain __________. A) amino acids B) nucleotides C) fatty acids D) phosphate groups E) monosaccharides
14) A double-stranded DNA strand contains 26 pyrimidines. How many purines will the copied strand contain? A) 0 B) 13 C) 26 D) 52
15) The simplest and the most common monosaccharide is a six-carbon sugar called __________. A) galactose B) lactose C) cellulose D) glucose E) sucrose
Version 1
4
16)
Humans are unable to get metabolic energy from cellulose because __________. A) cellulose contains very little chemical energy B) cellulose is not part of a normal diet C) cellulose digesting enzymes are absent from the human gut D) cellulose does not taste good E) cellulose is present in large quantities in the gut
17) Chitin, a modified form of cellulose, is not only cross-linked with proteins but its glucose units are modified with atoms of __________. A) nitrogen B) hydrogen C) oxygen D) sulfur E) phosphorous
18)
The biological macromolecule that is least soluble in water is a/an __________. A) protein B) nucleic acid C) enzyme D) lipid E) carbohydrate
19)
Common lipids for energy storage are __________.
Version 1
5
A) phospholipids B) steroids C) triglycerides D) cholesterols E) waxes
20)
Three fatty acids bonded together with a glycerol are found in a/an __________. A) alcohol B) enzyme C) phospholipid D) chlorophyll pigment E) triglyceride
21)
Biological membranes contain bilayers containing mostly ______. A) phospholipids B) oils C) prostaglandins D) triglycerides E) cholesterol
22) This type of lipid is found in pigments such aschlorophyll, in the visual pigment retinal, and in rubber. A) Phospholipid B) Terpene C) Steroid D) Prostaglandin E) Wax
Version 1
6
23)
Amino acids are linked together by peptide bonds in a __________. A) phospholipid B) nucleic acid C) polysaccharide D) polypeptide E) starch
24)
The specific amino acid sequence of a protein is its __________. A) primarystructure B) secondarystructure C) tertiarystructure D) quaternarystructure
25)
What is the difference in the bonding between alpha-helix and beta sheets?
A) Hydrogen bondingbetween the amides only occurs in one of these two types of secondarystructures. B) Hydrogen bondingoccurs between the amides of adjacent chains in the sheetsinstead ofbetween the amides in different parts of the helices. C) Sulfur bridges allow the spring like turns in alpha helices and van der Waals dispersion allows the folds inbeta sheets. D) Hydrogen bonding always occurs in the alpha helices but only occurs inparallel beta sheets.
26) Denaturation disrupts the secondary and tertiary structure of proteins. However, denaturation is not strong enough to disrupt what interaction that is relevant to protein structure?
Version 1
7
A) Hydrogen bonds B) Ionic bonds C) van der Waals attraction D) Hydrophobic exclusion E) Peptide bonds
27)
The information storage molecules of cells are called __________. A) proteins B) phospholipids C) enzymes D) hormones E) nucleic acids
28)
The nitrogenous base that is only found in DNA is and not in RNA is __________. A) adenine B) cytosine C) guanine D) thymine E) uracil
29)
The nitrogenous base that is only found in RNA and not DNA is __________. A) adenine B) cytosine C) guanine D) thymine E) uracil
Version 1
8
30) Fructose and galactose both have six carbon atoms, twelve hydrogen atoms and six oxygen atoms, just like glucose. A friend of yours says: "Since those sugars have the same number of atoms, they should have the same name." Which of the following choices would be the correct way to answer? A) "While they do have the same number and type of atoms, they are all polysaccharides and have slightly different functions within organisms and therefore have different names." B) "While they do have the same number and type of atoms, they are named differently because they have different secondary structure." C) "While they do have the same number and type of atoms, they are named differently because they cannot be converted into glucose." D) "While they do have the same number and type of atoms, they are named differently because one is found in RNA and the other in DNA." E) "While they do have the same number and type of atoms, they are named differently because they are isomers of the monosaccharide, glucose."
31)
A nucleic acid contains uracil, allowing you to conclude that __________. A) this nucleic acid is DNA B) this nucleic acid is RNA C) this nucleic acid could be either DNA or RNA
32)
A nucleic acid contains thymidine, allowing you to conclude that __________. A) this nucleic acid is DNA B) this nucleic acid is RNA C) this nucleic acid could be either DNA or RNA
33)
Proteins are created with __________.
Version 1
9
A) phosphodiester bonds B) peptide bonds C) triglycerides D) polysaccharides
34)
Phospholipids are made up of __________. A) a glycerol and three fatty acids B) four fused carbon rings C) a phosphate, two fatty acids and a glycerol D) five-carbon rings with two nonpolar tails
35)
Carbohydrates are composed of __________. A) amino acids B) fatty acids and glycerol C) nucleotides D) monosaccharides
36)
Enzymes function to __________. A) react with other enzymes to form a product B) catalyze chemical reactions C) transport molecules and ions across membranes D) regulate messages between cells
37)
The types of bonds linking nucleotidesin nucleic acids are __________.
Version 1
10
A) phosphodiester bonds B) peptide bonds C) ionic bonds D) nonpolar covalent bonds
38)
Which of the following reactions is a dehydration reaction? A) A + H 2O → B + C B) A + B → C + H 2O C) A + B + H 2O → C D) A → B + C+ H 2O
39)
Which of the following reactions is a hydrolysis reaction? A) A + H 2O → B + C B) A + B → C + H 2O C) A + B + H 2O → C D) A → B + C+ H 2O
40)
The DNA sequence capable of pairing with 5' CGATTAGT 3'is __________. A) 3' GCTAATCA 5' B) 3' CGATTAGT 5' C) 5' GCTAATCA 3' D) 5' CGATTAGT 3'
41) You recently identified a novel protein that contains several membrane-spanning domains. You would predict that the amino acids embedded in the membrane would have ____ side groups.
Version 1
11
A) nonpolar B) polaruncharged C) positivelycharged D) negativelycharged
42) L-lysine is an essential amino acid and must be supplied in the diet. By comparison, the stereoisomer D-lysine is not biologically active. Why can your body only utilize one form? A) Since the L form and D form are enantiomers, they will bind together and inhibit utilization of the D form. B) Since the L form and D form are chiral molecules, they will bind together and inhibit utilization of the D form. C) Antibodies recognize the D form and destroy it before your body can use it for nutritional purposes. D) Enzymes can only recognize a single, specific stereoisomer.
43) People who are lactose intolerant can often consume some products made from milk such as cheese and yogurt because microorganisms in their digestive system consume the lactose. By comparison, people with a true milk allergy, which involves an immune response to milk protein, cannot consume milk or products made from milk. Given this information, what can you conclude about milk? A) The protein that causes a true milk allergy is lactose. B) All people who are lactose intolerant are also allergic to milk. C) Yogurt contains live and active cultures of Lactobacillus acidophilus bacteria and therefore prevents an allergic response in people with a true milk allergy. D) Lactose is a carbohydrate, not a protein and is therefore not the cause of true milk allergies.
44) Egg whites consist primarily of water and the protein albumin. When you fry an egg, why does the egg white turn from clear to white?
Version 1
12
A) The protein becomes dissociated. B) The protein acquires tertiary structure. C) The protein becomes denatured. D) The protein becomes dehydrated.
45) Margarine is made by hydrogenating vegetable oils so that they take on the consistency of butter. Propose achemical basis for this change from a liquid to a solid. A) Hydrogenation offats changes themfrom unsaturated to saturated, which are more solid. B) Hydrogenation offats changes themfrom saturated to unsaturated, which are more solid. C) Trans-fatty acids are changed from unsaturated to saturated, allowing them to solidify. D) Trans-fatty acid are changed from saturated to unsaturated, allowing them to solidify.
46) Vitamin D can be synthesized by the body from a derivative of cholesterol. Given this information, predict vitamin D's solubility. A) Vitamin D iswater soluble. B) Vitamin D is fat soluble. C) Vitamin D is soluble in both water and fat. D) Vitamin D is not soluble in either water or fat.
47) You arrive late to a biological seminar. However, just as you enter the room, you hear the speaker referring to the "amino end" and the "carboxyl end" of a macromolecule. Immediately, you know that they are talking about a __________. A) carbohydrate B) protein C) DNA D) lipid
Version 1
13
48) You arrive late to a biological seminar. However, just as you enter the room, you hear the speaker referring to the "five-prime end" and the "three-prime end" of a macromolecule. Immediately, you know that they are talking about a __________. A) carbohydrate B) protein C) DNA D) lipid
49)
What type of monomer does ATP represent? A) Monosaccharide B) Amino acid C) Nucleotide D) Fatty acid E) Glycerol
50) The digestive enzyme pepsin works in the acidic environment of the stomach to hydrolyze peptide bonds in ______. A) carbohydrate B) protein C) DNA D) lipid
51) You have recently identified a novel protein and obtained its sequence. How could you use this sequence to predict the function of the protein? A) By determining the secondary structure of the protein B) By using the sequence to make antibodies against the protein C) By looking for conserved domains D) By determining the DNA sequence that could encode the protein
Version 1
14
52) At the pH of most bodily fluids, which functional group will accept a proton ?
functional group will lose a proton and which
A) Sulfhydryl; carbonyl B) Hydroxyl; amino C) Amino; carboxyl D) Carboxyl; amino E) Carbonyl; hydroxyl
53) A bobcat is feeding on a white-tailed deer. It first eats the liver, which is rich in stored sugars.In what form would sugar be stored in the deer’s liver? A) Glucose B) Maltose C) Glycogen D) Amylose E) Cellulose
54) Seth purchased five pounds of live crayfish and realized that the outer shells were made of a tough carbohydrate similar to that used to make the carapace of a cockroach. What do you think the outer shells of the crawfish are made of? A) Fibrin B) Cellulose C) Glycogen D) Chitin E) Amylopectin
55) Yogurt contains galactose and glucose, which are generated when bacterial enzymes act on milk sugars. What is the main source of the galactose in yogurt?
Version 1
15
A) Maltose B) Sucrose C) Lactose D) Fructose E) Glycogen
56) Luke sprained his ankle, tearing some of the collagen protein that forms his ligaments. Some types of collagen consist of three polypeptide chains twisted together to form a rope-like strand. What level ofprotein structure does this rope-like strand represent? A) Primary B) Secondary C) Tertiary D) Quaternary
57)
In a phospholipid, the phosphate would be found in which part of a membrane? A) On the surface of the membrane, interacting with water. B) In the bilayer, interacting with water. C) On the surface of the membrane, shielded fromwater. D) In the bilayer, shielded fromwater.
58)
Which amino acidis least soluble in water? A) Isoleucine B) Alanine C) Glutamine D) Glycine
59)
Which type of RNA is used as a template for translation of proteins?
Version 1
16
A) Messenger RNA B) Ribosomal RNA C) Transfer RNA D) Micro RNA
60)
Which amino acid is mostsoluble in water? A) Serine B) Valine C) Glycine D) Leucine
61) The formula for glucose is C 6H 12O 6. What is the formula for a polymer made by linking ten glucose molecules together by dehydration synthesis?
A) C 60H 120O 60 B) C 60H 102O 51 C) C 60H 100O 50 D) C 60H 111O 51
62) The myoglobin protein, which carries oxygen in muscle cells, has only the first three levels of protein structure. What can you infer about myoglobin? A) Myoglobin lacks hydrogen bonds. B) Myoglobin is made of nucleic acids. C) Myoglobin is not helical or pleated. D) Myoglobin is made of only one polypeptide chain.
63)
A phosphorus shortage in soil would make it difficult for plants to produce __________.
Version 1
17
A) cellulose B) amylose C) starch D) DNA
64) The nucleotide-containing molecule commonly used to carry electrons in the cell is __________. A) ADP B) ATP C) NAD + D) pyruvic acid
65) Benedict's reagent is a chemical that is used as a test for the presence of a free aldehyde functional group on glucose. The aldehyde is used when glucose and fructose bind together to formsucrose, and the Benedict's test is negative. Starch contains a great deal of glucose but gives a negative Benedict's test because __________. A) all of the aldehyde groups on the starch are oxidized and cannot react with the Benedict's reagent B) starch is not soluble in water and cannot react with Benedict's reagent C) glucose in starch has lost a carbon atom and cannot react with Benedict's reagent D) only the glucose at the end of a chain of starch will have an exposedaldehyde functional group to react with Benedict's reagent
66) What characteristicwould the R groups of amino acids have if they arelocated within the interior of biological membranes? A) The R groups wouldbe hydrophilic. B) The R groups wouldhydrophobic. C) The R groups wouldbe polar. D) The R groups wouldbe able to form ionic bonds.
Version 1
18
67) Sudan IV, a dye that binds to nonpolar molecules,is used to detect the presence of macromolecules in food. Sudan IV would test positive in the presence of what macromolecule? A) Starch B) DNA C) Chitin D) Hemoglobin E) Fats
68) How many water molecules are produced during the formation of a triglyceride from fatty acids and glycerol? A) 0 B) 1 C) 2 D) 3 E) 4
69) Acyclovir is an antiviral chemical that resembles a nucleotide but lacks a 3' OH groupand cannot form phosphodiester bonds. Why is acyclovir useful in treating some viral infections? A) Acyclovir stops viral protein synthesis. B) Acyclovir halts viral DNA replication. C) Acyclovir blocksthe assembly of viral cell capsules. D) Acyclovirhydrolyzes viral polysaccharides.
70) α-Glucose is present in potatoes and β-glucose is present in wood. Both are glucose molecules but one can be used by humans to synthesize ATP and one cannot. Based on this information, what is the best explanation on why humans can only digest one of the glucose types?
Version 1
19
A) α-Glucose is C 6H 12O 6and β-glucose is C 5H 12O 6. B) α-Glucose and β-glucose are stereoisomers with OH groups oriented differently. C) Only α-glucose can form polysaccharides. D) α-Glucose is a chainand β-glucose is a ring.
71) You label cells with a radioactive isotope of sulfur ( be radioactively labeled in these cells are __________.
35
S). The only molecules that would
A) lipids B) proteins C) carbohydrates D) nucleic acids
72)
A polymer with the sequence 5' ACGTACGAATAG 3' is a __________. A) protein B) piece of RNA C) piece of DNA D) polysaccharide
73)
The function of chaperone proteins is to _____. A) help proteins fold to their correct conformation B) facilitate the formation of peptide bonds between amino acids C) hold together multiple polypeptide chains in a quaternary structure D) link together glucose monomers to form cellulose molecules
Version 1
20
Answer Key Test name: Chap 03_3e 1) C 2) A 3) B 4) A
Version 1
21
Clarify Question ● What is the key concept addressed by the question? question asks about base pairing in DNA.
● The
● What type of thinking is required? ● You are being asked to analyze the percentage of thymine in a DNA molecule and calculate the amount of cytosine. ● What key words does the question contain and what do they mean? ● 32% thymine—32% of the total nucleotides in the DNA molecule are thymine Gather Content ● What do you know about base pairing? What other information is related to the question? ● DNA is double stranded and between the strands a thymine pairs with adenine and cytosine pairs with guanine through hydrogen bonds. ● If 32% of the bases are thymine then there will be an equal amount of adenine (32%). The sum of thymine and adenine is thus 64%. ● The rest of the nucleotides must be cytosine and guanine, 100% 64% = 36%. Choose Answer ● Given what you now know, what information is most likely to produce the correct answer? ● If cytosine and guanine add up to 36% of the total nucleotides, and there are equal amounts of both nucleotides, then cytosine and guanine each comprise 18% of the total nucleotides in the DNA molecule. Reflect on Process ● Did your problem-solving process lead you to the correct answer? If not, where did the process break down or lead you astray? How can Version 1
22
you revise your approach to produce a more desirable result? ● This question asked you to analyze the composition of a DNA molecule to predict the amount of cytosine. If you got the correct answer, great job! If you got an incorrect answer, where did the process break down? Did you recall that thymine and adenine pair and will be present in equal amounts? Did you recall that cytosine and guanine also pair and will be present in equal amounts, accounting for the remaining percentage of nucleotides?Did you realize that the total percentage of nucleotides add up to 100%?
5) C 6) A 7) B 8) A 9) E 10) D 11) E 12) B 13) C 14) C
Version 1
23
Clarify Question ● What is the key concept addressed by the question? ● The question asks about base pairing between pyrimidines and purines. ● What type of thinking is required? ● You are being asked to apply your knowledge of how pyrimidines and purines bind to each other in forming a double strand of DNA. ● What key words does the question contain and what do they mean? ● Pyrimidines—the nucleotides cytosine and thymine ● Purines—the nucleotides adenine and guanine Gather Content ● What do you know about base pairing? What other information is related to the question? ● Pyrimidines and purines on complementary strands of DNA through hydrogen bonding. Thymine pairs with adenine and cytosine pairs with guanine. ● Because pyrimidines and purines form base pairs, for each pyrimidine there will be a purine. Choose Answer ● Given what you now know, what information is most likely to produce the correct answer? ● If there are 26 pyrimidines in a double strand of DNA, then there will be 26 purines. Reflect on Process ● Did your problem-solving process lead you to the correct answer? If not, where did the process break down or lead you astray? How can you revise your approach to produce a more desirable result? ● This question asked you to apply the structure of DNA to predict the number of purines if you know the number of pyrimidines. If you got the correct answer, great job! If you got an incorrect answer, where did the process Version 1
24
break down? Did you recall what pyrimidines and purines are?Did you realize that the two form base pairs in DNA?
15) D 16) C 17) A 18) D 19) C 20) E 21) A 22) B 23) D 24) A 25) B 26) E 27) E 28) D 29) E 30) E 31) B 32) A 33) B 34) C 35) D 36) B 37) A 38) B
Version 1
25
Clarify Question ● What is the key concept addressed by the question? question asks about dehydration reactions.
● The
● What type of thinking is required? ● You are being asked to apply your knowledge of dehydration reactions to identify one. ● What key words does the question contain and what do they mean? ● Dehydration reaction—reactions that link two monomers together, releasing water Gather Content ● What do you know about dehydration reactions? What other information is related to the question? ● Dehydration reactions are used to join two monomers together. ● The two reactions with water on the reactant side of the reaction (left side) are using water and are thus not dehydration reactions. A + B + H2O → C and A + H2O → B + C. ● Reactions with one reactant and two products are breaking down a compound and not synthesizing molecules. A + H2O → B + C and A → B + C+ H2O. Choose Answer ● Given what you now know, what information is most likely to produce the correct answer? ● The best answer is A + B → C + H2O. In this case the two reactants A and B are being joined together to form the product C. Water is generated in the reaction making it a dehydration. Reflect on Process ● Did your problem-solving process lead you to the correct answer? If not, where did the process break down or lead you astray? How can Version 1
26
you revise your approach to produce a more desirable result? ● This question asked you to apply the definition of a dehydration reaction to identify it from several examples. If you got the correct answer, great job! If you got an incorrect answer, where did the process break down? Did you recall the definition of a dehydration reaction and that water was a product? Did you realize that in addition to producing water these involve synthesizing a polymer from two monomers?
39) A
Version 1
27
Clarify Question ● What is the key concept addressed by the question? question asks about hydrolysis reactions.
● The
● What type of thinking is required? ● You are being asked to apply your knowledge of hydrolysis reactions to identify one. Gather Content ● What do you know about hydrolysis reactions? What other information is related to the question? ● Hydrolysis reactions are used to break a polymer down into monomers. The roots “hydro” meaning water and “lysis” meaning to break give clues to what this reaction is doing. ● The two reactions with water on the product side of the reaction (right side) are releasing water and are thus not hydrolysis reactions. A → B + C+ H2O and A + B → C + H2O. ● Reactions with one product and two reactants are synthesizing molecules and not breaking them down A + B → C + H2O and A + B + H2O → C. Choose Answer ● Given what you now know, what information is most likely to produce the correct answer? ● The best answer is A + H2O → B + C. In this case the reactant A is being broken down to form B and C. Water is used in the reaction to break down A, making it a hydrolysis reaction. Reflect on Process ● Did your problem-solving process lead you to the correct answer? If not, where did the process break down or lead you astray? How can you revise your approach to produce a more desirable result? ● This question asked you to apply the definition of a hydrolysis reaction to Version 1
28
identify it from several examples. If you got the correct answer, great job! If you got an incorrect answer, where did the process break down? Did you recall the definition of a hydrolysis reaction and that water used to lyse or break down a molecule?Did you realize that in addition to using water these involve breaking down a polymer into two monomers?
40) A
Version 1
29
Clarify Question ● What is the key concept addressed by the question? question asks about base pairing in DNA.
● The
● What type of thinking is required? ● You are being asked to apply your knowledge of base pairing in DNA to pick the complementary strand for a DNA sequence. ● What key words does the question contain and what do they mean? ● 5' CGATTAGT 3' – 5’ and 3’ indicate the orientation of the strand of DNA, the letters represent the abbreviations for the nucleotide bases Gather Content ● What do you know about base pairing? What other information is related to the question? ● The two strands of DNA will pair with each other in a reverse orientation, so if one strand is in the 5’ →3’ orientation the other will be 3’ ← 5’. ● On each strand guanine (G) will pair with cytosine (C) and adenine (A) will pair with thymine (T). ● The strand in the question is in the orientation 5’ →3’, so the complementary strand will be 3’ ← 5’andhas the sequence CGATTAGT so the complementary sequence will be GCTAATCA. Choose Answer ● Given what you now know, what information is most likely to produce the correct answer? ● Putting these two requirements together, the correct answer is 3' GCTAATCA 5'. Reflect on Process ● Did your problem-solving process lead you to the correct answer? If not, where did the process break down or lead you astray? How can Version 1
30
you revise your approach to produce a more desirable result? ● This question asked you to apply the structure DNA to predict the complementary sequence to a given strand of DNA. If you got the correct answer, great job! If you got an incorrect answer, where did the process break down? Did you recall that DNA binds in a reverse complementary orientation? Did you remember that G pairs with C and A with T?
41) A
Version 1
31
Clarify Question ● What is the key concept addressed by the question? ● The question asks about membrane spanning domains of a protein. ● What type of thinking is required? ● You are being asked to apply your knowledge of membrane-spanning domains to predict the properties of amino acids in these domains. ● What key words does the question contain and what do they mean? ● Membrane—lipid bilayer that has polar head groups interacting with water and long non-polar fatty tails that point into the membrane away from water ● Membrane-spanning domains—segments of a protein that cross a lipid bilayer Gather Content ● What do you know about amino acid side groups? What other information is related to the question? ● There are 20 common amino acid side groups with different functional groups. Some are charged or polar which will interact positively with water. Other side groups are non-polar and hydrophobic and will be repelled by water. ● The center of a membrane is a non-polar or hydrophobic environment. In order for a protein to cross a membrane the side chains would need to be soluble in this lipid environment. Choose Answer ● Given what you now know, what information is most likely to produce the correct answer? ● Non-polar side chains on a protein would be most soluble in a membrane. Reflect on Process ● Did your problem-solving process lead you to the correct answer? Version 1
32
If not, where did the process break down or lead you astray? How can you revise your approach to produce a more desirable result? ● This question asked you to apply the properties of a membrane to predict which amino acid side chains would be found in a membrane-spanning domain. If you got the correct answer, great job! If you got an incorrect answer, where did the process break down? Did you understand that a membrane would be a non-polar and hydrophobic environment? Did you recall that polar and charged amino acid side chains are hydrophilic and would not dissolve well in hydrophobic membranes.
42) D 43) D
Version 1
33
Clarify Question ● What is the key concept addressed by the question? ● The question asks about different disorders related to the consumption of milk. ● What type of thinking is required? ● You are being asked to apply your knowledge of the effects of hydrogenation of fats on consistency. ● What key words does the question contain and what do they mean? ● Lactose intolerant—cannot break down lactose ● Milk allergy—proteins in milk trigger an allergic reaction Gather Content ● What do you know about milk proteins and lactose? What other information is related to the question? ● Lactose is a disaccharide that is broken into glucose and galactose by the enzyme lactase. People who lack the enzyme cannot break down lactose and are considered lactose intolerant.When milk is made into cheese or yogurt, microorganisms chew up the lactose, so people who are lactose intolerant can often eat these foods. ● Milk proteins are digested by enzymes in the stomach, but fragments can trigger an immune response in some individuals. These milk proteins are not degraded in making cheese or yogurt, so people with milk allergies cannot eat these foods. Choose Answer ● Given what you now know, what information is most likely to produce the correct answer? ● Lactose is a sugar and not a protein, so it does not trigger an allergic response. Lactose intolerance and milk allergies are different disorders with separate causes. Milk allergies are Version 1
34
to milk proteins, and people with these allergies cannot eat cheese or yogurt make by active cultures of bacteria. The best answer is that lactose is a carbohydrate and while it contributes to lactose intolerance, it is not a cause of allergies. Reflect on Process ● Did your problem-solving process lead you to the correct answer? If not, where did the process break down or lead you astray? How can you revise your approach to produce a more desirable result? ● This question asked you to analyze the description of lactose intolerance and milk allergies to explain the role of carbohydrate and proteins in each. If you got the correct answer, great job! If you got an incorrect answer, where did the process break down? Did you realize that lactose is a carbohydrate and not likely to cause allergies? Did you realize that lactose intolerance and milk allergies were separate disorders?
44) C 45) A
Version 1
35
Clarify Question ● What is the key concept addressed by the question? ● The question asks about the effect of hydrogenating an oil on its consistency. ● What type of thinking is required? ● You are being asked to apply your knowledge of the effects of hydrogenation of fats on consistency. ● What key words does the question contain and what do they mean? ● Hydrogenating vegetable oil—refers to adding more hydrogens to an unsaturated fat to make it more saturated ● Trans-fatty acids—the product formed when unsaturated fatty acids are hydrogenated Gather Content ● What do you know about the effect of saturation on the consistency of a fat? What other information is related to the question? ● Saturated fats do not have C=C double bonds, so their fatty acid tails can compact together tightly, makeingthem more likely to be a solid at room temperature. Butter is a good example of a saturated fat. If heat is added the butter can melt from a solid to a liquid. ● Unsaturated fats have C=C double bonds that introduce kinks or bends into the fatty acid tails.This prevents the fats from getting as close to each other, which makes them more likely to be a liquid at room temperature. Corn oil is a good example of this, if the oil is put in a freezer it will eventually form a solid. ● Adding hydrogens to an unsaturated fat will make it have fewer bends and make it more solid. These hydrogenated unsaturated fats are often trans-fats. Choose Answer Version 1
36
● Given what you now know, what information is most likely to produce the correct answer? ● The correct answer is that hydrogenation of fats changes them from unsaturated to saturated, which are more solid. Trans-fatty acid answers are not correct because they are the products of hydrogenation. Reflect on Process ● Did your problem-solving process lead you to the correct answer? If not, where did the process break down or lead you astray? How can you revise your approach to produce a more desirable result? ● This question asked you to apply the structure of a fat to predict its melting temperature. If you got the correct answer, great job! If you got an incorrect answer, where did the process break down? Did you recall what was meant by unsaturated fat, saturated fat, and trans-fat? Did you understand the impact of saturation on how solid a fat would be? Did you recall that trans-fats and saturated fats are made by hydrogenating unsaturated fats.
46) B
Version 1
37
Clarify Question ● What is the key concept addressed by the question? question asks about the solubility of vitamin D.
● The
● What type of thinking is required? ● You are being asked to apply the solubility of cholesterol to vitamin D made from cholesterol. ● What key words does the question contain and what do they mean? ● Derivative of cholesterol—cholesterol was the reactant in a reaction and vitamin D was made from the cholesterol Gather Content ● What do you know about the solubility of cholesterol? What other information is related to the question? ● Cholesterol is a lipid and it is fat soluble. Lipids are hydrophobic, non-polar molecules that do not dissolve well in water. ● Because vitamin D is made from cholesterol, it would have a similar solubility. Choose Answer ● Given what you now know, what information is most likely to produce the correct answer? ● Vitamin D is fat soluble because it is a lipid molecule like cholesterol. Reflect on Process ● Did your problem-solving process lead you to the correct answer? If not, where did the process break down or lead you astray? How can you revise your approach to produce a more desirable result? ● This question asked you to apply the structure of vitamin D to predict its solubility. If you got the correct answer, great job! If you got an incorrect answer, where did the process break down? Did you recall that cholesterol is a lipid and thus fat-soluble and not water-soluble? Did you Version 1
38
realize that if vitamin D was made from cholesterol it would have similar solubility?
47) B 48) C 49) C 50) B 51) C
Version 1
39
Clarify Question ● What is the key concept addressed by the question? question asks about domains of a protein.
● The
● What type of thinking is required? ● You are being asked to apply your knowledge of domains to explain that they determine the function of a protein. ● What key words does the question contain and what do they mean? ● Protein sequence—the primary structure of a protein Gather Content ● What do you know about protein function? What other information is related to the question? ● The function of a protein is determined by its tertiary or three dimensional structure. ● The primary structure or amino acid sequence contains information that allows proteins to fold into specific secondary structures (alpha helix or beta sheet). This information is typically obtained by determining the DNA sequence of the gene encoding the protein and then translating it on the computer. ● The secondary structure of a protein is important in determining the final tertiary structure of a protein, but do not always have specific activities. Choose Answer ● Given what you now know, what information is most likely to produce the correct answer? ● The tertiary structure, or threedimensional shape, of a protein determines its function. This structure can be broken into smaller conserved domains that fold into threedimensional structures with distinct activities. By looking for these conserved domains in a sequence you can make predictions about the Version 1
40
function of the complete protein. Reflect on Process ● Did your problem-solving process lead you to the correct answer? If not, where did the process break down or lead you astray? How can you revise your approach to produce a more desirable result? ● This question asked you to apply your knowledge of the different levels of protein structure to explain which would determine the function of a protein. If you got the correct answer, great job! If you got an incorrect answer, where did the process break down? Did you understand that the DNA sequence would give you the same sequence you already had? Did you think that the secondary structure of a protein determined the function of a protein? Did you realize that domains are threedimensional structures that have specific functions?
52) D
Version 1
41
Clarify Question ● What is the key concept addressed by the question? ● The question asks about functional groups that are acids and bases. ● What type of thinking is required? ● You are being asked to apply your knowledge of functional groups that are acids and bases to predict which would gain or lose a proton at a neutral pH. ● What key words does the question contain and what do they mean? ● Proton – H+ ion that is lost by acids and gained by bases at neutral pH Gather Content ● What do you know about functional groups? What other information is related to the question? ● Functional groups have specific activities based on their chemical structure. They can be polar or non-polar, some can behave as acids or bases. ● Sulfhydryl groups are –SH, these are polar but don’t readily behave as acids. ● Hydroxyl groups are –OH, these are polar but don’t behave as acids. ● Carbonyl groups are –CH=O and do not behave as acids. Choose Answer ● Given what you now know, what information is most likely to produce the correct answer? ● Carboxyl groups are –CO2H and the proton (H+) can be lost as an acid forming –CO2-. ● Amino groups are –NH2 and can gain a proton as a base forming – NH3+. Reflect on Process ● Did your problem-solving process lead you to the correct answer? Version 1
42
If not, where did the process break down or lead you astray? How can you revise your approach to produce a more desirable result? ● This question asked you to apply the structure of functional groups to predict if they will form an acid or base. If you got the correct answer, great job! If you got an incorrect answer, where did the process break down? Did you remember the structures of each of the functional groups? Did you know which would behave as an acid or a base?
53) C
Version 1
43
Clarify Question ● What is the key concept addressed by the question? ● The question asks about how sugars are stored in an organ in animals. ● What type of thinking is required? ● You are being asked to apply your knowledge of the storage of sugars in animals to a specific example. ● What key words does the question contain and what do they mean? ● Deer liver—an organ from an animal Gather Content ● What do you know about stored sugars in animals? What other information is related to the question? ● Stored sugars are complex sugars stored for long term use as opposed to simple sugars in the blood stream for immediate use.These complex carbohydrates contain multiple glucose molecules stuck together with α1-4 linkages. In animals this is called glycogen while in plants it is amylose or starch. ● For short term use, monosaccharides and disaccharides are used more often. Examples include glucose and maltose. Choose Answer ● Given what you now know, what information is most likely to produce the correct answer? ● The correct answer is glycogen. Liver glycogen is used to keep blood glucose levels constant.If blood glucose becomes low, liver glycogen is broken down, releasing glucose into the blood. If blood glucose rises, some is taken up by the liver and converted into glycogen. Reflect on Process ● Did your problem-solving process lead you to the correct answer? If not, where did the process break down or lead you astray? How can Version 1
44
you revise your approach to produce a more desirable result? ● This question asked you to apply the definitions of different carbohydrates to explain which would be found in the liver of a deer. If you got the correct answer, great job! If you got an incorrect answer, where did the process break down? Did you recall which forms of carbohydrate would be found in animals? Did you recall which would be used for long term energy storage?
54) D 55) C 56) D 57) A 58) A
Version 1
45
Clarify Question ● What is the key concept addressed by the question? ● The question asks about the solubility of functional groups in water. ● What type of thinking is required? ● You are being asked to apply your knowledge of functional groups to predict which would be least soluble in water. Gather Content ● What do you know about solubility in water? What other information is related to the question? ● The more polar a molecule is, the better it can dissolve in water. The more nonpolar functional groups a molecule has, the less soluble it will be in water. ● All four of the amino acids will have the same backbone structure with an amino group, an α-carbon, and a carboxyl group. The differences will be with the structure of the side chains. ● Glutamine is a polar, charged amino acid and will be the most soluble amino acid. Choose Answer ● Given what you now know, what information is most likely to produce the correct answer? ● Among the remaining three amino acids, each has the following side chains: glycine (H), alanine (CH3), and isoleucine (C4H9). While none of these has polar groups, isoleucine has the most carbons and will be the most nonpolar and hydrophobic. His will make isoleucine the least soluble in water. Reflect on Process ● Did your problem-solving process lead you to the correct answer? If not, where did the process break down or lead you astray? How can you revise your approach to produce a more desirable result? ● This Version 1
46
question asked you to apply the structure of an amino acid to predict its solubility. If you got the correct answer, great job! If you got an incorrect answer, where did the process break down? Did you know the structures of the amino acid side chains? Did you know which structures would be the most nonpolar?
59) A 60) A
Version 1
47
Clarify Question ● What is the key concept addressed by the question? ● The question asks about the solubility of functional groups in water. ● What type of thinking is required? ● You are being asked to apply your knowledge of functional groups to predict which would be most soluble in water. Gather Content ● What do you know about solubility in water? What other information is related to the question? ● The more polar a molecule is, the better it can dissolve in water. The more non-polar functional groups a molecule has, the less soluble it will be in water. ● All four of the amino acids will have the same backbone structure with an amino group, an α-carbon, and a carboxyl group. The differences will be with the structure of the side chains. ● Three of the amino acids have non-polar side chains: glycine (H), valine (C3H7), and leucine (C4H9). While none of these has polar groups, leucine has the most carbons and will be the most non-polar and hydrophobic. This will make isoleucine the least soluble in water. Choose Answer ● Given what you now know, what information is most likely to produce the correct answer? ● Serine is the only amino acid with a polar side chain (CH2OH), the hydroxyl group (OH) will interact well with water, making serine the most soluble in water. Reflect on Process ● Did your problem-solving process lead you to the correct answer? If not, where did the process break down or lead you astray? How can you revise your approach to produce a more desirable result? ● This Version 1
48
question asked you to apply the structure of an amino acid to predict its solubility. If you got the correct answer, great job! If you got an incorrect answer, where did the process break down? Did you know the structures of the amino acid side chains? Did you know which structures would be the most polar?
61) B
Version 1
49
Clarify Question ● What is the key concept addressed by the question? ● The question asks about the net reaction for a dehydration synthesis. ● What type of thinking is required? ● You are being asked to analyze choices for the product of a dehydration synthesis reaction and choose the correct answer. Gather Content ● What do you know about dehydration synthesis? What other information is related to the question? ● Dehydration synthesis is used to make polymers out of monomers. If 10 glucose molecules are joined together then it would make sense to simply multiply the number of each atom in glucose by 10 to calculate how many would be present in the polymer. Based on this you would obtain the answer C6H12O6 x 10 = C60H120O60. ● One additional thing to consider is that a water molecule is lost as each glucose is joined together. If there are 10 glucoses joined together, there would be 9 total dehydration reactions that took place. Choose Answer ● Given what you now know, what information is most likely to produce the correct answer? ● Water has the chemical structure H2O. If a water molecule is lost in each dehydration reaction a total of 9 H2O would be lost for a total loss of H18O9. The formula of the final polymer will then be C60H120O60 - H18O9 = C60H102O51. Reflect on Process ● Did your problem-solving process lead you to the correct answer? If not, where did the process break down or lead you astray? How can you revise your approach to produce a more desirable result? ● This Version 1
50
question asked you to analyze a dehydration synthesis reaction to determine the chemical formula of the final product. If you got the correct answer, great job! If you got an incorrect answer, where did the process break down? Did you realize that if there are 10 glucose molecules in the final polymer you would need to start by multiplying the formula of glucose by 10? Did you realize that even though there were 10 glucose molecules there would only be 9 dehydration reactions? Did you know to subtract the 9 water molecules from the formula to get the final answer?
62) D 63) D
Version 1
51
Clarify Question ● What is the key concept addressed by the question? ● The question asks about which biological molecules contain phosphorus. ● What type of thinking is required? ● You are being asked to apply your knowledge of the atoms found in different biological molecules to predict which could not be made by plants in a shortage of phosphorus. Gather Content ● What do you know about molecules that contain phosphorus? What other information is related to the question? ● The primary molecules in cells that contain phosphorus are nucleic acids. ● The word carbohydrate has the roots carbo- (carbon) and hydrate (water). Carbohydrates all have the base structure CH2O repeated many times, and do not contain phosphorus. Choose Answer ● Given what you now know, what information is most likely to produce the correct answer? ● DNA is deoxyribonucleic acid and contains nucleotides connected together by phosphates. The other options are all carbohydrates and do not contain phosphate. Reflect on Process ● Did your problem-solving process lead you to the correct answer? If not, where did the process break down or lead you astray? How can you revise your approach to produce a more desirable result? ● This question asked you to apply the structure of different biological molecules to determine which would contain phosphate. If you got the correct answer, great job! If you got an incorrect answer, where did the process break down? Did you realize that the carbohydrates do not Version 1
52
contain phosphate and only contain C, H and O? Did you remember that DNA is a nucleic acid and contains phosphate?
64) C 65) D
Version 1
53
Clarify Question ● What is the key concept addressed by the question? ● The question asks about the role of the aldehyde groups in glucose in the formation of starch. ● What type of thinking is required? ● You are being asked to analyze the results of an experiment to explain the structure of starch. Gather Content ● What do you know about the structure of starch? What other information is related to the question? ● Starch is formed from multiple glucose molecules joined together through the aldehyde on one glucose and a hydroxyl group on another. Thus the aldehyde group is no longer present on a glucose molecule when it is in starch. ● Once a starch molecule is broken down into glucose, the glucose molecules contain the aldehyde group and can react with Benedict’s reagent again. Thus the aldehyde group and the glucose molecule are not permanently altered. Choose Answer ● Given what you now know, what information is most likely to produce the correct answer? ● Even though there are many glucose molecules in a molecule of starch, most of these will have lost their aldehyde group when they were joined to the next glucose. Only the glucose at the end of the starch will have a free aldehyde to react with the Benedict’s reagent. Reflect on Process ● Did your problem-solving process lead you to the correct answer? If not, where did the process break down or lead you astray? How can you revise your approach to produce a more desirable result? ● This Version 1
54
question asked you to analyze the results of an experiment to explain the structure of starch. If you got the correct answer, great job! If you got an incorrect answer, where did the process break down? Did you understand that the aldehyde group on a glucose is lost when it forms a bond with another glucose molecule to form starch? Did you realize that this reaction is reversible, so no permanent changes to the glucose occur? Based on the structure of starch, did you understand that the glucose on the end of the starch molecule still has a free aldehyde?
66) B
Version 1
55
Clarify Question ● What is the key concept addressed by the question? ● The question asks about the nature of amino acids in a membrane. ● What type of thinking is required? ● You are being asked to apply your knowledge of the properties of amino acid side chains to predict which would be found in a membrane. ● What key words does the question contain and what do they mean? ● R groups—side chains of amino acids, can be polar or nonpolar Gather Content ● What do you know about proteins in membranes? What other information is related to the question? ● Membranes contain lipids and are hydrophobic, nonpolar environments. Amino acids with side chains that are nonpolar or hydrophobic will be more soluble in this environment. ● R groups that are hydrophilic, polar or able to form ionic bonds will all be soluble in water. These will not be soluble in a lipid membrane. Choose Answer ● Given what you now know, what information is most likely to produce the correct answer? ● R groups that are hydrophobic are repelled by water and will be the most soluble in the membrane.Because of this, proteins that span membranes contain long stretches of hydrophobic or nonpolar amino acids. Reflect on Process ● Did your problem-solving process lead you to the correct answer? If not, where did the process break down or lead you astray? How can Version 1
56
you revise your approach to produce a more desirable result? ● This question asked you to apply the properties of different amino acid R groups to predict which would be present in a biological membrane. If you got the correct answer, great job! If you got an incorrect answer, where did the process break down? Did you have trouble recalling that a biological membrane contains lipids and is thus hydrophobic and nonpolar? Did you understand that nonpolar amino acid R groups would be more soluble in a lipid membrane?Did you realize that the other options were all R groups that would be soluble in water and not in a membrane?
67) E
Version 1
57
Clarify Question ● What is the key concept addressed by the question? question asks about nonpolar molecules.
● The
● What type of thinking is required? ● You are being asked to apply your knowledge of the polarity of different macromolecules to predict which would bind to a dye. ● What key words does the question contain and what do they mean? ● Sudan IV—dye that binds to nonpolar molecules and turns those samples a color Gather Content ● What do you know about nonpolar molecules? What other information is related to the question? ● Nonpolar molecules are hydrophobic and do not dissolve in water. Lipids and fats are common nonpolar biological molecules. ● Hemoglobin is a protein found in blood, which is a water based solution, so it is a hydrophilic and polar molecule. ● Chitin and starch are complex carbohydrates with lots of hydroxyl groups, they are hydrophilic and polar molecules. ● DNA is highly charged and also a polar and hydrophilic molecule. Choose Answer ● Given what you now know, what information is most likely to produce the correct answer? ● Fats are nonpolar hydrophobic molecules, so they will bind to Sudan IV and turn any foods containing fats a color. Reflect on Process ● Did your problem-solving process lead you to the correct answer? If not, where did the process break down or lead you astray? How can Version 1
58
you revise your approach to produce a more desirable result? ● This question asked you to apply the properties of a macromolecule to determine if it would bind to a hydrophobic dye. If you got the correct answer, great job! If you got an incorrect answer, where did the process break down? Did you recall what nonpolar meant? Could you remember which biological molecules were nonpolar and which were polar?
68) D
Version 1
59
Clarify Question ● What is the key concept addressed by the question? ● The question asks about the number of water molecules produced in a dehydration synthesis reaction. ● What type of thinking is required? ● You are being asked to analyze a reaction to predict the number of water molecules produced. Gather Content ● What do you know about triglycerides? What other information is related to the question? ● Glycerol is a 3 carbon sugar with 3 hydroxyl (OH) groups.Each of these OH groups reacts with a fatty acid, forming a triglyceride. In each of these steps a water molecule is released in a dehydration synthesis reaction. It is dehydration because water is released, it is a synthesis, because you are making triglyceride from glycerol and fatty acids. ● The “tri” in triglyceride refers to the three fatty acids attached to the glycerol. Choose Answer ● Given what you now know, what information is most likely to produce the correct answer? ● Because fatty acids are attached to glycerol through a dehydration reaction, one water molecule is released with each reaction. Three fatty acids are attached to glycerol to form one triglyceride, so three water molecules would be released. Reflect on Process ● Did your problem-solving process lead you to the correct answer? If not, where did the process break down or lead you astray? How can you revise your approach to produce a more desirable result? ● This question asked you to analyze a reaction to predict the number of water Version 1
60
molecules produced. If you got the correct answer, great job! If you got an incorrect answer, where did the process break down? Did you realize that this is a dehydration synthesis reaction? Did you recall that three fatty acids are added to the glycerol? Did you understand that one water molecule would be released as each fatty acid was attached to glycerol?
69) B
Version 1
61
Clarify Question ● What is the key concept addressed by the question? ● The question asks about the role a 3’ OH group in the formation of phosphodiester bonds. ● What type of thinking is required? ● You are being asked to analyze statements about why a nucleotide lacking a 3’ OH group would serve as an antiviral drug. ● What key words does the question contain and what do they mean? ● Antiviral chemical—a drug that kills viruses ● 3’ OH group—hydroxyl group on the deoxyribose of a nucleotide Gather Content ● What do you know about the synthesis of DNA? What other information is related to the question? ● In the synthesis of DNA, the new strand grows from the 3’ end. The free 3’ OH group attacks the phosphate on the next nucleotide to be added, forming a phosphodiester bond. If the growing end lacks a 3’ OH group then no new nucleotides can be added. ● Proteins and carbohydrates do not contain phosphodiester bonds, so these would not be affected by Acyclovir. Choose Answer ● Given what you now know, what information is most likely to produce the correct answer? ● When Acyclovir is incorporated into a strand of viral DNA that strand now lacks a 3’ OH group. As a result it cannot add the next nucleotide because it cannot form a phosphodiester bond. Viral DNA replication ends where the Acyclovir was introduced into the viral DNA and the infection stops. Reflect on Process Version 1
62
● Did your problem-solving process lead you to the correct answer? If not, where did the process break down or lead you astray? How can you revise your approach to produce a more desirable result? ● This question asked you to analyze explanations for how a nucleotide lacking a 3’ OH group would function as an antiviral drug. If you got the correct answer, great job! If you got an incorrect answer, where did the process break down? Did you realize the role of the 3’ OH group in the formation of a phosphodiester bond? Did you know that phosphodiester bonds are found in nucleic acids like DNA but not in proteins or carbohydrates? Did you predict that the lack of a 3’ OH group would inhibit viral replication?
70) B
Version 1
63
Clarify Question ● What is the key concept addressed by the question? ● The question asks about the differences in glucose found in starch and cellulose. ● What type of thinking is required? ● You are being asked to apply your knowledge of the structures of starch and cellulose to explain the differences between these two forms of carbohydrate. ● What key words does the question contain and what do they mean? ● α-and β-glucose—chemically identical glucose molecules, but they differ in how their OH groups are oriented ● Synthesize ATP—means that the glucose can be broken down and metabolized Gather Content ● What do you know about the structure of carbohydrates? What other information is related to the question? ● Complex carbohydrates used to store energy include starch (plants) and glycogen (animals). These contain α-glucose joined together. These α-glucose can be broken off by enzymes and the glucose used to make ATP. ● Structural complex carbohydrates include cellulose (plants) and chitin (animals). These contain β-glucose joined together. These βglucose cannot be broken off by enzymes, which is why these carbohydrates are structural. Choose Answer ● Given what you now know, what information is most likely to produce the correct answer? ● α-glucose and β-glucose are chemically identical glucose molecules called stereoisomers which differ in how their OH groups are oriented. Version 1
64
Reflect on Process ● Did your problem-solving process lead you to the correct answer? If not, where did the process break down or lead you astray? How can you revise your approach to produce a more desirable result? ● This question asked you to apply the structure of a polysaccharide to explain some of its properties. If you got the correct answer, great job! If you got an incorrect answer, where did the process break down? Did you recall that both structural and energy storing forms of complex carbohydrates are made of glucose?Did you remember that α-glucose contains bonds that can be broken by enzymes while β-glucose does not?
71) B
Version 1
65
Clarify Question ● What is the key concept addressed by the question? ● The question asks about the composition of different polymers. ● What type of thinking is required? ● You are being asked to apply your knowledge of which atoms are found in different monomers to a specific example. ● What key words does the question contain and what do they mean? ● Radioactive isotope of sulfur—an atom of sulfur with an extra neutron that can go through radioactive decay ● Radioactively labeled—growing cells in the presence of the radioactive sulfur such that ny molecules containing sulfur will become radioactive Gather Content ● What do you know about composition of different monomers? What other information is related to the question? ● Lipids and carbohydrates contain only carbon, oxygen and hydrogen. ● Nucleic acids contain nitrogen, carbon, oxygen and hydrogen. ● Amino acids contain sulfur, nitrogen, carbon, oxygen and hydrogen. Choose Answer ● Given what you now know, what information is most likely to produce the correct answer? ● The only monomers that contain sulfur are the amino acids cysteine and methionine, therefore, only proteins will become radioactively labeled with 35S. Reflect on Process ● Did your problem-solving process lead you to the correct answer? If not, where did the process break down or lead you astray? How can Version 1
66
you revise your approach to produce a more desirable result? ● This question asked you to apply the composition of amino acids to explain why only proteins would be labeled with 35S. If you got the correct answer, great job! If you got an incorrect answer, where did the process break down? Did you understand what was meant by radioactively labeling a cell? Did you recall that only the amino acids methionine and cysteine contain sulfur?
72) C
Version 1
67
Clarify Question ● What is the key concept addressed by the question? question asks you to identify a macromolecule.
● The
● What type of thinking is required? ● You are being asked to apply your knowledge of the representation of a macromolecule to identify it. ● What key words does the question contain and what do they mean? ● 5’ and 3’—ends of a strand of nucleic acid, 5’ and 3’ refer to the OH groups on the ribose sugar that are not in a phosphodiester bond Gather Content ● What do you know about nucleic acid structure? What other information is related to the question? ● The 5’ and 3’ ends are the first clue that this is a nucleic acid. ● Carbohydrates or polysaccharides do not have a 5’ or 3’ end. ● Proteins have ends that are labeled by a free amino terminus or carboxy terminus. Choose Answer ● Given what you now know, what information is most likely to produce the correct answer? ● While all nucleic acids have a 5’ and 3’ end, these could be either RNA or DNA. RNA has nucleotides with the abbreviations A, C, G, and U.DNA has nucleotides with the abbreviations A, C, G, and T. Since this sequence has T and no U it must be DNA. Reflect on Process ● Did your problem-solving process lead you to the correct answer? If not, where did the process break down or lead you astray? How can you revise your approach to produce a more desirable result? ● This Version 1
68
question asked you to apply the structure of a polymer to determine its type. If you got the correct answer, great job! If you got an incorrect answer, where did the process break down? Did you realize that the 5’ and 3’ ends meant it was a nucleic acid? Did you notice that it had T’s and no U’s?
73) A
Version 1
69
CHAPTER 4 MULTIPLE CHOICE - Choose the one alternative that best completes the statement or answers the question. 1) Thesemifluid matrix that surrounds organelles in a cell is called the __________. A) ribosome B) cytoplasm C) stroma D) nucleoplasm
2) A distinctive feature of eukaryotes is the organization of tightly packaged chromosomal DNAwith proteins into a membrane-bound structure called a __________. A) nucleolus B) cell C) nucleus D) plasma membrane
3) The organelle that cancollect, package, modify, and transport molecules is called the __________. A) mitochondria B) plasma membrane C) vesicle D) Golgi apparatus
4) Many antibiotics selectivelykill bacteria and not mammalian cellsbecause they block synthesis of the _______.
Version 1
1
A) nucleus B) cytoskeleton C) cell wall D) plasma membrane E) cytoplasm
5)
The evolutionary process that created chloroplasts and mitochondria is _________. A) endosymbiosis B) evolution C) symbiosis D) punctuated equilibrium
6) Microfilaments such as actin, microtubules, and the intermediate filaments form the cellsupporting structure called the __________. A) cytoplasm B) plasma membrane C) cytoskeleton D) nucleus
7)
Integrins are glycoproteins that help link the cytoskeleton and the __________. A) cell wall B) extracellular matrix C) cytoplasm D) ribosomes
8) Small cells function more effectively, because as cells become larger their surface area to volume ratio __________.
Version 1
2
A) increases proportionally to the radius of the cell (r) B) decreases proportionally to the radius of the cell (r) C) stays the same D) increases proportionally to the cell's radius squared (r 2) E) decreases proportionally to the cell's radius squared (r 2)
9)
Organelles that breakdown hydrogen peroxide into water and oxygen are __________. A) lysosomes B) plastids C) vacuoles D) liposomes E) peroxisomes
10) A circular molecule of DNA devoid of any histone proteins describes the chromosome of a/an __________. A) bacteria B) protozoa C) insects D) flowering plants E) yeasts
11)
Who first described cells? A) Darwin B) Golgi C) Linnaeus D) Hooke E) Leeuwenhoek
Version 1
3
12) Schleiden and Schwann stated the "cell theory," which in its modern form says __________. A) all organisms are composed of one or more cells, all cells require oxygen, and all cells arise by division of other cells B) all organisms are composed of one or more cells, all cells are the smallest living things, and all cells arise by division of other cells C) all cells will arise spontaneously, all cells require oxygen, and all cells are composed of one or more cells D) all organisms are composed of one or more cells, all cells arise spontaneously, and all cells vary in size and shape
13) Antibiotics that block the synthesis of a cell wall would be least likely to affect cells from which organisms? A) Bacteria B) Archaea C) Plants D) Animals E) Fungi
14) The peptidoglycan of bacterial cell walls contains a carbohydrate matrix linked together by short chains of __________. A) amino acids B) fatty acids C) nucleotides D) water molecules E) steroids
15) If you added a drug to a suspension of bacteria that inhibited their flagella, they would not be able to __________.
Version 1
4
A) eat B) respire C) reproduce D) make proteins E) move
16)
Photosynthetic pigments in bacteria are located in the __________. A) nuclei B) chloroplasts C) cell wall D) plasma membrane E) plasmids
17) If you put plant cells in a hypotonic solution they swell up. The water entering the plant cell will be found primarily in the __________. A) nucleus B) chloroplast C) Golgi body D) centriole E) central vacuole
18)
This cellular structurelacks a membrane. A) Endoplasmic reticulum B) Microbody C) Golgi body D) Nucleoid E) Nucleus
Version 1
5
19)
As a plant cell grows a new cell wall the secondary wall forms __________. A) between the primary cell wall and plasma membrane B) outside of the primary cell wall C) inside of the plasma membrane D) in the middle lamella E) in thecytoplasm
20) The cytoplasmic space in eukaryotic cells is occupied by many diverse membrane-bound structures with specific cellular functions called __________. A) flagella B) organelles C) cilia D) chromosomes E) receptors
21)
Ribosomes are made up of __________. A) only DNA molecules B) only RNA molecules C) DNA and RNA D) only protein molecules E) protein and RNA
22)
Cells that produce a lot of lipids for membranes would have an enlarged __________. A) mitochondrion B) rough ER C) smooth ER D) Golgi body E) nucleus
Version 1
6
23)
In eukaryotes, mitochondria are the organelles primarily involved in __________. A) energy release/capture B) phospholipid assembly C) export of enzymes D) lipid synthesis E) protein synthesis
24)
The major histocompatibility complex is important in a T-cell's ability to __________. A) recognize specific bacteria B) recognize specific viruses C) recognize self from nonself D) recognize specific parasites
25)
Which of the following does not move through a nuclear pore? A) RNA B) Proteins C) DNA
26)
The nucleolus is the site of __________. A) protein synthesis B) ribosome assembly C) chromosome replication D) lipid synthesis E) uncoiling and unraveling of chromosomes
27) Chromosomes can be condensed into compact structures, visible with the light microscope, but usually only __________. Version 1
7
A) after the cell is dead B) during cell division C) while the DNA is being copied into RNA D) while the proteins are being assembled E) while the nuclear pores are open
28) A doctor diagnoses a child with adrenoleukodystrophy (ALD) a disorder in which there is an accumulation of long fatty acid chains accumulating in the cells of the central nervous system. The issues arise with malfunctions in enzymes associated with which organelle? A) Golgi bodies B) Ribosomes C) Vacuoles D) Peroxisomes E) Endoplasmic reticulum
29)
Peroxisomes are examples of __________. A) chromosomes B) lysosomes C) microbodies D) nucleosomes E) ribosomes
30) Mitochondria and chloroplasts are the other organelles besides the nucleus that contain __________. A) DNA B) pores C) channels D) plasma membranes E) pigments
Version 1
8
31) The organelle involved in the oxygen-requiring process by which the energy harvested from macromolecules is used to generate ATP is the __________. A) nucleus B) lysosome C) ER D) mitochondria E) chloroplasts
32)
Cilia and flagella of eukaryotic cells are composed of __________. A) microtubules in a 9 + 2 arrangement B) actin filaments in a 9 + 2 arrangement C) protein fibers in a helical arrangement D) nine triplets of microtubules in a circle
33)
Chloroplasts contain a green photosynthetic pigment called __________. A) Gram stain B) chlorophyll C) hemoglobin D) chromatin E) keratin
34) The spreading of cancer cells, wound healing, and blood clotting all rely on the same type of cell movement called __________. A) swimming B) crawling C) whipping D) waving
Version 1
9
35) A team of researchers is studying a new species of spider mite that is approximately 0.05mm. While they can see the dorsal surface of the spider mitewith their naked eye, they cannot see the mite clearly. The researchers are writing a grant to provide support for a microscope to use to study these new mites. Based on the information given, what type of microscope would you suggest? A) Transmission electron microscope B) Scanning electron microscope C) Binocular compound light microscope D) Dissecting microscope
36) A cytologist is examining a tissue under an electron microscope. He notices that the endoplasmic reticulum of each cell is extremely rough in appearance, and he knows that the rough appearance is because of the ribosomes embedded there. He asks why there are so many ribosomes. You respond, __________. A) "This tissueexports lipids and is very involved with mRNA production." B) "This tissue exports proteins to other areas of the body." C) "This tissue exports various nucleic acids, hence the large number of ribosomes present on the endoplasmic reticulum in each of those cells."
37) You stain a cell with a dye that reacts with an enzyme found in the lysosome. You would also expect to see this dye in the __________. A) endoplasmic reticulum B) cytoplasm C) nucleus D) mitochondria
38) A cell biologist has developed a new drug that blocks vesicle transport at the cis face of the Golgi apparatus. What will this drug prevent from happening inside a cell?
Version 1
10
A) The synthesis of proteins or lipids on the endoplasmic reticulum B) The movement of the lipids and proteins from the endoplasmic reticulum to the Golgi apparatus C) The movement of materials from the Golgi apparatus to the plasma membrane D) The movement of vesicles from mitochondria to the Golgi apparatus
39)
Plants, fungi and bacterial all have __________. A) cell walls B) chloroplasts C) nuclei D) lysosomes E) mitochondria
40) Your research project involves characterizing the expression pattern of a nuclear pore protein. After using immunohistochemistry to fluorescently label the protein, you decide that you would like to capture the data as a 3-D image. Which microscope should you use? A) Phase-contrast microscope B) Fluorescence microscope C) Confocal microscope D) Scanning electron microscope
41) Sulfolobus are archaea that are commonly found in geothermal environments, with an optimum growth temperature of about 80°C. What feature allows archaea to thrive at such temperatures? A) Archaea use theheat from their environment to produce ATP. B) Archaea are highly adaptable to changing environmental temperatures. C) Archaeal membranes contain special lipids. D) Archaea have two cell walls that helps prevent heat absorption.
Version 1
11
42) Alcohol consumption adversely affects the synthesis of molecules involved in protein degradation, which can eventually lead to liver damage. Given this information, which organelle in liver cells is most directly affected by alcohol consumption? A) Nucleus B) Golgi apparatus C) Rough endoplasmic reticulum D) Lysosome
43) You have identified an enzyme that appears to be involved in the addition of palmityl group (a fatty acid) to certain proteins. Knowing that palmitoylated proteins are usually targeted for delivery to the plasma membrane, in which organelles might this enzyme be expressed? A) Golgi apparatus and endoplasmic reticulum B) Golgi apparatus and nucleus C) Endoplasmic reticulum and ribosomes D) Endoplasmic reticulum and nucleus
44) Given what is known about the minimal essential functions of cellular life, if you could create an artificial cell, what cellularstructures must you include? A) Cell wall B) Ribosomes C) Mitochondria D) Nucleus
45) When a vesicle movesalong microtubules toward the center of the cell, the molecular motor(s) _________ are used?
Version 1
12
A) kinesin B) dynein C) both kinesin and dynein D) molecular motors are not required for movement along microtubules.
46) A protein that is destined to reach the plasma membrane is making its way through the Golgi. At that moment, a drug was added to cells, blocking trafficking at the trans face of the Golgi. As a result, what would happen to the protein? A) Theproteinwould continue on to the cell surface. B) The protein wouldbe stuck in the Golgi. C) The protein would return to the endoplasmic reticulum via the trans face of the Golgi. D) Theproteinwould exit the Golgi, but instead be targeted to thecytoplasm.
47) If you were to use antibodies that were chemically bonded to a stain in order to visualize the expression pattern of a protein in a fixed tissue, what experimental techniques and equipment would be applicable? A) Immunohistochemistry, bright-field microscopy B) DAPI or Hoechst stains, fluorescence microscopy C) Antibody interference assay, bright-field microscopy D) Immunohistochemistry, dark-field microscopy
48)
During which cellular process do all three forms of RNA associate? A) Ribosome assembly B) Chromatin formation C) Formation of the nucleolus D) Protein synthesis
Version 1
13
49) Cardiac muscle cells are connected by special junctions called intercalated discs. Intercalated discsprovide strength between cells of tissues which exhibit highmechanical stress, strong flexible connections between tissue cells,and electrical excitability between the cells. Using this information, which junctions are present in intercalated discs? A) Tight, plasmodesmata, andadherens B) Tight, plasmodesmata, and desmosomes C) Gap, adherens,and desmosomes D) Gap, desmosomes,and hemidesmosomes E) Adherens, tight, and gap
50) Based on the function of mitochondria, what substance is able to pass through the outer membrane of the mitochondria easily? A) Oxygen B) Sugars C) Amino acids D) Potassium ions
51) Individuals that are heavy smokers may eventually have trouble clearing the mucous from their lungs because the structures that move fluid along the epithelial lining of the lung become damaged over time. What are these structures called? A) Pili B) Flagella C) Cell walls D) Cilia
52) Streptomycin is an antibiotic that interferes with the function of the 30S subunit of the prokaryotic ribosome. What is the consequence of treating a bacteria with streptomycin?
Version 1
14
A) Impaired oxygen synthesis B) Impaired protein synthesis C) Impaired DNA synthesis D) Impaired ATP synthesis
53) A dish of cells is treated with centrinone, a centriole inhibitor. These cells would no longer be able to __________. A) make ATP B) produce lysosomes C) synthesize proteins D) conduct photosynthesis E) divide
54) When looking at plant and animal cells with an electron microscope, you notice that the plant cells have more Golgi membranes than the animal cells. This is most likely because unlike in animal cells, in plant cells the Golgi apparatus is involved in the production of __________. A) cell walls B) cytoplasm C) chloroplasts D) mitochondria
55) Glycogen storage disease type II (GSD II), also known as Pompe disease, is a metabolic disorder that leads to the accumulation of glycogen in muscles and neurons throughout the body. It is caused by a defect in an enzyme that is normally housed with other enzymes that are involved in breaking down large macromolecules. Which organelle is most likely affected by this disease?
Version 1
15
A) Mitochondrion B) Golgi body C) Lysosome D) Rough ER E) Nucleus
56) Scientists believe that stimulating the growth of algae and other organisms can reduce carbon dioxide amounts in the atmosphere. Where is carbon dioxide consumed in a plant cell? A) Stroma in chloroplasts B) Matrix in mitochondria C) Number of amyloplasts D) Central vacuole E) Cellulose in cell wall
57) Cardiac muscle has prolonged contraction due to a sodium induced-calcium released process into the cytoplasm of the cell. Cardiac cells can communicate through pore-like channels between cells. List the junction and the organelle involved in cardiac muscle contraction? A) Gap; rough ER B) Gap; smooth ER C) Gap; Golgi apparatus D) Adherens; rough ER E) Adherens; smooth ER F) Adherens; golgi apparatus
58) Mitochondrial disorders can be caused by mutations in all the following except __________. A) mitochondrialDNA B) nuclear DNA C) cytoplasmicDNA
Version 1
16
59)
What is the difference between mitochondria and chloroplasts?
A) Mitochondria have three membranes, and chloroplasts have two membranes. B) Mitochondria have linear strandsof DNA, and chloroplasts have rings of DNA. C) Mitochondria are oxidative metabolic bacteriaderivatives, and chloroplasts are photosynthetic bacteriaderivatives. D) Mitochondria produce oxygen, and chloroplasts produce carbon dioxide.
60) If a water molecule passes through the outer membrane of a chloroplast, how many more membranes will it have to move through to be inthe stroma? A) 1 B) 2 C) 3 D) 4
61) As ATP moves from the matrix of the mitochondrion to the cytoplasm of a cell, how many membranes will it pass? A) 1 B) 2 C) 3 D) 4
62)
Prokaryotic cells are smaller than eukaryotic cells. What is a plausible explanation? A) Prokaryotic cell walls limit their cell growth. B) Prokaryotes lackthe genetic material needed for protein synthesis. C) Eukaryotes havecompartmentalization, which allows for specialization. D) Prokaryotes have more diverse energy sources. E) Eukaryotes are immobile, allowing them to grow larger.
Version 1
17
63) A cuboidal shaped cell with six sidesis 1mm× 1mm× 1mm. What is the volume and the surface areaof the cell? A) 1 mm 3 : 6 mm 2 B) 3 mm 3 : 3 mm 2 C) 1 mm 3 : 3 mm 2 D) 3 mm 3 : 1 mm 2
64) Matthew noticed that his house plants were wilting badly. Which cell structure had most likely contracted? A) Cytoplasm B) Cell wall C) Plasma membrane D) Vacuole
Version 1
18
Answer Key Test name: Chap 04_3e 1) B 2) C 3) D 4) C 5) A 6) C 7) B 8) B
Version 1
19
Clarify Question ● What is the key concept addressed by the question? ● The question asks about changes in the surface are to volume of a sphere. ● What type of thinking is required? ● You are being asked to apply your knowledge of changes in the surface are to volume of a sphere. Gather Content ● What do you know about changes in the surface are to volume of a sphere? What other information is related to the question? ● The advantage of small cell size is readily apparent in terms of the surfacearea-to-volume ratio. As a cell’s size increases, its volume increases much more rapidly than its surface area. For a spherical cell, the surface area is proportional to the square of the radius (r2), whereas the volume is proportional to the cube of the radius (r3). The ratio of r2 /r3=1/r. Choose Answer ● Given what you now know, what information is most likely to produce the correct answer? ● As the radius increases the volume will increase r times faster.Flipped around, the surface area to volume ratio will decrease by r as it increases.This makes it more difficult for molecules to diffuse through the cell. Reflect on Process ● Did your problem-solving process lead you to the correct answer? If not, where did the process break down or lead you astray? How can you revise your approach to produce a more desirable result? ● This question asked you to apply your understanding of changes in the surface are to volume of a sphere. If you got the correct answer, great job! If you got an incorrect answer, where did the process break down? Version 1
20
Did you recall that surface area is proportional to r2 and volume to r3?Could you set up the ratio of surface area to volume?
9) E 10) A 11) D 12) B 13) D
Version 1
21
Clarify Question ● What is the key concept addressed by the question? question asks about production of a cell wall.
● The
● What type of thinking is required? ● You are being asked to apply your knowledge of production of a cell wall. Gather Content ● What do you know about production of a cell wall? What other information is related to the question? ● Members of plants, fungi, bacteria and archea can all make cell walls. Choose Answer ● Given what you now know, what information is most likely to produce the correct answer? ● Animals all can move and do not make cell walls, as their cells need to be more flexible. Reflect on Process ● Did your problem-solving process lead you to the correct answer? If not, where did the process break down or lead you astray? How can you revise your approach to produce a more desirable result? ● This question asked you to apply your understanding of how cell walls are made. If you got the correct answer, great job! If you got an incorrect answer, where did the process break down? Did you recall that animals need to move and a cell wall would interfere with this? Did you understand that organisms in all of the other groups can make cell walls?
14) A 15) E
Version 1
22
Clarify Question ● What is the key concept addressed by the question? question asks about the movement of flagella.
● The
● What type of thinking is required? ● You are being asked to apply your knowledge of the movement of flagella. Gather Content ● What do you know about the movement of flagella? What other information is related to the question? ● Prokaryotes use flagella to move.When protons move through the flagellum it twists, creating a beating motion that moves the cell. Choose Answer ● Given what you now know, what information is most likely to produce the correct answer? ● If you inhibited the function of flagella, bacteria would not be able to move. Reflect on Process ● Did your problem-solving process lead you to the correct answer? If not, where did the process break down or lead you astray? How can you revise your approach to produce a more desirable result? ● This question asked you to apply your understanding of the movement of flagella. If you got the correct answer, great job! If you got an incorrect answer, where did the process break down? Did you recall that flagella are used by bacteria to move?Did you understand that blocking the activity of flagella would prevent the bacteria from moving?
16) D 17) E Version 1
23
Clarify Question ● What is the key concept addressed by the question? question asks about where water goes in a plant cell.
● The
● What type of thinking is required? ● You are being asked to apply your knowledge of where water goes in a plant cell. Gather Content ● What do you know about where water goes in a plant cell? What other information is related to the question? ● Plant cells have a cell wall which gives them some structure even when they dry out.So they can wilt or droop, but will still be somewhat firm.The water taken up by the plant cell will go into the vacuole and cause the plasma membrane up against the cell wall. Choose Answer ● Given what you now know, what information is most likely to produce the correct answer? ● A plant stores excess water in the vacuole to help maintain the structure of the plant. Reflect on Process ● Did your problem-solving process lead you to the correct answer? If not, where did the process break down or lead you astray? How can you revise your approach to produce a more desirable result? ● This question asked you to apply your understanding of where water goes in a plant cell. If you got the correct answer, great job! If you got an incorrect answer, where did the process break down? Did you recall that water is stored in the vacuole?Did you think that water went into the nucleus or cytoplasm?
Version 1
24
18) D 19) A 20) B 21) E 22) C 23) A 24) C 25) C 26) B 27) B 28) D
Version 1
25
Clarify Question ● What is the key concept addressed by the question? question asks about accumulation of fatty acids.
● The
● What type of thinking is required? ● You are being asked to apply your knowledge of the breakdown of fatty acids to explain a disease. Gather Content ● What do you know about fatty acid accumulation in cells? What other information is related to the question? ● A decrease in enzymes used to break down fatty acids would result in an increase of these fatty acids in a cell.This is most likely because the enzyme cannot cleave the fatty acids any more. ● Endoplasmic reticulum and ribosomes are used in protein synthesis. The Golgi is involved in protein transport.Vacuoles are used to store water and carbohydrates in cells. Choose Answer ● Given what you now know, what information is most likely to produce the correct answer? ● Peroxisomes contain enzymes used to digest fatty acids.If an enzyme needed to break down fatty acids was missing from the peroxisome then fatty acids would accumulate in the cell. Reflect on Process ● Did your problem-solving process lead you to the correct answer? If not, where did the process break down or lead you astray? How can you revise your approach to produce a more desirable result? ● This question asked you to apply your understanding of the role of the peroxisome in a disease. If you got the correct answer, great job! If you Version 1
26
got an incorrect answer, where did the process break down? Did you realize that a decrease in the amount of enzyme would cause an accumulation of the substrate it normally cleaved? Did you recall the function of the other organelles listed?
29) C 30) A 31) D 32) A 33) B 34) B 35) C 36) B 37) A
Version 1
27
Clarify Question ● What is the key concept addressed by the question? question asks about an enzyme in the lysosome.
● The
● What type of thinking is required? ● You are being asked to apply your knowledge of how proteins move into the lysosome to explain the results of an experiment. Gather Content ● What do you know about lysosome proteins? What other information is related to the question? ● Enzymes from the lysosome are first made in the endoplasmic reticulum, where they move through the Golgi, and then on to the lysosome. ● Proteins are not made in the nucleus. Proteins made in the cytoplasm remain in the cytoplasm. Some proteins are made in the mitochondria, but remain there. Choose Answer ● Given what you now know, what information is most likely to produce the correct answer? ● Proteins that are secreted or transported into organelles like the lysosome are first made in the endoplasmic reticulum. Reflect on Process ● Did your problem-solving process lead you to the correct answer? If not, where did the process break down or lead you astray? How can you revise your approach to produce a more desirable result? ● This question asked you to apply your understanding of protein secretion to explain where an enzyme in the lysosome was made. If you got the correct answer, great job! If you got an incorrect answer, where did the process break down? Did you recall that proteins made in the Version 1
28
endoplasmic reticulum are transported to other organelles like the lysosome? Did you think that proteins made in the cytoplasm could cross a membrane and enter an organelle like the lysosome?
38) B
Version 1
29
Clarify Question ● What is the key concept addressed by the question? question asks about the role of the cis Golgi.
● The
● What type of thinking is required? ● You are being asked to analyze an experiment to explain the effect of a drug that would block vesicle transportin the cis Golgi. Gather Content ● What do you know about vesicle transport? What other information is related to the question? ● Proteins made in the endoplasmic reticulum are packaged in vesicles and transported to the cis Golgi. In the Golgi the proteins have carbohydrates added. Next the proteins are packaged into vesicles that bud off of the trans Golgi for transport to the plasma membrane or other organelles. ● Lipids are made in the smooth endoplasmic reticulum, but this would not affect vesicle transport. Movement of vesicles to the plasma membrane comes from the trans Golgi. Vesicles don’t move from the mitochondria to the Golgi. Choose Answer ● Given what you now know, what information is most likely to produce the correct answer? ● The best answer is that vesicles released from the endoplasmic reticulum would not be able to fuse with the cis Golgi.Proteins and lipids made in the endoplasmic reticulum would not be able to enter the Golgi apparatus if the drug blocked this step. Reflect on Process ● Did your problem-solving process lead you to the correct answer? If not, where did the process break down or lead you astray? How can Version 1
30
you revise your approach to produce a more desirable result? ● This question asked you to analyze the results of an experiment to predict the impact of a drug that blocks transport through the cis Golgi.To do this you needed to understand the function of the cis Golgi and then what would happen if this function was blocked. If you got the correct answer, great job! If you got an incorrect answer, where did the process break down? Did you recall the function of the cis Golgi? Did you confuse it with the trans Golgi? Did you recall the function of the cis Golgi, but could not predict what would happen if the drug blocked the cis Golgi?
39) A 40) C 41) C 42) C
Version 1
31
Clarify Question ● What is the key concept addressed by the question? ● The question asks about the organelle involved in making enzymes used in protein degradation. ● What type of thinking is required? ● You are being asked to apply your knowledge of the function of organelles to explain how alcohol can block the production of enzymes used to break down proteins. Gather Content ● What do you know about where lysosome proteins are made? What other information is related to the question? ● The enzymes in the endoplasmic reticulum are made in the endoplasmic reticulum and transported to the lysosome. Once in the lysosome these enzymes can break down proteins. ● The key to this question is that alcohol blocks the synthesis of the enzymes used in protein degradation. Choose Answer ● Given what you now know, what information is most likely to produce the correct answer? ● The best answer is that alcohol blocks the endoplasmic reticulum from producing enzymes that move to the lysosome to degrade proteins. A common mistake would be to choose the lysosome, and if the question had stated that alcohol blocked the molecules involved in protein degradation this would be correct.However, the question states that alcohol blocks the synthesis of these molecules, which occurs in the endoplasmic reticulum. Reflect on Process ● Did your problem-solving process lead you to the correct answer? Version 1
32
If not, where did the process break down or lead you astray? How can you revise your approach to produce a more desirable result? ● This question asked you to apply your understanding of where lysosomal proteins are made to explain the effect of alcohol on the liver. If you got the correct answer, great job! If you got an incorrect answer, where did the process break down? Did you miss that the question was asking where the synthesis of enzymes involved in protein degradation was occurring? Did you think that alcohol was inhibiting the enzymes themselves and not their synthesis? Did you recall that proteins are made in the endoplasmic reticulum?
43) A
Version 1
33
Clarify Question ● What is the key concept addressed by the question? ● The question asks about the transport of proteins to the plasma membrane. ● What type of thinking is required? ● You are being asked to apply your knowledge of protein transport in a cell to predict the location of an enzyme involved in the process. Gather Content ● What do you know about protein transport to the plasma membrane? What other information is related to the question? ● Proteins targeted for delivery to the plasma membrane are made in the endoplasmic reticulum and then move through the Golgi and on to the plasma membrane through vesicles. ● Ribosomes are structures used to synthesize proteins by translating a messenger RNA. The nucleus contains DNA and is where transcription to synthesize RNA occurs. Choose Answer ● Given what you now know, what information is most likely to produce the correct answer? ● Proteins targeted for delivery to the plasma membrane are made in the endoplasmic reticulum and then move through the Golgi and on to the plasma membrane through vesicles.An enzyme that attaches fatty acids to proteins targeted for the plasma membrane would be found in the endoplasmic reticulum or Golgi.Addition of the fatty acid would make the protein more hydrophobic and bind to the membrane in the vesicle and eventually the plasma membrane. Reflect on Process ● Did your problem-solving process lead you to the correct answer? Version 1
34
If not, where did the process break down or lead you astray? How can you revise your approach to produce a more desirable result? ● This question asked you to apply your understanding of protein secretion to predict where an enzyme would be found. If you got the correct answer, great job! If you got an incorrect answer, where did the process break down? Did you recall which organelles were involved in protein secretion? Did you realize that adding a fatty acid to a protein would make the protein more hydrophobic and bind to a membrane? Did you think that the nucleus was involved in protein secretion?
44) B 45) B 46) B
Version 1
35
Clarify Question ● What is the key concept addressed by the question? question asks about the function of the trans Golgi.
● The
● What type of thinking is required? ● You are being asked to analyze the design of an experiment to predict what would happened if a drug blocked transport from the trans Golgi. Gather Content ● What do you know about transport to the plasma membrane? What other information is related to the question? ● Proteins targeted for the plasma membrane are first made in the endoplasmic reticulum and are then transported through vesicles to the cis Golgi. The proteins then leave through the trans Golgi to be transported to the plasma membrane. ● The drug that blocks transport from the trans Golgi is added right when a protein is in the Golgi. This means that proteins would not be able to enter vesicles from the Golgi and would be blocked from moving to the plasma membrane. Choose Answer ● Given what you now know, what information is most likely to produce the correct answer? ● Proteins leave the Golgi through the trans Golgi, so if this step is blocked the proteins will accumulate in the Golgi. Reflect on Process ● Did your problem-solving process lead you to the correct answer? If not, where did the process break down or lead you astray? How can you revise your approach to produce a more desirable result? ● This question asked you to analyze an experiment and predict what would Version 1
36
happen if a drug was added that blocks transport from the trans Golgi. To do this you needed to understand that the trans Golgi is where proteins leave the Golgi for the plasma membrane. If you got the correct answer, great job! If you got an incorrect answer, where did the process break down? Did you recall where the trans Golgi was or its function? Did you understand that a drug blocking the trans Golgi would prevent proteins from leaving the Golgi and getting to the plasma membrane?
47) A 48) D 49) C
Version 1
37
Clarify Question ● What is the key concept addressed by the question? ● The question asks about the types of junctions found in an intercalated disk. ● What type of thinking is required? ● You are being asked to apply your knowledge of the attributes found in each junction to predict which would be present in an intercalated disk. Gather Content ● What do you know about junctions? What other information is related to the question? ● There are several classes of junctions with different functions: ● Tight junctions prevent molecules from passing between cells, for example in the lining of the intestines. ● Adherens junctions connect cells together in areas of high mechanical stress that need to stretch like the skin. ● Desmosomes create strong flexible connections for tissues that need to move yet be strong such as the skin. ● Hemidesmosomes are involved in cell migration and are important in development. ● ap junctions create a pore between adjacent cells that allows small molecules to move from one cell to another, important in transmitting electrical impulses between muscle cells. ● Plasmodesmata are junctions found in plant cells that allow cytoplasmic proteins to be exchanged between cells. Choose Answer ● Given what you now know, what information is most likely to produce the correct answer? ● Because heart muscle cells need to contract and relax constantly for the life of the organism, the cells must be connected by strong flexible connections capable of withstanding Version 1
38
high mechanical stress. Desmosomes and adherens junctions are capable of these two functions respectively. In addition, once a region of the heart starts to contract, that electrical impulse must move through the wall of the heart so all of the muscle beats in a smooth rhythm. Gap junctions allow signaling molecules to move from one heart muscle cell to the next carrying this message and keeping the heart in rhythm. Reflect on Process ● Did your problem-solving process lead you to the correct answer? If not, where did the process break down or lead you astray? How can you revise your approach to produce a more desirable result? ● This question asked you to apply the definitions of different types of junctions to predict which would be present in the intercalated disks holding heart muscle cells together. If you got the correct answer, great job! If you got an incorrect answer, where did the process break down? Did you have trouble recalling the definitions of the different cellular junctions? Did you realize that there were three different junctions being described in the question?
50) A 51) D 52) B
Version 1
39
Clarify Question ● What is the key concept addressed by the question? question asks about the function of the ribosome.
● The
● What type of thinking is required? ● You are being asked to apply your knowledge of the function of a ribosome to explain what would happen to a cell if this function were inhibited. Gather Content ● What do you know about the ribosome? What other information is related to the question? ● Because streptomycin inhibits the 30S subunit of the ribosome, the ribosome will no longer be able to function properly.The function of the ribosome is to synthesize proteins from a messenger RNA in translation. ● The ribosome is not directly involved in oxygen, DNA, or ATP production in a cell. Choose Answer ● Given what you now know, what information is most likely to produce the correct answer? ● The best answer is that inhibition of the 30S subunit of a ribosome by streptomycin will block translation of a mRNA and thus impair protein synthesis. If a bacteria cannot make new proteins then it will not be able to divide, and will eventually die. Reflect on Process ● Did your problem-solving process lead you to the correct answer? If not, where did the process break down or lead you astray? How can you revise your approach to produce a more desirable result? ● This question asked you to apply your knowledge of the function of a ribosome to explain how streptomycin would work as an antibiotic. If you got the correct answer, great job! If you got an incorrect answer, Version 1
40
where did the process break down? Did you recognize that because the 30S subunit was part of the ribosome, if it was inhibited then the ribosome would be inhibited? Did you remember that the ribosome is involved in protein synthesis, so if the ribosome was inhibited protein synthesis would be impaired?
53) E
Version 1
41
Clarify Question ● What is the key concept addressed by the question? question asks about centrioles.
● The
● What type of thinking is required? ● You are being asked to apply your knowledge of the function of a centriole to predict what would happen to a cell if the centrioles were inhibited. Gather Content ● What do you know about centrioles? What other information is related to the question? ● Centrioles are found in the nucleus and they organize the microtubules that bind to chromosomes during cell division in mitosis or meiosis. ● The drug centrinone does not affect other structures in the cell that are involved in protein synthesis, photosynthesis or ATP production. Choose Answer ● Given what you now know, what information is most likely to produce the correct answer? ● Centrioles are involved in forming the spindles from the edges of the nucleus to chromosomes.These microtubule spindles then pull the chromosomes to separate ends of the cell during mitosis, an essential step in cell division.If the centrioles are inhibited then the cells cannot divide. Reflect on Process ● Did your problem-solving process lead you to the correct answer? If not, where did the process break down or lead you astray? How can you revise your approach to produce a more desirable result? ● This question asked you to apply the function of centrioles to predict what would happen to cells in which centriole activity was blocked. If you got the correct answer, great job! If you got an incorrect answer, where did Version 1
42
the process break down? Did you recall the function of centrioles? Could you predict what would happen if centriole activity was blocked? Did you think that centrioles had one of the other functions listed?
54) A
Version 1
43
Clarify Question ● What is the key concept addressed by the question? ● The question asks about comparing the Golgi in plant and animal cells. ● What type of thinking is required? ● You are being asked to analyze the results showing a difference in the size of the Golgi in plant and animal cells and provide an explanation. Gather Content ● What do you know about Golgi in plant and animal cells? What other information is related to the question? ● Animal cells have about 20 Golgi organelles, and these are involved in adding carbohydrates to proteins produced in the endoplasmic reticulum and being secreted or targeted to other organelles. Plant cells can have 100 Golgi organelles, and in addition to adding sugars to proteins, sugars are joined together to produce cell walls in the Golgi. ● The Golgi do not produce other organelles, but they do make some proteins that are transported to organelles like the lysosome. Choose Answer ● Given what you now know, what information is most likely to produce the correct answer? ● The biggest difference between plant and animal cells is that plant cells have a cell wall and animal cells do not. Cell walls are made of cellulose, which consists of many sugar residues attached to each other. The cell walls are made in the Golgi, so the plant cells need more Golgi organelles to keep up with the demand for cellulose for new cell walls. Reflect on Process ● Did your problem-solving process lead you to the correct answer? If not, where did the process break down or lead you astray? How can Version 1
44
you revise your approach to produce a more desirable result? ● This question asked you to analyze the results of an observation to explain why plant cells have more Golgi than animal cells. If you got the correct answer, great job! If you got an incorrect answer, where did the process break down? Did you recall that the cell wall is made in the Golgi of plant cells but not animal cells? Did you pick chloroplasts because they were present in plant cells but not animal cells?
55) C
Version 1
45
Clarify Question ● What is the key concept addressed by the question? ● The question asks about the glycogen storage disorder Pompe disease. ● What type of thinking is required? ● You are being asked to apply your knowledge of different organelles to predict which would be involved in the breakdown of large macromolecules like glycogen. Gather Content ● What do you know about breaking down large macromolecules like glycogen? What other information is related to the question? ● Enzymes used to break down large macromolecules are typically housed in the lysosome.If one of these enzymes has a mutation and no longer functions then the macromolecule that it degrades will accumulate. ● The endoplasmic reticulum and Golgi are involved in protein transport, but would not store enzymes used to break down large macromolecules. ● The nucleus would encode genes for these enzymes, but would not contain the enzymes themselves. ● Mitochondria are responsible for ATP production in the presence of oxygen. Choose Answer ● Given what you now know, what information is most likely to produce the correct answer? ● The best answer is that these enzymes would be present in the lysosome along with other digestive enzymes.The lysosome contains many enzymes, and if any of these are defective it can lead to the accumulation of large macromolecules like glycogen.The accumulation of these large macromolecules eventually inhibit the normal function of the cell, causing a disease state in the patient. Version 1
46
Reflect on Process ● Did your problem-solving process lead you to the correct answer? If not, where did the process break down or lead you astray? How can you revise your approach to produce a more desirable result? ● This question asked you to apply your understanding of the functions of different organelles to explain where a defective enzyme would be located. If you got the correct answer, great job! If you got an incorrect answer, where did the process break down? Did you understand that the question was asking for the location of enzymes involved in degrading large macromolecules? Did you think that the question was asking about where these enzymes would be made in the cell?
56) A
Version 1
47
Clarify Question ● What is the key concept addressed by the question? ● The question asks about carbon dioxide consumption in plant cells. ● What type of thinking is required? ● You are being asked to apply your knowledge about different organelles to explain which would be involved in removing carbon dioxide from the atmosphere. Gather Content ● What do you know about plants using carbon dioxide? What other information is related to the question? ● Plants use carbon dioxide during photosynthesis, converting carbon dioxide and water into glucose and oxygen. By increasing algae growth and photosynthesis, the theory is that carbon dioxide levels would decrease. ● Amyloplasts and the vacuole store starch and water in plant cells. ● Cellulose is made of glucose, so the carbon dioxide that is made into glucose will end up in cellulose (and in starch) eventually. However, these are the end products of the reaction, not the location where the reaction occurred. ● Mitochondria are responsible for the reverse reaction, using oxygen to break glucose into water and carbon dioxide. So increased mitochondria would lead to increased carbon dioxide release into the atmosphere. Choose Answer ● Given what you now know, what information is most likely to produce the correct answer? ● Photosynthesis occurs in chloroplasts, and specifically in the stroma of a chloroplast.As more algae grow they will produce more chloroplasts and more carbon dioxide will be converted into glucose, removing the carbon dioxide from the atmosphere. Version 1
48
Reflect on Process ● Did your problem-solving process lead you to the correct answer? If not, where did the process break down or lead you astray? How can you revise your approach to produce a more desirable result? ● This question asked you to apply your understanding of different organelles to explain why an increase in algae would lead to a decrease in carbon dioxide in the atmosphere. If you got the correct answer, great job! If you got an incorrect answer, where did the process break down? Did you understand that the reaction that plants use to remove carbon dioxide from the atmosphere was photosynthesis? Did you know that photosynthesis occurs in the stroma of chloroplasts? Did you realize that mitochondria perform the reverse reaction and release carbon dioxide into the atmosphere? Did you think that the storage of glucose in starch or cellulose would explain the decrease in carbon dioxide in the atmosphere?
57) B 58) C
Version 1
49
Clarify Question ● What is the key concept addressed by the question? ● The question asks about the location of genes encoding mitochondrial proteins. ● What type of thinking is required? ● You are being asked to apply your knowledge of the location of genes encoding mitochondrial proteins. Gather Content ● What do you know about the location of genes encoding mitochondrial proteins? What other information is related to the question? ● Mitochondriacontain their own chromosome that synthesizes some mitochondrial proteins.Other mitochondrial proteins are encoded by genes on the nucleus and are transported into the mitochondria.DNA is not found in the cytoplasm of eukaryotic cells. Choose Answer ● Given what you now know, what information is most likely to produce the correct answer? ● Mitochondrial proteins can be encoded by genes on nuclear and mitochondrial DNA but not cytoplasmic DNA. Reflect on Process ● Did your problem-solving process lead you to the correct answer? If not, where did the process break down or lead you astray? How can you revise your approach to produce a more desirable result? ● This question asked you to apply your understanding of the location of genes encoding mitochondrial proteins. If you got the correct answer, great job! If you got an incorrect answer, where did the process break down? Did you recall that mitochondria contain DNA?Did you understand that Version 1
50
genes encoded in the nucleus can also produce proteins that go to the mitochondria?
59) C 60) A
Version 1
51
Clarify Question ● What is the key concept addressed by the question? ● The question asks about the location of the stroma in a chloroplast. ● What type of thinking is required? ● You are being asked to apply your knowledge of the structure of a chloroplast to explain how many membranes are present. Gather Content ● What do you know about the structure of a chloroplast? What other information is related to the question? ● A chloroplast has two membranes much like a mitochondria, an outer membrane and an inner membrane. Inside of the chloroplast is the stroma and within it are suborganelles called thylakoids. ● The water molecule has crossed the outer membrane, so it is in the inter-membrane space between the outer and inner membranes. Choose Answer ● Given what you now know, what information is most likely to produce the correct answer? ● The water molecule only needs to cross the inner membrane to be in the stroma, so it must cross one membrane. Reflect on Process ● Did your problem-solving process lead you to the correct answer? If not, where did the process break down or lead you astray? How can you revise your approach to produce a more desirable result? ● This question asked you to apply your understanding of the structure of a chloroplast to explain the movement of water across its membranes. If you got the correct answer, great job! If you got an incorrect answer, where did the process break down? Did you recall the structure of the Version 1
52
chloroplast and that it contains an outer membrane, an inner membrane and a thylakoid membrane? Did you remember where the stroma was in the chloroplast? Did you think that the question asked how many total membranes were in a chloroplast?
61) B
Version 1
53
Clarify Question ● What is the key concept addressed by the question? question asks about the structure of a mitochondria.
● The
● What type of thinking is required? ● You are being asked to apply your knowledge of the structure of the mitochondria to explain how many membranes an ATP molecule will cross moving from the matrix to the cytoplasm. Gather Content ● What do you know about the structure of a mitochondria? What other information is related to the question? ● Mitochondria contain two membranes, an outer and an inner membrane. The center of the mitochondria is called the matrix. ● The ATP molecule is starting in the matrix and is moving to the cytoplasm, which is entirely outside of the mitochondria. Choose Answer ● Given what you now know, what information is most likely to produce the correct answer? ● Because a mitochondria has two membranes, for an ATP molecule to move from the matrix to the cytoplasm it will need to cross both the inner and outer mitochondrial membranes.Thus the ATP molecule will cross two membranes. Reflect on Process ● Did your problem-solving process lead you to the correct answer? If not, where did the process break down or lead you astray? How can you revise your approach to produce a more desirable result? ● This question asked you to apply your understanding of the structure of a mitochondria to explain how many membranes a molecule of ATP will need to cross to move from the matrix to the cytoplasm. If you got the Version 1
54
correct answer, great job! If you got an incorrect answer, where did the process break down? Did you understand the structure of a mitochondria? Did you recall that a mitochondria has an inner and an outer membrane? Did you remember that the matrix is inside of the mitochondria and the cytoplasm is on the outside of the mitochondria?
62) C
Version 1
55
Clarify Question ● What is the key concept addressed by the question? ● The question asks about the difference in size between prokaryotic and eukaryotic cells. ● What type of thinking is required? ● You are being asked to apply your knowledge of the structure of prokaryotic and eukaryotic cells to explain why the prokaryotic cells are smaller. Gather Content ● What do you know about the structure of cells? What other information is related to the question? ● Eukaryotes are diverse organisms that contain organelles, some can move (animals and some protists) while others have cells walls (fungi, plants, some protists).Some are multicellular (plants, animals, and fungi) while some can be single celled (plants, protists, fungi). ● While prokaryotes are single celled and do not contain organelles they have the normal functions of all cells, namely they can replicate, produce energy through metabolism, and have a plasma membrane. Choose Answer ● Given what you now know, what information is most likely to produce the correct answer? ● Some eukaryotes can move and some have a cell wall. Bacteria can make proteins and their choice of energy source should not influence their size. The best answer is that eukaryotic cells all contain organelles such as a nucleus, mitochondria, chloroplasts, lysosomes, etc. These internal structures take up space making eukaryotic cells larger than prokaryotic cells. Reflect on Process ● Did your problem-solving process lead you to the correct answer? Version 1
56
If not, where did the process break down or lead you astray? How can you revise your approach to produce a more desirable result? ● This question asked you to apply your understanding of the differences in the cell structure of prokaryotes and eukaryotes to explain why eukaryotes are larger. If you got the correct answer, great job! If you got an incorrect answer, where did the process break down? Did you have trouble recalling that eukaryotes have internal organelles?Did you forget that many eukaryotes can move or have cell walls like prokaryotes so this would not explain the difference in size?Did you think that prokaryotes could not make proteins, an essential function for any living cell?
63) A
Version 1
57
Clarify Question ● What is the key concept addressed by the question? ● The question asks about the volume and surface area of a cube. ● What type of thinking is required? ● You are being asked to apply your knowledge of the volume and surface area of a cube to calculate both. Gather Content ● What do you know about area and volume of a cube? What other information is related to the question? ● The volume of a cube is calculated by multiplying the height x width x depth of a cube. If the dimensions of the cube are given in millimeters (mm) then the final volume will be mm x mm x mm = mm3. ● To calculate the surface area of a cube first calculate the area of one face of the cube. This will be the height x the width. A cube has 6 faces, so you then multiply the area of one face x 6 to calculate the total surface area. If the dimensions of the cube are given in millimeters (mm) then the final area will be mm x mm = mm2. Choose Answer ● Given what you now know, what information is most likely to produce the correct answer? ● The volume of the cube will be 1mm x 1mm x 1mm =1mm3. ● The area of one face of the cube will be 1mm x 1mm = 1mm2. There are 6 faces on a cube, so the total area of the cube will be 6 x 1mm2 =6mm2. Reflect on Process ● Did your problem-solving process lead you to the correct answer? If not, where did the process break down or lead you astray? How can Version 1
58
you revise your approach to produce a more desirable result? ● This question asked you to apply your understanding of the volume and area of a cube to a specific example. If you got the correct answer, great job! If you got an incorrect answer, where did the process break down? Did you have trouble remembering the formulas for the volume of a cube and the area of a cube? Did you think that you needed to add the dimensions instead of multiplying them? Did you remember that a cube has 6 faces, so you need to multiply the area of one face times 6 to calculate the entire area?
64) D
Version 1
59
Clarify Question ● What is the key concept addressed by the question? question asks about the structure of a plant cell.
● The
● What type of thinking is required? ● You are being asked to apply your knowledge of the structure of a plant cell to explain what is happening inside of the cell in a wilted plant. ● What key words does the question contain and what do they mean? ● Wilting—when a plant becomes limp and droopy ● Contracted—to squeeze down to a smaller size Gather Content ● What do you know about the structure of a plant cell? What other information is related to the question? ● Plant cells contain a plasma membrane and a cell wall. The cell wall helps maintain structural support. However, if a plant lacks enough water the leaves of the plant will wilt and become droopy.Once the plant is watered it will swell back up and stand up straight again. ● The plasma membrane and cell wall are structural elements of a plant cell, but they do not contract or swell when a plant becomes wilted. The cytoplasm does contain water, but it is not the primary water storage organelle in a plant. Choose Answer ● Given what you now know, what information is most likely to produce the correct answer? ● The primary water storage organelle in a plant is the vacuole.When a plant becomes wilted, the vacuole contracts or squeezes in. This pushes water out of the plant cell causing it to lose volume.When enough plant cells lose volume the plant itself starts to droop or wilt.When you water the plant, the cells take up the Version 1
60
water in their vacuoles, causing the cells to swell up again. Once enough cells swell up the plant will regain its normal shape. Reflect on Process ● Did your problem-solving process lead you to the correct answer? If not, where did the process break down or lead you astray? How can you revise your approach to produce a more desirable result? ● This question asked you to apply your understanding of the structure of a plant to explain what happens inside a cell when a plant wilts. If you got the correct answer, great job! If you got an incorrect answer, where did the process break down? Did you understand that when a plant is wilted it is not getting enough water? Did you recall that the vacuole is responsible for storing water in a plant cell?Did you understand that when a vacuole contracts it pushes water out of the cell, causing the plant to become droopy and wilt?
Version 1
61
CHAPTER 5 CHECK ALL THE APPLY. Choose all options that best completes the statement or answers the question. 1) The cell's plasma membrane mediates which of the followingtransactions with the environment? Checkall that apply. A) ingesting food molecules and sometimes entire cells B) returning waste and other molecules back to the environment C) responding to a host of chemical cues D) directing synthesis of various food-digesting proteins E) passing messages to other cells
2) Receptor-mediated endocytosis brings in specific material by engulfing it in vesicles. What factors, either directly or indirectly, are related to this process? Choose all that apply. A) It is a passive process. B) It helps to increase membrane fluidity by helping to transport cholesterol into the cell. C) It is an active process. D) It uses carrier proteins. E) A clathrin coat is present.
3)
A concentration gradient is exhibited when _______.Choose all that apply.
A) the room temperature is 24 degree Celsius and the outside air is 34 degrees Celsius B) intracellular fluid has 5% solute and extracellular fluid has 0.8% solute concentration C) during rest, muscle and liver cells contain high glycogen concentration and blood contains lower concentrations D) the pCO2 changes from 45 to 35 mmHgin the alveoli of lungs
4) If ATP production is blocked, decreased transport of material across a membrane occurs. Which type of transport is affected? Choose all that apply.
Version 1
1
A) Osmosis B) Diffusion C) Facilitated diffusion D) Active transport E) Endocytosis
5) A symporter is moving glucose, and sodium levels are being depleted in extracellular fluids. What can be concluded about the movement of glucose? Choose all that apply A) Glucose would be moving against its gradient into the cell. B) Glucose follows the gradient of sodium into the cell. C) Glucose would be moving with its gradient into the cell. D) ATP is not required to move sodium down its concentration gradient. E) Energy is needed to move glucose down its concentration gradient. F) Glucose's movement would still be considered diffusion. G) Glucose's movement is considered a part of coupled transport.
6) Insulin stimulates an increase in number of glucose carrier proteins at a cell membrane’s surface. What effect will insulin have on cell function? Choose all that apply. A) An increase ininsulin, which will increase the diffusion rate. B) ATP amounts neededfor this transport will increase. C) Low insulin levelswill decrease the number of available carrier proteins. D) Lack of membranereceptors for insulinwill decrease amount of glucose taken in.
MULTIPLE CHOICE - Choose the one alternative that best completes the statement or answers the question. 7) The lipid layer that forms the foundation of cell membranes is primarily composed of molecules called __________.
Version 1
2
A) phospholipids B) fats C) proteins D) carbohydrates
8) While water continually orients phospholipids into a lipid bilayer, it does not fix the lipids permanently into position. Thus, the bilayer is considered to be _______. A) static B) fluid C) fluctuating D) charged
9)
The net movement of substances to regions of lower concentration is called A) osmosis. B) activetransport. C) facilitation. D) diffusion.
10) Osmosis is the diffusion of water across a membrane in response to the concentration of one or more of the ______. A) solutes B) membranes C) compartments D) cells
11) Which type of protein is embedded in the cell membrane in both active transport and facilitated transport?
Version 1
3
A) carrier B) cytoskeletal C) structural D) targeted
12) Osmosis occurs as water can cross the lipid bilayer through specialized channels for water movement called _________. A) proteins B) transmembranecarbohydrates C) membrane pores D) aquaporins
13)
The plasma membrane is a thin sheet of lipid embedded with A) proteins. B) carbohydrates. C) polymers. D) nucleotides. E) sodium and potassium ions.
14) Membrane proteins are not very soluble in water because they possess long stretches of nonpolar amino acids that are A) too long tointeract with the water molecules. B) hydrophobic. C) are transmembranal. D) hydrophilic. E) used as transportchannels.
Version 1
4
15) If a cell has the same concentration of dissolved molecules as its outside environment, the cell's condition is referred toas being A) isotonic. B) hypertonic. C) hypotonic. D) hydrophobic. E) hydrophilic.
16) A type of transport of a solute across a membrane, up its concentration gradient, using protein carriers driven by the expenditure of chemical energy is known as A) osmosis. B) diffusion. C) facilitated transport. D) active transport. E) exocytosis.
17)
How are the tails and heads of membrane phospholipids oriented in their environment?
A) The hydrophobic heads are oriented toward each other and the hydrophilic tails are oriented towards the extracellular fluid andthe intracellular fluid. B) The hydrophilic tails are oriented toward each other and the hydrophobic heads are oriented towards the extracellular fluid andthe intracellular fluid. C) The hydrophobic tails are oriented toward each other and the hydrophilic heads are oriented towards the extracellular fluid andthe intracellular fluid. D) The hydrophilic heads are oriented toward each other and the hydrophobic tails are oriented towards the extracellular fluid andthe intracellular fluid.
18)
The fluid nature of the membranes is attributed to a lateral movement of
Version 1
5
A) protein channels. B) phospholipid molecules. C) antigen molecules. D) pumps such as the proton pump. E) the entire lipid bilayer.
19) On the outer surface of the plasma membrane, there are marker molecules that identify the cell type. Often these molecules are A) ATP. B) amino acids. C) nucleotides. D) carbohydrate chains. E) inorganic ions.
20)
Which protein classes are not found as membrane proteins? A) transport channels B) hormones C) receptors D) enzymes E) identity markers
21) The part of a membrane protein that extends through the phospholipid bilayer is primarily composed of amino acids that are A) highly polar. B) negatively charged. C) nonpolar. D) positively charged. E) water soluble.
Version 1
6
22) The biconcave shape of red blood cells is determined by membrane-anchored internal proteins called A) clathrins. B) phospholipids. C) cytoskeleton. D) spectrins. E) actinfilaments.
23)
The movement of substances to regions of lower concentration is called A) active transport. B) diffusion. C) pinocytosis. D) pumping. E) exocytosis.
24) If two solutions have unequal concentrations of a solute, the solution with the lower concentration is called A) isotonic. B) hypertonic. C) hypotonic. D) osmosis.
25) In bacteria, fungi, and plants, the high internal pressure generated by osmosis is counteracted by the mechanical strength of their
Version 1
7
A) plasma membranes. B) organelles. C) cytoskeletons. D) cell walls.
26) The actual transport of protons by the proton pump is mediated by a transmembrane protein that undergoes a change in its A) conformation. B) amino acid sequence. C) net charge. D) solubility. E) immunity.
27)
The process often referred to as "cellular eating" is A) osmosis. B) pinocytosis. C) phagocytosis. D) diffusion. E) active transport.
28)
Carrier-mediated transport is also called A) facilitated diffusion. B) active transport. C) exocytosis. D) endocytosis. E) phagocytosis.
29)
Osmosis can only occur if water travels through the
Version 1
8
A) cell wall. B) semipermeable membrane. C) vacuole. D) ER. E) cytoskeleton.
30) You do an experiment in which you increase the concentration of phosphate in a solution and measure the uptake by plant cells. You find that once the concentration of phosphate reaches 1 gram per liter adding more phosphate does not increase uptake any further. From this you would conclude that phosphate is taken up by ______ . A) exocytosis B) facilitated diffusion C) active transport D) endocytosis E) osmosis
31)
The type of movement that is specific and requires carrier molecules and energy is A) exocytosis. B) facilitated diffusion. C) active transport. D) endocytosis. E) osmosis.
32)
In a single sodium-potassium pump cycle, ATP is used with the result that A) three sodium ions leave and two potassium ions enter. B) two sodium ion enters and one potassium ion leaves. C) two sodium ion leaves and one potassium ion enters. D) three sodium ions enter and two potassium ions leave. E) sodium and potassium ions enter and water leaves.
Version 1
9
33)
The accumulation of amino acids and sugars in animal cells can occur through A) ATP pumps. B) sodium-potassium pumps. C) glucose pumps. D) coupled transport. E) proton pumps.
34)
Cholesterol functions in the plasma membrane to A) transport ions. B) serve as an energy molecule. C) maintain fluidity. D) mediate steroid action. E) maintain hypertension.
35) A phospholipid molecule has a polar and a nonpolar end. Because of this, water molecules form A) polar bonds with the nonpolar end of the phospholipid molecule. B) polar bonds with the polar end of the phospholipid molecule. C) hydrogen bonds with the nonpolar end of the phospholipid molecule. D) hydrogen bonds with the polar end of the phospholipid molecule. E) covalent bonds with the nonpolar end of the phospholipid molecule.
36)
The fluid mosaic model proposed by Singer and Nicolson in 1972 explained that
Version 1
10
A) the cell membrane was composed of lipids and proteins. B) the cell membrane was composed of a phospholipid bilayer between two layers of globular proteins. C) the cell membrane was composed of a phospholipid bilayer with globular proteins actually inserted into the bilayer. D) the cell membrane was composed of a phospholipid bilayer but the polar ends of the phospholipid molecules were reversed.
37) Onefunction of membrane proteinsis transportation of substances across the membrane. If a cell biologist placed cells into an environment with an addedchemical that blocks the function of only carrier proteins, ______ will be affected. A) active transport B) osmosis C) simple diffusion D) phagocytosis E) pinocytosis
38)
For the process of diffusion to occur, molecules must
A) move from areas of high concentration to areas of lesser concentration until an equilibrium is reached. B) move from areas of low concentration to areas of higher concentration until an equilibrium is reached. C) remain stationary until their molecular motion allows for an equilibrium to be reached. D) move from areas of high concentration to areas of lesser concentration until facilitated transport can assist the molecular equilibrium. E) move from areas of high concentration to areas of lesser concentration until an equilibrium is reached by active transport.
39) Facilitated diffusion is an important method for cells in obtaining necessary molecules and removing other ones. Requirements for facilitated diffusion include which of the following?
Version 1
11
A) The carrier molecule must be specific to the molecule that is transported. The direction of movement is always with the concentration gradient, never against the gradient. B) The carrier molecule is nonspecific to the molecule that is transported. The direction of movement is always with the concentration gradient, never against the gradient. C) The carrier molecule is nonspecific to the molecule that is transported. The direction of movement is always against the concentration gradient, never with the gradient. D) The carrier molecule must be specific to the molecule that is transported and an ATP molecule must be attached to the specific carrier. The direction of movement is always against the concentration gradient, never with the gradient.
40) A research laboratory is attempting to collect the content of human red blood cells. The type of salt solution researchers should use to cause blood cells to lyse (burst) would be A) hyperosmotic. B) isosmotic. C) hypoosmotic. D) osmotic.
41) One day during the summer, you and some friends make ice cream using an electric ice cream maker. To get the ice cream to form, a mixture of salt and ice is packed around the ice cream maker. A few days later, you notice a yellow circle of dead grass where the ice cream maker had been placed. You hypothesize that some of the salt and ice mixture spilled onto the grass and ___. A) froze the grass B) was isosmotic to the grass cells causing the yellow circle of dead grass C) was hyperosmotic to the grass cells causing the yellow circle of dead grass D) was hypoosmotic to the grass cells causing the yellow circle of dead grass
42)
What is the main component of an animal cell membrane?
Version 1
12
A) phospholipids B) polynucleotides C) aquaporins D) glycolipids E) cholesterol
43) A scientist performs an experiment in which they create an artificial cell with a selectively permeable membrane through which only water can pass. They injecta 5 M solution of glucose into the cell and then place the cellinto a beaker of water. After an hour, what effect do you expect to observe? A) Water moves out of the cell B) Glucose moves out of the cell C) No net change in cell weight D) Water moves into the cell E) Glucose moves into the cell
44) A scientist performs an experiment in which they create an artificial cell with a selectively permeable membrane through which only water can pass. They inject a 5 M solution of glucose into the cell and then place the cellinto a beaker containing 10 M glucose. What effect do you expect to observe? A) Water moves out of the cell B) Glucose moves out of the cell C) No net change in cell weight D) Water moves into the cell E) Glucose moves into the cell
45)
Turgor pressure is observed when aplant cells environment is __________.
Version 1
13
A) hypotonic B) isotonic C) osmosis D) hypertonic
46)
What initiatesthe sodium-potassium pump?
A) three sodium ions bind to the cytoplasmic side of the protein. B) three sodium ions are translocated out of the cell. C) phosphorylated pump has low affinity for sodium allowing sodium to leave and potassium to bind. D) two potassium ions are transported into the cell. E) ATP binds to the protein which becomes phosphorylated (ADP is released).
47) Cyclodextrins are a group of compounds capable of removing cholesterol from membranes. The effect most likely to have a large impact on cells as a result of cyclodextrin treatment would be A) increasedconcentration of transmembrane proteins. B) increasedpermeability of the membrane. C) increased membranefluidity at lower temperatures. D) increased membranefluidity at higher temperatures. E) disruption oftransmembrane protein structure.
48)
Membrane fluidity is affected by A) temperature, cholesterol, and types of fatty acids. B) size of molecules, polarity of molecules, and temperature. C) types of fatty acids, and waters affinity for hydrogen bonding. D) hyrophilic tails, hydrophobic heads, and cholesterol.
Version 1
14
49) GABA receptors play an essential role in neurotransmission in the central nervous system. In response to the appropriate signal, these receptors open up and allow Cl– to flow into the cell. Given this information, GABA receptors can be classified as a A) gated ion channel. B) gated carrier protein. C) symporter. D) Cl – pump.
50) Protein X contains four transmembrane domains, a short N-terminus and a long Cterminus. Following protein synthesis, the N-terminus of the protein faces the lumen (the inside) of the ER. After protein X is transported to the cell surface via exocytosis, you would expect the N-terminus of protein X to be A) extracellular. B) cytoplasmic. C) in the lumen. D) in the lipid bilayer of the plasma membrane.
51) Incubation of cells at 20°C blocks the release of proteins from the trans-Golgi. Under such conditions, you would expect to see A) an increase in exocytosis. B) a decrease in exocytosis. C) an increase in endocytosis. D) a decrease in endocytosis.
52) While eating lobster for dinner one night your friend asks whatmight happen if this marine invertebrate, normally isosmotic with respect to its surrounding seawater, were moved to a bay with a slightly lower salt concentration instead of being caught. You reply that, in order for the lobster to be isosmotic with respect to new environment,
Version 1
15
A) it must move water in to match the solute concentration of its new environment. B) it regulates its internal concentration of solutes to match that of its new environment. C) it must move water out to match the solute concentration of its new environment. D) it moves water inor out and regulates its internal concentration of water as needed.
53) If a cell has a normal osmotic pressure of 4500 mmHg and is placed into a solution with an osmotic pressure of 3500 mmHg, what will happen to the cell? A) The cell will lose water and its osmotic pressure will decrease. B) The cell will gain water and its osmotic pressure will decrease. C) The cell will gain water and its osmotic pressure will increase. D) The cell will lose water and its osmotic pressure will increase.
54) In individuals with normal blood sugar levels glucose is reabsorbed into the bloodstream in the kidney by members of the GLUT transporter family. Glucose moves passively across an aqueous pore made by GLUT in the kidney membraneand as a result, no glucose is excreted in urine. In people with untreated diabetes mellitus, however, blood sugar levels are high and glucose is often present in the urine. What can explain this occurrence? A) High blood glucose levels reverse the concentration gradient, allowing untransported glucose to be excreted in urine. B) High blood glucose levels interfere with the coupled transport of water and glucose, allowing untransported glucose to be excreted in urine. C) The GLUT transporters become saturated, allowing untransported glucose to be excreted in urine. D) The GLUT transporters cannot hydrolyze ATP quickly enough for ATP to transport the extra glucose, thereby allowing untransported glucose to be excreted in urine.
55) Which mutation of a membrane protein could interfere with its proper insertion in the plasma membrane?
Version 1
16
A) A polar amino acid to a nonpolar amino acid within a transmembrane domain B) A nonpolar amino acid to a polar amino acid within a transmembrane domain C) A polar amino acid to a nonpolar amino acid within a cytoplasmic domain D) A nonpolar amino acid to a polar amino acid within a cytoplasmic domain
56) During neuronal signaling, a change in membrane potential will cause sodium channels to open and let Na + ions diffuse down their concentration gradient into the cell. Which of the following helps establish the concentration gradient necessary for this process to occur? A) Na +/K + channel B) voltage-gated Na + channel C) Na + symporter D) Na +/K + pump
57) If you were to replace the phosphate group of a phospholipid with a fatty acid chain, you would have A) glycerol. B) a triglyceride. C) cholesterol. D) glycolipid.
58) If you were a very thirsty cell, which process would you use to take in some nutrients secreted by one of your neighbors? A) phagocytosis B) pinocytosis C) receptor-mediated endocytosis D) exocytosis
59)
How can freeze-fracture be used to determine the orientation of a protein in a membrane?
Version 1
17
A) Freeze-fracture allows a cell to be cleaved in between the lipid bilayer, splitting the plasma membrane into two layers. B) Freeze-fracture allows a cell to be cleaved in such a way as to preserve the embedded proteins. C) Freeze-fracture cuts the membrane in protein-sized components for viewing under an electron microscope.
60) Diapedesis is the movement of white blood cells through intact blood vessel walls into surrounding body tissue. It typically happens when an area of the body is injured or damaged, and an inflammation response is required. What role would you expect cell adhesion molecules to play in this process? A) Cell adhesion molecules will allow the white blood cells to cling to one another to form a migrating clot. B) Cell adhesion molecules help white blood cells adhere to the blood vessel wall so that they can migrate across the wall and infiltrate into the underlying tissue. C) Cell adhesionmolecules will allow the white blood cells to attach to the injured area sothey become immobile.
61)
Membrane potential will affect the directional movement of A) glucose (C 6H 12O 6). B) cholesterol. C) potassium (K +). D) oxygen (O 2).
62)
The rate of diffusion of molecules would be fastest in a cell with an
Version 1
18
A) internalconcentration of45 percent and an external concentration of 50 percent incold temperatures. B) internalconcentration of 50 percent and an external concentration of 12 percent in coldtemperatures. C) internalconcentration of 50 percent and an external concentration of 12 percent in warmtemperatures. D) internalconcentration of 40 percent and an external concentration of 35 percent in warmtemperatures.
63) When the solute concentration of solution A is lower than the solute concentration ofsolution B, solution A is considered A) hypertonic. B) hypotonic. C) isotonic.
64) Tetrodotoxin is a potent poison, produced by some newts, pufferfish, and blue-ringed octopus that affects sodium transport involved with the voltage gates in neurons. A friend swimming in the Caribbean is pricked by a pufferfish and gets very sick. The most likely action the poison takes is A) mimicking a symporter and allowing sodium to rush out and potassium to rush in. B) mimicking an antiporter and allowing sodium to rush out. C) blocking an antiporter so sodium cannot rush in. D) blocking a symporter and keeping sodium from rushing in.
65) An individual is constipated. They take magnesium salts to help loosen the stool. Why does this work?
Version 1
19
A) Water moves from the intestines to the blood because the blood is hypertonic. B) Water moves fromthe intestines to the blood because the blood is hypotonic. C) Water moves from the blood to the intestines because the blood is hypertonic. D) Water moves fromthe blood to the intestines because the blood is hypotonic.
66) Cholesterol is a structural component of cell membranes. The amount of cholesterol present can affect membrane stability and permeability. What is therole of cholesterol in a membrane? A) A membrane thatneeds to be more rigid and more permeable, would have high cholesterolratios. B) A membrane thatneeds to be rigid, such as a protective barrier, would have low cholesterolratios. C) Membranes with low amounts ofcholesterol are very stable and have increased permeability. D) Membranes withhigh amounts of cholesterol are very stable and have decreasedpermeability.
67)
A dehydrated runner drinks a lot of water after a race. They rehydrate because A) the stomach ishypotonic compared to the bloodstream. B) the bloodstream is hypotoniccompared to the stomach. C) the stomach andbloodstream are isotonic compared to each other. D) water will diffuse from a low tohigh concentration.
68) To test for the presence of starch, iodine can be used. A positive reaction results in a black color. A dialysis bag filled with starch solution and secured on both ends is placed in iodine solution. After 30 minutes the dialysis bag appears black. Based on this information, what most accurately describes what happened?
Version 1
20
A) Due to its sizeand polarity, starch was able to exit the dialysis bag and interact with theiodine solution. B) Due to its sizeand polarity, iodine was able to enter the dialysis bag and interact withstarch. C) Over time, both iodine and starch are equally distributed throughout the twosolutions.
69) A mixed solution of water and small molecules is exposed to a cell membrane and allowed to interact for an hour. Which molecule would be most likely to cross the phospholipid bilayer without the aid of transport proteins and diffuse across the membrane the fastest? A) Water B) NH 4 + C) NH 3 D) CH 3COO – E) Na +
70)
If Na
+
levels were depleted in the extracellular fluid, rate of glucose movement would
A) increase. B) decrease. C) stay the same.
71) A gardener notices that many plants in his garden look wilted. What osmotic solution would you suggest the gardener use to best return the plants to full health? A) Place the plantsin a 15% solute solution so water can be actively transported out of the plantcells. B) Place the plantsin a 5% solute solution so water can be actively transported out of the plantcells. C) Place the plantsin a 5% solute solution so water can diffuse into the plant cells. D) Place the plantsin a 15% solute solution so water can diffuse out of the plant cells.
Version 1
21
72) Matt is studying how the protein transferrin enters cells. He examines cells that have taken up transferrin, and finds clathrin-coated vesicles. What mechanism was used to take transferrin into the cell? A) phagocytosis B) pinocytosis C) exocytosis D) receptor-mediator endocytosis
SECTION BREAK. Answer all the part questions. 73) Three (3) eggs are boiled and all weigh 2g. The eggs are placed in a 15% solution of NaCl and reweighed after 30 minutes. The results: Egg 1 weighed 2.5 grams Egg 2 weighed 2.0 grams Egg 3 weighed 1.8 grams
73.1)
The solution in the container is hypertonic to which egg?
A) egg 1 B) egg 2 C) egg 3
73.2)
Which egg is isotonic to the solution?
A) Egg 1 B) Egg 2 C) Egg 3
73.3)
Version 1
When compared to the solution, the egg that gained weight was ______.
22
A) hypertonic B) hypotonic C) isotonic
74)
The diagram shows the rate of diffusion into a cell using carrier proteins.
74.1)
Carrier proteins are saturated at which point?
A) 1 B) 2 C) 3 D) 4
74.2)
The rate of diffusion will increase as
A) the number ofglucose molecules decreases. B) the number ofglucose molecules increases. C) temperaturedecreases. D) pH rises.
Version 1
23
Answer Key Test name: Chap 05_3e 1) [A, B, C, E] 2) [B, C, E]
Version 1
24
Clarify Question ● What is the key concept addressed by the question? question asks about receptor-mediated endocytosis.
● The
● What type of thinking is required? ● You are being asked to apply your knowledge about receptor-mediated endocytosis. ● What key words does the question contain and what do they mean? ● Receptor-mediated endocytosis—transport of targeted molecules into the cell viaenvelopment by the plasma membrane Gather Content ● What do you know about receptor-mediated endocytosis? What other information is related to the question? ● Molecules are often transported into eukaryotic cells through receptor-mediated endocytosis. These molecules first bind to specific receptors in the plasma membrane—they have a conformation that fits snugly into the receptor. The portion of the receptor molecule that lies inside the membrane is trapped in an indented pit coated on the cytoplasmic side with the protein clathrin. Each pit acts like a molecular mousetrapto form an internal vesicle when the right molecule enters the pit. ● Large molecules, like low density lipoprotein that carry cholesterol through the blood, are taken up by this process. Choose Answer ● Given what you now know, what information is most likely to produce the correct answer? ● Receptor-mediated endocytosis is an active process in which clatherin coated pits bring in molecules bound to receptors on the surface of the cell.It is not passive diffusion of molecules through carrier proteins in the membrane. Reflect on Process Version 1
25
● Did your problem-solving process lead you to the correct answer? If not, where did the process break down or lead you astray? How can you revise your approach to produce a more desirable result? ● If you got the correct answer, great job! If you got an incorrect answer, where did the process break down? Did you recall that clathrin coated pits were involved? Did you think the process involved passive diffusion through a carrier protein?
3) [B, C, D]
Version 1
26
Clarify Question ● What is the key concept addressed by the question? question asks about concentration gradients.
● The
● What type of thinking is required? ● You are being asked to apply your knowledge about concentration gradients. ● What key words does the question contain and what do they mean? ● Concentration gradient—difference in concentration of substances across a barrier or distance Gather Content ● What do you know about concentration gradients? What other information is related to the question? ● Concentration gradients are formed when the concentration of a molecule is different on two sides of a membrane. Concentration can be measured by mass per volume, percentage or partial pressure for gasses. Choose Answer ● Given what you now know, what information is most likely to produce the correct answer? ● Temperature is not a unit of concentration. The other examples all show a difference in concentration across a membrane in either percentage, glucose concentration, or partial pressure of CO2 gas. Reflect on Process ● Did your problem-solving process lead you to the correct answer? If not, where did the process break down or lead you astray? How can you revise your approach to produce a more desirable result? ● If you got the correct answer, great job! If you got an incorrect answer, where did the process break down? Did you think that temperature was a measure of concentration? Did you think that pCO2 was not a Version 1
27
concentration?
4) [D, E]
Version 1
28
Clarify Question ● What is the key concept addressed by the question? question asks about forms of transport that require ATP.
● The
● What type of thinking is required? ● You are being asked to apply your knowledge about forms of transport that require ATP. ● What key words does the question contain and what do they mean? ● Adenosine triphosphate (ATP)—an energy molecule Gather Content ● What do you know about forms of transport that require ATP? What other information is related to the question? ● Movement down a concentration gradient is a passive process and does not require ATP. These include osmosis, diffusion and facilitated diffusion. Active transport uses ATP to move molecules against their concentration gradient. Endocytosis requires ATP to pinch the membrane forming a vesicle inside of the cell. Choose Answer ● Given what you now know, what information is most likely to produce the correct answer? ● Active transport and endocytosis require ATP, so if ATP formation is blocked they will cease. The other forms of transport do not require ATP as molecules move down their concentration gradient. Reflect on Process ● Did your problem-solving process lead you to the correct answer? If not, where did the process break down or lead you astray? How can you revise your approach to produce a more desirable result? ● If you got the correct answer, great job! If you got an incorrect answer, where did the process break down? Did you think that movement down Version 1
29
a concentration gradient requires ATP? Did you realize that endocytosis requires ATP?
5) [A, B, D, E, G]
Version 1
30
Clarify Question ● What is the key concept addressed by the question? ● The question asks about coupled transport of glucose and sodium. ● What type of thinking is required? ● You are being asked to analyze statements about coupled transport of glucose and sodium. ● What key words does the question contain and what do they mean? ● Symporter—specific class ofcarrier proteins Gather Content ● What do you know about coupled transport of glucose and sodium? What other information is related to the question? ● The active glucose transporter uses the Na+ gradient produced by the Na+/K+ pump as a source of energy to power the movement of glucose into the cell. In this system, both glucose and Na+ bind to the transport protein, which allows Na+ to pass into the cell down its concentration gradient, capturing the energy and using it to move glucose into the cell through a symporter. Choose Answer ● Given what you now know, what information is most likely to produce the correct answer? ● Na+ moves down its concentration gradient, moving glucose against its concentration gradient through coupled transport. The movement of Na+ ions down its gradient is a form of energy, so glucose is being moved by a form of active transport. Reflect on Process ● Did your problem-solving process lead you to the correct answer? If not, where did the process break down or lead you astray? How can you revise your approach to produce a more desirable result? ● If you got the correct answer, great job! If you got an incorrect answer, Version 1
31
where did the process break down? Did you think that sodium and glucose were transported by an antiporter? Did you think that glucose transport was not linked to Na+ transport?
6) [A, C, D]
Version 1
32
Clarify Question ● What is the key concept addressed by the question? question asks about glucose transport.
● The
● What type of thinking is required? ● You are being asked to analyze statements about glucose transport. ● What key words does the question contain and what do they mean? ● Insulin—metabolic hormone produced in the pancreas of mammals Gather Content ● What do you know about glucose transport? What other information is related to the question? ● Passive transport of glucose indicates that glucose is moving down its concentration gradient through the GLUT transporter. This means that it does not require ATP or cotransport of water. The GLUT transporter is a protein that makes a pore through the membrane of cells allowing glucose to enter. The number of glucose transporters on the surface of the cell is controlled in part by insulin. Insulin binds to receptors on the surface of the cell and causes more transporters to be present on the cell, increasing the rate of glucose diffusion into the cell. Choose Answer ● Given what you now know, what information is most likely to produce the correct answer? ● As insulin increases, more GLUT transporters appear on the surface of the cell, increasing the rate of glucose diffusion into the cell. If insulin levels or the number of insulin receptors drop there will be no increase in GLUT transporters on the surface of the cell and glucose uptake will decrease. Reflect on Process Version 1
33
● Did your problem-solving process lead you to the correct answer? If not, where did the process break down or lead you astray? How can you revise your approach to produce a more desirable result? ● If you got the correct answer, great job! If you got an incorrect answer, where did the process break down? Did you think that ATP was necessary for glucose diffusion into a cell? Did you think that the insulin receptor was not needed for glucose uptake?
7) A 8) B 9) D 10) A 11) A 12) D 13) A 14) B 15) A 16) D 17) C 18) B 19) D 20) B 21) C 22) D 23) B 24) C 25) D 26) A 27) C Version 1
34
28) A 29) B 30) B
Version 1
35
Clarify Question ● What is the key concept addressed by the question? ● The question asks about saturation of phosphate movement across a membrane. ● What type of thinking is required? ● You are being asked to apply your knowledge of saturation of phosphate movement across a membrane. ● What key words does the question contain and what do they mean? ● Phosphate—charged molecule Gather Content ● What do you know about saturation of phosphate movement across a membrane? What other information is related to the question? ● Phosphatecannot move across a membrane by simple diffusion because it is polar.Because increasing the external concentration of phosphate increases uptake by the cell, this is most likely diffusion down a concentration gradient and not active transport. Osmosis is the movement of water across membranes, not phosphate ions.Endocytosis and exocytosis are used to move large bulky molecules across membranes. Choose Answer ● Given what you now know, what information is most likely to produce the correct answer? ● The best answer is facilitated diffusion because increasing external phosphate concentration will increase movement of phosphate across the membrane through transporters.Once all transporters are occupied by phosphate ions facilitated diffusion is saturated and increasing the phosphate concentration will not increase uptake. Version 1
36
Reflect on Process ● Did your problem-solving process lead you to the correct answer? If not, where did the process break down or lead you astray? How can you revise your approach to produce a more desirable result? ● This question asked you to analyze explanations about saturation of phosphate movement across a membrane. If you got the correct answer, great job! If you got an incorrect answer, where did the process break down? Did you think that phosphate was taken up by diffusion or osmosis?Did you understand that diffusion is the movement of molecules down a concentration gradient?
31) C 32) A 33) D 34) C 35) D 36) C 37) A
Version 1
37
Clarify Question ● What is the key concept addressed by the question? question asks about carrier proteins.
● The
● What type of thinking is required? ● You are being asked to apply your knowledge of carrier proteins to predict which process will be affected if they are inhibited. ● What key words does the question contain and what do they mean? ● Carrier proteins—channels that use ATP to move molecules across a membrane Gather Content ● What do you know about transport of molecules across membranes? What other information is related to the question? ● Molecules that can move directly through a membrane do so by simple diffusion, or osmosis in the case of water. ● Large macromolecules or fluids enter cells through phagocytosis or pinocytosis in which vesicles bud off of the plasma membrane, bringing the molecules into the cell. Choose Answer ● Given what you now know, what information is most likely to produce the correct answer? ● Osmosis and diffusion do not require proteins. Carrier proteins use ATP to move molecules across the membrane by active transport. Therefore, inhibiting carrier proteins will inhibit active transport. Reflect on Process ● Did your problem-solving process lead you to the correct answer? If not, where did the process break down or lead you astray? How can you revise your approach to produce a more desirable result? ● If Version 1
38
you got the correct answer, great job! If you got an incorrect answer, where did the process break down? Did you recall that carrier proteins are used to move molecules across membranes in active transport? Did you understand that molecules capable of osmosis or simple diffusion do not require proteins to move across membranes? Did you recall that pinocytosis and phagocytosis move molecules across membranes by forming vesicles?
38) A 39) A 40) C
Version 1
39
Clarify Question ● What is the key concept addressed by the question? question asks about osmotic concentration.
● The
● What type of thinking is required? ● You are being asked to apply your knowledge of osmotic concentration to choose what solution would cause a red blood cell to lyse. ● What key words does the question contain and what do they mean? ● Cell lysis—bursting of cell due to extreme excess of water Gather Content ● What do you know about osmotic concentration? What other information is related to the question? ● Hyperosmotic solutions would contain more salt than the cells, hyper isGreek for "more than". ● Hypoosmotic solutions would contain less salt than the cells, hypo is Greek for "less than". ● Isoosmotic solutions would contain the same salt concentration as the cells, iso is Greek for "equal". ● Water moves across membranes by osmosis. Water will move from an area of lower solute concentration to an area of higher solute concentration until the concentrationis equal. Choose Answer ● Given what you now know, what information is most likely to produce the correct answer? ● If you want red blood cells to swell, you want water to move into the blood cells. For this to happen the cells need to have more salt inside them than the surrounding solution. The solution would then be hypoosmotic relative to the cells, and contain less salt than the cells. Reflect on Process Version 1
40
● Did your problem-solving process lead you to the correct answer? If not, where did the process break down or lead you astray? How can you revise your approach to produce a more desirable result? ● If you got the correct answer, great job! If you got an incorrect answer, where did the process break down? Did you recall that for a cell to swell up it would need to be in a solution with a lower salt concentration than the inside of the cell? Did you remember what the roots hyper, hypo and iso mean? Did you understand that in osmosis water moves from an area of lower salt to higher salt?
41) C
Version 1
41
Clarify Question ● What is the key concept addressed by the question? question asks about osmotic concentration.
● The
● What type of thinking is required? ● You are being asked to analyze an observation and explain what happened to grass exposed to salt. ● What key words does the question contain and what do they mean? ● Ice and salt—when making ice cream, salt is mixed with ice to make a super cooled solution that help the ice cream form Gather Content ● What do you know about osmotic concentration? What other information is related to the question? ● Plants are normally hyperosmotic to the water in the ground. This causes water to move into the plant cells through osmosis. When salt water comes into contact with the grass this process is reversed and water moves from the inside of the plant (lower salt concentration) to the ground (now with a higher salt concentration). ● Hyperosmotic solutions would contain more salt than the cells, hyper isGreek for "more than". ● Hypoosmotic solutions would contain less salt than the cells, hypo is Greek for "less than". ● Isoosmotic solutions would contain the same salt concentration as the cells, iso is Greek for "equal". Choose Answer ● Given what you now know, what information is most likely to produce the correct answer? ● The salt and ice mixture has a higher concentration of salt than the grass cells, so it is hyperosmotic. The Version 1
42
higher salt concentration pulls water from the grass, causing it to die. Reflect on Process ● Did your problem-solving process lead you to the correct answer? If not, where did the process break down or lead you astray? How can you revise your approach to produce a more desirable result? ● If you got the correct answer, great job! If you got an incorrect answer, where did the process break down? Did you realize that it was the salt in the salt and ice mixture that was killing the grass? Did you understand that the ice and salt mixture was hyperosmotic relative to the grass cells?
42) A 43) D
Version 1
43
Clarify Question ● What is the key concept addressed by the question? question asks about osmotic concentration.
● The
● What type of thinking is required? ● You are being asked to apply your understanding of osmotic concentration to predict what will happen to a cell if it is injected with 5M glucose. ● What key words does the question contain and what do they mean? ● 5M glucose—M is molar or moles per liter, glucose is a polar molecule and will not be able to cross the membrane Gather Content ● What do you know about osmotic concentration? What other information is related to the question? ● Increasing the amount of glucose in the synthetic cell will increase its osmotic concentration. ● With an increased osmotic concentration of glucose inside of the cell, the cell will be hyperosmotic and the water hypoosmotic. Water moves from areas of lower osmotic concentration to higher osmotic concentration until the osmotic concentrations on both sides of the membrane are the same. Choose Answer ● Given what you now know, what information is most likely to produce the correct answer? ● Since the osmotic concentration on the inside of the cell is higher with the 5 M glucose added, the water will move into the cell by osmosis. Reflect on Process ● Did your problem-solving process lead you to the correct answer? If not, where did the process break down or lead you astray? How can you revise your approach to produce a more desirable result? ● If Version 1
44
you got the correct answer, great job! If you got an incorrect answer, where did the process break down? Did you understand what would happen to the osmotic concentration inside the cell when it was injected with 5 M glucose? Did you recall that water will move from an area of lower to higher osmotic concentration? Did you recall that a semipermeable membrane will not allow polar molecules like glucose to cross while water can cross the membrane?
44) A
Version 1
45
Clarify Question ● What is the key concept addressed by the question? question asks about osmotic concentration.
● The
● What type of thinking is required? ● You are being asked to apply your understanding of osmotic concentration to predict what will happen to a cell if it is injected with 5M glucose and then placed into a solution that is 10M glucose. ● What key words does the question contain and what do they mean? ● 5 M glucose—M is molar or moles per liter,glucose is a polar molecule and will not be able to cross the membrane Gather Content ● What do you know about osmotic concentration? What other information is related to the question? ● Increasing the amount of glucose in the synthetic cell to 5 M will increase its osmotic concentration relative to water. ● When the cell is placed into a 10 M solution, the cell will have a lower osmotic concentration relative to the 10 M solution. ● With a decreased osmotic concentration of glucose inside of the cell, the cell will be hypoosmotic and the 10 M solution hyperosmotic. Water moves from areas of lower osmotic concentration to higher osmotic concentration until the osmotic concentrations on both sides of the membrane are the same. Choose Answer ● Given what you now know, what information is most likely to produce the correct answer? ● Since the osmotic concentration on the inside of the cell (5 M) is lower than the solution (10 M), the water will move out of the cell by osmosis. Version 1
46
Reflect on Process ● Did your problem-solving process lead you to the correct answer? If not, where did the process break down or lead you astray? How can you revise your approach to produce a more desirable result? ● If you got the correct answer, great job! If you got an incorrect answer, where did the process break down? Did you understand what would happen to the osmotic concentration inside the cell when it was injected with 5 M glucose? Did you recall that water will move from an area of lower to higher osmotic concentration? Did you recall that a semipermeable membrane will not allow polar molecules like glucose to cross while water can cross the membrane?
45) A 46) A 47) D
Version 1
47
Clarify Question ● What is the key concept addressed by the question? ● The question asks about the role of cholesterol in a membrane. ● What type of thinking is required? ● You are being asked to apply your knowledge of the role of cholesterol in a membrane to predict what would happen if it were removed. ● What key words does the question contain and what do they mean? ● Cholesterol in membranes—steroid with a polar hydroxyl group andcomponent of plasma membranes Gather Content ● What do you know about the role of cholesterol in membranes? What other information is related to the question? ● Biological membranes are usually comprised of mostly phospholipids. Cholesterol in a membrane will prevent the fatty acid tails of the phospholipids from packing together very tightly. This keeps the membrane more fluid. ● The drug cyclodextrin removes cholesterol from the membrane. Because cholesterol makes the membrane fluid, removal of cholesterol could make the membrane less fluid. Choose Answer ● Given what you now know, what information is most likely to produce the correct answer? ● The best answer is that removal of cholesterol would decrease membrane fluidity. ● Decreased membrane fluidity should not affect the abundance or activity of membrane proteins. ● Decreased membrane fluidity would also decrease rather than increase permeability and osmosis. Reflect on Process Version 1
48
● Did your problem-solving process lead you to the correct answer? If not, where did the process break down or lead you astray? How can you revise your approach to produce a more desirable result? ● If you got the correct answer, great job! If you got an incorrect answer, where did the process break down? Did you recall that cholesterol increases membrane fluidity? Could you predict then that removal of cholesterol would decrease membrane fluidity? Did you understand that decreased fluidity would decrease permeability or osmosis and not increase these activities?
48) A 49) A
Version 1
49
Clarify Question ● What is the key concept addressed by the question? ● The question asks about the transport of molecules across a membrane. ● What type of thinking is required? ● You are being asked to apply your knowledge of transport proteins to explain how Cl- ions cross a membrane. ● What key words does the question contain and what do they mean? ● GABA receptors—proteins in a membrane that bind to a ligand, in this case the neurotransmitter GABA ● Cl-—chloride ion with a negative charge Gather Content ● What do you know about protein transporters? What other information is related to the question? ● Charged and polar molecules cannot cross a membrane without the assistance of a protein channel. GABA receptors can bind a ligand (GABA) and then open up a channel that allows Cl- ions to move through. In this way the GABA receptor is acting as a gate that can open to allow ions to flow through, down their concentration gradient. ● Symporters open channels that allow two molecules to enter a cell simultaneously. One of the molecules is flowing down its concentration gradient and that energy is used to move the second molecule against its concentration gradient. In the case of the GABA receptor, no mention is made of a second molecule moving through the channel. ● A pump is a protein that uses ATP or another energy source to drive active transport of a molecule against its concentration gradient. No mention is made of ATP being used in the case of the GABA receptor.
Version 1
50
Choose Answer ● Given what you now know, what information is most likely to produce the correct answer? ● The best answer is that the GABA receptor is a gated ion channel. When it is closed, no ions can move across the channel. When GABA binds to the receptor, the channel opens and the Cl- ion can move down its concentration gradient and enter the neuron. Reflect on Process ● Did your problem-solving process lead you to the correct answer? If not, where did the process break down or lead you astray? How can you revise your approach to produce a more desirable result? ● If you got the correct answer, great job! If you got an incorrect answer, where did the process break down? Did you understand that Cl- was an ion? Did you recognize that the GABA receptor was acting as a gate, preventing Cl- ions from crossing the membrane until GABA bound to the receptor? Did you observe that no mention was made of ATP or another molecule moving across the membrane?
50) A
Version 1
51
Clarify Question ● What is the key concept addressed by the question? ● The question asks about the orientation of a protein in a membrane. ● What type of thinking is required? ● You are being asked to apply your knowledge of how proteins are oriented in a membrane and the process of exocytosis. ● What key words does the question contain and what do they mean? ● Transmembrane domain—region of membrane protein that cross the plasma membrane Gather Content ● What do you know about how proteins are oriented in a membrane? What other information is related to the question? ● When a protein is made in the endoplasmic reticulum, the part facing the inside is said to be in the lumen. Proteins move from the endoplasmic reticulum to the plasma membrane through small lipid vesicles that pinch off the endoplasmic reticulum. Proteins facing the lumen will remain on the inside of these vesicles. When the vesicles fuse with the plasma membrane the contents that were inside the vesicle willbe outside of the cell. Similarly the proteins attached to the vesiclemembrane will now be facing outside of the cell as part of the plasma membrane. Choose Answer ● Given what you now know, what information is most likely to produce the correct answer? ● Proteins on the inside or lumen of the endoplasmic reticulum will be on the extracellular or outside of the membrane after being transported through vesicles. Reflect on Process Version 1
52
● Did your problem-solving process lead you to the correct answer? If not, where did the process break down or lead you astray? How can you revise your approach to produce a more desirable result? ● If you got the correct answer, great job! If you got an incorrect answer, where did the process break down? Did you think that proteins on the inside of a vesicle remained on the inside of a cell when it fused with the plasma membrane? Did you think that the protein would be buried in the lipid bilayer?
51) B 52) B
Version 1
53
Clarify Question ● What is the key concept addressed by the question? ● The question asks about adaptation to a change in the salinity. ● What type of thinking is required? ● You are being asked to apply your knowledge about adaptation to a change in the salinity. ● What key words does the question contain and what do they mean? ● Isosmotic—same osmotic concentration Gather Content ● What do you know about adaptation to a change in the salinity? What other information is related to the question? ● Some organisms that live in the ocean adjust their internal concentration of solutes to match that of the surrounding seawater. Because they are isosmotic with respect to their environment, no net flow of water occurs into or out of these cells. Choose Answer ● Given what you now know, what information is most likely to produce the correct answer? ● If the lobster allowed a lot of water to enter its body to keep the salt concentration equal to that of the surrounding water it would become swollen. Instead it adjusts its internal concentration of salts to maintain water balance. Reflect on Process ● Did your problem-solving process lead you to the correct answer? If not, where did the process break down or lead you astray? How can you revise your approach to produce a more desirable result? ● If you got the correct answer, great job! If you got an incorrect answer, where did the process break down? Did you think the lobster would take in large amounts of water to maintain salt balance?Did you think that the Version 1
54
lobster could secrete water from its body?
53) A
Version 1
55
Clarify Question ● What is the key concept addressed by the question? ● The question asks about how a cell responds to differences in osmotic pressure. ● What type of thinking is required? ● You are being asked to apply your knowledge about how a cell responds to differences in osmotic pressure. ● What key words does the question contain and what do they mean? ● Osmotic pressure—the force needed to stop osmotic flow Gather Content ● What do you know about how a cell responds to differences in osmotic pressure? What other information is related to the question? ● Water moves from an area of high pressure to low pressure. This will occur until the pressure in both regions is equal. Choose Answer ● Given what you now know, what information is most likely to produce the correct answer? ● If the inside of the cell is 4500 mmHg and the surrounding fluid was 3500 mmHg then water would leave the cell until it also reached a pressure of 3500 mmHg. Thus the pressure inside the cell would decrease. Reflect on Process ● Did your problem-solving process lead you to the correct answer? If not, where did the process break down or lead you astray? How can you revise your approach to produce a more desirable result? ● If you got the correct answer, great job! If you got an incorrect answer, where did the process break down? Did you think that water would move from low pressure to high pressure? Did you think that a cell Version 1
56
losing water would experience an increase in pressure?
54) C
Version 1
57
Clarify Question ● What is the key concept addressed by the question? ● The question asks about glucose movement through the GLUT transporter. ● What type of thinking is required? ● You are being asked to apply your knowledge about glucose movement through the GLUT transporter. Gather Content ● What do you know about glucose movement through the GLUT transporter? What other information is related to the question? ● Passive transport of glucose indicates that glucose is moving down its concentration gradient through the GLUT transporter. This means that it does not require ATP or cotransport of water. The GLUT transporter is a protein that makes a pore through the membrane of kidney cells. Choose Answer ● Given what you now know, what information is most likely to produce the correct answer? ● Because the GLUT transporter forms a pore of a given size, only so many glucose molecules can move through it at a time. In diabetes blood glucose levels increase dramatically. So if glucose levels increase, not all of the glucose molecules may be reabsorbed in time and some will be secreted in the urine. Reflect on Process ● Did your problem-solving process lead you to the correct answer? If not, where did the process break down or lead you astray? How can you revise your approach to produce a more desirable result? ● If you got the correct answer, great job! If you got an incorrect answer, where did the process break down? Did you think that passive Version 1
58
movement of glucose through the GLUT transporter required ATP? Did you think that elevating the glucose level would reverse the glucose concentration gradient?
55) B
Version 1
59
Clarify Question ● What is the key concept addressed by the question? ● The question asks about the amino acids found in a transmembrane domain. ● What type of thinking is required? ● You are being asked to apply your knowledge about the amino acids found in a transmembrane domain. ● What key words does the question contain and what do they mean? ● Membrane protein—protein embedded within the plasma membrane Gather Content ● What do you know about the amino acids found in a transmembrane domain? What other information is related to the question? ● Transmembrane domains contain nonpolar amino acids because the membrane itself is nonpolar. Polar amino acids are found more on the cytoplasmic portions of a protein that interact with water. Choose Answer ● Given what you now know, what information is most likely to produce the correct answer? ● If a nonpolar amino acid was changed to a polar amino acid in a transmembrane domain it would interfere with the insertion of a protein into a membrane. Changes in the cytoplasmic domain would not affect this process and nonpolar amino acids will interact better than the polar amino acids with the membrane. Reflect on Process ● Did your problem-solving process lead you to the correct answer? If not, where did the process break down or lead you astray? How can you revise your approach to produce a more desirable result? ● If you got the correct answer, great job! If you got an incorrect answer, Version 1
60
where did the process break down? Did you think that polar amino acids would be found on a membrane? Did you think that the inside of a membrane was polar?
56) D 57) B
Version 1
61
Clarify Question ● What is the key concept addressed by the question? question asks about the structure of a phospholipid.
● The
● What type of thinking is required? ● You are being asked to apply your knowledge about the structure of a phospholipid. Gather Content ● What do you know about the structure of a phospholipid? What other information is related to the question? ● Phospholipids contain two fatty acid chains attached to a glycerol backbone. There is also a phosphate attached to the glycerol. Choose Answer ● Given what you now know, what information is most likely to produce the correct answer? ● If the phosphate was replaced with a fatty acid chain then the glycerol would have three fatty acids attached to it, forming a triglyceride. Reflect on Process ● Did your problem-solving process lead you to the correct answer? If not, where did the process break down or lead you astray? How can you revise your approach to produce a more desirable result? ● If you got the correct answer, great job! If you got an incorrect answer, where did the process break down? Did you recall that phospholipids have two fatty acid chains? Did you recall that triglycerides have three fatty acid chains attached to a glycerol backbone?
58) B 59) A Version 1
62
60) B
Version 1
63
Clarify Question ● What is the key concept addressed by the question? ● The question asks about the role of adhesion molecules in white blood cell movement. ● What type of thinking is required? ● You are being asked to apply your knowledge about the role of adhesion molecules in white blood cell movement. ● What key words does the question contain and what do they mean? ● White blood cells—motile immune cells Gather Content ● What do you know about the role of adhesion molecules in white blood cell movement? What other information is related to the question? ● Adhesion molecules allow two cells to attach to each other. In some tissues, these are very stable long term attachments. Blood cells need to be able to move, so adhesion molecule attachments are short term for these cells. Choose Answer ● Given what you now know, what information is most likely to produce the correct answer? ● By binding to adhesion molecules on cells lining the blood vessels, white blood cells can crawl out of the blood vessel and into the surrounding tissues. Reflect on Process ● Did your problem-solving process lead you to the correct answer? If not, where did the process break down or lead you astray? How can you revise your approach to produce a more desirable result? ● If you got the correct answer, great job! If you got an incorrect answer, where did the process break down? Did you think that white blood cells Version 1
64
binding to a region of damage would be immobile? Did you think that white blood cells would bind to each other in the blood stream?
61) C
Version 1
65
Clarify Question ● What is the key concept addressed by the question? ● The question asks about the impact of membrane potential on the movement of molecules. ● What type of thinking is required? ● You are being asked to analyze statements about the impact of membrane potential on the movement of molecules. ● What key words does the question contain and what do they mean? ● Membrane potential—electric potential difference across the membrane Gather Content ● What do you know about the impact of membrane potential on the movement of molecules? What other information is related to the question? ● Membrane potentialswill affect the movement of charged ions across the membrane with positively charged ions moving towards the side of the membrane with a net negative charge and negatively charged ions moving towards the net positively charged side. Choose Answer ● Given what you now know, what information is most likely to produce the correct answer? ● Of the options, only potassium is an ion and will be affected by a membrane potential, moving towards the negatively charged side of the membrane. Reflect on Process ● Did your problem-solving process lead you to the correct answer? If not, where did the process break down or lead you astray? How can you revise your approach to produce a more desirable result? ● If you got the correct answer, great job! If you got an incorrect answer, Version 1
66
where did the process break down? Did you realize that a membrane potential involved different charges across a membrane? Did you think that molecules without a charge would be affected by the membrane potential?
62) C
Version 1
67
Clarify Question ● What is the key concept addressed by the question? ● The question asks about factors that affect the rate of diffusion. ● What type of thinking is required? ● You are being asked to apply your knowledge about factors that affect the rate of diffusion. ● What key words does the question contain and what do they mean? ● Diffusion—movement of substances from regions of high to low concentration Gather Content ● What do you know about factors that affect the rate of diffusion? What other information is related to the question? ● Diffusion relies on the rapid movement of molecules. This occurs most rapidly in warmer temperatures when the molecules are moving faster. The greater the difference in concentration across a membrane, the larger the gradient and the faster molecules will diffuse. Choose Answer ● Given what you now know, what information is most likely to produce the correct answer? ● The answer in which the internal concentration of 50 percent and an external concentration of 12 percent in warm temperatures would have the fastest diffusion. The warm temperatures and large gradient would both make diffusion faster in these conditions. Reflect on Process ● Did your problem-solving process lead you to the correct answer? If not, where did the process break down or lead you astray? How can you revise your approach to produce a more desirable result? ● If you got the correct answer, great job! If you got an incorrect answer, Version 1
68
where did the process break down? Did you think that molecules diffuse most rapidly in the cold? Did you think that molecules diffuse most rapidly down smaller gradients?
63) B 64) C
Version 1
69
Clarify Question ● What is the key concept addressed by the question? ● The question asks about the effects of tetrodotoxin on ion movement in neurons. ● What type of thinking is required? ● You are being asked to evaluate statements about the effects of tetrodotoxin on ion movement in neurons. ● What key words does the question contain and what do they mean? ● Voltage gates—ion channels allowing specific ions across the membrane ● Neurons—signaling cells of the nervous system Gather Content ● What do you know about the effects of tetrodotoxin on ion movement in neurons? What other information is related to the question? ● When a neuron fires, sodium ions enter the neuron and potassium ions leave the neuron. Both move in opposite direction through an antiporter. Tetrodotoxin blocks sodium entry into the neuron causing paralysis. Choose Answer ● Given what you now know, what information is most likely to produce the correct answer? ● Because tetrodotoxin blocks sodium from entering the cell it is acting on an antiporter. If it mimicked a symporter or antiporter then it would stimulate sodium ion transport. If sodium and potassium both entered a cell then the transporter would by a symporter. Reflect on Process ● Did your problem-solving process lead you to the correct answer? Version 1
70
If not, where did the process break down or lead you astray? How can you revise your approach to produce a more desirable result? ● If you got the correct answer, great job! If you got an incorrect answer, where did the process break down? Did you think that sodium and potassium both entered the cell through a symporter? Did you think that tetrodotoxin stimulated sodium ion transport?
65) D
Version 1
71
Clarify Question ● What is the key concept addressed by the question? question asks about osmosis. ● What type of thinking is required? apply your knowledge about osmosis.
● The
● You are being asked to
● What key words does the question contain and what do they mean? ● Constipated—difficulty in entirely completing the digestive process ● Stool—solid waste that is eliminated from body Gather Content ● What do you know about osmosis? What other information is related to the question? ● Osmosis occurs when water moves from a hypotonic region to a hypertonic region. When a person has constipation, they want more water to move from the blood into the intestines. Choose Answer ● Given what you now know, what information is most likely to produce the correct answer? ● By eating magnesium, they increase the concentration of salt in the intestines, making it more hypertonic. As a result water moves from the blood into the intestines. Reflect on Process ● Did your problem-solving process lead you to the correct answer? If not, where did the process break down or lead you astray? How can you revise your approach to produce a more desirable result? ● If you got the correct answer, great job! If you got an incorrect answer, where did the process break down? Did you think that water moved towards a hypotonic solution? Did you think that to relieve constipation Version 1
72
you wanted water to move out of the intestines?
66) D
Version 1
73
Clarify Question ● What is the key concept addressed by the question? ● The question asks about the impact of adding cholesterol in a membrane. ● What type of thinking is required? ● You are being asked to apply your knowledge about the impact of adding cholesterol in a membrane. ● What key words does the question contain and what do they mean? ● Cholesterol—a steroid with a polar hydroxyl group ● Membrane—barrier between internal and external cell environment made of lipids and proteins Gather Content ● What do you know about the impact of adding cholesterol in a membrane? What other information is related to the question? ● In animal cells, cholesterol may make up as much as 50% of membrane lipids in the outer leaflet. The cholesterol can fill gaps left by unsaturated fatty acids. This has the effect of decreasing membrane fluidity, but it increases the strength of the membrane. Overall this leads to a plasma membrane with intermediate fluidity that is more durable and less permeable. Choose Answer ● Given what you now know, what information is most likely to produce the correct answer? ● The best answer is that membranes with high amounts of cholesterol are very stable and have decreased permeability. Reflect on Process ● Did your problem-solving process lead you to the correct answer? If not, where did the process break down or lead you astray? How can Version 1
74
you revise your approach to produce a more desirable result? ● If you got the correct answer, great job! If you got an incorrect answer, where did the process break down? Did you think that cholesterol would make a membrane more rigid? Did you think the cholesterol would increase membrane permeability?
67) A
Version 1
75
Clarify Question ● What is the key concept addressed by the question? question asks about osmosis. ● What type of thinking is required? apply your knowledge about osmosis.
● The
● You are being asked to
● What key words does the question contain and what do they mean? ● Water—polar molecule able to move across membranes via specialized channels Gather Content ● What do you know about osmosis? What other information is related to the question? ● In osmosis, water moves from an area that is hypotonic to an area that is hypertonic. Choose Answer ● Given what you now know, what information is most likely to produce the correct answer? ● After a race if a person is dehydrated then there is less water in their blood and the blood is hypertonic. The water the runner just drank will make their stomach hypotonic. The water will then move from the hypotonic stomach to the hypertonic blood. Reflect on Process ● Did your problem-solving process lead you to the correct answer? If not, where did the process break down or lead you astray? How can you revise your approach to produce a more desirable result? ● If you got the correct answer, great job! If you got an incorrect answer, where did the process break down? Did you think that water moved towards a hypotonic area? Did you think that drinking water would make the stomach more hypertonic? Version 1
76
68) B
Version 1
77
Clarify Question ● What is the key concept addressed by the question? question asks about dialysis. ● What type of thinking is required? analyze statements about dialysis.
● The
● You are being asked to
● What key words does the question contain and what do they mean? ● Dialysis bag—container made of a semipermeable membrane Gather Content ● What do you know about dialysis? What other information is related to the question? ● In dialysis small molecules can pass through tiny gaps in the membrane. Larger molecules and cells cannot cross the membrane. Starch is a large polymer made up of multiple glucose molecules and cannot diffuse across a dialysis membrane. Iodine is a small ion that can easily diffuse across the dialysis membrane. Choose Answer ● Given what you now know, what information is most likely to produce the correct answer? ● For the iodine to react with the starch, one of the molecules must diffuse across the dialysis membrane. Iodine is much smaller and could do this while starch is very large and could not. Reflect on Process ● Did your problem-solving process lead you to the correct answer? If not, where did the process break down or lead you astray? How can you revise your approach to produce a more desirable result? ● If you got the correct answer, great job! If you got an incorrect answer, Version 1
78
where did the process break down? Did you think that starch was a small molecule and could diffuse across the dialysis membrane? Did you think that iodine was a large molecule and could not diffuse across the dialysis membrane?
69) C
Version 1
79
Clarify Question ● What is the key concept addressed by the question? question asks about diffusion. ● What type of thinking is required? apply your knowledge about diffusion.
● The
● You are being asked to
● What key words does the question contain and what do they mean? ● Phospholipid bilayer—plasma membrane that consists of polar and nonpolar regions ● Diffusion—movement of substances from high to low concentration Gather Content ● What do you know about diffusion? What other information is related to the question? ● Diffusion occurs when a molecule can move down its concentration gradient. If aprotein must cross a membrane, polar or charged molecules require the aid of transport proteins. Gases and small non-polar molecules can diffuse directly across the membrane. Water moves by osmosis, but typically only in response to differences in salt concentrations across the membrane. Choose Answer ● Given what you now know, what information is most likely to produce the correct answer? ● NH3 is ammonia gas and can diffuse most rapidly across a membrane. Water can diffuse, but would do so mostly in response to differences in ion concentration. The other three choices are all ions and would not diffuse rapidly without the aid of transport proteins. Reflect on Process ● Did your problem-solving process lead you to the correct answer? Version 1
80
If not, where did the process break down or lead you astray? How can you revise your approach to produce a more desirable result? ● If you got the correct answer, great job! If you got an incorrect answer, where did the process break down? Did you think that ions could diffuse across the membrane without the aid of transport proteins? Did you think that water would diffuse more rapidly than a gas?
70) B
Version 1
81
Clarify Question ● What is the key concept addressed by the question? ● The question asks about coupled transport of glucose and sodium. ● What type of thinking is required? ● You are being asked to apply your knowledge about coupled transport of glucose and sodium. ● What key words does the question contain and what do they mean? ● Extracellular fluid—environment external to the cell body Gather Content ● What do you know about coupled transport of glucose and sodium? What other information is related to the question? ● The active glucose transporter uses the Na+ gradient produced by the Na+/K+ pump as a source of energy to power the movement of glucose into the cell. In this system, both glucose and Na+ bind to the transport protein, which allows Na+ to pass into the cell down its concentration gradient, capturing the energy and using it to move glucose into the cell through an antiporter. Choose Answer ● Given what you now know, what information is most likely to produce the correct answer? ● If Na+ levels drop on the outside of a cell, less Na+ is available to move to the inside of the cell, bringing glucose along with it through the symporter. As a result glucose transport will decrease. Reflect on Process ● Did your problem-solving process lead you to the correct answer? If not, where did the process break down or lead you astray? How can you revise your approach to produce a more desirable result? ● This question asked you to apply your knowledge about coupled transport of Version 1
82
glucose and sodium. If you got the correct answer, great job! If you got an incorrect answer, where did the process break down? Did you think that sodium and glucose were transported by an antiporter? Did you think that glucose transport was not linked to Na+ transport?
71) C
Version 1
83
Clarify Question ● What is the key concept addressed by the question? question asks about water uptake in plants.
● The
● What type of thinking is required? ● You are being asked to apply your knowledge about water uptake in plants. ● What key words does the question contain and what do they mean? ● Wilted—drooping plants Gather Content ● What do you know about water uptake in plants? What other information is related to the question? ● Water diffuses from a hypotonic region to a hypertonic region. Most plant cells are hypertonic to their immediate environment, containing a high concentration of solutes in their central vacuoles. The resulting internal hydrostatic pressure, known as turgor pressure, presses the plasma membrane firmly against the interior of the cell wall, making the cell rigid. Most green plants depend on turgor pressure to maintain their shape, andwilt when they lack sufficient water. Choose Answer ● Given what you now know, what information is most likely to produce the correct answer? ● If a plant is wilted it need to be placed in a hypotonic solution so that the inside of the plant is hypertonic. This will allow water to enter the plant and restore its shape. The correct answer is to place the plants in a 5% solution so that water will enter the plant cells. Reflect on Process ● Did your problem-solving process lead you to the correct answer? If not, where did the process break down or lead you astray? How can Version 1
84
you revise your approach to produce a more desirable result? ● If you got the correct answer, great job! If you got an incorrect answer, where did the process break down? Did you think that water moves from a hypertonic solution to a hypotonic solution? Did you think that a wilted plant needs to lose more water?
72) D 73) Section Break 73.1) C
Version 1
85
Clarify Question • What is the key concept addressed by the question? o The question asks about osmosis. • What type of thinking is required? o You are being asked to apply your knowledge about osmosis. Gather Content • What do you know about osmosis? What other information is related to the question? o In osmosis, water moves from an area that is more hypotonic to an area that is more hypertonic. Choose Answer • Given what you now know, what information and/or problem solving approach is most likely to produce the correct answer? o If egg 3 lost weight, then water was leaving the egg. If the solution was 15% salt, then it was hypertonic relative to the egg, pulling water from the egg. Reflect on Process • Did your problem-solving process lead you to the correct answer? If not, where did the process break down or lead you astray? How can you revise your approach to produce a more desirable result? o This question asked you to apply your knowledge about osmosis. If you got the correct answer, great job! If you got an incorrect answer, where did the process break down? Did you realize that water was leaving the egg which made it lose weight? Did you think Version 1
86
that water moved towards a hypotonic solution? 73.2) B
Version 1
87
Clarify Question ● What is the key concept addressed by the question? question asks about osmosis. ● What type of thinking is required? apply your knowledge about osmosis.
● The
● You are being asked to
● What key words does the question contain and what do they mean? ● Isotonic—two solutions that have the same osmotic concentration Gather Content ● What do you know about osmosis? What other information is related to the question? ● In osmosis, water moves from an area that is more hypotonic to an area that is more hypertonic. If no water moves then the areas are isotonic. Choose Answer ● Given what you now know, what information is most likely to produce the correct answer? ● If egg 3 did not change weight, then water was not entering or leaving the egg. If the solution was 15% salt, then it was isotonic relative to the egg. Reflect on Process ● Did your problem-solving process lead you to the correct answer? If not, where did the process break down or lead you astray? How can you revise your approach to produce a more desirable result? ● If you got the correct answer, great job! If you got an incorrect answer, where did the process break down? Did you realize that water was not leaving or entering the egg? Did you think that water moved towards a hypotonic solution?
Version 1
88
73.3) A
Version 1
89
Clarify Question ● What is the key concept addressed by the question? question asks about osmosis. ● What type of thinking is required? apply your knowledge about osmosis.
● The
● You are being asked to
Gather Content ● What do you know about osmosis? What other information is related to the question? ● In osmosis, water moves from an area that is more hypotonic to an area that is more hypertonic. Choose Answer ● Given what you now know, what information is most likely to produce the correct answer? ● If egg 1 gained weight, then water was entering the egg. If the solution was 15% salt, then the egg was hypertonic relative to the solution. This caused water to enter the egg. Reflect on Process ● Did your problem-solving process lead you to the correct answer? If not, where did the process break down or lead you astray? How can you revise your approach to produce a more desirable result? ● If you got the correct answer, great job! If you got an incorrect answer, where did the process break down? Did you realize that water was entering the egg which made it gain weight? Did you think that water moved towards a hypotonic solution?
74) Section Break Version 1
90
74.1) D
Version 1
91
Clarify Question • What is the key concept addressed by the question? o The question asks about carrier proteins. • What type of thinking is required? o You are being asked to analyze a graph on carrier proteins. Gather Content • What do you know about carrier proteins? What other information is related to the question? o Carriers must bind to the molecule they transport, so the relationship between concentration and rate of transport differs from that due to simple diffusion. As concentration increases, transport by simple diffusion shows a linear increase in rate of transport. But when a carrier protein is involved, a concentration increase means that more of the carriers are bound to the transported molecule. At high enough concentrations all carriers will be occupied, and the rate of transport will be constant. This means that the carrier exhibits saturation. Choose Answer • Given what you now know, what information and/or problem solving approach is most likely to produce the correct answer? o Saturation occurs when the number of glucose molecules exceed the ability of the carrier proteins available to bind them. So at higher concentrations glucose saturates the carrier protein. Reflect on Process • Did your problem-solving process lead you to the correct Version 1
92
answer? If not, where did the process break down or lead you astray? How can you revise your approach to produce a more desirable result? o This question asked you to analyze a graph on carrier proteins. If you got the correct answer, great job! If you got an incorrect answer, where did the process break down? Did you think that the carrier proteins were saturated at lower concentrations of glucose? Did you think that saturation occurred at middle concentrations of glucose? 74.2) B
Version 1
93
Clarify Question ● What is the key concept addressed by the question? ● The question asks about factors that affect the rate of diffusion. ● What type of thinking is required? ● You are being asked to apply your knowledge about factors that affect the rate of diffusion. ● What key words does the question contain and what do they mean? ● Diffusion—movement of substances from high to low concentration Gather Content ● What do you know about factors that affect the rate of diffusion? What other information is related to the question? ● Diffusion relies on the rapid movement of molecules. This occurs most rapidly in warmer temperatures when the molecules are moving faster. The greater the difference in concentration across a membrane, the larger the gradient and the faster molecules will diffuse. Choose Answer ● Given what you now know, what information is most likely to produce the correct answer? ● Diffusion will increase as the concentration of glucose increases. A decrease in temperature will decrease the rate of diffusion. pH should not affect the rate of diffusion of glucose since it is not charged. Reflect on Process ● Did your problem-solving process lead you to the correct answer? If not, where did the process break down or lead you astray? How can you revise your approach to produce a more desirable result? ● If you got the correct answer, great job! If Version 1
94
you got an incorrect answer, where did the process break down? Did you think that as glucose levels decrease diffusion would increase? Did you think that lower temperature would increase the rate of diffusion.
Version 1
95
CHAPTER 6 MULTIPLE CHOICE - Choose the one alternative that best completes the statement or answers the question. 1) Why do we have storage macromolecules, such as fats,in our bodies? A) We can break downthese macromolecules to provide energy for the endergonic reactions in ourbodies. B) Human cells can directly capture the energy of sunlight throughphotosynthesis and store it as macromolecules such as fats. C) Macromolecules,such as fats,are a convenient way to store kinetic energy. D) Breaking downmacromolecules, such as fats, is an endergonic process.
2)
Energy is defined as __________. A) heat B) the capacity to do work C) change D) movement
3)
A calorie is a commonly used unit of chemical energy. It is also the unit of __________. A) light B) magnetism C) sound D) heat E) radioactivity
Version 1
1
4) It is summer, and you are excited about going to your local amusement park, and specifically about ridingthe new roller coaster. You imagine waiting at the top of the stairs for the roller coaster to pull into the station, climbing into the car, strapping yourself into the seatbelt, and pulling down the harness.You can imagine the cars slowly chugging up to the top of the first hill, coming down on the other side, and the excitement you expect to feel as you go along for the ride. Of all of the things that you have imagined, which is an example of potential energy? A) Waiting at the top of the stairs for the roller coaster to pull into the station B) The roller coaster car going down the first hill C) Pulling down the harness D) The roller coaster car going up the first hill
5) While standing at the top of the stairs, you have a potential energy of 40 Joules. If you walk all the way down the stairs, what would your potential energy be at the bottom of the stairs? (Ignore any chemically-stored potential energy.) A) 0 Joules B) 20 Joules C) 40 Joules D) 80 Joules
6) Oxidation and reduction reactions are chemical processes that result in a gain or loss of __________. A) atoms B) neutrons C) electrons D) molecules E) protons
7)
When an atom or molecule gains one or more electrons, it is said to be __________.
Version 1
2
A) energized B) oxidized C) polarized D) reduced
8) The term oxidation is derived from the name of the element oxygen. This is reasonable, because oxygen __________. A) attracts electrons very strongly B) can be oxidized by accepting electrons C) contains more electrons than are needed D) passes electrons to many other types of molecules
9) In an experiment described in a chemistry lab book, the directions state that after mixing two chemicals (A and B) and waiting 5 minutes that A will be oxidized. This means that __________. A) chemical A has lost electrons to chemical B B) chemical A has gained electrons from chemical B C) chemical A has gained energy in the form of heat from chemical B D) chemical A has lost energy in the form of heat to chemical B E) chemical A has reacted with oxygen
10) In a chemical reaction, glyceraldehyde-3-phosphate + NAD + yields 1,3bisphosphoglycerate + NADH. In this reaction, what happened to NAD +? A) It was oxidized to form NADH. B) It was reduced to form NADH. C) It was activated to form NADH. D) It served as an enzyme to catalyze the reaction, and at the end of the reaction formed NADH.
Version 1
3
11)
The energy used by humans ultimately originates from what source? A) The sun B) Plants C) Water D) Air E) Animals
12) The Law of Thermodynamics that states that energy cannot be created or destroyed is the __________. A) First Law ofThermodynamics B) Second Law ofThermodynamics C) Third Law ofThermodynamics D) Fourth Law ofThermodynamics
13) You eat a bowl of beans as part of your dinner. As you digest the beans, the proteins that are present get broken down to their component amino acids. As your body destroys the macromolecules that were present in the beans, is the energy present in those molecules destroyed? A) Yes. By breakingdown these macromolecules, some of the energy they contained is destroyed. B) No. While the vastmajority of the energy contained in these macromolecules is converted to heat,it is not actually destroyed. C) No.Theenergy contained within these macromolecules is converted into other forms ofchemical energy and kinetic energy, though some is lost as heat. D) No. Breaking downmolecules does not lead to the release of energy.
14) If you were able to increase the kinetic energy of the molecules inside your body, how would this affect your body temperature?
Version 1
4
A) It would increase. B) It would decrease. C) It would remain the same.
15) The Law of Thermodynamics that states that increases in entropy are favored is the __________. A) First Law of Thermodynamics B) Second Law of Thermodynamics C) Third Law of Thermodynamics D) Fourth Law of Thermodynamics
16) Glucose is broken down through cellular respiration, which involves a large number of chemical reactions. At the end of the cellular respiration process, a large number of ATP molecules are generated, yet not all of the possible energy that is contained in a molecule of glucose can be harnessed through these chemical reactions to generate ATP. In other words, during cellular respiration, not all of the energy that is contained in a molecule of glucose is converted into the energy stored in ATP.The remaining energyis __________. A) destroyed B) used to drive exergonic reactions C) donated to molecules in the cellular respiration process to reduce them D) lost as heat
17) You return home to find that your baby brother has scattered his toy trains and trucks all over the floor of your room. As you begin to pick up the toys and put them away, you realize that even though he is just a baby, he has clearly mastered __________. A) the First Law of Thermodynamics B) the Second Law of Thermodynamics C) potential energy D) free energy
Version 1
5
18)
The amount of energy available to do work is called __________. A) activation energy B) free energy C) kinetic energy D) potential energy
19) The energy needed to destabilize existing chemical bonds and start a chemical reaction is called __________. A) activation energy B) free energy C) kinetic energy D) potential energy
20) The synthesis of sugar molecules through the process of photosynthesis requires energy absorbed from sunlight. Bearing this in mind, what kind of reaction is photosynthesis? A) Exergonic B) Endergonic C) Catabolic D) Feedback
21) A particular chemical reaction is exergonic. What can you say about the relationship between the reactants and the products in this exergonic reaction? A) The reactants have more free energy than the products. B) The reactants are likely more disordered and the products are likely more ordered. C) The reactants cannot spontaneously react to generate the products. D) The reactants likely have lower enthalpy than the products.
Version 1
6
22) In a chemical reaction, 1,3-bisphosphoglycerate + ADP yields 3-phosphoglycerate plus ATP. What is the delta G for this reaction? A) Greater than zero B) Less than zero C) Equal to zero D) Cannot be determined
23)
If the ΔG of a reaction was –31.45 kJoules, you would know that __________. A) the products have more free energy than the reactants B) the reaction requires an energy input of 31.45 kJoules to proceed C) the reaction most likely leads to development of a more ordered system D) the reaction is spontaneous
24) Many metabolic pathways involve ATP; either generating ATP, or requiring ATP. Why is ATP so important to metabolism? A) Hydrolysis of ATPis used to drive exergonic reactions. B) Hydrolysis of thebond between adenine and ribose in ATP is commonly used to release energy thatcan be used to drive other cellular reactions. C) The phosphategroups of ATP are held together by unstable bonds that can be broken to releaseenergy. D) ATP is a proteinthat serves as the energy currency of cells.
25) Under standard conditions, ATP can release _______for every molecule converted to ADP. A) less than 1 cal ofenergy B) 1 to 2 cal ofenergy C) 7.3 kcal of energy D) different amountsof energy depending on the cell
Version 1
7
26)
Does ADP have the capacity to provide energy for the cell? A) No. ADP does notcontain any bonds that can be broken to provide energy for the
cell. B) Yes.ADP hasthe same capacity to provide energy for the cell as ATP. C) Yes. Cleaving thebond between the ribose sugar and the two phosphate groups can provide energyfor the cell. D) Yes. Cleaving thebond between the terminal phosphate and the phosphate attached to the ribosesugar can provide energy for the cell.
27) AMP-activated protein kinase (AMPK) is an enzyme that is activated by high levels of AMP in cells. If levels of AMP are high in cells, that means that levels of ATP are low.Once active, AMPKactivates catabolic pathways and inhibits anabolic pathways in the cell. Why do you think that is the case? Choose the answer that best explains the role of AMPK. A) High levels ofAMP indicate that there is a high amount of energy stored in the cell, thusactivating catabolic pathways and inhibiting anabolic pathways are mechanismsto use stored energy. B) By inhibitinganabolicpathways, AMPK provides a mechanism to generate heat for thecell, which is important if AMP levels are high in the cell. C) By activating catabolic pathways, AMPK provides a mechanism to activateexergonic pathways, which is important if AMP levels are high in the cell. D) Activatingcatabolic pathways and inhibiting anabolic pathways will ultimately lead tohigher consumption of ATP, which is important if AMP levels are high in thecell.
28) A particular reaction has a negative delta G. However, this reaction takes many years to proceed in the absence of enzyme. Why is this the case? A) This reaction does not obey the Second Law of Thermodynamics. B) This reaction doesnot proceed spontaneously. C) The initial freeenergy of the reactants is much less than the final free energy of theproducts. D) A certain amountof activation energy is required for the reaction to proceed.
Version 1
8
29) Carbonic anhydrase, an enzyme widely present in plant and animal tissues, catalyzes the reaction:
This reaction is important for the maintenance of acid-base balance in the blood and other tissues and to help transport CO 2 out of tissues. Without a catalyst, this reaction would not occur at rates that could maintain homeostasis. In the figure below, which line(s) represents the reaction catalyzed by carbonic anhydrase?
A) Line B; the activation energy has been lowered B) Line A; the activation energy increased C) Line A; more energy is supplied D) Either, as the free energy is the same for both
30)
In an enzyme-catalyzed reaction, the reactant is called the __________.
Version 1
9
A) coenzyme B) catalyst C) substrate D) product
31) When the substrate is bound to the enzyme, the shape of the enzyme may change slightly, leading to __________. A) an induced fit B) a great range of possible catalytic activities C) more permanent binding through intimate total contact D) more possible products of the reaction
32) In order to reuse an enzyme after the conclusionof an enzyme-catalyzed reaction, what must occur? A) The enzyme hastobe resynthesized. B) The enzyme has toseparate itself from the product. C) The enzyme must use ATP to return to an active form. D) The enzyme has todecrease entropy.
33) The organic nonprotein components that aid in enzyme functioning are called __________. A) reactants B) cofactors C) coenzymes D) substrates E) products
Version 1
10
34) The inorganic nonprotein components that participate in enzyme catalysis are known as __________. A) coenzymes B) cofactors C) products D) substrates E) reactants
35)
RNA molecules that actas enzymes are given the name __________. A) ribozymes B) cofactors C) coenzymes D) allosteric enzymes
36) A ribozyme catalyzes a reaction on itself and actually changes shape. This is an example of what? A) Interenzyme inhibition B) Allosteric inhibition C) Intramolecular catalysis D) Intermolecular catalysis
Version 1
11
37) Based on the graph below, what are the optimal temperatures for the human enzyme and the hotsprings prokaryote enzyme?
A) The optimal temperature for the human enzyme is 30°C; the optimal temperature for the hotsprings prokaryote enzyme is 60°C. B) The optimal temperature for the human enzyme is 46°C; the optimal temperature for the hotsprings prokaryote enzyme is 79°C. C) The optimal temperature for the human enzyme is 40°C; the optimal temperature for the hotsprings prokaryote enzyme is 72°C. D) The optimal temperature for the human enzyme is 35°C; the optimal temperature for the hotsprings prokaryote enzyme is 65°C.
38) The normal body temperature of a bat is similar to the normal body temperature of a human with a body temperature of 37 degrees C. Which of the curves below most likely represents an enzyme from a bat?
Version 1
12
A) A B) B C) C
39) A drug binds to the active site of an enzyme. If it is bound to the active site of the enzyme, it prevents substrate binding.This drug would be considered a __________. A) noncompetitive inhibitor B) allosteric inhibitor C) allosteric activator D) competitive inhibitor
40) A molecule that closely resembles the shape of a substrate for an enzyme would most likely serve as a __________. A) noncompetitive inhibitor B) allosteric inhibitor C) competitive inhibitor D) allosteric activator
41) A current problem in modern medicine is the development of drug-resistance mutations. This occurs when a mutation arises in a disease-causing microbe making it resistant, and rendering the drug useless for treating an infection. Many useful drugs are competitive inhibitors of specific enzymes, and the drug-resistance mutations prevent the binding of the drug. These types of mutations, in addition to preventing competitive inhibitor binding, can also sometimes reduce the activity of the enzyme. Why is that the case? A) Binding to the competitive inhibitor is essential for the function of the enzyme. B) These mutations most likely affect an allosteric site on the enzyme. C) These mutations lower the activation energy of the reaction catalyzed by the enzyme. D) These mutations most likely change the shape of the active site of the enzyme.
Version 1
13
42) Tacrolimus (FK-506) is a drug that inhibits an enzyme called calcineurin. Calcineurin is a phosphatase – one of the cellular enzymes that dephosphorylateproteins (removephosphates). When added to cells, tacrolimus can inhibit the dephosphorylation of a protein called NFAT, but it cannot prevent the dephosphorylation of a protein called CDK1. What is the most likely explanation for this finding? A) Calcineurin requires an additional cofactor to dephosphorylate NFAT. B) NFAT is a substrate of calcineurin, but CDK1 is not. C) Tacrolimus is a competitive inhibitor of calcineurin for NFAT and CDK11. D) Tacrolimus changes the optimum pH for calcineurin.
43) While conducting an experiment, you realize that a competitive inhibitor was interfering with your reaction. How could you overcome this problem? A) Add a noncompetitive inhibitor to the reaction B) Add a cofactor to the reaction C) Increase the concentration of the correct substrate in the reaction D) Add an allosteric inhibitor to the reaction
44) You are working with a specific enzyme-catalyzed reaction in the lab. You are a very careful experimentalist, so at the beginning of each of your experiments you measure the temperature in the lab. On days 1 through 5, the temperature in the lab was 20°C. Today is day 6 of your experiment, and the temperature in the lab is 30°C. How do you predict that this will alter the rate of your enzyme-catalyzed reaction? A) It will decrease the rate. B) It will increase the rate. C) It could possibly increase or decrease the rate. D) It will not affect the rate.
45) You are studying an enzyme-catalyzed reaction that induces a particular cellular function in the lab.If you wanted to slow down that particular cellular function by controlling the enzyme, what could you do?
Version 1
14
A) Decrease the temperature of the incubator below the optimal temperature B) Increase the pH of the media the cells are growing in to the optimum pH C) Add cofactors to the media the cells are growing in D) Add an allosteric activator to the cells
46) A new antibiotic has been developed that inhibits the activity of an enzyme by competitive inhibition. What effect will this have on the activation energy of the enzymecatalyzed reaction? A) The activation energy required for the reactionin the presence of the antibiotic would be greater than the activation energyrequired in the absence of the antibiotic. B) The activation energy required for the reaction in the presence of the antibiotic would be less than the activation energyrequired in the absence of the antibiotic. C) The activation energy required for the reaction in the presence of the antibiotic will be the same as the activation energy required in the absence of the antibiotic.
47) A new antibiotic has been developed. It acts as anoncompetitive inhibitor of a key bacterial enzyme. How will this antibiotic affect delta G for the enzyme-catalyzed reaction? A) Delta G will increase. B) Delta G will decrease. C) Delta G will be unaffected.
48) The chemistry of living systems representing all chemical reactions is called __________. A) catabolism B) anabolism C) metabolism D) enzymology E) thermodynamics
Version 1
15
49) Metabolic reactions fall under two general categories: anabolic and catabolic. What type of chemical reactions are these two classes of metabolic reactions? A) Anabolic reactionsare exergonic reactions, whereas catabolic reactions are endergonic. B) Both anabolic and catabolic reactions are exergonic. C) Both anabolic and catabolic reactions are endergonic. D) Anabolic reactionsare endergonic reactions, whereas catabolic reactions are exergonic.
50) Glucose is converted into glucose 6-phosphate by hexokinase. Glucose 6-phosphate then serves as the substrate for the enzyme phosphoglucose isomerase, which converts this reactant into fructose 6-phosphate. Fructose 6-phosphate serves as the substrate for phosphofructokinase, which converts fructose 6-phosphate into fructose 1,6-bisphosphate. Based on the information provided, this is an example of __________. A) feedback inhibition B) allosteric regulation C) a metabolic pathway D) enzyme inhibition
51) Enzyme 1 converts substrate A into product B. Is this an example of a metabolic pathway? A) Yes. This is ametabolic pathway that includes feedback inhibition. B) No. This onlydescribes one chemical reaction. A metabolic pathway includes multiplechemical reactions. C) Yes.This isa simple metabolic pathway. D) No.Ametabolic pathway must include an inhibitory step.
52) Hexokinase is an enzyme that binds specifically to glucose and converts it into glucose 6phosphate.The activity of hexokinase is suppressed by glucose 6-phosphate, which binds to hexokinase at a location that is distinct from the active site.This is an example of what? Version 1
16
A) Feedback inhibition B) Competitive inhibition C) Cofactor binding D) Allosteric activation
53) Phosphofructokinase (PFK)is an enzyme that converts fructose 6-phosphate to fructose 1,6-bisphosphate, by adding a phosphate group. This is the first committed step of the metabolic pathway of glycolysis, and thus it is very tightly regulated. ATP binds to PFK at a site distinct from the binding site for fructose 6-phosphate, and inhibits the formation of fructose 1,6bisphosphate. What is the role of ATP in this example? A) Competitive inhibitor B) Allosteric inhibitor C) Allosteric activator D) Catalyst
Version 1
17
Answer Key Test name: Chap 06_3e 1) A Fat molecules have many bonds, and breaking those bonds provides energy, as discussed in the text. Therefore, these macromolecules are a convenient way of storing energy, and stored energy is considered potential energy.Human cells cannot perform photosynthesis. Breaking down macromolecules is an exergonic process. 2) B 3) D 4) A
Version 1
18
Clarify Question ● What is the key concept addressed by the question? question asks about different forms of energy.
● The
● What type of thinking is required? ● You are being asked to apply your knowledge of potential and kinetic energy. Gather Content ● What do you know about potential and kinetic energy? What other information is related to the question? ● Potential energy is stored in chemical bonds, gradients, or height. Kinetic energy is released by breaking bonds, or allowing things to move down gradients or drop in height. Choose Answer ● Given what you now know, what information is most likely to produce the correct answer? ● At the top of the stairs, you have gravitational potential energy. This is the only choice where you or the roller coaster car are not moving. Reflect on Process ● Did your problem-solving process lead you to the correct answer? If not, where did the process break down or lead you astray? How can you revise your approach to produce a more desirable result? ● This question asked you to apply your understanding of kinetic and potential energy. If you got the correct answer, great job! If you got an incorrect answer, where did the process break down? Did you think that kinetic energy was stored? Did you understand that when you were standing at the top of the stairs you had potential energy?
Version 1
19
5) A Clarify Question ● What is the key concept addressed by the question? question asks about potential energy.
● The
● What type of thinking is required? ● You are being asked to apply your knowledge of potential energy. Gather Content ● What do you know about potential energy? What other information is related to the question? ● Potential energy in this example is related to how high you are on the stairs. As you move down the stairs your potential energy will decrease. If you went down half of the steps your potential energy would drop to 20 Joules. Choose Answer ● Given what you now know, what information is most likely to produce the correct answer? ● If you go all the way down the stairs and cannot go any further then you would have 0 Joules. This is just related to your height on the stairs. Reflect on Process ● Did your problem-solving process lead you to the correct answer? If not, where did the process break down or lead you astray? How can you revise your approach to produce a more desirable result? ● This question asked you to apply your understanding of potential energy. If you got the correct answer, great job! If you got an incorrect answer, where did the process break down? Did you understand that as you went down the stairs you would lower your potential energy?
Version 1
20
6) C 7) D 8) A 9) A 10) B
Version 1
21
Clarify Question What is the key concept addressed by the question? • The question asks you to determine what happens to NAD+ during a chemical reaction. What type of thinking is required? • You are being asked to break down, or analyze, the chemical reaction to determine what happens to NAD+. Gather Content What do you already know about oxidation and reduction chemical reactions? What other information is related to the question? • Remember that many chemical processes participate in oxidationreduction (redox) reactions. Oxidation reactions occur when electrons are liberated from a molecule whereas reduction reactions occur when electrons are added to a molecule. Reduction is frequently associated with addition of hydrogen atoms as well. • As you may remember, redox reactions are quite common in biochemical pathways such as the reactiondescribed in the question. Given what you know about redox reactions, what happens to NAD+ when it becomes NADH? Choose Answer Do you have all the information needed to determine the correct answer? • Redox reactions involve one molecule becoming oxidized and another molecule becoming reduced via the removal and addition of electrons, respectively. • NAD+ is a molecular acceptor of electrons, and is therefore an oxidized molecule. The question indicates that NADH is a product of the Version 1
22
reaction. So what is the relationship between NAD+ and NADH? • When NAD+ accepts electrons, it becomes reduced. The reduced form of NAD+ is therefore NADH. Reflect on Process Did your problem-solving process lead you to the correct answer? If not, where did the process break down or lead you astray? How can you revise your approach to produce a more desirable result? • This question asked you to determine what happens to NAD+ in the reaction. • If you got the answer correct, good work! If you got an incorrect answer, did you remember how redox reactions occur? Were you able to identify which molecules participated in redox reactions? Were you able to discover what happened to NAD+ when it became NADH? 11) A 12) A 13) C 14) A
Version 1
23
Clarify Question What is the key concept addressed by the question? • The question asks for you to determine how increased kinetic energy in your body might affect your body temperature. What type of thinking is required? • You are being asked to take what you already know about potential and kinetic energy and apply your understanding to determine how increased kinetic energy might affect body temperature. Gather Content What do you already know about potential and kinetic energy? What other information is related to the question? • To solve this problem you’ll need to apply what you know about kinetic energy and use this to infer what might happen to body temperature. • Remember that kinetic energy is energy associated with things in motion. Choose Answer Do you have all the information needed to answer the question? • Objects in motion are affected by temperature. When something is frozen, it is immobile, and its kinetic energy is very low or absent. The opposite would also be true; if you could increase the kinetic energy of your body, the molecules that make up your body would be in motion, and more intensely moving. This would increase the number of contact events between molecules, which would elevate the cellular temperature and thus your body temperature. Reflect on Process Version 1
24
Did your problem-solving process lead you to the correct answer? If not, where did the process break down or lead you astray? How can you revise your approach to produce a more desirable result? • Answering this question correctly depended on your ability to use your understanding of kinetic energy and relate it to temperature. If you got an incorrect answer, did you remember that kinetic energy is energy associated with objects in motion? Did you remember that objects in motion frequently generate friction, and friction generates heat? Were you able to apply that understanding to predict that body temperature would increase as kinetic energy increases? 15) B 16) D It is lost as heat. In fact, this is one of the ways we maintain our body temperature. 17) B 18) B 19) A 20) B Because this process requires energy input, it is an endergonic reaction. 21) A
Version 1
25
Clarify Question What is the key concept addressed by the question? • The question asks for you to determine the relationship between reactants and products in an exergonic reaction. What type of thinking is required? • You are being asked to take what you already know about enzymes and free energy and apply your understanding to determine the energy characteristics of an exergonic reaction. Gather Content What do you already know about the energy associated with reactants and products in reactions? What other information is related to the question? • To solve this problem you’ll need to apply what you know about the free energy associated with reactants and products and determine how it is associated with exergonic and endergonic reactions. What free energy characteristics determine whether or not a reaction is exergonic (spontaneous) or endergonic (nonspontaneous)? Choose Answer Do you have all the information needed to answer the question? • Recall that the change in free energy in enzyme-based reactions is classified as exergonic or endergonic. Exergonic reactions have a negative delta G and are spontaneous, whereas endergonic reactions have a positive delta G and are nonspontaneous. Remember also that nonspontaneous reactions can be made to work if there is sufficient investment of energy (e.g. coupled reactions). • The only way to be able to meet the criteria for an exergonic reaction is to choose an answer that will create a negative delta G. In practical Version 1
26
terms, that means the free energy of the reactants must be higher than the free energy of the products. All other answer choices are implausible because they violate that basic energy assumption. Reflect on Process Did your problem-solving process lead you to the correct answer? If not, where did the process break down or lead you astray? How can you revise your approach to produce a more desirable result? • Answering this question correctly depended on your ability to use your understanding of reactions and free energy to determine the energy characteristics for an exergonic reaction. • If you got an incorrect answer, did you remember that reactions can be classified as exergonic and endergonic based on whether or not their change in free energy is positive or negative? Did you remember that exergonic reactions have a negative delta G and endergonic reactions have a positive delta G? Were you able to infer from this information that the reactant must have more free energy than the products in order to obtain a negative delta G? 22) B
Version 1
27
Clarify Question What is the key concept addressed by the question? • The question asks for you to determine the free energy characteristics for a chemical reaction. What type of thinking is required? • You are being asked to take what you already know about enzymes and free energy and apply your understanding to determine the energy characteristics of the enzyme-based reaction. Gather Content What do you already know about free energy in enzymatic reactions? What other information is related to the question? • To solve this problem you’ll need to apply what you know about free energy and how it is associated with different types of enzyme reactions. What free energy characteristics determine whether a reaction will occur or not? • What clues does the question give you about delta G? Does the reaction require the breakdown of ATP or produce ATP? What types of reaction require ATP hydrolysis in order to run? Choose Answer Do you have all the information needed to answer the question? • Recall that a reaction is classified as exergonic or endergonic. Exergonic reactions have a negative delta G and are spontaneous, whereas endergonic reactions have a positive delta G and are nonspontaneous. Remember also that nonspontaneous reactions can be made to work if there is sufficient investment of energy (e.g. coupled reactions). • The question says nothing about any need to useATP to provide Version 1
28
energy. In fact, the opposite is true − the reactionconverts ADP to ATP, storing energy. This indicates that the reaction must not have a positive delta G, which would indicate an endergonic reaction requiring energy input. Clearly, an energy conversion of some kind has occurred, so delta G definitely does change. Since the reaction is able to store energy in ATP, it is an exergonic reaction with a negative delta G. Reflect on Process Did your problem-solving process lead you to the correct answer? If not, where did the process break down or lead you astray? How can you revise your approach to produce a more desirable result? • Answering this question correctly depended on your ability to use your understanding of enzymes and free energy to determine what the free energy change would be for the reaction. • If you got an incorrect answer, did you remember that enzyme reactions can be classified as exergonic and endergonic based on whether or not their change in free energy is positive or negative? Did you remember that exergonic reactions have a negative delta G and endergonic reactions have a positive delta G? Were you able to see that ADP was converted to ATP and infer that the reaction would have to be exergonic and therefore have a negative delta G? 23) D 24) C ATP does serve as the energy currency of cells, by storing energy in its triphosphate group. By cleaving the unstable bonds holding the phosphates together in the ATP molecule, energy can be released to drive endergonic reactions. ATP can also be used as a building block for RNA molecules. It is not a protein. 25) C 26) D Version 1
29
While ADP does not have the same capacity to provide energy for the cell as ATP, cleaving the terminal phosphate of ADP has the capacity to release energy for the cell, as this is a high-energy bond. 27) C
Version 1
30
Clarify Question What is the key concept addressed by the question? • The question asks you to determine the role of AMPK in balancing catabolic and anabolic pathways in cells. What type of thinking is required? • You are being asked to dissect, or analyze, a scenario to ascertain the role of AMPK in cells. Gather Content What do you already know about AMP, ADP, and ATP in cells? What other information is related to the question? • Remember that ATP is the energy currency of the cell. High levels of ATP indicate the cell is fully powered and has the energy reserves it needs to conduct cellular work. Recall also that ADP and AMP are lower-level and lowest-level energy molecules, respectively. • If the cell is in an energy deficit, which molecule will be most prevalent: ATP, ADP, or AMP? Choose Answer Do you have all the information needed to make a determination about the change in free energy? • If AMP levels are high in the cell, this indicates that energy levels in the cell are low. This in turn means that the cell would have lower levels of ATP, the high-energy currency of the cell. • Recall that AMP can be converted to ADP and then to ATP by adding inorganic phosphate. When energy levels are low (AMP), the cell needs to generate more high energy molecules (ATP). This requires the cell to turn on energy-generating exergonic pathways and turn offendergonic pathways that require energy. Since reactions that break chemical bonds Version 1
31
in molecules are required to capture stored energy and generate ATP, and reactions that require energy to build molecules are less needed, AMPK will most likely activate catabolic pathways and inhibit anabolic pathways. Reflect on Process Did your problem-solving process lead you to the correct answer? If not, where did the process break down or lead you astray? How can you revise your approach to produce a more desirable result? • This question asked you to analyze the most likely role of AMPK in cells when AMP levels are high and ATP levels are low. • If you got the answer correct, congratulations! If you got an incorrect answer, were you able to correlate AMP levels with low energy and ATP levels with high energy in the cell? Were you able to determine that catabolic reactions are required to generate ATP and anabolic reactions require ATP? Were you able to connect the two ideas together to determine that high AMP levels would lead to increased catabolic (exergonic) and decreased anabolic (endergonic) reactions? 28) D Reactions with a negative delta G proceed spontaneously, but not instantaneously. They need a certain amount of activation energy to proceed. 29) A 30) C 31) A 32) B 33) C 34) B 35) A Version 1
32
36) C 37) C Clarify Question ● What is the key concept addressed by the question? reactions. ● What type of thinking is required? enzyme reactions.
● The question asks about enzyme
● You are being asked to apply your knowledge of
Gather Content ● What do you know about enzyme reactions? What other information is related to the question? ● Enzyme reactions are affected by temperature. At low temperatures, the rate of the reaction is low because chemical bonds between the enzyme and its substrates cannot form. At high temperatures most enzymes denature and can no longer catalyze the reaction. Enzyme reactions have an optimal temperature at which the rate of the reaction is highest. Choose Answer ● Given what you now know, what information is most likely to produce the correct answer? ● The optimal temperature is the temperature at which the enzyme functions best. Based on the graph, the rate of the reaction is highest for the human enzyme around 40 degrees C, and for the hotsprings prokaryote enzyme around 72 degrees C. At 46 and 79 degrees C each enzyme becomes denatured and can no longer catalyze the reaction. Reflect on Process ● Did your problem-solving process lead you to the correct answer? If not, where did the process break down or lead you astray? How can you revise your approach to produce a more desirable result? ● This question asked you to apply your understanding of enzyme reactions. If you got the correct answer, great job! If you got an incorrect answer, where did the process break down? Did you recall that at high temperatures enzymes denature and cannot catalyze reactions any longer? Did you understand that the points on each curve represented the points with the highest reaction rate?
38) A
Version 1
33
Clarify Question ● What is the key concept addressed by the question? reactions. ● What type of thinking is required? enzyme reactions.
● The question asks about enzyme
● You are being asked to apply your knowledge of
Gather Content ● What do you know about enzyme reactions? What other information is related to the question? ● Enzyme reactions are affected by temperature. At low temperatures, the rate of the reaction is low because chemical bonds between the enzyme and its substrates cannot form. At high temperatures, most enzymes denature and can no longer catalyze the reaction. Enzyme reactions have an optimal temperature at which the rate of the reaction is highest, and these are commonly around the body temperature of the organism they are from. Choose Answer ● Given what you now know, what information is most likely to produce the correct answer? ● Humans and bats both have a body temperature of 37 degrees C, so their enzyme most likely has an optimal temperature near this point. So curve A is the best fit for humans and bats. Reflect on Process ● Did your problem-solving process lead you to the correct answer? If not, where did the process break down or lead you astray? How can you revise your approach to produce a more desirable result? ● This question asked you to apply your understanding of enzyme reactions. If you got the correct answer, great job! If you got an incorrect answer, where did the process break down? Did you understand that enzymes tend to have an optimal temperature near that of the body temperature of their organism.
39) D 40) C
Version 1
34
Clarify Question What is the key concept addressed by the question? • The question asks for you to identify the most likely function of a molecule that closely resembles the shape of the substrate. What type of thinking is required? • You are being asked to take what you already know about enzymes and molecular shape and apply your understanding to determine which function is most likely. Gather Content What do you already know about enzymes and inhibition? What other information is related to the question? • To solve this problem you’ll need to apply what you know about the different ways that enzyme inhibition can occur and molecular shape and determine the most likely role for a molecule with similar shape to the substrate. Recall that enzyme function can be inhibited competitively, noncompetitively, and allosterically. Why is the shape of substrate molecules important, and what possible role could a molecule that mimics the substrate’s shape play in enzyme function? Choose Answer Do you have all the information needed to answer the question? • Enzymes can be competitively, noncompetitively, and allosterically inhibited. What are the key differences between these methods? • Noncompetitive inhibition means molecules are not trying to outcompete substrate molecules for the enzyme’s active site. As such, the shape of noncompetitive inhibitors is unlikely to closely match the shape of the substrate molecule. • Likewise, allosteric inhibitors, which are noncompetitive by Version 1
35
definition, bind at a site remote from the active site to inhibit enzyme function. Again, the shape of allosteric inhibitor is unlikely to closely resemble the shape of a substrate molecule because it doesn’t need to function as a mimic. • That leaves competitive inhibition. In order to compete for the active site of an enzyme, an inhibitor needs to be able to fit into the same space as a substrate molecule. This means the inhibitor must closely resemble the substrate’s shape in order to inhibit enzyme function. Reflect on Process Did your problem-solving process lead you to the correct answer? If not, where did the process break down or lead you astray? How can you revise your approach to produce a more desirable result? • Answering this question correctly depended on your ability to use your understanding of enzymes and inhibition. • If you got an incorrect answer, did you recall the ways in which enzymes can be inhibited? Did you remember that noncompetitive and allosteric inhibition do not involve the active site, and therefore their shape would have no need to resemble the substrate’s shape? Were you able to rationalize competitive inhibition considering the shape of the inhibitor needed to resemble the substrate? 41) D
Version 1
36
Clarify Question What is the key concept addressed by the question? • The question asks you ascertain how drug resistance development in bacteria is related to reduced enzyme activity. What type of thinking is required? • You are being asked to break down, or analyze, a possible means by which drug resistance in bacteria might work to reduce enzyme activity. Gather Content What do you already know about enzymes and how their activity is regulated? How does this drug affect enzyme activity under normal circumstances? What other information is related to the question? • Remember that the activity of an enzyme is directly related to its structure and function. Recall that there are several structural elements of an enzyme that affect its function; these include the active site and the allosteric site. These structural elements as well as inhibitors influence an enzyme’s ability to facilitate reactions. The question also indicates that the drug acts as a competitive inhibitor of the enzyme. Considering the structure and function of enyzmes, which most directly affected by bacterial drug resistance that reduces enzyme activity? Choose Answer Do you have all necessary information to analyze how drug resistance would reduce enzyme activity? • The question states that the drug is a competitive inhibitor of the enzyme, and that changes in the enzyme’s structure render the drug ineffective. Recall that competitive inhibitors mimic the shape of a substrate, which binds to the active site. This implies that the shape of the drug was a close match to the substrate in order to be a competitive inhibitor in the first place. • Amutation in the bacteriumrenders the drug ineffective. • This implies that the active site of the enzyme must be directly involved, since that would prevent binding of the drug Reflect on Process Did your problem-solving process lead you to the correct answer? If not, where did the process break down or lead you astray? How can you revise your approach to produce a more desirable result? • This question asked you to analyze possible reasons for why a previously-effective drug that works as a competitive inhibitor is rendered ineffective by bacterial mutation. Version 1
37
• If you got the answer correct, well done! If you got an incorrect answer, were you able to break down the structural elements of the enzyme into the active and allosteric sites? Were you able to connect the drug being a competitive inhibitor and the bacterial mutation eliminating drug effectiveness? Were you able to inferthat the active site was the most likely aspect of the enzyme to be affected?
42) B
Version 1
38
Clarify Question What is the key concept addressed by the question? • The question asks why the drug tacrolimus works in cells to inhibit the enzyme calcineurin, prevent dephosphorylation of NFAT, and yet allow phosphorylation of CDK1. What type of thinking is required? • You are being asked to break down, or analyze, a possible means by which the drug tacrolimus works on calcineurin phosphatase enzyme. Gather Content What do you already know about phosphates and enzymestructure? What other information is related to the question? • Since calcineurin is an enzyme, that means it works to convert substrates to products – just like any other enzyme. In the case of this enzyme, calcineurin dephosphorylates other proteins. The trick to answering this question correctly is to try and decipher how NFAT and CDK1 relate to calcineurin Choose Answer Do you have all necessary information to analyze how tacrolimus works on NFAT and CDK1? • It is unlikely that tacrolimus would affect calcineurin’s pH. Enzymes operate in very specific pH ranges and if tacrolimus affected pH, other enzymes besides calcineurin would also be affected. • The question mentions nothing about any cofactors needed for calcineurin to dephosphorylate NFAT. • If tacrolimus were a competitive inhibitor of calcineurin, it would interact with the enzyme as a substrate but prevent calcineurin’s dephosphorylation activity, leading to greater phosphorylation of NFAT and CDK1. The question indicates that tacrolimus acts to prevent dephosphorylation of NFAT but not CDK1, so that answer is not plausible. • That leaves NFAT as a substrate of calcineurin, which makes sense when you consider that adding tacrolimus prevents NFAT being dephosphorylated by calcineurin. CDK1 is unaffected because it is not a substrate of calcineurin. Reflect on Process Did your problem-solving process lead you to the correct answer? If not, where did the process break down or lead you astray? How can you revise your approach to produce a more desirable result? Version 1
39
• This question asked you to determine the likely explanation for relationships between calcineurin, NFAT, and CDK1 proteins based on tacrolimus treatment. • If you got the answer correct, good job! If you got an incorrect answer, were you able to decipher that tacrolimus resulted in less calcineurin activity and therefore less NFAT dephosphorylation? Were you able to correlate the activity of calcineurin and NFAT together? Were you infer that their relationship would only be possible if NFAT were a substrate for calcineurin?
43) C
Version 1
40
Clarify Question What is the key concept addressed by the question? • The question asks for you to overcome a problem where a competitive inhibitor is interfering with a reaction. What type of thinking is required? • You are being asked to take what you already know about enzymes and competitive inhibition and apply your understanding to determine how to rectify the problem. Gather Content What do you already know about enzymes and competitive inhibition? What other information is related to the question? • Recall that enzyme function can be inhibited competitively, noncompetitively, and allosterically. How does competitive inhibition compare to the other types of inhibition? Keeping that mechanism in mind, what could you do to overcome the inhibition? Choose Answer Do you have all the information needed to answer the question? • If you added a noncompetitive inhibitor, you would likely still have decreased enzyme function,because it changes the enzyme structure and prevents substrate binding altogether. An allosteric inhibitor would essentially function in the same way as a noncompetitive inhibitor, and likewise not help your reaction. • A cofactor might conceivably help overcome the latent inhibition, but only if the cofactor outcompeted the inhibitor for the active site, which may not be the case. • That leaves increasing substrate concentration as the best solution. Recall that in competitive inhibition, the inhibitor directly competes with the substrate for the enzyme’s active site. Both inhibitor and substrate try to occupy the same physical space within the structure of the enzyme. Increasing substrate concentration would serve up more reactants to the active site per unit time than the inhibitor, effectively washing out the effects of the inhibitor. Reflect on Process Did your problem-solving process lead you to the correct answer? If not, where did the process break down or lead you astray? How can you revise your approach to produce a more desirable result? • Answering this question correctly depended on your ability to use your understanding of enzymes and competitive inhibition to determine what would be the most likely solution to restore your reaction. Version 1
41
• If you got an incorrect answer, did you recall how competitive inhibition works? Did you contrast that with noncompetitive and allosteric inhibition, which don’t involve the active site and therefore wouldn’t solve the problem at all? Did you decide that the best solution was to simply outcompete the competitive inhibitor by increasing substrate?
44) C
Version 1
42
Clarify Question What is the key concept addressed by the question? • The question asks you predict what effect temperature change will have on the rate of an enzyme-catalyzed reaction. What type of thinking is required? • You are being asked to weigh and judge, or evaluate, which prediction is most plausible from a given change in environmental temperature. Gather Content What do you already know about enzyme rates and temperature? What other information is related to the question? • Remember that temperature affects protein structure and function, and as such can influence enzyme rate of reaction because enzymes are proteins. The real question here is to predict how a 10 degree temperature change might affect enzyme reaction rate. Choose Answer Do you have all necessary information to make a judgment? • Recall that enzyme protein structure and function operates best within a relatively narrow window. Therefore, enzyme reaction rates will fluctuate, depending on the degree of temperature change. • The enzyme rates could increase if the 10 degree temperature increase brings the enzyme closer to its optimal temperature. Conversely, enzyme reaction rates could decrease if the optimal temperature is closer to the 20 degree room temperature. Without more information, it is impossible to predict how exactly the enzyme rates will change, so the most acceptable prediction is to say that enzyme reaction rates could increase or decrease. Reflect on Process Did your problem-solving process lead you to the correct answer? If not, where did the process break down or lead you astray? How can you revise your approach to produce a more desirable result? • This question asked you to evaluate by predicting the effects of environmental temperature on enzyme reaction rates. • If you got the answer correct, good job! If you got an incorrect answer, did you remember that enzymes are proteins, and that temperature affects protein structure and function? Did you recall that enzymes operate best at an optimal temperature? Were you able to determine that enzyme rate could increase or decrease due to the 10 degree environmental temperature change? Version 1
43
45) A Decreasing the temperature could work to slow down the reaction. All of these other choices would either increase or maintain the rate of the reaction.
46) C
Version 1
44
Clarify Question What is the key concept addressed by the question? • The question asks you how an antibiotic that competitively inhibits an enzyme will affect the enzyme’s activation energy. What type of thinking is required? • You are being asked to break down, or analyze, how a competitive inhibitor antibiotic will influence an enzyme’s activation energy. Gather Content What do you already know about enzymes and activation energy? What factors influence activation energy? What other information is related to the question? • Remember that the activity of an enzyme is directly related to its structure and function. Recall that there are several structural elements of an enzyme that affect its function; these include the active site, which can be regulated by competitive inhibitors. • The question indicates that the antibiotic acts as a competitive inhibitor of the enzyme. Considering the structure and function of enyzmes and how competitive inhibitors work, how might the activation energy of the reaction be affected? Choose Answer Do you have all necessary information to analyze how the antibiotic might influence activation energy? • Recall that competitive inhibitors bind to the active site of the enzyme, which precludes the substrate binding to the same location. Thus, it will take more substrate to out-compete the competitive inhibitor, but with enough substrate this can be done and the enzyme will achieve the same maximum rate eventually, with or without the competitive inhibitor. Version 1
45
• Regardless of how fast the enzyme can process the reaction (meaning with or without the competitive inhibitor), the actual energy costs associated with the reaction do not change. As a result, the competitive inhibitor antibiotic should have no effect on the activation energy required for the enzyme-catalyzed reaction. Reflect on Process Did your problem-solving process lead you to the correct answer? If not, where did the process break down or lead you astray? How can you revise your approach to produce a more desirable result? • This question asked you to analyze the effect of a competitive inhibitor antibiotic on the activation energy for an enzyme-catalyzed reaction. • If you got the answer correct, good job! If you got an incorrect answer, did you remember that competitive inhibitors bind to the active site? Did you recall how competitive inhibitors affect overall reaction rate as substrate concentration changes? Were you able to determine that activation energy, the actual cost of the reaction, doesn’t change regardless of the presence or absence of a competitive inhibitor? 47) C
Version 1
46
Clarify Question What is the key concept addressed by the question? • The question asks you how an antibiotic that noncompetitively inhibits an enzyme will affect the reaction’s change in free energy. What type of thinking is required? • You are being asked to break down, or analyze, how a noncompetitive inhibitor antibiotic will influence an enzyme-catalyzed change in free energy. Gather Content What do you already know about enzymes and free energy? What other information is related to the question? • Remember that an enzyme helps to accelerate the rate of a reaction by making the reaction more likely to occur. Recall that noncompetitive inhibitors bind not to the active site, but to an enzyme’s secondary site which changes the protein’s three-dimensional confirmation, changes the shape of the active site, and thereby reduces the overall speed of the reaction. • Considering the structure and function of enzymes and how noncompetitive inhibitors work, how might the free energy of the reaction be affected? Choose Answer Do you have all necessary information to analyze how the noncompetitive inhibitor might influence free energy? • Recall that noncompetitive inhibitors bind to a secondary site on the enzyme, which changes the shape of the enzyme and precludes the substrate from binding to the active site. Thus, no amount of substrate will allow the enzyme to reach its top speed because the shape of the active site has been changed. • Regardless of how fast the enzyme can process the reaction (meaning with or without the noncompetitive inhibitor), the actual difference in free energy between the reactants and the products will not change. Whether or not a reaction is exergonic or endergonic is a function of whether or not the reaction has a positive or negative delta G. The presence of an enzyme, or a noncompetitive inhibitor, has no effect on the difference in free energy. As a result, the noncompetitive inhibitor antibiotic should have no effect on the free energy required for the enzyme-catalyzed reaction. Reflect on Process Did your problem-solving process lead you to the correct answer? If not, where did the process break down or lead you astray? How can you revise your approach to produce a more Version 1
47
desirable result? • This question asked you to analyze the effect of a noncompetitive inhibitor antibiotic on the free energy for an enzyme-catalyzed reaction. • If you got the answer correct, well done! If you got an incorrect answer, did you remember that noncompetitive inhibitors bind to a secondary site on an enzyme? Did you recall how noncompetitive inhibitors affect overall reaction rate as substrate concentration changes? Were you able to determine that free energy, or the difference in free energy from reactants to products, doesn’t change regardless of the presence or absence of a noncompetitive inhibitor?
48) C 49) D Anabolic reactions require energy to build up molecules, and thus are endergonic reactions. Catabolic reactions harvest energy by breaking down molecules, and thus are exergonic. 50) C 51) B 52) A In this case, the product of the reaction, glucose 6-phosphate, is feeding back to inhibit the enzyme that catalyzes the reaction, hexokinase. As a result, this is an example of feedback inhibition.
53) B
Version 1
48
Clarify Question What is the key concept addressed by the question? • The question asks you to determine the role of ATP in regulating PFK enzyme in the glycolysis metabolic pathway. What type of thinking is required? • You are being asked to dissect, or analyze, how ATP works to regulate the activity of the PFK enzyme based on the scenario. Gather Content What do you already know about enzymes, feedback inhibition, and glycolysis? What other information is related to the question? • Remember that several enzymes can function together in a biochemical pathway, and the efficient regulation of that pathway frequently involves feedback inhibition, which occurs when the final product regulates an earlier step in the pathway. Choose Answer Do you have all the information needed to determine how ATP helps to regulate PFK activity? • Consider that ATP is an important high-energy molecule used by the cell to power its processes. If a cell has enough ATP, is it useful and efficient for that cell to make more when it already has enough? When enough ATP is present, it will bind to PFK and down-regulate its activity so that more ATP isn’t made until it is needed. Reflect on Process Did your problem-solving process lead you to the correct answer? If not, where did the process break down or lead you astray? How can you revise your approach to produce a more desirable result? • This question asked you to analyze the role for ATP in regulating PFK and glycolysis. • If you got the answer correct, fantastic! If you got an incorrect answer, did you remember how biochemical pathways work? Were you able to determine how PFK is regulated? Were you able to infer how ATP works to activatePFK?
Version 1
49
CHAPTER 7 MULTIPLE CHOICE - Choose the one alternative that best completes the statement or answers the question. 1) Organisms that can manufacture their own chemical energy are called ____________. A) autotrophs B) heterotrophs C) oligotrophs D) chemotrophs
2) Organisms that depend on the energy stored in chemical bonds by other organismsfor their food energy are called ____________. A) autotrophs B) heterotrophs C) oligotrophs D) chemotrophs
3) In animals that take in oxygen from their environment, glucose is broken down into carbon dioxide and water in a process called __________. A) anaerobic respiration B) organic compound respiration C) glucose respiration D) aerobic respiration
4) To form NADH from NAD +, two electrons and a proton are removed from an organic molecule. What term best describes the reaction in which electrons are removed from an organic molecule?
Version 1
1
A) Condensation B) Reduction C) Oxidation D) Decarboxylation E) Isomerization
5) What oxidizing agentis used to temporarily store high energy electrons harvested from glucose molecules in a series of gradual steps in the cytoplasm? A) FADH 2 B) ADP C) NAD + D) oxygen
6) Aerobic respiration is ultimately the reaction of glucose with oxygen to generate carbon dioxide, water, and energy.However, in a cell this process involves the transfer of electrons from glucose to carriers such as NAD + over a lengthy series of steps.Why don't cells employ the direct reaction of glucose with oxygen to generate energy? A) The direct transfer of electrons from glucose to oxygen occurs via a combustion reaction that is incompatible with life. B) Oxygen andglucose are localized in different subcellular compartments. C) The directreaction of oxygen with glucose would be much less efficient. D) The direct reaction of oxygen with glucose does not spontaneously occur.
7) What is the oxidized form of the most common electron carrier that is needed for both glycolysis and the citric acid cycle?
Version 1
2
A) ATP B) FAD C) Pyruvate D) NAD + E) Acetyl-CoA
8)
When substrate-level phosphorylation occurs, it means that __________. A) NAD + is converted into NADH B) ATP is converted into ADP plus a phosphate group C) ADP is converted into ATP by the addition of a phosphate group D) NADH is converted into NAD + plus a proton
9) Why is the energy generated from the catabolism of sugars and other macromolecules ultimately harnessed to generate ATP? A) ATP can be used bycellsto drive endergonic reactions. B) ATP can be used tomake RNA, which is an energy storage molecule in the cell. C) ATP synthesis isan exergonic reaction. D) ATP is required togenerate the proton gradient in the intermembrane space of mitochondria.
10)
How and where is ATP made in a eukaryoticcell?
Version 1
3
A) ATP is only madein the mitochondria in response to chemiosmosis. B) ATP is made inall compartments of the cell inresponse to endergonic reactions and isused to drive exergonic reactions in the cell. C) ATP can be madeby direct phosphorylation of ADP in the cytoplasm, and by an enzyme complexthat uses the energy from a proton gradient to drive ATP synthesis in themitochondria. It can also be made in other locations in the cell, depending onthe cell type. D) ATP can be made byan enzyme complex that uses the energy of protons moving down theirconcentration gradient from the mitochondrial matrix to the cytoplasm to makethe ATP.
11) A biochemist wants to control the initial substrate level phosphorylation that occurs in the tracheal cells of grasshoppers once glucose has crossed the plasma membrane. He has access to the following inhibitors: Rotenone, an electron transport chain inhibitor; Oligomycin, an ATP synthase inhibitor;andTLN232, an inhibitor of glycolysis. Which inhibitor should he use to slow down initial substrate level phosphorylation that occurs once glucose has crossed the plasma membrane? A) Rotenone B) Oligomycin C) TLN-232 D) None of these inhibitors would be effective in preventing substrate-level phosphorylation.
12)
What aspect of cellular respiration occurs in the cytoplasm in eukaryotic cells? A) Glycolysis B) Pyruvate oxidation C) The citric acid cycle D) The electron transport chain E) ATP synthesis
13) In glycolysis, a major portion of the energy remains in the final product, which is called __________.
Version 1
4
A) glucose B) citrate C) glyceraldehyde 3-phosphate (G3P) D) pyruvate
14) What is the net number of ATP generated directly during glycolysis per molecule of glucose? A) 0 B) 2 C) 4 D) 6 E) 8
15)
What is an end-product of glycolysis? A) Oxaloacetate B) NAD + C) Alcohol D) ADP E) Pyruvate
16)
During what step of glycolysisare two ATP molecules required? A) Cleavage and rearrangement B) Glucose priming C) Oxidation D) Pyruvate formation E) Acetyl-CoA formation
17)
Which process iscommon to nearly all living cells?
Version 1
5
A) Glycolysis B) Alcohol fermentation C) The citric acid cycle D) Electron transport chain reactions E) Pyruvate oxidation
18)
What molecule can oxidize NADH? A) Acetaldehyde B) Lactate C) Ubiquinone D) Glucose E) Isocitrate
19) CO
One way to generate acetyl-CoA is to convert pyruvate into acetyl-CoA by stripping off a 2 molecule. The removal of CO 2 is referred to as what type of reaction? A) Decarboxylation B) Glycolytic C) Carboxylation D) Acetylation
20) All of the reactions of cellular respiration that occur after glycolysis take place in what part of the eukaryotic cell? A) The chloroplast B) The nucleus C) The mitochondria D) The plasma membrane E) The cytoplasm
Version 1
6
21)
Where does pyruvate oxidation occur in eukaryotic cells? A) In the cytoplasm B) In the nucleus C) In the Golgi body D) In the mitochondria E) In the plasma membrane
22) Starting with one molecule of acetyl-CoA as an input, how many oxidation reactions occur during the citric acid cycle? A) One B) Two C) Four D) Six E) Eight
23)
What are the products of one turn of the citric acid cycle? A) 1 CO 2,2 NADH, 1 FADH 2, 1 ATP B) 2 CO 2,3 NADH, 1 FADH 2, 1 ATP C) 2 CO 2,6 NADH, 2 FADH 2, 2 ATP D) 4 CO 2,6 NADH, 2 FADH 2, 2 ATP E) 4 CO 2,12 NADH, 4 FADH 2, 4 ATP
24)
What is common to all of the oxidation reactions in the citric acid cycle?
A) They all lead tothe generation of NADH. B) They are alldecarboxylation reactions. C) They all involve a loss of electrons from an organic molecule coupled tothe reduction of an electron acceptor. D) They all lead tosubstrate-level phosphorylation of ADP to generate ATP.
Version 1
7
25) The equation for cellular respiration is: C6H12O6 + 6O2 6CO2 + 6H2O. At what specific point in the cellular respiration process has glucose been broken down completely from a sixcarbon molecule to form six molecules of CO2? A) After the priming reactions in glycolysis B) After the oxidation and ATP formation reactions in glycolysis C) After pyruvate oxidation D) After the condensation reaction in the citric acid cycle E) After the second oxidation in the citric acid cycle
26) In the reaction catalyzed by aconitase, the conversion of citrate to isocitrate can be inhibited by fluoroacetate. Fluoroacetate isused as a pesticide. Why is this an effective pesticide? A) It inhibits glycolysis. B) It inhibits pyruvate oxidation. C) It inhibits the citric acid cycle. D) It inhibits the electron transport chain. E) It inhibits ATP synthase.
27) Arsenic poisoning can lead to organ failure and death. Though arsenic can inhibit or interfere with a number of cellular enzymes, arsenic poisoning is thought to be mainly due to indirect inhibition of enzymes involved in both pyruvate oxidation and the citric acid cycle. As a result, this compound must be able to enter which cellular compartment? A) The cytoplasm B) The nucleus C) The intermembrane space of the mitochondria D) The mitochondrial matrix E) The endoplasmic reticulum
28) The citric acid cycle occurs in the mitochondria. There are nine biochemical reactions involved in the citric acid cycle, and they are highly ordered. Select the correct order from the following choices. (Note: These are abbreviated and do not show NAD, ADP, ATP, or FAD.)
Version 1
8
A) Acetyl-CoA joins the citric acid cycle and unites with oxaloacetate →forming citrate →which forms beta-ketoglutarate → which forms succinyl-CoA → which forms succinate → which forms fumarate → which forms malate → which forms oxaloacetate B) Acetyl-CoA joins the citric acid cycle and unites with oxaloacetate →forming citrate →which forms alpha-ketoglutarate → which forms succinyl-CoA → which forms succinate → which forms malate → which forms fumarate → which forms oxaloacetate C) Acetyl-CoA joins the citric acid cycle and unites with oxaloacetate →which forms alpha-ketoglutarate → forming citrate → which forms succinyl-CoA → which forms succinate → which forms fumarate → which forms malate → which forms oxaloacetate D) Acetyl-CoA joins the citric acid cycle and unites with oxaloacetate →forming citrate →which forms alpha-ketoglutarate → which forms succinyl-CoA → which forms succinate → which forms fumarate → which forms malate → which forms oxaloacetate
29) Why are the components of the electron transport chain embedded in the inner mitochondrial membrane rather than floating freely in the cytoplasm of mitochondrial matrix? A) To generate and maintain the proton gradient essential for ATP production. B) To separate the ATP from the ADP. C) Because electrons cannot float in the matrix. D) Because NADH cannot localize to the mitochondrial matrix.
30)
In aerobic respiration, chemiosmotic generation of ATP is driven by __________.
A) P i transfer through the plasma membrane B) the Na +/K + pump C) a difference in H + concentration on the two sides of the inner mitochondrial membrane D) osmosis of macromolecules E) large quantities of ADP
31) Cytochromes contain a heme group similar to that in hemoglobin. The iron atom (Fe) in the center of the group can be oxidized and reduced. If someone was suffering from iron deficiency anemia, what stage of cellular respiration would be most affected? Version 1
9
A) Glycolysis B) The citric acid cycle C) The electron transport chain D) Fermentation
32) The energy released in the mitochondrial electron transport chain is used to transport protons where? A) Into the mitochondrial matrix B) Into the cytoplasm C) Into the endoplasmic reticulum D) Into the intermembrane space of the mitochondria E) Into the nucleus
33) What is different about the way that NADH and FADH electron transport chain?
2 donate electrons to the
A) NADH is oxidizedand FADH 2 is reduced. B) NADH contributesits electrons to the first transmembrane complex in the electron transportchain and FADH 2 contributes its electronsafter the first transmembrane complex. C) More protons aretransported into the intermembrane space of the mitochondria in response to onemolecule of FADH 2 as compared to the number ofprotons transported in response to one molecule of NADH. D) The electrons fromNADH ultimately go on to reduce oxygen to generate water, whereas the electronsfrom FADH 2 are used to reduce pyruvate to lactate.
34)
What happens to the oxygen that is used in cellular respiration?
Version 1
10
A) It is converted to carbon dioxide. B) It is used to make glucose. C) It is used to make citric acid cycle intermediates. D) It is reduced to form water. E) It is converted to acetyl-CoA.
35) As electrons move along the electron transport chain, they lose potential energy.How is the energy that is released used by the cell? A) The energy is used to transport protons against their concentration gradient. B) The energy is used to pump electrons along the electron transport chain. C) The energy is converted directly into ATP. D) The energy is used to pump NAD + into the cytoplasm so it can be used in glycolysis.
36) As a forensic pathologist, you perform an autopsy on a poisoning victim. After a thorough examination, you conclude that the victim died of cyanide poisoning. You know that cyanide binds to the cytochrome oxidase complex, and therefore list the official cause of death as suffocation due to cyanide exposure. However, if you wanted to provide a more technical explanation as to the cause of death, which process was specifically inhibited directly by cyanide? A) The reduction of NAD + B) The oxidation of FADH 2 C) All proton pumping into the intermembrane space D) The formation of water from oxygen
37) If ATP synthase had a mutation in the F1 complex portion of the protein, whichfunction of ATP synthasewould most likely be affected?
Version 1
11
A) The rotation of the rotor B) The flow of protons through the channel C) The conversion of ADP and Pi to ATP D) The insertion of the enzyme into the membrane
38) Your friend is having difficulty keeping track of the energy flow from glucose through glycolysis, the citric acid cycle, and electron transport. Your best advice would be to __________. A) follow ATP production B) follow the protons C) follow NAD + production D) follow the electrons
39) How does the ATP yield from aerobic respiration of one glucose molecule differ in prokaryotes and eukaryotes? A) Both yield the same 32 ATP molecules, because the processes are so similar. B) Eukaryotes yield 30 instead of 32 ATP, because of the cost of transporting two NADH into the mitochondria. C) Both yield the same 30 ATP molecules, because the processes are so similar. D) Prokaryotes yield 30 instead of 32 ATP, because they lack mitochondria. E) Prokaryotes yield 7 instead of 32 ATP, because they lack mitochondria.
40) In eukaryotic cells,what is the predicted energy yield from glycolysis of one glucose molecule? Take into account thenetnumber of ATP molecules produced by substrate-level phosphorylation, the ATP value per molecule of NADH produced, and the fact that NADH molecules produced in the cytoplasm have to be transported into the mitochondria.
Version 1
12
A) 2 ATP B) 5 ATP C) 7 ATP D) 30 ATP
41) What is the predicted energy yield of the citric acid cycle, per molecule of glucose in eukaryotic cells? Take into account theATP value per molecule of NADH and FADH2, andthe number of ATP molecules produced by substrate-level phosphorylation. A) 2 ATP B) 10 ATP C) 20 ATP D) 30 ATP
42) Phosphofructokinase is regulated by a number of factors, including high levels of ATP. Why is this enzyme regulated by ATP levels? A) If ATP levels are high, this provides a mechanism to directlyinhibit the citric acid cycle, thus preventing further generation of NADH, FADH 2, and ATP molecules that are not needed. B) If ATP levels arehigh, it is important to directlyinhibit the reaction that commits thesubstrate to glycolysis to allow the substrate to be available for otherreactions, since the cell has ample energy. C) If ATP levels arehigh, it is important to inhibit ATP synthase, and phosphofructokinase directlyinhibitsATP synthase. D) If ATP levels are high, this provides a mechanism to directly inhibit theelectron transport chain, thus preventing the formation of a proton gradient inthe intermembrane space of mitochondria.
43) If citrate levels are high in the cell, but ATP levels are low, what do you think will happen in the cell?
Version 1
13
A) In the presence of glucose, glycolysis will run to generate energy for the cell, but the citric acid cycle will be inhibited. B) Glycolysis will be inhibited, but the citric acid cycle will be functional, allowing it to be utilized to breakdown acetyl-CoA generated from beta-oxidation. C) The electron transport chain will be inhibited, causing a build-up of NADH and FADH2. This will inhibit the citric acid cycle, but in the presence of glucose, glycolysis will still run coupled with fermentation to regenerate NAD+. D) Glycolysis and the citric acid cycle will both be inhibited, thus under these conditions there will be no mechanism to generate ATP.
44) You decide to go on vacation in the mountains, where you are staying in a cabin. Unfortunately, when you turn on the water in the cabinyou smell hydrogen sulfide (H 2S) gas. After some research, you find out that the H 2S may be due to the presence of sulfur bacteria living in your pipes. What molecule dothese bacteria use as an electron acceptor? A) O 2 B) H 20 C) SO 4 2– D) H 2S
45)
In the absence of oxygen, can cells utilize the electron transport chain?
A) Yes, all cells canmake use of the electron transport chain in the absence of oxygen viafermentation. B) No, oxygen is arequired cofactor for the complexes in the electron transport chain. C) Yes, in the casethat a cell can use a terminal electron acceptor other than oxygen, it can makeuse of the electron transport chain. D) No, oxygen is theprimary electron acceptor in electron transport chains in all cell types.
46) What stage of cellular respiration can occur in human cellswith or without oxygen present?
Version 1
14
A) The citric acid cycle B) Glycolysis C) The electron transport chain D) Pyruvate oxidation
47) Regardless of the electron or hydrogen acceptor used, one of the products of fermentation is always __________. A) ADP B) ATP C) NAD + D) pyruvate E) alcohol
48) Many of the antiviral drugs currently used to treat HIV/AIDS also interfere with an enzyme that helps mitochondria multiply. Treatment can therefore result in a decrease in the number of mitochondria found in certain tissues. Given this information, what might you expect to see in patients treated with antiviral drugs? A) An increase in oxidative phosphorylation B) An increase in phosphofructokinase activity C) An increase in NADH dehydrogenase activity D) An increase in lactic acid levels
49) When oxygen is unavailable during heavy exercise what process domuscle cells use for energy generation? A) Glycolysis coupled with alcohol fermentation B) Anaerobic respiration C) Aerobic respiration D) Glycolysis coupled with lactic acid fermentation
Version 1
15
50) A human cell has a mutation in the gene that encodes the enzyme that generates lactate from pyruvate, rendering that enzyme completely non-functional.Assuming that there is ample glucose present, how would this cell generate energy in the presence of oxygen? A) Glycolysis coupled with ethanol fermentation B) Aerobic respiration C) Primarily through the breakdown of proteins into amino acids D) No way to generate energy under these conditions
51) When amino acids are degraded in cells, into what components of aerobic respiration are they converted? A) Pyruvate B) Acetyl-CoA C) Citric acid cycle intermediates D) Pyruvate and acetyl-CoA E) Pyruvate and citric acid cycle intermediates
52)
What must happen to amino acids before they can be used in catabolic reactions? A) They must bedecarboxylated. B) They must bedeoxygenated. C) They must bedehydrogenated. D) They must bedeaminated.
53) In order to lose weight and reduce body fat, a friend of yours hasdecided to eliminate all fat from his diet, while consuming unrestricted amounts of carbohydrates. What do you think of this idea?
Version 1
16
A) It is a good idea,because if your friend doesn't eat any fat, he cannot store any additionalfat. B) It is a bad idea,because consumption of fat is required to provide cofactors for the electrontransport chain. C) It is a good idea, because under conditions where ATP levels are low, carbohydrates will be stored, and fat stores will be catabolized via beta-oxidation to generate energy. D) It is a bad idea, because if ATP levels are high, excess acetyl-CoA from the metabolism of carbohydrates can be used for fatty acid synthesis.
54) Many types of cancer cells have been detected to secrete significant levels of lactate. Do you think these cells are likely undergoing beta-oxidation? A) Yes, becausebeta-oxidation can generate intermediates that would lead to the production oflactate. B) No, because if lactate is being produced, the cell is not likelymaking use of the pathways needed to make use of the products of beta-oxidation. C) Yes, becauselactate stimulates beta-oxidation. D) No, because lactate is consumed in beta-oxidation.
55) Glucose is not our only food source, nor the only one we can utilize in our bodies to generate energy. Other primary sources of energy include other sugars, proteins, and fats. What metabolic intermediate are fats primarily converted into? A) Pyruvate B) Acetyl-CoA C) Citric acid cycle intermediates D) Electron transport chain components E) ATP synthase
56) Cardiac muscle cells need to generate significant amounts of ATP to allow for constant contractile activity. As a result, they primarily depend upon beta-oxidation of fatty acids, which has a higher energy yield than the catabolism of glucose. What would be the ATP yield for betaoxidation of a hypothetical 10-carbon fatty acid?
Version 1
17
A) 32 ATP B) 35 ATP C) 50 ATP D) 62 ATP
57) You discover a new organism while on vacation in Yellowstone Park. You use a series of experiments to establish that it carries out aerobic respiration, but find that it has a unique ATP synthase that requires 16 protons per rotation.What does this tell you about the P/O ratio in this organism? A) The P/O ratio in the new organism is the same as the typical P/O ratio. B) The P/O ratio in the new organism is higher than the typical P/O ratio. C) The P/O ratio in the new organism is lower than the typical P/O ratio. D) The new organism only makes ATP by substrate-level phosphorylation.
58)
Which statement about the P/O ratio is true? A) It represents theamount of ATP produced by oxidative phosphorylation. B) It depends on thenumber of electron acceptor sites on ATP synthase. C) It shows thatcellular respiration is an inefficient process. D) Itrepresentsthe amount of ATP made per glucose molecule.
59) An organism acquires a mutation in the gene for pyruvate dehydrogenase that renders the protein unable to bind NADH. What is the most likely consequence of this mutation? A) ATP synthesis willproceed at a high rate even when the cell has an ample supply. B) Pyruvate oxidationwill cease to occur. C) Fats will beoxidized at a higher rate than normal. D) Substrate levelphosphorylation will cease to occur. E) Pyruvate will notbe able to enter the mitochondria.
Version 1
18
Answer Key Test name: Chap 07_3e 1) A 2) B 3) D 4) C 5) C 6) A 7) D 8) C 9) A 10) C 11) C
Version 1
19
Clarify Question What is the key concept addressed by the question? • The question asks you to determine which chemical inhibitor would slow down initial substrate-level phosphorylation. What type of thinking is required? • You are being asked to dissect, or analyze, the effects of each inhibitor on initial substrate-level phosphorylation once glucose has been imported into the cell. Gather Content What do you already know about substrate-level phosphorylation during cellular respiration? What other information is related to the question? • Recall that cellular respiration can produce ATP using two mechanisms; one that uses substrate-level phosphorylation and one that uses oxidative phosphorylation. Oxidative phosphorylation provides the majority of ATP that a cell needs, provided oxygen is present. Choose Answer Do you have all the information needed to determine which inhibitor would have the biggest impact on substrate-level phosphorylation? • The key to answering this question correctly is to realize when substrate-level phosphorylation occurs during cellular respiration, and more specifically when substrate-level phosphorylation first occurs after glucose enters the cell. • Remember that substrate-level phosphorylation occurs during glycolysis and the citric acid cycle. Since the question indicates that the inhibitor should slow initial substrate-level phosphorylation, that means the inhibitor should restrict ATP production during glycolysis. Version 1
20
• Of the listed chemicals, TLN-232 is the only inhibitor to restrict glycolysis, and is therefore the correct answer. Reflect on Process Did your problem-solving process lead you to the correct answer? If not, where did the process break down or lead you astray? How can you revise your approach to produce a more desirable result? • Answering this question correctly depended on your ability to analyze which inhibitor a biochemist should choose in order to slow down initial substrate-level phosphorylation. • If you got the correct answer, good thinking! If you got an incorrect answer, were you able to remember when during cellular respiration substrate-level phosphorylation occurs? Were you able to figure out that the substrate-level phosphorylation following the import of glucose would be during glycolysis? Were you able to use that information to choose TLH-232 from the list of inhibitors? 12) A 13) D 14) B 15) E 16) B 17) A 18) A 19) A 20) C 21) D 22) C 23) B 24) C
Version 1
21
While all of the oxidation reactions are characterized by the donation of electrons from an organic molecule to an electron acceptor, they are not all decarboxylation reactions, nor do they all involve the generation of NADH. The third oxidation reaction is not a decarboxylation reaction, and involves the generation of FADH 2. None of the oxidation reactions directly induce substrate-level phosphorylation of ADP. 25) E
Version 1
22
Clarify Question What is the key concept addressed by the question? • The question asks you to determine the point at which glucose has been completely broken down to carbon dioxide during cellular respiration. What type of thinking is required? • You are being asked to dissect, or analyze, cellular respiration to determine when the six carbon molecules in glucose have been completely oxidized to carbon dioxide. Gather Content What do you already know about cellular respiration and the oxidation of carbon? What other information is related to the question? • Recall that cellular respiration involves oxidizing carbon in glucose to become carbon dioxide. Consider each stage of cellular respiration: glycolysis, pyruvate oxidation, citric acid cycle, and electron transport. Which of these stages involves the production of carbon dioxide? Choose Answer Do you have all the information needed to determine at which point glucose is completely oxidized to carbon dioxide? • Let’s review each step of cellular respiration in order to determine the last point at which carbon dioxide is produced. Glycolysis involves converting a 6-carbon glucose molecule into two 3-carbon pyruvate molecules. Hopefully it is clear that no carbon dioxide is lost during glycolysis, so no carbon oxidation has occurred. • Pyruvate oxidation involves conversion of the two 3-carbon pyruvate molecules into two 2carbon acetyl-CoA molecules. Since matter can’t be created or destroyed, and you have to account for all of the carbon, the only way this can make sense is if two carbon dioxide molecules are given off during pyruvate oxidation. Since two carbon atoms have been oxidized, that leaves four to go in order to account for the six carbons that are oxidized during cellular respiration. • If you recall, the citric acid cycle pairs a 2-carbon acetyl-CoA molecule with a 4-carbon oxaloacetate molecule to produce a 6-carbon citrate molecule. However, the 4-carbon oxaloacetate molecule must be continuously regenerated in order to accept more acetyl-CoA molecules. That means that, per citrate molecule from the citric acid cycle, two carbon atoms must be oxidized per turn of the citric acid cycle. This involves two oxidation events. If you account for the fact that the citric acid cycle must turn twice per glucose molecule, then you know that it is the second oxidation during the citric acid cycle when glucose becomes completely oxidized to carbon dioxide. Reflect on Process
Version 1
23
Did your problem-solving process lead you to the correct answer? If not, where did the process break down or lead you astray? How can you revise your approach to produce a more desirable result? • Answering this question correctly depended on your ability to analyze cellular respiration for when the 6 carbon atoms in glucose become completely oxidized to carbon dioxide. • If you got the correct answer, well done! If you got an incorrect answer, were you able to determine which stages of cellular respiration involved carbon oxidation? Were you able to follow the trail and account for each carbon as it becomes oxidized from glucose? Were you able to determine that the final carbon atoms become oxidized during the citric acid cycle?
26) C
Version 1
24
Clarify Question What is the key concept addressed by the question? • The question asks you to determine why fluoroacetate is an effective pesticide. What type of thinking is required? • You are being asked to take what you know about citrate to isocitrate conversion, and apply your knowledge to determine why fluoroacetate is an effective pesticide. Gather Content What do you already know about cellular respiration and when citrate is converted to isocitrate? What other information is related to the question? •Recall that cellular respiration involves glycolysis, pyruvate oxidation, the citric acid cycle, and the electron transport chain. During which of these stages is citrate converted to isocitrate by the aconitase enzyme, and which process would be inhibited by the fluoroacetate pesticide? Choose Answer Do you have all necessary information to determine why fluoroacetate is an effective pesticide? • Fluoroacetate prevents conversion of citrate to isocitrate, chemical reactions that only occur during the citric acid cycle. Citrate is not involved in glycolysis, pyruvate oxidation, or the electron transport chain, which would also preclude anything having to do with ATP synthase. Essentially, fluoroacetate would prevent aerobic respiration in organisms treated with this chemical. Reflect on Process Did your problem-solving process lead you to the correct answer? If Version 1
25
not, where did the process break down or lead you astray? How can you revise your approach to produce a more desirable result? • Answering this question correctly depended on your ability to apply what you know about the different stages of cellular respiration to identify which stage involves citrate to isocitrate conversion and use that to determine why fluoroacetate would be an effective pesticide.. • If you got the correct answer, good thinking! If you got an incorrect answer, were you able to recall the different stages of cellular respiration? Were you able to identify which stage involves conversion of citrate to isocitrate? Were you able to infer that inhibition of this reaction would prevent the citric acid cycle from functioning? 27) D
Version 1
26
Clarify Question What is the key concept addressed by the question? • The question asks you to determine which cellular compartments would be involved with arsenic poisoning, which inhibits pyruvate oxidation and the citric acid cycle. What type of thinking is required? • You are being asked to use what you know about pyruvate oxidation and the citric acid cycle, and apply this information to determine where arsenic poisoning would occur within the cell. Gather Content What do you already know about pyruvate oxidation and the citric acid cycle? What other information is related to the question? • Recall that cellular respiration involves four stages; glycolysis, pyruvate oxidation, the citric acid cycle, and the electron transport chain. Remember also that glycolysis occurs in the cytoplasm, and the products of glycolysis can be metabolized under either aerobic or anaerobic conditions. If you assume aerobic conditions, then pyruvate oxidation and the citric acid cycle will be used. Where do pyruvate oxidation and the citric acid cycle physically occur within cells? Choose Answer Do you have all necessary information to determine where arsenic poisoning would have its greatest impact within the cell? • As we discussed earlier, glycolysis occurs in the cytoplasm, and its products must be imported into the mitochondria to participate in pyruvate oxidation and the citric acid cycle, particularly when aerobic conditions exist. • Mitochondria are divided into several spaces which include the intermembrane space and the matrix. Version 1
27
Reflect on Process Did your problem-solving process lead you to the correct answer? If not, where did the process break down or lead you astray? How can you revise your approach to produce a more desirable result? • Answering this question correctly depended on your ability to apply what you know about pyruvate oxidation and the citric acid cycle, and apply this information to determine where arsenic poisoning would occur within the cell. • If you got the correct answer, good thinking! If you got an incorrect answer, were you able to recall the different regions of mitochondria? Did you realize that the matrix was the region most likely to be affected by arsenic? 28) D 29) A 30) C 31) C
Version 1
28
Clarify Question What is the key concept addressed by the question? • The question asks you to determine the stage of cellular respiration that would be most affected by iron deficiency anemia. What type of thinking is required? • You are being asked to break down, or analyze, the options to determine the likely stage most affected by iron deficiency anemia. Gather Content What do you already know about redox reactions and when they take place during cellular respiration? What other information is related to the question? • You know that the stages of cellular respiration include glycolysis, pyruvate oxidation, the citric acid cycle, and electron transport. Under anaerobic conditions fermentation can also occur. • The question indicates that cytochromes that are similar to hemoglobin and can participate in oxidation and reduction reactions would be affected if an individual was deficient in iron and developed anemia. Essentially what this means is iron deficiency will reduce the ability of the cell to conduct redox reactions. During which stage of cellular respiration do most redox reactions occur? Choose Answer Do you have all the information needed to determine which stage of cellular respiration would be most affected by an iron deficiency? • Determining the right answer for this question is relatively straightforward. Let’s see if we can eliminate some options. Certainly oxidation and reduction reactions happen during glycolysis, but none of them involve anything resembling cytochromes. That answer seems less Version 1
29
likely. • Similarly, the citric acid cycle also contains redox reactions, but again no cytochromes. The same would hold true for fermentation; the absence of cytochromes makes these answers less likely to be correct. • That leaves the electron transport chain as a possible option, let’s see why it might or might not make sense as the correct answer. We know that redox reactions occur during the electron transport chain; what types of molecules help facilitate those redox reactions? In fact, it is cytochromes that participate in the redox reactions. Therefore, the stage of cellular respiration that would be most affected in someone suffering from iron deficiency anemia would be the electron transport chain. Reflect on Process Did your problem-solving process lead you to the correct answer? If not, where did the process break down or lead you astray? How can you revise your approach to produce a more desirable result? • Answering this question correctly depended on your ability to analyze each option to determine which stage of cellular respiration would be most affected by someone suffering from iron deficiency anemia. • If you got the correct answer, nice job! If you got an incorrect answer, did you remember which stages of cellular respiration use redox reactions? Were you able to distinguish this from those that use cytochromes? Were you able to infer that the electron transport chain would be most affected in individuals suffering from iron deficiency anemia? 32) D 33) B 34) D 35) A 36) D
Version 1
30
Clarify Question What is the key concept addressed by the question? • The question asks you to provide a technical explanation as to which cellular respiration process facilitated by cytochrome oxidase would be inhibited by cyanide poisoning. What type of thinking is required? • You are being asked to break down, or analyze, the processes associated with cytochrome oxidase that would be inhibited by cyanide poisoning. Gather Content What do you already know about cellular respiration and cytochrome oxidase? What other information is related to the question? • You know that the stages of cellular respiration include glycolysis, pyruvate oxidation, the citric acid cycle, and electron transport. Cytochromes, particularly cytochrome oxidase, are found in the electron transport chain. Essentially what this means is that cyanide works to inhibit the electron transport chain, but how? What specific processes in the electron transport chain would be most affected by inhibiting cytochrome oxidase? Choose Answer Do you have all the information needed to determine which specific process is directly inhibited by cyanide? • Determining the right answer for this question is fairly easy if you know the right clues to look for. As before, let’s see if we can eliminate some of the incorrect options and increase our chances of getting the correct answer. • FADH2 does in fact become oxidized, and specifically at complex II Version 1
31
of the electron transport chain. However this option is not connected to cytochrome oxidase. Nor is the reduction of NAD+ related to cytochrome oxidase. We know that proton pumping occurs using multiple complexes in the electron transport chain, so that seems an unlikely option also. • So what exactly does cytochrome oxidase do? In fact, that complex takes the reduced-energy electrons that have been passed through the electron transport chain via the redox reactions and reduces atmospheric oxygen to form water. That is the correct answer. Reflect on Process Did your problem-solving process lead you to the correct answer? If not, where did the process break down or lead you astray? How can you revise your approach to produce a more desirable result? • Answering this question correctly depended on your ability to analyze the specific processes associated with the electron transport chain to determine which is involved with cytochrome oxidase and which would be affected by cyanide poisoning. • If you got the correct answer, well done! If you got an incorrect answer, were you able to see that the available answers involved something having to do with the electron transport chain? Were you able to associate cytochrome oxidase with the end of the electron transport chain? Were you able to infer that cyanide poisoning would affect the conversion of atmospheric oxygen into water? 37) C
Version 1
32
Clarify Question What is the key concept addressed by the question? • The question asks you to determine which function of ATP synthase would be affected if the F1 portion of the protein complex became mutated. What type of thinking is required? • You are being asked to analyze the possible outcomes of mutations in the F1 portion of the ATP synthase complex. Gather Content What do you already know about ATP synthase structure and function? What other information is related to the question? • You likely already know that ATP synthase is the enzyme complex that produces ATP via a proton gradient established by the electron transport chain. What you may not realize is that ATP synthase is actually a molecular machine that is comprised of multiple moving parts, each of which contributes to the overall function of ATP synthase. • How is the ATP synthase complex structured? Located in the inner mitochondrial membrane, ATP synthase is comprised of a domain that transverses the membrane (F0), and another domain that projects into the mitochondrial matrix (F1). Both F0 and F1 domains of ATP synthase contain multiple protein subunits. Choose Answer Do you have all necessary information to determine which outcome is most likely from a mutation in the F1 part of ATP synthase? • Considering that the F0 domain of ATP synthase transverses the inner mitochondrial membrane, it is more likely to be involved in proton transport than actual ATP synthesis. Recall that ATP synthase rotates Version 1
33
within the membrane, so the function of F1, the other domain, is unlikely to act as the rotor or the proton channel. • The F1domain is thecatalytic domain that connects inorganic phosphate and ADP to create ATP. So a mutation would disrupt that process. Reflect on Process Did your problem-solving process lead you to the correct answer? If not, where did the process break down or lead you astray? How can you revise your approach to produce a more desirable result? • Answering this question correctly depended on your ability to analyze the options to determine which function of the F1 ATP synthase domain would be affected by a mutation. • If you got the correct answer, well done! If you got an incorrect answer, were you aware of the overall structure and function of ATP synthase? Were you able to determine the functions of the F0 and F1 domains? Were you able to infer that mutations in F1 would inhibit the actual process of ATP production? 38) D 39) B
Version 1
34
Clarify Question What is the key concept addressed by the question? • The question asks you to describe how the ATP yields from aerobic respiration differ for prokaryotes and eukaryotes. What type of thinking is required? • You are being asked to take what you know about the ATP yield of aerobic respiration, and use, or apply, your knowledge to describe how it differs in prokaryotes and eukaryotes. Gather Content What do you already know about ATP yield from aerobicrespiration? What other information is related to the question? • Recall that cellular respiration is comprised of glycolysis, pyruvate oxidation, the citric acid cycle, and electron transport. The overall ATP yield depends on the ATP generated by substrate-level phosphorylation, as well as the ATP generated in chemiosmosis after electrons are donated from NADH and FADH2, minus any ATP costs to manage the process. • Also recall that although prokaryotes and eukaryotes use similar pathways, in eukaryotes everything but glycolysis takes place in the mitochondria. Choose Answer Do you have all the information needed to calculate the ATP yield from pyruvate? • The complete theoretical yield of aerobic respiration is 32 ATP. This represents a net of 2 ATP direct from glycolysis, 5 ATP from the NADH generated in glycolysis, 5 ATP from the NADH generated from pyruvate oxidation, 2 ATP directly from the citric acid cycle, 15 ATP from Version 1
35
NADH generated by the citric acid cycle, and 3 ATPfrom FADH2 generated by the citric acid cycle. However, in eukaryotes 2 ATP must be spent to transport the NADH from glycolysis into the mitochondria, leaving a net total of only 30 ATP. Reflect on Process Did your problem-solving process lead you to the correct answer? If not, where did the process break down or lead you astray? How can you revise your approach to produce a more desirable result? • Answering this question correctly depended on your ability to apply what you know about the ATP yield of aerobic respiration in prokaryotes and eukaryotes. • If you got the correct answer, nice job! If you got an incorrect answer, were you able to recall the total theoretical yield from aerobic respiration? Did you also remember that eukaryotes alone have the added cost of transporting NADH into the mitochondria? 40) B
Version 1
36
Clarify Question What is the key concept addressed by the question? • The question asks you to determine the predicted yield of glycolysis from associated NADH molecules and net ATP produced. What type of thinking is required? • You are being asked to dissect, or analyze, glycolysis for ATP and NADH production to determine the total yield of ATP during glycolysis. Gather Content What do you already know about glycolysis and ATP and NADH yield? What energy costs are required to transport NADH produced in the cytosol via glycolysis into the mitochondria? What other information is related to the question? • Recall that the first part of cellular respiration is glycolysis, which occurs in the cytosol. Products from glycolysis include ATP and NADH, among other molecules. • Glycolysis requires energy investment, which takes away from the total number of ATP it can produce via substrate-level phosphorylation. NADH, another key product of glycolysis, holds high energy electrons that can ultimately be used to produce ATP via chemiosmosis and oxidative phosphorylation in the mitochondria using ATP synthase during electron transport. • How many ATP molecules can conceivably be produced in the mitochondria from one NADH molecule? Choose Answer Do you have all necessary information to determine which products would affect cellular respiration? • Remember that during glycolysis 4 total ATP are produced, 2 ATP Version 1
37
need to be invested in the early stages, leaving a net of 2 ATP molecules produced overall. In addition, 2 molecules of NADH are produced during glycolysis. • Each molecule of NADH, whether made in the cytosol or in the mitochondria, will deposit their high-energy electrons into the electron transport chain. ATP synthase, the key enzyme that produces ATP via the chemiosmotic proton gradient, is predicted to yield 2.5 molecules of ATP per NADH molecule. Thus, during glycolysis, we generate 2 net ATP via substrate-level phosphorylation and another 5 for the 2 molecules of NADH that are produced. • However, each molecule of NADH generated by glycolysis has to be transported into the mitochondria via active transport, which requires 1 ATP molecule per NADH to be transported. • Therefore, the total energy yield of glycolysis in a eukaryotic cell should be: 2 ATP (glycolysis net ATP) + 5 ATP (2 NADH molecules from glycolysis making 2.5 ATP each during chemiosmosis) - 2ATP (NADH active transport into mitochondria) = 5 ATP. Reflect on Process Did your problem-solving process lead you to the correct answer? If not, where did the process break down or lead you astray? How can you revise your approach to produce a more desirable result? • Answering this question correctly depended on your ability to analyze the different ways that ATP could be produced both during glycolysis as well as from glycolytic products and to account for the ATP costs across the board. • If you got the correct answer, well done! If you got an incorrect answer, were you able to accurately tally how many ATP were made via substrate-level phosphorylation in glycolysis? Did you consider how many ATP could be produced from the NADH molecules produced Version 1
38
during glycolysis? Did you account for the energy costs associated with glycolysis itself as well as NADH active transport into the mitochondria? 41) C
Version 1
39
Clarify Question What is the key concept addressed by the question? • The question asks you to determine the predicted ATP yield of the citric acid cycle per molecule of glucose. What type of thinking is required? • You are being asked to dissect, or analyze, the citric acid cycle for ATP and NADH production to determine the total yield of ATP per glucose molecule. Gather Content What do you already know about the citric acid cycle and ATP and NADH yield? What other information is related to the question? • Recall that a key part of cellular respiration is the citric acid cycle, which occurs in the mitochondria. Products from the citric acid cycle include NADH, FADH2, and a few ATP, among other molecules. • The citric acid cycle must be able to continuously receive acetyl-CoA, a 2-carbon molecule that is converted from a 3-carbon pyruvate (generated during glycolysis) immediately preceding the citric acid cycle. Each 6-carbon glucose molecule is first broken into two 3-carbon pyruvate molecules during glycolysis, which are in turn oxidized into two 2-carbon acetyl-CoA molecules with two molecules of carbon dioxide produced as well. Each acetyl-CoA molecule is able to enter the citric acid cycle. • At the beginning of the citric acid cycle each 2-carbon acetyl-CoA molecule joins with a 4-carbon oxaloacetate molecule to form a 6carbon citrate molecule. In order to take in another acetyl-CoA, the 4carbon oxaloacetate has to be regenerated, prompting the release of two carbon dioxide molecules per pyruvate. Recall that each time a carbon dioxide molecule is produced, bonds are broken and high-energy electrons are released, which are captured by NADH. As it turns out, this Version 1
40
kind of bond breaking happens several times per acetyl-CoA during the citric acid cycle, prompting both NADH and FADH2 to be produced. • How many acetyl-CoA molecules are produced from one glucose molecule? What does that tell you about how many times the citric acid cycle will turn per glucose? Choose Answer Do you have all necessary information to determine which products would affect cellular respiration? • As we figured out earlier, the citric acid cycle runs twice per glucose molecule. This is because glucose is broken down into twomolecules of pyruvate, each of which are converted to acetyl-CoA through pyruvate oxidation. Both acetyl-CoA molecules can enter the citric acid cycle. • Remember that bond breakage produces high-energy electrons that are captured by NADH. This happens several times each time the citric acid cycle turns, which produces 3 NADH, 1 FADH2, and 1 ATP molecule. Since the citric acid cycle turns twice per glucose molecule, this means that 6 NADH, 2 FADH2, and 2 ATP are made. Since 2.5 ATP are made per NADH and 1.5 ATP are made per FADH2 from ATP synthase, the number of ATP that are made from high-energy electron carriers is 2.5 x 6 NADH + 1.5 x 2 FADH2 = 18 ATP. • Finally, remember that the citric acid cycle also produces 1 ATP per turn via substrate-level phosphorylation. Since there are 2 turns per glucose, 2 ATP are made. When added to the 18 ATP produced from NADH and FADH2, the total is 20 ATP. Reflect on Process Did your problem-solving process lead you to the correct answer? If not, where did the process break down or lead you astray? How can you revise your approach to produce a more desirable result? • Answering this question correctly depended on your ability to analyze Version 1
41
the citric acid cycle for the different ways that ATP could be produced both from NADH and FADH2 as well as from substrate-level phosphorylation. • If you got the correct answer, good thinking! If you got an incorrect answer, were you able to accurately tally how many NADH and FADH2 molecules were produced during one turn of the citric acid cycle? Did you realize that the citric acid cycle turns twice per glucose? Were you able to calculate the number of ATP produced from NADH, FADH2 as well as the 2 ATP produced via substrate-level phosphorylation in the citric acid cycle? 42) B 43) B
Version 1
42
Clarify Question What is the key concept addressed by the question? • The question asks you to determine what would happen in a cell that has high citrate levels and low ATP levels. What type of thinking is required? • You are being asked to dissect, or analyze, cellular outcomes when citrate levels are high and ATP levels are low. Gather Content What do you already know about the stages of cellular respiration and how they are regulated? What other information is related to the question? • Remember that cellular respiration is comprised of glycolysis, pyruvate conversion, the citric acid cycle, and electron transport. Recall also that regulating each of these stages involves feedback loops that help the cell to optimize efficient use of resources. • Remember that citrate results from the joining together of acetyl-CoA and oxaloacetate at the beginning of the citric acid cycle. Choose Answer Do you have all the information needed to determine what would happen to cellular respiration? • Citrate levels help the cell to sense when ATP production needs to be increased or decreased. Citrate actually inhibits phosphofructokinase, a key enzyme in the early stages of glycolysis. Thus, you can conclude that glycolysis is probably largely shut down in this question. • However, the fact that ATP levels are low would normally stimulate cellular respiration due to an increased need for ATP. Low levels of ATP will actually stimulate the citric acid cycle, but since glycolysis is essentially shut down, where can a cell obtain the acetyl-CoA it needs to Version 1
43
run the citric acid cycle? Remember that glucose is not the only fuel that can be metabolized by cells. Fats, and in some cases proteins, can be used as well. • Since no acetyl-CoA will be arriving from glycolysis, and the demand for increased ATP remains undiminished, a fuel source other than glucose will need to be used. That fuel source is most likely to be fatty acid breakdown via beta-oxidation. By converting fatty acids into 2carbon acetyl-CoA molecules, the citric acid cycle can continue to turn, NADH and FADH2 molecules will still be made, the proton gradient will continue to be established, and ATP synthase will continue to turn and produce ATP. Reflect on Process Did your problem-solving process lead you to the correct answer? If not, where did the process break down or lead you astray? How can you revise your approach to produce a more desirable result? • Answering this question correctly depended on your ability to analyze what would happen to a cell with high citrate levels and low ATP levels. • If you got the correct answer, nice work! If you got an incorrect answer, were you able to recall the stages of cellular respiration and how they are regulated? Were you able to use information from the question to realize that glycolysis was not working, and therefore could not supply acetyl-CoA to the citric acid cycle? Were you able to reason that organic molecules other than glucose would have to supply the necessary ATP? 44) C 45) C 46) B 47) C 48) D
Version 1
44
Clarify Question What is the key concept addressed by the question? • The question asks you to determine what might happen in patients treated with antiviral drugs. What type of thinking is required? • You are being asked to dissect, or analyze, the possible outcomes of antiviral drug treatment. Gather Content What do you already know about which cellular respiration processes occur in mitochondria? How would a reduction in mitochondria affect the ability of the cell to harvest energy? What other information is related to the question? • Remember that cellular respiration is comprised of glycolysis, pyruvate oxidation, the citric acid cycle and electron transport. With the exception of glycolysis, all of these processes are related to aerobic respiration, which occurs inside the mitochondria. • If you follow the line of logic, what possible options are a cell left with, if it cannot perform pyruvate oxidation, the citric acid cycle, or electron transport? Choose Answer Do you have all necessary information to determine the outcome on respiration from taking antiviral drugs? • Let’s examine each answer in order to determine the correct one. We know that oxidative phosphorylation occurs during the electron transport chain and results from aerobic conditions. Considering that the electron transport chain is physically located inside the inner mitochondrial membrane, if the number of mitochondria decreases, so does the ability to conduct oxidative phosphorylation. That can’t be the right answer. Version 1
45
• Phosphofructokinase is an enzyme that operates during glycolysis. Even though glycolysis doesn’t involve aerobic conditions, the question indicates that there would be an increase in phosphofructokinase activity. That typically happens when you need to ramp up ATP production, and since there are no mitochondria present due to the antiviral drugs, phosphofructokinase activity is more likely to go down. Likewise, NADH dehydrogenase activity is likely to go down for the same reasons described above. • By process of elimination, that leaves an increase in lactic acid levels. But to make sure that is the correct response let’s examine it further. If you have no mitochondria, aerobic respiration is not possible. Cells still need energy, and so how will they obtain it? Using the same mechanism that is required when cells have no oxygen but still have mitochondria present: lactic acid fermentation. Reflect on Process Did your problem-solving process lead you to the correct answer? If not, where did the process break down or lead you astray? How can you revise your approach to produce a more desirable result? • Answering this question correctly depended on your ability to analyze the likely outcome for energy metabolism in patients who take antiviral drugs. • If you got the correct answer, nice job! If you got an incorrect answer, were you able to associate the stages of cellular respiration with mitochondrial anatomy? Were you able to eliminate pyruvate oxidation, the citric acid cycle, and the electron transport chain as possible options because no mitochondria were present? Were you able to infer that the inability of cells exposed to antiviral drugs to aerobically harvest energy would necessitate anaerobic energy harvesting processes like lactic acid fermentation? Version 1
46
49) D 50) B
Version 1
47
Clarify Question What is the key concept addressed by the question? • The question asks you to determine how a cell that can’t convert pyruvate into lactate would be able to generate energy in the presence of oxygen if ample glucose is present. What type of thinking is required? • You are being asked to dissect, or analyze, how a cell with a mutated enzyme that can convert pyruvate into lactate would be able to generate energy, considering that plenty of oxygen and glucose are present. Gather Content What do you already know about how cells use multiple fuels to generate energy? How does this relate to the inability of the cell to convert pyruvate into lactate? What other information is related to the question? • Recall that cells are able to use multiple fuel sources, including carbohydrates, fats, and protein, as necessary. Carbohydrates are most easily metabolized, followed by fats and then protein. The conditions under which each is metabolized during cellular respiration is largely determined by the internal and external environment and the energy needs of the cell. • At the end of glycolysis, cells can choose to use aerobic processes like the citric acid cycle if there is sufficient oxygen available, or they can use anaerobic processes like lactic acid fermentation when there is no oxygen. In this question, the cell has a mutation that prevents the enzyme that converts pyruvate into lactate from functioning at all. This means that lactic acid fermentation is not an option for the cell. • Recall that glycolysis does not depend on whether the environment is aerobic or anaerobic; however what happens to pyruvate at the end of Version 1
48
glycolysis is dependent on environmental conditions. In this case, the question indicates there is plenty of oxygen, so downstream processes like the citric acid cycle and electron transport are possible. Choose Answer Do you have all necessary information to determine which products would affect cellular respiration? • Let’s try to narrow down the possible options for this question. Recall that human cells are capable of lactic acid fermentation, not ethanol fermentation. Furthermore the question indicates there is plenty of glucose available, so there would be no need to break down proteins into amino acids to derive energy for the cell. Otherwise, the cell would have no problem going through glycolysis and, if conditions are right, carrying out the citric acid cycle and electron transport to generate cellular energy. • The question also indicates that oxygen is present. If oxygen is present, these cells should have no problem doing aerobic respiration. Just because the cell contains a mutated enzyme and can’t convert pyruvate into lactate shouldn’t really matter, because there is plenty of oxygen around and nothing to prevent aerobic processes like the citric acid cycle and electron transport chain from functioning normally. Reflect on Process Did your problem-solving process lead you to the correct answer? If not, where did the process break down or lead you astray? How can you revise your approach to produce a more desirable result? • Answering this question correctly depended on your ability to analyze how a human cell that has a mutated enzyme and can’t convert pyruvate into lactate would generate energy assuming ample glucose and oxygen are available. • If you got the correct answer, excellent! If you got an incorrect Version 1
49
answer, were you able to remember which stages of cellular respiration require oxygen and which do not? Were you able to eliminate ethanol fermentation and protein breakdown as less likely options? Were you able to infer that the cell could still participate in aerobic respiration because there was plenty of glucose and oxygen available? 51) E 52) D 53) D
Version 1
50
Clarify Question What is the key concept addressed by the question? • The question asks you to determine whether or not it is a good idea for your friend to only consume carbohydrates in an attempt to lose weight. What type of thinking is required? • You are being asked to analyze the feasibility of your friend losing weight by only consuming carbohydrates. Gather Content What do you already know about consuming carbohydrates and weight loss? Is excessive carbohydrate intake a good or bad idea? What other information is related to the question? • We hear in the news all the time that we consume too many carbohydrates in our diets. Why the concern? What harm could it be? Since we know that our bodies metabolize organic fuels through cellular respiration, let’s use what we already know about how cellular respiration works. • Organic fuels are broken down via cellular respiration, including glycolysis, pyruvate oxidation, the citric acid cycle, and electron transport. These are catabolic processes that convert bonds in organic molecules into energy stored in ATP that cells use to do work. But what happens when we have all the energy our cells need? What do we do with the excess molecules? Choose Answer Do you have all the information needed to determine whether or not excessive consumption of carbohydrates will help or harm your friend? • The metabolism in our cells is a finely-tuned balance of catabolic and Version 1
51
anabolic processes. We use catabolic processes to break down organic molecules and harness their energy via cellular respiration. However, when our cells have all the energy (ATP) they need, they tend to save the excess organic molecules. This allows the cell to store reserve energy for when it needs organic molecules but none are available. • If your friend consumes nothing but carbohydrates, ATP levels will be high and anabolic processes will engage. This will prompt excess acetylCoA to be converted into fatty acids, which is converted into fat. So, consuming nothing but carbohydrates is bad because there is no way for your cells to use it all and it will eventually get stored as fat. Reflect on Process Did your problem-solving process lead you to the correct answer? If not, where did the process break down or lead you astray? How can you revise your approach to produce a more desirable result? • Answering this question correctly depended on your ability to analyze whether or not your friend could lose weight by consuming nothing but carbohydrates. • If you got the correct answer, well done! If you got an incorrect answer, were you aware that metabolism is a balance of catabolic and anabolic processes that sense cellular ATP levels? Were you able to infer that excess carbohydrate would lead to high ATP levels, prompting the extra acetyl-CoA to be converted into fatty acids? Were you able to draw the conclusion that eating only carbohydrate is a bad idea? 54) B
Version 1
52
Clarify Question What is the key concept addressed by the question? • The question asks you to decide whether or not cancer cells that secrete lactate are undergoing beta-oxidation. What type of thinking is required? • You are being asked to dissect, or analyze, the likelihood that cancer cells that secrete lactic acid are undergoing beta-oxidation. Gather Content What do you already know about lactate fermentation? How do products from this process indicate how cancer cells derive energy? What other information is related to the question? • Recall that cellular respiration is a process of oxidizing glucose in order to produce NADH and ultimately the ATP used to power cellular processes. Respiration is comprised of glycolysis, oxidation of 3-carbon into 2-carbon molecules that then are incorporated into the citric acid cycle, and electron transport and oxidative phosphorylation. With the exception of glycolysis, all of these processes are related to aerobic respiration. • At the end of glycolysis, cells can choose to use aerobic processes like the citric acid cycle if there is sufficient oxygen available, or they can use anaerobic processes like lactic acid fermentation when there is no oxygen. Cancer cells in particular tend to use anaerobic processes. • Recall that molecules other than glucose are metabolized for energy in cells; this includes breakdown of fatty acids and glycerol and, in certain circumstances proteins into amino acids. Each of these molecules plugs into cellular respiration in different places and under different conditions. The process of breaking down fatty acids is called betaoxidation and involves cutting fatty acid into two-carbon fragments Version 1
53
(acetyl-CoA) that plug into the citric acid cycle. Choose Answer Do you have all necessary information to determine which products would affect cellular respiration? • The Warburg effect, discovered by Otto Warburg and since confirmed by observations from other scientists, indicates that many cancer cells derive their energy mainly through the glycolytic pathway coupled with lactic acid (lactate) fermentation. If lactate is produced, that suggests these cells are not engaging in aerobic respiration, including the citric acid cycle, and/or the electron transport chain. This is because cancer cells are using fermentation to regenerate NAD+, instead of using the electron transport chain. • Since beta-oxidation generates acetyl-CoA, which can only be used in the citric acid cycle, the observation that these cells are generating lactate suggests that they are not undergoing beta-oxidation. Betaoxidation does not lead to generation of intermediates that can lead to lactate production, nor does lactate stimulate beta-oxidation, and lactate is not consumed through the process of beta-oxidation. • The long story short here is that cancer cells use anaerobic processes to generate lactate, and since beta-oxidation of fatty acids requires oxygen, cancer cells that operate in the absence of oxygen aren’t using beta-oxidation. Reflect on Process Did your problem-solving process lead you to the correct answer? If not, where did the process break down or lead you astray? How can you revise your approach to produce a more desirable result? • Answering this question correctly depended on your ability to analyze whether or not cancer cells that secrete higher levels of lactate are likely undergoing beta-oxidation. Version 1
54
• If you got the correct answer, well done! If you got an incorrect answer, were you able to recall the stages of cellular respiration and which require oxygen? Were you able to correlate higher levels of lactate with anaerobic metabolism? Were you able to infer that the anaerobic conditions needed for lactate metabolism would prevent aerobic processes like beta-oxidation? 55) B 56) D
Version 1
55
Clarify Question What is the key concept addressed by the question? • The question asks you to determine the ATP yield for beta-oxidation of a 10-carbon fatty acid. What type of thinking is required? • You are being asked to use what you already know about betaoxidation of fatty acids and use it to determine, or analyze, the ATP yield for a 10-carbon fatty acid. Gather Content What do you already know about cellular respiration and betaoxidation of fatty acids? How do these molecules plug into the process of cellular respiration? What other information is related to the question? • Recall that cellular respiration is a process of oxidizing glucose and other organic molecules in order to produce NADH and ultimately the ATP used to power cellular processes. Respiration is comprised of glycolysis, pyruvate oxidation, the citric acid cycle, and electron transport. • Molecules other than glucose are metabolized for energy in cells; this includes the breakdown of fatty acids. The process of breaking down fatty acids is called beta-oxidation and involves cutting fatty acid into two-carbon fragments (acetyl-CoA) that enter into the citric acid cycle. Choose Answer Do you have all necessary information to determine the ATP yield from beta-oxidation of a 10-carbon fatty acid? • For each round of the citric acid cycle, the ATP yield is 10 ATP. To calculate the 10 ATP, remember that one FADH2and three Version 1
56
NADHmolecules are produced per turn of the citric acid cycle. The NADH molecules have an ATP value of 7.5 (3 NADH x 2.5 ATP each), and theFADH2molecule has an ATP value of 1.5 (1 NADH x 1.5 ATP each), giving a total of 9 ATP. Add to that theone ATP produced via substrate-level phosphorylationfor a total of 10 ATP. • For each round of beta-oxidation, one molecule of ATP is invested, and a molecule of NADH and FADH2 are generated. This means that for each round of beta-oxidation, a net of 3 ATP are generated. For a 10carbon fatty acid, there would be four rounds of beta-oxidation to cleave a 10-carbon molecule into five 2-carbon molecules. Thus, from betaoxidation there would be a yield of 12 ATP (4 rounds of beta-oxidation x 3 ATP per round). •Each of these five 2-carbon acetyl-CoA molecules could enter the citric acid cycle; thus the citric acid cycle will run five times for a total yield of 50 ATP. Taken together, the oxidation of a 10-carbon fatty acid would yield 62 ATP. Reflect on Process Did your problem-solving process lead you to the correct answer? If not, where did the process break down or lead you astray? How can you revise your approach to produce a more desirable result? • Answering this question correctly depended on your ability to remember how beta-oxidation works during cellular respiration and use that to analyze the ATP yield of a 10-carbon fatty acid. • If you got the correct answer, well done! If you got an incorrect answer, were you able to recall how beta-oxidation of fatty acids works? Were you able to correctly calculate how many NADH and FADH2 molecules would be produced from a single 2-carbon fragment from a fatty acid and a turn of the citric acid cycle? Were you able to determine the total number of ATP from beta-oxidation reactions, ATP synthasegenerated ATP from NADH and FADH2, and ATP made via substrateVersion 1
57
level phosphorylation? 57) C
Version 1
58
Clarify Question ● What is the key concept addressed by the question? question asks about factors that affect the P/O ratio. ● What type of thinking is required? analyze factors that affect the P/O ratio.
● The
● You are being asked to
Gather Content ● What do you know about factors that affect the P/O ratio? What other information is related to the question? ● The value for the amount of ATP synthesized per O2 molecule reduced is called the phosphate-to-oxygen ratio (P/O ratio). For each NADH or FADH2 that goes through the electron transport chain, one molecule of oxygen is consumed. We need to know the number of protons pumped during electron transport: 10 H+ per NADH, and 6 H+ per FADH2. Then we need to know the number of protons needed per ATP. Because ATP synthase is a rotary motor, this calculation depends on the number of binding sites for ATP and the number of protons required for rotation. We know that ATP synthase has three binding sites for ATP. If 12 protons are used per rotation, you get the value of 4 H+ per ATP. If NADH pumps 10 H+ per oxygen the P/O ratio would be 10 ATP/4 O2 = 2.5. Choose Answer ● Given what you now know, what information is most likely to produce the correct answer? ● If the number of protons to turn the ATPase are increased to 16, then that would correlate to just over 5 protons per ATP. The P/O ratio would then be 10 ATP / 5 O2 = 2. So the ratio would decrease. Reflect on Process Version 1
59
● Did your problem-solving process lead you to the correct answer? If not, where did the process break down or lead you astray? How can you revise your approach to produce a more desirable result? ● This question asked you to analyze factors that affect the P/O ratio. If you got the correct answer, great job! If you got an incorrect answer, where did the process break down? Did think that more protons required for one turn would produce more ATP? Did you understand that the P/O ration reflected ATP produced per oxygen consumed?
58) A 59) A
Version 1
60
Clarify Question ● What is the key concept addressed by the question? ● The question asks about the regulation of pyruvate dehydrogenase. ● What type of thinking is required? ● You are being asked to apply your knowledge about the regulation of pyruvate dehydrogenase. Gather Content ● What do you know about the regulation of pyruvate dehydrogenase? What other information is related to the question? ● The main control point in the oxidation of pyruvate occurs at the committing step in the citric acid cycle with the enzyme pyruvate dehydrogenase, which converts pyruvate to acetyl-CoA. This enzyme is inhibited by high levels of NADH, a key product of the citric acid cycle. This allows the cell to shut down NADH production when it has enough to meet its needs for producing ATP. Choose Answer ● Given what you now know, what information is most likely to produce the correct answer? ● If pyruvate dehydrogenase cannot bind NADH it will not be inhibited when the cell has enough energy. This will cause the cell to produce ATP at a high rate and more than the cell needs. Reflect on Process ● Did your problem-solving process lead you to the correct answer? If not, where did the process break down or lead you astray? How can you revise your approach to produce a more desirable result? ● This question asked you to apply your understanding of the regulation of pyruvate dehydrogenase. If you got the correct answer, great job! If you got an incorrect answer, where did the process break down? Did you Version 1
61
recall that NADH inhibits pyruvate dehydrogenase? Did you understand that without regulation of pyruvate dehydrogenase ATP production would increase?
Version 1
62
CHAPTER 8 MULTIPLE CHOICE - Choose the one alternative that best completes the statement or answers the question. 1) Carbon dioxide and water can combine to form glucose, water and oxygen. What is required for that process to occur? A) Nothing, this is a spontaneous reaction. B) energy from the process of cellular respiration C) light energy from the sun D) mitochondria
2)
Flattened sacs of internal membranes associated with photosynthesis are called A) chloroplasts. B) photosystems. C) the stroma. D) thylakoids. E) cristae.
3) The chloroplast is a membrane-bound organelle that is surrounded by an outer membrane. Aside from the outer membrane of the chloroplast, how many other types of membranes can be found inside the chloroplast? A) zero B) one C) two D) three E) four
4)
Inside the chloroplast, where are organic molecules made?
Version 1
1
A) stroma B) on the thylakoid membrane C) between the outer and inner membranes D) inside the thylakoid E) on the cristae
5)
In eukaryotes, photosynthesis takes place inside ________. A) chloroplasts B) mitochondria C) the cytoplasm D) the Golgi apparatus E) central vacuoles
6) Jan Baptista van Helmont did an important experiment related to photosynthesis. He weighed a small willow tree and a pot of soil. Janthen planted the tree in the pot. Over the course of five years, he added only water to the pot. At the end of five years, he weighed the tree and found that it had gained 74.4 kg. He weighed the soil, and it was only 57 g less. What do the results of this specific experiment demonstrate about what is needed for a plant to grow and increase its mass? A) All of the food aplant needs to grow comes from the soil. B) Plants do not needsoil to grow. C) Plants must beable to increase their mass using substances in addition to what is found insoil. D) All of the food aplant needs to grow comes from the water. E) Plants need oxygento grow.
7) In his 1771 experiments, Joseph Priestly determined that plants added something to the air that allow an animal to survive. Now we know that the essential thing that plants contribute to the air is
Version 1
2
A) oxygen. B) carbon dioxide. C) organic sugars. D) water.
8) The aquatic plant Elodea is commonly used to study photosynthesis. You put some Elodea in a test tube, shine light on the plant, and count the bubbles that were released under varying conditions. Which results would you expect to see in your analysis of photosynthetic rate? A) At high light intensities, the bubbles increase as carbon dioxide concentration increases. B) At low light intensities, the bubbles increase as temperature increases and the bubbles decrease as carbon dioxide concentration decreases. C) At high light intensities, the bubbles increase suddenly when the temperature is increased to human body temperature (37 degrees C). D) The bubbles increase indefinitely with increased illumination. E) As light intensity increases, the bubbles increase and then rapidly decrease.
9) F. F. Blackman performed experiments to investigate the effects of various factors on photosynthesis. In one of his experiments, Blackman found that if light intensity was low, photosynthesis could be accelerated by increasing the amount of light, but not by increasing the temperature or carbon dioxide concentration. What can you conclude from this specificexperiment? A) Light is important for photosynthesis. B) Carbon dioxide is important for photosynthesis. C) High temperatures are important for photosynthesis. D) Photosynthesis generates oxygen.
10) Most atmospheric oxygen comes fromphotosynthesis. From what molecule is oxygen derived?
Version 1
3
A) water B) carbon dioxide C) glucose D) chlorophyll a E) chlorophyll b
11) The aquatic plant Elodea is commonly used to study photosynthesis. You put some Elodea in a test tube filled with water containing radiolabeled oxygen (H 2 18O), shine light on the test tube, and measure the rate of photosynthesis by counting oxygen bubbles released. At the end of your experiment, you wouldfind the most 18O in A) CO 2. B) glucose. C) O 2. D) ATP.
12)
In plants, production of NADPH A) requires electronsoriginally found in water. B) requires theproduction of a proton gradient. C) occurs during theCalvin cycle. D) occurs in theabsence of light.
13) A researcher isolates chloroplasts in a tube and illuminates them, but does not provide any carbon dioxide. The chloroplasts will A) produce ATP and NADPH, but will not produce glucose. B) not produce any ATP, NADPH, or glucose. C) produce NADH and glucose, but will notproduce any ATP. D) produce glucose, but will not produce ATP or NADPH.
Version 1
4
14)
Visible light has a wavelength range of: A) 0.001–100,000 nanometers. B) 200–800 nanometers. C) 200–740 nanometers. D) 200–400 nanometers. E) 400–740 nanometers.
15)
Light consists of units of energy called A) electrons. B) photons. C) calories. D) neutrons. E) pigments.
16)
What is the color of light that chlorophylls absorb thathas the highest energy? A) green B) violet-blue C) red D) yellow-orange
17)
What are the characteristics of red light, which is absorbed by chlorophylls? A) Its photons have theshortest wavelength in the visual spectrum. B) Its photons have the highest energy in the visual spectrum. C) Its photons have the longest wavelength in the visual spectrum. D) It is not considered a part of the visual spectrum.
Version 1
5
18)
Based on a variety of experiments, it is known that the rate of photosynthesis is highest when wavelengths of light between 400 and 500 nm are used, and when wavelengths around 700 nm are used. What does this indicate about the relative importance of chlorophyll a and b in photosynthesis? A) Reflection of light by chlorophyll a and b is essential for the process of photosynthesis. B) Absorption of light by chlorophyll a and b is essential for the process of photosynthesis. C) Transmission of light by chlorophyll a and b is essential for the process of photosynthesis. D) Chlorophyll a and b are much less important than other pigments in the reflection, absorption, and transmission of light relevant to photosynthesis.
19)
Molecules that absorb light are called
Version 1
6
A) enzymes. B) electron carriers. C) pigments. D) photosynthesizers. E) absorbers.
20)
What color of light is not strongly absorbed by chlorophyll? A) green B) violet-blue C) red D) yellow-orange
21) If chlorophyll a is blocked from absorbing light, what would be the effect on the ability of the plant to do photosynthesis? A) The plant will have an enhanced ability to carry out light-dependent reactions. B) The plant will have an enhanced ability to carry out light-independent reactions. C) The plant will have less ability to carry out light-dependent reactions. D) The plant's ability to carry out photosynthesis will be unchanged, as one of the accessory pigments can fill in for chlorophyll a.
22)
Fall leaf color on deciduous trees is a result of
A) the production of more accessory pigments because of the cooler temperatures. B) the reduction in the production of accessory pigments. C) absence of chlorophyll, which allows the accessory pigments to be revealed. D) the increased angle of the sun during the fall, which reflects more of the accessory pigments.
23)
Carotenoids are important to many plants because these pigments are able to
Version 1
7
A) remove carbon dioxide from the air. B) absorb wavelengths of light that neither chlorophyll a nor b can absorb. C) absorb water so that hydrolysis can be carried out in the chloroplasts. D) capture UV radiation that is harmful to the DNA in the nucleus of plant cells. E) store electrons for use during the "dark" reaction of photosynthesis.
24)
Why is eating carrots associated with vision?
A) High levels ofchlorophyll in carrots helps with photosynthesis in the eyes. B) Carotenoid pigments in carrots are structurally similar to the pigment retinal, used in the vertebrate eye. C) High levels ofcarotenoids in carrots can allow vertebrate eyes to perform the darkreactions. D) High levels ofNADPH in carrots can allow vertebrate eyes to perform the light reactions.
25) Chlorophyll b absorbs green wavelengths of light that chlorophyll a cannot absorb. In this respect, chlorophyll b acts as A) an accessory pigment. B) an energizer for photosynthetic bacteria. C) a light absorber in the green light. D) a more efficient pigment.
26) In 1932, researchers Emerson and Arnold showed that the rate of photosynthesis saturated at a lower light intensity than expected for the number of individual chlorophyll molecules. From this observation they determined
Version 1
8
A) that they had overestimated the number of chlorophyll molecules present in their test organism. B) that high light intensities destroyed most pigment molecules. C) that pigments were clustered in photosystems and they were measuring saturation of pigments in reaction centers. D) that plants must be able to fix carbon before their pigments can absorb light.
27) A photosystem channels the excitation energy gathered by absorption of light by any one of the pigment molecules to a specific reaction center chlorophyll, which in turn passes the energy to A) accessory pigments. B) photosystem II. C) the primary electron acceptor. D) the secondary electron center. E) cytochrome.
28)
The role of the antenna complex in a photosystem is to A) transfer excitedelectrons to the primary electron acceptor. B) generateglucose. C) generate NADPH. D) capture photonsfrom sunlight.
29) If the antenna complexes were smaller in a particular plant mesophyll cell, what effect would that have on photosynthesis? A) There would be no expected effect on photosynthesis in that cell. B) That cell would be unable to absorb any light energy. C) That cell would have a reduced capacity to generate glucose. D) That cell would be able to fix more carbon dioxide.
Version 1
9
30)
Clusters of chlorophyll and accessory pigments are called ________. A) the Golgi apparatus B) chloroplasts C) photosystems D) photosynthetic membranes E) cristae
31)
In plants, the light-dependent reactions require A) ATP. B) O 2. C) NADP +. D) glucose. E) Calvin cycleenzymes.
32)
For photosynthesis in green plants, the electron donor for the light-dependent reaction is A) carbon dioxide. B) oxygen. C) RuBP. D) chlorophyll II. E) water.
33) In the photosystem Ireaction center of green plants, light energy captured by pigment molecules is passed to a special reaction center chlorophyll a called
Version 1
10
A) P 680. B) P 700. C) chlorophyll I. D) chlorophyll II. E) P 870.
34) You are working on aresearch project thatinvolves a mutational analysis of the lightharvesting complex Lhca1, which is an important light-harvesting complex found in plants. Your labmate asks where this complex is located. You say it is in the A) outer chloroplast membrane. B) inner chloroplast membrane. C) thylakoid membrane. D) stroma. E) matrix.
35) If the Calvin cycle were to betemporarily restricted, the highest concentration of NADPH would likely be in the A) thylakoidspace. B) thylakoidmembrane. C) stroma. D) chloroplast outermembrane.
36)
The splitting of water and the generation of oxygen occur where? A) photosystem I B) the citric acid cycle C) the Calvin cycle D) photosystem II
37)
What is common to both photosystems I and II?
Version 1
11
A) Both involve the splitting of water to donate an electron to the reaction center. B) Both involve the generation of oxygen. C) Both lose an electron to a primary electron acceptor that passes the electron down an electron transport chain leading to the generation of ATP. D) Both contain a reaction center composed of chlorophyll a. E) Both are found in the stroma.
38)
NADPH is made by A) chemiosmosis through NADPH synthase. B) the Krebs cycle in the stroma. C) the Calvin cycle in the stroma. D) the passing ofelectrons from photosystem I to an electron transport chain.
39) In green plants, which photosystem absorbs photons to excite electrons in the reaction center? A) photosystem I B) photosystem II C) photosystem III D) photosystems I and II E) photosystems I, II, and III
40)
What products of light reactions of photosynthesis are used in the Calvin cycle? A) oxygen and protons B) carbon dioxide and water C) ATP and NADPH D) ADP and NADP E) glucose and oxygen
Version 1
12
41)
Light-dependent reactions generate A) ATP andNADPH. B) O 2, ATP, and NADPH. C) light. D) ATP and CO 2. E) glucose and O 2.
42)
In photosynthesis conducted by plants, ATP is made by A) chemiosmosis. B) glycolysis. C) the citric acid cycle. D) the Calvincycle. E) the passing ofelectrons from photosystem I to an electron transport chain.
43) A mutant tomato plant was created usinggamma radiation. A cellular analysis of the mutant reveals that ATP synthase complexes found in the thylakoid membrane are inserted backwards into the membrane. What is the most likely outcome of this defect? A) A proton gradient will not be established in the thylakoid spacein response to the activation of photosystem II. B) The proton gradient in the thylakoid space will not be able to be used to generate ATP. C) The thylakoid membrane will become permeable to protons as a result of this change. D) The ATP synthase will pump protons into the stroma in this orientation. E) This change is not likely to have any effect on the function of the ATP synthase enzyme with respect to its role in photosynthesis.
44) During the light-dependent reactions,electron transport leads to the thylakoid space becoming
Version 1
13
A) the site of ATP synthesis. B) the site of NADPH production. C) the site where the Calvin cycle occurs. D) more acidic than the stroma.
45) As electrons travel from the primary electron acceptor in photosystem II down the electron transport chain to photosystem I, they drive the movement of protons from the stroma into the thylakoid compartment. Why is the development of this chemical gradient useful to photosynthesis? A) Protons are directly used in the thylakoid to make glucose. B) Protons provide the energy needed to drive the light-dependent reactions. C) The proton gradient can drive the production of ATP, which is needed for the lightindependent reactions. D) This proton gradient is used to make NADPH, which is needed for the Calvin cycle.
46)
In photosynthesis, carbon fixation occurs A) in photosystem I. B) in the electron transport chain. C) in photosystem II. D) in the Calvin cycle. E) during photorespiration.
47) If the gene encoding the enzyme rubisco is mutated such that it is nonfunctional, the process that would be affected is the ability to A) make ATP. B) harvest photons. C) fix carbon. D) make O2. E) make NADPH.
Version 1
14
48) Most plants incorporate carbon dioxide into sugars by means of a cycle of reactions called the A) CAM cycle. B) carbon cycle. C) Calvin cycle. D) Krebs cycle. E) citric acid cycle.
49)
The light-independent reactions of photosynthesis are those that A) convert glucoseinto energy. B) convertchlorophylls into light energy. C) convert water intohydrogen and oxygen. D) convert CO 2 into reduced molecules (sugars). E) occur only atnight.
50) In the light-independent reactions of photosynthesis, CO molecule known as
2 is added to a five-carbon
A) cellulose. B) ribose. C) deoxyribose. D) RuBP. E) CAM.
51)
The cyclic carbon fixation reactions are also known as the
Version 1
15
A) Krebs cycle. B) Calvin cycle. C) citric acidcycle. D) tri carboxylic acidcycle. E) glycolysis.
52)
The Calvin cycle requires all of the following except A) carbon dioxide. B) oxygen. C) ATP. D) NADPH.
53)
In order to generate one glucose molecule, the Calvin cycle has to turn how many times? A) 1 B) 2 C) 3 D) 6 E) 12
54) The number ofglyceraldehyde-3-phosphatemolecules that would be producedfrom 24 turns of the Calvin cycle would be A) 6 B) 8 C) 4 D) 12 E) 48
55)
How many molecules of CO
Version 1
2 are needed to generate one molecule of glucose?
16
A) 2 B) 3 C) 4 D) 5 E) 6
56)
At night, the Calvin cycle in a C
3 plant
A) cannot run, as itrequires light energy directly. B) can still run aslong as there is ATP, CO 2, and NADPHpresent. C) runs in adifferent place in the plant. D) uses a differentsource of carbon.
57) The ATP generated from cellular respiration is not sufficientto drive the Calvin cycle in plants, even if appropriate levels of NADPH and CO2 are present. Why not? A) Plants do not undergo cellular respiration. B) Breaking down sugar to provide the energy needed to synthesize sugar would be a futile cycle. C) The Calvin cycle requires more ATP than can possibly be made from cellular respiration. D) Cellular respiration and photosynthesis do not happen in the same cells in plants.
58) In the light-independent reactions, when CO bisphosphate (RuBP), the immediate product is
2 is added to a molecule of Ribulose 1,5-
A) citric acid. B) glucose. C) glyceraldehyde-3-phosphate. D) 3-phosphoglycerate. E) pyruvate.
Version 1
17
59) A plant researcher wants to construct a synthetic cellulose fiber from 20 glucose molecules. How many molecules of CO 2would be needed to construct the glucose for the fiber? A) 20 B) 40 C) 60 D) 80 E) 120
60) Which experimental modification would most effectively help to determine the sequence of reactions and reaction intermediates in the Calvin cycle? A) Exposing the cells to 14CO2 for various time intervals B) Exposing the cells to alternating periods of light and darkness C) Exposing the cells to radiolabeled 18O2for various time intervals D) Varying the amount of water the cells are exposed to
61)
In which part of the chloroplasts are the Calvin cycle enzymes located? A) stroma B) thylakoids C) grana D) envelope E) cristae
62) Which plantsutilize a specialized carbon fixation enzyme and a unique cell type/organization to reduce the problems of photorespiration?
Version 1
18
A) C 3 B) C 4 C) desert D) arctic E) CAM
63) Plants that show a pattern of stomatal opening and closing that is the reverse of C plants are called
3
A) C 4. B) temperate. C) CAM. D) Calvin cycle.
64) If a plant's stomata were closed more often, how would this affectthe plant's ability to make glucose? A) There would be no effect on the ability of the plant to make glucose. B) Plants would be able to make more sugar than normal. C) Plants would make less sugar than normal.
65) Since rubisco can either fix carbon or oxidize RuBP, what condition will be most favorable for glucose production? A) moderatetemperatures B) hightemperatures C) dry climate D) high oxygenenvironment
66) One of the disadvantages of the C photosynthesis. Version 1
4 pathway is that it requires _____ than C
3
19
A) more O 2 B) more NADPH C) more light D) more ATP E) a much higher temperature
67)
Which type of plant can utilize the Calvin cycle? A) C 3plants B) CAM plants C) C 4plants D) C 3, C 4, and CAMplants
68) The plant hormone abscisic acid (ABA) is involved in the regulation of photorespiration. Based on this information, ABA likely plays a role in A) the opening and closing of stomata. B) the photoelectric effect. C) cyclic photophosphorylation. D) noncyclic photophosphorylation.
69) If you exposed a C4 plant to 14CO2 in the light, which would be the first organic molecule labeled with 14C? A) oxaloacetate B) malate C) pyruvate D) phosphoenolpyruvate (PEP)
70)
CAM and C
Version 1
4 plants likely grow in what type of environment?
20
A) cool B) hot and arid C) high altitude D) in water
71) A protein that has been transported past the outer chloroplast membrane would have to pass how many more membranes to reach the stroma and thylakoid lumen, respectively? A) 0, 1 B) 1, 2 C) 2, 3 D) 3, 4
72) A plant is treated with a chemical that blocks the flow of electrons between photosystem II and photosystem I, such that protons are not transported from the stroma into the thylakoid compartment. What is the effect of this chemical on photosynthesis? A) The increased number of protons will be directly used in the thylakoid to produce glucose. B) An increased proton gradient will provide the energy needed to produce ATP from the light reactions. C) A decreased proton gradient will cause less ATP to be produced from the light reactions. D) The increased proton gradient will be used to make NADPH for the Calvin cycle.
73)
Where do the carbon atoms in glucose come from? A) sunlight B) carbon dioxide C) water D) NADPH E) oxygen
Version 1
21
Answer Key Test name: Chap 08_3e 1) C This describes the process of photosynthesis, which is an endergonic reaction requiring light energy. 2) D 3) C There are two types of membranes inside the outer membrane of the chloroplast: the inner membrane, and the thylakoid membrane. 4) A Organic molecules are made in the stroma, which contains the necessary enzymes to make these compounds. 5) A 6) C
Version 1
22
Clarify Question ● What is the key concept addressed by the question? question asks about the origin of the mass in a plant.
● The
● What type of thinking is required? ● You are being asked to apply your knowledge of the origin of the mass in a plant. Gather Content ● What do you know about the origin of the mass in a plant? What other information is related to the question? ● Photosynthesis uses energy from the sun to combine carbon dioxide gas molecules to produce glucose. Some minerals are taken up from the soil, but most of the mass of a plant is cellulose which is made from the glucose molecules. Choose Answer ● Given what you now know, what information is most likely to produce the correct answer? ● Because the plant increased its mass to a greater extent than the soil mass was depleted, this suggests that the plant can increase its mass using substances other than what is found in the soil. Reflect on Process ● Did your problem-solving process lead you to the correct answer? If not, where did the process break down or lead you astray? How can you revise your approach to produce a more desirable result? ● This question asked you to apply your understanding of the origin of the mass in a plant. If you got the correct answer, great job! If you got an incorrect answer, where did the process break down? Did you think that the mass of the plant was less than the loss of mass in the soil? Did you understand that the carbon in glucose comes from carbon dioxide gas? Version 1
23
7) A 8) A
Version 1
24
Clarify Question ● What is the key concept addressed by the question? ● The question asks about factors that affect the rate of photosynthesis. ● What type of thinking is required? ● You are being asked to apply your knowledge of factors that affect the rate of photosynthesis. Gather Content ● What do you know about the factors that affect the rate of photosynthesis? What other information is related to the question? ● Photosynthesis uses carbon dioxide and water as substrates and produces oxygen and glucose as products.The reaction is driven by energy provided by light.At low light less photosynthesis will occur and eventually as light is increased photosynthesis will plateau. Choose Answer ● Given what you now know, what information is most likely to produce the correct answer? ● Photosynthesis will increase as the intensity of light and amount of carbon dioxide increases. Reflect on Process ● Did your problem-solving process lead you to the correct answer? If not, where did the process break down or lead you astray? How can you revise your approach to produce a more desirable result? ● This question asked you to apply your understanding of factors that affect the rate of photosynthesis. If you got the correct answer, great job! If you got an incorrect answer, where did the process break down? Did you recall that carbon dioxide is a substrate for photosynthesis? Did you understand that at some point increasing the intensity of light will no longer increase photosynthesis?
Version 1
25
9) A
Version 1
26
Clarify Question What is the key concept addressed by the question? • The question asks you to draw a conclusion about photosynthesis based on Blackman’s experimental results. What type of thinking is required? • You are being asked to weigh and judge, or evaluate, Blackman’s experimental results and use that information to draw a conclusion. Gather Content What do you already know about photosynthesis? What other information is related to the question? • Recall that photosynthesis is a process of creating organic molecules from inorganic carbon sources using light energy. Photosynthesis is comprised of the light reactions and the Calvin cycle. Light and carbon dioxide are inputs of photosynthesis and oxygen and carbohydrate (among other molecules) are outputs of photosynthesis. • The observation that photosynthesis could be accelerated by increasing the amount of light but not by increasing temperature or carbon dioxide concentration is telling. How is this observation important to drawing a conclusion about photosynthesis in this experiment? Choose Answer Do you have all the information needed to draw a supported conclusion? • Let’s unpack each option and see how well it is supported by experimental observations. We know that plants need to carbon dioxide, but increasing carbon dioxide did not increase photosynthesis in this experiment. Likewise, increasing temperature did not increase Version 1
27
photosynthesis either. And while we know that photosynthesis generates oxygen, there is no connection between experimental observations and that outcome, so that is not a supported conclusion. • The process of elimination leaves us with light being important for photosynthesis as the logical conclusion, but why else would that make sense? Experimental observations showed that as light increased, so did photosynthesis. The fact that increased carbon dioxide did not increase photosynthesis would support the conclusion that light is essential. Reflect on Process Did your problem-solving process lead you to the correct answer? If not, where did the process break down or lead you astray? How can you revise your approach to produce a more desirable result? • Answering this question correctly depended on your ability to evaluate Blackman’s experimental results and draw an accurate conclusion based on those observations. • If you got the correct answer, good thinking! If you got an incorrect answer, were you able to weigh Blackman’s observations and correlate that with a conclusion? Were you able to eliminate carbon dioxide and high temperatures as options based on the observations? 10) A 11) C 12) A 13) A
Version 1
28
Clarify Question ● What is the key concept addressed by the question? ● The question asks about the role of carbon dioxide in photosynthesis. ● What type of thinking is required? ● You are being asked to apply your knowledge of the role of carbon dioxide in photosynthesis. Gather Content ● What do you know about the role of carbon dioxide in photosynthesis? What other information is related to the question? ● Photosynthesis uses carbon dioxide and water as substrates and produces oxygen and glucose as products. The reaction is driven by energy provided by light, producing NADPH and ATP in the light reaction. This energy is then used in the Calvin cycle to convert carbon dioxide into glucose. Choose Answer ● Given what you now know, what information is most likely to produce the correct answer? ● Carbon dioxide is used in the Calvin cycle where energy in ATP and NADPH is used to convert this into glucose. If carbon dioxide is absent, then ATP and NADPH will not be used and will accumulate. Glucose levels will drop as it cannot be produced without carbon dioxide. Reflect on Process ● Did your problem-solving process lead you to the correct answer? If not, where did the process break down or lead you astray? How can you revise your approach to produce a more desirable result? ● This question asked you to apply your understanding of the role of carbon dioxide in photosynthesis. If you got the correct answer, great job! If you got an incorrect answer, where did the process break down? Did you Version 1
29
recall that carbon dioxide, ATP and NADPH are all substrates for the Calvin cycle? Did you understand that without carbon dioxide present substrates would accumulate?
14) E 15) B 16) B Chlorophylls absorb violet-blue, blue, orange-red, and redlight. Of these, violet-blue light has the shortest wavelength and thus the highest energy. 17) C 18) B
Version 1
30
Clarify Question What is the key concept addressed by the question? • The question asks you to determine the relative importance of chlorophyll a and b in photosynthesis. What type of thinking is required? • You are being asked to decipher, or analyze, data from the graph to determine the relative importance of chlorophylls a and b in photosynthesis. Gather Content What do you already know about the role of pigments during photosynthesis? What other information is related to the question? • The first phase of photosynthesis includes the light reactions, where energy from the sun is captured using concentrated chlorophyll clusters organized in photosystems to remove electrons from water, excite them to a higher energy level, and to use redox reactions in order to eventually produce ATP and NADPH. • Each type of chlorophyll contributes differently to the overall ability of the plant to absorb solar energy and fix carbon. Given the data in the graph, what is the relative importance of chlorophylls a and b? Choose Answer Do you have all the information needed to analyze the graph and answer the question? • Chlorophylls are the main light-harvesting molecules that plants use to photosynthesize. This graph shows three types of pigment molecules; carotenoids, chlorophyll a, and chlorophyll b. • Let’s consider the graph itself for a moment. The x-axis shows the wavelength of light whereas the y-axis shows the amount of light that can be absorbed. Considering that photosynthesis is about absorbing light, it makes sense to eliminate answers that talk about reflection or transmission of light. Using the location and size of their absorption peaks, it is clear from reading the graph that chlorophylls a and b are important, so we can also eliminate the answer that indicates they are less important. • Based on the data presented in the graph, and process of elimination for incorrect answers, the correct response is chlorophyll a and b are essential for photosynthesis. Reflect on Process Did your problem-solving process lead you to the correct answer? If not, where did the process break down or lead you astray? How can you revise your approach to produce a more desirable result? Version 1
31
• Answering this question correctly depended on your ability to analyze the graph to determine the relative importance of chlorophylls a and b. • If you got a correct answer, well done! If you got an incorrect answer, did you recall that a plant’s ability to photosynthesize relies on pigments like chlorophyll? Did you remember that there are different types of chlorophyll molecules? Were you able to decipher data from the graph that shows the importance of chlorophylls a and b for light absorption and therefore photosynthesis?
19) C 20) A 21) C
Version 1
32
Clarify Question What is the key concept addressed by the question? • The question asks you to determine what would happen to photosynthesis if chlorophyll a was blocked from absorbing light. What type of thinking is required? • You are being asked to use what you know about photosynthetic pigments and use, or apply, this knowledge to determine what would happen to a plant when chlorophyll a was blocked from absorbing light. Gather Content What do you already know about the role of pigments during photosynthesis? What other information is related to the question? • Photosynthesis is comprised of the light reactions, which produces ATP, NADPH, and oxygen and the Calvin cycle, which produces organic molecules from inorganic carbon dioxide. The first phase of photosynthesis includes the light reactions, where energy from the sun is captured using concentrated chlorophyll clusters organized in photosystems to remove electrons from water, excite them to a higher energy level, and to use redox reactions in order to eventually produce ATP and NADPH. • Each of the two photosystems in plants is comprised of multiple chlorophyll molecules, one of which is chlorophyll a. If you blocked chlorophyll a, what you think would happen to the plant's ability to photosynthesize? Choose Answer Do you have all the information needed to determine what would happen? • Let’s see if we can eliminate some possible answers. First, we know Version 1
33
that blocking one of the main light-harvesting molecules will have an impact on photosynthesis, so the plant's ability to photosynthesize will certainly be changed. Second, light-independent reactions like the Calvin cycle don’t depend on light directly, so all other things being equal that would also not be a correct choice. The question then becomes about whether blocking chlorophyll a will enhance or decrease lightdependent reactions. • Let’s consider now what makes up a photosystem. Chlorophyll a molecules arranged in a cluster absorb photons from the sun and elevate the potential energy of an electron stripped from a water molecule. During this process the high-energy electron is passed to the photosystem reaction center and eventually on to an electron acceptor that begins the downstream process of electron transport, establishment of a proton gradient, and ATP production. Therefore, if you block chlorophyll a from absorbing light, fewer of those processes will take place, and the plant's ability to photosynthesize will suffer. Reflect on Process Did your problem-solving process lead you to the correct answer? If not, where did the process break down or lead you astray? How can you revise your approach to produce a more desirable result? • Answering this question correctly depended on your ability to apply what you know about chlorophyll a and its role in photosynthesis. • If you got a correct answer, good job! If you got an incorrect answer, did you recall that a plant’s ability to photosynthesize relies on pigments like chlorophyll a? Did you remember that chlorophyll a molecules are a key component of photosystems, and that photosystems are core to the light reactions ability to function? Were you able to infer that blocking chlorophyll a would then decrease photosynthesis? 22) C Version 1
34
23) B 24) B In no case can vertebrates perform the reactions of photosynthesis within their own cells. 25) A 26) C 27) C 28) D 29) C
Version 1
35
Clarify Question What is the key concept addressed by the question? • The question asks you to determine what would happen to photosynthesis if a particular plant mesophyll cell had smaller antenna complexes? What type of thinking is required? • You are being asked to dissect, or analyze, the photosynthetic outcome for this plant cell. Gather Content What do you already know about the light reactions, photosystems, and antenna complexes? What other information is related to the question? • The first phase of photosynthesis includes the light reactions, where energy from the sun is captured using chlorophyll to remove electrons from water, excite them to a higher energy level, and to use redox reactions in order to produce ATP and NADPH. The ability to capture solar energy depends on photosystems, which are collections of many pigment molecules, some of which have specialized functions. • Antenna complexes contain hundreds of chlorophyll molecules and have the specialized ability to vastly increase the amount of energy that can be harvested by the plant. The antenna complex serves to harvest and direct its energy so that the electrons stripped from water can move to the reaction center complex of the photosystem. Choose Answer Do you have the necessary information to dissect the problem? • A cell with reduced antenna complexes would havereduced capture of light energy, and thusless energy to participate in electron transport, Version 1
36
less proton pumping and a lower gradient, and ultimately less ATP and NADPH. • For the affected cell, less ATP and NADPH means less potential energy to drive the Calvin cycle, and less glucose will be produced as a result. Reflect on Process Did your problem-solving process lead you to the correct answer? If not, where did the process break down or lead you astray? How can you revise your approach to produce a more desirable result? • Answering this question correctly depended on your ability to analyze the photosynthetic outcome on a mesophyll plant cell with reduced antenna complexes. • If you got a correct answer, well done! If you got an incorrect answer, did you recall that the light reactions rely on photosystems? Did you remember that photosystems contain antenna complexes and reaction centers? Were you able to infer that less ATP and NADPH in the affected mesophyll cell would mean less energy for the Calvin cycle to fix carbon and would produce less glucose as a result? 30) C 31) C The light-dependent reactions generate ATP, NADPH, and oxygen. Calvin cycle enzymes are required for the light-independent reactions. Glucose is the ultimate product of photosynthesis 32) E 33) B 34) C
Version 1
37
Clarify Question What is the key concept addressed by the question? • The question asks you to determine where the light-harvesting complex Lhca1 is located so you can communicate this to your lab mate who is performing a mutational analysis. What type of thinking is required? • You are being asked to take what you know about photosystems and light-harvesting complexes and use, or apply, your knowledge to answer your labmate’s question. Gather Content What do you already know about how light-harvesting occurs in photosystems during photosynthesis? What other information is related to the question? • Photosynthesis is comprised of the light reactions, which produce ATP, NADPH, and oxygen and the Calvin cycle, which uses products from the light reactions to construct organic molecules from inorganic carbon dioxide. The light reactions capture and concentrate solar energy chlorophyll molecules clustered into light-harvesting complexes like Lhca1. Can you recall where the light reactions and the Calvin cycle take place within chloroplasts? Choose Answer Do you have all the information needed to provide an explanation? • Now it should be clearwhere particular photosynthetic processes take place within chloroplasts. During the light reactions, chlorophyll molecules arranged in a cluster, or light-harvesting complex, absorb photons from the sun and elevate the potential energy of an electron stripped from a water molecule. The light reactions take place in the Version 1
38
thylakoid membrane, which is the location of the photosystems and therefore complex Lhca1. The other answers can be eliminated because neither the outer nor the inner chloroplast membranes contain lightharvesting molecules, and the stroma is where the Calvin cycle takes place, not the light reactions. Reflect on Process Did your problem-solving process lead you to the correct answer? If not, where did the process break down or lead you astray? How can you revise your approach to produce a more desirable result? • Answering this question correctly depended on your ability to apply what you know about the light reactions, photosystems, and lightharvesting clusters of pigment molecules like Lhca1 to provide an explanation to your labmate. • If you got a correct answer, nice job! If you got an incorrect answer, did you recall that the light reactions in photosynthesis rely on photosystems? Did you remember that photosystems contain reaction centers? Did you remember that the light reactions occur in the thylakoid membrane, and therefore the light-harvesting complex Lhca1 must be located there as well? 35) C
Version 1
39
Clarify Question What is the key concept addressed by the question? • The question asks you to decipher where the highest concentration of NADPH would be located if the Calvin cycle were to be temporarily restricted. What type of thinking is required? • You are being asked to break down, or analyze, photosynthetic processes and where they occur to determine where the highest concentration of NADPH would be located following restriction of the Calvin cycle. Gather Content What do you already know about NADPH, where it is produced, and where it is used? What other information is related to the question? • Recall that photosynthesis is comprised of the light reactions, which produces ATP, NADPH, and oxygen and the Calvin cycle, which produces organic molecules from inorganic carbon dioxide. The first phase of photosynthesis includes the light reactions, where energy from the sun is captured using molecules like chlorophyll to remove electrons from water, excite them to a higher energy level, and to use redox reactions in order to produce ATP and NADPH. • The second phase of photosynthesis includes the Calvin cycle, which uses the ATP and NADPH products of the light reactions to fix inorganic carbon into an organic form within chloroplasts. If the Calvin cycle were to be restricted, what would happen to you NADPH concentration as a result? Choose Answer Do you have all necessary information to determine what type of Version 1
40
reaction is taking place? • The light reactions of photosynthesis, which occurs in the thylakoid membrane, produce ATP and NADPH. While both of these molecules are important to the Calvin cycle, if the Calvin cycle were restricted they would still continue to be produced because there is no restriction in light. • Since NADPH is required to help drive the Calvin cycle, and this occurs in the stroma, a restriction of the Calvin cycle would allow NADPH to accumulate, leading to a high concentration. This leads one to conclude the stroma is where the highest concentration of NADPH would be located. Reflect on Process Did your problem-solving process lead you to the correct answer? If not, where did the process break down or lead you astray? How can you revise your approach to produce a more desirable result? • Answering this question correctly depended on your ability to apply what you know about chloroplast anatomy and NADPH production to determine where the highest concentration of NADPH would be located if the Calvin cycle were to be temporarily restricted. • If you got a correct answer, excellent! If you got an incorrect answer, did you recall that the light reactions produce the NADPH used by the Calvin cycle? Did you recall that the light reactions occur in the thylakoid membrane and involved in the thylakoid space? Were you able to infer restriction of the Calvin cycle would allow NADPH to accumulate in the stroma where it is normally used? 36) D 37) D
Version 1
41
Clarify Question What is the key concept addressed by the question? • The question asks you to determine what photosystems I and II have in common. What type of thinking is required? • You are being asked to break down, or analyze, each photosystem to compare their feature and determine what they have in common. Gather Content What do you already know about the role that photosystems I and II in photosynthesis? What other information is related to the question? • Photosynthesis is comprised of the light reactions, which produces ATP, NADPH, and oxygen and the Calvin cycle, which produces organic molecules from inorganic carbon dioxide. The first phase of photosynthesis includes the light reactions, where energy from the sun is captured using concentrated chlorophyll clusters organized in photosystems to remove electrons from water, excite them to a higher energy level, and to use redox reactions in order to eventually produce ATP and NADPH. • Photosystems I and II are key to the function of the light reactions. Each photosystem is comprised of multiple chlorophyll molecules that have specialized functions that serve to focus and amplify high-energy electrons to bring them to a reaction center. Comparing these photosystem features, how do they differ, and what do they share in common? Choose Answer Do you have all the information needed to make a comparison? • First let’s consider what makes up a photosystem. Chlorophyll Version 1
42
molecules arranged in a cluster absorb photons from the sun and elevate the potential energy of an electron stripped from a water molecule. During this process the high-energy electron is passed to the photosystem reaction center and eventually on to an electron acceptor that begins the downstream process of electron transport, establishment of a proton gradient, and ATP production. So how do photosystem I and II compare? • Let’s see if we can eliminate some possible answers. First, the light reactions, the processes where photosystems operate, happen in the thylakoid membrane and not the stroma. Second, the splitting of water occurs early on in the light reactions, and photosystem II is the complex that serve this function. If you recall, oxygen is a byproduct of water splitting, so anything having to do with the splitting of water or oxygen production can’t be associated with photosystem I, only photosystem II. So these are differences, not similarities. We also know that only photosystem II operates the proton pump needed to generate ATP. Yet again, another difference between the two photosystems. • That leaves the only possible commonality between the two photosystems is a reaction center composed of chlorophyll a. Reflect on Process Did your problem-solving process lead you to the correct answer? If not, where did the process break down or lead you astray? How can you revise your approach to produce a more desirable result? • Answering this question correctly depended on your ability to analyze photosystems I and II to determine what they have in common. • If you got a correct answer, well done! If you got an incorrect answer, did you recall that the light reactions in photosynthesis rely on photosystems? Did you remember that photosystems contain reaction centers? Were you able to eliminate functions that are specific to one Version 1
43
photosystem or the other and use that to compare functions they have in common? 38) D 39) D 40) C 41) B 42) A 43) B
Version 1
44
Clarify Question What is the key concept addressed by the question? • The question asks you to consider what would result in a mutant tomato plant which has ATP synthase inserted into the thylakoid membrane in a reverse orientation. What type of thinking is required? • You are being asked to consider results of this defect andjudge, or evaluate, the most likely outcome. Gather Content What do you already know about ATP synthase and thylakoid membranes? What other information is related to the question? • The light reactions are the first phase of photosynthesis, where energy from the sun is used to split water, excite electrons to a higher energy level, and use redox reactions and electron transport in order to establish a proton gradient across the thylakoid membrane. • The thylakoid membrane serves to establish a proton gradient that is used by ATP synthase to produce ATP. The orientation of ATP synthase is crucial to its ability to harness the gradient. In a normally functioning tomato plant, where is the proton concentration highest and lowest across the thylakoid membrane? Choose Answer Do you have all the information needed to solve the problem? • Recall that energy harnessed from redox passage of electrons during the light reactions is used to establish a proton gradient across the thylakoid membrane. The movement of protons down their concentration gradient through ATP synthase provides the ATP needed for the Calvin cycle, which takes place in the stroma. Version 1
45
• If ATP synthase were inserted backwards into the membrane, that would mean that the portion of the enzyme through which protons enter to flow down their concentration gradient would be facing towards the stroma. As a result, the buildup of protons in the thylakoid space, in response to photosystem II, could not enter the ATP synthase enzyme and be used to generate ATP. This proton gradient would still be generated, as photosystem II and its downstream electron transport chain are still intact. • The permeability of the thylakoid membrane is not likely to be changed by the insertion of the ATP synthase in the wrong orientation. While ATP synthase can be used to pump protons across a membrane, as an active transport process, in the new orientation, ATP synthase would only be able to pump protons into the thylakoid space. Reflect on Process Did your problem-solving process lead you to the correct answer? If not, where did the process break down or lead you astray? How can you revise your approach to produce a more desirable result? • Answering this question correctly depended on your ability to evaluate possible outcomes for a plant that has ATP synthase inserted backwards in the thylakoid membrane. • If you got a correct answer, nice job! If you got an incorrect answer, did you recall that electron transport during the light reactions produces a proton gradient? Did you remember that ATP synthase produces ATP based on the proton gradient? Were you able to determine that a mutation that reorients ATP synthase backwards would not be able to harness the proton gradient to produce ATP? 44) D
Version 1
46
Clarify Question What is the key concept addressed by the question? • The question asks you to determine what happens to the thylakoid space due to electron transport. What type of thinking is required? • You are being asked to use, or apply, what you know about the light reactions and electron transport to determine what happens to the thylakoid space. Gather Content What do you already know about thylakoid membranes and proton gradients? What other information is related to the question? • First, let’s orient you to chloroplast membrane structure and function. The innermost membrane in the chloroplast is the thylakoid membrane, and it is important because it contains the proteins used to produce both ATP and NADPH. The thylakoid membrane maintains an impermeable barrier to protons, the positively charged subatomic particles used to generate the proton gradient needed to produce ATP. • Remember that a biological outcome of photosynthesis is to produce carbohydrate. In order to do that, plants and similarly photosynthesizing organisms use the light reactions, where energy from the sun is captured using chlorophyll molecules localized in photosystems I and II to strip electrons from water, excite them to a higher energy level, and use redox reactions in order to produce ATP and NADPH via proton gradients. The reason why thylakoid membranes are important is because this is where the protein complexes used to capture sunlight and produce ATP and NADPH are located, and it is the barrier that establishes and maintains the proton gradient. • Remember also that the ATP and NADPH produced from the light Version 1
47
reactions are used by the Calvin cycle, located in the chloroplast stroma. Given this information, what happens to the thylakoid space? Choose Answer Do you have the information needed to make a determination? • Under normal conditions, if both photosystems and their electron transport chains are working properly, a plant will harness light energy, split water and excite electrons to higher potential energy, and produce ATP and NADPH, based partly on the integrity of thylakoid membranes. The light reactions rely on the integrity of the thylakoid membrane, which establishes and maintains separation of protons, to allow an electrochemical gradient to be formed. This gradient is crucial to production of ATP in particular, without which the Calvin cycle that produces sugar cannot function. • Electrochemical gradient formed across the thylakoid membrane represents a difference in proton concentration. Another way to look at changes in proton concentration is pH. Thus, any difference in proton concentration across a membrane also represents a pH change. In this case, since there are more protons on the lumen side of the thylakoid membrane than the stroma side, the thylakoid space is in fact more acidic than the stroma. Reflect on Process Did your problem-solving process lead you to the correct answer? If not, where did the process break down or lead you astray? How can you revise your approach to produce a more desirable result? • Answering this question correctly depended on your ability to apply what you know about the light reactions and electron transport to determine what happens to the thylakoid space. Version 1
48
• If you got a correct answer, nice job! If you got an incorrect answer, did you recall where the light reactions and the Calvin cycle physically take place within the chloroplast? Did you remember that the light reactions and electron transport product ATP via a proton gradient across the thylakoid membrane? Were you able to correlate the difference in proton concentration across the thylakoid membrane with a pH difference as well? 45) C
Version 1
49
Clarify Question ● What is the key concept addressed by the question? ● The question asks about the function of a proton gradient in photosynthesis. ● What type of thinking is required? ● You are being asked to apply your knowledge of the function of a proton gradient in photosynthesis. Gather Content ● What do you know about the function of a proton gradient in photosynthesis? What other information is related to the question? ● Photosynthesis uses carbon dioxide and water as substrates and produces oxygen and glucose as products. The reaction is driven by energy provided by light, exciting electrons that then release energy as they pass through the electron transport chain. The proton gradient is then used to produce ATP. This ATP is then used in the Calvin cycle to convert carbon dioxide into glucose. Choose Answer ● Given what you now know, what information is most likely to produce the correct answer? ● The proton gradient is used to drive the production of ATP through chemiosmosis when the protons pass through ATP synthase. Reflect on Process ● Did your problem-solving process lead you to the correct answer? If not, where did the process break down or lead you astray? How can you revise your approach to produce a more desirable result? ● This question asked you to apply your understanding of the function of a proton gradient in photosynthesis. If you got the correct answer, great job! If you got an incorrect answer, where did the process break down? Version 1
50
Did you recall that protons move down their concentration gradient and as they release energy it is used to produce ATP? Did you understand that this is an example of chemiosmosis?
46) D 47) C
Version 1
51
Clarify Question What is the key concept addressed by the question? • The question asks you to determine which photosynthetic process would be affected by a rubisco enzyme loss-of-function mutation. What type of thinking is required? • You are being asked to dissect, or analyze, the photosynthetic process most likely to be affected by a rubisco enzyme mutation that destroys its function. Gather Content What do you already know about photosynthetic processes? What other information is related to the question? • The first phase of photosynthesis includes the light reactions, where energy from the sun is captured using chlorophyll molecules localized in Photosystems I and II to remove electrons from water, excite them to a higher energy level, and to use redox reactions in order to produce ATP and NADPH. • Remember that the light reactions, which include photosystems I and II, absorb light, make oxygen by splitting water and produce ATP and NADPH as major products. Electrons focused and amplified by photosystems I and II undergo a series of oxidation-reduction reactions as part of their electron transport chains. The Calvin cycle then takes products from the light reactions and uses it to make carbohydrate. During what photosynthetic process is rubisco active? Choose Answer Do you have the necessary information to dissect the problem? • The light reactions of photosynthesis capture photons from sunlight using chlorophyll embedded in photosystems I and II and use that Version 1
52
energy to establish a proton gradient, produce ATP, and reduce NADP+ to NADPH. These products are then used to fix inorganic carbon into organic form during the Calvin cycle. • The key enzyme involved in carbon fixation is rubisco. Since carbon fixation occurs during the Calvin cycle, no answers that involve products from the light reactions are plausible. That means making ATP, harvesting photons, making oxygen, and producing NADPH are infeasible answers. Therefore, the likely outcome of a rubisco enzyme mutation would be an inability to fix carbon and produce carbohydrate. Reflect on Process Did your problem-solving process lead you to the correct answer? If not, where did the process break down or lead you astray? How can you revise your approach to produce a more desirable result? • Answering this question correctly depended on your ability to analyze the possible photosynthetic effect of a mutation in the rubisco enzyme. • If you got a correct answer, well done! If you got an incorrect answer, did you recall that carbon fixation occurs during the Calvin cycle? Did you remember that the key enzyme that fixes carbon is rubisco? Were you able to infer that mutations in rubisco would affect carbon fixation? 48) C 49) D 50) D 51) B 52) B 53) D 54) B
Version 1
53
Clarify Question What is the key concept addressed by the question? • The question asks you to determine how many molecules of glyceraldehyde 3-phosphate would be produced from 24 turns of the Calvin cycle. What type of thinking is required? • You are being asked to calculate, or analyze, the number of glyceraldehyde 3-phosphate produced when the Calvin cycle turns 24 times. Gather Content What do you already know about the Calvin cycle and photosynthesis? What other information is related to the question? • Recall that the biological function of the Calvin cycle is to fix carbon from the atmosphere into inorganic molecule. The Calvin cycle uses ATP and NADPH produced during the light reactions to fix inorganic carbon into inorganic form within the stroma of chloroplasts. • Remember that the Calvin cycle is an iterative process, meaning that carbon dioxide is accepted, molecules are transformed, excess carbohydrate is harvested, and new acceptor molecules are regenerated. Given what you know about the Calvin cycle, how many turns are required in order to produce one glyceraldehyde-3-phosphate molecule? Choose Answer Do you have all the information needed to determine the likely outcome? • Remember that during the Calvin cycle, RuBP binds to a single carbon dioxide molecule and produces a six-carbon molecule that is then converted into two three-carbon molecules of 3-phosphoglycerate. Each Version 1
54
of these molecules is subsequently converted into two three-carbon glyceraldehyde 3-phosphate molecules. • The ability of the Calvin cycle to keep operating is dependent on the regeneration of RuBP. Excess glyceraldehyde 3-phosphate molecules are harvested and use to make glucose, whereas other glyceraldehyde 3phosphate molecules are eventually converted back into RuBP. The trick is knowing how many turns of the Calvin cycle are needed, given the need to regenerate RuBP. • Essentially, one carbon dioxide molecule is incorporated into an organic molecule for every turn of the Calvin cycle. Glyceraldehyde-3phosphate is a 3-carbon molecule, so that means the Calvin cycle will have to turn three times. Since the problem indicates there are 24 turns total, that means 24 turns divided by 3 carbons in each turn of the Calvin cycle for each glyceraldehyde-3-phosphate molecule would equal 8 glyceraldehyde-3-phosphate molecules would be produced. Reflect on Process Did your problem-solving process lead you to the correct answer? If not, where did the process break down or lead you astray? How can you revise your approach to produce a more desirable result? • Answering this question correctly depended on your ability to analyze the Calvin cycle to determine how many glyceraldehyde 3-phosphate molecules would be produced from 24 turns. • If you got the correct answer, nice job! If you got an incorrect answer, did you recall that the Calvin cycle essentially requires a turn for each carbon that is fixed? Did you remember that eachglyceraldehyde-3phosphate molecule contained three carbons? Were you able to calculate the number of glyceraldehyde-3-phosphate molecules given 24 turns of the Calvin cycle? 55) E Version 1
55
Glucose is C 6H 12O 6. Since there are 6 carbons in this molecule, 6 CO 2 molecules are needed to make one molecule of glucose. 56) B
Version 1
56
Clarify Question ● What is the key concept addressed by the question? ● The question asks about the activity of the Calvin cycle at night. ● What type of thinking is required? ● You are being asked to apply your knowledge of the activity of the Calvin cycle at night. Gather Content ● What do you know about the activity of the Calvin cycle at night? What other information is related to the question? ● Photosynthesis uses carbon dioxide and water as substrates and produces oxygen and glucose as products. The reaction is driven by energy provided by light, exciting electrons that then release energy as they pass through the electron transport chain. The proton gradient is then used to produce ATP. This ATP is then used in the Calvin cycle to convert carbon dioxide into glucose. Choose Answer ● Given what you now know, what information is most likely to produce the correct answer? ● The Calvin cycle includes the lightindependent reactions that use ATP, NADPH, and carbon dioxide to produce glucose. Even if no light is available, if these substrates are present the reaction will occur. Reflect on Process ● Did your problem-solving process lead you to the correct answer? If not, where did the process break down or lead you astray? How can you revise your approach to produce a more desirable result? ● This question asked you to apply your understanding of the activity of the Calvin cycle at night. If you got the correct answer, great job! If you got an incorrect answer, where did the process break down? Did you recall Version 1
57
that the Calvin cycle is a light independent reaction? Did you understand that as long as the substrates for the Calvin cycle are present the reaction will occur?
57) B
Version 1
58
Clarify Question What is the key concept addressed by the question? • The question asks you to determine why ATP generated during cellular respiration is insufficient to drive the Calvin cycle in plants even when there is plenty of NADPH and CO2. What type of thinking is required? • You are being asked to break down possible scenarios, or analyze, why there isn’t enough ATP produced from cellular respiration to drive the Calvin cycle even when NADPH and carbon dioxide are not limited. Gather Content What do you already know about the Calvin cycle, photosynthesis, and cellular respiration in plants? What other information is related to the question? • Recall that photosynthesis is comprised of the light reactions and the Calvin cycle. ATP and NADPH, the main products of the light reactions, provide the energy and reducing power needed during the Calvin cycle to fix carbon dioxide from the atmosphere into an organic form. • Have you ever wondered how plants continue to operate during the night when the light reactions are not producing ATP and NADPH? Remember that plants also contain mitochondria in addition to chloroplasts; that means they are able to metabolize the sugars they produce during photosynthesis to produce ATP needed to operate a wide variety of cellular processes. • How efficient do you think it would be for a plant to operate the Calvin cycle using only ATP from cellular respiration? Choose Answer Do you have all the information need to determine why ATP from Version 1
59
cellular respiration needs to be supplemented with ATP from the light reactions? • Let’s approach answering this question logically. Plants possess mitochondria, so the answer that says plants don’t undergo cellular respiration is clearly incorrect. And the fact that mitochondria exist in the same cell at the same time as chloroplasts indicates that answer is also not an option. • The Calvin cycle and the process of fixing carbon into an organic form from atmospheric carbon is an inherently energy-intensive process; it simply takes a great deal of energy to overcome entropy and create the more ordered organic molecules in the Calvin cycle. And while cellular respiration does produce ATP, it doesn’t produce enough to overcome the inherent energy inefficiency, nor does cellular respiration produce the NADPH needed during to fix carbon. • It’s not that cellular respiration can’t produce enough ATP, it’s that there wouldn’t be enough ATP production to cover the plant cell’s energy costs and still have enough left over to make excess carbohydrate. Thus, breaking down the sugar needed to provide the energy needed to make more sugar would be a zero sum proposition. Reflect on Process Did your problem-solving process lead you to the correct answer? If not, where did the process break down or lead you astray? How can you revise your approach to produce a more desirable result? • Answering this question correctly depended on your ability to analyze why ATP produced during plant cellular respiration is insufficient to drive the Calvin cycle even when NADPH and carbon dioxide levels are sufficient. • If you got the correct answer, well done! If you got an incorrect answer, did you recall that plant cells contain mitochondria and Version 1
60
chloroplasts? Were you able to determine why ATP from the light reactions and cellular respiration are needed to meet plant cell energy costs? Were you able to infer how inherently inefficient making sugar to break it down for energy to make sugar again would be? 58) D 59) E
Version 1
61
Clarify Question What is the key concept addressed by the question? • The question asks you to decipher how many molecules of carbon dioxide would be needed to construct a cellulose fiber comprised of 20 glucose molecules. What type of thinking is required? • You are being asked to dissect, or analyze,the scenario to determine how many molecules of carbon dioxideare needed. Gather Content What do you already know about the Calvin cycle and carbon dioxide? What other information is related to the question? • Recall that the biological function of the Calvin cycle is to fix carbon from the atmosphere into an organic molecule. The Calvin cycle uses ATP and NADPH produced during the light reactions to fix inorganic carbon into an organic form within the stroma of chloroplasts. One turn of the Calvin cycle is required for each carbon dioxide molecule that is fixed by the enzyme rubisco. • Remember that the Calvin cycle is an iterative process, meaning that carbon dioxide is accepted, molecules are transformed, and glucose is produced. How many carbons make up one glucose molecule, and how many glucose would be needed for the cellulose molecule? Choose Answer Do you have all the information needed to determine the likely outcome? • If you understand how many carbons make up glucose, the rest should be fairly straightforward math. A synthetic cellulose fiber that contains Version 1
62
20 glucose molecules would contain six times that number in carbon atoms because glucose has six carbons in its structure, meaning 120 molecules of carbon dioxide would be required to make the fiber. Reflect on Process Did your problem-solving process lead you to the correct answer? If not, where did the process break down or lead you astray? How can you revise your approach to produce a more desirable result? • Answering this question correctly depended on your ability to analyze the Calvin cycle to determine how many carbon dioxidemolecules would be needed to make a synthetic cellulose fiber that contained 20 glucose molecules. • If you got the correct answer, well done! If you got an incorrect answer, didyou remember how many carbon atoms are in a glucose molecule? 60) A
Version 1
63
Clarify Question What is the key concept addressed by the question? • The question asks you to determine which experimental modification would most effectively help determine reactions and intermediates in the Calvin cycle. What type of thinking is required? • You are being asked to weigh and judge, or evaluate, the different experimental modifications to determine which approach would be most effective for investigating the Calvin cycle. Gather Content How would you begin to narrow down the experimental options? What key molecules could you focus on considering the investigation is about the Calvin cycle? • Recall that photosynthesis is comprised of the light reactions, which produces ATP, NADPH, and oxygen and the Calvin cycle, which produces organic molecules from inorganic carbon dioxide. • The first phase of photosynthesis includes the light reactions, where energy from the sun is captured using molecules like chlorophyll to remove electrons from water, excite them to a higher energy level, and to use redox reactions in order to produce ATP and NADPH. The second phase of photosynthesis includes the Calvin cycle, which uses the ATP and NADPH products of the light reactions to fix inorganic carbon into an organic form within chloroplasts. • Considering what you know about the Calvin cycle, which experimental options make the most sense? Choose Answer Do you have all the information needed to determine which Version 1
64
experimental approach would be most effective? • Since this question focuses on the Calvin cycle and not the light reactions, exposing the cells to alternating periods of light and darkness is unlikely to affect the Calvin cycle directly. Likewise, molecules associated with the light reactions like water and oxygen are probably less useful as well. That leaves using a carbon dioxide-based approach as the most effective for determining reactions and reaction intermediates in the Calvin cycle. Reflect on Process Did your problem-solving process lead you to the correct answer? If not, where did the process break down or lead you astray? How can you revise your approach to produce a more desirable result? • Answering this question correctly depended on your ability to apply what you know about key molecules in both the light reactions and the Calvin cycle to determine which experimental modification would most effectively determine reactions and intermediates in the Calvin cycle. • If you got a correct answer, well done! If you got an incorrect answer, were you able to identify the key molecules in each answer? Were you able to use that information to systematically eliminate options that were associated with the light reactions and not the Calvin cycle? Were you able to conclude that the carbon dioxide based experimental modification would be most suitable for investigating the Calvin cycle? 61) A 62) B 63) C 64) C
Version 1
65
Clarify Question What is the key concept addressed by the question? • The question asks you to decipher what would happen to glucose production if a planthad its stomata closed more often. What type of thinking is required? • You are being asked to dissect, or analyze, how a plant’s ability to produce glucose would be affected by closed stomata. Gather Content What do you already know about how stomata and gas exchange in plants? What other information is related to the question? • Recall that plants import carbon dioxide and export oxygen due to concentration differences on both the inside and outside the plant. For example, oxygen is in its highest concentration inside the plant, and because the concentration of oxygen is lower in the atmosphere, diffuses from inside to outside the plant. Conversely, carbon dioxide is in higher concentration outside the plant and thus diffuses into the plant where the concentration of carbon dioxide is lower. • The anatomical gateways a plant uses to regulate gas exchange are the stomata that are located primarily in leaves. Analogous to a human nose or mouth, stomata can open or close, depending on environmental suitability and photosynthetic need of the plant. Considering the importance of carbon dioxide in photosynthesis, what do you think would happen to a plant whose stomata are usually closed? Choose Answer Do you have all the information needed to determine what would happen to the plant with closed stomata? • Remember that photosynthesis is comprised of the light reactions and Version 1
66
the Calvin cycle. The Calvin cycle in particular requires carbon dioxide in order to fix that carbon and produce carbohydrates. • If a plant's stomata were usually closed, it would not be able to take in carbon dioxide from the environment. As a result, the ability of that plant to make glucose would be significantly decreased. Reflect on Process Did your problem-solving process lead you to the correct answer? If not, where did the process break down or lead you astray? How can you revise your approach to produce a more desirable result? • Answering this question correctly depended on your ability to analyze what would happen to a plant’s capacity to produce glucose if its stomata were closed. • If you got the correct answer, well done! If you got an incorrect answer, did you remember that oxygen and carbon dioxide gases move from high to low concentration? Did you remember that plant stomata regulate gas exchange? Were you able to infer that preventing carbon dioxide from entering the plant would restrict the Calvin cycle and therefore sugar production? 65) A
Version 1
67
Clarify Question What is the key concept addressed by the question? • The question asks you to determine the most favorable conditions for glucose production given that rubisco can either fix carbon or oxidize RuBP. What type of thinking is required? • You are being asked to weigh and judge, or evaluate, the different environmental conditions to determine which would most favorably produce glucose. Gather Content What do you already know about RuBP, rubisco, and the Calvin cycle? What other information is related to the question? • Recall that plants are evolutionarily adapted to particular climates and environments and respond differently depending on whether the environment is hot, dry, or humid. Part of a plant’s ability to physiologically adapt to an environment has to do with how it metabolizes carbon, and what happens to the machinery that fixes carbon when conditions are not optimal. • Remember that the molecule that accepts atmospheric carbon during the Calvin cycle is ribulose-1,5-bisphosphate, or RuBP. Remember also that the key enzyme involved in carbon fixation during the Calvin cycle is RuBP carboxylase/oxygenase, or rubisco. This enzyme is key in taking inorganic carbon dioxide from the air and fixing it in an organic molecule, although under less ideal conditions can oxidize RuBP. Considering the different environmental conditions indicated, how might rubisco’s function change? Choose Answer Version 1
68
Do you have all the information needed to determine the most favorable condition for glucose production? • Let’s take each option and consider it individually to determine the most correct answer. If environmental conditions are hot or dry, the plant will close its stomata in order to prevent water loss, thereby restricting carbon dioxide diffusion into the leaves. Under those conditions, if light were still available, the light reactions would still occur and oxygen levels would increase. • Anything that increases the relative level of oxygen will not be favorable for glucose production, as rubisco will favor oxidizing RuBP instead of fixing carbon. As a result, anything that leads to the closing of stomata, or changes the overall level of oxygen will be unfavorable. • Conversely, moderate temperatures allow stomata to remain open, and thus allow carbon dioxide to enter the leaves. This is why moderate temperatures are the most favorable condition for glucose production. Reflect on Process Did your problem-solving process lead you to the correct answer? If not, where did the process break down or lead you astray? How can you revise your approach to produce a more desirable result? • Answering this question correctly depended on your ability to evaluate several environmental conditions for which would be most favorable for glucose production, considering that rubisco can fix carbon or oxidize RuBP. • If you got the correct answer, excellent! If you got an incorrect answer, were you able to determine what would happen to stomata and rubisco function under high temperatures and dry climate? Were you able to figure out that oxygen levels would increase if stomata were closed during the day? Were you able to infer moderate temperatures would be preferable to any of the other conditions? Version 1
69
66) D 67) D All of these plants can do the Calvin cycle. The difference between them is how they fix carbon. 68) A
Version 1
70
Clarify Question What is the key concept addressed by the question? • The question asks you to determine the role of ABA based on its role in photorespiration during photosynthesis. What type of thinking is required? • You are being asked to dissect, or analyze, how ABA might help to regulate photo respiration in plants. Gather Content What do you already know about photo respiration in photosynthesis? What other information is related to the question? • Recall that the key enzyme during Calvin cycle carbon fixation is rubisco. This enzyme is essential in taking inorganic carbon dioxide from the air and fixing it in an organic molecule, and under those conditions works as a carboxylase enzyme. However, rubisco also operates as an oxygenase enzyme. Rubisco carboxylase is associated with normal Calvin cycle production of G3P and regeneration of RuBP, whereas rubisco oxygenase activity prevents RuBP from proceeding in the Calvin cycle. • Given the potentially adverse effects of photo respiration, how can ABA be used to regulate this process? Choose Answer Do you have all the information needed to determine what processes ABA regulates in plants? • Although the answers seem a little complicated, the correct answer is fairly straightforward. If you want to minimize rubisco oxygenase activity and maximize carboxylase activity, what environmental factors would you change in order to do so? Photo respiration is increased when Version 1
71
carbon dioxide is low or oxygen is high. By manipulating when stomata open and close, a plant can regulate the relative concentration of both gases, and in this way promote rubisco carboxylase activity. The molecule a plant uses to help regulate stomata opening and closing is ABA. Reflect on Process Did your problem-solving process lead you to the correct answer? If not, where did the process break down or lead you astray? How can you revise your approach to produce a more desirable result? • Answering this question correctly depended on your ability to analyze how ABA might help regulate photorespiration during photosynthesis. • If you got the correct answer, good job! If you got an incorrect answer, did you remember how photo respiration works? Did you remember that rubisco is both a carboxylase and an oxygenase that is sensitive to levels of carbon dioxide and oxygen? Were you able to correlate ABA with the ability of the plant to open and close stomata and thereby regulate the relative amounts of oxygen and carbon dioxide? 69) A
Version 1
72
Clarify Question What is the key concept addressed by the question? • The question asks you to determine which organic molecule would be the first to be labeled with 14C in a C4 plant. What type of thinking is required? • You are being asked to dissect, or analyze, which molecule in a C4 plant would be labeled with 14C first. Gather Content What do you already know about how carbon fixation in C3, C4, and CAM plants? How does photosynthesis differ in these plants? What other information is related to the question? • Recall that plants are evolutionarily adapted to particular climates and environments. Part of a plant’s ability to adapt to an environment has to do with how and where it metabolizes carbon, and what happens to the machinery that fixes carbon when conditions are not optimal. It also has to do with the plant’s ability to balance intake of carbon dioxide with loss of water in hot, dry environments. C4 and CAM plants essentially use two strategies to address environmental stressors; separation of light reactions and carbon fixation by space and by time. Choose Answer Do you have all necessary information to determine which organic molecule would be first to be labeled with 14C in a C4 plant? • C4 and CAM plants use different strategies to deal with hot temperatures and water loss issue during the day. C4 plants acquire carbon dioxide and transport it in a different form to a separate location before making it available for the Calvin cycle. CAM plants acquire carbon dioxide at night when it is cooler and water loss is less likely to Version 1
73
happen and store it until daytime when photosynthesis can take place. • Since we are talking about a C4 plant here, let’s focus on how that plant takes in carbon dioxide in its adverse environment. Recall that a C4 plant first locks carbon dioxide onto a three-carbon molecule to form a four-carbon oxaloacetate molecule. Thus, a carbon dioxide molecule labeled with 14C would first be incorporated into oxaloacetate before any of the other molecules. Reflect on Process Did your problem-solving process lead you to the correct answer? If not, where did the process break down or lead you astray? How can you revise your approach to produce a more desirable result? • Answering this question correctly depended on your ability to analyze which molecule would be the first to be labeled with 14C in a C4 plant. • If you got the correct answer, well done! If you got an incorrect answer, were you able to identify which molecules receive carbon dioxide in C4 plants? Did you remember that oxaloacetate is a fourcarbon molecule? Were you able to recognize that 14C from the carbon dioxide must end up in oxaloacetate because it is the first molecule formed during carbon fixation in C4 plants? 70) B Both of these types of plants have adaptations that allow them to minimize photorespiration. As a result, they likely grow in hot, arid environments where stomata closeto minimize water loss. 71) B
Version 1
74
Clarify Question What is the key concept addressed by the question? • The question asks you to determine how many more chloroplast membranes would be required for a protein to be transported into the thylakoid lumen once it is already past the outer chloroplast membrane? What type of thinking is required? • You are being asked to use what you already know about chloroplasts and membranes, and apply that to figure out how many more membranes a protein inside the outer chloroplast membrane would need to pass through to get to the inside of the thylakoid lumen. Gather Content What do you already know about chloroplasts and membranes? What other information is related to the question? • Let’s first re-orient you to chloroplast structure and function. Chloroplasts are the cellular organelle involved in photosynthesis. They are enclosed by multiple membranes, each of which serves to compartmentalize the interior of the chloroplast. • The innermost membrane, called the thylakoid membrane, contains a dense concentration of proteins and chlorophyll molecules that capture sunlight and use it to drive the process of photosynthesis. For example, the thylakoid membrane constituents produce both ATP and NADPH. The thylakoid membrane also maintains an impermeable barrier to protons, the positively charged subatomic particles used to generate the proton gradient needed to produce ATP. What other membranes are associated with the chloroplast, and how many would a protein that is already inside the outer chloroplast membrane need to pass through to get to the thylakoid lumen?
Version 1
75
Choose Answer Do you have all necessary information to determine what was produced during the plant experiment? • Once you understand chloroplast anatomy it’s fairly straightforward to figure out the answer. Chloroplasts have an outer membrane, and inner membrane, and a thylakoid membrane. In this particular question, the protein has already passed through the outer chloroplast membrane and is in the space just outside of the inner mitochondrial membrane. • So, in order to be transported into the thylakoid lumen, the protein must pass through the inner chloroplast membrane and the thylakoid membrane, bringing the total number of membranes that need to be passed through to two. Reflect on Process Did your problem-solving process lead you to the correct answer? If not, where did the process break down or lead you astray? How can you revise your approach to produce a more desirable result? • Answering this question correctly depended on your ability to apply what you know about chloroplast anatomy and the number of membranes included in a chloroplast to determine how many more membranes a protein would need to cross in order to get into the thylakoid lumen. • If you got the correct answer, well done! If you got an incorrect answer, were you able to determine the total number of membranes in the chloroplast? Were you able to spatially position the protein in the space between the outer and inner chloroplast membranes? Were you able to count the remaining number of membranes needed to get to the thylakoid lumen? 72) C Version 1
76
Clarify Question What is the key concept addressed by the question? • The question asks you to determine what would happen to a plant that was treated with a chemical that blocks electron flow between photosystems II and I. What type of thinking is required? • You are being asked to use what you know about electron transport and proton gradients and use, or apply, that knowledge to predict what would happen to a plant treated with a chemical that blocks electron transport between photosystems II and I. Gather Content What do you already know about photosystems II and I and electron transport? What other information is related to the question? • The first phase of photosynthesis includes the light reactions, where energy from the sun is captured using chlorophyll molecules localized in Photosystems II and I to remove electrons from water, excite them to a higher energy level, and to use redox reactions and electron transport in order to eventually produce ATP and NADPH. • Remember that the light reactions, which include photosystems II and I, absorb light, make oxygen by splitting water and produce ATP and NADPH as major products. Electrons focused and amplified by photosystems I and II undergo a series of oxidation-reduction reactions as part of their electron transport chains. What is done with the energy released during electron transport? Choose Answer Do you have the necessary information to solve the problem? • The light reactions of photosynthesis capture photons from sunlight Version 1
77
using chlorophyll embedded in photosystems II and I. Under normal conditions, if both photosystems and their electron transport chains are working properly, the plant will harness light energy, split water and excite electrons to higher potential energy, and produce ATP and NADPH, with oxygen as a byproduct. • Two electron transport chains occur during the light reactions. Energy given off during the passage of electrons is used to to move protons across the thylakoid membrane in order to generate the proton gradient used to produce ATP via ATP synthase. Therefore, if electron transport between photosystem II and I were to be disrupted by the chemical, the direct effect would be a decreased proton gradient that would lead to less production of ATP. Reflect on Process Did your problem-solving process lead you to the correct answer? If not, where did the process break down or lead you astray? How can you revise your approach to produce a more desirable result? • Answering this question correctly depended on your ability to apply what you know about electron transport and proton gradients to determine what happens when chemical treatment of a plant disrupts electron transport between photosystem II and I. • If you got a correct answer, nice job! If you got an incorrect answer, did you recall that the light reactions rely on redox reactions during electron transport? Did you remember that photosystems and electron transport work together during the light reactions to produce a proton gradient? Were you able to determine that a chemical that blocks electron transport would decrease the proton gradient and reduce ATP production? 73) B
Version 1
78
Through the process of photosynthesis, the carbons in carbon dioxide are fixed and ultimately utilized to generate glucose.
Version 1
79
CHAPTER 9 MULTIPLE CHOICE - Choose the one alternative that best completes the statement or answers the question. 1) The protein Sonic Hedgehog (SHH) plays a role in specifying the identity of digits (such as fingers and toes) in mammals. Several digits arise directly from cells that both express and respond to secreted SHH. What type of signaling would this be considered? A) direct contact B) paracrine C) endocrine D) synaptic E) autocrine
2) One cell sends second messengers to another cell through gap junctions.What type of signaling would this be considered? A) direct contact B) paracrine C) endocrine D) synaptic E) autocrine
3)
In synaptic signaling, neurotransmitters are released into a space that is referred to as a A) chemicalsynapse. B) neuronjunction. C) paracrinespace. D) gap junction. E) plasmodesmata.
4) In response to injury, cell fragments called platelets get activated to induce clotting. Activated platelets release factors that can in turn bind to specific membrane receptors on nearby cells.What type of signaling would this be considered?
Version 1
1
A) direct contact B) paracrine C) endocrine D) synaptic E) autocrine
5) A researcher is working to generate a new cancer drug. Thus far, he has identified a compound that can reduce the size of tumors in the lung. However, in order for the drug to work, the lung tumor has to be small. In addition, the tumor cannot have metastasized (spread to other areas of the body). Furthermore, he knows that the drugacts toprohibit the signaling from one tumor cell to another tumor cell. Given the above information, this new drug prohibits A) synaptic signalingbetween tumor cells. B) autocrine signaling between tumor cells. C) paracrine signaling between tumor cells. D) endocrinesignaling between tumor cells.
6) Using a specific chemical, acell biologist has blocked one type of channel-linked receptorin a lab rat's liver tissue. What was the likely mechanism of the chemical she used? A) The chemicalprohibits binding of a ligand to the receptor. B) The chemicalprohibits ions from entering the cell via active transport. C) The chemicalcauses ions to move in the opposite direction of the ion gradient. D) The chemicalprohibits second messenger signaling.
7) Leptin is a circulating hormone that is produced by fat cells and plays a role in body metabolism and obesity. It normallybinds to receptors in the brain and inhibits appetite.Studies have demonstrated that when obese mice that are leptin deficientare injected with leptin they quickly lose their excess weight. Interestingly, however, many overweight people have high levels of leptin in their bloodstream. Why do you think that the high levels of leptin in obese individuals are insufficient to curb their appetite?
Version 1
2
A) Paracrine signaling may be disrupted in obese people with high levels of leptin. B) Obese people whohave high levels of leptin may have a leptin receptor with decreasedsensitivity. C) The form of leptin released by obese people may lack protein kinase activity. D) In obese people with high levels of leptin, the leptin is a nonfunctional second messenger.
8) Vasopressin is a hormone that is primarily released when the body is low on water. Release of vasopressin causes the kidneys to conserve water by concentrating urine and decreasing urine output. However, vasopressin also has additional functions in other tissues such as the brain and blood vessels, which are not directly involved in urine production. How could it be possible for vasopressin to trigger different responses in different tissues? A) Vasopressin canbind to, and act on, different receptor subtypes, leading to different cellularresponses in different tissues. B) Vasopressin can bereleased at different times to result in different effects. C) Vasopressin bindsto the same exact receptor in different tissues, but causes a differentconformational change to the receptor in each tissue. D) The kidneys have a receptor forvasopressin, but cells in the brain and blood vessels do not have a receptorfor vasopressin.
9) Addison's disease is a disorder that results from a reduction in production of glucocorticoids and mineralocorticoids. Individuals with Addison's disease suffer from a variety of systemic symptoms including: muscle weakness, fever, issues with the gastrointestinal tract, and increased tanning. Considering this information, what type of signaling do you think that glucocorticoids and mineralocorticoids stimulate? A) Directcontact B) Paracrine C) Endocrine D) Synaptic E) Autocrine
10)
What is the function of a protein kinase?
Version 1
3
A) To removephosphate groups from proteins B) To cleavemembrane phospholipids C) To phosphorylate GDP to generate GTP D) To add phosphategroups onto proteins
11) You have identified a novel cytoplasmic protein. Through sequence analysis you are able to identify the presence of a number of serine/threonine phosphorylation motifs. Based on these data, you hypothesize that this protein may be a substrate for what kind of enzyme? A) a protein kinase B) a receptor tyrosine kinase C) a G protein-coupled receptor D) a tyrosine phosphatase E) a phospholipase
12) You have identified a molecule, molecule X,that can bind to the ATP-binding site of a specific protein kinase, and prevent ATP binding. You are conducting a kinase assay using this protein kinase and a known substrate of this kinase, and have set up the reaction using the proper conditions and cofactors to allow for optimal protein kinase activity. If you add molecule Xto the reaction, what do you predict will be the outcome? A) The substrate willbe phosphorylated normally in the presence of molecule X. B) Lessphosphorylation of the substrate will be seen in the presence of moleculeX. C) Morephosphorylation of the substrate will be seen in the presence of moleculeX.
13) Why are phosphorylation and dephosphorylation reactions commonly used to regulate signal transduction pathways?
Version 1
4
A) Phosphate groupsare efficient second messengers. B) Protein kinases andphosphatases are abundant in most cells. C) The hydrolysis ofbound GTP generates GDP, which can change the activity of proteins bound toGDP. D) The addition orremoval of a phosphate group can expose or hide potential binding sites inproteins or change protein activity.
14) You are performing an experiment to assay phosphorylation of a substrate by a protein kinase. Based on the data in the table below, what do these experimental results suggest? Substrate
Protein kinase
GTP
Phosphorylation detected
Test sample
+
+
+
yes
Control #1
-
+
+
no
Control #2
+
-
+
yes
Control #3
+
+
-
no
A) GTP is not required for phosphorylation. B) The substrate can undergo autophosphorylation. C) The substrate is not required for phosphorylation. D) The protein kinase is required for phosphorylation.
15)
These molecules can directly convert extracellular signals into intracellular signals. A) second messengers B) scaffolds C) membrane receptors D) adapter proteins E) G proteins
16) When bound to ligand, this type of receptor changes conformation to allow ions to move down their concentration gradient across the membrane.
Version 1
5
A) channel-linked receptor B) enzymatic receptor C) G protein-coupled receptor D) steroid hormone receptor
17) Growth of new blood vessels is stimulated by vascular endothelial growth factor (VEGF). This ligand binds to the VEGF receptor, which leads to the dimerization and autophosphorylation of this receptoron cytoplasmic tyrosines. What type of receptor is the VEGF receptor? A) channel-linked receptor B) enzymatic receptor C) G protein-coupledreceptor D) steroid hormone receptor
18) What type of receptor is not an enzyme, but is directly associated with an enzyme that can hydrolyze GTP? A) channel-linked receptor B) enzymatic receptor C) G protein-coupled receptor D) steroid hormone receptor
19) You are planning to perform some protein-protein interaction studies to identify a receptor for a steroid hormone you have been working on. You start by fractionating the cell. What cell fraction would you want to work with to identify the steroid hormone receptor of interest?
Version 1
6
A) the plasma membrane fraction B) the extracellular fraction C) the cytoplasmic fraction D) the mitochondrial fraction E) the ribosomal fraction
20)
What property defines most ligands for intracellular receptors? A) They are lipid-soluble. B) They arehydrophilic. C) They are moleculesthat can perform significant hydrogen bonding interactions with
water. D) They are largemolecules.
21) You are interested in developing an inhibitor for the estrogen receptor. You have identified a molecule that is small and hydrophilic. In a test tube, this inhibitor binds tightly to the estrogen receptor and inhibits the interaction of the receptor with estrogen. Do you think that this molecule will be an effective inhibitor of the estrogen receptor in cells? A) Yes, because itcan prevent estrogen binding. B) Yes, because itis small. C) No, because it ishydrophilic. D) No, because itwill need to bind to the DNA-binding domain to act as an inhibitor.
22) A mutation in the DNA-binding domain of a steroid hormone receptor is most likely to affect what aspect of receptor function? A) the conformational change of the receptor B) the binding of the hormone to the receptor C) the translocation of the receptor to the nucleus D) the cellular response to the hormone
Version 1
7
23) You are having lunch with an elderly friend when suddenly he begins to experience chest pain and tightness. He reaches into his pocket, gets out one of his nitroglycerin tablets and quickly takes it. After the episode passes, your friend asks you how the nitroglycerin helps his angina. You explain that nitroglycerin gets converted by cells to nitric oxide (NO)causing the smooth muscle cells of the blood vessel to relax and increase blood flow to the heart. He pushes you for a more detailed explanation, so you explain further that A) NO activates guanylyl cyclase, which catalyzes the synthesis of cGMP, which acts as an intracellular messenger in a pathway leading to smooth muscle relaxation. B) NO activates a protein kinase, which catalyzes the synthesis of cGMP, which acts as an intracellular messenger in a pathway leading to smooth muscle relaxation. C) NO inhibits guanylyl cyclase, which allows the build up of cGMP, which acts as an intracellular messenger in a pathway leading to smooth muscle relaxation. D) NO activates cGMP, which catalyzes the synthesis of guanylyl cyclase, which acts as an intracellular messenger in a pathway leading to smooth muscle relaxation.
24) What is the mechanism by which binding of testosterone to the testosterone receptor leads to upregulated gene transcription? A) Binding oftestosterone to the testosterone receptor leads to the increase in secondmessengers that activate a kinase cascade, ultimately leading to the activationof transcription factors that promote gene transcription. B) Binding oftestosterone to the testosterone receptor leads to the activation of a Gprotein that stimulates the activation of enzymes that promote genetranscription. C) Binding oftestosterone to the testosterone receptor leads to a conformational change thatallows the receptor to enter the nucleus where it can bind to DNA and promotegene transcription. D) Binding oftestosterone to the testosterone receptor leads to the opening of an ionchannel that leads to an increase in calcium in the cytoplasm.Theincreased calcium levels activate calmodulin, which can go on to activate otherproteins that promote gene transcription. E) Binding of testosterone to the testosterone receptor activates guanylyl cyclase, and the resulting cGMP promotes gene transcription.
25)
How are receptor tyrosine kinases and steroid hormone receptors similar?
Version 1
8
A) Both are activatedby autophosphorylation. B) Both aretransmembrane proteins with a single transmembrane domain. C) Both have aDNA-binding domain. D) Both have a sitefor binding ligand.
26) Some cancers are caused by the overexpression of receptor tyrosine kinases (RTKs). It is known that RTK-signaling pathways commonly stimulate cell division. Why would the overexpression of receptor tyrosine kinases lead to cancer development? A) RTKs are activated by dimerization, caused by ligand binding. If there are too many receptors on the cell surface, it is possible that these receptors dimerize in the absence of ligand binding, thus stimulating cell division at inappropriate times. B) If there are too many RTKs on the cell surface, this will tend to allow cells to adhere to each other. Once they adhere, RTKs from one cell can bind to RTKs from another cell, and they can activate each other leading to activation of downstream signaling pathways in both cells. C) If there are too many RTKs on the cell surface, it is less likely that inhibitors will bind to all of the available RTKs and block their ability to enter the nucleus. As a result, some of the RTKs will be able to enter the nucleus to stimulate transcription. D) If there are too many RTKs on the cell surface, this willinhibitautophosphorylation. In the absence of autophosphorylation, it is more likely that the RTK downstream signaling pathway will be active.
27)
What signaling pathway commonly makes use of a G protein to transduce the signal? A) channel-linked receptor pathways B) steroid hormone receptor pathways C) receptor tyrosine kinase pathways
Version 1
9
28) Your research project involves the characterization of a particular signal transduction pathway. Specifically, you have been studying a receptor tyrosine kinase and protein X, which initiates downstream events. You are certain that protein X is part of the signal transduction pathway, but have been unable to identify a direct protein-protein interaction between the receptor tyrosine kinase and the protein X. What should you do next in your attempt to identify a link between the receptor tyrosine kinase and protein X? A) try to identify a coactivator protein B) try to identify proteins that act downstream of protein X C) try to identify an adapter protein that can interact with both the receptor and protein X D) confirm the interaction between the receptor tyrosine kinase and its ligand
29) The protein SOS is a guanine nucleotide exchange factor for Ras. SOS helps facilitate the exchange of GDP for GTP. What would be the effect of a mutation that inhibits the interaction between SOS and Ras? A) GTP would remainbound to Ras, thereby keeping Ras constitutively active. B) GDP would remainbound to Ras, thereby preventing Ras activation. C) Ras would be morelikely to hydrolyze GTP to GDP. D) There would be noeffect.
30) Many receptor tyrosine kinase (RTK)pathways lead to the activation of Ras.To activate Ras, it is necessary to recruit a guanine nucleotide exchange factor to the plasma membrane, because Ras is a membrane-associated protein.Guanine nucleotide exchange factors, such as SOS, stimulate the exchange of GDP for GTP. However, SOS cannot bind directly to most RTKs.The protein Grb2 has a domain that can bind to phosphorylated tyrosines, and another domain that can bind to SOS. Therefore, Grb2 can bind to active RTKs and recruit SOS to the plasma membrane. Grb2 is an example of what kind of signaling molecule? A) scaffold B) adapter protein C) second messenger D) enzyme
Version 1
10
31) Ste5 binds to MAPKKK, MAPKK, and MAPK to organize the kinase cascade. What type of signaling molecule is Ste5? A) scaffold B) adapter protein C) second messenger D) enzyme
32) Many receptor tyrosine kinase pathways have MAPK as a downstream signaling component. MAPK can phosphorylate target proteins. What type of signaling molecule is MAPK? A) scaffold B) adapter protein C) second messenger D) enzyme
33) A small number of cell surface receptors can ultimately generate a large intracellular response due to activation of kinase cascades. This is referred to as A) signalamplification. B) signaltransduction. C) signaldampening. D) signalactivation.
34)
What enzyme phosphorylates MAP kinase kinase? A) MAP kinase B) MAP kinase kinase C) MAP kinase kinase kinase D) MAP phosphatase
Version 1
11
35) The cell surface receptors that can act onlyindirectly on plasma membrane-bound enzymes or ion channels are called A) G protein-coupledreceptors. B) gated ion channels. C) enzymatic receptors. D) second messenger receptors.
36) G protein-coupled receptors are the largest family of cell surface receptors. Each receptor passes through the plasma membrane how many times? A) once B) three times C) five times D) seven times
37) In the nitric oxide signaling pathway, nitric oxide activates an enzyme that can convert many molecules of GTP into molecules of cGMP. cGMP is a small molecule that can diffuse through the cytoplasm and bind to an enzyme called protein kinase G (PKG). Binding of cGMP to PKG leads to a change in the confirmation of PKG that leads to its activation. What type of signaling molecule is cGMP? A) scaffold B) adapter protein C) second messenger D) enzyme
38) Some enzymatic receptors and most G protein-coupled receptors transmit the signal into the cytoplasmby utilizing other substances within the cytoplasm that are referred to as
Version 1
12
A) genes. B) intracellular receptors. C) second messengers. D) first messengers.
39) Calmodulin is a calcium-binding protein. A mutation that affects Ca ++ binding to calmodulin is most likely to interfere with what aspect of a signal transduction pathway? A) the flow of Ca ++ ions through ion channel receptors B) the cellular response to the second messenger C) the MAP kinase cascade D) G protein activity
40) If the effector protein phospholipase C failed to cleave phosphatidylinositol-4,5bisphosphate (PIP 2), the action of what enzyme would be affected? A) cAMP B) IP 3 C) protein kinase A (PKA) D) protein kinase C (PKC)
41) What is similar about G protein-coupled receptors (GPCRs)and receptor tyrosine kinases (RTKs)? A) Both are directlybound to G proteins. B) Both can activatephospholipase C. C) Both areenzymes. D) Both are activatedby autophosphorylation. E) Both bind tohydrophobic ligands.
42)
The function of adenylyl cyclase is to
Version 1
13
A) catalyze theconversion of ATP to cAMP. B) directly activateprotein kinase A (PKA). C) stimulate therelease of calcium from the endoplasmic reticulum. D) cleave PIP 2 into DAG and IP 3.
43) What is the difference between Ras and the G proteins bound to G protein-coupled receptors (GPCRs)? A) Ras is an enzyme, and the G proteins bound to GPCRs are not enzymes. B) Ras is a second messenger, and the G proteins bound to GPCRs are not second messengers. C) Ras is a small G protein, and the G proteins bound to GPCRs are not small G proteins. D) Ras can activate different effector molecules, and the G proteins bound to GPCRs cannot activate different effector molecules.
44)
What second messenger is directlyinvolved in activating protein kinase C (PKC)? A) DAG B) cAMP C) cGMP D) phospholipase C E) G protein
Version 1
14
Answer Key Test name: Chap 09_3e 1) E
Version 1
15
Clarify Question ● What is the key concept addressed by the question? question asks about autocrine signaling.
● The
● What type of thinking is required? ● You are being asked to apply your knowledge of autocrine signaling. Gather Content ● What do you know about autocrine signaling? What other information is related to the question? ● Cells communicate in four ways. a. Two cells in direct contact with each other may send signals across gap junctions. b. In paracrine signaling, secretions from one cell have an effect only on cells in the immediate area. c. In endocrine signaling, hormones are released into the organism’s circulatory system, which carries them to the target cells. d. Chemical synapse signaling involves transmission of signal molecules, called neurotransmitters, from a neuron over a small synaptic gap to the target cell. In addition to using these four basic mechanisms, some cells actually send signals to themselves, secreting signals that bind to specific receptors on their own plasma membranes. This process, called autocrine signaling, is thought to play an important role in reinforcing developmental changes, and it is an important component of signaling in the immune system Choose Answer ● Given what you now know, what information is most likely to produce the correct answer? ● If a cell responds to the ligand that it secretes, this is considered autocrine signaling. Reflect on Process ● Did your problem-solving process lead you to the correct answer? If not, where did the process break down or lead you astray? How can Version 1
16
you revise your approach to produce a more desirable result? ● This question asked you to apply your understanding of autocrine signaling. If you got the correct answer, great job! If you got an incorrect answer, where did the process break down? Did you recall that most forms of signaling involve a molecule secreted by one cell activating a different cell? Did you understand that the same cell was secreting and responding to sonic hedgehog?
2) A Gap junctions require direct contact between cells, and thus this is an example of direct contact signaling. 3) A 4) B
Version 1
17
Clarify Question ● What is the key concept addressed by the question? question asks about paracrine signaling.
● The
● What type of thinking is required? ● You are being asked to apply your knowledge of paracrine signaling. Gather Content ● What do you know about? paracrine signaling What other information is related to the question? ● Cells communicate in four ways. a. Two cells in direct contact with each other may send signals across gap junctions. b. In paracrine signaling, secretions from one cell have an effect only on cells in the immediate area. c. In endocrine signaling, hormones are released into the organism’s circulatory system, which carries them to the target cells. d. Chemical synapse signaling involves transmission of signal molecules, called neurotransmitters, from a neuron over a small synaptic gap to the target cell. In addition to using these four basic mechanisms, some cells actually send signals to themselves, secreting signals that bind to specific receptors on their own plasma membranes. This process, called autocrine signaling, is thought to play an important role in reinforcing developmental changes, and it is an important component of signaling in the immune system Choose Answer ● Given what you now know, what information is most likely to produce the correct answer? ● Because the platelets are activating nearby cells, this is an example of paracrine signaling. Reflect on Process ● Did your problem-solving process lead you to the correct answer? If not, where did the process break down or lead you astray? How can Version 1
18
you revise your approach to produce a more desirable result? ● This question asked you to apply your understanding of. paracrine signaling If you got the correct answer, great job! If you got an incorrect answer, where did the process break down? Did you recall that paracrine signaling worked on nearby, but not adjacent cells? Did you understand that platelets were not activating themselves, so this was not autocrine signaling?
5) C
Version 1
19
Clarify Question • What is the key concept addressed by the question? o The question asks about signaling involved in cancer growth. • What type of thinking is required? o You are being asked to analyze the signaling involved in cancer growth. Gather Content • What do you know about signaling involved in cancer growth? What other information is related to the question? o Synaptic signaling occurs between neurons across a synapse. Autocrine signaling is a cell talking to itself. Endocrine signaling involves hormones that travel through the blood to act on different organs. Paracrine signaling involves communication with nearby cells. Choose Answer • Given what you now know, what information is most likely to produce the correct answer? o Since the drug only works on small tumors that have not metastasized, and prevents signaling from one cell to another cell, it must inhibit paracrine signaling. Synaptic signaling is irrelevant in this case, as signaling does not involve neurons. Reflect on Process • Did your problem-solving process lead you to the correct answer? If not, where did the process break down or lead you astray? How can you revise your approach to produce a more desirable result? o This question asked you to analyze the signaling involved in cancer Version 1
20
growth. If you got the correct answer, great job! If you got an incorrect answer, where did the process break down? Did you think that synaptic signaling would stimulate growth in a cell? Did you think that endocrine signaling worked on nearby cells? 6) A
Version 1
21
Clarify Question • What is the key concept addressed by the question? o The question asks about specifically inhibiting one class of receptors. • What type of thinking is required? o You are being asked to apply your knowledge about specifically inhibiting one class of receptors. Gather Content • What do you know about specifically inhibiting one class of receptors? What other information is related to the question? o Receptors bind to specific ligands. This then can activate the opening of ion channels or production of second messengers. Multiple receptors can activate similar pathways, but only one ligand binds each receptor. Choose Answer • Given what you now know, what information is most likely to produce the correct answer? o To specifically inhibit one receptor a chemical that binds to the ligand binding site would work best. Chemicals that block the movement of ions do not specifically block the action of one receptor. Reflect on Process • Did your problem-solving process lead you to the correct answer? If not, where did the process break down or lead you astray? How can you revise your approach to produce a more desirable result? o This question asked you to apply your knowledge about specifically inhibiting one class of receptors. If you got the correct answer, great job! If you got an incorrect answer, where did the process break down? Did Version 1
22
you think that blocking movement of ions would be specific to one receptor? Did you think that ion transport would use second messenger signaling? 7) B
Version 1
23
Clarify Question • What is the key concept addressed by the question? o The question asks about resistance to leptin. • What type of thinking is required? o You are being asked to analyze why some obese people are resistance to leptin. Gather Content • What do you know about resistance to leptin? What other information is related to the question? o For a hormone like leptin to function it must bind to a receptor. This then allows a signal to pass into the cell. If the hormone levels are low, or the receptor does not work properly then the cells do not respond to the hormone. Choose Answer • Given what you now know, what information is most likely to produce the correct answer? o If a person lacks leptin or the receptor then their brain will not receive a signal that they are full and their appetite will not be inhibited. Obese individuals who have normal leptin levels but low leptin receptor activity will not respond to the leptin, will be hungry more often and become obese. Reflect on Process • Did your problem-solving process lead you to the correct answer? If not, where did the process break down or lead you astray? How can you revise your approach to produce a more desirable result? Version 1
24
o This question asked you to analyze why some obese people are resistance to leptin. If you got the correct answer, great job! If you got an incorrect answer, where did the process break down? Did you think that leptin was a paracrine signal instead of endocrine? Did you think that leptin was a second messenger or kinase? 8) A 9) C
Version 1
25
Clarify Question • What is the key concept addressed by the question? o The question asks about different types of signaling. • What type of thinking is required? o You are being asked to analyze what type of signaling glucocorticoids and mineralocorticoids perform. Gather Content • What do you know about different types of signaling? What other information is related to the question? o Signaling with adjacent cells include paracrine, direct contact, synaptic and autocrine. Endocrine signaling works at a distance with the hormones moving through the blood to distant organs. Choose Answer • Given what you now know, what information is most likely to produce the correct answer? o Since the symptoms of this disease are systemic and occur throughout the body, this is an example of endocrine signaling. In fact, glucocorticoids and mineralocorticoids are hormones, and this disease is considered an endocrine disorder. Reflect on Process • Did your problem-solving process lead you to the correct answer? If not, where did the process break down or lead you astray? How can you revise your approach to produce a more desirable result? o This question asked you to analyze what type of signaling glucocorticoids and mineralocorticoids perform. If you got the correct Version 1
26
answer, great job! If you got an incorrect answer, where did the process break down? Did you think that by affecting many organs glucocorticoids and mineralocorticoids only worked on nearby cells? Did you think that glucocorticoids and mineralocorticoids were not hormones? 10) D Protein kinases function to phosphorylate proteins. 11) A
Version 1
27
Clarify Question • What is the key concept addressed by the question? o The question asks about phosphorylation of a protein. • What type of thinking is required? o You are being asked to apply your knowledge about phosphorylation of a protein. Gather Content • What do you know about phosphorylation of a protein? What other information is related to the question? o The class of enzyme that adds phosphate groups from ATP to proteins is called a protein kinase. These phosphate groups can be added to the three amino acids that have an OH as part of their R group, namely serine, threonine, and tyrosine. We categorize protein kinases as either serine–threonine or tyrosine kinases based on the amino acids they modify. Most cytoplasmic protein kinases fall into the serine–threonine kinase class. Choose Answer • Given what you now know, what information is most likely to produce the correct answer? o Since this protein has serine/threonine phosphorylation motifs, it may be a substrate for serine/threonine phosphatases and kinases. Phospholipases cleave phospholipids. GPCRs are not enzymes. Phosphatases remove phosphates from proteins. Reflect on Process • Did your problem-solving process lead you to the correct answer? If Version 1
28
not, where did the process break down or lead you astray? How can you revise your approach to produce a more desirable result? o This question asked you to apply your knowledge about phosphorylation of a protein. If you got the correct answer, great job! If you got an incorrect answer, where did the process break down? Did you think that the serine/threonine phosphorylation sites were a substrate for a tyrosine kinase? Did you think that a phosphatase adds a phosphate to a protein? 12) B
Version 1
29
Clarify Question • What is the key concept addressed by the question? o The question asks about the effects of a kinase inhibitor. • What type of thinking is required? o You are being asked to analyze statements about the effects of a kinase inhibitor. Gather Content • What do you know about kinase inhibitors? What other information is related to the question? o The class of enzyme that adds phosphate groups from ATP to proteins is called a protein kinase. These phosphate groups can be added to the three amino acids that have an OH as part of their R group, namely serine, threonine, and tyrosine. We categorize protein kinases as either serine–threonine or tyrosine kinases based on the amino acids they modify. Most cytoplasmic protein kinases fall into the serine–threonine kinase class. Choose Answer • Given what you now know, what information is most likely to produce the correct answer? o If this molecule prevents ATP binding to the kinase, this will inhibit the reaction catalyzed by the kinase. This is because the kinase acts to transfer the terminal phosphate of ATP onto its substrate. If binding to ATP is inhibited, then the kinase will not have a source of phosphate nor the energy source needed to drive this endergonic reaction. In fact, this is an approach that is used by multiple drugs that are known kinase inhibitors.
Version 1
30
Reflect on Process • Did your problem-solving process lead you to the correct answer? If not, where did the process break down or lead you astray? How can you revise your approach to produce a more desirable result? o This question asked you to analyze statements about the effects of a kinase inhibitor. If you got the correct answer, great job! If you got an incorrect answer, where did the process break down? Did you think that the substrate could be phosphorylated without the activity of the kinase? Did you think that the inhibitor would increase kinase activity? 13) D 14) B
Version 1
31
Clarify Question • What is the key concept addressed by the question? o The question asks about the results of a phosphorylation assay. • What type of thinking is required? o You are being asked to analyze the results of a phosphorylation assay. Gather Content • What do you know about phosphorylation? What other information is related to the question? o The class of enzyme that adds phosphate groups from GTP to substrates is called a kinase. These phosphate groups can be added to the substrate by the enzyme. In the assay, a “+” indicates that that component was added to the reaction and a “–“ indicates that it was left out. Choose Answer • Given what you now know, what information is most likely to produce the correct answer? o In the experiment the substrate was phosphorylated in the two conditions where the substrate and GTP were present. The substrate could still be phosphorylated when the kinase was absent, indicating that the kinase was not necessary for phosphorylation. The best answer is that the substrate could phosphorylate itself even in the absence of the kinase. Reflect on Process • Did your problem-solving process lead you to the correct answer? If not, where did the process break down or lead you astray? How can you revise your approach to produce a more desirable result? o This question asked you to analyze the results of a phosphorylation assay. If you got the correct answer, great job! If you got an incorrect answer, where did the process break down? Did you think that the kinase was necessary for phosphorylation of the substrate? Did you think that GTP was not necessary for phosphorylation of the substrate?
15) C 16) A 17) B 18) C 19) C
Version 1
32
Steroid hormone receptors are intracellular receptors, and of these choices, would most likely be found in the cytoplasm. 20) A They are mostly lipophilic molecules so that they can penetrate the lipid bilayer and interact with their receptors. 21) C
Version 1
33
Clarify Question • What is the key concept addressed by the question? o The question asks about an inhibitor of the estrogen receptor. • What type of thinking is required? o You are being asked to analyze statements about an inhibitor of the estrogen receptor. Gather Content • What do you know about an inhibitor of the estrogen receptor? What other information is related to the question? o The nonpolar structure of steroid hormones like estrogen allows these hormones to cross the membrane and bind to intracellular receptors. The location of steroid hormone receptors prior to hormone binding is cytoplasmic, but their primary site of action is in the nucleus. Binding ofthe hormone to the receptor causes the complex to shift from the cytoplasm to the nucleus. As the ligand–receptor complex makes it all the way to the nucleus of the cell, these receptors are often called nuclear receptors. Choose Answer • Given what you now know, what information is most likely to produce the correct answer? o Because this molecule is hydrophilic, it will have a difficult time passing through the plasma membrane. Because the estrogen receptor is an intracellular receptor, this will not likely be an effective inhibitor. If the molecule were hydrophobic then it would be a good inhibitor. Reflect on Process
Version 1
34
• Did your problem-solving process lead you to the correct answer? If not, where did the process break down or lead you astray? How can you revise your approach to produce a more desirable result? o This question asked you to analyze statements about an inhibitor of the estrogen receptor. If you got the correct answer, great job! If you got an incorrect answer, where did the process break down? Did you think that the inhibitor could diffuse across the membrane even though it is hydrophilic? Did you think that the estrogen receptor was on the surface of the cell? 22) D
Version 1
35
Clarify Question • What is the key concept addressed by the question? o The question asks about steroid receptors. • What type of thinking is required? o You are being asked to analyze statements about steroid receptors. Gather Content • What do you know about steroid receptors? What other information is related to the question? o The nonpolar structure of steroid hormones allows these hormones to cross the membrane and bind to intracellular receptors. The location of steroid hormone receptors prior to hormone binding is cytoplasmic, but their primary site of action is in the nucleus. Binding ofthe hormone to the receptor causes the complex to shift from the cytoplasm to the nucleus. In the nucleus the steroid receptor binds to DNA turning on different genes will be turned on in different cell types. Choose Answer • Given what you now know, what information is most likely to produce the correct answer? o A mutation in the DNA-binding domain of a steroid receptor would prevent it from binding to the promoters of some genes. If these genes were not expressed normally it would affect the cellular response to the hormone. A mutation in the DNA binding domain would not affect the binding of the hormone to the receptor, its conformational change after binding, or moving into the nucleus. Reflect on Process
Version 1
36
• Did your problem-solving process lead you to the correct answer? If not, where did the process break down or lead you astray? How can you revise your approach to produce a more desirable result? o This question asked you to analyze statements about steroid receptors. If you got the correct answer, great job! If you got an incorrect answer, where did the process break down? Did you think that mutations in the DNA binding domain of a steroid receptor would affect its ability to bind to the hormone? Did you think that mutations in the DNA binding domain of a steroid receptor would affect its movement to the nucleus? 23) A 24) C 25) D 26) A
Version 1
37
Clarify Question • What is the key concept addressed by the question? o The question asks about over expression of receptor tyrosine kinases. • What type of thinking is required? o You are being asked to analyze statements about over expression of receptor tyrosine kinases. Gather Content • What do you know about receptor tyrosine kinases? What other information is related to the question? o Receptor tyrosine kinases have a relatively simple structure consisting of a single transmembrane domain, an extracellular ligand-binding domain, and an intracellular kinase domain. This kinase domain contains the catalytic site of the receptor, which acts as a protein kinase that adds phosphate groups to tyrosines. On ligand binding to a specific receptor, two of these receptor–ligand dimerize and phosphorylate each other,. Choose Answer • Given what you now know, what information is most likely to produce the correct answer? o Dimerization leads to activation of RTKs, and thus if there are too many receptors on the cell surface, it is more likely that dimerization and activation of the signaling pathway will happen in the absence of ligand. This can lead to the inappropriate activation of a signaling pathway that promotes cell division. Dimerization and autophosphorylation has to occur with RTKs in the same cell. There is no mechanism for this occurring between two different cells. RTKs are membrane bound proteins that do not enter the nucleus. Autophosphorylation is necessary for activation of this pathway and thus if autophosphorylation were Version 1
38
inhibited, it would be less likely that the downstream pathway would be active. Reflect on Process • Did your problem-solving process lead you to the correct answer? If not, where did the process break down or lead you astray? How can you revise your approach to produce a more desirable result? o This question asked you to analyze statements about over expression of receptor tyrosine kinases. If you got the correct answer, great job! If you got an incorrect answer, where did the process break down? Did you think that receptor tyrosine kinases allow cells to bind to each other? Did you think that receptor tyrosine kinases enter the nucleus of a cell once they are activated? 27) C Receptor tyrosine kinase pathways commonly use the protein Ras to transduce the signal.Ras is a small G protein. 28) C
Version 1
39
Clarify Question • What is the key concept addressed by the question? o The question asks about the signaling pathway activated by a receptor tyrosine kinase. • What type of thinking is required? o You are being asked to evaluate an experiment on the signaling pathway activated by a receptor tyrosine kinase. Gather Content • What do you know about the signaling pathway activated by a receptor tyrosine kinase? What other information is related to the question? o When a receptor tyrosine kinase is activated it dimerizes and autophosphorylates itself. This then passes a signal to adapter proteins that carry the signal to other proteins in the cell. At the end of the signaling pathway a transcription factor is typically activated increasing gene expression. Choose Answer • Given what you now know, what information is most likely to produce the correct answer? o If the receptor and protein X do not seem to be interacting directly then it is possible that another adapter protein is necessary to carry the signal from the receptor to protein X. Reflect on Process • Did your problem-solving process lead you to the correct answer? If not, where did the process break down or lead you astray? How can Version 1
40
you revise your approach to produce a more desirable result? o This question asked you to evaluate an experiment on the signaling pathway activated by a receptor tyrosine kinase. If you got the correct answer, great job! If you got an incorrect answer, where did the process break down? Did you think that proteins downstream of protein X would affect the interaction between the receptor and protein X? Did you think that the interaction between the ligand and receptor would interfere with binding between the receptor and protein X? 29) B
Version 1
41
Clarify Question • What is the key concept addressed by the question? o The question asks about the interaction between Ras and SOS. • What type of thinking is required? o You are being asked to analyze what would happen if Ras and SOS could not interact. Gather Content • What do you know about the interaction between Ras and SOS? What other information is related to the question? o The protein SOS is a guanine nucleotide exchange factor for Ras. SOS helps facilitate the exchange of GDP for GTP. Ras is a G protein that is active when GTP is bound to it. Ras can then cleave a phosphate off of GTP forming GDP to go into the inactive state. When SOS replaces GDP with GTP it reactivates the Ras. Choose Answer • Given what you now know, what information is most likely to produce the correct answer? o Without SOS, Ras would have GDP bound to it and could not replace the GDP with GTP. Without GTP, the Ras would be inactive. Reflect on Process • Did your problem-solving process lead you to the correct answer? If not, where did the process break down or lead you astray? How can you revise your approach to produce a more desirable result? o This question asked you to analyze what would happen if Ras and SOS could not interact. If you got the correct answer, great job! If you Version 1
42
got an incorrect answer, where did the process break down? Did you think that GTP would remain bound to Ras in the absence of SOS? Did you think that Ras would increase breakdown of GTP into GDP? 30) B 31) A 32) D 33) A 34) C 35) A 36) D 37) C 38) C 39) B 40) D
Version 1
43
Clarify Question • What is the key concept addressed by the question? o The question asks about enzymes activated by inositol triphosphate. • What type of thinking is required? o You are being asked to apply your knowledge aboutenzymes activated by inositol triphosphate. Gather Content • What do you know about enzymes activated by inositol triphosphate? What other information is related to the question? o A common second messenger is produced from the molecules called inositol phospholipids. These are inserted into the plasma membrane by their lipid ends and have the inositol phosphate portion protruding into the cytoplasm. The most common inositol phospholipid is phosphatidylinositol-4,5-bisphosphate (PIP2). This molecule is a substrate of the effector protein phospholipase C, which cleaves PIP2 to yield diacylglycerol (DAG) and inositol-1,4,5-trisphosphate (IP3).Both of these compounds then act as second messengers with a variety of cellular effects. DAG, like cAMP, can activate a protein kinase, in this case protein kinase C (PKC). Choose Answer • Given what you now know, what information is most likely to produce the correct answer? o When inositol triphosphate (IP3) is produced by phospholipase C it actives protein kinase C. Protein kinase A is activated by cAMP. cAMP and IP3 are second messengers and not enzymes. Reflect on Process Version 1
44
• Did your problem-solving process lead you to the correct answer? If not, where did the process break down or lead you astray? How can you revise your approach to produce a more desirable result? o This question asked you to apply your knowledge about enzymes activated by inositol triphosphate. If you got the correct answer, great job! If you got an incorrect answer, where did the process break down? Did you think thatcAMPor IP3were enzymes? Did you think that protein kinase A was activated by IP3? 41) B 42) A 43) C 44) A
Version 1
45
CHAPTER 10 CHECK ALL THE APPLY. Choose all options that best completes the statement or answers the question. 1) Which of the following describe changes in how scientists understand the genetics of cancer as a result of new data collected? Select all that apply. A) Malignant transformation is a linear progression of cellular mutations. B) Most cancer gene mutations found in solid tumors are single-base substitutions. C) Mutations that confer growth advantages to cells (i.e. cause cancer) can occur in oncogenes or tumor suppressor genes. D) Cancer is the result of interactions between biological processes such as gene mutations, metabolic pathways, genomic stability, and patterns of transcription. E) Cancer may or may not include over proliferation of cells in various tissues/organs. F) Tens of thousands of cancer genomes have been analyzed and the data are publically available to help identify mutations that drive cancer development.
MULTIPLE CHOICE - Choose the one alternative that best completes the statement or answers the question. 2) In bacterial cell division, the cell divides into two nearly equal halves. This process is referred to as __________. A) binaryfission B) mitosis C) fusion D) meiosis E) cytokinesis
3) How does the organization of the bacterial genome differ from the organization of the eukaryotic genome?
Version 1
1
A) The compaction ofthe eukaryotic genome involves structural maintenance of chromosome (SMC)proteins, and the compaction of the bacterial genome does not. B) Most bacterialchromosomes are circular and the eukaryotic chromosomes contained in thenucleus are not. C) Bacterialchromosomes are made up of RNA and eukaryotic chromosomes are made up ofDNA. D) The eukaryoticgenome is found on chromosomes and there are no chromosomes in bacterialcells. E) Eukaryotic chromosomes have to be tightly packed to fit into the nucleus,and bacterial chromosomes do not require tight packing to fit into thecell.
4)
The division of a bacterial cell occurs as the __________. A) cell wall develops cracks around the equator of the cell B) chromosomes are pulled toward the ends of the cell C) actin and microtubules constrict the cytoplasm D) new membrane and cell wall materials begin to grow and form a septum
5) If a cell contained a mutation in the gene that encodes FtsZ, which process would be affected? A) Septation B) Cytokinesis C) Prophase D) DNA Synthesis E) Cohesin cleavage
6) The accommodation of the very long DNA strands that are part of a chromosomeinto the limited space of the nucleus is achieved by coiling the DNAaround beads of histones into repeating subunits. These DNA-wrapped histones are called __________.
Version 1
2
A) solenoids B) nucleosomes C) chromatin loops D) rosettes
7) The point of constriction on chromosomes thatcontains certain repeated DNA sequences that bind specific proteins is called __________. A) the kinetochore B) the centromere C) the cohesin complex D) the centriole E) the centrosome
8) Eukaryotic chromosomes are composed of a complex of 60% protein and 40% DNA. This complex is referred to as __________. A) the histone complex B) chromatin C) the kinetochore D) cohesin
9) The number of chromosomes characteristic of diploid eukaryotic cells, in general __________. A) is 46 B) can be predicted bythe size of the organism C) changes as each organismgrows and ages D) varies considerably from 2 to over 1000 in different species
Version 1
3
10) A person whose cells all contain a chromosome number of 2n+1 most likely has what type of condition? A) Monosomy B) Trisomy C) Cancer D) Obesity E) None, this is thenormal chromosome number for humans.
11) In later chapters, you will learn more about the regulation of gene expression. One way to regulate gene expression is to make changes to the histone proteins to alter how tightly the DNA is coiled and wrapped. The more tightly coiled and wrapped a particular region of DNA is, the less likely it is that the genes in that region will be expressed. Bearing this in mind, how tightly do you think regions of heterochromatin are compacted? A) Very tightly B) Moderately C) Loosely D) Not at all
12) You are assembling a model of a chromosome, but begin having some trouble when you get to the step of forming chromatin loops. If you are unable to resolve this problem, what step of chromosome structure would you be unable to achieve? A) Histone/DNA complex B) Nucleosome C) Solenoid D) Rosettes
13)
A scientist wants to study histones. Histones are __________.
Version 1
4
A) proteins that double-stranded DNA molecules wrap around in eukaryotes B) proteins that double-stranded DNA molecules wrap around in prokaryotes and eukaryotes C) proteins that regulate checkpoints in the mitotic cell cycle D) proteins that serve as the spindle fiber to pull sister chromatids apart during anaphase E) proteins that required for the activity of Cdk enzymes
14) The two copies of each type of chromosome foundin normal somatic (body) cells in an organism, throughout the cell cycle,are called __________. A) sister chromatids B) homologous chromosomes C) daughter chromosomes D) kinetochores
15) Before cell division of somatic cells, each chromosome must be replicated. After replication, the resulting two parts of each chromosome are held together by cohesin at the centromere. These two parts are referred to as __________. A) sister chromatids B) homologous chromosomes C) daughter chromosomes D) kinetochores E) genes
16)
Which one of these structures is held together by cohesin? A) Nucleosomes B) Sister chromatids C) Homologous chromosomes D) Solenoids
Version 1
5
17) A cell biologist produces a karyotype of mouse somatic cells arrested in mitosis. She sees 40 chromosomes, which is completely normal for mice. Based on this information, what is the haploid number of chromosomes for mice? A) 10 B) 20 C) 40 D) 80 E) It cannot be determined from the information provided.
18) If there are 32 sister chromatids in a normal somatic cell, what is the haploid number for that cell? A) 8 B) 16 C) 32 D) 64
19) If there are 32 sister chromatids in a normal somatic cell, how many chromosomes are there? A) 8 B) 16 C) 32 D) 64
20) A somatic cell from a garden pea normally contains 14 chromosomes.How many sister chromatids would that cell contain during G1 of the cell cycle?
Version 1
6
A) 0 B) 7 C) 14 D) 28
21) A somatic cell from a corn plantnormally contains 20 chromosomes. How many sister chromatids would that cell contain during G 2 of the cell cycle? A) 0 B) 10 C) 20 D) 40
22)
What is the sequence of events in a typical eukaryotic cell cycle? A) G 1 to G 2 to S to mitosis to cytokinesis B) G 1 to S to G 2 to mitosis to cytokinesis C) G 1 to S to G 2 to cytokinesis to mitosis D) G 1 to G 2 to mitosis to S to cytokinesis E) S to G 1 to G 2 to mitosis to cytokinesis
23) The portion of the cell cycle when the cell is growing and does not contain a replicated genome is referred to as __________. A) G 1 B) S C) G 2 D) mitosis E) cytokinesis
Version 1
7
24) The stage of the cell cycle during which the cytoplasm divides to form two cells is called __________. A) G 1 B) S C) G 2 D) mitosis E) cytokinesis
25) Which stage of the cell cycle is characterized by growth and contains a checkpoint to verify that all of the DNA has been replicated prior to mitosis? A) G 1 B) S C) G 2 D) Mitosis E) Cytokinesis
26) A duplicate copy of all of the hereditary information contained in the nucleus of eukaryotic cellsis made during what stage of the cell cycle? A) G 1 B) S C) G 2 D) Mitosis E) Cytokinesis
27)
How is cytokinesis in plant cells different from cytokinesis in animal cells?
Version 1
8
A) Cytokinesis in plant cells requires formation of a cell plate; cytokinesis in animal cells requires formation of a ring of actin. B) Cytokinesis in plant cells occurs during G 2; cytokinesis in animal cells occurs in the M phase of the cell cycle. C) Cytokinesis in plant cell results in one large cell with two nuclei; cytokinesis in animal cells results in two cells, each with one nucleus. D) Cytokinesis in plant cells results in haploid cells; cytokinesis in animal cells results in diploid cells.
28) If a cell has 32 chromosomes prior to S phase and undergoes mitosis followed by cytokinesis, each new daughter cell will have how many chromosomes? A) 64 B) 32 C) 16 D) 8
29) Embryonic cell cycles allow the rapiddivision of cells in the early embryo. These complete mitotic cell cyclesare much shorter in length than the mitotic cell cyclesof cells in a mature organism. The specific phase of mitosis, however, takes approximately the same amount of time during the embryonic cell cycles as it does in the cell cycles of mature cells. What do you think is a result of the embryonic cycle? A) Resultingdaughter cells are smaller than the mother cell in the embryonic cellcycles. B) Resulting daughter cells do not contain the same genetic information as themother cell in the embryonic cell cycles. C) Resulting daughter cells cannot form a mitotic spindle in the embryonic cellcycle. D) Mother cells in the embryonic cell cycle spend the majority of their time in G 0.
30) What isthe portion of the cell cycle during which the chromosomes are invisible under the light microscope because they are not yet condensed?
Version 1
9
A) Interphase B) Prophase C) Metaphase D) Anaphase E) Telophase
31)
Interphase is made up of what stages of the cell cycle? A) G 1 + G 2 + S B) S +cytokinesis C) Prophase + metaphase + anaphase + telophase D) Cytokinesis + mitosis E) G 0 + G 1 + G 2
32)
During what stages of the cell cycle are sister chromatids bound together by cohesin? A) G 1, S,G 2 B) S,G 2 C) G 1,S D) S, G 2,prophase, metaphase E) S, G 2, prophase, metaphase,anaphase, telophase
33) Following S phase, a human cell would have how many pairs of sister chromatids and individual DNA molecules? A) 23 pairs of sister chromatids and 46 individual DNA molecules B) 23 pairs of sister chromatids and 92 individual DNA molecules C) 46 pairs of sister chromatids and 46 individual DNA molecules D) 46 pairs of sister chromatids and 92 individual DNA molecules E) 46 pairs of sister chromatids and 184 individual DNA molecules
Version 1
10
34) If a chromosome contains a mutation such that it cannot bind to the kinetochore complex, what would be the consequence? A) That chromosomewould not be able to be replicated. B) That chromosomewould not be able to condense. C) That chromosomewould not be able to bind to the mitotic spindle. D) That chromosomewould not be able to interact with histone proteins.
35) Consider the cell cycle of a human cell. During G2, what is the state of the homologous chromosomes? A) The homologouschromosomes are lined up on the equator of the cell. B) The homologouschromosomes have all been copied through DNA replication and are beginning tocondense. C) The homologous chromosomes have been pulled to their respective poles by the spindle apparatus. D) The homologous chromosomes have not been replicated yet. E) The homologous chromosomes are now in the haploid or n condition.
36) Which stage of mitosis is characterized by the alignment of the chromosomes in a ring along the inner circumference of the cell? A) Interphase B) Telophase C) Prophase D) Metaphase
37) The stage of mitosis characterized by the physical separation of sister chromatids is called __________.
Version 1
11
A) anaphase B) metaphase C) prometaphase D) telophase
38) Which stage of mitosis is characterized by the disassembly of spindle apparatus, the reestablishment of the nuclear membrane, and the decondensation of the chromosomes? A) Prometaphase B) Telophase C) Anaphase D) Metaphase
39) During which stage of mitosis does the nuclear envelope begin tobreak down and the spindle begins to form? A) Anaphase B) Metaphase C) Prophase D) Telophase E) Prometaphase
40) In prophase, ribosomal RNA synthesis stops when the chromosomes condense, and as a result __________. A) the chromosomes lengthen B) the nuclear envelope reforms C) the nucleolus disappears D) the chromosomes line up at the equator of the cell
Version 1
12
41) During which stage of mitosis do the chromosomes become attached to the spindle at their kinetochores? A) Prophase B) Prometaphase C) Metaphase D) Anaphase E) Telophase
42)
What happens during Anaphase B? A) Kinetochores arepulled toward the poles. B) The spindle polesmove apart. C) The spindleapparatus disassembles. D) The nuclearenvelope reforms. E) The APC/C getsactivated for the first time.
43) You are conducting a genetic screen using Caenorhabditis elegans embryos to isolate mutations affecting anaphase (A). Therefore, you need to look for embryos in which __________. A) the centromeres do not move toward the poles B) the poles do not move apart C) the spindle apparatus does not disassemble D) sister chromatids are mismatched and therefore fail to separate
44)
What stage of mitosisis essentially the reverse of prophase?
Version 1
13
A) Anaphase B) Prometaphase C) Metaphase D) Telophase E) Cytokinesis
45) The drug taxol, or paclitaxel, is used to treat patients with a variety of cancers, including breast, lung and ovarian cancers. The drug works by stabilizing microtubules, and preventing their disassembly. The goal of the drug is to prevent dividing cells from being able to complete mitosis. As a result, cancerous cells can no longer divide. In a cell treated with taxol, at what stage of mitosiswill the cells arrest? A) Prior to metaphase B) Anaphase C) Telophase D) Cytokinesis
46) Why is it so important that all of the chromosomes align on the metaphase plate during metaphase? A) This is the onlyplace in the cell where the cyclins and Cdks are located. B) If they cannot,it suggests that they aren't properly attached to the spindle microtubules, andthus won't separate properly during anaphase. C) This is thelocation where the chromosomes can become attached to the spindlemicrotubules. D) This allows astersto form. E) This allows sister chromatids toform.
47) If a cell was capable of bypassing metaphase and going directly from prometaphase to anaphase, what is the most likelyconsequence?
Version 1
14
A) The resulting daughter cells would not have a nuclear envelope. B) The resultingdaughter cells would have significantly different quantities of cytoplasmicmaterials. C) The resultingdaughter cells would have different numbers of chromosomes. D) The resultingdaughter cells would be completely normal.
48)
Animal cells typically achieve cytokinesis by __________. A) binary fission B) forming a cell plate across the middle of the cell C) forming a cleavage furrow that pinches the cell into two D) chromosome condensation E) chromosome elongation
49)
Plant cells typically achieve cytokinesis by __________. A) binary fission B) forming a cell plate across the middle of the cell C) forming a cleavage furrow that pinches the cell into two D) chromosome condensation E) chromosome elongation
50) If a drug that inhibited transport from the trans face of the Golgi was applied to plant cells, which stage of the cell cycle would be directly affected? A) G 2 B) S C) Metaphase D) Anaphase E) Cytokinesis
Version 1
15
51)
The progress of the eukaryoticcell cycleis regulated primarily by what proteins? A) Cyclins B) Histone proteins C) Condensins D) FtsZ
52)
At what checkpoint(s) does the cell arrest in response toDNA damage? A) G 1/S B) G 2/M C) Spindle D) G 1/S and G 2/M E) G 2/M and spindle
53) You are examining the effect of maturation-promoting factor (MPF) in sea urchin cells, which have a diploid number of 36. If you fuse a dividing sea urchin cell with a G 1 arrested oocyte, what would be the outcome? A) The G 1 cell would enter mitosis, but would likely arrestat the spindle checkpoint because the chromosomes have not been properlyreplicated. B) The G 1 cell would undergo mitosis and its daughter cells would each have 36 chromosomes. C) The G 1 cell would undergo mitosis and its daughter cellswould each have 18 chromosomes. D) The G 1 cell would first go through S phase and then mitosis. Its daughter cells would have 36 chromosomes.
Version 1
16
54)
What time point represents G2?
A) 1 B) 2 C) 3 D) 4
55) You are studying cell cycle progression in yeast cells. If you could prevent cdc2 from associating with themitotic cyclin, the cells would __________. A) arrest in G 1 B) arrest in G 2 C) arrest in S D) arrest in prometaphase E) arrest in metaphase
56) You are studying cell cycle progression in an early frog embryo. If you were to inject a protein synthesis inhibitor into a cell during S phase, where do you predict that the cells would arrest?
Version 1
17
A) G 1 B) G 2 C) Metaphase D) Telophase
57) In G 2, there are typically high levels of the mitotic cyclin. Why is cdc2 not active during G 2 if the mitotic cyclin is present? A) Cdc2 is alsoregulated by phosphorylation. B) Cdc2 does notbind to the mitotic cyclin. C) Cdc2 requiresubiquitination to be activated. D) Cdc2 also has tobind to cohesin to be activated.
58)
What is separase? A) A protein that marks a protein called securin for destruction B) A protein that is part of the cohesin complex C) A protein that destroys cohesin through its protease activity D) A protein that targets the mitotic cyclin for degradation
59)
What is one of the roles of the APC/C during anaphase? A) To directlytarget the mitotic cyclins for destruction B) To directly target cohesin for destruction C) To directly target separase for destruction D) To directly target microtubules for destruction
60) What would you expect to happen if the anaphase-promoting complex/cyclosome (APC/C) failed to ubiquitinate securin?
Version 1
18
A) The cohesin complex will be destroyed, and the cell will remain in metaphase. B) The cohesin complex will persist, preventing the cell from entering anaphase. C) Separase will be marked for degradation by securin, preventing the cell from entering anaphase. D) Securin willremain intact and therefore will degrade cohesin, allowing the cell to enteranaphase.
61) If you were to think of the cell as a car, and mitosis as a process that drives that car to go, what would be a good analogy for a cell that has a mutation in both copies of a tumor-suppressor gene? A) The gas pedal of a car gets stuck while pushed down. B) The gas pedal of a car does not work at all. C) The brake pedal of a car gets stuck while pushed down. D) The brake pedal of a car does not work at all.
62) You are leading a team of researchers at a pharmaceutical company. Your goal is to design drugs that help fight cancer. Specifically, you want to focus on drugs that bind to and inactivate certain proteins, thereby halting cell cycle progression. One of your team members suggests targeting the retinoblastoma (Rb) protein and inhibiting this protein. Will this approach be successful? Why or why not?
Version 1
19
A) This approach will not be successful. Rb is tumor-suppressor protein and functions to inhibit the action of a number of cell cycle regulatory proteins. A drug designed to inactivate the Rb protein would essentially create the same situation as in as a cell that lacks both copies of the Rb gene. Lack of Rb activity would release the inhibition of cell cycle regulatory proteins, thereby promoting cell cycle progression, rather than halting it. B) This approach will be successful. Rb is an oncogene and functions to activate a number of cell cycle regulatory proteins. A drug designed to inactivate the Rb protein would halt the cell cycle in cells that contain an active Rb. As a result, cancer cells expressing a constitutively active Rb protein would be good targets for this type of therapeutic. C) This approach will be successful. Rb is tumor-suppressor protein and functions to inhibit the action of a number of cell cycle regulatory proteins. A drug designed to inactivate the Rb protein would activate cell cycle inhibition. Lack of Rb activity would therefore inhibit the cell cycle regulatory proteins. D) This approach will not be successful. Rb is an oncogene and functions to activate a number of cell cycle regulatory proteins. A drug designed to inactivate the Rb protein would actually activate cell cycle progression. As a result, this drug would likely make this situation worse for patients whose cancer cells contain mutant Rb.
63) Which protein or protein complexfunctions in the cell to stop cell division if the cell has experienced extensive DNA damage? A) APC/C B) p53 C) FtsZ D) Condensin
64)
In bacteria, DNA replication is initiated at the __________. A) 3' end B) 5' end C) origin of replication D) nucleoid E) terminus
Version 1
20
Version 1
21
Answer Key Test name: Chap 10_3e 1) [B, C, D, F] 2) A 3) B Both eukaryotic and prokaryotic chromosomes have to be tightly packed to fit into the cell, and the compaction process involves SMC proteins in both cases. Both eukaryotic and prokaryotic chromosomes are made up of double stranded DNA. Both eukaryotic and prokaryotic cells contain chromosomes, though the chromosomes are of different structure. 4) D 5) A
Version 1
22
Clarify Question ● What is the key concept addressed by the question? question asks about the role of FtsZ.
● The
● What type of thinking is required? ● You are being asked to apply your knowledge of FtsZ to predict what would happen if it was mutated. Gather Content ● What do you know about FtsZ? What other information is related to the question? ● FtsZ is a protein found in a ring around the center of a bacterium.As this ring closes down the cell splits into two new cells. ● FtsZ is only found in bacteria so it is not involved in eukaryotic processes like cytokinesis, cohesin cleavage, and prophase, nor is itinvolved in DNA synthesis. Choose Answer ● Given what you now know, what information is most likely to produce the correct answer? ● The best answer is that FtsZ is involved in pinching bacterial cells into two new cells through the formation of a ring around the center of the cell called a septum. The process is called septation and will be blocked if FtsZ activity is lost. Reflect on Process ● Did your problem-solving process lead you to the correct answer? If not, where did the process break down or lead you astray? How can you revise your approach to produce a more desirable result? ● This question asked you to apply your understanding of the role of FtsZ in a bacterial cell to predict what would happen if it was mutated. If you got the correct answer, great job! If you got an incorrect answer, where did the process break down? Did you recall that FtsZ is found in the center Version 1
23
of the cell?Did you remember that the septum contracts in septation?
6) B Review Figure 10.5 7) B The centromere is the point of constriction on the chromosome that contains repeated DNA sequences that bind specific proteins. The proteins that bind at the centromere are referred to as the kinetochore. The cohesin complex keeps the sister chromatids attached at the centromere, but the cohesin complex is a protein complex, and does not contain repeated DNA sequences. The centrosome is a microtubule organizing center that contains a pair of centrioles. These are not part of chromosomes. 8) B 9) D 10) B
Version 1
24
Clarify Question ● What is the key concept addressed by the question? question asks about a cell with 2n+1 chromosomes.
● The
● What type of thinking is required? ● You are being asked to apply your knowledge of the normal number of chromosomes in a cell. Gather Content ● What do you know about the number of chromosomes in a human cell? What other information is related to the question? ● The normal chromosome number for humans is 2n, where n = 23 for the number of different types of chromosomes that human cells contain. 2n + 1 indicates that the individual has the diploid number of chromosomes (i.e., 46) plus one additional chromosome. ● Monosomy would be 2n - 1, because this refers to a condition where one chromosome is missing. ● While cancer cells can sometimes gain or lose one or more chromosomes, this would not affect all the cells in the organism - only the cancer cells. ● Chromosome number is not relevant to obesity. Choose Answer ● Given what you now know, what information is most likely to produce the correct answer? ● The presence of an extra chromosome indicates that they most likely have an extra copy of one chromosome, which is how trisomy is defined.Because this is in all cells, this is most likely a case of trisomy. Reflect on Process ● Did your problem-solving process lead you to the correct answer? If not, where did the process break down or lead you astray? How can Version 1
25
you revise your approach to produce a more desirable result? ● This question asked you to apply your understanding of the number of chromosomes in a normal cell to define how an individual 2n+1 chromosomes would be classified. If you got the correct answer, great job! If you got an incorrect answer, where did the process break down? Did you think that 2n-1 was a trisomy and not a monosomy? Did you think that a 2n+1 number of chromosomes in all cells would contribute to cancer or obesity?
11) A
Version 1
26
Clarify Question ● What is the key concept addressed by the question? ● The question asks about the role of DNA bound to histones in gene expression. ● What type of thinking is required? ● You are being asked to apply your knowledge of the effects of histone binding in heterochromatin on gene expression. ● What key words does the question contain and what do they mean? ● Heterochromatin—wound DNA bound to histones ● Histones—positively charged proteins that bind to negatively charged DNA Gather Content ● What do you know about gene expression? What other information is related to the question? ● For a gene to be expressed, enzymes like RNA polymerase need access to the DNA to synthesize RNA from the DNA template. ● If a region of DNA is loosely bound to histones and not at all compacted, the genes would be more likely to be expressed. Choose Answer ● Given what you now know, what information is most likely to produce the correct answer? ● Since heterochromatin domains are not expressed, the DNA must be very tightly compacted in those regions. Reflect on Process ● Did your problem-solving process lead you to the correct answer? If not, where did the process break down or lead you astray? How can you revise your approach to produce a more desirable result? ● This Version 1
27
question asked you to apply your understanding of gene expression to predict if heterochromatin is tightly bound to histones. If you got the correct answer, great job! If you got an incorrect answer, where did the process break down? Did you recall that heterochromatin genes are not expressed? Could you make the connection that tightly wound heterochromatin genes could not be accessed and expressed?
12) D
Version 1
28
Clarify Question ● What is the key concept addressed by the question? question asks about the organization of chromosomes.
● The
● What type of thinking is required? ● You are being asked to apply your knowledge of chromosomes to predict what would happen if chromatin loops could not form. Gather Content ● What do you know about chromosome structure? What other information is related to the question? ● A chromatin loop forms from a solenoid and several of these loops then form a rosette. ● Histone/DNA complexes could coil tightly to form a nucleosome and these in turn coil tightly to form a solenoid, so all of these structures could still form even if the chromatin loop could not. Choose Answer ● Given what you now know, what information is most likely to produce the correct answer? ● If a chromatin loop cannot form, then the next higher order structure formed from the chromatin loop cannot form. In this case the rosettes cannot form. Reflect on Process ● Did your problem-solving process lead you to the correct answer? If not, where did the process break down or lead you astray? How can you revise your approach to produce a more desirable result? ● This question asked you to apply your understanding of chromosome organization to predict what would happen if chromatin loops could not form. If you got the correct answer, great job! If you got an incorrect answer, where did the process break down? Did you recall the order of the structures from smallest to largest is DNA/histone complex, Version 1
29
nucleosome, solenoid, chromatin loop, rosette? Did you understand that if the step of chromatin loop was blocked, the next step would be blocked so no rosettes would form?
13) A 14) B 15) A 16) B 17) B
Version 1
30
Clarify Question ● What is the key concept addressed by the question? ● The question asks about predicting the number of chromosomes in a haploid cell. ● What type of thinking is required? ● You are being asked to apply your knowledge of diploid cells to calculate the number of chromosomes in a haploid cell. ● What key words does the question contain and what do they mean? ● Somatic cell arrested in mitosis—a normal body cell in the mouse which would be diploid Gather Content ● What do you know about diploid cells? What other information is related to the question? ● The karyotype was done with somatic cells arrested in mitosis.The 40 chromosomes observed represents the diploid number (2n). ● If there are 40 chromosomes in a diploid cell then 80 chromosomes would be tetraploid. 10 chromosomes would be half the number in a haploid cell and would not occur naturally. Choose Answer ● Given what you now know, what information is most likely to produce the correct answer? ● The haploid number (n) will be half the number of chromosomes that was observed in the diploid cell (2n). Therefore, the haploid number is 20. Reflect on Process ● Did your problem-solving process lead you to the correct answer? If not, where did the process break down or lead you astray? How can you revise your approach to produce a more desirable result? ● This Version 1
31
question asked you to apply the definitions of haploid and diploid to calculate the number of chromosomes in a haploid cell. If you got the correct answer, great job! If you got an incorrect answer, where did the process break down? Did you remember the definitions of haploid and diploid?Did you recognize that the somatic cell was diploid?Did you know how to calculate the number of chromosomes in a haploid cell if you knew the number in a diploid cell?
18) A
Version 1
32
Clarify Question ● What is the key concept addressed by the question? question asks about sister chromatids.
● The
● What type of thinking is required? ● You are being asked to apply your knowledge of sister chromatids and chromosomes to calculate the number of chromosomes in a haploid cell. ● What key words does the question contain and what do they mean? ● Sister chromatids—after DNA replication, each chromosome contains two sister chromatids Gather Content ● What do you know about sister chromatids? What other information is related to the question? ● If there are 32 sister chromatids and two per chromosome, that means that there are 16 chromosomes. ● Since this is a normal somatic cell it is diploid. So the diploid number of chromosomes is 16. A cell with 32 chromosomes would be tetraploid and a cell with 64 chromosomes would be octaploid. Choose Answer ● Given what you now know, what information is most likely to produce the correct answer? ● Since this is a normal somatic cellthe haploid number would be half of 16 or 8 chromosomes. Reflect on Process ● Did your problem-solving process lead you to the correct answer? If not, where did the process break down or lead you astray? How can you revise your approach to produce a more desirable result? ● This question asked you to apply the definition of sister chromatids to calculate the number of chromosomes in a haploid cell. If you got the Version 1
33
correct answer, great job! If you got an incorrect answer, where did the process break down? Did you recall that there are two sister chromatids in a single chromosome? Did you realize that a normal somatic cell will be diploid? Did you know how to calculate the haploid number by dividing the diploid number by two?
19) B
Version 1
34
Clarify Question ● What is the key concept addressed by the question? question asks about sister chromatids.
● The
● What type of thinking is required? ● You are being asked to apply the definition of sister chromatids to calculate the number of chromosomes in a cell. ● What key words does the question contain and what do they mean? ● Sister chromatids—after DNA replication, each chromosome contains two sister chromatids Gather Content ● What do you know about sister chromatids? What other information is related to the question? ● If there are 32 sister chromatids and two per chromosome, that means that there are 16 chromosomes. ● Sine this is a normal somatic cell it is diploid. So the diploid number of chromosomes is 16. A cell with 32 chromosomes would be tetraploid and a cell with 64 chromosomes would be octaploid. Choose Answer ● Given what you now know, what information is most likely to produce the correct answer? ● Since this is a normal somatic cell the 32 sister chromatids would consist of 16 chromosomes. Reflect on Process ● Did your problem-solving process lead you to the correct answer? If not, where did the process break down or lead you astray? How can you revise your approach to produce a more desirable result? ● This question asked you to apply the definition of sister chromatids to predict the number of chromosomes in a diploid cell. If you got the correct Version 1
35
answer, great job! If you got an incorrect answer, where did the process break down? Did you recall that there are two sister chromatids in a chromosome? Did you think that each sister chromatid was one chromosome?
20) A
Version 1
36
Clarify Question ● What is the key concept addressed by the question? ● The question asks about the number of sister chromatids in G1. ● What type of thinking is required? ● You are being asked to apply your knowledge of the cell cycle to predict how many sister chromatids there are in G1. ● What key words does the question contain and what do they mean? ● G1—before the cell enters the cell cycle so the DNA has not yet replicated Gather Content ● What do you know about sister chromatids? What other information is related to the question? ● Sister chromatids form after the DNA is replicated in S phase but before they separate in mitosis. ● If a somatic cell contains 14 chromosomes, this will be the diploid number. The haploid number will be 7. A cell with 28 chromosomes would be tetraploid. Choose Answer ● Given what you now know, what information is most likely to produce the correct answer? ● Because the cell is still in G1it will not have replicated its DNA and will have no sister chromatids. It will not matter how many chromosomes the cell has. Reflect on Process ● Did your problem-solving process lead you to the correct answer? If not, where did the process break down or lead you astray? How can you revise your approach to produce a more desirable result? ● This question asked you to apply your knowledge of when chromosomes replicate to form sister chromatids. If you got the correct answer, great Version 1
37
job! If you got an incorrect answer, where did the process break down? Did you recall that DNA replicates in S phase after G1? Did you think that the sister chromatids were present in G1?
21) D
Version 1
38
Clarify Question ● What is the key concept addressed by the question? ● The question asks about the number of sister chromatids in G2. ● What type of thinking is required? ● You are being asked to apply your knowledge of the cell cycle to predict how many sister chromatids there are in G2. ● What key words does the question contain and what do they mean? ● G2—after S phase of the cell cycle so the DNA has been replicated Gather Content ● What do you know about sister chromatids? What other information is related to the question? ● Sister chromatids form after the DNA is replicated in S phase but before they separate in mitosis. ● If a somatic cell contains 20 chromosomes, this will be the diploid number. The haploid number will be 10. If the cell had gone through mitosis and entered G1 then there would be 0 sister chromatids. Choose Answer ● Given what you now know, what information is most likely to produce the correct answer? ● Because the cell is in G2 it will have replicated its DNA and will have two sister chromatids per chromosome. Since there are 20 chromosomes there will be 40 sister chromatids. Reflect on Process ● Did your problem-solving process lead you to the correct answer? If not, where did the process break down or lead you astray? How can you revise your approach to produce a more desirable result? ● This question asked you to apply your knowledge of when chromosomes replicate to form sister chromatids. If you got the correct answer, great Version 1
39
job! If you got an incorrect answer, where did the process break down? Did you recall that DNA replicates in S phase before G2 so there would be two sister chromatids per chromosome?Did you think that the sister chromatids had already separated in mitosis which actually occurs after GG22?
22) B 23) A During both G 1 and G 2 the cells are growing, but during G cells do not contain a replicated genome.
1, the
24) E 25) C 26) B 27) A Plant cells are eukaryotic, and therefore like in animals, the division of cytoplasm is referred to as cytokinesis. 28) B
Version 1
40
Clarify Question ● What is the key concept addressed by the question? ● The question asks about the number of chromosomes following mitosis. ● What type of thinking is required? ● You are being asked to apply your knowledge of mitosis to predict the number of chromosomes a cell will have. ● What key words does the question contain and what do they mean? ● G2—after S phase of the cell cycle so the DNA has been replicated Gather Content ● What do you know about mitosis? What other information is related to the question? ● In mitosis the chromosomes are replicated in S phase and then divide evenly into two daughter cells. ● Before S phase the cell will be diploid, so there are 32 chromosomes in this diploid cell. ● If the diploid number is 32 then a cell with 64 chromosomes is tetraploid. A cell with 16 chromosomes is haploid. A cell with 8 chromosomes would not be able to survive. Choose Answer ● Given what you now know, what information is most likely to produce the correct answer? ● If a diploid cell has 32 chromosomes and goes through mitosis each of the new daughter cells would also be diploid and have 32 chromosomes. Reflect on Process ● Did your problem-solving process lead you to the correct answer? If not, where did the process break down or lead you astray? How can you revise your approach to produce a more desirable result? ● This Version 1
41
question asked you to apply your understanding of mitosis to predict how many chromosomes daughter cells would have. If you got the correct answer, great job! If you got an incorrect answer, where did the process break down? Did you recall that the number of chromosomes in the daughter cells does not change following mitosis? Did you confuse mitosis and meiosis? Did you think that the number of chromosomes increase after mitosis?
29) A
Version 1
42
Clarify Question ● What is the key concept addressed by the question? question asks about the rate of cell division.
● The
● What type of thinking is required? ● You are being asked to analyze several predictions about what would happen in the cell cycle was shortened. Gather Content ● What do you know about the cell cycle? What other information is related to the question? ● The cell cycle is made up of four phases; G1 and G2 are gaps where proteins and membranes are made for the newly dividing cells, S where DNA is replicated, and mitosis where the cells divide. ● The question states that the replication of DNA and mitosis occurs at the normal rate. However the overall rate of the cell cycle is shorter. ● For the daughter cells to survive, they must receive the same amount of DNA as the embryonic stem cell, and to do so they need to be able to form a mitotic spindle. Choose Answer ● Given what you now know, what information is most likely to produce the correct answer? ● The embryonic stem cells have a shorter cell cycle, but S and mitosis occur at the same rate, so G1 and G2 must be shorter. As a result, the daughter cells will likely be smaller than the mother cell in the embryonic cell cycles. Reflect on Process ● Did your problem-solving process lead you to the correct answer? If not, where did the process break down or lead you astray? How can you revise your approach to produce a more desirable result? ● This Version 1
43
question asked you to analyze different predictions of what would happen in embryonic stem cells with shorter cell cycles. If you got the correct answer, great job! If you got an incorrect answer, where did the process break down? Did you think that the cells could not be able to divide? Did you think that the daughter cells would not get as much DNA as the embryonic stem cells.
30) A 31) A 32) D Sister chromatids are held together by cohesin from the time they are synthesized in S phase until they enter anaphase and cohesin is cleaved by separase. 33) D
Version 1
44
Clarify Question ● What is the key concept addressed by the question? ● The question asks about numbers of chromatids and individual DNA molecules. ● What type of thinking is required? ● You are being asked to apply your knowledge of chromosomes to predict the numbers of chromatids and DNA molecules after S phase. Gather Content ● What do you know about sister chromatids? What other information is related to the question? ● Sister chromatids form after a chromosome is replicated during S phase. For each initial chromosome there will now be a pair of sister chromatids. ● Each pair of sister chromatids then contains two individual DNA molecules that were created from the single DNA molecule before replication in S phase. ● A human cell starts with 46 chromosomes.These are then replicated during S phase. Choose Answer ● Given what you now know, what information is most likely to produce the correct answer? ● If a cell starts with 46 chromosomes and replicates in S phase it will produce 46 pairs of sister chromatids. Each sister chromatid contains two strands of DNA so there will be 92 strands of DNA. Reflect on Process ● Did your problem-solving process lead you to the correct answer? If not, where did the process break down or lead you astray? How can you revise your approach to produce a more desirable result? ● This Version 1
45
question asked you to apply your understanding of sister chromatids to calculate how many would be in a human cell after S phase. If you got the correct answer, great job! If you got an incorrect answer, where did the process break down? Did you recall that a normal human cell has 46 chromosomes? Did you understand that each chromosome would produce two sister chromatids after its DNA was replicated? Did you realize that each sister chromatid was an individual DNA molecule?
34) C
Version 1
46
Clarify Question ● What is the key concept addressed by the question? question asks about the kinetochore complex.
● The
● What type of thinking is required? ● You are being asked to apply your knowledge of the kinetochore complex to predict what would happen if a chromosome could not bind to it. Gather Content ● What do you know about the kinetochore complex? What other information is related to the question? ● It allows the chromosome to bind to the mitotic spindle. ● Microtubules do not affect chromosome replication, condensation, or binding to histones. Choose Answer ● Given what you now know, what information is most likely to produce the correct answer? ● Without being able to bind to the kinetochore complex a chromosome cannot bind to the mitotic spindle. Reflect on Process ● Did your problem-solving process lead you to the correct answer? If not, where did the process break down or lead you astray? How can you revise your approach to produce a more desirable result? ● This question asked you to apply your knowledge of the function of the kinetochore complex. If you got the correct answer, great job! If you got an incorrect answer, where did the process break down? Did you recall that the kinetochore complex binds to the mitotic spindle? Did you think that it was involved in replication of the chromosome or binding to histones?
Version 1
47
35) B 36) D 37) A 38) B 39) C 40) C 41) B 42) B 43) A 44) D 45) A
Version 1
48
Clarify Question ● What is the key concept addressed by the question? ● The question asks about the role of microtubules in moving chromosomes during mitosis. ● What type of thinking is required? ● You are being asked to apply your understanding of how chromosomes move by the assembly and disassembly of microtubules to predict where taxol would arrest a cell in mitosis. Gather Content ● What do you know about movement of chromosomes in mitosis? What other information is related to the question? ● Chromosomes move by being pushed and pulled by microtubules. If more tubulin is added to a microtubule it will grow and push a chromosome that it is attached to. If tubulin is removed from a microtubule by disassembly, then any attached chromosome will be pulled. ● In anaphase, telophase and cytokinesis the chromosomes are being pulled to either end of the cell. This will require disassembly of the tubulin molecules attached to the chromosome. Choose Answer ● Given what you now know, what information is most likely to produce the correct answer? ● Even though the later stages do require microtubule disassembly, the movement of the chromosome to the equator of the cell also requires some pushing and pulling of the chromosome. Thus if taxol is added it will affect this movement of the chromosome before the later stages and the cell will arrest before it gets to metaphase. Reflect on Process Version 1
49
● Did your problem-solving process lead you to the correct answer? If not, where did the process break down or lead you astray? How can you revise your approach to produce a more desirable result? ● This question asked you to apply your understanding of how chromosomes move during mitosis to predict the effects of taxol. If you got the correct answer, great job! If you got an incorrect answer, where did the process break down? Did you recall that chromosomes needed to be pushed and pulled by the assembly and disassembly of microtubules to get them to the equator in metaphase?Did you realize that pulling chromosomes requires the disassembly of microtubules?
46) B 47) C
Version 1
50
Clarify Question ● What is the key concept addressed by the question? question asks about the role of metaphase.
● The
● What type of thinking is required? ● You are being asked to analyze different explanations of what would happen to daughter cells if they skipped metaphase. ● What key words does the question contain and what do they mean? ● Metaphase—during metaphase the sister chromatids are attached to the mitotic spindle as they are moved to the equator of the cell Gather Content ● What do you know about metaphase? What other information is related to the question? ● Metaphase is critical for ensuring that all sister chromatids are properly attached to the mitotic spindle. Itinvolves the movement of chromosomes to the center of the dividing cell. Metaphasedoes not affect formation of the nuclear envelope or movement of cytoplasm. Choose Answer ● Given what you now know, what information is most likely to produce the correct answer? ● If this phase were bypassed that would mean that there was no mechanism to check for proper attachment to the mitotic spindle.As a result, it is possible that some chromosomes were not properly attached and therefore, some chromosomes may not properly separate and/or segregate. This would lead to daughter cells with different numbers of chromosomes, which is not a goal of mitosis. Reflect on Process Version 1
51
● Did your problem-solving process lead you to the correct answer? If not, where did the process break down or lead you astray? How can you revise your approach to produce a more desirable result? ● This question asked you to analyze statements about what would happen to a cell if it skipped metaphase and divided. If you got the correct answer, great job! If you got an incorrect answer, where did the process break down? Did you recall that metaphase is where sister chromatids attach to the mitotic spindle? Did you understand that if the spindle does not attach properly that chromosomes will not segregate equally between the two daughter cells.Did you think that metaphase is where the nuclear envelope forms?Did you confuse metaphase and telophase where the cytoplasm separates into the two daughter cells?
48) C 49) B 50) E
Version 1
52
Clarify Question ● What is the key concept addressed by the question? ● The question asks about the role of the Golgi in a step in mitosis. ● What type of thinking is required? ● You are being asked to apply your knowledge of mitosis in a plant to explain the role of the Golgi. Gather Content ● What do you know about the role of the Golgi in mitosis in plants? What other information is related to the question? ● In plants, the cell plate forms from the fusion of Golgi-derived vesicles. ● S is where DNA synthesis occurs, and does not involve the Golgi. ● Metaphase and anaphase involve the movement of chromosomes and do not require the Golgi. Choose Answer ● Given what you now know, what information is most likely to produce the correct answer? ● The drug will prevent the building blocks of the cell plate from being released from the Golgi. As a result cytokinesis is the most likely process to be affected directly by this drug. Reflect on Process ● Did your problem-solving process lead you to the correct answer? If not, where did the process break down or lead you astray? How can you revise your approach to produce a more desirable result? ● This question asked you to apply your understanding of the role of the Golgi in mitosis in plants. If you got the correct answer, great job! If you got an incorrect answer, where did the process break down? Did you recall that the cell plate in plant cells arises from the Golgi? Did you think that anaphase and metaphase require the Golgi? Did you think that with the Version 1
53
Golgi functioning the cell would arrest in G2?
51) A 52) D 53) A
Version 1
54
Clarify Question ● What is the key concept addressed by the question? ● The question asks about the effect of maturation promotion factor on a cell in G1. ● What type of thinking is required? ● You are being asked to analyze different predictions of what would happen if a cell in G1 is fused to a rapidly dividing cell which would express MPF. Gather Content ● What do you know about MPF? What other information is related to the question? ● While MPF will induce a cell to enter mitosis and not S phase, if the DNA has not replicated then the cell will not be able to split the DNA into equal amounts between the two daughter cells. Choose Answer ● Given what you now know, what information is most likely to produce the correct answer? ● In the presence of MPF the G1 cell would go immediately into mitosis. The nuclear envelope would break down, the spindle would assemble and chromosomes would move in the cell. But because the cell has not gone through S phase, the chromosomes would not be replicated, and thus the cell would likely arrest at the spindle checkpoint. Reflect on Process ● Did your problem-solving process lead you to the correct answer? If not, where did the process break down or lead you astray? How can you revise your approach to produce a more desirable result? ● This question asked you to analyze several predictions of what would happen if a rapidly dividing cell was fused with a cell arrested in G1. If you got the correct answer, great job! If you got an incorrect answer, where did Version 1
55
the process break down? Did you recall that MPF in the rapidly dividing cell would cause the cell arrested in G1 to immediately enter mitosis without going through S phase? Did you think that the cell would divide its existing chromosomes without going through S phase? Did you forget that the checkpoints would prevent cells from dividing if the spindles did not form properly?
54) C 55) B
Version 1
56
Clarify Question ● What is the key concept addressed by the question? ● The question asks about the effect on inhibiting a cdc2/mitotic cyclin complex. ● What type of thinking is required? ● You are being asked to apply your knowledge of cyclin/cdc2 interactions to predict what would happen if it was inhibited. Gather Content ● What do you know about cdc2/cyclin complexes? What other information is related to the question? ● Because the question specifies a mitotic cyclin, the cdc2/cyclin complex would normally allow the cell to enter mitosis. Without the complex formation the cell would not enter mitosis, so it could not arrest in a stage of mitosis like prometaphase or metaphase. Separate checkpoints and cyclins regulate movement of the cell through S and G1. Choose Answer ● Given what you now know, what information is most likely to produce the correct answer? ● Without a cdc2/mitotic cyclin complex the cell could not enter mitosis. G2 is the stage before mitosis, so the cell would be stuck in G2. Reflect on Process ● Did your problem-solving process lead you to the correct answer? If not, where did the process break down or lead you astray? How can you revise your approach to produce a more desirable result? ● This question asked you to apply your understanding of the function of the cdc2/mitotic cyclin to predict what would happen if this complex could not form. If you got the correct answer, great job! If you got an incorrect Version 1
57
answer, where did the process break down? Did you recall that the mitotic cyclin would allow the cell to enter mitosis when bound to cdc2?Did you realize that G2 is the stage before mitosis, so the cell would be stuck, or arrest, in G2 if this activity was blocked?
56) B
Version 1
58
Clarify Question ● What is the key concept addressed by the question? ● The question asks about the role of protein synthesis in cell cycle check points. ● What type of thinking is required? ● You are being asked to apply your knowledge of which proteins need to be synthesized for a cell to move through the cell cycle. Gather Content ● What do you know about protein synthesis and the cell cycle? What other information is related to the question? ● Cyclins need to be made new for the cell to move through each stage of the cell cycle. If cyclins cannot be synthesized then the cell will be stuck in that stage of the cell cycle. Choose Answer ● Given what you now know, what information is most likely to produce the correct answer? ● Because the protein synthesis inhibitor is added during S phase the next checkpoint is the G2/M checkpoint. Without newly made cyclin the cell will not be able to pass this checkpoint and will become stuck in G2. Reflect on Process ● Did your problem-solving process lead you to the correct answer? If not, where did the process break down or lead you astray? How can you revise your approach to produce a more desirable result? ● This question asked you to apply your understanding of the synthesis of cyclins to predict what would happen if protein synthesis was blocked. If you got the correct answer, great job! If you got an incorrect answer, where did the process break down? Did you recall that cells in S phase Version 1
59
had already passed the G1/S check point? Did you remember that the G2/M checkpoint would be next? Did you understand that cyclins need to be synthesized for cells to pass through each stage of the cell cycle?
57) A 58) C 59) A 60) B
Version 1
60
Clarify Question ● What is the key concept addressed by the question? question asks about the role of securing in the cell cycle.
● The
● What type of thinking is required? ● You are being asked to analyze several statements predicting what would happen in a cell if APC/C failed to ubiquitinate securin. ● What key words does the question contain and what do they mean? ● Ubiquitinate securin—when ubiquitin is added to a protein, that protein is targeted for degradation Gather Content ● What do you know about securin? What other information is related to the question? ● Securin is an inhibitor of the enzyme separase.Separase will cleave the cohesin complex that holds sister chromatids together.Once cleaved, the sister chromatids can separate in anaphase. ● APC/C ubiquitinates proteins and targets them for destruction.If it cannot ubiquitinate securin, then securin will remain active. Choose Answer ● Given what you now know, what information is most likely to produce the correct answer? ● The cohesin complex needs to be destroyed for the cell to enter anaphase.If securin was not targeted for destruction, it would continue to inhibit separase which would not cleave the cohesion complex preventing the cell from entering anaphase. Reflect on Process ● Did your problem-solving process lead you to the correct answer? If not, where did the process break down or lead you astray? How can you revise your approach to produce a more desirable result? ● This Version 1
61
question asked you to analyze several statements to choose which best explains what would happen if securin remained active in a cell. If you got the correct answer, great job! If you got an incorrect answer, where did the process break down? Did you understand that ubiquitination of securin would lead to its destruction?Did you recall that securin is an inhibitor of separase, and if securin was not degraded separase would remain inactive? Did you predict that if separase remained inactive, cohesin would not be degraded and the sister chromatids could not separate allowing to cell to enter anaphase?
61) D Tumor suppressor genes are like the brakes on the cell cycle. Normally, they prevent cell division in response to problems, such as damaged DNA or incomplete replication. If both copies of a tumor suppressor gene are mutated, that would mean that a cell could not stop dividing in response to normal signals. This is analogous to cutting the brakes on a car. No matter what "signals" emerge (a stop sign, a red light, etc.), it is not possible to stop the car using the brake because there is something wrong with the brakes. 62) A
Version 1
62
Clarify Question ● What is the key concept addressed by the question? ● The question asks about the role of Rb in regulating the cell cycle. ● What type of thinking is required? ● You are being asked to analyze statements about the benefits of inhibiting Rb as a cancer therapy. ● What key words does the question contain and what do they mean? ● Rb—a tumor suppressor protein, it normally halts the cell cycle Gather Content ● What do you know about Rb? What other information is related to the question? ● Rb is a tumor suppressor that halts the cell cycle. Mutations that block Rb activity lead to an increase in the cell cycle and possibly cancer. ● The researchers propose to inhibit the tumor suppressor Rb, this would have the same effect as mutations in Rb that lead to cancer. Choose Answer ● Given what you now know, what information is most likely to produce the correct answer? ● The best answer is that this experiment would not be successful, because Rb is a tumor suppressor you do not want not inactivate it to prevent cancer. Reflect on Process ● Did your problem-solving process lead you to the correct answer? If not, where did the process break down or lead you astray? How can you revise your approach to produce a more desirable result? ● This question asked you to analyze several statements to see which best explains if an experiment would work. If you got the correct answer, Version 1
63
great job! If you got an incorrect answer, where did the process break down? Did you recall that Rb was a tumor suppressor? Did you realize that inhibiting a tumor suppressor would actually increase the risk of cancer?
63) B 64) C
Version 1
64
CHAPTER 11 CHECK ALL THE APPLY. Choose all options that best completes the statement or answers the question. 1) Which cells never divide by meiosis? (Check all that apply.) A) Haploid cells B) Diploid cells C) Somatic cells D) Germ-line cells E) Zygotes
2) A cell biologist examinesa skin cell from a lizard during metaphase of mitosis and determines that 20 chromatids are present. The role of meiosis in this species is to (Check all that apply) A) produce 4 daughter cells that are genetically different from each other. B) produce 4 daughter cells that are genetically different from the original parental cell. C) reduce the number of chromosomes per cell from 10 to 5. D) reduce the number of chromosomes per cell from 20 to 10.
3) A cell biologist examinesa leaf cell from an alfalfa plantduring metaphase of mitosis and determines that 32 chromatids are present. The role of fertilization in this species is to (Check all that apply) A) produce a new cell that has a combination of chromosomes from 2 different parents. B) increase the number of chromosomes per cell from 16 to 32. C) increase the number of chromosomes per cell from 8 to 16. D) combine the chromosomes from 2 haploid cells into a single diploid cell.
4)
Sister chromatids move to opposite poles of the cell during (Check all that apply.)
Version 1
1
A) Anaphase I of meiosis. B) Metaphase I of meiosis. C) Anaphase II of meiosis. D) Anaphase of mitosis.
MULTIPLE CHOICE - Choose the one alternative that best completes the statement or answers the question. 5) Edouard van Beneden proposed that an egg and a sperm, each containing half the complement of chromosomes found in somatic cells, fuse to produce a single cell called a(n) ______. A) zygote B) karyotype C) embryo D) oocyte
6) The process of nuclear division that reduces the number of chromosomes per cell from 2 sets to 1 set is called _____. A) mitosis B) meiosis C) binary fission D) syngamy
7)
What type of cells contain only one set of chromosomes? A) Germ-line B) Somatic C) Diploid D) Haploid
Version 1
2
8) In life cycles that alternate between haploid and diploid stages, fertilization doubles the number of chromosomes per cell while the number of chromosomes is reduced by half by which process? A) Mitosis B) Meiosis C) Binary fission D) Syngamy
9) Homologous chromosomes pair along their length during prophase I of meiosis. While two homologues are paired, genetic exchange may occur between them in a process called ________. A) syngamy B) synapsis C) independentassortment D) crossing over
10)
Compared to asexual reproduction, the main advantage of sexual reproduction is that it A) requires lessenergy. B) increases thegenetic diversity of the offspring. C) can produce morecomplex offspring. D) can produce agreater number of offspring.
11) If there were no suppression of DNA replication between meiotic divisions but cytokinesis proceeded normally, what is the most likely outcome of meiosis?
Version 1
3
A) 4 diploidcells B) 2 diploid cells C) 4 haploid cells D) 2 haploid cells E) 2 diploid cells and 2 haploid cells
12) Evidence of crossing over can often be seen under the light microscope as a structure called a _______. A) kinetochore B) centromere C) chiasma D) centriole
13)
The fusion of a male gamete with a female gamete is called A) syngamy. B) meiosis. C) mitosis. D) recombination. E) synapsis.
14) Diploid organisms use meiosis to produce haploid cells. Meiosis consists of ____ round(s) of nuclear division. A) one B) two C) three D) four E) None of the answer choices are correct.
Version 1
4
15) The pairing of chromosomes along their lengths, which is essential for crossing over, is referred to as A) syngamy. B) synapsis. C) prophase. D) recombination. E) centromere.
16)
The cell produced by the fusion of an egg and a sperm is the A) gamete. B) oocyte. C) zygote. D) germ line cell. E) somatic cell.
17)
The zygote has A) one copy of each chromosome. B) one full haploidcomplement of chromosomes. C) chromosomes identical to those of a sperm cell. D) chromosomes identical to those of an egg cell. E) two copies of each chromosome.
18)
Animal cells that are haploid are called A) gametes. B) muscle cells. C) nerve cells. D) germ-line cells. E) somatic cells.
Version 1
5
19) The point of connection between two sister chromatids, before anaphase II of meiosis separates them, is called the A) centriole B) kinetochore. C) centromere. D) spindleapparatus. E) centrosome.
20) The process that produces new cells that are genetically identical to the original cell is _____. A) meiosis B) mitosis C) crossing over D) independent assortment E) fertilization
21) In animals, the cells that will eventually undergo meiosis to produce gametes are set aside early in development. These are called A) somaticcells. B) germ-linecells. C) sex cells. D) gametophytes. E) reproductivecells.
22)
What event occurs first during meiosis?
Version 1
6
A) Homologous chromosomes separate and are pulledto opposite poles of the cell. B) Sister chromatids separate and are pulledto opposite poles of the cell. C) Chromosomes decondense. D) synapsis of homologous chromosomes E) Pairs of homologous chromosomes align along the equatorial plate.
23)
Chromosomes exchange genetic information by A) fertilization. B) mitosis. C) syngamy. D) DNA replication. E) crossing over.
24)
Crossing over between homologouschromosomes takes place during A) prophase II. B) prophase I. C) interphase II. D) interphase I. E) metaphase II.
25)
The synaptonemal complex is A) a cluster ofmicrotubules at each poleof the cell. B) a network ofmicrotubules that forms the spindle apparatus. C) a network ofproteins that holds homologues together. D) the area wheremicrotubules attach to the centromere of each chromosome. E) a region ofhighly coiled DNA.
26)
At the end of meiosis II, each of the four resulting cells contains
Version 1
7
A) one full set of chromosomes, each with 2 molecules of DNA. B) two full sets of chromosomes, each with 2 molecules of DNA. C) one full set of chromosomes, each with 1 molecule of DNA. D) two full sets of chromosomes, each with 1 molecule of DNA.
27)
Which best describes the process of independent assortment?
A) The way one pair ofhomologues lines up along the metaphase plate does not affect how any otherpair lines up. B) Segregation ofchromosomes during meiosis I is independent of their segregation during meiosisII. C) During synapsis,chromosomes pair up at random. D) Crossing overalong one pair of chromosomes is independent of crossing over along the otherpairs. E) During synapsis,homologues pair independently of each other.
28)
In plants and animals, the zygote develops into an embryo by _____. A) mitosis B) meiosis C) syngamy D) synapsis E) reduction division
29)
The pairing ofhomologous chromosomes is called A) syngamy. B) synapsis. C) alignment. D) independentassortment. E) crossing over.
Version 1
8
30)
Which of the following statements is the most consistant with the process of meiosis?
A) The cells that result from meiosis I are haploid, and each chromosome consists of 1 chromatid. B) The cells that result from meiosis I are haploid, and each chromosome consists of 2 chromatids. C) The cells that result from meiosis II are haploid, and each chromosome consists of 2 chromatids. D) The cells that result from meiosis I are diploid, and each chromosome consists of 2 chromatids. E) The cells that result from meiosis I are diploid, and each chromosome consists of 1 chromatid.
31)
During meiosis, sister chromatids are held togetherby A) a commoncentromere and kinetochore microtubules. B) a commoncentromere and chiasmata. C) a commoncentromere and polar microtubules. D) a common centromere and a cohesin protein complex. E) the kinetochores and a cohesin protein complex.
32)
What immediately follows meiosis I? A) DNA replication B) Prophase II C) Metaphase II D) Synapsis of homologues E) Crossing over of homologues
33)
Which event normally contributes to genetic variation?
Version 1
9
A) Crossing over during meiosis II. B) Fusion of a random egg and sperm to generate a zygote. C) Independent assortment of homologous chromosomes during mitosis. D) Cytokinesis during mitosis.
34) One of a pair of chromosomes with similar genetic information and from different sources like the sperm and egg is called a(n) A) chiasma. B) homologue. C) kinetochore. D) synapsis. E) synaptonemal.
35)
Which structures indicate where crossing over has occurred? A) Chiasmata B) Centromeres C) Kinetochores D) Centrioles E) Spindle fibers
36)
Which structureholds two homologous chromosomes together? A) Centromere B) Kinetochore C) Polar microtubules D) Spindle apparatus E) Synaptonemal complex
Version 1
10
37) During anaphase I, which best represents segregation of the chromatids that make up one pairof homologues? (M represents a maternal chromatid and P represents a paternal chromatid. Assume no crossing over occurs.) A) M and M to one pole; P and P to the other pole B) M and P to one pole; M and P to the other pole C) M to one pole; P to the other pole D) Half of M and half of P to each pole E) The segregation of these chromatids is completely random.
38) You are studying meiosis in an organism where 2n =28. How many chromosomes will bepresent in each cell after meiosis Iis complete but before meiosis II begins? A) 7 B) 14 C) 28 D) 56
39) You are studying meiosis in an organism where 2n = 24. How many chromosomes will each nucleus have after meiosis II is complete? A) 24 B) 12 C) 6 D) 48
40) You are comparing the events of meiosis I in cells from several different organisms. You come across one species in which you do not observe any chiasmata. The best conclusion to make is
Version 1
11
A) there is no crossing over between nonsister chromatids. B) kinetochores ofsister chromatids do not fuse. C) the chromosomepairs will not assort independently. D) chiasmata willform during meiosis II.
41)
When do homologous chromosomes separate from each other during meiosis? A) Prophase I B) Metaphase I C) Anaphase I D) Metaphase II E) Anaphase II
42) The Rec8 protein holds sister chromatids together during meiosis. If an individual has a mutation that prevents degradation of Rec8, this would most likely prevent A) normal segregation during meiosis I. B) normal segregation during meiosis II. C) synapsis of homologous chromosomes. D) crossing over during prophase I.
43) Which of the following is likely to occur if you were to prevent cyclin B from associating with its cyclin-dependent kinase near the end of meiosis I? A) Failure to maintain sister chromatid cohesion at the centromere B) Failure to form initiation complexes necessary for DNA replication to proceed C) Suppression of DNA replication D) Activation ofDNA replication
44) If it were possible to prevent crossing over, what would be the result of inhibiting this event? Version 1
12
A) Sister chromatids would not have cohesion during metaphase I. B) Sister chromatids would not segregate properly during mitosis. C) Homologous chromosomes would not align properly during metaphase I. D) Homologous chromosomes would not segregate properly during mitosis.
45) Some organisms have a life cycle that is not an alternation between haploid and diploid stages. What is an example of an organism that has a life cycle that is not an alternation between diploid and haploid chromosome numbers? A) Dogs B) The bacterium E. coli C) Alfalfa plants D) The mold N. crassa
46) In 95% of cases of Down's syndrome, there is one extra chromosome (number 21) in every cell. This aneuploid condition is most likely the result of A) failure to suppress DNA replication between meiosis I and meiosis II. B) failure of the chromosomes to assort independently during meiosis. C) failure of 1 homologous pair to segregate during meiosis. D) failure of 2 homologous pairs to segregate during meiosis. E) failure of the cytoplasm to divide at the end of meiosis II.
47)
Why does sexual reproduction require both meiosis and syngamy?
Version 1
13
A) The process of meiosis results in the production of gametes in which the number of chromosomes remains the same. During syngamy, two gametes fuse to form a new cell, and the number of chromosomes is restored to the full amount. Therefore, by coupling meiosis and syngamy, the organism ensures that the proper number of chromosomes will be maintained. B) The process of meiosis results in the production of gametes in which the number of chromosomes is reduced by half. During syngamy, two gametes fuse to form a new cell, and the number of chromosomes is restored to the full amount. Therefore, by coupling meiosis and syngamy, the organism ensures that the proper number of chromosomes will be maintained. C) The process of meiosis results in the production of gametes in which the number of chromosomes is doubled. During syngamy, gametes are reduced by half, and the number of chromosomes is restored to the full amount. Therefore, by coupling meiosis and syngamy, the organism ensures that the proper number of chromosomes will be maintained.
48) The most common form of gene therapy involves inserting a normal gene into cells that contain a defective version of the gene. In order to use gene therapy to prevent a man from passing a defective gene on to future generations, you should try to insert normal copies of the gene into A) blood cells. B) germ-line cells. C) somatic cells in the testes. D) bone marrow cells.
49) A geneticist examinesa somatic cell from a fly during metaphase of mitosis and determines that 16 chromatids are present. If a germ-line cell from this species divides by meiosis, then at the end of meiosis I (including the first cytokinesis) each cell will contain A) 8 chromosomes with 8 DNA molecules. B) 8 chromosomes with 16 DNA molecules. C) 4 chromosomes with 4 DNA molecules. D) 4 chromosomes with 8 DNA molecules.
Version 1
14
50) Nondisjunction is the failure of homologous chromosomes to separate during meiosis I, or the failure of sister chromatids to separate during meiosis II or mitosis.As a result, both homologous chromosomes or both sister chromatids migrate to the same pole of the cell.This produces daughter cells with an imbalance of chromosomes.A cell biologist examines the final products of meiosis in an earthworm (2n = 36) and finds 2 cells with 20 chromosomes, and 2 cells with 16 chromosomes.Most likely this was because A) 2 pairs of sister chromatids failed to separate during meiosis II. B) 1 pair of sister chromatids failed to separate during meiosis II. C) 2 pairs of homologous chromosomesfailed to separate during meiosis I. D) 1 pair of homologous chromosomesfailed to separate during meiosis I.
51) Nondisjunction is the failure of homologous chromosomes to separate during meiosis I, or the failure of sister chromatids to separate during meiosis II,or during mitosis.As a result, both homologous chromosomes or both sister chromatids migrate to the same pole of the cell.This produces daughter cells with an imbalance of chromosomes.If 18 pairs of sister chromatids segregate normally during meiosis II in cats (n = 19) but we have nondisjunction of 1 pair, then at the end of meiosis II we will have A) 3 cells with 20 chromosomes and 1 cell with 18. B) 2 cells with 20 chromosomes and 2 cells with 18. C) 2 cells with 19 chromosomes, 1 with 20, and 1 with 18. D) 3 cells with 18 chromosomes and 1 cell with 20.
52) Meiosis results in a reassortment of maternal chromosomes (inherited from the mother) and paternal chromosomes (inherited from the father). If n = 4 for a given species, and ignoring the effects of crossing over, what is the probability that a gamete will receive only paternal chromosomes? A) 1/2 B) 1/4 C) 1/8 D) 1/16
53) A cell in G 2before meiosis begins, compared with one of the four cells produced at the end of meiosis II, has Version 1
15
A) twice as much DNAand twice as many chromosomes. B) four times asmuch DNA and twice as many chromosomes. C) twice as much DNAbut the same number of chromosomes. D) four times asmuch DNA and four times as many chromosomes. E) twice as much DNAand half as many chromosomes.
54) In meiosis, sister kinetochores are attached to the same pole of the cell during meiosis I, and sister chromatid cohesion is released during anaphase II. What would be the likely result if sister kinetochores were attached to different poles of the cell during meiosis I and sister chromatid cohesion was released during anaphase I? A) Sister chromatids would migrate to opposite poles during anaphase I. B) Sister chromatids would migrate to opposite poles during anaphase II. C) Sister chromatids would migrate to the samepole during anaphase I. D) Sister chromatids would migrate to the samepole during anaphase II.
FILL IN THE BLANK. Write the word or phrase that best completes each statement or answers the question. 55) If a germ-line cell from an owl contains 8 picograms of DNA during G1 of interphase, how many picograms of DNA would be present in each cell during prophase I of meiosis? (Enter the number only, not the units.)
56) If a somatic cell from a cat contains 40 picograms of DNA during G 2 of interphase, how many picograms of DNA would be present in each cell during metaphase II of meiosis? (Enter the number only, not the units.)
57) If a germ-line cell from a salamander contains 10 picograms of DNA during G1 of interphase, how many picograms of DNA would be present in each gamete produced by this species?(Enter the number only, not the units.)
58) How many tetrads are present in a single elephant cell (2n = 56) during metaphase I of meiosis? Version 1
16
59) A cell biologist examinesa diploid cell from a particular species of Ragoletis during metaphase of mitosis and determines that 8 centromeres are present. Based on this finding, how many centromeres should be present in a single cell from this species during anaphase II of meiosis?
60) A cell biologist examinesa diploid cell from a particular species ofbutterfly during prometaphase of mitosis and determines that 10 centromeres are present. Based on this finding, how many chromatids should be present in a single cell from this species in metaphase I of meiosis?
Version 1
17
Answer Key Test name: Chap 11_3e 1) [A, C, E] 2) [A, B, C]
Version 1
18
Clarify Question ● What is the key concept addressed by the question? ● The question asks about the number of chromosomes in mitosis and meiosis. ● What type of thinking is required? ● You are being asked to use your knowledge of mitosis and meiosis to analyze statements about the products of meiosis. Gather Content ● What do you know about meiosis? What other information is related to the question? ● If there are 20 chromatids in a cell, this means the diploid cell has 10 chromosomes. These will separate during meiosis to form gametes that are haploid, each containing 5 chromosomes. Choose Answer ● Given what you now know, what information is most likely to produce the correct answer? ● Due to independent assortment and crossing over, these gametes will be genetically different from each other. Because the gametes are haploid and the parental cell is diploid, these cells will also be genetically different from each other. Furthermore, because the parental cell has 10 chromosomes, after meiosis it will be haploid with 5 chromosomes. Reflect on Process ● Did your problem-solving process lead you to the correct answer? If not, where did the process break down or lead you astray? How can you revise your approach to produce a more desirable result? ● This question asked you to analyze statements about meiosis and choose which are correct. If you got the correct answer, great job! If you got an incorrect answer, where did the process break down? Did you recall that Version 1
19
a cell with 20 chromatids would have 10 chromosomes? Did you understand that during meiosis these cells would drop from 10 to 5 chromosomes? Did you realize that after meiosis the gametes would be genetically distinct from each other and from the parental diploid cell?
3) [A, C, D]
Version 1
20
Clarify Question ● What is the key concept addressed by the question? question asks about what happens during fertilization.
● The
● What type of thinking is required? ● You are being asked to analyze statements about fertilization to choose which is correct. Gather Content ● What do you know about fertilization? What other information is related to the question? ● In fertilization, two haploid gametes fuse to form a diploid fertilized egg. If a cell has 32 chromatids, these chromatids are paired as a chromosome. So the diploid number for this cell will be 16 chromosomes. If this cell goes through meiosis forming haploid gametes, each gamete will have eight chromosomes. Choose Answer ● Given what you now know, what information is most likely to produce the correct answer? ● In this case, the haploid gametes each have eight chromosomes, so they would fuse to form a cell with 16 chromosomes. Because these gametes contain chromosomes from two different individuals, the chromosomes in the fertilized cell will be a combination of those from the two different parents. Reflect on Process ● Did your problem-solving process lead you to the correct answer? If not, where did the process break down or lead you astray? How can you revise your approach to produce a more desirable result? ● This question asked you to analyze statements about fertilization, to explain properties of the cell that is formed. If you got the correct answer, great job! If you got an incorrect answer, where did the process break down? Did you recall that if a cell has 32 chromatids its diploid number is 16 Version 1
21
chromosomes? Could you calculate that the haploid gametes would have half the number of chromosomes or eight?
4) [C, D] 5) A 6) B 7) D 8) B 9) D 10) B 11) A 12) C 13) A 14) B 15) B 16) C 17) E 18) A 19) C 20) B 21) B 22) D 23) E 24) B 25) C 26) C 27) A 28) A 29) B Version 1
22
30) B 31) D 32) B 33) B 34) B 35) A 36) E 37) A 38) B
Version 1
23
Clarify Question ● What is the key concept addressed by the question? ● The question asks about the number of chromosomes present after meiosis I. ● What type of thinking is required? ● You are being asked to apply your knowledge of meiosis to calculate how many chromosomes will be in a cell after meiosis I. Gather Content ● What do you know about meiosis? What other information is related to the question? ● In meiosis I, the chromosome pairs line up on the equator and separate. Then in meiosis II, the sister chromatids separate. ● If2n = 28 in a cell,the only two possible answers are n = 14 or 2n = 28 for haploid or diploid cells, respectively. If a cell had 7 chromosomes, that would be less than half the chromosomes of a haploid cell. If a cell had 56 chromosomes, the cell would be tetraploid. Choose Answer ● Given what you now know, what information is most likely to produce the correct answer? ● If a cell going into meiosis has 28 chromosomes, these will form 14 pairs that line up during meiosis I and separate, forming two cells each with 14 chromosomes.After meiosis II, the sister chromatids would separate, forming four cells that are each haploid. Reflect on Process ● Did your problem-solving process lead you to the correct answer? If not, where did the process break down or lead you astray? How can you revise your approach to produce a more desirable result? ● This question asked you to apply your understanding of meiosis to predict Version 1
24
how many chromosomes would be in a cell after meiosis I. If you got the correct answer, great job! If you got an incorrect answer, where did the process break down? Did you recall that the chromosome pairs separate during meiosis I? Did you think that the chromosome with two sister chromatids was actually two separate chromosomes?
39) B
Version 1
25
Clarify Question ● What is the key concept addressed by the question? ● The question asks about the number of chromosomes in a cell after meiosis. ● What type of thinking is required? ● You are being asked to apply your knowledge of meiosis to predict the number of chromosomes in a cell. Gather Content ● What do you know about meiosis? What other information is related to the question? ● In meiosis I, the chromosome pairs line up on the equator and separate. Then in meiosis II, the sister chromatids separate. ● If 2n = 24 in a cell,the only two possible answers are n = 12 or 2n = 24 for haploid or diploid cells, respectively. If a cell had 6 chromosomes, that would be less than half the chromosomes of a haploid cell. If a cell had 48 chromosomes, the cell would be tetraploid. Choose Answer ● Given what you now know, what information is most likely to produce the correct answer? ● If a cell going into meiosis has 24 chromosomes, these will form 12 pairs that line up during meiosis I and separate, forming two cells each with 12 chromosomes. After meiosis II, the sister chromatids would separate, forming four cells that are each haploid, still with 12 chromosomes. Reflect on Process ● Did your problem-solving process lead you to the correct answer? If not, where did the process break down or lead you astray? How can you revise your approach to produce a more desirable result? ● This question asked you to apply your understanding of meiosis to predict Version 1
26
how many chromosomes would be in a cell after meiosis II. If you got the correct answer, great job! If you got an incorrect answer, where did the process break down? Did you recall that meiosis forms haploid cells? Did you confuse meiosis with mitosis and think that meiosis forms diploid cells?
40) A 41) C 42) B
Version 1
27
Clarify Question ● What is the key concept addressed by the question? ● The question asks about the role of Rec8 on chromatid separation. ● What type of thinking is required? ● You are being asked to apply your knowledge of chromatids to predict what would happen if they could not separate. Gather Content ● What do you know about meiosis? What other information is related to the question? ● In meiosis, there are two stages. in meiosis I, the chromosome pairs line up on the equator and separate. Then in meiosis II, the sister chromatids separate. If the sister chromatids cannot separate, this would happen in meiosis II and not meiosis I. Choose Answer ● Given what you now know, what information is most likely to produce the correct answer? ● The best answer is that without being able to degrade Rec8 and cohesin the chromosomes will not be able to segregate normally during meiosis II. Reflect on Process ● Did your problem-solving process lead you to the correct answer? If not, where did the process break down or lead you astray? How can you revise your approach to produce a more desirable result? ● This question asked you to apply your understanding of the role of Rec8 and cohesin to predict what would happen if it was inhibited during meiosis. If you got the correct answer, great job! If you got an incorrect answer, where did the process break down? Did you recall that Rec8 was part of cohesin and holds sister chromatids together until it is degraded in meiosis II?Did you think that the sister chromatids separate in meiosis Version 1
28
I?Did you think that Rec8 was involved in crossing over or the synapse instead of being part of cohesin?
43) D
Version 1
29
Clarify Question ● What is the key concept addressed by the question? ● The question asks about the effect of loss of cyclin on a kinase activity. ● What type of thinking is required? ● You are being asked to analyze several statements that predict what would happen if a cyclin cannot bind a kinase. Gather Content ● What do you know about cyclins and cell check points? What other information is related to the question? ● There are three main check points in the cell cycle. G1/S, G2/M, and one in mitosis/meiosis to check for proper spindle formation. ● During mitosis, the destruction of mitotic cyclin is necessary for a cell to enter another division cycle. The result of this maintenance of cyclin B between meiotic divisions in germ-line cells is the failure to form initiation complexes necessary for DNA replication to proceed. This failure to form initiation complexes appears to be critical to suppressing DNA replication. Choose Answer ● Given what you now know, what information is most likely to produce the correct answer? ● Since cyclin B suppresses a cell from replicating DNA, loss of cyclin B would lead to an increase in DNA replication. Reflect on Process ● Did your problem-solving process lead you to the correct answer? If not, where did the process break down or lead you astray? How can you revise your approach to produce a more desirable result? ● This question asked you to analyze different statements explaining what Version 1
30
would happen if cyclin B was inhibited in a cell. If you got the correct answer, great job! If you got an incorrect answer, where did the process break down? Did you recall that cyclin B needs to be degraded for a cell to replicate DNA?Did you realize that the cyclin B would not affect the steps in chromosome separation of meiosis?
44) C 45) B
Version 1
31
Clarify Question ● What is the key concept addressed by the question? ● The question asks about species that have both haploid and diploid stages in their life cycle. ● What type of thinking is required? ● You are being asked to apply your knowledge of haploid and diploid stages in a life cycle. Gather Content ● What do you know about haploid and diploid stages in a life cycle.? What other information is related to the question? ● The life cycles of all sexually reproducing organisms follow a pattern of alternation between diploid and haploid chromosome numbers, but there is variation in the pattern’s timing. Most plants and some algae alternate between a multicellular haploid phase and a multicellular diploid phase. In most animals, by contrast, the diploid state dominates. The zygote first undergoes mitosis to produce diploid cells. Then, later in the life cycle, some of these diploid cells undergo meiosis to produce haploid gametes.Bacteria and other prokaryotes do not exist as diploids are reproduce asexually. Choose Answer ● Given what you now know, what information is most likely to produce the correct answer? ● Plants, animals and fungi all have both haploid and diploid stages in their life cycles.Only bacteria are always haploid. Reflect on Process ● Did your problem-solving process lead you to the correct answer? If not, where did the process break down or lead you astray? How can you revise your approach to produce a more desirable result? ● This Version 1
32
question asked you to apply your understanding of haploid and diploid stages in a life cycle. If you got the correct answer, great job! If you got an incorrect answer, where did the process break down? Did you recall that bacteria are always haploid?Did you understand that molds are fungi and have a diploid stage in their life cycle?
46) C
Version 1
33
Clarify Question ● What is the key concept addressed by the question? question asks about origin of a trisomy.
● The
● What type of thinking is required? ● You are being asked to apply your knowledge of meiosis to predict when a third copy of a chromosome would occur. Gather Content ● What do you know about meiosis? What other information is related to the question? ● During meiosis chromosome pairs separate during meiosis I. If these do not segregate, then a daughter cell would get an extra chromosome. The presence of an extra chromosome is due to a problem with segregation, not DNA replication or independent assortment, which would affect more than one chromosome. Choose Answer ● Given what you now know, what information is most likely to produce the correct answer? ● The best answer is that if one pair of chromosome does not segregate during meiosis then one daughter cell will obtain two copies of that chromosome.When this gamete fuses with a normal gamete that contains one copy of each chromosome, there will be three copies of that chromosome. Reflect on Process ● Did your problem-solving process lead you to the correct answer? If not, where did the process break down or lead you astray? How can you revise your approach to produce a more desirable result? ● This question asked you to apply your understanding of meiosis to explain how a trisomy arises. If you got the correct answer, great job! If you got an incorrect answer, where did the process break down? Did you recall Version 1
34
that homologous chromosome pairs separate during meiosis? Did you understand that if this does not happen one gamete will end up with an extra copy of that chromosome while the other will have no chromosome?Did you think that the extra chromosome arose because of increased DNA replication or a failure of independent assortment?
47) B 48) B
Version 1
35
Clarify Question ● What is the key concept addressed by the question? ● The question asks about the transfer of genetic information to the next generation. ● What type of thinking is required? ● You are being asked to apply your knowledge of fertilization to explain which types of cells produce gametes. Gather Content ● What do you know about fertilization? What other information is related to the question? ● Gametes pass genetic information from one generation to the next. If you want to correct a defective gene so it cannot be passed on, it would need to be done in gametes. Gametes are produced by meiosis in specific cells in the body, notin somatic cells like blood cells or bone marrow cells. Choose Answer ● Given what you now know, what information is most likely to produce the correct answer? ● Gametes are produced in germ-line cells in the testes orovaries. If gene therapy is done on these cells, the gametes they produce would contain the fixed gene preventingthe inheritance of the defective gene. Reflect on Process ● Did your problem-solving process lead you to the correct answer? If not, where did the process break down or lead you astray? How can you revise your approach to produce a more desirable result? ● This question asked you to apply your understanding of gamete production to predict which cells would pass genetic changes on to the next generation. If you got the correct answer, great job! If you got an Version 1
36
incorrect answer, where did the process break down? Did you recall that germ-line cells produce gametes? Did you realize that the changes would need to be made in the gametes to be passed on to the next generation? Did you think that changes in somatic cells like blood and bone marrow cells could be passed on to the next generation?
49) D
Version 1
37
Clarify Question ● What is the key concept addressed by the question? ● The question asks about the number of chromosomes present in a cell after meiosis I. ● What type of thinking is required? ● You are being asked to calculate the number of chromosomes present in a cell after meiosis. Gather Content ● What do you know about meiosis? What other information is related to the question? ● If a diploid cell has 16 chromatids, these will consist of 8 pairs of sister chromatids which each make a chromosome. ● In meiosis I, chromosome pairs align and then separate. So after meiosis I, the cells will be diploid, containing one copy of each chromosome where each chromosome consists of two sister chromatids. Choose Answer ● Given what you now know, what information is most likely to produce the correct answer? ● If the cell has 16 chromatids during metaphase of mitosis, the diploid number of the cell is 8 chromosomes. After meiosis I, the cells will now be haploid with 4 chromosomes each. ● In metaphase I of meiosis, the chromosome pairsalign. As each chromosome contains a pair of sister chromatids, the pairsforms a tetrad. After meiosis I, the cells will now be haploid, but each chromosome will contain two chromatids for each chromosome,so the 4 chromosomes will contain 8 DNA molecules. Reflect on Process ● Did your problem-solving process lead you to the correct answer? If not, where did the process break down or lead you astray? How can Version 1
38
you revise your approach to produce a more desirable result? ● This question asked you to analyze a problem to calculate the number of chromosomes and DNA molecules in a cell after meiosis I. If you got the correct answer, great job! If you got an incorrect answer, where did the process break down? Did you understand that the diploid number would be 8 chromosomes? Could you calculate that after meiosis I the cell would now be haploid with 4 chromosomes? Did you understand that each chromosome would contain 2 sister chromatids, meaning the cell has 8 DNA molecules?
50) C
Version 1
39
Clarify Question ● What is the key concept addressed by the question? ● The question asks about the effect of nondisjunction on the number of chromosomes in a gamete. ● What type of thinking is required? ● You are being asked to analyze different statements about nondisjunction to explain how many chromosomes would be present in a gamete. Gather Content ● What do you know about nondisjunction? What other information is related to the question? ● In nondisjunction, chromosomes fail to segregate, leading to gametes with extra or missing chromosomes. If n = 18, and the haploid gametes have either 16 or 20 chromosomes, then both cells either gained or lost twoadditional chromosomes. ● If the nondisjunction happens during meiosis I,the two daughter cells will have either missing or extra chromosomes. When these daughter cells go through a second division in meiosis II,both of their daughter cells would inherit the same number of chromosomes. So the four daughter cells would all contain extra or missing chromosomes. ● If the nondisjunction happens during meiosis II, one of the original daughter cells will divide normally producing two new daughter cells with the correct number of chromosomes. The cell that goes through nondisjunction will produce two daughter cells with abnormal chromosomes. So in this case, two gametes would end up with a normal number of chromosomes and two would have extra or missing chromosomes. Choose Answer ● Given what you now know, what information is most likely to produce the correct answer? ● Because the original diploid cell Version 1
40
contains 2n = 36 chromosomes, the haploid cell would end up with 18 chromosomes.Because there are twocells with 20 chromosomes, and twocells with 16 chromosomes the nondisjunction must have happened in the first round of meiosis. Nondisjunction would have led toonecell with 20 chromosomes and onewith 16, eachchromosomes consisting of2 sister chromatids. In meiosis II the twocells would divide again producing twohaploid cells with 20 chromosomes and twowith 16 chromosomes. Reflect on Process ● Did your problem-solving process lead you to the correct answer? If not, where did the process break down or lead you astray? How can you revise your approach to produce a more desirable result? ● This question asked you to analyze the results of an experiment to predict when nondisjunction occurred. If you got the correct answer, great job! If you got an incorrect answer, where did the process break down? Did you recall -that if nondisjunction occurs in the first round of meiosis I the twodaughter cells would then divide producing identical gametes? Did you realize that if nondisjunction occurred in meiosis II that 1 daughter cell would produce twogametes with the correct number of chromosomes and the second undergoing nondisjunction would produce gametes with extra or missing chromosomes?
51) C
Version 1
41
Clarify Question ● What is the key concept addressed by the question? ● The question asks about the effect of nondisjunction on the number of chromosomes in a gamete. ● What type of thinking is required? ● You are being asked to analyze different statements about nondisjunction to explain how many chromosomes would be present in a gamete. Gather Content ● What do you know about nondisjunction? What other information is related to the question? ● In nondisjunction, chromosomes fail to segregate, leading to gametes with extra or missing chromosomes. If n = 19 then a normally segregating cell would have 19 chromosomes and those with an extra or missing chromosome would have 20 or 18 chromosomes respectively. ● If the nondisjunction happens during meiosis I, the two daughter cells will have either missing or extra chromosomes. When these daughter cells go through a second division in meiosis II, both of their daughter cells would inherit the same number of chromosomes and the four daughter cells would all contain extra or missing chromosomes. ● If the nondisjunction happens during meiosis II, one of the original daughter cells will divide normally producing two new daughter cells with the correct number of chromosomes. The cell that goes through nondisjunction will produce two daughter cells with abnormal chromosomes. Choose Answer ● Given what you now know, what information is most likely to produce the correct answer? ● Because the normal haploid cell has 19 chromosomes and the nondisjunction happens in meiosis II, after Version 1
42
meiosis I both daughter cells will still have 19 chromosomes with chromatid pairs. After meiosis II one cell would go through normal separation of the sister chromatids producing two haploid cells with 19 chromosomes.For the second cell that goes through nondisjunction, 18 of the sister chromatids would separate normally while one sister chromatid pair would not separate andone gamete would end up with 18 total chromosomes whilethe other would have an extra chromosome for20 total. Reflect on Process ● Did your problem-solving process lead you to the correct answer? If not, where did the process break down or lead you astray? How can you revise your approach to produce a more desirable result? ● This question asked you to analyze the results of an experiment to predict how many chromosomes a cell would have when nondisjunction occurred. If you got the correct answer, great job! If you got an incorrect answer, where did the process break down? Did you recall that if nondisjunction occurs in meiosis II half of the gametes will have the normal number of chromosomes and half will have extra or missing chromosomes? Did you think that three of the cells would have the normal number of chromosomes and only one would end up with an abnormal number?
52) D
Version 1
43
Clarify Question ● What is the key concept addressed by the question? ● The question asks about the probabilities in independent assortment. ● What type of thinking is required? ● You are being asked to apply your knowledge of independent assortment to calculate the probability of all chromosomes in a gamete coming from one parent. Gather Content ● What do you know about independent assortment? What other information is related to the question? ● For each chromosome pair lining up during metaphase I of meiosis there is a 50:50 probability for it to move into one new cell or the other. The probability of independent assortment is like flipping a coin. If the odds of a head is 50% or ½, then the probability of two heads is ½ x ½ or ¼. Choose Answer ● Given what you now know, what information is most likely to produce the correct answer? ● Following the coin tossing analogy,if a cell has 4 chromosomes that segregate independently the probability that the chromosomes from the father would all move to one cell is ½ x ½ x ½ x ½ = 1/16. Reflect on Process ● Did your problem-solving process lead you to the correct answer? If not, where did the process break down or lead you astray? How can you revise your approach to produce a more desirable result? ● This question asked you to apply your understanding of independent assortment. If you got the correct answer, great job! If you got an incorrect answer, where did the process break down? Did you recall that the probability of two chromosomes segregate together is the probability Version 1
44
of each event separately multiplied?Did you think that the probabilities would be added?
53) B 54) A
Version 1
45
Clarify Question ● What is the key concept addressed by the question? ● The question asks about separation of sister chromatids in meiosis. ● What type of thinking is required? ● You are being asked to apply your knowledge of sister chromatids to predict what would happen if they separated in anaphase I of meiosis. Gather Content ● What do you know about separation of sister chromatids? What other information is related to the question? ● Normally sister chromatids remain attached during meiosis I and anaphase I. The sister chromatids then separate during anaphase II of meiosis II. Choose Answer ● Given what you now know, what information is most likely to produce the correct answer? ● The cohesin being released in anaphase I and kinetochore microtubules attached to opposite poles of the cell would result in the sister chromatids migrating to opposite poles in anaphase I. Reflect on Process ● Did your problem-solving process lead you to the correct answer? If not, where did the process break down or lead you astray? How can you revise your approach to produce a more desirable result? ● This question asked you to apply your understanding of the separation of sister chromatids to predict what would happen if this occurred in anaphase I. If you got the correct answer, great job! If you got an incorrect answer, where did the process break down? Did you recall that once cohesin is released the sister chromatids separate? Did you realize that if the kinetochore microtubules attach to different poles of the cell, Version 1
46
the sister chromatids will migrate to opposite poles.
55) 16
Version 1
47
Clarify Question ● What is the key concept addressed by the question? ● The question asks about the amount of DNA in a cell at different stages of meiosis. ● What type of thinking is required? ● You are being asked to analyze the amount of DNA present at different stages of meiosis. Gather Content ● What do you know about meiosis? What other information is related to the question? ● After G1 a cell enters S phase where the chromosomes replicate. The chromosomes then condense in prophase I but do not divide until anaphase of meiosis. The chromosomes replicate but do not divide, so the cells will have twice the mass of DNA as in G1. Choose Answer ● Given what you now know, what information is most likely to produce the correct answer? ● Because the DNA has duplicated, but not divided again, the cells will have twice as much DNA. So, if they had 8 picograms to start with they will now have 16 picograms. Reflect on Process ● Did your problem-solving process lead you to the correct answer? If not, where did the process break down or lead you astray? How can you revise your approach to produce a more desirable result? ● This question asked you to calculate the amount of DNA in a cell at a specific stage in meiosis. If you got the correct answer, great job! If you got an incorrect answer, where did the process break down? Did you recall that after G1 the cells will duplicate their DNA? Did you realize that the cells in prophase I will not have divided their DNA yet?
Version 1
48
56) 20
Version 1
49
Clarify Question ● What is the key concept addressed by the question? ● The question asks about the amount of DNA in a cell at different stages of meiosis. ● What type of thinking is required? ● You are being asked to analyze the amount of DNA present at different stages of meiosis. Gather Content ● What do you know about meiosis? What other information is related to the question? ● After G2 a cellgoes through meiosis I and the amount of DNA in each cell is halved. In metaphase II the cells do notdivideagain. Choose Answer ● Given what you now know, what information is most likely to produce the correct answer? ● The DNA has already duplicated in G2 and will go through one division in meiosis.So, if a cell had 40 picograms of DNA in G2,after one division the cells will each have 20 picograms. Reflect on Process ● Did your problem-solving process lead you to the correct answer? If not, where did the process break down or lead you astray? How can you revise your approach to produce a more desirable result? ● This question asked you to calculate the amount of DNA in a cell at a specific stage in meiosis. If you got the correct answer, great job! If you got an incorrect answer, where did the process break down? Did you recall that after G2 the cells had duplicated their DNA? Did you realize that the cells in metaphase II will have divided their DNA once?
Version 1
50
57) 5
Version 1
51
Clarify Question ● What is the key concept addressed by the question? ● The question asks about the amount of DNA in a cell at different stages of meiosis. ● What type of thinking is required? ● You are being asked to analyze the amount of DNA present at different stages of meiosis. Gather Content ● What do you know about meiosis? What other information is related to the question? ● After G1, a cell enters S phase and the chromosomes replicate. The chromosomes then divide twice through meiosis,so the mass of DNA will be half as much as in G1. Choose Answer ● Given what you now know, what information is most likely to produce the correct answer? ● Because the DNA has duplicated after S phase the cell will have 20 picograms of DNA. After meiosis I, the two daughter cells will have 10 picograms of DNA. After meiosis II the four gametes will have 5 picograms of DNA. Reflect on Process ● Did your problem-solving process lead you to the correct answer? If not, where did the process break down or lead you astray? How can you revise your approach to produce a more desirable result? ● This question asked you to calculate the amount of DNA in a cell at a specific stage in meiosis. If you got the correct answer, great job! If you got an incorrect answer, where did the process break down? Did you recall that after G1 the cells will duplicate their DNA? Did you realize that after meiosis two rounds of division would occur?
Version 1
52
58) 28
Version 1
53
Clarify Question ● What is the key concept addressed by the question? question asks about tetrads.
● The
● What type of thinking is required? ● You are being asked to apply your knowledge of tetrads to calculate how many will be present in a cell during metaphase I of meiosis. Gather Content ● What do you know about tetrads? What other information is related to the question? ● If a cell is diploid, it has pairs of chromosomes equal to half the number of chromosomes in the diploid cell. Choose Answer ● Given what you now know, what information is most likely to produce the correct answer? ● If a cell has a 2n = 56, then it has 28 pairs of chromosomes. During metaphase Iof meiosis these pairs form tetrads. Thus, if there are 28 pairs of chromosomes, there will be 28 tetrads. Reflect on Process ● Did your problem-solving process lead you to the correct answer? If not, where did the process break down or lead you astray? How can you revise your approach to produce a more desirable result? ● This question asked you to apply your understanding of tetrads to calculate the number in a cell during metaphase I of meiosis. If you got the correct answer, great job! If you got an incorrect answer, where did the process break down? Did you recall that tetrads are formed from pairs of homologous chromosomes? Did you realize that if a diploid cell had 56 chromosomes that it would have 28 pairs? Did you think that each Version 1
54
chromosome would form a tetrad by itself? Did you think that a tetrad contained four chromosomes?
59) 4
Version 1
55
Clarify Question ● What is the key concept addressed by the question? ● The question asks about the number of centromeres at different stages of the cell cycle. ● What type of thinking is required? ● You are being asked to analyze an experiment to predict how many centromeres would be present. Gather Content ● What do you know about centromeres? What other information is related to the question? ● Centromeres are at the center of a chromosome. They pair up during metaphase.So if a cell has 8 centromeres in its diploid state, it has 8 chromosomes. In anaphase II, the cells will now be haploid and contain half as many chromosomes.Each chromosome still contains one centromere. Choose Answer ● Given what you now know, what information is most likely to produce the correct answer? ● If the diploid number is 8, the haploid number will be 4 chromosomes, each with one centromere. Thus, the cell will have 4 centromeres. Reflect on Process ● Did your problem-solving process lead you to the correct answer? If not, where did the process break down or lead you astray? How can you revise your approach to produce a more desirable result? ● This question asked you to analyze the results of an experiment to calculate the number of centromeres on chromosomes in cells in anaphase II of meiosis. If you got the correct answer, great job! If you got an incorrect answer, where did the process break down? Did you recall that each Version 1
56
chromosome contains a centromere?Did you realize that in anaphase II of meiosis the cells would be haploid?
60) 20
Version 1
57
Clarify Question ● What is the key concept addressed by the question? ● The question asks about the number of chromatids in a cell during meiosis. ● What type of thinking is required? ● You are being asked to analyze an experiment to predict the number of chromatids present in metaphase I of meiosis. Gather Content ● What do you know about the number of chromatids in a cell? What other information is related to the question? ● Sister chromatids are formed after DNA replicates. Pairs of sister chromatids are chromosomes, and pairs of homologous chromosomes pair up in metaphase I of meiosis. Choose Answer ● Given what you now know, what information is most likely to produce the correct answer? ● If a cell has 10 centromeres in prometaphase, it has 10 chromosomes. The cell is still diploid, so 2n=10. In metaphase Iof meiosis there will still be 10 chromosomes. Each chromosome contains two sister chromatids, so there will be 20 chromatids in total. Reflect on Process ● Did your problem-solving process lead you to the correct answer? If not, where did the process break down or lead you astray? How can you revise your approach to produce a more desirable result? ● This question asked you to analyze the results of an experiment to calculate how many chromatids are present in metaphase I. If you got the correct answer, great job! If you got an incorrect answer, where did the process break down? Did you understand that the cell would still be diploid in Version 1
58
metaphase I? Did you realize that each chromosome would have a centromere? Did you understand that each chromosome would have two sister chromatids?
Version 1
59
CHAPTER 12 CHECK ALL THE APPLY. Choose all options that best completes the statement or answers the question. 1) Mendel demonstrated which of the following about inherited traits? Check all that apply. A) Traits can behidden in some generations, but subsequently reappear unchanged in futuregenerations. B) Traits segregateamong the offspring of a cross. C) Certain traits aremore likely to appear than their alternatives. D) Some traits showblending inheritance. E) Parents that bothhave the same trait can have offspring with an alternative trait.
2) Knight followed up on attempts of English farmers to improve varieties of agriculture with his studies on garden peas. When he crossed two true-breeding varieties, he found all of the following: Check all that apply. A) All the F 1 offspring had the dominant trait. B) If both parents have the same trait, then all of their offspring will also have that trait. C) Some F 2 offspring had the dominant trait and some had the recessive trait. D) Among the F 2 offspring, more had the dominant traitand fewer had the recessive trait.
3) Mendel chose the garden pea for his work on inheritance for which of the following reasons? Check all that apply. A) Some traits in pea plants are controlled by one dominant allele and one recessive allele. B) Earlier investigators had shown segregation among the offspring. C) A large number of true breeding varieties were available. D) The generation time was short; many offspring can be grown easily. E) He could choose to self- or cross-pollinate.
Version 1
1
4) Mendel's understanding of the inheritance of traits in peas, expressed in modern language, included what? Check all that apply. A) Parents transmit information encoded in genes. B) Each individual contains two genes for each trait. C) Not all genes are identical; alternative forms (alleles) exist. D) Each of the alleles present in an individual is discrete. E) If a given allele is present, its effects will be seen in the individual.
5) Based on his monohybrid crosses, Mendel's proposed which of the following? "Check all that apply." A) Alternative forms of a trait are encoded by alternative alleles. B) The 2 alleles for a given traitseparate when gametes form. C) Each allele has an equal probability of being passed on to the gametes. D) Allele pairs segregate independently of each other. E) Diploid individuals have 2 alleles for each trait.
6) Before Mendel's experiments with pea plants, which ideas formed the basis for most thinking about heredity? Check all that apply. A) Species graduallychange over time B) The traits ofboth parents are blended together in their offspring C) Heredity occurswithin species D) Some traits aredominant and some are recessive E) Traits aretransmitted directly from parents to offspring
7) A scientist has an uncharacterized pea plant. She wants to determine the plant’s genotype for seed color. In order to be certain that she will get conclusive results from a single cross, with what plant could she cross it? Check all that apply.
Version 1
2
A) A green-seeded pea plant (yy) B) A yellow-seeded pea plant (YY) C) Itself
8) A woman with blood type O has a child with blood type O. She claims that a man named Ralphis the child's father.Her claim would be proven false if Ralph has blood type (check all that apply) A) O. B) A. C) B. D) AB.
MULTIPLE CHOICE - Choose the one alternative that best completes the statement or answers the question. 9) The first detailed and quantitative studies on inheritance were carried out by an Austrian monk named _____. A) Mendel B) Darwin C) Mendeleev D) Benedict
10)
Mendel used what plant for his studies on inheritance? A) lily B) carrot C) onion D) pea
Version 1
3
11) The phenotypic ratio for the F 1 generation that results from a testcross between a homozygous recessive individual and a heterozygous individual is A) 1:1. B) 2:1. C) 3:1. D) 1:2:1.
12)
In modern terminology, Mendel's heredity factorsare called A) DNA. B) chromosomes. C) genes. D) RNA.
13)
The observable expression of the genes present in an organism is called its A) phenotype. B) genotype. C) pedigree. D) karyotype.
14)
Alternate forms of the same gene are called A) homologues. B) genotypes. C) cofactors. D) alleles.
15) Which of Mendel's Principles states that the two alleles present at each gene locusseparate from one another during gamete formation and remain distinct?
Version 1
4
A) Principle of Alleles B) Principle of Segregation C) Principle of Crossing Over D) Principle of Dominance E) Principle of Independent Assortment
16) Mendel used what approach to determine whether an individual with the dominant phenotypewas homozygous or heterozygous for that trait? A) backcross B) testcross C) dihybrid cross D) F 2 generation E) double cross
17) This term can be used to describe a diploid organism that has two identical alleles for a given trait. A) homozygous B) heterozygous C) dominant D) recessive E) codominant
18) An allelefor a particular trait that is only expressed in the presence of a second copy of the samealleleis called A) dominant. B) codominant. C) incompletelydominant. D) recessive. E) pleiotropic.
Version 1
5
19) During his experiments with pea plants, Mendel referred to the trait that was expressed in the F 1 or first filial generation as A) recessive. B) dominant. C) codominant. D) independent. E) homozygous.
20) In Mendel's experiments on seed color inpea plants, when a dominant yellow seedbearing plant was crossed with a recessive green seed-bearing plant, what was the approximate phenotypic ratio among the F 2 generation? A) 1 yellow: 3 green B) 1 yellow: 1 green C) 3 yellow: 1 green D) all yellow E) 9:3:3:1
21)
Individuals that have 2 alleles for most gene lociare best described as A) haploid. B) diploid. C) dihybrid. D) homozygous. E) heterozygous.
22) If fertilization involvestwo gametes that contain different alleles of a given gene, the resulting offspring is
Version 1
6
A) dihybrid. B) haploid. C) heterozygous. D) homozygous. E) polygenic.
23)
In a heterozygous individual, the allele being expressed is A) recessive. B) codominant. C) pleiotropic. D) dominant. E) epistatic.
24)
An allele that is present but unexpressed is A) codominant. B) dominant. C) pleiotropic. D) epistatic. E) recessive.
25)
The allelic make up of a cell orindividual is referred to as its A) blueprint. B) genotype. C) phenotype. D) pedigree.
26)
The observable outward manifestation of the genes of an individual is referred to as its
Version 1
7
A) blueprint. B) genotype. C) phenotype. D) genetic map. E) pedigree.
27) What type of cross is used to determine if an individual with the dominant form of a trait is homozygous or heterozygous? A) double cross B) dihybrid cross C) reciprocal cross D) testcross E) back cross
28)
A cross where we follow the inheritance of 2 pairs of alleles is called A) homozygous. B) monohybrid. C) dihybrid. D) heterozygous. E) diallelic.
29) Let P = purple flowers and p = white, and T = tall plants and t = dwarf. If the upper case letters represent the dominant alleles, what is the phenotype of a plant with the genotype PpTt? A) purple flowers, tall B) purple flowers, dwarf C) white flowers, tall D) white flowers, dwarf E) pale purple flowers, intermediate height
Version 1
8
30) Let P = purple flowers and p = white, and T = tall plants and t = dwarf. What are the genotypes of the gametes thatare produced by a plant that is heterozygous for both traits? A) PpTt only B) PT and pt C) P, p, T,andt D) PT, Pt, pT, andpt E) Tt, TT, tt, Pp,PP, and pp
31) Let P = purple flowers and p = white, and T = tall plants and t = dwarf. Of the 16 possible gamete combinations in the dihybrid cross between 2 double heterozygotes, how many would produce the phenotype white, tall? A) None B) 1 C) 3 D) 9 E) 16
32)
Mendel's Principle of Independent Assortment states thatdifferent pairs of A) nonhomologous chromosomessegregate independently of each other. B) sister chromatidssegregate independently of each other. C) nonsister chromatids segregate independently of each other. D) alleles segregateindependently of each other. E) gametes segregateindependently of each other.
33)
The independent assortment of allele pairs is due to
Version 1
9
A) the independentsegregation of homologous chromosome pairs during anaphase II. B) the randomcombination of gametes during fertilization. C) the independentsegregation of sister chromatid pairs during anaphase I. D) the independent segregation of nonsister chromatid pairsduring anaphase II. E) the independentsegregation of homologous chromosome pairs during anaphase I.
34)
If a single gene has 3 or more alternative forms, this is called A) pleiotropy. B) multiple alleles. C) epistasis. D) blendinginheritance. E) codominance.
35) Sometimes, one gene pair will interfere withthe expression of a second gene pair causing divergance from standard Mendelian ratios in an interaction called A) incompletedominance. B) codominance. C) blendinginheritance. D) pleiotropy. E) epistasis.
36)
If an individual allele has more than one effect on the phenotype, this is called A) pleiotropy. B) epistasis. C) blendinginheritance. D) multiplealleles. E) polygenicinheritance.
Version 1
10
37)
ABO blood group determination is an example of A) epistasis. B) incompletedominance. C) polygenicinheritance. D) multiple alleles. E) pleiotropy.
38) When Mendel crossed purple-flowered pea plants with white-flowered pea plants, he never got offspring with flowers that had an intermediate color. This was counter to the theory of A) independentassortment. B) blendinginheritance. C) directtransmission of traits. D) segregation ofalleles. E) continuousvariation of traits.
39) Sometimes, when Mendel crossed two pea plants with each other, he obtained a phenotypic ratio of 3:1 purple-flowered pea plants to white-flowered pea plants. These results are consistent with which set of parents? A) Homozygous purple pea plant and homozygous white pea plant B) Heterozygous purple pea plant and homozygous white pea plant C) Heterozygous purple pea plant and homozygous purple pea plant D) Heterozygous purple pea plant and heterozygous purple pea plant
40) Height is a trait that showscontinuous variation in humans. In pea plants, on the other hand, the tall allele is dominant over the short allele andthere are no intermediate heights. What is the best explanation for this difference?
Version 1
11
A) The alleles thatcontrol height in pea plants are epistatic. B) The alleles thatcontrol height in pea plants are pleiotropic. C) The alleles thatcontrol height in humans are pleiotropic. D) Height is apolygenic trait in humans. E) Height is apolygenic trait in pea plants.
41) An extensive study was conducted on identical twins who were separated at birth. Among other things, the study showed that the individual from each pair who received better nutrition during childhood tended to score higher on standardized intelligence tests.This can best be described as an example of how A) mutation alters phenotype. B) environment altersphenotype. C) environment altersgenotype. D) mutation altersgenotype. E) pleiotropic genesaffect more than one trait.
42) Irene knows her blood type is A, but William does not know his blood type. However, William knows that his mother and father both had blood typeB. Irene and William'sfirst child is a boy with type O blood. Based on this information, William's blood type could be A) B only. B) either B or O. C) O only. D) either AB or O. E) A only.
43) A person who has lost a large amount of blood is being transported to the hospital. Several people are helping the paramedics load the victim into the ambulance. After the ambulance has departed for the hospital, you overhear the following conversation from the persons who helped the paramedics: "I am certain that when that guy gets to the hospital, they will transfuse him with any blood that they have in the blood bank since he has lost so much blood." The other person says, "Yeah, I bet you're right." Having had a biology course, you know which blood could be safely given to anyone. You speak up and say, "As long as it's type __________ blood!" Version 1
12
A) A B) B C) AB D) O
44)
A diploid individual carrying two identical alleles at a given gene locus is called A) homologous. B) heterozygous. C) homozygous. D) dihybrid.
45)
This term refers to the alleles that are present for a particular trait. A) genotype B) karyotype C) pedigree D) phenotype E) pleiotropy
46)
The observable expression of the genes present is called A) genotype. B) karyotype. C) epistasis. D) phenotype.
47)
A diploid individual carrying two different alleles at a given gene locus is called
Version 1
13
A) homologous. B) heterozygous. C) homozygous. D) polygenic.
48) A malefruit fly has the genotype PpYYrrTt. In terms of these 4 allele pairs, how many different types of gametes can he form? A) 2 B) 4 C) 8 D) 16
49) In white tigers, the absence of fur pigmentation is caused by a recessive allele that also causes the tigers to be cross-eyed. If two tigers heterozygous for this allele mate, what do you expect to see among the offspring? A) 1/4 will be both white and cross-eyed, 3/4 will be orange with normal eyes. B) 1/4 will be white with normal eyes, 1/4 will be white and cross-eyed, 1/4 will be orange with normal eyes, and 1/4 will be orange and cross-eyed. C) 1/16 will be white and cross-eyed, 15/16 will be orange with normal eyes. D) 1/3 will be both white and cross-eyed, 2/3 will be orange with normal eyes. E) 1/4 will be orange with normal eyes, 3/4 will be both white and cross-eyed.
50) Let Y = yellow and y = green, and R = round and r = wrinkled. You cross YYRR peas with yyrr peas. All of the F 1 individuals are yellow and round with a genotype of YyRr. You then perform an F 2 cross and get the expected 9:3:3:1 phenotypic ratio. What proportion of the F 2 plants are expected to be heterozygous for both traits? A) 1/16 B) 1/4 C) 1/2 D) 9/16
Version 1
14
51) Let R = red pigment and r = no pigment. In carnations, RR offspring make a lot of red pigment, rr offspring make no pigment, and Rr offspring make a small amount of red pigment, thus appearing pink. Pink carnations are therefore an example of A) codominance. B) incomplete dominance. C) epistasis. D) pleiotropy.
52)
A Punnett square is generally used to A) determine thegenotype of each parent from its phenotype. B) determine thephenotype of each parent from its genotype. C) predict thegenotypic ratio among the offspring. D) predict the numberof offspring that will exhibit each genotype.
53) As a genetic counselor, you are constructing a human pedigree for a particular disease. You note that every generation shows the trait, suggesting that the trait is A) sex-linked. B) recessive. C) dominant. D) polygenic. E) pleiotropic.
54) Achondroplasia is a common form of dwarfism caused by an autosomal dominant mutation in the fibroblast growth factor receptor 3 (FGFR3)gene. Two copies of the mutant gene are invariably fatal before or shortly after birth.If a person with achondroplasia marries a person of normal height, what is the probability that both their first child and second child will have achondroplasia?
Version 1
15
A) 1/8 B) 1/4 C) 1/2 D) 1
55) Mendel performed reciprocal crosses in which he used pollen from a white-flowered plant to fertilize a purple-flowered plant and pollen from a purple-flowered plant to fertilize a white-flowered plant. What did these reciprocal crosses demonstrate? A) The female gametemade a largergenetic contribution to the offspring than the malegamete. B) Each parent madean equal genetic contribution to the offspring. C) Flower color is asex-linked trait. D) The traits of flower color and sex assort independently. E) The male gamete made alargergenetic contribution to the offspring than the female gamete.
56) You toss a coin twice. Because the outcome of the first toss does not affect the outcome of the second toss, these 2 events are called A) mutually exclusive. B) independent. C) dependent. D) random.
57) If you are tossing a six-sided die, what is the probability of getting either a 1 or a 2 on your first toss AND a 1 or a 2 on your second toss?
Version 1
16
A) 1/18 B) 1/9 C) 1/6 D) 1/3 E) 2/3
58) In humans, the sickle-cell trait is caused by a single defective allele, but sickle-cell disease only occurs in individuals that are homozygous for the sickle-cell allele. A man and woman each carry the trait, but do not have sickle-cell disease. What is the probability that their first two children will both have sickle-cell disease? A) 1/16 B) 1/8 C) 1/4 D) 3/8 E) 1/2
59) In rabbits, there is a gene that controls ear length, with a dominant allele "T" for long ears and a recessive allele "t" for short ears. At another gene locus, there are alleles “B” for black coat and “W” for white coat. Neither the B or W allele is dominant, and BW produces a gray coat. These two allele pairs assort independently. If a gray rabbit that is heterozygous at the gene locus controlling ear length is mated with a white rabbit that is also heterozygous at the gene locus controlling ear length, what is the probability that their first offspring will be gray with long ears? A) 1/16 B) 3/16 C) 1/8 D) 3/8 E) 1/2
60) What proportion of the long-eared offspring will be homozygous for the long-eared allele?
Version 1
17
A) 1/8 B) 1/4 C) 1/3 D) 1/2
61) Parent Blood Types
A and B
A and A
B and O
Child Blood Type
O
B
AB
Three babies were recently mixed up in a hospital. Based on the data in the table above, the couple with blood types A and B are the actual parents of the child with blood type A) O B) B C) AB D) none of these babies could be the child of the couple with blood types A and B
62) Within the human body, different cells have different characteristics or phenotypes. For example, neurons look and behave very differently from hepatocytes (liver cells). How can this be explained considering that all cells within a person were derived from the same starting cell? A) Each cell typewithin a person must contain different genes. B) Each cell typewithin a person must contain different proteins. C) Each cell typewithin a person must contain different alleles for specific genes. D) Each cell typewithin a person must undergo continuous variation.
63) The actual number of alleles for most genes in an outbreeding population is between ______.
Version 1
18
A) 1-2 B) 2-3 C) 2-4 D) 3-24 E) 20-500
SECTION BREAK. Answer all the part questions. 64) A secretor is a person who secretes their blood type antigens into body fluids such as saliva. By comparison, a nonsecretor does not. A person's status as a secretor or nonsecretor is independent of blood type. Consider the following inheritance pattern of this trait: secretor x secretor
------>
all offspring are secretors
nonsecretor x nonsecretor
------>
all offspring are nonsecretors
secretor x nonsecretor
------>
all offspring are secretors
F1 generation secretors
------>
F2 generation = 75% secretors, 25% nonsecretors
64.1)
The most likely explanation for this inheritance pattern is that the secretor trait is
A) incompletely dominant. B) recessive. C) dominant. D) codominant.
64.2)
The genotypic ratio produced in the last cross is most likely
A) 3:1 B) 1:2:1 C) 2:1 D) 9:3:3:1 E) 1:1
Version 1
19
65) Let Cy = curly wings and Cy + = wild type. In Drosophila, the Cy allele behaves as a dominant mutation that produces curly wings in the heterozygous condition (Cy/Cy +), but also behaves as a recessive lethal mutation. Flies homozygous for the Cy allele die before reaching adulthood.
65.1) If you crossed flies heterozygous for the Cy allele, what phenotypes would you find in the adultF1 generation? A) Both wild type and curly wings B) Wild type only C) Curly wings only D) None, because allthe F1 flies would die before reaching adulthood.
Version 1
20
Answer Key Test name: Chap 12_3e 1) [A, B, C, E] 2) [A, C, D] 3) [B, C, D, E] 4) [A, B, C, D] 5) [A, B, C, E] 6) [B, C, E] 7) [A, C]
Version 1
21
Clarify Question ● What is the key concept addressed by the question? ● This question asks you to consider the results of various crosses, and indicate which would serve as a testcross to determine the parental genotype. ● What type of thinking is required? ● You must analyze the information to determine which plant(s) would work. Gather Content ● What do you know about testcrosses? What other information is related to the question? ● We learned about Mendel’s use of the testcross. He used homozygous recessive plants to cross to the uncharacterized plant. Because they carry two recessive alleles, only the traits from the tested individual contribute to the phenotype. ● What strategy can you use to determine which cross(es) will provide the needed information? What about considering the results of each possibility in turn? Choose Answer ● Given what you now know, what information is most likely to produce the correct answer? ● First, consider that the uncharacterized plant may be YY, Yy, or yy. Our goal is to distinguish between these possibilities. Which crosses will differentiate them? With a pen and paper, diagram each of the three possible answer choices: 1.If you cross the unknown plant to a green-seeded plant (yy), what will result? If the unknown plant is YY, then YY x yy → 100% Yy yellow seeds. If it is Yy, then Yy x yy → 50% Yy yellow seeds, 50% yy green seeds. If it is yy, then yy x yy →100% yy green seeds. The three possibilities distinguish the genotype. This is the traditional testcross. 2.If you cross the unknown plant to a yellow-seeded plant (YY), what will result? If the unknown plant is YY, then YY x YY → 100% Version 1
22
YY yellow seeds. If it is Yy, then Yy x YY → 50% YY yellow seeds, 50% Yy yellow seeds. If it is yy, then yy x YY →100% Yy yellow seeds. Although the genotypes vary, the phenotypes are ALL yellow seeds. This cross provides no information at all. 3.What about selfing the mystery plant? If the unknown plant is YY, then YY x YY → 100% YY yellow seeds. If it is Yy, then Yy x Yy → 25% YY yellow seeds, 50% Yy yellow seeds, and 25% yy green seeds. If it is yy, then yy x yy →100% yy green seeds. Three different phenotype ratios distinguish the possibilities. This also works! Reflect on Process ● Did your problem-solving process lead you to the correct answer? If not, where did the process break down or lead you astray? How can you revise your approach to produce a more desirable result? ● This question required you to analyze the information given, using logic, to dissect the problem and determine the answer. If you got the correct answer, great job! If you got an incorrect answer, where did the process break down? To answer this question, did you try using pen and paper to write out the crosses? With three possible mystery plant genotypes, and three possible tester plants, that’s 9 different crosses to consider. So you can see that writing it out will help!
8) [D]
Version 1
23
Clarify Question ● What is the key concept addressed by the question? ● This question addresses a single gene cross with recessive and codominant alleles for blood type. ● What type of thinking is required? ● You need to analyze the information given to determine which blood type(s) are incompatible with Ralph being the father. Gather Content ● What do you know about monohybrid crosses and recessive and codominant alleles? What other information is related to the question? ● This question asks which paternal genotypes are incompatible with a child of blood type O, when the mother’s blood type is also O. ● You know that blood type O is determined by the genotype i i, that is, two recessive alleles. ● So both the mother’s genotype and the child’s must be i i. ● What are the genotypes for each of Ralph’s possible blood types? Blood type A may be IA IA or IA i. Blood type B may be IB IB or IB i. Blood type O must be i i. Blood type AB must be IA IB. Choose Answer ● Given what you now know, what information is most likely to produce the correct answer? ● Since the child has two recessive alleles, we know that his father must have contributed a recessive allele. Test out each possible cross in turn: ● If the father is blood type A, he could be IA i and contributed the i allele. ● If the father is blood type B, he could be IB i and contributed the i allele. ● If the father is blood type O, he is i i and contributed an i allele. ● However, if the father is blood type AB, he can only be IA IB. Version 1
24
There is no i allele to contribute. Reflect on Process ● Did your problem-solving process lead you to the correct answer? If not, where did the process break down or lead you astray? How can you revise your approach to produce a more desirable result? ● This question required you to analyze the information given, using logic, to dissect the problem and determine the answer. If you got the correct answer, great job! If you got an incorrect answer, where did the process break down? Did you remember that IA and IB are codominant alleles, but that the i allele is recessive? Did you realize that blood type A could be determined by a heterozygous IA i genotype?
9) A 10) D 11) A 12) C 13) A 14) D 15) B 16) B 17) A 18) D 19) B 20) C 21) B 22) C 23) D 24) E Version 1
25
25) B 26) C 27) D 28) C 29) A 30) D
Version 1
26
Clarify Question ● What is the key concept addressed by the question? ● This question requires two steps. First, you must determine the genotype of a plant that is heterozygous for the two genes, then you can decide what gametes that plant will produce. ● What type of thinking is required? ● You need to use your understanding of heterozygosity and independent assortment, and apply it to a plant with this particular set of genes. Gather Content ● What do you know about heterozygous crosses? What other information is related to the question? ● A diploid organism has two copies of each gene. In this case, it is heterozygous for both genes – it has two different alleles for each. So the plant is Pp Tt. ● Gametes are haploid and carry only one allele for each gene. They assort independently (unless you have reason to believe that they are linked on a chromosome). So any gamete may receive P or p, and any gamete may receive T or t. Furthermore, gametes must be haploid and carry only one copy of each gene, and they must also contain a copy of every gene. ● This creates four possible combinations: PT, Pt, pT, and pt. Choose Answer ● Given what you now know, what information is most likely to produce the correct answer? ● Determine the genotype of the double heterozygous plant. Then determine every possible combination of the four alleles to describe the gametes. Reflect on Process ● Did your problem-solving process lead you to the correct answer? Version 1
27
If not, where did the process break down or lead you astray? How can you revise your approach to produce a more desirable result? ● This question required you to apply your knowledge and understanding to this unfamiliar situation. If you got the correct answer, great job! If you got an incorrect answer, where did the process break down? You will benefit from reading slowly and carefully. This question required determining the plant genotype before figuring out the gametes produced.
31) C
Version 1
28
Clarify Question ● What is the key concept addressed by the question? ● This question requires you to work out the results of a dihybrid cross, using the Punnett square approach. Jotting on scrap paper will help. ● What type of thinking is required? ● You must analyze the situation to determine the 16 gamete combinations produced by the cross, and then count how many of them have a white, tall phenotype. Gather Content ● What do you know about dihybrid crosses? What other information is related to the question? ● A dihybrid cross is a cross between two dihybrids, or double heterozygotes:organisms that are heterozygous for two different genes. So a dihybrid cross is Aa Bb x Aa Bb. Or in this case, Pp Tt x Pp Tt. ● You may remember that Punnett squares are a good way to work out the results of a dihybrid cross. A pen and paper will help. ● You may decide there are only two genotypes that produce white, tall plants: TT pp and Tt pp. This is correct. But careful reading of the question shows that it asks for the proportion of the 16 gamete combinations. These 16 combinations are the results of drawing the Punnett square, which differentiates between the T (maternal) t (paternal) pp combination and the t (maternal) T (paternal) pp combination. Choose Answer ● Given what you now know, what information is most likely to produce the correct answer? ● You could do this in your head, but drawing a Punnett square on scrap paper is an easier approach. Write the possible parental gametes along both the top and side: PT, Pt, pT, and pp. Fill in the 16 combinations, then circle the ones that have a white, Version 1
29
tall phenotype. Reflect on Process ● Did your problem-solving process lead you to the correct answer? If not, where did the process break down or lead you astray? How can you revise your approach to produce a more desirable result? ● This question required you to analyze the information given, using logic, to dissect the problem and determine the answer. If you got the correct answer, great job! If you got an incorrect answer, where did the process break down? Some questions may be deceptively short, but require some time to work out the correct answer. Don’t be shy about using pen and paper to help you by diagramming the cross—geneticists do it all the time!
32) D 33) E 34) B 35) E 36) A 37) D 38) B 39) D
Version 1
30
Clarify Question ● What is the key concept addressed by the question? ● Instead of asking you to derive the phenotype ratio from the cross, here the question asks you to determine which genotypes could be crossed to produce a certain phenotypic ratio. ● What type of thinking is required? ● You have to use your understanding of monohybrid crosses and apply it to this scenario of a 3:1 ratio of purple:white flowers. Gather Content ● What do you know about crosses and phenotype ratios? What other information is related to the question? ● Punnett squares are very helpful for determining phenotype ratios. A pen and paper might be helpful. ● Remember that purple (P) is dominant over white (p). ● If you don’t know the answer right away, look at the answer choices. Could you determine the phenotypic ratios that would result from each choice to find the one that matches 3:1 purple:white? Choose Answer ● Given what you now know, what information is most likely to produce the correct answer? ● Look at the answer choices. For each, you will need to translate the words (homozygous purple) into the genotype (PP). What phenotypes result from each of these crosses? ● Recall that P is dominant for the purple phenotype. So a cross of homozygous purple (PP) x homozygous white (pp) will produce only Pp purple offspring. ● Recall that gametes receive one allele randomly in equal amounts. So a cross of heterozygous purple (Pp) x homozygous white (pp) will produce half purple (Pp) offspring and half white (pp) offspring. Version 1
31
Reflect on Process ● Did your problem-solving process lead you to the correct answer? If not, where did the process break down or lead you astray? How can you revise your approach to produce a more desirable result? ● This question required you to apply your knowledge and understanding to determine parental genotypes/phenotypes. If you got the correct answer, great job! If you got an incorrect answer, where did the process break down? If you didn’t write out the Punnett square, you may have decided that Pp x Pp produces just three types of offspring: PP, Pp, and pp. Don’t forget that there are two categories of Pp offspring: P (maternal) p (paternal) and p (maternal) P (paternal). So three of the four combinations are purple, producing a 3:1 ratio. Even if you didn’t remember that monohybrid crosses produce a 3:1 ratio, you could solve the problem by considering each of the answer choices.
40) D 41) B
Version 1
32
Clarify Question ● What is the key concept addressed by the question? ● This question focuses on the difference between genotype and phenotype. ● What type of thinking is required? ● You must apply your understanding of genotype and phenotype to this situation where identical twins experience different nutrition. Gather Content ● What do you know about genotype and phenotype? What other information is related to the question? ● It is important to distinguish genotype and phenotype, and mutation and environment. Genotype is the set of genes in an individual. The genotype does not change, unless it experiences a mutation. Phenotype is the description of the individual’s traits—even behavioral traits, like intelligence. The phenotype is determined not just by genes, but also by environmental influences— anything from outside the body. ● We know that identical twins share the same genes—the same genotype. Genotype doesn’t change during the lifetime of the individual. ● Intelligence is a trait—an aspect of the phenotype. We can imagine that better nutrition allowed a healthier brain to grow in the lucky twin. Nutrition is a type of environmental influence. ● You might consider that the twin with poor nutrition may have experienced a mutation. That is possible—good nutrition can help prevent mutation-based diseases like cancer. Nutrients like antioxidants help cells to neutralize mutation-causing chemicals. However, a mutation is not the likely basis for the lower intelligence score in the unlucky twin, because mutations happen in individual cells. Choose Answer ● Given what you now know, what information is most likely to Version 1
33
produce the correct answer? ● Since intelligence is part of the phenotype and nutrition is part of the environment, we can conclude that this example shows how environment can alter the phenotype. ● Does this mean that genotype does not affect intelligence? Can you think of other examples in which genotype and environment work together to affect phenotype? Reflect on Process ● Did your problem-solving process lead you to the correct answer? If not, where did the process break down or lead you astray? How can you revise your approach to produce a more desirable result? ● This question required you to apply your knowledge and understanding to this unfamiliar situation. If you got the correct answer, great job! If you got an incorrect answer, where did the process break down? When a question like this trips you up, consider whether the scientific terms were too unfamiliar. Jotting down key terms as you read the chapter, or making flash cards may help you remember them. As you learn about genetics, remember that nature is more complicated than Mendel’s early experiments would indicate. Genes are not the only influence on an individual’s phenotype—the environment is important too.In real life, most traits are a combination of nature and nurture (the nature/nurturedichotomy).
42) B
Version 1
34
Clarify Question ● What is the key concept addressed by the question? ● This question deals with blood type alleles, which have multiple alleles and have a codominance relationship. ● What type of thinking is required? ● This question asks you to analyze the scenario, and use logic to deduce William’s genotype. Gather Content ● What do you know about blood type alleles? What other information is related to the question? ● Blood type is controlled by multiple alleles. The phenotypes A, B, and O are encoded by the alleles IA, IB, and i. ● The i allele is recessive to IA and IB. So a person with type A blood could have a genotype of either IAIA or IAi. Someone with type O must have the genotype ii. ● The A and B alleles are codominant, so IAIB gives the blood type AB. ● William says both his parents were type B. They may each have been either IBIB or IBi. Based on this, William may be either IBIB, IBi, or ii. He could not have received the IA allele. So which answer choices can we rule out? Choose Answer ● Given what you now know, what information is most likely to produce the correct answer? ● We can narrow down the possibilities for William’s genotype based on what we learned about his parents and his son. ● Based on his two type B parents, what are possible genotypes for William? (IBIB, IBi, or ii) ● What are the possible genotypes for William’s type O son? (only Version 1
35
ii) ● What allele must William have contributed to his son? (i) So what genotype can be ruled out for William? (We can rule out IBIB as a possibility) ● The two possibilities we have left cannot be ruled out—IBi, or ii. What blood type do these correspond to? (Type B or Type O) Reflect on Process ● Did your problem-solving process lead you to the correct answer? If not, where did the process break down or lead you astray? How can you revise your approach to produce a more desirable result? ● This question required you to analyze the information given, using logic, to dissect the problem and determine the answer. If you got the correct answer, great job! If you got an incorrect answer, where did the process break down? A pen and paper may help keep track of the information in questions like these. In your diagram, show three generations: William’s parents, William and Irene, and their son. Note both phenotype and possible genotypes for each person. Did you get confused by the information about Irene? Is her blood type an important piece of information
43) D 44) C 45) A 46) D 47) B 48) B
Version 1
36
Clarify Question ● What is the key concept addressed by the question? question asks about allele combinations in gametes.
● This
● What type of thinking is required? ● The question requires you to apply your understanding of alleles and gametes to four particular genes in a fly. ● Gather Content ● What do you know about alleles and gametes? What other information is related to the question? ● Gametes get one copy of each gene. Thus they get one of the alleles, randomly assorted. So a gamete can get P or p for the P gene. Only Y is available for the Y gene (both alleles are the same). Only r is available for the R gene. T or t is possible for the T gene. ● Does the choice of one allele affect the choice of the others? No. Generally speaking, genes assort independently. If genes are close together on the chromosome, they maybe linked, but this question does not indicate linkage. So the P allele can pair with either the T or t allele. Choose Answer ● Given what you now know, what information is most likely to produce the correct answer? ● Now it is a simple matter of counting the possible combinations. Pen and paper may be handy for this: ● The P allele can assort with what other alleles? (Only Y, only r, and either T or t. That makes two possibilities: P Y r T and P Y r t) ● The p allele can assort with what other alleles? (Only Y, only r, and either T or t. That makes two possibilities: p Y r T and p Y r t) ● Add them up and you get how many total possible allele combinations? (4) Version 1
37
Reflect on Process ● Did your problem-solving process lead you to the correct answer? If not, where did the process break down or lead you astray? How can you revise your approach to produce a more desirable result? ● This question required you to apply your knowledge and understanding to this unfamiliar situation. If you got the correct answer, great job! If you got an incorrect answer, where did the process break down? Did you use pen and paper to help figure the combinations? Did you recall that each gamete receives one allele of each gene? Did you recall that genes assort independently?
49) A
Version 1
38
Clarify Question ● What is the key concept addressed by the question? ● This question asks about the phenotypic ratio that results from a cross of two heterozygotes. This is similar to the monohybrid cross we studied in the example of Mendel’s purple and white pea flowers. ● What type of thinking is required? ● You need to analyze the information to determine the phenotype ratio. Gather Content ● What do you know about monohybrid crosses? What other information is related to the question? ● Recall that we saw a monohybrid cross like this for purple and white flower color in peas. Choose Answer ● Given what you now know, what information is most likely to produce the correct answer? ● You could do this in your head, but drawing a Punnett square on scrap paper is an easier approach. Write the possible parental gametes along both the top and side. Just make up your own abbreviation—say, W (orange) and w (white). Fill in the 4 combinations. ● You should get WW, Ww, wW and ww. Which are the phenotypes of each? ● Remember, the w allele is recessive for both white fur and crossed-eyes. So only ww are white and cross-eyed. The other three categories are orange with normal eyes. Reflect on Process ● Did your problem-solving process lead you to the correct answer? If not, where did the process break down or lead you astray? How can you revise your approach to produce a more desirable result? ● This Version 1
39
question required you to analyze the information given, using logic, to dissect the problem and determine the answer. If you got the correct answer, great job! If you got an incorrect answer, where did the process break down? For questions like this, try drawing a Punnett square. Remember that even though there are three genotypes—WW, Ww, and ww—they are not in equal proportions. There are two categories of Ww offspring, giving a 1:2:1 genotype ratio, and a 3:1 phenotype ratio. So only 1/4 of offspring are the white, cross-eyed ww tigers.
50) B
Version 1
40
Clarify Question ● What is the key concept addressed by the question? ● You may recognize that this is a dihybrid cross, like the example in the text. ● What type of thinking is required? ● You need to analyze the information to determine the genotype ratios. Gather Content ● What do you know about dihybrid crosses? What other information is related to the question? ● Dihybrid crosses produce numerous categories of offspring. It can be hard to keep track of them all without a method of diagramming the cross. What method have you learn for keeping track of genotypes and phenotypes from a cross? Choose Answer ● Given what you now know, what information is most likely to produce the correct answer? ● Try drawing a Punnett square. Write the possible gametes on the top and the side. ● The different gametes for a YyRr parent are: YR, Yr, yR, and yr. ● There are 16 combinations of offspring. ● Which offspring are heterozygous for both traits , or YyRr? (Four of the sixteen categories have this genotype, or 1/4.) Reflect on Process ● Did your problem-solving process lead you to the correct answer? If not, where did the process break down or lead you astray? How can you revise your approach to produce a more desirable result? ● This question required you to analyze the information given, using logic, to dissect the problem and determine the answer. If you got the correct answer, great job! If you got an incorrect answer, where did the process break down? Writing down the cross is helpful with questions like these. Version 1
41
Even better, draw the Punnett square.
51) B 52) C 53) C 54) B
Version 1
42
Clarify Question ● What is the key concept addressed by the question? ● This question requires two steps: figuring out the likelihood of an affected offspring in a single pregnancy, then calculating the probability of the first and second children both being affected. Although each pregnancy results in just one child, you can calculate the expected phenotype ratio from the cross to determine the probability of the dwarfism phenotype. ● What type of thinking is required? ● You need to analyze the information to determine the probability of the two children being affected. Gather Content ● What do you know about phenotype ratios from crosses? What other information is related to the question? ● What is the genotype of the parent with normal height? ● What is the genotype of the parent with achondroplasia? (Hint: Remember that the question mentioned that two copies of the mutant gene are fatal.) ● Treat each pregnancy as a cross. The expected phenotype ratio will tell you the likelihood that the child will have the dwarfism phenotype. What method have you learn for keeping track of genotypes and phenotypes from a cross? Choose Answer ● Given what you now know, what information is most likely to produce the correct answer? ● Try drawing a Punnett square. Write the possible gametes on the top and the side. ● The only gametes for the normal height parent areFGFR3+. ● The gametes for the parent with dwarfism are FGFR3+ and FGFR3-. Version 1
43
● There are 2 combinations of offspring : FGFR3+/ FGFR3- and FGFR3+ / FGFR3+, in equal ratio. So there is a 50% chance that a child will have dwarfism. ● If the first child has dwarfism (50%), there is again a 50% chance that the second child will have dwarfism. The second child is unaffected by the result for the first child. So the total probability that both child 1 and 2 will be affected is 1/2 x 1/2, or 1/4. Reflect on Process ● Did your problem-solving process lead you to the correct answer? If not, where did the process break down or lead you astray? How can you revise your approach to produce a more desirable result? ● This question required you to analyze the information given, using logic, to dissect the problem and determine the answer. If you got the correct answer, great job! If you got an incorrect answer, where did the process break down? Did you see the connection between phenotype ratios and probability for one child? Now you may appreciate another aspect of how Punnett squares can be useful. Remember, in genetics treat each child as an independent event—like a coin flip.
55) B 56) B 57) B
Version 1
44
Clarify Question ● What is the key concept addressed by the question? ● This question is asking about your understanding of probability. ● What type of thinking is required? ● You need to apply your understanding of probability to this particular example of rolling a die. Gather Content ● What do you know about probability? What other information is related to the question? ● We have learned about probability in the context of genetics. Like rolling a die, in genetics the outcome from one reproductive event does not affect the likelihood of a subsequent event. ● Break this question into two events: the likelihood from the first roll and the second. Then multiply the two probabilities to get the overall probability. Choose Answer ● Given what you now know, what information is most likely to produce the correct answer? ● What is the likelihood of rolling a 1 or 2 on the first roll? Those are 2 out of 6 possibilities, so 2/6 or 1/3. ● The second roll has the same probability—it is unaffected by the first result. So 1/3 for the second roll. ● The probability of the combined scenario is found by multiplying the two individual probabilities. 1/3 x 1/3 = 1/9. Reflect on Process ● Did your problem-solving process lead you to the correct answer? If not, where did the process break down or lead you astray? How can you revise your approach to produce a more desirable result? ● This question required you to apply your knowledge and understanding to this unfamiliar situation. If you got the correct answer, great job! If you Version 1
45
got an incorrect answer, where did the process break down? Make sure to read questions carefully. Would the result be the same if you were asked about rolling a 1 or 2 on the first roll, and then rolling the same number on the second roll? With dice and with genetics, remember that independent events do not affect each other. Even if a couple has four sons in a row, their fifth baby still has a 50% chance of being a boy!
58) A
Version 1
46
Clarify Question ● What is the key concept addressed by the question? ● This question asks about a monohybrid cross and combines that with a probability problem. ● What type of thinking is required? ● You need to analyze the information to determine the probability of both the two children having sickle cell disease. Gather Content ● What do you know about monohybrid crosses and probability? What other information is related to the question? ● Since the parents carry the trait but do not have the disease, they must be heterozygous (HbA/HbS). The question asks for the probability that the first two children will both have the disease (both be HbS/HbS). ● Did you recognize that this cross is a monohybrid cross, much like the example with Mendel’s purple pea flowers? Do you remember the classic phenotype ratio? Punnett squares are a useful way to determine both genotype and phenotype ratios. Choose Answer ● Given what you now know, what information is most likely to produce the correct answer? ● Try drawing a Punnett square. Write the possible gametes on the top and the side. ● The gametes for each parent are HbA and HbS. There are four categories of offspring: HbA/HbA, HbA/HbS, HbS/HbA, and HbS/HbS. ● Only the last category will have sickle cell disease: HbS/HbS. So there is a ¼ probability for each child. ● Since each child is an independent event, we must multiply the two probabilities to get the overal probability. ¼ x ¼ = 1/16
Version 1
47
Reflect on Process ● Did your problem-solving process lead you to the correct answer? If not, where did the process break down or lead you astray? How can you revise your approach to produce a more desirable result? ● This question required you to analyze the information given, using logic, to dissect the problem and determine the answer. If you got the correct answer, great job! If you got an incorrect answer, where did the process break down? Did you try writing out the genotypes and the cross? Did you draw a Punnett square? Did you remember to treat each child as an independent event? The outcome of the first child does not affect subsequent children.
59) D
Version 1
48
Clarify Question ● What is the key concept addressed by the question? ● This question asks about a cross involving the independent assortment of two genes. This is similar to Mendel’s dihybrid cross, although in this case one parent is homozygous for one of the genes. Also, one gene has dominant and recessive alleles, but the other gene has an incompletely dominant allele: when heterozygous, the “black” allele (B) only turns the fur gray. ● What type of thinking is required? ● This question requires you to analyze the information to determine the phenotype ratio of the offspring. ● What key words does the question contain and what do they mean? ● Pay close attention to the description of the alleles. The dominant allele will exhibit its phenotype in the hybrid, but the recessive allele will exhibit its phenotype only when homozygous. The two genes assort independently–this tells you that they are either on separate chromosomes or far apart on a chromosome, and the alleles end up in the gametes randomly. If a gamete gets a T allele, it could also get either a B or a W from the other gene. ● Don’t be confused by the gene abbreviations. The abbreviation for the ear length gene is T or t. For the fur gene, however, the abbreviations are different letters: B or W. From the question, though, we know that these are alleles of the same gene. So there are two genes in this scenario: the ear length gene T/t, and the fur color gene B/W. Gather Content ● What do you know about dihybrid crosses and independent assortment? What other information is related to the question? ● Recall the dihybrid cross from the text. A two-gene cross can produce as Version 1
49
many as 16 categories of offspring. This is not a problem to do in your head! ● Punnett squares are a useful way to determine both genotype and phenotype ratios. ● What is the genotype of the gray rabbit heterozygous for ear length? (BW Tt) ● What is the genotype of the white rabbit heterozygous for ear length? (WW Tt) ● What is the genotype of the offspring in question, gray with long ears? (BW TT or BW Tt) Choose Answer ● Given what you now know, what information is most likely to produce the correct answer? ● Try drawing a Punnett square. Write the possible gametes on the top and the side. ● The gametes for the gray heterozygous (BW Tt) rabbit are B T, B t, W T, and W t. Gametes for the white heterozygous (WW Tt) rabbit are W T and W t. ● This creates 8 categories of offspring. Of these 8 categories, 3 are gray with long ears: BW TT, BW Tt, and BW tT. So the phenotype ratio is 3/8, and the likelihood that any individual rabbit will be gray with long ears is 3/8. Reflect on Process ● Did your problem-solving process lead you to the correct answer? If not, where did the process break down or lead you astray? How can you revise your approach to produce a more desirable result? ● This question required you to analyze the information given, using logic, to dissect the problem and determine the answer. If you got the correct answer, great job! If you got an incorrect answer, where did the process break down? Did you diagram the categories of offspring with a Punnett Version 1
50
square? Did you recognize that there are two different genotypes that correspond to “gray with long ears”? (BW TT and BW Tt)? Did you recognize that even though the two BW Tt categories have the same genotype, they contribute twice to the ratio? One category receives the T allele from the mother, the other category from the father. This is where a Punnett square can be particularly helpful.
60) C
Version 1
51
Clarify Question ● What is the key concept addressed by the question? ● This question seems to be asking about a cross involving two genes, similar to Mendel’s dihybrid cross. But in fact, does the coat color gene need to be considered to answer the question? Since the question addresses only the ear phenotype, you can ignore the coat color gene entirely. Remember, for unlinked genes one gene does not affect the assortment of the other. So you can treat this as a cross between two heterozygotes for the ear length gene–a monohybrid cross. ● What type of thinking is required? ● This question requires you to analyze the information to determine the genotype ratio of the offspring. ● What key words does the question contain and what do they mean? ● Pay close attention to the description of the alleles. The dominant allele will exhibit its phenotype in the hybrid, but the recessive allele will exhibit its phenotype only when homozygous. The two genes assort independently–this tells you that they are either on separate chromosomes or far apart on a chromosome, and the alleles end up in the gametes randomly. If a gamete gets a T allele, it could also get either a B or a W from the other gene. ● Don’t be confused by the gene abbreviations. The abbreviation for the ear length gene is T or t. For the fur gene, however, the abbreviations are different letters: B or W. From the question, though, we know that these are alleles of the same gene. So there are two genes in this scenario: the ear length gene T/t, and the fur color gene B/W. Gather Content ● What do you know about monohybrid crosses? What other information is related to the question? ● You may recall that Version 1
52
monohybrid crosses produce a 3:1 phenotype ratio, and a 1:2:1 genotype ratio. What does this question ask you to determine? (genotype ratio) ● What is the genotype of the parental rabbits heterozygous for ear length? (Tt) ● What is the genotype of the offspring in question with long ears? (TT or Tt) Choose Answer ● Given what you now know, what information is most likely to produce the correct answer? ● Try drawing a Punnett square. Write the possible gametes on the top and the side. ● The gametes for both heterozygous parents are T and t. (Remember, we can ignore the fur color gene to answer this question) ● This creates 4 categories of offspring: TT, Tt, tT, and tt. Even though Tt and tT are the same genotype, they constitute 2 of the 4 categories, because one is T (maternal) t (paternal) and the other is t (maternal) T (paternal). ● Of the four categories, which have long ears? (TT, Tt, and tT) ● So, considering JUST the long-eared offspring, what proportion of them will be homozygous TT? One of the three categories. So 1/3 of long-eared offspring are homozygous. Reflect on Process ● Did your problem-solving process lead you to the correct answer? If not, where did the process break down or lead you astray? How can you revise your approach to produce a more desirable result? ● This question required you to analyze the information given, using logic, to dissect the problem and determine the answer. If you got the correct answer, great job! If you got an incorrect answer, where did the process break down? Did you diagram the categories of offspring with a Punnett square? Did you recognize that you could ignore fur color to answer the Version 1
53
question? Did you realize that you should answer the proportion of just the long-eared offspring, not all offspring? You can still use the Punnett square ratios–just ignore the short-eared tt category, and determine how many of the three remaining categories are homozygous (one).
61) C
Version 1
54
Clarify Question ● What is the key concept addressed by the question? question addresses blood type alleles.
● This
● What type of thinking is required? ● You need to analyze the information given to determine which baby goes with which parents. Gather Content ● What do you know about blood type alleles? What other information is related to the question? ● Blood type alleles can be recessive (i) or codominant (IA and IB). ● Some blood types are specified by a single genotype (e.g., blood type O is always i i), but some can be specified by two possible genotypes (blood type A could be IA IA or IA i). ● The A and B parents could each be homozygous or heterozygous with an i allele. They could have produced the blood type O baby by contributing their i alleles. They could have produced the B baby by contributing an i and a IB allele. Or they could have produced the AB baby by contributing the IA and IB alleles. You need to figure out the other combinations to correctly answer the question! Choose Answer ● Given what you now know, what information is most likely to produce the correct answer? ● Let’s consider each of the babies in turn: ● The baby of blood type O must have received two i alleles. What parents are possible? (A and B, A and A, or B and O) ● The baby of blood type B must have received alleles IB and i (since there aren’t two B parents). What parents are possible? (A and B, or B and O ● The baby of blood type AB must have received alleles IA and IB. What parents are possible? (only A and B!) So we can assign the AB Version 1
55
baby to parents A and B. ● We can now determine that the baby of blood type B must be from the B and O parents, and finally the only parents left for the O baby are the A and A parents, who must have each contributed an i allele. Reflect on Process ● Did your problem-solving process lead you to the correct answer? If not, where did the process break down or lead you astray? How can you revise your approach to produce a more desirable result? ● This question required you to analyze the information given, using logic, to dissect the problem and determine the answer. If you got the correct answer, great job! If you got an incorrect answer, where did the process break down? Did you recognize that the A and B parents could in theory have produced any of the babies (by contributing one or more i alleles)?
62) B
Version 1
56
Clarify Question ● What is the key concept addressed by the question? ● The question asks about phenotypic differences between cells. ● What type of thinking is required? ● You are being asked to apply your knowledge of phenotypic differences between cells. Gather Content ● What do you know about phenotypic differences between cells? What other information is related to the question? ● All cells in an organism contain the same DNA sequences in their chromosomes. However, different genes can be expressed as proteins in different cell types. This gives different cells different traits. Choose Answer ● Given what you now know, what information is most likely to produce the correct answer? ● Hepatocytes and neurons within a particular person should contain exactly the same genetic information. However, the pattern of expression of these genes, and therefore what proteins are expressed in these cells, is likely to be different. Since proteins are what determines the phenotype of cells, these cells must contain different proteins. Reflect on Process ● Did your problem-solving process lead you to the correct answer? If not, where did the process break down or lead you astray? How can you revise your approach to produce a more desirable result? ● This question asked you to apply your understanding of phenotypic differences between cells. If you got the correct answer, great job! If you got an incorrect answer, where did the process break down? Did you recall that all cells in an organism contain the same DNA? Did you Version 1
57
understand that cells from different organs will express different proteins?
63) D 64) Section Break 64.1) C
Version 1
58
Clarify Question ● What is the key concept addressed by the question? question focuses on allele dominance relationships.
● This
● What type of thinking is required? ● You must evaluate the information, and decide the most likely explanation. Gather Content ● What do you know about allele dominance relationships? What other information is related to the question? ● Recall that many genes have alleles with a dominant/recessive relationship. In a heterozygote, the dominant trait is seen. Some genes are codominant, and the heterozygote phenotype is an expression of both alleles. Other genes are incompletely dominant, with the heterozygote in-between two extremes. ● The most important result is the cross between secretor and nonsecretor. This would produce a heterozygote. Since the heterozygote phenotype is “secretor,” what does this suggest? Choose Answer ● Given what you now know, what information is most likely to produce the correct answer? ● Since the secretor phenotype is expressed in the heterozygote, the allele for that trait is dominant. This is also consistent with the results of the last cross—a monohybrid cross that produces a 3:1 phenotype ratio. Reflect on Process ● Did your problem-solving process lead you to the correct answer? If not, where did the process break down or lead you astray? How can you revise your approach to produce a more desirable result? ● This question required you to weigh and Version 1
59
judge evidence—to evaluate—and choose the best of the possible answers. If you got the correct answer, great job! If you got an incorrect answer, where did the process break down? When reading a question, don’t let details like gene names distract you. To determine whether a trait is dominant, recessive, codominant, or incompletely dominant simply depends on whether the phenotype is exhibited in the heterozygote. If fully expressed, it is dominant. If not expressed, it is recessive. If expressed alongside another allele, it is codominant. If partially expressed, it is incompletely dominant.
64.2) B
Version 1
60
Clarify Question ● What is the key concept addressed by the question? ● This question is is basically a monohybrid cross like that seen in the text for Mendel’s purple pea flowers. ● What type of thinking is required? ● You need to apply what you know about monohybrid crosses and the use of Punnett squares. Gather Content ● What do you know about monohybrid crosses? What other information is related to the question? ● Recall that we saw a monohybrid cross like this for purple and white flower color in peas. Choose Answer ● Given what you now know, what information is most likely to produce the correct answer? ● You could do this in your head, but drawing a Punnett square on scrap paper is an easier approach. Write the possible parental gametes along both the top and side: seS and seNS. Fill in the 4 combinations. ● Note that two are the same, seS/seNS– thus we get a 1:2:1 ratio. Reflect on Process ● Did your problem-solving process lead you to the correct answer? If not, where did the process break down or lead you astray? How can you revise your approach to produce a more desirable result? ● This question required you to apply your knowledge and understanding to this unfamiliar situation. If you got the correct answer, great job! If you got an incorrect answer, where did the process break down? For questions like this, try drawing a Version 1
61
Punnett square. If you chose the answer “3:1”, think about the difference between and genotypic ratio and a phenotypic ratio.
65) Section Break 65.1) A
Version 1
62
Clarify Question ● What is the key concept addressed by the question? ● This question asks about a complex example of one allele that is both recessive and dominant, for different traits. ● What type of thinking is required? ● This question requires you to analyze the information to predict an outcome. Gather Content ● What do you know about dominant and recessive alleles? What other information is related to the question? ● You know that dominant alleles are exhibited in the heterozygote, and recessive alleles are exhibited only in the homozygote. Perhaps you did not know that one allele can be both recessive and dominant, for different traits! ● Don’t let the complexity throw you. Simply write out the cross: Cy/Cy+ x Cy/Cy+ Choose Answer ● Given what you now know, what information is most likely to produce the correct answer? ● Try drawing a Punnett square. Write the possible gametes on the top and the side: Cy and Cy+ on the top and the side. ● The combinations in the offspring are thus Cy/Cy (dead), Cy/Cy+(curly wings, alive), Cy+/Cy (curly wings, alive), and Cy+/Cy+ (straight wings, alive). ● So the two phenotypes seen in adults are curly wings and wild type (straight wings). Reflect on Process ● Did your problem-solving process lead you to the correct Version 1
63
answer? If not, where did the process break down or lead you astray? How can you revise your approach to produce a more desirable result? ● This question required you to analyze the information given, using logic, to dissect the problem and determine the answer. If you got the correct answer, great job! If you got an incorrect answer, where did the process break down? Writing down the cross is helpful with questions like these. Even better, draw the Punnett square.
Version 1
64
CHAPTER 13 MULTIPLE CHOICE - Choose the one alternative that best completes the statement or answers the question. 1) The distances between gene loci measured in terms of the frequency of recombination is indicated by a genetic ______. A) map B) profile C) pedigree D) clone E) karyotype
2) Of the 23 pairs of human chromosomes, 22 pairs are homologous and are found in both males and females. These are called ________. A) bivalents B) autosomes C) recombinantchromosomes D) somaticchromosomes
3) Traits that are controlled by geneslocated on the X chromosome are said to be ________________. A) autosomal B) gametal C) sex-linked D) pleiotropic
4)
Allele pairsare most likely to assort independently of one another when __________.
Version 1
1
A) they control unrelatedtraits B) they control related traits C) they are on the same chromosome D) they are sex linked E) they are on different chromosomes
5) The number of allele pairs that assort independently in an organism is generally much higher than the number of chromosome pairs. This phenomenon is due to __________. A) independent assortment B) segregation C) crossing over D) sex-linkage E) chromosome inactivation
6)
The theory of chromosomal inheritance was first proposed by __________. A) Mendel B) Morgan C) Knight D) Sutton E) Stern
7)
In Drosophila, the sex of an individual is determined by __________. A) one pair of alleles B) the number of X chromosomes C) the number of Y chromosomes D) one pair ofautosomes E) two pairs of alleles
Version 1
2
8) In Morgan's experiments, the white eye allele in Drosophila was shown to be __________. A) located on the X chromosome B) located on the Y chromosome C) dominant D) located on an autosome E) codominant
9) The geneticist who discovered the white eye mutation in Drosophila and helped establish that genes are carried on chromosomes was __________. A) Mendel B) Sutton C) Sturtevant D) Janssens E) Morgan
10)
Genetic exchange between two homologous chromosomesis called __________. A) synapsis B) pleiotropy C) crossing over D) allelic exchange E) independent assortment
11) Occasionally, chromosomes fail to separate during meiosis, leading to daughter cells that have an abnormal number of chromosomes. This phenomenon is called __________.
Version 1
3
A) epistasis B) nondisjunction C) crossing over D) pleiotropy E) chromosome inactivation
12)
Humans who have lost one copy of an autosome are called __________. A) haploid B) trisomic C) bisomic D) monosomic E) monoploid
13) In humans, individuals with trisomy of which chromosome are most likely to survive until adulthood? A) 13th B) 15th C) 18th D) 21st
14)
If a human female has two Barr bodies per cell, it is almost certain that __________. A) her father had one Barr body per cell B) her mother also had two Barr bodies per cell C) she developed from a fertilized egg with three X chromosomes D) she is genetically a male with female characteristics E) she is genetically a normal fertile female
Version 1
4
15) A human female with only one X chromosome is said to have a condition called __________. A) X chromosome inactivation B) Angelman syndrome C) Turner syndrome D) Klinefelter syndrome E) Down syndrome
16) A married couple has a son with sickle-cell disease. If this couple has more children, what is the likelihood that a daughter will inherit sickle-cell disease? A) A daughter would be less likely to inherit sickle-cell disease than a son. B) A daughter would be more likely to inherit sickle-cell disease than a son. C) A daughter would be equally likely to inherit sickle-cell disease as compared to a son. D) A daughter cannot inherit sickle-cell disease, while a son can.
17)
Hemophilia is caused by a __________. A) recessive allele on the X chromosome B) dominant allele on the X chromosome C) codominant allele on the X chromosome D) recessive allele on an autosome E) dominant allele on an autosome
18) Which human hereditary disease is caused by a dominant allele but does not show up in affected individuals until they are in middle age?
Version 1
5
A) Cystic fibrosis B) Sickle cell anemia C) Tay-Sachs disease D) Huntington disease E) Hemophilia
19)
Amniocentesis is a procedure that is normally used __________. A) to reduce the risk of genetic disease B) for gene therapy C) to change the sex of the fetus D) for diagnosis of genetic disorders E) for nourishing the fetus
20) Huntington disease is caused by a single dominant allele. It is a lethal disease, yet it persists in the human population. Which of the following statements best describes why? A) Huntington disease is sex-linked and every human has at least one X chromosome; thus, the chances are extremely high for this allele to be maintained in the human population. B) Huntington disease canpresent symptoms so mild that they appear to lack dominant expression of the allelein some individuals; in those cases, the allele is passed on to the offspring. C) While lethal to a parent, Huntington disease will not be lethal to the offspring since it can skip a generation. D) Huntington disease presents symptoms in midlife, after most people have already had offspring. E) Even though Huntington disease is lethal, it improves chances for reproduction before the person dies.
21) In humans, the male has an X and aY sex chromosome. The human female has two X chromosomes. In birds, the female has a Z and a W sex chromosome while the male has two Z chromosomes. Which of the following statements is accurate about which parent determines the gender of the offspring based on inheritance of the necessary sex chromosome?
Version 1
6
A) In humans andbirds, the male determines the gender of all the offspring. B) In humans andbirds, the female determines the gender of all the offspring. C) In humans, themale determines the gender of the offspring, and in birds the female determinesthe gender. D) In humans, thefemale determines the gender of the offspring, and in birds the male determinesthe gender. E) Determination ofthe gender of any human or bird offspring is related to the environmentalconditions at the time of conception.
22)
Sickle cell anemia is caused by a defect in the __________. A) oxygen-carrying pigment hemoglobin B) ability of the blood to clot C) ability ofred blood cells to fight infection D) chloride ion transport protein
23)
How many Barr bodies does a normal human female contain in each diploid cell? A) 0 B) 1 C) 2 D) 3
24)
Which of the following animals is a genetic male? A) Bird ZW B) Grasshopper XO C) Honeybee diploid D) Drosophila XX
Version 1
7
25) In humans, if nondisjunction led to an individual with a genotype of XO, that personwould __________. A) be female because each cell lacks aY chromosome B) be male because each cell has onlyone X chromosome C) display both male and female characteristics D) not survive
26) In humans, if nondisjunction led to an individual with a genotype of XXY, that personwould __________. A) be female because each cell hastwo X chromosomes B) be male because each cell hasone Y chromosome C) be phenotypically normal D) not survive
27)
Any genetic differences between individuals in a population are called __________. A) markers B) alleles C) polymorphisms D) SNPs
28)
The classic experiments performed by Creighton and McClintock in Maize __________.
A) provided the initial evidence for genetic recombination B) provided evidence that genes located on the same chromosome do not assort independently C) allowed for the establishment of the first genetic map D) provided evidence for the physical exchange of genetic material between homologues
Version 1
8
29) In humans, if an XY individual had a deletion of the SRY gene, that personwould __________. A) develop as a female B) have both male and female characteristics C) have ambiguous genitalia D) develop as a male
30)
Assuming no chromosomal aneuploidies, a calico cat would be which gender? A) Female B) Male C) It cannot be determined from the information provided
31) If an XY human had a genetic disorder that causesinsensitivity to androgens, that person'sgenotype and phenotype would be __________. A) XX, female B) XX, male C) XY, female D) XY, male
32) Which offspring inherit all their mitochondrial DNA from their mother and none from their father? A) Daughters B) Sons C) Both sons and daughters D) Neither sons nor daughters
Version 1
9
33) If you needed to determine the order of genes on a chromosome, you should perform __________. A) a testcross B) a two-point cross C) a three-point cross D) a SNP test
34) A 39-year-old woman is in her sixth week of pregnancy. Due to her advanced age, she is at higher risk for having a baby with Down's syndrome than younger pregnant women. She would like to find out as early as possible whether or not her baby has Down's syndrome. Her doctor should suggest __________. A) amniocentesis B) genetic counseling C) chorionic villi sampling D) a pedigree analysis
35) In Drosophila, dosage compensation is controlled by the male-specific lethal (MSL) complex consisting of MSL proteins and roX RNAs. Based on what you know about dosage compensation, the role of the MSL complex in males would be to __________. A) double the level of expression of genes on the X chromosome B) increase the level of expression of genes on the X chromosome by 50% C) decrease the level of expression of genes on the X chromosome by 50% D) decrease the level of expression of genes on the X chromosome by 100% E) double the level of expression of genes on the Y chromosome
36) What is the relationship between recombination frequency and the actual physicaldistance on a chromosome?
Version 1
10
A) As physicaldistance increases, the recombination frequency increases in a linearfashion. B) As physicaldistance increases, the recombination frequency decreases in alinearfashion. C) As physicaldistance increases, the recombination frequency first increases in a linearfashion, but gradually levels off to a frequency of 0.5. D) As physicaldistance increases, the recombination frequency first increases, but thendecreases.
37) In a two-point cross to map genes A and B, you obtained 98 recombinant types and 902 parental types among the offspring. How far apart are these genes? A) 9.8 cM B) 0.98 cM C) 90.2 cM D) 9.02 cM E) .098 cM
38) Morgan's student Sturtevant demonstrated that the recombination frequencies between a series of linked genes is additive. Examine the following recombination data from Sturtevant, and determine the proper order of the genes on the Drosophila X chromosome. Assume y is in the 0.0 position. Gene 1 Gene 2 Recombination frequency yellow(y) vermilion(v) 0.322 vermilion miniature(m) 0.030 white(w) vermilion 0.297 yellow white 0.010 white miniature 0.337
A) y m v w B) y w v m C) y m w v D) y w m v
Version 1
11
39) A deletion of a particular stretch of chromosome 15 can cause either Prader-Willi syndrome or Angelman syndrome, depending on __________. A) the parental origin of the normal and deleted chromosome B) whether or not the region is methylated properly C) whether a translocation event has occurred D) whether a nondisjunction event has occurred
40)
Why isn't mitochondrial DNA a unique identifier?
A) Mitochondrial DNA is inherited through the paternal lineage. All offspring inherit their father's mitochondria and therefore the same mitochondrial DNA. As a result, all family members that share a paternal lineage would have the same mitochondrial DNA. Mitochondrial DNA can therefore be used to confirm or eliminate a person's relationship within a paternal line, but cannot be used to identify a specific individual. B) Mitochondrial DNA is inherited through the maternal lineage. All offspring inherit their mother's mitochondria and therefore the same mitochondrial DNA. As a result, all family members that share a maternal lineage would have the same mitochondrial DNA. Mitochondrial DNA can therefore be used to confirm or eliminate a person's relationship within a maternal line, but cannot be used to identify a specific individual. C) Mitochondrial DNA is inherited through the maternal lineage. All female offspring inherit their mother's mitochondria and therefore the same mitochondrial DNA. As a result, all female family members that share a maternal lineage would have the same mitochondrial DNA. Mitochondrial DNA can therefore be used to confirm or eliminate a person's relationship within a maternal line, but cannot be used to identify a specific individual.
41) Suppose you are carrying out a series of crosses with an insect where the mechanism of sex determination is unknown. You discover a mutant female with short bristles and decide to cross it with a wild type male that has normal bristles. Half of the F1progeny have short bristles but all of these short-bristled F1progeny are males. Based onthese results, a valid hypothesis would be __________.
Version 1
12
A) males are ZW, females are ZZ, and short bristles are caused by a dominant allele on the Z chromosome B) males are ZZ, females are ZW, and short bristles are caused by a recessive allele on the Z chromosome C) males are ZZ, females are ZW, and short bristles are caused by a dominant allele on the W chromosome D) males are ZZ, females are ZW, and short bristles are caused by a dominant allele on the Z chromosome
42) In 1910, Morgan did a series of experiments with the fruit fly Drosophila, an organism where females are XX and males are XY. When a mutant male fly with white eyes was crossed with a wild type female with red eyes, none of the F1 progeny had white eyes but 18% of the F2 progeny had white eyes. Unexpectedly, all of these white-eyed F2 flies were males. Based on these results, Morgan concluded that white eyes is caused by a recessive X-linked allele. Suppose Morgan had found that half of the F1 progeny had white eyes but all of these whiteeyed F1 flies were females. In this case, a valid hypothesis would be __________. A) white eyes is caused by a recessive Y-linked allele B) white eyes is caused by a dominant Y-linked allele C) white eyes is caused by a dominant X-linked allele D) white eyes is caused by a dominant autosomal allele
43) Genetic maps are based on recombination frequencies. Because both odd and even numbers of crossovers can occur between any two gene loci, as the physical distance between two loci increases, the maximum recombination frequency levels off at 50%. However, suppose you discovered a species where only an even number of crossovers can occur between any two gene loci.In this case, as the physical distance between two loci increases, you would expect the maximum recombination frequency to __________. A) remain at zero B) increase with no limit C) level off at 25% D) level off at 75% E) level off at 100%
Version 1
13
44) Below is a pedigree for a particular human trait. Affected individuals are illustrated by the shaded in circles or squares. This particular pedigree does not yet have a particular shading pattern for carriers. As a result, some of these individuals may be carriers for the trait. What is the mechanism of inheritance of this trait?
A) Sex-linked recessive B) Autosomal dominant C) Autosomal recessive D) Mitochondrial
FILL IN THE BLANK. Write the word or phrase that best completes each statement or answers the question. 45) In humans, hemophilia is caused by a recessive allele on the X chromosome. Suppose a man with hemophilia has children with a healthy woman whose mother had hemophilia. What is the probability that their second child will have hemophilia? (Enter the probability as a percent. Enter the number only without the percent sign. For example, enter 100% as 100 and enter 12.5% as 12.5.)
46) In humans, hemophilia is caused by a recessive allele on the X chromosome. Suppose a man with hemophilia has children with a healthy woman whose mother had hemophilia. If an ultrasound test shows that their first child is a girl, what is the probability that she has hemophilia? (Enter the probability as a percent. Enter the number only without the percent sign. For example, enter 100% as 100 and enter 12.5% as 12.5.)
Version 1
14
47) In the fruit fly Drosophila, there is a dominant gene for normal wings and its recessive allele for vestigial wings. At another gene locus on the same chromosome, there is a dominant gene for red eyes and its recessive allele for purple eyes. A male that was heterozygous at both gene loci was mated with a female that was homozygous for both recessive alleles and the following results were observed among the offspring: Normal wings and red eyes—420 Vestigial wings and red eyes—80 Normal wings and purple eyes—70 Vestigial wings and purple eyes—430 According to these data, what is the distance, in centimorgans, between these two gene loci? (Enter the number only without the units. For example, 100 cM would be entered as 100.)
48) In fruit flies (Drosophila melanogaster) there is a dominant allele for red eyes and a recessive allele for white eyes. These alleles are located on the X chromosome.If a heterozygous red-eyed female is mated with a white-eyed male, what percentage of the offspring are expected to be white-eyed females?(Enter the number only without the percent sign. For example, enter 100% as 100 and enter 12.5% as 12.5.)
49) At an autosomal gene locus in humans, the allele for brown eyes is dominant over the allele for blue eyes. At another gene locus, located on the X chromosome, a recessive allele produces colorblindness while the dominant allele produces normal color vision.A heterozygous brown-eyed woman who is a carrier of colorblindness has children with a blue-eyed man who is not colorblind. What is the probability that their first child will be a blue-eyed female who has normal color vision?(Enter the probability as a percent. Enter the number only without the percent sign. For example, enter 100% as 100 and enter 12.5% as 12.5.)
50) At an autosomal gene locus in humans, the allele for brown eyes is dominant over the allele for blue eyes. At another gene locus, located on the X chromosome, a recessive allele produces colorblindness while the dominant allele produces normal color vision.A heterozygous brown-eyed woman who is a carrier of colorblindness has a child with a blue-eyed man who is not colorblind.An ultrasound test shows that the child is a girl. What is the probability that she will be colorblind?(Enter the probability as a percent. Enter the number only without the percent sign. For example, enter 100% as 100 and enter 12.5% as 12.5.)
Version 1
15
SECTION BREAK. Answer all the part questions. 51) In Drosophila, the allele red eyes (bw+) is dominant to the allelefor brown eyes (bw). At another gene locus on the same chromosome, the allele for thin wing veins (hv+) is dominant to the allele for heavy wing veins (hv). Flies homozygous forbw and hv+ are crossed to flies homozygous for bw+ and hv to obtain doubly heterozygous F 1 progeny.
51.1) Given that these two gene loci are very closely linked, the genotypic ratio in the F2 generation should be closest to __________. A) 1:2:1 B) 1:1:1:1 C) 9:3:3:1 D) 3:1
51.2) Given that these two gene loci are very closely linked, the phenotypic ratio in the F2 generation should be closest to __________. A) one brown eye, heavy wing veins: two red eye, thin wing veins: one brown eye, thin wing veins B) one brown eye, thin wing veins: two red eye, thinwing veins: one red eye, heavywing veins C) three red eye, thinwing veins: one brown eye, heavywing veins D) one brown eye: one red eye: one heavywing veins: one thinwing veins
51.3)
What would be the results of a test cross with the F1 flies?
A) one brown eye, thin wing veins: one red eye, heavy wing veins B) one brown eye, heavy wing veins: one red eye, thin wing veins C) one brown eye, thin wing veins: two red eye, thin wing veins: one red eye, heavy wing veins D) one brown eye, heavy wing veins: two red eye, thin wing veins: one red eye, thin wing veins
Version 1
16
52) You are a forensic technician working on a DNA sample obtained from a crime scene. Your job is to compare the unknown DNA sample with known DNA samples collected from five different suspects. Preliminary analysis using only a few DNA markers revealed that the unknown sample could possibly match two of the suspects. After consulting the case file, you find out that these two suspects are brothers (but not twins). You realize that you will have to do a more detailed analysis on the samples so that you can distinguish between the brothers and determine which brothers' DNA matches the unknown sample.
52.1)
Which of the following will help you distinguish between the two final suspects?
A) Single nucleotide polymorphisms (SNPs) B) Human genome map C) Linkage data D) Markers on the Y chromosome
52.2)
Why can't you use mitochondrial DNA to distinguish between these two suspects?
A) The sequence of mitochondrial DNA has not yet been determined. B) The brothers sharethe same mitochondrial DNA. C) There are no molecular techniques available that allow one to analyze mitochondrial DNA. D) Because mitochondrial DNA is inherited in a paternal pattern.
Version 1
17
Answer Key Test name: Chap 13_3e 1) A 2) B 3) C 4) E 5) C 6) D 7) B 8) A 9) E 10) C 11) B 12) D 13) D 14) C 15) C 16) C
Version 1
18
Clarify Question ● What is the key concept addressed by the question? ● The question asks about the mode of inheritance of sickle cell anemia. ● What type of thinking is required? ● You are being asked to apply your knowledge of the mode of inheritance of sickle cell anemia. Gather Content ● What do you know about the mode of inheritance of sickle cell anemia? What other information is related to the question? ● Sickle cell anemia is an autosomal recessive disease. This means that two parents who are carriers for the sickle cell allele would have a 25% probability of having a child who inherited both sickle cell alleles from their parents. Because the sickle cell trait is not on an X chromosome, the probability of a son or a daughter getting sickle cell anemia is the same. Choose Answer ● Given what you now know, what information is most likely to produce the correct answer? ● Sickle-cell disease is a recessive autosomal disorder. Therefore, sons and daughters would be equally likely to inherit this disease, as it is not sex-linked. The probability of inheriting this disease from heterozygous parents is 1/4. Reflect on Process ● Did your problem-solving process lead you to the correct answer? If not, where did the process break down or lead you astray? How can you revise your approach to produce a more desirable result? ● This question asked you to apply your understanding of the mode of inheritance of sickle cell anemia. If you got the correct answer, great job! If you got an incorrect answer, where did the process break down? Version 1
19
Did you recall that sickle cell anemia is an autosomal recessive disease? Did you understand that boys and girls would have the same probability of getting the disease?
17) A 18) D 19) D 20) D 21) C
Version 1
20
Clarify Question • What is the key concept addressed by the question? o This question addresses sex determination. • What type of thinking is required? o Although you may not have seen this example before, you know enough about the concept to apply your knowledge and understanding to this unfamiliar situation. • What key words does the question contain? o Sex chromosomes are the chromosomes that determine the sex of offspring.X and Y are the names of the sex chromosomes in some animals. When the female carries different sex chromosomes, they are called Z and W. Gather Content • What do you already know about sex determination? o In humans, the Y chromosome determines maleness, due to the SRY gene. o In some fish and reptiles, sex is determined by environmental factors like temperature. o In birds, the female is the heterogametic sex–she carries two different sex chromosomes. In those species the female is ZW and the male is ZZ. Consider Possibilities • Consider the different answer options. Which can you rule out? o In both humans and birds, sex is determined by the chromosomes, not the environment, so we can rule out that option. Choose Answer
Version 1
21
• Given what you now know, what information and/or problem solving approach is most likely to produce the correct answer? o Write out the gametes produced by a human male and female. o The male produces half X gametes and half Y gametes. The female only produces X gametes. o So the baby always receives an X from the mother. If she gets another X from her father she will become a girl; if he gets a Y from his father he will be a boy. o Thus we say the male’s contribution determines the gender. o The opposite is true for birds. The male always contributes a Z gamete. So the gender of the offspring is determined by whether the female contributed a Z or W. Reflect on Process • Did your problem-solving process lead you to the correct answer? If not, where did the process break down or lead you astray? How can you revise your approach to produce a more desirable result? o This question required you to apply your knowledge and understanding of sex determination to answer the question. o Did you understand that the sex is decided by the contribution of one particular parent? o Did you consider the types of gamete each parent produces? 22) A 23) B 24) B 25) A 26) B 27) C 28) D 29) A Version 1
22
30) A 31) C 32) C 33) C 34) C 35) A
Version 1
23
Clarify Question • What is the key concept addressed by the question? o This question addresses dosage compensation. • What type of thinking is required? o Although you may not have seen this example before, you know enough about dosage compensation to apply your knowledge and understanding to this unfamiliar situation. • What key words does the question contain? o Dosage compensation is the process that ensures an equal level of expression from the sex chromosomes despite a differing number of sex chromosomes in males and females. The MSL complex and roX RNAs are the names of genes that participate in the process in flies. Gather Content • What do you already know about dosage compensation? o Dosage compensation is the process that ensures an equal level of expression from the sex chromosomes despite a differing number of sex chromosomes in males and females. o If there were no dosage compensation, females would have twice the level of X-linked gene expression as males. o This would create problems in the cell because the products of Xlinked genes have to work in the correct proportions with the products of autosomal genes. Consider Possibilities • Consider the different answer options. Which can you rule out? o There are few genes on the Y chromosome, and most of those are on the X as well (in the pseudo-autosomal region). So dosage compensation does not affect Y-linked genes. Version 1
24
Choose Answer • Given what you now know, what information and/or problem solving approach is most likely to produce the correct answer? o So for male fruit flies, would dosage compensation need to increase or decrease gene expression? o Flies use the XX/XY system, so male flies have one copy of the X chromosome. o Therefore, male flies need to have increased gene expression relative to females. o How much more expression do the males need to equal that of females (who have two X chromosomes)? o Twice as much–so dosage compensation doubles the expression of Xlinked genes. Reflect on Process • Did your problem-solving process lead you to the correct answer? If not, where did the process break down or lead you astray? How can you revise your approach to produce a more desirable result? o This question required you to apply your knowledge and understanding about dosage compensation to this unfamiliar situation. o Did you read carefully enough to understand that this question asked specifically about male fruit flies? o Did you recognize that the function of dosage compensation is to equalize gene expression between the sexes? o Did you realize that dosage compensation in flies uses a different mechanism than in mammals? In mammals, the dosage of the X is lowered in females, rather than raised in males. o Why do you think the genes were named “male-specific lethals”? Version 1
25
36) C 37) A
Version 1
26
Clarify Question • What is the key concept addressed by the question? o This question addresses a two-point mapping cross. • What type of thinking is required? o This question is asking you to take what you already know and apply it to this unfamiliar situation. • What key words does the question contain and what do they mean? o A two-point cross is a cross involving the alleles of two genes. o Recombinant types are chromosomes with new allele combinations, different than the parents carried. o Parental types are chromosomes with the same allele combinations as the parents carried. Gather Content • What do you already know about two-point mapping? o Genetic distances are described in centimorgans (cM). o One cM represents 1% recombination between the genes. o Recombination events are recognized by counting the number of recombinant chromosomes. Consider Possibilities • What other information is related to the question? Which information is most useful? o To calculate the distance in cM, just determine what percent of progeny have recombinant chromosomes. Choose Answer • Given what you now know, what information and/or problem solving approach is most likely to produce the correct answer? Version 1
27
o From this cross, the progeny were 98 recombinant types and 902 parental types. o 98 + 902 = 1000 total progeny o Recombinant progeny = 98/1000 = 9.8/100 = 9.8% o Since 1 cM represents 1% recombination, we can say the distance is 9.8 cM. Reflect on Process • Did your problem-solving process lead you to the correct answer? If not, where did the process break down or lead you astray? How can you revise your approach to produce a more desirable result? o This question asked how many cM apart are genes A and B, based on the recombination between them. o The question required you to take what you already know and apply it to this unfamiliar situation. o Did you recognize that the distance in centiMorgans is just the percent recombination? 38) B
Version 1
28
Clarify Question • What is the key concept addressed by the question? o This question addresses recombination frequency. • What type of thinking is required? o This answer requires you to weigh and judge evidence–to evaluate–and choose the best of the possible answers. • What key words does the question contain? o Linked genes are genes that lie near each other on a chromosome–near enough that they do not segregate independently. They will segregate together unless there is a recombination event between them, which takes place at some frequency, depending on distance. This is the recombination frequency. Gather Content • What do you already know about recombination frequency? o Recombination frequency reflects the distance between two genes. If the genes are close, the frequency will be low; if far apart, it will be higher, up to 0.5. o The frequency between two genes will be the sum of the frequencies between those flanking genes and a middle gene (up to 0.5). Consider Possibilities • What other information is related to the question? Which information is most useful? o For this problem, drawing a diagram can be helpful. Choose Answer • Given what you now know, what information and/or problem solving approach is most likely to produce the correct answer? o We are told that yellow is at the 0.0 position–in other words, at the end of the chromosome, so start by drawing y on the left. o What recombination data do we have for yellow and other genes? 0.322 between y and v, and 0.010 between y and w. So white is closer to yellow and vermilion is much further. o Now you can draw y-w----------v. o What other data do we have, to help place miniature? 0.030 between v and m, and 0.337 between w and m. So miniature is close to vermilion, and far from white. o But is miniature between white and vermilion, or on the far side of vermilion? The recombination frequency is greater for white and miniature (0.337), than for white and vermilion (0.297), so miniature must be the furthest away. Version 1
29
o Thus: y-w----------v ---m, or y w v m. Reflect on Process • Did your problem-solving process lead you to the correct answer? If not, where did the process break down or lead you astray? How can you revise your approach to produce a more desirable result? o This answer requires you to weigh and judge evidence–to evaluate–and choose the best of the possible answers. o Did you recognize that recombination frequency reflects gene distance? o Did you realize that recombination frequencies are additive? o Did you draw a diagram to help you interpret the data?
39) A 40) B 41) D
Version 1
30
Clarify Question • What is the key concept addressed by the question? o This question addresses “X-linked inheritance”, but in the context of the ZZ/ZW sex determination system. • What type of thinking is required? o This answer requires you to weigh and judge evidence–to evaluate– and choose the best of the possible answers. • What key words does the question contain? o Sex determination is the mechanism by which the gender of the individual is specified. Gather Content • What do you already know about sex determination systems? o Remember, there are two common types of chromosomal sex determination systems: XX/XY and ZZ/ZW. o In the ZZ/ZW system, the female is the heterogametic sex, with Z and W chromosomes. Consider Possibilities • Consider the different answer options. Which can you rule out? o Looking at the answer options, we see they all involve the ZZ / ZW system. o You may recall that in the ZZ / ZW system, it is the females that are ZW. (Otherwise we just call it XX and XY.) So you can rule out the answer that says males are ZW. Choose Answer • Given what you now know, what information and/or problem solving Version 1
31
approach is most likely to produce the correct answer? o If the insect uses the ZZ / ZW system, then the female (with the short bristles) is ZW. The short bristles allele is likely on the Z. Let’s call her genotype ZS / W. o So the male with normal bristles is Z+/ Z+. o Only the sons exhibit the short bristles phenotype. o Yet they are ZS/ Z+. Therefore the mutant allele is dominant. Reflect on Process • Did your problem-solving process lead you to the correct answer? If not, where did the process break down or lead you astray? How can you revise your approach to produce a more desirable result? o This question required you to weigh and judge evidence–to evaluate– and choose the best of the possible answers. o Did you remember that in the ZZ / ZW system, males are ZZ? o Did you try writing out the cross to help answer the question? o Are there other possible explanations? Could the allele have been on the W chromosome? Could the allele have been on an autosome? 42) C
Version 1
32
Clarify Question • What is the key concept addressed by the question? o This question addresses X-linked inheritance. • What type of thinking is required? o This answer requires you to weigh and judge evidence–to evaluate– and choose the best of the possible answers. • What key words does the question contain? o Dominant alleles are expressed when heterozygous, but recessive alleles are expressed only when homozygous or on the only X chromosome in males (hemizygous). Gather Content • What do you already know about X-linked inheritance? o X-linked genes have different patterns of gene expression because males have only the one X chromosome. o Since females have two X chromosomes, they must always receive the one X chromosome from their father. Males always receive one of their mother’s X chromosomes, and the Y from their father. Consider Possibilities • Consider the different answer options. Which can you rule out? o Since the phenotype is only seen in one sex of the progeny, we can rule out the gene sitting on an autosome. If it was, both sexes would receive it equally. Choose Answer • Given what you now know, what information and/or problem solving approach is most likely to produce the correct answer? Version 1
33
o Write out the imaginary cross (“Suppose...”) to answer this question. Call the white-eyed allele w-, and the red allele w+. o The original male is w- / ?, and the female is w+/ w+. o In this imaginary scenario, all the females of the F1 are white-eyed. What would this mean? o They would have received the mutant allele from the white-eyed father, so it would have to be on the X. Females never receive a Y–or they would become males! o If the w- / w+ females exhibited the phenotype, that would mean that w- was dominant over w+. o Thus, a dominant X-linked allele. Reflect on Process • Did your problem-solving process lead you to the correct answer? If not, where did the process break down or lead you astray? How can you revise your approach to produce a more desirable result? o This question required you to weigh and judge evidence–to evaluate– and choose the best of the possible answers. o Did you remember that females never inherit the Y chromosome? o Did you try writing out the cross to help answer the question? 43) A
Version 1
34
Clarify Question • What is the key concept addressed by the question? o This question addresses recombination frequency. • What type of thinking is required? o You need to analyze the information given, using logic, to dissect the problem and determine the answer. • What key words does the question contain? o “Gene loci” is just another way of saying genes. The loci are the locations on the chromosome. Recombination frequency is the percent of progeny with a recombined chromosome–that is, a set of alleles that differs from the original parental chromosomes. Gather Content • What do you already know about recombination frequency? o Recombination frequency is the percent of progeny with a recombined chromosome–that is, a set of alleles that differs from the original parental chromosomes. o Normally, the frequency increases with distance because there is more physical room for the crossover of DNA strands to occur. Consider Possibilities • What other information is related to the question? Which information is most useful? o Drawing a diagram is a useful way to attack this problem. Choose Answer • Given what you now know, what information and/or problem solving approach is most likely to produce the correct answer? Version 1
35
o Draw a chromosome pair with two genes, A and B on the top chromosome, and a and b on the bottom chromosome. o If one crossover event occurs (draw an X between the chromosomes), you can now see that A and b are connected on one chromosome, and a and B are connected on the other chromosome. o But if a second crossover event occurs (draw another X next to the first), the A allele is NOW connected to the B allele–just like the parental chromosome. o So normally, as distance increases, the even numbered crossover events reconnect the parental combination. This counteracts the effect of the first crossover. Eventually, at large distance, only half the events will be odd numbered, and thus visible in the progeny. o What happens if all the events are even numbered? Then the parental chromosomes will always result (A and B, and a and b). This would appear to be a 0% recombination frequency. Reflect on Process • Did your problem-solving process lead you to the correct answer? If not, where did the process break down or lead you astray? How can you revise your approach to produce a more desirable result? o This question required you to analyze the information given, using logic, to dissect the problem and determine the answer. o Did you recognize that double crossover events connect the genes like on the original chromosome? o Did you try drawing a diagram to help answer the question? o What if two genes always had exactly one crossover between them? How would they segregate? 44) C
Version 1
36
Clarify Question ● What is the key concept addressed by the question? interpreting a pedigree. ● What type of thinking is required? pedigrees.
● The question asks about
● You are being asked to apply your knowledge of
Gather Content ● What do you know about pedigrees? What other information is related to the question? ● In a pedigree, colored symbols represent individuals with the indicated trait. In a dominant trait, for a child to inherit the disease, at least one parent would need to have the trait. In an Xlinked trait, males only have one X chromosome and are thus more likely to have the trait. In recessive traits two unaffected parents could be carriers and have a child with the trait. Choose Answer ● Given what you now know, what information is most likely to produce the correct answer? ● There are children with the trait without parents showing the trait, so it is not likely to be dominant. The trait seems to distribute fairly evenly between males and females, so it is not sexlinked. It is not following a pattern where an affected mother passes it on to all of her offspring, soit is not the result of a mitochondrial gene. There are unaffected parents with affected children, so the mechanism of inheritance is autosomal recessive. Reflect on Process ● Did your problem-solving process lead you to the correct answer? If not, where did the process break down or lead you astray? How can you revise your approach to produce a more desirable result? ● This question asked you to apply your understanding of interpreting pedigrees. If you got the correct answer, great job! If you got an incorrect answer, where did the process break down? Did you recall that in a recessive trait two unaffected parents could have a child with the trait? Did you understand that an X-linked trait would be more common in males?
45) 50
Version 1
37
Clarify Question • What is the key concept addressed by the question? o This question addresses X-linked inheritance. • What type of thinking is required? o Although you may not have seen this example before, you know enough about the concept to apply your knowledge and understanding to this unfamiliar situation. • What key words does the question contain? o Hemophilia is a disease that interferes with blood clotting. To answer this question, you just need to know that it is a recessive, X-linked disease. That is, it is due to a gene on the X chromosome, and the disease is only expressed when the allele is homozygous (in females) or hemizygous (on the only X, in males). Gather Content • What do you already know about X-linked inheritance? o X-linked inheritance works differently than autosomal inheritance. For females, genes act similarly. But males only carry one X chromosome. o Since males only carry one X, recessive disease genes on the X are more likely to be expressed. X-linked diseases are more common in males, and females are usually carriers. Consider Possibilities • What other information is related to the question? Which information is most useful? o In this case, we are told that the mother’s mother had hemophilia. She must have been homozygous (since hemophilia is caused by a recessive allele). But her daughter is healthy. Version 1
38
o Therefore we know that the mother is a carrier of the gene–she is heterozygous. o Punnett squares are a useful way to determine both genotype and phenotype ratios. Choose Answer • Given what you now know, what information and/or problem solving approach is most likely to produce the correct answer? o Try drawing a Punnett square. Write the possible gametes on the top and the side. o The gametes for the father are XH and Y. o The gametes for the mother are XH and X+. o There are 4 combinations of offspring : XH / Y, X+ / Y, XH / X+ and XH / XH. o So whether boy or girl, in this case the child has a 50% chance of expressing the hemophilia phenotype. o Whether it is the first child, the second child, or the tenth child, each child has the same probability. Reflect on Process • Did your problem-solving process lead you to the correct answer? If not, where did the process break down or lead you astray? How can you revise your approach to produce a more desirable result? o This question required you to apply your knowledge and understanding to this unfamiliar situation. o Did you remember that recessive X-linked genes are expressed in males that carry the allele? o Did you recognize the information provided by knowing that the maternal grandmother had hemophilia? Version 1
39
o Did you try drawing a Punnett square to help answer the question? o This example is unusual because the father has the disease and the mother is a carrier. So in this case the risk is equal for both males and females. In the population at large, though, we see X-linked diseases more commonly expressed in males. Why? 46) 50
Version 1
40
Clarify Question • What is the key concept addressed by the question? o This question addresses X-linked inheritance. • What type of thinking is required? o Although you may not have seen this example before, you know enough about the concept to apply your knowledge and understanding to this unfamiliar situation. • What key words does the question contain? o Hemophilia is a disease that interferes with blood clotting. To answer this question, you just need to know that it is a recessive, X-linked disease. That is, it is due to a gene on the X chromosome, and the disease is only expressed when the allele is homozygous (in females) or hemizygous (on the only X, in males). Gather Content • What do you already know about X-linked inheritance? o X-linked inheritance works differently than autosomal inheritance. For females, genes act similarly. But males only carry one X chromosome. o Since males only carry one X, recessive disease genes on the X are more likely to be expressed. X-linked diseases are more common in males, and females are usually carriers. Consider Possibilities • What other information is related to the question? Which information is most useful? o In this case, we are told that the mother’s mother had hemophilia. She must have been homozygous (since hemophilia is caused by a recessive allele). But her daughter is healthy. o Therefore we know that the mother is a carrier of the gene–she is Version 1
41
heterozygous. o Punnett squares are a useful way to determine both genotype and phenotype ratios. Choose Answer • Given what you now know, what information and/or problem solving approach is most likely to produce the correct answer? o Try drawing a Punnett square. Write the possible gametes on the top and the side. o The gametes for the father are XH and Y. o The gametes for the mother are XH and X+. o There are 4 combinations of offspring : XH / Y, X+ / Y, XH / X+ and XH / XH. o So whether boy or girl, in this case the child has a 50% chance of expressing the hemophilia phenotype. Reflect on Process • Did your problem-solving process lead you to the correct answer? If not, where did the process break down or lead you astray? How can you revise your approach to produce a more desirable result? o This question required you to apply your knowledge and understanding to this unfamiliar situation. o Did you remember that recessive X-linked genes are expressed in males that carry the allele? o Did you recognize the information provided by knowing that the maternal grandmother had hemophilia? o Did you try drawing a Punnett square to help answer the question? o This example is unusual because the father has the disease and the mother is a carrier. So in this case the risk is equal for both males and Version 1
42
females. In the population at large, though, we see X-linked diseases more commonly expressed in males. Why? 47) 15
Version 1
43
Clarify Question • What is the key concept addressed by the question? o This question addresses linkage mapping by recombination. • What type of thinking is required? o Although you may not have seen this example before, you know enough about the concept to apply your knowledge and understanding to this unfamiliar situation. • What key words does the question contain? o Dominant alleles are expressed in the heterozygote; recessive alleles are only expressed in the homozygote. Centimorgans are a unit of map distance. 1 centimorgan is equivalent to a 1% recombination frequency. Gather Content • What do you already know about linkage mapping by recombination? o Linkage mapping uses recombination frequencies as a proxy for distance along the chromosome. Consider Possibilities • What other information is related to the question? Which information is most useful? o Centimorgans are the unit of map distance. 1 centimorgan is equivalent to a 1% recombination frequency. Choose Answer • Given what you now know, what information and/or problem solving approach is most likely to produce the correct answer? o To analyze the data, we should first determine which were the likely parental chromosomes in the father. Since vg+ pr+ / vg pr (normal wings Version 1
44
and red eyes) and vg pr / vg pr (vestigial wings and purple eyes) are the most common classes, those are the original paternal chromosomes. o So the recombinant chromosomes are vg pr+ / vg pr (vestigial wings and red eyes) and vg+ pr / vg pr (normal wings and purple eyes). o The recombinant chromosomes constitute the fraction of progeny: 80 + 70 / 420 + 80 + 70 + 430, or 150 / 1000. o To determine the percentage recombination frequency, 150 / 1000 = 15 / 100 = 15%. o Since 1% recombination = 1 cM, then 15% recombination is 15 cM. Reflect on Process • Did your problem-solving process lead you to the correct answer? If not, where did the process break down or lead you astray? How can you revise your approach to produce a more desirable result? o This question required you to apply your knowledge and understanding to this unfamiliar situation. o Did you remember that 1% recombination = 1 centimorgan? o Did you recognize that you needed to determine which two classes were the recombinant classes? o Did you determine the percent recombination using the equation : 80 + 70 / 420 + 80 + 70 + 430, or 150 / 1000? 48) 25
Version 1
45
Clarify Question • What is the key concept addressed by the question? o This question addresses X-linked inheritance. • What type of thinking is required? o Although you may not have seen this example before, you know enough about the concept to apply your knowledge and understanding to this unfamiliar situation. • What key words does the question contain? o Dominant alleles are expressed in the heterozygote; recessive alleles are only expressed in the homozygote. Gather Content • What do you already know about X-linked inheritance? o X-linked inheritance works differently than autosomal inheritance. For females, genes act similarly. But males carry just one X chromosome and the other paired chromosome is the Y chromosome. Consider Possibilities • What other information is related to the question? Which information is most useful? o Punnett squares are a useful way to determine both genotype and phenotype ratios. Choose Answer • Given what you now know, what information and/or problem solving approach is most likely to produce the correct answer? o Try drawing a Punnett square. Write the possible gametes on the top and the side. Version 1
46
o The female gametes are w+ and w-. o The male gametes are w- and Y. o There are four combinations of offspring : w+/ w-, w-/ w- , w+/ Y and w-/ Y in equal ratios. o So 25% of the offspring will be white-eyed females: w-/ w-. Reflect on Process • Did your problem-solving process lead you to the correct answer? If not, where did the process break down or lead you astray? How can you revise your approach to produce a more desirable result? o This question required you to apply your knowledge and understanding to this unfamiliar situation. o Did you remember that the X chromosome is paired with the Y chromosome? o Did you try drawing a Punnett square to help answer the question? 49) 25
Version 1
47
Clarify Question • What is the key concept addressed by the question? o This question addresses both X-linked and autosomal inheritance. • What type of thinking is required? o You need to analyze the information given, using logic, to dissect the problem and determine the answer. • What key words does the question contain? o Autosomal refers to genes on a nonsex chromosome. X-linked genes are on the X chromosome, like the colorblindness gene here. Dominant alleles are expressed when heterozygous, but recessive alleles are expressed only when homozygous or on the only X chromosome in males (hemizygous). Gather Content • What do you already know about X-linked and autosomal inheritance? o X-linked genes have different patterns of gene expression because males have only the one X chromosome. o The segregation of an allele on the X will not affect the segregation of an allele on an autosome. Consider Possibilities • What other information is related to the question? Which information is most useful? o Punnett squares are a useful way to determine both genotype and phenotype ratios. Choose Answer
Version 1
48
• Given what you now know, what information and/or problem solving approach is most likely to produce the correct answer? o Try drawing a Punnett square. Write the possible gametes on the top and the side. You will have to create your own abbreviations for the gene alleles. o Let’s call the colorblindness gene C, and the eye color gene B. o The mother is a carrier for colorblindness (C/c) and heterozygous for brown eyes (B/b). Her gametes are C B, C b, c B, and c b. o The father is not colorblind (C/Y) and is blue-eyed (b/b). His gametes are C b and Y b. o This creates 8 combinations in the offspring. o Consider just the ½ of the offspring which are female. The females are CC Bb, CC bb, Cc Bb, and Cc bb. Half of these females are blueeyed, and ALL will have normal color vision. o So ½ (female) x ½ (blue-eyed) x 1 (noncolorblind) = 1/4. or 25% o You can also solve this with a simpler approach: o ½ the offspring will always be female. Those females will ALL receive the C allele on the one X from their father. (A daughter always receives her father’s X chromosome, not the Y.) Of those females, half will get the blue-eyes allele from their mother (and guaranteed that allele from the homozygous recessive father). o So ½ x 1 x ½ = ¼ or 25%. Reflect on Process • Did your problem-solving process lead you to the correct answer? If not, where did the process break down or lead you astray? How can you revise your approach to produce a more desirable result? o This question required you to analyze the information given, using logic, to dissect the problem and determine the answer. o Did you recognize that a daughter will always receive the X from her father? Version 1
49
o Did you try drawing a Punnett square, or at least writing out the cross, to help answer this question? 50) 0
Version 1
50
Clarify Question • What is the key concept addressed by the question? o This question addresses both X-linked and autosomal inheritance. • What type of thinking is required? o You need to analyze the information given, using logic, to dissect the problem and determine the answer. • What key words does the question contain? o Autosomal refers to genes on a non-sex chromosome. X-linked genes are on the X chromosome, like the colorblindness gene here. Dominant alleles are expressed when heterozygous, but recessive alleles are expressed only when homozygous or on the only X chromosome in males (hemizygous). Gather Content • What do you already know about X-linked and autosomal inheritance? o X-linked genes have different patterns of gene expression because males have only the one X chromosome. o Since girls have two X chromosomes, they must always receive the one X chromosome from their father. Consider Possibilities • What other information is related to the question? Which information is most useful? o Punnett squares are a useful way to determine both genotype and phenotype ratios. o Do you need to worry about the eye color gene to answer this question? No!
Version 1
51
Choose Answer • Given what you now know, what information and/or problem solving approach is most likely to produce the correct answer? o Try drawing a Punnett square. Write the possible gametes on the top and the side. You will have to create your own abbreviations for the gene alleles. o Let’s call the colorblindness gene C. To answer this question, we can ignore the eye color gene. o The mother is a carrier for colorblindness (C/c). Her gametes are C and c. o The father is not colorblind (C/Y) . His gametes are C and Y. o This creates four combinations in the offspring. o Consider just the offspring which are female (since we know the child is a female). The females are CC or Cc. o Since the colorblindness allele is recessive, both categories of females will have normal vision. There is a 0% chance of a daughter being colorblind. Reflect on Process • Did your problem-solving process lead you to the correct answer? If not, where did the process break down or lead you astray? How can you revise your approach to produce a more desirable result? o This question required you to analyze the information given, using logic, to dissect the problem and determine the answer. o Did you recognize that a daughter will always receive the X from her father? o Did you try drawing a Punnett square, or at least writing out the cross, to help answer this question? 51) Section Break Version 1
52
51.1) A
Version 1
53
Clarify Question • What is the key concept addressed by the question? o This question addresses genetic linkage. • What type of thinking is required? o You need to analyze the information given, using logic, to dissect the problem and determine the answer. • What key words does the question contain? o This question describes two gene loci (singular is gene locus). Each gene’s abbreviation uses two letters–bw for brown and hv for heavy veins. The genotypic ratio refers to the alleles in the DNA, not to the physical phenotypes. Gather Content • What do you already know about genetic linkage? o Genetic linkage refers to genes that lie close to each other on the chromosome. o Because their loci are linked, they are not segregated by independent assortment. Consider Possibilities • Consider the different answer options. Which can you rule out? o Since the genes are linked, they do not segregate like typical unlinked genes. Therefore we would NOT expect the 9:3:3:1 answer which would be typical for two independent, unlinked genes. Choose Answer • Given what you now know, what information and/or problem solving approach is most likely to produce the correct answer? Version 1
54
o Since the genes are tightly linked, we can approximate them as one gene. Let’s call the bw hv+ combination A, and the bw+ hv combination B. o So the P cross is AA x BB... o Creating the F1 double heterozygotes who cross together, AB x AB. o This cross will produce four categories of offspring in the F2: AA, AB, BA, and BB. But AB and BA are the same genotypically. So the genotypic ratio is 1:2:1. Reflect on Process • Did your problem-solving process lead you to the correct answer? If not, where did the process break down or lead you astray? How can you revise your approach to produce a more desirable result? o This question required you to analyze the information given, using logic, to dissect the problem and determine the answer. o Did you recognize that you can treat the two tightly linked genes like a single gene? o Did you write out the crosses and draw a Punnett square to determine the answer? 51.2) B
Version 1
55
Clarify Question • What is the key concept addressed by the question? o This question addresses genetic linkage. • What type of thinking is required? o You need to analyze the information given, using logic, to dissect the problem and determine the answer. • What key words does the question contain? o This question describes two gene loci (singular is gene locus). Each gene’s abbreviation uses two letters–bw for brown and hv for heavy veins. The genotypic ratio refers to the alleles in the DNA, and the phenotypic ratio refers to the physical phenotypes. Gather Content • What do you already know about genetic linkage? o Genetic linkage refers to genes that lie close to each other on the chromosome. o Because their loci are linked, they are not segregated by independent assortment. Consider Possibilities • Consider the different answer options. Which can you rule out? o In the previous question, we determined the genotypic ratio to be 1:2:1. o The phenotypes are a reflection of the genotypes, so we should expect phenotypes in the same 1:2:1 ratio. o We can rule out the answer with a 1:1:1:1 ratio. o The three genotypes produce three distinct phenotypes, so we can also rule out the answer with a 3:1 ratio.
Version 1
56
Choose Answer • Given what you now know, what information and/or problem solving approach is most likely to produce the correct answer? o The cross will produce four categories of offspring in the F2: AA, AB, BA, and BB. But AB and BA are the same genotypically. So the genotypic ratio is 1:2:1 of AA: AB: BB. o AA stands for bw hv+/ bw hv+, or brown eyes and thin wing veins. o AB stands for bw hv+/ bw+ hv, or red eyes and thin wing veins (the dominant phenotypes). o BB stands for bw+ hv/ bw+ hv, or red eyes and heavy wing veins. o So we get one brown, thin: two red, thin: one red, heavy. Reflect on Process • Did your problem-solving process lead you to the correct answer? If not, where did the process break down or lead you astray? How can you revise your approach to produce a more desirable result? o This question required you to analyze the information given, using logic, to dissect the problem and determine the answer. o Did you recognize that you can treat the two tightly linked genes like a single gene? o Did you write out the crosses and draw a Punnett square to determine the answer? o Did you correctly match the genotypes to the phenotypes? 51.3) A
Version 1
57
Clarify Question • What is the key concept addressed by the question? o This question addresses linked genes and testcrosses. • What type of thinking is required? o You need to analyze the information given, using logic, to dissect the problem and determine the answer. • What key words does the question contain? o A testcross is a cross used to determine the genotype of one of the parents by looking at the phenotypes that result when crossed to a tester strain. Gather Content • What do you already know about linked genes and testcrosses? o Tightly linked genes segregate together, so you can treat them as a unit. o Testcrosses typically use a double homozygous recessive strain, so that any dominant alleles in the other parent are revealed in the progeny. Consider Possibilities • What other information is related to the question? Which information is most useful? o Punnett squares are a useful way to determine both genotype and phenotype ratios. Choose Answer • Given what you now know, what information and/or problem solving approach is most likely to produce the correct answer? Version 1
58
o Since testcrosses typically use a fully recessive homozygous strain, we should assume a homozygous bw hv is crossed to an F1 individual. o The F1 are heterozygous bw hv+ / bw+ hv . o So the cross is bw hv+ / bw+ hv x bw hv / bw hv. o Draw a Punnett square. The tester parent produces only bw hv gametes. o Since the genes segregate together, the other parent produces just two kinds of gametes: bw hv+ and bw+ hv. o This creates just two combinations of offspring in equal proportion: bw hv+ / bw hv and bw+ hv / bw hv. o Their phenotypes are: brown eyes, thin wing veins and red eyes, heavy wing veins. Reflect on Process • Did your problem-solving process lead you to the correct answer? If not, where did the process break down or lead you astray? How can you revise your approach to produce a more desirable result? o This question required you to analyze the information given, using logic, to dissect the problem and determine the answer. o Did you remember the point of a testcross? o Did you recognize that the tester strain should be recessive for all genes? o Did you use a Punnett square to help you determine the genotype ratios? o Did you correctly match phenotypes to the genotypes? 52) Section Break 52.1) A
Version 1
59
Clarify Question • What is the key concept addressed by the question? o This question addresses the use of genetic maps and DNAmarkers. • What type of thinking is required? o You need to analyze the information given, using logic, to dissect the problem and determine the answer. • What key words does the question contain? o DNA markers are features in the DNA that can be analyzed, and often differ between individuals. Forensics is the use of scientific methods to analyze crime data. Gather Content • What do you already know about genetic maps and DNA markers? o Genetic maps provide information about the location and distances between genes. o The human genome map was created using a few individuals to represent a generic human genome–but it does not encompass all of human genetic diversity. o DNA markers are features in the DNA that can differ between individuals and be used to match or disinguish two samples of DNA. Consider Possibilities • Consider the different answer options. Which can you rule out? o Since we need to distinguish the DNA of the two brothers based on the fine details, the human genome map will not be useful. It would be far too expensive and complicated to produce a genome Version 1
60
map for each of the suspects. o Linkage data is also not useful–that only provides general distances between genes and is not specific for individuals. Choose Answer • Given what you now know, what information and/or problem solving approach is most likely to produce the correct answer? o Consider the two remaining choices: markers on the Y chromosome, and SNPs. o Since the two suspects are brothers, how different are their Y chromosomes? Not very! They will likely be the same since they presumably received their Y from the same father (With the exception of some possible recombination in the pseudoautosomal region). o Would single nucleotide polymorphisms (SNPs) be useful? Yes! They are exactly the kind of detailed fine feature that is likely to vary even between closely related individuals. Reflect on Process • Did your problem-solving process lead you to the correct answer? If not, where did the process break down or lead you astray? How can you revise your approach to produce a more desirable result? o This question required you to analyze the information given, using logic, to dissect the problem and determine the answer. o Did you recognize that you needed a source of information that reflected the fine structure differences in individual DNA? o Did you recognize that their Y chromosomes would be the same? 52.2) B Version 1
61
Clarify Question • What is the key concept addressed by the question? o This question addresses mitochondrial inheritance. • What type of thinking is required? o Although you may not have seen this example before, you know enough about the concept to apply your knowledge and understanding to this unfamiliar situation. • What key words does the question contain? o MItochondrial DNA is the DNA that encodes some of the proteins used by mitochondria (other mitochondrial proteins are encoded by regular nuclear DNA). The mitochondrial DNA is carried within the mitochondria. Gather Content • What do you already know about mitochondrial DNA? o Mitochondrial DNA is carried within mitochondria, and mitochondria are inherited only from the mother–the sperm have too little cytoplasm to carry mitochondria. o There is generally only one type of mitochondrial DNA in any individual. That is, all the mitochondria in an individual have the same mitochondrial DNA. Consider Possibilities • Consider the different answer options. Which can you rule out? o Mitochondrial genomes are small and easy to sequence–they have been sequenced. o Mitochondrial DNA is inherited maternally, not paternally. o Mitochondrial DNA is the same as nuclear DNA, and regular molecular biology techniques work fine. Version 1
62
Choose Answer • Given what you now know, what information and/or problem solving approach is most likely to produce the correct answer? o The correct answer is left–since the two men are brothers, they both inherited the same mitochondrial DNA from their mother. Thus, that information will not help distinguish the samples. Reflect on Process • Did your problem-solving process lead you to the correct answer? If not, where did the process break down or lead you astray? How can you revise your approach to produce a more desirable result? o This question required you to apply your knowledge and understanding to this unfamiliar situation. o Did you remember that mitochondrial DNA is maternally inherited? o Did you recognize that brothers will have inherited the same mitochondrial DNA?
Version 1
63
CHAPTER 14 MULTIPLE CHOICE - Choose the one alternative that best completes the statement or answers the question. 1) Viruses that attack bacteria are called __________. A) phages B) proviruses C) virulent D) lytic
2)
Which of the choices is the genetic material for all cellular organisms and some viruses? A) RNA B) DNA C) Protein D) Chromatin
3)
Nucleotides have a phosphate groupattached at which carbon atom of the sugar? A) 2' B) 3' C) 5' D) 1'
4) Watson and Crick developed a model of DNA in which the two strands twist into the shape of a ________________. A) circle B) helix C) double helix D) double pleated sheet
Version 1
1
5) Either strand of a DNA molecule can be used as a template to reconstruct the other because the twostrands of a DNA molecule are __________. A) identical B) antiparallel C) bound D) complementary
6) The synthesis of a growing strand of DNA is carried out by adding nucleotides to which end of an existing strand? A) 2' B) 3' C) 5' D) 1'
7)
DNA consists of two antiparallel strands of nucleotides held together by __________. A) peptide bonds B) covalent bonds C) ionic bonds D) hydrogen bonds
8) The method of DNA replication, where each original strand is used as a template to build a new strand, is called the __________. A) conservative method B) semiconservative method C) disruptive method D) continuous method E) replication and amplification method
Version 1
2
9)
Which enzyme adds new nucleotides to the end of a growing strand? A) Ligase B) Polymerase C) Gyrase D) Helicase E) Endonuclease
10)
Who originally discovered the process of bacterial transformation? A) Watson andCrick B) Chargaff C) Hershey andChase D) Griffith E) Meselson and Stahl
11) When a mixture of live nonvirulent bacteria and dead virulent bacteria was injected into mice, Griffith unexpectedly found that the injected mice died. He explained this result by suggesting that the nonvirulent bacteria are being __________. A) activated B) transformed C) translated D) transcribed E) expressed
12) Avery and his coworkers showed that the agent responsible for changingnonvirulent bacteria into virulent bacteria was __________.
Version 1
3
A) protein B) polysaccharide C) DNA D) RNA E) lipid
13) Based on their experiment with T2 bacteriophages, Hershey and Chase concluded that __________. A) DNA replication is semiconservative B) the phage coatcontains the genetic material C) DNA functions asthe genetic material D) A always pairs with T and G always pairs with C E) virulent bacteria can transform nonvirulent bacteria
14)
After attaching to a bacterial cell, a bacteriophage typically __________. A) injects protein into the cell B) extracts protein fromthe cell C) injects DNA into the cell D) extracts DNA from the cell E) transforms the cell into a virulent strain
15)
DNA primase __________. A) creates a short RNA primer that is complementary to an RNA template B) creates a short DNA primer that is complementary to an RNA template C) creates a short DNA primer that is complementary to a DNA template D) creates a short DNA template that is complementary to an RNA primer E) creates a short RNA primer that is complementary to a DNA template
Version 1
4
16) As the two strands of DNA are unraveled, which enzyme relieves the strain on the two strands? A) DNA polymerase B) DNA ligase C) DNA gyrase D) DNAendonuclease E) DNAexonuclease
17) The chemical bond connecting one nucleotide with the next along one strand of a DNA molecule is called a __________. A) glycosidic bond B) hydrogen bond C) phosphate bond D) phosphodiester bond E) peptide bond
18)
Chargaff's rules for the pairing of nitrogen bases is __________. A) A = C and G = T B) A = T and G = C C) A = G and C = T D) A+T = G+C
19) Information obtained by Franklin from X-ray crystallography on DNA suggested that it is shaped like a __________.
Version 1
5
A) helix B) ribbon C) hollow cylinder D) pleated sheet E) icosahedron
20)
In DNA, complementary nitrogenous bases are held together by __________. A) nitrogen bonds B) hydrogen bonds C) hydrophobic bonds D) peptide bonds E) phosphodiester bonds
21)
The two strands of aDNA molecule contain nitrogenous bases that are __________. A) identical B) parallel C) complementary D) proportionate E) random
22)
Replication of DNA is __________. A) conservative B) redundant C) dispersive D) semiconservative E) semidispersive
23)
During DNA replication, each new strand begins with a short __________.
Version 1
6
A) amino acid primer B) lipid primer C) hydrophilic primer D) DNA primer E) RNA primer
24) The lagging strand is replicated with a seriesof Okazaki fragments and that is why its synthesis is considered to be __________. A) discontinuous B) continuous C) bidirectional D) antiparallel E) semiconservative
25) DNA replication always proceeds by adding new bases to which end of an existing strand? A) 1' B) 2' C) 3' D) 4' E) 5'
26)
The double helix model of DNA structure was proposed by __________. A) Watson and Crick B) Griffith C) Avery D) Franklin E) Chargaff
Version 1
7
27) Griffith, a British microbiologist, used bacteria and mice to demonstrate the process of transformation. During transformation, __________. A) a bacterial chromosome produces an exact copy of itself B) the nucleus of a mouse cell repairs itself when damaged by bacterial infection C) DNA from an infected mouse cell changes bacteria from a nonpathogenic to a pathogenic form D) genetic material is transferred from one bacterial cell to another E) a bacterial cell is transformed into a spore during unsuitable environmental conditions
28)
In 1952, Hershey and Chase confirmed Avery's (1944) conclusion that __________. A) proteins are the repositories for hereditary information B) DNA is the repository for hereditary information C) RNA is the repository for hereditary information D) each DNA molecule is composed of two strands that are twisted into a double helix E) the replication of DNA is semiconservative
29) If 14% of the nucleotides from a DNA moleculecontain the base T, what percent will contain the base G? A) 14% B) 18% C) 28% D) 36% E) 72%
30)
If a short sequence of DNA is 5' AATTGCCGT 3', its complement is __________.
Version 1
8
A) 5' AAAACGCCA 3' B) 3' TTAACGGCT 5' C) 3' TTAACGGCA 5' D) 3' TTAAGCCGA 5' E) 5' TTAACGGCA 3'
31) After DNA replication is complete, each strand of the original molecule is bound to a new complementary strand. This process is known as __________. A) disruptive replication B) conservative replication C) semiconservative replication D) dispersive replication E) stabilizing replication
32) The site where the two original DNA strands are separated and active replication occurs is called the __________. A) replication fork B) replisome C) primosome D) lagging strand E) sliding clamp subunit
33)
The replisome has two main subcomponents. They are __________. A) the replication fork and the sliding clamp subunit B) the primosome and the sliding clamp subunit C) the replication fork and the DNA pol III enzymes D) the DNA pol III enzymes and the sliding clamp subunits E) the primosome and the DNA pol III enzymes
Version 1
9
34)
During replication, which enzyme unwinds the DNA double helix? A) DNA primase B) DNA polymerase I C) DNA helicase D) DNA gyrase E) DNA ligase
35)
Eukaryotic organisms speed up the process of DNA replication by __________. A) shortening the initiation phase B) producing several sliding clamp complexes that provide more binding sites forDNA
pol III C) using DNA gyrase to unravel the double helix rather than DNA helicase D) using multiple origins of replication on each chromosome
36) Endonucleases and exonucleases are enzymes that can remove nucleotides from a polynucleotide chain. Where do endonucleases and exonucleases remove nucleotides from? A) An endonuclease removes nucleotides from the 5' end of the chain; an exonuclease removes nucleotides from the 3' end of the chain. B) An endonuclease removes nucleotides from the 3' end of the chain; an exonuclease removes nucleotides from the 5' end of the chain. C) An endonuclease removes nucleotides interally; an exonuclease removes nucleotides from the ends of the chain. D) An endonuclease removes nucleotides from the ends of the chain; an exonuclease removes nucleotides internally.
37) During DNA replication, DNA pol III synthesizes the lagging strand in segments, called __________.
Version 1
10
A) replication fragments B) Chargaff's segments C) repeating primers D) replisomes E) Okazaki fragments
38) Who proposed that the structure of DNA is a double helix with two polynucleotide chains running in opposite directions and held together by hydrogen bonding between pairs of nitrogenous bases? A) Hershey and Chase B) Chargaff C) Franklin D) Watson and Crick E) Meselson and Stahl
39)
Who demonstrated that phage genetic materialis DNA and not protein? A) Hershey and Chase B) Chargaff C) Franklin D) Watson and Crick E) Avery, MacLeod, and McCarty
40) Who provided X-ray diffraction photographsof purified DNA fibers that suggested a helical structure with a consistent diameter of about 2 nm and a complete helical turn every 3.4 nm? A) Hershey and Chase B) Chargaff C) Franklin D) Watson and Crick
Version 1
11
41)
Who proposed that in any DNA molecule, A =T and G = C? A) Hershey and Chase B) Chargaff C) Franklin D) Watson and Crick
42)
In Griffith's experiments, __________.
A) mice infected with live nonvirulent bacteria developed pneumonia and died B) nonvirulent bacteria transformed the virulent bacteria into a nonvirulent variety C) mice infected with heat-killed virulent bacteriaand live nonvirulent bacteriadeveloped pneumonia and died D) mice infected with heat-killed virulent bacteriaand heat-killed nonvirulent bacteriadeveloped pneumonia and died
43) If we think of the DNA double helix as a twisted ladder, what makes up the rungs or steps of the ladder? A) Phosphate groups only B) Alternating phosphate groups and sugars C) Sugars only D) Purines and pyrimidines E) Phosphate groups and bases
44) Deoxyribose has a carbon atom that is not part of the pentose ring. In a nucleotide, what is attached to this carbon?
Version 1
12
A) A nitrogenous base B) A phosphate group C) Three hydrogen atoms D) One hydrogen atom
45)
Which of the following DNA sequences is complementary to 5' ATGGTCAGT 3'? A) 5' ATGGTCAGT 3' B) 5' TGACTGGTA 3' C) 5' TACCAGTCA 3' D) 5' ACTGACCAT 3'
46) In DNA, a purine must always pair with a pyrimidine and vice versa in order to ensure that __________. A) the distance between the two phosphodiester backbonesremains constant B) the two strands are antiparallel C) the distance between one base pair and the nextremains constant D) each base pair is held together by three hydrogen bonds
47) gap?
During DNA replication, which enzyme removes the RNA primers and then fills in the
A) DNA polymerase I B) DNA primase C) DNA polymerase III D) DNA ligase
48) If a mutation prevented synthesis of the beta subunit of DNA pol III, which would be most affected?
Version 1
13
A) DNA unwinding B) Formation of RNA primers C) Formation of the replication fork D) Processivity E) Removal of RNA primers
49)
Why does DNA polymerase I carry the number one? A) It was the firstpolymerase isolated from E. coli. B) It is the firstpolymerase to be activated during DNA replication. C) It is the onlypolymerase that has both 3' to 5' and 5' to 3' exonuclease activity. D) It is the smallestpolymerase.
50) You are asked to give a classroom demonstration of DNA supercoiling. To do so, you take two different colored pieces of rubber tubing, twist them very tightly and excessively around each other and join the ends of the tubing appropriately. Next, you relieve the supercoiling by breaking one strand and unwinding it. In this demonstration, the breaking and unwinding to relieve the supercoiling simulates the action of __________. A) exonucleases B) polymerases C) ligases D) topoisomerases E) primases
51)
In which cells would you expect to find the highest level of telomerase? A) Muscle cells B) Adipose tissue cells C) Neurons D) Cells that replenish the lining of the gut
Version 1
14
52) You are asked to participate in a clinical trial for a new drug that can activate telomerase. Which of the following is a likely risk of such a drug? A) Decreased capacity for tissue repair B) Increased vulnerability to cancer C) Premature aging D) Decreased telomere length
53) When planning a long week of sunbathing on the beach, what type of DNA damage should sunbathers be aware of before hitting the beach? A) Photolyase-induced mutations B) Decreased telomere length C) Thymine dimers D) Activation of uvr A, B, and C genes
54) Xeroderma pigmentosum (XP) is a rare autosomal recessive disorder. Patients with XP exhibit a cellular hypersensitivity to ultraviolet (UV) radiation, a high incidence of skin cancer, and premature aging. Based on these clinical characteristics, what is the most likely cause for this disease? A) Defects in DNA repair B) Defects in DNA replication C) Lack of telomerase activity D) Shortened telomeres
55) If a mutation produced helicase that was unable to hydrolyze ATP, DNA replication would be __________.
Version 1
15
A) stopped B) sped up C) unaffected D) more prone to errors
56) Suppose a new form of DNA polymerase III is discovered that does not require a primer to begin synthesis of a new strand. All other properties of the enzyme remain unchanged. If this new enzyme were to be expressed in the same organism that normally expresses DNA polymerase III, what would no longer be necessary to completely replicate all of the cellular DNA? A) DNA polymerase I B) Telomerase C) DNA ligase D) DNA helicase E) Single strand-binding protein
57) Suppose a new species of bacterium is discovered. It contains a formof DNA pol III that can add new nucleotides to either the 5' end or the 3' end of an existing strand. All other properties of the enzyme remain unchanged. How would DNA replication in these bacteria differ from normal replication? A) Both new strands could be synthesized continuously like the leading strand. B) The same polymerase molecule could synthesize both strands at the same time. C) Helicase would no longer be required for replication. D) DNA gyrase would no longer be required for replication.
58) If 16% of the nucleotides in one strand of a DNA molecule contain the base G, what percent of the nucleotides on the complementary strand will also contain the base G?
Version 1
16
A) 16% B) 8% C) 34% D) 32% E) It is impossible to determine from the information given.
59) You decide to repeat the Meselson-Stahl experiment, except this time you plan to grow the E. coli cells on light 14N medium for many generations and then transfer them to heavy 15 N medium and allow them to grow for 2 additional generations (2 rounds of DNA replication). If the semiconservative model of DNA replication is correct, what isthe expected distribution of DNA in the density gradient after two rounds of replication? A) One band of intermediate density B) One band of intermediate density and one band of light density C) One band of intermediate density and one band of heavy density D) One band of heavy density and one band of light density E) One band of heavy density
60) You are working with a strain of E. coli that has a mutation in the DNA polymerase II gene, such that the encoded protein is exonuclease deficient, but is otherwise normal. How do you predict that this would most likely affect the activity of this enzyme? A) This mutation would prevent the ability of DNA polymerase II to engage in lagging strand synthesis. B) This mutation would prevent the ability of DNA polymerase II to form phosphodiester bonds between nucleotides. C) This mutation would prevent the ability of DNA polymerase II to remove RNA nucleotides in the 5' to 3' direction. D) This mutation would prevent the ability of DNA polymerase II to remove DNA nucleotides that are damaged.
Version 1
17
61) You are performing an experiment to study replication of DNA in E. coli. Replication initiation was synchronized and DNA was recovered every 5 minutes and an analysis was done to look for DNA replication in specific regions of the E. coli chromosome. Assume that these regions of the chromosome are adjacent to each other in the following order: Region A, Region B, Region C, Region D. The data below indicate the times during which DNA replication was observed in each of these regions. Using these data, which region or regions of the bacterial chromosome most likely contained an origin of replication? Times When DNA Replication Was Observed in Each Region: Region A: 40 minutes, 45 minutes, and 50 minutes Region B: 35 minutes, 40 minutes and 45 minutes Region C: 30 minutes, 35 minutes, and 40 minutes Region D: 35 minutes, 40 minutes, and 45 minutes A) Region A B) Region B C) Region C D) Region D E) Regions A, B, C, and D
62) Each origin of replication in eukaryotic cells is recognized by an origin recognition complex, which allows the assembly of the prereplicative complex including the replicative helicase. This complex ultimately allows the recruitment of DNA polymerase. After DNA polymerase is recruited to an origin and replicates the DNA in that region during S phase, the replicative helicase cannot be loaded onto the DNA in that region again until early G1 due to a variety of regulatory mechanisms. Why is it important that, once used, the replicative helicase cannot be loaded onto the DNA again until G1 of the cell cycle? A) This ensures that multiple origins must be used to replicate the DNA in all cellular chromosomes. B) This ensures that DNA cannot be replicated again in that region until the next cell cycle. C) This ensures thattelomeres on all chromosomes will be replicated properly. D) This ensures that DNA repair processes can only take place during G1 of the cell cycle.
Version 1
18
63) An organism has been found to contain a defective form of photolyase. What function might be impaired in this organism? A) Lengthening of the tips of chromosomes B) Repair of DNA damage caused by UV light C) Stabilization of single-stranded DNA during DNA replication D) Recognition of damaged DNA by the UvrABC complex
Version 1
19
Answer Key Test name: Chap 14_3e 1) A 2) B 3) C 4) C 5) D 6) B 7) D 8) B 9) B 10) D 11) B 12) C 13) C 14) C 15) E 16) C 17) D 18) B 19) A 20) B 21) C 22) D 23) E 24) A 25) C 26) A Version 1
20
27) D 28) B 29) D
Version 1
21
Clarify Question • What is the key concept addressed by the question? o This question addresses base-pairing. • What type of thinking is required? o You need to analyze the information given, using logic, to dissect the problem and determine the answer. • What key words does the question contain? o Nucleotides are the modular units of DNA composed of a phosphate group, a sugar, and a nitrogenous base. The four bases, adenine, thymine, guanine and cytosine are abbreviated A, T, G, and C, respectively. Gather Content • What do you already know about base pairing? o Base pairing is the interaction that connects the two strands of a DNA molecule. o A pairs with T, and G pairs with C. Consider Possibilities • What other information is related to the question? Which information is most useful? o Since T pairs with A, and G pairs with C, the % of T nucleotides does not equate to the number of G nucleotides. o But, it does equate to the number of A nucleotides. For every T in the DNA, there will be an A. o So we know there is 14% T and 14% A. o We also know there are just four kinds of nucelotides that make up the DNA.
Version 1
22
Choose Answer • Given what you now know, what information and/or problem solving approach is most likely to produce the correct answer? o Now we can solve this as a math problem. o The four types of nucleotides add up to 100%. A + T + C + G = 100% o A = T = 14% o Since G pairs with C, they also have the save value. G = C o 14% + 14% + C + G = 100% o 28% + 2G = 100% o 2G = 72% o 72%/2 = G = 36% Reflect on Process • Did your problem-solving process lead you to the correct answer? If not, where did the process break down or lead you astray? How can you revise your approach to produce a more desirable result? o This question required you to analyze the information given, using logic, to dissect the problem and determine the answer. o Did you remember that A pairs with T, and G pairs with C? o Did you recognize that since they pair, the percent of T is equal to the percent of A? o Did you recognize that all four nucleotides must add up to 100%? 30) C
Version 1
23
Clarify Question • What is the key concept addressed by the question? o This question addresses base-pairing. • What type of thinking is required? o Although you may not have seen this example before, you know enough about the concept to apply your knowledge and understanding to this unfamiliar situation. • What key words does the question contain? o A DNA sequence has a direction, which is referred to as 5’ to 3’, or 3’ to 5’. These refer to the carbons on the sugar molecule which connect between the phosphates in the backbone. The complement of a DNA sequence is the sequence of its opposite strand. Gather Content • What do you already know about base pairing? o Base pairing is the interaction that connects the two strands of a DNA molecule. o A pairs with T, and G pairs with C. Consider Possibilities • Consider the different answer options. Which can you rule out? o We have learned that the DNA strands are antiparallel. That means that one strand runs 5’ to 3’, and the other runs 3’ to 5’. o One of the answers appears to base paired properly, but would not be antiparallel, so you can rule that out. Choose Answer Version 1
24
• Given what you now know, what information and/or problem solving approach is most likely to produce the correct answer? o Starting at the 5’ of the original sequence, we can deduce the complementary sequence. o The original strand starts with 5’, so it’s complement begins at the 3’ end. o The original strand starts out 5’ AATT. o We know that A base pairs with T, and vice versa, so its complement is 3’ TTAA. o We know that G base pairs with C, and vice versa, so continuing on the complete complement is: 3' TTAACGGCA 5'. Reflect on Process • Did your problem-solving process lead you to the correct answer? If not, where did the process break down or lead you astray? How can you revise your approach to produce a more desirable result? o This question required you to apply your knowledge and understanding to this unfamiliar situation. o Did you remember that A binds with T, and G binds with C? o Did you recognize that the two DNA strands are antiparallel, so the answer should begin 3’ TTAA? o Note: It is permissible to write a DNA strand in either direction. So an acceptable answer could also have been 5’ ACGGCAATT 3’. How is this different from the wrong answer 5’ TTAACGGCA 3’? 31) C 32) A 33) E 34) C 35) D 36) C Version 1
25
37) E 38) D 39) A 40) C 41) B 42) C 43) D 44) B 45) D
Version 1
26
Clarify Question • What is the key concept addressed by the question? o This question addresses base pairing. • What type of thinking is required? o You need to analyze the information given, using logic, to dissect the problem and determine the answer. • What key words does the question contain? o A DNA sequence has a direction, which is referred to as 5’ to 3’, or 3’ to 5’. These refer to the carbons on the sugar molecule which connect between the phosphates in the backbone. The complementary sequence is the sequence of its opposite strand. Gather Content • What do you already know about base pairing? o Base pairing is the interaction that connects the two strands of a DNA molecule. o A pairs with T, and G pairs with C. Consider Possibilities • What other information is related to the question? Which information is most useful? o We have learned that the DNA strands are antiparallel. That means that one strand runs 5’ to 3’, and the other runs 3’ to 5’. o We can write the DNA either way, as long as we clearly note the direction. So 3’ ATCG 5’ is the same exact sequence as 5’ GCTA 3’. o Note that all the answer options are written to begin with 5’, just like the original sequence. So to match them, we should read from the 3’ Version 1
27
end. Choose Answer • Given what you now know, what information and/or problem solving approach is most likely to produce the correct answer? o Starting at the 5’ of the original sequence, we can deduce the complementary sequence. o The original strand starts with 5’, so it’s complement begins at the 3’ end. o The original strand is 5' ATGGTCAGT 3'. o We know that A base pairs with T, and vice versa, and G base pairs with C, and vice versa. o So the complement of the original strand is 3’ TACCAGTCA 5’. o BUT, note that all the answer options are written from 5’ to 3’. So we need to flip our notation like so: 5’ ACTGACCAT 3’. Reflect on Process • Did your problem-solving process lead you to the correct answer? If not, where did the process break down or lead you astray? How can you revise your approach to produce a more desirable result? o This question required you to analyze the information given, using logic, to dissect the problem and determine the answer. o Did you remember that A binds with T, and G binds with C? o Did you recognize that the two DNA strands are antiparallel? o Did you recognize that all the answer options were written from 5’ to 3’? 46) A 47) A 48) D Version 1
28
Clarify Question • What is the key concept addressed by the question? o This question addresses the function of DNA polymerase. • What type of thinking is required? o You need to analyze the information given, using logic, to dissect the problem and determine the answer. • What key words does the question contain? o DNA pol III is the abbreviation for DNA Polymerase III. One particular protein subunit is the beta (β) subunit. Gather Content • What do you already know about DNA polymerase? o DNA polymerase III is responsible for the bulk of DNA synthesis in E. coli. o The beta subunit is known as a sliding clamp. It forms a ring that can slide along the DNA. Consider Possibilities • Consider the different answer options. Which can you rule out? o DNA pol III is a DNA polymerase–it synthesizes DNA, not RNA. So it is unrelated to the formation of RNA primers. Choose Answer • Given what you now know, what information and/or problem solving approach is most likely to produce the correct answer? o The beta subunit forms a ring that slides along the DNA. Its role is to keep the polymerase attached to the DNA. As long as the polymerase Version 1
29
stays on the DNA, it can continue to synthesize DNA on the leading strand. o The continuous synthesis of DNA is called processivity. Thus, if the beta subunit sliding clamp was missing, the polymerase would fall off the DNA and lack processivity. Reflect on Process • Did your problem-solving process lead you to the correct answer? If not, where did the process break down or lead you astray? How can you revise your approach to produce a more desirable result? o This question required you to analyze the information given, using logic, to dissect the problem and determine the answer. o Did you remember that the beta subunit is shaped like a ring–a sliding clamp? o Did you recognize that if the beta subunit was missing, the polymerase would fall off the DNA more often? o Did you remember that the quality of continuing to synthesize DNA is called processivity? 49) A 50) D
Version 1
30
Clarify Question • What is the key concept addressed by the question? o This question addresses supercoiling and the function of different enzymes. • What type of thinking is required? o Although you may not have seen this example before, you know enough about the concept to apply your knowledge and understanding to this unfamiliar situation. • What key words does the question contain? o Supercoiling is the twisting that can occur in DNA, particularly after helicase has unwound part of the double-stranded portion for replication. Gather Content • What do you already know about supercoiling? o DNA can become highly twisted, or supercoiled. These twisted loops interfere with replication, and the torsion is relieved by enzymes. Consider Possibilities • What other information is related to the question? Which information is most useful? o Look at the answer options. Each is the name of an enzyme. o An enzyme often ends in -ase. The beginning of the name is usually a prefix that hints at the enzymes job. The name can help us remember what the enzyme does. Choose Answer • Given what you now know, what information and/or problem solving Version 1
31
approach is most likely to produce the correct answer? o You are being asked to name the type of enzyme that relieves supercoiling by cutting, unwinding, and reconnecting the DNA. o You may remember that DNA gyrase does this, but that option isn’t here. What is the name of the class of enzymes to which DNA gyrase belongs? o If you don’t remember the name of the enzyme, go through each of the options and figure out their names: o Exonuclease: An enzyme that cuts nucleotides (nucle) from the outside (the end, exo). o Ligase: An enzyme that ties (ligates) DNA together. o Polymerase: An enzyme that makes polymers, or strings of multiple units. o Primase: An enzyme that primes, or starts the process of synthesis by making a primer. o Topoisomerase: An enzyme that changes the shape (topology) of the DNA into another version (isomer). That’s it! Reflect on Process • Did your problem-solving process lead you to the correct answer? If not, where did the process break down or lead you astray? How can you revise your approach to produce a more desirable result? o This question required you to apply your knowledge and understanding to this unfamiliar situation. o Did you remember that topoisomerases are the enzymes that relieve supercoiling? o If not, did you try analyzing the enzyme names to infer their functions? 51) D
Version 1
32
Clarify Question • What is the key concept addressed by the question? o This question addresses the function of telomerase. • What type of thinking is required? o This answer requires you to weigh and judge evidence–to evaluate– and choose the best of the possible answers. • What key words does the question contain? o Telomerase is an enzyme that creates telomeres on the ends of DNA. Gather Content • What do you already know about telomeres and telomerase? o Telomeres are short repeated sequences of DNA, which are found at the tips of each chromosome. o They are made by an enzyme called telomerase, which uses an internal RNA as a template and not the DNA itself. o Telomeres serve to cap off the end of the DNA, protecting it and preventing the molecule from getting shorter in every replication (due to the usual need for a primer). Consider Possibilities • What other information is related to the question? Which information is most useful? o A gradual shortening of the ends of chromosomes occurs in the absence of telomerase activity. This is part of the reason cells senesce, or eventually age and die. o During embryonic and childhood development in humans, telomerase activity is high, but it is low in most somatic cells of the adult. Version 1
33
o The exceptions are cells that must divide as part of their function, such as lymphocytes. Choose Answer • Given what you now know, what information and/or problem solving approach is most likely to produce the correct answer? o So what cells must divide multiple times during the life of an adult? o Not neurons, which do not divide. Not muscle cells. Not fat cells (adipose tissue). o But cells that replenish the gut lining do divide multiple times, and therefore you could expect high levels of telomerase. Reflect on Process • Did your problem-solving process lead you to the correct answer? If not, where did the process break down or lead you astray? How can you revise your approach to produce a more desirable result? o This question required you to weigh and judge evidence–to evaluate– and choose the best of the possible answers. o Did you remember that telomerase acts to build telomeres on newly replicated DNA? o Did you recognize that you should pick the cell type that undergoes rounds of cell division? 52) B
Version 1
34
Clarify Question • What is the key concept addressed by the question? o This question addresses the function of telomerase. • What type of thinking is required? o This answer requires you to weigh and judge evidence–to evaluate– and choose the best of the possible answers. • What key words does the question contain? o A clinical trial is a test of a new drug using human patients. Telomerase is the enzyme that builds telomeres on the ends of DNA. Gather Content • What do you already know about telomeres and telomerase? o Telomeres are short repeated sequences of DNA, which are found at the tips of each chromosome. o They are made by an enzyme called telomerase, which uses an internal RNA as a template and not the DNA itself. o Telomeres serve to cap off the end of the DNA, protecting it and preventing the molecule from getting shorter in every replication (due to the usual need for a primer). Consider Possibilities • What other information is related to the question? Which information is most useful? o A gradual shortening of the ends of chromosomes occurs in the absence of telomerase activity. This is part of the reason cells senesce, or eventually age and die. o During embryonic and childhood development in humans, telomerase Version 1
35
activity is high, but it is low in most somatic cells of the adult. o The exceptions are cells that must divide as part of their function, such as lymphocytes. Consider Possibilities • Consider the different answer options. Which can you rule out? o Having higher telomerase activity would give cells longer telomeres, allowing them to continue to divide. o So you can rule out the answer of decreased telomere length.. o This is likely to increase, not decrease, capacity for tissue repair, so you can rule out that answer. o This is likely to decrease aging, so you can rule out that answer. Choose Answer • Given what you now know, what information and/or problem solving approach is most likely to produce the correct answer? o The remaining answer is increased vulnerability to cancer.. Does this make sense? o Cancer cells divide multiple times. Repeated division gradually shortens and loses the telomeres and degrades the DNA ends. o But with telomerase present, telomeres can be continually reformed. o This removes one of the hurdles for cancer cells to proliferate. Reflect on Process • Did your problem-solving process lead you to the correct answer? If not, where did the process break down or lead you astray? How can you revise your approach to produce a more desirable result? o This question required you to weigh and judge evidence–to evaluate– and choose the best of the possible answers. o Did you remember that telomerase adds telomeres to the ends of Version 1
36
DNA? o Did you recognize that cancer cells need to divide multiple times? 53) C
Version 1
37
Clarify Question • What is the key concept addressed by the question? o This question addresses light-induced DNA damage. • What type of thinking is required? o Although you may not have seen this example before, you know enough about the concept to apply your knowledge and understanding to this unfamiliar situation. • What key words does the question contain? o DNA damage refers to a defect in the structure of DNA, often caused by chemicals (mutagens) or UV light. Gather Content • What do you already know about light-induced DNA damage? o Sunlight includes UV radiation which can be damaging to DNA. o UV light damages DNA by causing formation of thymine dimers. o When two thymines are adjacent, they can become bound together, distorting the helix of the DNA. This can lead to a permanent mutation after DNA replication. Consider Possibilities • Consider the different answer options. Which can you rule out? o Two of the answer options refer not to DNA damage, but to DNA repair mechanisms: photolyase and the uvr genes act to repair DNA. We are looking for an example of UV light-induced DNA damage. Choose Answer • Given what you now know, what information and/or problem Version 1
38
solving approach is most likely to produce the correct answer? o Decreased telomere length would be damaging to DNA, but this is not a common result from UV light. o Instead, formation of thymine dimers is the most common photoproduct of DNA damage. o It activates the photorepair pathway, involving recognition of the thymine dimer by photolyase. (So, photolyase is a good guy!) Reflect on Process • Did your problem-solving process lead you to the correct answer? If not, where did the process break down or lead you astray? How can you revise your approach to produce a more desirable result? o This question required you to apply your knowledge and understanding to this unfamiliar situation. o Did you remember that thymine dimers are a common result of DNA damage? o Did you recognize that photolyase and the uvr genes help repair DNA? o Next time you plan a day in the sun, how can you reduce your risk of thymine dimer formation? 54) A
Version 1
39
Clarify Question • What is the key concept addressed by the question? o This question addresses DNA replication and repair. • What type of thinking is required? o This answer requires you to weigh and judge evidence–to evaluate– and choose the best of the possible answers. • What key words does the question contain? o Xeroderma pigmentosum is a genetic disease. For this question, you simply need to understand its symptoms as described: sensitivity to UV, skin cancer, and aging. Gather Content • What do you already know about DNA replication and repair? o DNA replication is the process that makes more DNA before cell division. o Along with replication, the enzyme telomerase adds telomeres to the ends of the DNA to protect them. o DNA repair is the process of recognizing and fixing damage to the DNA. o The damage can come from chemicals or from the UV radiation in sunlight. Consider Possibilities • What other information is related to the question? Which information is most useful? o We are told that XP patients have three types of symptoms: sensitivity to UV, skin cancer, and premature aging. What ties together these three Version 1
40
things? o You might consider a connection relating to telomeres. After all, an increase in telomerase activity is seen in cancer cells, and telomerase is thought to counteract aging. o But, the main problem from UV light is that it damages DNA by creating thymine dimers, not damaging telomeres. So that doesn’t fit. Choose Answer • Given what you now know, what information and/or problem solving approach is most likely to produce the correct answer? o What does it mean exactly to be hypersensitive to UV light? It means that UV is even more damaging than usual. o First consider what UV does to a healthy individual. The UV causes damage to the structure of DNA, which is recognized and repaired by enzymes. o If the DNA damage is not repaired, cancer can develop. DNA damage also contributes to aging, and DNA repair can slow the effects of aging. o So imagine an individual with a nonfunctional DNA repair process, due to a missing enzyme. UV-induced DNA damage would go unrepaired, leading to a high rate of skin cancer and increased effects of aging. This describes Xeroderma pigmentosum. Reflect on Process • Did your problem-solving process lead you to the correct answer? If not, where did the process break down or lead you astray? How can you revise your approach to produce a more desirable result? o This question required you to weigh and judge evidence–to evaluate– and choose the best of the possible answers. o Did you recognize that sensitivity to UV, skin cancer, and aging all Version 1
41
related to DNA repair? o Xeroderma pigmentosum cannot be cured. But with special precautions, individuals can live into middle age. What precautions would you recommend to an XP patient? 55) A
Version 1
42
Clarify Question • What is the key concept addressed by the question? o This question addresses the function of helicase. • What type of thinking is required? o You need to analyze the information given, using logic, to dissect the problem and determine the answer. • What key words does the question contain? o Helicase is the enzyme that unwinds DNA during DNA replication (the process of making new copies of DNA). Hydrolyzing ATP means breaking ATP down into ADP and a phosphate. Hydrolysis means breaking (lysis) using water (hydro). Gather Content • What do you already know about the function of helicase? o Helicase unwinds the double helix of DNA at the replication fork. The strands have to be separated in order to be copied. o Helicase uses ATP as a source of energy to separate the strands, which are more stable bound together. Consider Possibilities • Consider the different answer options. Which can you rule out? o We know that helicase requires ATP. So a mutation that prevented hydrolysis of ATP would mean helicase couldn’t get energy to do its job. o We also know that helicase is an important part of the replication machinery, so having it broken would definitely interfere with DNA replication. o So we can rule out the answer that says DNA replication would be Version 1
43
unaffected. Choose Answer • Given what you now know, what information and/or problem solving approach is most likely to produce the correct answer? o So if helicase was broken, what would result? The DNA would stay double stranded and wound together. o If DNA stayed double stranded, the whole replication fork would stop and could not continue. o It would not speed up or even continue with errors, because the DNA polymerase would not have access to the template strand for copying. Reflect on Process • Did your problem-solving process lead you to the correct answer? If not, where did the process break down or lead you astray? How can you revise your approach to produce a more desirable result? o This question required you to analyze the information given, using logic, to dissect the problem and determine the answer. o Did you remember that helicase acts to unwind the double stranded DNA? o Did you recognize that helicase must hydrolyze ATP to have energy to drive the unwinding? 56) A
Version 1
44
Clarify Question ● What is the key concept addressed by the question? ● The question asks about the function of primers in DNA synthesis. ● What type of thinking is required? ● You are being asked to apply your knowledge of the function of primers in DNA synthesis. Gather Content ● What do you know about production and removal of DNA primers? What other information is related to the question? ● All three DNA polymerases in E. coli synthesize polynucleotide strands only in the 5′-to-3′ direction and require a primer. In addition to adding nucleotides to a growing DNA strand, some polymerases can also remove nucleotides, or act as a nuclease. DNA Pol I also has a 5′-to-3′ exonuclease activity, which can be used to remove RNA primers. Telomerase is found in organisms with linear chromosomes. Ligase seals nicks in chromosomes. Helicase unwinds DNA at which point single strand binding protein prevent the DNA from rehybridizing. Choose Answer ● Given what you now know, what information is most likely to produce the correct answer? ● This enzyme doesn't require primers to start DNA synthesis, but is otherwise identical to DNA polymerase III. As a result, to replicate DNA, DNA polymerase I would no longer be needed, as it is involved in removing primers and filling in the gaps. DNA polymerase III is expressed in E. coli. As a result, telomerase was never needed in this organism. DNA ligase, DNA helicase and single strand-binding protein would all still be needed for the replication process. Reflect on Process Version 1
45
● Did your problem-solving process lead you to the correct answer? If not, where did the process break down or lead you astray? How can you revise your approach to produce a more desirable result? ● This question asked you to apply your understanding of the function of primers in DNA synthesis. If you got the correct answer, great job! If you got an incorrect answer, where did the process break down? Did you recall that primers need to be removed from the newly replicated DNA? Did you understand that the other enzymes do not require or remove a primer?
57) A
Version 1
46
Clarify Question • What is the key concept addressed by the question? o This question addresses DNA replication. • What type of thinking is required? o You need to analyze the information given, using logic, to dissect the problem and determine the answer. • What key words does the question contain? o DNA Pol III is the polymerase that synthesizes DNA during replication. 5’ and 3’ are directions of the DNA backbone. Gather Content • What do you already know about DNA replication? o Normally, DNA replication proceeds differently on each strand, due to the directionality of the DNA pol III enzyme. o On the leading strand, DNA pol III can add nucleotides continuously after a single primer starts the process. o On the lagging strand, DNA pol III has to add DNA in short segments called Okazaki fragments, each proceeding from a primer. Consider Possibilities • What other information is related to the question? Which information is most useful? o The question suggests that the hypothetical enzyme is different from normal DNA pol III–it can add nucleotides in either direction. o What would this mean for the leading strand? Not much–synthesis on the leading strand can already go quickly and continuously, following the replication fork. o So this would most affect the lagging strand. Version 1
47
Choose Answer • Given what you now know, what information and/or problem solving approach is most likely to produce the correct answer? o What would occur on the lagging strand in this scenario? After primase adds the first primer, the mutant DNA pol III could add nucleotides normally in the 5’ to 3’ direction. But it could ALSO add nucleotides starting from the other side of the RNA primer–in the 3’ to 5’ direction. o Thus, in this scenario, even the lagging strand could proceed with a single, continuous activity of the DNA polymerase following the unwinding of the replication fork. So it would be continuous, just like the leading strand. o Helicase and gyrase would still be required–the replcation fork would still need the DNA to be separated, unwound, and have the supercoiling relieved. o Although each strand of DNA could be synthesized continuously, they would still require separate enzymes to do so. Remember, even in a circular bacterial genome, the two strands are two separate strands–their backbones do not connect. Reflect on Process • Did your problem-solving process lead you to the correct answer? If not, where did the process break down or lead you astray? How can you revise your approach to produce a more desirable result? o This question required you to analyze the information given, using logic, to dissect the problem and determine the answer. o Did you recognize that lagging strand synthesis is normally discontinuous because of the directionality of DNA pol III? Version 1
48
o Did you realize that the functions of helicase and gyrase would not be changed? 58) E
Version 1
49
Clarify Question • What is the key concept addressed by the question? o This question addresses base pairing in DNA. • What type of thinking is required? o You need to analyze the information given, using logic, to dissect the problem and determine the answer. • What key words does the question contain? o The nucleotides in DNA are the modular units that contain bases adenine, thymine, cytosine, guanine, known as A, T, C, and G, respectively. The complementary strand of DNA is the other strand of the double helix that must base pair with the first strand. Gather Content • What do you already know about base pairing in DNA? o Base pairing in DNA follows the rules: A binds with T, and C binds with G. o One strand of the double helix must bind its complementary strand according to these rules. Consider Possibilities • What other information is related to the question? Which information is most useful? o There are only four bases in DNA, so the percentage of those four must add up to 100% of the DNA bases. Choose Answer • Given what you now know, what information and/or problem Version 1
50
solving approach is most likely to produce the correct answer? o What can we infer from the information that one strand contains 16% G? o We know that G binds with C on the opposite strand, so we can infer that the complement contains 16% C. o What else can we infer? Not much! o Try the approach of imagining examples and testing each one. o What about a sequence like ATGCGC? In this case, there would be an equal number of Gs in each strand because the top strand has an equal number of Gs and Cs. But does that have to be the case? o What about the sequence ATGGGG? Now, the top strand has only Gs, and the bottom strand has only Cs! o We know that in the top strand, (A + T + C) + 16% = 100%, and in the bottom strand (A + T + G) + 16% = 100%. But without further information, there is no way to know how much the remaining three bases are contributing to the DNA. The answer is impossible to determine. Reflect on Process • Did your problem-solving process lead you to the correct answer? If not, where did the process break down or lead you astray? How can you revise your approach to produce a more desirable result? o This question required you to analyze the information given, using logic, to dissect the problem and determine the answer. o Did you remember that G binds with C? o Did you recognize that Gs and Cs don’t have to equate on the same strand? o Did you test examples of your own to analyze possibilites? o Could a piece of DNA be all Gs on one strand, and all Cs on another? 59) C Version 1
51
Clarify Question • What is the key concept addressed by the question? o This question addresses the Meselson-Stahl experiment. • What type of thinking is required? oYou need to analyze the information given, using logic, to dissect the problem and determine the answer. • What key words does the question contain? o The Meselson-Stahl experiment was a classic experiment that demonstrated that DNA replication is semiconservative. The medium is the nutrient broth the cells are grown in–when grown with 14N, the DNA is light, when grown with 15N, the DNA is heavy. Gather Content • What do you already know about the Meselson-Stahl experiment? o The Meselson-Stahl experiment was a clever, classic experiment that tested whether DNA replication was conservative, semiconservative, or dispersive. o The bands on the density gradient indicated how much of the 15N was incorporated into the new strands of DNA. Heavy bands were all 15N, light bands were all 14N, and intermediate bands had a mixture. Consider Possibilities • What other information is related to the question? Which information is most useful? o The results of the real experiment showed that in each round of replication, the new DNA has one old strand and one newly synthesized strand. This supported the semiconservative model. Version 1
52
o The template strand was conserved, but each new DNA double helix contained one fresh strand. Choose Answer • Given what you now know, what information and/or problem solving approach is most likely to produce the correct answer? o A diagram can help to answer this question. You can use two colors to indicate light 14N strands and heavy 15N strands. o Remember, the question asks you to assume that replication is semiconservative–that would mean that each new double helix has one strand of new DNA, and one strand of old DNA. o We start out with all light DNA. In the first round, there would result two new pieces of DNA–each a hybrid of heavy and light. o In the second round, there would be two hybrid heavy/light molecules, plus two all heavy molecules. o Thus, a heavy band and an intermediate density band. Reflect on Process • Did your problem-solving process lead you to the correct answer? If not, where did the process break down or lead you astray? How can you revise your approach to produce a more desirable result? o This question required you to analyze the information given, using logic, to dissect the problem and determine the answer. o Did you remember the meaning of the semiconservative model? o Did you recognize what types of DNA would result from this experiment? o Did you try drawing a diagram to help answer the question? 60) D
Version 1
53
Clarify Question ● What is the key concept addressed by the question? ● The question asks about mutations in the exonuclease activity of DNA polymerase II. ● What type of thinking is required? ● You are being asked to analyze the effect of a mutation in the exonuclease activity of DNA polymerase II. Gather Content ● What do you know about DNA polymerase II? What other information is related to the question? ● The three different polymerases have different roles in the replication process. DNA Pol III is the main replication enzyme; it is responsible for the bulk of DNA synthesis. DNA Pol I acts on the lagging strand to remove primers and replace them with DNA. The Pol II enzyme does not appear to play a role in DNA replication. It is involved instead in DNA repair processes. To do this, it has 3’ to 5’ exonuclease activity so it can back up and remove bases that are incorrect and replace them with correct bases. Choose Answer ● Given what you now know, what information is most likely to produce the correct answer? ● E. coli DNA polymerase II is thought to be involved in DNA repair processes. It is not important for lagging strand synthesis, nor does it have a 5'-3' exonuclease activity to remove RNA primers. It can synthesize DNA. This ability will not be compromised if the mutation only affects the exonuclease activity of this enzyme. However, an exonuclease-deficient DNA polymerase II would not be able to remove damaged bases, which is necessary for certain types of DNA repair.
Version 1
54
Reflect on Process ● Did your problem-solving process lead you to the correct answer? If not, where did the process break down or lead you astray? How can you revise your approach to produce a more desirable result? ● This question asked you to analyze the effect of a mutation in the exonuclease activity of DNA polymerase II. If you got the correct answer, great job! If you got an incorrect answer, where did the process break down? Did you recall that DNA polymerase II does not have 5’ to 3’ exonuclease activity? Did you understand that not all DNA polymerases are involved in DNA synthesis?
61) C
Version 1
55
Clarify Question ● What is the key concept addressed by the question? question asks about identifying the origin of replication.
● The
● What type of thinking is required? ● You are being asked to analyze data to identify the origin of replication. Gather Content ● What do you know about origins of replication? What other information is related to the question? ● Replication in E. coli initiates at a specific site, the origin, and ends at a specific site, the terminus. After initiation, replication proceeds bidirectionally from this unique origin to the unique terminus. Because this replication is bidirectional, regions on either side of the origin will be replicated at about the same time. Choose Answer ● Given what you now know, what information is most likely to produce the correct answer? ● Replication begins in origin C and proceeds bidirectionally. This is why replication is seen in Regions D and B at approximately the same time, but it takes longer to see replication in Region A. Reflect on Process ● Did your problem-solving process lead you to the correct answer? If not, where did the process break down or lead you astray? How can you revise your approach to produce a more desirable result? ● This question asked you to analyze data to identify the origin of replication. If you got the correct answer, great job! If you got an incorrect answer, where did the process break down? Did you recall that replication is bidirectional? Did you understand that regions further from the origin Version 1
56
will take longer to replicate?
62) B
Version 1
57
Clarify Question ● What is the key concept addressed by the question? ● The question asks about blocking reinitiation of DNA replication. ● What type of thinking is required? ● You are being asked to analyze the need to block reinitiation of DNA replication. Gather Content ● What do you know about blocking re-initiation of DNA replication? What other information is related to the question? ● Although most DNA polymerases can unwind DNA during synthesis, replication is more efficient if the helix is unwound ahead of the polymerase. An enzyme with DNA-unwinding activity is called a helicase. The replication machinery of eukaryotes is similar to that found in bacteria, but it is larger and more complex. The initiation phase of replication requires more factors to assemble both helicase and primase complexes onto the template, then load the polymerase with its sliding clamp unit. Choose Answer ● Given what you now know, what information is most likely to produce the correct answer? ● Preventing the reloading of the helicase onto a region where the DNA has already been replicated is how the cell ensures that replication happens once and only once per cell cycle. While it is true that there are multiple origins of replication, preventing the reloading of the helicase does not ensure that there are multiple origins of replication. There are prereplicative complexes loaded at most, if not all, origins of replication at the beginning of S phase. While it is true that telomeres need to be replicated properly in some cell types, preventing the reloading of the helicase is irrelevant to this process. As well, DNA repair processes can occur throughout the Version 1
58
cell cycle. Preventing the reloading of the helicase does not restrict when DNA repair can occur. Reflect on Process ● Did your problem-solving process lead you to the correct answer? If not, where did the process break down or lead you astray? How can you revise your approach to produce a more desirable result? ● This question asked you to analyze the need to block re-initiation of DNA replication. If you got the correct answer, great job! If you got an incorrect answer, where did the process break down? Did you recall that a region of DNA should only be replicated once in each cell division? Did you understand that by preventing the helicase from binding to the origin again, DNA replication would be blocked?
63) B
Version 1
59
Clarify Question • What is the key concept addressed by the question? o This question addresses the function of photolyase. • What type of thinking is required? o Although you may not have seen this example before, you know enough about the concept to apply your knowledge and understanding to this unfamiliar situation. • What key words does the question contain? o Photolyase is an enzyme in the photorepair pathway. Gather Content • What do you already know about photolyase? o Photolyase is an enzyme in the photorepair pathway. o It recognizes thymine dimers–the main damage caused by UV light– and repairs them. Consider Possibilities • Consider the different answer options. Which can you rule out? o Does photolyase lengthen the tips of chromosomes? No, that’s telomerase. o Does photolyase stabilize single-stranded DNA? No, those are the SSBs, single-stranded DNA binding proteins. o Does photolyase recognize damaged DNA with the UvrABC complex? Close, but that is a different DNA repair pathway. Choose Answer • Given what you now know, what information and/or problem Version 1
60
solving approach is most likely to produce the correct answer? o Without photolyase, the organism would be less equipped to repair UV-induced DNA damage. Reflect on Process • Did your problem-solving process lead you to the correct answer? If not, where did the process break down or lead you astray? How can you revise your approach to produce a more desirable result? o This question required you to apply your knowledge and understanding to this unfamiliar situation. o Did you remember that photolyase recognizes and fixes the DNA damage caused by UV light?
Version 1
61
CHAPTER 15 CHECK ALL THE APPLY. Choose all options that best completes the statement or answers the question. 1) A difference in transcription in eukaryotes that is not present in prokaryotes is ______. Check all that apply. A) eukaryotes have many transcription factors to recognize promoters B) eukaryotes have well-defined RNA polymerase II termination sites C) many eukaryotic genes have a paused polymerase D) RNA polymerase binds to a promoter to initiate transcription E) RNA polymerase recruits elongation factors via its carboxyl terminal domain (CTD)
MULTIPLE CHOICE - Choose the one alternative that best completes the statement or answers the question. 2) Gene expression involves two phases, translation and _____________. A) replication B) transcription C) initiation D) condensation
3)
Messenger RNA molecules contain information that is usedto synthesize___________. A) nucleotides B) amino acids C) polypeptides D) fatty acids
4)
The strand of DNA that is not transcribed is called the ______.
Version 1
1
A) coding strand B) noncoding strand C) template strand D) complementary strand
5) During this process, a ribosome assembles a polypeptide whose amino acid sequence is specified by the nucleotide sequence in a molecule ofmRNA. A) Transcription B) Translation C) Replication D) Posttranscriptional modification
6) Crick and his colleagues proposed that the genetic code consists of a series of blocks of information, each corresponding to one amino acid in an encoded protein. What were these blocks of information called? A) Alleles B) Codons C) Genes D) Polypeptides
7) During transcription of mRNA in eukaryotes, some sequences are cut out of the primary transcript and the remaining sequencesare joined together. This processing of mRNA is called _________. A) termination B) translation C) splicing D) capping E) elongation
Version 1
2
8) To remove noncoding sequencesin the pre-mRNA of eukaryotes, multiple snRNPs combine with proteins to form a larger complex called the ___________ . A) 5' cap B) introsome C) ribosome D) spliceosome E) 3' poly-A tail
9) The connection that exists between genes and hereditary traits is based onusing the information encoded in genes to synthesize A) codons. B) nucleotides. C) proteins. D) histones. E) complementary bases.
10)
Both DNA and RNA are made up of building blocks known as A) nucleotides. B) nucleicacids. C) amino acids. D) genes. E) codons.
11)
The one-gene/one-enzyme hypothesis was proposed by
Version 1
3
A) Watson and Crick. B) Griffith. C) Garrod. D) Franklin. E) Beadle and Tatum.
12) The polypeptide-making organelles, which consist of protein combined withRNA, are called A) ribosomes. B) Golgi bodies. C) lysosomes. D) centrosomes. E) mitochondria.
13)
During translation, amino acids are carried to the ribosome by A) mRNA. B) tRNA. C) snRNA. D) rRNA. E) miRNA.
14) During ______, RNA polymerase synthesizes a molecule ofRNA usingDNA as a template. A) MRNA splicing B) translation C) transcription D) gene sequencing E) termination
Version 1
4
15)
Which base in an anticodon will pair with the baseadenine in a codon? A) Thymine B) Cytosine C) Guanine D) Uracil
16)
In eukaryotes, translation takes place A) on the plasmamembrane. B) inside thenucleus. C) on ribosomes. D) on the nuclearmembrane. E) onspliceosomes.
17)
Ribosomes are complex aggregates of A) RNA and DNA. B) RNA andproteins. C) RNA and sugars. D) DNA and proteins. E) nucleosomes and RNA.
18) The A, P, and E sites are progressively occupied by amino acids being assembled into a polypeptide. These sites are part of A) DNA. B) the largeribosomal subunit. C) mRNA. D) tRNA. E) thespliceosome.
Version 1
5
19)
In eukaryotic cells, transcription occurs A) on the surface ofthe nuclear membrane. B) on ribosomes. C) onspliceosomes. D) inside thenucleus. E) on the surface ofthe plasma membrane.
20) In prokaryotes, the form of RNA polymerase that canaccurately initiate synthesis of RNA is called A) theholoenzyme. B) the corepolymerase. C) RNA polymeraseII. D) RNA polymeraseIII. E) the sigmasubunit.
21)
The genetic code uses how manynucleotide(s) to specify oneamino acid. A) 1 B) 2 C) 3 D) 4 E) 64
22) How many unique mRNA codons can be constructed from the four different RNA nucleotides?
Version 1
6
A) 4 B) 12 C) 16 D) 61 E) 64
23) During translation, the nucleotides that make up themRNA are read in groups of three. These groups arecalled A) codons. B) anticodons. C) exons. D) introns. E) templates.
24)
The tRNA nucleotide sequence that pairs withbases on the mRNA is called a(n) A) intron. B) exon. C) codon. D) initiationfactor. E) anticodon.
25)
Ribosome movement along the mRNA is called A) transcription. B) initiation. C) replication. D) translocation. E) activation.
Version 1
7
26)
Specific amino acids are attached to tRNA molecules by A) aminoacyl-tRNAsynthetases. B) hydrogenbonds. C) anticodons. D) deactivatingenzymes. E) initiation factors.
27)
Codons that serve as stop signals for translation are recognized by A) tRNA. B) releasefactors. C) anticodons. D) translationterminators. E) aminoacyl-tRNA synthetases
28) When a polypeptide is being assembled, the bond that forms between a newly added amino acid and the previous amino acid in the chain is a ______ bond. A) hydrogen B) hydrophobic C) terminal D) phosphodiester E) peptide
29) Eukaryotic mRNA molecules may contain noncoding sequences that must be removed before translation. These are called
Version 1
8
A) anticodons. B) introns. C) exons. D) nucleosomes. E) noncodons.
30)
The location of translation in prokaryotic cells is A) in thenucleoid. B) on ribosomes. C) on the plasmamembrane. D) on mesosomes. E) on chromosomes
31)
During translation, uncharged tRNA molecules leave the ribosome from the _____ site. A) E B) P C) A D) termination E) release
32)
The central dogma of biology can bestated as A) proteins→ RNA→ DNA. B) RNA→ DNA→ proteins. C) DNA→ proteins→ RNA. D) DNA→ RNA→ proteins.
33) If the sequence of bases in the template strandof a DNA molecule is 3' ATCGCTCC 5', what is the sequence of bases in the RNA that is transcribed from this molecule? Version 1
9
A) 3' UAGCGAGG 5' B) 3' TAGCGAGG 5' C) 5' UAGCGAGG 3' D) 5' TAGCGAGG 3' E) 5' AUCGCUCC 3'
34) The template strand of aDNA segment that codes for mRNA has the sequence: ATGCGT. Which tRNA anticodons would pair with the mRNA that is coded for by this sequence? A) AUG CGU. B) ATG CGT. C) UAC GCA. D) UAG CGU.
35) Eukaryotic and prokaryotic organisms differ in how they process genetic information. Which statements best explain one of these differences? A) In prokaryotes, translation of the mRNA begins before transcription is complete. Ineukaryotes, transcription and modification of themRNA is completed before translation begins. B) In prokaryotes, genes are transcribed directly into polypeptides. In eukaryotes, genes are transcribed into RNA that is used to assemblepolypeptides. C) In prokaryotes, translation occurs before genes aretranscribed into mRNA. In eukaryotes, genes are transcribed into mRNA that is then translated into polypeptides. D) In prokaryotes, introns are removed before genes are transcribed into mRNA. In eukaryotes, introns are removedafter genes are transcribed into mRNA.
36)
What is the first step during transcription initiation in prokaryotes?
Version 1
10
A) The transcription bubble is formed. B) RNA polymerase binds to the promoter. C) The DNA double helix is unwound. D) RNA polymerase synthesizes a short primer. E) Transcription factors bind to the TATA box sequence.
37) Initiation of transcription differs from initiation of DNA replication in several ways. One difference is that initiation of transcription does not require A) a promoter. B) enzymes. C) a primer. D) a DNA template strand.
38) Transcription in prokaryotes is carried out by ______, which unwinds and transcribes the gene. A) RNA synthetase B) RNA polymeraseII C) RNA polymeraseIII D) RNA polymerase holoenzyme
39)
Eukaryotes have how many different types of RNA polymerase enzymes? A) 2 B) 3 C) 4 D) 1 E) 64
40)
In eukaryotes, each type of RNA polymerase recognizes a different
Version 1
11
A) start codon. B) stop codon. C) promoter. D) releasefactor. E) transcriptionfactor.
41)
Eukaryotic pre-mRNA molecules are modified A) in the cytoplasm. B) at the ribosome. C) inside thenucleus. D) as they passthroughthe nuclear membrane. E) at thetranscription bubble.
42)
In eukaryotes, the 3' poly-A tail is attached to A) poly-Apolymerase. B) mRNA. C) tRNA. D) the ribosome. E) the templatestrand of DNA.
43) Why are there fewer tRNA anticodons than the 61 needed to match each mRNA codon that codes for an amino acid?
Version 1
12
A) There is someflexibility in pairing between the 5' base of the codon and the 3' base of theanticodon. B) There is someflexibility in pairing between the middle baseof the codon and the middlebase of the anticodon. C) There is someflexibility in pairing between the 3' base of the codon and the 5' base of theanticodon. D) There is someflexibility in pairing between all 3bases of the codon and all 3 bases ofthe anticodon.
44)
During translation, translocation refers to A) releasing a tRNAmolecule from the ribosome. B) joining an aminoacid to a tRNA molecule. C) joining an aminoacid to the next amino acid in the chain. D) joining a tRNAmolecule to the ribosome. E) movingtheribosome along the mRNA molecule.
45)
What happens during RNA splicing in eukaryotes?
A) The product oftranslation, called the primary transcript, is cut and some pieces arejoinedback together to form the mature mRNA. B) The product oftranscription, called the primary transcript, is cut and some pieces are joinedback together to form the mature tRNA. C) The product oftranscription, called the secondary transcript, is cut and some piecesarejoined back together to form the mature mRNA. D) The product oftranscription, called the primary transcript, is cut and some piecesarejoined back together to form the mature mRNA. E) The product oftranscription, called the primary transcript, is cut and all piecesarejoined back together to form the mature mRNA.
46) During protein synthesis in eukaryotes, which molecule passes from the nucleus to the cytoplasm and specifies the sequence of amino acids in the newpolypeptide?
Version 1
13
A) DNA B) RNA polymerase C) mRNA D) rRNA E) tRNA
47) Which molecule combines with proteins to form both the large and small ribosomal subunits? A) DNA B) RNA polymerase C) miRNA D) rRNA E) tRNA
48) Given the sentence "THE FAT CAT ATE THE RED RAT," which of the following would represent a frameshift mutation? A) THE FAT RAT ATETHE RED CAT B) THE CAT ATE THE RED RAT C) THE FAC ATA TET HER EDR AT D) THE FAT CAT ATE THERED RAT
49)
What would happen if snRNAs did not recognize the branch point within an intron? A) A lariat wouldnot form. B) snRNAs would not base pair with the 5' end of the intron. C) A 3' poly A tailwould not be added to the transcript. D) A 5' cap wouldnot be added to the transcript.
50)
During the splicing reaction, the intron-exon junctions are recognized by
Version 1
14
A) snRNPs. B) miRNAs. C) SRP RNAs. D) the lariat. E) the branch point.
51)
In prokaryotes, the RNA polymerase holoenzyme consists of A) the core polymeraseplus two alpha subunits. B) the core polymeraseplus two beta subunits. C) the core polymeraseplus two alpha subunits, two beta subunits, and a sigma subunit. D) the corepolymerase plus a sigma subunit. E) two alpha subunits, two beta subunits,and two sigma subunits.
52) Two 6-base sequences are present inbacterial promoters: TATAAT (located 10 nt upstream from the start site) and TTGACA (located 35 nt upstream from the start site). What is the significance of the fact that these two base sequences are different? A) Binding sites forboth the holoenzyme and ATP are provided. B) Both the locationof the start site and the direction of transcription can be established. C) Binding sites forboth the core polymerase and holoenzyme are provided. D) The transcription bubble can be properly formed. E) It allows RNA polymerase to distinguishbetween the template strand and the coding strand of the DNA molecule.
53) Within the transcription bubble, the 9 most recently added nucleotidesin the newly synthesized RNA strand temporarily form a helix with the template DNA strand. How might transcription be affected if helix formation did not occur?
Version 1
15
A) Rewinding the DNAmolecule wouldbe inhibited. B) Unwinding the DNAmolecule wouldbe inhibited. C) The position of the5' end of the RNA would be unstable, inhibiting elongation. D) The position of the3' end of the RNA would be unstable, inhibiting elongation. E) The position of the 5' end of the RNAwould be unstable, stimulating elongation.
54) Cells conserve energy and resources by making active proteins only when they are needed. If a protein is not needed, which of the following methods of control would bethe most energy efficient? A) Block transcription B) Degrade the mRNA after it is made C) Prevent translation of the mRNA D) Degrade the protein after it is made
55)
What is required for formation of the transcription initiation complex in eukaryotes?
A) Binding of a transcription factor to the TATA box, followed by recruitment of additional transcription factors and recruitment of RNA polymerase II B) Binding of a transcription factor to the transcription bubble, followed by recruitment of additional transcription factors and recruitment of RNA polymerase III C) Binding of the sigma subunit to the start sitefollowed by recruitment of RNA polymerase II D) Binding of RNA polymerase II to the TATA box, followed byrecruitment of transcription factors E) Binding of the sigma subunit to promoter elements at –35 and –10, followed by recruitment of the core polymerase
56)
What would happen if snRNPs did not recognize the branch point within an intron?
Version 1
16
A) A lariat wouldnot form. B) snRNPs would not base-pair with the 5' end of the intron. C) A 3' poly A tailwould not be added to the transcript. D) A 5' cap wouldnot be added to the transcript.
57) What is the likely consequence of a mutation that alters the branch point within an intron? A) No effect, since introns are not expressed B) Failure to form a lariat C) Failure of snRNPs to recognize the5' end of intron D) No exon shuffling E) Failure of snRNAs to combine with protein and form snRNPs
58) You are working to characterize a novel protein in mice. Analysisshows that high levels of the primary transcript that codes for this protein are found in tissue from thebrain, muscle, liver, and pancreas. However, an antibody that recognizes the C-terminal portion of the protein indicates thatthe protein is present inbrain, muscle, and liver, but not inthe pancreas. What is the most likely explanation for this result? A) The gene that codes for this protein is not transcribed in the pancreas. B) The transcript is immediately degraded in the pancreas. C) The transcript receives a 5’ cap in the pancreas. D) Alternative splicing in the pancreas creates a protein missing the domain that the antibody recognizes. E) The transcript receives a 3’ poly-A tail in the pancreas.
59)
The mutation responsible for Huntington's disease is a
Version 1
17
A) missensemutation. B) nonsense mutation. C) frameshift mutation. D) triplet repeat expansion mutation.
60) How would a large chromosomal inversion affect the expression of a gene if the gene is located between the two break points but no breaks occur within the gene? A) The inversionwould probablyhave no effect on gene expression. B) The gene would notbe transcribed because it would be oriented in the wrong direction. C) The gene would betranscribedin the 3' to 5' direction. D) The gene would be transcribednormally, but the mRNA would be translated in the 3' to 5' direction.
61)
How does DNA polymerase differ fromRNA polymerase?
A) Only RNA polymeraseadds new nucleotides to the 3' end of a growing chain. B) Only RNA polymeraserequires a primer. C) Only DNA polymeraseuses a template DNA strand to direct synthesis of a new nucleotide strand. D) Only DNApolymerase has a proofreading ability.
Version 1
18
62) A scientist makes three artificial mRNA strands: AAAUUUAAAUUUAAAUUUAAAUUUAAA 3' (y) 5' UUUCCCUUUCCCUUUCCCUUUCCCUUU 3' (z) 5' AUAUAUAUAUAUAUAUAUAUAUAUAU 3'
(x) 5'
When she analyzes the polypeptides produced, she finds that: x produces a polypeptide that is50% phenylalanine and 50% lysine. y produces a polypeptide that is 50% phenylalanine and 50% proline. z produces a polypeptide that is 50% isoleucineand 50% tyrosine. Based on these results only, the best conclusion to make is that
A) AUA codes for isoleucine. B) AAA codes for phenylalanine. C) AAA codes for lysine. D) AAA codes for lysine and AUA codes for isoleucine. E) AAA codes for phenylalanine and AUA codes for isoleucine.
63) A bacterial cell has a nonsense mutation that prevents it from producing a functionalsigma subunit for RNA polymerase. Inability tosynthesize a functional sigma subunitwould have the most direct effect on A) transcriptioninitiation. B) transcriptionelongation. C) transcriptiontermination. D) translationinitiation. E) translation termination.
64) What is the best way to describe our current understanding of the one-gene/onepolypeptide hypothesis?
Version 1
19
A) It applies to bothprokaryotes and eukaryotes. B) It applies toprokaryotes but not to eukaryotes. C) It applies toeukaryotes but not to prokaryotes. D) It has beenreplaced by the one-gene/one-enzyme hypothesis.
65) You areattempting to synthesize rRNA in a test tube using DNA isolated from mouse cells. In addition to the template DNA, ribonucleotides, and the necessary transcription factors, you should also add _________ to the test tube. A) poly-Apolymerase B) RNA polymeraseIII C) RNA polymeraseII D) RNA polymeraseI
66) You are studying an individual with very low levels of insulin in her blood. Further analysis indicates that cells of her pancreas are producing normal levels of this protein, but it is accumulating in the cytoplasm rather than being secreted from the cells. Which hypothesis makes the most sense to explain this observation? A) A small deletion has removed the nucleotides that code forthe signal sequence at the amino terminus of the protein. B) A missense mutationhas caused premature termination during translation of this protein. C) A chromosomalsegment that includes the gene for insulin has been inverted. D) A two-basedeletionnear the middle of the gene has altered the reading frame duringtranslation of the protein. E) A missense mutation has altered theribosome-binding sequence at the 5' end of the mRNA.
67) A gene that codes for a protein was removed from a eukaryotic cell and inserted into a prokaryotic cell. Although the gene was successfully transcribed and translated, it produced a differentprotein than it produced in the eukaryotic cell. What is the most likely explanation?
Version 1
20
A) There are slightdifferences in the genetic code for prokaryotes and eukaryotes. B) Unlike eukaryotes,which have three differentRNA polymerases, prokaryotes have a single RNApolymerase. C) Eukaryotic genesoften contain introns while prokaryotic genes do not. D) Eukaryotictranscripts have a 5' cap while prokaryotic transcripts do not.
68) A mutation has occurred in the sequence in the promoter region of a bacterial gene, such that the sequence TTGACA has been lost. What process will be most directly affected by this mutation? A) Transcription initiation B) Splicing C) Translation initiation D) RNA polymerase II binding
69) In eukaryotic cells, the 5' cap is bound to a protein complex called cap binding complex. In addition, during the splicing reaction, proteins are deposited at sites where the exons are fused together. These proteins, in combination with the cap binding complex, are essential for the recruitment of an additional complex of proteins that is essential for mRNA export from the nucleus to the cytoplasm. This complex is called the transcription-export, or TREX, complex. Taking this into account, would an mRNA encoded by a virus infecting a eukaryotic cell need to bind to the TREX complex? A) Yes, mRNAs encoded by viruses infecting eukaryotic cells would still need to move from the nucleus to the cytoplasm to be translated. B) No, only eukaryotic mRNAs are modified, and therefore viral mRNAs do not need to bind to the TREX complex. C) Yes, mRNAs encoded by viruses infecting eukaryotic cells would need to bind these complexes to allow RNA processing events to occur. D) No, mRNAs encoded by viruses infecting eukaryotic cells can be translated in the nucleus of these cells.
Version 1
21
70) A particular tRNA molecule includes the anticodon sequence AUG. What would be the corresponding sequence in the noncoding strand of the DNA molecule that encodes the mRNA that this tRNA would pair with? A) TAC B) ATG C) AUG D) UAC
71) You are interested in developing a new antibiotic for treating bacterial illnesses in humans, which is specifically toxic to bacterial cells. What processes or proteins would you want to inhibit in bacteria to generate this antibiotic? A) The enzymes that add a 5' cap to the RNA B) The SRP receptor C) RNA polymerase II D) Sigma factor binding to the holoenzyme
72)
What protein would be the most likely to be expected to contain a signal sequence? A) An antibody B) SRP C) Aminoacyl-tRNA synthetase D) Sigma subunit E) Phosphofructokinase
73) Which of the following occurs in prokaryote translation initiation but NOT eukaryotic initiation?
Version 1
22
A) The initiating amino acid is methionine in prokaryotes. B) The initiation complex is more complicted in prokaryotes. C) Translation is not energy intensive in prokaryotes. D) The initiaton complex includes a modified molecule called N-formylmethionine in prokaryotes.
SECTION BREAK. Answer all the part questions. 74) Shown below is a hypothetical DNA sequence from a virus. Also shown is the sequence of the mRNA that is synthesized from this DNA. DNA sequence: 5'-AGCACCTGCCGAATGGGCCAAATCCTGCCGAATAAA-3' 3'-TCGTGGACGGCTTACCCGGTTTAGGACGGCTTATTT -5' mRNA sequence (G* = G cap): 5'-G*AGCACCUGCCGCCUGCCGAAUAAAAAAA....-3'
74.1)
What is the base sequence of the intron in the template strand of the DNA?
A) 3'-TCGTGGACGGC-5' B) 3'-TTACCCGGTTTA-5' C) 3'-GGACGGCTTATTT-5' D) 3'-GCTTACCCGGTT-5'
74.2)
The RNA was most likely transcribed by
A) RNA polymeraseholoenzyme. B) RNA polymeraseI. C) RNA polymeraseII. D) RNA polymeraseIII.
Version 1
23
Answer Key Test name: Chap 15_3e 1) [A, C, E] 2) B 3) C 4) A 5) B 6) B 7) C 8) D 9) C 10) A 11) E 12) A 13) B 14) C 15) D 16) C 17) B 18) B 19) D 20) A 21) C 22) E 23) A 24) E 25) D 26) A Version 1
24
27) B 28) E 29) B 30) B 31) A 32) D 33) C
Version 1
25
Clarify Question ● What is the key concept addressed by the question? question addresses transcription.
● This
● What type of thinking is required? ● This question is asking you to analyze the information given, using logic, to dissect the problem and determine the answer. ● What key words does the question contain and what do they mean? ● Template strand—DNA strand used to make the RNA copy, RNA is paired up with the template strand, so it is complementary to it, not the same ● 3’ and 5’—designations show the directionality of each DNA strand Gather Content ● What do you know about transcription? What other information is related to the question? ● Transcription is the process of making an RNA copy off the DNA. After the DNA opens up, one strand is the coding strand, the other is the template strand. ● The template strand is the one directly used to make the RNA, by base pairing. Therefore it is the complementary sequence to the RNA, not the same sequence. Choose Answer ● Given what you now know, what information is most likely to produce the correct answer? ● We know that the RNA is produced by base pairing to the tempate strand, with strand directionality in the opposite direction, and using Us instead of Ts. ● Thus the RNA will be 5’ UAGCGAGG 3’. Reflect on Process Version 1
26
● Did your problem-solving process lead you to the correct answer? If not, where did the process break down or lead you astray? How can you revise your approach to produce a more desirable result? ● If you got the correct answer, great job! If you got an incorrect answer, where did the process break down? Did you recognize that the RNA is complementary to the template strand, not the same? Did you remember that RNA contains U, not T? Did you remember that the paired strands are oriented in opposite directions, so the RNA should read 5’ to 3’?
34) A
Version 1
27
Clarify Question ● What is the key concept addressed by the question? question addresses transcription and translation.
● This
● What type of thinking is required? ● This question is asking you to analyze the information given, using logic, to dissect the problem and determine the answer. ● What key words does the question contain and what do they mean? ● Template strand—DNA strand used to make the RNA copy, RNA is paired up with the template strand, so it is complementary to it, not the same ● tRNA anticodons—three nucleotide sequences that base pair with each codon of the mRNA Gather Content ● What do you know about transcription and translation? What other information is related to the question? ● You know that there are several steps from DNA to tRNAs. The DNA is transcribed into mRNA, then the mRNA is translated by the ribosome using tRNAs whose anticodons base pair with the mRNA. ● What is different about the bases in RNA compared to DNA? RNA uses uracil (U), not thymine (T). This is true for tRNA as well as mRNA. So you can rule out the answer with T in the RNA. Choose Answer ● Given what you now know, what information is most likely to produce the correct answer? ● What specifically are you being asked to describe? The question is asking you to name the tRNA anticodons that relate to the given DNA template strand. So you need to determine the corresponding mRNA, and then determine which anticodons would Version 1
28
pair with the mRNA. ● Is an mRNA the same as its DNA template strand, or complementary to it? The mRNA is complementary to the sequence of the template strand, because it is transcribed by base-pairing to it. So the mRNA is UACGCA. ● The tRNAs have to base pair to the mRNA. So the tRNAs would read AUG CGU. Reflect on Process ● Did your problem-solving process lead you to the correct answer? If not, where did the process break down or lead you astray? How can you revise your approach to produce a more desirable result? ● If you got the correct answer, great job! If you got an incorrect answer, where did the process break down? Did you recognize that the tRNAs would be complementary to the mRNA? Did you remember that RNAs always use U instead of T?
35) A 36) B 37) C 38) D 39) B 40) C 41) C 42) B 43) C 44) E 45) D 46) C
Version 1
29
47) D 48) C
Version 1
30
Clarify Question ● What is the key concept addressed by the question? question addresses frameshift mutations.
● This
● What type of thinking is required? ● This question is asking you to take what you already know and apply it to this unfamiliar situation. ● What key words does the question contain and what do they mean? ● Frameshift mutation—change in the DNA that shifts the reading frame Gather Content ● What do you know about frameshift mutations? What other information is related to the question? ● A frameshift mutation causes the bases to be read in different groups of three, so that the new codons no longer encode the same amino acids. ● An insertion or deletion of nucleotides will cause a frameshift– unless it happens to insert or delete three nucleotides or a multiple of three. Choose Answer ● Given what you now know, what information is most likely to produce the correct answer? ● What is a frameshift mutation in DNA? An insertion or deletion that shifts the reading frame. ● What is the result of a frameshift mutation? It changes the codons downstream of the mutation, usually into gibberish (wrong amino acids that don’t make a functioning protein). ● So the answer option THE FAC ATA TET HER EDR AT is a good representation of a frameshift mutation. A single nucleotide deletion causes a shift in the frame that turns the codons downstream into gibberish. Version 1
31
Reflect on Process ● Did your problem-solving process lead you to the correct answer? If not, where did the process break down or lead you astray? How can you revise your approach to produce a more desirable result? ● If you got the correct answer, great job! If you got an incorrect answer, where did the process break down? Did you recognize that the key feature of a frameshift mutation is that it shifts the reading frame, messing up all the codons downstream?
49) A
Version 1
32
Clarify Question ● What is the key concept addressed by the question? question asks about the role of snRNAs in splicing.
● The
● What type of thinking is required? ● You are being asked to apply your knowledge of snRNAs in splicing. Gather Content ● What do you know about splicing? What other information is related to the question? ● The splicing process begins with when snRNAs recognize an A at the branch point and 5′ end of the intron. This 5′ end becomes attached to the 2′ OH of the branch point A, forming a branched structure called a lariat due to its resemblance to the noose in a cowboy’s lariat. The 3′ end of the first exon is then used to displace the 3′ end of the intron, joining the two exons together and releasing the intron as a lariat. Choose Answer ● Given what you now know, what information is most to produce the correct answer? ● If the branch point is not recognized, the 2’ OH of the intron will not be able to attach the 3’ end of the intron and a lariat would not form. The snRNAs are not involved in the 5’ cap or 3’ poly A tail.The lariat is at the 3’ end of the intron, not the 5’ end. Reflect on Process ● Did your problem-solving process lead you to the correct answer? If not, where did the process break down or lead you astray? How can you revise your approach to produce a more desirable result? ● This question asked you to apply your understanding of your knowledge of snRNAs in splicing. If you got the correct answer, great job! If you got an incorrect answer, where did the process break down? Did you recall Version 1
33
that the branch point is involved in formation of a lariat?Did you understand that the 5’ and 3’ modifications of the mRNA do not involve the branch point?
50) A 51) D 52) B 53) D 54) A
Version 1
34
Clarify Question ● What is the key concept addressed by the question? question addresses transcription and translation.
● This
● What type of thinking is required? ● This question is asking you to analyze the information given, using logic, to dissect the problem and determine the answer. ● What key words does the question contain and what do they mean? ● Energy—in a cell, refers to chemical energy, stored in molecules like ATP ● Proteins—final output from the transcription and translation of DNA; they are made of amino acids ● Energy efficient—energy saving Gather Content ● What do you know about transcription and translation? What other information is related to the question? ● Transcription is the process of turning the stored information from a DNA sequence and turning it into a temporary mRNA copy. ● Translation takes the information from mRNA and turns it into a protein, or polypeptide–a string of amino acids. ● What is meant by energy efficient? How would making an unnecessary protein waste energy? Because both transcription and translation require energy in the form of ATP to drive the enzymes to carry out the work. They also use up ribonucleotides and amino acids, which were produced using more energy from ATP. Choose Answer ● Given what you now know, what information is most likely to produce the correct answer? ● Only by blocking transcription can the Version 1
35
cell save all that energy. If the gene is never transcribed, then ATP, ribonucleotides, and amino acids are all saved. Reflect on Process ● Did your problem-solving process lead you to the correct answer? If not, where did the process break down or lead you astray? How can you revise your approach to produce a more desirable result? ● If you got the correct answer, great job! If you got an incorrect answer, where did the process break down? Did you recognize that blocking transcription would save the most energy? Despite this, cells do often regulate at the level of translation as well. Why do you think this might benefit a cell despite the wasted energy?
55) A 56) A
Version 1
36
Clarify Question ● What is the key concept addressed by the question? question addresses splicing.
● This
● What type of thinking is required? ● This question is asking you to analyze the information given, using logic, to dissect the problem and determine the answer. ● What key words does the question contain and what do they mean? ● snRNPs—small nuclear ribonucleoprotein particles, the complexes that carry out splicing ● Intron—portion of the sequence of a pre-mRNA that is removed by splicing ● Branch point—a particular adenosine nucleotide that lies within the intron and is recognized by snRNPs during the splicing process Gather Content ● What do you know about splicing? What other information is related to the question? ● Splicing is the process that removes introns from pre-mRNA and is carried out by the spliceosome, a complex formed of snRNPs. The snRNPs bind to specific sites on the 5’ end of the intron and to the branch point near the 3’ end. The snRNPs come together, forming the intron into a lariat, which is cut as the exons are joined together. Choose Answer ● Given what you now know, what information is most likely to produce the correct answer? ● The snRNPs bind at the 5’ end and at the branch point near the 3’ end. Then by coming together, the two sets of snRNPs create the lariat which is cleaved as the exons are ligated. ● If the snRNPs couldn’t recognize the branch point, they could still Version 1
37
recognize and bind the 5’ end. ● However, the lariat would fail to form. Reflect on Process ● Did your problem-solving process lead you to the correct answer? If not, where did the process break down or lead you astray? How can you revise your approach to produce a more desirable result? ● If you got the correct answer, great job! If you got an incorrect answer, where did the process break down? Did you recognize that the branch point is near the 3’ end? Did you recognize that binding the branch point is critical for lariat formation?
57) B
Version 1
38
Clarify Question ● What is the key concept addressed by the question? question addresses splicing.
● This
● What type of thinking is required? ● This question is asking you to analyze the information given, using logic, to dissect the problem and determine the answer. ● What key words does the question contain and what do they mean? ● Intron—portion of the sequence of a pre-mRNA that is removed by splicing ● Branch point—a particular adenosine nucleotide that lies within the intron and is recognized by snRNPs during the splicing process Gather Content ● What do you know about splicing? What other information is related to the question? ● Splicing is the process that removes introns from pre-mRNA and is carried out by the spliceosome, a complex formed of snRNPs. The snRNPs bind to specific sites on the 5’ end of the intron and to the branch point near the 3’ end. The snRNPs come together, forming the intron into a lariat, which is cut as the exons are joined together. Choose Answer ● Given what you now know, what information is most likely to produce the correct answer? ● The snRNPs bind at the 5’ end and at the branch point near the 3’ end. Then by coming together, the two sets of snRNPs create the lariat which is cleaved as the exons are ligated. ● If the snRNPs couldn’t recognize the branch point, they could still recognize and bind the 5’ end. ● However, the lariat would fail to form. Version 1
39
Reflect on Process ● Did your problem-solving process lead you to the correct answer? If not, where did the process break down or lead you astray? How can you revise your approach to produce a more desirable result? ● If you got the correct answer, great job! If you got an incorrect answer, where did the process break down? Did you recognize that the branch point is near the 3’ end? Did you recognize that binding the branch point is critical for lariat formation?
58) D
Version 1
40
Clarify Question ● What is the key concept addressed by the question? ● This question addresses posttranscriptional regulatory mechanisms. ● What type of thinking is required? ● This question is asking you to weigh and judge evidence, or evaluate, to choose the best of the possible answers. ● What key words does the question contain and what do they mean? ● Novel protein—one that has just recently been discovered, and isn’t well understood by scientists ● Primary transcript—the initial pre-mRNA transcript before splicing takes place ● C-terminal—portion of a protein is the end near the final –COOH Gather Content ● What do you know about post-transcriptional regulatory mechanisms? What other information is related to the question? ● Cells use several kinds of post-transcriptional regulatory mechanisms. A gene can be transcribed or not in different cells, or the transcript can be degraded. Alternative splicing creates proteins with different domains. ● Most mRNAs receive a 5’ cap and poly-A tail to improve stability. Choose Answer ● Given what you now know, what information is most likely to produce the correct answer? ● What could explain why the antibody reacts with the protein in some cells but not others? Either the protein is not present in the pancreas cells or the protein is different and not recognizable by the antibody. ● What types of regulation lead to different sets of proteins in different cells? In addition to transcriptional regulation, alternative Version 1
41
splicing is a mechanism that ensures that different cell types produce specialized sets of proteins. ● The best explanation of the answer options is that alternative splicing produced a different form of the protein in pancreas cells that could not be recognized by the antibody. Reflect on Process ● Did your problem-solving process lead you to the correct answer? If not, where did the process break down or lead you astray? How can you revise your approach to produce a more desirable result? ● If you got the correct answer, great job! If you got an incorrect answer, where did the process break down? Did you recognize that the regulation must be post-transcriptional? Did you realize that antibodies may recognize only a portion of a protein?
59) D 60) A 61) D 62) C
Version 1
42
Clarify Question ● What is the key concept addressed by the question? question addresses the genetic code.
● This
● What type of thinking is required? ● This question is asking you to weigh and judge evidence, or evaluate, to choose the best of the possible answers. ● What key words does the question contain and what do they mean? ● Artificial mRNAs—similar to mRNAs in a cell, but they were designed by a scientist and produced in vitro. ● Polypeptides—proteins, in this question, resulting from translation of the artificial mRNAs Gather Content ● What do you know about the genetic code? What other information is related to the question? ● The genetic code was elucidated by experiments similar to this one. ● By producing artificial mRNAs with a defined sequence, Nirenberg and Khorana could infer the sequence of codons that encoded each amino acid. ● Is there enough information to say anything about what AUA encodes? No, because it could either be isoleucine or tyrosine. We don’t have enough information to distinguish between them. Choose Answer ● Given what you now know, what information is most likely to produce the correct answer? ● Is there enough information to say what AAA encodes? Yes. ● From experiment (x), we know that the AAA and UUU together make phenylalanine and lysine. But we don’t know which is which. Version 1
43
● From experiment (y), we know that the CCC and UUU together make phenylalanine and proline. But we don’t know which is which. ● Putting both together, we realize that UUU must encode phenylalanine, since that codon and that amino acid are in both experiments. ● That means that AAA must make the other amino acid from experiment (x), namely lysine. Reflect on Process ● Did your problem-solving process lead you to the correct answer? If not, where did the process break down or lead you astray? How can you revise your approach to produce a more desirable result? ● If you got the correct answer, great job! If you got an incorrect answer, where did the process break down? Did you recognize that you needed to use experiment (x) and (y) together to determine what AAA encoded?
63) A
Version 1
44
Clarify Question ● What is the key concept addressed by the question? question addresses RNA polymerase.
● This
● What type of thinking is required? ● This question is asking you to take what you already know and apply it to this unfamiliar situation. ● What key words does the question contain and what do they mean? ● Nonsense mutation—causes an early STOP codon, resulting in a protein that is shorter than normal ● RNA polymerase—the enzyme that transcribes DNA into mRNA ● Sigma (σ) subunit—subunit of RNA polymerase is important in transcription initiation Gather Content ● What do you know about RNA polymerase? What other information is related to the question? ● RNA polymerase is the enzyme that does transcription–it produces RNA. ● There are three main steps to transcription: initiation, elongation, and termination. ● Initiation requires the polymerase to recognize and bind to the promoter. Promoter binding is controlled by the σ subunit, which recognizes the–35 sequence in the promoter and positions the RNA polymerase at the correct start site, oriented to transcribe in the correct direction. Choose Answer ● Given what you now know, what information is most likely to produce the correct answer? ● Of the three aspects of transcription, loss of the sigma subunit is likely to disrupt initiation, because the sigma subunit is necessary to recognize and bind the promoter. Version 1
45
Reflect on Process ● Did your problem-solving process lead you to the correct answer? If not, where did the process break down or lead you astray? How can you revise your approach to produce a more desirable result? ● If you got the correct answer, great job! If you got an incorrect answer, where did the process break down? Did you recognize that RNA polymerase acts in transcription, not translation? Did you remember that the sigma subunit binds the promoter?
64) B 65) D
Version 1
46
Clarify Question ● What is the key concept addressed by the question? question addresses RNA polymerases.
● This
● What type of thinking is required? ● This question is asking you to take what you already know and apply it to this unfamiliar situation. ● What key words does the question contain and what do they mean? ● Ribosomal RNA (rRNA)—the RNA component of the ribosome ● Template DNA—the DNA that is used to transcribe an RNA ● Ribonucleotides—the building blocks of RNA ● Transcription factors—proteins that help carry out and regulate transcription Gather Content ● What do you know about RNA polymerases? What other information is related to the question? ● There are three kinds of RNA polymerases, which are the enzymes that carry out transcription: RNA Polymerase I, II, and III, each with specialized functions. ● RNA Polymerase I transcribes rRNA. ● RNA Polymerase II transcribes mRNA and snRNAs. ● RNA Polymerase III transcribes tRNA and some other small RNAs. Choose Answer ● Given what you now know, what information is most likely to produce the correct answer? ● To decide which RNA polymerase should be added, you must remember that they each have specialized target genes, and that RNA Polymerase I is the enzyme that transcribes rRNA. Version 1
47
Reflect on Process ● Did your problem-solving process lead you to the correct answer? If not, where did the process break down or lead you astray? How can you revise your approach to produce a more desirable result? ● If you got the correct answer, great job! If you got an incorrect answer, where did the process break down? Did you remember that RNA Polymerase I is specialized to transcribe rRNA?
66) A
Version 1
48
Clarify Question ● What is the key concept addressed by the question? question addresses transcription and translation.
● This
● What type of thinking is required? ● This question is asking you to analyze the information given, using logic, to dissect the problem and determine the answer. ● What key words does the question contain and what do they mean? ● Insulin—a protein produced by the pancreas, that plays a critical role in blood sugar regulation ● If the protein is accumulating in the cytoplasm rather than being secreted, that means it is building up inside the cells instead of being released from the cells Gather Content ● What do you know about transcription and translation? What other information is related to the question? ● First, a protein like insulin is transcribed into mRNA, then the mRNA is translated. ● A secreted protein like insulin will be synthesized on the rough ER (RER) so that the protein ends up in the lumen of the ER, ready for release by the cell. A signal sequence on the amino terminus of the polypeptide is responsible for recognition of the peptide by the SRP, and docking onto the RER. Choose Answer ● Given what you now know, what information is most likely to produce the correct answer? ● What specifically seems to be wrong with the insulin protein in this individual? The protein is being produced fine, just not secreted. ● What type of mutation could interfere with that specific aspect? A Version 1
49
mutation must have disrupted the signal sequence, which helps the growing peptide dock at the ER for later secretion. Reflect on Process ● Did your problem-solving process lead you to the correct answer? If not, where did the process break down or lead you astray? How can you revise your approach to produce a more desirable result? ● If you got the correct answer, great job! If you got an incorrect answer, where did the process break down? Did you recognize that lack of secretion indicates a problem with the signal sequence on the ER?
67) C
Version 1
50
Clarify Question ● What is the key concept addressed by the question? question addresses transcription and translation.
● This
● What type of thinking is required? ● This question is asking you to analyze the information given, using logic, to dissect the problem and determine the answer. ● What key words does the question contain and what do they mean? ● Eukaryotic cell—from an organism with a nucleus, like a human, fly, plant, or fungus ● Prokaryotic cell—a single-celled organism such as a bacterium like E. coli Gather Content ● What do you know about transcription and translation? What other information is related to the question? ● One of the main differences in transcription between eukaryotes and prokaryotes is that eukaryotic genes typically have introns, which are spliced out of the premRNA by specialized splicing machinery. Prokaryotes do not have introns or the machinery to splice them out. ● Could the different RNA polymerases explain the change in the protein? No, that doesn’t make sense. ● Could the 5’ cap explain the change in the protein? No, because that affects stability but not the sequence of the protein itself. Choose Answer ● Given what you now know, what information is most likely to produce the correct answer? ● The best explanation is that the eukaryotic gene contained introns. In a prokaryote they would not be removed, so the protein would have extra sections of translated Version 1
51
sequence. ● For this reason, scientists usually use a cDNA, or complementary DNA copy from a processed mRNA, rather than the DNA gene itself when they clone a eukaryotic gene into bacteria for protein expression. Reflect on Process ● Did your problem-solving process lead you to the correct answer? If not, where did the process break down or lead you astray? How can you revise your approach to produce a more desirable result? ● If you got the correct answer, great job! If you got an incorrect answer, where did the process break down? Did you recognize that prokaryotes lack the splicing machinery?
68) A
Version 1
52
Clarify Question ● What is the key concept addressed by the question? question asks about a mutation in the promoter.
● The
● What type of thinking is required? ● You are being asked to apply your knowledge of mutations in the promoter. ● What key words does the question contain and what do they mean? ● Mutation—any disruption to the sequence of the DNA ● Promoter—region of a gene (DNA) that forms a recognition and binding site for RNA polymerase Gather Content ● What do you know about the function of the promoter? What other information is related to the question? ● RNA polymerase binds to the promoter and initiates transcription of an RNA.Splicing occurs after RNA is made and translation occurs after splicing.RNA polymerase II binding is not affected. Choose Answer ● Given what you now know, what information is most likely to produce the correct answer? ● The loss of this sequence from the promoter will prevent proper binding of the RNA polymerase holoenzyme.As a result, transcription initiation will be most directly affected.Splicing and RNA polymerase II binding are not relevant to bacterial genes. Reflect on Process ● Did your problem-solving process lead you to the correct answer? If not, where did the process break down or lead you astray? How can you revise your approach to produce a more desirable result? ● If you got the correct answer, great job! If you got an incorrect answer, Version 1
53
where did the process break down? Did you recall that the promoter determines where transcription starts?Did you understand that translation and splicing do not involve the promoter.
69) A
Version 1
54
Clarify Question ● What is the key concept addressed by the question? question asks about the translation of viral genes. ● What type of thinking is required? analyze how viral genes are translated.
● The
● You are being asked to
Gather Content ● What do you know about translation of viral genes? What other information is related to the question? ● Viral mRNAs are made in the nucleus but still need to move into the cytoplasm to be translated. Because of this, viruses must use the existing cell transport systems to get their mRNA into the cytoplasm. Choose Answer ● Given what you now know, what information is most likely to produce the correct answer? ● If the virus is infecting a eukaryotic cell, the machinery needed to translate the viral genome is found in the nucleus. As a result, virally encoded mRNAs will need to find a way to be exported from the nucleus into the cytoplasm to be translated. Therefore, virally encoded mRNAs do need to bind to the TREX complex or find another way to be exported into the cytoplasm. Reflect on Process ● Did your problem-solving process lead you to the correct answer? If not, where did the process break down or lead you astray? How can you revise your approach to produce a more desirable result? ● This question asked you to analyze how viral genes are translated. If you got the correct answer, great job! If you got an incorrect answer, where did the process break down? Did you recall that viruses do not carry their own machinery for translation? Did you understand that translation Version 1
55
occurs in the cytoplasm?
70) B
Version 1
56
Clarify Question ● What is the key concept addressed by the question? ● The question asks about tRNA and the template strand of DNA. ● What type of thinking is required? ● You are being asked to analyze the relationship between the tRNA and template strand of DNA. Gather Content ● What do you know about the relationship between the tRNA and template strand of DNA? What other information is related to the question? ● If the tRNA sequence was AUG, then the mRNA codon would be UAC.The DNA template sequence would be ATG because DNA has a T where a U would be found in mRNA. Choose Answer ● Given what you now know, what information is most likely to produce the correct answer? ● This tRNA would complement an mRNA codon with the sequence UAC. That mRNA would be coded by a coding strand TAC. The noncoding strand would have the sequence ATG. Reflect on Process ● Did your problem-solving process lead you to the correct answer? If not, where did the process break down or lead you astray? How can you revise your approach to produce a more desirable result? ● If you got the correct answer, great job! If you got an incorrect answer, where did the process break down? Did you recall that the anticodon of the tRNA would have the complementary sequence to the mRNA?Did you understand that the template strand of the DNA would have the complementary sequence of the mRNA?
Version 1
57
71) D
Version 1
58
Clarify Question ● What is the key concept addressed by the question? ● The question asks about an antibiotic that specifically targets bacteria. ● What type of thinking is required? ● You are being asked to analyze which steps in transcription are specific to bacteria. Gather Content ● What do you know about which steps in transcription are specific to bacteria? What other information is related to the question? ● Bacteria do not have SRP receptors or 5’ caps on mRNA, so an antibiotic against these would not be specific to bacteria. Sigma factor is only found in bacteria. Choose Answer ● Given what you now know, what information is most likely to produce the correct answer? ● Sigma factor binding to the holoenzyme is the only choice that is specific to bacteria. All of these other processes/proteins are specific to eukaryotic cells.As a result, inhibiting these processes/proteins would be expected to affect the host's cells and would leave the bacterial cells intact.This is the opposite of what you would want an antibiotic to do. Reflect on Process ● Did your problem-solving process lead you to the correct answer? If not, where did the process break down or lead you astray? How can you revise your approach to produce a more desirable result? ● This question asked you to analyze which steps in transcription are specific to bacteria. If you got the correct answer, great job! If you got an incorrect answer, where did the process break down? Did you recall that sigma factor is only found in bacteria? Did you understand that a good Version 1
59
antibiotic should target something found only in bacteria?
72) A
Version 1
60
Clarify Question ● What is the key concept addressed by the question? question asks about proteins with a signal peptide.
● The
● What type of thinking is required? ● You are being asked to apply your knowledge of signal peptides. Gather Content ● What do you know about signal peptides? What other information is related to the question? ● Signal peptides cause proteins to be translated into the endoplasmic reticulum where they are eventually secreted. Choose Answer ● Given what you now know, what information is most likely to produce the correct answer? ● Signal sequences are found on eukaryotic proteins that will ultimately be either secreted from the cell or found as resident proteins in the organelles found in the secretory pathway. Of all the proteins listed, only antibodies will be secreted from the cell. The rest of the listed proteins play roles within cells or, in the case of the sigma subunit, are prokaryotic proteins. Reflect on Process ● Did your problem-solving process lead you to the correct answer? If not, where did the process break down or lead you astray? How can you revise your approach to produce a more desirable result? ● This question asked you to apply your understanding of signal peptides. If you got the correct answer, great job! If you got an incorrect answer, where did the process break down? Did you recall that proteins with a signal peptide are secreted out of the cell? Did you understand that antibodies are secreted out of a cell? Version 1
61
73) D 74) Section Break 74.1) B
Version 1
62
Clarify Question ● What is the key concept addressed by the question? question asks about the sequence of an intron.
● The
● What type of thinking is required? ● You are being asked to analyze the sequence of an intron based on a mRNA. ● What key words does the question contain and what do they mean? ● Intron—noncoding DNA that interrupts the sequence of a gene Gather Content ● What do you know about introns? What other information is related to the question? ● An intron sequence will not appear in the final mRNA after it was spliced out.The template sequence will be the reverse complement sequence of the intron. Choose Answer ● Given what you now know, what information is most likely to produce the correct answer? ● For the DNA sequence: ● 5'AGCACCTGCCGAATGGGCCAAATCCTGCCGAATAAA-3' ● 3'TCGTGGACGGCTTACCCGGTTTAGGACGGCTTATTT-5' ● And the mRNA sequence: ● 5’AGCACCUGCCGCCUGCCGAAUAAAAAAA-3’ ● The intron will be the sequence that is in the mRNA and not the DNA: ● 5’-AATGGGCCAAAT-3’ ● The template DNA for this will be the reverse complement: ● 3'-TTACCCGGTTTA-5' Version 1
63
Reflect on Process ● Did your problem-solving process lead you to the correct answer? If not, where did the process break down or lead you astray? How can you revise your approach to produce a more desirable result? ● If you got the correct answer, great job! If you got an incorrect answer, where did the process break down? Did you recall that introns are spliced out of a mRNA? Did you understand that the template would have the reverse complement sequence of the mRNA?
74.2) C
Version 1
64
CHAPTER 16 CHECK ALL THE APPLY. Choose all options that best completes the statement or answers the question. 1) Control of the lac operon and lactose utilization in bacteria is __________. (Check all that apply.) A) not induced in the presence of both glucose and lactose B) only induced when there is glucose but not lactose C) under negative control, mediated by a repressor D) controlled by the expression of three downstream genes
2) In a 32-cell stage tunicate embryo, if the vegetal marginal cells lack a functional FGF receptor, which structure(s) will fail to develop? (Select all that apply.) A) Notochord B) Mesenchyme C) Muscle D) Nerve cord
3) Nucleases are enzymes that degrade mRNA. In order for nuclease to overcome the inherent protections in mRNA that prevent its degradation, nucleases must use __________. (Check all that apply.) A) 5' decapping enzymes B) A and U sites that promote removal of the poly A tails C) C and G recognition sites that activate removal of poly A tails D) 3' end recognition sites that initiate quick degradation E) 3' cap and poly A tail sites that activate degradation
4) When stimulating formation of heterochromatin during RNA silencing in plants, the key changes that would be observed include __________. (Check all that apply.)
Version 1
1
A) tightening of chromatin via methylated histones B) reduction in expression of methyl groups C) increasing histone methylation D) increasing histone acetylation E) the RNAi pathway, which enhances methylation
5)
Select all changes that occur as a result of RNA editing. A) Insertion of uracil into RNA transcripts B) Deletion of uracil in RNA transcripts C) Deamination of cytosine to uracil D) Deamination of cytosine to guanine E) Deamination of adenine to inosine
MULTIPLE CHOICE - Choose the one alternative that best completes the statement or answers the question. 6) As a microbiologist, you have been asked to investigate the regulation of a novel gene in a bacterial species. Given what is known about bacteria, the logical place to begin your investigation is __________. A) transcriptional regulation B) translational regulation C) alternative splicing regulation D) regulation by enhancer elements E) regulation by general transcription factors
7) You have been asked to design a synthetic DNA motif, able to bind transcriptional regulatory proteins. The location on this motif that you will design for protein binding is the __________.
Version 1
2
A) minor groove of the DNA double helix B) major groove of the DNA double helix C) phosphate backbone of the DNA double helix D) sugar backbone of the DNA double helix E) available hydrogen bonds of bases in single-stranded DNA
8) You lead a research team challenged with the task of creating a regulatory protein able to shut off transcription of a particular gene. You focus your design around a binding site called an operator that is associated with the promoter. The physical location of the operator most likely to affect transcription would be __________. A) upstream of the gene promoter B) downstream of the gene promoter C) internal to the gene itself D) internal to the gene promoter
9) Histones that are tightly wound by DNA and are the basic unit of chromatin are called __________. A) operons B) nucleosomes C) protein clusters D) repressor genes E) facilitators sites
10)
Enhancers are the binding sites for the __________. A) promoters of DNA synthesis B) suppressor factors C) co-activation factors D) mediator factors E) specific transcription factors
Version 1
3
11) The basic tool of genetic regulation is the ability of certain proteins to bind to specific __________. A) regulatory RNA sequences B) regulatory DNA sequences C) repressor parts of the gene D) promoter parts of the gene E) enzymes of the cell
12)
High levels of DNA methylation correlate with __________.
A) the methylated gene functioning without interruption B) no errors being made during transcription C) quick nucleosome formation on the methylated sequence, which assists in mRNA formation D) reuse of the mRNA after the methylated gene is transcribed E) inactive genes
13) You work for a pharmaceutical company that designs small RNAs, used to control expression of disease genes. The primary focus area of your research should be __________. A) transcriptional repression B) transcriptional activation C) translational repression D) translational activation
14) Although the specific mechanism of RNA interference has not been fully defined, it involves __________.
Version 1
4
A) double-stranded RNA interference with mRNA B) double-stranded RNA interference with DNA C) double-stranded DNA interference with mRNA D) double-stranded mRNA interference with DNA
15) The DNA-binding proteins of almost all regulatory proteins use one of a small set of shapes that enable them to fit into the DNA major groove. These shapes are called __________. A) DNA-binding motifs B) DNA prints C) fingerprints D) repressors E) transcriptional domains
16) Small RNAs can regulate gene expression. One type, called micro RNA (miRNA), acts by binding directly to __________. A) mRNA to prevent translation B) tRNA to prevent transcription C) mRNA to prevent transcription D) tRNA to prevent translation
17) If a strain of bacteria had a mutation that blocked expression of the lac repressor, what would you expect as a result? A) The mutant strain would outcompete wildype strains, since it could always utilize lactose. B) The mutant strain would grow at the same rate as wild-type if lactose was not present. C) The mutant strain would waste energy producing enzymes in the absence of lactose. D) The mutant strain would act the same, because it would still require lac activator protein to turn on.
Version 1
5
18) In eukaryotes, specific transcription factors have two distinct domains, that isa DNAbinding domain and __________. A) an RNA-binding domain B) an activating domain C) a repressor domain D) an enhancer domain E) an operator domain
19)
In orderfor transcription to be initiated, __________.
A) DNA polymerase must have access to the DNA double helix and must alsobe capable of binding to the gene's promoter B) RNA polymerase must have access to the DNA double helix and must alsobe capable of binding to the gene's promoter C) DNA polymerase must have access to the RNA and must alsobe capable of binding to the gene's promoter D) RNA ligase must have access to the DNA double helix and must alsobe capable of binding to the gene's promoter E) RNA polymerase must have access to the DNA double helix and also must be capable of binding to the gene's operator
20)
Certain proteins can bind to specific DNA regulatory sequences by entering __________. A) the major groove of the DNA and reading the nucleotide base pairs B) the minor groove of the DNA and reading the nucleotide base pairs C) the major groove of RNA and reading the nucleotide base pairs D) DNA's major groove by using DNA polymerase and reading the nucleotide base
pairs E) DNA's minor groove by using DNA polymerase and reading the nucleotide base pairs
Version 1
6
21) Regulatory proteins can identify specific sequences on the DNA double helix without unwinding the helix. This is accomplished by inserting __________. A) RNA promoters into either the major groove or the minor groove of the double helix where the edges of the nitrogen bases protrude B) DNA-binding motifs into the minor groove of the double helix where the edges of the nitrogen bases protrude C) DNA polymerase into the major groove of the double helix where the edges of the nitrogen bases protrude D) RNA polymerase into the major groove of the double helix where the edges of the nitrogen bases protrude E) DNA-binding motifs into the major groove of the double helix where the edges of the nitrogen bases protrude
22) When E. coli cells produce the amino acid tryptophan, a cluster of five genes is transcribed together. This cluster of genes is referred to as the __________. A) trp transcriptional operator B) trp regulator C) trp suppressor D) trp operon E) trp promoter
23) The proteins necessary for the use of lactose in E. coli are collectively called the __________. A) lac regulator B) lac suppressor C) lac operon D) lac promoter E) lac transcriptional operator
24)
A defining characteristic of eukaryotic organisms is that they __________.
Version 1
7
A) have their transcription occurring in the cytoplasm and translation in the nucleus B) have their transcription occurring in the nucleus and translation in the cytoplasm C) have only operons to assist in gene expression D) carry out protein synthesis only in the presence of the cAMP molecule E) use the leucine zipper primarily for the production of the amino acid tryptophan
25) A protein that regulates transcription by binding to the operator is known as a/an __________. A) operon B) repressor C) promoter protein D) operator E) induction protein
26) A cluster of functionally related genes that are regulated together and encoded into a single mRNA molecule is referred to as a/an __________ and can be found in __________ cells. A) operon; prokaryotic B) repressor; eukaryotic C) promoter; prokaryotic D) operator; eukaryotic E) CAP; all
27) A site of negative genetic regulation wherebinding by repressor blocks transcription is the __________. A) operon B) repressor C) promoter D) operator E) CAP
Version 1
8
28) A site at the 5' end of a gene where RNA polymerase attaches to initiate transcription and control of gene expression occurs is called a/an __________. A) operon B) repressor C) promoter D) operator E) CAP
29)
The main form of glucose repression in the lac operon is __________. A) induction B) repression C) inducer exclusion D) the CAP/cAMP system
30) If you were given a bacterial strain with a mutation in the promoter region of the trp operon, the most likely effect would be __________. A) interference with RNA polymerase binding B) to block the repressor from binding C) derepression of the trp operon D) enabling the trp operon to be expressed in the absence of tryptophan E) lack of expression of just the first gene in the operon
31) You are working to identify enhancer regions of a particular gene. The best place to look is __________. A) immediately upstream of the promoter B) immediately downstream of the promoter C) either upstream or downstream of the promoter, possibly some distance away D) on a separate chromosome from the promoter
Version 1
9
32) The p300/CBP (CREB-binding protein) proteins are histone acetylases that help regulate the transcription of many genes. Based on this information, you can conclude that these proteins __________. A) acetylate purines B) remodel chromatin C) recruit helicases D) physically connect activator proteins
33)
DNA methylation is the only known natural modification of DNA. It affects __________. A) adenine bases B) guanine bases C) cytosine bases D) thymine bases
34) If you were able to look very closely at a portion of DNA and find methylated histones, you would __________. A) be mistaken since only DNA can be methylated, not histones B) be looking at a region of active chromatin C) be looking at a region of inactive chromatin D) be looking at a chromatin remodeling complex
35) Production of the iron-storing protein ferritin is regulated by aconitase, which binds to a 30-nucleotide sequence at the beginning of the ferritin mRNA and interferes with ribosome binding. Aconitase must be a __________. A) transcription repressor protein B) translation repressor protein C) RNA interference protein D) translation initiation protein
Version 1
10
36) You are studying the effects of transcription factors on the activation of gene expression. However, when first starting your study, you notice that one of the transcription factors binds far away from the gene it is supposed to activate. What description best explains this finding? A) The transcription factor transcribes small RNAs that then bind to the promoter and activate the gene's expression. B) Signals originatevia inducersfrom the transcription factor to the site of promotion thatactivate transcription of the gene. C) DNA loopingtransports the transcription factor closer to the promoter andinitiatesgene transcription. D) The assistance ofRNA looping moves the transcription factor closer to the promoter.
37) In invertebrate embryos, the cells created by cleavage contain different developmental signals from the egg called cytoplasmic __________. A) stem cells B) signals C) determinants D) junctions
38)
Differentiated human cells may be distinguished from one another by __________. A) their genomes B) the genes they express C) their morphologyand function D) their morphology, function, and the genes they express E) their morphology, function, genome, and genes they express
39)
A muscle precursor cell would be considered to be __________.
Version 1
11
A) pluripotent B) determined C) differentiated D) induced
40)
What can be said aboutanimal and vegetal poles?
A) When animal pole cells are cultured with vegetal pole cells, some of the animal pole cells become mesoderm. B) When animal pole cells are cultured with vegetal pole cells, some of the vegetal pole cells become mesoderm. C) Isolated animal cells develop as endoderm. D) Isolated vegetal cells develop as ectoderm.
41) In a 32-cell stage tunicate embryo, vegetal marginal cells that lack functional genes for both macho-1 and the FGF receptor would develop as what cell type? A) Mesenchyme B) Muscle C) Notochord D) Nerve cord
42) A gene is coded for only one protein and associated with only one specific tissue type. Is this statement correct and why? A) No, some genes can go through alternative splicing leading to the production of different proteins in different tissues. B) Yes, all genes only code for one protein normally specific to one tissue type. C) No, all genes can encode more than one protein in multiple tissue types due to exon splicing formation. D) Yes, all tissue-specific proteins are only expressed from a single gene for that specific tissue.
Version 1
12
43) The progesterone receptor (PR) is a steroid hormone receptor and transcription factor. The protein SRC-1, which does not bind DNA by itself, can bind to PR and increase expression of genes regulated by PR. Based on this information, SRC-1 is best described as a/an __________. A) coactivator B) inducer C) general transcription factor D) specific transcription factor
44)
What is the primary objective of gene regulation in prokaryotes? A) To enter apoptosis B) To adjust a cell's activity to its environment C) To begin a developmental program D) To express all of the genes in the cell at the same level
45) You are studying the function of a recently identified gene in C. elegans. You perform genetic screens for several months in an attempt to isolate loss-of-function gene mutations, but your efforts are unsuccessful. Your advisor suggests you try another approach to eliminate gene function. The besttechnique toaccomplish this goal would be __________. A) to design a repressor to bind to the operon of this gene B) use a histone deacetylase to induce a transcriptionally inactive state C) use a C. elegans strain with a homozygous TFIID mutation to prevent the translation initiation complex from forming D) use RNA interference to prevent mRNA translation
Version 1
13
Answer Key Test name: Chap 16_3e 1) [A, C, D] 2) [A, B]
Version 1
14
Clarify Question • What is the key concept addressed by the question? o This question addresses FGF induction. • What type of thinking is required? o This question is asking you to analyze the information given, using logic, to dissect the problem and determine the answer. • What key words does the question contain and what do they mean? o Tunicates are marine invertebrates also known as sea squirts. o The vegetal marginal cells lie on the edge of the vegetal pole and can respond to the FGF signal, and eventually make either mesenchyme or notochord. Gather Content • What do you already know about FGF induction? o The example of induction in the vegetal marginal cells of tunicates illustrates how different cells can have different responses to the same inductive signal, depending on what other genes are active in the cell. o If the vegetal marginal cells receive the FGF signal and also have the Macho-1 gene expressed, those cells will become mesenchyme. o If the vegetal marginal cells receive the FGF signal but do not have the Macho-1 gene expressed, those cells will become notochord. o If the cells did not receive the FGF signal, they would become either muscle or nerve cord. Consider Possibilities • Consider the different answer options. Which can you rule out? o Would cells lacking FGF receptor fail to make muscle or nerve cord? No, because those normally result when the FGF signal is not received.
Version 1
15
Choose Answer • Given what you now know, what information and/or problemsolving approach is most likely to produce the correct answer? o If the cells lacked the FGF receptor, they would not be able to receive the FGF signal, and would develop as if there were no signal. o They would not be able to develop into the fates that depend on the FGF signal–notochord and mesenchyme. Reflect on Process • Did your problem-solving process lead you to the correct answer? If not, where did the process break down or lead you astray? How can you revise your approach to produce a more desirable result? o This question asked which cells would fail to develop if the tunicate vegetal marginal cells lacked FGF receptor. o The question required you to analyze the information given, using logic, to dissect the problem and determine the answer. o Did you recognize that the two cell fates that depended on FGF were mesenchyme and notochord? 3) [A, B, D]
Version 1
16
Clarify Question ● What is the key concept addressed by the question? question asks about degradation of mRNA.
● The
● What type of thinking is required? ● You are being asked to apply your knowledge of degradation of mRNA. Gather Content ● What do you know about degradation of mRNA? What other information is related to the question? ● Some mRNA contain specific sequences near their 3′ ends that make them targets for enzymes that degrade mRNA. A sequence of A and U nucleotides near the 3′ poly-A tail of a transcript promotes removal of the tail, which destabilizes the mRNA. Loss of the poly-A tail leads to rapid degradation by 3′-to-5′ RNA exonucleases. Another consequence of this loss is the stimulation of decapping enzymes that remove the 5′ cap, leading to degradation by 5′-to-3′ RNA exonucleases. Other mRNA transcripts contain sequences near their 3′ ends that are recognition sites for endonucleases, which cause these transcripts to be digested quickly. The short half-lives of the mRNA transcripts of many regulatory genes are critical to the function of those genes, because they enable the levels of regulatory proteins in the cell to be altered rapidly. Choose Answer ● Given what you now know, what information is most likely to produce the correct answer? ● mRNA 5’ degradation can be stimulated by decapping enzymes, A and U sites that promote removal of the poly A tail and sequences near the 3’ ends. There is not a 3’ cap on the mRNA. Reflect on Process Version 1
17
● Did your problem-solving process lead you to the correct answer? If not, where did the process break down or lead you astray? How can you revise your approach to produce a more desirable result? ● This question asked you to apply your understanding of degradation of mRNA. If you got the correct answer, great job! If you got an incorrect answer, where did the process break down? Did you recall that mRNA has a 5’ cap and poly A tail? Did you understand that some mRNA has a sequence that increases degradation?
4) [A, C, E]
Version 1
18
Clarify Question ● What is the key concept addressed by the question? ● The question asks about RNA silencing through chromatin formation. ● What type of thinking is required? ● You are being asked to apply your knowledge of RNA silencing through chromatin formation. Gather Content ● What do you know about RNA silencing through chromatin formation? What other information is related to the question? ● Modifications include acetylation and methylation of lysine as well as phosphorylation of serine, threonine, and tyrosine. In general, acetylation, especially of H3, is correlated with active sites of transcription, both in regulatory regions and in the transcribed region of the gene itself. But methylation of the same histone (H3) can have the opposite effect, depending on the lysine methylated. Choose Answer ● Given what you now know, what information is most likely to produce the correct answer? ● Methylation of a histone can lead to decreased RNA production. DNA binds more tightly to the methylated histone. Anything that enhances methylation would decrease transcription. Reduction of methylation would not lead to a decrease in RNA. Reflect on Process ● Did your problem-solving process lead you to the correct answer? If not, where did the process break down or lead you astray? How can you revise your approach to produce a more desirable result? ● This question asked you to apply your understanding of RNA silencing through chromatin formation. If you got the correct answer, great job! If Version 1
19
you got an incorrect answer, where did the process break down? Did you recall that methylation of histones can lead to tighter chromatin? Did you understand that chromatin does not allow as much RNA transcription?
5) [A, C, E] 6) A
Version 1
20
Clarify Question • What is the key concept addressed by the question? o This question addresses gene regulation in bacteria. • What type of thinking is required? o This question is asking you to take what you already know and apply it to this unfamiliar situation. • What key words does the question contain and what do they mean? o Regulation of a gene refers to the control of its expression. o A novel gene is a new gene–that is, one that has not been well-studied or understood. o Bacteria are prokaryotes like E. coli or Salmonella. Their gene regulation is similar to eukaryotes, but has distinct features. Gather Content • What do you already know about gene regulation in bacteria? o Gene regulation in bacteria is similar to that of eukaryotes. o However, bacteria do not have a nucleus, therefore there is less separation between transcription and translation. o Because of this, bacteria tend to rely on transcriptional regulation more than translational or posttranslational regulation. Consider Possibilities • Consider the different answer options. Which can you rule out? o Is a prokaryotic gene likely to use alternative splicing regulation? No, prokaryotic genes don’t have introns, and don’t have splicing machinery. o Is a prokaryotic gene likely to use regulation by enhancer elements? No, prokaryotic genes don’t have distant enhancers like eukaryotic genes do. o Is a prokaryotic gene likely to use regulation by general transcription Version 1
21
factors? No, this term is used for eukaryotes to differentiate the basal transcriptional machinery from more specific transcriptional regulatory proteins. Moreover, the general transcription factors do not play an important role in regulation. Choose Answer • Given what you now know, what information and/or problemsolving approach is most likely to produce the correct answer? o Is a prokaryotic gene likely to use translational regulation? No. Since bacteria lack a nuclear membrane, transcription and translation are coupled. o Thus it is important that they regulate gene expression at the transcriptional level. o Transcriptional regulation is the most common mode of gene regulation in bacteria. Reflect on Process • Did your problem-solving process lead you to the correct answer? If not, where did the process break down or lead you astray? How can you revise your approach to produce a more desirable result? o This question asked about the most common type of regulation in bacteria. o The question required you to take what you already know and apply it to this unfamiliar situation. o Did you recognize that transcriptional regulation is by far the most common type of regulation in bacteria? o Did you remember that bacteria lack splicing and lack enhancers? 7) B
Version 1
22
Clarify Question • What is the key concept addressed by the question? o This question addresses transcriptional regulatory proteins. • What type of thinking is required? o This question is asking you to take what you already know and apply it to this unfamiliar situation. • What key words does the question contain and what do they mean? o Synthetic DNA is an artificially designed and produced piece of DNA. o A DNA motif is a specific sequence that is recognizable by DNAbinding proteins. o Transcriptional regulatory proteins, also called transcription factors, are proteins that bind DNA to regulate transcription. Gather Content • What do you already know about transcriptional regulatory proteins? o Transcriptional regulatory proteins are proteins that bind DNA to control when, where, and how much a gene is transcribed. o These proteins have special DNA-binding domains, such as the helixturn-helix or the zinc finger domain. Consider Possibilities • Consider the different answer options. Which can you rule out? o For a protein to bind a specific DNA motif, can it recognize the phosphate backbone? No, because that will be identical on every piece of DNA. o For a protein to bind a specific DNA motif, can it recognize the sugar backbone? No, because that will be identical on every piece of DNA. Version 1
23
o For a protein to bind a specific DNA motif, can it recognize the open hydrogen bonds of single-stranded DNA? No, because the DNA is double stranded before transcription begins. Choose Answer • Given what you now know, what information and/or problemsolving approach is most likely to produce the correct answer? o So, it makes sense that the regulatory protein is going to need to recognize the unique sequence of bases while the DNA is double stranded. o In fact, regulatory proteins bind DNA in the major groove and not the minor groove because it gives enough room for the protein to bind along the bases. Reflect on Process • Did your problem-solving process lead you to the correct answer? If not, where did the process break down or lead you astray? How can you revise your approach to produce a more desirable result? o This question asked what part of the DNA is use for motif recognition by regulatory proteins. o The question required you to take what you already know and apply it to this unfamiliar situation. o Did you recognize that regulatory proteins must recognize the unique parts of the DNA, i.e., the sequence of bases? o Did you realize that the proteins bind the major groove because the minor groove is not wide enough? 8) B
Version 1
24
Clarify Question • What is the key concept addressed by the question? o This question addresses the function of operators an repressors. • What type of thinking is required? o This question is asking you to take what you already know and apply it to this unfamiliar situation. • What key words does the question contain and what do they mean? o An operator is a site on the DNA that can attract a repressor protein to block transcription. Gather Content • What do you already know about operators and repressors? o Repressors are proteins that bind to regulatory sites on DNA called operators to prevent or decrease the initiation of transcription. They act as a kind of roadblock to prevent the polymerase from initiating effectively. Consider Possibilities • What other information is related to the question? Which information is most useful? o When we learned about the lac repressor, we learned that it binds the operator site just downstream of the promoter. When the repressor protein is bound, it blocks RNA polymerase and prevents transcription. o Likewise, when we learned about the trp repressor, we learned that it binds the operator site just downstream of the promoter. When the repressor protein is bound, it blocks RNA polymerase and prevents transcription. Choose Answer Version 1
25
• Given what you now know, what information and/or problemsolving approach is most likely to produce the correct answer? o Operator sites are location just downstream of the promoter, because when the repressor protein binds, it acts like a roadblock to prevent transcription by RNA polymerase. Reflect on Process • Did your problem-solving process lead you to the correct answer? If not, where did the process break down or lead you astray? How can you revise your approach to produce a more desirable result? o This question asked where an operator site is typically located to block transcription. o The question required you to take what you already know and apply it to this unfamiliar situation. o Did you recognize that operator sites lie downstream of the promoter, so that when the repressor binds it acts like a roadblock? 9) B 10) E 11) B 12) E 13) C
Version 1
26
Clarify Question • What is the key concept addressed by the question? o This question addresses small RNAs. • What type of thinking is required? o This question is asking you to take what you already know and apply it to this unfamiliar situation. • What key words does the question contain and what do they mean? o A pharmaceutical company is a company that designs new medicinal drugs. o Small RNAs are either naturally occuring or manufactured RNAs such as micro-RNAs (miRNAs) or small interfering RNAs (siRNAs) that regulate gene expression. Gather Content • What do you already know about small RNAs? o Small RNAs are used naturally by organisms, and also by researchers, to silence genes. Consider Possibilities • What other information is related to the question? Which information is most useful? o Small RNAs typically work by silencing genes–that is, repressing gene expression. o Their mechanism of action involves binding of the small RNA to the native mRNA, attracting enzymes that inhibit translation or cut up the mRNA. Choose Answer
Version 1
27
• Given what you now know, what information and/or problemsolving approach is most likely to produce the correct answer? o Since small RNAs act on the native mRNA, their regulation is posttranscriptional (after transcription), and specifically translational regulation. o Since they inhibit the expression of genes, they act as translational repressors. Reflect on Process • Did your problem-solving process lead you to the correct answer? If not, where did the process break down or lead you astray? How can you revise your approach to produce a more desirable result? o This question asked what type of regulation you should focus on if you want to cure disease with small RNAs. o The question required you to take what you already know and apply it to this unfamiliar situation. o Did you recognize that small RNAs typically work posttranscriptionally, binding with mRNA to block translation? o Did you consider what RNAs are most likely to be able to bind? 14) A 15) A 16) A 17) C
Version 1
28
Clarify Question • What is the key concept addressed by the question? o This question addresses regulation of the lac operon. • What type of thinking is required? o This question is asking you to analyze the information given, using logic, to dissect the problem and determine the answer. • What key words does the question contain and what do they mean? o The lac repressor is a protein that binds the operator site to regulate transcription of the lac operon. Gather Content • What do you already know about the lac operon? o The lac operon encodes proteins that help the cell use lactose as an energy source. o The operon only is transcribed when lactose is present. o When present, lactose binds the repressor protein and keeps it from binding the operator site. That allows transcription to occur. Consider Possibilities • Consider the different answer options. Which can you rule out? o Would the mutant strain outcompete wildype strains, since it could always utilize lactose? No, because lactose is not always present, and it would waste energy synthesizing useless enzymes. o Would the mutant strain grow at the same rate as wild-type if lactose was not present? No, because it would waste energy synthesizing useless enzymes. o Would the mutant strain act the same, because it would still require lac activator protein to turn on? No. In fact there is no lac activator protein. The lac operon is controlled by the activity of the lac repressor. Version 1
29
Choose Answer • Given what you now know, what information and/or problemsolving approach is most likely to produce the correct answer? o Unless lactose was always present at high levels, the mutant strain would be harmed by this mutation. It would waste energy producing useless enzymes that were not needed. Reflect on Process • Did your problem-solving process lead you to the correct answer? If not, where did the process break down or lead you astray? How can you revise your approach to produce a more desirable result? o This question asked what would result for a bacterium mutant for the lac repressor. o The question required you to analyze the information given, using logic, to dissect the problem and determine the answer. o Did you remember that the lac operon encodes proteins that help the cell to utilize lactose? o Did you recognize that the lac operon is regulated so that the cell doesn’t waste energy unless lactose is present? 18) B 19) B 20) A 21) E 22) D 23) C 24) B 25) B 26) A Version 1
30
27) D 28) C 29) C 30) A
Version 1
31
Clarify Question • What is the key concept addressed by the question? o This question addresses the promoter. • What type of thinking is required? o This question is asking you to analyze the information given, using logic, to dissect the problem and determine the answer. • What key words does the question contain and what do they mean? o The promoter region is the region of a gene where the RNA polymerase binds and initiates transcription. o The trp operon is a set of genes in bacteria that are transcribed together to produce enzymes for the biosynthesis of tryptophan. Gather Content • What do you already know about promoters? o A promoter is a region at the beginning of a gene, in the –35 and –10 region, that attracts the binding of RNA polymerase to initiate transcription. Consider Possibilities • Consider the different answer options. Which can you rule out? o If the promoter was mutated, would this prevent the repressor from binding? No, because the repressor binds the operator, which is downstream of the promoter. o If the promoter was mutated, would this cause derepression of the trp operon? In other words, prevent repression to allow expression? No, because repression depends on the repressor binding the operator, not the promoter. o If the promoter was mutated, would this enable the trp operon to be expressed in the absence of tryptophan? No, because that would require Version 1
32
preventing the function of the repressor, which binds the operator. o If the promoter was mutated, would this cause lack of expression of just the first gene in the operon? No, because the promoter is necessary for expression of all the genes in the operon. Choose Answer • Given what you now know, what information and/or problemsolving approach is most likely to produce the correct answer? o What is the role of the promoter region? To allow RNA polymerase to bind and initiate transcription. o What binds to the promoter region? RNA polymerase. o So if the promoter was mutated, RNA polymerase would likely fail to bind and fail to initiate transcription. Reflect on Process • Did your problem-solving process lead you to the correct answer? If not, where did the process break down or lead you astray? How can you revise your approach to produce a more desirable result? o This question asked what would result from a mutation in the promoter of the trp operon. o The question required you to analyze the information given, using logic, to dissect the problem and determine the answer. o Did you recognize that the promoter is necessary for the binding of RNA polymerase? o Did you remember that the repressor acts on the operator region, not the promoter? 31) C 32) B
Version 1
33
Clarify Question • What is the key concept addressed by the question? o This question addresses histone acetylases. • What type of thinking is required? o This question is asking you to analyze the information given, using logic, to dissect the problem and determine the answer. • What key words does the question contain and what do they mean? o Histone acetylases are enzymes that acetylate (that is, add a -COCH3 group) to histone proteins. Gather Content • What do you already know about histone acetyltransferases? o Histone acetylases are a class of proteins that modify histones. o Histones are proteins that form much of the structure of chromatin, that is, the DNA and proteins of the chromosome. Consider Possibilities • Consider the different answer options. Which can you rule out? o Would histone acetylases act to acetylate purines? No, they acetylate histone proteins, not the DNA itself. o Would histone acetylases act to recruit helicases? No, they change the structure of chromatin by modifying histones. o Would histone acetylases act to physically connect activator proteins? No. Although sometimes called coactivators, they activate by modifying chromatin structure, not directly connecting transcription factors. Choose Answer • Given what you now know, what information and/or problemVersion 1
34
solving approach is most likely to produce the correct answer? o Would histone acetylases act to remodel chromatin? Yes! That is their mode of action. Reflect on Process • Did your problem-solving process lead you to the correct answer? If not, where did the process break down or lead you astray? How can you revise your approach to produce a more desirable result? o This question asked about the role of p300/CBP proteins. o The question required you to analyze the information given, using logic, to dissect the problem and determine the answer. o Did you recognize that histone acetylases modify chromatin structure? 33) C 34) C 35) B 36) C
Version 1
35
Clarify Question ● What is the key concept addressed by the question? ● The question asks about transcription factors acting at a distance. ● What type of thinking is required? ● You are being asked to analyze how transcription factors act at a distance. Gather Content ● What do you know about transcription factors acting at a distance? What other information is related to the question? ● Enhancers are the binding site of specific transcription factors. The ability of enhancers to act over large distances is accomplished by DNA bending to form a loop, positioning the enhancer closer to the promoter. The linear distance separating two sites on the chromosome does not have to translate to great physical distance, because the flexibility of DNA allows bending and looping. An activator bound to an enhancer can thus be brought into contact with the transcription factors bound to a distant promoter . Choose Answer ● Given what you now know, what information is most likely to produce the correct answer? ● The best answer is that DNA is flexible and can form a loop, bringing the transcription factor close to the promoter. Binding of a transcription factor does not lead to small RNA production or release of signals. Reflect on Process ● Did your problem-solving process lead you to the correct answer? If not, where did the process break down or lead you astray? How can you revise your approach to produce a more desirable result? ● This question asked you to analyze how transcription factors act at a distance. Version 1
36
If you got the correct answer, great job! If you got an incorrect answer, where did the process break down? Did you recall that DNA is flexible and can form loops? Did you understand that transcription factor binding does not work through forming signals?
37) C 38) D 39) B 40) A 41) D 42) A
Version 1
37
Clarify Question ● What is the key concept addressed by the question? ● The question asks about the number of proteins made per gene. ● What type of thinking is required? ● You are being asked to apply your knowledge of the number of proteins made per gene. Gather Content ● What do you know about the number of proteins made per gene? What other information is related to the question? ● Splicing of premRNA is one of the processes leading to mature mRNA. Many of these splicing events may produce different mRNAs from a single primary transcript by alternative splicing. This mechanism allows another level of control of gene expression. Alternative splicing can change the splicing events that occur during different stages of development or in different tissues. Choose Answer ● Given what you now know, what information is most likely to produce the correct answer? ● Most genes go through alternative splicing, so a single RNA can have some exons left out producing different proteins. Reflect on Process ● Did your problem-solving process lead you to the correct answer? If not, where did the process break down or lead you astray? How can you revise your approach to produce a more desirable result? ● This question asked you to apply your understanding of the number of proteins made per gene. If you got the correct answer, great job! If you got an incorrect answer, where did the process break down? Did you recall that alternative splicing produces multiple proteins from a single Version 1
38
RNA? Did you understand that a single gene can encode multiple proteins?
43) A
Version 1
39
Clarify Question • What is the key concept addressed by the question? o This question addresses coactivators. • What type of thinking is required? o This question is asking you to take what you already know and apply it to this unfamiliar situation. • What key words does the question contain and what do they mean? o A steroid hormone receptor is a special kind of specific transcription factor. They respond to hormones, then enter the nucleus and regulate transcription. Gather Content • What do you already know about coactivators? o In our text, coactivators are described as transcription factors that stabilize the transcription complex by bridging activator proteins with the complex. They act by binding the transcription factor and then binding to another part of the transcription apparatus. Consider Possibilities • Consider the different answer options. Which can you rule out? o Is SRC-1 best described as a general transcription factor? No, because it does not act in the transcription of every gene. o Is SRC-1 best described as an inducer? No. Our text gave us several examples of inducers. All of them were small molecules in prokaryotes that activate genes by inhibiting the binding of a repressor protein. o Is SRC-1 best described as a specific transcription factor? Perhaps– but its role is to enhance the stimulation by another specific transcription factor, the progesterone receptor.
Version 1
40
Choose Answer • Given what you now know, what information and/or problem solving approach is most likely to produce the correct answer? o SRC-1 is best described as a coactivator–a protein that acts to enhance the of other specific transcription factors, by bridging activator proteins with the complex. Reflect on Process • Did your problem-solving process lead you to the correct answer? If not, where did the process break down or lead you astray? How can you revise your approach to produce a more desirable result? o This question asked what is the best description of the protein SRC-1. o The question required you to take what you already know and apply it to this unfamiliar situation. o Did you recognize that as a “helper” for another specific transcription factor, SRC-1 is best described as a coactivator? 44) B 45) D
Version 1
41
Clarify Question • What is the key concept addressed by the question? o This question addresses loss-of-function mutations. • What type of thinking is required? o This question is asking you to weigh and judge evidence, or evaluate, to choose the best of the possible answers. • What key words does the question contain and what do they mean? o A loss-of-function gene mutation is a mutation that breaks the gene, preventing it from working. Gather Content • What do you already know about loss-of-function mutations? o Loss-of-function mutations are any mutation that results in less of the normal gene activity. o They may be due to total deletion of the gene, or a lower level of expression of the mRNA, or because the protein cannot work at the normal rate. Consider Possibilities • Consider the different answer options. Which can you rule out? o To mimic a loss-of-function mutation in this gene, should you design a repressor to bind to the operon of this gene? Bad idea. First of all, eukaryotes like C. elegans don’t have operons. Second, even if they did, that would knock out several genes at once. Third, that would be very difficult to do. o To mimic a loss-of-function mutation in this gene, should you use a histone deacetylase to induce a transcriptionally inactive state? No, that would probably inactivate many many genes and kill the worms. o To mimic a loss-of-function mutation in this gene, should you use a Version 1
42
C. elegans strain with a homozygous TFIID mutation to prevent the translation initiation complex from forming? No, that would definitely affect all the genes and kill the worm. Choose Answer • Given what you now know, what information and/or problemsolving approach is most likely to produce the correct answer? o To mimic a loss-of-function mutation in this gene, should you use RNA interference to prevent mRNA translation? Yes! That is an excellent way to quickly mimic a loss-of-function mutation in one specific gene. Reflect on Process • Did your problem-solving process lead you to the correct answer? If not, where did the process break down or lead you astray? How can you revise your approach to produce a more desirable result? o This question asked what approach would be a good substitute for finding a mutation in a gene. o The question required you to weigh and judge evidence, or evaluate, to choose the best of the possible answers. o Did you recognize that RNAi is a technique that allows a researcher to mimic a loss-of-function mutation?
Version 1
43
CHAPTER 17 CHECK ALL THE APPLY. Choose all options that best completes the statement or answers the question. 1) A gel electrophoresis is performed, but when the gel imager is used, no bands appear. The researcher might have forgotten to __________. (Check all that apply.) A) add electric current B) add antibody C) ligate the DNA fragments together D) include DNA dye E) include an RNA probe
2) The genes incorporated into the genome of Golden Rice include __________. (Check all that apply.) A) daffodil phytoene synthase gene B) bacterial carotene desaturase gene C) bacterial beta-galactosidase D) daffodil lycopene beta-cyclase gene
3) You digest DNA from the frog Xenopus laevis with the enzyme NotI that produces the sticky DNA ends 5'-GGCC-3'. You can ligate your DNA to __________. (Check all that apply.) A) blunt AluI ends from Xenopus laevis B) sticky NotI ends from Xenopus laevis C) blunt AluI ends from bacteria D) sticky NotI ends from bacteria E) sticky EcoRI ends (5'-AATT-3')from Xenopus laevis F) sticky EcoRI ends (5'-AATT-3')from bacteria
4) Some of the useful applications of genetic engineering include __________. (Check all that apply.)
Version 1
1
A) bacteria that can digest oil in an oil spill B) growing glyphosate-resistant cotton C) development of hybrid corn to increase yield D) creating Golden Rice
5) When giving a lecture on GM crops, what concerns may come up in conversation and what research supports or does not support these concerns? (Check all that apply.) A) Selective pressure on weedy species by continued use of a single herbicide. Management practices have been introduced to slow herbicide resistance. B) Gene flow from GM crops into wild plants. Certain plants such as soy are grown in places where there are no related plants and therefore genetic information can not cross over. C) Can GM foods be harmful when ingested? There has been no documentation of adverse health effects for existing GM crops. D) The destruction of all insects with inserted genes for toxins. Research shows us that even with the death of many insects some will always survive.
6) Your city is considering a proposal to build a new algae-based biofuel plant. What benefits would this have over traditional fossil fuels? (Check all that apply.) A) The kelp fronds would provide habitat for endangered sea otters. B) It contributes less carbon to the atmosphere overall. C) It is a renewable resource. D) Biofuel doesn’t need to be burned to provide energy. E) The system could be paired with wastewater treatment. F) The algae can be filtered from the water and used for fuel without refinement.
7) Select the techniques that rely on complementary DNA and fluorescent molecules. (Choose all that apply.)
Version 1
2
A) FISH B) RT-qPCR C) The CRISPR-Cas9 system D) PCR
8) Select diseases or conditions for which stem cell therapy has shown promise. (Choose all that apply.) A) Heart attack B) Blood disorders C) Multiple sclerosis D) Alzheimer's disease
MULTIPLE CHOICE - Choose the one alternative that best completes the statement or answers the question. 9) You want to insert your favorite gene, which has been digested with Ncol endonuclease on both ends, into a vector. Ncol recognizes the DNA sequence C*CATGG. The vector you want to use contains many endonuclease recognition sites, but does not include an Ncol site. Which restriction endonuclease would be the most suitable replacement to digest the vector? A) Sphl - G*CATGC B) Sunl - C*GTACG C) Kpnl - GGTAC*C D) CViAll - C*ATG
10) Enzymes that cleave DNA at specific sites are called _____________. Cutting by some of these enzymes results in single-stranded, complementary tails called _________ . A) vectors; blunt B) peptidases; lagging C) endonucleases; sticky D) DNases; leading
Version 1
3
11) What best describes the way a single-stranded complementary tail is formed by a restriction enzyme? A) Staggered cuts inDNA leaves sticky ends. B) Uneven cuts lead toorphaned ends of DNA. C) Same positionalcutting sites lead to vector ends. D) Same base cuts leadto 5' end production.
12) Bacterial plasmids and artificial chromosomes are used as ______ to insert foreign DNA into host cells and create recombinant genomes. A) clones B) vectors C) endonucleases D) subunits
13) Gel ________ is a process that separates DNA or protein fragments according to their size, by causing them to migrate within a gel in response to an electric field. A) duplication B) blotting C) electrophoresis D) migration
14) Single-stranded ends generated by the same restriction enzyme are complementary to each other. They can be joined together with DNA ligase, __________.
Version 1
4
A) even when the source of the DNA is different B) only when the source of the DNA is the same C) but the sticky endswill most likely have to be modified D) but the hybridization of the two ends may cause a problem with cloning E) only if the subunits have been methylated
15) Two strands of DNA that have been cut by an endonuclease can be sealed together by a/an __________. A) polymerase enzyme B) ligase enzyme C) exonuclease enzyme D) protease enzyme E) methylase enzyme
16) An example of a commercially significant human protein produced in bacteria is _________. The gene for the protein is inserted into bacteria using specifically designed _________. A) hemoglobin; vectors B) globulins; hosts C) AZT; plasma D) insulin; plasmids
17) Scientists can distinguish between DNA of different individuals, thus making this information useful in criminal investigations. The technique used is called __________. A) DNA fingerprinting B) PCR C) CRISPR/Cas9 D) fluorescent in situ hybridization
Version 1
5
18)
A key difference between transforming plant cells and human cells is __________.
A) genes to be expressed in human cells require a eukaryotic promoter, and genes to be expressed in plants don't B) DNA cannot be introduced into human cells C) Agrobacterium is commonly used as a tool to directly transform plant cells but not human cells D) a vector is needed to introduce DNA into human cells and not plant cells
19)
Which technique is used to amplify a desired piece of DNA? A) Gel electrophoresis B) FISH C) PCR D) Transgenesis E) RNA interference
20) It is believed that a nucleotide change from T to C in the gene encoding a green pigment protein is responsible for certain cases of color blindness. In order to test that a mutation in this gene is responsible for color blindness, you would want to __________. A) run gel electrophoresis to ensure it is the right gene B) carry out knockout analysis to observe what happens in mice when the gene is not expressed C) perform plasmid transformation of the DNA into a vector and observe changes in bacteria D) perform a RT-PCR to account for the C to T change in the sequence
21) BamHI is a Type II restriction enzyme that recognizes the sequence 5' G*GATCC 3', and cuts between the two Gs, leaving a 5' overhang. If you were to digest DNA with BamHI, what would be the sequence of the overhanging sticky ends of the two strands?
Version 1
6
A) 5' GATCC 3' and 5' GATCC 3' B) 5' GATCC 3' and 5' GGATC 3' C) 5' GATC 3' and 5' GATC 3' D) 5' GATC 3' and 5' CTAG 3' E) 5' G 3' and 5' G 3' F) 5' G 3' and 5' C 3'
22) You are the scientific consultant for a television show about forensic analysis. In an upcoming episode, investigators will compare the DNA of a suspect and a DNA sample collected from the victim’s fingernails. You are to write up a brief explanation of the technique required for comparing the DNA. Which technique should you explain? A) DNA fingerprinting B) Fluorescent in situ hybridization C) RNAi D) RT-PCR
23)
Which of the following two enzymes are needed to construct a cDNA library? A) Reverse transcriptase and DNA ligase B) Reverse transcriptase and Taq polymerase C) DNA ligase and Taq polymerase D) Taq polymerase and RNA polymerase
24) A researcher wants to create a mouse model for a disease, to allow her team to study possible therapies. The disease is known to be caused by a small deletion that removes a certain gene. Which type of genetic technique would be the best choice for this purpose? A) Knockout mouse B) Knockin mouse C) PCR-based random mutagenesis D) CRISPR-Cas9 system to delete the gene in specific tissues
Version 1
7
25) You have discovered a very small amount of DNA from an ancient organism that you want to save and study. What is the very first thing you should do to allow you to study this DNA in the lab? A) RT-PCR B) Gel electrophoresis C) Polymerase chain reaction (PCR) D) Insert the DNA into a vector
26) As a farmer you spray your weed-infested crops with a herbicide containing glyphosate. Glyphosate stops plants from producing aromatic amino acids needed for development. The main reason you are not worried this will affect the growth of your crops is that __________. A) glyphosate-resistant crops are produced to create higher amounts of enzymes that make aromatic amino acids B) crops are genetically engineered to extract higher amounts of aromatic amino acid from the soil bacteria C) different plants respond differently to glyphosate; some crops have an enzyme that breaks down glyphosate to a harmless substrate D) crops have been sprayed with a soil bacteria that protects the plants from the mechanism of glyphosate
27)
Identify a sequence that is most likely to be recognized by a restriction enzyme. A) CGATGC B) CCCTTT C) TGGCCA D) TCATCA
28) The PCR technique requires a DNA polymerase from an organism that can endure high heat, such as Thermus aquaticus. What step of the protocol makes the heat-stability most necessary?
Version 1
8
A) Synthesis B) Primer annealing C) Denaturation D) Antibody binding
29) Imagine that a very unusual animal species madeits own aromatic amino acids using a 5enolpyruvylshikimate-3-phosphate (EPSP) synthase enzyme similar to that of plants. What would happen if this animal foraged in a field of glyphosate-resistant plants? A) There would be no effect. B) They would become sick from the glyphosate that was sprayed on the plants. C) They would also become glyphosate resistant. D) They would no longer need to make their own aromatic amino acids.
30) A fourth type of restriction enzyme is identified that cleaves at precise locations like a type II enzyme, but it also occasionally makes mistakes and cuts elsewhere. Would this type of enzyme be a useful enzyme to use in molecular cloning? A) Yes, because it cuts correctly sometimes. B) No, because it is a precise enzyme. C) No, because cutting mistakes could lead to incorrect cloning. D) Yes, because cutting mistakes don't matter in molecular cloning.
Version 1
9
31) A 6.85 kb EcoRI fragment of DNA is shown below. The location of several restriction sites is indicated. Scale is approximate.
If you were to completely digest this fragment of DNA with PstI, how many pieces of DNA would you obtain? A) 1 B) 2 C) 3 D) 4
Version 1
10
32) A 6.85 kb EcoRI fragment of DNA is shown below. The location of several restriction sites is indicated. Scale is approximate.
If you were to digest this DNA with PstI, which length of DNA fragment would migrate the fastest on an agarose gel? A) 803 B) 2622 C) 3425 D) 235
33) Investigators have found a new species bacteria that lives under both aerobic and anaerobic conditions. They hypothesize that different genes are expressed in each condition. What experiment could test this hypothesis and identify important genes? A) Completely sequence the genome of the bacteria in both aerobic and anaerobic conditions B) Collect mRNA from both anaerobic and aerobic conditions and generate two cDNA libraries C) Measure levels of RNA polymerase in both conditions D) Design primers to a specific region of the genome and sequence this region during both anaerobic and aerobic conditions
Version 1
11
34) Three scientists are using different forms of PCR for their research. Abby wants to determine changes in gene expression levels in a tissue, so she uses __________. Bob is trying to extract genetic information from a wooly mammoth fossil, so he uses __________. Chris needs to express a gene in bacteria that matches the form expressed in kidney cells, so she uses __________. A) quantitative RT-PCR; PCR; RT-PCR B) PCR; quantitative RT-PCR; RT-PCR C) RT-PCR; PCR; quantitative RT-PCR D) RT-PCR; quantitative RT-PCR; PCR E) quantitative RT-PCR; RT-PCR; PCR
35) Which type of disease is least likely to be able to be treated using genome editing with TALEN or CRISPR? A) Trisomy 21 B) Leukemia C) Cystic fibrosis D) Huntingtondisease
36) In some ciliate protozoa, the codons UAA and UAG encode glutamate, rather than acting as stop codons. How does this compare to other species? A) This is unusual–almost all species share the same universal genetic code. B) This is typical–every species has its own unique genetic code. C) This is not unusual–while most species have a similar genetic code, there are often variations for some of the amino acids.
37) After an oil spill, bacteria that can metabolize hydrocarbons are released for bioremediation. After a month, two areas affected by the spill show different levels of improvement. What factor should be measured to determine the difference in effectiveness between the two sites?
Version 1
12
A) Temperature B) Oxygen levels C) pH D) Salinity E) Nutrient levels F) All of these factors may play a role.
38) A medical researcher is trying to develop a diagnostic technique for a disease that is characterizedby a large duplicationof a particular chromosomal region. What would be the most appropriate technique? A) FISH B) Gene chip C) Genome sequencing D) White blood cell count
39) A farmer is growing glyphosate-resistant transgenic soybeans. What are you unlikely to see in those fields during the growing season? A) A tilling machine B) Caterpillars C) Field mice D) Irrigation
40) What technique revolutionized the way genome sequencing was carried out, reducing cost and increasing efficiency? A) PCR B) RT-PCR C) FISH D) DNA fingerprinting E) RNAi
Version 1
13
41) Which technique allows a researcher to change a single specific nucleotide in a gene sequence in vitro? A) PCR-based random mutagenesis B) PCR-based site-specific mutagenesis C) The CRISPR-Cas9 system D) RT-PCR
42)
Select the genetic technique that acts directly on mRNA rather than DNA. A) TALE proteins B) The CRISPR/Cas9 system C) RNAi D) FISH
43)
What advantages do Bt crops have over conventional crops? A) These crops are able to use the Bt toxin to kill nearby weeds. B) These crops are able to use the Bt toxin to kill insects that attempt to eat them. C) The Bt toxin makes plants resistant to glyphosate. D) The Bt toxin prevents infection by Agrobacterium.
Version 1
14
Answer Key Test name: Chap 17_3e 1) [A, D]
Version 1
15
Clarify Question • What is the key concept addressed by the question? o This question addresses the gel electrophoresis technique. • What type of thinking is required? o This question is asking you to take what you already know and apply it to this unfamiliar situation. • What key words does the question contain and what do they mean? o Gel electrophoresis is a technique to separate DNA fragments by size. o The bands represent the different size pieces of DNA. Gather Content • What do you already know about gel electrophoresis? o Gel electrophoresis uses a soft agarose gel to separate DNA. o The DNA is pipetted into wells at one end; then electrical current drags the DNA through the gel. o The different sized fragments travel at different speeds, so they separate across the gel. o The bands are visualized using a DNA dye. Consider Possibilities • Consider the different answer options. Which can you rule out? o Could the researcher have forgotten to add antibody or an RNA probe? No, because those are not used with the simple gel electrophoresis technique. o Could the researcher have forgotten to ligate the DNA fragments? No. The point of a gel is to separate DNA fragments, not connect them. Choose Answer
Version 1
16
• Given what you now know, what information and/or problemsolving approach is most likely to produce the correct answer? o The researcher could have forgotten to turn on the electric current– then the DNA would stay in the wells and not separate properly. o The researcher could also have forgotten to add the DNA dye–then the DNA would be separated, but the bands would not be visible. Reflect on Process • Did your problem-solving process lead you to the correct answer? If not, where did the process break down or lead you astray? How can you revise your approach to produce a more desirable result? o This question asked what could have gone wrong with gel electrophoresis. o The question required you to take what you already know and apply it to this unfamiliar situation. o Did you recognize that antibodies, RNA probes, and membranes are not involved with simple gel electrophoresis? o Did you realize that gel electrophoresis requires electric current, and a DNA dye? 2) [A, B, D] 3) [B, D]
Version 1
17
Clarify Question • What is the key concept addressed by the question? o This question addresses restriction enzymes and cloning. • What type of thinking is required? o This question is asking you to take what you already know and apply it to this unfamiliar situation. • What key words does the question contain and what do they mean? o The frog Xenopus laevis is often used for cell biology research. o Digesting DNA means to treat it with restriction enzymes. o Sticky DNA ends are the result of cutting with most restriction enzymes–they have a staggered cut with one strand longer than the other. Gather Content • What do you already know about restriction enzymes? o Restriction enzymes recognize a specific DNA sequence, ranging from 4 bases to 12 bases, and cleave the DNA at a spcific base within this sequence. o The recognition sites for most type II enzymes are palindromes. The palindromic DNA sequence reads the same from 5′ to 3′ on one strand as it does on the complementary strand. o Given this kind of sequence, cutting the DNA at the same base on either strand produces staggered cuts that leaves sticky ends,or overhangs. Consider Possibilities • Consider the different answer options. Which can you rule out? o Is your NotI-cut frog DNA going to ligate with blunt AluI ends from Xenopus laevis? No, sticky ends need to bind compatible sticky ends. Version 1
18
o Is your NotI-cut frog DNA going to ligate with blunt AluI ends from bacteria? No, sticky ends need to bind compatible sticky ends. o Is your NotI-cut frog DNA going to ligate with sticky EcoRI ends from Xenopus laevis? No, the sticky ends need to have compatible overhangs. o Is your NotI-cut frog DNA going to ligate with sticky EcoRI ends from bacteria? No, the sticky ends need to have compatible overhangs. Choose Answer • Given what you now know, what information and/or problemsolving approach is most likely to produce the correct answer? o Is your NotI-cut frog DNA going to ligate with sticky NotI ends from Xenopus laevis? Yes, that should work fine. o Is your NotI-cut frog DNA going to ligate with sticky NotI ends from bacteria? Yes, that should also work fine–as long as the sticky ends are the same, it doesn’t matter where the DNA came from. Reflect on Process • Did your problem-solving process lead you to the correct answer? If not, where did the process break down or lead you astray? How can you revise your approach to produce a more desirable result? o This question asked what types of DNA would be compatible. o The question required you to take what you already know and apply it to this unfamiliar situation. o Did you recognize that all that matters when cloning DNA is whether the pieces have compatible ends? o Did you realize that frog DNA and bacterial DNA can ligate together if properly cut? 4) [A, B, D] Version 1
19
5) [A, B, C] 6) [B, C, E]
Version 1
20
Clarify Question • What is the key concept addressed by the question? o This question addresses biofuels from algae. • What type of thinking is required? o This question is asking you to take what you already know and apply it to this unfamiliar situation. Gather Content • What do you already know about biofuels from algae? o Biofuels produced from microalgae have several advantages over petroleum-based fossil fuels. They are renewable as long as carbon dioxide can be fixed into biomass by photosynthesis. They are more environmentally friendly than fossil fuels, given that carbon in the fuel is derived from atmospheric carbon dioxide, a greenhouse gas that contributes to climate change. The production of biodiesel from microalgae can also be combined with processes such as wastewater treatment or coupled to carbon dioxide capture systems of fossil fuel burning industrial plants. o A variety of techniques can be used to harvest the lipids from the algae, and this is one of the most expensive parts of algal biodiesel production. Once the lipids have been isolated from the algae, chemical treatments separate the glycerol from the fatty acid chains in the triglyceride. Further refinement yields a biodiesel product. Choose Answer • Given what you now know, what information and/or problemsolving approach is most likely to produce the correct answer? o In the proposed biofuel system, would kelp fronds provide habitat for endangered sea otters? No. Biofuels are made from microalgae in ponds Version 1
21
or tanks, not kelp in the ocean. o Is a benefit of biofuel that it doesn’t need to be burned to provide energy? No, it still needs to be burned like any other fuel. o Is a benefit of biofuel that the algae can be filtered from the water and used for fuel without refinement? No. The lipids need to be refined from the algae, and this expensive procedure limits the efficiency of the process. o Is a benefit of biofuel that it contributes less carbon to the atmosphere overall? Yes. Since the algae first captures carbon from the atmosphere (CO2 that it uses to grow), the overal release of carbon from burning the fuel is balanced. o Is a benefit of biofuel that it is a renewable resource? Yes, unlike fossil fuels, we can grow more algae. o Is a benefit of biofuel that the system could be paired with wastewater treatment? Yes, and could also be paired with an industrial carbon capture system. Reflect on Process • Did your problem-solving process lead you to the correct answer? If not, where did the process break down or lead you astray? How can you revise your approach to produce a more desirable result? o The question required you to take what you already know and apply it to this unfamiliar situation. o Did you recognize that the benefits of algae biofuels are that they are renewable, they release less carbon overall, and they can be paired with wastewater treatment? 7) [A, B] 8) [A, B, C] 9) A
Version 1
22
• What is the goal of digesting both the gene and the vector with restriction enzymes? • What must be true of the DNA ends for the gene to be able to insert and ligate into the vector successfully? • Can you draw out the results of each enzyme to help answer this question?
Version 1
23
Clarify Question • What is the key concept addressed by the question? o This question addresses restriction enymes. Gather Content • What do you already know about restriction enzymes? o Restriction enzymes recognize a specific DNA sequence, ranging from 4 bases to 12 bases, and cleave the DNA at a specific base within this sequence. o The recognition sites for most type II enzymes are palindromes. The palindromic DNA sequence reads the same from 5′ to 3′ on one strand as it does on the complementary strand. o Given this kind of sequence, cutting the DNA at the same base on either strand produces staggered cuts that leaves sticky ends,or overhangs. Consider Possibilities • What other information is related to the question? Which information is most useful? o What is the goal of digesting both the gene and the vector with restriction enzymes? The goal is to allow the insert ends to stick to the ends of the vector, so that it can be ligated in. o What must be true of the DNA ends for the gene to be able to insert and ligate into the vector successfully? The sticky ends of the gene and vector must complement each other–they must be compatible. o Can you draw out the results of each enzyme to help answer this question? o On the vector, Ncol - C*CATGG would create 5’ sticky ends of CATG Version 1
24
o Sunl - C*GTACG would create 5’ sticky ends of GTAC o Kpnl - GGTAC*C would create 3’ sticky ends of GTAC o CViAll - C*ATG would create 5’ sticky ends of ATG o Sphl - G*CATGC would create 5’ sticky ends of CATG Choose Answer • Given what you now know, what information and/or problemsolving approach is most likely to produce the correct answer? o For digesting the gene to be inserted, only Sphl - G*CATGC would create the same compatible sticky ends as NcoI. o Both create 5’ ends of CATG. o Remember, since the enzyme cleaves on both strands, the other strand is cleaved by SphI like 3’ CGTAC*G 5’. So the overhang with both enzymes is four nucleotides, 5’ CATG 3’. They are compatible ends even though the recognition sites are slightly different. Reflect on Process • Did your problem-solving process lead you to the correct answer? If not, where did the process break down or lead you astray? How can you revise your approach to produce a more desirable result? o This question asked you to choose the best restriction enzyme. o Did you recognize that the overhangs needed to complement, that is, they needed to be the same? o Did you recognize that a 5’ overhang cannot bind with a 3’ overhang? 10) C 11) A 12) B 13) C
Version 1
25
14) A 15) B 16) D 17) A 18) C 19) C 20) B
Version 1
26
Clarify Question ● What is the key concept addressed by the question? question asks about testing the effect of a mutation.
● The
● What type of thinking is required? ● You are being asked to apply your knowledge of the effect of a mutation. Gather Content ● What do you know about testing the effect of a mutation? What other information is related to the question? ● One way to measure the effect of a mutation in DNA on the phenotype of an organism is to produce the mutation in an organism. For examining a mutation in vision, an organism with eyes would be best. Using bacteria would not work as well as mice in this example. Tests like gels and RT-PCR would look at the DNA or RNA but not the effect on a whole organism’s phenotype. Choose Answer ● Given what you now know, what information is most likely to produce the correct answer? ● The best answer is to make the mutation in mice and study their vision. Bacteria would not be as good for a model organism because they don’t have eyes. Reflect on Process ● Did your problem-solving process lead you to the correct answer? If not, where did the process break down or lead you astray? How can you revise your approach to produce a more desirable result? ● This question asked you to apply your understanding of the effect of a mutation. If you got the correct answer, great job! If you got an incorrect answer, where did the process break down? Did you recall that model organisms without eyes would not be as effective for studying Version 1
27
colorblindness? Did you understand that measuring the DNA would not tell you anything about the phenotype of the organism?
21) C
Version 1
28
Clarify Question • What is the key concept addressed by the question? o This question addresses restriction enzymes. • What type of thinking is required? o This question is asking you to analyze the information given, using logic, to dissect the problem and determine the answer. • What key words does the question contain and what do they mean? o Type II restriction enzymes cut DNA at specific sites, typically palindromes. o The overhangs or sticky ends are the jagged ends of DNA left after restriction digestion, since one strand ends up longer than the other. Gather Content • What do you already know about restriction enzymes? o Restriction enzymes recognize a specific DNA sequence, ranging from 4 bases to 12 bases, and cleave the DNA at a specific base within this sequence. o The recognition sites for most type II enzymes are palindromes. The palindromic DNA sequence reads the same from 5′ to 3′ on one strand as it does on the complementary strand. o Given this kind of sequence, cutting the DNA at the same base on either strand produces staggered cuts that leaves sticky ends,or overhangs. Consider Possibilities • Consider the different answer options. Which can you rule out? o Could the resulting sticky ends be 5' GATCC 3' and 5' GATCC 3'? No, because the last C is paired, not overhanging, since the enzyme cut one nucleotide up on the other strand. Version 1
29
o Could the resulting sticky ends be 5' GATCC 3' and 5' GGATC 3' No, the sticky ends are always the same on both strands. o Could the resulting sticky ends be 5' GATC 3' and 5' CTAG 3'? No, the sticky ends are always the same on both strands. o Could the resulting sticky ends be 5' G 3' and 5' G 3' or 5' G 3' and 5' C 3'? No, the overhanging portion is the four nucleotides on the other side. Choose Answer • Given what you now know, what information and/or problemsolving approach is most likely to produce the correct answer? o Do this: o If we write out both strands, showing asterisks for the cuts: o 5' G*GATCC 3' o 3’ CCTAG*G 5’ o We see that the two overhangs look like: o 5' GATCC 3' o G 5’ o and o 5' G o 3’ CCTAG 5’ o If you read both 5’ to 3’, they are both the same: 5' GATC 3' and 5' GATC 3'. Reflect on Process • Did your problem-solving process lead you to the correct answer? If not, where did the process break down or lead you astray? How can you revise your approach to produce a more desirable result? o This question asked what the sticky ends would look like on each Version 1
30
strand after being cut with a restriction enzyme. o The question required you to analyze the information given, using logic, to dissect the problem and determine the answer. o Did you recognize that the sticky ends are the same for both strands? o Did you realize that the overhanging portion would be only four nucleotides? o Did you try writing out the two strands to help answer the question? 22) A
Version 1
31
Clarify Question • What is the key concept addressed by the question? o This question addresses molecular techniques. • What type of thinking is required? o This question is asking you to take what you already know and apply it to this unfamiliar situation. • What key words does the question contain and what do they mean? o Forensic analysis is the science of analzying data from crime scenes. Gather Content • What do you already know about molecular techniques? o There are a variety of molecular techniques for many purposes. o Some are used on RNA, some on proteins, and some on DNA. Consider Possibilities • Consider the different answer options. Which can you rule out? o Would fluorescent in situ hybridization (FISH) allow the investigators to compare the DNA? No, because FISH is used to detect one particular small target sequence in a mixture of DNA, not to compare two samples overall. o Would RNAi allow the investigators to compare the DNA? No, because that is used to inhibit expression of a gene. o Would RT-PCR allow the investigators to compare the DNA? No, because that is used to create DNA copies of mRNAs. Choose Answer • Given what you now know, what information and/or problemsolving approach is most likely to produce the correct answer? o Would DNA fingerprinting allow the investigators to compare the Version 1
32
DNA? Yes, that is the purpose of that technique. Reflect on Process • Did your problem-solving process lead you to the correct answer? If not, where did the process break down or lead you astray? How can you revise your approach to produce a more desirable result? o This question asked which technique is used to identify the source of DNA at a crime scene. o The question required you to take what you already know and apply it to this unfamiliar situation. o Did you remember that DNA fingerprinting is the tecnique used to compare DNA samples? 23) A 24) A
Version 1
33
Clarify Question • What is the key concept addressed by the question? o This question addresses transgenic mouse technology. • What type of thinking is required? o This question is asking you to weigh and judge evidence, or evaluate, to choose the best of the possible answers. Gather Content • What do you already know about transgenic mouse technology? o The knockout mouse technique creates a loss-of-function of the target gene by inserting a neomycin cassette. o The knockin mouse replaces the normal gene with a modified version created by the experimenter. o PCR-based random mutagenesis introduces random mutations into the DNA being amplified by PCR. o CRISPR-Cas9 can be used to introduce mutations into specific genes. Choose Answer • Given what you now know, what information and/or problemsolving approach is most likely to produce the correct answer? oGiven that the experimenter wants to mimic a loss-of-function of a gene throughout the body, the best approach would be to create a knockout mouse. oSince the gene is known, random mutagenesis would not be helpful. Similarly, taking a tissue-specific approach with CRISPR-Cas9 would only inactivate thegene in certain cells. The question asks about a genetic disease, so we can assume that the gene is deleted from all tissues. Reflect on Process Version 1
34
• Did your problem-solving process lead you to the correct answer? If not, where did the process break down or lead you astray? How can you revise your approach to produce a more desirable result? o The question required you to weigh and judge evidence, or evaluate, to choose the best of the possible answers. o Did you recognize that the knockout mouse is the best choice, because it mimics the disease in all tissues? 25) C
Version 1
35
Clarify Question • What is the key concept addressed by the question? o This question addresses DNA techniques. • What type of thinking is required? o This question is asking you to take what you already know and apply it to this unfamiliar situation. • What key words does the question contain and what do they mean? o Researchers who study ancient DNA are often trying to get DNA out of fossils or specimens that are extremely old, where most of the DNA is long gone. Gather Content • What do you already know about DNA techniques? o Different DNA techniques allow researchers to take a small amount of DNA and make more, or determine the size of DNA fragments, ormatch DNA to a particular probe. o In this case, when you have a vanishingly tiny amount of DNA like that from an ancient sample, the first thing you must do is to amplify it into a larger amount Consider Possibilities • Consider the different answer options. Which can you rule out? o Does RT-PCR amplify DNA? Yes, but it uses mRNA as a starting material. o Does gel electrophoresis amplify DNA? No, it sorts DNA fragments by length. o Does inserting the DNA into a vector amplify the DNA? Yes, this can be a way to amplify DNA. But it is not a good way to amplify very tiny amounts of precious DNA, because most DNA will be lost in the Version 1
36
process. Choose Answer • Given what you now know, what information and/or problemsolving approach is most likely to produce the correct answer? o Does polymerase chain reaction amplify DNA? Yes! PCR is the best technique for amplify DNA from tiny amounts. Reflect on Process • Did your problem-solving process lead you to the correct answer? If not, where did the process break down or lead you astray? How can you revise your approach to produce a more desirable result? o This question asked what would be the best technique for a tiny amount of DNA. o The question required you to take what you already know and apply it to this unfamiliar situation. o Did you recognize that PCR is the best way to amplify small amounts of DNA? 26) A
Version 1
37
Clarify Question ● What is the key concept addressed by the question? question asks about glyphosate resistance.
● The
● What type of thinking is required? ● You are being asked to apply your knowledge of glyphosate resistance. Gather Content ● What do you know about glyphosate resistance? What other information is related to the question? ● Agriculturally important broadleaf plants such as soybeans have been genetically engineered to be resistant to glyphosate, a powerful, biodegradable herbicide that kills most actively growing plants. Glyphosate works by inhibiting an enzyme called 5-enolpyruvylshikimate-3-phosphate (EPSP) synthase, which plants require to make aromatic amino acids. Animals do not make aromatic amino acids; rather, they are obtained from the diet, so animals living in and around crops, or in waterways receiving crop runoff, are unaffected by glyphosate. To make glyphosate-resistant plants, scientists used a Ti plasmid to insert extra copies of the EPSP synthase gene into plants. These engineered plants produce 20 times the normal level of EPSP synthase, enabling them to synthesize aromatic amino acids and grow despite glyphosate’s inhibition of the enzyme. Choose Answer ● Given what you now know, what information is most likely to produce the correct answer? ● Plants resistant to glyphosate overexpress production of aromatic amino acids. Glyphosate is a chemical, and not bacteria. The soil contains low levels of amino acids, so increasing uptake from soil is unlikely. Resistance to some drugs can be due to metabolism of the drug, but this is not the case for glypshosate.
Version 1
38
Reflect on Process ● Did your problem-solving process lead you to the correct answer? If not, where did the process break down or lead you astray? How can you revise your approach to produce a more desirable result? ● This question asked you to apply your understanding of glyphosate resistance. If you got the correct answer, great job! If you got an incorrect answer, where did the process break down? Did you recall that in glyphosate-resistance productionof aromatic amino acids increase? Did you understand that glyphosate is a chemical and not a bacterium?
27) C
Version 1
39
Clarify Question • What is the key concept addressed by the question? o This question addresses sequences recognized by restriction enzymes. Gather Content • What do you already know about restriction enzymes? o Restriction enzymes recognize a specific DNA sequence, ranging from 4 bases to 12 bases, and cleave the DNA at a specific base within this sequence. o The recognition sites for most type II enzymes are palindromes. The palindromic DNA sequence reads the same from 5′ to 3′ on one strand as it does on the complementary strand. o Given this kind of sequence, cutting the DNA at the same base on either strand produces staggered cuts that leaves sticky ends,or overhangs. Consider Possibilities • What other information is related to the question? Which information is most useful? o We have learned that restriction enzymes usually recognize sites that are palindromes, that is, the sequence reads the same from 5’ to 3’ on each strand. Choose Answer • Given what you now know, what information and/or problemsolving approach is most likely to produce the correct answer? o The only sequence listed that is a palindrome is TGGCCA. o The double strand would read the same forwards and back: Version 1
40
o 5’ TGGCCA 3’ o 3’ ACCGGT 5’ Reflect on Process • Did your problem-solving process lead you to the correct answer? If not, where did the process break down or lead you astray? How can you revise your approach to produce a more desirable result? o This question asked you to identify the most likely restriction enzyme recognition site. o The question required you to take what you already know and apply it to this unfamiliar situation. o Did you recognize that restriction sites are usually palindromes? 28) C
Version 1
41
Clarify Question • What is the key concept addressed by the question? o This question addresses the PCR technique. Gather Content • What do you already know about the PCR technique? o PCR is a method for amplifying DNA from a starting template. o It has three steps that are repeated in multiple rounds: denaturation, annealing of Primer, and synthesis. o Each of these steps takes place at a different optimum temperature. Consider Possibilities • Consider the different answer options. Which can you rule out? o Does antibody binding require the enzyme to be heat stabile? No, there is no antibody binding in the PCR technique. o Does the synthesis step require the enzyme to be heat stabile? No,synthesis uses the enzyme, but it takes place at 72°C. o Does the primer annealing step require the enzyme to be heat stabile? No. Annealing takes place at around 50–60°C. Choose Answer • Given what you now know, what information and/or problemsolving approach is most likely to produce the correct answer? o Does the denaturation step require the enzyme to be heat stabile? Yes! denaturation takes place at 95°C, which is hot enough to quickly destroy normal enzymes. Reflect on Process Version 1
42
• Did your problem-solving process lead you to the correct answer? If not, where did the process break down or lead you astray? How can you revise your approach to produce a more desirable result? o This question asked why PCR requires a heat stable polymerase. o The question required you to take what you already know and apply it to this unfamiliar situation. o Did you recognize that the denaturation step has the highest temperature? 29) B
Version 1
43
Clarify Question • What is the key concept addressed by the question? o This question addresses herbicide-resistance transgenes. • What key words does the question contain and what do they mean? o EPSP synthase is an enzyme normally only in plants that allows them to synthesize aromatic amino acids. o Glyphosate is an herbicide that kills plants by blocking EPSP. o Glyphosate-resistant plants are resistant because they have multiple copies of the EPSP gene. Gather Content • What do you already know about herbicide resistance transgenes? o Glyphosate is a powerful, biodegradable herbicide that kills most actively growing plants. o It works by inhibiting an enzyme called 5-enolpyruvylshikimate-3phosphate (EPSP) synthase, which plants require to make aromatic amino acids. o Animals do not make aromatic amino acids, so animals living in and around crops, or in waterways receiving crop runoff, are unaffected by glyphosate. o To make glyphosate-resistant plants, scientists used a Ti plasmid to insert extra copies of the EPSP synthase gene into plants so they produce 20 times the normal level of EPSP synthase, enabling them to grow despite glyphosate’s inhibition. Consider Possibilities • Consider the different answer options. Which can you rule out? o Would the animal have no effect from eating in the glyphosateVersion 1
44
resistant field? Most animals do not, but then animals normally do not have the EPSP gene. o Would the animals also become glyphosate-resistant? No, that could only happen if they acquired the transgene in multiple copies–very unlikely to happen. o Would the animals no longer need to make their own aromatic amino acids? Not likely. Choose Answer • Given what you now know, what information and/or problemsolving approach is most likely to produce the correct answer? o Would the animals become very sick from the glyphosate that was sprayed on the plants? Yes, very likely. If they had the plant enzyme, they would be affected just like the plants. Remember that glyphosate is sprayed in the field to kill weeds, so the trangenic plants are not affected but weeds are killed. Reflect on Process • Did your problem-solving process lead you to the correct answer? If not, where did the process break down or lead you astray? How can you revise your approach to produce a more desirable result? o This question asked what would happen to an animal with EPSP if it ate out of a field of glyphosate resistant plants. o The question required you to take what you already know and apply it to this unfamiliar situation. o Did you recognize that the animal could not acquire glyphosate resistance? o Did you realize that the glyphosate would block the animals’ EPSP enzymes? Version 1
45
30) C
Version 1
46
Clarify Question • What is the key concept addressed by the question? o This question addresses restriction enzymes. Gather Content • What do you already know about restriction enzymes? o Restriction enzymes recognize a specific DNA sequence, ranging from 4 bases to 12 bases, and cleave the DNA at a specific base within this sequence. o The recognition sites for most type II enzymes are palindromes. The palindromic DNA sequence reads the same from 5′ to 3′ on one strand as it does on the complementary strand. o Given this kind of sequence, cutting the DNA at the same base on either strand produces staggered cuts that leaves sticky ends, or overhangs. o These cuts and overhangs are used to cut and paste together pieces of DNA to create new combinations. Consider Possibilities • What other information is related to the question? Which information is most useful? o Would a messy enzyme be useful in molecular cloning? Not likely, because its behavior would be unpredictable. Choose Answer • Given what you now know, what information and/or problemsolving approach is most likely to produce the correct answer? o In molecular cloning, precision is paramount. If the enzyme cut in the Version 1
47
wrong spot, the resulting product would be wrong. o It would be much more difficult to piece together the desired sequence if the enzyme was unpredictable. Reflect on Process • Did your problem-solving process lead you to the correct answer? If not, where did the process break down or lead you astray? How can you revise your approach to produce a more desirable result? o This question asked whether a messy restriction enzyme would be a problem. o The question required you to weigh and judge evidence, or evaluate, to choose the best of the possible answers. o Did you recognize that having enzymes that cut reliably in precise places is crucial for successful cloning? 31) C
Version 1
48
Clarify Question • What is the key concept addressed by the question? o This question addresses restriction enzymes. • What key words does the question contain and what do they mean? o A 6.85 kb EcoRI fragment of DNA is a piece of DNA 6850 base pairs long, that was already cut with the enzyme EcoRI. o Restriction sites are recognition sites for different restriction enzymes. o Digesting the DNA with PstI means to treat the DNA with the enzyme PstI, cleaving it. Gather Content • What do you already know about restriction enzymes? o Restriction enzymes recognize a specific DNA sequence, ranging from 4 bases to 12 bases, and cleave the DNA at a specific base within this sequence. o The recognition sites for most type II enzymes are palindromes. The palindromic DNA sequence reads the same from 5ʹ to 3ʹ on one strand as it does on the complementary strand. o Enzymes can cut in more than one spot on the same DNA fragment, as long as the treatment is given enough time. Consider Possibilities • What other information is related to the question? Which information is most useful? o Where will the enzyme cut? Only at the sites marked PstI. o Assume that the reaction is set up so that the enzyme cuts each DNA completely. Choose Answer • Given what you now know, what information and/or problem-solving approach is most likely to produce the correct answer? o Since there are two PstI sites, the fragment will be cut twice, leaving three pieces of DNA. o Remember this was already a DNA fragment, so the ends are free. Reflect on Process • Did your problem-solving process lead you to the correct answer? If not, where did the process break down or lead you astray? How can you revise your approach to produce a more desirable result? o This question asked how many fragments would result for the DNA with a PstI digestion. Version 1
49
o Did you recognize that by cutting in two spots, the DNA is broken into three fragments? o Did you read carefully enough to understand that the DNA had already been cut with EcoRI, and now you are cutting with PstI?
32) A
Version 1
50
Clarify Question • What is the key concept addressed by the question? o This question addresses restriction enzymes. • What key words does the question contain and what do they mean? o A 6.85 kb EcoRI fragment of DNA is a piece of DNA 6850 base pairs long, that was already cut with the enzyme EcoRI. o Restriction sites are recognition sites for different restriction enzymes. o Digesting the DNA with PstI means to treat the DNA with the enzyme PstI, cleaving it. o Migrating on a gel means moving down a gel to the end in the gel electrophoresis technique. Gather Content • What do you already know about restriction enzymes? o Restriction enzymes recognize a specific DNA sequence, ranging from 4 bases to 12 bases, and cleave the DNA at a specific base within this sequence. o The recognition sites for most type II enzymes are palindromes. The palindromic DNA sequence reads the same from 5ʹ to 3ʹ on one strand as it does on the complementary strand. o Enzymes can cut in more than one spot on the same DNA fragment, as long as the treatment is given enough time. Consider Possibilities • What other information is related to the question? Which information is most useful? o What kind of DNA fragments migrate fastest on a gel? The shortest fragments. Choose Answer • Given what you now know, what information and/or problem-solving approach is most likely to produce the correct answer? o Since there are two PstI sites, the fragment will be cut twice, leaving three pieces of DNA. o Remember this was already a DNA fragment, so the ends are free. o What are the lengths of the different PstI fragments? 803, 2622, and 3425. o Which is the shortest and fastest? 803. Reflect on Process • Did your problem-solving process lead you to the correct answer? If not, where did the process break down or lead you astray? How can you revise your approach to produce a more Version 1
51
desirable result? o This question asked how many fragments would result for the DNA with a PstI digestion. o The question required you to analyze the information given, using logic, to dissect the problem and determine the answer. o Did you recognize that by cutting in two spots, the DNA is broken into three fragments? o Did you read carefully enough to understand that the DNA had already been cut with EcoRI, and now you are cutting with PstI? o Did you remember that the shortest fragments migrate the fastest on a gel?
33) B
Version 1
52
Clarify Question • What is the key concept addressed by the question? o This question addresses gene expression. • What key words does the question contain and what do they mean? o Aerobic and anaerobic conditions refer to conditions with or without oxygen. Gather Content • What do you already know about gene expression? o Gene expression is controlled at multiple levels: transcriptional, posttranscriptional, transational, and posttranslational. o In bacteria, however, the most common and important level of regulation is transcriptional. Consider Possibilities • Consider the different answer options. Which can you rule out? o Would an experiment to sequence the genome address the hypothesis? No, because the genome always stays the same, and the hypothesis was specifically about gene expression. o Would an experiment to PCR a region of the genome address the hypothesis? No, because the entire genome will remain the same in both conditions, and the hypothesis was specifically about gene expression. o Would an experiment to measure levels of RNA polymerase address the hypothesis? No, because levels of RNA polymerase are unlikely to change, and would not be a likely way of regulating gene expression. Choose Answer • Given what you now know, what information and/or problemVersion 1
53
solving approach is most likely to produce the correct answer? o Would creating two cDNA libraries address the hypothesis? Yes, because that would allow you to see differences in transcript levels for different genes in each condition. Reflect on Process • Did your problem-solving process lead you to the correct answer? If not, where did the process break down or lead you astray? How can you revise your approach to produce a more desirable result? o This question asked how to test a hypothesis about gene expression in two different conditions. o The question required you to take what you already know and apply it to this unfamiliar situation. o Did you recognize that the hypothesis was saying that gene expression would be different, and that basically means the set of expressed mRNAs? 34) A
Version 1
54
Clarify Question • What is the key concept addressed by the question? o This question addresses PCR techniques. Gather Content • What do you already know about PCR techniques? o PCR, polymerase chain reaction, mimics the processes of DNA replication. An individual reaction is cycled through a series of steps where each step is analogous to a step in DNA replication: denaturation, annealing, and synthesis. It starts with small amounts of DNA and makes more DNA. o RT-PCR, reverse transcription PCR, makes amplified DNA from mRNA. It can allow you to create an intron-less gene matching an expressed mRNA. o Quantitative RT-PCR is a sensitive way to determine relative levels of expression of an mRNA in a particular tissue or sample. Choose Answer • Given what you now know, what information and/or problemsolving approach is most likely to produce the correct answer? o Abby wants to determine changes in gene expression levels in a tissue. She needs a way to quantify mRNA levels, so she uses quantitative RT-PCR. o Bob is trying to extract genetic information from a wooly mammoth fossil. He should amplify DNA, not RNA, since DNA is more stable over millions of years. He uses PCR. o Chris needs to express a gene in bacteria. Bacteria cannot do alternative splicing, and she wants to use a particular expression variant from kidney cells. So she uses RT-PCR to create a DNA copy of the Version 1
55
mRNA from those cells. Reflect on Process • Did your problem-solving process lead you to the correct answer? If not, where did the process break down or lead you astray? How can you revise your approach to produce a more desirable result? o Did you recognize that PCR is best when simple amplification of DNA is needed? o Did you recognize that RT-PCR is used when you need to create a DNA copy of an mRNA? o Did you recognize that quantitative RT-PCR is best for determining changes in levels of an mRNA? 35) A
Version 1
56
Clarify Question • What is the key concept addressed by the question? o This question addresses genome editing. • What type of thinking is required? o This question is asking you to weigh and judge evidence, or evaluate, to choose the best of the possible answers. Gather Content • What do you already know about genome editing? o Genome editing uses either a TALE protein or an RNA to target a nuclease to a particular gene in the genome. o The nuclease then cuts the targeted DNA, which can be repaired to leave a deletion, or repaired to incorporate a new desired sequence of DNA. Choose Answer • Given what you now know, what information and/or problemsolving approach is most likely to produce the correct answer? o All of the diseases listed originate with a single abnormal gene that could be targeted by genome editing, except for trisomy 21, which involves a whole extra chromosome. o It would be difficult to regulate the expression of a whole chromosome using the single gene targeting techniques of genome editing. Reflect on Process • Did your problem-solving process lead you to the correct answer? If not, where did the process break down or lead you astray? How can Version 1
57
you revise your approach to produce a more desirable result? o The question required you to weigh and judge evidence, or evaluate, to choose the best of the possible answers. o Did you recognize that genome editing works by targeting and modifying a particular gene in the genome? It would be difficult to use the technique to target multiple genes–or a whole chromosome–at once. 36) A
Version 1
58
Clarify Question • What is the key concept addressed by the question? o This question addresses the genetic code. Gather Content • What do you already know about the genetic code? o The universal nature of the genetic code allows transgenesis to work. o A human gene can be placed into a mouse genome, and the cells of the mouse will express the gene. Similarly, a human gene can be placed into the genome of an E. coli bacterium, and the bacteria will make the encoded protein. o This is because the cells of organisms of different species interpret genetic information identically. o The genetic code used to decipher the information in a gene is the same code used in bacteria, archaea, and eukaryotes. o There are minor variations to the genetic code, however. For example, the genetic code of mitochondriais slightly different from the genetic code used to interpret nuclear genes. Choose Answer • Given what you now know, what information and/or problemsolving approach is most likely to produce the correct answer? o The ciliates are unusual–the genetic code is nearly universal. That is why scientists are able to express transgenes correctly in different species. Reflect on Process • Did your problem-solving process lead you to the correct answer? If Version 1
59
not, where did the process break down or lead you astray? How can you revise your approach to produce a more desirable result? o The question required you to take what you already know and apply it to this unfamiliar situation. o Did you recognize that the genetic code is nearly (but not quite) universal? 37) F
Version 1
60
Clarify Question • What is the key concept addressed by the question? o This question addresses bioremediation. Gather Content • What do you already know about bioremediation? o Certain microorganisms are capable of metabolizing hydrocarbon pollutants. The degradation or metabolism of hydrocarbon pollutants by microorganisms is called bioremediation. o The rate at which these microorganisms metabolize hydrocarbons is affected by a number of variables including temperature, physical and chemical properties of the pollutant, oxygen availability, pH, salinity, other microorganisms present, and perhaps most critically, nutrient availability. o Lab studies have shown that nutritional supplementation of certain microorganisms leads to a significant increase in the rate at which hydrocarbon pollutants can be metabolized. Choose Answer • Given what you now know, what information and/or problemsolving approach is most likely to produce the correct answer? o All of the listed factors could be important for allowing the bacteria to thrive so they can clean up the oil. Reflect on Process • Did your problem-solving process lead you to the correct answer? If not, where did the process break down or lead you astray? How can you revise your approach to produce a more desirable result? Version 1
61
o Did you recognize that a wide variety of factors can affect the ability of bacteria to survive and multiply, so they are able to metabolize pollutants? 38) A
Version 1
62
Clarify Question • What is the key concept addressed by the question? o This question addresses FISH and gene chip diagnostics. • What type of thinking is required? o This question is asking you to take what you already know and apply it to this unfamiliar situation. Gather Content • What do you already know about FISH and gene chip diagnostics? o Fluorescent in situ hybridization (FISH) takes advantage of the ability of DNA to be reversibly denatured and renatured. Hybridization uses a probe with either a fluorescent. In the case of FISH, the target DNA is an actual metaphase spread of chromosomes, or cells with intact chromosomes. This allows the detection of gross chromosomal abnormalities, such as large deletions, inversions, duplications, and translocations. o Gene chips also known as DNA microarrays, are collections of hundreds to thousands of different DNA sequences that are spotted onto a solid surface such as a microscope slide. The DNA sequences are usually short chemically synthesized pieces of DNA, or short sequences of DNA amplified by PCR or RT-PCR. Biomarkers can be diagnostic of a disease state, be informative about the progress of a disease, or help monitor the response to therapy. In the case of gene chips, the biomarker is the mRNA synthesized from a particular gene. For example, individuals with certain autoimmune diseases or chronic inflammatory conditions will express different sets of genes in immune cells than will healthy individuals. Gene chips can be used to determine which genes are being expressed and the relative levels of expression for different genes. Version 1
63
Choose Answer • Given what you now know, what information and/or problemsolving approach is most likely to produce the correct answer? o Since the disease is characterized by a gross chromosomal abnormality, FISH is the best technique to use here. o Whole genome sequencing would work as well, but is too complex and expensive. FISH is more practical. Reflect on Process • Did your problem-solving process lead you to the correct answer? If not, where did the process break down or lead you astray? How can you revise your approach to produce a more desirable result? o The question required you to take what you already know and apply it to this unfamiliar situation. o Did you recognize that the best technique for recognizing a chromosomal duplication is FISH, because there would be an extra fluorescent spot present? 39) A
Version 1
64
Clarify Question • What is the key concept addressed by the question? o This question addresses transgenic crops. Gather Content • What do you already know about transgenic crops? o Agriculturally important broadleaf plants such as soybean have been genetically engineered to be resistant to glyphosate, a powerful, biodegradable herbicide that kills most actively growing plants. Herbicide-resistant crops allow for no-till planting. o Tilling is the process where soils are turned over to prepare them for planting. This acts to remove weeds, which compete with crop plants for nutrients, and improve irrigation. Soils that are not tilled have greater levels of organic and inorganic nutrients that promote growth, often retain more water, and are less prone to soil erosion. Spraying glyphosate on the crop during the season means that weeds are killed eliminating the need to till while the crops are growing. o Bt crops are resistant to some insect pests. The approach is to insert into crop plants genes encoding proteins harmful to insect pests, but harmless to other organisms. The most commonly used protein is a toxin produced by the soil bacterium Bacillus thuringiensis (Bt toxin). When insects ingest Bt toxin, endogenous enzymes convert it into an insectspecific toxin, causing paralysis and death. Because these enzymes are not found in other animals, the protein is harmless to them. Choose Answer • Given what you now know, what information and/or problemsolving approach is most likely to produce the correct answer? o Farmers use glyphosate to destroy weeds without having to till the Version 1
65
soil, so a tiller would not be used on the fields. Reflect on Process • Did your problem-solving process lead you to the correct answer? If not, where did the process break down or lead you astray? How can you revise your approach to produce a more desirable result? o Did you recognize that glyphosate resistance allows farmers to target weeds without tilling? 40) A 41) B 42) C 43) A
Version 1
66
CHAPTER 18 CHECK ALL THE APPLY. Choose all options that best completes the statement or answers the question. 1) Which of the following statements is/are accurate about the role of genome analysis in crop improvement? Check all that apply. A) Conservation of synteny will hinder our ability to find agriculturally important genes in plants. B) Arabidopsis is being genetically modified as a crop plant. C) Sequencing of the rice genome was important because it is related to many other cereal crop plants. D) The genes in one crop plant type can help to identify useful genes in another crop.
2) As the director of a new nonprofit functional genomics research center, it is your job to oversee the set up of each laboratory necessary for achieving the center's research goals. Therefore, you plan to include a(n) (check all that apply) A) DNA microarray facility. B) proteomics lab. C) lab to improve sequencing speed and cost. D) mutagenesis facility. E) ancient DNA laboratory.
3) When a segment of DNA duplicates, causing two copies of a gene to be present in the chromosome, the duplicate copy could possibly (check all that apply) A) lose function due to mutation. B) cause the organism to become a new species. C) gain a new function due to mutation. D) become a pseudogene. E) become a centromere.
Version 1
1
4) Genome sequencing of our human ancestor, Neanderthals (Homo neanderthalensis), revealed that Neanderthals have a FOXP2 gene with the same two amino acid changes observed in H. sapiens FOXP2. What can be concluded based on this finding? (Check all that apply) A) Neanderthals evolved from H. sapiens. B) H. sapiens evolved from Neanderthals. C) Neanderthals may have had the ability to use language. D) These two amino acid changes arose prior to the common ancestor of Neanderthals and H. sapiens. E) Verbal speech was impossible for Neanderthals.
5) Select features that make next-generation sequencing technologies faster and less expensive than dideoxy terminator sequencing. Choose all that apply. A) A genomic library is not needed. B) Millions of reactions are performed simultaneously. C) Electrophoresis is not required. D) Primers are not required. E) Fluorescent dyes are not used.
6) Select challenges that scientists faced in sequencing the wheat genome. Choose all that apply. A) The wheat genome contains nonstandard nucleotides. B) The wheat genome is polyploid. C) The wheat genome is highly repetitive. D) The wheat genome is enormous. E) Wheat DNA is difficult to extract for sequencing.
7) The important advances stemming from comparative genomics include the ability to (check all that apply)
Version 1
2
A) draw particular conclusions about species. B) draw general conclusions about evolution. C) infer function about an unsequenced genome using synteny. D) develop ahaplotype map. E) distinguish how proteins bind to receptors.
8)
Driver mutations in cancer A) can be in oncogenes. B) can be in tumor suppressor genes. C) affect progress to cancer. D) accumulate but do not lead to cancer.
MULTIPLE CHOICE - Choose the one alternative that best completes the statement or answers the question. 9) Mobile bits of DNA that can move from one genomic location to another and do not code for a protein (although some code for proteins required for their movement) are called A) simple sequence repeats. B) transposons. C) exons. D) introns. E) pseudogenes.
10) Your research team has been asked to quantify levels of cellular RNAs in rats before and after exercise. The primary focus of your work will be to analyze the A) proteome. B) spliceosome. C) nucleosome. D) different RNA motifs. E) transcriptome.
Version 1
3
11) When looking at microarray data verses phenotypic behaviors in a model organism, you notice that certain genes seem to have lost their function. Therefore you decide to classify them as A) nonfunctioning introns. B) inactivatedexons. C) templates. D) transposons. E) pseudogenes.
12) Nearly half of the human genome is composed of a noncoding DNA that can move from one chromosomal location to another. This type of noncoding DNA is/are called A) introns. B) microRNA genes. C) tandem clusters. D) transposons. E) pseudogenes.
13) To study gene expression, microarrays are created by robotically placing DNA on to a microscope slide and probing with A) mRNA from different sources. B) DNA from different sources. C) SNPs from different sources. D) STSs from different sources. E) proteins from different sources.
14) Sequence-tagged sites (STS) are used to assemble DNA maps from a given species. STSs are useful in this regard because
Version 1
4
A) each STS is found many times within the genome. B) they provide a way to order DNA fragments. C) they generate DNA fragments of a reasonable size for mapping. D) they are only found in genes, not in noncoding DNA.
15) The sequencing method that cuts DNA segments into fragments, arranges those fragments based on overlapping nucleotide sequences, and then clones these fragments is called A) shotgun sequencing. B) clone-contig sequencing. C) RFLP sequencing. D) consensus sequencing.
16) The sequencing method that cuts the DNA of an entire chromosome into small fragments and then clones these fragments is called A) shotgun sequencing. B) clone-contig sequencing. C) RFLP sequencing. D) consensus sequencing.
17) Rice and its grain relatives, maize, barley, and wheat, diverged from a common ancestor about 50 million years ago. What main advantage does this afford research in crop genomics? A) Genome information from these plants can be used interchangably. B) Single nucleotide polymorphisms are clearly defined in many plants used as crops. C) STS can be isolated in more than one plant leading to better crop production. D) It allows for useful genes in crops to be identified in multiple species of plants.
18) Because it has undergone whole-genome duplication, the animal with the highest number of genes is Version 1
5
A) pufferfish. B) human. C) raccoon. D) chicken.
19) Traditionally, toxicology studies have involved numerous bioassays on rodents. However, a more reliable and reproducible approach to study the genome-wide effects of a toxin on gene expression is A) a protein microarray. B) automated DNA sequencing. C) mass spectroscopy. D) a DNA microarray.
20)
How did the rice genome project influence other genome projects? A) It was the first genome sequenced, so it proved it could be done. B) It sped up the genome analysis of other cereal crops, due to synteny. C) It sped up the analysis of the human genome, since rice has even more genes. D) It helped analysis of all genomes, since it was the basis of the ENCODE algorithms.
21)
Using the genome sequencing data from each of the kingdoms, it is currently possible
A) to determine genetic differences between species very directly, examining DNA changes and identifying unknown genes. B) to build, from scratch, complex organisms with trans-kingdom genomes using newly-sequenced genes. C) to sequence genes and build, from scratch, simple organisms with trans-kingdom genomes. D) to design humans with a variety of inter-kingdom abilities built into their DNA.
22)
The protein-coding gene sequences in humans and chimpanzees
Version 1
6
A) are vastly different. B) are about 50% alike. C) are about 80% alike. D) are more than 99% alike. E) are identical.
23) The best explanation for the phenotypic differences between humans and chimps despite the great similarities in their gene-coding sequences is A) the vast differences in exons between their genomes. B) the vast differences in the introns between their genomes. C) the junk DNA of humans contains important sequences not present in chimps. D) differences in gene expression between species.
24) What type of genes would be most likely to exhibit different expression patterns between humans and nonhuman primates, such as gorillas? A) genes controlling brain development B) genes controlling mammary gland development C) genes controlling lung development D) genes controlling heart development
25) Recent research has suggested that some pseudogenes and non-protein-coding DNA code for RNAs that affect transcription of coding regions. This suggests that A) pseudogenes code for proteins. B) non-protein-coding DNA has no function. C) non-protein-coding DNA may regulate gene expression. D) pseudogenes have no function.
26) A science student asks you why there are a lower number of unique genes compared to mRNAs. The most accurate explanation is Version 1
7
A) exons are alternatively spliced creating many more mRNAs than genes. B) introns can be spliced together in multiple ways making variable mRNA sequences. C) alternative splicing add introns to exons in multiple ways forming more types of mRNA. D) splicing both introns and exons creates variable forms of mRNA.
27) What main concept accounts for the higher similarity of the chimpanzee genome to the human genome compared to the baboon or dog genome? A) Over time mammalian species have diverged, leading to genetic differences, some of which are bigger than others. B) Some species share identical genomes while others are highly variable. C) The primary differences among mammals are due to changes in gene expression. D) Species have become less diverse over time, accounting for the existance of more mutations when comparing species.
28) Given a sequencing reaction length of approximately 500 successive nucleotides, what is the absolute minimum number of sequencing reactions needed to determine the complete human genome, excluding overlap and redundancy? A) 3.1 billion B) 6200 C) 12.4 million D) 6.2 million
29) Your research group has recently determined the complete genomic sequence of the organism Examus biologisium. You instruct your group to begin genomeannotation, which involves A) shotgun cloning. B) the alignment of contigs. C) the identification of open reading frames. D) the construction of a genetic map.
Version 1
8
30) In vertebrates, the family of myogenic regulatory factors (MRFs) plays a major role in muscle development. This family consists of four related genes that have distinct but overlapping functions. This gene family is represented by only a single gene in Drosophila, called nautilus. You want to know whether nautilus plays a role like all of the MRFs put together, or more like one in particular. What is a quick and easy way to begin to address this question? A) Compare the DNA and protein sequences of all the genes to determine if nautilus is more similar to one MRF than another. B) Use a DNA microarray to compare gene activation patterns between nautilus and the MRFs. C) Compare the proteome of a cell expressing nautilus to the those of cells expressing each of the MRFs in turn. D) Generate four transgenic Drosophila strains that express the MRFs in a mutant nautilus background.
31) A pharmaceutical company is developing a new drug to treat a rare disease. They need to determine which of the cell’s proteins can bind the drug. What tool can help them survey the proteome for drug interactions? A) Protein microarray B) DNA microarray C) ENCODE D) A monoclonal antibody
32)
Dideoxy terminator sequencing relies on A) a nucleotide lacking an -OH group on the 3' carbon of the sugar. B) a nucleotide lacking a phosphate on the 5' carbon of the sugar. C) a nucleotide lacking an -OH on the 5' carbon of the sugar. D) a nucleotide lacking a phosphate on the 3' carbon of the sugar.
33) For many years scientists belived that humans had around ______ genes. The Human Genome Project revealed that humans actually have around ______ genes.
Version 1
9
A) 100,000; 20,000 B) 20,000; 100,000 C) 3 billion; 3 million D) 3 million; 3 billion E) 45,000; 20,000 F) 20,000; 45,000
34) The relative location of genes on a chromosome as determined by recombination frequencies is illustrated in a _______ map. A) DNA B) genetic C) chromosomal D) physical
35)
Using actual landmarks within DNA sequences, scientists can generate a _______ map. A) genetic B) DNA C) physical D) chromosomal
36)
Distances on a genetic map are measured in _____ units. A) centromere B) DNA C) dalton D) centimorgan
37)
Recombination frequency between genes can beused to generate a
Version 1
10
A) genetic map. B) physical map. C) restriction map. D) haplotype map.
38) Sequences of DNA assembled by identifying overlaps among smaller DNA segments are known as A) single strand polymorphisms. B) synteny. C) draft sequences. D) the proteome. E) a contig.
39)
Among the organisms listed below, which has the highest number of genes? A) Rice B) Humans C) Fruit flies D) Yeast
40) Humans are diploid organisms, containing two sets of chromosomes. How many sets of chromosomes does wheat, Triticum aestivum,have? A) 1 B) 2 C) 4 D) 6 E) 8
41)
Long interspersed elements (LINEs) are a type of
Version 1
11
A) pseudogene. B) proteome. C) exon. D) intron. E) transposon.
42) Some regions of chromosomes remain highly condensed, tightly coiled, and untranscribed throughout the cell cycle. These regions are referred to as A) transposable elements. B) single sequence repeats. C) noncoding DNA. D) short interspersed elements. E) constitutive heterchromatin.
43)
The ENCODE project seeks to A) sequence the genomes of all animals. B) sequence the DNA of all organisms. C) identify the coding sequences of human DNA. D) identify the functional elements of the human genome. E) identify the proteins encoded by the human genome.
44) The current estimates of the ENCODE project for percent functionality of the human genome based on the ENCODE definition of functionality is _____ the estimate of functionality based on the selected-effect functionality definition. A) greater than B) less than C) equal to
Version 1
12
45)
Conserved arrangements of segments of DNA in related genomes are referred to as A) synteny. B) homology. C) analogous DNA. D) a contig. E) a comparative genome.
46) What overlapping features of newly constructed genetic and physical maps enables them to more easily be correlated with one another? A) The units of distance for newly constructed genetic maps, cM, now correspond to fixed numbers of physical base-pairs making the comparison more straightforward with physical maps. B) Newly constructed genetic maps are so complete there is only one way for them to be arrangedinto a physical map. C) Newly constructed genetic maps use molecular markers allowing their location in the genome to be identified, correlating with the physical map of cloned genes. D) Advanced sequencing technologies make the genetic map obsolete, only the physical map is necessary.
47)
What fraction of genes in cottonwood and rice appear to be plant-specific genes? A) 1/3 B) 2/3 C) 1/2 D) 3/4
48)
What could account for the difference in mutation rate between rodents and primates?
Version 1
13
A) Difference in generation time B) Difference in genome size C) Differences in gene transcription D) Differences in exposure to mutagens
49) An organism you are studying has about 14,000 genes in its genome, but makes about 60,000 unique mRNAs. What is the best explanation for this observation? A) Exons are alternatively spliced creating many more mRNAs than genes. B) Introns can be spliced together in multiple ways making many more mRNAs. C) Alternative splicing randomly adds introns to exons making multiple mRNAs. D) Splicing either exons or introns creates variation in mRNA sequences.
FILL IN THE BLANK. Write the word or phrase that best completes each statement or answers the question. 50) Genes that have DNA sequences similar to functional genes but that do not appear to produce any functional product are ________________.
Version 1
14
Answer Key Test name: Chap 18_3e 1) [C, D] 2) [A, B, D]
Version 1
15
Clarify Question • What is the key concept addressed by the question? o This question addresses functional genomics. • What type of thinking is required? o This question is asking you to weigh and judge, or evaluate, evidence to choose the best of the possible answers. • What key words does the question contain and what do they mean? o Functional genomics focuses on understanding the connection between genes and phenotype. In other words, how exactly does a gene influence the workings of the cell? Gather Content • What do you already know about functional genomics? o We can think of functional genomics as the study of what genes do. o By focusing on the function of genes, we can understand how certain mutations or disruptions of gene expression can be connected to disease. Consider Possibilities • Consider the different answer options. Which can you rule out? o Would a lab to improve sequencing speed and cost fit in with the goals of a functional genomics center? Not really. While important, sequencing efficiency does not directly address functional genomics. o Would an ancient DNA laboratory fit in with the goals of a functional genomics center? Not really. Sometimes ancient DNA could provide some insight into gene changes and disease, but studying ancient DNA is not a direct approach to understanding gene function. It is more likely to provide insight into evolution. Choose Answer Version 1
16
• Given what you now know, what information and/or problem solving approach is most likely to produce the correct answer? o Would a DNA microarray facility fit in with the goals of a functional genomics center? Yes, that allows researchers to study how the genes are expressed in different cell types and conditions–an important aspect of gene function. o Would a proteomics lab fit in with the goals of a functional genomics center? Yes, this allows the researchers to study large numbers of proteins in different cell types and conditions. o Would a mutagenesis facility fit in with the goals of a functional genomics center? Yes, that would allow researchers to see directly what the phenotypic outcome is for loss-of-function of a particular gene. Reflect on Process • Did your problem-solving process lead you to the correct answer? If not, where did the process break down or lead you astray? How can you revise your approach to produce a more desirable result? o This question asked what should be included in a functional genomics center. o Did you recognize that functional genomics is about connecting genes to phenotype? 3) [A, C, D]
Version 1
17
Clarify Question • What is the key concept addressed by the question? o This question addresses gene duplication. • What key words does the question contain and what do they mean? o Duplication of a single gene or a segment of a chromosome means to create a second copy. Gather Content • What do you already know about gene duplication? o Duplication of a genomic region, either within a chromosome, or to another chromosome, allows genes with the same function to diverge because a backup pair of genes exist. o A duplicated gene can lose function; it is not selected against because there is another functioning gene, not just an allele. o The duplicated gene can also diverge and acquire new functions because the original persists. Consider Possibilities • Consider the different answer options. Which can you rule out? o Could a duplicated gene cause an organism to become another species? This seems unlikely as a number of genetic changes would have to occur to allow species divergence. o Could a duplicated gene become a centromere? No, because centromeres have special DNA sequences that allow them to be recognized as such. Choose Answer
Version 1
18
• Given what you now know, what information and/or problem solving approach is most likely to produce the correct answer? o Could a duplicated gene lose function due to mutation? Yes. This is possible because the other copy of the gene is still carrying out the original function. In fact it is the most common outcome. o Could a duplicated gene gain a new function due to mutation? Yes, this is also possible, again because the other copy of the gene is still carrying out the original function. o Could a duplicated gene become a pseudogene? Yes. It could acquire a mutation that makes it nonfunctional, but still linger in the genome. Reflect on Process • Did your problem-solving process lead you to the correct answer? If not, where did the process break down or lead you astray? How can you revise your approach to produce a more desirable result? o This question asked what can occur to a duplicated gene. o Did you recognize that a duplicated gene can more freely mutate, and either lose or gain function, because the other copy of the gene can carry out the original function. 4) [C, D]
Version 1
19
Clarify Question • What is the key concept addressed by the question? o This question addresses FOXP2. • What type of thinking is required? o This question is asking you to analyze the information given, using logic, to dissect the problem and determine the answer. • What key words does the question contain and what do they mean? o Neanderthals (Homo neanderthalensis) are an extinct close relative of humans. o H. sapiens is the abbreviation for Homo sapiens—the modern human species. o FOXP2 is a transcription factor gene that functions to develop centers for spoken language in the brain. Gather Content • What do you already know about FOXP2? o FOXP2 is a transcription factor that controls the expression of other genes. o It is expressed in the brain. o The gene is found in other primates and mammals, but humans have a special version of the sequence that is thought to be essential for spoken language. Consider Possibilities • Consider the different answer options. Which can you rule out? o Can you conclude that Neanderthals evolved from H. sapiens? No, sharing a gene does not indicate that one species evolved from another. o Can you conclude that H. sapiens evolved from Neanderthals? No, sharing a gene does not indicate that one species evolved from another. Version 1
20
o Can you conclude that verbal speech was impossible for Neanderthals? No, if anything the presence of the shared gene suggests the opposite. Choose Answer • Given what you now know, what information and/or problem solving approach is most likely to produce the correct answer? o Can you conclude that Neanderthals may have had the ability to use language? Yes, but the emphasis is on may. We know that FOXP2 seems to be essential for human spoken language, but we don’t know what other genes are necessary. o Can you conclude that these two amino acid changes arose prior to the common ancestor of Neanderthals and H. sapiens? Yes, that is the most parismonious explanation for the shared sequence. Reflect on Process • Did your problem-solving process lead you to the correct answer? If not, where did the process break down or lead you astray? How can you revise your approach to produce a more desirable result? o This question asked what you can conclude from the fact that Neanderthals had the same human version of the gene FOXP2. o Did you recognize that the shared FOXP2 suggests that it arose before the common ancestor of humans and Neanderthals? o Did you recognize that this gene suggests—but does not prove—that Neanderthals may have had language? 5) [A, B] 6) [B, C, D] 7) [A, B, C] 8) [A, B, C] 9) B Version 1
21
10) E
Version 1
22
Clarify Question • What is the key concept addressed by the question? o This question addresses the transcriptome. • What type of thinking is required? o This question is asking you to take what you already know and apply it to this unfamiliar situation. • What key words does the question contain and what do they mean? o Quantifying levels of cellular RNA means to measure the amount of RNAs such as mRNAs and rRNAs. Gather Content • What do you already know about the transcriptome? o The transcriptome is the set of transcripts, or RNAs, that are expressed in a cell. o It will differ between different cell types, and between different conditions. Consider Possibilities • Consider the different answer options. Which can you rule out? o Does studying RNAs involve studying the proteome? No, because the proteome refers to all the proteins of a cell. o Does studying RNAs involve studying the spliceosome? No, because the spliceosome is the name for the cellular machinery that splices mRNAs. o Does studying RNAs involve studying the nucleosome? No, because the nucleosome is the wound up module of chromatin in the chromosome. o Does studying RNAs involve studying different RNA motifs? No, because you were asked to measure levels of large numbers of RNAs, Version 1
23
not studying details of particular RNAs. Choose Answer • Given what you now know, what information and/or problem solving approach is most likely to produce the correct answer? o Does studying RNAs involve studying the transcriptome? Yes, because that is the term for all the RNAs of a cell. Reflect on Process • Did your problem-solving process lead you to the correct answer? If not, where did the process break down or lead you astray? How can you revise your approach to produce a more desirable result? o This question asked what you would study to measure RNAs. o Did you recognize that the RNAs of a cell represent the transcriptome? 11) E
Version 1
24
Clarify Question ● What is the key concept addressed by the question? question asks about inactive genes. ● What type of thinking is required? apply your knowledge of inactive genes.
● The
● You are being asked to
Gather Content ● What do you know about inactive genes? What other information is related to the question? ● Genes normally contain introns that donot encode proteins and exons that do encode proteins. Transposons and templates are not parts of genes. Pseudogenes are genes that once encoded proteins and have accumulated mutations that now make them inactive. Choose Answer ● Given what you now know, what information is most likely to produce the correct answer? ● Genes that have lost their function are an example of pseudogenes. Reflect on Process ● Did your problem-solving process lead you to the correct answer? If not, where did the process break down or lead you astray? How can you revise your approach to produce a more desirable result? ● This question asked you to apply your understanding of inactive genes. If you got the correct answer, great job! If you got an incorrect answer, where did the process break down? Did you recall that introns and exons are parts of a gene? Did you understand that a gene that becomes inactive with mutations is a pseudogene?
Version 1
25
12) D 13) A 14) B 15) B 16) A 17) D 18) A 19) D
Version 1
26
Clarify Question ● What is the key concept addressed by the question? ● The question asks about measuring genome-wide expression of genes. ● What type of thinking is required? ● You are being asked to apply your knowledge of measuring genome-wide expression of genes. Gather Content ● What do you know about measuring genome-wide expression of genes? What other information is related to the question? ● Functional genomics uses a range of high-throughput techniques to learn about the products produced from a genome, when those products are produced, and how they change during development or in response to the environment. DNA microarrays were created for the analysis of gene expression at the whole-genome level. The arrays allow questions to be asked about how gene expression patterns change during development, in response to environmental stimuli, and even during the development of disease. Choose Answer ● Given what you now know, what information is most likely to produce the correct answer? ● DNA microarrays can be used to measure all of the genes with increased or decreased expression after exposure to a toxin. Protein microarrays do exist but they are used to track the interactions and activities of proteins. Mass spectroscopy is used to identify proteins. Genome sequencing tells you the sequence of DNA molecules, but cannot be used to determine changes in gene expression. A DNA microarray is the best choice for this question. Reflect on Process ● Did your problem-solving process lead you to the correct answer? Version 1
27
If not, where did the process break down or lead you astray? How can you revise your approach to produce a more desirable result? ● This question asked you to apply your understanding of measuring genomewide expression of genes. If you got the correct answer, great job! If you got an incorrect answer, where did the process break down? Did you understand that DNA microarrays are used to assay gene expression? Did you understand the function of the other three assays?
20) B 21) A 22) D 23) D 24) A
Version 1
28
Clarify Question • What is the key concept addressed by the question? o This question addresses gene expression patterns. • What key words does the question contain and what do they mean? o Gene expression patterns are the patterns of where and when genes are expressed in the developing embryo or adult. o Primates are the group of mammals that includes humans, other apes, monkeys, and lemurs. Gather Content • What do you already know about gene expression patterns? o Gene expression refers to the amount of gene product produced in different times and places in the developing animal. o Gene expression is the key to understanding why different organisms end up looking different. o The best explanation for why a mouse develops into a mouse and not a human is that the same or similar genes are expressed at different times, in different tissues, and in different amounts and combinations. Consider Possibilities • Consider the different answer options. Which can you rule out? o Are the genes for the development of mammary glands, the lungs, or the heart likely to have different expression patterns in humans versus other primates? No, because these are not particularly different in humans than in other primates–certainly not as different as our brains are. Choose Answer • Given what you now know, what information and/or problem solving Version 1
29
approach is most likely to produce the correct answer? o Are the genes for the development of the brain likely to have different expression patterns in humans versus other primates? Yes! Human brains are much larger for our size than the brains of other primates, and have more complex capabilities. Reflect on Process • Did your problem-solving process lead you to the correct answer? If not, where did the process break down or lead you astray? How can you revise your approach to produce a more desirable result? o This question asked which type of genes were most likely to be expressed differently in humans compared to other primates. o Did you recognize that genes for brain developent were the most likely to have changed in humans? 25) C
Version 1
30
Clarify Question • What is the key concept addressed by the question? o This question addresses non-protein-coding DNA. • What key words does the question contain and what do they mean? o Pseudogenes are old genes that have become inactive, often due to a stop codon. o Non-protein-coding DNA is any DNA that does not lead to a protein product. Gather Content • What do you already know about non-protein-coding DNA? o Non-protein-coding DNA is responsible for much of the variation in genome size between different species. o Non-protein-coding DNA was once called junk DNA, but scientists have begun to appreciate ways it may be functionally important. Consider Possibilities • Consider the different answer options. Which can you rule out? o Do the RNAs from non-protein-coding DNA suggest that pseudogenes code for proteins? No, this does not necessarily mean they produce proteins. o Do the RNAs from non-protein-coding DNA suggest that nonprotein-coding DNA has no function? No. In fact this suggests that it may have a regulatory function. o Do the RNAs from non-protein-coding DNA suggest that pseudogenes have no function? No. Some pseudogenes may also have a previously-unrecognized regulatory function. Choose Answer Version 1
31
• Given what you now know, what information and/or problem solving approach is most likely to produce the correct answer? o Do the RNAs from non-protein-coding DNA suggest that nonprotein-coding DNA may regulate gene expression? Yes. If the RNAs regulate transcription of other genes, this may be an important regulatory function, maintaining the presence of the DNA. Reflect on Process • Did your problem-solving process lead you to the correct answer? If not, where did the process break down or lead you astray? How can you revise your approach to produce a more desirable result? o This question asked what is suggested by the fact that non-proteincoding DNA may produce RNAs that affect transcription. o Did you recognize that these regions may function to regulate the expression of other genes? 26) A
Version 1
32
Clarify Question ● What is the key concept addressed by the question? ● The question asks about the link between numbers of genes and numbers of mRNAs. ● What type of thinking is required? ● You are being asked to analyze the link between numbers of genes and numbers of mRNAs. Gather Content ● What do you know about the link between numbers of genes and numbers of mRNAs? What other information is related to the question? ● In RNA splicing, introns are removed and the exons are joined together. Due to alternative splicing of RNA, a single gene can code for multiple mRNAs and proteins. This happens when some exons are skipped leaving them out of the mRNA. The exons encode different parts of the protein, so leaving out an exon changes the final activity of the protein. Choose Answer ● Given what you now know, what information is most likely to produce the correct answer? ● Because of alternative splicing of exons, many different mRNAs can be produced by the same gene. This leads to multiple different proteins also being expressed. Reflect on Process ● Did your problem-solving process lead you to the correct answer? If not, where did the process break down or lead you astray? How can you revise your approach to produce a more desirable result? ● This question asked you to analyze the link between numbers of genes and numbers of mRNAs. If you got the correct answer, great job! If you got an incorrect answer, where did the process break down? Did you recall Version 1
33
that splicing joins exons and not introns? Did you understand that by leaving out some exons different mRNAs can be produced?
27) A
Version 1
34
Clarify Question ● What is the key concept addressed by the question? ● The question asks about similarities between human and chimpanzee genomes. ● What type of thinking is required? ● You are being asked to apply your knowledge of similarities between human and chimpanzee genomes. Gather Content ● What do you know about similarities between human and chimpanzee genomes? What other information is related to the question? ● When two species are more closely related, their genomic DNA sequences are more similar. As the time to a common ancestor is extended, more mutations can occur, leading to greater differences in the genomic sequence. Choose Answer ● Given what you now know, what information is most likely to produce the correct answer? ● The best answer is that over time mutations occur between species. The more closely related the species, the fewer differences. Because chimpanzees are the closest living relative to humans, we have more similar genomic DNA sequences than with a dog or baboon. Reflect on Process ● Did your problem-solving process lead you to the correct answer? If not, where did the process break down or lead you astray? How can you revise your approach to produce a more desirable result? ● This question asked you to apply your understanding of similarities between human and chimpanzee genomes. If you got the correct answer, great Version 1
35
job! If you got an incorrect answer, where did the process break down? Did you recall that humans and chimpanzees are closely related? Did you understand that more distantly related species like dogs and baboons would have more differences in their genomes compared to humans?
28) D
Version 1
36
Clarify Question • What is the key concept addressed by the question? o This question addresses genome size. • What type of thinking is required? o This question is asking you to analyze the information given, using logic, to dissect the problem and determine the answer. • What key words does the question contain and what do they mean? o A sequencing reaction is the reaction used to determine the sequence of a piece of DNA. o The complete human genome refers to one complete set of 23 chromosomes. Gather Content • What do you already know about genome size? o Genome size is highly variable, and not always correlated with organismal complexity. o The human genome size is about 3 billion base pairs. Consider Possibilities • Consider the different answer options. Which can you rule out? o Would 3.1 billion sequencing reactions cover the genome? Yes, but that is too many. Each reaction could be only 1 nucleotide long and that would be enough! o Would 6200 sequencing reactions cover the genome? No, that is not nearly enough. That would only cover 3.1 million base pairs. (6200 x 500 = 3,100,000) o Would 12.4 million sequencing reactions cover the genome? Yes, but that is too many. That would cover over 6 billion nucleotides. (12,400,000 x 500 = 6,200,000,000) Version 1
37
Choose Answer • Given what you now know, what information and/or problem solving approach is most likely to produce the correct answer? o The human genome size is 3.1 billion base pairs. o Would 6.2 million sequencing reactions cover the genome? Yes, that is the bare minimum number. That would cover 3 billion nucleotides. (6,200,000 x 500 = 3,100,000,000) Reflect on Process • Did your problem-solving process lead you to the correct answer? If not, where did the process break down or lead you astray? How can you revise your approach to produce a more desirable result? o This question asked you to determine the minimum number of sequencing reactions to sequence the entire human genome. o The question required you to analyze the information given, using logic, to dissect the problem and determine the answer. o Did you remember that the human genome is about 3 billion base pairs long? 29) C
Version 1
38
Clarify Question • What is the key concept addressed by the question? o This question addresses genome analysis. • What key words does the question contain and what do they mean? o Genomeannotation refers to making notes in the genome database about the locations and details of genes. Gather Content • What do you already know about genome analysis? o Sequencing a genome is not enough to be useful. o To make sense of the sequence of nucleotides, we have to figure out where the genes lie. Consider Possibilities • Consider the different answer options. Which can you rule out? o Would the next step be shotgun cloning? No, that would be part of the sequencing effort. o Would the next step be the alignment of contigs? No, that would be part of the sequencing effort. o Would the next step be the construction of a genetic map? No, that would not directly annotate the genome sequence unless the genes were aligned to molecular markers. Choose Answer • Given what you now know, what information and/or problem solving approach is most likely to produce the correct answer? o Would the next step be the identification of open reading frames? Yes, this would show where the likely genes are. This is an important Version 1
39
part of genomeannotation. Reflect on Process • Did your problem-solving process lead you to the correct answer? If not, where did the process break down or lead you astray? How can you revise your approach to produce a more desirable result? o This question asked you to describe what would occur as part of genomeannotation. o Did you recognize that finding open reading frames—i.e., the locations of genes—is an important part of sequence annotation? 30) A
Version 1
40
Clarify Question • What is the key concept addressed by the question? o This question addresses sequence analysis. • What type of thinking is required? o This question is asking you to analyze the information given, using logic, to dissect the problem and determine the answer. • What key words does the question contain and what do they mean? o Myogenic regulatory factors (MRFs) are a gene family in vertebrates that are important for muscle development. o Drosophila is the genus name for fruit flies, the workhorse of genetic research. Gather Content • What do you already know about sequence analysis? o Sequence analysis can use a DNA sequence to reveal information about a protein, by translating the DNA on a computer. o By looking at protein domains and motifs, we can learn possible roles for that protein. o Comparing two proteins can show us how similar they are. If the protein sequences are similar, that suggests they play similar roles. Consider Possibilities • Consider the different answer options. Which can you rule out? o Is it quick and easy to generate four transgenic Drosophila strains that express the MRFs in a mutant nautilus background? No, this is not quick and easy–although it would be interesting. o Is it quick and easy to use a DNA microarray to compare gene activation patterns between nautilus and the MRFs? Not really. You would need to establish transgenic lines so that the MRFs were in Version 1
41
Drosophila, to compare gene activation to nautilus (or vice versa). o Is it quick and easy to compare the proteome of a cell expressing nautilus to the those of cells expressing each of the MRFs? No, because again you would need to establish the transgenic lines to get the MRFs in Drosophila. Choose Answer • Given what you now know, what information and/or problem solving approach is most likely to produce the correct answer? o Is it quick and easy to compare the DNA and protein sequences of all the genes to determine if nautilus is more similar to one MRF than another? Yes! This could be done in an afternoon, thanks to the genome data available. This would give you some idea of what to expect, based on how similar the protein sequences are. Reflect on Process • Did your problem-solving process lead you to the correct answer? If not, where did the process break down or lead you astray? How can you revise your approach to produce a more desirable result? o This question asked what would be the quick and easy way to see which vertebrate MRF gene was most similar to the nautilus gene. o The question required you to analyze the information given, using logic, to dissect the problem and determine the answer. o Did you recognize that sequence analysis is a quick and easy way to learn about the likely functions of a gene? 31) A
Version 1
42
Clarify Question • What is the key concept addressed by the question? o This question addresses proteomics. • What type of thinking is required? o This question is asking you to analyze the information given, using logic, to dissect the problem and determine the answer. • What key words does the question contain and what do they mean? o The proteome is the complete set of proteins in a cell. Gather Content • What do you already know about proteomics? o Proteomics techniques involve ways of analyzing hundreds or thousands of proteins at a time. Consider Possibilities • Consider the different answer options. Which can you rule out? o Would a DNA microarray help to survey the proteome for proteins that bind a new drug? No, because that only reveals the presence of DNA or RNA in a sample. o Would ENCODE help to survey the proteome for proteins that bind a new drug? No, because ENCODE is the name of a project that identifies elements in the genome. o Would using a monoclonal antibody help to survey the proteome for proteins that bind a new drug ? No, because each monoclonal antibody can only bind one protein—you would need thousands of antibodies to survey the proteome. Choose Answer • Given what you now know, what information and/or problem solving Version 1
43
approach is most likely to produce the correct answer? o Would a protein microarray help to survey the proteome for proteins that bind a new drug? Yes. That is a technique that allows you to recognize the presence of up to thousands of proteins at once. o You could treat cells with the drug, use a column to capture the drug and bound proteins, then apply those bound proteins to a protein microarray to identify them. Reflect on Process • Did your problem-solving process lead you to the correct answer? If not, where did the process break down or lead you astray? How can you revise your approach to produce a more desirable result? o This question asked which technique could be used to survey the proteome for drug interactions. o Did you recognize that protein microarrays can be used to recognize the presence of multiple proteins in a sample? 32) A 33) A 34) B 35) C 36) D 37) A 38) E 39) A 40) D 41) E 42) E 43) D 44) A 45) A Version 1
44
46) C 47) A 48) A 49) A
Version 1
45
Clarify Question ● What is the key concept addressed by the question? question asks about alternative splicing.
● The
● What type of thinking is required? ● You are being asked to apply your knowledge of alternative splicing. Gather Content ● What do you know about alternative splicing? What other information is related to the question? ● In alternative splicing, in a mRNA some exons are included and some are left out. When the mRNA is then translated unique proteins are made with different exons encoding different protein domains. Choose Answer ● Given what you now know, what information is most likely to produce the correct answer? ● Introns are spliced out of mRNA, only exons are included. So exons are alternatively spliced to create more mRNAs than genes. Reflect on Process ● Did your problem-solving process lead you to the correct answer? If not, where did the process break down or lead you astray? How can you revise your approach to produce a more desirable result? ● This question asked you to apply your understanding of alternative splicing. If you got the correct answer, great job! If you got an incorrect answer, where did the process break down? Did you recall that exons are spliced together to create a mRNA? Did you understand that some exons can be skipped, creating proteins with different combinations of domains?
Version 1
46
50) pseudogenes
Version 1
47
CHAPTER 19 CHECK ALL THE APPLY. Choose all options that best completes the statement or answers the question. 1) In a population, allele S is strongly favored. However, over time, the frequency of the s allele does not significantly decrease, but rather is maintained. What evolutionary process(es) could be responsible for countering the effect of selection for the S allele? (Select all that apply.) A) Mutation B) Genetic drift C) Gene flow D) Assortative mating
2) A selection experiment you are doing requires certain conditions in order for evolutionary change to occur. What conditions would cause this change? (Check all that apply.) A) Variation between individuals in a population B) Genetic variations being heritable C) No variation existing between individuals in the next generation D) Survival rate of offspring varying due to parental genetic differences
3) In both field and lab studies of guppies, you observe a rapid evolutionary change in the physiological traits that help the animals avoid predation. This observation illustrates which main points? (Check all that apply.) A) Allele frequency is maintained in all conditions. B) Natural selection is an ongoing process. C) Dominant genes will always increase survival rates. D) Evolutionary change is facilitated by natural selection.
4) Hardy-Weinberg states out that original genotype proportions within a population remain constant across generations if certain assumptions were met. These assumptions include __________. (Check all that apply.) Version 1
1
A) the occurance of random mating B) the lack of selection C) the absence of polymorphic loci in the population D) a large population size E) an absence of gene flow
5) In a populationof purebred cats, over 75% of the alleles present can be dated back to just 16 individuals belonging to a cat breeder from the late eighteenth century. Based on this, what statement(s) about the cat population are accurate? A) Further evolutionary change in the purebred cats is unlikely. B) Rapid evolutionary change will be seen in the purebred cats over time. C) Very little genetic variation exists in the purebred cats. D) Inbreeding has led to high levels of genetic variation in the purebred cats. E) Mutations rates will be higher in the purebred cat population than in most domestic cat populations.
6) What are likely outcomes fora female who is not choosy enough and has too broad of an acceptance of reproductive signals? (Check all that apply.) A) Her sons may havelow success attracting mates. B) Her fitness willbe improved because she never misses a chance to mate. C) She may mate withmales who are sick with parasites or disease. D) She may mate withmales with less adaptivegenes. E) Her fitness goesup due to increased efficiency by not wasting time in lengthy evaluations. F) She might inadvertentlymate with a different species.
7) Swordtail fish are known for the long extensions on the males' tails. What types of experiments might help you determine whether the females are using visual input about tail length to choose between males? (Check the best three answers.)
Version 1
2
A) Add artificial tails of different lengths to females and determine their response to normal males. B) Add artificial tails of different lengths to male platys (a similar fish without tail extensions) and measure female response. C) Create artificial fish models with different size tails and measure female response. D) Measure hormone levels in males who have different size tails. E) Measure sperm count in males who have different size tails. F) Trim the tails of male fish to different lengths and measure female response.
MULTIPLE CHOICE - Choose the one alternative that best completes the statement or answers the question. 8) Many male songbirds are brightly colored. However, the color of the birds is usually determined by a balance of __________. A) natural selection against bright colors due to predation and sexual selection in favor of bright colors B) mutations that reducebright colors and gene flow in favor of bright colors C) natural selection against bright colors due to predation and mutations that introducebright colors D) mutations that reduce bright colors and sexual selection in favor of bright colors E) sexual selection that reduces bright colors and natural selection in favor of bright colors
9) Female cardinals select male mates in part based on their bright red color. What effect would this have on a cardinal population that was inHardy-Weinberg equilibrium? A) The frequency ofred alleles would be greater than those predicted by HardyWeinbergequilibrium. B) The frequency ofred alleles would be less than those predicted by HardyWeinbergequilibrium. C) The red allelefrequencies would remain unchanged because of equilibrium. D) The frequency of red alleles would be equal to those predicted byHardy-Weinberg equilibrium. E) The red allelefrequencies would decrease because of equilibrium.
Version 1
3
10) Two parents who do not have sickle cell anemia have a child that has the disease.The parents are both __________. A) homozygous for the sickle cell allele B) heterozygous for the sickle cell allele C) homozygous for the normal allele D) epistatic for the sickle cell allele E) pleiotropic for the sickle cell allele
11) An island is on the migration route of sea birds. This island also has abundant treenesting birds that live on the island permanently and are not found on any other island. The two bird species do not interbreed. The tree-nesting birds are more likely than the sea birds to be in Hardy-Weinberg equilibrium because __________. A) of high immigration and emigration in the sea bird population B) the sea bird population is larger C) there are fewer mutations in the sea birds D) mating is random in the tree-nesting birds E) natural selection is stronger in the tree-nesting birds
12) Coloration in the peppered moth (Biston betularia) is determined by a single gene with two alleles showing complete dominance. Dark moths are homozygous dominant or heterozygous for the gene, light moths are homozygous recessive. In a sample of 100 moths, you determine that 64 of the moths are dark. According to the Hardy-Weinberg rule, the expected frequency of the dominant allele is __________. A) 0.4 B) 0.36 C) 0.6 D) 0.64 E) It cannot be determined.
13) In three-spined sticklebacks males rear the young. Byselectively choosing a mate a female stickleback is most likely evaluating traits that would improve her __________. Version 1
4
A) fitness B) longevity C) allele frequency D) gene flow
14)
Darwin proposed that natural selection occurs in an environment by __________.
A) favoring heritable features that make the organism better suited to survive and reproduce B) producing a constant number of offspring while in that environment C) surviving for a fixed amount of time D) resisting the environment and keeping the environment from changing E) favoring those individuals with the most favorable acquired characteristics
15)
Natural selection for a certain phenotype will affect the __________. A) allele frequency of a population B) allele frequency of an individual C) genotype of an individual D) genotype of a population
16) When bird populations migrate and mix with populations that do not migrate, _____________ will occur. A) gene blending B) gene flow C) mutation D) genetic drift
17) Genotypes are said to be in ________ equilibrium if there is random mating and no other forces tend to alter the proportions of alleles from one generation to the next. Version 1
5
A) steady-state B) homeostatic C) Hardy-Weinberg D) Mendelian
18) The key point in Darwin's proposal is that the ________ imposes the conditions that determine the direction of selection. A) parent B) gene C) individual D) environment
19) Natural selection, as a mechanism of evolution that acts on variants within populations and ultimately leads to the evolution of different species, was proposed by __________. A) Mendel B) Lyell C) Malthus D) Darwin E) Founder
20)
In the Hardy-Weinberg equation, p and q are __________. A) allele frequencies B) genotypes C) phenotypes D) measures of fitness E) mutation rates
Version 1
6
21) A restriction in genetic variability caused by a drastic reduction in population size is called a __________. A) founder effect B) Hardy-Weinberg effect C) bottleneck effect D) polymorphic effect E) adaptive effect
22)
Reproductive success of an individual is known as __________. A) variation B) microevolution C) macroevolution D) fitness E) adaptive makeup
23) In the Hardy-Weinberg equations, the frequencies of two alleles in a population (where there are only two alleles to consider) can be designated as __________. A) (p + q) 2 B) p and q C) p 2 and q 2 D) 2pq E) 1 - p and 1 - q
24)
The inheritance of acquired characteristics proposal was put forward by __________.
Version 1
7
A) Darwin B) Lamarck C) Wallace D) Founder E) Hardy-Weinberg
25) The frequency of a particular allele within a population can be changed over time by __________. A) genetic outflow B) a large population size C) a lack of selection D) the inheritance of acquired characteristics E) random mating
26) The genetic preservation of the features that increase the likelihood of survival and reproduction of some individuals within a population is called the process of __________. A) natural selection B) creation of new species C) genetic drift D) inheritance of acquired characteristics E) increasing evolutionary resistance
27) There are more than 30 blood group genes in humans, in addition to the ABO locus. This increases ____ in human populations. A) geneticvariability B) homozygosity C) selectivemating D) gene flow E) foundereffects
Version 1
8
28) For a gene with two alternative alleles, A (with a frequency of p) and B (with a frequency of q), the term in the algebraic form of the Hardy-Weinberg equilibrium that represents the heterozygote genotype frequency is __________. A) p 2 B) q 2 C) 2pq D) (p+q) 2 E) 2Aa
29) The observation that most purebloodNative Americans have type O blood is best explained by __________. A) a founder effect B) gene flow C) genetic drift D) assortative mating E) frequent mutations
30) You spray your kitchen with an insecticide to kill the cockroaches. A few survive and reproduce, producing a large healthy population in a few generations that all have similar genetic backgrounds. This is an example of __________. A) mutation B) migration C) genetic drift D) assortative mating E) bottleneck effect
31) Gene flow, defined as the movement of genes from one population to another, can take place by migration as well as __________.
Version 1
9
A) mating with certain trait-containing individuals B) mating with dominant phenotypes C) mating between individuals of adjacent populations D) removing the barriers between the populations E) physical movement of genes within an individual by transposons
32) Some flowering plants cannot self-pollinate. This increases their tendency to mate with phenotypically different mates, a process called disassortative mating. What effect would this have on a population in Hardy-Weinberg equilibrium? A) An increase inheterozygotes would be observed. B) A decrease inheterozygotes would be observed. C) An increase inhomozygotes would be observed. D) No changehomozygotes or heterozygotes would be observed. E) The populationwould remain in Hardy-Weinberg equilibrium.
33) Being born with extra fingers on a hand is called polydactyly. This is more common in some populations in North America than others due to __________. A) a founder effect because their ancestors from Europe carried the alleles B) an increase in mutation rate in these populations C) random mating with other populations in North America D) natural selection in favor of extra fingers in these populations E) migration of people with multiple fingers out of the population
34) In a forest, trees that grow taller receive more sunlight and end up producing more viable seeds. Shorter trees of the same species receive less sunlight and produce few viable seeds. Over time, what type of selection will take place?
Version 1
10
A) Sexual selection B) Stabilizing selection C) Disruptive selection D) Directional selection E) Artificial selection
35) Certain small towns in the western United States have remained isolated since their settlement many years ago. Some alleles are more common in these communities as compared to the rest of the US population. This difference is a result of __________. A) artificial selection B) directional selection C) disrupting selection D) the Hardy-Weinberg principle E) the founder effect
36) Cheetahs that have experienced a genetic bottleneck would be characterized by evidence showing that __________. A) rates of genetic mutation are low in this species B) the body is long, thin, and graceful C) the population has low levels of genetic variability D) these cats are members of an endangered species E) they originally came from small areas of Africa
37) Animals that select mates that are phenotypically similar will have ________ when compared with Hardy-Weinberg predictions.
Version 1
11
A) fewerhomozygotes B) less naturalselection C) more heterozygotes D) more homozygotes. E) more mutations
38) If a population was inHardy-Weinberg equilibrium then ________ would occur in that population. A) directionalselection B) assortivemating C) naturalselection D) anincreaseof recessive alleles E) no evolutionarychanges
39) Large ears is a rare trait in a mainland population of mice. A female mouse homozygous for very large ears sneaks onto a boat and arrives at an island that already contains mice. Assuming she mates with the island mice, a higher incidence of big ears in the island population than in the mainland population can be explained by __________. A) a bottleneck effect B) gene flow C) increased mutation D) genetic drift E) natural selection
40) In some instances, environmental change causes a situation where one phenotype is favored for a period of time and then a different phenotype is favored for a period of time. This oscillating selection is most likely to lead to __________.
Version 1
12
A) the maintenance of genetic variation in the population B) the elimination of rarer genotypes because of directional selection C) an increase in point mutations D) an increasing population to maintain genetic variation E) a population with decreasing levels of genetic variation
41) People homozygous for the sickle-cell anemia allele develop a life-threatening disease, while those homozygous for the normal allele are at the highest risk of dying frommalaria. Carriers have some resistance to malaria but do not develop sickle cell anemia.This is an example of __________. A) founder effect B) genetic bottleneck C) point mutation D) heterozygote advantage E) heterozygosity
42) During a drought on the Galapagos islands, finches with larger beaks were able to crack the large tough seeds produced by plants that survived the dry conditions. These finches were able to survive and reproduce at a greater rate than were finches with smaller beaks. This is an example of __________. A) directional selection B) stabilizing selection C) disruptive selection D) genetic drift E) a founder effect
43) __________ would produce the smallest evolutionary change in a given period of time in a population of birds.
Version 1
13
A) Mutation B) Naturalselection C) Migration D) Assortivemating E) Gene flow
44) The disease sickle-cell anemia is common in malaria-infested areas because individuals that are heterozygous for the gene ( AS) have enhanced resistance to malaria over normal individuals ( AA). Individuals with severe sickle-cell anemia ( SS) usually die before reproduction. If malaria were to be eradicated, what would happen to the frequency of the A allele over time? A) It would go up because there is no longer a heterozygote advantage. B) It would go up because there will no longer be cases of sickle cell anemia. C) It would go down because there is no longer a herterozygote advantage. D) It would go down because there will no longer be cases of sickle cell anemia.
45) The California populations of the Northern elephant seal are descendants from a very small population of seals that was overhunted in the 1890s. Heterozygosity in this population would be expected to be ________ due to ________. A) slight; a bottleneck effect B) slight; the founder effect C) great; disruptive selection D) great; a bottleneck effect E) great; assortive mating
46) The several hundred species of picture-winged fruit flies of the Hawaiian Islands are genetically very similar, yet they all differ markedly from their ancestral population in Asia. This is probably accounted for by __________.
Version 1
14
A) sexual selection B) directional selection C) disruptive selection D) the founder effect E) gene flow
47) An insect population that becomes resistant to a commonly used insecticide is an example of __________. A) natural selection B) genetic drift C) disruptive selection D) a founder effect E) inheritance of acquired characteristics
48)
The organism with the highest fitness is an animal __________. A) that lives 10 years and produces 10 offspring per year B) that lives 100 years and produces 10 offspring total C) that lives 2 years and produces 10 offspring per year D) that lives 10 years and produces 2 offspring per year E) that lives one year and produces 10 offspring total
49) A population of lizards lives in a rocky area next to a desert. Some lizards are light colored and blend into the sand. Others are dark and blend into the rocks. What is likely to happen to this population of lizards over time? A) They may evolveinto two separate species as a result of disruptive selection. B) They may evolveinto two separate species as a result of directional selection. C) They may remainone species as a result of disruptive selection. D) They may remainone species as a result of directional selection. E) They may remainone species as a result of stabilizing selection.
Version 1
15
50)
Both extremes of an array of phenotypes are favored in __________. A) directional selection B) stabilizing selection C) disruptive selection D) gene flow E) genetic drift
51) Farmerschoosing to consistently breed only corn with plumper kernels is an example of __________. A) directional selection B) stabilizing selection C) disruptive selection D) sexualselection E) genetic drift
52)
The midrange of an array of phenotypes is favored in __________. A) directional selection B) stabilizing selection C) disruptive selection D) gene flow E) genetic drift
53) In a small population of cockroaches living in your kitchen, only a few roaches mate in one year. This can lead to random changes in allele frequency in the population through __________.
Version 1
16
A) genetic drift B) mutation C) selection D) migration
54) In the lab, you show that guppies under different predatory conditions change in rapid evolution. In the field, you wish to design an experiment that would generate data to support your observation. What experiment would best help you achieve your goal? A) Transfer wild guppies from a predator-free pond into one with predators but no guppies; over time observe the guppies for physiological changes. B) Use the guppies from the lab and introduce them into a predator-rich pond free of wild populations of guppies; observe trait changes. C) Place predators from the lab into a pond free of other predators but with wild populations of guppies and monitor changes in physical traits. D) Create a mixed population of wild guppies and place them into a controlled environment with predators to record the genetic changes.
55)
Mutations affect Hardy-Weinberg equilibrium by __________. A) altering frequencies of particular alleles within a population B) keeping allele frequencies unchanged between generations C) decreasing the genetic variation that affects natural selection D) allowing gene flow to increase
Version 1
17
56) The percentage of different colored water boatmen (an aquatic insect) eaten by fish were graphed relative tothe frequency of that color in different populations. Based on the graph, what conclusion can you draw? A) Rare phenotypes will survive better due to negative-frequency selection. B) Common phenotypes will survive better due to negative-frequency selection. C) Rare phenotypes will survive better due to positive-frequency selection. D) Common phenotypes will survive better due to positive-frequency selection.
57)
Assortative and disassortative mating are similar in that both __________.
A) change only the expected Hardy-Weinberg allele frequencies in a population B) change only the expected Hardy-Weinberg genotype frequencies in a population C) change both the expected Hardy-Weinberg allele and genotype frequencies in a population D) increase the proportion of homozygous individuals in a population E) are examples of random mating
Version 1
18
58) In the experiment above, guppy color patterns (spots) were measured in populations exposed to increasing amounts of predation. From this you could conclude that __________. A) predators aremore likely to catch and eatbrightly colored guppies B) predators areless likely to catch and eatbrightly colored guppies C) brightly coloredguppies are more likely to reproduce in the presence of predators D) predators do notaffect the color patterns of guppies E) evolutionarychanges take millions of years to appear
59)
Evolution acts at the level of the __________. A) ecosystem B) population C) species D) community
60) The degree of parental investment required often results in the nonrandom mating behavior called __________.
Version 1
19
A) allopatry B) fidelity C) mate choice D) preferability
61) By evaluating and selecting mates with superior qualities, an animal can increase its __________. A) reproductive success B) learning C) competitive strategies D) foraging efficiency E) length of life
62) Pea fowl (peacocks and peahens) show sexual dimorphism—males have long blue-green tail feathers with eyespots, while females have short brown tailfeathers.Which statement best describes the graphed data about peacocks?
Version 1
20
A) There are no peacocks with less than 140 eyespots. B) The fewer eyespots that a peacock has in his tail, the more mates he attracts. C) Actually eyespots have very little to do with mate-attracting activities. D) The more eyespots that a peacock has in his tail, the more mates he attracts. E) There are no peacocks with more than 165 eyespots.
63) Male Mormon crickets choose larger females as their mates. Which of the following statements best interprets the graph?
A) Larger females live longer and thus produce more eggs. B) Larger females are capable of storing sperm. C) Larger females reproduce earlier than smaller females. D) Larger females lay more eggs. E) Larger females defend themselves better.
64) Male hummingbirds flash a patch ofiridescent feathers on their neck called the gorget.This behavior makes them very visible. Why advertise their presenceto a predator?
Version 1
21
A) Hummingbirds are so fast that predation is not a large factorforthem. B) Predators cannotsee colors so are not alerted. C) The benefits ofdefending a territory exceed the costs of predation. D) This is an exampleof a maladaptive behavior that decreases overall fitness.
65) In the phalarope, or wadepiper bird, the male is unusual in playing the larger parenting role—he performs all the egg incubation and chick care. What unusual behavior might you predict for the female of the species? A) The female providessperm to fertilize the male's eggs. B) The female is partof a large harem of females, under the domain of a single male. C) The females competeto mate with the males, who choose among them. D) The females havedull, brown coloring that keeps them well-camouflaged.
66) A study finds a positive correlation between the number of spots on the tail of male peacocks and their mating success. What would be the best followup experiment? A) Count the number ofspots on female tails and measure mating success. B) Remove spots or addartificial spots to male tailsand determine female responses. C) Measure the spermcount of males and compare to spot count. D) Inject testosteroneand observe changes in spot count.
67) Platys and swordtails are related tropical freshwater fish. In studies, researchers have shown that female platys prefer males with swordtails, even though males of their own species do not have them. This was discovered by attaching artificial swordtails to platys males.What does this suggest about the origin of the swordtail feature? A) Female preferencefor swords may have predated the origin of the feature itself. B) Evolution of swords later led to evolution of female preference forswords. C) The same gene thatcauses sword development also causes development of neural circuits for femalepreference. D) Adjacent genes onthe same chromosome cause sword development and development of neural circuitsfor female preference.
Version 1
22
Version 1
23
Answer Key Test name: Chap 19_3e 1) [A, B, C] In theory, mutation and genetic drift can counter the effects of selection. However, mutation rates are rarely high enough to counter the effects of natural selection andnatural selection is expected to overwhelm drift except when populations are very small.Gene flow can serve to counter or enhance natural selection. 2) [A, B, D] 3) [B, D] 4) [A, B, D, E] 5) [A] Because the purebred cat population contains the genetic information of so few individuals, the genetic variation available for evolutionary forces to act upon is very low. Further evolutionary change in the purebred cats is unlikely. 6) [A, C, D, F]
Version 1
24
Clarify Question • What is the key concept addressed by the question? o The results when a female isn't maximizing fitness with mate choice. • What type of thinking is required? o This is an analyze question, you need to break down all of the given outcomes and pick the best ones. You need to select which of these outcomes would result from a female not being careful when picking a mate. The goal of mate choice is to pick the highest quality mate, thus maximizing fitness. Gather Content • What do you already know about mate choice and fitness? What other information is related to the question? o This question comes from the section of the textbook that discusses the role of courtship. Picking the best male gives the female the best chances of producing high-quality offspring with higher fitness than other potential offspring. Choose Answer • Given what you now know, what information is most likely to produce the correct answer? o Anything that reduces her fitness or the fitness of her offspring is a potential downfall of not being picky when it comes to mate choice. Reflect on Process • Did your problem-solving process lead you to the correct answer? If not, where did the process break down or lead you astray? How can you revise your approach to produce a more desirable result? o The key to this analyze question is to understand the link between mate choice and fitness. Her lack of male selection may effect her Version 1
25
reproductive fitness. 7) [B, C, F]
Version 1
26
Clarify Question • What is the key concept addressed by the question? o How to study sexual selection behaviors in swordtail fish. • What type of thinking is required? o This is an analyze question; you need to break down each answer into its components and decide which are the best choices. You are being asking to choose between a series of experiments concerning mate choice is swordtails. There are three correct answers in the set. Gather Content • What do you already know about sexual selection behaviors? What other information is related to the question? o This question comes from the section of the textbook that discusses the nature of signals and mate choice. All of the choices would provide some data, but only three of the choices deal with the length of the swordtail. The length of a male swordtail is the main visual cue that females use when selecting mates. Choose Answer • Given what you now know, what information is most likely to produce the correct answer? o Lengthening tails, shortening tails, or using models with varying tail lengths would all be experiments that would provide information about the visual cue that female use to pick mates. Reflect on Process • Did your problem-solving process lead you to the correct answer? If not, where did the process break down or lead you astray? How can you revise your approach to produce a more desirable result? Version 1
27
o Were you able to select the three correct choices? The key to this question is to understand that female swordtails use the visual cue of tail length to pick mates. 8) A Selection favors phenotypes with the greatest fitness, but fitness is a combination of survival, mating success, and number of offspring per mating. Atrait favored for one component of fitness may be at a disadvantage for others. In this case, bright coloration is favored by females (sexual selection) and increases mating success. However, bright coloration also puts birds at a higher risk of predation, decreasing survival. The balance of the two forms of selection determines male bird coloration in the population. 9) A
Version 1
28
Clarify Question • What is the key concept addressed by the question? o This question is asking you to predict what will happen to allele frequencies if female cardinals select male mates based upon their red color. • What type of thinking is required? o You are being asked to take what you already know and use, or apply, it to the concept of sexual selection to the Hardy-Weinberg principle. • What key words does the question contain and what do they mean? o Hardy-Weinberg equilibrium, which refers to populations whose genotype frequencies do not change from generation to generation. o Frequency, which refers to the number of times a particular event occurs in a sample. o Allele, which refers to a version of a gene. Gather Content • What do you already know about the Hardy-Weinberg principle? How does it relate to the question? o The Hardy-Weinberg principle states that the original proportions of the genotypes in a population will remain constant from generation to generation, as long as the following assumptions are met: 1. No mutation takes place. 2. No genes are transferred to or from other sources. 3. Mating is random. 4. The population size is very large. 5. No selection occurs. o The question stem violates one of the five assumptions. Which assumption is violated, and how will this violation influence changes in the allele frequency? Version 1
29
Consider Possibilities • What other information is related to the question? Which information is most useful? o The question states that female cardinals will select male mates based upon a characteristic they possess. If a male does not possess that characteristic, then they will not be able to find a female to mate with and thus their fitness will decrease. What would happen to the frequency of alleles that do not code for bright red color? Choose Answer • Given what you now know, what information and/or problem solving approach is most likely to produce the correct answer? o To answer this question, you may choose to identify the HardyWeinberg condition that is not being met and then identify how that condition may influence allele frequency in a population. In this specific example, female cardinals prefer to mate with bright red males. How will this influence the frequency of the allele that codes for the bright red color? Reflect on Process • Did your problem-solving process lead you to the correct answer? If not, where did the process break down or lead you astray? How can you revise your approach to produce a more desirable result? o This question asked you to determine the effect selection would have on Hardy-Weinberg equilibrium. o Answering this question correctly depended on your ability to use the Hardy-Weinberg principle in a new situation. o If you got the correct answer, excellent! If you got an incorrect Version 1
30
answer, where did the process break down? Did you remember the five assumptions for Hardy-Weinberg equilibrium, or that selection for a certain phenotype can influence an allele’s frequency? Did you have trouble extending how a phenotype is tied to a set of alleles? 10) B
Version 1
31
Clarify Question • What is the key concept addressed by the question? o The question stem is asking you to determine the genotype of parents when you know the parent’s and offspring’s phenotypes. • What type of thinking is required? o This question is asking you to take the concept of allele dominance and use, or apply, it to identify genotypes of individuals. • What key words does the question contain and what do they mean? o Sickle cell anemia, which refers to a hereditary disease affecting hemoglobin in humans. o Homozygous, which means that both alleles for a gene are the same. o Heterozygous, which means that the alleles for a gene are different. o Epistatic, which is the phenomenon in which an allele for one gene may have different effects, depending on alleles present at other genes. o Pleiotropic, which means alleles affect multiple aspects of a phenotype. Gather Content • What do you already know about sickle cell anemia? How does it relate to the question? o The sickle cell allele is recessive to the normal allele. This means that only individuals that are homozygous recessive will express sickle cell anemia. Consider Possibilities • What other information is related to the question? Which information is most useful? o Epistasis is the phenomenon in which an allele for one gene may have different effects, depending on alleles present at other genes. In addition, Version 1
32
pleiotropic alleles often affect multiple aspects of a phenotype. When would a gene be either epistatic or pleiotropic? Choose Answer • Given what you now know, what information and/or problem solving approach is most likely to produce the correct answer? o Recall that Punnett squares may be a useful tool for determining the genotype of parents and offspring. Perhaps the creation of Punnett squares will be a useful tool for determining the genotype of the parents based upon their phenotype. Reflect on Process • Did your problem-solving process lead you to the correct answer? If not, where did the process break down or lead you astray? How can you revise your approach to produce a more desirable result? o This question asked you to determine the genotype of the parents. o Answering this question correctly depended on your ability to use Mendelian genetics in a new situation. o If you got the right answer correct, great job! If you got an incorrect answer, where did the process break down? Did you remember how dominant and recessive alleles are expressed as phenotypes? Do you understand the difference between an organism’s genotype and an organism’s phenotype? Did you have trouble extending the application of Punnett squares to answer this question? 11) A
Version 1
33
Clarify Question • What is the key concept addressed by the question? o This question is asking you to analyze how two different populations of birds will evolve based on where the populations live throughout the year. • What type of thinking is required? o This is an analyze question because you have to determine the effect of migratory birds on Hardy-Weinberg equilibrium. • What key words does the question contain and what do they mean? o Hardy-Weinberg equilibrium, which refers to populations whose genotype frequencies do not change from generation to generation. o Migration, which means that individuals move from one location to another o Population, which refers to a group of individuals living in the same area that can interbreed and produce fertile offspring. Gather Content • What do you already know about the Hardy-Weinberg principle? How does it relate to the question? o The Hardy-Weinberg principle states that the original proportions of the genotypes in a population will remain constant from generation to generation, as long as the following assumptions are met: 1. No mutation takes place. 2. No genes are transferred to or from other sources. 3. Mating is random. 4. The population size is very large. 5. No selection occurs. Consider Possibilities Version 1
34
• What other information is related to the question? Which information is most useful? o The question states that an island populated by tree-nesting birds is on the migration route of sea birds. In regards to the assumptions for HardyWeinberg equilibrium, these two populations are different in one key assumption. One of the populations in the question stem violates one of the five assumptions. Which population violates which assumption? Choose Answer • Given what you now know, what information and/or problem solving approach is most likely to produce the correct answer? o To answer this question, you may choose to identify the HardyWeinberg condition that is not being met and then identify which population is not meeting that condition. In this specific example, an island populated by tree-nesting birds is on the migration route of sea birds. Information in the question stem provides clues to which HardyWeinberg condition is not being met. Reflect on Process • Did your problem-solving process lead you to the correct answer? If not, where did the process break down or lead you astray? How can you revise your approach to produce a more desirable result? o This question asked you to determine the impact of migratory birds versus nonmigratory birds on Hardy-Weinberg equilibrium in both populations. o Answering this question correctly depended not only on distinguishing between migratory sea birds and resident tree-nesting birds, but on your ability to break down, or analyze, how the HardyWeinberg principle is applied to this particular situation. Version 1
35
o If you got the correct answer, excellent! If you got an incorrect answer, where did the process break down? Did you remember the five assumptions for Hardy-Weinberg equilibrium, or that statements within the question stem could provide clues for identifying the conditions for Hardy-Weinberg equilibrium? Did you have trouble breaking down how the Hardy-Weinberg principle could be applied to this situation to determine the correct answer? 12) A
Version 1
36
Clarify Question • What is the key concept addressed by the question? o This question is asking you to calculate various frequencies for a population in Hardy-Weinberg equilibrium. What are the formulas involved in calculating the frequencies for a population in HardyWeinberg equilibrium? • What type of thinking is required? o This is an analyze question because you have to calculate various frequencies for a population in Hardy-Weinberg equilibrium. • What key words does the question contain and what do they mean? o Dominant phenotype, which refers to the physical trait of a version of a gene inherited from a single parent. o Recessive allele, which refers to the physical trait of a version of a gene inherited from both parents. o Hardy-Weinberg equilibrium, which refers to populations whose genotype frequencies do not change from generation to generation. o Allele, which refers to a version of a gene. o Homozygous, which means that both alleles for a gene are the same. o Heterozygous, which means that the alleles for a gene are different. Gather Content • What do you already know about the Hardy-Weinberg principle? How does it relate to the question? o To solve this problem you will need to apply the Hardy-Weinberg equations. Recall that there are two equations involved in the HardyWeinberg principle. One equation is p + q = 1; where p is the frequency of the dominant allele, q is the frequency of the recessive allele, and 1 is 100% of the alleles. The other equation is p2 + 2pq + q2 = 1; where p2 is the frequency of the homozygous dominant genotype, 2pq is the Version 1
37
frequency of the heterozygous genotype, q2 is the frequency of the homozygous recessive genotype, and 1 is 100% of the individuals. How are these two equations related? How can we go from a phenotype frequency to an allele frequency? Which phenotype frequency is the same as one of the genotype frequencies? Consider Possibilities • What other information is related to the question? Which information is most useful? o The question provides you with the frequency of the dominant phenotype. The recessive phenotype is equivalent to the homozygous recessive genotype. If you can calculate the frequency of the homozygous recessive genotype, then which allele frequency can you determine? Choose Answer • Given what you now know, what information and/or problem solving approach is most likely to produce the correct answer? o To answer this question you must determine which phenotype frequency matches a genotype frequency. Keep in mind that you may have a dominant phenotype with either a homozygous dominant or heterozygous genotype. What genotype do you have if you express the recessive phenotype? o How do you get from a value that is squared, like p2, to a value that is not squared but has the same variable, like p? What is the mathematical function involved in this process? Reflect on Process • Did your problem-solving process lead you to the correct answer? If Version 1
38
not, where did the process break down or lead you astray? How can you revise your approach to produce a more desirable result? o This question asked you to determine the frequency of the dominant allele in peppered moths. o Answering this question correctly depended not only on distinguishing between genotypic and phenotypic frequencies, but on your ability to break down, or analyze, various frequencies. o If you got the correct answer, excellent! If you got an incorrect answer, where did the process break down? Did you remember how to calculate the square root of a decimal? Do you understand that the recessive phenotype must have a homozygous recessive genotype? Therefore, the frequency of the recessive phenotype must equal the frequency of which genotype? Did you have trouble breaking down the Hardy-Weinberg equations to determine the correct answer? 13) A 14) A 15) A 16) B 17) C 18) D 19) D 20) A 21) C 22) D 23) B 24) B 25) A 26) A 27) A 28) C Version 1
39
29) A Isolated human populations that were started by relatively few individuals often have a different allelic composition than the populations from which they originated. They tend to bedominated by thegenetic features characteristic of their founders. 30) E Spraying the insecticide drastically reduced the cockroach population in size. The few surviving cockroaches represent a random genetic sample of the original population. The loss of genetic variability in the new population is called thebottleneck effect. 31) C 32) A 33) A 34) D In this situation, selection is favoring taller tree phenotypes over shorter tree phenotypes. Eventually, selection may eliminate the extremely short phenotypes. This form of selection is calleddirectional selection. 35) E Isolated human populations that were started by relatively few individuals often have a different allelic composition than the populations from which they originated. They tend to bedominated by thegenetic features characteristic of their founders. <!--Markup Copied from Habitat--> 36) C
Version 1
40
When a population is drastically reduced in size, it often experiences a genetic bottleneck, whereby the few surviving individuals make up a random genetic sample of the original population. This leads to a low level of genetic variability as compared to the original population. <!-Markup Copied from Habitat--> 37) D 38) E 39) B 40) A In a changing environment, selection may favor one phenotype at one time and another phenotype at another time, a phenomenon called oscillating selection. If selection repeatedly oscillates in this fashion, the effect is to maintain genetic variation in the population. <!--Markup Copied from Habitat--> 41) D 42) A 43) A 44) A In an environment withoutmalaria, there is no longer an advantage to being an AS hetereozygote.Homozygous SS individualswould continue to die without reproducing, leading to a decrease (and eventual elimination) of the S allele over time. Homozygous AA individualswould be selected for, leading to an increase in the frequency of A over time. 45) A 46) D 47) A 48) A 49) A Version 1
41
50) C 51) A 52) B 53) A 54) A 55) A Mutation is the ultimate source of genetic variation and thus makes evolution possible.Mutation from one allele to another can change the allele frequencies within a population. However, mutation rates are so low that they usually have only a very small effect on the HardyWeinberg proportions of common alleles; other evolutionary processes are usually more important in determining how allele frequencies change. 56) A The graph shows that each of the color types is preyed upon disproportionately when it is the most common in the population. Rare color types survive better as a result of fish forming a search image for common color types, a phenomenon known as negative frequency-dependent selection.
57) B Assortative mating and disassortative mating are types of nonrandom mating. Neither process changes thefrequency of the individual alleles but, rather, both causethe frequencies of particular genotypes to differ greatly from those predicted by the Hardy-Weinberg principle. Assortative mating increases theproportion of homozygous individuals, whiledisassortative matingproduces an excess of heterozygotes. 58) A 59) B 60) C 61) A 62) D Version 1
42
Clarify Question • What is the key concept addressed by the question? o It comes from the section of the textbook that discusses sexual selection. Does the huge ornate tail of the peacock provide some kind of advantage? The data on this graph addresses that question. Once you’ve understood the graph you need to pick between the choices • What type of thinking is required? o This is an analyze question. You need to use the data in the graph to pick the best choice. Gather Content • What do you already know about sexual selection? What other information is related to the question? o There is a positive relationship, more spots = more mates. When you look at the choices, the graph is best described by only one of the choices. There is no information about peacocks with fewer than 140 or more than 165 spots. They may or may not exist, but the graph has nothing to say about that. Eye spots clearly have a relationship to mating, and it is a strong positive relationship. Choose Answer • Given what you now know, what information is most likely to produce the correct answer? o The answer is “The more eyespots that a peacock has in his tail, the more mates he attracts.” o That is the only choice that is supported by the data presented. Reflect on Process • Did your problem-solving process lead you to the correct answer? If not, where did the process break down or lead you astray? How can you revise your approach to produce a more desirable result? o This was an analyze question. You needed to extract the information from the graph and pick the correct choice. The information presented was about sexual selection. Knowing about sexual selection makes the graph more understandable, but the key to answering this question lies in your ability to extract the information from the graph.
63) D
Version 1
43
Clarify Question • What is the key concept addressed by the question? o The question is about mate choice in Mormon crickets, but to answer the question, you need to interpret the data in a graph. • What type of thinking is required? o This is an analyze question. You need to use the data in the graph to pick the best choice. Gather Content • What do you already know about mate choice? What other information is related to the question? o This question is from the section of the textbook that discusses mate choices and parental investment in offspring. Some animals provide lots of care, to a small number of offspring; some animals provide no parental care. If an animal does not provide parental care then the most common strategy is to produce as many offspring as possible. Mormon crickets do not care for their young. Is there a relationship between female body weight and the number of eggs produced? Is the relationship positive or negative? Is an increase in body mass increase in the number of eggs? Choose Answer • Given what you now know, what information is most likely to produce the correct answer? o There are five choices presented, but the graph really only provides evidence for one of them. The only information that the graph provides is to show that larger females have more eggs. The other choices may or may not be true, but the graph does not say anything about those choices. Reflect on Process • Did your problem-solving process lead you to the correct answer? If not, where did the process break down or lead you astray? How can you revise your approach to produce a more desirable result? o Did you get the question right? You needed to extract the information from the graph and pick the correct choice. The information presented was about the relationship between mass and number of eggs, but that wasn’t going to immediately point you to the correct answer. The key to answering this question lies in your ability to extract the information from the graph.
64) C
Version 1
44
Clarify Question • What is the key concept addressed by the question? o Determining the advantages of a male hummingbird's highly visible plumage. • What type of thinking is required? o This is an analyze question; all of the choices are potential explanations for the presence of a bright iridescent path on the hummingbird’s plumage. Only one of the choices is correct. Gather Content • What do you already know about the tradeoff of being visible to potential mates and predators? What other information is related to the question? o The main goal of any organism is to survive and pass on its genes. Something that may seem maladaptive (having bright patches of feather) may have an advantage that outweighs the costs. The key to this question is to recognize the concept of an evolutionary trade-off. Only one of the choices explains the presence of a gorget in the context of an evolutionary trade-off. Behaviors that decrease overall fitness are very rapidly selected out of a population. Hummingbirds certainly face predation and many predators use vision to detect prey. Choose Answer • Given what you now know, what information is most likely to produce the correct answer? o The gorget helps the hummingbird maintain a specific territory. By monopolizing a territory, the male hummingbird can have greater access to females, increasing his chance of passing on his genes. Reflect on Process Version 1
45
• Did your problem-solving process lead you to the correct answer? If not, where did the process break down or lead you astray? How can you revise your approach to produce a more desirable result? o Were you able to get the correct answer? Recognizing and understand the concept of an evolutionary trade-off is the key to this question. 65) C
Version 1
46
Clarify Question • What is the key concept addressed by the question? o Mate selection and parental investment involved in this species where the male and female roles are reversed. • What type of thinking is required? o This is an analyze question; you will need to break down the choices into their components to find the correct answer. You are given choices for female breeding behaviors and are being asked to pick the best one. Gather Content • What do you already know about traditional roles of mate selection and parental investments? What other information is related to the question? o The key is to recognize the role of parental investment in raising offspring. Normally male animals provide less care; female choose the best quality male they can find and then do most of the parental care (if any parental care is being provided). The situation with these birds is reversed. Females still produce eggs and males still produce sperm. The behaviors are different but the sexes are the same. A single male could not possible maintain a large number of females if she is providing all of the parental care. If females are going to compete for males, then the expectation is that the females will be the brightly colored sex (that is the case with these birds). Choose Answer • Given what you now know, what information is most likely to produce the correct answer? o The correct choice is that females compete and males choose the females. Version 1
47
Reflect on Process • Did your problem-solving process lead you to the correct answer? If not, where did the process break down or lead you astray? How can you revise your approach to produce a more desirable result? o Did you select the correct answer? Recognizing that the roles are reversed in this species is the key of finding the answer to this question. 66) B
Version 1
48
Clarify Question • What is the key concept addressed by the question? o Planning an experiment to determine if the spots are the only thing affecting sexual selection. • What type of thinking is required? o This is an analyze question; the choices you are being given are follow-up experiments to see if the correlation between spots and mating hold true. Gather Content • What do you already know about sexual selection and planning an experiment? What other information is related to the question? o This question concerns sexual selection. In peafowl, more spots equals more mating, but there could be a third factor involved. The goal of the next experiment is to test only the variable of spots and see if just a variation in spots alone produces a change in mating success. Is the number of spots the main factor when a peahen picks a peacock? In order to determine if the link were real, a researcher would need several peacocks that were similar in every way (same size, same age, same level of fitness) but they differed in spot number. An experiment like that would be able to provide evidence to show that the link between spots and mating success is real. Choose Answer • Given what you now know, what information is most likely to produce the correct answer? o The experimental design that involves adding and removing spots is the best choice. Reflect on Process Version 1
49
• Did your problem-solving process lead you to the correct answer? If not, where did the process break down or lead you astray? How can you revise your approach to produce a more desirable result? o Did you get the correct answer? The key to this question is to understand the difference between correlation and causation. 67) A
Version 1
50
Clarify Question • What is the key concept addressed by the question? o What is the origin of a feature in swordtail fish that is involved in sexual selection in a related species. • What type of thinking is required? o This is an analyze question. You are given four different explanations for the observation of female preference for long tails in species of fish where the males do not normally have long tail fins. Gather Content • What do you already know about the evolutionary origins of a characteristic involved in sexual selection? What other information is related to the question? o Male swordtails have long tail fins that they use to attract females. Closely related fish species (platys) also preferentially mate with males with long tail fins, but the long tail fins aren’t found in the platys males. This was discovered by attaching artificial swordtails to platys males. The question is asking you to pick be best explanation for the observation in female platys. If the genes for the production of swordtails and the neural mechanism for preference were the same genes, then in all species where females show a preference for swordtails, the male would have swordtails. Choose Answer • Given what you now know, what information is most likely to produce the correct answer? o The genes for preference may have evolved first in an ancestral species, followed by the development of swordtails in only one of the species that evolved. Version 1
51
Reflect on Process • Did your problem-solving process lead you to the correct answer? If not, where did the process break down or lead you astray? How can you revise your approach to produce a more desirable result? o Did you choose the correct answer? The key here is to recognize that the preference for a particular male trait precedes the development of that trait.
Version 1
52
CHAPTER 20 CHECK ALL THE APPLY. Choose all options that best completes the statement or answers the question. 1) Which observations illustrate shared ancestry by anatomical comparison studies? (Check all that apply.) A) The same structure has evolved to carry out different functions. B) Different structures evolve in similar ways when subjected to the same selective pressures. C) A structure becomes vestigial. D) Features are similar during embryonic development. <!--Markup Copied from Habitat-->
2) Select the pairs of organism characteristics that are homologous structures. (Check all that apply.) A) Cactus spines; tree leaves B) Human hand bones; whale fin bones C) Bat wing; butterfly wing D) Octopus eye; mouse eye
3)
Indicate which factors support the industrial melanism hypothesis. (Check all that apply.) A) Pollution inhibits the growth of dark colored lichen. B) Dark moths have greater camouflage on trees in polluted woodlands. C) Pollutants darken bark surfaces. D) Light moths have greater camouflage on trees in highly polluted woodlands.
4) Choose the true statement(s) about the evolutionary divergence of horses in North America. (Check all that apply.)
Version 1
1
A) Most changes are seen as adaptations for life in grassland habitats. B) The general trends during horse evolution are increase in body size, lengthening of limbs, reduction of digits on both sets of limbs, and tooth development for a grazing life style. C) Horse evolution is unusual in that rates of change were uniform and constant throughout most lineages. D) Living lineages of horses show much less diversity than existed in the past.
5) Which of the following features of ancient horses are adaptations for living on open grasslands? (Check all that apply.) A) Multiple toes that increased flexibility B) Complex patterns of ridges on molar and premolars that increased chewing ability C) Lengthening of limbs that increased running speed over long distances D) Decrease in body size that increased agility in confined spaces
6) Choose the true statements about species that show convergent evolution. (Check all that apply.) A) They do not share a recent common ancestor. B) They must be found in the same biogeographical region. C) They are subject to similar selection pressures. D) They have similar phenotypes.
7)
Choose the scientific evidence that supports evolutionary theory. (Check all that apply.) A) The fossil record B) Homologous structures C) The molecular record D) Intelligent design E) Vestigial structures
Version 1
2
8) Select all of the following that are valid counterarguments to the idea that evolution is just a theory. (Check all that apply.) A) Scientists use the word theory to describe unifying ideas. B) Evolution is just a collection of facts without a central theme and this is not a theory. C) Scientific theories are supported by many types of evidence and no empirical data refutes them. D) Evolution occurred in the past, but it is not occurring in the present, so it cannot be a theory.
9) Select the statements that provide direct biogeographical evidence for evolutionary theory. (Check all that apply.) A) Unrelated organisms from similar habitats exhibit similar characters. B) Terrestrial mammals are not found naturally on any island greater than 300 miles from a mainland. C) Organisms on islands are most similar to organisms on the nearby mainland, even when habitats on the mainland and island are quite different. D) Invasive rats have colonized more than 80% of the world's islands.
10) Which aspect(s) of natural selection is(are) illustrated by the figure below? (Check all that apply.)
Version 1
3
A) Variation must exist in the population. B) Variation among individuals must be genetically transmissible to the next generation. C) Variation among individuals leads to differences in lifetime reproductive success. D) More individuals are produced each generation than the environment can support, resulting in competition for survival.
11) A new winged species is discovered in the Amazon rainforest.Select the evidence that supports that this species’ wings are homologous to vertebrate animals and analogous to insect wings. (Check all that apply.) A) The wings have an underlying bone structure. B) The wings are poisonous. C) Blood vessels are networked through the wings. D) The wings are covered in chitinous scales.
MULTIPLE CHOICE - Choose the one alternative that best completes the statement or answers the question. 12) A cluster of very closely related flower species is found on an island chain in the South Pacific Ocean. It is likely that each species evolved to fit a different _____________ through the process of ________________. A) island; convergent evolution B) island; adaptive radiation C) niche; adaptive radiation D) niche; convergent evolution
13) A finch species living in the Galapagos Islands feeds on small insects found in flower buds. Its bill is most likely __________.
Version 1
4
A) small and narrow B) broad and powerful C) broad and sharp D) mid-sized and very long
14)
The most likely explanation for why toothed whales have a blowhole is __________. A) they evolved from an animal with gills B) they evolved from an animal with nostrils C) blowholes are more efficient for large animals D) blowholes are more efficient for breathing underwater than gills are E) blowholes are vestigial structures passed down from the first marine vertebrates
15) Flamingo beaks are upside down compared to other birds, forcing them to feed by putting their heads upside down in the water. The beak of a flamingo provides an example of a/an __________. A) homologous structure B) vestigial structure C) structure that follows a common embryonicdevelopmental pattern D) convergent structure E) imperfectly designed structure
16) In 2006, scientists discovered a fossil that had scales and gills, a flat head with eyes on top like a frog, and fin and neck bones that would allow it to prop itself out of the water.The best explanation for this fossil is that it is __________. A) a transitional fossil between fish and amphibians B) a transitional fossil between amphibians and reptiles C) a ray-finned fish D) a modern amphibian
Version 1
5
17) During development in the mother's uterus, human embryos have pharyngeal pouches (sometimes called gill slits) that later develop into various glands. Fish also have pharyngeal pouches as embryos, but these develop into gills. The best explanation for why human and fish embryos develop pharyngeal pouches is __________. A) humans and fish share a common ancestor that had gills B) human embryos need gill slits to breathe in the uterus C) humans and fish both develop pharyngeal pouches by random chance D) fish evolved from humans
18)
The pair of traits that best illustrates convergent evolution is __________. A) an elephant's tusks and a beaver's teeth B) a lizard's arm and a bird's wing C) a dragonfly's wing and a butterfly's wing D) a cartilage skeleton in a shark and a bone skeleton in a dolphin
19) The extinctmarsupial wolfin Australia closely resembled canines inthe rest of the world. This is an example of __________. A) the fossil record B) homology C) convergent evolution D) biogeography E) development
20) The streamlined body shape seen in sharks, tuna, and dolphins is most likely due to __________.
Version 1
6
A) a recent shared common ancestor B) different selection pressures C) the physical properties of water D) the need to escape fast-moving predators E) imperfect design
21) Plants in the desert-adapted families Cactacea (cactus) and Euphorbia (euphorbs) share many of the same physical characteristics, such as succulent stems that store water and CAM photosynthesis, but they do not share a recent common ancestor. This evidence suggests that these families are an example of __________. A) convergent evolution B) homologous structures C) vestigial structures D) divergent evolution
Version 1
7
22) The figure below shows the relationship between beak depth in the medium ground finch on Daphne Major and relative fitness.Assuming that beak depth is heritable, what do you predict about beak depth in the next generation?
A) Mean beak depth will decrease. B) Mean beak depth will increase. C) Mean beak depth will stay about the same. D) Mean beak depth in the next generation cannot be predicted using this data.
23) A scientist studying evolution of peppered moths ( Biston betularia) in Michigan observed that the percentage of melanic moths increased by around 2% from 1960 to 1961. Has evolution occurred in this population?
Version 1
8
A) No, because melanism wasn't adaptive at that time. B) No, because the frequency of the melanic moths did not change enough to be considered evolution. C) Yes, because the frequency of the melanic moths in the population changed over time. D) Yes, because the frequency of melanic moths decreased.
24) Comparing the evolution of domesticated dogs to the evolution of wolves over the same time period illustrates what relationship between artificial and natural selection? A) Artificial selection is slower than natural selection. B) Artificial selection is reversible; natural selection is not. C) Artificial selection is more likely to produce maladaptive structures than natural selection. D) Artificial selection cannot produce changes as large as changes produced by natural selection. E) In artificial selection, the magnitude of the selection pressure can be varied; in natural selection, it cannot.
25) Considerably more phenotypic variation exists in domesticated varieties of species like dogs and cabbages than exists in nondomesticated species like lions and maple trees. Which one of the following statements best explains why this is true? A) There is no selection and mating is random under domestication. B) During domestication, very high rates of mutation are induced. C) Genetic drift is important because domestication involves small populations. D) Many domesticated varieties would not survive in the natural world. E) Domesticated species exhibit hybrid vigor (heterosis).
26)
Which statement applies toartificial selection, but not natural selection?
Version 1
9
A) Organisms are capable of producing more offspring than typically will survive. B) The phenotypic variation of a species can be increased very rapidly. C) Phenotypic variation exists within populations. D) Phenotypic variation can influence reproductive success. E) Phenotypic variation within a species is due in part to inherited characteristics.
27) In studying the medium ground finch on Daphne Major, the Grants noted that each generation of finches had beaks __________. A) best suited for their parents' environment B) best suited for their current environment C) smaller than those of the previous generation D) larger than those of the previous generation
28)
Industrial melanism is a term __________.
A) describing the color change induced by living in industrialized areas B) explaining that the darker moths have higher mutation rates because of industrialization C) describing the evolutionary process in which initially light-colored organisms become dark as a result of natural selection D) explaining that the darker moths having higher reproductive success because of their pigmentation E) describing the widespread implementation of pollution controls
29) The evidence for industrial melanism being caused by an increase in the dark allele was provided from field tests carried out by __________.
Version 1
10
A) Kettlewell B) Lamarck C) Darwin D) Tutt E) Wallace
30) The shape of the beaks of Darwin's finches and industrial melanism are often cited as examples of the process of _______________ leading to evolutionary change. A) artificial selection B) coevolution C) intelligent design D) founder effect E) natural selection
31) Natural selection has favored the dark form of the peppered moth in areas subject to severe air pollution, perhaps because on darkened trees, moth-eating birds see them less easily. As pollution abated, the light forms increased in the population because __________. A) light moths had lower fitness than dark moths B) light moths were able to produce more offspring than dark moths C) light moths were more genetically variable than dark moths D) birds ate more light moths
32) Industrial melanism involving the peppered moths ( Biston sp.) is cited as an example of natural selection that has been observed in the last 100 years. Recall that the peppered moth exhibits two distinct morphological types with dark and light colored wings. Which of the following statements about changes in these two types of moths as a result of industrial melanism is true?
Version 1
11
A) The dark forms are selected against in nonpolluted forests. B) The dark forms are distasteful to birds and are thus safe in polluted forests. C) The light forms are selected against in nonpolluted forests. D) Birds prey more on the dark forms in polluted forests E) Dark forms emigrate from nonpolluted forests to polluted forests.
33)
Domestication of dogs has led to __________. A) a variety of vestigial structures B) a variety of phenotypes C) convergent evolution D) reproductively isolated breeds E) different species
34) Some of the most dramatic evidence for evolution has come from human agriculture. One of the most highly artificially selected crop plants is __________. A) peas B) tomatoes C) potatoes D) corn E) asparagus
35) A technique used in dating a rock can be used to accurately predict the age of the fossils occurring in the rocks. This technique involves __________. A) fossil dating B) successive rock layering C) radioactive isotope decay D) structural geology E) developmental geology
Version 1
12
36)
Today, the fossil record __________.
A) can be dated only with one layer in relation to older layers below and younger layers above, no actual dates can be estimated B) can be dated reasonably well only with uranium-238 C) can be dated reasonably well using a variety of radioactive isotopes with known halflives D) can be dated precisely, to within a single year, using radioactive isotopes E) can be dated precisely, to within a single year, using DNA hybridization
37)
Progressive changes in the fossil record are evidence for evolution because __________. A) they show descent with modification B) they show inheritance of acquired characteristics C) they show evolutionary change at the level of the individual D) they show gaps in the fossil record
38) Radioactive isotopes are used in dating materials from the distant past. Which of the following statements about radioactive dating techniques is true? A) It gives the general ages of rock strata. (±1 million years). B) It gives exact ages of rock strata. (± 1 year). C) It uses a technique in which the degree of radioactive decay is measured; the younger the rock the more radioactive decay. D) It uses a technique in which the degree of radioactive decay is measured; the older the rock the more radioactive decay. E) It does not work well with fossil remains that have not absorbed radiation.
39) Which one of the following features of Archaeopteryx clearly demonstrates that it was on the evolutionary line leading from dinosaurs to birds?
Version 1
13
A) Egg-laying ability B) Teeth C) Feathers D) Bony tail E) Claws on forelimbs
40) The most precise method of obtaining estimates of the absolute ages of geological deposits is to __________. A) measure rates of sedimentary rock formation B) measure the ratios of various radioactive isotopes in the deposits C) apply the principle of superposition (younger deposits above older deposits) D) make salinity measurements in ocean sediment deposits E) study the sequence of fossil types in the deposits
41)
The size and shape of a finch's beak correlates with its __________. A) choice ofmates B) choice of food C) habitat D) size
42) Structures that have no apparent function but had a function in ancestral species are __________. A) the fossil record B) the molecular record C) homology D) development E) vestigial structures
43)
Homologous structures are __________.
Version 1
14
A) structures of animals that appear to have evolved from different parts of their bodies B) structures of animals that have different appearances and functions but seem to have evolved from the same body part in a common ancestor C) structures of animals that have the same appearances and functions but obviously no common ancestor D) structures of animals that have different appearances and functions but different ancestors
44) The side toes of a horse, the pelvis of the whale, and the human appendix are all examples of structures that resemble structures of presumed ancestors, which are known as __________. A) developmental structures B) homologous structures C) vestigial structures D) homeotic mutations E) acquired structures
45) You would predict to see the fewest amino acid sequence substitutions in a protein between a human and a __________. A) fish B) bacterium C) snake D) orangutan E) bird
46) The evolution of similar forms in different lineages when exposed to the same selective pressures is __________.
Version 1
15
A) called divergence B) called convergence C) referred to as successive homologies D) referred to as descent E) also known as natural selection
47) The observation that different geographical areas sometimes exhibit plant and animal communities of similar appearance, even though the individual plants and animals are not closely related, is called __________. A) the fossil record B) homology C) convergent evolution D) divergent evolution E) adaptation
48)
Convergent evolution occurs when two species living in __________.
A) the same area become reproductively isolated B) different areas become reestablished and are able to reproduce C) the same area are competing for the same resource thus causing one to evolve away from the other D) different areas evolve similarities through natural selection acting on those characteristics
49) Inhabitants of oceanic islands resemble forms of the nearest mainland but show some differences. This is an example of __________.
Version 1
16
A) the fossil record B) homology C) convergent evolution D) biogeography E) development
50)
Evolution does not violate the second law of thermodynamics because __________. A) evolution is a random process; it does not require an input of energy B) Earth is not a closed system; it constantly receives an input of energy from the sun C) living organisms are not subject to the second law D) the disorder generated by extinction balances the order created by evolution
51) Why shouldn'tthe origin of species according to a religious text be included in the curriculum of biology courses? A) Religious teachings are not scientifically testable theories. B) Most religious texts have beenproven to contain false information. C) The origin of species from an intelligent designerhas been superseded by more modern theories. D) It is contrary to the beliefs of most scientists.
52) A new plant species is introduced to Daphne Major and produces small, thin seeds.This plant species is highly adapted to drought and after 5 years has replaced over 80% of the native plants that produce large seeds.How will this change affect the evolution of beak size in the medium ground finch population? A) Small beaks will be favored under all rainfall conditions. B) Small beaks will be favored in wet years and large beaks will be favored in dry years. C) Large beaks will be favored in wet years and small beaks will be favored in dry years. D) Large beaks will be favored under all rainfall conditions.
Version 1
17
53) In a hypothetical study, equal numbers of marked dark and light moths are released in polluted and unpolluted woodlands and recaptured 10 days later. According to the industrial melanism hypothesis, which of the following results is unexpected? A) More marked dark moths should be recaptured in polluted woodlands than in unpolluted woodlands. B) Fewer marked light moths should be recaptured in polluted woodlands than in unpolluted woodlands. C) Fewer marked dark moths should be recaptured in unpolluted woodlands than in polluted woodlands. D) More marked light moths should be recaptured in unpolluted woodlands than in polluted woodlands. E) Fewer unmarked dark moths should be captured in polluted woodlands than unmarked light moths.
54)
Which alternate hypothesis might also explain industrial melanism?
A) Dark moths are more resistant to the toxic effects of pollution than light moths. B) Dark moths emigrate out of polluted areas to escape the pollution. C) Pollution kills important tree species that peppered moths depend on for egg laying. D) Because dark moths absorb more heat, they are more active and better able to avoid bird predation. E) Birds used the marks that Kettlewell placed on the moths when he released them to help them find and capture the moths.
Version 1
18
55) The figure below shows results of bristle number in Drosophila flies after 35 generations of artificial selection. This figure suggests that __________.
A) bristle number has evolved beyond the original range of phenotypic variation for this trait B) after 35 generations of selection, populations no longer exhibit variation in bristle number C) natural selection cannot lead to large phenotypic changes D) at the end of the experiment, "high population" flies were unable to interbreed with "low population" flies
56) A rock contains 18 mg of the radioactive isotope carbon-14. The half-life of carbon-14 is 5,600 years. How many half-lives and years will it take before the carbon-14 decays to less than 4 mg? A) 1 half-lives; 5,600 years B) 2 half-lives; 11,200 years C) 3 half-lives; 16,800 years D) 4 half-lives; 22,400 years E) 9 half-lives; 50,400 years
57) The half-life of carbon-14 is 5,600 years. A fossil that has one-eighth the normal proportion of carbon-14 to carbon-12 is probably __________.
Version 1
19
A) 2,800 years old B) 5,600 years old C) 11,200 years old D) 16,800 years old E) 22,400 years old
SECTION BREAK. Answer all the part questions.
58)
58.1) The pattern of wet years and dry years shown will probably move the population toward __________. A) all having larger beaks B) all having smaller beaks C) having intermediate size beaks D) having two, distinct populations E) maintaining abundant variation in beak depth
58.2) Which statement best describes the correlation between beak depths of Geospiza fortis on the Galápagos island Daphne major and the dryness of the environment?
Version 1
20
A) Birds with large beaks are favored during wet years. B) Birds with small beaks are favored in dry years. C) Birds with large beaks are favored during dry years. D) Birds with either size beak (large or small) are favored equally.
59) In the laboratory, fruit flies (Drosophila) were artificially selected for the number of bristles on their bodies. One population (R) was selected for low numbers of bristles, a second population (S) for high numbers of bristles.
59.1) In order for the scientists to artificially select Drosophila for their number of bristles, __________. A) they had to induce mutations into their populations of Drosophila B) they had to study the population for millions of years C) genetic variation had to be present in the population D) dramatic mutations had to be produced by the selection E) the number of bristles in each Drosophila had to be random
59.2) After 35 generations (about one year's time), the two populations R and S were studied. Researchers found __________.
Version 1
21
A) no change was seen; it's much too soon B) some slight increase of bristle numbers was already evident C) a vast, overlapping range of bristle numbers was seen D) a slight increase in bristles in S, a slight decrease in R, with a little overlap in the middle ranges E) a huge increase in S, a huge decrease in R, with no overlap whatsoever
60) Marsupials are a very ancestral form of mammal, evolving before placental mammals over 70 million years ago. Today Australian marsupials can be compared to placental mammals on other continents.
60.1) When you compare Australian marsupials to placental mammals today __________. A) the marsupials are all very primitive, having changed very little during the last 70 million years B) the marsupials are very similar to placental mammals in the ways they have adapted to similar ecological niches C) the marsupials have evolved much more rapidly than placental mammals and are more highly adapted D) the marsupial fossils look remarkably similar to the placental mammals of today E) living marsupials are little changed from the earliest marsupial fossil forms
60.2) Australian marsupial kangaroosand placental deer graze on grass with similar teeth. These species have __________. A) undergone divergent evolution B) undergone selective advantage C) undergone heterozygote advantage D) undergone convergent evolution E) developed homologous structures
Version 1
22
Version 1
23
Answer Key Test name: Chap 20_3e 1) [A, C, D] 2) [A, B] 3) [B, C] 4) [A, B, D] 5) [B, C] 6) [A, C, D] 7) [A, B, C, E] 8) [A, C] 9) [B, C] 10) [A, B] 11) [A, C]
Version 1
24
Clarify Question What is the key concept addressed by the question? • The question stem is asking you to select evidence supporting that the species’ wings are homologous to vertebrate wings and analogous to insect wings. What type of thinking is required? • You are being asked to take what you already know and use, or apply, it to comparing homologous structures and analogous structures. Gather Content What do you already know about homologous structures? What other information is related to the question? • Homologous structures may have different appearances and functions even though they are derived from the same common ancestral body part. Analogous structures may have different appearances, but similar functions even though they are not derived from the same common ancestral body part. Choose Answer Given what you now know, what information is most likely to produce the correct answer? • Of the options available, which ones describe structures you would find within vertebrate limbs, such as wings? Bones and blood vessels are found in vertebrate animals. Reflect on Process Did your problem-solving process lead you to the correct answer? If not, where did the process break down or lead you astray? How can Version 1
25
you revise your approach to produce a more desirable result? • Answering this question correctly depended on your ability to use homologous structures in a new situation. If you got an incorrect answer, did you remember that homologous structures are derived from a common ancestor? Did you have trouble extending this evidence for evolution to determine the correct answer? 12) C When a group of closely related species evolve to occupy a variety of different habitats, or niches, within a region, it is called adaptive radiation. <!--Markup Copied from Habitat--> 13) A The finch species of the Galapagos Islands show differences in their bills which correspond to differences in their feeding habits. Species with small and narrow bills, such as the warbler finch, eat insects. <!-Markup Copied from Habitat--> 14) B
Version 1
26
Clarify Question What is the key concept addressed by the question? ● The question is asking you to determine the most likely evolutionary reason that toothed whales have blowholes. What type of thinking is required? ● You are being asked to evaluate, or weigh, various explanations against each other to determine the most likely one. Gather Content What do you already know about the evolution of toothed whales? What other information is related to the question? ● Fossils have revealed the evolutionary path of whales. These intermediate forms show gradual changes to the skull, limbs, and swimming motion. ● Anatomical structures can be convergent, homologous, or vestigial. Make sure that you understand the difference between these terms. Choose Answer Given what you now know, what information is most likely to produce the correct answer? ● Fossils show that modern whales evolved from hoofed mammal ancestors. The earliest of these intermediate forms lived on land, and therefore would have used nostrils to breathe. The nostrils of whale ancestors and the blowholes of modern whales are homologous structures. Reflect on Process Did your problem-solving process lead you to the correct answer? If not, where did the process break down or lead you astray? How can you revise your approach to produce a more desirable result? ● Answering this question correctly depended on your ability to weight and judge, or evaluate, the various explanations for the blowholes of toothed whales. If you got an incorrect answer, did you Version 1
27
understand the difference between convergent, homologous, and vestigial structures? Did you review the fossil evidence for intermediate forms of aquatic mammals? 15) E Natural selection can only act upon variation that is present in a population—it cannot purposefully create perfect adaptations.The argument can be made that a flamingo's beak is a workable, but imperfect structure. 16) A 17) A Many developmental features reflect the common ancestry of vertebrates. Many vertebrate embryos are nearly indistinguishable in the early stages to all but developmental biologists; the different species begin to differentiate as development progresses. 18) D
Version 1
28
Clarify Question What is the key concept addressed by the question? • The question stem is asking you to select which pair of traits is an example of convergent evolution. What type of thinking is required? • You are being asked to take what you already know and use, or apply, it to convergent evolution. Gather Content What do you already know about convergent evolution? What other information is related to the question? • Convergent evolution explains how natural selection appears to favor parallel evolutionary adaptations in similar environments. Because selection in these instances has tended to favor changes that made the two groups more alike, their phenotypes have converged. Choose Answer Given what you now know, what information is most likely to produce the correct answer? • Convergent evolution occurs when traits are similar due to the natural selection of traits best adapted to similar environments. The traits are not similar due to a common ancestor between the two organisms. The skeletons of a shark (made from cartilage) and a dolphin (made from bone) are not a result of having a common ancestor. Reflect on Process Did your problem-solving process lead you to the correct answer? If not, where did the process break down or lead you astray? How can Version 1
29
you revise your approach to produce a more desirable result? • Answering this question correctly depended on your ability to use convergent evolution in a new situation… If you got an incorrect answer, did you remember that convergent evolution explains why two unrelated species may share similar phenotypic traits, or that the environment affects adaptations by the natural selection of the most adaptive phenotype? Did you have trouble extending convergent evolution to determine the correct answer? 19) C Environmental pressures provide the forces upon which natural selection acts. When organisms with some similar characteristics (size, diet, etc.) occupy similar niches in similar geographical areas, they may each evolve similar adaptations best suited for that environment. 20) C 21) A 22) C
Version 1
30
Clarify Question What is the key concept addressed by the question? • The question stem is asking you to determine how the relationship between beak depth and relative fitness will influence future beak depth. What type of thinking is required? • This is an analyze question because you have to break beak depth and relative fitness, or survivorship, into their component pieces to understand how they function in relation to one another. Gather Content What do you already know about relative fitness? What other information is related to the question? • Fitness is a relative concept; the most fit phenotype is simply the one that produces, on average, the greatest number of offspring. Choose Answer Given what you now know, what information is most likely to produce the correct answer? • Based on the graph, which phenotype presents the highest survivorship? How does survivorship influence an organism’s fitness relative to other organisms of different survivorship? If the survivorship for one phenotype is low, then how would the phenotypic variation be influenced? The mean beak size has the highest survivorship so it will be maintained. Reflect on Process Did your problem-solving process lead you to the correct answer? If not, where did the process break down or lead you astray? How can you revise your approach to produce a more desirable result? • Answering this question correctly depended not only on distinguishing between beak depth and survivorship, but on your ability to break down, or analyze, how variation is influenced by beak depth and survivorship. If you got an incorrect answer, did you remember that relative fitness would likely affect variation by selecting for the phenotype best adapted to the environment? Did you have trouble breaking down how relative fitness influences variation levels to determine the correct answer?
23) C 24) C Version 1
31
25) D 26) B Artificial selection involves a selective breeding to enhance desirable traits. Substantialincreases in phenotypic variation can be seen in short periods of time, often in just a few generations. 27) A Each generation of finches inherited beak size from the previous (parental) generation; it was the parental generation on which selection had acted to produce the offspring generation's distribution of beak size. 28) C Industrial melanism is a specific form of evolution in which the selective pressure is environmental pollution; it results in a predominance of dark individuals over light individuals. 29) A Tutt first proposed the hypothesis that those moths more similar in color to the tree trunks on which they rested would be better camouflaged and thus survive longer and reproduce more. Kettlewell, however, set up experimental field tests to test the hypothesis. 30) E Natural selection occurred in both of these examples; the pressures leading to the selection of deeper beaks following drought or darker pigmentation following pollution were environmental changes. Natural selection is the change in allele frequencies from one generation to the next in response to changes in environmental pressures. 31) B
Version 1
32
While the darker phenotype was selected for after pollution killed the lichens and deposited soot on the tree trunks, after the Clean Air Act of 1956, pollutants were removed and light-colored lichens regrew, resulting in selection for the lighter phenotype. 32) A In nonpolluted forests, light-colored lichens cover the tree trunks, making the dark forms more susceptible to predation. 33) B In dogs, artificial selection has led to a variety of different phenotypes; the different dog breeds are not considered separate species nor are they reproductively isolated. 34) D 35) C 36) C 37) A 38) D 39) C 40) B 41) B 42) E 43) B 44) C 45) D Homologous structures are those shared among different species due to common ancestry. 46) B 47) C 48) D Version 1
33
49) D 50) B 51) A 52) A
Version 1
34
Clarify Question What is the key concept addressed by the question? • The question stem is asking you to determine how rainfall and the introduction of this new plant species will influence beak size selection. What type of thinking is required? • You are being asked to take what you already know and use, or apply, it to natural selection of beak size. Gather Content What do you already know about natural selection? What other information is related to the question? • Natural selection is a method by which organisms can evolve. The seeds from the new plant species will be common during times of drought. When rains are abundant, native plants will produce small, tender seeds. Choose Answer Given what you now know, what information is most likely to produce the correct answer? • Recall that the Grants found birds with small beaks were favored in wet years because of the plentiful small, tender seeds and that birds with large beaks were favored in dry years because of the lack of small, tender seeds and the multitude of large, hard seeds. Reflect on Process Did your problem-solving process lead you to the correct answer? If not, where did the process break down or lead you astray? How can you revise your approach to produce a more desirable result? Version 1
35
• Answering this question correctly depended on your ability to use natural selection in a new situation. If you got an incorrect answer, did you remember that natural selection is influenced by biotic and abiotic factors or that rainfall affects natural selection by influencing the type of seeds present in the environment? Did you have trouble extending natural selection to determine the correct answer? 53) E
Version 1
36
Clarify Question What is the key concept addressed by the question? • The question stem is asking you to predict the results of a hypothetical study involving the release of marked dark and light colored moths into polluted and unpolluted woodlands. What type of thinking is required? • This question is asking you to weight and judge, or evaluate, how natural selection will influence the moth color phenotypic frequencies. Gather Content What do you already know about natural selection? What other information is related to the question? • Natural selection is a method by which organisms can evolve. Regardless of whether the moths are marked or unmarked, one phenotype will be favored in the polluted woods and the other phenotype will be favored in the unpolluted woods. Choose Answer Given what you now know, what information is most likely to produce the correct answer? • Note that within the polluted woods, the tree bark was darkened by soot from nearby factories. Moths with similarly colored wings would be harder to be seen by predators than moths with light-colored wings. Therefore, moths with wings colored like their environment would be more common than the moths with wing color that contrasts with the environment. Reflect on Process
Version 1
37
Did your problem-solving process lead you to the correct answer? If not, where did the process break down or lead you astray? How can you revise your approach to produce a more desirable result? • Answering this question correctly depended on your ability to weight and judge, or evaluate, natural selection and phenotypic frequencies. If you got an incorrect answer, did you remember that wing color confers a selective advantage if it matches the environment? Did you have trouble weighing the merits of wing color relative to the levels of pollution to determine the correct answer? 54) A
Version 1
38
Clarify Question What is the key concept addressed by the question? • The question stem is asking you to identify which alternative hypothesis also explains why moths wing color frequencies changed as woodland pollution levels changed. What type of thinking is required? • You are being asked to take what you already know and use, or apply, it to how natural selection will influence the moth color phenotypic frequencies. Gather Content What do you already know about natural selection? What other information is related to the question? • Natural selection is a method by which organisms can evolve. A valid alternative hypothesis will provide a logical explanation for the observed phenomenon, industrial melanism. Choose Answer Given what you now know, what information is most likely to produce the correct answer? • If dark moths emigrated, we would expect the preponderance of this morph to decrease, not increase. If the tree species was killed by pollution, then both moth morphs would be equally affected. There is no evidence that pollution increased heat, nor is there evidence that black moths absorb more heat and are subsequently more active. In any case, moths avoid predation by cryptic, hiding behavior, not by fleeing from their predators. Finally, Kettlewell placed the marks under the moths' wings and they were thus undetectable by the birds.
Version 1
39
Reflect on Process Did your problem-solving process lead you to the correct answer? If not, where did the process break down or lead you astray? How can you revise your approach to produce a more desirable result? • Answering this question correctly depended on your ability to use natural selection in a new situation. If you got an incorrect answer, did you remember that natural selection favors one morph over another or that pollution affects morph frequency by selecting for the more cryptic morph? Did you have trouble extending natural selection to determine the correct answer? 55) A
Version 1
40
Clarify Question What is the key concept addressed by the question? • The question stem is asking you to look at how artificial selection has caused the bristle number to change from the initial population to the low bristle number population and the high bristle number population. What type of thinking is required? • You are being asked to take what you already know and use, or apply, it to the effects of artificial selection on variation within populations. Gather Content What do you already know about artificial selection? What other information is related to the question? • Artificial selection occurs when humans select for a desired trait within a population. Humans have artificially selected for traits in dogs, resulting in different breeds of dogs. Can different breeds of dogs interbreed and produce mixed offspring? Choose Answer Given what you now know, what information is most likely to produce the correct answer? • Artificial selection occurs when humans select for a desired trait within a population. Ultimately, the desired trait may become exaggerated within the population far beyond the levels of variation found within the initial population. Reflect on Process Did your problem-solving process lead you to the correct answer? If not, where did the process break down or lead you astray? How can you revise your approach to produce a more desirable result? • Answering this question correctly depended on your ability to use artificial selection in a new situation. If you got an incorrect answer, did you remember that artificial selection is process commonly used by humans to select for desired traits or that artificial selection affects variation by selecting specific characteristics? Did you have trouble extending your understanding of artificial selection to determine the correct answer?
56) C
Version 1
41
Clarify Question What is the key concept addressed by the question? • The question stem is asking you to calculate how many half-lives would occur in order to have less than 4 mg left of the original 18 mg of radioactive carbon-14. What type of thinking is required? • You are being asked to take what you already know and use, or apply, it to the half-life of the radioactive isotope, carbon-14. Gather Content What do you already know about half-lives of radioactive isotopes? What other information is related to the question? • Isotopes decay at a known rate, called their half-life. During each halflife, one-half of the original amount of parent isotope has transformed into a daughter isotope. Therefore, if a sample of material had 10 g of radioactive isotope and a half-life of 3 hours, then after 3 hours, or one half-life, the sample would have 5g of radioactive isotope and 5 g of the daughter isotope. After 6 hours, or two half-lives, the sample would have 2.5 g of radioactive isotopes and 7.5 g of the daughter isotope. Choose Answer Given what you now know, what information is most likely to produce the correct answer? • The half-life of carbon-14 is 5,600 years. Therefore, after 5,600 years, the 18 mg sample of carbon-14 would have 9 mg of carbon-14 left. How many mg would be left after 11,200 years? What about after three halflives? What about after four or even nine half-lives? Reflect on Process Version 1
42
Did your problem-solving process lead you to the correct answer? If not, where did the process break down or lead you astray? How can you revise your approach to produce a more desirable result? • Answering this question correctly depended on your ability to use the concept of half-life in a new situation. If you got an incorrect answer, did you remember that half-life is the time it takes for one half of a sample of radioactive isotope to decay into the daughter isotope or that the amount of time affects how many half-lives have occurred? Did you have trouble extending half-life to determine the correct answer? 57) D
Version 1
43
Clarify Question What is the key concept addressed by the question? • The question stem is asking you to calculate how many half-lives would occur in order to have one-eighth of radioactive carbon-14 sample left. What type of thinking is required? • You are being asked to take what you already know and use, or apply, it to the half-life of the radioactive isotope, carbon-14. Gather Content What do you already know about half-lives of radioactive isotopes? What other information is related to the question? • Isotopes decay at a known rate, called their half-life. During each halflife, one-half of the original amount of parent isotope has transformed into a daughter isotope. Therefore, if a sample of material had 10 g of radioactive isotope and a half-life of 3 hours, then after 3 hours, or one half-life, the sample would have 5g of radioactive isotope and 5 g of the daughter isotope. After 6 hours, or two half-lives, the sample would have 2.5 g of radioactive isotopes and 7.5 g of the daughter isotope. Choose Answer Given what you now know, what information is most likely to produce the correct answer? • The half-life of carbon-14 is 5,600 years. Therefore, after 5,600 years, one half of your sample of carbon-14 would still be of carbon-14 left. What fraction of your sample would be left after 11,200 years? In order for 1/8 of a sample to remain 3 half-lives have passed. (1 divided by 2 equals 1/2, divided by 2 equals 1/4, divided by 2 equals 1/8)
Version 1
44
Reflect on Process Did your problem-solving process lead you to the correct answer? If not, where did the process break down or lead you astray? How can you revise your approach to produce a more desirable result? • Answering this question correctly depended on your ability to use the concept of half-life in a new situation. If you got an incorrect answer, did you remember that half-life is the time it takes for one half of a sample of radioactive isotope to decay into the daughter isotope or that the amount of time affects how many half-lives have occurred? Did you have trouble extending half-life to determine the correct answer? 58) Section Break 58.1) E
Version 1
45
Clarify Question What is the key concept addressed by the question? • The question stem is asking you to assess how beak depth changes during the wet years and dry years. What type of thinking is required? • This is an analyze question because you have to break beak depth and changes in precipitation into their component pieces to understand how they function in relation to one another. Gather Content What do you already know about natural selection? What other information is related to the question? • Changes in the amount of precipitation influence the food available for consumption by the birds. Beak depth limits the birds to only certain food types. Choose Answer Given what you now know, what information is most likely to produce the correct answer? • Natural selection can only be applied to the variation found within the population. If the environment fluctuates between wet and dry years, then what kind of beak depth would be best adapted to the environment? It may seem like an intermediate beak size would also be favored in this example, however, the lack of food choices for that size of beak would be limited and therefore not favored. Reflect on Process Did your problem-solving process lead you to the correct answer? Version 1
46
If not, where did the process break down or lead you astray? How can you revise your approach to produce a more desirable result? • Answering this question correctly depended not only on distinguishing between precipitation and beak depth, but on your ability to break down, or analyze, how natural selection is influenced by the environment. If you got an incorrect answer, did you remember that components of natural selection include adaptations selected from the available variations within the population or that environment would likely affect beak depth by changing which beak depth is best adapted to the changing environment? Did you have trouble breaking down how natural selection works to determine the correct answer? 58.2) C
Version 1
47
Clarify Question What is the key concept addressed by the question? • The question stem is asking you to assess how beak depth changes during the wet years and dry years. What type of thinking is required? • This is an analyze question because you have to break beak depth and changes in precipitation into their component pieces to understand how they function in relation to one another. Gather Content What do you already know about natural selection? What other information is related to the question? • Changes in the amount of precipitation influence the food available for consumption by the birds. Beak depth limits the birds to only certain food types. Choose Answer Given what you now know, what information is most likely to produce the correct answer? • Natural selection can only be applied to the variation found within the population. If the environment fluctuates between wet and dry years, then what kind of beak depth would be best adapted to the environment? Reflect on Process Did your problem-solving process lead you to the correct answer? If not, where did the process break down or lead you astray? How can you revise your approach to produce a more desirable result? Version 1
48
• Answering this question correctly depended not only on distinguishing between precipitation and beak depth, but on your ability to break down, or analyze, how natural selection is influenced by the environment. If you got an incorrect answer, did you remember that components of natural selection include adaptations selected from the available variations within the population or that environment would likely affect beak depth by changing which beak depth is best adapted to the changing environment? Did you have trouble breaking down how natural selection works to determine the correct answer? 59) Section Break 59.1) C 59.2) E 60) Section Break 60.1) B 60.2) D
Version 1
49
CHAPTER 21 CHECK ALL THE APPLY. Choose all options that best completes the statement or answers the question. 1) Select the postzygotic isolating mechanisms. (Check all that apply.) A) Hybrids are sterile. B) Hybrids develop abnormally. C) Hybrids fail to become established in nature. D) Hybrids cannot be formed.
2)
Members of the same biological species can __________. (Check all that apply.) A) have several isolated gene pools B) differ from other species in one or more characteristics C) generate fertile offspring with the members of a different species D) interbreed freely with each other
3) Select the concepts emphasized by Ernst Mayr's biological species concept. (Check all that apply.) A) Hybridizationbetween different species B) Production offertile offspring within the species C) Populations inzoos D) Reproductiveisolation from other species
4) Why do the Galápagos Islands and the Hawaiian Islands have so many endemic species of flies and birds? (Check all that apply.)
Version 1
1
A) There are numerous islands in each archipelago. B) There is adequate distance between islands to allow divergence. C) The founder effect was common when the islands were colonized. D) Each bird or fly species has many common ancestors.
5) Choose the factors contributing to adaptive radiation of Drosophila in the Hawaiian Islands. (Check all that apply.) A) New islands are constantly being formed. B) There are high rates of mutation due to radioactive materials in the lava flows. C) The islands are highly volcanic. D) Drosophila species have no predators on the Hawaiian Islands.
6) Choose the true statements about the diversity of Darwin's finches on the Galápagos Islands. (Check all that apply.) A) All of the finch species share a common ancestor that colonized the islands. B) Prior to the adaptive radiation of finches, there were no unoccupied habitats in the Galápagos Islands. C) Competition among finches for food sources led to character displacement of beak shapes. D) Beak shape is not an important key innovation.
7)
Which statements about the Lake Victoria cichlid fishes are true? (Check all that apply.)
Version 1
2
A) The adaptive radiation of the estimated 300 species of cichlid fish was very rapid and took place during the last 200,000 years. B) The adaptive radiation of the Lake Victoria cichlids may have resulted from a key evolutionary innovation. C) Speciation of the Lake Victoria cichlids was mainly sympatric. D) The diversity of the Lake Victoria cichlids continues today because the lake is devoid of predators. E) Many of the Lake Victoria cichlids have very specialized adaptations that allow them to occupy diverse habitats within the lake.
8) Which statements about the adaptive radiation of New Zealand alpine buttercups are true? (Check all that apply.) A) Allopatric speciation occurred mainly during periods of glacial recession. B) Divergence occurred mainly during periods of glacial advance. C) The availability of a variety of habitats favored diversification. D) Reproductive isolation was reinforced mainly during periods of glacial recession.
9) Which statements about gradualism and punctuated equilibrium are true? (Check all that apply.) A) Darwin's publication On the Origin of Species changed the view of evolution from gradualism to that of punctuated equilibrium. B) According to the theory of punctuated equilibrium, evolutionary change occurs only during brief periods of speciation. C) Stasis is likely caused by periods of disruptive and directional selection. D) Gradualism suggests that evolutionary change is not linked to speciation.
10) Select the reasons why adaptive radiations are often observed on remote volcanic islands. (Check all that apply.)
Version 1
3
A) When a new volcanic island forms, there are many empty niches. B) Only a few individuals reach remote islands. C) Remote islands have more plant diversity than the mainland does. D) Evolution is intrinsically faster on islands than on the mainland.
11) The most famous and well-studied mass extinction occurred at the end of the Cretaceous period (65 million years ago), at which time the dinosaurs and a variety of other terrestrial and aquatic organisms went extinct. Recent findings have supported the hypothesis that this extinction event was triggered when a large asteroid slammed into Earth. Which of the following could cause a global mass extinction following an asteroid strike? (Check all that apply.) A) The impact triggered global forest fires that obscured the sun. B) The asteroid killed animals it hit upon impact. C) The asteroid was very hot when it entered the atmosphere and heated up the planet. D) The asteroid created a massive dust cloud that obscured the sun.
12) Scientists estimate that as much as one-fourth of all species will become extinct in the near future, a rate of extinction not seen on Earth since the Cretaceous mass extinction. How are humans contributing to this mass extinction? (Check all that apply.) A) Contributing to climate change B) Intentionally killing all members of a species C) Destroying habitat D) Blocking evolution
13) Scientists observed that marine bryozoan fossils lack intermediate forms. This evidence supports which hypotheses? (Check all that apply.)
Version 1
4
A) Evolution occurs in spurts, alternating with long periods of little change. B) Evolution proceeds steadily, with gradual, incremental change in a given evolutionary line. C) There is variability in the rate of evolution. D) Evolution proceeds at a constant rate that is the same for all organisms.
14) Two hundred years agothe fly species Rhagoletis pomonella only laid its eggs on hawthorn fruit. Todaydifferent populations of R. pomonella lay their eggs on hawthorns OR apples. Apples occur within the range of hawthorns so divergence between apple flies and hawthorn flies could be the first step in sympatric speciation. Choose the evidence that would suggest that R. pomonella is currently undergoing sympatric speciation. (Check all that apply.) A) Apple flies typically mate with apple flies, and hawthorn flies typically mate with hawthorn flies. B) Apple flies and hawthorn flies emerge from their hosts at different times of the year. C) Apple flies and hawthorn flies are able to form fertile hybrids. D) Apple flies and hawthorn flies are physically indistinguishable from each other.
15) Which examples would be considered different species under the biological species concept? (Check all that apply.) A) Two bear species are able to breed in zoos, but have separate ranges in nature. B) Two skunk species look morphologically similar, but are unable to interbreed. C) Two snake species look different, but often form fertile hybrids. D) Two plant species with no gene flow between them. E) Two tree species have different leaf shapes, but form hybrids in their overlapping ranges.
16) Two species regularly come into contact and form hybrids that have higher fitness than either of the parental species. What are some likely consequences of regular hybridization between these species? (Check all that apply.)
Version 1
5
A) Evolution of a third species B) Reinforcement of prezygotic isolating mechanisms C) Extincton of one or both of the parental species. D) Increased gene flow between the parental species E) Complete reproductive isolation of the two original species
17) Indicate the scenario(s) that could directly result in sympatric speciation. Check all that apply. A) Two sunflower species cross to form a healthy hybrid individual with equal copies of chromosomes from each parental species. B) A new river channel divides a population of beetles into two populations. C) A large fence blocks the intermingling of two populations of a species. D) A subset of flies within a larger population colonizes a new host plant.
MULTIPLE CHOICE - Choose the one alternative that best completes the statement or answers the question. 18) Scientists investigating speciation in fruit flies divided a single population of flies into separate populations that each lived in their own cage on one of two food sources: maltose-based food or starch-based food. After several generations, the flies’ mate preference was tested, and it was found that flies strongly preferred to mate with other flies that were raised on the same food source.This result suggests that __________. A) selection on food source indirectly led to the evolution of reproductive isolating mechanisms B) selection on reproductive isolating mechanisms indirectly led to the evolution of different food preferences C) reproductive isolating mechanisms evolved primarily through genetic drift D) different food preferences evolved primarily through genetic drift
19) The model that assumes that evolution proceeds with slow successive change in a given evolutionary line is referred to as __________.
Version 1
6
A) successive descent model B) punctuated equilibrium C) gradualism D) the allopatric model E) the sympatric model
20) The type of speciation that results in the formation of separate species as a result of geographic isolationis called ____________ speciation. A) gradual B) sympatric C) allopatric D) sexual E) adaptive
21) When two species cannot form hybrid zygotes, the mechanism that keeps the species separate is known asa _____________ isolating mechanism. A) hybridization B) postzygotic C) prezygotic D) adaptive E) differential
22) A type of isolating mechanism that leads to reproductive isolation after a hybrid zygote has formed is called a _____________ isolating mechanism. A) hybridization B) postzygotic C) prezygotic D) adaptive E) differential
Version 1
7
23)
Which isolating mechanism is postzygotic? A) Hybridsterility B) Behavioralisolation C) Ecologicalisolation D) Mechanicalisolation E) Temporalisolation
24) Hybridization regularly occurs between three species of Darwin’s finches on Daphne Major, but the finches are still considered different species because __________. A) they have different fitnesses B) they are reproductively isolated C) they are mechanically isolated D) they occupy different niches
25) According to the Biological Species Concept, two groups of organisms that are unable to form fertile, viable hybrids are considered to be different __________. A) races B) species C) kingdoms D) hybrids E) cohabitants
26) If populations within the samearea split into species, the process is known as __________.
Version 1
8
A) sympatric speciation B) allopatric speciation C) racial speciation D) ecotypical speciation E) reproductive speciation
27)
If species are competing for the same resource, natural selection will lead to ______. A) prezygotic isolating mechanisms B) hybrids C) character displacement D) polyploid individuals
28)
Once species have formed, they keep their identity by __________. A) developmental differences B) genetic blocks C) isolating mechanisms D) somatic mutations E) structural modifications
29) Many species might coexist in a particular environment by occupying ecological __________. A) biomes B) niches C) territories D) domains E) ranges
Version 1
9
30) Prezygotic mechanisms lead to reproductive isolation by preventing the formation of __________. A) mating pairs B) fertile eggs C) fertile sperm or pollen cells D) somatic cells E) hybrid zygotes
31) Very rapid speciation occurred when cichlid fishes were isolated in Lake Victoria. Widespread extinction was dramatically seen when __________. A) artificial selection stopped B) predatory fish were added C) hybridization stopped D) fishing was prohibited E) insect larvae stocks were depleted
32) Species formation among alpine buttercups in New Zealand was promoted by __________. A) mechanical isolation B) gametic isolation C) periodic isolation by glaciation D) temporal isolation E) geographical isolation
33) Punctuated equilibrium proposed by Eldredge and Gould in 1972 is an evolutionary process that involves __________.
Version 1
10
A) gradual change observable in the fossil record with very little, if any, stasis B) "spurts" of change observable in the fossil record followed by periods of stasis C) gradual change observable in the fossil record followed by periods of stasis D) "spurts" of change observable in the fossil record with very little if any stasis E) gradual change observable in the fossil record followed by periods of no change at all
34)
Adaptive radiation is best described as the __________.
A) existence of groups of closely related species recently evolved from a common ancestor B) existence of groups of distantly related species recently evolved from a common ancestor C) existence of groups of closely related species recently evolved from different ancestors through hybridization D) existence of individuals of closely related species that originated in different areas within diverse habitats but that have rejoined as a single species
35)
The most common mechanism for sympatric speciation in plants is __________. A) polyploidy B) geographic isolation C) spontaneous mutation D) punctuated equilibrium E) disruptive selection
36) Lake Victoria is famous for its variety of cichlid fishes. In fact, they offer the most diverse assembly of vertebrates presently known to science. One trait that scientists have focused on as one possible reason for their adaptability is __________.
Version 1
11
A) a second set of scales that lie beneath the top outermost scales B) a second set of jaws in their throats C) gill arches that are not covered by an operculum, which prevents hiding places for gill parasites D) digestive tracts that are able to process most substrates, which has eliminated the need for feeding specialization E) ability to change color in relationship to their environment
37) Two populations that inhabit the same area but remain genetically distinct can best be called __________. A) species B) subspecies (races) C) polymorphic D) allopatric species E) sympatric species
38)
The biological species concept of Ernst Mayr cannot be applied to __________. A) sympatric organisms B) allopatric organisms C) eukaryotic organisms D) prokaryotic organisms E) asexually reproducing organisms
39)
Which statement is true about speciation on island archipelagos? A) Speciation is slow because it depends on emigration from the mainland. B) Most speciation occurs allopatrically. C) Ecological competition may cause new species to go extinct, slowing speciation. D) Speciation by polyploidy is most common. E) Species clusters are less common.
Version 1
12
40)
Which form of speciation is most rapid? A) Allopatric speciation on the mainland B) Allopatric speciation on island archipelagos C) Sympatric speciation via polyploidy D) Sympatric speciation via disruptive selection E) Sympatric speciation via character displacement
41) Due to character displacement, two closely related species may differ most morphologically when they __________. A) are allopatric B) are sympatric C) occupy the same niche D) occupy different niches E) share a common ancestor
42) Two individuals of the same speciesmate and produce offspring that can mate with each other but are unable to mate with other members of the species. This is an example of ____________. A) allopatric speciation via autopolyploidy B) sympatric speciation via disruptive selection C) sympatric speciation via autopolyploidy D) allopatric speciation via allopolyploidy E) sympatric speciation via allopolyploidy
43) If a new species is produced by the interbreeding of two members of distinct species, this would be __________ speciation achieved via __________.
Version 1
13
A) allopatric; autopolyploidy B) allopatric; allopolyploidy C) sympatric; autopolyploidy D) sympatric; allopolyploidy E) sympatric; divergence
44)
Reinforcement is most likely to drive the evolution of __________.
A) prezygotic isolating mechanisms because selection favors alleles that prevent hybridization B) postzygotic isolating mechanisms because selection acts after the formation of hybrids C) prezygotic isolating mechanisms because selection favors alleles that promote hybridization D) postzygotic isolating mechanisms because hybrids have low fitness relative to the parents
45) In 1938, scientists found a population of coelacanths in the Indian Ocean. This fish species is considered a “living fossil” and appears not to have changed morphologically for 80 million years.This discovery supports the ___________ model of evolution. A) gradualism B) punctuated equilibrium C) adaptation D) natural selection E) common descent
46) Scientists found that different beak shapes correlate with different mating songs in Darwin's finches. Which hypothesis does this support?
Version 1
14
A) Darwin's finches are not different species according to the biological species concept. B) Adaptation to different food sources led to behavioral isolation in Darwin's finches. C) Mating songs are primarily determined by the acoustics of different beaks. D) Adaptation to different food sources likely occurred after reproductive isolation in Darwin's finches.
47) Males of many marsupial species have forked penises that are incompatible with the anatomy of female placental mammals. What type of isolating mechanism does this represent? A) Ecological isolation B) Temporal isolation C) Mechanical incompatibility D) Hybrid sterility E) Prevention of gamete fusion
48) In the southeastern United States, two species of wild lettuce ( Lactuca) do not usually form hybrids, because they bloom at different seasons. The means of isolation appears to be __________. A) postzygotic and behavioral B) prezygotic and temporal C) prezygotic and geographical D) prezygotic and mechanical E) postzygotic and ecological
49) The hundreds of species of Drosophila in Hawaii exhibit different mating rituals. This isolating mechanism is __________.
Version 1
15
A) prezygotic and behavioral B) prezygotic and temporal C) prezygotic and geographical D) postzygotic and mechanical E) prezygotic and ecological
50) Scientists studying a group of related bird species on an island archipelago discover that these species are always more closely related to each other within an island than they are between any two islands. The most likely explanation for this result is that __________. A) each bird species evolved different adaptations in allopatry and then colonized the other islands B) one bird species colonized each island and then evolved into distinct species on that island through character displacement C) each bird species became reproductively isolated by living on a different island, then later colonized the other islands and acquired new adaptations D) many ancestral bird species colonized each island
51) The Isthmus of Panama lies between the Caribbean Sea and the Pacific Ocean. Studies of snapping shrimp have shown that each Caribbean shrimp species is more closely related to a Pacific shrimp species than it is to any other Caribbean shrimp species. The most likely explanation for this result is that __________. A) snapping shrimp in the Caribbean and the Pacific were never part of the same population B) snapping shrimp species in each ocean diverged prior to the formation of the Isthmus of Panama C) snapping shrimp species in each ocean diverged after the formation of the Isthmus of Panama D) creation of the Panama Canal allowed interbreeding between populations of shrimp
Version 1
16
52) The sunflower species Helianthus anomalus is thought to have been formed by hybridization between the sunflower species H. petiolaris and H. annuus because it contains a set of chromsomes from both parental species. H. anomalus is most likely an example of instantaneous speciation through __________. A) allopolyploidy B) autopolyploidy C) parthenogenesis D) hybrid sterility
53) Two bird species have different mating calls, occupy different habitats in nature, and when they hybridize, the hybrids are infertile. Which isolating mechanisms maintain these species? A) Ecological isolation, behavioral isolation, and hybrid sterility B) Mechanical isolation, hybrid sterility, geographical isolation C) Ecological isolation, mechanical isolation, hybrid sterility D) Geographical isolation, temporal isolation, behavioral isolation
54) A single red squirrel species lives in avolcanic valley. One day ahuge lava flowseparates the valleyproducingtwo populations of squirrels. What must occur for the two populationsto be considered separate species? A) The two squirrel populations must feed on different species of plants. B) The environment must change so that the populations exist in distinct habitats. C) The fur color or other morphological characteristic of the two squirrel populations must become distinct. D) The two squirrel populations must select mates using different behaviors. E) The two squirrel populations must be reproductively isolated.
55) Several species of sea urchins may inhabit the same tide pool and their reproductive periods overlap. Although they practice external fertilization, they seldom interbreed because eggs produce chemicals that are only attractive to sperm of their own species. This is an example of a _________ barrier that _______________.
Version 1
17
A) prezygotic; causes geographic isolation B) prezygotic; causes temporal isolation C) prezygotic; prevents gamete fusion D) postzygotic; prevents zygote fusion E) postzygotic; causes hybrid infertility
56) Several new tetraploid species of a plant called salsify (Tragopogon) have arisen in western North America. These new species each arose from a diploid species that went through the process of allopolyploidy. This mode of speciation is a type of ________ and it occurs _______. A) allopatric speciation; almost instantaneously B) allopatric speciation; very slowly C) sympatric speciation; almost instantaneously D) sympatric speciation; very slowly E) adaptive radiation; very slowly
57) Many male songbirds have very distinctive songs that are used to attract females. These songs are an example of a ________ isolating mechanism. A) postzygotic,ecological B) prezygotic,behavioral C) prezygotic,temporal D) postzygotic,mechanical E) prezygotic, gamete fusion
58) Imagine two species of birds with similar beak sizes and diets. Where the two species are allopatric, we would expect that their beak sizes __________. A) should be more different than when sympatric B) should be more similar than when sympatric C) should be larger than when sympatric D) should be smaller than when sympatric
Version 1
18
59) Choose the best cause and effect sequence for the process of speciation by natural selection. A) Variation --> Selection --> Reproductive Isolation --> Mutation --> Speciation B) Mutation --> Variation --> Selection --> Reproductive Isolation --> Speciation C) Speciation --> Reproductive Isolation --> Selection --> Mutation --> Variation D) Variation --> Reproductive Isolation --> Speciation --> Mutation --> Selection E) Selection --> Speciation --> Reproductive Isolation --> Variation --> Mutation
60)
Which is an example of temporal isolation?
A) One plant species blooms in June-July, and a second plant species blooms in AugustSeptember. B) One plant species is pollinated by bees and a second plant species is pollinated by moths. C) Two plant species live on the same mountainside, but one lives on rocky outcrops and the other in open meadows. D) Two plant species have incompatible gametes.
61) NASA scientists send a small population of dogs to a new planet in order to test if it will be hospitable for human colonization.Through a random lottery, only Great Dane and St. Bernard dog breeds are sent, and the resulting population consists of tall, long-haired dogs.After 1,000 humanyears these dogs return to Earth and are no longer able to mate with Earth dogs.This scenario is an example of A) allopatric speciation through genetic drift. B) allopatric speciation through natural selection. C) sympatric speciation through genetic drift. D) sympatric speciation through natural selection.
62) Diversification of mammals after the extinction of the dinosaurs is considered an adaptive radiation because a few mammal species rapidly evolved into most of the current mammalian lineages in a relatively short amount of time. This diversification was partially driven by the evolution of endothermic homeostasis in mammals, which is an example of Version 1
19
A) allopatric speciation. B) reproductive isolation. C) a key innovation. D) hybridization.
63) Courtship for Anolis lizards involves the male displaying a dewlap (skin under the throat area) that varies in appearance based upon the species and their environment. Lizards that are found in darker, more shaded environments will have dewlaps that are lighter in color because they are more noticeable by potential mates. Whatmechanism is at work in speciation among Anolislizards? A) Postzygotic isolation B) Reinforcement C) Genetic drift D) Natural selection
FILL IN THE BLANK. Write the word or phrase that best completes each statement or answers the question. 64) The __________ __________ model assumes that evolution occurs in spurts, between which there are long periods of stasis with little evolutionary change.
65) The _______ definition of species says that a speciesis a group of organisms that are reproductively isolated from other such groups.
66)
Speciation may result when reproductive isolation is ______ by natural selection.
67) The phenomenon where one species diverges into many by occupying a series of different habitats within a region is called __________ _________.
Version 1
20
68) The process by which related species diverge in order to reduce competition is called ___________ _________.
Version 1
21
Answer Key Test name: Chap 21_3e 1) [A, B, C] 2) [A, B, D] 3) [B, D] 4) [A, B, C] 5) [A, C] 6) [A, C] 7) [A, B, E] 8) [B, C, D] 9) [B, D] 10) [A, B] 11) [A, D] To have a global impact on terrestrial and aquatic life, the amount of sunlight reaching the earth would have to be blocked. 12) [A, C] Altering ecosystems has the biggest effect on where a species can live. 13) [A, C]
Version 1
22
Clarify Question • What is the key concept addressed by the question? o This question addresses the pace of evolution. • What type of thinking is required? o This question is asking you to analyze the information given, using logic, to dissect the problem and determine the answer. Gather Content • What do you already know about modes of evolutionary change? o The punctuated equilibrium hypothesis argues that species have long periods of little or no evolutionary change (termed stasis), punctuated by quick bursts of evolutionary change. oGradualism is the accumulation of almost imperceptible changes that eventually result in major differences. Choose Answer • Given what you now know, what information and/or problemsolving approach is most likely to produce the correct answer? o Does the lack of intermediate fossils suggest that evolution proceeds steadily? No, because then you would expect to see the gradual change reflected in the fossil record. o Does the lack of intermediate fossils suggest that evolution proceeds at a constant rate that is the same for all organisms? No, because you would expect to see intermediate fossils represented. o Does the lack of intermediate fossils suggest that there is variability in the rate of evolution? Yes. Infrequent, rapid change means that intermediate forms may not become fossilized. o Does the lack of intermediate fossils suggest that evolution occurs in spurts, alternating with long periods of little change? Yes. This can Version 1
23
explain why intermediate forms were not well-represented in the fossil record. Reflect on Process • Did your problem-solving process lead you to the correct answer? If not, where did the process break down or lead you astray? How can you revise your approach to produce a more desirable result? o If you got the right answer, good job! If not, did you recognize that a lack of intermediate fossils suggests that change happens quickly, in keeping with the punctuated equilibrium model? 14) [A, B]
Version 1
24
Clarify Question • What is the key concept addressed by the question? o This question addresses sympatric speciation. • What type of thinking is required? o This question is asking you to analyze the information given, using logic, to dissect the problem and determine the answer. Gather Content • What do you already know about sympatric speciation? o Sympatric speciation is speciation that occurs without geographic separation. o Reproductive isolating mechanisms can contribute to speciation. o Prezygotic isolating mechanisms include ecological isolation, behavioral isolation, temporal isolation, mechanical isolation, and prevention of gamete fusion. o Postzygotic isolation occurs when hybrids between species are either inviable (poor at surviving) or infertile (unable to reproduce themselves). Choose Answer • Given what you now know, what information and/or problemsolving approach is most likely to produce the correct answer? o Which pieces of evidence indicate that the host preference is leading towards less interbreeding between the two populations? Reflect on Process • Did your problem-solving process lead you to the correct answer? If not, where did the process break down or lead you astray? How can Version 1
25
you revise your approach to produce a more desirable result? o If you got the right answer, way to go! If not, did you recognize that a host preference can drive sympatric speciation, particularly if the adults are ready to mate at different times, and typically mate only with flies on the same host? 15) [A, B, D]
Version 1
26
Clarify Question • What is the key concept addressed by the question? o This question addresses the biological species concept. • What type of thinking is required? o This question is asking you to take what you already know and apply it to this unfamiliar situation. Gather Content • What do you already know about the biological species concept? o The biological species concept says that a species is composed of natural populations whose members mate with each other and produce fertile offspring. Conversely, natural populations whose members do not mate with each other or who cannot produce fertile offspring are said to be reproductively-isolated and, therefore, are members of different species. Choose Answer • Given what you now know, what information and/or problemsolving approach is most likely to produce the correct answer? o Which option(s) suggest(s) that reproductive isolation occurs in natural populations? Reflect on Process • Did your problem-solving process lead you to the correct answer? If not, where did the process break down or lead you astray? How can you revise your approach to produce a more desirable result? o If you got the right answer, well done! If not, did you recognize that the biological species concept only distinguishes populations as separate Version 1
27
species when they are reproductively isolated in nature? 16) [A, C, D]
Version 1
28
Clarify Question • What is the key concept addressed by the question? o This question addresses gene flow before speciation. • What type of thinking is required? o This question is asking you to analyze the information given, using logic, to dissect the problem and determine the answer. Gather Content • What do you already know about gene flow before speciation? o If hybrids are LESS fit than the parents then selection would improve prezygotic isolating mechanisms andreinforcement is likely to occur. o However, if hybrids are MORE fit than the parents then there will likely be increased gene flow between the parental species. Choose Answer • Given what you now know, what information and/or problemsolving approach is most likely to produce the correct answer? o In this casehybrids are more fit. So, reinforcement is not likely to occur, and complete reproductive isolation and speciation are not likely to occur. o There may be evolution of a third speciesor even extinction of one of both of the parental speciesif they becomeoutcompeted by the hybrid. Reflect on Process • Did your problem-solving process lead you to the correct answer? If not, where did the process break down or lead you astray? How can you revise your approach to produce a more desirable result? o If you got the right answer, well done. If not, did you recognize that reinforcement of reproductive isolation requires that the hybrids are less Version 1
29
fit than the parents? 17) [A, D]
Version 1
30
Clarify Question • What is the key concept addressed by the question? o This question addresses sympatric speciation. • What type of thinking is required? o This question is asking you to take what you already know and apply it to this unfamiliar situation. Gather Content • What do you already know about sympatric and allopatric speciation? o A population can diverge into two species because the population has been separated geographically: this is allopatric speciation. o Or, the population can diverge into separate species even when the populations are not separated: sympatric speciation. Choose Answer • Given what you now know, what information and/or problemsolving approach is most likely to produce the correct answer? o Sympatric speciation can only occur when two populations are reproductively isolated but not geographically isolated. o The two examples here of geographically overlapping populations are the hybridizing sunflower species and the flies that prefer a new host plant. Reflect on Process • Did your problem-solving process lead you to the correct answer? If not, where did the process break down or lead you astray? How can you revise your approach to produce a more desirable result? Version 1
31
o If you got the right answer, well done. If not, did you recognize that sympatric speciation occurs when two populations that overlap geographically are reproductively isolated? 18) A
Version 1
32
Clarify Question ● What is the key concept addressed by the question? ● The key concept addressed is the effect of natural selection and genetic drift on speciation. ● What type of thinking is required? ● The question is asking you to analyze the results of an experiment in which fruit flies raised in different environments developed differentmating preferences. Gather Content ● What do you already know about the effect of natural selection and genetic drift on speciation? What other information is related to the question? ● Over time any two isolated populations will diverge because of genetic drift. This random divergence can affect traits responsible for reproductive isolation. ● Although random processes such as genetic drift and the founder effect are always actingnatural selection probably also plays a role in the speciation process. ● As populations of a species adapt to different circumstances, such as a different primary food source, they may accumulate genetic differences that lead to reproductive isolation. Choose Answer ● Given what you now know, what information is most likely to produce the correct answer? ● Laboratory experiments on fruit flies and other fast-reproducing organisms have shown that genetic drift by itself can lead to reproductive isolation. However, reproductive isolation evolves more rapidly when the populations are forced to adapt to a new laboratory environment (for example, an altered temperature or food type). In such experiments, natural selection does not directly favor traits because they lead to reproductive isolation. Still, the end effect of Version 1
33
adaptive divergence is that the populations reared in different environments become reproductively isolated. Reflect on Process ● Did your problem-solving process lead you to the correct answer? If not, where did the process break down or lead you astray? How can you revise your approach to produce a more desirable result? ● If you got the correct answer, hooray! If you got an incorrect answer, did you remember that adaptation via selection to different environments indirectly leads to populations becoming reproductively isolated from each other?
19) C 20) C 21) C 22) B 23) A 24) D 25) B 26) A 27) C 28) C 29) B 30) E 31) B 32) C 33) B 34) A 35) A
Version 1
34
36) B 37) A 38) E 39) B 40) C 41) B 42) C 43) D 44) A 45) B
Version 1
35
Clarify Question • What is the key concept addressed by the question? o This question addresses the pace of evolution. • What type of thinking is required? o This question is asking you to take what you already know and apply it to this unfamiliar situation. Gather Content • What do you already know about modes of evolutionary change? o The punctuated equilibrium hypothesis argues that species have long periods of little or no evolutionary change (termed stasis), punctuated by quick bursts of evolutionary change. oGradualism is the accumulation of almost imperceptible changes that eventually result in major differences. Choose Answer • Given what you now know, what information and/or problemsolving approach is most likely to produce the correct answer? o The unusual thing about the coelacanth is that it appears to have not changed over a long time. This is an example of stasis. o Of the listed options, the one that predicts that species will sometimes stay the same is punctuated equilibrium. Reflect on Process • Did your problem-solving process lead you to the correct answer? If not, where did the process break down or lead you astray? How can you revise your approach to produce a more desirable result? o If you got the correct answer, well done! If not, did you recognize that Version 1
36
the coelacanth is an example of a species in stasis, which is a prediction of punctuated equilibrium? 46) B
Version 1
37
Clarify Question • What is the key concept addressed by the question? o This question addresses reproductive isolation. • What type of thinking is required? o This question is asking you to weigh and judge evidence, or evaluate, to choose the best of the possible answers. Gather Content • What do you already know about reproductive isolation? o Populations are considered to be different species when the members are reproductively isolated from each other. o Reproductive isolating mechanisms prevent genetic exchange between species. Some prevent the formation of zygotes : prezygoticisolating mechanisms. Others prevent the proper functioning of zygotes after they form : postzygotic-isolating mechanisms. o Behavioral isolation is when related species differ in their courtship rituals, keeping these species distinct. Choose Answer • Given what you now know, what information and/or problemsolving approach is most likely to produce the correct answer? o Is it likely that Darwin's finches are not different species according to the biological species concept? No. The biological species concept defines species as groups which are reproductively isolated. Having different mating songs is a mechanism of reproductive isolation. o Is it likely that mating songs are primarily determined by the acoustics of different beaks? No, this doesn’t seem likely. Mating songs vary mostly in the pattern and tempo of the notes, a feature of brain circuits, not the beak. Version 1
38
o Is it likely that adaptation to different food sources occurred after reproductive isolation in Darwin's finches? Not likely. Reproductive isolation wouldn’t necessarily encourage two new species to diverge in their food preferences. However, different food preferences CAN set the stage for reproductive isolation to follow. o Is it likely that adaptation to different food sources led to behavioral isolation in Darwin's finches? Yes. Having different niches, ecological isolation, limits the hybridization of two groups, causing reproductive isolation, and further behavioral isolation due to changes in mating songs. Reflect on Process • Did your problem-solving process lead you to the correct answer? If not, where did the process break down or lead you astray? How can you revise your approach to produce a more desirable result? o If you got the right answer, well done! If not, did you recognize that preferences for different foods can lead to ecological isolation, which allows development of behavioral isolation barriers? 47) C
Version 1
39
Clarify Question • What is the key concept addressed by the question? o This question addresses reproductive isolation. • What type of thinking is required? o This question is asking you to take what you already know and apply it to this unfamiliar situation. Gather Content • What do you already know about reproductive isolation? o Reproductive isolating mechanisms prevent genetic exchange between species. Some prevent the formation of zygotes : prezygoticisolating mechanisms. Others prevent the proper functioning of zygotes after they form : postzygotic-isolating mechanisms. o Mechanical isolation is when the structural morphology of two species (e.g. copulatory organs) prevents mating. Choose Answer • Given what you now know, what information and/or problemsolving approach is most likely to produce the correct answer? o In this example, it is the morphologythat prevents mating, this is an example of mechanical isolation. Reflect on Process • Did your problem-solving process lead you to the correct answer? If not, where did the process break down or lead you astray? How can you revise your approach to produce a more desirable result? o If you got the right answer, hooray! If not, did you recognize that when two groups have incompatible genitalia, we call it mechanical isolation? Version 1
40
48) B
Version 1
41
Clarify Question • What is the key concept addressed by the question? o This question addresses reproductive isolating mechanisms. • What type of thinking is required? o This question is asking you to take what you already know and apply it to this unfamiliar situation. Gather Content • What do you already know about reproductive isolation? o Prezygotic-isolating mechanisms include ecological isolation, behavioral isolation, temporal (relating to time) isolation, mechanical isolation, and prevention of gamete fusion. o Postzygotic isolation occurs when hybrids between species are either inviable (poor at surviving) or infertile (unable to reproduce themselves). Choose Answer • Given what you now know, what information and/or problemsolving approach is most likely to produce the correct answer? o The differentlettuce speciescannot cross because they bloom at different times of year. This is an example of temporal isolation, isolation based on timing. o Because the pollen and egg never meet at alla zygote never has a chance to form. Thus this is a prezygotic-isolating mechanism. Reflect on Process • Did your problem-solving process lead you to the correct answer? If not, where did the process break down or lead you astray? How can Version 1
42
you revise your approach to produce a more desirable result? o If you got the right answer, well done! If not, did you recognize that blooming at different times is an example of temporal isolation, which is a prezygotic mechanism of reproductive isolation? 49) A
Version 1
43
Clarify Question • What is the key concept addressed by the question? o This question addresses reproductive isolation mechanisms. • What type of thinking is required? o This question is asking you to take what you already know and apply it to this unfamiliar situation. Gather Content • What do you already know about reproductive isolation mechanisms? o Prezygotic isolating mechanisms include ecological isolation, behavioral isolation, temporal isolation, mechanical isolation, and prevention of gamete fusion. o Postzygotic isolation occurs when hybrids between species are either inviable (poor at surviving) or infertile (unable to reproduce themselves). Choose Answer • Given what you now know, what information and/or problemsolving approach is most likely to produce the correct answer? o The Hawaiian Drosophila speciesdo not interbreed because they have different courtship rituals. This is a behavioral isolating mechanism. o Without interbreeding, the sperm and egg never meet at all, so a zygote never has a chance to form. Thus this is a prezygotic isolating mechanism. Reflect on Process • Did your problem-solving process lead you to the correct answer? If Version 1
44
not, where did the process break down or lead you astray? How can you revise your approach to produce a more desirable result? o If you got the right answer, congrats! If not, did you recognize that when two species do not interbreed because of incompatible courtship rituals, whichis a behavioral, prezygotic isolating mechanism? 50) B
Version 1
45
Clarify Question • What is the key concept addressed by the question? o This question addresses adaptive radiation. • What type of thinking is required? o This question is asking you to weigh and judge evidence, or evaluate, to choose the best of the possible answers. Gather Content • What do you already know about adaptive radiation? o Species may evolve different adaptations while separated (allopatric) or after sharing the same island (sympatric). o Two species sharing an island creates pressure to minimize their competition for resources. In the process of character displacement, natural selection favors those individuals that can use resources not used by the other species. Thus the two species will diverge in their favored resources and habitat. Choose Answer • Given what you now know, what information and/or problemsolving approach is most likely to produce the correct answer? o The question states that the bird species are always more closely related to each other within an island than they are between any two islands. Which sequence of events may lead to this scenario? o If each bird species evolved different adaptations in allopatry and then colonized the other islands then the closest related species would be found on different islands. o If onebird species colonized each island and then evolved into distinct species on that island through character displacement then the closest related species would be found within an island. Version 1
46
Reflect on Process • Did your problem-solving process lead you to the correct answer? If not, where did the process break down or lead you astray? How can you revise your approach to produce a more desirable result? o If you got the right answer, three cheers! If not, did you recognize that if species on the same island are most closely related, they likely radiated from a progenitor on that island? 51) B
Version 1
47
Clarify Question • What is the key concept addressed by the question? o This question addresses sympatric and allopatric speciation. • What type of thinking is required? o This question is asking you to weigh and judge evidence, or evaluate, to choose the best of the possible answers. Gather Content • What do you already know about sympatric and allopatric speciation? o Species may evolve different adaptations while separated (allopatric) or living in the same geographic region(sympatric). o In this question, theIsthmus of Panama is a clear geographic separationthat separates the two oceans. However, each Caribbean species is actually more closely related to a sister species across the isthmus. Which sequence of evolutionary events may have led to this scenario? Choose Answer • Given what you now know, what information and/or problemsolving approach is most likely to produce the correct answer? o Highrelatedness suggests that the Caribbean and Pacific species were part of the same populations at one time. o If the snapping shrimp species in each ocean diverged after the formation of the Isthmusthen the species in each ocean would be most closely related to each other. o Extra info: The species in the same ocean diverged many million yearsago, but the species across the isthmus only diverged 3 million years ago when it formed. Version 1
48
Reflect on Process • Did your problem-solving process lead you to the correct answer? If not, where did the process break down or lead you astray? How can you revise your approach to produce a more desirable result? o If you go the right answer, good work! If not, did you recognize that the relatedness of different species can allow us to interpret when and how their speciation occurred? 52) A
Version 1
49
Clarify Question • What is the key concept addressed by the question? o This question addresses speciation through polyploidy. • What type of thinking is required? o This question is asking you to take what you already know and apply it to this unfamiliar situation. Gather Content • What do you already know about speciation through polyploidy? o Instantaneous sympatric speciation occurs when an individual is born that is reproductively isolated from all other members of its species. This often occurs in plants due to their tolerance of polyploidy. o In autopolyploidy, all of the chromosomes may arise from a single species (“self” polyploidy). oIn allopolyploidythe chromosomes arisewhen two species hybridize (“other” polyploidy). Choose Answer • Given what you now know, what information and/or problemsolving approach is most likely to produce the correct answer? o In this example, two different species contributed DNA to the new species, so it is an example of allopolyploidy. Reflect on Process • Did your problem-solving process lead you to the correct answer? If not, where did the process break down or lead you astray? How can you revise your approach to produce a more desirable result? o If you got the right answer, good on you! If not, did you recognize that when two species hybridize and create a polyploid offspring, this is Version 1
50
an example of instantaneous speciation by allopolyploidy? 53) A
Version 1
51
Clarify Question • What is the key concept addressed by the question? o This question addresses reproductive isolation mechanisms. • What type of thinking is required? o This question is asking you to take what you already know and apply it to this unfamiliar situation. Gather Content • What do you already know about reproductive isolation? o Reproductive isolating mechanisms prevent genetic exchange between species. Some prevent the formation of zygotes—prezygotic isolating mechanisms. Others prevent the proper functioning of zygotes after they form—postzygotic isolating mechanisms. o Prezygotic isolating mechanisms include ecological isolation, behavioral isolation, temporal isolation, mechanical isolation, and prevention of gamete fusion. o Postzygotic isolation occurs when hybrids between species are either inviable (poor at surviving) or infertile (unable to reproduce themselves). Choose Answer • Given what you now know, what information and/or problemsolving approach is most likely to produce the correct answer? o The two bird species have different mating calls. This is a type of behavioral isolation. o The two species occupy different habitats in nature. This is ecological isolation. o When the two species hybridize, the hybrids are infertile. This is hybrid sterility. Version 1
52
Reflect on Process • Did your problem-solving process lead you to the correct answer? If not, where did the process break down or lead you astray? How can you revise your approach to produce a more desirable result? o If you got the right answer, good job! If not, did you recognize that different isolating mechanisms can be acting at the same time? 54) E
Version 1
53
Clarify Question • What is the key concept addressed by the question? o This question addresses the biological species concept. • What type of thinking is required? o This question is asking you to take what you already know and apply it to this unfamiliar situation. Gather Content • What do you already know about speciation? o The biological species concept says that a species is composed of natural populations whose members mate with each other and produce fertile offspring. Conversely, natural populations whose members do not mate with each other or who cannot produce fertile offspring are said to be reproductively-isolated and, therefore, are members of different species. Choose Answer • Given what you now know, what information and/or problemsolving approach is most likely to produce the correct answer? o Which option directly addresses thebiological species concept ofreproductive isolation? Reflect on Process • Did your problem-solving process lead you to the correct answer? If not, where did the process break down or lead you astray? How can you revise your approach to produce a more desirable result? o If you got the right answer, good work! If not, did you recognize that the #1 essential criterion that defines different species is that they are reproductively-isolated? Version 1
54
55) C
Version 1
55
Clarify Question • What is the key concept addressed by the question? o This question addresses reproductive isolation. • What type of thinking is required? o This question is asking you to take what you already know and apply it to this unfamiliar situation. Gather Content • What do you already know about reproductive isolation? o Reproductive-isolating mechanisms prevent genetic exchange between species. Some prevent the formation of zygotes: prezygoticisolating mechanisms. Others prevent the proper functioning of zygotes after they form: postzygotic-isolating mechanisms. o Prezygotic-isolating mechanisms include ecological isolation, behavioral isolation, temporal isolation, mechanical isolation, and prevention of gamete fusion. o Postzygotic-isolation occurs when hybrids between species are either inviable (poor at surviving) or infertile (unable to reproduce themselves). Choose Answer • Given what you now know, what information and/or problemsolving approach is most likely to produce the correct answer? o In this example of the different sea urchin species, the eggs and sperm cannot fuse due to incompatible chemical signals. o This prevents a zygote (embryo) from even forming, so it is a prezygotic-isolating mechanism. The incompatibility prevents gamete fusion (fusion of egg and sperm).
Version 1
56
Reflect on Process • Did your problem-solving process lead you to the correct answer? If not, where did the process break down or lead you astray? How can you revise your approach to produce a more desirable result? o If you got the right answer, good job! If not, did you recognize that even eggs and sperm have to be compatible to allow mating to take place? 56) C
Version 1
57
Clarify Question • What is the key concept addressed by the question? o This question addresses speciation through polyploidy. • What type of thinking is required? o This question is asking you to take what you already know and apply it to this unfamiliar situation. Gather Content • What do you already know about speciation through polyploidy? o Allopolyploidy is a type of sympatric speciation, whichdoesn’t require geographical isolation. Choose Answer • Given what you now know, what information and/or problemsolving approach is most likely to produce the correct answer? o Because the new polyploid offspring is reproductively isolated from the parental species, we can say that this speciation occurs almost instantaneously. Reflect on Process • Did your problem-solving process lead you to the correct answer? If not, where did the process break down or lead you astray? How can you revise your approach to produce a more desirable result? o If you got the right answer, you know your stuff! If not, did you recognize that allopolyploidy allows speciation to occur almost instantly? 57) B
Version 1
58
Clarify Question • What is the key concept addressed by the question? o This question addresses reproductive-isolating mechanisms. • What type of thinking is required? o This question is asking you to take what you already know and apply it to this unfamiliar situation. Gather Content • What do you already know about reproductive isolation? o Reproductive-isolating mechanisms prevent genetic exchange between species. Some prevent the formation of zygotes: prezygoticisolating mechanisms. Others prevent the proper functioning of zygotes after they form: postzygotic-isolating mechanisms. o Prezygotic-isolating mechanisms include ecological isolation, behavioral isolation, temporal isolation, mechanical isolation, and prevention of gamete fusion. o Postzygotic-isolation occurs when hybrids between species are either inviable (poor at surviving) or infertile (unable to reproduce themselves). Choose Answer • Given what you now know, what information and/or problemsolving approach is most likely to produce the correct answer? o In this example, the bird songs cannot attract the females of a different species such thatno mating occurs. Thus, a zygote does not form. o Since it is based on mating behavior, this is a behavioral isolation mechanism. Reflect on Process Version 1
59
• Did your problem-solving process lead you to the correct answer? If not, where did the process break down or lead you astray? How can you revise your approach to produce a more desirable result? o If you got the right answer, well done! If not, did you recognize that when mating behavior is incompatible between different species, this is a prezygotic, behavioral isolation mechanism? 58) B
Version 1
60
Clarify Question • What is the key concept addressed by the question? o This question addresses character displacement. • What type of thinking is required? o This question is asking you to analyze the information given, using logic, to dissect the problem and determine the answer. Gather Content • What do you already know about character displacement? o Two species sharing a habitat have pressure to minimize their competition for resources. In the process of character displacement, natural selection favors those individuals that can use resources not used by the other species. Thus, the two species will diverge in their favored resources and habitat. This may be reflected in morphological features that allow them to take advantage of different resources. Choose Answer • Given what you now know, what information and/or problemsolving approach is most likely to produce the correct answer? o Since the bird species are likely to compete for resources, there will be pressure for them to adapt via character displacement. o The beak sizes should be more similar when they are allopatric (not overlapping in the same area) than when they are sympatric (present in the same area). Reflect on Process • Did your problem-solving process lead you to the correct answer? If not, where did the process break down or lead you astray? How can you revise your approach to produce a more desirable result? Version 1
61
o If you got the right answer, good job! If not, did you recognize that when species do not overlap or compete there may be no character displacement? 59) B
Version 1
62
Clarify Question • What is the key concept addressed by the question? o This question addresses natural selection and speciation. • What type of thinking is required? o This question is asking you to analyze the information given, using logic, to dissect the problem and determine the answer. Gather Content • What do you already know about natural selection and speciation? o As populations of a species adapt to different circumstancesthey likely accumulate many differences that may lead to reproductive isolation and eventually speciation. Choose Answer • Given what you now know, what information and/or problemsolving approach is most likely to produce the correct answer? o The random occurrence of a mutation in DNA may produce variation in phenotype, which is what selection can act upon. o The new phenotype may lead to reproductive isolation by a variety of mechanisms. And reproductive isolation is the necessary prerequisite for speciation. o The full sequence is: mutation --> variation --> selection --> reproductive Isolation --> speciation. Reflect on Process • Did your problem-solving process lead you to the correct answer? If not, where did the process break down or lead you astray? How can you revise your approach to produce a more desirable result? Version 1
63
o If you got the right answer, good work! If not, did you recognize that mutations are sourcesof variation that selection can actupon?Did you also recognize that reproductive isolation leads to speciation? 60) A
Version 1
64
Clarify Question • What is the key concept addressed by the question? o This question addresses temporal isolation. • What type of thinking is required? o This question is asking you to take what you already know and apply it to this unfamiliar situation. Gather Content • What do you already know about temporal isolation? o Populations are considered to be different species when the members are reproductively isolated from each other. o Temporal (relating to time) isolation is when species reproduce in different seasons. Choose Answer • Given what you now know, what information and/or problemsolving approach is most likely to produce the correct answer? o Plant species that bloom during different months is anexample in which the plants are isolated by time. Reflect on Process • Did your problem-solving process lead you to the correct answer? If not, where did the process break down or lead you astray? How can you revise your approach to produce a more desirable result? o If you got the right answer, bravo! If not, did you recognize that plants blooming during different months is an example of temporal isolation? 61) A
Version 1
65
Clarify Question • What is the key concept addressed by the question? o This question addresses genetic drift. • What type of thinking is required? o This question is asking you to take what you already know and apply it to this unfamiliar situation. Gather Content • What do you already know about genetic drift? o Random changes may cause reproductive isolation. o Given enough timeany two isolated populations will diverge because of genetic drift. o In some cases, this random divergence may affect traits responsible for reproductive isolation, and speciation may occur. Choose Answer • Given what you now know, what information and/or problemsolving approach is most likely to produce the correct answer? o In this hypothetical example the astronaut dogs are geographically separate from Earth dogsthus they are allopatric populations. o The incompatibility is likely due to random genetic changes from genetic drift. Reflect on Process • Did your problem-solving process lead you to the correct answer? If not, where did the process break down or lead you astray? How can you revise your approach to produce a more desirable result? o If you got the right answer, huzzah!If notdid you recognize that a Version 1
66
small distant population is likely to become reproductively isolated by allopatric genetic drift? 62) C
Version 1
67
Clarify Question • What is the key concept addressed by the question? o This question addresses adaptive radiation. • What type of thinking is required? o This question is asking you to take what you already know and apply it to this unfamiliar situation. Gather Content • What do you already know about adaptive radiation? o Adaptive radiations are particularly common in situations in which a species occurs in an environment with few other species and many available resources. o Adaptive radiation can result when a new trait, called a key innovation, evolves within a species allowing it to use resources or other aspects of the environment that were previously inaccessible. Choose Answer • Given what you now know, what information and/or problemsolving approach is most likely to produce the correct answer? o The evolution of endothermic homeostasis by the early mammals is an example of a key innovation. Reflect on Process • Did your problem-solving process lead you to the correct answer? If not, where did the process break down or lead you astray? How can you revise your approach to produce a more desirable result? o If you got the right answer, good job!If not, did you recognize that sometimes adaptive radiations are promoted by the evolution of a key Version 1
68
innovation—a major evolutionary advance? 63) D Clarify Question • What is the key concept addressed by the question? o Mechanisms that lead to speciation. • What type of thinking is required? o The question requires you to understanda situation that leads to speciation in lizards. Gather Content/Choose Answer • What do you already know about speciation? What other information is related to the question? o Postzygotic-isolation occurs when hybrids between species are either inviable (poor at surviving) or infertile (unable to reproduce themselves). o In small populations, genetic drift may cause populations to diverge. o If hybrids areLESSfit than the parents then selection would improve prezygotic isolating mechanisms andreinforcement is likely to occur. o Natural selection can select for traits that increase reproductive isolation Reflect on Process • Did your problem-solving process lead you to the correct answer? If not, where did the process break down or lead you astray? How can you revise your approach to produce a more desirable result? o If you figured out the correct answer, gold star! If not, did you realize that sexual selected traitsmayincrease reproductive isolation? 64) punctuated equilibrium 65) biological Version 1
69
66) reinforced Incomplete isolating mechanisms are reinforced by natural selection until they are completely effective. 67) adaptive radiation 68) character displacement
Version 1
70
CHAPTER 22 CHECK ALL THE APPLY. Choose all options that best completes the statement or answers the question. 1) A phylogenetically based taxonomy is important not only in creating a logical way to name organisms, but also in learning about ________ of organisms using information in related species. (Check all that apply.) A) physiology B) behavior C) development D) morphology
2)
Select all of the following that can result in homoplasies. (Check all that apply.) A) Convergentevolution B) Evolutionaryreversals C) Divergentevolution D) Sharedancestry
3) Select the true statements describing why cladistics may not always be the best method for reconstructing phylogenies based on data from the DNA genome. (Check all that apply.) A) The rate of evolution of the DNA genome is rapid. B) The assumptions of the principle of parsimony are not always met. C) Homoplasy often dominates the data sets. D) The number of character states for DNA is large.
4)
Choose the true statements about molecular clocks. (Check all that apply.)
Version 1
1
A) All molecular clocks tick at the same rate. B) Molecular clocks can be calibrated using fossil evidence. C) The rate of a molecular clock for a given DNA character might vary depending on how constrained the character is by natural selection. D) Ultimately, all molecular clocks depend on the rate of DNA mutation and DNA repair mechanisms.
5) There are many features that distinguish bacteria and archaea. Which of the following is/are characteristic of bacteria but not archaea? (Check all that apply.) A) Protein synthesis begins with formyl-methionine B) Presence of membrane bound organelles C) Growth inhibited by streptomycin and chloramphenicol D) Peptidoglycan in cell wall E) Nuclear envelope
6)
Which of the following are properties of eukaryotes? (Check all that apply.) A) Compartmentalization B) Multicellularity C) Peptidoglycan in cell wall D) Sexual reproduction
7) What are some reasons phenotypic similarity may not accurately predict evolutionary relationships? (Check all that apply.) A) It's not necessarily a good predictor of how long it has been since two species shared a common ancestor B) Two species that are not closely related may end up more similar to each other than they are to their close relatives due to convergent evolution C) Evolutionary change is not constant in rate and direction
Version 1
2
MULTIPLE CHOICE - Choose the one alternative that best completes the statement or answers the question. 8) Characteristics between the branch points of a cladogram that are shared by all organisms above the branch point and are not present in any below it are called __________. A) homologous characters B) homoplastic characters C) ancestral characters D) derived characters E) novel characters
9) Characteristics that have arisen in organisms as a result of common evolutionary descent are said to be ______________ characteristics. A) homologous B) homoplastic C) adaptive D) derived E) ancestral
10) One day after a biology class four of your friends argue about the difference between phylogeny and systematics. Which friend is right? A) Friend A states that systematics and phylogenies are really the same, one is more recent than the other, but basically they are the same. B) Friend B says that systematics is the same as cladistics and cladistics is reconstructing clades, which ultimately lead to the development of phylogenies. C) Friend C argues that systematics is the actual collecting and cataloguing of specimens into museums that can be used later by scientists to construct clades and phylogenies. D) Friend D says that the way she remembers is that systematics is the reconstruction and study of phylogenies.
Version 1
3
11) The evolutionary sequence in the development of a complex character can be best analyzed through __________. A) homoplasies B) phylogenetics C) taxonomy D) classification E) synapomorphies
12)
The study and reconstruction of phylogenies is __________. A) evolution B) systematics C) taxonomy D) taxidermy E) cladistics
13)
Derived character states shared by clade members are called __________. A) ancestral traits B) homoplasies C) synapomorphies D) plesiomorphies E) symplesiomorphies
14)
Character states shared by the ancestor or outgroup of a clade are called __________. A) derived traits B) homoplasies C) synapomorphies D) plesiomorphies E) symplesiomorphies
Version 1
4
15)
A group is considered monophyletic if __________. A) all members of the group share a common ancestor that is included in the group B) not all descendants of the common ancestor are included C) all members share homoplastic traits D) the group does not contain the most recent common ancestor E) it is the most parsimonious grouping
16)
A group is considered paraphyletic if __________. A) all members of the group share a common ancestor B) not all descendants of the common ancestor are included C) the common ancestor of the group is not included in the group D) the group does not contain the most recent common ancestor E) it is the most parsimonious grouping
17)
A group is considered polyphyletic if __________. A) all members of the group share a common ancestor B) not all descendants of the common ancestor are included C) all members share homoplastic traits D) the group does not contain the most recent common ancestor E) it is the most parsimonious grouping
18) The phylogenetic species concept (PSC) declares that a group is a species if it __________.
Version 1
5
A) is reproductively isolated from other species B) has evolved one or more of its own derived characters C) looks different than other species D) is allopatric E) has no synapomorphies
19)
Parental care in dinosaurs, crocodiles, and birds is an example of __________. A) homoplasy caused by convergence B) homoplasy caused by common descent C) homoplasy caused by evolutionary reversal D) homology caused by convergence E) homology caused by common descent
20) The evolution of conducting tubes (sieve tubes) in land plants and aquatic brown algae is an example of __________. A) homoplasy caused by convergence B) homoplasy caused by common descent C) homoplasy caused by evolutionary reversal D) homology caused by convergence E) homology caused by common descent
21) The evolution of saber teeth in a both extinct carnivorous mammals and marsupials is an example of __________. A) homoplasy caused by convergence B) homoplasy caused by common descent C) homoplasy caused by evolutionary reversal D) homology caused by convergence E) homology caused by common descent
Version 1
6
22)
Identify the polyphyletic group. A) Protists B) Chloroplasts C) Eukarya D) Archaea E) Bacteria
23) What can you conclude from the fact that DNA analysis of all chloroplasts reveals that they aremonophyletic? A) Endosymbiosis of the same species ofcyanobacteria occurred multiple times in cellsthat evolved into red, green, and brown algae. B) Cyanobacteria reproduced by sexual reproduction after being engulfed by brown algae. C) All eukaryotic cells engulfed cyanobacteriaat some point in their evolutionary history. D) Aerobic bacteria were engulfed by eukaryotic cells, forming plant cells.
24)
Most infectious diseases in humans are caused by __________. A) bacteria B) archaea C) plantae D) fungi
25) Choose the true statement about the morphological data in the chart shown below. The "1" in the box means the trait is present and the "0" indicates that the trait is absent. (Note: all of the characteristics across the top of the columns are considered "TRAITS" for purposes of reading the chart.) Traits Organism
Version 1
Jaws
Lungs
Amniotic
Hair
No tail
Bipedal
7
membrane Lamprey
0
0
0
0
0
0
Shark
1
0
0
0
0
0
Salamander
1
1
0
0
0
0
Lizard
1
1
1
0
0
0
Tiger
1
1
1
1
0
0
Gorilla
1
1
1
1
1
0
Human
1
1
1
1
1
1
A) All organisms in this chart share all of the derived characteristics. B) Only the gorilla and humans share all of the derived characteristics. C) The lamprey is the only outgroup since it has none of the derived characteristics. D) The salamander and the tiger together are the outgroup because they only share two of the derived characteristics (jaws and lungs). E) The shark is the outgroup since it only has one of the derived characteristics (jaws).
26) The following are steps used to construct a cladogram that best reflects the evolutionary relationships of a group of species. Which step involves deciding which tree is most likely to have occurred, based on the fewest character state changes? A) Polarize the characters B) Establish the character states C) Gather data on characters to be used D) Apply the principle of parsimony E) Select an outgroup
27) Birds, snakes, lizards, turtles, and crocodiles are all thought to share a common ancestor and several homologous traits. Assuming that this is true, these groups of animals and their common ancestor would represent __________. A) a polyphyletic group B) a monophyletic group C) homoplastic convergence D) an outgroup E) a species cluster
Version 1
8
28) Evidence suggests that the crocodiles are more closely related to the birds than the turtles and snakes. If so, then including crocodiles, but not birds, in the Class Reptilia would make the Class Reptilia __________. A) a monophyletic group B) a paraphyletic group C) a polyphyletic group D) parsimonious E) not homologous
FILL IN THE BLANK. Write the word or phrase that best completes each statement or answers the question. 29) The biological specialty that deals with the grouping and naming of organisms is called taxonomy or _________.
30) The biological specialty that deals with the reconstruction and study of phylogenies is called _________.
31) The behavior of parental care exhibited by dinosaurs is thought to be similar to this behavior in crocodiles and birds because phylogeny shows that these three groups share a _________.
32) Saber teeth found in different groups of extinct carnivorous mammals are considered ___________ structures, as the fossil record shows that saber teeth evolved independently among the groups.
33) Systematists often use DNA sequencing in ______________, the study of the order of evolutionary events within a group sharing derived characters.
Version 1
9
34) A ______________ group consists of the most recent common ancestor and all of its descendants.
35) A ______________ group consists of the most recent common ancestor and some of its descendants.
36)
A ______________ group does not contain the most recent common ancestor.
37) The principle of ______________ favors the hypothesis that requires the fewest assumptions.
38) A derived character that is shared by all members of a clade is called a ______________ of that clade.
39)
_________ is the science of classifying living things.
SECTION BREAK. Answer all the part questions.
40)
Version 1
10
40.1) Based on the phylogeny shown, we can conclude that species 2 is most closely related to which species? A) 1 B) 3 C) 4 D) 5 E) 1 or 3 (you can't tell)
40.2)
Based on the phylogeny shown, the outgroup would be which species?
A) 1 B) 2 C) 3 D) 4 E) 5
40.3) Based on the phylogeny shown, the group of species including (2–5) can best be described as a __________. A) clade B) paraphyletic group C) polyphyletic group D) cladogram E) synapomorphy
Version 1
11
41)
41.1) Consider the cladogram shown. Based on this cladogram, which species has the greatest number of ancestral character states (pleisiomorphies)? A) 1 B) 2 C) 3 D) 4 E) 5 F) 6
41.2) Consider the cladogram shown. Which pair of species shares the greatest number of derived characters (synapomorphies)? A) 1 and 3 B) 6 and 4 C) 6 and 5 D) 2 and 3 E) 6 and 3
42) Trait Species
Version 1
A
B
C
D
E
12
1
1
1
0
1
1
2
1
1
0
1
0
3
1
1
1
0
0
4
1
0
0
0
0
5
1
1
1
0
0
6
1
1
0
1
1
42.1) The table shows the distribution of traits (A–E) in six extant species (1–6). A "0" indicates the ancestral condition, while "1" indicates the derived condition. Based on this data, which species would be considered the outgroup? A) 1 B) 2 C) 3 D) 4 E) 5
42.2) The table shows the distribution of traits (A–E) in six extant species (1–6). A "0" indicates the ancestral condition, while "1" indicates the derived condition. Which trait is least informative of phylogenetic relationships within the group? A) A B) B C) C D) D E) E
43) Version 1
13
43.1) Consider the cladogram shown. Which pair of species shares the greatest number of derived characters (synapomorphies)? A) 4 and 2 B) 5 and 4 C) 6 and 1 D) 6 and 2 E) 5 and 2
43.2) Consider the cladogram shown. Based on this cladogram, which species has the greatest number of ancestral character states? A) 1 B) 2 C) 3 D) 4 E) 5
44) Trait Species
A
B
C
D
E
1
1
0
0
0
0
2
1
1
1
1
1
3
1
1
1
1
1
4
1
1
1
1
0
5
1
1
1
0
0
6
1
1
0
0
0
44.1) The table shows the distribution of traits (A–E) in six extant species (1–6). A "0" indicates the ancestral condition, while a "1" indicates the derived condition. Which trait is represented by the greatest number of plesiomorphies? Version 1
14
A) A B) B C) C D) D E) E
44.2) The table shows the distribution of traits (A–E) in six extant species (1–6). A "0" indicates the ancestral condition, while a "1" indicates the derived condition. Based on this data, which pair of species diverged most recently? A) 1 and 4 B) 2 and 3 C) 2 and 4 D) 6 and 4 E) 6 and 5
45)
45.1) Consider the cladogram shown. Which pair of species shares the greatest number of derived characters (synapomorphies)?
Version 1
15
A) 1 and 4 B) 2 and 3 C) 2 and 4 D) 6 and 4 E) 6 and 5
45.2) Consider the cladogram shown. If we designate species 1 as the outgroup, which species has the greatest number of ancestral character states (pleisiomorphies)? A) 2 B) 3 C) 4 D) 5 E) 6
46) Trait Species
A
B
C
D
E
1
0
1
1
1
1
2
0
0
1
1
1
3
1
0
1
0
1
4
0
1
1
1
1
5
0
0
0
0
1
6
1
0
1
0
1
46.1) The table shows the distribution of traits (A–E) in six extant species (1–6). A "0" indicates the ancestral condition, and a "1" indicates the derived condition. Which trait is least informative of phylogenetic relationships within the group?
Version 1
16
A) A B) B C) C D) D E) E
46.2) The table shows the distribution of traits (A–E) in six extant species (1–6). A "0" indicates the ancestral condition, and a "1" indicates the derived condition. Which species would you designate as the outgroup? A) 1 B) 4 C) 2 D) 3 E) 5
46.3) The table shows the distribution of traits (A–E) in six extant species (1–6). A "0" indicates the ancestral condition, and a "1" indicates the derived condition. Which pair of species shares the greatest number of derived characters (synapomorphies)? A) 1 and 4 B) 6 and 3 C) 6 and 1 D) 5 and 6 E) 5 and 1
46.4) The table shows the distribution of traits (A–E) in six extant species (1–6). A "0" indicates the ancestral condition, and a "1" indicates the derived condition. Which species has the greatest number of ancestral character states (pleisiomorpohies)?
Version 1
17
A) 1 B) 2 C) 3 D) 4 E) 5
47) Trait Species
A
B
C
D
E
1
1
1
0
1
1
2
1
0
0
1
0
3
1
0
0
1
1
4
1
1
1
1
1
5
1
1
1
1
1
6
0
0
0
1
0
47.1) The table shows the distribution of traits (A–E) in six extant species (1–6). A "0" indicates the ancestral condition, and a "1" indicates the derived condition. Which trait is least informative of phylogenetic relationships within the group? A) A B) B C) C D) D E) E
47.2) The table shows the distribution of traits (A–E) in six extant species (1–6). A "0" indicates the ancestral condition, and a "1" indicates the derived condition. Which species would you designate as the outgroup?
Version 1
18
A) 1 B) 2 C) 3 D) 4 E) 5 F) 6
47.3) The table shows the distribution of traits (A–E) in six extant species (1–6). A "0" indicates the ancestral condition, and a "1" indicates the derived condition. Which pair of species shares the greatest number of derived characters (synapomorphies)? A) 4 and 5 B) 2 and 4 C) 1 and 4 D) 3 and 4 E) 2 and 5
47.4) The table shows the distribution of traits (A–E) in six extant species (1–6). A "0" indicates the ancestral condition, and a "1" indicates the derived condition. Which species has the greatest number of ancestral character states (pleisiomorphies)? A) 1 B) 2 C) 3 D) 4 E) 5 F) 6
48) Trait Species
A
B
C
D
E
1
0
0
1
0
0
2
1
0
1
0
1
Version 1
19
3
1
0
1
0
1
4
1
1
1
0
0
5
1
1
1
1
0
6
1
1
1
1
0
48.1) The table shows the distribution of traits (A–E) in six extant species (1–6). A "0" indicates the ancestral condition, and a "1" indicates the derived condition. Which trait is least informative of phylogenetic relationships within the group? A) A B) B C) C D) D E) E
48.2) The table shows the distribution of traits (A–E) in six extant species (1–6). A "0" indicates the ancestral condition, and a "1" indicates the derived condition. Which pair of species shares the fewest number of derived characters (synapomorphies)? A) 1 and 6 B) 1 and 4 C) 4 and 6 D) 2 and 3 E) 6 and 3
Version 1
20
49)
49.1)
Based on the phylogeny shown, which species is not a tree but has seeds?
A) Psaronius B) Ginkgo C) Fern D) Rose E) Oak F) Pine
49.2)
Based on the phylogeny shown, which characteristic is homoplastic?
A) Tree B) Needles C) Flowers D) Seeds
49.3) Assume that the ancestor of this phylogeny had vascular tissue and chlorophyll. If this is true, which answer lists all key traits of a ginkgo?
Version 1
21
A) Vascular tissue, needles, tree, and chlorophyll B) Tree and seeds C) Seeds, tree, vascular tissue, and chlorophyll D) Needles, tree, and seeds
49.4) Based on the phylogeny shown, indicate the groups that are monophyletic (Select all that apply). A) Rose and oak B) Pine, ginkgo, oak, and rose C) Fern, pine, and ginkgo D) Psaronius, fern, pine, ginkgo, oak, and rose
49.5) Based on the phylogeny shown, choose the true statement about evolutionary relationships. A) Ginkgo is more closely related to oak than rose. B) Oak is equally related to rose and pine. C) Fern is more closely related to pine than to rose. D) Fern is equally related to ginkgo and oak.
50)
Version 1
22
50.1) Based on the phylogeny shown, choose the true statements about evolutionary relationships. (Check all that apply.) A) A snake is more closely related to a newt than to a mouse. B) A newt is equally related to a snake and a chimp. C) A mouse is more closely related to a chimp than to a snake. D) A salamander is more closely related to a mouse than to a chimp.
50.2) Indicate the phylogenies that show the same relationship as the phylogeny shown here.(Check all that apply.)
Version 1
23
A)
B)
C)
Version 1
24
D)
51)
51.1)
A frog should be added to the phylogeny above at which point?
A) A B) B C) C D) D E) E
51.2) The common ancestor shared by mouse and gorilla is at point ______ on the phylogeny shown.
Version 1
25
A) A B) B C) C D) D E) E
Version 1
26
Answer Key Test name: Chap 22_3e 1) [A, B, C, D] 2) [A, B] 3) [A, B, C] 4) [B, C, D] 5) [A, C, D] Bacteria and archaea have many differences. 6) [A, B, D] Eukaryotes with cell walls lack peptidoglycan. 7) [A, B, C] 8) D 9) E 10) D 11) E 12) B 13) C 14) E 15) A 16) B 17) D 18) B 19) E 20) A These do not share a recent common ancestor. 21) A Marsupials and mammals do not have a recent common ancestor. Version 1
27
22) A Protists are very diverse. 23) A Chloroplasts arose from cyanobacteria, The same cyanobacteria were engulfed by multiple hosts that were ancestral to the red and green algae. Brown algae share the same ancestral chloroplast DNA but most likely gained it by engulfing red algae. <!--Markup Copied from Habitat--> 24) A Certain bacteria are responsible for many forms of disease. <!--Markup Copied from Habitat-->The archaea seem to have diverged very early from the bacteria and are more closely related to eukaryotes than to bacteria. <!--Markup Copied from Habitat--> <!--Markup Copied from Habitat--> 25) C
Version 1
28
Clarify Question • What is the key concept addressed by the question? o This question addresses outgroups and derived characters. • What type of thinking is required? o This question is asking you to take what you already know and apply it to this unfamiliar situation. Gather Content • What do you already know about outgroups and derived characters? o The method of outgroup comparison is used to assign character polarity. To use this method, a species related to, but not a member of, the group under study is designated as the outgroup. o When a group has multiple character states, and one of those states is seen in the outgroup, that state is considered to be ancestral and other states are derived. Choose Answer • Given what you now know, what information and/or problemsolving approach is most likely to produce the correct answer? o The lampreyshares none of the derived characteristics; itis the outgroup to the other organisms. Reflect on Process • Did your problem-solving process lead you to the correct answer? If not, where did the process break down or lead you astray? How can you revise your approach to produce a more desirable result? o If you got the right answer, well done. If not, did you recognize that the lamprey is the outgroupbecause it lacked the derived characteristics? 26) D Version 1
29
Clarify Question • What is the key concept addressed by the question? o This question addresses cladograms. • What type of thinking is required? o This question is asking you to analyze the information given, using logic, to dissect the problem and determine the answer. Gather Content • What do you already know about cladograms? o A cladogram is a diagram that depicts an evolutionary tree. o A simple cladogram is a nested set of clades, each clade characterized by its own synapomorphies. Choose Answer • Given what you now know, what information and/or problemsolving approach is most likely to produce the correct answer? o The principle of parsimony states that the best hypothesis is the one that requires the fewest assumptions (divergences). o So the phylogeny that requires the fewest evolutionary events is considered the best hypothesis of phylogenetic relationships when using the principle of parsimony. Reflect on Process • Did your problem-solving process lead you to the correct answer? If not, where did the process break down or lead you astray? How can you revise your approach to produce a more desirable result? o If you got the right answer, well done. If not, did you recognize that when using the principle of parsimony the most likely tree is the one with the fewest character state changes? Version 1
30
27) B
Version 1
31
Clarify Question • What is the key concept addressed by the question? o This question addresses monophyly. • What type of thinking is required? o This question is asking you to take what you already know and apply it to this unfamiliar situation. Gather Content • What do you already know about common ancestry and homologous traits? o The common ancestor of a group is the last organism that gave rise to all the members of the group, i.e., the organism at the bottom node on the tree of the group. o Homologous traits are traits in different species that are the same, or very similar, because they arose from a common evolutionary origin. o A monophyletic group is a group that includes the most recent common ancestor of the group and all of its descendants. Choose Answer • Given what you now know, what information and/or problemsolving approach is most likely to produce the correct answer? o Could you call this group of organisms a monophyletic group? Yes. A monophyletic group includes the most recent common ancestor of the group and all of its descendants. Reflect on Process • Did your problem-solving process lead you to the correct answer? If not, where did the process break down or lead you astray? How can you revise your approach to produce a more desirable result? Version 1
32
o If you got the right answer, well done. If not, did you recognize that the group of organisms sharing traits, and including the common ancestor, constitutes a monophyletic group. 28) B
Version 1
33
Clarify Question • What is the key concept addressed by the question? o This question addresses monophyly. • What type of thinking is required? o This question is asking you to take what you already know and apply it to this unfamiliar situation. Gather Content • What do you already know about monophyly? o A monophyletic group is a group that includes the most recent common ancestor of the group and all of its descendants. o A polyphyletic group is a grouping of distant relatives that ismissing the common ancestor. o A paraphyletic group is a grouping of the most recent common ancestor of the group, but not all its descendants. Choose Answer • Given what you now know, what information and/or problemsolving approach is most likely to produce the correct answer? o In this case, would Class Reptilia be a paraphyletic group? Yes, that is the correct term for a group that is missing one of its members. Reflect on Process • Did your problem-solving process lead you to the correct answer? If not, where did the process break down or lead you astray? How can you revise your approach to produce a more desirable result? o If you got the right answer, well done. If not, did you recognize that when a member is missing from a grouping, that grouping is called paraphyletic? Version 1
34
29) classification 30) systematics 31) common ancestor 32) homoplastic 33) cladistics 34) monophyletic 35) paraphyletic 36) polyphyletic 37) parsimony 38) synapomorphy 39) Taxonomy 40) Section Break 40.1) B
Version 1
35
Clarify Question • What is the key concept addressed by the question? o This question addresses phylogenetic trees. • What type of thinking is required? o This question is asking you to take what you already know and apply it to this unfamiliar situation. Gather Content • What do you already know about phylogenetic trees? o A phylogenetic tree is a way of diagramming a hypothesis about evolution. o The tips of the branches represent existing species. As one works down the tree, the joining of twigs and branches reflects the pattern of common ancestry. Choose Answer • Given what you now know, what information and/or problemsolving approach is most likely to produce the correct answer? o When a V-style tree is drawn with the Vs upwards, the highest nodes are the most recently diverged species, and the lower nodes are older divergences between groups. o Since the groups with the highest node are most closely related, the closest relative of species 2 is species 3. Reflect on Process • Did your problem-solving process lead you to the correct answer? If not, where did the process break down or lead you astray? How can you revise your approach to produce a more Version 1
36
desirable result? o If you got the right answer, well done. If not, did you recognize that the most closely related species are drawn with nodes highest in the chart? 40.2) A
Version 1
37
Clarify Question • What is the key concept addressed by the question? o This question addresses phylogenetic trees. • What type of thinking is required? o This question is asking you to take what you already know and apply it to this unfamiliar situation. Gather Content • What do you already know about phylogenetic trees? o A phylogenetic tree is a way of diagramming a hypothesis about evolution. o The tips of the twigs represent existing species. As one works down the tree, the joining of twigs and branches reflects the pattern of common ancestry. Choose Answer • Given what you now know, what information and/or problemsolving approach is most likely to produce the correct answer? o An outgroup is a distantly related species or group of species, relative to the group of species being focused on. The outgroup has ancestral characters, not the derived characters of the group of species in question. o So the outgroup here would be the oldest, lowest node: group 1. Reflect on Process • Did your problem-solving process lead you to the correct answer? If not, where did the process break down or lead you astray? How can you revise your approach to produce a more desirable result? o If you got the right answer, well done. If not, did you recognize Version 1
38
that the outgroup would be the most distantly related to the other species with the lowest node? 40.3) A
Version 1
39
Clarify Question • What is the key concept addressed by the question? o This question addresses phylogeny. • What type of thinking is required? o This question is asking you to take what you already know and apply it to this unfamiliar situation. Gather Content • What do you already know about phylogenetic trees? o A phylogenetic tree is a way of diagramming a hypothesis about evolution. o The tips of the twigs represent existing species. As one works down the tree, the joining of twigs and branches reflects the pattern of common ancestry. Choose Answer • Given what you now know, what information and/or problemsolving approach is most likely to produce the correct answer? o A monophyletic group includes the most recent common ancestor of the group and all of its descendants. By definition, a clade is a monophyletic group. o Can you describe species 2–5 as a clade? Yes. Reflect on Process • Did your problem-solving process lead you to the correct answer? If not, where did the process break down or lead you astray? How can you revise your approach to produce a more desirable result? Version 1
40
o If you got the right answer, well done. If not, did you recognize that a clade is a group containing all descendants of a common ancestor? 41) Section Break 41.1) A
Version 1
41
Clarify Question • What is the key concept addressed by the question? o This question addresses cladistics. • What type of thinking is required? o This question is asking you to take what you already know and apply it to this unfamiliar situation. Gather Content • What do you already know about character states? o Character states exhibited by the outgroup are assumed to be ancestral, and other character states are considered derived. Choose Answer • Given what you now know, what information and/or problemsolving approach is most likely to produce the correct answer? o The outgroup would be the most distantly related to the other species with the lowest node. Reflect on Process • Did your problem-solving process lead you to the correct answer? If not, where did the process break down or lead you astray? How can you revise your approach to produce a more desirable result? o If you got the right answer, well done. If not, did you recognize that the outermost species in the cladogram is most distantly related to the others, and this “outgroup” has the most ancestral characters? 41.2) C
Version 1
42
Clarify Question • What is the key concept addressed by the question? o This question addresses cladistics. • What type of thinking is required? o This question is asking you to take what you already know and apply it to this unfamiliar situation. Gather Content • What do you already know about synapomorphies? o A synapomorphy is a shared derived character. In other words, a newly evolved trait shared by a group. Choose Answer • Given what you now know, what information and/or problemsolving approach is most likely to produce the correct answer? o Since the nested clades are defined by synapomorphies, the species in the innermost nested clade must share more synapomorphies than more distantly related pairs of species. o So species 6 and 5 share the most characters. Reflect on Process • Did your problem-solving process lead you to the correct answer? If not, where did the process break down or lead you astray? How can you revise your approach to produce a more desirable result? o If you got the right answer, well done. If not, did you recognize thatthe two most closely related species that are highest on the tree have the most shared derived characters? 42) Section Break
Version 1
43
42.1) D
Version 1
44
Clarify Question • What is the key concept addressed by the question? o This question addresses cladistics. • What type of thinking is required? o This question is asking you to analyze the information given, using logic, to dissect the problem and determine the answer. Gather Content • What do you already know about character states? o Character states exhibited by the outgroup are assumed to be ancestral, and other character states are considered derived. o To help construct a phylogeny, a trait must have derived characters shared between members of a group and the ancestral character in the outgroup. Choose Answer • Given what you now know, what information and/or problemsolving approach is most likely to produce the correct answer? o Since an outgroup has ancestral characters, the best outgroup here is species 4, which has ancestral characters for all the useful traits (traits A is shared by everybody and thus not useful). Reflect on Process • Did your problem-solving process lead you to the correct answer? If not, where did the process break down or lead you astray? How can you revise your approach to produce a more desirable result? o If you got the right answer, well done. If not, did you recognize that the outgroup is the species with the most ancestral characters?
Version 1
45
Did you try drawing the cladogram for this data? 42.2) A
Version 1
46
Clarify Question • What is the key concept addressed by the question? o This question addresses synapomorphies. • What type of thinking is required? o This question is asking you to analyze the information given, using logic, to dissect the problem and determine the answer. Gather Content • What do you already know about synapomorphies? o Only shared derived characters (synapomorphies) are useful for inferring phylogenies, and they must be contrasted to an outgroup that has the ancestral characters. Choose Answer • Given what you now know, what information and/or problemsolving approach is most likely to produce the correct answer? o Since trait A is the same (derived) in all the species listed here, that trait cannot help us to understand their relationships. Reflect on Process • Did your problem-solving process lead you to the correct answer? If not, where did the process break down or lead you astray? How can you revise your approach to produce a more desirable result? o If you got the right answer, well done. If not, did you recognize that a trait is only useful for building a cladogram if the trait varies (from ancestral to derived) in different species? Did you try drawing the cladogram for this data? 43) Section Break
Version 1
47
43.1) C
Version 1
48
Clarify Question • What is the key concept addressed by the question? o This question addresses cladistics. • What type of thinking is required? o This question is asking you to take what you already know and apply it to this unfamiliar situation. Gather Content • What do you already know about synapomorphies? o Only shared derived characters (synapomorphies) are useful for inferring phylogenies, and they must be contrasted to an outgroup that has the ancestral characters. Choose Answer • Given what you now know, what information and/or problemsolving approach is most likely to produce the correct answer? o Since the nested clades are defined by synapomorphies, the species in the innermost nested clade must share more synapomorphies than more distantly related pairs of species. o So species 6 and 1 share the most characters. Reflect on Process • Did your problem-solving process lead you to the correct answer? If not, where did the process break down or lead you astray? How can you revise your approach to produce a more desirable result? o If you got the right answer, well done. If not, did you recognize that a large number of shared characters indicates that two species are closely related? Version 1
49
43.2) D Clarify Question • What is the key concept addressed by the question? o This question addresses cladistics. • What type of thinking is required? o This question is asking you to take what you already know and apply it to this unfamiliar situation. Gather Content • What do you already know about ancestral characters? o The outermost species—the outgroup—must have the fewest synapomorphies and the most ancestral states (at least relative to the other species shown in this tree). Choose Answer • Given what you now know, what information and/or problemsolving approach is most likely to produce the correct answer? o The outermost species—the outgroup—must have the fewest synapomorphies and the most ancestral states (at least relative to the other species shown in this tree). Reflect on Process • Did your problem-solving process lead you to the correct answer? If not, where did the process break down or lead you astray? How can you revise your approach to produce a more desirable result? o If you got the right answer, well done. If not, did you recognize that the outermost species in the cladogram is most distantly related to the others, and this “outgroup” has the most ancestral characters? Version 1
50
44) Section Break 44.1) E
Version 1
51
Clarify Question • What is the key concept addressed by the question? o This question addresses plesiomorphies. • What type of thinking is required? o This question is asking you to analyze the information given, using logic, to dissect the problem and determine the answer. Gather Content • What do you already know about plesiomorphies? o The ancestral condition of a trait is the original condition; the derived condition is the more newly evolved version. o Plesiomorphies are ancestral characters. Choose Answer • Given what you now know, what information and/or problemsolving approach is most likely to produce the correct answer? o The question is asking which trait is most often in the ancestral condition in this data chart. The ancestral state is marked as “0” in this chart, so the trait represented by the most plesiomorphies is the correct answer. Reflect on Process • Did your problem-solving process lead you to the correct answer? If not, where did the process break down or lead you astray? How can you revise your approach to produce a more desirable result? o If you got the right answer, well done. If not, did you recognize that the trait with the most “0”s was the one with the most pleiomorphies? 44.2) B Version 1
52
Clarify Question • What is the key concept addressed by the question? o This question addresses cladistics. • What type of thinking is required? o This question is asking you to analyze the information given, using logic, to dissect the problem and determine the answer. Gather Content • What do you already know about cladistics? o The ancestral condition of a trait is the original condition; the derived condition is the more newly evolved version. Choose Answer • Given what you now know, what information and/or problemsolving approach is most likely to produce the correct answer? o If two species are not differentiated by a character, they must be closely related. Which two species share the most characters than other species? Reflect on Process • Did your problem-solving process lead you to the correct answer? If not, where did the process break down or lead you astray? How can you revise your approach to produce a more desirable result? o If you got the right answer, well done. If not, did you recognize that the species with the most shared derived traits are most closely related? 45) Section Break 45.1) B
Version 1
53
Clarify Question • What is the key concept addressed by the question? o This question addresses cladistics. • What type of thinking is required? o This question is asking you to take what you already know and apply it to this unfamiliar situation. Gather Content • What do you already know about cladistics? o A synapomorphy is a shared derived character. In other words, a newly-evolved trait shared by a group of species. Choose Answer • Given what you now know, what information and/or problemsolving approach is most likely to produce the correct answer? o Since the nested clades are defined by synapomorphies, the species in the innermost nested clade, the most closely related species in this cladogram, must share more synapomorphies than more distantly related pairs of species. Reflect on Process • Did your problem-solving process lead you to the correct answer? If not, where did the process break down or lead you astray? How can you revise your approach to produce a more desirable result? o If you got the right answer, well done. If not, did you recognize that a large number of shared characters indicates that two species are closely related? So the two species closest on the tree have the most shared derived characters. Did you remember that nodes are Version 1
54
established by the appearance of a new derived character on one branch? 45.2) E
Version 1
55
Clarify Question • What is the key concept addressed by the question? o This question addresses cladistics. • What type of thinking is required? o This question is asking you to take what you already know and apply it to this unfamiliar situation. Gather Content • What do you already know about cladistics? o The ancestral condition of a trait is the original condition; the derived condition is the more newly evolved version. o Plesiomorphies are ancestral characters. Choose Answer • Given what you now know, what information and/or problemsolving approach is most likely to produce the correct answer? o The outermost species—the outgroup—must have the fewest synapomorphies and the most ancestral states (at least relative to the other species shown in this tree). o So species 6 is the one (besides the outgroup, 1) with the most ancestral states. Reflect on Process • Did your problem-solving process lead you to the correct answer? If not, where did the process break down or lead you astray? How can you revise your approach to produce a more desirable result? o If you got the right answer, well done. If not, did you recognize that the outermost species in the cladogram is most distantly related to the others, and this group has the most ancestral characters? Version 1
56
46) Section Break 46.1) E
Version 1
57
Clarify Question • What is the key concept addressed by the question? o This question addresses cladistics. • What type of thinking is required? o This question is asking you to analyze the information given, using logic, to dissect the problem and determine the answer. Gather Content • What do you already know about cladistics? o Cladistics is the method used to construct a cladogram—a graphically represented hypothesis of evolutionary relationships. o Only shared derived characters (synapomorphies) are useful for inferring phylogenies, and they must be contrasted to an outgroup that has the ancestral characters. Choose Answer • Given what you now know, what information and/or problemsolving approach is most likely to produce the correct answer? o Since trait E is the same (derived) in all the species listed here, that trait cannot help us to understand their relationships. Reflect on Process • Did your problem-solving process lead you to the correct answer? If not, where did the process break down or lead you astray? How can you revise your approach to produce a more desirable result? o If you got the right answer, well done. If not, did you recognize that a trait is only useful for building a cladogram if the trait varies (from ancestral to derived) in different species? Did you try drawing
Version 1
58
the cladogram for this data? 46.2) E Clarify Question • What is the key concept addressed by the question? o This question addresses cladistics. • What type of thinking is required? o This question is asking you to take what you already know and apply it to this unfamiliar situation. Choose Answer • Given what you now know, what information and/or problemsolving approach is most likely to produce the correct answer? o The outgroup must be the species with the fewest synapomorphies and the most ancestral states (at least relative to the other species). o Which specieshas the most 0 traits? Reflect on Process • Did your problem-solving process lead you to the correct answer? If not, where did the process break down or lead you astray? How can you revise your approach to produce a more desirable result? o If you got the right answer, well done. If not, did you recognize that the outgroup has the most ancestral characters? Did you try drawing the cladogram for this data? 46.3) A
Version 1
59
Clarify Question • What is the key concept addressed by the question? o This question addresses cladistics. • What type of thinking is required? o This question is asking you to take what you already know and apply it to this unfamiliar situation. Gather Content • What do you already know about cladistics? o A synapomorphy is a shared derived character. In other words, a newly-evolved trait shared by a group of species. Choose Answer • Given what you now know, what information and/or problemsolving approach is most likely to produce the correct answer? o Which two species share the mostderived characters? Reflect on Process • Did your problem-solving process lead you to the correct answer? If not, where did the process break down or lead you astray? How can you revise your approach to produce a more desirable result? o If you got the right answer, well done. If not, did you recognize that the derived characters are marked with 1s in the chart? 46.4) E
Version 1
60
Clarify Question • What is the key concept addressed by the question? o This question addresses cladistics. • What type of thinking is required? o This question is asking you to take what you already know and apply it to this unfamiliar situation. Gather Content • What do you already know about cladistics? o The ancestral condition of a trait is the original condition; the derived condition is the more newly evolved version. o Plesiomorphies are ancestral characters. Choose Answer • Given what you now know, what information and/or problemsolving approach is most likely to produce the correct answer? o Which species has the most ancestral states? Reflect on Process • Did your problem-solving process lead you to the correct answer? If not, where did the process break down or lead you astray? How can you revise your approach to produce a more desirable result? o If you got the right answer, well done. If not, did you recognize that the ancestral characters are marked with 0s in the chart? 47) Section Break 47.1) D
Version 1
61
Clarify Question • What is the key concept addressed by the question? o This question addresses cladistics. • What type of thinking is required? o This question is asking you to analyze the information given, using logic, to dissect the problem and determine the answer. Gather Content • What do you already know about cladistics? o Cladistics is the method used to construct a cladogram—a graphically represented hypothesis of evolutionary relationships. o Only shared derived characters (synapomorphies) are useful for inferring phylogenies, and they must be contrasted to an outgroup that has the ancestral characters. Choose Answer • Given what you now know, what information and/or problemsolving approach is most likely to produce the correct answer? o Since trait D is the same (derived) in all the species listed here, that trait cannot help us to understand their relationships. Reflect on Process • Did your problem-solving process lead you to the correct answer? If not, where did the process break down or lead you astray? How can you revise your approach to produce a more desirable result? o If you got the right answer, well done. If not, did you recognize that a trait is only useful for building a cladogram if the trait varies (from ancestral to derived) in different species? Did you try drawing Version 1
62
the cladogram for this data? 47.2) F Clarify Question • What is the key concept addressed by the question? o This question addresses cladistics. • What type of thinking is required? o This question is asking you to take what you already know and apply it to this unfamiliar situation. Choose Answer • Given what you now know, what information and/or problemsolving approach is most likely to produce the correct answer? o The outgroup must be the species with the fewest synapomorphies and the most ancestral states (at least relative to the other species). o Which specieshas the most 0 traits? Reflect on Process • Did your problem-solving process lead you to the correct answer? If not, where did the process break down or lead you astray? How can you revise your approach to produce a more desirable result? o If you got the right answer, well done. If not, did you recognize that the outgroup has the most ancestral characters? Did you try drawing the cladogram for this data? 47.3) A
Version 1
63
Clarify Question • What is the key concept addressed by the question? o This question addresses cladistics. • What type of thinking is required? o This question is asking you to take what you already know and apply it to this unfamiliar situation. Gather Content • What do you already know about cladistics? o A synapomorphy is a shared derived character. In other words, a newly-evolved trait shared by a group of species. Choose Answer • Given what you now know, what information and/or problemsolving approach is most likely to produce the correct answer? o Which species share the most derived characters? Reflect on Process • Did your problem-solving process lead you to the correct answer? If not, where did the process break down or lead you astray? How can you revise your approach to produce a more desirable result? o If you got the right answer, well done. If not, did you recognize that the derived characters are marked with 1s in the chart? 47.4) F
Version 1
64
Clarify Question • What is the key concept addressed by the question? o This question addresses cladistics. • What type of thinking is required? o This question is asking you to take what you already know and apply it to this unfamiliar situation. Gather Content • What do you already know about cladistics? o The ancestral condition of a trait is the original condition; the derived condition is the more newly evolved version. o Plesiomorphies are ancestral characters. Choose Answer • Given what you now know, what information and/or problemsolving approach is most likely to produce the correct answer? o Which species has the most ancestral states ("0"s)? Reflect on Process • Did your problem-solving process lead you to the correct answer? If not, where did the process break down or lead you astray? How can you revise your approach to produce a more desirable result? o If you got the right answer, well done. If not, did you recognize that the ancestral characters are marked with 0s in the chart? 48) Section Break 48.1) C
Version 1
65
Clarify Question • What is the key concept addressed by the question? o This question addresses cladistics. • What type of thinking is required? o This question is asking you to analyze the information given, using logic, to dissect the problem and determine the answer. Gather Content • What do you already know about cladistics? o Cladistics is the method used to construct a cladogram—a graphically represented hypothesis of evolutionary relationships. o Only shared derived characters (synapomorphies) are useful for inferring phylogenies, and they must be contrasted to an outgroup that has the ancestral characters. Choose Answer • Given what you now know, what information and/or problemsolving approach is most likely to produce the correct answer? o Since trait C is the same (derived) in all the species listed here, that trait cannot help us to understand their relationships. Reflect on Process • Did your problem-solving process lead you to the correct answer? If not, where did the process break down or lead you astray? How can you revise your approach to produce a more desirable result? o If you got the right answer, well done. If not, did you recognize that a trait is only useful for building a cladogram if the trait varies (from ancestral to derived) in different species? Did you try drawing Version 1
66
the cladogram for this data? 48.2) A Clarify Question • What is the key concept addressed by the question? o This question addresses cladistics. • What type of thinking is required? o This question is asking you to take what you already know and apply it to this unfamiliar situation. Gather Content • What do you already know about cladistics? o A synapomorphy is a shared derived character. In other words, a newly-evolved trait shared by a group of species. Choose Answer • Given what you now know, what information and/or problemsolving approach is most likely to produce the correct answer? o Which species share the least number of derived characters ("1"s)? Reflect on Process • Did your problem-solving process lead you to the correct answer? If not, where did the process break down or lead you astray? How can you revise your approach to produce a more desirable result? o If you got the right answer, well done. If not, did you recognize that the derived characters are marked with 1s in the chart? 49) Section Break 49.1) D Version 1
67
Clarify Question • What is the key concept addressed by the question? o This question addresses cladistics. • What type of thinking is required? o This question is asking you to take what you already know and apply it to this unfamiliar situation. Gather Content • What do you already know about cladistics? oWhen a V-style tree is drawn with the Vs upwards, the highest nodes are the most recently diverged species, and the lower nodes are older divergences between groups. o A branch above a character labelimplies that the higher clade possesses the character. Choose Answer • Given what you now know, what information and/or problemsolving approach is most likely to produce the correct answer? o Which species is shown above two character labels: “Seeds”but not above “Tree”? Reflect on Process • Did your problem-solving process lead you to the correct answer? If not, where did the process break down or lead you astray? How can you revise your approach to produce a more desirable result? o If you got the right answer, well done. If not, did you recognize that the gray labels marked characters? 49.2) A
Version 1
68
Clarify Question • What is the key concept addressed by the question? o This question addresses cladistics. • What type of thinking is required? o This question is asking you to take what you already know and apply it to this unfamiliar situation. Gather Content • What do you already know about cladistics? o Homoplastic means that a character arose by convergent evolution. It is not a synapomorphy because it arose in separate unrelated lineages. Choose Answer • Given what you now know, what information and/or problemsolving approach is most likely to produce the correct answer? o Needles, flowers, or seedsonly appear once on the tree. Reflect on Process • Did your problem-solving process lead you to the correct answer? If not, where did the process break down or lead you astray? How can you revise your approach to produce a more desirable result? o If you got the right answer, well done. If not, did you recognize that the feature “tree” in the phylogeny is homoplastic since it appears in multiple clades? 49.3) C
Version 1
69
Clarify Question • What is the key concept addressed by the question? o This question addresses cladistics. • What type of thinking is required? o This question is asking you to analyze the information given, using logic, to dissect the problem and determine the answer. Gather Content • What do you already know about cladistics? o The ancestor of the phylogeny would be the extinct plant species at the base of the diagram. Choose Answer • Given what you now know, what information and/or problemsolving approach is most likely to produce the correct answer? o Since the question posits that all the species have vascular tissue and chlorophyll, ginkgo must have these too. o By tracing the branches from the bottom to the ginkgo branch, you can find which features the ginkgo must have. Reflect on Process • Did your problem-solving process lead you to the correct answer? If not, where did the process break down or lead you astray? How can you revise your approach to produce a more desirable result? o If you got the right answer, well done. If not, did you recognize that you could trace the evolution of features by moving from the bottom to the top of the tree? 49.4) [A, B, D]
Version 1
70
Clarify Question • What is the key concept addressed by the question? o This question addresses monophyly. • What type of thinking is required? o This question is asking you to take what you already know and apply it to this unfamiliar situation. Gather Content • What do you already know about monophyly? o True clades are monophyletic—they are groups including the most recent common ancestor and all of its descendants. Choose Answer • Given what you now know, what information and/or problemsolving approach is most likely to produce the correct answer? o Do “rose and oak” form a monophyletic group? Yes, because they share the synapomorphy “flowers” and form a clade together. o Do “pine, ginkgo, oak, and rose” form a monophyletic group? Yes, because they share the synapomorphy “seeds” and form a clade together. o Do “Psaronius, fern, pine, ginkgo, oak, and rose” form a monophyletic group? Yes, because they form a clade together as indicated on the cladogram (although their synapomorphy is not indicated here). Reflect on Process • Did your problem-solving process lead you to the correct answer? If not, where did the process break down or lead you Version 1
71
astray? How can you revise your approach to produce a more desirable result? o If you got the right answer, well done. If not, did you recognize that the question was asking which are true clades? 49.5) D
Version 1
72
Clarify Question • What is the key concept addressed by the question? o This question addresses cladistics. • What type of thinking is required? o This question is asking you to take what you already know and apply it to this unfamiliar situation. Gather Content • What do you already know about cladistics? o Cladistics is the method used to construct a cladogram—a graphically represented hypothesis of evolutionary relationships. o Only shared derived characters (synapomorphies) are useful for inferring phylogenies, and they must be contrasted to an outgroup that has the ancestral characters. o Each nested clade is defined by a new shared derived character. These new characters are typically labeled on the branch. Choose Answer • Given what you now know, what information and/or problemsolving approach is most likely to produce the correct answer? o Is it true that fern is equally related to ginkgo and oak? Yes, because they are members of the same distant clade. o Is ginkgo more closely related to oak than rose? No, it is equally related to them both, since they are members of the same clade. o Is oak equally related to rose and pine? No, oak is a member of the same clade with rose, so it is more closely related to rose. o Is fern more closely related to pine than to rose? No, it is equally related to them both, since they are members of the same clade.
Version 1
73
Reflect on Process • Did your problem-solving process lead you to the correct answer? If not, where did the process break down or lead you astray? How can you revise your approach to produce a more desirable result? o If you got the right answer, well done. If not, did you recognize that a species is equally related to all the species of a distant clade? 50) Section Break 50.1) [B, C]
Version 1
74
Clarify Question • What is the key concept addressed by the question? o This question addresses cladistics. • What type of thinking is required? o This question is asking you to take what you already know and apply it to this unfamiliar situation. Gather Content • What do you already know about cladistics? o One key to interpreting a phylogeny is to look at how recently species share a common ancestor, rather than looking at the arrangement of species across the top of the tree Choose Answer • Given what you now know, what information and/or problemsolving approach is most likely to produce the correct answer? o Is a snake more closely related to a newt than to a mouse? No. Snakes are within the same clade as mice. o Is a salamander more closely related to a mouse than to a chimp? No, because mice and chimps are both members of the same distantly related clade. o Is a newt equally related to a snake and a chimp? Yes, because snakes and chimps are both members of the same clade. o Is a mouse more closely related to a chimp than to a snake? Yes, because mice and chimps are members of the same clade. Reflect on Process • Did your problem-solving process lead you to the correct answer? If not, where did the process break down or lead you Version 1
75
astray? How can you revise your approach to produce a more desirable result? o If you got the right answer, well done. If not, did you recognize that looking at the nested clades of a phylogeny can help you to understand how closely related species are? 50.2) [A, D] Clarify Question • What is the key concept addressed by the question? o This question addresses cladograms. • What type of thinking is required? o This question is asking you to weigh and judge, or evaluate, evidence to choose the best of the possible answers. Gather Content • What do you already know about cladograms? o The important thing about a cladogram is the branching pattern and location of nodes. So if the species branch off with a different pattern, they do not represent the same evolutionary tree. Choose Answer • Given what you now know, what information and/or problem-solving approach is most likely to produce the correct answer? o Two of the cladograms are the same except that a clade is rotated above a node. The order of listed species doesn’t matter as long as the branching pattern is the same, so these represent the same tree. Reflect on Process • Did your problem-solving process lead you to the correct answer? If not, where did the process break down or lead you astray? How can you revise your approach to produce a more desirable result? o If you got the right answer, well done. If not, did you recognize that you can “spin” the branches above a node on the cladogram, and it is the same cladogram?
51) Section Break 51.1) A Version 1
76
Clarify Question • What is the key concept addressed by the question? o This question addresses cladistics. • What type of thinking is required? o This question is asking you to take what you already know and apply it to this unfamiliar situation. Gather Content • What do you already know about cladistics? o One key to interpreting a phylogeny is to look at how recently species share a common ancestor, rather than looking at the arrangement of species across the top of the tree Choose Answer • Given what you now know, what information and/or problemsolving approach is most likely to produce the correct answer? o Frogs have many more features in common with the other amphibians, salamanders and newts. Reflect on Process • Did your problem-solving process lead you to the correct answer? If not, where did the process break down or lead you astray? How can you revise your approach to produce a more desirable result? o If you got the right answer, well done. If not, did you recognize that frogs are amphibians, and they share many features with salamanders and newts? 51.2) E
Version 1
77
Clarify Question • What is the key concept addressed by the question? o This question addresses cladistics. • What type of thinking is required? o This question is asking you to take what you already know and apply it to this unfamiliar situation. Gather Content • What do you already know about cladistics? o One key to interpreting a phylogeny is to look at how recently species share a common ancestor, rather than looking at the arrangement of species across the top of the tree Choose Answer • Given what you now know, what information and/or problemsolving approach is most likely to produce the correct answer? o Gorillas, mice and chimps are all mammals Reflect on Process • Did your problem-solving process lead you to the correct answer? If not, where did the process break down or lead you astray? How can you revise your approach to produce a more desirable result? o If you got the right answer, well done. If not, did you realize that the correct answer is the closet shared node to the branches for mouse and gorilla?
Version 1
78
CHAPTER 23 TRUE/FALSE - Write 'T' if the statement is true and 'F' if the statement is false. 1) Retroviruses are unique because they are a combination of RNA and DNA. ⊚ ⊚
true false
2) Tetracycline is effective against viruses because it disrupts the action of the viral ribosomes. ⊚ ⊚
true false
3) Bacteria typically use binary fission, but under special circumstances sexual reproduction is possible via horizontal gene transfer. ⊚ ⊚
true false
MULTIPLE CHOICE - Choose the one alternative that best completes the statement or answers the question. 4) A mutation in the gene encoding the integrase enzyme renders the protein nonfunctional. How would this affect-the HIV infection cycle? A) The gp120 glycoprotein would not be able to bind the CCR5 receptor. B) Reverse transcription would not be possible. C) The viral DNA would not be able to integrate the viral genome into a chromosome. D) Cutting and assembling the capsid would be halted.
5) Once a prophage is present in a bacterial genome, viral genes are reproduced every time the bacterium multiplies. Expression of viral genes, however, is inhibited by a repressor protein. Cell stress can induce the formation of proteases that degrade the repressor. What will result?
Version 1
1
A) The virus will enter the lysogenic cycle. B) The bacterium will reject the viral genes. C) The virus will enter the lytic cycle. D) The bacterial genome will be denatured.
6)
What best describes whether a virus is likely to infect a particular cell? A) Viruses infect the first cells they encounter. B) If a host cell is weakened, viruses are likely to infect it. C) A virus will only infect host cells with the correct surface receptors. D) Viruses only attack epithelial cells, such as the cells lining the lungs. E) The initial choice is random but once a cell type is chosen it becomes the preferred
host .
7) An experimental chemical has been shown to selectively cleave phosphodiester bonds. How would this affect a virus? A) Fragment thegenome B) Digest thecapsid C) Dissociate theenvelope D) Alter the hostbinding site E) Hydrolyze the protein coating
8)
Can a virus be killed? A) Yes, destroying its genome kills it. B) No, viruses are capable of self-assembly and cannot be permanently killed. C) Yes, separating the genes from the capsid kills it. D) No, viruses are nonliving and cannot die.
9) A doctor’s office has run out of the latest flu vaccine, but many patients are asking for it. One of the nurses finds some vials from last year’s batch in a refrigerator. Should they be used?
Version 1
2
A) Yes, because the preservatives and refrigeration ensure that the vaccine is still fresh. B) No, because each year the technology to make a good vaccine improves. C) No, because each year the vaccine is made against different subtypes of the flu virus. D) Maybe, the vaccine should be mixed in a test tube with the virus to see if it still inactivates it.
10) Which class of drugs would be effective against HIV but not against most other pathogenic viruses? A) Nucleotide analogs that halt replication B) Receptor antagonists that block viral entry C) Non-nucleoside reverse transcriptase inhibitors that block reverse transcription D) Translation inhibitors that stop assembly of viral proteins
11) If the gp120 glycoprotein were damaged as a result of a mutation, HIV would have difficulty A) binding to a host cell. B) leaving a macrophage. C) inserting its genes into the host cell's genome. D) assembling the capsid. E) attaching its RNA to a host cell's ribosome.
12) Plasma from a sick mouse is filtered across a membrane that has a 200 nm pore size. If the filtrate is still infectious when injected into a healthy mouse, then the pathogen is most likely a(n) A) bacterium. B) archaean. C) fungus. D) virus. E) parasitic worm.
Version 1
3
13) the
A layer of lipoprotein and glycoprotein that covers the outer surface of some viruses is
A) capsid. B) casing. C) envelope. D) membrane. E) viroid.
14)
Viruses that become established as stable parts of the host cell genome are called A) lethal. B) potent. C) temperate. D) virulent.
15) Which of the following statements about the types and subtypes of the influenza (flu) virus is false? A) Of the three major types of flu virus (A, B, and C), only Type A can occur in humans, other mammals, and birds. B) Flu subtype is determined by the kinds of proteins representing the H and N protein spikes making up the capsid of the virus. C) The A(H2N2) andthe A(H3N2) strains of the virus require different vaccines. D) High mutation rates create more diversity in strains of flu than does genetic recombination. E) Genetic recombination between flu strains from different species is common.
16) Phil has chronic ulcers; he has been taking acid blockers and drinking vast amounts of liquid antacids. These help relieve the symptoms, but he still suffers from ulcers. What would be the best course of action to treat the source of his ulcers?
Version 1
4
A) Taking several different antacids B) Increasing the dosage of acid blockers C) Taking a course of antibiotics D) Reducing the stress in his daily life E) Changing his diet to include more fiber
17)
Why are children often told that if they eat too much candy their teeth will fall out?
A) Sugar attacks tooth enamel. B) Sugars gradually replace the tooth enamel. C) Bacteria use the sugar to make ATP. This fermentation process produces acids as a by-product. D) Bacteria secrete acids in order to digest the sugar molecules.
18) Amanda is a microbiologist who studies the pathogenic bacteria Streptococcus pyogenes. In what ways are they (Amanda and the bacteria)similar? A) Both have cell walls. B) Both have linear chromosomes. C) Both have mitochondria for ATP production. D) Both are capable ofsexual reproduction. E) Both obtain carbon from consuming organic molecules.
19)
Are transformation and transduction the same process? A) Yes, they both involve the acquisition of viral genes. B) No, only transformation passes on genes for pathogenicity. C) Yes, both processes transfer specific gene products to a specific bacterium. D) No, although both involve gene transfer, the sources of the genetic material differ.
Version 1
5
20) Which method of horizontal gene transferis most likely to transfer two genes that are near each other on a bacterial chromosome? A) F+ conjugation B) Transformation C) Lytic viralinfection D) Transduction
21) The cells wall of bacteria Z contains high concentrations of phospholipids along with proteins and relatively low amounts of lipopolysaccharide. When bacteria Z is gram stained, it will most like be colored? A) Purple B) Pink C) Blue D) Clear, it does notretain any stain
22) Many cells of the immune system have receptors that detect peptidoglycans and lipopolysaccharides. How do pathogenic bacteria avoid detection? A) Pathogenic bacteria lack cell walls. B) Pathogens often encase themselves in a gelatinous capsule. C) Peptidoglycans are absent from pathogenic bacteria. D) Disease is only possible if the immune system has been compromised by some other event.
23) A researcher observed a reduction in the number of chromosomes during cell division for a microbe, he concludes that the microbe must be A) eukaryotic. B) prokaryotic. C) an archaean. D) a virus.
Version 1
6
24) DNA replication and gene expression in archaeans more closely resemble eukaryotic cells than the same processes in bacteria. However both Bacteria and Archaea are prokaryotic. What does this suggest? A) Eukaryotic cells are descended from modern archaeans. B) Bacterial cells are not related to archaeans. C) Eukaryotic cells shared a more recent common ancestor with archaeans than with bacterial cells. D) There must have been two origin events, one producing the bacteria and the other producing archaeans and eukaryotic cells.
25)
If the label on a slide reads Gram-positive bacillus, what would a students expect to see? A) Pink helicalcells B) Purple rod shapedcells C) Red sphericalcells D) Blue coiledcells
26) A high concentration of chlorophyll embedded in the plasma membrane fold of a newly discovered species of bacteria suggests a A) Chemolithoautotrophic metabolism B) Chemoheterotrophicmetabolism C) Photoautotrophicmetabolism D) Photoheterotrophicmetabolism
27)
Current classification of prokaryotes is based on
Version 1
7
A) morphology of flagella. B) sequencing of proteins, DNA, and RNA. C) occurrence of transverse binary fission. D) type of colony formation. E) ability to stain the cell wall with Gram stain.
28)
Prokaryotes undergo a process that produces cells that are identical. This process is called A) mitosis. B) meiosis. C) conjugation. D) binary fission. E) syngamy.
29)
The prokaryotic DNA is located in the A) capsid region. B) nucleoid region. C) endospore region. D) peptidoglycan region. E) pili region.
30) Early classification systems of the bacteria employed all of the following characteristics except A) the ability to photosynthesize. B) rRNA sequences. C) motility. D) colony or filament forming. E) sporulation or binary fission.
Version 1
8
31)
In all of the following characteristics, prokaryotes differ from eukaryotes except in A) cell size. B) multicellularity. C) chromosomes. D) nucleic acids as the hereditary material. E) organelles.
32)
Which of the following is not a shape seen in prokaryotes? A) Spheres B) Rods C) Spirals D) Icosahedrons
33)
Archaea differ from Bacteria in all of the following except A) cell wall composition. B) plasma membrane make up. C) DNA replication. D) gene expression. E) the presence of a nucleoid region.
34)
All of the following are ways that prokaryotes can exchange genetic material except A) vertical genetransfer. B) conjugation. C) transduction. D) transformation. E) exchange of Rplasmids.
Version 1
9
35) Some prokaryotes attach to other substrates by hairlike outgrowths, which are shorter than flagella. These are called A) endospores. B) flagellin. C) pili. D) plasmids. E) fission products.
36) The enzymes for cellular respiration in eukaryotic organisms are located within the mitochondria. In prokaryotic organisms, these enzymes are attached to the A) cell wall. B) cellmembrane. C) ribosomes. D) circular DNA. E) nuclear membrane.
37) A microbiologist explains to one of her biology classes that her research focuses on the nucleoid region of the cholera bacteria. This means that she is studying a region within the bacterium that contains the A) nucleic acids. B) nuclear membrane. C) R plasmids. D) peptidoglycan.
38)
Which of the following statements about Hfr cells and bacterial conjugation is false?
Version 1
10
A) An Hfr cell becomes an F – cell if its integrated F plasmid is excised. B) If excision of the F plasmid from an Hfr cell includes some bacterial genes, a partial diploid can result. C) The genes that are transferred from an Hfr cell into a recipient cell during conjugation replace the equivalent genes in the recipient cell. D) An F' plasmid contains some bacterial genes. E) Cells with integrated F' plasmid have two copies of some genes.
39)
Plasmids are distinguished from bacterial chromosomes in that A) plasmids are circular, while bacterial chromosomes are linear. B) plasmids occur in the cytoplasm, while bacterial chromosomes occur in the nucleus. C) plasmids are composed of RNA, while bacterial chromosomes are composed of
DNA. D) plasmids have few genes, while bacterial chromosomes have many genes. E) plasmids consist of single-stranded nucleic acids, while bacterial chromosomes consist of double-stranded nucleic acids.
40)
Which of the following statements about generalized transduction is false?
A) "Headfull" packaging always places only phage DNA into the phage head during phage packaging in generalized transduction. B) Generalized transduction always involves lytic phage. C) Generalized transduction can be used to map bacterial genes. D) Genes that are close together on the bacterial chromosome will have a relatively large cotransduction frequency. E) Two recombination events are necessary to integrate the transferred DNA during generalized transduction.
41)
Which of the following events is least likely to result in a genetic change in a bacterium?
Version 1
11
A) Pilus formation B) Conjugation C) Specialized transduction D) Generalized transduction E) Natural transformation
FILL IN THE BLANK. Write the word or phrase that best completes each statement or answers the question. 42) Prokaryotes split into two lines early in the history of life. The differences are so fundamental that they are assigned to two different domains, the ________________ and the Bacteria.
43) Cell walls of bacteria (domain Bacteria) usually consist of _____________, a network of polysaccharide molecules connected by polypeptide cross-links.
Version 1
12
Answer Key Test name: Chap 23_3e 1) FALSE 2) FALSE 3) FALSE 4) C
Version 1
13
Clarify Question • What is the key concept addressed by the question? o This question addresses the HIV infection cycle. • What type of thinking is required? o This question is asking you to take what you already know and apply it to this unfamiliar situation. Gather Content • What do you already know about the HIV infection cycle? o The HIV infection cycle has multiple steps, using several viralencoded enzymes. o The gp120 protein binds to the CD4 receptor and then the CCR5 receptor to dock and fuse into the cell. o Reverse transcriptase creates a double-stranded DNA copy of the viral RNA genome. o Integrase allows the viral DNA to merge into a host cell chromosome. o The genome is transcribed, translated, and processed by proteases, and then new viruses are assembled and released. Choose Answer • Given what you now know, what information and/or problemsolving approach is most likely to produce the correct answer? o If integrase was mutated, would the gp120 glycoprotein be unable to bind the CCR5 receptor? No, gp120 can interact with CCR5 directly after undergoing a conformational change. o If integrase was mutated, would reverse transcription be impossible? No, that is the job of reverse transcriptase. o If integrase was mutated, would cutting and assembling the capsid be halted? No, that is the job of protease. Version 1
14
o If integrase was mutated, would the viral DNA be unable to incorporate into the host cell chromosome? Yes. Integrase is necessary to integrate into the viral DNA into the host cell chromosome. Reflect on Process • Did your problem-solving process lead you to the correct answer? If not, where did the process break down or lead you astray? How can you revise your approach to produce a more desirable result? o The question required you to take what you already know and apply it to this unfamiliar situation. o Did you recognize that the integrase enzyme is necessary to integrate the viral DNA into the host cell genome? 5) C
Version 1
15
Clarify Question • What is the key concept addressed by the question? o This question addresses phage induction. • What type of thinking is required? o This question is asking you to analyze the information given, using logic, to dissect the problem and determine the answer. Gather Content • What do you already know about phage induction? o In a lysogen, expression of the prophage genome is repressed by a viral regulatory protein. o If the cell is stressed, the prophage can be derepressed, expressing the enzymes necessary for excision of the genome and the lytic cycle can commence. o The switch from a lysogenic prophage to a lytic cycle is called induction because it requires turning on the gene expression necessary for the lytic cycle. o It can be stimulated in the laboratory by stressors such as starvation or ultraviolet radiation. o The molecular basis of induction is a stress-induced protease that can destroy the repressor protein that keeps the prophage silent. The normal function of this protease is to degrade a host repressor that controls DNA repair genes. Choose Answer • Given what you now know, what information and/or problemsolving approach is most likely to produce the correct answer? o When cell stress degrades the repressor, will the bacterial genome be denatured? No, it shouldn’t affect the bacterial genome. (If anything, the Version 1
16
protease will induce DNA repair.) o When cell stress degrades the repressor, will the bacterium reject the viral genes? No, this isn’t a mechanism to do that. o When cell stress degrades the repressor, will the virus enter the lysogenic cycle? No, it was already in the lysogenic cycle. Instead, it will express viral genes and enter the lytic cycle Reflect on Process • Did your problem-solving process lead you to the correct answer? If not, where did the process break down or lead you astray? How can you revise your approach to produce a more desirable result? o The question required you to analyze the information given, using logic, to dissect the problem and determine the answer. o Did you recognize that when the repressor is degraded, then viral genes are expressed and the prophage can excise and enter the lytic cycle? 6) C
Version 1
17
Clarify Question • What is the key concept addressed by the question? o This question addresses viral host range. • What type of thinking is required? o This question is asking you to take what you already know and apply it to this unfamiliar situation. Gather Content • What do you already know about viral host range? o Viruses occur as obligate intracellular parasites in every kind of organism that has been investigated for their presence. o Viruses infect fungal cells, bacterial cells, and protists as well as cells of plants and animals; however, each type of virus can replicate in only a very limited number of cell types. A virus that infects bacteria would be ill-equipped to infect a human or plant cell. o The suitable cells for a particular virus are collectively referred to as its host range. o The first step of infection is called attachment, in which the virus contacts and becomes specifically bound to the cell. This step limits the host range of the virus as they bind to specific proteins on the surface of the cell. Choose Answer • Given what you now know, what information and/or problemsolving approach is most likely to produce the correct answer? o Because of the need to bind receptors on the cell to enter, a virus will only infect host cells with the correct surface receptors. Reflect on Process Version 1
18
• Did your problem-solving process lead you to the correct answer? If not, where did the process break down or lead you astray? How can you revise your approach to produce a more desirable result? o The question required you to take what you already know and apply it to this unfamiliar situation. o Did you recognize that a virus has to match the receptors on its specific host cell? 7) A
Version 1
19
Clarify Question • What is the key concept addressed by the question? o This question addresses viral structure. • What type of thinking is required? o This question is asking you to take what you already know and apply it to this unfamiliar situation. Gather Content • What do you already know about viral structure? o A virus will contain a genome made of DNA or RNA. o The genome will lie within a capsid made of protein. o It may have an envelope made of lipid membrane with proteins. Choose Answer • Given what you now know, what information and/or problemsolving approach is most likely to produce the correct answer? o Would cleaving phosphodiester bonds digest the capsid? No, because that is formed of protein, and phosphodiester bonds are not found in proteins. o Would cleaving phosphodiester bonds dissociate the envelope? No, because that is formed of protein and lipids, and phosphodiester bonds are not found in proteins or most lipids. o Would cleaving phosphodiester bonds alter the host binding site? No, because that is likely a protein, and phosphodiester bonds are not found in proteins. o Would cleaving phosphodiester bonds hydrolyze the protein coating? No, because phosphodiester bonds are not found in proteins. o Would cleaving phosphodiester bonds fragment the genome? Yes, because viral genomes are made of either DNA or RNA, which contain Version 1
20
phosphodiester bonds. Reflect on Process • Did your problem-solving process lead you to the correct answer? If not, where did the process break down or lead you astray? How can you revise your approach to produce a more desirable result? o The question required you to take what you already know and apply it to this unfamiliar situation. o Did you recognize that phosphodiester bonds are found only in DNA and RNA, thus they are found in the viral genome? 8) D
Version 1
21
Clarify Question • What is the key concept addressed by the question? o This question addresses whether viruses are alive. • What type of thinking is required? o This question is asking you to weigh and judge evidence, or evaluate, to choose the best of the possible answers. Gather Content • What do you already know about viruses as organisms? o Viruses are genetic elements enclosed in protein; they are not considered organisms because they lack many of the features associated with life, including cellular structure, and independent metabolism or replication. o For this reason viral particles are not called viral cells, but virions, and they are generally not described as living or dead but as active or inactive. Choose Answer • Given what you now know, what information and/or problemsolving approach is most likely to produce the correct answer? o You cannot kill a virus, because they are nonliving entities. If they are never really alive, they can never die. Reflect on Process • Did your problem-solving process lead you to the correct answer? If not, where did the process break down or lead you astray? How can you revise your approach to produce a more desirable result? o The question required you to weigh and judge evidence, or evaluate, Version 1
22
to choose the best of the possible answers. o Did you recognize that viruses cannot be killed since they are not truly alive? 9) C
Version 1
23
Clarify Question • What is the key concept addressed by the question? o This question addresses the flu virus. • What type of thinking is required? o This question is asking you to weigh and judge evidence, or evaluate, to choose the best of the possible answers. Gather Content • What do you already know about the flu virus? o Unlike most other viruses, the influenza virus is known for high rates of mutation and recombination. o Different strains of flu virus, called subtypes, differ in their protein spikes. One of these proteins, hemagglutinin (H), aids the virus in gaining access to the cell interior. The other, neuraminidase (N), helps the daughter viruses break free of the host cell after replication. o Parts of the H molecule contain “hotspots” with a high rate of mutation. o Also, viral RNA segments are readily reassorted by genetic recombination when two different subtypes simultaneously infect the same cell. This may put together novel combinations of H and N spikes unrecognizable by human antibodies specific for the old configuration. Choose Answer • Given what you now know, what information and/or problemsolving approach is most likely to produce the correct answer? o Does the technology to make a good vaccine improve each year? Not really. o Should the vaccine be mixed in a test tube with the virus to see if it still inactivates it? No. Remember that the vaccine is a mimic of the Version 1
24
virus that trains the immune system to recognize it. It does not directly inactivate the virus. o Will the preservatives and refrigeration ensure that the vaccine is still fresh? Yes, probably, but it will likely be ineffective. o Each year, is the vaccine made against different subtypes of the flu virus? Yes. This is why last year’s vaccine is likely to be ineffective and should not be used. Reflect on Process • Did your problem-solving process lead you to the correct answer? If not, where did the process break down or lead you astray? How can you revise your approach to produce a more desirable result? o The question required you to weigh and judge evidence, or evaluate, to choose the best of the possible answers. o Did you recognize that the flu vaccine is different every year, to match the common new subtypes? 10) C
Version 1
25
Clarify Question • What is the key concept addressed by the question? o This question addresses HIV replication. • What type of thinking is required? o This question is asking you to take what you already know and apply it to this unfamiliar situation. Gather Content • What do you already know about HIV replication? o Human immunodeficiency virus (HIV), the agent that causes acquired immunodeficiency syndrome (AIDS), is a retrovirus. o Retroviruses have an RNA genome that is reverse-transcribed into DNA by the enzyme reverse transcriptase. The DNA fragments produced by reverse transcription are often integrated into a host’s chromosomal DNA. Choose Answer • Given what you now know, what information and/or problemsolving approach is most likely to produce the correct answer? o Would most viruses be affected by nucleotide analogs that halt replication? Yes, because all viruses use nucleotides in their RNA or DNA genome. o Would most viruses be affected by receptor antagonists that block viral entry? Yes, because viruses often take advantage of cell receptors to enter the cell. o Would most viruses be affected by translation inhibitors that stop assembly of viral proteins? Yes, because all viruses (although not viroids) use proteins in their capsid. o Would most viruses be affected by non-nucleoside reverse Version 1
26
transcriptase inhibitors that block reverse transcription? No. Most viruses don’t use reverse transcriptase. Only retroviruses like HIV use reverse transcriptase, so it would be affected but most other viruses would not be. Reflect on Process • Did your problem-solving process lead you to the correct answer? If not, where did the process break down or lead you astray? How can you revise your approach to produce a more desirable result? o The question required you to take what you already know and apply it to this unfamiliar situation. o Did you recognize that unlike most viruses, HIV relies on a reverse transcriptase enzyme? 11) A
Version 1
27
Clarify Question • What is the key concept addressed by the question? o This question addresses HIV infection. • What type of thinking is required? o This question is asking you to take what you already know and apply it to this unfamiliar situation. Gather Content • What do you already know about HIV infection? o HIV infects key immune-system cells: macrophages and T cells. o The viral glycoprotein gp120 precisely fits on the cell-surface marker protein CD4+ on those cells. When HIV attaches to two receptors CD4+ and CCR5, it is able to pull itself across the cell membrane and enter the cell. Choose Answer • Given what you now know, what information and/or problemsolving approach is most likely to produce the correct answer? o The gp120 glycoprotein binds to the CD4+ protein on the surface of macrophages and T cells, allowing the HIV virus to enter the cell. o So if it was mutated, it would not be able to bind to the host cell, and that would prevent viral entry. Reflect on Process • Did your problem-solving process lead you to the correct answer? If not, where did the process break down or lead you astray? How can you revise your approach to produce a more desirable result? o The question required you to take what you already know and apply it Version 1
28
to this unfamiliar situation. o Did you recognize that the gp120 glycoprotein binds to the CD4+ cellsurface marker protein on macrophages and T cells, allowing viral entry? 12) D
Version 1
29
Clarify Question • What is the key concept addressed by the question? o This question addresses virus size. • What type of thinking is required? o This question is asking you to take what you already know and apply it to this unfamiliar situation. Gather Content • What do you already know about virus size? o A traditional definition of viruses was that they were “filterable agents.” that is, infectious agents that could pass through a 500 nm filter. o Some viruses, like Mimivirus or Pandoravirus, can be larger. However, anything smaller than 200 nm is likely to be a virus. o The smallest bacterium, Mycoplasma, is between 200–300 nm across. Choose Answer • Given what you now know, what information and/or problemsolving approach is most likely to produce the correct answer? o Viruses are the smallest of the options listed. Bacteria, archaea, a fungus or worm would all be caught in the filter. Reflect on Process • Did your problem-solving process lead you to the correct answer? If not, where did the process break down or lead you astray? How can you revise your approach to produce a more desirable result? o The question required you to take what you already know and apply it to this unfamiliar situation. o Did you recognize that viruses are smaller than bacteria, archaea, fungi, or worms? Version 1
30
13) C 14) C 15) E 16) C
Version 1
31
Clarify Question • What is the key concept addressed by the question? o You are Phil’s doctor. You have been treating his ulcers with acid blockers (such as omeprazole) and liquid antacids, but these treatments don’t seem to be solving the underlying problem. What should you do to get the ulcers to go away? • What type of thinking is required? o This question is an “apply” question, so it is asking you to use the information you already know about ulcers and apply it to this new situation. Gather Content • What do you already know about ulcers? How does it relate to the question? o Doctors used to think that ulcers were caused by stress or eating acidic foods. One researcher proposed that peptic ulcers were caused by a bacterial infection. He was completely ignored until, during a presentation at a conference, he drank a solution of the bacterium Heliobacter pylori, which gave him peptic ulcers. • What other information is related to the question? Which information is most useful? o Which treatments has Phil taken so far? Why haven’t they worked? Choose Answer • Given what you now know, what information and/or problemsolving approach is most likely to produce the correct answer? o Given that Phil’s ulcers may be caused by Heliobacter pylori, what would you propose as his next treatment? Reflect on Process Version 1
32
• Did your problem-solving process lead you to the correct answer? If not, where did the process break down or lead you astray? How can you revise your approach to produce a more desirable result? o This question asked you to figure out which treatment option would be most likely to treat the source of Phil’s ulcers. o If you got the correct answer, great job! o If you got an incorrect answer, where did you get stuck? • Phil has already been taking anacids and acid blockers. These treated his symptoms, but not the source of the problem.It is probably time to try something new. • Heliobacter pylori is a bacterium, which could be killed using antibiotics. If Phil’s ulcers are due to an infection, antibiotics would kill the bacteria and stop the ulcers. 17) C
Version 1
33
Clarify Question • What is the key concept addressed by the question? o Eating too much candy can cause you to get dental cavities. Why? • What type of thinking is required? o This question is an “evaluate” question. You need to determine which hypothesis is the best explanation of the phenomenon you have observed. Gather Content • What do you already know sugars? How does it relate to the question? o Sugars are simple carbohydrates. Many sugars linked together form more complex carbohydrates such as starch or cellulose. • What other information is related to the question? Which information is most useful? o The dental plaque you brush off your teeth each night is made of biofilm, which is made of many different bacterial species. Some of these bacteria perform aerobic respiration, while others produce anaerobic respiration. What are the byproducts of aerobic and anaerobic respiration? Choose Answer • Given what you now know, what information and/or problemsolving approach is most likely to produce the correct answer? o Cover up the options and try to answer the question as if it were a short answer question. Now, uncover the options. Do any of the options match your answer? o What else do you know about sugars? What else do you know about bacteria? Version 1
34
o Once you’ve narrowed down your options, pick the one with the most support. Reflect on Process • Did your problem-solving process lead you to the correct answer? If not, where did the process break down or lead you astray? How can you revise your approach to produce a more desirable result? o This question asked you to explain why eating sugars results in tooth decay. o If you got the correct answer, great job! o If you got an incorrect answer, where did you get stuck? • Sugars themselves do not attack the tooth enamel. Bacteria using anaerobic respiration to consume the sugars produce lactic acid, which breaks down the enamel. Would bacteria using aerobic respiration break down your enamel? 18) E
Version 1
35
Clarify Question • What is the key concept addressed by the question? o How are bacteria and eukaryotes (specifically, humans) similar? • What type of thinking is required? o This question is an “apply” question, so it is asking for you to find similarities and differences between two different concepts (bacteria and eukaryotes) that you learned about individually. Gather Content • What do you already know about eukaryotes and prokaryotes? How does it relate to the question? o Humans are a group of eukaryotes. In general, eukaryotes have linear chromosomes made of DNA. Do bacteria have chromosomes? Are they linear or circular? o Mitochondria are organelles that produce the molecule ATP. ATP can be broken apart to release energy anywhere it is needed in the cell. Do eukaryotes have mitochondria? Do bacteria? o Sexual reproduction involves the production of gametes (cells that contain half the copies of chromosomes of the normal cells). Two gametes fuse to form a zygote, which has half of its DNA from one parent and half of its DNA from another parent. Conjugation is similar to sexual reproduction, but it is not true of sexual reproduction. Which organisms do sexual reproduction? Which organisms use meiosis to produce gametes? o Cell walls are rigid structures outside of the plasma membrane. They help protect the cell and keep the cell from exploding in hypotonic solutions. Cell walls can be made of many things, including cellulose, lignin, silica, and peptidoglycan.Do bacteria have cell walls? Do humans have cell walls? o Organisms can get nutrients a variety of ways, including heterotrophy Version 1
36
and autotrophy. Heterotrophs get energy and raw materials by consuming other organisms.Autotrophs produce their own energy. For example, plants use the sunlight to make energy and get their carbon from carbon dioxide. They do not need to consume other organisms to get these things. How do humans get energy and raw materials? How do bacteria do these things? Choose Answer • Given what you now know, what information and/or problemsolving approach is most likely to produce the correct answer? o Make a Venn diagram showing how eukaryotes and bacteria are similar and different. You should end up with some grey areas. For example, some eukaryotes and some bacteria are autotrophs while others are heterotrophs. Make another Venn diagram showing similarities and differences between humans and Streptococcus pyogenes. Use this diagram to answer the question. Reflect on Process o Did your problem-solving process lead you to the correct answer? If not, where did the process break down or lead you astray? How can you revise your approach to produce a more desirable result? o This question asked you to find similarities between humans and the pathogenic bacterium Streptococcus pyogenes. o If you got the correct answer, great job! o If you got an incorrect answer, where did you get stuck? o Did you remember that bacteria do not have organelles? Humans and other eukaryotes have mitochondria, but bacteria do not. o Did you recognize that both humans and S. pyogenes are heterotrophs? This means they consume other organisms (organic Version 1
37
molecules) to obtain energy and raw materials, including carbon. 19) D 20) A 21) B
Version 1
38
Clarify Question • What is the key concept addressed by the question? •You have discovered a new bacteria species: bacteria Z. You analyze its cell wall and determine that it contains a lot of phospholipids, some proteins, and some carbohydrates.Is this a gram-negative or grampositive bacterium?What color will it be after gram staining? • What type of thinking is required? o This question is an “apply” question. You need to apply your previous knowledge (about gram staining, gram-negative bacteria, and grampositive bacteria) to a new situation. Gather Content • What do you already know about gram staining? How does it relate to the question? o During gram staining, you use crystal violet and iodine, which stay under the peptidoglycan layer of gram-positive cells. In gram-negative cells, there are pores between sections of the peptidoglycan layer, which allow the crystal violet and iodine to escape. o You then use saffranin, which stains all of the cells. The grampositive cells are already purple, so their color does not change much. The gram-negative cells change from clear to pink. o Is bacteria Z gram-positive or gram-negative? How are the cell walls of these two groups different? How could this help you answer the question? Choose Answer • Given what you now know, what information and/or problemsolving approach is most likely to produce the correct answer? o First, determine whether bacteria Z is gram-negative or gram-positive. Version 1
39
Use the information about the contents of bacteria Z’s cell wall. Which type of bacterium is going to have a lot of phospholipids, a few proteins, and a few carbohydrates in its cell wall? o Then, use this information to determine what color the bacterium will be after gram staining. Reflect on Process • Did your problem-solving process lead you to the correct answer? If not, where did the process break down or lead you astray? How can you revise your approach to produce a more desirable result? o This question gave you information about a new bacterium, then asked you to determine what color it would be after a gram stain. The question required a two-step process to answer. First, you needed to determine whether bacteria Z is gram-positive or gram-negative. Then, you needed to use that information to determine what color it would be after a gram stain. o If you got the correct answer, great job! o If you got an incorrect answer, where did you get stuck? •Bacteria Z had a lot of phospholipids, a few carbohydrates, and a few proteins in its cell wall. Did you figure out whether it was gram-positive or gram-negative? • Gram-positive bacteria have a huge layer of peptidoglycan (a carbohydrate) for a cell wall. • Gram-negative bacteria have a thin layer of peptidoglycan surrounded by the plasma membrane on one side and the outer membrane on the other. These membranes are made of phospholipids and proteins. 22) B
Version 1
40
Clarify Question • What is the key concept addressed by the question? o You need to determine which option best explains how bacterial cells avoid being detected by the host’s immune system. • What type of thinking is required? o This question is an “apply” question. You need to use your knowledge of bacterial anatomy and apply it to this new situation. Gather Content • What do you already know about bacterial cell walls? How does it relate to the question? o Bacteria can be gram-positive, with a cell wall made of a thick layer of peptidoglycan (a lipopolysaccharide). o They can also be gram-negative, with a cell wall made of a thin layer of peptidoglycan sandwiched between the plasma membrane and the outer membrane. o The cell wall has several purposes. It protects the cell from damage. It also keeps the cell from bursting if it is put in a hypotonic solution.What do you know about the cell walls of pathogenic bacteria?What do they do to keep the hosts’ immune system from recognizing them? Choose Answer • Given what you now know, what information and/or problem solving approach is most likely to produce the correct answer? o Pick any pathogenic bacterium you remember from class or the textbook. Does it have a cell wall? What is the cell wall made of? Read over the options. There are a couple you can cross off fairly easily. Then, review the information in the text to choose between the last couple options. Version 1
41
Reflect on Process • Did your problem-solving process lead you to the correct answer? If not, where did the process break down or lead you astray? How can you revise your approach to produce a more desirable result? o This question asked you to explain how pathogenic bacteria can avoid the immune cells if the immune cells can detect peptidoglycan. o If you got the correct answer, great job! o If you got an incorrect answer, where did you get stuck? • All bacteria have cell walls, but many pathogenic ones also have a gelatinous capsule.This capsule allows them to stick to surfaces and avoid detection. There is more information about this in your text. 23) A
Version 1
42
Clarify Question • What is the key concept addressed by the question? o You are observing a new microorganism under the microscope. You see a cell replicate its chromosomes and divide into multiple daughter cells, but each daughter cell has fewer chromosomes than the original parent cell. How would you classify this new organism? • What type of thinking is required? o This question is an “apply” question. You need to use your knowledge about the different taxa (eukaryotes, prokaryotes, viruses) and how they replicate to answer the question about this new organism. Gather Content • What do you already know about cell replication? How does it relate to the question? o Cells can replicate in a number of ways, including: binary fission, mitosis, meiosis, and lysogenesis. Which of these types of replication is found in bacteria? in archaea? in eukarya?in viruses? o Review each of these types of cell replication and division. Which of them produces daughter cells that have fewer chromosomes than the original parent cell? Choose Answer • Given what you now know, what information and/or problemsolving approach is most likely to produce the correct answer? o First, figure out which type of cell replication produces daughter cells with fewer chromosomes than the parent cell. o Now that you’ve figured out which process you observed under the microscope, which group of organisms can do this process?
Version 1
43
Reflect on Process • Did your problem-solving process lead you to the correct answer? If not, where did the process break down or lead you astray? How can you revise your approach to produce a more desirable result? o This question asked you to classify a new microorganism based on the fact that you observed the microorganism replicate and produce daughter cells that had fewer chromosomes than the original parent cell. o If you got the correct answer, great job! o If you got an incorrect answer, where did you get stuck? •There is only one type of cell replication that involves a decrease in chromosome number: meiosis. Meiosis produces daughter cells that each have half the chromosomes of the original parent cell. If the parent cell was diploid, it produces four haploid daughter cells. •Eukaryotes are the only organisms that do meiosis. Did you know that there are single-celled eukaryotes? In fact, most eukaryotes are singlecelled. 24) C
Version 1
44
Clarify Question • What is the key concept addressed by the question? o You observe that Eukaryotes are more similar to Archaea than to Bacteria. Which hypothesis best explains this observation? • What type of thinking is required? o This question is an “evaluate” question. You must use your previous knowledge, along with logic and reason, to determine which hypothesis best explains the observed phenomenon. Gather Content • What do you already know about phylogenetic trees? How does it relate to the question? o How do phylogenetic trees help explain how organisms are related to each other? These trees use similarities and differences to map out relatedness. If two organisms are very similar, they are likely to be closely related. How would you draw a phylogenetic tree of the three domains? Are all three equally related?If so, they would all branch out of the same node. Are two domains more similar to each other? If so, first one domain branches off, then the other two. o Scientists used to think that Bacteria and Archaea were closely related to each other because they are both prokaryotes. These two domains were grouped together, and Eukarya was their “cousin” on the tree. Does modern evidence agree with this? What would a modern tree of the three domains look like? Choose Answer • Given what you now know, what information and/or problemsolving approach is most likely to produce the correct answer? o You can evaluate some of these options using probability. What is the Version 1
45
probability that life originated on Earth twice?Is it greater or less than the probability that Eukaryotes are related to Archaea? o Next, think about what you know of phylogenetic trees. If Eukaryotes are more similar to Archaea, does that mean that Archaea are not related to Bacteria at all? Does it mean that Eukaryotes evolved from modernday Archaeans? Reflect on Process • Did your problem-solving process lead you to the correct answer? If not, where did the process break down or lead you astray? How can you revise your approach to produce a more desirable result? o This question asked you to pick the hypothesis that best explains why Eukaryotes are more similar to Archaea than to Bacteria. o If you got the correct answer, great job! o If you got an incorrect answer, where did you get stuck? • Did you think that Eukaryotes descended from modern Archaeans because they are so similar? Eukaryotes did not descend from modern Archaeans, just like you did not descend from your sibling. The two domains descended from a common ancestor, and that common ancestor was more recent than the common ancestor with Bacteria. Think of Eukarya and Archaea as sisters, both descended from a mother. Bacteria is their cousin, related to them through everyone’s grandmother. •Phylogenetic trees are sometimes difficult to understand at first. If you are struggling with this concept, talk to your instructor or TA and try to do more practice problems. The best way to build understanding and confidence for this concept is practice! 25) B
Version 1
46
Clarify Question • What is the key concept addressed by the question? o If you had a slide of a Gram-positive bacillus bacterium that was stained using the Gram-staining process, what would it look like? • What type of thinking is required? o This question is an “apply” question. You need to combine your knowledge from two different areas (Gram-staining and bacterial shapes) to answer the question. Gather Content • What do you already know about Gram staining? How does it relate to the question? o During Gram staining, you use crystal violet and iodine, which stay under the peptidoglycan layer of Gram-positive cells. In Gram-negative cells, there are pores between sections of the peptidoglycan layer, which allow the crystal violet and iodine to escape. o You then use saffranin, which stains all of the cells.The Grampositive cells are already purple, so their color does not change much. The Gram-negative cells change from clear to pink. • What other information is related to the question? Which information is most useful? o Bacteria can be round, rod-shaped, or spiral-shaped. They can also be in chains, pairs, or clusters. What vocabulary terms describe each of these shapes and arrangements? Try looking up bacillus, coccus, spirochete, diplo, and strepto. Choose Answer • Given what you now know, what information and/or problemsolving approach is most likely to produce the correct answer? Version 1
47
o First, figure out which color the bacteria should be. Then figure out what shape the bacteria should be. For an option to be correct, both parts of the answer need to be correct. Reflect on Process • Did your problem-solving process lead you to the correct answer? If not, where did the process break down or lead you astray? How can you revise your approach to produce a more desirable result? o This question asked you to determine what a Gram-positive bacillus would look like. o If you got the correct answer, great job! o If you got an incorrect answer, where did you get stuck? •Gram-positive bacteria are purple after a Gram staining procedure. Bacilli are rod-shaped. Cocci are round, and spirochetes are spiralshaped. 26) C
Version 1
48
Clarify Question • What is the key concept addressed by the question? o You have discovered a new bacterium in a lake. Looking at it under the microscope, you notice a lot of green chlorophyll molecules embedded in folds in its plasma membrane.How does this bacterium get energy? • What type of thinking is required? o This question is an “apply” question.You need to apply your knowledge of bacterial metabolism to a new situation. Gather Content • What do you already know about bacterial metabolism? How does it relate to the question? o In general, bacteria can either produce their own energy or get energy by consuming other organisms. Autotrophic organisms produce their own energy (auto = self).Heterotrophic organisms get energy by consuming others (hetero = other). o Bacteria such as cyanobacteria get energy from sunlight by doing photosynthesis. Are these bacteria autotrophs or heterotrophs? o Bacteria such as rhizobia, which live in legume roots, get energy by oxidizing nitrogen. These bacteria are categorized as chemolithotrophs. There are a variety of chemolithotrophs, and they can oxidize nitrogen, sulfur, or hydrogen. Some live at the bottom of the ocean next to volcanic vents.Are these bacteria autotrophs or heterotrophs? • What other information is related to the question? Which information is most useful? o Chlorophyll is a green pigment found in a variety of organisms, including many green plants. What is it used for?
Version 1
49
Choose Answer • Given what you now know, what information and/or problemsolving approach is most likely to produce the correct answer? o This question can actually be broken into two parts. First, determine whether the organisms is a chemolithotroph or a phototroph. Then determine whether the organisms is an autotroph or a heterotroph. Use this information to pick between the four options. Reflect on Process • Did your problem-solving process lead you to the correct answer? If not, where did the process break down or lead you astray? How can you revise your approach to produce a more desirable result? o This question asked you to determine the type of metabolism used by a bacterium that has a lot of chlorophyll in its membrane. o If you got the correct answer, great job! o If you got an incorrect answer, where did you get stuck? •Did you figure out that chlorophyll is used for photosynthesis? Green plants have it in their chloroplasts and cyanobacteria have it embedded in their plasma membranes. Your new bacterium probably does photosynthesis to get energy.A few bacteria, such as purple sulfur bacteria, are photoheterotrophs, but the presence of a lot of chlorophyll makes it more likely that your bacterium is a photoautotroph. 27) B 28) D 29) B 30) B 31) D 32) D 33) E Version 1
50
34) A 35) C 36) B 37) A 38) A 39) D 40) A 41) A
Version 1
51
Clarify Question • What is the key concept addressed by the question? o The options list various processes a bacterium could undergo. Which process is least likely to result in the bacterium getting new DNA? • What type of thinking is required? o This question is an “evaluate” question. You need to judge the probability of each option resulting in genetic change. You should then pick the option that has the least probability of the bacterium getting new DNA. Gather Content • What do you already know about horizontal gene transfer? How does it relate to the question? o Horizontal gene transfer occurs when the DNA from a donor cell is inserted into the DNA of a receiving cell. This DNA could be new or could look like the original DNA. There are three types of horizontal gene transfer: conjugation, transformation, and transduction. o Conjugation occurs when a donor cell attaches to a recipient cell using a sex pilus. The donor cell then transfers a copy of the F-plasmid to the recipient cell. o Transduction occurs when a virus accidentally picks up DNA from an infected bacterium’s chromosome. It then inserts this DNA into the next bacterium that the virus infects. o Transformation occurs when a bacterium picks up DNA from its environment, either by taking in free-floating DNA or by taking DNA from a cell it just consumed using phagocytosis. The bacterium inserts this segment of DNA into a similar region in its own chromosome. Sometimes this new DNA contains new genes. These new genes could be beneficial or detrimental. Version 1
52
Choose Answer • Given what you now know, what information and/or problemsolving approach is most likely to produce the correct answer? o Look at each option and picture the process in your mind. It may help to draw cartoons of each process. Which ones involve DNA transfer? Which direction is the DNA moving? What is the probability that the recipient cell will get any DNA at all? Reflect on Process • Did your problem-solving process lead you to the correct answer? If not, where did the process break down or lead you astray? How can you revise your approach to produce a more desirable result? o This question asked you to determine which of the options would be the least likely to result in a change in the bacterium’s genotype. o If you got the correct answer, great job! o If you got an incorrect answer, where did you get stuck? •Four of the options discuss processes of horizontal gene transfer. All four have relatively the same probability of a bacterium getting new DNA. •“Pilus formation” is different. Bacteria can form pili that are not sex pili. These pili are used for movement or to pull the bacterium closer to a surface. If it is a sex pilus, it will only form on a donor cell, and donor cells do not get new DNA. 42) Archaea 43) peptidoglycan
Version 1
53
CHAPTER 24 TRUE/FALSE - Write 'T' if the statement is true and 'F' if the statement is false. 1) Cercozoans are a widely diverse group of amoeboid flagellated protists that have been grouped together based on genetic similarities. ⊚ ⊚
2)
true false
Mitosis requires full dissolution of the nuclear membrane. ⊚ ⊚
true false
CHECK ALL THE APPLY. Choose all options that best completes the statement or answers the question. 3) Which of the following supports the hypothesis that chloroplasts originated from cyanobacteria? Check all that apply. A) Replication by mitosis B) Chloramphenicol, which binds to the50S ribosomal subunit andinhibits protein sythesis in bacteria, also affects chloroplasts. C) Chloroplasts contain their own DNA, which is remarkably similar to the DNA of bacteria in size and character. D) Replication by binary fission
MULTIPLE CHOICE - Choose the one alternative that best completes the statement or answers the question. 4) Sponges (phylum: Porifera) are animals that possess feeding cells called collar cells. These cells have a central flagellum surrounded by a collar of microvilli. The collar filters organic matter out of the water column. Which protist shares a more recent common ancestor with the poriferans?
Version 1
1
A) Ciliates B) Rhodophyta C) Choanoflagellates D) Radiolarians E) Myxomycota
5) The polysaccharides agarose and carrageenan are isolated from red algae for commercial use. These polysaccharides are found in cell walls. What plant polysaccharide are they analogous to? A) Amylose B) Collagen C) Cellulose D) Elastin E) Mannose
6)
The Irish potato famine in the 1840s was caused in large part by a proliferation of a(n) A) paramecia. B) oomycote. C) dinoflagellate. D) chlorophyte. E) fungus.
7) What type of cellular damage in a malarial parasite would prevent it from penetrating a red blood cell? A) Mutation in thevariable surface glycoprotein genes B) A disruption of thecytoskeletal proteins C) Failure of thecross-linking of the peptidoglycans D) Damage to thechoanocytes
Version 1
2
8) Which feature unites paramecia, malarial parasites, and dinoflagellates into a single group? A) The use ofcilia B) A parasitic lifestyle C) The presence of anucleus D) The type ofribosomes E) The possession ofalveoli
9) Transferring Euglena into a dark environment results in the loss of photosynthetic pigment. What conclusion does this support? A) Euglenoids are obligate photoautotrophs. B) Light triggers heterotrophy. C) Euglenoids are closely related to plants. D) Euglenoids are neither plant-like or animal-like protists.
10) A patient comes to you in the urgent care center with severe stomach cramps and diarrhea. He was recently swimming in unclean water. Which protist would be the most likely cause of his symptoms? A) Toxoplasma gondii B) Plasmodium C) Leishmaniasis D) Giardia
11) Unlike virtually all other eukaryotes, diplomonads lack mitochondria. Why aren't these organisms considered to represent a state most like the common ancestorto all other eukaroyotes?
Version 1
3
A) They have endosymbiotic relationships with other bacteria. B) They do not use ATP to power their cells. C) They have nonfunctional mitochondrial remnants. D) They are phototrophic.
12) During cell division the progeny of a trypanosome express adifferent VSG gene than their parents. Why is this done? A) To hide from the host's immune system B) To signal endocytosis of the parasites C) VSG genes inhibit phagocytosis. D) VSG genes activate photoheterotrophy.
13)
What would be strong evidence to suggest a protist is phototrophic? A) Many lysosomes B) Locomotion viaseveral flagella C) The presence ofchloroplasts D) Silicon dioxidecell walls E) A benthic lifestyle
14) Which feature of mitochondria provides the least amount of information about their endosymbiotic origins? A) Circularchromosome B) Mode ofreplication C) Independentreplication D) Phospholipids inthe membranes
Version 1
4
15) A biologist uses a time machine to collect living material from two different time periods. Sample A is 3 billion years old and sample B is 1.5 billion years old. How would these samples differ? A) Prokaryotic organisms would be abundant in sample A but absent from sample B. B) Eukaryotic organisms would be present in both samples but much more common in sample B. C) Prokaryotic organisms would only occur in sample B. D) Eukaryotic organisms would be absent from sample A.
16)
The common characteristic of Alveolata is A) a single, unique mitochondria. B) a layer of flattened vesicles beneath the plasma membrane. C) silica covering. D) funnel-shaped, contractile collar. E) presence of a plasmodium.
17)
A common characteristic of Choanoflagellida is A) a single, unique mitochondria. B) space beneath the plasma membrane. C) silica covering. D) funnel-shaped, contractile collar. E) presence of a plasmodium.
18)
The most diverse of the four eukaryotic kingdoms is the A) Protista Kingdom. B) Bacteria Kingdom. C) Plantae Kingdom. D) Animalia Kingdom.
Version 1
5
19)
Amoebas move from place to place by means of their A) cysts. B) pseudopods. C) pseudomonads. D) sporocarps. E) plasmodia.
20)
The marine multicellular protists including the larger brown algae belong to the A) dinoflagellates. B) Choanoflagellida. C) Stramenopiles. D) euglenoids. E) foraminifera.
21) Organisms that are characterized by being unicellular, photosynthetic and having double shells made of silica are known as A) dinoflagellates. B) diatoms. C) kelps. D) euglenoids. E) foraminifera.
22) The tropical diseases such as African sleeping sickness, Chaga's disease, and leishmaniasis are caused by
Version 1
6
A) foraminifera. B) sporozoans. C) phytomastigotes. D) trypanosomes. E) oomycetes.
23) All ________ have two very different types of nuclei within their cells, small micronuclei and larger macronuclei. A) sporozoans B) protists C) oomycetes D) ciliates E) dinoflagellates
24)
Protists are classified into a single kingdom because of which of the following features? A) They are eukaryotes, but not fungi, plants, or animals. B) They are microscopic. C) They are unicellular. D) All types of nutritional modes are found. E) They are monophyletic.
25) The organism, Phytophthora infestans, responsible for the Irish Potato Famine of 1845 and 1847 belongs to A) Euglenozoa. B) Alveolata. C) Rhodophyta. D) Chlorophyta. E) Stramenopila.
Version 1
7
26)
Oomycetes are characterized by all of the following except they A) produce zoospores asexually. B) are able to reproduce sexually. C) contain motile spores. D) contain two unequal flagella. E) are consideredfungi.
27) The White Cliffs of Dover, a famous landmark in England, is made of limestone, which is really a collection of the pore-studded shells (tests) of A) diatoms. B) dinoflagellates. C) brown algae. D) spirochetes. E) foraminifera.
28)
Which of the following is not one of the distinctive features of dinoflagellates? A) They are unicellular. B) They are photosynthetic. C) Some exist in fresh water, while some are part of marine plankton. D) They form large, spherical colonies. E) Some planktonic members are luminous and cause flashes in the night.
29)
Vacuoles for ingesting food and regulating their water balance are characteristic of
Version 1
8
A) sporozoans. B) trypanosomes. C) ciliates. D) algae. E) spirochetes.
30) Conjugating ciliates exchange a pair of haploid micronuclei that undergo mitosis as a diploid micronucleus, while the macronucleus in each A) is expelled from the cell. B) disintegrates. C) divides by meiosis. D) fuses with each other. E) shrinks and becomes diploid micronuclei.
31) Some protists use multiple fission (cell division preceded by several nuclear divisions) as a means of reproduction. Multiple fission is also referred to as A) budding. B) zygotic meiosis. C) schizogony. D) syngamy. E) intermediary meiosis.
32)
Members of the Euglenozoa move within their habitats with the aid of A) tests. B) pseudopodia. C) cilia. D) flagella. E) pellicles.
Version 1
9
33)
In addition to the true amoebas (Rhizopoda), another group with pseudopods are the A) Euglenozoa. B) Stramenopila. C) Radiolarians. D) Diplomonads. E) Alveolata.
34) Which of the following statements would not be consistent with the theory for the endosymbiotic origin of mitochondria and chloroplasts? A) Mitochondria and chloroplasts have their own DNA enclosed within a nuclear envelope. B) The composition of the outer membrane of chloroplasts and mitochondria is similar to the plasma membrane of eukaryotic cells. C) The composition of the inner membrane of chloroplasts and mitochondria is similar to the plasma membrane of prokaryotic cells. D) Mitochondria and chloroplasts have some enzymes unique to them. E) Genes encoded within mitochondria and chloroplasts indicate a closer relationship to prokaryotes than do most nuclear genes.
35) Some photosynthetic euglenoids can also ingest dissolved or particulate food. Which one of the following terms would best describe such a protist? A) Autotrophic B) Heterotrophic C) Mixotrophic D) Osmotrophic E) Phagotrophic
36)
Which of these is not part of the life cycle of brown algae?
Version 1
10
A) Meiosis B) Multicellularhaploid stage C) Multicellulardiploid stage D) Gametes producedby mitosis E) Spores produced bymitosis
37)
Which one of the following statements about diplomonads and parabasilids is false? A) Absence of mitochondria in these groups is an ancestral trait. B) Some members are parasitic, but some also engage in mutualism. C) Both groups are in different clades. D) Both groups are flagellated protists.
38) Which one of the following statements about the trypanosome causing African sleeping sickness does not explain why the disease has been difficult to control? A) Trypanosomes have a protective glycoprotein coat. B) Trypanosomes have many variable antigen genes. C) Rearrangement of antigen genes takes place during the asexual cycle. D) By infesting the red blood cells, trypanosomes can "hide" from the immune system. E) Only one gene determining the antigenic nature of the glycoprotein coat is expressed at a time.
39)
How do humans use red algae polysaccharides? A) Thickeners B) Herbicides C) Adhesives D) Detergents
Version 1
11
40) What features of charophytes provides evidence for charophytes being the closests relative to land plants? A) Charophytespossess plasmodesmata. B) Charophytes arethe only photosynthetic protists. C) Charophytes arefully terrestrial. D) Charophytesproduce seeds and flowers.
41) Which feature would exclude a newly discovered organism from being categorized as a protist? A) Lacking functional mitochondria B) The absence of a nuclear envelope C) Having cellulose cell walls D) Using pseudupodiafor locomotion
42)
The grouping Protista is
A) a valid evolutionary grouping that includes organisms that are more closely related to each other than to other organisms B) A convenient grouping of eukaryotic organisms that have some similarity but are hard to classify C) A group of exclusively microscopic organisms
43) Which feature would be the least useful in determining if a protist were a close relative of plants? A) Cellulose cell walls B) Capacity for photosynthesis C) Plasmodesmata between cells D) Enzymes for ATP production
Version 1
12
44)
An easy way to describe a radiolarian would be to say that it is A) Ameoba in a diatom shell B) A diatom with many flagella C) A red algae trapped in a glass casing D) A paramecium that is capable of photosynthesis
45) The plasmodial and cellular slime molds differ in a number of features. Select the following choice that is not a difference. A) Aggregation of individuals when conditions worsen B) Ability to form spore-producing structures C) Ability to form an acellular plasmodium D) Ability to form individual amoebas E) Synchronization of mitosis
46) Conjugation in most ciliates involves a series of steps that leads to increased genetic diversity. Which one of the following choices is not one of those steps? A) Mitosis of a haploid nucleus B) Mitosis of a diploid nucleus C) Meiosis of a haploid nucleus D) Meiosis of a diploid nucleus E) Sexual reproduction
47)
How do protists provide evidence for the step-wise evolution of mitosis? A) In some protiststhe nuclear membrane fails to breakdown during mitosis. B) Protists use conjugation instead of mitosis. C) Only meiosis occurs in protists. D) Mitosis has not been observedin any protist species.
Version 1
13
48)
What would be strong evidence to suggest a protist is phagotrophic? A) An abundance oflysosomes B) Locomotion usingpolar flagella C) Silicon dioxidecell walls D) Benthiclifestyle E) Presence of chloroplasts
49)
Select the closest relative to a squirrel. A) Plants B) Fungi C) Kelp D) Diatoms E) Green algae F) Bacteria
50)
The concept of endosymbiosis is widely accepted. What kind of relationship is it? A) One organism in the partnership benefits, and the other is harmed. B) Both organisms inthe partnership benefit. C) Neither organism benefits. D) One organism benefits, but the other is not affected.
51) As a freshwater ecologist you are tasked with identifying microscopic species in a local pond. The latest specimen is a multicellular sphere with two flagella on each cell. There appear to be additional spherical colonies within the sphere. To which genus does this organism belong? A) Chlamydomonas B) Chlorella C) Volvox D) Ulva
Version 1
14
FILL IN THE BLANK. Write the word or phrase that best completes each statement or answers the question. 52) The members of ___________ include dinoflagellates, apicomplexans, and ciliates.
Version 1
15
Answer Key Test name: Chap 24_3e 1) TRUE Refer to Section 24.6 for a discussion of Cercozoans. 2) FALSE In fungi and in some groups of protists, the nuclear membrane does not dissolve. Review S/section 24.1. 3) [B, C, D] Review S/section 24.1 4) C
Version 1
16
Clarify Question • What is the key concept addressed by the question? The question is asking you to determine which of the options shares a most recent common ancestor with Porifera. On a family tree, you and your sister would share a more recent common ancestor than you and your cousin. Look at the Eukarya family tree. Which groups have more recent common ancestors with Porifera? • What type of thinking is required? This question is an “apply” question, so it is asking for you to use the knowledge you already have about reading phylogenetic trees to figure out the most recent common ancestor of Porifera. Gather Content • What do you already know about Porifera? How does it relate to the question? o Porifera are in Kingdom Animalia. Where is Kingdom Animalia on the Eukaryote family tree? o Common ancestors are populations that were originally one species, but diverged into two species. They can be seen as nodes on a phylogenetic tree. o More closely related groups tend to have more traits in common. In this case, close relatives to Porifera are likely to have a similar feeding collar. Choose Answer • Given what you now know, what information and/or problemsolving approach is most likely to produce the correct answer? o Now that you know Porifera are in Kingdom Animalia, find them on the Eukaryote family tree. If you have a pencil, circle them. Now trace Version 1
17
your finger back along the branch that becomes Animalia. When it joins another line, you have found a common ancestor. Let’s call that ancestor “Mom.” How many daughters does Mom have? Now trace the line back farther to find the next common ancestor. Let’s call that ancestor “Grandma.” How many granddaughters does Grandma have? Which of the options are a sister to Porifera? Which of the options are a cousin to Porifera? Are there second cousins? Reflect on Process • Did your problem-solving process lead you to the correct answer? If not, where did the process break down or lead you astray? How can you revise your approach to produce a more desirable result? o This question asked to you figure out which group was most closely related to Porifera. To answer this question correctly, you needed to apply your knowledge about phylogenies to the Eukaryote family tree. o If you got the correct answer, great job! Keep using the strategy you used here on other “apply” questions. o If you got the incorrect answer, where did the process break down? Did you remember that the closest relatives share the most recent common ancestor? Did you have trouble reading the phylogenetic tree? Reading phylogenies is difficult, and many students struggle with it at first. You should go back to the chapter on reading phylogenies to brush up. If you are still having difficulty, go talk to your professor or TA. Once you understand phylogenies, questions like this will be a lot easier. 5) C
Version 1
18
Clarify Question • What is the key concept addressed by the question? o This question is asking you to find the molecule that meets two requirements: • It is a plant polysaccharide. • It is analogous to agarose and carrageenan. • What type of thinking is required? o This question is an “analyze” question, so it is asking you to break the question apart into a list of requirements. Then, you use what you’ve learned so far to figure out which answer meets all of the requirements. Gather Content • What do you already know about agarose and carrageenan? How does this help you decide which statements are true? o Agarose and carrageenan are polysaccharides that make up the cell walls of red algae. o Polysaccharides are very long chains of sugar molecules. Starch is another polysaccharide.Which of the possible answers is a polysaccharide? The others might be proteins or fats. • What other information do you need to determine the answer? o The correct answer is • a polysaccharide • found in plants • is analogous to agarose and carrageenan in red algae. o Now that you’ve determined which of the answers are polysaccharides, you should figure out which are found in plants. Choose Answer • Given what you now know, what information and/or problemVersion 1
19
solving approach is most likely to produce the correct answer? o The best way to answer this type of question is to break the question into smaller parts. Then, figure out which answer is correct for all three parts. o You figured out which options were polysaccharides. o You then figured out which of those polysaccharides occur in plants. o You now need to figure out which are analogous to agarose and carrageenan in red algae. What role do those compounds play in red algae? They make up the cell wall. Which of the options make up plant cell walls? Reflect on Process • Did your problem-solving process lead you to the correct answer? If not, where did the process break down or lead you astray? How can you revise your approach to produce a more desirable result? o This question asked you to determine which polysaccharid in plants is analogous to agarose and carrageenan in red algae. Answering this question required you analyze each option and determine whether it fulfilled all three parts of the question. It also required you to combine your knowledge of red algae with your knowledge of macromolecules and plants. o If you got the correct answer, great job! o If you got an incorrect answer, where did your process break down? • Did you have trouble breaking down the question into three smaller components? 6) B 7) B
Version 1
20
Clarify Question • What is the key concept addressed by the question? o This question is asking you to determine which type of damage to the Plasmodium cell would prevent it from entering a red blood cell. Many of the options would damage the Plasmodium, possibly killing it, but only one would prevent it from entering a red blood cell. Gather Content • What type of thinking is required? o This question is an “apply” question, so it is asking for you to take a concept you already know (the anatomy of Plasmodium) and apply it to a new scenario to solve the problem. • What do you already know about Plasmodium? How does it relate to the question? o Plasmodium is a member of the Apicomplexans. This group of protists is able to invade host cells using the apical complex. How does the apical complex work? What sort of damage would prevent the apical complex from working? Choose Answer • Given what you now know, what information and/or problem solving approach is most likely to produce the correct answer? o Plasmodium is able to infect red blood cells using its apical complex. How does the apical complex work? • Does it use surface proteins? • Does it use the cytoskeleton? • Does it depend on peptidoglycans? • Does it use choanocytes? o You might be able to rule out some of the answers if they do not occur Version 1
21
in protists. Reflect on Process • Did your problem-solving process lead you to the correct answer? If not, where did the process break down or lead you astray? How can you revise your approach to produce a more desirable result? o This question asked you to determine which type of damage to Plasmodium would prevent it from infecting red blood cells. Answering this question required you understand how an apical complex works. You then had to apply this information to determine which type of damage would prevent the apical complex from working. o If you got the correct answer, great job! o If you got an incorrect answer, where did you get stuck? • Did you remember that peptidoglycans only occurs in Bacteria? • Did you remember that the apical complex depends on the cytoskeleton and secretory glands? 8) E
Version 1
22
Clarify Question • What is the key concept addressed by the question? o This question asks you to compare what you know about paramecia, malarial parasites, and dinoflagellates. What characteristics do these three taxa have in common? • What type of thinking is required? o This question is an “apply” question, so it is asking for you to take a concept you already know (each of these taxa individually) and apply it to a new scenario to solve the problem. Gather Content • What do you already know about each of these three groups? How does it relate to the question? o Do all three have cilia? "A nucleus Alveoli?" o Are they all parasitic? o Do they all have the same type of ribosomes? • What other information is related to the question? Which information is most useful? o Once you have figured out the traits all three groups have in common, you need to address the other part of the question: Which of these traits unites the three taxa on the phylogenetic tree? Choose Answer • Given what you now know, what information and/or problemsolving approach is most likely to produce the correct answer? o Look at the Protist family tree. Which traits do these three taxa have but the other Protists do not? All three taxa probably have mitochondria, but so do the other Protists, so that feature isn’t particularly useful when grouping taxa on the family tree. Version 1
23
Reflect on Process • Did your problem-solving process lead you to the correct answer? If not, where did the process break down or lead you astray? How can you revise your approach to produce a more desirable result? o This question asked you to determine which of the traits unites the three taxa on the phylogenetic tree. Answering this question required you to: • compare the traits of the three taxa to see which traits they have in common • compare the three taxa to the other protists to see which traits are unique to these three taxa. o If you got the correct answer, great job! o If you got an incorrect answer, where did you get stuck? • Did you choose “the presence of a nucleus”? While paramecia, sporozoans, and dinoflagellates all have nuclei, so do all of the other protists. This trait cannot be used to group these three taxa on the tree. • Did you look at the tree? Did you notice that these three taxa are grouped under Alveolata? The names of groups can give you a hint as to the distinguishing feature that grouped the taxa together. 9) D
Version 1
24
Clarify Question • What is the key concept addressed by the question? o This question is asking you to determine which statement is supported by the results of an experiment where Euglena was moved from light to dark. • What type of thinking is required? o This is an “evaluate” question, so you need to look at each option and evaluate whether the results of your experiment support your conclusion. Remember, you moved Euglena from the light into the dark and saw the photosynthetic pigment disappear. Gather Content • What do you already know about Euglena? How does it relate to the question? o What sort of photosynthetic pigments have you learned about? One is chlorophyll, which is essential for photosynthesis. The very first step of photosynthesis involves chlorophyll absorbing energy from sunlight, so it is a very useful pigment. On the other hand, maintaining chlorophyll uses up energy. If a plant does photosynthesis intermittently, during nonphotosynthetic periods it has to use up valuable sugars maintaining its chlorophyll. • What other information is related to the question? Which information is most useful? o What are photoautotrophs? How are they different from heterotrophs? Choose Answer • Given what you now know, what information and/or problemsolving approach is most likely to produce the correct answer? o You can go about answering this question in a couple of ways: Version 1
25
• You have four hypotheses. What experiment would you conduct to test each one? How well does the experiment described in the question’s stem match those methods? • You have observed that Euglena has photosynthetic pigment when growing in the light but digests this pigment when growing in the dark. Which conclusion best explains what is going on? o Be careful when evaluating each option. Some of the statements themselves can be correct without being the correct answer. The correct answer is supported by the results of your experiment. Reflect on Process • Did your problem-solving process lead you to the correct answer? If not, where did the process break down or lead you astray? How can you revise your approach to produce a more desirable result? o This question asked you to evaluate which statement was supported by the results discussed in the stem of the question. Answering the question depended on your ability to evaluate each of the statements. o If you got the correct answer, well done! o If you got an incorrect answer, where did the process break down? Did you pick “Light triggers heterotrophy”? Remember, you moved Euglena to the dark, triggering heterotrophy. This statement would be correct if you said “Dark triggers heterotrophy.” 10) D
Version 1
26
Clarify Question • What is the key concept addressed by the question? o The quesiton addresses your knowledge of parasitic and pathogenic protists. • What type of thinking is required? o This is an “apply” question, so you need to apply your knowledge about the various parasitic and pathogenic protists to the patient's case. Gather Content • What do you already know about these organisms? How does it relate to the question? o Toxoplasma gondii: Infection usually occurs by eating undercooked contaminated meat, exposure from infected cat feces, or mother-to-child transmission during pregnancy. o Plasmodium:This causes malaria, and has a complex life cycle that alternates between mosquitoes and mammals. o Leishmaniasis: Thistrypanosomic diseaseis transmitted by sand flies. o Giardia: Thisdiplomonad can pass from human to human via contaminated water and cause diarrhea. Choose Answer • Given what you now know, what information and/or problemsolving approach is most likely to produce the correct answer? o You know two things about the parasite: • It can be transmitted by water (particularly unclean water) • It causes stomach cramps and diarrhea. o Which of the organisms meets both criteria? Reflect on Process • Did your problem-solving process lead you to the correct answer? If Version 1
27
not, where did the process break down or lead you astray? How can you revise your approach to produce a more desirable result? o If you got the correct answer, well done! o If you got an incorrect answer, where did the process break down? Did you remember that only oneof the mentioned protists is most commonly found in unclean water? 11) C 12) A 13) C
Version 1
28
Clarify Question • What is the key concept addressed by the question? o This question gives you five characteristics of a protist. You need to determine which characteristic would tell you that the protist is phototrophic. • What type of thinking is required? o This question is an “evaluate” question, so it is asking you to judge which of the statements provides the best evidence of phototrophy. All of the statements could be correct, but only one provides evidence of phototrophy. Gather Content • What do you already know about the nutritional strategies of protists? How does it relate to the question? o Protists can be autotrophic, heterotrophic, or mixotrophic. o Autotrophs get their energy from photosynthesis or chemosynthesis. An organism that does photosynthesis would be called “phototrophic.” What characteristics would you expect in a protist that does photosynthesis? o Heterotrophs get their energy by eating things, but single-celled organisms don’t eat things the way we do. An amoeba is an example of a heterotrophic protist. How does an amoeba get food? o Mixotrophs, such as Euglena, can use either photosynthesis or heterotrophy. • What other information is related to the question? Which information is most useful? o Think of the different protists that do photosynthesis (for example, green algae and euglena). Is there a trait they all share?
Version 1
29
Choose Answer • Given what you now know, what information and/or problemsolving approach is most likely to produce the correct answer? o You know phototrophy is a nutritional strategy, and that it involves getting energy through photosynthesis. Which characteristic would be essential for a protist that wants to do photosynthesis? Other statements could still be true about the organism, but one of them is essential. Reflect on Process • Did your problem-solving process lead you to the correct answer? If not, where did the process break down or lead you astray? How can you revise your approach to produce a more desirable result? o This question asked you to determine which characteristics would be essential for an organism that is phototrophic. o If you got the correct answer, great job! o If you got an incorrect answer, where did you get stuck? • This was an “evaluate” question, which required you to weigh and judge which piece of evidence would convince you that the protist is phototrophic. Some protists, like red algae, are able to do photosynthesis with a different organelle, but most use chloroplasts. Of the options you were given, “the presence of chloroplasts” is the best answer because that organelle would allow a protist to do photosynthesis. 14) D
Version 1
30
Clarify Question • What is the key concept addressed by the question? o This question is asking you to evaluate four categories of evidence for endosymbiosis and determine which one is the weakest. • What type of thinking is required? o This question is an “evaluate” question, so it is asking for you to weigh and compare the different answers to determine which one provides the poorest evidence for endosymbiosis. Gather Content • What do you already know mitochondria and endosymbiosis? How does it relate to the question? o Mitochondria have many traits that are similar to bacteria. What are they? Some of them involve cell processes, such as replication, while others are characteristics of the cell itself. o What happens to mitochondria during mitosis? Choose Answer • Given what you now know, what information and/or problemsolving approach is most likely to produce the correct answer? o Go through each possible answer and write down how it shows that mitochondria evolved by endosymbiosis. Some things, like DNA and cellular processes, are less likely to evolve multiple times. If mitochondria and bacteria share these traits, they are most likely due to homology instead of convergent evolution. Reflect on Process • Did your problem-solving process lead you to the correct answer? If Version 1
31
not, where did the process break down or lead you astray? How can you revise your approach to produce a more desirable result? o This question asked you to determine which of the options provided the weakest evidence for endosymbiosis. o If you got the correct answer, great job! o If you got an incorrect answer, where did you get stuck? • Did you have a hard time judging the options? The textbook highlights the best evidence for endosymbiosis, and it does not mention phospholipids. It is true that the phospholipids on the inner membrane of mitochondria are similar to bacteria, but phospholipids easily change. Things like a circular chromosome, binary fission, and independent replication do not change easily, so they are most likely a homology with present-day bacteria. 15) D
Version 1
32
Clarify Question • What is the key concept addressed by the question? o Biodiversity is the number of species present and how abundant each species is. This question asks you to compare and contrast the biodiversity present on Earth 3 billion years ago with biodiversity present 1.5 billion years ago. • What type of thinking is required? o This question is an “evaluate” question. You should use the information you learned from the text to evaluate which statement is the most accurate. Gather Content • What do you already know about Prokaryotes and Eukaryotes? How does it relate to the question? o When did Prokaryotes evolve? When did they become abundant? o When did Eukaryotes evolve? When did they become abundant? Choose Answer • Given what you now know, what information and/or problemsolving approach is most likely to produce the correct answer? o Use your answers to the questions in “Gather Content” to predict what would be in each sample. Then, figure out which option in the question best matches your predictions. Reflect on Process • Did your problem-solving process lead you to the correct answer? If not, where did the process break down or lead you astray? How can you revise your approach to produce a more desirable result? Version 1
33
o This question asked you to determine what would be present in two different biological samples, one from 3 billion years ago and one from 1.5 billion years ago. You needed to use your understanding of Prokaryote and Eukaryote evolution to determine the best answer. o If you got the correct answer, great job! o If you got an incorrect answer, where did you get stuck? • The options for this question are complicated because you are answering for both Prokaryotes and Eukaryotes. For a question like this, it is best to first treat it like a short answer question. Ignore the multiplechoice options and just write down your answer. Then, compare each option to your answer and pick the one that matches. 16) B 17) D 18) A 19) B 20) C 21) B 22) D 23) D 24) A 25) E 26) E 27) E 28) D 29) C 30) B 31) C 32) D 33) C 34) A Version 1
34
Clarify Question • What is the key concept addressed by the question? • Three of the options are evidence that mitochondria and chloroplasts became organelles through endosymbiosis. You need to find the option that does not provide evidence of endosymbiosis. • What type of thinking is required? o This question is an “evaluate” question. You need to judge which of the options provides the poorest evidence. Gather Content • What do you already know about the origin of mitochondria and chloroplasts? How does it relate to the question? o Mitochondria and chloroplasts were once free-living bacteria. They were engulfed by a larger cell and became endosymbionts. Over a very long period of time, they lost the ability to be free-living and became organelles. What traits do mitochondria and chloroplasts share with bacteria? Choose Answer • Given what you now know, what information and/or problemsolving approach is most likely to produce the correct answer? o The endosymbiosis theory states that mitochondria and chloroplasts were once bacteria that were engulfed by a larger, eukaryotic cell. Which option is inconsistent with that theory? Which option does not suggest that mitochondria and chloroplasts were once free-living bacteria? Reflect on Process
Version 1
35
• Did your problem-solving process lead you to the correct answer? If not, where did the process break down or lead you astray? How can you revise your approach to produce a more desirable result? o This question asked you to determine which statement would not be possible if mitochondria and chloroplasts evolved from endosymbiotic bacteria. Bacteria do not have nuclear envelopes (neither do mitochondria or chloroplasts). o If you got the correct answer, great job! o If you got an incorrect answer, where did you get stuck? • The options were very wordy. Did you have trouble comparing them? Try rewriting them or drawing pictures to help you quickly see what each option is saying. That will help you compare them. 35) C 36) E 37) A 38) D 39) A 40) A 41) B Refer to Section 24.2 for an explanation. 42) B Refer to section 24.3 43) D Refer to Section 24.7 for a discussion of protist/plant relations. 44) A Refer to section 24.6 for a discussion of Radiolarians 45) B
Version 1
36
Clarify Question • What is the key concept addressed by the question? o The question asks you to compare and contrast the traits of plasmodial slime mold and cellular slime mold. • What type of thinking is required? o This question is an “apply” question. You need to take the traits from each group and compare them. Gather Content • What do you already know about these two organisms? How does it relate to the question? o Plasmodial slime mold has many nuclei in a single, giant cell, while cellular slime mold has one nucleus per cell. What are other differences between the two groups? Are there any similarities? Choose Answer • Given what you now know, what information and/or problemsolving approach is most likely to produce the correct answer? o The best way to answer this question is to make two lists, one with the traits for plasmodial slime mold and one with the traits of cellular slime mold. You can use the options from the question to make your list, but don’t depend on them at first. Figure out at least two similarities between the two taxa. Are either of those similarities an option? o Alternately, make a table with the options on the left and a column for each taxon. Figure out which of the two taxa has each characteristic. For example, cellular slime mold individuals aggregate when conditions worsen. Do plasmodial slime mold individuals do the same? Eventually, you will get to an option that applies to both taxa.
Version 1
37
Reflect on Process • Did your problem-solving process lead you to the correct answer? If not, where did the process break down or lead you astray? How can you revise your approach to produce a more desirable result? o This question asked you to compare and contrast plasmodial slime mold with cellular slime mold. o If you got the correct answer, great job! o If you got an incorrect answer, where did you get stuck? • Did you have trouble finding similarities between the two groups? Learning to compare and contrast groups is a great way to remember they characteristics. You will remember traits of plasmodial slime mold better if you can remember how it is similar to other protists and how it is different from other protists. 46) C
Version 1
38
Clarify Question • What is the key concept addressed by the question? o This question asks you to determine which steps are part of conjugation. One of the options is not part of conjugation. • What type of thinking is required? o This question is an “apply” question. You need to use your knowledge of conjugation, mitosis, and meiosis to answer the question. Gather Content • What do you already know about mitosis and meiosis? How does it relate to the question? o Mitosis duplicates a nucleus, so a diploid nucleus would produce two diploid nuclei. o Meiosis duplicates, then divides a nucleus, so a diploid nucleus would produce four haploid nuclei. o Do both of these occur in conjugation? When do they occur? Choose Answer • Given what you now know, what information and/or problemsolving approach is most likely to produce the correct answer? o Go through each option and see first whether it makes sense. Then, think about whether it occurs in conjugation. Reflect on Process • Did your problem-solving process lead you to the correct answer? If not, where did the process break down or lead you astray? How can you revise your approach to produce a more desirable result? o This question asked you to determine which steps occur during Version 1
39
conjugation and which do not. o If you got the correct answer, great job! o If you got an incorrect answer, where did you get stuck? • Meiosis cannot happen to a haploid nucleus because there is nothing to divide. Meiosis can only happen to a nucleus that has 2, 4, 6, etc. copies of the chromosomes. 47) A
Version 1
40
Clarify Question • What is the key concept addressed by the question? o This question is asking you to explain which part of Protist mitosis shows that mitosis evolved step by step, not all at once. • What type of thinking is required? o This question is an “apply” question, so you need to apply what you already know about Protist mitosis to a new situation. It is also an “evaluate” question because you need to assess which statement provides the best evidences of a step-wise evolution of mitosis. Gather Content • What do you already know about mitosis in Protists? How does it relate to the question? o Prokaryotes divide by binary fission, while Eukaryotes divide by mitosis. Protists divide using an interesting early version of mitosis. In what ways is their cell division similar to Eukaryotes? In which ways is it different? • What other information is related to the question? Which information is most useful? o Generally, in Eukaryotes, mitosis occurs in the following steps: • chromosomes replicate and condense • the nuclear envelope breaks down • chromosome pairs are separated and moved to either side of the cell • two nuclear envelopes form • the cell divides (cytokinesis) o Which of these steps is present in Protists? Which are absent? Choose Answer • Given what you now know, what information and/or problemVersion 1
41
solving approach is most likely to produce the correct answer? o Review what you know about Protists. Do they do mitosis? Do they do meiosis? o What is conjugation? What steps does it involve? Reflect on Process • Did your problem-solving process lead you to the correct answer? If not, where did the process break down or lead you astray? How can you revise your approach to produce a more desirable result? o This question asked you to find the statement that provides the best evidence that mitosis evolved step-by-step (Merriam-Webster's 11th edition). o If you got the correct answer, great job! o If you got an incorrect answer, where did you get stuck? • Did you remember that Protists use both conjugation and mitosis? Look at the diagram of conjugation, and you’ll see that mitosis is part of conjugation. 48) A
Version 1
42
Clarify Question • What is the key concept addressed by the question? o This question gives you five characteristics of a protist. You need to determine which characteristic would tell you that the protist is phagotrophic. • What type of thinking is required? o This question is an “evaluate” question, so it is asking you to judge which of the statements provides the best evidence of phagotrophy. All of the statements could be correct, but only one provides evidence of phagotrophy. Gather Content • What do you already know about the nutritional strategies of protists? How does it relate to the question? o Protists can be autotrophic, heterotrophic, or mixotrophic. o Autotrophs get their energy from photosynthesis or chemosynthesis. What characteristics would you expect in a protist that does photosynthesis? o Heterotrophs get their energy by eating things, but single-celled organisms don’t eat things the way we do. An amoeba is an example of a heterotrophic protist. How does an amoeba get food? o Mixotrophs, such as Euglena, can use either photosynthesis or heterotrophy. • What other information is related to the question? Which information is most useful? o A similar word you may have heard before is “phagocytosis.” Do you remember this vocabulary term from cellular biology? Choose Answer Version 1
43
• Given what you now know, what information and/or problemsolving approach is most likely to produce the correct answer? o You know phagotrophy is a nutritional strategy and that it involves eating things. Which characteristic would be essential for a protist that wants to eat a food particle? Other statements could still be true about the organism, but one of them is essential. Reflect on Process • Did your problem-solving process lead you to the correct answer? If not, where did the process break down or lead you astray? How can you revise your approach to produce a more desirable result? o This question asked you to determine which characteristics would be essential for an organism that is phototrophic. o If you got the correct answer, great job! o If you got an incorrect answer, where did you get stuck? • This was an “evaluate” question, which required you to weigh and judge which piece of evidence would convince you that the protist is phagotrophic. 49) B
Version 1
44
Clarify Question • What is the key concept addressed by the question? o This question addresses the phylogenetic relationships between the provided options and animals. • What type of thinking is required? o This question is an “apply” question, so it is asking for you to take a concept you already know and apply it to a new scenario to solve the problem. The best way to solve this question is to look at a phylogenetic tree (e.g. page533) that contains all of the groups listed in the question. Gather Content • What do you already know about squirrels? How does it relate to the question? o Squirrels are mammals, like us, so they are in Kingdom Animalia, in Domain Eukarya. Which of the organisms listed is also in Domain Eukarya? Are any of them in Animalia? • What other information is related to the question? Which information is most useful? o Look at the protist family tree in Chapter 29 (also known as a phylogenetic tree). How do you know which organisms are most closely related? You can use the branches of the tree to do this. For example, Land Plants are most closely related to Charophytes. The group “Land Plants and Charophytes” is most closely related to Chlorophytes (green algae). Choose Answer • Given what you now know, what information and/or problemsolving approach is most likely to produce the correct answer? o Animals andChoanoflagellidaboth share a common ancestor withFungi; they comprisethe supergroup Opisthokonta. Version 1
45
Reflect on Process • Did your problem-solving process lead you to the correct answer? If not, where did the process break down or lead you astray? How can you revise your approach to produce a more desirable result? o This question asked you to determine which group is most closely related to squirrels. You needed to use your understanding of phylogenetic trees and apply it to a new situation. o If you got the correct answer, great job! o If you got an incorrect answer, where did you get stuck? • Phylogenetic trees can be hard to read at first, but once you understand them, this type of question will be easy. Go back to the chapter on reading phylogenetic trees, then try to answer this question again. If you are still having difficulty, work through some problems with your teacher or TA. The best way to get comfortable with phylogenetic trees is to have a lot of practice with them. 50) B
Version 1
46
Clarify Question • What is the key concept addressed by the question? o The question is asking you whether, in endosymbiosis, both partners benefit. • What type of thinking is required? o This question is an “apply” question, so it is asking for you to take a concept you already know (endosymbiosis) and apply it to a new scenario to solve the problem. Gather Content • What do you already know about organism interactions? How does it relate to the question? o There are three types of symbiosis interactions: mutualism, parasitism, and commensalism. In mutualism, both partners benefit. In parasitism, one partner benefits while the other is harmed. In commensalism, one partner benefits but has no affect on the other. o The question is asking whether endosymbiosis is an example of mutualism or parasitism. Choose Answer • Given what you now know, what information and/or problemsolving approach is most likely to produce the correct answer? o Think about the first endosymbiosis that led to mitochondria. How did this arrangement benefit the bacterium? How did it benefit the eukaryotic cell? How did it harm the bacterium? How did it harm the eukaryotic cell? o Overall, do you think each partner had a net benefit or a net harm? Reflect on Process
Version 1
47
• Did your problem-solving process lead you to the correct answer? If not, where did the process break down or lead you astray? How can you revise your approach to produce a more desirable result? o This question asked you to determine whether endosymbiosis was an example of mutualism or parasitism. To answer this question, you needed to understand that the larger cell provided protection, while the smaller cell provided energy. Both received a net benefit. o If you got the correct answer, great job! o If you got an incorrect answer, where did you get stuck? • Did you have trouble thinking of mitochondria as an independent organism? 51) C Refer to S/section 24.7. All are green algae but only one of the genera features elaborate multicellular colonies. 52) Alveolata
Version 1
48
CHAPTER 25 TRUE/FALSE - Write 'T' if the statement is true and 'F' if the statement is false. 1) Primary mycelia contain twice as much DNA as secondary mycelia. ⊚ ⊚
true false
2) Recent phylogenetic analysis has provided evidence to justify classifying microsporidians as fungi. ⊚ ⊚
3)
true false
Zygospores are used to resist harsh conditions. ⊚ ⊚
true false
4) Comparisons of DNA sequences have revealed that glomeromycetes are not in the same clade as the zygomycetes. ⊚ ⊚
true false
CHECK ALL THE APPLY. Choose all options that best completes the statement or answers the question. 5) Select any true statements about the Chytridiomycota. A) Most members of Chytridiomycota are terrestrial pathogens. B) Massive amphibian die-offs have been caused by a species of Chytridiomycota. C) They produce flagellated, haploid zoospores. D) They are named for the shape of their zoosporangia. E) They are currently a paraphyletic group.
Version 1
1
MULTIPLE CHOICE - Choose the one alternative that best completes the statement or answers the question. 6) The fungal group that is most closely related to ancestral fungi are the A) Zygomycota. B) Chytridomycota. C) Neocallimastigomycota. D) Blastocladiomycota. E) Microsporidia.
7) Baker's yeast is a single-celled ascomycete that is used in bread making. Why does yeast make bread dough rise? A) Yeast cells take in carbon dioxide, causing them to swell and the dough to expand. B) Yeast cells produce carbon dioxide during fermentation, causing the dough to expand. C) Exponential growth of the yeast cells physically expands the dough. D) The fungal cells burst and release spores, causing the dough to expand.
8)
Why are neocallimastigomycetes being studied for biofuel production? A) If provided with an aerobic environment, they can break down cellulose and lignin. B) They can break down cellulose and lignin in a hot, anaerobic environment. C) They can use their hydrogenosomes to generate methane. D) They can degrade pollutants and toxinsinto benign substances.
9) In leaf cutter ant colonies, the ants clip leaves to feed a fungus garden and then the ants feed on the fungus. Is this a mutualistic relationship?
Version 1
2
A) No, the ants are pathogens of both the trees from which they clip the leaves and the fungus. B) Yes, the ants and the trees are in a mutualistic relationship. C) No, the ants are parasites of the trees from which they clip the leaves, but have a commensal relationship with the fungus. D) Yes, the ants and the fungus are engaged in a mutualistic relationship.
10) Alexander Fleming kept finding mold contamination that was killing his bacterial cultures. This observation eventually lead to the development of the antibiotic penicillin. Why would the fungi produce a compound that is selectively toxic to bacteria? A) The fungi are parasites of the bacteria. B) The bacteria produce toxins that can kill the fungi. C) The fungi compete with the bacteria for resources. D) The fungi and the bacteria have a symbiotic relationship.
11) Many antibacterial antibiotics interfere with protein synthesis. Why wouldn't this strategy be suitable in the selection of antifungal drugs? A) Fungi produce few proteins. B) Fungi can rapidly metabolize drugs. C) Chitin cell walls prevent uptake of drugs. D) Human and fungal proteins are too similar. E) Fungi can counteract such drugs by absorbing new proteins during decomposition.
12)
What type of symbiotic relationship is exemplified by mycorrhizal fungi? A) Mutualism B) Commensalism C) Parasitism D) Atavism
Version 1
3
13) Which gene product would be most useful for distinguishing between a microsporidian and a blastocladiomycete? A) Rubisco B) Chitin C) DNA polymerase D) Phospholipidase E) ATP synthase
14)
What type of symbiotic relationship is exemplified by athlete's foot? A) Mutualism B) Commensalism C) Parasitism D) Atavism
15)
In what way are bacteria and fungi similar? A) Ecologicalfunction B) Cell wallcomponents C) Design of thenucleus D) Ribosomestructure
16)
Why are fungi used to clean oil spills? A) Fungi use the hydrocarbons in oil instead of CO 2 during photosynthesis. B) The chemicals in oil kill off the fungus's competitors. C) Fungi use the organic molecules in the oil as food. D) Fungi store the hydrocarbons in their large central vacuoles.
17)
Why are microsporidia so hard to classify?
Version 1
4
A) When compared to other eukaryotic organisms, microsporidia are extremely small. B) The intracellular parasitic lifestyle greatly reduces the need for functional organelles. C) They lack nuclei. D) They are very rare.
18)
Which molecule would be an indicator of fungal contamination in a drug preparation? A) Cellulose B) Collagen C) Chitin D) Deoxyribose E) Lignin
19)
Specialized symbiotic associations between the roots of plants and fungi are called A) lichens. B) hyphal associations. C) heterokaryotic junctions. D) mycorrhizae. E) a mycelial unit.
20)
A mass of hyphae in an ascomycete is commonly referred to as a(n) A) mycelium. B) mycorrhizae. C) conidium. D) sporangium. E) ascus.
21)
The cell walls of fungi are made up of polysaccharides including
Version 1
5
A) polyglycans. B) phospholipids. C) bilipid layers. D) glycolipids. E) chitin.
22)
Fungi are ecologically important for all of the following reasons, except which one? A) They break down organic materials and return them to the ecosystem. B) All can exist in symbiotic relationships. C) Some can ferment and produce bread, beer, wine, cheese, and soy sauce. D) Some cause diseases in plants and animals. E) They can break down lignin.
23) a
A symbiotic association between a fungus and a green alga or a cyanobacterium is called
A) lichen. B) mycorrhiza. C) protist. D) kelp. E) mushroom.
24) a(n)
The system of fungal filaments that penetrates and grows through the substrate is called
A) exoskeleton. B) hyphal compartment. C) mushroom. D) mycelium. E) sporangium.
Version 1
6
25) Mycorrhizae, specialized symbiotic associations between the roots of plants and fungi, are characteristic of A) all plants. B) about 90% of all plants. C) about 60% of all plants. D) about 40% of all plants. E) about 20% of all plants.
26)
Diploid cells of animals and plants are genetically similar to fungal A) monokaryotic hyphae. B) dikaryotic hyphae (1 n+ 1n). C) homokaryotic hyphae. D) spores. E) triploid cells.
27)
All of the following are reproductive structures found in some fungi except A) ascus. B) basidium. C) sporangiophore. D) mycorrhiza. E) conidia.
28) A division of fungi that lacks septa in their hyphae except when they form sporangia or gametangia is the
Version 1
7
A) ascomycetes. B) zygomycetes. C) chytridomycetes D) basidiomycetes.
29) The fungi that cause serious plant disease, such as Dutch elm disease and chestnut blight, belong to the phylum A) Ascomycota. B) Zygomycota. C) Glomeromycota. D) Basidiomycota. E) Actinomycetes.
30)
Asexual reproduction in ascomycetes takes place by means of A) ascogonia. B) ascospores. C) conidia. D) motile gametes. E) zygospores.
31)
A maturewalled ascus of an ascomycete usually contains A) eight diploid nuclei. B) eight haploid nuclei. C) many multinucleate conidia. D) an egg and a sperm. E) a zygote.
32)
The most economically useful unicellular fungi are
Version 1
8
A) basidiomycetes. B) actinomycetes. C) yeasts. D) mushrooms.
33)
The fruiting structures formed by secondary mycelia of basidiomycetes are known as A) mushrooms. B) yeasts. C) lichen. D) ascocarps. E) basidia.
34) by
The only diploid cell of the life cycle of basidiomycetes is produced within the basidium
A) mitosis. B) meiosis. C) karyogamy. D) ascus bursting. E) budding.
35)
The rusts and smuts that seriously affect crop plants are A) ascomycetes. B) zygomycetes. C) chytrids. D) basidiomycetes. E) actinomycetes.
36)
Lichens are adversely affected by
Version 1
9
A) humidity. B) sunlight. C) pollution. D) sexual reproduction of the fungal partner. E) asexual reproduction of the photosynthetic partner.
37) Which of the following fungi usually form arbuscular mycorrhizal associations in which the outer cells of the plant roots are penetrated by the fungal hyphae? A) Ascomycetes B) Glomeromycetes C) Deuteromycetes D) Basidiomycetes E) Chytridiomycetes
38) one?
Arbuscular mycorrhizae are characterized by all of the following features, except which
A) They are the most common mycorrhizal type. B) Fungal hyphae penetrate plant root cells. C) They are seen in the earliest vascular plant fossils. D) The fungus can associate with several species of plants. E) The fungal component is a glomeromycete.
39) Ectomycorrhizae, associations in which the fungal hyphae surround but do not penetrate the cell walls of the plant roots, usually involve A) ascomycetes. B) basidiomycetes. C) zygomycetes. D) ascomycetes and basidiomycetes. E) ascomycetes and zygomycetes.
Version 1
10
40) A mycologist visits your biology class and discusses her research on a particular fungus that lives on the bark of damaged spruce trees. She explains that the fungal hyphae of her research organism is heterokaryotic. You raise your hand and request that she define that word. Her correct response is A) heterokaryotic means that the nuclei of the fungal hyphae are genetically the same. B) heterokaryotic means that the nuclei of each compartment of the hyphae are genetically distinct. C) heterokaryotic means that each compartment of the hyphae contain two distinct nuclei. D) heterokaryotic means that each compartment of the hyphae contains only one nucleus.
41)
Select the incorrect statement related to fungi.
A) All fungi secrete digestive enzymes and practice external digestion. B) Some fungi can break down cellulose into its glucose subunits. In addition, some fungi are predaceous. C) One method of reproduction in fungi is by flagellated sperm. Water is a requirement for those particular fungi to reproduce. D) Some fungi are able to grow to many hectares in size. E) Cheese, wine, and soy sauce are but a few examples of foods dependent on fungi for their production.
42)
Lichens are mutualisms between A) yeast and another fungi. B) fungi and an alga. C) fungi and a green plant. D) fungi and a mycorrhizae. E) mycorrhizae and yeast.
43)
Which of the phyla of fungi is incorrectly matched with its example?
Version 1
11
A) Ascomycota—yeast B) Basidiomycota—rust C) Zygomycota—bread mold D) Ascomycota—mushrooms E) Basidiomycota—toadstools
44)
One major difference between ectomycorrhizae and arbuscular mycorrhizae is A) ectomycorrhizae are parasitic on plant roots while arbuscular mycorrhizae are not. B) ectomycorrhizae do not penetrate plant roots; arbuscular mycorrhizae penetrate plant
roots. C) ectomycorrhizae are more common on plant roots than arbuscular mycorrhizae. D) ectomycorrhizae form mutualisms while arbuscular mycorrhizae are parasitic. E) most ectomycorrhizae are zygomycetes while most endomycorrhizae are basidiomycetes.
45)
Ruminant animals have symbiotic fungi that A) digest cellulose and lignin. B) infect milk-producing glands, fermenting the milk. C) produce toxins in muscle tissue. D) produce natural antibiotics.
46) Which one of the following fungal groups has an alternation of multicellular gametophyte and sporophyte generations? A) Zygomycetes B) Ascomycetes C) Blastocladiomycota D) Chytridiomycetes E) Glomeromycetes
Version 1
12
47)
In the life cycle of a zygomycete, after the zygospore is formed, the next event will be A) mitosis. B) meiosis. C) plasmogamy. D) karyogamy. E) dikaryon formation.
48) A nonphotosynthetic plant sharing a mycorrhizal fungus with a photosynthetic plant is best described as a A) mutualist. B) epiparasite. C) parasite. D) host. E) symbiont.
49) Which one of the following terms best applies to the fungus participant in a typical lichen? A) Mutualist B) Ectomycorrhizal fungus C) Arbuscular mycorrhizal fungus D) Epiparasite
50)
How does fungal mitosis differ from animal mitosis? A) Cytokinesis onlyoccurs in fungal mitosis. B) The nuclearenvelope remains intact in fungal mitosis. C) The spindleapparatus is unique to fungal mitosis. D) Fungi only undergo meiosis; true mitosis is only found in animals.
Version 1
13
51)
In basidiomycetes, the _______ is diploid. A) basidium B) primarymycelium C) secondarymycelium D) zygote
52) The fungus Penicillium produces a compound that blocks peptidoglycan assembly. Why is a fungus trying to kill bacteria? A) Fungi eat bacteria. B) Fungi and bacteria are both decomposers and compete for resources. C) The compound produced by the fungus is used for nitrogen fixation, the antibacterial properties are accidental. D) The fungal compound is produced in response to a bacterial invasion of the fungal hyphae.
FILL IN THE BLANK. Write the word or phrase that best completes each statement or answers the question. 53) During asexual reproduction, hyphae produce clumps of erect stalks, called ___________.
Version 1
14
Answer Key Test name: Chap 25_3e 1) FALSE 2) TRUE 3) TRUE 4) TRUE 5) [B, D] 6) B 7) B
Version 1
15
Clarify Question What is the key concept addressed by the question? • The key concept addressed here is the relationship between yeast and rising bread dough. What type of thinking is required? o This question is asking you to weigh and judge, or evaluate, why yeast makes bread dough rise. Gather Content What do you already know about yeast? What other information is related to the question? • Baker’s yeast is a one-celled fungus that is widely used commercially, from bread making to the production of beer and wine. • In the presence of sugar, yeast undergoes anaerobic respiration in the form of alcoholic fermentation. • The products of alcoholic fermentation are carbon dioxide and ethanol. Choose Answer Given what you now know, what information is most likely to produce the correct answer? • Yeast ferment anaerobically; they do not suck in air. • Although yeast may multiply under the right environmental conditions, a population increase of these single-celled fungi would not result in a significant increase in the size of bread dough. • Yeast do not burst as they ferment. • Carbon dioxide, which occurs as gas bubbles at room temperature, is produced by fermentation. The presence of carbon dioxide causes bread dough to rise. Version 1
16
Reflect on Process Did your problem-solving process lead you to the correct answer? If not, where did the process break down or lead you astray? How can you revise your approach to produce a more desirable result? o Answering this question correctly depended on your ability to weigh and judge, or evaluate, the role of yeast in rising bread dough. If you got an incorrect answer, did you remember that yeast undergo anaerobic respiration, or that a product of this respiration is carbon dioxide gas? Did you have trouble connecting yeast to rising bread dough to determine the correct answer? 8) B 9) D
Version 1
17
Clarify Question What is the key concept addressed by the question? • The key concept here is the nature of the relationship between leaf cutter ants and fungi. What type of thinking is required? o This question is asking you to weigh and judge, or evaluate, the relationship between leaf cutter ants and fungi. Gather Content What do you already know about leaf cutter ants and fungi? What other information is related to the question? • Leaf cutter ants cultivate fungi within their colony. • Fungi are fed leaves that the ants cut from surrounding trees. • Within the colony, the fungi receive a sheltered place in which they can grow. Choose Answer Given what you now know, what information is most likely to produce the correct answer? ● Parasites feed on the host without consuming it entirely. Ants do not feed directly upon the leaves of the trees. ● Pathogens cause disease that is often fatal to the host. Ants do not cause disease in the treeorthe fungus.. ● Since the ants feed on the fungus, they do have an ecological relationship with the fungus. ● The trees do not obtain any benefits from the ants, so the relationship between the ants and the trees is not mutualistic. ● The fungi benefit from their association with the ants by the secure habitat and food that the ant colony provides. Version 1
18
● The ants benefit by obtaining a food source from the fungi.Since both organisms benefit, this is a mutualistic relationship. Reflect on Process Did your problem-solving process lead you to the correct answer? If not, where did the process break down or lead you astray? How can you revise your approach to produce a more desirable result? o Answering this question correctly depended on your ability to weigh and judge, or evaluate, leaf cutter ants and fungi. If you got an incorrect answer, did you remember that leaf cutter ants provide a secure location and nutrients for the fungi, or that the ants benefit by using the fungi as food? Did you have trouble relating leaf cutter ants and fungi to determine the correct answer? 10) C
Version 1
19
Clarify Question What is the key concept addressed by the question? • This question is asking about the necessity of toxin production by fungi. What type of thinking is required? o This question is asking you to weigh and judge, or evaluate, the reason that fungi produce antibacterial compounds. Gather Content What do you already know about fungi and bacteria? What other information is related to the question? • Fungi feed by releasing chemicals into their environment and then absorbing the available nutrients. • Heterotrophic bacteria obtain nutrients by absorbing them from the environment. Choose Answer Given what you now know, what information is most likely to produce the correct answer? • If the fungi and bacteria were in symbiosis, it would not be an evolutionarily sound strategy for the fungi to kill the bacteria. • If the fungi can be killed by bacterial toxins, the production of antibiotic compounds as a defense would likely betoo late to be useful. • If the fungi are parasites of bacteria it would not be necessary to kill them first. • Since the fungi and heterotrophic bacteria may be feeding on the same resources, they are competitors for those resources. The production of antibacterial compounds by fungi would lessen that competition and would give an advantage to the fungi. Version 1
20
Reflect on Process Did your problem-solving process lead you to the correct answer? If not, where did the process break down or lead you astray? How can you revise your approach to produce a more desirable result? o Answering this question correctly depended on your ability to weigh and judge, or evaluate, bacteria and fungi. If you got an incorrect answer, did you remember that heterotrophic bacteria and fungi feed on the same materials, or that the elimination of bacteria would make more resources available to the fungi? Did you have trouble weighing the merits of bacteria and fungi to determine the correct answer? 11) D 12) A 13) E
Version 1
21
Clarify Question What is the key concept addressed by the question? • This question is asking about the differences between microsporidians and blastocladiomycetes. What type of thinking is required? o This question is asking you to compare and contrast microsporidians and blastocladiomycetes, making it an evaluate-level question. Gather Content What do you already know about microsporidians and blastocladiomycetes? What other information is related to the question? • Microsporidians do not possess mitochondria, so they do not carry out aerobic respiration. • Blastocladiomycetes do posses mitochondria. Choose Answer Given what you now know, what information is most likely to produce the correct answer? • Since a general characteristic of fungi is a cell wall that is made of chitin, this answer can be disregarded. • Rubisco is used in photosynthesis, so this answer can also be disregarded. • Phospholipidases and DNA polymerases are produced by many fungi, so neither is a differentiating factor. • Since ATP synthase is an enzyme that is used in the mitochondria during aerobic respiration, it would not be present in the microsporidians. ATP synthase could be used to differentiate between microsporidians and other fungi. Version 1
22
Reflect on Process Did your problem-solving process lead you to the correct answer? If not, where did the process break down or lead you astray? How can you revise your approach to produce a more desirable result? o Answering this question correctly depended on your ability to weigh and judge, or evaluate, microsporidians and blastocladiomycetes. If you got an incorrect answer, did you remember that microsporidians lack mitochondria, or the that aerobic respiration occurs in the mitochondria? Did you have trouble comparing microsporidians and blastocladiomycetes to determine the correct answer? 14) C
Version 1
23
Clarify Question What is the key concept addressed by the question? • This question is asking about the nature of the relationship between the fungus that causes athlete’s foot and the human that has the infection. What type of thinking is required? o You are being asked to take what you already know and use, or apply, it to the symbiotic relationship that is exemplified by athlete’s foot. Gather Content What do you already know about athlete’s foot? What other information is related to the question? • Athlete’s foot is caused by Tinea pedis, which is a fungus. • Tinea pedis is a dermatophyte, which means it feeds on skin. Choose Answer Given what you now know, what information is most likely to produce the correct answer? • Atavism involves a return to a primitive form, which is not what occurs with athlete’s foot. • Since commensalism involves a relationship in which one species benefits while the other species is unaffected, it can be disregarded as an answer. • Mutualism is a relationship in which both species benefit. This is not what occurs in the case of athlete’s foot. • Since the fungus is feeding on the skin of the host and the host is receiving no benefit, athlete’s foot is an example of parasitism. Reflect on Process Version 1
24
Did your problem-solving process lead you to the correct answer? If not, where did the process break down or lead you astray? How can you revise your approach to produce a more desirable result? o Answering this question correctly depended on your ability to use symbiotic relationships in a new situation. If you got an incorrect answer, did you remember that commensalism benefits one species while the other is unaffected, or that mutualism benefits both species? Did you have trouble extending the concept of symbiotic relationships to determine the correct answer? 15) A 16) C
Version 1
25
Clarify Question What is the key concept addressed by the question? • This question is asking about the use of fungi in resolving an oil spill. What type of thinking is required? o You are being asked to take what you already know and use, or apply, it to fungi as agents of remediation for an oil spill. Gather Content What do you already know about fungi? What other information is related to the question? • Fungi absorb and then metabolize materials from their surrounding environment. • Fungi are known for their ability to break down almost any organic compound, even jet fuel. Choose Answer Given what you now know, what information is most likely to produce the correct answer? • Fungi are not photosynthetic, nor do they have central vacuoles, so those answers can be disregarded. • Since fungi are well-known for their ability to metabolize anything that is carbon based, and oil is carbon based, it is reasonable to conclude that they may be used to break down oil. Reflect on Process Did your problem-solving process lead you to the correct answer? If not, where did the process break down or lead you astray? How can you revise your approach to produce a more desirable result? Version 1
26
o Answering this question correctly depended on your ability to use fungi in a new situation. If you got an incorrect answer, did you remember that fungi absorb and metabolize materials from their environment, or that fungi can break down a wide variety of organic compounds? Did you have trouble extending the abilities of fungi to determine the correct answer? 17) B
Version 1
27
Clarify Question What is the key concept addressed by the question? • This question is asking about the nature of microsporidia as it relates to their classification. What type of thinking is required? o You are being asked to take what you already know and use, or apply, it to the classification of microsporidia. Gather Content What do you already know about microsporidia? What other information is related to the question? • Microsporidia are unicellular parasites that utilize a variety of creatures as their hosts. • Microsporidia are associated with human disease. • Mitochondria are absent in microsporidians;they utilize anaerobic respiration. Choose Answer Given what you now know, what information is most likely to produce the correct answer? • Their unicellular nature and small size are not factors in the difficulty of classifying microsporidia. • As eukaryotes, microsporidia do have nuclei. • Microsporidia are not rare. • The lack of organelles like mitochondria are a factor in the classification of the microsporidia. Reflect on Process
Version 1
28
Did your problem-solving process lead you to the correct answer? If not, where did the process break down or lead you astray? How can you revise your approach to produce a more desirable result? o Answering this question correctly depended on your ability to use microsporidia in a new situation. If you got an incorrect answer, did you remember that microsporidia lack mitochondria, or that they utilize anaerobic respiration? Did you have trouble extending the characteristics of microsporidia to determine the correct answer? 18) C
Version 1
29
Clarify Question What is the key concept addressed by the question? • This question is asking about how fungi can be differentiated from other living things. What type of thinking is required? o You are being asked to take what you already know and use, or apply, it to the identification of fungi. Gather Content What do you already know about fungi? What other information is related to the question? • Fungi absorb nutrients from their surroundings. • Fungi possess cell walls that are composed of chitin. • Fungi can be unicellular or multicellular. Choose Answer Given what you now know, what information is most likely to produce the correct answer? • All living things possess deoxyribose, which is a component of DNA. This would not differentiate fungi. • Lignin and cellulose are characteristic of plants, not fungi. • Collagen is characteristic of animals. • Chitin is present in the cell walls of fungi, so this could be used to differentiate them from plants, and most animals. Reflect on Process Did your problem-solving process lead you to the correct answer? If not, where did the process break down or lead you astray? How can Version 1
30
you revise your approach to produce a more desirable result? o Answering this question correctly depended on your ability to use fungi in a new situation. If you got an incorrect answer, did you remember that fungi have cell walls that are composed of chitin, or that other microbes lack this polymer? Did you have trouble recognizing the characteristics of fungi to determine the correct answer? 19) D 20) A 21) E 22) B 23) A 24) D 25) B 26) B 27) D 28) B 29) A 30) C 31) B 32) C 33) A 34) C 35) D 36) C 37) B 38) D 39) B 40) B 41) C 42) B Version 1
31
43) D 44) B 45) A 46) C 47) B 48) B 49) A 50) B 51) D 52) B
Version 1
32
Clarify Question • What is the key concept addressed by the question? o You have a petri dish covered in bacteria, which form a translucent white coating on the dish. You then place pieces of Penicillium in the dish. Twenty-four hours later, there is a clear ring around each of the pieces of Penicilium because all the bacteria within 2 millimeters of the fungus are dead. Why does Penicilium produce a chemical that is toxic to bacteria? • What type of thinking is required? o This is an “evaluate” question, so use your knowledge about fungi and bacteria to evaluate the relative merit of each hypothesis. Remember, a hypothesis is an educated guess, so you should be able to back up your choice with facts. Gather Content • What do you already know about fungi? How does it relate to the question? o Fungi occur in a variety of environments, including soil and water. They can be multicellular or unicellular. Multicellular fungi have bodies made of hyphae, which are long strands one cell wide and many cells long. No fungi are autotrophs; they all need to consume other organisms to get energy. Some are decomposers, consuming material from dead organisms. Others are predators or parasites. One predatory fungus creates traps for nematodes. When the nematode gets stuck in the trap, the fungus sends hyphae into the nematode’s body and digests the nematode from the inside out. • What other information is related to the question? Which information is most useful? o As decomposers, fungi are very important for carbon cycling. They Version 1
33
can digest wood, which is very difficult. Which nutrient cycles are bacteria involved in? Choose Answer • Given what you now know, what information and/or problemsolving approach is most likely to produce the correct answer? o Each of the options provides a valid hypothesis as to why Penicilium produces a toxin that kills bacteria. You need to determine which option is the best hypothesis. Which evidence supports each hypothesis. Which evidence argues against each hypothesis? Reflect on Process • Did your problem-solving process lead you to the correct answer? If not, where did the process break down or lead you astray? How can you revise your approach to produce a more desirable result? o This question asked you to hypothesize why Penicilium produces a toxin that kills bacteria. Answering the question depended on your ability to judge which option was the best hypothesis. o If you got the correct answer, well done! o If you got an incorrect answer, where did the process break down? • Fungi could eat bacteria, but they generally consume things that are larger. There isn’t much evidence against this hypothesis, but there isn’t much in favor of it either. • We know that bacteria and fungi compete for resources, so a fungus that kills neighboring bacteria would have access to more resources. This hypothesis had the best evidence in favor and the least evidence opposed to it. • There is no evidence that fungi participate in nitrogen cycling. • The toxin could be produced in response to an attack of pathogenic Version 1
34
bacteria, but there is no evidence to support this hypothesis.Penicilium produces the toxin when exposed to any bacteria, not just pathogenic bacteria. 53) sporangiophores
Version 1
35
CHAPTER 26 TRUE/FALSE - Write 'T' if the statement is true and 'F' if the statement is false. 1) Liverworts can reproduce asexually by releasing spores from the thallus. ⊚ ⊚
2)
true false
As the endosperm matures, it swells and eventually becomes the fruit. ⊚ true ⊚ false
3) Pollination and fertilization are essentially the same process. If pollination occurs, fertilization will follow quickly. ⊚ ⊚
true false
4) In flowering plants, after fertilization the zygote continuously develops until the seed germinates. ⊚ true ⊚ false
5)
All gymnosperms have needle-like leaves and produce some type of resin. ⊚ true ⊚ false
6)
In seed plants, the entire male gametophyte is transported to the female. ⊚ ⊚
Version 1
true false
1
MULTIPLE CHOICE - Choose the one alternative that best completes the statement or answers the question. 7) Welwitschia has vessel elements, seeds on cones, and partially exposed megaspores, and is insect pollinated. It is classified as a gymnosperm but shares most of these features with the A) angiosperms. B) pterophytes. C) hepaticophytes. D) bryophytes.
8)
What group is most closely related to the tracheophytes? A) Bryophytes. B) Charophytes. C) Rhodophytes (red algae). D) Chlorophytes.
9)
In which of these groups does it appear that the cuticle evolved? A) Streptophytes. B) Bryophytes. C) Green plants. D) Green algae. E) Land plants.
10)
The gametophyte is photosynthetic and nutritionally independent A) only in the liverworts. B) in all seeds plants. C) in all tracheophytes. D) only in the ferns. E) in the bryophytes and pterophytes.
Version 1
2
11)
Hornworts differ from mosses and liverworts in containing A) small leaves. B) sporophytes. C) photosynthetic sporophytes. D) gametophytes. E) spores.
12)
Tracheophyte plants are characterized by all of the following except A) a specialized water conducting system. B) a waxy cuticle. C) stomata. D) homosporous or heterosporous. E) seeds.
13)
The dominant stage in the life cycle of tracheophyte plants is the A) epiphytegeneration. B) sporophyte generation. C) spore generation. D) gametophyte generation. E) angiosperm generation.
14)
Why is a rhizoid not considered a true root? A) Rhizoids lack the capacity to anchor mosses into the substrate. B) Rhizoids are not capable of absorbing water. C) True roots have mycorrhizal fungi. D) Rhizoids lack xylem tissue. E) Only true roots are capable of nitrogen fixation.
Version 1
3
15) Azolla filiculoides is a small aquatic fern that is typically 1–2 cm tall. How would a botanist distinguish it from a bryophyte? A) It produces seeds. B) It has xylem tissue. C) It has a gametophyte. D) It has chloroplasts.
16)
The aquatic ancestry of bryophytes is most clearly demonstrated by what character? A) The use of flagellated motile sperm. B) Mycorrhizal associations with fungi. C) A nutritionally dependent sporophyte. D) The production of a sporangium.
17)
Why are most bryophytes less than 10 cm tall? A) They have underdeveloped stomata. B) They are seedless. C) They lack a cuticle. D) The have conducting cells instead of tracheids. E) They have no H2O transport mechanisms.
18)
The development of tracheids helped land plants to A) resist desiccation. B) protect the genome from UV exposure. C) grow to very large sizes. D) absorb H2O from stems and leaves. E) take in more CO2.
19)
What is a 15-meter tall tree fern?
Version 1
4
A) A gametophyte B) Haploid C) Produced by meiosis D) A sporophyte
20)
A waxy cuticle helps prevents desiccation. What is the cost of this adaptation? A) Lowered lightabsorption B) Reduced gaspermeability C) SlowerH 2O transport D) Increased exposureto damaging UV radiation E) Increased susceptibility to pathogenicinfection
21) Stomata are used by plants to facilitate gas exchange. Why are they absent in charophytes? A) Gasses freely diffuse through uncoated charophyte tissues. B) Charophytes use organic acids as a carbon source. C) They are present in charophytes but have been reduced to a vestigial state. D) Stomata are an exclusively aquatic adaptation.
22) Plants lacking well-developed conducting systems such as mosses, liverworts, and hornworts have been called A) gametophytes. B) sporophytes. C) angiosperms. D) gymnosperms. E) bryophytes.
23)
Which of the following represents a monophyletic group?
Version 1
5
A) Mosses, liverworts, and hornworts B) Ferns and club mosses C) Ferns, whisk ferns, and horsetails D) Liverworts and hornworts E) Charophytes and mosses
24) Which plants are characterized by a conducting system, the possession of cuticles, specialized stems, and roots, stomata, and, in many species, seeds? A) Liverworts B) Mosses C) Tracheophyte D) Horsetails E) Club mosses
25)
Bryophytes, like club mosses and certain other tracheophyte plants, require A) air to reproduce sexually. B) water to reproduce sexually. C) soil to reproduce sexually. D) shade to reproduce sexually. E) sunlight to reproduce sexually.
26) The rootlike projections of mosses, liverworts, and hornworts that anchor them to the substrate are called A) rhizoids. B) tap roots. C) fibrous roots. D) specialized roots. E) storage roots.
Version 1
6
27)
Well-developed vascular systems are absent in all of the following except A) mosses. B) liverworts. C) hornworts. D) Charophytes. E) Pterophytes.
28)
Survival on land for organisms is difficult because of the problem of A) desiccation. B) pollination. C) syngamy. D) spore germination. E) coevolution.
29) Plants as well as brown, green, and red algae show a basic life cycle that involves the alternation of generations between a(n) A) spore and pollen. B) megaspore and a microspore. C) seed and a cone. D) sporophyte and a gametophyte. E) egg and a sperm.
30)
Which of the following features are common to both moss leaves and true leaves? A) Vascular strands B) Chlorophyll a and b C) Haploid cells throughout D) Only one cell thick except at the midrib E) Stomata
Version 1
7
31)
To reproduce sexually, bryophytes require A) free water external to the plant. B) high temperatures. C) solar energy. D) insect pollination.
32)
In tracheophyte plants, water and dissolved minerals are conducted away from roots by A) water vessels. B) stomata. C) phloem tubes. D) xylem tubes. E) chloroplasts.
33) Select the best answer to complete the following statement. Stomata are openings used by plants for the passage of A) only oxygen and carbon dioxide. B) only carbon dioxide and pollen. C) only carbon dioxide and water vapor. D) carbon dioxide, oxygen, and water vapor. E) only carbon dioxide and spores or gametes.
34)
All of the following statements about ferns are correct except A) ferns flourish in many habitats, but the majority are located in the tropics. B) the sporophyte is a conspicuous generation. C) the gametophyte is a conspicuous generation. D) ferns have underground stems called rhizomes. E) sori are located on the back or ventral surface of fronds.
Version 1
8
35) A bryologist (a scientist that studies mosses, and their allies) gives a lecture to your biology class. In her lecture, she makes a reference to fern reproduction. She says, "Fern spores germinate into ____." A) gametophytes, which are often heart-shaped and have rhizoids that anchor the fern to the substrate B) gametophytes, which are often heart-shaped and have roots that anchor the fern to the substrate C) sporophytes, which are often heart-shaped and have rhizoids that anchor the fern to the substrate D) sporophytes, which are often heart-shaped and have roots that anchor the fern to the substrate
36) Whisk ferns, club mosses, and horsetails have all the following characteristics in common with ferns except: A) all of these plants are seedless tracheophyte plants. B) all of these plants produce flagellated sperm. C) all of these plants form antheridia and archegonia. D) all of these plants require free water for the process of fertilization to occur. E) all of these plants have no vascular tissue.
37) Advances in molecular systematics show that whisk ferns and horsetails are the closest living relatives of A) mosses. B) club mosses. C) seed plants. D) gymnosperms. E) ferns.
38)
Which one of the following statements about haplodiplontic life cycles is false?
Version 1
9
A) For each gene, cells of the gametophyte stage have a single allele. B) Spore mother cells produce haploid spores by meiosis. C) The gametophyte stage produces gametes by mitosis. D) The zygote is the only diploid cell in the sporophyte stage. E) Mitosis occurs in both sporophyte and gametophyte stages.
39)
Which one of the following structures in the moss life cycle is not haploid? A) Germinating spore B) Antheridium C) Sperm D) Spore mother cell E) Rhizoid tissue
40) For which of the following organisms is the sporophyte stage the dominant stage in the life cycle? A) Charophyte B) Fern C) Green alga Chlamydomonas D) Moss E) Liverwort
41)
How do lycophytes differ from bryophytes? A) Lycophytes havetracheids. B) Lycophytes produceseeds. C) Lycophytes useflowers for reproduction. D) The dominant stage in a lycophyte is the gametophyte.
42)
What are fruits derived from?
Version 1
10
A) Styles B) Ovaries C) Stigmas D) Anthers E) Filaments
43)
In flowering plants, the mature male gametophyte is produced A) in the anther. B) within the pollen granule during transfer. C) when the pollen makes contact with the stigma. D) in the pollen tube when the generative cell divides. E) within the ovule just prior to fertilization.
44)
Double fertilization occurs when A) the sperm and tube nuclei fertilize the egg. B) two sperm cells fertilize the two polar nuclei. C) one sperm fuses with the egg and the other sperm fuses with both polar nuclei. D) the polar nuclei fuse with the egg after it has fused with the sperm.
45)
Petals A) protect the developing flower. B) enclose and protect the ovule. C) produce pollen. D) are part of the gametophyte. E) attract pollinators.
46)
Why do bees visit flowers?
Version 1
11
A) To transfer pollen to the carpel B) To maintain a healthy flower population C) To help the plant reproduce D) To collect food and feed it to their larvae
47)
Pollen comes from the A) carpel. B) petal. C) stigma. D) anther. E) style.
48)
Double fertilization produces a __________ embryo and a __________ endosperm. A) haploid; diploid B) diploid; haploid C) diploid; triploid D) triploid; diploid E) haploid; triploid
49)
What is the function of the generative cell in conifers? A) Growth of themegaspore B) Dispersal of thepollen C) Production of thearchegonium D) Formation of thesperm E) Maturation of thefemale cone
50)
Ephedrine is a stimulant that is derived from a member of the phylum
Version 1
12
A) Hepaticophyta. B) Gnetophyta. C) Pterophyta. D) Cycadophyta.
51) A mutation prevents division of the generative cellin a flowering plant. Where would this mutation most likely manifest itself? A) Anther B) Ovule C) Ovary D) Style
52)
All angiosperms have A) nectaries. B) animal pollinators. C) scented flowers. D) fleshy fruits for dispersal. E) ovules enclosed in carpels.
53)
Why is external water not essential for fertilization in seed plants? A) The sperm cells in seed plants are capable of resisting desiccation. B) Seed plants use pollen instead of sperm for fertilization. C) Pollen grains are transported by wind or an animal. D) Sperm cells are used to transport pollen to the archegonia. E) Ovules are contained within the pollen.
54)
Which part of the seed is responsible for providing nutrition to the embryo?
Version 1
13
A) Integument B) Endosperm C) Cotelydons D) Seed coat E) Procambium
55)
Pollen tubes A) transport pollen to the stigma. B) facilitate the growth of pollen. C) package sperm into pollen granules. D) allow sperm to fertilize the egg. E) are present during spermatogenesis.
56) Due to their pinnately compound leaves, tropical habitat, and overall appearance, members ofwhich phylum are often thought of as a type of palm? A) Coniferophyta B) Gentophyta C) Cycadophyta D) Ginkophyta
57)
What is the function of pine resins? A) transportation of sucrose to the roots. B) reduction of fungal and insect attack. C) triggering of seed maturation. D) aiding in pollen dispersal.
58)
What happens to the ovary after fertilization in angiosperms?
Version 1
14
A) It dies. B) It forms the seed coat. C) It becomes food for the embryoinside the seed. D) It becomes the fruit.
59)
Why are conifers and cycads considered gymnosperms?
A) The ovule forms the seed. B) The female gametophyte is not completely enclosed in sporophyte tissue at the time of pollination. C) The fruit is not fleshy. D) The pollen is wind dispersed. E) Pollination and fertilization occur at different times.
60)
Based on the number of species, the most successful extant phylum of plants is the A) Anthophyta(angiosperms). B) Coniferophyta. C) Pterophyta. D) Gnetophyta. E) Charophyta. F) Hepaticophyta.
61) Cooksonia is a genus of very early tracheophyte plants. Unlike modern tracheophytes, they only grew to a few centimeters in height. What feature provides the best explanation for their diminutive size? A) They completely lacked vascular tissues. B) They did not have stomata. C) They consisted of only photosynthetic stems, not roots or leaves. D) Cooksonia species were not autotrophic.
Version 1
15
62) Hepaticophytes lack stomata and tracheids. What would provide evidence to justify their inclusion in the Bryophytes and not the Charophytes? A) Cellulose in theircell walls B) Genes for ABAsignalling C) The use ofchlorophyll D) A haplodiplonticlifecycle
63)
Are any of these vegetables actually fruit? A) Sugar beets B) Corn C) Carrots D) Celery E) All of these are vegetables; none of them are fruit.
64) A botanist examines two plants. Plant A has large colorful petals; plant B has extremely tiny white flowers. Which one is most likely animal pollinated? A) Plant A B) Plant B
65)
Based on the outcome for plants, a monkey eating a banana is a A) pollinator. B) fertilizer. C) disperser. D) parasite.
66) Old pine stumps are dug up, split into pieces and sold as "fatwood." Fatwood is used as a fire starter. Why is this done?
Version 1
16
A) The wood in the stumps is porous and dry, thus it burns easily. B) The high concentration of volatile organic compounds makes the wood flammable. C) Pine bark does not absorb water and is flammable.
67)
Why aren't ferns commonly found in deserts? A) Fern fronds are sensitive to UV light. B) Fern spores are unable to resist prolongeddry conditions. C) Ferns are nonvascular and cannot absorb water from deep within the soil. D) Fern pollen is spread by insects that do not occur in deserts.
68)
Why can corn seeds germinate in total darkness?
A) The seed coating absorbs water. B) Seeds are heterotrophic and can capture nutrients from the soil. C) The endosperm provides the embryo with the nutrition needed during the initial germination stages. D) Soil microbes degrade the seed coating, allowing the seed to germinate.
69) A researcher brings a sample of dry soil to the lab, wets it, provides ample amounts of fertilizer, and places the soil in a container in a greenhouse. Which type of plant would be least likely to emerge? A) Pine seedling B) Sunflower C) Corn D) Wheat E) Liverwort
Version 1
17
FILL IN THE BLANK. Write the word or phrase that best completes each statement or answers the question. 70) The first plants clearly evolved from an organism that, if it existed today, would be classified as a multicellular green __________.
71) Plants have a __________ life cycle in which the haploid and diploid stages are multicellular.
Version 1
18
Answer Key Test name: Chap 26_3e 1) FALSE Liverworts reproduce asexually byreleasing fragments of tissue from structures called gemmae, found on the surface of the thallus. Spores are involved in sexual reproduction and are released by the sporophyte. <!-Markup Copied from Habitat--> 2) FALSE 3) FALSE Pollination and fertilization are different processes that may be separated by a long delay. Pollination is the mechanical transfer of pollen from its source to a receptive area on the receiving plant. Sperm are subsequently conducted to the egg by a pollen tube, and fertilization—the joining of sperm with egg—may occur much later. For instance, in a typical pine tree, fertilization follows 15 months after pollination. 4) FALSE 5) FALSE 6) TRUE 7) A Review the features in the question stem and thecharacteristicsof each plant group. 8) A 9) E Review plant adaptations and innovations in Sections 26.1 and 26.3. Comparison of Figures 26.1 and 26.8 may help determine the correct answer.
Version 1
19
10) E Review the characteristics of the gametophyte in the major plant groups. 11) C 12) E 13) B 14) D
Version 1
20
Clarify Question What is the key concept addressed by the question? • The concept addressed here is the nature of a true root. What type of thinking is required? • This question is asking you to compare and contrast rhizoids with true roots, making it an evaluate-level question. Gather Content What do you already know about true roots and rhizoids? What other information is related to the question? • Rhizoids are root-like structures in that they anchor the mosses and other plants that possess them. • While roots are multicellular tissues, rhizoids are unicellular and therefore are not tissues. • Roots contain vascular tissue. • Unlike roots, rhizoids lack vascular tissue. Choose Answer Given what you now know, what information is most likely to produce the correct answer? • Rhizoids do anchor mosses to the substrate. • Although they do not have vascular tissue, rhizoids can conduct water. • Roots do not fix nitrogen; it is the symbiotic bacteria associated with roots that fix nitrogen. • True roots have vascular tissue, which includes xylem and phloem. So if a plant lacks xylem, it stands to reason that it does not have true roots. Reflect on Process
Version 1
21
Did your problem-solving process lead you to the correct answer? If not, where did the process break down or lead you astray? How can you revise your approach to produce a more desirable result? • Answering this question correctly depended on your ability to weigh and judge, or evaluate, true roots and rhizoids. If you got an incorrect answer, did you remember that rhizoids differ from true roots by lacking vascular tissue, or the criteria to compare rhizoids and true roots includes the shared function of anchoring the plant? Did you have trouble weighing the merits of rhizoids relative to true roots to determine the correct answer? 15) B
Version 1
22
Clarify Question What is the key concept addressed by the question? • This question is highlighting the differences between ferns and bryophytes. What type of thinking is required? • You are being asked to take what you already know and use, or apply, it to the identification of a fern versus a bryophyte. Gather Content What do you already know about ferns and bryophytes? What other information is related to the question? • Bryophytes include mosses, liverworts, and hornworts. • Bryophytes lack vascular tissue. • Ferns are seedless vascular plants. Choose Answer Given what you now know, what information is most likely to produce the correct answer? • Both bryophytes and ferns have gametophytes and chloroplasts, so those features could not be used to distinguish between the two plants. • Neither bryophytes nor ferns produce seeds. • A key factor that differentiates bryophytes and ferns is the presence of vascular tissue. Vascular tissue is present in ferns, but absent in bryophytes. Since xylem is a component of vascular tissue, it could be used to differentiate a fern from a bryophyte. Reflect on Process Did your problem-solving process lead you to the correct answer? If not, where did the process break down or lead you astray? How can Version 1
23
you revise your approach to produce a more desirable result? • Answering this question correctly depended on your ability to use ferns and bryophytes in a new situation. If you got an incorrect answer, did you remember that vascular tissue includes xylem, or that ferns have vascular tissue but bryophytes do not have vascular tissue? Did you have trouble extending the characteristics of ferns and bryophytes to determine the correct answer? 16) A 17) D
Version 1
24
Clarify Question What is the key concept addressed by the question? • The key concept addressed here is the limiting factor for bryophyte size. What type of thinking is required? • This question is asking you to weigh and judge, or evaluate, sizelimiting features of bryophytes. Gather Content What do you already know about bryophytes? What other information is related to the question? • Bryophytes include mosses, liverworts, and hornworts. • Although they are found in a variety of environments, most bryophytes are found in moist habitats. • Bryophytes lack vascular tissues. Choose Answer Given what you now know, what information is most likely to produce the correct answer? • The lack of seeds in bryophytes does not limit their size. • Mosses may have stomata at certain life stages, but bryophytes overall do not possess stomata. • Bryophytes do possess water-conducting tissues, but they lack tracheids and vascular tissues. Since vascular tissues increase the efficiency of water and nutrient circulation, they make tall heights possible for plants. Reflect on Process
Version 1
25
Did your problem-solving process lead you to the correct answer? If not, where did the process break down or lead you astray? How can you revise your approach to produce a more desirable result? • Answering this question correctly depended on your ability to weigh and judge, or evaluate, bryophytes and the size-limitations that are relevant for them. If you got an incorrect answer, did you remember that bryophytes lack vascular tissue or that vascular tissue includes tracheids, which facilitate the efficient delivery of water in plants? Did you have trouble weighing the merits of internal structure relative to plant size to determine the correct answer? 18) C
Version 1
26
Clarify Question What is the key concept addressed by the question? • This question is asking you to apply the benefits of tracheids to land plant adaptations. What type of thinking is required? • You are being asked to take what you already know and use, or apply, it to the advantages conferred by tracheids. Gather Content What do you already know about tracheids? What other information is related to the question? • Tracheids are the first known components of vascular tissues. • Vascular tissues are used to move water and other nutrients efficiently within a plant. • Water and nutrients are necessary for a plant to maintain its structure and metabolic activities. Choose Answer Given what you now know, what information is most likely to produce the correct answer? • Although they move materials including water within a plant, tracheids do not prevent desiccation, nor are they protective against UV exposure. • Tracheids deliver water to leaves and stems rather than absorbing it from same. • Tracheids are passages for delivery of materials; they do not take in CO2. • Since tracheids increase the efficiency of water and nutrient circulation, they make large sizes possible for plants. Version 1
27
Reflect on Process Did your problem-solving process lead you to the correct answer? If not, where did the process break down or lead you astray? How can you revise your approach to produce a more desirable result? • Answering this question correctly depended on your ability to use tracheids in a new situation. If you got an incorrect answer, did you remember that tracheids are components of vascular tissue or that vascular tissue is used to efficiently transport water and nutrients? Did you have trouble extending tracheids to determine the correct answer? 19) D
Version 1
28
Clarify Question What is the key concept addressed by the question? • The question is asking about the nature of a fern that reaches a large size. What type of thinking is required? • You are being asked to take what you already know and use, or apply, it to determine the identity of a very large fern. Gather Content What do you already know about ferns? What other information is related to the question? • The sporophyte is the dominant life stage of the fern. • Sporophytes are diploid. • Fern sporophytes have vascular tissues, but fern gametophytes do not have vascular tissues. • Gametophytes are haploid. Choose Answer Given what you now know, what information is most likely to produce the correct answer? • Since fern gametophytes lack vascular tissues, they remain very small, so this answer can be excluded. • Gametophytes, but not sporophytes, are haploid, so this answer can also be excluded. • Gametes are produced by meiosis, so this answer can be disregarded. • Sporophytes possess vascular tissue and can thereby grow to very large sizes. Fifteen meters is within an expected size for the sporophyte of a tree fern.
Version 1
29
Reflect on Process Did your problem-solving process lead you to the correct answer? If not, where did the process break down or lead you astray? How can you revise your approach to produce a more desirable result? • Answering this question correctly depended on your ability to use fern life stages in a new situation. If you got an incorrect answer, did you remember that ferns can exist as sporophytes and gametophytes, or that sporophytes alone can grow to large sizes? Did you have trouble extending fern life stages to determine the correct answer? 20) B 21) A 22) E 23) C
Version 1
30
Clarify Question What is the key concept addressed by the question? • This question is asking you to determine which group of plants is most closely related. What type of thinking is required? • You are being asked to take what you already know and use, or apply, it to the analysis of the evolutionary history of plants. Gather Content What do you already know about phylogeny? What other information is related to the question? • Phylogeny refers to the evolutionary history of different species. • Information to build an evolutionary history of different species can be drawn from morphology, cytology, nucleic acid analysis, and other sources. • A proposed evolutionary history can be diagrammed on a cladogram, which shows the relatedness of species. • A monophyletic relationship shows the closest evolutionary relatedness. • A monophyletic grouping includes a common ancestor and all descendant species. Choose Answer Given what you now know, what information is most likely to produce the correct answer? • If a cladogram is examined, it is clear that the horsetails, ferns, and whisk ferns share a common ancestor. There are no descendant species from that common ancestor that are not included int his group.
Version 1
31
Reflect on Process Did your problem-solving process lead you to the correct answer? If not, where did the process break down or lead you astray? How can you revise your approach to produce a more desirable result? • Answering this question correctly depended on your ability to use phylogeny in a new situation. If you got an incorrect answer, did you remember that a monophyletic group includes species that share a common ancestor or that a monophyletic grouping must include all descendant species from the common ancestor? Did you have trouble extending phylogeny to determine the correct answer? 24) C 25) B 26) A 27) E 28) A 29) D 30) B
Version 1
32
Clarify Question What is the key concept addressed by the question? • This question is asking you to identify features that are possessed by mosses and green leaves. What type of thinking is required? • You are being asked to take what you already know and use, or apply, it to the similarities between mosses and true leaves. Gather Content What do you already know about mosses and true leaves? What other information is related to the question? • Mosses and true leaves are both land plants. • Mosses and plants with true leaves undergo photosynthesis, which requires pigments to absorb light. • The main photosynthetic pigments are chlorophyll a and chlorophyll b. • Mosses do not have vascular tissue, but plants with true leaves do have vascular tissue. • The leaf-like structures of mosses are one cell thick, while true leaves have many cell layers. Choose Answer Given what you now know, what information is most likely to produce the correct answer? • While true leaves do have vascular strands, mosses do not have vascular strands. • True leaves have multiple cell layers, so this answer can be disregarded. • While mosses may have stomata in the capsule portion of the Version 1
33
sporophyte, stomata are not present in the leaf-like structures that mosses possess. • Since Mosses and true leaves undergo photosynthesis, and they both possess chlorophyll a and chlorophyll b. Reflect on Process Did your problem-solving process lead you to the correct answer? If not, where did the process break down or lead you astray? How can you revise your approach to produce a more desirable result? • Answering this question correctly depended on your ability to use mosses and true leaves in a new situation. If you got an incorrect answer, did you remember that true leaves are multicellular and have vascular tissue or that both mosses and true leaves are photosynthetic? Did you have trouble extending mosses and true leaves to determine the correct answer? 31) A 32) D 33) D 34) C 35) A 36) E 37) E 38) D 39) D 40) B 41) A 42) B 43) D 44) C 45) E Version 1
34
46) D 47) D 48) C 49) D 50) B 51) D 52) E 53) C 54) B 55) D 56) C 57) B 58) D 59) B 60) A 61) C
Version 1
35
Clarify Question What is the key concept addressed by the question? • The concept addressed by this question is the benefit of the smaller stature of Cooksonia. What type of thinking is required? • You are being asked to take what you already know and use, or apply, it to the features that contribute to the smaller size of Cooksonia. Gather Content What do you already know about Cooksonia? What other information is related to the question? • Cooksonia is the first known vascular land plant. • Cooksonia is now extinct. • Little competition existed for Cooksonia. • Roots and leaves were absent from Cooksonia. Choose Answer Given what you now know, what information is most likely to produce the correct answer? • Cooksonia was a tracheophyte, so it did have vascular tissue. • Photosynthesis was utilized by Cooksonia, and some members of this genus did have stomata that were likely utilized for gas exchange. • Since Cooksonia lacked roots, taller stature would not be possible. Since it lacked leaves, and competition was low, photosynthesis could occur on the stem of Cooksonia. Reflect on Process Did your problem-solving process lead you to the correct answer? If Version 1
36
not, where did the process break down or lead you astray? How can you revise your approach to produce a more desirable result? • Answering this question correctly depended on your ability to use Cooksonia in a new situation. If you got an incorrect answer, did you remember that Cooksonia lacked roots and leaves or that there was little competition for space in the habitat of Cooksonia? Did you have trouble extending the features of Cooksonia to determine the correct answer? 62) B
Version 1
37
Clarify Question What is the key concept addressed by the question? • This question is asking about the factor that would differentiate a hepaticophyte from a bryophyte. What type of thinking is required? • You are being asked to take what you already know and use, or apply, it to the characteristics of hepaticophytes. Gather Content What do you already know about hepaticophytes? What other information is related to the question? • Charophytes are green algae that live in fresh water. • Hepaticophytes are commonly known as liverworts. • Liverworts are part of a larger group known as bryophytes. • Bryophytes lack vascular tissue, but can exist on land in various environments. Choose Answer Given what you now know, what information is most likely to produce the correct answer? • The presence of cellulose in the cell wall, the use of chloroplasts, and the haplodiplontic life cycle are features that are common to charophytes and hepaticophytes, so these feature would not allow differentiation. • ABA (abscisic acid) signaling is found in plants that must tolerate temperature and other stresses associated with life on land. Since liverworts exist on land and charophytes do not, this would be a key factor in their differentiation. Reflect on Process Version 1
38
Did your problem-solving process lead you to the correct answer? If not, where did the process break down or lead you astray? How can you revise your approach to produce a more desirable result? • Answering this question correctly depended on your ability to apply the characteristics of hepaticophytes in a new situation. If you got an incorrect answer, did you remember that hepaticophytes are vascular, land plants or that ABA signaling aids in responding to the stresses that are encountered by land plants? Did you have trouble extending hepaticophytes to determine the correct answer? 63) B
Version 1
39
Clarify Question What is the key concept addressed by the question? • The key concept addressed here is the difference between vegetables and fruit. What type of thinking is required? o This question is asking you to compare and contrast fruit and vegetables, making it an evaluate-level question. Gather Content What do you already know about fruit? What other information is related to the question? • Fruits are the mature ovaries of angiosperms. • Fruits usually contain seeds. Choose Answer Given what you now know, what information is most likely to produce the correct answer? • Sugar beets, carrots, and celery are all vegetative structures. They are not fruit because they are not mature ovarian structures. • Corn does represent a mature ovary, so it is considered to be a fruit. Reflect on Process Did your problem-solving process lead you to the correct answer? If not, where did the process break down or lead you astray? How can you revise your approach to produce a more desirable result? o Answering this question correctly depended on your ability to weigh and judge, or evaluate, fruits and vegetables. If you got an incorrect answer, did you remember that fruit differs from vegetables because it is Version 1
40
a mature ovary or the criteria to compare fruits and vegetables includes the origin of the fruit or vegetable? Did you have trouble distinguishing fruits relative to vegetables to determine the correct answer? 64) A
Version 1
41
Clarify Question What is the key concept addressed by the question? • The key concept addressed here is the nature of animal-pollinated plants. What type of thinking is required? o You are being asked to take what you already know and use, or apply, it to the comparison of animal pollinated versus other plants. Gather Content What do you already know about animal-pollinated plants? What other information is related to the question? • Animal-pollinated plants tend to be colorful and showy. • Wind-pollinated plants may have no petals at all. Choose Answer Given what you now know, what information is most likely to produce the correct answer? • Plant B’s small, white flowers would not be attractive to animal pollinators. • Since animal-pollinated plants have colorful flowers, plant A is the likely candidate for an animal-pollinated plant. Reflect on Process Did your problem-solving process lead you to the correct answer? If not, where did the process break down or lead you astray? How can you revise your approach to produce a more desirable result? o Answering this question correctly depended on your ability to use pollination in a new situation. If you got an incorrect answer, did you Version 1
42
remember that animal-pollinated plants have colorful flowers or that wind-pollinated plants may have no petals? Did you have trouble recognizing different pollination characteristics to determine the correct answer? 65) C
Version 1
43
Clarify Question What is the key concept addressed by the question? • This question is asking about the role of an animal who eats fruit in the lifecycle of the plant that made the fruit. What type of thinking is required? o You are being asked to take what you already know and use, or apply, it to the function of fruit. Gather Content What do you already know about fruit? What other information is related to the question? • Fruit represents the mature ovary of a plant. • Fruit usually contains seeds. • Seeds are the products of fertilization. • Animals that consume fruit play a role in the lifecycle of the plant. Choose Answer Given what you now know, what information is most likely to produce the correct answer? • Since seeds are the products of fertilization, and fertilization happens after pollination, pollination and fertilization can be excluded as functions of the consumption of seeds. • Parasites feed on the body fluids of the host. A banana is a product of a plant, not part of its body, so a monkey that feeds on a banana is not a parasite. • By consuming a banana, a monkey disperses the seeds as it passes them along with waste from its body. Reflect on Process Version 1
44
Did your problem-solving process lead you to the correct answer? If not, where did the process break down or lead you astray? How can you revise your approach to produce a more desirable result? o Answering this question correctly depended on your ability to use fruit in a new situation. If you got an incorrect answer, did you remember that fruit often contains seeds or that seeds from consumed fruit pass with waste from an animal’s body? Did you have trouble extending the definition of fruit to determine the correct answer? 66) B
Version 1
45
Clarify Question What is the key concept addressed by the question? • This question is asking about the logic of using old pine stumps as fire starters. What type of thinking is required? o You are being asked to take what you already know and use, or apply, it to the reason for the flammability of fat wood. Gather Content What do you already know about pine? What other information is related to the question? • The resin from certain pines can yield turpentine, which is a volatile liquid. • Turpentine is a flammable compound. Choose Answer Given what you now know, what information is most likely to produce the correct answer? • Pine bark is fire resistant, so that answer can be ruled out. • Stumps are generally the last section of a tree to dry out, so that answer can be excluded. • Since turpentine is flammable and since it is present in the resin of pine, old pine trunks would be good fire starters because of the presence of this volatile organic compound. Reflect on Process Did your problem-solving process lead you to the correct answer? If not, where did the process break down or lead you astray? How can Version 1
46
you revise your approach to produce a more desirable result? o Answering this question correctly depended on your ability to use pine in a new situation. If you got an incorrect answer, did you remember that pine resin contains turpentine or that turpentine is flammable? Did you have trouble extending the components of pine to determine the correct answer? 67) B
Version 1
47
Clarify Question What is the key concept addressed by the question? • This question is asking about the limitations of fern reproduction. What type of thinking is required? o You are being asked to take what you already know and use, or apply, it to the exclusion of ferns from a desert habitat. Gather Content What do you already know about ferns? What other information is related to the question? • Ferns require water for fertilization to occur. • Ferns produce spores, which require a moist environment in which to grow. Choose Answer Given what you now know, what information is most likely to produce the correct answer? • Ferns are vascular plants, so that answer can be disregarded. • Ferns do not produce flowers, so they do not produce pollen. • Fern fronds do utilize UV light to drive photosynthesis. • Since fern reproduction and spore growth is dependent on the presence of water, a dry environment such as a desert would not sustain a fern. Reflect on Process Did your problem-solving process lead you to the correct answer? If not, where did the process break down or lead you astray? How can you revise your approach to produce a more desirable result? o Answering this question correctly depended on your ability to use Version 1
48
ferns in a new situation. If you got an incorrect answer, did you remember that ferns require water to reproduce or that fern spores also require moist environments? Did you have trouble extending the characteristics of ferns to determine the correct answer? 68) C
Version 1
49
Clarify Question What is the key concept addressed by the question? • The key concept addressed here is the initial source of nutrients in corn seedlings. What type of thinking is required? o This question is asking you to weigh and judge, or evaluate, the way that corn seedlings obtain nutrients without the presence of light. Gather Content What do you already know about corn seeds? What other information is related to the question? • Flowering plants such as corn have extensive endosperm in their seeds. • Endosperm is a source of nutrition for a plant embryo before its leaves emerge from the soil. Choose Answer Given what you now know, what information is most likely to produce the correct answer? • Soil microbes are not a key factor in the degradation of the seed coat and the germination of a seed. • Seeds do not take in nutrients from the soil in significant amounts. • Although seeds require water for germination, water does not provide nutrients for the growing plant. • Because endosperm is abundant in corn seeds and because it contains nutrients that are available to the growing plant, it is the endosperm that allow germination and growth to occur in the dark. Reflect on Process
Version 1
50
Did your problem-solving process lead you to the correct answer? If not, where did the process break down or lead you astray? How can you revise your approach to produce a more desirable result? o Answering this question correctly depended on your ability to weigh and judge, or evaluate, corn seeds and the nutrients that are available to them. If you got an incorrect answer, did you remember that endosperm is abundant in corn seeds or that the endosperm is a nutrient source for newly germinated corn seedlings? Did you have trouble weighing the merits of corn seeds relative to available nutrients to determine the correct answer? 69) E
Version 1
51
Clarify Question What is the key concept addressed by the question? • This question is asking about the likelihood of the emergence of various plant types from dry soil. What type of thinking is required? o You are being asked to take what you already know and use, or apply, it to the propagation of different types of plants. Gather Content What do you already know about plant propagation? What other information is related to the question? • Liverworts are bryophytes, which is a group that includes the mosses. • Mosses require water to reproduce. • Pines are gymnosperms, and they place their seeds in cones. They do not require water to reproduce. • Sunflower, corn, and wheat are angiosperms. As seed plants, they do not require water to reproduce. Choose Answer Given what you now know, what information is most likely to produce the correct answer? • Seed plants, pines, corn, wheat, and sunflowers could all have seeds in the dry soil that would germinate if sufficient levels of light and water were present. • Since liverworts are bryophytes, they must have water to reproduce. Young would not form in dry soil, and since adults are not present in the dry soil, these plants would not appear. Reflect on Process Version 1
52
Did your problem-solving process lead you to the correct answer? If not, where did the process break down or lead you astray? How can you revise your approach to produce a more desirable result? o Answering this question correctly depended on your ability to use plant propagation in a new situation. If you got an incorrect answer, did you remember that pines, corn, sunflowers, and wheat are seed plants or that liverworts require water to reproduce? Did you have trouble extending methods of plant propagation to determine the correct answer? 70) algae 71) haplodiplontic
Version 1
53
CHAPTER 27 MULTIPLE CHOICE - Choose the one alternative that best completes the statement or answers the question. 1) Damage to its flagellum would prevent a choanocyte from __________. A) circulating water through the sponge B) stinging and capturing prey C) secreting spongin to form the sponge body D) producing the supportive spicules
2) Predict why a box jelly can eat fish while a sponge can only consume bacteria and other microscopic organic matter? A) Box jellies useintracellular digestion. B) Sponges lack venomin their stinging cells. C) Box jellies startdigestion in their gastrovascular cavity. D) Sponges are toosmall to consume large prey items.
3)
Damage to the collar would prevent the sponge from __________. A) circulating water through the sponge B) extracting organic matter from the water C) secreting spongin to form the sponge body D) producing gametes for sexual reproduction
4) A mixture of DNA fragments is amplified from a sample of ocean water. Which gene(s) could be definitively categorized as originating from a species of animal? A) Genes for ATP synthase B) Gene for collagen C) Cell cycle regulatory genes D) Genes for biosynthesis of chitin
Version 1
1
5) Animals are thought to have descended from choanoflagellate protists. Which poriferan cell type lends evidence to this claim? A) Gamete B) Choanocyte C) Spicule D) Amoebocyte
6) Sea urchins and jellyfish are both radially symmetrical, but sea urchins and humans are much more closely related than sea urchins and jellyfish. Why is this true? A) Humans are radiallysymmetrical during most of their embryonic development. B) Sea urchins havebilaterally symmetrical larvae and exhibit radial cleavage. C) Jellyfish havebilaterally symmetrical, triploblastic larvae. D) Humans aredescendents of sea urchins not jellyfish.
7) Linnaeus lumped all of the worm-like animals into the category Vermes (Vermis is Latin for worm). Is this a valid classification scheme? A) No, because someworms are protists. B) Yes, because allworms are members of the same phylum. C) Yes, because allworm-like animals are more closely related to each other than they are to otheranimals. D) No, because the worm-like body plan has appeared many times through evolutionary history and is not a useful trait for classification.
8)
Is the group "invertebrates" a valid monophyletic group?
Version 1
2
A) Yes, because it includes all organisms without backbones B) No, because the common ancestor to all the invertebrates is also an ancestor of the vertebrates C) Yes, because only vertebrates are deuterostomes D) No, because most animals are vertebrates
9)
Of the following features, which is the most widely shared in the animal kingdom? A) Protostome development B) Radial cleavage C) Molting of a cuticle or exoskeleton D) A body plan with symmetry E) A triploblastic embryo
10)
Can a blue crab find its identical twin? A) No, spiralcleavage is determinate. B) Yes, crab larvaereproduce via mitosis. C) Yes, theseparation of embryonic cells is rare but not lethal. D) No, arthropods are deuterostomes, and twinning only occurs in protostomes.
11)
Pseudocoelomate nematodes lack a body cavity. A) Correct, theirpseudocoel does not allow for the development and expansion of
organs. B) Incorrect, theirbody cavity is fully enclosed by ectoderm but is a functional body cavity. C) Correct, they aresolid from the gut to the cuticle. D) Incorrect, although not fully enclosed by mesoderm, the pseudocoel is a body cavity.
Version 1
3
12) Given the distribution of the 30 or so animal phyla across the Earth's various ecosystems, what would be the most likely habitat for the origin of animals? A) Rain forests B) Freshwaterhabitats C) Deserts D) Oceans E) Temperate grassland
13) Of the following combination of statements about protostomes and deuterostomes, which choice is correct? A) Protostomes are animals in which the mouth develops from the blastopore, and the anus or anal pore develops from the second opening. Deuterostomes are animals in which the anus develops from the blastopore, and the mouth develops secondarily later in their development. B) Protostomes are animals in which the anus develops from the blastopore, and the mouth develops from the second opening. Deuterostomes are animals in which the anus develops from the blastopore, and the mouth develops secondarily later in their development. C) Protostomes are animals in which the mouth develops from the blastopore, and the anus or anal pore develops from the second opening. Deuterostomes are animals in which the mouth develops from the blastopore, and the anus develops secondarily later in their development. D) Protostomes are animals in which the mouth or anus develops from the blastopore, depending on the species. Deuterostomes are animals in which the mouth and anus develops from the blastopore, depending on the species.
14)
Protostomes develop through __________. A) spiral cleavage B) radial cleavage C) axial cleavage D) polar cleavage E) protocleavage
Version 1
4
15) A biologist discovered a new animal. Upon studying its embryonic development, she observed radial cleavage with the blastopore developing into an anus. This animal was categorized as a __________. A) parazoan B) radiata C) deuterostome D) protostome E) pseudocoelomate
16)
Solid worms that lack a body cavity are known as __________. A) acoelomates B) pseudocoelomates C) triploblastic D) coelomates E) diploblastic
17)
Which of the following terms is mismatched with its meaning or characteristics? A) Diploblastic: ectoderm and mesoderm B) Triploblastic: ectoderm, mesoderm, and endoderm C) Parazoa: lacks symmetry; no tissues D) Eumetazoa: definite symmetry; tissues organized E) Chordate: animal with notochord
18)
What are the three animal phyla that dominate animal life on land?
Version 1
5
A) Cnidaria, Mollusca, Platyhelminthes B) Porifera, Arthropoda, Nematoda C) Nematoda, Chordata, Cnidaria D) Mollusca, Arthropoda, Chordata E) Platyhelminthes, Nematoda, Arthropoda
19) Segmentation is the subdivision of the body into segments. True segmentation is found in which of the following phyla? A) Annelida, Mollusca, Arthropoda B) Echinodermata, Platyhelminthes, Cnidaria C) Mollusca, Chordata, Annelida D) Arthropoda, Chordata, Porifera E) Arthropoda, Annelida, Chordata
20) Molecular data confirm that cnidarians branched off from the rest of animals before ____________ evolved. A) echinoderms B) Porifera C) mollusks D) bilaterial symmetry E) arthropods
21)
Parazoa __________. A) are pseudocoelomates B) exhibit bilateral symmetry C) include Cnidaria and Ctenophora D) exhibit primitive segmentation E) have no true tissues
Version 1
6
22)
Which one of the following terms applies to the phylum Platyhelminthes? A) Radial cleavage B) Deuterostome C) Diploblastic D) Acoelomate E) Radial symmetry
23)
Which one of the following terms applies to the phylum Nematoda? A) Parazoa B) Diploblastic C) Deuterostome D) Pseudocoelomate
24)
Which one of the following terms applies to the phylum Mollusca? A) Protostome B) Pseudocoelomate C) Parazoa D) Diploblastic E) Asymmetry
25)
Which one of the following terms applies to the phylum Cnidaria? A) Radial symmetry B) Pseudocoelomate C) Parazoa D) Triploblastic E) Protostome
Version 1
7
26) Select the phylum that includes deuterostome animals that are exclusively marine and have radial symmetry as adults. A) Porifera B) Rotifera C) Echinodermata D) Chordata E) Cnidaria
27)
Which one of the following terms applies to the phylum Bryozoa? A) Radial symmetry B) Lophophore C) Parazoa D) Deuterostome E) Radial cleavage
28) Select the following phylum that includes protostome animals that are segmented, bilaterally symmetric, and have a chitinous exoskeleton. A) Arthropoda B) Annelida C) Mollusca D) Brachiopoda E) Platyhelminthes
29) Select the following phylum that includes coelomate, bilaterian animals that have a structure called a notochord.
Version 1
8
A) Echinodermata B) Annelida C) Arthropoda D) Mollusca E) Chordata
30) Select the following phylum that includes animals that are unsegmented, acoelomate, and have a digestive cavity with a single opening. A) Annelida B) Platyhelminthes C) Nematoda D) Echinodermata E) Rotifera
31)
Which one of the following features is unique to animals? A) Cells organized into tissues B) Movement associated with muscle tissue and nervous tissue C) Heterotrophism D) Cells without walls E) Embryonic development
32) Dibenzoylhydrazines are a category of molecules that have been used as insecticides. They work by blocking ecdysteroid receptors. How does this kill an insect? A) The insect can nolonger excrete nitrogenous wastes. B) The insect cannotmolt. C) The insect can nolonger coordinate its movements. D) The chitinousexoskeleton is dissolved.
33)
Which feature is an evolutionary novelty of hexapods?
Version 1
9
A) Jointedappendages B) Mandibles C) Chitinousexoskeletons D) Wings E) Antennae
34)
In what way are a human and a crab similar? A) Vertebrates andarthropods are deuterostomes. B) Both have musclesthat pull against rigid skeletons. C) Both have closedcirculatory systems. D) Both have a continuous growth pattern.
35) Scorpions have a prosoma, pedipalps that are modified into claws, and chelicerae. Which taxonomic group are they a member of? A) Chelicerata B) Hexapoda C) Crustacea D) Myriapoda
36)
The lophophore of a brachiopod serves the same function as the __________ in a bivalve. A) mantle B) foot C) shell D) gills
37) Pinworm is a nematode parasite that lives in the human rectum and causes itching in the anal area. What best explains the itching symptom?
Version 1
10
A) Any kind of parasite will naturally irritate the skin. B) The itching induces the person to scratch, helping to spread the parasite to another person. C) The itching is a side-effect when the immune system has successfully attacked the nematode. D) The itching is because of the mismatch between humans and the parasite’s natural host.
38) How could a paleontologist distinguish between a bryozoan fossil and a brachiopod fossil? A) Bryozoans are large and solitary. B) Zoecia are always found in aggregates. C) You can only distinguish the two by examining the lophophore. D) Bryozoans often look like bivalves.
39)
What is a simple way for determining if an annelid is a polychaete or a clitellate? A) The number ofsegments B) Possession of a closed circulatory system C) The amount of chaetae (setae) D) The layout of thedigestive system
40) A snail in your garden and an oyster are quite different in appearance. What would be evidence to justify grouping them together? A) Both use a radulafor feeding. B) Both have amantle capable of secreting calcium carbonate. C) Both use siphonsto obtain oxygen. D) Both have sensoryantenna.
41)
If a hiker picked up a mollusk on a trail in a rain forest, what class would it belong to?
Version 1
11
A) Gastropoda B) Polyplacophora C) Bivalvia D) Cephalopoda
42) Paramecia use cilia for locomotion and for gathering organic matter. How are rotifers similar? A) Rotifers are single-celled protists. B) The corona is made of cilia, and used for swimming and feeding. C) Rotifers and ciliates are thought to have given rise to all other animal phyla. D) Both lack a gut.
43) Turbellarian flatworms have a distict head. When comparing the heads of these flatworms to insect heads, what is itmissing? A) Sensoryorgans B) Light sensitivestructures C) Anteriorcollection of nerve cells D) A mouth
44)
Why doesn't a tapeworm have a mouth on its scolex? A) The mouth is on the proglottids. B) Endoparasites don't need a mouth. C) The mouth is located at the end of the pharynx. D) Tapeworms use a radula instead of a mouth to feed.
45) According to the modern classification scheme, which animal would be a spider's closest relative?
Version 1
12
A) A snail B) An earthworm C) A tapeworm D) A roundworm E) A squid
46)
The phylum that includes snails, clams, oysters, and octopuses is the __________. A) Ectoprocta B) Brachiopoda C) Mollusca D) Annelida E) Phoronida
47) Many marine mollusks have distinctive larvae that have their bodies encircled by a row of cilia. These larvae are called __________. A) planulae B) polyps C) miracidia D) zoecia E) trochophores
48)
The phyla Bryozoa and Brachiopodahave which feature in common? A) Nephridia B) A lophophore C) A radula D) Setae E) A muscular foot
Version 1
13
49)
Annelids possess all of the following except __________. A) muscles to swim, crawl, and burrow B) ganglia to respond to light and respond to other environmental cues C) circulatory, excretory, and neural elements in each segment D) setae in each segment E) adductor muscles
50) All of the following are either structures or characteristics of members of the Class Bivalvia of the Phylum Mollusca except __________. A) the mantle B) a radula C) a shell D) gills E) open circulation
51) The evolutionary innovation that first appeared in arthropods and is characteristic of the most successful of all animal groups is that of __________. A) bilateral symmetry B) coelomic body architecture C) jointed appendages D) segmentation E) three primary types of tissues
52)
In terms of the number of species, the most successful class of arthropods is __________.
Version 1
14
A) Hexapoda B) Chelicerata C) Crustacea D) Chilopoda E) Myriapoda
53)
The passage of an arthropod through stages from egg to adult is called __________. A) differentiation B) evolution C) graduation D) metamorphosis E) succession
54)
Nauplius, a unique kind of larva, is characteristic of __________. A) mites B) crustaceans C) horseshoe crabs D) insects E) chelicerates
55)
In arthropods, locomotion is accomplished by muscles that work against __________. A) each other B) the exoskeleton C) a hydrostatic skeleton D) mineralized bones E) the pseudocoel
Version 1
15
56) The rigid chitinous exoskeleton of an arthropod has all of the following functions except it __________. A) provides a place for muscle attachment B) protects the animal from predators C) prevents water loss D) maintains a uniform size for all individuals of that species E) prevents injury to the animal
57) An invertebrate biologist is conducting research with crabs. Her interest is ecdysis. This implies that she studies __________. A) tagmatization processes in crustaceans B) how ommatidia function individually in a compound eye C) how the HOX gene controls the fusion of the head and thorax into the cephalothorax D) how ocelli evolved from mere photoreceptors to imaging lens E) the molting process
58) A marine biologist makes a presentation about crustaceans to your biology class. He explains that he has recently received an NSF grant to study sessile crustaceans. You are only aware of one group of sessile crustaceans. This crustacean must be __________. A) a lobster B) a crayfish C) a shrimp D) a barnacle E) the nauplius
59)
What do echinoderms and chordates have in common?
Version 1
16
A) Endoskeletons B) Dorsal nerve cords C) Protostome development D) Ecdysis
Version 1
17
Answer Key Test name: Chap 27_3e 1) A
Version 1
18
Clarify Question •What is the key concept addressed by the question? o This question addresses sponge morphology. • What type of thinking is required? o This question is asking you to take what you already know and apply it to this unfamiliar situation. Gather Content • What do you already know about sponge morphology? o A small, anatomically simple sponge has a vaselike shape. The walls of the “vase” have three functional layers. o Facing the internal cavity are flagellated cells called choanocytes, or collar cells. Once water has passed through a flagellated chamber, it travels through channels that converge at a large opening called an osculum through which water is expelled from the sponge. o The body of a sponge is bounded by an outer epithelium consisting of flattened cells somewhat like those that make up the outer layers of animals in other phyla. o Pores on the sponge allow water to enter the channels that course through its body, leading to and from the flagellated chambers. The name of the phylum, Porifera, refers to these pores, or ostia. o Some epithelial cells are specialized to surround the ostia; they can contract when touched or exposed to appropriate stimuli, causing the ostia to close, thereby protecting the delicate inner cells from the entry of potentially harmful substances such as sand and noxious chemicals. Choose Answer • Given what you now know, what information and/or problemsolving approach is most likely to produce the correct answer? o The choanocyte flagella are necessary to circulate water through the Version 1
19
sponge. Reflect on Process • Did your problem-solving process lead you to the correct answer? If not, where did the process break down or lead you astray? How can you revise your approach to produce a more desirable result? o The question required you to take what you already know and apply it to this unfamiliar situation. o Did you recognize that the choanocytes move water through the sponge body using their flagella? 2) C
Version 1
20
Clarify Question •What is the key concept addressed by the question? o This question addresses sponge and jellyfish morphology. • What type of thinking is required? o This question is asking you to compare the process of digestion between sponges and box jellies. Gather Content • What do you already know about sponge morphology? o A small, anatomically simple sponge has a vaselike shape. The walls of the “vase” have three functional layers. o Facing the internal cavity are flagellated cells called choanocytes, or collar cells. Once water has passed through a flagellated chamber, it travels through channels that converge at a large opening called an osculum through which water is expelled from the sponge. o The body of a sponge is bounded by an outer epithelium consisting of flattened cells somewhat like those that make up the outer layers of animals in other phyla. o Pores on the sponge allow water to enter the channels that course through its body, leading to and from the flagellated chambers. The name of the phylum, Porifera, refers to these pores, or ostia. o Some epithelial cells are specialized to surround the ostia; they can contract when touched or exposed to appropriate stimuli, causing the ostia to close, thereby protecting the delicate inner cells from the entry of potentially harmful substances such as sand and noxious chemicals. What do you know about jelly fish morphology? o Jellyfish are Cnidarians that can capture prey. oCnidarianshave tentaclesaround their mouth containing nematocytes. The nematocytes have a nematocyst that sting prey, functioning like a harpoon. Version 1
21
o Cnidarians have an extracellular cavity that contains enzymes to breakdown food that is then absorbed into cells that line the cavity. oExtracellular digestion (having a cavity)is what allows an organismto eat larger prey. Choose Answer • Given what you now know, what information and/or problemsolving approach is most likely to produce the correct answer? o Jellyfish have a gastrovascular cavity for digestion. Reflect on Process • Did your problem-solving process lead you to the correct answer? If not, where did the process break down or lead you astray? How can you revise your approach to produce a more desirable result? o The question required you to take what you already know and apply it to this unfamiliar situation. o Did you recognize that thejellyfish are cnidarians that have extracellular digestion? 3) A
Version 1
22
Clarify Question •What is the key concept addressed by the question? o This question addresses sponge morphology. • What type of thinking is required? o This question is asking you to take what you already know and apply it to this unfamiliar situation. Gather Content • What do you already know about sponge morphology? o A small, anatomically simple sponge has a vaselike shape. The walls of the “vase” have three functional layers. o Facing the internal cavity are flagellated cells called choanocytes, or collar cells. Once water has passed through a flagellated chamber, it travels through channels that converge at a large opening called an osculum through which water is expelled from the sponge. o The body of a sponge is bounded by an outer epithelium consisting of flattened cells somewhat like those that make up the outer layers of animals in other phyla. o Pores on the sponge allow water to enter the channels that course through its body, leading to and from the flagellated chambers. The name of the phylum, Porifera, refers to these pores, or ostia. o Some epithelial cells are specialized to surround the ostia; they can contract when touched or exposed to appropriate stimuli, causing the ostia to close, thereby protecting the delicate inner cells from the entry of potentially harmful substances such as sand and noxious chemicals. Choose Answer • Given what you now know, what information and/or problemsolving approach is most likely to produce the correct answer? o The collar is made up of choanocytes, and they are necessaryfor Version 1
23
circulating water through the sponge. Reflect on Process • Did your problem-solving process lead you to the correct answer? If not, where did the process break down or lead you astray? How can you revise your approach to produce a more desirable result? o The question required you to take what you already know and apply it to this unfamiliar situation. o Did you recognize that the collar is where the choanocytes are located and that they move water through the sponge body using their flagella? 4) B
Version 1
24
Clarify Question • What is the key concept addressed by the question? o This question addresses animal genes. • What type of thinking is required? o This question is asking you to take what you already know and apply it to this unfamiliar situation. Gather Content • What do you already know about animal genes? o Some of the genes listed are expressed in animals, some are not. Only one is expressed in animals but not other organisms. o ATP synthase: This enzyme synthesizes ATP, which is used by many organisms. The enzyme is not unique to animals. o Cell cycle regulatory genes: All multicellular organisms have to regulate their cell cycle. The genes are not unique to animals. o Genes for biosynthesis of chitin: Chitin is a polysaccharide found in arthropods and mollusks, but also in fungi and algae. Chitin is not unique to animals. Choose Answer • Given what you now know, what information and/or problemsolving approach is most likely to produce the correct answer? oCollagen isa protein sometimes used to hold the framework of animal cells together. Animal cells are flexible since they lack cell walls and therefore need to be connected in some manner. Collagen is only found in animals. Reflect on Process •Did your problem-solving process lead you to the correct answer? If Version 1
25
not, where did the process break down or lead you astray? How can you revise your approach to produce a more desirable result? o The question required you to take what you already know and apply it to this unfamiliar situation. o Did you recognize that collagen is an extracellular protein used to hold animals cells together and is unique to animals? 5) B 6) B 7) D
Version 1
26
Clarify Question • What is the key concept addressed by the question? o This question addresses animal classification. • What type of thinking is required? o This question is asking you to weigh and judge evidence, or evaluate, to choose the best of the possible answers. Gather Content • What do you already know about animal classification? o Animals are classified according to their evolutionary relationships. o In cladistics, scientists attempt to group animals into monophyletic clades, that is, a group in which all members are more closely related to each other than any members are to another type of organism. Choose Answer • Given what you now know, what information and/or problemsolving approach is most likely to produce the correct answer? o Consider some different types of “worms”: o Acoel flatworms: in the phylum Acoela, the sister group to all other Bilateria. o Platyhelminthes: flatworms like planaria. o Annelida: segmented worms, including earthworms and leeches. o Nematoda: tiny roundworms, such as C. elegans. o Nemertea: a group of marine worms. o Chaetognatha: arrow worms, which have an uncertain phylogenetic placement. o It is clear that the worm body plan has been a common solution hit upon during evolution of the animals. It has appeared many times through evolutionary history and is not a useful trait for classification. Version 1
27
Reflect on Process • Did your problem-solving process lead you to the correct answer? If not, where did the process break down or lead you astray? How can you revise your approach to produce a more desirable result? o The question required you to weigh and judge evidence, or evaluate, to choose the best of the possible answers. o Did you recognize that “worm” body plans are common in distantly related animal phyla, and thus provide no useful classification information? 8) B
Version 1
28
Clarify Question •What is the key concept addressed by the question? o This question is asking about animal phylogeny. • What type of thinking is required? o This question is asking you to weight and judge, or evaluate whether the category of invertebrates could also be called monophyletic. Gather Content • What do you already know about animal phylogeny and invertebrates? What other information is related to the question? o Amonophyletic group includes the most recent common ancestor of the group and all of its descendants. Aparaphyleticgroup includes the most recent common ancestor of the group, but not all its descendants, and apolyphyleticgroup does not include the most recent common ancestor of all members of the group. o Invertebrates are organisms that lack a vertebral column. Choose Answer • Given what you now know, what information is most likely to produce the correct answer? o Invertebrates share a common ancestor with vertebrates, therefore they cannot be monophyletic. Reflect on Process o Answering this question correctly depended on your ability to weight and judge, or evaluate, animal phylogeny and the category of invertebrates. If you got an incorrect answer, did you remember that a monophyletic group contains the common ancestor and all descendants? 9) D
Version 1
29
Clarify Question • What is the key concept addressed by the question? o This question addresses features of animals. • What type of thinking is required? o This question is asking you to analyze the information given, using logic, to dissect the problem and determine the answer. Gather Content • What do you already know about features of animals? o The animal features listed were generally maintained in large clades after they evolved. o These features describe aspects of development, but they also characterize large clades that share each feature. Choose Answer • Given what you now know, what information and/or problemsolving approach is most likely to produce the correct answer? o Consider each feature in turn: o Molting of a cuticle or exoskeleton: This is a characteristic of the Ecdysozoa, a subset of the protostomes. o Protostome development: This is found naturally in all the protostomes. o Radial cleavage: This is a characteristic of the deuterostomes. o A triploblastic embryo: These are found in all the Bilateria. o A body plan with symmetry: This is a feature not just of the bilateral animals (Bilateria), but also of animals with radial symmetry (Cnidaria). Only the Porifera have an asymmetrical body plan. Thus, “symmetry” is the most widespread feature. Reflect on Process Version 1
30
• Did your problem-solving process lead you to the correct answer? If not, where did the process break down or lead you astray? How can you revise your approach to produce a more desirable result? o The question required you to analyze the information given, using logic, to dissect the problem and determine the answer. o Did you recognize that symmetry—whether radial or bilateral—is the oldest and most widely shared animal feature? Only the sponges lack symmetry. 10) A 11) D 12) D
Version 1
31
Clarify Question • What is the key concept addressed by the question? o This question addresses animal phyla. • What type of thinking is required? o This question is asking you to take what you already know and apply it to this unfamiliar situation. Gather Content • What do you already know about animal phyla? o Consider the major animal phyla from Table 33.1: o Arthropoda: marine, terrestrial, a few freshwater o Mollusca: mostly marine, a few terrestrial and freshwater o Chordata: marine, terrestrial, and freshwater o Platyhelminthes: marine and freshwater, a few terrestrial o Nematoda: marine, freshwater and terrestrial o Annelida: marine, freshwater and terrestrial o Porifera: marine o Echinodermata: marine o Cnidaria: marine, a few terrestrial o Bryozoa: marine Choose Answer • Given what you now know, what information and/or problemsolving approach is most likely to produce the correct answer? o The commonality between the animal phyla is marine habitat. Some phyla are entirely or almost entirely restricted to a marine habitat. o This makes sense since the Cambrian explosion of animal phyla occurred in the oceans.
Version 1
32
Reflect on Process • Did your problem-solving process lead you to the correct answer? If not, where did the process break down or lead you astray? How can you revise your approach to produce a more desirable result? o The question required you to take what you already know and apply it to this unfamiliar situation. o Did you recognize that many of the animal phyla live mostly or entirely in the ocean, reflecting their evolutionary origin? 13) A 14) A 15) C
Version 1
33
Clarify Question • What is the key concept addressed by the question? o This question addresses animal development. • What type of thinking is required? o This question is asking you to take what you already know and apply it to this unfamiliar situation. Gather Content • What do you already know about animal development? o In spiralian protostomes, embryonic cells cleave in a spiral pattern and exhibit determinate development; the blastopore becomes the animal’s mouth, and the coelom originates from a split among endodermal cells. o In deuterostomes, embryonic cells cleave radially and exhibit indeterminate development; the blastopore becomes the animal’s anus, and the coelom originates from an invagination of the archenteron. Choose Answer • Given what you now know, what information and/or problemsolving approach is most likely to produce the correct answer? o The group of animals known for radial cleavage, and development of the blastopore into the anus is the deuterostomes. Reflect on Process • Did your problem-solving process lead you to the correct answer? If not, where did the process break down or lead you astray? How can you revise your approach to produce a more desirable result? o The question required you to take what you already know and apply it to this unfamiliar situation. Version 1
34
o Did you recognize that the deuterostomes are the group of animals that are characterized by radial cleavage, and development of the blastopore into the anus? 16) A 17) A 18) D 19) E 20) D 21) E 22) D 23) D 24) A 25) A 26) C 27) B 28) A 29) E 30) B 31) B 32) B
Version 1
35
Clarify Question • What is the key concept addressed by the question? o This question addresses ecdysis. • What type of thinking is required? o This question is asking you to analyze the information given, using logic, to dissect the problem and determine the answer. Gather Content • What do you already know about ecdysis? o The Ecdysozoa comprises animals that molt. When an animal grows large enough that it completely fills its hard external skeleton, it must lose that skeleton by molting (ecdysis). o While the animal grows, it forms a new exoskeleton underneath the existing one. The first step in molting is for the body to swell until the existing exoskeleton cracks open and is shed. Upon molting that skeleton, the animal inflates the soft, new one, expanding it using body fluids (and sometimes air). When the new exoskeleton hardens, it is larger than the old one and has room for growth. o So unlike other animals, the growth of ecdysozoans is step-wise. Choose Answer • Given what you now know, what information and/or problemsolving approach is most likely to produce the correct answer? o Ecdysteroid hormones bind to ecdysteroid receptors to regulate molting. o So these insecticides kill insects by blocking the ability to molt. Reflect on Process
Version 1
36
• Did your problem-solving process lead you to the correct answer? If not, where did the process break down or lead you astray? How can you revise your approach to produce a more desirable result? o The question required you to analyze the information given, using logic, to dissect the problem and determine the answer. o Did you recognize that ecdysteroid hormones and their receptors are essential for insect molting? 33) D 34) B
Version 1
37
Clarify Question • What is the key concept addressed by the question? o This question addresses crustaceans. • What type of thinking is required? o This question is asking you to take what you already know and apply it to this unfamiliar situation. Gather Content • What do you already know about crustaceans? o A crab is a crustacean, which is a member of the arthropods, part of the Ecdysozoa. o Like other arthropods, crabs molt periodically, they have an open circulatory system, and they have an external rigid skeleton called the exoskeleton. Choose Answer • Given what you now know, what information and/or problemsolving approach is most likely to produce the correct answer? o Are both vertebrates and arthropods deuterostomes? No. There are a few invertebrates that are deuterostomes, but arthropods like a crab are protostomes. o Do both vertebrates and arthropods have closed circulatory systems? No. Vertebrates have a closed circulatory system, while arthropods have an open circulatory system. o Do both vertebrates and arthropods have a continuous growth pattern? No. Vertebrates do, but arthropods growth in steps at each molt. o Do both vertebrates and arthropods have muscles that pull against rigid skeletons? Yes! In vertebrates, muscles pull against a rigid endoskeleton of bone; in arthropods, muscles pull against a rigid Version 1
38
exoskeleton of chitin. Reflect on Process • Did your problem-solving process lead you to the correct answer? If not, where did the process break down or lead you astray? How can you revise your approach to produce a more desirable result? o The question required you to take what you already know and apply it to this unfamiliar situation. o Did you recognize that arthropods like a crab also use muscles against a rigid skeleton? Their skeleton, however, is on the outside, an exoskeleton. 35) A 36) D
Version 1
39
Clarify Question • What is the key concept addressed by the question? o This question addresses the lophophore. • What type of thinking is required? o This question is asking you to take what you already know and apply it to this unfamiliar situation. Gather Content • What do you already know about the lophophore? o The two marine phyla Bryozoa and Brachiopoda have a lophophore, a circular or U-shaped ridge around the mouth with one or two rows of ciliated tentacles. o The lophophore functions as a surface for gas exchange, and the cilia of the lophophore serve to guide the organic detritus and plankton on which the animal feeds to the mouth. o Because of the lophophore, bryozoans and brachiopods have been considered related to one another, but some recent data indicate that the structures may have evolved convergently. Choose Answer • Given what you now know, what information and/or problemsolving approach is most likely to produce the correct answer? o Lophophores function as both respiratory and feeding structures. o Their feathery structure provides high surface area for gas exchange. o In bivalves—mollusks like clams and oysters—a similar structure for gas exchange is the set of internal gills. Reflect on Process • Did your problem-solving process lead you to the correct answer? If Version 1
40
not, where did the process break down or lead you astray? How can you revise your approach to produce a more desirable result? o The question required you to take what you already know and apply it to this unfamiliar situation. o Did you recognize that the lophophore is both a respiratory and feeding structure? 37) B
Version 1
41
Clarify Question • What is the key concept addressed by the question? o This question addresses nematode diseases. • What type of thinking is required? o This question is asking you to take what you already know and apply it to this unfamiliar situation. Gather Content • What do you already know about nematode diseases? o About 50 species of nematodes, including ones in the United States, parasitize humans. o Pinworms, Enterobius vermicularis, infect 11% of the people in the United States, mainly children. Adult pinworms live in the human rectum where they usually cause nothing more serious than itching of the anus; large numbers, however, can lead to prolapse of the rectum. The worms can easily be killed by drugs. Choose Answer • Given what you now know, what information and/or problemsolving approach is most likely to produce the correct answer? o A common need for every parasite is the necessity for offspring to travel to a new host. This is achieved in a variety of ways. Some use an intermediate host, like the mosquito. o In the pinworm, however, the itching symptom induces the host to scratch, carrying eggs on the hands and allowing them to possibly infect a new host. Reflect on Process
Version 1
42
• Did your problem-solving process lead you to the correct answer? If not, where did the process break down or lead you astray? How can you revise your approach to produce a more desirable result? o The question required you to take what you already know and apply it to this unfamiliar situation. o Did you recognize that causing itching benefits the parasite by helping it to spread? 38) B
Version 1
43
Clarify Question • What is the key concept addressed by the question? o This question addresses lophophorates. • What type of thinking is required? o This question is asking you to take what you already know and apply it to this unfamiliar situation. Gather Content • What do you already know about lophophorates? o Both Bryozoa and Brachiopoda are phyla of marine organisms with a lophophore, a circular or U-shaped ridge around the mouth with one or two rows of ciliated tentacles. The lophophore functions for gas exchange, and the cilia of the lophophore help bring food to the mouth. o Because of the lophophore, bryozoans and brachiopods have been considered related to one another, but some recent data indicate that the structures may have evolved convergently. o Bryozoans are tiny and live in colonies that look like patches of moss on the surfaces of submerged objects. Bryozoans are the only exclusively colonial animals. o Brachiopods and phoronids are solitary lophophorates. o The gut in some brachiopods is U-shaped, as in bryozoans, whereas in others there is no anus at all. Choose Answer • Given what you now know, what information and/or problemsolving approach is most likely to produce the correct answer? o The Bryozoa are always colonial, whereas the brachiopods are solitary. In other words, the zoecia (the shell-like chambers secreted by each bryozoan zooid) are always found in aggregates. Version 1
44
Reflect on Process • Did your problem-solving process lead you to the correct answer? If not, where did the process break down or lead you astray? How can you revise your approach to produce a more desirable result? o The question required you to take what you already know and apply it to this unfamiliar situation. o Did you recognize that bryozoans are always colonial, but brachiopods are solitary? 39) C
Version 1
45
Clarify Question • What is the key concept addressed by the question? o This question addresses annelids. • What type of thinking is required? o This question is asking you to take what you already know and apply it to this unfamiliar situation. Gather Content • What do you already know about annelids? o The annelids comprise two major classes, Polychaeta and Clitellata (which combines the old groups Oligochaeta and Hirudinea). o Polychaetes (“many chaetae”) are marine annelids characterized by numerous bristles, or setae/chaetae. o The Clitellata include earthworms and leeches. They haver fewer chaetae (setae). (“Oligochaete" means few chaetae.) Choose Answer • Given what you now know, what information and/or problemsolving approach is most likely to produce the correct answer? o The most obvious anatomical difference between polychaetes and clitellates is reflected in their names: polychaetes have more chaetae (setae). Reflect on Process • Did your problem-solving process lead you to the correct answer? If not, where did the process break down or lead you astray? How can you revise your approach to produce a more desirable result? o The question required you to take what you already know and apply it Version 1
46
to this unfamiliar situation. o Did you recognize that polychaetes are “bristly worms”? 40) B
Version 1
47
Clarify Question • What is the key concept addressed by the question? o This question addresses mollusks. • What type of thinking is required? o This question is asking you to take what you already know and apply it to this unfamiliar situation. Gather Content • What do you already know about mollusks? o One of the best-known characters of the mollusk phylum is the shell, which protects against predators and adverse environmental conditions. o A mollusk shell is secreted by the outer surface of the mantle. o A typical mollusk shell consists of two layers of calcium carbonate, which is precipitated extracellularly. The outer layer consists of densely packed crystals. In some species, the inner layer is pearly in appearance and is called mother-of-pearl or nacre. Choose Answer • Given what you now know, what information and/or problemsolving approach is most likely to produce the correct answer? o Do a land snail and an oyster both have a radula for feeding? No. The snail does, but oysters are bivalves: they use their gills to filter feed. o Do a land snail and an oyster both use siphons to obtain oxygen? No. The oyster does, but in a land snail, the mantle acts like a lung o Do a land snail and an oyster both have sensory antenna? No. Oysters lack these. o Do a land snail and an oyster both have a mantle capable of secreting calcium carbonate? Yes. This is how they both create their shells.
Version 1
48
Reflect on Process • Did your problem-solving process lead you to the correct answer? If not, where did the process break down or lead you astray? How can you revise your approach to produce a more desirable result? o The question required you to take what you already know and apply it to this unfamiliar situation. o Did you recognize that both land snails and oysters use their mantles to secrete the calcium carbonate that forms their shells? 41) A
Version 1
49
Clarify Question • What is the key concept addressed by the question? o This question addresses mollusks. • What type of thinking is required? o This question is asking you to take what you already know and apply it to this unfamiliar situation. Gather Content • What do you already know about mollusks? o Mollusks (phylum Mollusca) are morphologically diverse and live in different environments. o Mollusks include snails, slugs, clams, scallops, oysters, cuttlefish, octopuses, and others. The shells of many mollusks are beautiful and elegant; some mollusks lack a shell. Mollusks range in size from almost microscopic to huge. o Like all major animal groups, mollusks evolved in the oceans, and most groups have remained there. Marine mollusks are widespread and many are abundant. Snails and slugs have invaded freshwater and terrestrial habitats, and freshwater mussels live in lakes and streams. The flat foot of a snail or slug allows it to crawl on land. Choose Answer • Given what you now know, what information and/or problemsolving approach is most likely to produce the correct answer? o A rain forest trail is not a typical habitat for a mollusk. Nearly all mollusks are aquatic, and most of them are marine aquatic species. Only one particular group of mollusks has successfully invaded terrestrial environments, the snails and slugs. o Therefore, it could only be a gastropod. Version 1
50
Reflect on Process • Did your problem-solving process lead you to the correct answer? If not, where did the process break down or lead you astray? How can you revise your approach to produce a more desirable result? o The question required you to take what you already know and apply it to this unfamiliar situation. o Did you recognize that of all the mollusks, only gastropods are terrestrial? 42) B
Version 1
51
Clarify Question • What is the key concept addressed by the question? o This question addresses rotifers and ciliates. • What type of thinking is required? o This question is asking you to take what you already know and apply it to this unfamiliar situation. Gather Content • What do you already know about rotifers and ciliates? o Rotifers are tiny, bilateral, unsegmented pseudocoelomates. Their ancestors may have resembled flatworms, with which they are classified in the spiralian Platyzoa. o At 50 to 500 μm long, rotifers are smaller than some ciliate protists. But they have complex bodies with three cell layers, highly developed internal organs, and a complete gut. o The corona, a conspicuous ring of cilia at the anterior end, is used for locomotion, and its cilia also sweep food into the rotifer’s mouth. Once food is swallowed, it is crushed with a complex jaw in the pharynx. Choose Answer • Given what you now know, what information and/or problemsolving approach is most likely to produce the correct answer? o Like paramecia, rotifers use cilia for both swimming and feeding. Reflect on Process • Did your problem-solving process lead you to the correct answer? If not, where did the process break down or lead you astray? How can you revise your approach to produce a more desirable result? Version 1
52
o The question required you to take what you already know and apply it to this unfamiliar situation. o Did you recognize that rotifers use the ciliated corona for both locomotion and feeding? 43) D 44) B
Version 1
53
Clarify Question • What is the key concept addressed by the question? o This question addresses tapeworms. • What type of thinking is required? o This question is asking you to analyze the information given, using logic, to dissect the problem and determine the answer. Gather Content • What do you already know about tapeworms? o An adult tapeworm hangs onto the inner wall of its host’s intestine. o It lacks a digestive cavity as well as digestive enzymes, absorbing food from the host’s gut through its outer surface. o The long, flat body of a tapeworm is divided into three zones: the scolex, or attachment structure; the neck; and a series of repetitive sections, the proglottids. o The scolex of many species bears four suckers and may also have hooks. The scolex is not a head: It has neither concentrated nervous tissue nor a mouth. o Each proglottid is a complete hermaphroditic unit, containing both male and female reproductive organs. Proglottids are formed continuously in a growth zone at the base of the neck, with maturing ones being pushed posteriorly as new ones are formed. Choose Answer • Given what you now know, what information and/or problemsolving approach is most likely to produce the correct answer? o The scolex is not a head, and the tapeworm doesn’t have a mouth anywhere. o Since the tapeworm is an intestinal parasite, it doesn't need a mouth. It Version 1
54
absorbs food directly across its outer surface. Reflect on Process • Did your problem-solving process lead you to the correct answer? If not, where did the process break down or lead you astray? How can you revise your approach to produce a more desirable result? o The question required you to analyze the information given, using logic, to dissect the problem and determine the answer. o Did you recognize that endoparasites like tapeworms don’t have a mouth? 45) D
Version 1
55
Clarify Question • What is the key concept addressed by the question? o This question addresses protostome phylogeny. • What type of thinking is required? o This question is asking you to take what you already know and apply it to this unfamiliar situation. Gather Content • What do you already know about protostome phylogeny? o The spider is an arthropod. Arthropods are part of Ecdysozoa, the molting animals. o The other phyla in Ecdysozoa are: Loricifera, Kinorhyncha, Nematoda, Tardigrada, and Onychophora. Choose Answer • Given what you now know, what information and/or problemsolving approach is most likely to produce the correct answer? o The most common representative of these other ecdysozoan groups is the roundworm, a member of Nematoda. Reflect on Process • Did your problem-solving process lead you to the correct answer? If not, where did the process break down or lead you astray? How can you revise your approach to produce a more desirable result? o The question required you to take what you already know and apply it to this unfamiliar situation. o Did you recognize that arthropods and nematodes are closely related? 46) C Version 1
56
47) E 48) B 49) E 50) B 51) C 52) A 53) D 54) B 55) B 56) D 57) E 58) D 59) A
Version 1
57
CHAPTER 28 TRUE/FALSE - Write 'T' if the statement is true and 'F' if the statement is false. 1) All mammals have hair, mammary glands, and a placenta. ⊚ ⊚
true false
2) The major diversification of fishes occured in the Devonian period; since that time fish have changed little. ⊚ ⊚
true false
CHECK ALL THE APPLY. Choose all options that best completes the statement or answers the question. 3) Which of the following characteristics can be found in all fish? Check all that apply. A) Nutritional deficiencies B) Jaws C) Lungs D) Bones E) Vertebral column
MULTIPLE CHOICE - Choose the one alternative that best completes the statement or answers the question. 4) What would be an easy way to determine if a skull is a mammal's skull? A) Look for air sacsin the skull. B) Examine theteeth. C) Look for theremains of the tympanum. D) Measure the braincase volume.
5)
Of the primates listed below, a human’s closest extant relative is a(n)
Version 1
1
A) South American spider monkey. B) mandrill. C) orangutan. D) ring-tailed lemur.
6) For human races to be considered valid biological groupings, the genetic diversity within the group would have to be _________ the genetic diversity between groups. A) lower than B) equal to C) higher than
7)
A chimpanzee'sclosest extant relative is a(n) A) orangutan. B) mandrill. C) South Americanspider mokey. D) human.
8)
The breast meat in a bucket of fried chicken is a muscle that normally allows a bird to A) run. B) jump. C) swim. D) feed. E) fly.
9)
A Tyrannosaurus rex' s closest relative is a
Version 1
2
A) Komodo dragon. B) bullfrog. C) squirrel. D) chicken.
10) A vertebrate paleontologist is hunting through chalk deposits and finds a set of fused clavicles with some other bone fragments. What has she found? A) The remains of an amphibian B) The remains of a lobe-finned fish C) The remains of a bird D) The remains of a mammal
11)
What feature do monkeys share with iguanas but not with bullfrogs? A) eyes B) eggs C) thoracic breathing D) four-chambered heart
12)
What would happen to an egg that had a nonfunctional chorion? A) The embryo wouldsuffocate or dehydrate. B) The embryo wouldstarve in the egg. C) The embryo woulddie due to high levels of toxic metabolic wastes. D) The embryo woulddevelop normally.
13) The single ventricle in an amphibian heart means the oxygenated and deoxygenated blood mix. How do amphibians manage to survive despite this inefficiency?
Version 1
3
A) The absorb large amounts of fluid. B) They transport oxygen across the skin. C) They keep their body temperatures high. D) They move very slowly. E) They keep gills into adulthood.
14)
Frog eggs most closely resemble A) chicken eggs. B) fish eggs. C) platypuseggs. D) alligatoreggs.
15) One of your closest fish relatives is the coelacanth. Where is its equivalent to your forearm? A) The claspers B) The pectoral fin lobe C) The operculum D) The gill arch extensions E) The pelvic fin lobe
16) Sharks have a number of fleshy gill slits instead of a bony gill cover. How does this affect their behavior? A) Their movements are slow due to low oxygen. B) They live only in highly oxygenated water. C) They keep swimming to move water across their gills. D) They do not maintain neutral buoyancy.
17) Scuba divers use a BCD to control buoyancy. It is essentially a bag that can be inflated or deflated to prevent excessive sinking or floating. How do bony fish accomplish this?
Version 1
4
A) They pump their operculum. B) They adjust the level of gasses in their swim bladder. C) They adjust their position using the pectoral and pelvic fins. D) They swim rapidly to saturate the blood with oxygen.
18) A tunicate looks like a sponge. How could a biologist distinguish it from a member of the Porifera? A) Tunicates do not filter feed. B) Only tunicatesproduce gametes. C) Unlike tunicates,sponges are symmetrical. D) Tunicates have a stomach.
19) Manta rays, sting rays, and eagle rays are similar to lampreys in that they have cartilaginous skeletons. How are they different from lampreys? A) They have closedcirculatory systems. B) They havegills. C) They havejaws. D) They have bonyfin lobes.
20)
If an adult human suffered a traumatic injury, could their notochord be damaged?
A) Yes, because the notochord is deep within the body only a severe injury could damage it. B) No, the notochord is encased in bone and it not susceptible to damage. C) Yes, damage to the notochord often results in paralysis. D) No, adult humans do not have a notochord.
21) The mucous sheet produced by the endostyle traps the microscopic food particles in the water. This sheet is located in the pharynx of what animal?
Version 1
5
A) Arrow worm B) Tunicate C) Acorn worm D) Lancelet E) Crinoid
22) Small, fishlike marine chordates, pointed at both ends with no distinguishable head, in which the notochord runs the entire length of the nerve cord, are called A) arrow worms. B) tunicates. C) acorn worms. D) lancelets. E) lampreys.
23)
Most bony fishes have a hard plate that covers the gills on each side of the head called the A) radula. B) pharyngeal slits. C) lateral lines. D) operculum. E) gill arches.
24) One of the most critical adaptations of the reptiles in relation to their life on land is the evolution of the A) amniotic egg. B) allantois egg. C) chorionic egg. D) diploid egg. E) swim bladder.
Version 1
6
25) Birds, like mammals, can regulate their body temperatures within close limits. Therefore birds and mammals are called A) ectothermic. B) poikilothermic. C) endothermic. D) heterothermic.
26) Which of the following is considered the key innovation that started chordates along the evolutionary path that led to vertebrates? A) A flexible rod to which muscles are attached, which allowed lateral movement of the back B) An endoskeleton C) A skin covering the entire body to prevent desiccation D) A hard shell encasing the body E) Amniotic egg
27)
In addition to a vertebral column, all vertebrates have A) a distinctive head or skull. B) an open circulatory system. C) gills. D) scales. E) a tunic.
28) It is generally accepted that the vertebrates that evolved during the early Devonian period are the
Version 1
7
A) sharks and bony fish. B) hominoids. C) birds. D) dinosaurs. E) mammals.
29) Features that have been retained by reptiles from the time they replaced the amphibians as the dominant terrestrial vertebrates include all of the following, except A) the amniotic egg. B) dry skin. C) thoracic breathing. D) reoriented appendages. E) endothermy.
30)
Reptiles can maintain their internal body temperature by A) behavioral mechanisms. B) drinking warm or cool water. C) eating and oxidizing more food. D) an effective insulating covering. E) rerouting blood to distribute heat.
31) Which of the following living reptiles care for their young and have a four-chambered heart, as birds do? A) Turtles B) Crocodiles C) Lizards D) Snakes E) Ttuataras
Version 1
8
32) In terms of number of species, the most successful terrestrial vertebrates that invaded the air are A) insects. B) pterosaurs. C) birds. D) bats. E) flying monkeys.
33)
Birds are different from reptiles in all of the following features except A) endothermy. B) their lack of teeth. C) the presence of feathers. D) the presence of a flight skeleton. E) bipedalism.
34) The changes that were necessary to cope with the heavy energy demands of flight in birds include all of the following except A) efficient respiration. B) efficient circulation. C) endothermy. D) auxiliary air sacs and hollow bones. E) a three-chambered heart.
35)
Mammals are the only vertebrates to possess
Version 1
9
A) hair. B) single loop circulation. C) a four-chambered heart with a pulmonary circuit. D) endothermy. E) live birth.
36)
Monotremes differ from the other mammals in their A) laying of shelled eggs. B) lack ofmilk-producing glands. C) fur. D) carnivorous nature. E) containing a pouch for young to live.
37)
The only marsupial mammal living in North America is the A) opossum. B) raccoon. C) ring-tailed cat. D) weasel. E) wombat.
38) The key specializations that occurred in the evolution of mammals include all of the following except A) warm-bloodedness. B) the presence of a placenta in most. C) different teeth suited to their diet. D) digestive tract specialization. E) reduction in brain size.
Version 1
10
39)
Which of the following statements about tunicates is incorrect? A) Tunicates are nonvertebrate chordates. B) Tunicates are all marine animals. C) Tunicates are sessile as adults. D) Tunicate larvae are tadpole-like with all of the basic chordate characteristics. E) Tunicate larvae can reproduce.
40) Because of its association with tools, the earliest human was called __________ meaning "handy man." A) Homo habilis B) Homo erectus C) Homo sapiens D) Neanderthal E) Cro-Magnon
41) ago.
Our own species, __________, appears to have evolved in Africa less than 600,000 years
A) Homo habilis B) Homo erectus C) Homo sapiens D) Homo hominis E) Homo australopithecus
42)
The European Neanderthals were replaced about 34,000 years ago by
Version 1
11
A) Homo habilis. B) Homo erectus. C) Homo heidelbergensis. D) Neanderthals. E) Cro-Magnons.
43)
All of the following features are true of primates except A) they are mammals with grasping fingers and toes. B) they belong to the order containing lemurs, monkeys, and apes. C) they have binocular vision. D) they haveprehensile tails.
44)
All of the following are true about prosimians except A) they are big-eyedmammals with increased visual acuity. B) only two lineages survive today. C) they are nocturnal. D) they are widespread.
45)
Chimpanzees are considered to be our closest extant relative because A) they have prehensile tails. B) their genome is99% identical to the human genome. C) their upper/lower limbs equally account for bearing the body weight. D) they have flat, spreading noses. E) they are a highly social species.
46) Which of the following evidence obtained from fossil studies is generally accepted as marking the beginning of the hominids?
Version 1
12
A) Large brain development B) Walking upright C) Bipedalism D) Development of lower limbs longer than forelimbs E) Manufacture and use of tools
47) The hominid australopithecines are generally accepted to be different from modern humans because A) only humans are bipedal. B) the human skull is apelike in appearance with a strongly projecting face. C) human brains on an average are at least twice as large in size. D) human arms are longer than legs. E) only humans are meat eaters.
48)
All of the following about Homo habilis is true except A) the species lived in Africa for 500,000 years and then became extinct. B) the species coexisted with dinosaurs. C) the species was replaced by a new kind of human, which had a larger brain. D) the species was small in stature. E) the species was associated with tools.
49) New World monkeys evolved approximately 30 million years ago. These animals are easy to identify because they A) have opposable thumbs and binocular vision. B) have prehensile tails and hair. C) have protruding noses and prehensile tails. D) have flattened noses and prehensile tails. E) have opposable thumbs and protruding noses.
Version 1
13
50)
Which one of the following structures is not found in all adult and larval tunicates? A) Incurrent siphon B) Excurrent siphon C) Gill slits D) Notochord E) Pharynx
51)
Which one of the following was not an adaptation seen in the sharks (Chondrichthyes)? A) Lateral line system B) Internal fertilization and internal development C) Swim bladder D) Teeth E) Calcified cartilaginous skeleton
52) You dissect a specimen of a vertebrate during an anatomy laboratory. It has the following characteristics: lungs, abundant blood capillaries under thin skin, a partially divided heart, a pulmonary circuit, and four walking legs. What is this animal? A) lungfish B) amphibian C) reptile D) placental mammal E) marsupial mammal
53) The invasion of the land by amphibians posed a number of challenges requiring evolutionary innovations. Which one of the following does not match a challenge with a corresponding adaptation found in amphibians?
Version 1
14
A) The force of gravity was greater without water's buoying effect, selecting for wellmuscled legs. B) Additional oxygen was needed for larger muscles, selecting for the partially divided heart and pulmonary circuit. C) Eggs dry out in a terrestrial environment, selecting for the amniotic egg. D) Gills tend to collapse out of the water, selecting for lungs and cutaneous respiration.
54) Which one of the following does not match a component of the amniotic egg with its primary function? A) Shell—porous, protective covering B) Chorion—exchange of respiratory gases; retention of water C) Amnion—encloses embryo within an oxygen-filled cavity D) Allantois—segregates nitrogenous wastes from embryo E) Yolk sac—provides food for the embryo
55) You excavate a fossil of a small reptile during a paleontology field trip. It has the following characteristics: skull with a pair of dorsal temporal openings and a pair of lateral temporal openings; bipedal with legs positioned directly under the pelvic girdle. Dating with radioisotopes shows that the specimen lived 100 million year ago (MYA). You conclude that the specimen belongs to which reptilian group (be specific as possible)? A) Anapsid B) Synapsid C) Diapsid D) Archosaur E) Dinosaur
56) Which one of the following is not an adaptation seen in birds that is necessary for winged flight?
Version 1
15
A) Hollow bones and air sacs B) Bipedalism C) Expansion of the pectoral girdle D) Elongation of the digits of the forearm E) Feathers
57)
Which of the following statements is accurate? A) Anurans (frogs and toads) - amphibians without tails. B) Caudata (salamanders) - amphibians without tails. C) Apoda (caecilians) - burrowing amphibians with large eyes. D) Anura, caudata, and apoda are reptilian groups, not amphibian.
Version 1
16
Answer Key Test name: Chap 28_3e 1) FALSE 2) FALSE 3) [A, B, E] 4) B 5) C
Version 1
17
Clarify Question • What is the key concept addressed by the question? o This question addresses primate relationships. • What type of thinking is required? o This question is asking you to take what you already know and apply it to this unfamiliar situation. Gather Content • What do you already know about primate relationships? o The primates consist of several groups. o Tarsiers, lemurs, and lorises used to be considered prosimians, but it is now realized that this group is paraphyletic, with tarsiers more closely related to monkeys and apes than they are to lemurs and lorises. o Anthropoids include monkeys, apes, and humans. o The monkeys fall into two groups, New World monkeys and Old World monkeys. The mandrill, despite its short tail, is an Old World monkey, not an ape. o Anthropoids that remained in Africa gave rise to two lineages: the Old World monkeys and the hominoids (apes and humans). o The apes include gibbons, orangutans, gorillas, and chimpanzees. The chimpanzees and hominids (humans and human ancestors) are sister groups. Choose Answer • Given what you now know, what information and/or problemsolving approach is most likely to produce the correct answer? o The ring-tailed lemur is the most distantly related to humans. It lies at the base of the primate phylogeny, in the clade of lemurs and lorises. o The South American spider monkey is the second most distant from Version 1
18
humans. It is a New World monkey. o The mandrill is still distantly related to humans. It is an Old World monkey, not an ape. o The orangutan is a hominoid and an ape. Of these options, it is the closest relative to humans. Reflect on Process • Did your problem-solving process lead you to the correct answer? If not, where did the process break down or lead you astray? How can you revise your approach to produce a more desirable result? o The question required you to take what you already know and apply it to this unfamiliar situation. o Did you recognize that mandrills are Old World monkeys, not apes? 6) A
Version 1
19
Clarify Question • What is the key concept addressed by the question? o This question addresses human races. • What type of thinking is required? o This question is asking you to analyze the information given, using logic, to dissect the problem and determine the answer. Gather Content • What do you already know about human races? o Humans have differentiated in their characteristics as they have spread throughout the world. For example, northern Europeans often have blond hair, fair skin, and blue eyes, whereas Africans often have black hair, dark skin, and brown eyes. These traits may play a role in adapting the particular populations to their environments. o All humans are capable of mating with one another and producing fertile offspring. The reasons that they do or do not choose to associate with one another are purely psychological and behavioral (cultural). o The number of groups into which the human species might logically be divided has long been a point of contention. Some contemporary anthropologists divide people into as many as 30 “races,” others as few as three. o The characteristics used to define the races are usually not well correlated with one another, and so the determination of race is always somewhat arbitrary. Indeed, if one were to break the human species into subunits based on overall genetic similarity, the groupings would be very different from those based on skin color or other visual features. o In humans, it is not possible to delimit clearly defined races that reflect biologically differentiated and well-defined groupings. Relatively little of the variation in the human species represents differences Version 1
20
between the described races. The human racial categories do a very poor job in describing the vast majority of genetic variation that exists in humans. Choose Answer • Given what you now know, what information and/or problemsolving approach is most likely to produce the correct answer? o Human races are not valid biological groupings. The reason is simple. Different groups of people have constantly intermingled and interbred with one another during the entire course of history. This constant gene flow has prevented the human species from fragmenting into highly differentiated subspecies. o To be a valid grouping, each race would have to have less genetic variability within it that between races. But in fact, this is not the case. Reflect on Process • Did your problem-solving process lead you to the correct answer? If not, where did the process break down or lead you astray? How can you revise your approach to produce a more desirable result? o The question required you to analyze the information given, using logic, to dissect the problem and determine the answer. o Did you recognize that the human races are not valid biological groupings, because there is so much genetic overlap between them? People of different races may have less genetic variability than people of the “same” race. 7) D 8) E
Version 1
21
Clarify Question • What is the key concept addressed by the question? o This question addresses bird adaptations. • What type of thinking is required? o This question is asking you to take what you already know and apply it to this unfamiliar situation. Gather Content • What do you already know about bird adaptations? o Birds have several adaptations for flight. o The bones of birds are thin and hollow. Many of the bones are fused, making the bird skeleton more rigid than a reptilian skeleton. The fused sections of backbone and of the shoulder and hip girdles form a sturdy frame that anchors muscles during flight. o The power for active flight comes from large breast muscles that can make up 30% of a bird’s total body weight. They stretch down from the wing and attach to the breastbone, which is greatly enlarged and bears a prominent keel for muscle attachment. Choose Answer • Given what you now know, what information and/or problemsolving approach is most likely to produce the correct answer? o The large breast muscles of a bird are the muscles that power the wings in flight. o Chickens can fly, although not well because they have been selected to put on extra mass. However they still have large flight muscles that we call “breast meat.” Reflect on Process Version 1
22
• Did your problem-solving process lead you to the correct answer? If not, where did the process break down or lead you astray? How can you revise your approach to produce a more desirable result? o The question required you to take what you already know and apply it to this unfamiliar situation. o Did you recognize that breast meat is really the bird’s flight muscles? 9) D
Version 1
23
Clarify Question • What is the key concept addressed by the question? o This question addresses reptile-bird relationships. • What type of thinking is required? o This question is asking you to take what you already know and apply it to this unfamiliar situation. Gather Content • What do you already know about reptile-bird relationships? o The first known bird was Archaeopteryx, a 150-million-year-old fossil. It had a skull with teeth, and very few of its bones were fused to one another. Its bones are thought to have been solid, not hollow like a bird’s. Also, it had a long, reptilian tail and no enlarged breastbone such as modern birds use to anchor flight muscles. Finally, the skeletal structure of the forelimbs was nearly identical to those of theropods. o Because of its many dinosaur features, several Archaeopteryx fossils (in which the feathers had not been preserved) were originally misclassified as a small theropod dinosaur. However, it in fact had feathers on its wings and tail. o Almost all paleontologists have concluded that Archaeopteryx is the direct descendant of dinosaurs, indeed, today’s birds are “feathered dinosaurs.” o The dinosaur Caudipteryx, for example, is clearly intermediate between Archaeopteryx and dinosaurs, having large feathers on its tail and arms but also many features of dinosaurs like Velociraptor. Choose Answer • Given what you now know, what information and/or problemsolving approach is most likely to produce the correct answer? Version 1
24
o The closest extant relative to the dinosaurs are the birds. o Therefore the chicken is the correct option. Reflect on Process • Did your problem-solving process lead you to the correct answer? If not, where did the process break down or lead you astray? How can you revise your approach to produce a more desirable result? o The question required you to take what you already know and apply it to this unfamiliar situation. o Did you recognize that birds evolved from a group of dinosaurs? 10) C
Version 1
25
Clarify Question • What is the key concept addressed by the question? o This question addresses bird adaptations. • What type of thinking is required? o This question is asking you to take what you already know and apply it to this unfamiliar situation. Gather Content • What do you already know about bird adaptations? o Birds have several adaptations for flight. o The bones of birds are thin and hollow. Many of the bones are fused, making the bird skeleton more rigid than a reptilian skeleton. The fused sections of backbone and of the shoulder and hip girdles form a sturdy frame that anchors muscles during flight. o The power for active flight comes from large breast muscles that can make up 30% of a bird’s total body weight. They stretch down from the wing and attach to the breastbone, which is greatly enlarged and bears a prominent keel for muscle attachment. Choose Answer • Given what you now know, what information and/or problemsolving approach is most likely to produce the correct answer? o Birds are the group of vertebrates known for fused bones. The fused sections of backbone and of the shoulder and hip girdles form a sturdy frame that anchors muscles during flight. Reflect on Process • Did your problem-solving process lead you to the correct answer? If Version 1
26
not, where did the process break down or lead you astray? How can you revise your approach to produce a more desirable result? o The question required you to take what you already know and apply it to this unfamiliar situation. o Did you recognize that birds have many fused bones to help them fly? 11) C
Version 1
27
Clarify Question • What is the key concept addressed by the question? o This question addresses reptile innovations. • What type of thinking is required? o This question is asking you to take what you already know and apply it to this unfamiliar situation. Gather Content • What do you already know about reptile innovations? o All living reptiles share certain fundamental characteristics that were innovations not seen in their amphibian relatives. These innovations include amniotic eggs, dry skin, and thoracic breathing. Furthermore, their hearts became more efficient, although only one group of reptiles evolved a four-chambered heart. o Amniotic eggs: Most reptiles lay watertight eggs that contain yolk and a series of four membranes. All modern reptiles, as well as birds and mammals, share these membranes and are called amniotes. o Dry skin: Reptiles have dry, watertight skin with a layer of scales that reduces water loss. o Thoracic breathing: Amphibians breathe by squeezing their throat to pump air into their lungs; this limits their breathing capacity to the volume of their mouths. Reptiles developed pulmonary breathing, expanding and contracting the rib cage and diaphragm to suck air into the lungs and then force it out. The capacity of this system is limited only by the volume of the lungs. o Heart chambers: The circulatory system of reptiles is an improvement over that of fish and amphibians, providing oxygen to the body more efficiently. The improvement is achieved by extending the septum within the heart from the atrium partway across the ventricle. This Version 1
28
septum creates a partial wall that tends to lessen mixing of oxygen-poor blood with oxygen-rich blood within the ventricle. Crocodiles independently evolved a fully four-chambered heart, like that in birds and mammals, by extending the septum to completely divide the ventricle. Choose Answer • Given what you now know, what information and/or problemsolving approach is most likely to produce the correct answer? o Do monkeys and iguanas, but not bullfrogs, have eyes? No, they all have eyes. o Do monkeys and iguanas, but not bullfrogs, have eggs? No, iguanas and bullfrogs both lay external eggs, o Do monkeys and iguanas, but not bullfrogs, have a four-chambered heart? No. The monkey has a four-chambered heart, but the heart of an iguana is like those of most other reptiles (besides crocodiles), the septum does not completely cross the ventricle to create four chambers. o Do monkeys and iguanas, but not bullfrogs, have thoracic breathing? Yes, thoracic breathing was a new innovation in the reptiles, not found in the amphibians, and was maintained in the mammals. Reflect on Process • Did your problem-solving process lead you to the correct answer? If not, where did the process break down or lead you astray? How can you revise your approach to produce a more desirable result? o The question required you to take what you already know and apply it to this unfamiliar situation. o Did you recognize that one of the innovations of reptiles was thoracic breathing? Version 1
29
12) A
Version 1
30
Clarify Question • What is the key concept addressed by the question? o This question addresses the amniotic egg. • What type of thinking is required? o This question is asking you to analyze the information given, using logic, to dissect the problem and determine the answer. Gather Content • What do you already know about the amniotic egg? o Amphibianeggs must be laid in water or a moist setting to avoid drying out. Most reptiles lay watertight eggs that contain a food source (the yolk) and a series of four membranes: the yolk sac, the amnion, the allantois, and the chorion. Each membrane plays a role in making the egg an independent life-support system. All modern reptiles, as well as birds and mammals, show exactly this same pattern of membranes within the egg. These three classes are called amniotes. o The outermost membrane of the egg is the chorion, which lies just beneath the porous shell. It allows exchange of respiratory gases but retains water. o The amnion encases the developing embryo within a fluid-filled cavity. o The yolk sac provides food from the yolk for the embryo via blood vessels connecting to the embryo’s gut. o The allantois surrounds a cavity into which waste products from the embryo are excreted. Choose Answer • Given what you now know, what information and/or problemsolving approach is most likely to produce the correct answer? Version 1
31
o The function of the chorion is to allow gases to pass in and out of the egg, but it also allows it to retain water, So if there was a problem with the chorion, the embryo would either suffocate or dehydrate. Reflect on Process • Did your problem-solving process lead you to the correct answer? If not, where did the process break down or lead you astray? How can you revise your approach to produce a more desirable result? o The question required you to analyze the information given, using logic, to dissect the problem and determine the answer. o Did you recognize that the chorion is the outermost membrane of the amniotic egg, and it allows gas in and out but keeps water in? 13) B
Version 1
32
Clarify Question • What is the key concept addressed by the question? o This question addresses amphibian features. • What type of thinking is required? o This question is asking you to take what you already know and apply it to this unfamiliar situation. Gather Content • What do you already know about amphibian features? o Living amphibians have several key characteristics in common: o 1. Legs: These were one of the key adaptations to life on land. o 2. Lungs: Most amphibians possess a pair of lungs, although the internal surfaces have much less surface area than do reptilian or mammalian lungs. Amphibians breathe by lowering the floor of the mouth to suck in air and then raising it back to force the air down into the lungs. o 3. Cutaneous respiration: Frogs, salamanders, and caecilians all supplement the use of lungs by respiring directly through their skin, which is kept moist and provides an extensive surface area for gas exchange. o 4. Pulmonary veins: After blood is pumped through the lungs, two large veins called pulmonary veins return the aerated blood to the heart for repumping. In this way aerated blood is pumped to the tissues at a much higher pressure. o 5. Partially divided heart: A dividing wall helps prevent aerated blood from the lungs from mixing with nonaerated blood being returned to the heart from the rest of the body. The blood circulation is thus divided into two separate paths: pulmonary and systemic. The separation is imperfect, however, because no dividing wall exists in one chamber of Version 1
33
the heart, the ventricle. Choose Answer • Given what you now know, what information and/or problemsolving approach is most likely to produce the correct answer? o One reason that amphibians manage despite a heart that is less efficient than a reptile’s or mammal’s is that they use cutaneous respiration, that is, they can acquire oxygen through their skin. o A few species of amphibians do retain gills into adulthood, but most do not, so this explanation isn’t as powerful. Reflect on Process • Did your problem-solving process lead you to the correct answer? If not, where did the process break down or lead you astray? How can you revise your approach to produce a more desirable result? o The question required you to take what you already know and apply it to this unfamiliar situation. o Did you recognize that living amphibians can get some oxygen directly through their skin? 14) B
Version 1
34
Clarify Question • What is the key concept addressed by the question? o This question addresses eggs. • What type of thinking is required? o This question is asking you to take what you already know and apply it to this unfamiliar situation. Gather Content • What do you already know about eggs? o Both fish and amphibians’ eggs must be laid in water or a moist setting to avoid drying out. o Most reptiles lay watertight “amniotic” eggs: eggs that contain yolk and a series of four membranes. All modern reptiles, as well as birds and mammals, share these membranes and are called amniotes. o Each membrane of the amniotic egg plays a role in making the egg an independent life-support system. The outermost membrane of the egg is the chorion, which lies just beneath the porous shell. It allows exchange of respiratory gases but retains water. The amnion encases the developing embryo within a fluid-filled cavity. The yolk sac provides food from the yolk for the embryo via blood vessels connecting to the embryo’s gut. The allantois surrounds a cavity into which waste products from the embryo are excreted. Choose Answer • Given what you now know, what information and/or problemsolving approach is most likely to produce the correct answer? o Because both fish and amphibians lack the features of the amniotic egg, their eggs are more similar to each other than to eggs of any of the amniotes (reptiles, birds, and mammals). Version 1
35
Reflect on Process • Did your problem-solving process lead you to the correct answer? If not, where did the process break down or lead you astray? How can you revise your approach to produce a more desirable result? o The question required you to take what you already know and apply it to this unfamiliar situation. o Did you recognize that fish and amphibian eggs lack the innovations of the amniotic egg? 15) B
Version 1
36
Clarify Question • What is the key concept addressed by the question? o This question addresses the coelacanths. • What type of thinking is required? o This question is asking you to take what you already know and apply it to this unfamiliar situation. Gather Content • What do you already know about the coelacanths? o The two major groups of bony fish differ in the structure of their fins: the ray-finned fishes and the lobe-finned fishes. o In ray-finned fishes, the internal skeleton of the fin is composed of parallel bony rays that support and stiffen each fin. There are no muscles within the fins: they are moved by muscles within the body. o By contrast, coelacanths have paired fins that consist of a long fleshy muscular lobe supported by a central core of bones that form fully articulated joints with one another. There are bony rays only at the tips of each lobed fin. Muscles within each lobe can move the fin rays independently. o Lobe-finned fish evolved 390 million years ago, shortly after the first bony fishes appeared. Only eight species survive today, two species of coelacanth (subclass Actinistia) and six species of lungfish (subclass Dipnoi). Although rare today, lobe-finned fishes played an important part in the evolutionary story of vertebrates because it was a coelacanthlike animal that crawled out of water and colonized land, giving rise to the first amphibians. Lungfish are more closely related to tetrapods (legged vertebrates) than they are to coelacanths, thus making the Sarcopterygii a paraphyletic group. Choose Answer Version 1
37
• Given what you now know, what information and/or problemsolving approach is most likely to produce the correct answer? o The coelacanth’s lobed fins represent the limbs that evolved into the amphibian legs. o So the pectoral lobe fins (toward the front of the coelacanth’s body) would be homologous to your forearms. Reflect on Process • Did your problem-solving process lead you to the correct answer? If not, where did the process break down or lead you astray? How can you revise your approach to produce a more desirable result? o The question required you to take what you already know and apply it to this unfamiliar situation. o Did you recognize that the coelacanth lobe fins are homologous to your arms and legs? 16) C
Version 1
38
Clarify Question • What is the key concept addressed by the question? o This question addresses the operculum. • What type of thinking is required? o This question is asking you to take what you already know and apply it to this unfamiliar situation. Gather Content • What do you already know about the operculum? o Most bony fishes have a hard plate called the operculum, or gill cover, that covers the gills on each side of the head. Sharks lack an operculum. o In bony fish, flexing the operculum pumps water over the gills. The gills are suspended in the pharyngeal slits that form a passageway between the pharynx and the outside of the fish’s body. When the operculum is closed, it seals off the exit. When the mouth is open, closing the operculum increases the volume of the mouth cavity, so that water is drawn into the mouth. When the mouth is closed, opening the operculum decreases the volume of the mouth cavity, forcing water past the gills to the outside. Using this very efficient bellows, bony fishes can pass water over the gills while remaining stationary in the water. Choose Answer • Given what you now know, what information and/or problemsolving approach is most likely to produce the correct answer? o Since sharks lack an operculum (or gill cover), they keep moving to keep oxygenated water passing over the gills. Reflect on Process
Version 1
39
• Did your problem-solving process lead you to the correct answer? If not, where did the process break down or lead you astray? How can you revise your approach to produce a more desirable result? o The question required you to take what you already know and apply it to this unfamiliar situation. o Did you recognize that the reason sharks must keep moving for respiration is because they lack an operculum, or gill cover? 17) B 18) D
Version 1
40
Clarify Question • What is the key concept addressed by the question? o This question addresses tunicates. • What type of thinking is required? o This question is asking you to take what you already know and apply it to this unfamiliar situation. Gather Content • What do you already know about tunicates? o Tunicates are members of the subphylum Urochordata. Their morphology is simple on the outside, but more complex on the inside. o Water enters through the incurrent siphon and exits through the excurrent siphon. The pharynx is lined with cilia that draw a stream of water into the pharynx, where microscopic food particles become trapped in a mucous sheet secreted from a structure called an endostyle. Captured food particles pass into the stomach and through the intestine. o Tunicates change so much as they mature and adjust developmentally to an immobile, filter-feeding existence that it is difficult to discern their evolutionary relationships solely by examining an adult. Many adult tunicates secrete a tunic, a tough sac composed mainly of cellulose, a substance frequently found in the cell walls of plants and algae but rarely found in animals. The tunic surrounds the animal and gives the subphylum its name. Colonial tunicates may have a common sac and a common opening to the outside. Choose Answer • Given what you now know, what information and/or problemsolving approach is most likely to produce the correct answer? o Sponges are morphologically simple on the outside and the inside, but Version 1
41
tunicates have a gut and stomach inside. The stomach, therefore, identifies the sponge-like animal as a member of the Urochordata. Reflect on Process • Did your problem-solving process lead you to the correct answer? If not, where did the process break down or lead you astray? How can you revise your approach to produce a more desirable result? o The question required you to take what you already know and apply it to this unfamiliar situation. o Did you recognize that tunicates have a full digestive tract inside? 19) C
Version 1
42
Clarify Question • What is the key concept addressed by the question? o This question addresses the evolution of the jaw. • What type of thinking is required? o This question is asking you to take what you already know and apply it to this unfamiliar situation. Gather Content • What do you already know about the evolution of the jaw? o An important evolutionary advance in the late Silurian was the development of jaws. Jaws evolved from gill arches, which in jawless fish were cartilaginous struts used to reinforce the tissue between gill slits to hold the slits open. o Each gill arch was formed by a series of several cartilages (which later evolved to become bones) arranged somewhat in the shape of a V turned on its side, with the point directed outward. Imagine the fusion of the front pair of arches at top and bottom, with hinges at the points, and you have the primitive vertebrate jaw. o Armored fishes (placoderms) and spiny fishes (acanthodians) both had jaws. Spiny fishes and ostracoderms had internal skeletons made of cartilage, but their scales contained small plates of bone. Spiny fishes were jawed predators and far better swimmers than ostracoderms, with many fins reinforced with strong spines. o The diverse and successful placoderms dominated the seas of the late Devonian. The placoderm jaw was much improved over the primitive jaw of spiny fishes, with the upper jaw fused to the skull and the skull hinged on the shoulder. o At the end of the Devonian period, these pioneer vertebrates were replaced by sharks and bony fishes. Version 1
43
o In these fishes, the jaw was improved even further, with the upper part of the first gill arch behind the jaws being transformed into a supporting strut or prop, joining the rear of the lower jaw to the back of the skull. This allowed the mouth to open much wider than was previously possible. Soon, sharks became the dominant predators in the sea. Choose Answer • Given what you now know, what information and/or problemsolving approach is most likely to produce the correct answer? o Like their relatives the sharks, rays have jaws. Lampreys do not. Reflect on Process • Did your problem-solving process lead you to the correct answer? If not, where did the process break down or lead you astray? How can you revise your approach to produce a more desirable result? o The question required you to take what you already know and apply it to this unfamiliar situation. o Did you recognize that a major innovation of the cartilaginous fish was the refinement of the jaw? 20) D
Version 1
44
Clarify Question • What is the key concept addressed by the question? o This question addresses the notochord. • What type of thinking is required? o This question is asking you to weigh and judge evidence, or evaluate, to choose the best of the possible answers. Gather Content • What do you already know about the notochord? o One of the features that characterizes the chordates is the notochord. o The notochord is a flexible rod that forms on the dorsal side of the primitive gut in the early embryo and is present at some developmental stage in all chordates. The notochord is located just below the nerve cord. The notochord may persist in some chordates; in others, it is replaced during embryonic development by the vertebral column that forms around the nerve cord. o During development of the vertebra, the flexible notochord is surrounded and eventually replaced by a cartilaginous or bony covering, the centrum. The neural tube is protected by an arch above the centrum. The vertebral column functions as a strong, flexible rod that the muscles pull against when the animal swims or moves. Choose Answer • Given what you now know, what information and/or problemsolving approach is most likely to produce the correct answer? o Adult humans do not have a notochord. It is replaced by the vertebral column during embryonic development. Reflect on Process Version 1
45
• Did your problem-solving process lead you to the correct answer? If not, where did the process break down or lead you astray? How can you revise your approach to produce a more desirable result? o The question required you to weigh and judge evidence, or evaluate, to choose the best of the possible answers. o Did you recognize that in humans, the notochord is a feature only of the developing embryo? 21) B 22) D 23) D 24) A 25) C 26) A 27) A 28) A 29) E 30) A 31) B 32) C 33) E 34) E 35) A 36) A 37) A 38) E 39) E 40) A 41) C 42) E
Version 1
46
43) D 44) D 45) B 46) C 47) C 48) B 49) D 50) D 51) C 52) B
Version 1
47
Clarify Question • What is the key concept addressed by the question? o This question addresses amphibians. • What type of thinking is required? o This question is asking you to take what you already know and apply it to this unfamiliar situation. Gather Content • What do you already know about amphibians? o Amphibians include frogs, salamanders, and the wormlike, nearly blind organisms called caecilians.Amphibians have several key characteristics in common: o Legs: Frogs and most salamanders have four legs and can move about on land quite well. o Lungs: Most amphibians possess a pair of lungs, although the internal surfaces have much less surface area than do reptilian or mammalian lungs. o Cutaneous respiration: Frogs, salamanders, and caecilians all supplement the use of lungs by respiring directly through their skin, which is kept moist and provides an extensive surface area for gas exchange. o Pulmonary veins: After blood is pumped through the lungs, two large veins called pulmonary veins return the aerated blood to the heart for repumping. In this way aerated blood is pumped to the tissues at a much higher pressure. o Partially divided heart: A dividing wall helps prevent aerated blood from the lungs from mixing with nonaerated blood being returned to the heart from the rest of the body. The separation is imperfect, however, because no dividing wall exists in one chamber of the heart, the Version 1
48
ventricle. Choose Answer • Given what you now know, what information and/or problemsolving approach is most likely to produce the correct answer? o Could the animal be a lungfish? No, they don’t have four proper walking legs. o Could the animal be a reptile? No, they don’t have thin skin with abundant capillaries. Their skin is covered with scales. o Could the animal be a placental or marsupial mammal? No, they have a fully divided, four-chambered heart. o Could the animal be an amphibian? Yes. Amphibians respire partly through their skin, and they lack scales, so their skin is thin and there are abundant capillaries underneath for gas exchange. Reflect on Process • Did your problem-solving process lead you to the correct answer? If not, where did the process break down or lead you astray? How can you revise your approach to produce a more desirable result? o The question required you to take what you already know and apply it to this unfamiliar situation. o Did you recognize that amphibians respire partially through their skin? 53) C
Version 1
49
Clarify Question • What is the key concept addressed by the question? o This question addresses amphibian adaptations to land. • What type of thinking is required? o This question is asking you to take what you already know and apply it to this unfamiliar situation. Gather Content • What do you already know about amphibian adaptations to land? o The successful invasion of land by amphibians posed a number of major challenges: o Because amphibian ancestors had relatively large bodies, supporting the body’s weight on land as well as enabling movement from place to place was a challenge. Legs evolved to meet this need. o The delicate structure of fish gills requires the buoyancy of water to support them, and they cannot function in air. Therefore, other methods of obtaining oxygen were required. o Delivering great amounts of oxygen to the larger muscles needed for movement on land required modifications to the heart and circulatory system. o Reproduction still had to be carried out in water so that eggs would not dry out. o The body itself had to be prevented from drying out. Choose Answer • Given what you now know, what information and/or problemsolving approach is most likely to produce the correct answer? o Of all the features listed, only the amniotic egg is not a feature of amphibians. Version 1
50
o Reptiles were the first amniotes, because they were the first to lay their eggs out of the water. Reflect on Process • Did your problem-solving process lead you to the correct answer? If not, where did the process break down or lead you astray? How can you revise your approach to produce a more desirable result? o The question required you to take what you already know and apply it to this unfamiliar situation. o Did you recognize that amphibians lay their eggs in water? o Did you remember that reptiles first evolved the amniotic egg, not amphibians? 54) C 55) E
Version 1
51
Clarify Question • What is the key concept addressed by the question? o This question addresses reptile skulls and dinosaur adaptations. • What type of thinking is required? o This question is asking you to analyze the information given, using logic, to dissect the problem and determine the answer. Gather Content • What do you already know about reptile skulls and dinosaur adaptations? o An important feature of reptile classification is the presence and number of openings behind the eyes. Reptiles’ jaw muscles were anchored to these holes, which allowed them to bite more powerfully. The first group to rise to dominance were the synapsids, whose skulls had one hole behind the openings for the eyes. (Anapsids had no openings—they are represented among extant reptiles only by the turtles.) o Diapsids have skulls with two pairs of holes on each side of the head, and like amphibians and early reptiles, they were ectotherms. A variety of different diapsids occurred in the Triassic period (213 to 248 mya), but one group, the archosaurs, were of particular evolutionary significance because they gave rise to crocodiles, pterosaurs, dinosaurs, and birds. o Among the early archosaurs were the largest animals the world had seen, up to that point, and the first land vertebrates to be bipedal—to stand and walk on two feet. By the end of the Triassic period, however, one archosaur group rose to prominence: the dinosaurs. o Dinosaurs evolved about 220 mya. Unlike previous bipedal diapsids, their legs were positioned directly underneath their bodies. This design Version 1
52
placed the weight of the body directly over the legs, which allowed dinosaurs to run with great speed and agility. Subsequently, a number of types of dinosaur evolved to enormous size and reverted to a four-legged posture to support their massive weight. Choose Answer • Given what you now know, what information and/or problemsolving approach is most likely to produce the correct answer? o Based on the two sets of holes in the skull, you can rule out the skeleton being an anapsid or synapsid. It must be a diapsid. o The bipedal stance with legs directly under the pelvis tells you that this diapsid was not only an archosaur—it was specifically a dinosaur. Reflect on Process • Did your problem-solving process lead you to the correct answer? If not, where did the process break down or lead you astray? How can you revise your approach to produce a more desirable result? o The question required you to analyze the information given, using logic, to dissect the problem and determine the answer. o Did you recognize that if there are two temporal openings on each side, it must be some kind of diapsid? o Did you recognize that the vertical placement of legs under the pelvis was a dinosaur adaptation? 56) D 57) A
Version 1
53
CHAPTER 29 TRUE/FALSE - Write 'T' if the statement is true and 'F' if the statement is false. 1) Mesophyll tissue is part of the root system of a plant. ⊚ true ⊚ false
CHECK ALL THE APPLY. Choose all options that best completes the statement or answers the question. 2) Which of the following is a true statement about meristems? Select all that apply. A) They can be apical or lateral. B) Meristematic cells divide into two cells, one of which remains a meristem cell and another that becomes a plant body cell. C) Apical meristems give rise to three types of embryonic tissues. D) Meristematic cells are found only in eudicots and not in monocots. E) They are responsible for primary and secondary growth.
3) Which of the following describe characteristics of sclerenchyma cells? Select all that apply. A) Their function is primarily to transport oxygen and carbon dioxide throughout the leaf. B) They have thick, tough secondary walls. C) Their secondary walls may be impregnated with lignin. D) They form fibers and sclereids. E) They lack living protoplasts when mature.
4) In certain plants, some of the roots may be modified to carry out unusual functions. Which of these is an example of one of these special functions? Select all that apply.
Version 1
1
A) Help absorb oxygen B) Store food or water C) Carry out photosynthesis D) Parasitize other plants E) Discourage herbivores
5)
Which of these is a possible function of trichomes? Select all that apply. A) Reflect light from leaf surfaces B) Store or secrete salt C) Structural support D) Irritate potential herbivores E) Defend against insects
6)
Which of the following is a modified stem? Select all that apply. A) Rhizome B) Tendril C) Spine D) Tuber E) Corm
7) Various modifications in leaves can make plants better adapted to their habitats. Which of the following is an evolutionary modification of leaves? Select all that apply. A) Floral leaves B) Fruit-bearing leaves C) Spines D) Reproductive leaves E) Insectivorous leaves
8)
Which statement about secondary growth in plants is correct? Select all that apply.
Version 1
2
A) Secondary growth in plants is a result of lateral meristems. Trees and shrubs have active lateral meristems. B) Secondary growth in plants increases the girth (diameter) of woody plants. C) Secondary growth in woody plants results from two cylinders of actively dividing cells, the cork cambium and vascular cambium. D) The cork cambium produces secondary phloem while the vascular cambium produces secondary xylem. E) Only some plants have secondary growth.
9) Of the following structures, which are a specialized cell of the epidermis? Select all that apply. A) Guard cells B) Sclereids C) Root hairs D) Oil glands E) Trichomes
10)
Which statement concerning root hairs is/are correct? Select all that apply.
A) Root hairs are tubular extensions of individual epidermal cells. B) A root hair is isolated from its epidermal cell with a cross wall. C) Root hairs generally live only a few days before being sloughed off. D) Root hairs help the plant to adhere to the soil but have role in the absorption of water and minerals. E) Root hairs are located in the zone of maturation of a root.
11)
Which of these is/are a function of the root cap? Select all that apply. A) gravity perception B) rapid cell division C) protection of the root tip D) pushing away soil particles as the root grows E) releasing a slimy lubricant fluid
Version 1
3
12)
Which of the following are the three tissue systems of plants? A) Circulatory B) Dermal C) Endocrine D) Ground E) Lymphatic F) Vascular
MULTIPLE CHOICE - Choose the one alternative that best completes the statement or answers the question. 13) Plant cells that give rise to two cells, one of which is free to differentiate into various kinds of cells that contribute to the plant body, are called A) endodermal cells. B) primary cells. C) lateral cells. D) parenchyma cells. E) meristematic cells.
14) Cell division in the apical meristems at the tips of a plant that results in increases in height or length is called A) primary growth. B) secondary growth. C) vascular cambium growth. D) mitotic growth. E) herbaceous growth.
15) A growing root comes into contact with a chemical that inhibits the Golgi bodies. Which of the following would be a probable effect of this contact?
Version 1
4
A) The root would begin to growing in an upward direction. B) The root would not be slick enough to move through the soil easily. C) The root would begin to simultaneously grow in multiple directions. D) The root would produce too much mucigel and oxygen uptake would be inhibited. E) The root cap cells would begin to divide rapidly.
16) You propose an experiment where root hairs of a particular plant species are continuously treated with an antibiotic while nutrient levels in the plant tissue are monitored. Based on your knowledge of root hairs, you hypothesize the plant will experience a deficiency in what nutrient? A) potassium B) nitrogen C) calcium D) phosphorus E) carbon
17)
Why does trimming the top of a plant make the plant bushier?
A) Trimming disrupts vertical transport routes for water and nutrients while enhancing horizontal transport routes. B) Trimming damages or removes the apical meristem, which activates the axillary meristems to grow. C) Trimming stimulates the plant to produce more rhizomes, which results in more branches. D) Trimming forces leaves around the vicinity of the cuts to become reproductive leaves. E) Trimming stimulates the cork cambium to produce more secondary growth.
18)
An increase in plant diameter results from cell division in which type of meristem?
Version 1
5
A) Intercalary meristem B) Lateral meristem C) Primary meristem D) Secondary meristem E) Stele meristem
19)
Parenchyma, collenchyma, and sclerenchyma are all types of cells derived from A) procambium tissue. B) ground tissue. C) vascular cambium tissue. D) cork cambium tissue. E) epidermal tissue.
20)
Which leaf tissue would be most directly affected by a nonfunctional rubisco enzyme? A) Cuticle B) Mesophyll tissue C) Upper epidermis D) Lower epidermis E) Stoma
21)
A stem that cannot increase in diameter is lacking what type of tissue? A) Xylem B) Ground meristem C) Procambium D) Protoderm E) Vascular cambium
22)
The slender stalk that connects the flattened leaf blade to the stem in most eudicots is the
Version 1
6
A) meristem. B) petiole. C) stele. D) receptacle. E) vein.
23) You are given a sample of plant tissue and asked to identify the type of tissue and its location in the plant. Using your microscope, you notice two distinct layers of cells. One layer contains cells that are tightly compacted together while the other layer has loosely arranged cells. The cells in both layers contain many chloroplasts. Based on this information, you determine the tissue is A) mesophyll tissuefrom a leaf. B) ground tissue from a stem. C) vascular tissue from a leaf. D) mesophyll tissue from a stem. E) vascular tissue from a root.
24) You are asked to determine if a tissue sample taken from a plant stem comes from a monocot or eudicot. How will you accomplish this? A) Look at the ground tissue to see if it is arranged in concentric rings or in parallel bundles. B) Examine the vascular cambium and determine if it is divided into segments or is continuous throughout the stem. C) Determine if the xylem and phloem are located in separate vascular bundles or are bundled together. D) Examine the epidermis and determine the depth of the cuticle layer. E) Locate the vascular bundles and analyze their pattern.
25)
Xylem tissue may contain each of these cell types, except which?
Version 1
7
A) Vessel members B) Tracheids C) Sieve cells D) Fibers E) Rays
26)
Which of the following cell types does not move materials through the body of the plant? A) Sclerenchyma B) Xylem vessel members C) Sieve tube members D) Tracheids E) Sieve cells
27) by
A plant that has the mutant form of the WEREWOLF gene ( wer) would be identifiable
A) the excessive number of vascular bundles in stem. B) the lack of fibers in the outer portions of the stem. C) the abundance of microscopic hairs on the root. D) the extra thick layer of mesophyll cells in the leaf. E) the lack of microscopic hairs along the epidermis of the root.
28) Primary growth at the apical meristems can produce cells that differentiate into each of these cell types, except which? A) Leaves B) Ground tissue C) Procambium D) Cork cambium E) Epidermis
Version 1
8
29)
Primary growth in plants originates in A) apical meristems. B) lateral meristems. C) vascular cambium. D) cork cambium. E) tracheids.
30)
Which of the following cell types conducts water most rapidly through a plant? A) Parenchyma cells B) Sclerenchyma cells C) Sieve tubes D) Vessel members E) Phloem
31) You have been given a plant sample and asked to identify a specific tissue. Upon investigation, you find the cells are all long in length and some of them have small pores. They all contain fluid. You test the fluid using various biochemical tests and discover the fluid contains an abundance of disaccharides. What type of tissue are you looking at? A) Xylem B) Phloem C) Root hair cells D) Sclerenchyma E) Meristematic
32)
In plants with only primary growth, the epidermis is not
Version 1
9
A) one cell thick. B) the outer protective coating of the plant. C) produced by the protoderm. D) covered by a waxy layer that constitutes the cuticle. E) covered with bark.
33)
Root hairs grow actively in which area of the developing roots? A) Root cap B) Zone of maturation C) Zone of elongation D) Zone of cell division E) Endodermis
34)
Which of the following statements correctly describes the vascular cambium? A) It develops between the primary xylem and the primary phloem in dicots. B) It occurs only in monocot stems. C) It is important in the elongation of roots. D) It is formed by the ground meristem. E) It is responsible for all cell types that result from primary growth.
35)
Annual rings in a tree trunk result from A) deposition of colored materials in the oldest cells. B) the variation in cell size due to different growth conditions throughout a year. C) the alternation of xylem and phloem produced in one year. D) the kinds of cells, parenchyma or sclerenchyma, produced by the cambium. E) the proportion of wood cells and cork cells in the xylem.
36)
Compound leaves
Version 1
10
A) have two or more petioles per blade. B) have a blade divided into leaflets. C) are toothed at the edges. D) are alternately arranged. E) have palmate venation.
37)
Axillary buds A) add length to a plant. B) develop into roots when water is scarce. C) form flowers or branches. D) increase the diameter of a stem. E) form lateral meristems.
38)
The most distinctive characteristic of leaf mesophyll cells is that they are filled with A) centralvacuoles. B) many chloroplasts. C) oxygen bubbles. D) veins. E) stomata.
39) A birdhouse is nailed into a tree 6 feet up from the ground. If the tree grows about 2 feet taller each year, where will the birdhouse be 25 years later? A) 50 feet B) 56 feet C) 26 feet D) 6 feet E) 4 feet
Version 1
11
40) If you examined a cross section of a woody stem under the microscope and located the vascular cambium, the tissues on the inside of the vascular cambium ring (toward the center of the stem) would be A) xylem (primary and secondary). B) xylem (only primary). C) phloem (primary and secondary). D) phloem (only secondary). E) phelloderm.
41) A friend who is not a biologist tells you that she grows irises from iris roots. You explain to her that the "root" she is planting is not a root, but instead is called a rhizome. "Why?" she asked. You explain: A) "A root grows vertically, not horizontally." B) "A rhizome has nodes and internodes and is really a modified stem that can exist underground." C) "A root stores nutrients, but rhizomes are underground stems that do not store nutrients." D) "A rhizome, although a modified stem, acts as a root does. In other words a rhizome is a stem-root combination." E) "A rhizome is a modified root that is able to grow leaves."
42) the
The name of the meristem that ultimately gives rise to secondary xylem and phloem is
A) vascular cambium. B) cork cambium. C) intercalary meristems. D) ground meristem. E) primary meristem.
Version 1
12
43) The waterproof cuticle covering the epidermis of land plants helps prevent dehydration, much like the skin of some land animals. As a consequence of having a cuticle, what other evolutionary adaptation was important for most land plants? A) The endodermis in the root B) Root hairs on the root epidermis C) Collenchyma fibers just beneath the surface of the epidermis D) Stomata in the leaves E) Trichomes on leaf surfaces
44) In an experiment, some herbaceous, nonwoody plants were exposed to frequent windy conditions or mechanical shaking for several hours per day. The flexibility of these plants was due to the __________ cells. A) parenchyma B) aerenchyma C) periderm D) collenchyma E) sclerenchyma
45) Linen is woven from strands of sclerenchyma __________ that occur in the phloem of flax ( Linum spp.). A) tracheid B) sieve C) sclerid D) collenchyma E) fiber
46) the
A major distinguishing feature between monocot and eudicot stems is the organization of
Version 1
13
A) epidermis. B) conducting system. C) vascular tissue. D) secondary tissues. E) ground tissues.
47)
How could you distinguish between a root hair versus a very small root? A) Root hairs are white, while roots are brown. B) Root hairs are extensions of single cells, while roots are multicellular. C) Root hairs only absorb water, not nutrients like roots. D) Root hairs do not have a cuticle. E) The epidermis of a root hair is thinner than the epidermis of a root.
48) What type of cells are joined end-to-end, conduct water, and are connected to each other with strips of wall material? A) Sieve cells B) Tracheids C) Sieve tube members D) Vessel members E) Companion cells
49) In the vascular bundles of most eudicot stems, primary phloem differentiates toward the __________ of the stem, while primary xylem differentiates toward the __________ of the stem. A) middle; outside B) cork cambium; vascular cambium C) outside; middle D) shoot tip; roots E) middle; middle
Version 1
14
50)
Damage to the ground meristem of a plant would result in A) the inability to move water but not sugar through the plant. B) decreased sugar production. C) an increase of gas exchange at the surface of the leaf. D) dehydration of the plant. E) inability to move sugar but not water through the plant.
51)
Secondary phloem A) includes live companion cells and dead sieve cells. B) encompasses more stem volume than secondary xylem. C) primarily stores carbohydrates. D) is part of the inner bark. E) is produced by the cork cambium.
52) In the annual rings of woody trees, how does the cell size within the xylem of the spring and summer wood compare to the fall and winter wood? A) It is more dense. B) It has smaller vessels. C) The cell walls are thicker. D) It is younger. E) The cells are larger.
53)
Which plant cells are the most common and the least specialized? A) Epidermis B) Collenchyma C) Sclerenchyma D) Parenchyma E) Meristem
Version 1
15
54) Your lab instructor hands you a root slide and asks you to show her a cell with condensed chromosomes arranged in a straight line. In which section of the root would you begin your search? A) It doesn't matter; all zones have the same level of mitotic activity. B) Zone of elongation C) Zone of maturation D) Zone of cell division E) Root cap
55) Which of the following would not be useful in distinguishing between dermal and ground tissue cells? A) Presence of cellulose in the cell wall B) Thickness of the primary cell wall C) Presence of a secondary cell wall D) Whether the cell is living or dead at maturity E) Size of the cell's vacuole
SECTION BREAK. Answer all the part questions. 56) Choose the letter of the best match from the following: A. protoderm B. vascular cambium C. apical meristem D. procambium E. ground meristem
56.1)
Ultimate source of all cells that contribute to growth in length of stem or root.
56.2)
Lateral meristem that contributes to the increase in thickness of a stem or root.
Version 1
16
56.3) The three primary meristems are the __________, which forms the epidermis; the procambium, which produces primary vascular tissues (primary xylem and primary phloem); and the ground meristem, which differentiates further into ground tissue.
56.4)
Produces parenchyma, collenchyma, and sclerenchyma cells.
56.5)
Source of cells that differentiate into primary vascular tissues.
Version 1
17
Answer Key Test name: Chap 29_3e 1) FALSE Mesophyll tissue contains the photosytnthetic tissue of a leaf and is therefore part of the shoot system. 2) [A, B, C, E] Meristems are areas of a plant that produce growth. Apical meristems add length and produce the primary plant tissues while lateral meristems add girth and produce the secondary plant tissues. 3) [B, C, D, E] Unlike other cells derived from ground tissue, sclerenchyma die at maturity. Their secondary cell wall is lignified that makes the cell very rigid and this adds strength to the plant tissue. 4) [A, B, C, D] Evolution has resultedin many types of modified roots. Please read Section 29.3 for a description of these modifications 5) [A, B] Since trichomes are an external feature of plants found on stems, leaves, and the reproductive organs, they are not involved with structural support. Please read Section 29.2 for more detailed information on trichomes. 6) [A, B, D, E] Spines form from modified leaves not stems. For a detailed description of the types of modified stems please read Section 29.4. 7) [A, C, D, E] Please read Section 29.5 for more detailed information on the modifications of leaves. 8) [A, B, C, E] Version 1
18
Secondary growth, which increases the diameter of a plant, occurs in some eudicots. This increase in diameter, or girth, is due to the presence of lateral meristems. There are two types of lateral meristems, the cork cambium and the vascular cambium. The cork cambium contributes to the outer bark while the vascular cambium produces both secondary xylem and secondary phloem. Please read Section 29.1 for more information on lateral meristems. 9) [A, C, D, E] Sclereids are part of the ground tissues of plants. Ground tissue cells function in storage, photosynthesis, and secretion. Sclereids are a type of sclerenchyma cell that may have an irregular shape. Their function is to strengthen the tissues where they are found. Please read Section 29.2 for more information on sclereids. 10) [A, C, E] A root hair is an extension of an epidermal cell and is not separated from the cell by a cross-wall. Root hairs do increase the absorptive capacity of a root. Please read Section 29.2 for a more detailed description of root hairs. 11) [A, C, D, E] The area of the root responsible for rapid cell division is the Zone of cell division. Please see Section 29.3 for more information on the four regions of a typical root. 12) [B, D, F] 13) E Section 29.1 discusses apical and lateral meristems. 14) A Section 29.1 discusses primary and secondary growth of a plant. 15) B
Version 1
19
Clarify Question • What is the key concept addressed by the question? o The role of the Golgi bodies to a growing root in a plant. • What type of thinking is required? o You must apply what you know about the function of the Golgi bodies to deduce the effects of their inhibition in a plant. • What key words does the question contain and what do they mean? o Golgi bodies, which is an organelle that processes macromolecules. Gather Content • What do you already know about the function of golgi bodies in plant root cells? How does it relate to the question? o Golgi bodies produce mucigel. o Mucigel moves through the plant cell walls. o The passage of roots through soil is eased by mucigel. Consider Possibilities • What other information is related to the question? Which information is most useful? o Mucigel does not directly affect the cell division rate or growth direction of the root. Choose Answer • Given what you now know, what information and/or problemsolving approach is most likely to produce the correct answer? o Recall that Golgi bodies in general are in involved in macromolecule processing in cells. o In root cells, the Golgi bodies produce mucigel, which is a slimy Version 1
20
substance that moves through the cell wall to the outside of the cell. o Mucigel allows the root tip to slide through the soil as it grows. o If the Golgi bodies are inhibited, mucigel production will be inhibited. o If mucigel is not produced, the root tip will not move through the soil smoothly as it grows. Reflect on Process • Did your problem-solving process lead you to the correct answer? If not, where did the process break down or lead you astray? How can you revise your approach to produce a more desirable result? o This question asked you to utilize the importance of the Golgi bodies. Answering this question correctly depended on your ability to use the function of the Golgi bodies in a new situation. If you got the correct answer, great job! If you got an incorrect answer, where did the process break down? Did you remember that Golgi produce mucigel, or that mucigel allows root tips to move through the soil more readily? Did you have trouble extending the function of the Golgi bodies to determine the correct answer? 16) B
Version 1
21
Clarify Question • What is the key concept addressed by the question? o The question addresses the effects of the removal of symbiotic bacteria from a plant. • What type of thinking is required? o Apply, because you must take what you know about the effects of antibiotics and apply that to symbiotic relationships that occur between plants and microorganisms. What key words does the question contain and what do they mean? o Root hairs, which is an extension from a root that absorbs minerals and water. o Antibiotic, which is a chemical that kills bacteria. o Nutrient, which is an element of food that helps a living thing to survive. Gather Content • What do you already know about the funtion of symbiotic bacteria in plants? How does it relate to the question? o Certain antibiotics kill certain bacteria. o Plants have symbiotic bacteria that fix nitrogen from the atmosphere. o Root hairs provide a pathway for symbiotic bacteria into the plant. Consider Possibilities • What other information is related to the question? Which information is most useful? o Bacteria aid in the absorption and storage of nitrogen, particularly for some groups of plants (the legume family Fabaceae), acting as symbionts that form nodules on the roots. o Other nutrients do not require bacterial intervention. Version 1
22
Choose Answer • Given what you now know, what information and/or problemsolving approach is most likely to produce the correct answer? o The indicator that the question was focused on nitrogen is the use of the antibiotic. o Since antibiotics kill certain bacteria, the use of the antibiotic would remove the nitrogen-fixing symbionts from the plant. o Without the nitrogen-fixing bacteria, the plant would be deficient in nitrogen. Reflect on Process • Did your problem-solving process lead you to the correct answer? If not, where did the process break down or lead you astray? How can you revise your approach to produce a more desirable result? o This question asked you to understand the relationship between bacteria and plants. Answering this question correctly depended on your ability to use the role of bacteria in a new situation. If you got the correct answer, great job! If you got an incorrect answer, where did the process break down? Did you remember that bacteria fix nitrogen into a usable form for some plants,or that antibiotics kill bacteria? Did you have trouble extending the roleof bacteria to determine the correct answer? 17) B When the apical meristem is damaged, axillary buds that generally develop into branchesare activated. Please refer to Section 29.1 for more information on axillary buds. 18) B Section 29.1 discusses the roles of apical and lateral meristems in plant growth. Version 1
23
19) B Section 29.2 discusses the three types of cells that are derived from ground tissue. 20) B Section 29.5 discusses leaf structure. Rubisco is the enzyme that fixes carbon dioxide during the Calvin cycle of photosynthesis. Photosynthesis primarily occurs is the mesophyll tissue. 21) E Section 29.4 differentiates between monocot and eudicot stems. Monocot stems lack a vascular cambium (and cork cambium) and therefore do not increase in girth (diameter). 22) B Figure 29.21 shows a leaf attached to a branch by its petiole. 23) A
Version 1
24
Clarify Question • What is the key concept addressed by the question? o Plant tissue can be specifically identified by using a microscope and examining the shape, characteristics, and placement of cells. • What type of thinking is required? Apply, because you must remember what you know about plant tissues, and then apply it to an unknown sample • What key words does the question contain and what do they mean? o Chloroplast, which is the organelle in which photosynthesis occurs. o Loosely arranged cells, which can have a spongy appearance andin plants can refer to a specific mesophyll. o Compacted cells, which can refer to a specific type of mesophyll. Gather Content • What do you already know about the shape, characteristics, and arrangement of plant cells? How does it relate to the question? o Mesophyll tissue has spongy and a compact (palisade) layers in eudicots. o Chloroplasts are found throughout the mesophyll. Consider Possibilities • What other information is related to the question? Which information is most useful? o Vascular tissue would have bundles of xylem and phloem, and would not have chloroplasts. o Ground tissue would have parenchyma, sclerenchyma, and collenchyma Choose Answer Version 1
25
• Given what you now know, what information and/or problemsolving approach is most likely to produce the correct answer? o To answer this question, you need to recall the cellular characteristics of each of the major tissue types of a plant. o Focusing on the presence of chloroplasts, as well as the spongy and compacted cell layers, leads to only one conclusion. o The presence of the spongy and compact layers with chloroplasts is sufficient to determine this is mesophyll. Reflect on Process • Did your problem-solving process lead you to the correct answer? If not, where did the process break down or lead you astray? How can you revise your approach to produce a more desirable result? o This question asked you to identify cells in plant tissue. Answering this question correctly depended on your ability to use your knowledge of cells and tissues in a new situation. If you got the correct answer, great job! If you got an incorrect answer, where did the process break down? Did you remember that mesophyll has spongy and compact layers, or thatchloroplasts are plentiful in mesophyll? Did you have trouble extending your knowledge of plant tissues to determine the correct answer? 24) E Both monocots and eudicots have vascular bundles that contain both xylem and phloem. Monocots have these vascular bundles arranged randomly throughout the stem while eudicots have these bundles located in a circular pattern toward the outside of the stem. See Section 29.4 for more detailed information. 25) C Sieve cells are characteristic of phloem. 26) A Version 1
26
Sclerenchyma cells function in adding strength to plant tissues. Please read Section 29.2 for infomation on plant cells and their respective functions. 27) C The WEREWOLF gene (WER) inhibits root hair development in root epidermal cells. In contrast, the mutant form of the gene (wer) facilitates the development of root hairs. See Section 29.3 for a more detailed explanation of the WEREWOLF gene. 28) D Cork cambium as well as vascular cambium are tissues generated from a lateral meristem and are therefore referred to as secondary plant growth. Please see Section 29.1 for more detailed information. 29) A Apical meristems produce the primary plant body while the lateral meristems produce the secondary plant body. Please read Section 29.1 for more detailed information about apical and lateral meristems. 30) D Xylem, which transports water through the body of a plant, is composed of tracheids and vessel members. Both of these cell types are dead at maturity. Water moves between tracheids, which sit next to one another, through pits. The vessel member cells are joined end to end with the space between cells almost completely open or punctuated with strips of cell wall material. This allows water to travel faster through the vessel members. 31) B
Version 1
27
Clarify Question • What is the key concept addressed by the question? o Plant tissues can be identified by looking at their cellular composition. • What type of thinking is required? o You must apply what you know about plant tissues to identify an unknown sample. • What key words does the question contain and what do they mean? o Biochemical tests, which include tests that can be used to detect macromolecules such as carbohydrates, proteins, lipids, etc. o Monosaccharides, which are carbohydrates that are composed of one sugar building block. Gather Content • What do you already know about the cellular compostition of different types of plant cells? How does it relate to the question? o Glucose, a product of photosynthesis, is a monosaccharide. o Phloem transports nutrients, including monosaccharides. o Phloem cells are long and have small pores. Consider Possibilities • What other information is related to the question? Which information is most useful? o Xylem transports water. o Root hair cells are tubular and occur just behind the roots. o Sclerenchyma cells have tough, thick walls. Choose Answer • Given what you now know, what information and/or problemVersion 1
28
solving approach is most likely to produce the correct answer? o The abundance of monosaccharides is an important clue to the identification of this phloem tissue. Also, the physical characteristics noted in the question are only found in phloem. Reflect on Process • Did your problem-solving process lead you to the correct answer? If not, where o This question asked you to deduce an unknown tissue type. Answering this question correctly depended on your ability to use known characteristics of tissue in a new situation. If you got the correct answer, great job! If you got an incorrect answer, where did the process break down? Did you remember that phloem carries monosaccharides or that phloem cells tend to be long with small pores? Did you have trouble extending the characteristics of the tissues to determine the correct answer? 32) E Only plants with lateral meristems exhibit secondary growth. Bark results from secondary growth. Please read Section 29.1 for more information on primary and secondary growth. 33) B Root hairs are common only in the zone of maturation. Please read Section 29.3 for more information on root development. 34) A
Version 1
29
Some dicots have both primary growth (initiated by apical meristems) and secondary growth (initiated by lateral meristems). There are two types of lateral meristems, cork cambium and vascular cambium. The vascular cambium initially develops between the primary phloem and primary xylem. As it produces new vascular tissue, it becomes surrounded by secondary phloem and secondary xylem. Please read Section 29.1 for more information on lateral meristems. 35) B Tree rings result from variations in environmental conditions throughout the year. Please read Section 29.4 for more information about tree rings. 36) B Unlike simple leaves, compound leaves are divided into smaller sections. Please read Section 29.5 for more detailed information on leaves. 37) C Apical buds which form at the tips of stems add length to the plant. Axillary buds which form primary plant tissues are located in the space between a leaf and a stem and form either branches or flowers. Please read Sections 29.1 for more information about axillary buds. 38) B Since leaves are the main photosynthetic organ of a plant, the mesophyll cells within a leaf are filled with chloroplasts. The chloroplasts function to capture sunlight energy and convert it into chemical energy. Please read Section 29.5 for more information about leaves and mesophyll cells. 39) D
Version 1
30
Clarify Question • What is the key concept addressed by the question? o The mechanism of growth in a plant. • What type of thinking is required? You must apply what you know about plant growth to a new situation. • What key words does the question contain and what do they mean? o Tree, which is a perennial plant with an elongated stem and branches. o Twenty-five years later, which means the tree will have changed in height. o Six feet up from the ground, which is the original location on the stem/trunk for the birdhouse. Gather Content • What do you already know about a tree grows? How does it relate to the question? o Apical meristems cause an increase in plant height. o Apical meristems occur at the top of a plant. Consider Possibilities • What other information is related to the question? Which information is most useful? o Trees to not increase in height from the bottom up; rather they grow at their upper margins. Choose Answer • Given what you now know, what information and/or problemsolving approach is most likely to produce the correct answer? o To answer this question, you need to understand that increases in the Version 1
31
height of plant occur from the top. Any structures lower than the apical meristem do not change in elevation, even as the tree grows. As long as the bird house is not placed at the top of a tree, it will not change in elevation, even as the tree grows. Reflect on Process • Did your problem-solving process lead you to the correct answer? If not, where did the process break down or lead you astray? How can you revise your approach to produce a more desirable result? o This question asked you to extrapolate the position of a birdhouse in a growing tree. Answering this question correctly depended on your ability to use the mechanism of growth in a new situation. If you got the correct answer, great job! If you got an incorrect answer, where did the process break down? Did you remember that the apical meristem is where increases in height occur or that the apical meristem occurs at the top of a tree? Did you have trouble extending the mechanism of plant growth to determine the correct answer? 40) A
Version 1
32
Clarify Question • What is the key concept addressed by the question? o A cross section of a woody stem is made up of different tissues. • What type of thinking is required? o You must apply what you know about individual tissues of the woody to identify those in the central region of the stem. • What key words does the question contain and what do they mean? o Cross section, which is a section of tissue that is cut perpendicular to the long axis of the organism. o Woody stem, which is the trunk of the tree. o Vascular cambium, which is the tissue between the xylem and the phloem. Gather Content • What do you already know about the different tissues in a woody stem? How does it relate to the question? o The vascular cambium lies just inside the bark of a woody stem. o Xylem is added to the inside of the vascular cambium. Consider Possibilities • What other information is related to the question? Which information is most useful? o Phloem is added to the outside of the vascular cambium. Choose Answer • Given what you now know, what information and/or problemsolving approach is most likely to produce the correct answer? o To answer this question, you need to focus on the reference to the Version 1
33
tissue inside the vascular cambium ring. If you connect that to the location of xylem on the inside of that ring, you have the correct answer. Reflect on Process • Did your problem-solving process lead you to the correct answer? If not, where did the process break down or lead you astray? How can you revise your approach to produce a more desirable result? o This question asked you to asses a section of tissue. Answering this question correctly depended on your ability to use the characteristics of the tissue in a new situation. If you got the correct answer, great job! If you got an incorrect answer, where did the process break down? Did you remember that xylem is on the inside of the vascular cambium or that phloem is on the outside of the vascular cambium? Did you have trouble extending the characteristics of the plant tissues to determine the correct answer? 41) B A rhizome is a type of modified stem that functions in plant propagation. These tend to run horizontally underground. Because a rhizome is a stem, it contains nodes and internodes. The nodes contain leaves and axillary buds. Roots do not have nodes/internodes/leaves or axillary buds. Please read Section 29.4 for more information on rhizomes. 42) A Vascular cambium is one of two types of lateral meristems in woody tissues. The cork cambium contributes to the outer bark while the vascular cambium, as its name suggests, generates vascular tissues. Please read Section 29.4 for more information on meristems. 43) D
Version 1
34
The cuticle is a waxy coating of a leaf. Although the cuticle is an effective way to reduce water loss, it can interfere with the exchange of gases (carbon dioxide into the leaf, oxygen out of the leaf). So stomata, small pores in the leaf epidermis were another necessary evolutionary adaptation for land plants. Please read Section 29.2 for more information. 44) D
Version 1
35
Clarify Question • What is the key concept addressed by the question? o Different types of plant cell types have different functions within a plant. • What type of thinking is required? o You must apply what you know about the functions of individual cell types to recognize an unknown cell type in a sample. • What key words does the question contain and what do they mean? o Nonwoody plant, which refers to a plant that is herbaceous, with a green rather than a wood-based stem. o Flexibility, which means there is a great range of movement. Gather Content • What do you already know about the composition and function of various plant cells types? How does it relate to the question? o Collenchyma cells provide flexibility to a plant. o Collenchyma cells often form strands. Consider Possibilities • What other information is related to the question? Which information is most useful? o Parenchyma may have thin cell walls. o Sclerenchyma cells are typically dead at maturity and are not flexible. Choose Answer • Given what you now know, what information and/or problemsolving approach is most likely to produce the correct answer? o If you connect the ideas that the unknown cells are from the stem of Version 1
36
the plant and that they are flexible and strong, you can only conclude this is collenchyma. Collenchyma is known for its flexible nature. Reflect on Process • Did your problem-solving process lead you to the correct answer? If not, where did the process break down or lead you astray? How can you revise your approach to produce a more desirable result? o This question asked you to identify a plant tissue. Answering this question correctly depended on your ability to use the known characteristics of plant tissues in a new situation. If you got the correct answer, great job! If you got an incorrect answer, where did the process break down? Did you remember that collenchyma is known for its flexibility or that other tissues are not nearly as flexible? Did you have trouble the known characteristics of plant tissues to determine the correct answer? 45) E Please read Section 29.2 for more information on sclerenchyma cells. 46) C Monocots and eudicots vary in the arrangement of the vascular tissues within the stem. In monocots, the vascular bundles are scattered through the stem. In eudicots, the vascular bundles are arranged in a ring around the periphery of the stem. Please refer to Section 29.4 for more information on stems. 47) B
Version 1
37
Root hairs are simply extensions of individual root epidermal cells. So root hairs are much smaller than roots. Since root hairs are small and can access tiny areas within the soil, root hairs function to keep the plant in intimate contact with the soil. The majority of water and nutrient absorption takes place via the root hairs. Root hairs are confined to the zone of maturation on a root. Please read both Sections 29.2 and 29.3 for more information on root hairs. 48) D Sieve cells, companion cells, and sieve tube members are all part of the phloem, tissue that conducts food (sugar) materials throughout the plant. Tracheids and vessel members conduct water. Vessel members are cells that are joined end to end. Where two cells meet, they are connected by strips of cell wall material (called perforation plates). The gaps between these strips allows water to travel very efficiently through the plant. Please refer to Section 29.2 for more information on the cell types found in phloem and xylem. 49) C In eudicots, the primary vascular tissues are arranged in a ring toward the periphery of the stem. The xylem develops on the inside of this ring while the phloem develops toward the outside of the ring. Please refer to Section 29.4 for more information on vascular tissues in a eudicot stem. 50) B
Version 1
38
Clarify Question • What is the key concept addressed by the question? o The ground meristem plays an important role in the life of a plant. • What type of thinking is required? o You must apply what you know about the functions of the ground meristem to identify the effects of damage to that tissue. • What key words does the question contain and what do they mean? o Ground meristem, which has functions that include giving rise to photosynthetic tissues. Gather Content • What do you already know about the function of ground meristem in plants? How does it relate to the question? o Ground meristem functions in photosynthesis, storage, secretion, support, and protection. o Photosynthesis produces sugar. Consider Possibilities • What other information is related to the question? Which information is most useful? o Ground meristem is not involved in the transport of water and nutrients. o Ground meristem is not involved in gas exchange. Choose Answer • Given what you now know, what information and/or problemsolving approach is most likely to produce the correct answer? o You need to connect the functions of ground meristem to this example Version 1
39
by imagining what would happen to the plant if the ground meristem is compromised. Since the ground meristem gives rise to the photosynthetic tissues, if the ground meristem is compromised, there would be fewer photosynthetic tissues. With lower rates of photosynthesis, there would be less sugar produced. Reflect on Process • Did your problem-solving process lead you to the correct answer? If not, where did the process break down or lead you astray? How can you revise your approach to produce a more desirable result? o This question asked you to extrapolate the effects of tissue damage. Answering this question correctly depended on your ability to use the function of specific tissues in a new situation. If you got the correct answer, great job! If you got an incorrect answer, where did the process break down? Did you remember that ground meristem gives rise to tissues that photosynthesize? Did you have trouble extending the function of specific tissues to determine the correct answer? 51) D Woody plants have a vascular cambium that develops just beneath the bark. The vascular cambium produces secondary vascular tissues. Secondary xylem is produced toward the inside of the tree while secondary phloem is produced toward the outside of the tree and is part of the inner bark. Please refer to Section 29.1 for more information on lateral meristems. 52) E
Version 1
40
Annual growth rings reveal patterns of vascular cambium growth. Vascular cambium is the lateral meristem present in woody eudicots that produces secondary vascular tissues (secondary phloem and secondary xylem). When growth conditions are favorable, the cells produced by the vascular cambium are larger. Please see Section 29.4 for more information on annual growth rings. 53) D Ground tissue contains three primary cell types. Collenchyma cells remain alive for many years, have cell walls of various thicknesses, and provide flexibility and strength for the plant. Sclerenchyma cells are dead at maturity and have thick lignified secondary cell walls that function to strengthen plant tissues. Parenchyma cells are the least specialized, alive at maturity and function in storage, photosynthesis, and secretion. Please read Section 29.2 for more information on ground tissue. 54) D
Version 1
41
Clarify Question • What is the key concept addressed by the question? o The cellular activities of the root zones. • What type of thinking is required? o You must apply what you know about the nature of a root to identify the location of the process at hand. • What key words does the question contain and what do they mean? o Root tip, which is an area of active growth and cell division. Gather Content • What do you already know about the appearance of root cells in the different zones? How does it relate to the question? o The zone of cell division in a plant root is a site of active mitosis. o Students often use slides from this zone to examine all of the mitotic phases. o Chromosomes are visible using a light microscope during mitosis. Consider Possibilities • What other information is related to the question? Which information is most useful? o Zone of maturation contains cells that differentiate. o Zone of elongation contains cells that elongate via the merging of small vacuoles. Choose Answer • Given what you now know, what information and/or problemsolving approach is most likely to produce the correct answer? o You need to connect the zone of cell division to the highest amount of Version 1
42
mitosis. Abundant mitosis would provide the best opportunity to view condensed chromosomes that are in a straight line, as they would be in mitotic metaphase. You can rule out options that include functions not covered by the zone of cell division. Reflect on Process • Did your problem-solving process lead you to the correct answer? If not, where did the process break down or lead you astray? How can you revise your approach to produce a more desirable result? o This question asked you to differentiate the roles of the zones in a root tip. Answering this question correctly depended on your ability to use your knowledge of each zone in a new situation. If you got the correct answer, great job! If you got an incorrect answer, where did the process break down? Did you remember that the zone of cell division has active mitosis or that chromosomes condense and are visible during mitosis? Did you have trouble extending the functions of each zone to determine the correct answer? 55) A 56) Section Break 56.1) C 56.2) B 56.3) A 56.4) E 56.5) D
Version 1
43
CHAPTER 30 CHECK ALL THE APPLY. Choose all options that best completes the statement or answers the question. 1) Pea plants are self-pollinators. What can you further deduce about peas? (Check all that apply.) A) Peas will be moreresistant to garden pests than other plants. B) Peas are lesslikely to invade highly variable natural environments than other plants. C) A gardenerplanning to collect heirloom seeds can plant different varieties of peas neareach other. D) Peas will be moreresistant to new plant diseases. E) To perform his pea cross-pollinationexperiments, Gregor Mendel must have first removed the stamens fromflowersto prevent self-pollination.
2) If development of the shoot apical meristem failed in a bean embryo, which tissues would still be able to form? (Check all that apply.) A) Cotyledons B) Leaves C) Root D) Stem E) Suspensor
3)
Which "vegetables" are technically fruits? (Check all that apply.) A) Lettuce B) Green onion C) Tomato D) Spinach E) Peapod F) Carrot
Version 1
1
MULTIPLE CHOICE - Choose the one alternative that best completes the statement or answers the question. 4) Angiosperms use which temporary reproductive structures that are not present in any other group of plants? A) Cones B) Carpels C) Receptacles D) Flowers E) Seeds
5)
What is the process that eventually leads to the death of a plant? A) Outcrossing B) Dichogamy C) Abscission D) Perennial E) Senescence
6) The developmental transition during which plants become able to respond to external and/or internal signals that promote flowering is known as A) cloning. B) initiation. C) abscission. D) perennial development. E) phase change.
7) Ovules develop within the ovary, which is located in the swollen lower portion of which floral organ?
Version 1
2
A) Anther B) Stamen C) Filament D) Stigma E) Pistil
8)
Which of the following is not a floral structure? A) Endosperm B) Calyx C) Corolla D) Stigma E) Style
9) Your botany instructor likes to tell nerdy jokes, like this:"I wanted to buy a pink Toyota Corolla, but the dealer only had a green Calyx available." Impress your instructor with a clever response to this joke. A) "Oh, I wouldhave chosen the blue Receptacle." B) "Those aregreat cars, but the pinecone tires don't roll very well." C) "Me too! But Icould only afford the yellow Androecium." D) "The factorymust have run out of class A genes."
10) A researcher is investigating the morphology of complete flowers. Which of the following organs must his flowers contain? A) Calyx, corolla, receptacle, and carpels B) Calyx, corolla, carpels, and ovules C) Calyx, corolla, stamens, and anthers D) Calyx, corolla, carpels, and pistils E) Calyx, corolla, androecium, and gynoecium
Version 1
3
11) A genus of primarily wind-pollinated plants includes one unusual species that has evolved to be pollinated by insects. Several morphological changes have accompanied this adaptation. The ______________ are more colorful, there is a nectary at the base of the ______________, the ________________ are shorter and produce sticky pollen, and the __________________ are stickier for enhanced pollen capture. A) petals; stamens; sepals; carpels B) petals; sepals; stamens; carpels C) stamens; carpels; petals; sepals D) petals; sepals; carpels; stamens
12)
Microspore mother cells produce microspores. What do microspores then develop into? A) Egg cells B) Pollen grains C) Synergids D) Endosperm E) Megaspores
13)
Which of the following statements about the angiosperm embryo sac is false? A) It contains eight nuclei. B) It is the gametophyte generation of the plant. C) The nuclei within the embryo sac are diploid. D) The synergids flank the egg cell. E) The embryo sacnuclei are the product ofmitotic divisionof a megaspore.
14)
Which structure contains the angiosperm egg?
Version 1
4
A) Style B) Endosperm C) Anther D) Embryo sac E) Stigma
15) You are a landscape architect working on the grounds of a large estate. After you plant a dozen trees, the owner says, "There's been a mistake. The flowers on some of these willow trees look totally different! I specifically asked for Salix alba. You must have planted two different species." You refer to your tree guide, and read: "Salix alba - species of willow native to Europe and Asia, noted for the white undersides of leaves. Insect pollinated, dioecious. Grows 10-30m tall." How do you respond to the client? A) "Sorry aboutthat sir, I will dig up the wrong trees right away." B) "Don't worry,the trees with the smaller flowers just haven't undergone phase changeyet." C) "Don't worry,when the shorter trees reach full size, they will grow the other kind offlower." D) "Don't worry, Salix alba is dioecious, meaning each tree has either male or femaleflowers. That is why they look different."
16) Thanks to a novel recessive mutation, a certain plant species has begun to invade colder Arctic regions outside of its normal range. However, this cold-tolerant mutation has not yet become fixed in the population. A second mutation arises that promotes self-pollination. Do you predict that the self-pollination allele will be selected for in the Arctic regions? Why or why not? A) Yes. Theself-pollinated plant is more likely to maintain the cold-tolerant mutation,and plants in the Arctic are unlikely to be visited by pollinators. B) No. Outcrossing isalways beneficial, since it enhances genetic diversity. C) No. Plants thatproduce flowers must be cross-pollinated for proper seed development. D) Cannot make a prediction from the information given. It depends on whether the plant is normally insect pollinated or wind pollinated.
Version 1
5
17) Which of these characteristics would most likely discourage self-pollination in a flower with both stamens and pistils? A) Many more stamens than pistils B) Long pistils and short stamens C) Long stamens and short pistils D) Wind pollination E) Determinant plants
18) Oaks, birches and corn produce both staminate and pistillate flowers on the same plant. What term describes this type of plant? A) Monoecious B) Dioecious C) Hermaphroditic D) Complete E) Determinant
19) A gourmet ice cream company started a charitable campaign to save an insect species. Which species are they most likely to care about, and why? A) Cockroaches. Inthe 1950s, the company nearly eradicated a certain species of roach, butsubsequently learned they are valuable pollinators. B) Cochineal insects.They are the only natural source of red food dye. C) The honeybee. They explain that half of their ingredients, and a significant portion of our food supply in general, rely on bee pollination, and bee populations have been suffering from the poorly understood colony collapse disorder. D) Butterflies. Theyexplain that half of their ingredients, and a significant portion of ourgeneral food supply, rely on butterfly pollination, and butterfly populationshave suffered from the spread of "bug zappers" in the suburbs.
20)
Flowering plants use animals or wind to transfer ________ between flowers.
Version 1
6
A) microspores B) megaspores C) ovules D) seeds E) pollen
21) In a wind-pollinated species, which of these floral parts is likely to be inconspicuous or absent? A) Receptacle B) Stamens C) Pistils D) Petals E) Stigmas
22)
Early seed plants were pollinated by _______, as are present-day conifers. A) beetles B) bees C) trilobites D) wind E) water
23)
Which of these characteristics does not promote outcrossing in flowering plants? A) Self-incompatibility B) Dioecism C) Monoecism D) Physical separation of flower parts E) Apomixis
Version 1
7
24)
Which of the following factors is least likely to attract pollinators to a flower? A) the amount ofnectar B) odor C) color of theflower in visible light D) color of theflower in UV light E) taste of theovules
25)
Which of the following statements about plant reproduction is incorrect? A) Monoecious plants can have male and female flowers on the same plant. B) Dioecious plants have male and female flowers on different plants. C) Self-pollinatingangiosperms are usually adapted to stable habitats. D) Outcrossing promotes genetic diversity. E) Self-pollinatedplants must be visited by pollinators for fertilization to occur.
26) Some flowers have distinctive patterns that only reflect ultraviolet light and therefore are invisible to humans. Which statement about such markings is not correct? A) The ultravioletpatterns are highly visible and attractive to bees. B) Carotenoidpigments are responsible for the ultraviolet patterns. C) The ultravioletpatterns often form a conspicuous central bull's eye or identify the bestlanding sites. D) The ultravioletpatterns repel birds and bats from the flowers. E) Flowers withultraviolet patterns usually appear yellow or orange to humans.
27)
Which of these is not an example of coevolution between flowers and pollinating bees?
Version 1
8
A) Bees have tongues to lap up nectar. B) Some bees are hairy to collect and carry pollen. C) Flowers have ultraviolet markings visible to bees but invisible to most other insects. D) Some flowers are bright red with long tubes to hold nectar. E) Bees have strong preferences for particular types of flowers and remember where the plants are located.
28) Many plants devote significant resources to growing colorful, fragrant flowers filled with nectar. What does this illustrate the importance of? A) Nutrient storage in seeds B) Long-term nutrient storage in perennial plants C) Outcrossing D) Chemical signaling among plants
29) You and your neighbor each have a holly bush in your yards. You carefully water and care for your bush, but it fails to produce berries. Your neighbor, on the other hand, neglects his holly, yet it produces gorgeous red berries each year. Finally, in frustration, you replace your holly with a rose bush. The next year, your neighbor’s holly bush has no berries at all. What happened? A) Roses compete forthe same insect pollinators as holly bushes. B) Holly plants aredioecious. Yours was a male that was needed to pollinate the neighbor’sfemale. C) Roses secrete achemical that inhibits the reproduction of neighboring plants. D) The neighbor’sbush became infected with the same disease that yours had.
30)
What process results in the formation of the endosperm and the embryo in angiosperms?
Version 1
9
A) Alternation of generations B) Hydrolysis C) Determinate growth D) Double fertilization E) Secondary growth
31) Pollination begins when pollen is transferred from the ________ to the ________ of a flower. A) anther, ovary B) anther,receptacle C) anther,stigma D) stigma, embryosac E) synergid, egg
32)
Which step is not involved in the fertilization of angiosperm flowers?
A) Pollen istransported to a stigma of a flower. B) When pollen reachesthe stigma of a flower, it begins to grow a pollen tube that pierces thestyle. C) The style of thepistil contracts to bring the sperm closer to the embryo sac. D) The pollen tubegrows until it reaches the ovule in the ovary. E) Doublefertilization produces an embryo and the endosperm.
33)
What is the best description of double fertilization in angiosperms?
Version 1
10
A) One sperm fertilizes the egg, which develops into the embryo; the other sperm fertilizes the ovary, which becomes the fruit. B) One sperm fertilizes the egg, which develops into the embryo; the other sperm fertilizes the polar nuclei, which forms the endosperm. C) One sperm fertilizes the egg, which develops into the embryo; the other sperm fertilizes the polar nuclei, which forms the seed coat. D) One sperm fertilizes the egg, which develops into the embryo; the other sperm produces the pollen tube. E) One pollen grain fertilizes the embryo sac; the other sperm fertilizes the pollen grain.
34) You go to the movies with a friend, and he refuses to share your popcorn. He says that he heard that movie theatre popcorn is triploid. What is the most likely explanation for this? A) Most corn isgenetically modified by large agricultural companies. B) The butterytopping on movie theatre popcorn causes chromosomal abnormalities. C) Popcorn is madefrom a different species of maize than regular corn. D) All popcorn istriploid, because the starchy part of the kernel is triploid endospermtissue.
35) If a pollen grain fails to generate two sperm cells, what is the most likely result of pollination? A) Half the seedswould develop fine, but the other half would die. B) The resultingprogeny would be haploid. C) The seeds wouldlack either embryos or endosperm, and so would be nonviable. D) The endospermwould be smaller.
36)
What are the "seed leaves" that grow from bulges on the embryo called?
Version 1
11
A) Radicles B) Embryo sacs C) Petioles D) Cotyledons E) Ovules
37) What do the two daughter cells generated by the first asymmetric division of the plant zygote become? A) The suspensor and the embryo B) The epicotyl and the hypocotyl C) The root and the shoot D) The egg and pollen E) The xylem and the phloem
38)
What does the globular stage of plant embryo development give rise to? A) A heart-shaped embryo B) The suspensor of the germinating seed C) A double embryo D) The monocot scutellum
39)
What is the name of the meristematic tissue that gives rise to future vascular tissue? A) Procambium B) Cork cambium C) Ground meristem D) Dermal tissue E) Protoderm
Version 1
12
40) What is the name of theelongated structure that links the embryo to the nutrient tissue of the seed? A) Suspensor B) Cotyledon C) Epicotyl D) Style E) Ovule
41)
Which of these structures does not begin development during embryogenesis? A) Apical meristem B) Vascular tissue C) Floral primordia D) Leaf primordial E) Ground tissue
42)
Which of these is not part of a seed? A) Endosperm B) Embryo C) Diploid cells D) Seed coat E) Fruit
43)
Endosperm tissue of angiosperms has what ploidy level? A) n B) 2 n C) 3 n D) 4 n E) 8 n
Version 1
13
44) Germination requires the use of metabolic reserves in the seed. Which of these compartments contains starch used to fuel early seedling growth? A) Amyloplasts B) Protoplasts C) Chloroplasts D) Chromoplasts E) Mitochondria
45)
Which of the following is not a feature of a seed? A) It contains a protective coat formed from the outer layers of ovule cells. B) The embryo is surrounded by nutritive tissue or has cotyledons that contain nutritive
tissue. C) It contains a large percentage of water to support the embryo. D) It is a complex structure that allows the embryo to be dispersed to distant sites. E) Environmental signals such as light or temperature can trigger germination.
46)
Why do oils form a significant part of the nutritional reserves in many seeds? A) Oils help keep theembryo hydrated, allowing the seed to survive in dry habitats. B) The oils reducethe density of the seed and allow for water dispersal. C) Oils contain more mineral nutrients than carbohydrates. D) Oils store twiceas much energy as an equivalent mass of starch. E) Hormones diffusemore quickly through oils.
47)
What is the term for the single cotyledon within the kernels of cereal grains?
Version 1
14
A) Aleurone B) Hypocotyl C) Dicot D) Scutellum E) Amyloplast
48)
What is the portion of a dicot seedling below the cotyledons called? A) Internode B) Epicotyl C) Hypocotyl D) Plumule E) Subcotyl
49) Stratification refers to the extended exposure to ________________ that is required for some seeds to germinate. A) long wavelengths of light B) lowtemperatures C) high relative humidity D) short daylengths E) clay soil particles
50)
Which structure does not represent a mode of vegetative reproduction? A) runners B) rhizoids C) adventitiousplantlets D) suckers E) rhizomes F) stolons
Version 1
15
51) What are plants that flower only once before they die, normally after two seasons of growth, called? A) Annuals B) Biennials C) Perennials D) Monocots E) Dicots
52) Which part of the seed produces hydrolytic enzymes that break down the starchy endosperm during seed germination? A) Scutellum B) Amyloplasts C) Aleurone layer D) Plumule E) Coleorhiza
53) When a seed germinates, the embryo emerges from its dormancy and begins which phase of the plant life cycle? A) Sporophyte generation B) Gametophyte generation C) Dicot generation D) Monocot generation E) Alternate generation
54)
Which of the following statements about the epicotyl is true?
Version 1
16
A) It remains in the ground after a seed has germinated. B) It is the portion of the stem axis that is directly below the cotyledons. C) The shoot apical meristem of a seedling is located just above it. D) It takes up most of the volume within a monocot or dicot seed. E) It forms immediately above the secondary roots of a seedling.
55) Despite its name, Kentucky bluegrass is a widespread turfgrass used for lawns throughout the United States. In some regions, strains have developed desirable disease-resistant properties. However, Kentucky bluegrass typically develops by apomixis. Plant breeders say this creates both a challenge and an opportunity for engineering novel strains. Why? A) In apomixis, newplants develop from underground horizontal stems. This allows for fastpropagation of new strains, but all progeny are identical to theparent. B) Plants thatdevelop by apomixis are naturally tolerant of herbicides. As a result,herbicide tolerance genes cannot be used as selectable markers for a transgene,but herbicides can be used to keep the resulting culture pure. C) Since the progenyof apomixis are triploid, hybrids have an imbalance of parental genes. However,this may be desirable if a higher proportion of one allele is beneficial. D) If a seed developsby apomixis, it will be identical to its parent. Hybridization with anotherstrain requires special techniques. However, once a hybrid strain is developed,apomixis ensures its reliable propagation.
56) Which tissue cannot divide to give rise to identical progeny in a plant that reproduces asexually? A) Root B) Stem C) Leaf D) Pollen grain E) Ovule
57)
Which of the following statements about plant tissue culture is false?
Version 1
17
A) Virus-free cropspecies can be propagated by culturing meristems. B) Individual cells can give rise to whole plants through tissue cultures. C) Synthetic hormonescan be used to selectively promote root or shoot formation. D) Callus tissue canonly form roots, notshoots. E) A callus is a massof undifferentiated cells.
58) Your rich uncle, an eccentric botanist, has died. The will states: "Just like my favorite plants, I keep my wealth buried underground." Where do you dig for his treasure? A) In the peanutfield. Legumes have root nodules which house symbiotic ants that harvest honey,a nutritional "treasure". B) In the herbaceousperennial bed. Perennial stems die each fall, but underground nutrient storesprovide energy for regrowth each spring. C) In the annualplant bed. Annuals store nutrients in extensive underground root systems thatallow them to regrow each year. D) Dig up the rootsof the mistletoe plants, which store large quantities of water.
59) Two users of an internet gardening site are arguing. Sarah, an organic gardener from Maine, refers to her snapdragon plants as "annuals." Marco, a farmer from Italy, scoffs and declares them "perennials." Who is right? A) Sarah is right. A snapdragon plant only returns for three to four years, and thus is not a true perennial. B) Marco is right.With enough inorganic fertilizer, the snapdragon can regrow. All"annuals" are just undernourished perennials. C) They are victimsof bad translation software. The plant Marco refers to reseeds itself yearafter year and thus is a perennial. D) They are both right. In a Mediterranean climate, the snapdragon reemerges from its rootstock each spring. In colder climes, it must be replanted each year. A plant that is perennial in its native habitat may be an annual in other climates.
Version 1
18
60) Browsing through a seed catalog, you are tempted to order several beautiful floral varieties but worry that you will spend too much on your garden every year. Which plants should you buy? A) Herbaceous perennials, which will come back year after year. B) Woody perennials, which require little space and few resources. C) Annuals, which will come back year after year. D) Biennials, which will last twenty years before they die.
61) Heirloom plants are treasured for their flavorful produce and true-breeding progeny that allow gardeners to avoid repeatedly purchasing seed from agribusiness conglomerates. What special effort is required to save seed from heirloom biennials such as carrots? A) Since carrotsproduce large roots, they do not devote many resources to seed development. Theroots must be trimmed before collecting seed. B) Since carrotsbuild up so much beta-carotene, they do not devote many resources to seeddevelopment. A yellow carrot variety will produce better seeds than a darkerone. C) Since carrots arerelated to wild weeds, they are often cross-pollinated. The green top must bebagged to ensure self-pollination. D) Biennials do notproduce seed until the second year of growth. Although carrot roots arenormally harvested after one season, they must be allowed to grow another yearfor seedcollection.
62)
Which of the following events is not a characteristic of successful seed germination? A) The radicle breaks through the seed coat. B) The shoot becomes photosynthetic. C) The embryo becomes desiccated. D) New growth occurs at the meristems. E) The seedling orients itself relative to gravity.
63) Your friend has never gardened before. When none of the seeds he planted germinate, he asks you to suggest why. Which is the best explanation of what occurred?
Version 1
19
A) The monocot seeds he used were only able to produce a single cotyledon each. B) The seeds were planted too deeply and used up their reserves before reaching the surface and sunlight. C) The suspensor failed to develop into an embryo within each seed. D) All seeds require fire in order to germinate. E) Seeds take more than one season to germinate.
64) Why can scratching or chipping the seed coat speed up the germination of hard-coated seeds like alfalfa? A) The mechanicalstress activates hydrolytic enzymes. B) Water and oxygen can enter the seed to reach the embryo more quickly. C) Carbon dioxide canenter the seed and stimulate photosynthesis. D) Bacteria or fungican enter the seed and release growth factors. E) Damage to the seedcoat triggers the aleurone layer to release amylase.
65) Scientists have confirmed by carbon dating that seeds dormant for up to 2000 years can successfully germinate. Based on the requirements for germination, what research expedition should you join in order to retrieve some ancient but viable seeds? A) A trip toIndonesia, where the civet cat eats and expels coffee beans, producing agourmet coffee known as Kopi Luwak. B) A trip to theIrish marsh, where wet peat creates anaerobic conditions, and your colleagueshope to unearth a bog mummy. C) A trip to theBurgess Shale in Canada, where your paleontologist colleagues have foundfossils from the Cambrian era. D) A trip to the aridDead Sea region, where your historian colleagues have found a storeroom ofwax-sealed jars.
66) The gene wooden leg is necessary for development of phloem. If you stained a latestage embryo with an anti-Wooden Leg antibody, in what tissue would you expect to see staining?
Version 1
20
A) Endosperm B) Ground meristem C) Procambium D) Protoderm
67) A forest ranger surveys the burned landscape of his park following a destructive wildfire. “Well, there’s one silver lining,” he says, “We ought to see some unusual plants sprouting up in the next few years.” Why? A) Damage to predatorpopulations will allow vulnerable plants to survive the seedling stage. B) The growth of plants is normally limited by carbon availability in thesoil. C) Somespecies'pollen is only carried and activated by smoke. D) Some seeds areadapted to germinate only when heated by a fire. E) In many plants, the seedling’s suspensor can more easily push into ashesthan into overgrown soil.
68) A number of plants in a wilderness area carry a mutation that makes their soft, colorful fruits bitter and unpalatable. Would you expect this mutation to increase or decrease in frequency in the plant’s population? A) The mutation will decrease in frequency. B) The mutation will increase in frequency. C) It depends whether they are pollinated by the wind, birds, or insects. D) It depends whether the plants use sexual or asexual reproduction.
69) A seed contains an embryo mutant for gibberellic acid (GA) receptors. What phenotype would you expect to result in the seedling?
Version 1
21
A) No mutantphenotype until the plant is older. B) Poor growth due tounusable nutrient reserves. C) Malformedcotyledons due to defects in development. D) Yellow instead ofgreen due to chloroplast failure.
SECTION BREAK. Answer all the part questions. 70) Choose the letter of the best match from the following: A. hypocotyl B. endosperm C. scutellum D. radicle E. epicotyl
70.1)
The stem axis of the embryo below the cotyledons in some seedlings.
70.2)
A single cotyledon found in the kernels of cereal grains.
70.3)
Nutritive tissue within angiosperm seeds.
70.4)
The part of the stem axis that extends above the cotyledons in some seedlings.
Version 1
22
Answer Key Test name: Chap 30_3e 1) [B, C, E]
Version 1
23
Clarify Question • What is the key concept addressed by the question? The question is asking about the consequences of self-pollination. • What type of thinking is required? You are being asked to take what you already know and apply it to the effects of self-pollination to a plant and the plants in the surrounding area. Gather Content • What do you already know about pollination? What other information is related to the question? o Both pistils and stamens are found in the same flower on a selfpollinated plant. o Self-pollinated plants have lower genetic variability than crosspollinated plants. o Self-pollination is favored in environments that have greater stability. Choose Answer • Given what you now know, what information is most likely to produce the correct answer? o Self-pollinated plants are not necessarily more resistant to garden pests or diseases. o If self-pollinating plants are grown next to each other, they maintain their own identities and traits, so different varieties could be easily differentiated. o To turn a self-pollinator into a cross-pollinator, the stamens must be removed. Reflect on Process • Did your problem-solving process lead you to the correct answer? If Version 1
24
not, where did the process break down or lead you astray? How can you revise your approach to produce a more desirable result? o Answering this question correctly depended on your ability to use the characteristics of self-pollinating plants in a new situation. If you got an incorrect answer, did you remember that self-pollination removes the necessity for dispersion or that self-pollinated plants maintain their own unique identities? Did you have trouble extending the characteristics of self-pollinated plants to determine the correct answer? 2) [A, C, E] 3) [C, E] Fruits are formed by maturation of the flower ovary. Because they are formed from ovaries, peapods and tomatos are technically fruits. 4) D 5) E 6) E 7) E 8) A 9) C
Version 1
25
Clarify Question • What is the key concept addressed by the question? This question is asking you to connect the similarities that are present in the parts of a flower. • What type of thinking is required? • Apply level: o You are being asked to take what you already know and apply it to the use of the terms that describe flower anatomy. Gather Content • What do you already know about flower anatomy? What other information is related to the question? o The corolla is the entire whorl of petals in a flower. o A calyx is the term for the entire whorl of sepals. o An androecium is the entire whorl of stamens. Choose Answer • Given what you now know, what information is most likely to produce the correct answer? o The androecium is the most similar structure to the corolla and calyx, since all three involve smaller structures that are arranged in a whorl. The receptacle is the stem from which the flower parts arise, so that is morphologically very different. Class A genes are only partially responsible for petal production, and since they are genetic rather than morphological, they are the least related item. Reflect on Process • Did your problem-solving process lead you to the correct answer? If not, where did the process break down or lead you astray? How can you revise your approach to produce a more desirable result? Version 1
26
o Answering this question correctly depended on your ability to use your knowledge of flower structure in a new situation. If you got an incorrect answer, did you remember that sepals, petals, and stamen are arranged in whorls or that a receptacle is found at the base of a flower? Did you have trouble extending the similarities in the structures of a flower to determine the correct answer? 10) E 11) B 12) B 13) C 14) D 15) D
Version 1
27
Clarify Question • What is the key concept addressed by the question? This question is asking about the cause of different appearances of individuals within a species. • What type of thinking is required? • Apply level: o You are being asked to take what you already know and apply it to an explanation of the different morphologies of two plants of the same species. Gather Content • What do you already know about dioecious species? What other information is related to the question? o Dioecious means male and females are found as separate individuals. o Dioecious species may be sexually dimorphic, meaning that males and females have different appearances. Choose Answer • Given what you now know, what information is most likely to produce the correct answer? o Phase change occurs before flower formation, so that does not account for the different flowers. o Height is not a factor that influences the appearance of flowers. o Sexual dimorphism, or the different appearances of male and female plants of the same species, explains the differences in the plants in this example. Reflect on Process • Did your problem-solving process lead you to the correct answer? If not, where did the process break down or lead you astray? How can Version 1
28
you revise your approach to produce a more desirable result? o Answering this question correctly depended on your ability to use the concept of a dioecious species in a new situation. If you got an incorrect answer, did you remember that dioecious species have males and females as separate individuals or that males and females may look different from one another? Did you have trouble extending the term dioecious to determine the correct answer? 16) A
Version 1
29
Clarify Question What is the key concept addressed by the question? • This question is asking about the adaptiveness of self-pollination in an Artic climate. What type of thinking is required? • Evaluate level: o This question is asking you to evaluate the consequences of a mutation that results in the self-pollination of a plant in the Artic. Gather Content What do you already know about pollination? What other information is related to the question? • Self-pollination is favored in stable, unchanging conditions. • Artic regions would have a minimum of insect pollinators. Choose Answer Given what you now know, what information is most likely to produce the correct answer? • Cross-pollination is not required for flower production, so that answer can be excluded. • Self-pollinated plants would be likely to survive and produce offspring, so the self-pollination mutation would be maintained over time. Reflect on Process Did your problem-solving process lead you to the correct answer? If not, where did the process break down or lead you astray? How can you revise your approach to produce a more desirable result? Version 1
30
Answering this question correctly depended on your ability to evaluate the adaptiveness of a mutation that results in self-pollination. If you got an incorrect answer, did you remember that self-pollination differs from cross-pollination by the removal of the need for a pollinator or the criteria to compare self and cross-pollination includes the availability of potential pollinators? Did you have trouble weighing the merits of selfpollination and cross-pollination to determine the correct answer? 17) B 18) A 19) C 20) E 21) D 22) D 23) E 24) E 25) E 26) D 27) D 28) C 29) B
Version 1
31
Clarify Question • What is the key concept addressed by the question? This question is asking you to identify why a holly bush is no longer producing fruit. • What type of thinking is required? This is an analyze question because you have to break the removal of a holly bush and the following lack of fruit production in a neighboring plant into their component pieces to understand how they function. Gather Content • What do you already know about dioecious plants? What other information is related to the question? o Dioecious means male and females are found as separate individuals. o Seeds are produced when male and female gametes are present. o If a plant species has separate male and female individuals, fruit is only produced by female plants. o Fruit is produced to encase seeds and to encourage their dispersal. Choose Answer • Given what you now know, what information is most likely to produce the correct answer? o Roses do not compete with holly bushes for pollinators. o Roses do not produce inhibitors. o The neighboring holly plant must be female, as it was able to produce fruit. The holly plant that was removed must have been male, as it never produced fruit. Additionally, when the male plant was removed, the female plant stopped producing fruit. Reflect on Process • Did your problem-solving process lead you to the correct answer? If not, where did the process break down or lead you astray? How can Version 1
32
you revise your approach to produce a more desirable result? o Answering this question correctly depended not only on distinguishing between monoecious and dioecious plants, but on your ability to analyze what is necessary for fruit production. If you got an incorrect answer, did you remember that components of dioecious plants include separate male and female individuals or that if one gender is absent, fruit is not produced by dioecious plants? Did you have trouble breaking down dioecious plants and fruit production to determine the correct answer? 30) D 31) C 32) C 33) B 34) D
Version 1
33
Clarify Question • What is the key concept addressed by the question? This question is asking you to explain the triploid nature of popcorn. • What type of thinking is required? This question is asking you to identify key factors that qualify something as triploid. Gather Content • What do you already know about seed structure? What other information is related to the question? o Triploid cells have three sets of chromosomes. o Popcorn endosperm is triploid, and forms from the fusion of one sperm cell and two polar nuclei in the embryo sac of a plant seed. o The endosperm is the food supply of an embryo. Choose Answer • Given what you now know, what information is most likely to produce the correct answer? o Popcorn is naturally triploid; it is not modified by agricultural companies or by the butter that is put on the popcorn. Other species of corn also have endosperm that is triploid, so that answer can be excluded. In fact corn typically has starchy, triploid endosperm components that serve as a food source for the embryo. Reflect on Process • Did your problem-solving process lead you to the correct answer? If not, where did the process break down or lead you astray? How can you revise your approach to produce a more desirable result? o Answering this question correctly depended not only on distinguishing between the causes of triploidy, but on your ability to Version 1
34
analyze, the reasons that popcorn is triploid. If you got an incorrect answer, did you remember that causes of triploidy include the mechanism of endosperm formation or that corn in general has triploid endosperm? Did you have trouble breaking down triploidy in corn to determine the correct answer? 35) C
Version 1
35
Clarify Question • What is the key concept addressed by the question? This question is asking about the effect of missing sperm in a pollen grain. • What type of thinking is required? This is an analyze question because you have to break the function of sperm in a pollen grain and their role in the development of a plant embryo into their component pieces to understand how they function. Gather Content • What do you already know about plant gametes and fertilization? What other information is related to the question? o Two sperm cells are normally carried in a pollen grain. o One of the two sperm cells fuses with two polar nuclei in the embryo sac of a plant seed to form the endosperm. o The endosperm is the food supply of an embryo. o The other sperm cell from the pollen grain fuses with the egg cell, which forms a zygote. Choose Answer • Given what you now know, what information is most likely to produce the correct answer? o The endosperm cannot form without a sperm cell from the pollen grain, and without the endosperm, the embryo would not have a food source. Additionally, the zygote will not form without a sperm cell from the pollen grain. If the pollen grain was missing one of the two necessary sperm cells, the plant embryo could not grow. Reflect on Process • Did your problem-solving process lead you to the correct answer? If Version 1
36
not, where did the process break down or lead you astray? How can you revise your approach to produce a more desirable result? o Answering this question correctly depended not only on distinguishing between the necessary number of sperm in a pollen grain and the role of those sperm in embryo development, but on your ability to break down, or analyze, the consequences of a lack of sufficient sperm cells in the pollen grain. If you got an incorrect answer, did you remember that the functions of the sperm in the pollen grain include fertilizing the egg as well as facilitating the formation of the endosperm as a food source for the embryo? Did you have trouble breaking down the role of sperm cells in embryo development to determine the correct answer? 36) D 37) A 38) A 39) A 40) A 41) C 42) E 43) C 44) A 45) C 46) D 47) D 48) C 49) B 50) B 51) B 52) C 53) A Version 1
37
54) C 55) D
Version 1
38
Clarify Question • What is the key concept addressed by the question? This question is asking you to explain why apomixis is both challenging and beneficial in the production of Kentucky Bluegrass.. • What type of thinking is required? This is an analyze question because you have to break the challenges and opportunities presented by apomixis into their component pieces to understand how they function. Gather Content • What do you already know about apomixis? What other information is related to the question? o Apomixis is the process by which an embryo is produced asexually via the cloning of diploid cells. o Embryos produced through apomixis are diploid and are identical to the parent cell. Choose Answer • Given what you now know, what information is most likely to produce the correct answer? o Apomixis is really the formation of an embryo through mitosis; the process itself forms seeds, not extensions of the existing parent. o The embryos that result from apomixis are diploid, not triploid, so that answer can be excluded. o Embryos formed through apomixis are not pesticide resistant unless the parent plant is pesticide resistant. o Apomixis involves the creation of a clone of a parent cell, so the offspring are identical to the parent. Since this is asexual reproduction, hybridizing two different strains would need special techniques, but would result in plants that could be propagated. Version 1
39
Reflect on Process • Did your problem-solving process lead you to the correct answer? If not, where did the process break down or lead you astray? How can you revise your approach to produce a more desirable result? o Answering this question correctly depended not only on distinguishing between the benefits and challenges of apomixis, but on your ability to analyze the components of apomixis. If you got an incorrect answer, did you remember that components of apomixis include the formation of a diploid embryo or that the embryo is a clone of the parent? Did you have trouble breaking down the process of apomixis to determine the correct answer? 56) D 57) D 58) B 59) D 60) A
Version 1
40
Clarify Question • What is the key concept addressed by the question? This question is asking you to identify the most cost-effective way to enhance a garden. • What type of thinking is required? o This question is asking you to evaluate the best way to add flowers to a garden while keeping cost to a minimum. Gather Content • What do you already know about annuals and perennials? What other information is related to the question? o Annuals produce showy flowers, but they die after one year. o Biennials do not flower during the first year of their life, and may flower only once in their lifetime. o Herbaceous perennials live for multiple years and can flower every year. o Woody perennials may take longer to mature, but can flower every year after maturation. Choose Answer • Given what you now know, what information is most likely to produce the correct answer? o Since annuals and biennials may flower only one time in their life, although they can be beautiful displays of flowering, they are not costeffective foe a long-term garden. Woody perennials can produce flowers also. However they can take years to mature; they take up more space; and they may require a greater initial investment of money. Herbaceous perennials are a cost-effective choice because they produce flowers the first year they are planted and continue to flower for multiple years. Reflect on Process Version 1
41
• Did your problem-solving process lead you to the correct answer? If not, where did the process break down or lead you astray? How can you revise your approach to produce a more desirable result? o Answering this question correctly depended on your evaluation annuals, perennials, and biennials. If you got an incorrect answer, did you remember that annuals and biennials differ from perennials because their lifecycles differ, or the criteria to compare annuals, biennials, and perennials includes the number of seasons that the plants can be expected to exhibit flowering? Did you have trouble weighing the merits of annuals, biennials, and perennials to determine the correct answer? 61) D
Version 1
42
Clarify Question • What is the key concept addressed by the question? This question is asking you to recognize the stages in the lifecycle of a biennial plant. • What type of thinking is required? This is an analyze question because you have to break biennials and seed production into their component pieces to understand how they function. Gather Content • What do you already know about biennials? What other information is related to the question? o Biennials do not flower in the first year of their life. o Flowers are necessary reproductive structures that give rise to seeds. o Biennial plants that are harvested as crops are typically harvested for their leaves and/or their roots, which grow during the first year. o If a biennial plant is harvested during its first year, it will not have produced flowers or seeds. Choose Answer • Given what you now know, what information is most likely to produce the correct answer? o Trimming roots only removes tissue and possible food storage sites. This would not induce flowering, as the plant must mature before flowering occurs. o Beta-carotene is not a factor in seed development. o The green top of a carrot is not a flowering structure, so it would not contain pollen. o Keeping in mind that carrots are biennials, they would not flower or produce seeds until the second year of their life. If the plants are not Version 1
43
removed after the first year and instead are allowed to grow, in the second year of life seeds could be harvested from the flowers that occur at that time. Reflect on Process • Did your problem-solving process lead you to the correct answer? If not, where did the process break down or lead you astray? How can you revise your approach to produce a more desirable result? o Answering this question correctly depended not only on distinguishing between growth and flowering, but on your ability to analyze the lifecycle of a biennial. If you got an incorrect answer, did you remember that components of the lifecycle of a biennial include a year of growth before maturation or that flowering and seed production does not occur until the second year? Did you have trouble breaking down the biennial life cycle to determine the correct answer? 62) C 63) B
Version 1
44
Clarify Question • What is the key concept addressed by the question? The question is asking you about the key factors in germination. • What type of thinking is required? You are being asked to take what you already know and use, or apply, it to the reason for lack of germination in a group of seeds. Gather Content • What do you already know about germination? What other information is related to the question? o Germination is the development of a plant embryo as it gains photosynthetic structures. o Plant embryos obtain nutrition from nutritional reserves in the endosperm before they are capable of photosynthesis. o The suspensor is a link between a plant embryo and the nutritional reserves of the seed. o To undergo photosynthesis, a seedling must produce photosynthetic structures, and it must emerge from the soil. Choose Answer • Given what you now know, what information is most likely to produce the correct answer? o Monocots do produce one cotyledon, but this is a genetic and structural difference, not a disadvantage. o Suspensors help to nourish embryos, but they do not develop into embryos. o Seeds can germinate within one season, although older seeds can germinate also. o Although some seeds require fire to germinate, this is an exception rather than the rule. Version 1
45
o A plant embryo can be sustained for a limited time by the nutrients stored in the seed, but it must quickly establish photosynthetic structures and be exposed to light for its life to continue. If a seed is planted too deeply, it may use up the nutrient reserves in the seed before it emerges from the ground. In that case, it would die from lack nutrition. Reflect on Process • Did your problem-solving process lead you to the correct answer? If not, where did the process break down or lead you astray? How can you revise your approach to produce a more desirable result? o Answering this question correctly depended on your ability to use the requirements for germination and growth in a new situation. If you got an incorrect answer, did you remember that germination is fueled by nutrient reserves in the seed? Did you recall that these reserves are used up quickly and that the plant must undergo photosynthesis after the reserves are gone? Did you have trouble extending the nature of germination to determine the correct answer? 64) B 65) D
Version 1
46
Clarify Question • What is the key concept addressed by the question? This question is asking about the conditions that are necessary for longterm seed dormancy. • What type of thinking is required? This question is asking you to evaluate the environment that would be most favorable seed dormancy. Gather Content • What do you already know about germination? What other information is related to the question? o To germinate, a seed must absorb water and oxygen. o Oxygen is used in the process of cellular respiration, by which the embryo could obtain energy from nutrients stored within the seed. Choose Answer • Given what you now know, what information is most likely to produce the correct answer? o Peat is anaerobic and acidic, which would destroy seeds. o The Burgess Shale is a fossilized mass that would not contain living things. o Wax-sealed jars would prevent mold formation and would be a favorable environment for the long-term storage of seeds. Reflect on Process •Did your problem-solving process lead you to the correct answer? If not, where did the process break down or lead you astray? How can you revise your approach to produce a more desirable result? o Answering this question correctly depended on your ability to evaluate the conditions necessary for seed dormancy and long-term Version 1
47
survival. If you got an incorrect answer, did you remember that water is one of the factors that causes seeds to germinate or the criteria to compare dormancy and long-term survival includes protection from environmental conditions? Did you have trouble weighing the merits of the necessary conditions for dormancy to determine the correct answer? 66) C 67) D 68) A Fruits serve to disperse seeds. If the fruits are unpalatable they will not be eaten and so the seeds will not be dispersed. 69) B 70) Section Break 70.1) A 70.2) C 70.3) B 70.4) E
Version 1
48
CHAPTER 31 TRUE/FALSE - Write 'T' if the statement is true and 'F' if the statement is false. 1) Closing the stomata for an extended period of time would lead to an increase in G3P in the mesophyll cells. ⊚ ⊚
true false
2) If the soil around a plant is treated with a fungicide, the plant may experience a decline in DNA synthesis. ⊚ ⊚
3)
true false
Cavitation would have no effect on translocation. ⊚ ⊚
true false
4) An application of too much fertilizer will negatively affect the movement of materials in the phloem. ⊚ ⊚
true false
5) A decrease in the availability of ATP in a root cell could decrease the water potential of the cell. ⊚ ⊚
true false
6) An increase in root pressure will result in the rapid release of abscisic acid (ABA) and the subsequent opening of the stomata. ⊚ ⊚
Version 1
true false
1
7)
If the proton pumps of guard cell were damaged, transpiration would decrease. ⊚ ⊚
true false
8) While researching the effects of light on chlorophyll synthesis in sorghum, you find that brief pulses of red light stimulates the synthesis of chlorophyll a and b. This would suggest that chlorophyll synthesis is directed by the P r form of phytochrome. ⊚ ⊚
9)
true false
The removal of the root cap eliminates the ability of a root to respond to gravity. ⊚ ⊚
true false
CHECK ALL THE APPLY. Choose all options that best completes the statement or answers the question. 10) Which of the following statements about plant macronutrients is/are true? Check all that apply. A) They approach or exceed 1% of a healthy plant's dry weight. B) They reduce the need for CO 2 during photosynthesis. C) A deficiency in any one can have severe effects on a plant's growth. D) The amount of a macronutrient needed by a plant can be assessed by growing plants in hydroponic solutions.
11) Red light and phytochrome are involved in which of the following plant growth responses? Check all that apply.
Version 1
2
A) Stem elongation B) Inhibition of seed germination C) Elongation of etiolated shoots D) Detection of crowding by neighboring plants
12) Which of the following describe a role that ethylene plays in plant physiology. Check all that apply. A) Retarding stem and root elongation B) Hastening the ripening of fruit C) Inhibition of leaf abscission D) Inhibition of lateral bud growth E) Accelerates the loss of damaged fruits
13)
Which of the following statements about abscisic acid are true? Check all that apply. A) It is produced chiefly in mature green leaves and in fruits. B) It stimulates the growth of dormant buds. C) It promotes leaf senescence. D) It plays an important role in controlling the opening and closing of stomata. E) It plays a role in seed dormancy.
MULTIPLE CHOICE - Choose the one alternative that best completes the statement or answers the question. 14) The process by which water is pulled up through the xylem columns of plants as it evaporates out of the leaves is called A) respiration. B) transpiration. C) osmosis. D) anhydration. E) stomatization.
Version 1
3
15) The most important ion in controlling the movement of water into and out of the guard cells is A) sodium. B) hydrogen. C) carbon. D) nitrogen. E) potassium.
16)
Water is able to cross plant plasma membranes by a combination of A) osmosis and aquaporins. B) evaporation and diffusion. C) root pressure and turgidity. D) diffusion and phloem. E) transpiration and Casparian strips.
17)
On a short-term basis, water loss in plants may be controlled by the A) bending of the petioles on the leaves. B) closing of the stele in the roots. C) closing of the stomata in the leaves to limit transpiration. D) opening of the stomata in the leaves to allow more carbon dioxide to enter. E) opening the Casparian strips in the roots.
18)
Some plants are able to endure frequent flooding events because they form A) more xylem. B) an extra layer of spongy mesophyll. C) adventitious roots that reach above the surface of the water. D) an extra porpous epidermal tissue.
Version 1
4
19) Even in the absence of transpiration, some water can move into the roots and partially up the xylem columns. This phenomenon is due to A) flooding. B) stomatal opening. C) root pressure. D) proton pumps. E) phloem translocation.
20)
Which of the following statements about transport through phloem is true? A) Transport only occurs from the roots to the shoot. B) It is aided by root pressure. C) The direction of flow can change at different times if the sources and sinks change. D) No energy is required. E) Mostly dissolved starch is transported.
21)
Turgor is most directly related to the plant cells' A) total water potential. B) pressure potential. C) solute potential. D) osmosis. E) gravity potential.
22)
The movement of oxygen through the aquatic plant may depend on
Version 1
5
A) osmotic absorption by the roots. B) aquaporins. C) negative pressures created by transpiration. D) differences in the water potential of different tissues. E) larger lenticels.
23)
Most of the water that evaporates from leaves passes out through the A) cuticle. B) ends of xylem vessels. C) epidermis. D) spaces between epidermal cells. E) stomata.
24)
Most of the water absorbed by the plant enters through the A) root apical meristem. B) root cap. C) root hairs. D) stomata. E) lenticels.
25) The plasma membranes of root hair cells contain a variety of protein transport channels through which specific ions are transported, even against large concentration gradients, by A) electron pumps. B) carbohydrate pumps. C) water pumps. D) hydrogen ion pumps. E) root pumps.
Version 1
6
26)
A plant will usually wilt when which component of water potential reaches 0 MPa? A) Solute potential B) Turgor pressure C) Total water potential D) Gravity potential
27)
Cavitation can occur when what happens? A) Root pressure pushes water up the xylem. B) Starch grains block the sieve tubes. C) Gas bubbles expand inside a tracheid or vessel member. D) Stomata get stuck closed. E) Clouds block sunlight shining on the leaves.
28) Which of the following actions occurs in many plants when CO high?
2 concentrations are
A) Guard cells lose turgor, and the stomata close. B) Water evaporates at a higher rate than usual. C) CAM photosynthesis fixes CO 2 at night. D) Oxygen is used by plants for photosynthesis. E) Calcium ions are transported faster through the phloem.
29) The Calvin cycle can only produce sugar if __________ can enter through the pores of the leaves. A) O 2 B) CO 2 C) water D) potassium E) N 2
Version 1
7
30)
Plants living in standing water often face which of the following problems? A) Increased CO 2 levels B) Chilling of the roots C) Oxygen deprivation D) Loss of stomata E) Harmful root pressure
31) A plant is exposed to a toxin that blocks ABA receptor sites in the plasma membrane. What is the likely consequence of this exposure? A) Oxygen deprivation will trigger the release of ethylene, which will in turn suppress root growth. B) Gibberellin production will increase, allowing the plant to access more nutrients in the soil. C) ABA will enter the cells by an alternative route and normal function will be restored. D) The plant will lose excess water through transpiration and suffer dehydration. E) The guard cells will become flaccid and gas exchange will cease.
32) Which of the following must occur in order to maintain a high pressure potential within guard cells? A) Potassium ions must be actively transported out. B) Energy must be constantly expended. C) Water must exit guard cells by osmosis. D) Stomata must take up more oxygen and less carbon dioxide. E) The rate of transpiration must increase.
33) The uneven distribution of an impermeable solute on either side of a membrane will result in
Version 1
8
A) an increase in oxygen deprivation. B) proton pumping. C) root pressure. D) osmosis. E) stomatal closing.
34) Most carbohydrates manufactured in leaves and other photosynthetic parts are transported through the phloem to the rest of the plant by the process of A) transpiration. B) translocation. C) osmosis. D) receptor-mediated transport. E) turgor pressure.
35) The process of moving water through a plant by transpiration works because water molecules stick to each other with A) turgor pressure. B) osmosis. C) ionic bonds. D) adhesion. E) hydrogen bonds.
36)
Which of the following equations represents the total water potential of a plant? A) s 2 + p 2 = w 2 B) ψ s = ψ p + ψ w C) ψ p = ψ w + ψ s D) ψ w = ψ p + ψ s E) ψ w = ψ s ¸ ψ p
Version 1
9
37)
Pure water without applied pressure has a water potential that is equal to A) infinity. B) 0.0 MPa. C) 1.6 MPa. D) –2.0 MPa. E) atmospheric pressure.
38) When water molecules "stick"to something that is not another water molecule, this is called A) adhesion. B) cohesion. C) root pressure. D) water pressure. E) pneumatophores.
39) Water may be lost in the form of liquid from the surface of leaves through a process known as A) guttation. B) cohesion. C) phloem loading. D) mesophyll adhesion. E) aeration.
40)
The pressure-flow hypothesis describes
Version 1
10
A) how hormones move through the phloem. B) how carbohydrates enter the sieve tubes. C) how carbohydrates in solution move through the phloem. D) how water and minerals move through the xylem. E) how carbohydrates and minerals move through the xylem.
41)
Most often, the largest gradient in water potential is between A) roots and soil water. B) root xylem and shoot xylem. C) xylem and phloem. D) leaf cells and the intracellular spaces inside the leaves. E) the relative humidity inside the leaf and the relative humidity outside the plant.
42)
Mycorrhizal fungi interact with plants at the A) stomata. B) seed coat. C) roots. D) edge of ponds. E) cuticle.
43) A large watermelon fruit is very heavy and contains nearly 90% water. Since the skin of a watermelon is thick and lacks stomata, transpiration does not "pull" water into the watermelon. So, how does all that water get into the fruit? A) Root pressure pushes water into the watermelon. B) Water enters by osmosis from the soil. C) Water is pumped in by active transport. D) Water is transported in the phloem along with the sugars while they are being translocated into the fruit. E) Watermelons, like other fruit, contain an abundance of xylem.
Version 1
11
44) Halophytes are plants that live in saline soils. The high osmotic potential of the salt solution in the soil creates a very negative water potential. What can halophytes do so that water will flow into the roots? A) They can close their stomata so that less water is lost through transpiration. B) They can open all their stomata so that transpiration "pulls" more water into the roots. C) They can increase the solute concentration in their roots creating a water potential that is more negative than the soil. D) They can pump ions out of the plant creating a water potential in the roots that is more positive than the soil. E) They can remove the mycorrhizal fungi from their roots decreasing the competition for water.
45) You a given a plant tissue in the lab and asked to determine if the tissue is a source or a sink. Microscopic analysis reveals an abundance of colorless parenchyma cells with plastids. Only one biochemical test reacts positively with your sample. In the presence of iodine, the plastids turn a dark purplish color. Based on this information, you determine your sample is A) a source because your sample is rich in carbohydrates. B) a source because your sample contains plastids. C) a sink because your sample is rich in carbohydrates. D) both a source and sink because it contains both plastids and carbohydrates.
46)
Which of the following statements about plants grown in hydroponic culture is true? A) They no longer need to transport water through their xylem. B) They cannot be used for commercial purposes. C) They rarely get enough micronutrients. D) They are unable to photosynthesize. E) They are suspended with their roots in a nutrient solution.
47)
A deficiency in a plant micronutrient
Version 1
12
A) is less serious than a deficiency in a macronutrient. B) makes that plant susceptible to herbivores. C) can have a severe effect on plant growth. D) results in no visible symptoms. E) has no impact on the agriculture of a region.
48) Inserting a cloned gene for a certain plasma membrane transporter into crop plants could lead to A) the ability of a plant to produce its own micronutrients. B) extra uptake of minerals that would fortify the crop for human consumption. C) the evolution of nodule-formation on the crop plant. D) elevated CO 2 levels in the atmosphere. E) plants taking up enough minerals that they no longer need to photosynthesize.
49) A deficiency of nitrogen would inhibit the production of which of the following molecules? A) Ubiquitin and phytochrome B) Starch C) Short chain fatty acids D) Glucose and fructose E) Long chain fatty acids
50) Although symbiotic relationships between nitrogen-fixing bacteria and plants are rare, many plants have symbiotic relationships with A) other plants. B) nematode worms in the soil. C) filamentous algae. D) mycorrhizal fungi in the soil. E) soil insects.
Version 1
13
51)
Mycorrhizae are especially effective in helping plant roots uptake A) nitrogen. B) phosphorus. C) potassium. D) iron. E) magnesium.
52)
What makes a mineral element essential for plant growth? A) It is part of typical chemical fertilizers. B) It can only be found in organic fertilizers. C) If it is missing, the plant cannot grow or reproduce normally. D) As its concentration increases, the plant grows more. E) It contributes to the solute potential of the plant.
53) Gibberellins affect many processes in plants. In seeds embryos, gibberellins are believed to signal hydrolysis of the stored food reserves through inducing the synthesis of the enzyme αamylase. Which food source are gibberellins targeting with the release of α-amylase? A) Omega-3 fatty acids B) Starch C) Glycogen D) Fructose E) Protein
54)
__________ is directional growth responses of plants to a unidirectional source of light.
Version 1
14
A) Gravitropism B) Thigmotropism C) Dormancy D) Phototropism E) Thermotropism
55)
Which cell structure is most likely responsible for sensing gravity? A) Central vacuole B) Golgi appartus C) Chloroplast D) Amyloplast E) Apical meristem
56) While touring a farm, the farmer tells you that she uses cytokinin to increase crop productivity. But, from biology class you know that cytokinin alone has no effect on parenchyma cells. What other substance is the farmer most likely using in conjunction with cytokinin? A) Auxin B) Abscisic acid C) Strigolactones D) Ethylene E) Suberin
57) While touring the ancient Egypt exhibit at the local history museum, you learn that ancient Egyptians would slash figs in order to stimulate ripening. Based on your knowledge of plant hormones you hypothesize that wounding the figs must release
Version 1
15
A) abscisic acid. B) gibberellin. C) indoleacetic acid. D) ethylene. E) cytokinin.
58) Abscisic acid (ABA) is synthesized in the roots in response to decreased soil water potential. ABA then translocates to the leaves, where it rapidly alters the osmotic potential of guard cells. What effect would this have on the stomata and the process of transpiration? A) The stomata would open thereby increasing the rate of transpiration. B) The stomata would open thereby decreasing the rate of transpiration. C) The stomata would close thereby decreasing the rate of transpiration. D) The stomata would close thereby increasing the rate of transpiration. E) The change in the stomata would have no effect on the rate of transpiration.
59)
The elongation of stems is stimulated by A) auxin only. B) gibberellin only. C) ethylene only. D) auxin and gibberellin. E) auxin and ethylene.
60) When Darwin covered the tip of a grass seedling with metal foil and then illuminated it from one side, what happened to the seedling? A) It bent away from the directional light. B) It did not respond to the directional light. C) It grew toward the light, as expected. D) It waved back and forth. E) It wilted and died.
Version 1
16
61) When seeds are removed from strawberries, fruit growth is stopped. This suggests the seeds contain which of the following hormones? A) Ethylene B) Auxin C) Strigolactones D) Oligosaccharides E) Gibberellins
62)
A compound similar to auxin, 2,4,5-trichlorophenoxyacetic acid (2,4,5-T), has been used A) as a fertilizer for corn and wheat. B) as a spray to promote flowering of pineapple. C) as an organic substitute for chlorine bleach. D) to detoxify the dangerous poison dioxin. E) an herbicide for killing woody seedlings and dicot weeds.
63) Coconut milk has been used to promote the differentiation of organs in masses of plant tissue growing in culture. Coconut milk is effective for this reason because it contains A) auxins. B) gibberellins. C) cytokinins. D) ethylene. E) abscisic acid.
64) Which naturally occurring plant hormone has a major role in the phototropic response of plants?
Version 1
17
A) Auxin B) Gibberellin C) Abscisic acid D) Ethylene E) 2, 4-D
65) What internal process helps plants to keep their movements and other responses synchronized with the day-night cycle? A) Purine monitoring B) Solar tracking C) A circadian clock D) Auxin cycling E) Gravitropism
66) Several scientists, including Charles and Francis Darwin, Frits Went, and others, demonstrated that auxins caused stems to bend toward light in a series of elaborate experiments. Went, was the researcher who finally determined that bending of the stem toward light was a result of which mechanism? A) Light caused a reduction in auxin concentration and a reduction in stem elongation on the lighted side of the plant stem, and thus a bending in the direction of the light. B) The transport of auxin to the shaded side of the plant stem stimulated stem elongation and a bending in the direction of the light. C) Auxin concentration was equal on both sides (lighted and shaded) of the stem; however, the apical meristem responded to the presence of light. D) A combination of positive phototropism and a negative gravitropism. E) The stems actually were responding to the agar blocks that had been used in the experiments.
67)
Directional, light-triggered plant development is known as
Version 1
18
A) thigmotropism. B) phototropism. C) thigmonasty. D) photomorphogenesis. E) acid-growth movement.
68) What happens when plants are illuminated with directional light from the side instead of from directly overhead? A) More auxin is produced. B) Less auxin is produced. C) Auxin is transported away from the lighted side. D) Auxin is transported toward the lighted side. E) Auxin receptors are activated.
69) Before ARF transcription factors can trigger auxin-induced gene expression, what must happen? A) Auxin binds to ubiquitin, a degradation tag. B) Auxin degrades Aux/IAA proteins, which repress ARF transcription factors. C) Auxin binds to TIR1, an auxin receptor. D) Auxin must bind to the ARF transcription factor. E) Ubiquitin must destroy TIR1 to release Aux/IAA proteins.
70) In some genetically engineered tomatoes, fruit ripening was delayed by crippling the gene that produces which hormone? A) Auxin B) Ethylene C) Abscisic acid D) Gibberellin E) Cytokinin
Version 1
19
71) When one plant is shaded by another, why is the ratio of far-red to red light increased compared to an unshaded plant? A) Phytochrome in the taller plant absorbs most of the red light. B) Auxin absorbs mostly far-red light. C) As light passes through the leaves of the taller plant, most of the red light is absorbed by chlorophyll. D) Far-red light mostly reflects off the surface of the soil while red light is mostly absorbed. E) Shade is enriched in far-red wavelengths of light.
72) In Arabidopsis plants, about 10% of its genome is devoted to processes tied to which molecule? A) Auxin B) Ubiquitin C) Rubisco D) Phytochrome E) Gibberellins
73) Abscisic acid synthesized in the root travels to different parts of the shoot where it regulates transpirational water loss. What is the most likely way that ABA travels from the root to the various locations within the shoot? A) Vessels and tracheids B) Sieve tubes C) Between cell walls D) From cell to cell through the plasmodesmata
74) Plants maintain higher levels of phytochrome at their growing tips where phytochrome plays important roles in growth responses to light. Phytochrome levels are also high in areas of the plant that are "greening." What does this suggest about the role of phytochrome in these "greening" zones? Version 1
20
A) Phytochrome is absorbing green wavelengths of light. B) In these areas, phytochrome is the first step in the electron transport chain of photosynthesis. C) Phytochrome is facilitating the transcription and translation of photosynthesis related proteins. D) Phytochrome is facilitating the migration of chlorophyll pigments from other areas of the plant.
75) Research has shown that cytokinin levels fluctuate seasonally in temperate plants. What season would you expect to see the lowest levels of cytokinins and why would this be adaptive? A) Spring, because lower levels of cytokinin allow auxin to promote rapid growth in stem height. This will allow the plant to compete effectively for sunlight. B) Summer, because lower levels of cytokinin would slow growth down and delay leaf aging. This will allow the plant to maximize the production of starch and increase it's sugar reserves. This will help ensure the plant's survival through autumn and winter. C) Autumn, because lower levels of cytokinin at this time of year will induce the production of ethylene. This will result in maturation of fruit and ensure effective seed dispersal before winter. D) Winter,because lower levels of cytokinin will place buds into a dormant state. This will protect the underlying tissues from freezing until spring.
76) One effect of auxin, which triggers the elongation of cells, is to affect cell walls by increasing their A) permeability. B) plasticity. C) stiffness. D) thickness. E) water solubility.
77)
Charles Darwin and his son experimented with which plant growth response?
Version 1
21
A) Gravitropism B) Circadian rhythms C) Phototropism D) Heat shock E) Thermotropism
78) __________ are chemical substances produced in small quantities in one part of an organism and then transported to another part of the organism, where they bring about physiological responses. A) Glycoproteins B) Photosynthates C) Hormones D) Heat shock proteins E) Pulvini
79) A plant physiologist treats a leaf with abscisic acid and observes that the stomata close. What happened inside the guard cells during the experiment? A) Abscisic acid influences the movement of sodium ions out of the guard cells. B) Abscisic acid influences the movement of potassium ions out of the guard cells. C) Abscisic acid binds carbon dioxide and causes carbonate crystals to block the stomatal opening. D) Abscisic acid binds with auxins and induces water molecules to exit the guard cells. E) Abscisic acid causes the guard cells to become more turgid.
Version 1
22
Answer Key Test name: Chap 31_3e 1) FALSE While the stomata are closed, gases cannot be exchanged. As a consequence, oxygen levels rise and carbon dioxide levels decrease within the plant. Without carbon dioxide, the Calvin cycle lacks the material to produce G3P, the precursor to glucose. 2) TRUE
Version 1
23
Clarify Question • What is the key concept addressed by the question? The key concept is the role of fungi in the life of plants. • What type of thinking is required? You are being asked to take what you already know and apply it to a new situation in which fungi are removed from their plant symbionts. • What key words does the question contain and what do they mean? o Fungicide, which is a chemical that kills fungi. o DNA, which is the genetic material of living things. o Synthesis, which means to produce. Gather Content • What do you already know about the role that fungi play in plant cell function? How does it relate to the question? o Mycorrhizzal fungi are associated with the roots of certain plants. o Myco refers to fungi, while rhizzal pertains to roots. o These fungi aid the absorption of water and minerals from the soil. o Phosphorus absorption from the soil is especially enhanced. o Phosphorus is an important building block of DNA. Consider Possibilities • What other information is related to the question? Which information is most useful? o Since DNA synthesis is dependent on available building blocks, mycorrhizal fungi would aid this process by making phosphorus available. Choose Answer • Given what you now know, what information and/or problemsolving approach is most likely to produce the correct answer? Version 1
24
o To answer this question, you must connect the concept of associating with mycorrhizal fungi, the minerals they make available, and the role those minerals play in DNA synthesis. Reflect on Process • Did your problem-solving process lead you to the correct answer? If not, where did the process break down or lead you astray? How can you revise your approach to produce a more desirable result? o This question asked you to identify the contributions of mycorrhizal fungi to plant biology. Answering this question correctly depended on your ability to use the idea that these fungi contribute minerals to plants in a new situation. If you got the correct answer, great job! If you got an incorrect answer, where did the process break down? Did you remember that these fungi make phosphorus available or that phosphorus is a building block of DNA? Did you have trouble extending this benefit of mycorrhizal assocation to determine the correct answer? 3) FALSE
Version 1
25
Clarify Question • What is the key concept addressed by the question? The concept in this question is cavitation and its effects on a plant. • What type of thinking is required? You are being asked to take what you already know and apply it to the effects of cavitation. • What key words does the question contain and what do they mean? o Cavitation, which is the blockage of xylem. o Translocation, which refers to the movement of nutrients through a plant. Gather Content • What do you already know about cavitation in plants? How does it relate to the question? o Xylem carries water through plant tissue. o Phloem carries nutrients through plant tissue. o Water flows from xylem to phloem, since the water potential in xylem is greater. o Phloem relies on the influx of water to push nutrients into tissues. o Cavitation refers to the presence of an air bubble in xylem. o If a xylem vessel is blocked, associated phloem will be impeded too. Consider Possibilities • What other information is related to the question? Which information is most useful? o Content specific to key concept. Choose Answer • Given what you now know, what information and/or problemsolving approach is most likely to produce the correct answer? Version 1
26
o If water flow is blocked, nutrient flow will be blocked as well. Reflect on Process • Did your problem-solving process lead you to the correct answer? If not, where did the process break down or lead you astray? How can you revise your approach to produce a more desirable result? o This question asked you to apply what you know about cavitation. Answering this question correctly depended on your ability to use cavitation and its effects on translocation in a new situation. If you got the correct answer, great job! If you got an incorrect answer, where did the process break down? Did you remember that cavitation stops the flow of water in xylem or that if a xylem vessel is blocked, associated phloem will be impeded in function? Did you have trouble extending these ideas of cavitation and translocation to determine the correct answer? 4) TRUE
Version 1
27
Clarify Question • What is the key concept addressed by the question? For water to move into plant roots, the water potential must be less inside of the plant than it is outside of the plant. • What type of thinking is required? You are being asked to take what you already know and apply it to the effects of changing tonicity in the soil around a plant. • What key words does the question contain and what do they mean? o Fertilizer, which is an applied substance that increases plant growth. o Phloem, which is vascular tissue that transports nutrients. Gather Content • What do you already know about water potential and how it effects the movement of water in plants? How does it relate to the question? o Adding fertilizer to the soil increases the solute concentration of the soil. o If the solute concentration of the soil is too high, the water potential will decline below that of the plant. o If the water potential is lower outside of the plant, water will flow out of the plant. o If water flows out of the plant, it will not be drawn into the xylem. o If water flow in the xylem ceases, the phloem will also be affected. o Without water flow from the xylem, nutrients will not be pushed from the phloem into the tissue. Consider Possibilities • What other information is related to the question? Which information is most useful? o Since fertilizer is a solute, adding it to the soil would affect the tonicity of the soil, which will affect water flow in the plant. Version 1
28
Choose Answer • Given what you now know, what information and/or problemsolving approach is most likely to produce the correct answer? o You need to connect the presence of fertilizer with increased tonicity. o Increased tonicity in the soil can result in water loss from the plant. o With water loss, the flow of water and nutrients in the xylem and phloem are impeded. Reflect on Process • Did your problem-solving process lead you to the correct answer? If not, where did the process break down or lead you astray? How can you revise your approach to produce a more desirable result? o This question asked you to predict the effects of increasing tonicity by the additional of fertilizer. Answering this question correctly depended on your ability to use your knowledge of water flow in a new situation. If you got the correct answer, great job! If you got an incorrect answer, where did the process break down? Did you remember that water flows into xylem under normal conditions, or that water in the xylem flows into the phloem. which pushes nutrients into the tissues? Did you have trouble extending these ideas of nutrients and water flow to determine the correct answer? 5) FALSE A lack of ATP in a root cell would affect the cell's ability to actively pump ions into the cell from the soil. Without these ions, the water potential of the cell would increase and make it more difficult for the root hair to absorb water. The movement of water is dependent on the soil having a higher water potential than the root hair so that water will move into the plant via osmosis. 6) FALSE Version 1
29
Clarify Question • What is the key concept addressed by the question? The key concept is root pressure. • What type of thinking is required? You are being asked to take what you already know and apply it to changing root pressure in a plant. • What key words does the question contain and what do they mean? o Root pressure is an accumulation of ions in root tissue, which causes an influx of water. o ABA is a plant hormone that causes stomata to close. o Stomata are openings on leaves that allow gas exchange to occur. Gather Content • What do you already know about root pressure and its effect on ABA? How does it relate to the question? o Increasing root pressure causes guttation, which is the loss of water from small veins in the leaf. o Guttation bypasses the stomata. Consider Possibilities • What other information is related to the question? Which information is most useful? o ABA acts to close the stomata. Choose Answer • Given what you now know, what information and/or problemsolving approach is most likely to produce the correct answer? o If you connect root pressure to guttation and not ABA and stomata, you will arrive at the correct answer. Reflect on Process Version 1
30
• Did your problem-solving process lead you to the correct answer? If not, where did the process break down or lead you astray? How can you revise your approach to produce a more desirable result? o This question asked you to separate the effects of root pressure from those of ABA. Answering this question correctly depended on your ability to use root pressure in a new situation. If you got the correct answer, great job! If you got an incorrect answer, where did the process break down? Did you remember that root pressure is connected to guttation or that guttation is unrelated to the stomata? Did you have trouble extending root pressure to determine the correct answer? 7) TRUE
Version 1
31
Clarify Question • What is the key concept addressed by the question? The key concept is transpiration. • What type of thinking is required? You are being asked to take what you already know and use, or apply, it to proton pumps and transpiration. • What key words does the question contain and what do they mean? o Proton pump, which creates a proton gradient across a membrane, and thereby stores potential energy. o Transpiration, which is the evaporation of water in the stomata. o Guard cell, which is a cell that opens and closes the stomata. Gather Content • What do you already know about the relationship between guard cells and transpiration? How does it relate to the question? o Guard cells require ATP to actively transport ions. o Proton pumps are used in this active transport. o As ions move in, so does water. o Water increases the turgor od the guard cells. o Increased turgor in guard cells opens the stomata. o Open stomata allow transpiration to occur. Consider Possibilities • What other information is related to the question? Which information is most useful? o If proton pumps are damaged, guard cells could not stockpile ions, and so they would not fill with water. If guard cells do not fill with water, the stomata remain closed. If guard cells remain closed, transpiration would not occur. Choose Answer Version 1
32
• Given what you now know, what information and/or problemsolving approach is most likely to produce the correct answer? o To answer this question, you need to connect the function of the proton pumps to the opening of the stomata. Once the stomata are open, transpiration can occur. Reflect on Process • Did your problem-solving process lead you to the correct answer? If not, where did the process break down or lead you astray? How can you revise your approach to produce a more desirable result? o This question asked you to connect proton pump function with guard cells and transpiration. Answering this question correctly depended on your ability to use the mechanics of transpiration in a new situation. If you got the correct answer, great job! If you got an incorrect answer, where did the process break down? Did you remember that the opening of guard cells requires the build up of ions via the use of proton pumps or that if guard cells do not open, transpiration cannot occur? Did you have trouble extending transpiration to determine the correct answer? 8) FALSE
Version 1
33
Clarify Question • What is the key concept addressed by the question? o The question is asking about the different forms of phytochrome. • What type of thinking is required? o This question is asking you to evaluate-phytochrome use in a plant. • What key words does the question contain and what do they mean? o Chlorophyll, which is a pigment that absorbs light that can be used in photosynthesis. o Red light, which includes wavelengths of light that are used in photosynthesis. o Phytochrome, which is a pigment-containing protein. Gather Content • What do you already know about "phytochromes and photosynthesis? How does it relate to the question? o Active processes require the conversion of Pr to Pfr. Consider Possibilities • What other information is related to the question? Which information is most useful? o Since chlorophyll synthesis requires active phytochrome, the Pr would need to become Pfr. Choose Answer • Given what you now know, what information and/or problemsolving approach is most likely to produce the correct answer? o In the presence of red light, the Pr form of phytochrome is converted to the active form Pfr , which would direct the synthesis of chlorophyll. Version 1
34
Reflect on Process • Did your problem-solving process lead you to the correct answer? If not, where did the process break down or lead you astray? How can you revise your approach to produce a more desirable result? This question asked you to evaluate the different forms of phytochrome. Answering this question correctly depended on your ability to evaluate when Pr and Pfr act. If you got the correct answer, well done! If you got an incorrect answer, where did the process break down? Did you remember that Pr differs from Pfr in that Pfr is required for active processes? Did you have trouble weighing the merits of Pr relative to Pfr to determine the correct answer? 9) TRUE
Version 1
35
Clarify Question • What is the key concept addressed by the question? o This question is asking about the importance of the root cap. • What type of thinking is required? o You are being asked to take what you already know and apply it to the function of a root cap. • What key words does the question contain and what do they mean? o Root cap, which is the covering that occurs at the end of a root. Gather Content • What do you already know about root caps? How does it relate to the question? o The gravity-sensing cells of the root are located in the root cap. o Studies have shown that removing the cap eliminates the root's ability to respond to gravity. Consider Possibilities • What other information is related to the question? Which information is most useful? o Since gravity-sensing cells are located in the root cap, removal of the root cap would eliminate this ability. Choose Answer • Given what you now know, what information and/or problemsolving approach is most likely to produce the correct answer? o To answer this question you must connect the presence of gravitysensing cells with the root cap. Removal of the root cap would remove gravity-sensing cells from the root. Version 1
36
Reflect on Process • Did your problem-solving process lead you to the correct answer? If not, where did the process break down or lead you astray? How can you revise your approach to produce a more desirable result? o This question asked you to understand the nature of the root cap. Answering this question correctly depended on your ability to use the composition of the root cap in a new situation. If you got the correct answer, great job! If you got an incorrect answer, where did the process break down? Did you remember that the root cap contains gravitysensing cells? Did you have trouble extending the nature of the root cap to determine the correct answer? 10) [A, C, D] Section 31.7 discusses plant nutrients. 11) [B, C, D] Phytochrome (Section 31.9) is involved in all the responses except stem elongation, which is a function of gibberellins (31.8). 12) [A, B, D, E] The functions of ethylene are described in Section 31.8. 13) [A, C, D, E] Abscissic acid is discussed in Section 31.8. 14) B Transpiration is evaporation of water through the stomata. Because water molecules stick to one another through cohesion, as one water molecule evaporates, it pulls other water molecules up towards the stomata. 15) E The hormone ABA triggers the outflux of potassium from guard cells. As the potassium exits, the guard cells water follows. This results in a loss of turgor pressure, and the cells cover the stomatal opening.
Version 1
37
16) A Water moves from regions of higher pressure to lower pressure (osmosis). In cells, this movement of water is facilitated by small membrane channels called aquaporins. These channels allow water, which is a polar molecule, to rapidly cross the nonpolar cell membrane. 17) C When water needs to be conserved, guard cells block the openings of the stomata. This prevents water loss due to transpiration. 18) C Adventitious roots emerge from the stem. These roots can form above the water surface and facilitate the transportion of oxygen to the parts of the plant that are submerged. 19) C Root pressure is due to osmosis in the roots. The roots continue to accumulate ions even when the plant is not transpiring. Because the roots have such a high ion concentration, water enters via osmosis and enters the xylem. 20) C Sources and sinks change over time, and therefore, the direction of phloem flow changes. 21) B Turgor results from pressure against the cell wall. This pressure is also known as pressure potential. Turgor pressure increases as the pressure potential increases. 22) E Lenticels are corky pores or narrow lines on the surface of the stems of woody plants that allow the interchange of gases between the interior tissue and the surrounding air. Therefore, these facilitate the transport of oxygen from the above-water portion of the plant to the parts that are below the water. Version 1
38
23) E The majority of water exits a plant through the stomata. Since water exits through the stomata, the plant can regulate water loss. The opening of the stomata are regulated by guard cells. When the plant needs to conserve water, the guard cells close the openings to the stomata. 24) C Regions of the root with root hairs absorb the most water. Root hairs, which are extensions of the epidermis, greatly increase the surface area of the root for absorption. Please refer to Figure 31.2 for more information. 25) D The endodermal cells of the root actively transport nutrients using hydrogen ion (proton) pumps. 26) B When the pressure potential reaches 0 MPa within a cell, there is no more turgor pressure. This pressure is necessary to keep the cell wall rigid and to keep the plant from wilting. 27) C A gas bubble in the xylem can block the movement of water through the plant. This can cause dehydration and even death of the plant. 28) A When carbon dioxide levels are high, the stomata can close because the Calvin cycle has the material needed to produce sugar. It is only when the cells are deficient in carbon dioxide that the stomata need to open. 29) B The pores in the leaves are called stomata. When open, they allow the entry of carbon dioxide and the exit of oxygen. The carbon dioxide is fixed into sugar in the Calvin cycle of photosynthesis. 30) C
Version 1
39
Plants need oxygen for cellular respiration. Standing water does not contain as much oxygen as moving water. Plants that have evolved in habitats with standing water have adaptations that allow them to facilitate the flow of oxygen to the tissues below water. 31) D
Version 1
40
Clarify Question • What is the key concept addressed by the question? o The key concept is the function of ABA. • What type of thinking is required? o Apply, because you must take what you know about ABA and apply it to a new situation. • What key words does the question contain? o ABA receptor sites, which are molecules that receive ABA and signal the cell to open ion channels. Gather Content • What do you already know about ABA? How does it relate to the question? o The hormone ABA is abscisic acid. o ABA binds to receptors in the cell membrane. o As a result, K+ and other ion channels open. o This causes guard cells to lose K+. o Water loss follows, and turgor pressure decreases. o This causes guard cells to close. o Closed guard cells become flaccid and cover the openings of the stomata. o This prevents excess water loss. Consider Possibilities • What other information is related to the question? Which information is most useful? o ABA acts on the guard cells that surround the stomata, so root function is not impacted. o If guard cells do not become flaccid, stomata will remain open, and Version 1
41
gas exchange can occur. Choose Answer • Given what you now know, what information and/or problemsolving approach is most likely to produce the correct answer? o As soon as you identify the function of ABA, you can deduce what would happen to a plant that did not have functional ABA receptors. Reflect on Process • Did your problem-solving process lead you to the correct answer? If not, where did the process break down or lead you astray? How can you revise your approach to produce a more desirable result? o This question requires you to connect what you know about ABA function in a normal plant with a sudden loss of ABA activity due to the toxin. First, you must recognize that ABA functions in guard cells that surround the stomata of the leaf. When you recall that ABA causes the stomata to close, you know that by closing, the stomata are preventing excess water loss. Therefore, if ABA is not received by the guard cells, the cells will remain open, which would allow evaporation of water without limit. 32) B
Version 1
42
Clarify Question • What is the key concept addressed by the question? o The key concept is the mechanical function of the guard cells. • What type of thinking is required? o Apply, because knowledge of how the guard cells function must be applied to a different concept. • What key words does the question contain? o High pressure potential, which is a high hydrostatic pressure inside a cell. o Guard cells, which open and closethe stomata on the leaf surface. Gather Content • What do you already know about how guard cells function? How does it relate to the question? o High pressure potential is directly related to turgor. o Turgor is maintained in plant cells by the active uptake of K+ and other ions. o As solute concentration increases through this active uptake, water potential decreases. o Water flows into the cell, toward the area of lower water potential. o This and any active process requires the use of energy. Consider Alternatives • What other information is related to the question? Which information is most useful? o Gas exchange is not directly related to the mechanics of the guard cells. o Any loss of water or ions would cause the guard cells to lose turgor and pressure potential. Version 1
43
Choose Strategy • Given what you now know, what information and/or problem solving approach is most likely to produce the correct answer? o To answer this question you must connect the concepts of pressure potential and turgor.To maintain turgor, ions must be actively transported (using ATP) into the cell.This lowers the water potential, which brings water in and increases turgor and the pressure potential. Reflect on Process • Did your problem-solving process lead you to the correct answer? If not, where did the process break down or lead you astray? How can you revise your approach to produce a more desirable result? o The question is asking you to relate the active uptake of ions with increased turgor. If you arrived at the correct answer, well done! If you came up with an incorrect answer, where did the process break down?Did you remember that as pressure potential increases, turgor increases. Turgor is maintained by the active uptake of K+ and other ions. 33) D The side of the membrane with the most solute would be hypertonic relative to the other side. This means water would flow via osmosis toward the side with the most solute. 34) B Translocation is the process where the carbohydrates manufactured during photosynthesis are transported from the leaves and other green parts to the rest of the plant. This is a function of the phloem. 35) E
Version 1
44
Cohesion is the attraction of one water molecule to another. This attraction is caused by the formation of hydrogen bonds. Water is a polar molecule, which means that part of the molecule has a weak positive charge and part of it has a weak negative charge. When the positive charge on one water molecule is attracted to a weak negative charge on another water molecule, a hydrogen bond forms. 36) D The total water potential of a cell is the sum of its pressure potential and solute potential. 37) B Since pure water does not have any solutes, it does not have any solute potential. 38) A Adhesion is when a water molecule is attracted to something other than another water molecule. Water molecules are attracted via adhesion to the walls of the xylem vessels. Cohesion is where water molecules stick to other water molecules. 39) A Guttation occurs when root pressure is so strong that it forces water up to the leaves. 40) C The pressure-flow hypothesis describes how carbohydrates flow from a source to a sink. 41) E The driving force of transpiration is the gradient that exists between the outside of the leaf (low humidity) and the inside of the leaf (high humidity). 42) C
Version 1
45
Mycorrhizal fungi interact with the roots of many species of plants. They extend the surface area for absorption of water and nutrients and therefore are beneficial to the plant. 43) D
Version 1
46
Clarify Question • What is the key concept addressed by the question? The key concept in this question is the movement of water into a fruit. • What type of thinking is required? You are being asked to take what you already know and apply it to the heavy water content of a fruit. • What key words does the question contain and what do they mean? o Stomata are small opening on leaves that allow gas exchange to occur, but also allow water loss through evaporation. o Transpiration, which is the process of pulling water into a plant through the roots via the evaporation of water through the leaves. Gather Content • What do you already know about how water moves through the tissues in a plant? How does it relate to the question? o The question tells you that water cannot enter the watermelon through transpiration or through the stomata. You know that the phloem is heavy in solutes, as it carries sugars and other nutrients in a water-based solution. Since this material in the phloem is solute-heavy, it is hypertonic and has lower water potential. Solutions with lower water potential such as this would receive water from surrounding tissues, which would have higher water potential. Consider Possibilities • What other information is related to the question? Which information is most useful? o Actively transporting water into a watermelon would be energetically costly and unnecessary. Xylem already inside the watermelon would not help to transport water into the watermelon.The question tells you that the skin of the watermelon is thick, so that would not promote the Version 1
47
movement of water from the soil directly into the watermelon. Choose Answer • Given what you now know, what information and/or problemsolving approach is most likely to produce the correct answer? o The question itself tells you it is not feasible for water to flow directly from the environment into the watermelon due to the thick skin and lack of stomata on the watermelon. To answer this question, you must recall the internal consistency of the watermelon.The watermelon becomes a sink for the hypertonic sap that is carried in the phloem. Reflect on Process • Did your problem-solving process lead you to the correct answer? If not, where did the process break down or lead you astray? How can you revise your approach to produce a more desirable result? o This question asked you to take what you know about the flow of materials within a plant, and apply it to the synthesis of fruit.Answering this question correctly depended on your ability to use the nature of the fluid within the phloem in a new situation. If you got the correct answer, great job! If you got an incorrect answer, where did the process break down? Did you remember that the fluid within the phloem is hypertonic or that the skin of a watermelon is too thick to allow water to flow in? Did you have trouble extending the nature of the fluid in the phloem to determine the correct answer? 44) C
Version 1
48
Clarify Question • What is the key concept addressed by the question? The question addresses adaptations of plants that live in high salt environments. • What type of thinking is required? You are being asked to take what you already know to explain how certain plants can cope with an extreme environment. • What key words does the question contain and what do they mean? o Halophytes, which are plants that can tolerate high salt conditions. o Saline, which refers a salty environment. o High osmotic potential, which is a high solute concentration. o Negative water potential, which is a lower proportion of water. Gather Content • What do you already know about the adaptations that occurs in plants that live in salty environments? How does it relate to the question? o In salty soil, the water potential inside the plant is higher than that outside, so plants dry out. o Halophytes are salt-loving plants, which can tolerate much higher levels of salt in the soil compared to other plants. o Some halophytes add extra ions to their root tissue. o The addition of excess ions in the root reverses the usual water potential gradient. o Water flows into the plant, from high to low water potential. Consider Possibilities • What other information is related to the question? Which information is most useful? o Stomata must be able to open at some point, or gas exchange would Version 1
49
be negatively impacted. o If stomata were consistently to remain open, too much water loss would occur. o A loss of ions would negatively impact the physiology of the plant. Choose Answer • Given what you now know, what information and/or problemsolving approach is most likely to produce the correct answer? o Plants must have water enter through the roots.Water flows from high water potential to low water potential. In salty soil, the water potential gradient causes water to flow out of plants. Halophytes reverse this gradient by pumping ions into the root tissue, thereby creating a lower water potential inside the root.This makes water flow into the roots of halophytes. Reflect on Process • Did your problem-solving process lead you to the correct answer? If not, where did the process break down or lead you astray? How can you revise your approach to produce a more desirable result? o This question asked you to explain how a halophyte survives extreme conditions.Answering this question correctly depended on your ability to use concept your knowledge of halophytes and water potential in a new situation. If you got the correct answer, great job! If you got an incorrect answer, where did the process break down? Did you remember that water potential is greater inside the root of a plant in salty soil, but halophytes can reverse this gradient? Did you have trouble extending the concept of water potential to determine the correct answer? 45) C
Version 1
50
Clarify Question • What is the key concept addressed by the question? The key concept is that a sink can be differentiated by the presence of stored carbohydrates. • What type of thinking is required? You are being asked to take what you already know apply it to the analysis of unknown tissue to determine whether the tissue is a source or a sink. • What key words does the question contain and what do they mean? o Source, which is the part of the plant that is photosynthetic. o Sink, which is the part of the plant that uses or stores carbohydrates. o Iodine, can be used as an indicator for starch. Gather Content • What do you already know about the function of a sink in a plant? How does it relate to the question? o Colorless tissue is not photosynthetic. o The product of photosynthesis is glucose. o Glucose is stored by plants as starch. o A positive iodine test for starch turns blue black in color. Consider Possibilities • What other information is related to the question? Which information is most useful? o Content specific to key concept. Choose Answer • Given what you now know, what information and/or problemsolving approach is most likely to produce the correct answer? o Sources are photosynthetic, so they are green in color. They would not Version 1
51
contain colorless plastids. Reflect on Process • Did your problem-solving process lead you to the correct answer? If not, where did the process break down or lead you astray? How can you revise your approach to produce a more desirable result? • Apply level: o This question asked you to apply what you know about sources and sinks to a sample of unknown origin. Answering this question correctly depended on your ability to use these ideas about sources and sinks in a new situation. If you got the correct answer, great job! If you got an incorrect answer, where did the process break down? Did you remember that sinks are sites of carbohydrate use and storage or that iodine indicates starch by turning blue-black? Did you have trouble extending these concepts to determine the correct answer? 46) E In hydroponic cultures, plants are grown with their roots suspended in aerated water containing nutrients. 47) C A deficiency in a micronutrient can have severe effects on plant growth. 48) B For many plants, the uptake of nutrients from the soil is limited by plasma membrane transport. Genes that code for these membrane transporters are being inserted into crop plants to make them more efficient at obtaining nutrients from the soil. 49) A
Version 1
52
Clarify Question • What is the key concept addressed by the question? o The question is asking about the importance of ammonia to plants. • What type of thinking is required? o You are being asked to take what you already know and apply it to the use of ammonia in plants. • What key words does the question contain and what do they mean? o Ammonia, which is an important source of nitrogen to plants. Gather Content • What do you already know about ammonia in plants? How does it relate to the question? o Amino acids and proteins require nitrogen in their structure. o Ammonia is an important source of nitrogen. o Proline and alanine are amino acids. Consider Possibilities • What other information is related to the question? Which information is most useful? o Starch, glucose, and fructose are carbohydrates. o Fatty acids and carbohydrates lack nitrogen in their structure. Choose Answer • Given what you now know, what information and/or problemsolving approach is most likely to produce the correct answer? o To answer this question, you must first realize that ammonia can be used a nitrogen source in plants. You must then connect the fact that proline and alanine are amino acids with the fact that amino acids Version 1
53
require ammonia. Reflect on Process • Did your problem-solving process lead you to the correct answer? If not, where did the process break down or lead you astray? How can you revise your approach to produce a more desirable result? o This question asked you to identify proline and alanine as amino acids, and also to know that ammonia is a source of nitrogen. Answering this question correctly depended on your ability to use ammonia as a source of nitrogen in a new situation. If you got the correct answer, great job! If you got an incorrect answer, where did the process break down? Did you remember that nitrogen can be obtained from ammonia or that proline and alanine as amino acids require nitrogen? Did you have trouble extending ammonia as a nitrogen source to determine the correct answer? 50) D About 90% of all vascular plants have a mutualistic symbiotic relationship with mycorrhizal fungi. The fungi help the plant to extract phosphorus in the soil in exchange for sugar. 51) B Mycorrhizal fungi form a mutualistic symbiotic relationship with plants. In exchange for carbohydrates, the fungi transfer phosphate to the plant. 52) C Essential nutrients are nutrients necessary for normal growth of the plant. 53) B
Version 1
54
Clarify Question • What is the key concept addressed by the question? o This question is asking about the function of amylase. • What type of thinking is required? o You are being asked to take what you already know and apply it to the use of amylase. • What key words does the question contain and what do they mean? o Gibberellin, which is a plant hormone. o Hydrolysis, which refers to the splitting of larger molecules into smaller molecules. o Amylase, which is an enzyme that occurs in plants. Gather Content • What do you already know about hormone and enzyme function in plants? How does it relate to the question? o Gibberellins have multiple roles within plants. o Amylase is an enzyme that breaks down starch into individual glucose molecules. Consider Possibilities • What other information is related to the question? Which information is most useful? o "Amyl/o" is a word root that refers to starch, not other macromolecules. Choose Answer • Given what you now know, what information and/or problemsolving approach is most likely to produce the correct answer? o The key in this question is the reference to amylase. Amylase is an enzyme that is found in plants and other organisms, and it is used to Version 1
55
break down starch. Reflect on Process • Did your problem-solving process lead you to the correct answer? If not, where did the process break down or lead you astray? How can you revise your approach to produce a more desirable result? o This question asked you to understand the role of amylase. Answering this question correctly depended on your ability to use the function of amylase in a new situation. If you got the correct answer, great job! If you got an incorrect answer, where did the process break down? Did you remember that amyl/o is always a reference to starch or that amylase is an enzyme that hydrolyzes starch? Did you have trouble extending the function of amylase to determine the correct answer? 54) D Phototropic responses include the bending of growing stems and other plant parts toward sources of light with blue wavelengths. Section 31.9 discusses phototropism. 55) D Amyloplasts are the strucutures believed to be responsible for sensing gravity (Section 31.9). 56) A Cytokinin is a hormone that works in conjuction with auxin to stimulate cell division and differentiation. 57) D
Version 1
56
Clarify Question • What is the key concept addressed by the question? o The question is asking about chemical influences on fruit ripening. • What type of thinking is required? o You are being asked to take what you already know and apply it to the ripening of fruit. • What key words does the question contain and what do they mean? o Ripening, which is a maturation process by which fruit become sweeter and softer. Gather Content • What do you already know about fruit ripening? How does it relate to the question? o Hormones drive processes in plants, including ripening of fruit. Consider Possibilities • What other information is related to the question? Which information is most useful? o Gibberellin promotes stem elongation and production of enzymes in germinating seeds. o Auxin has many functions but is not involved in fruit ripining. o Abscisic acid inhibits bud growth, plays a role in seed dormancy, inhibits other hormones, and controls stomatal closure. o Cytokinin promotes cell division. Choose Answer • Given what you now know, what information and/or problemsolving approach is most likely to produce the correct answer? Version 1
57
o Ethylene is the only hormone listed that promotes fruit ripening. Reflect on Process • Did your problem-solving process lead you to the correct answer? If not, where did the process break down or lead you astray? How can you revise your approach to produce a more desirable result? o This question asked you to assess fruit ripening. Answering this question correctly depended on your ability to use the characteristics of plant hormones in a new situation. If you got the correct answer, great job! If you got an incorrect answer, where did the process break down? Did you remember that ripening is hormone driven or that ethylene promotes ripening? Did you have trouble extending the characteristics of plant hormones to determine the correct answer? 58) C
Version 1
58
Clarify Question • What is the key concept addressed by the question? o The question is asking about the role of stomata in transpiration. • What type of thinking is required? o This is an analyze question because you have to break the stomata and transpiration into their component pieces to understand how they function. • What key words does the question contain and what do they mean? o Stomata, which are openings on leaves. o Guard cells, which regulate the stomata. o Transpiration, which is the evaporation of water from plant leaves. Gather Content • What do you already know about the relationship between transpiration and stomata? How does it relate to the question? o Abscisic acid (ABA) plays an important role in the opening and closing of the stomata. o Stomata regulate water loss from the leaves. Consider Possibilities • What other information is related to the question? Which information is most useful? o Decreased water flow into the plant would not cause the stomata to open. Choose Answer • Given what you now know, what information and/or problemsolving approach is most likely to produce the correct answer? o Decreased soil water potential would result in a reduced water flow Version 1
59
into the plant. o To prevent dehydration, ABA would signal the guard cells to close. o Closed guard cells would reduce transpiration and prevent dehydration. Reflect on Process • Did your problem-solving process lead you to the correct answer? If not, where did the process break down or lead you astray? How can you revise your approach to produce a more desirable result? o This question asked you to understand the factors that regulate transpiration.Answering this question correctly depended not only on distinguishing between the roles of different hormones, but on your ability to break down, or analyze, the effects of hormones on the stomata and transpiration. If you got the correct answer, excellent! If you got an incorrect answer, where did the process break down? Did you remember that ABA influences stomatal activity or that stomata regulate transpiration? Did you have trouble breaking down the functions of the hormones to determine the correct answer? 59) D Both auxin and gibberllins (Section 31.8) enhance plant growth. 60) B Charles Darwin and his son Francis found that when a shoot tip was covered with foil, the shoot would not bend (Section 31.10). This finding eventually led to the discovery of auxin. 61) B
Version 1
60
Clarify Question • What is the key concept addressed by the question? o The question is asking about the effect of seed removal from a fruit. • What type of thinking is required? o You are being asked to take what you already know and apply it to the importance of seeds to fruit. • What key words does the question contain and what do they mean? o Seeds, which are plant embryos. o Fruit, which is a seed-bearing structure produced by a plant. o Hormones, which are chemicals that can regulate processes in tissues. Gather Content • What do you already know about the relationship between seeds and fruit? How does it relate to the question? o A strawberry lacking seeds will not develop. o If treated with a synthetic auxin, the fruit will grow. Consider Possibilities • What other information is related to the question? Which information is most useful? o Ethylene is involved with ripening. o Gibberellins influence seed germination. o Strigolactones and oligosaccharides do not appear to have a role in fruit formation. Choose Answer • Given what you now know, what information and/or problemVersion 1
61
solving approach is most likely to produce the correct answer? o Auxin is involved with the formation of fruit. Reflect on Process • Did your problem-solving process lead you to the correct answer? If not, where did the process break down or lead you astray? How can you revise your approach to produce a more desirable result? o This question asked you to understand the importance of seeds to fruit. Answering this question correctly depended on your ability to use the characteristics of seeds in a new situation. If you got the correct answer, great job! If you got an incorrect answer, where did the process break down? Did you remember that seeds contain hormones that influence fruit or that auxin is a hormone that stimulates fruit formation? Did you have trouble extending the characteristics of seeds to determine the correct answer? 62) E Synthetic auxins are discussed in Section 31.8. 63) C Coconut milk contains cytokinins (Section 31.10), which work with auxin to stimulate cell division and differentiation. 64) A The properties of auxin are described in Section 31.8. 65) C Circadian rhythms (Section 31.9) relate the day-night cycle to the eukaryotic organism that has them. 66) B The discovery of auxin is discussed in Section 31.8. 67) B Phototropism is defined in Section 31.9. 68) C
Version 1
62
Auxin and it's role in the response to light are discussed in Section 31.9. 69) C The five steps that lead from auxin perception to auxin-induced gene expression are listed in Section 31.8. 70) B An antisense copy of the gene for ethylene biosynthesis prevents the formation of ethylene and subsequent ripening of transgenic fruit. The antisense strand is complementary to the sequence for the ethylene biosynthesis gene. After transcription, the antisense mRNA pairs with the sense mRNA, and the double-stranded mRNA cannot be translated into a functional protein. Ethylene is not produced, and the fruit does not ripen. 71) C Chlorophyll absorbs light in the red spectrum (Section 31.9). So light passing out of a leaf will contain less red wavelengths. 72) D Phytochrome is discussed in Section 31.9. 73) A
Version 1
63
Clarify Question • What is the key concept addressed by the question? o This question is asking about the mode of transport from the roots to the stem for abscisic acid. • What type of thinking is required? o You are being asked to take what you already know and apply it to the movement of abscisic acid within a plant. • What key words does the question contain and what do they mean? o Abscisic acid, which is a plant hormone. o Transpirational water loss, which is the evaporation of water from the surface of a plant. Gather Content • What do you already know about transportation of materials within a plant? How does it relate to the question? • Xylem, which is composed of vessels and tracheids, moves water and other materials from the roots up through the shoots to the leaves. • Abscisic acid is made in the roots of plants. Consider Possibilities • What other information is related to the question? Which information is most useful? o Travel between cell walls or from cell to cell would be very inefficient. Choose Answer • Given what you now know, what information and/or problemsolving approach is most likely to produce the correct answer? o Abscisic acid is synthesized in the roots and must travel upward. The Version 1
64
xylem and tracheids carry materials upward to the stem and leaves. Reflect on Process • Did your problem-solving process lead you to the correct answer? If not, where did the process break down or lead you astray? How can you revise your approach to produce a more desirable result? o This question asked you to assess the route through which abscisic acid moves from the root to the stem of the plant.Answering this question correctly depended on your ability to use the characteristics of transport within a plant in a new situation. If you got the correct answer, great job! If you got an incorrect answer, where did the process break down? Did you remember that abscisic acid is made in the roots or that xylem carries material upward from the roots to the stem and leaves? Did you have trouble extending the routes of transport within a plant to determine the correct answer? 74) C
Version 1
65
Clarify Question • What is the key concept addressed by the question? o This question is asking about the role of phytochrome in plants. • What type of thinking is required? o This question is asking you to evaluate the importance of phytochrome. • What key words does the question contain and what do they mean? o Phytochrome, which is a pigment-containing protein. Gather Content • What do you already know about the role of phytochrome in plants? How does it relate to the question? o Phytochrome transduces information. o One of the functions of phytochrome is to activate the transcription of light-regulated genes. Consider Possibilities • What other information is related to the question? Which information is most useful? o If phytochrome absorbed green wavelengths of light, the tissue in which it occurred would not appear green. o Phytochrome does not play a direct role in the reactions of photosynthesis. Choose Answer • Given what you now know, what information and/or problemsolving approach is most likely to produce the correct answer? o To answer this question, you must connect phytochromes with the Version 1
66
indirect role that they play in photosynthesis. Phytochrome activates the expression of genes that code for photosynthesis-related proteins like the proteins necessary for chloroplast development. Reflect on Process • Did your problem-solving process lead you to the correct answer? If not, where did the process break down or lead you astray? How can you revise your approach to produce a more desirable result? This question asked you to evaluate the function of phytochromes. Answering this question correctly depended on your ability to evaluate the activities that occur in tissue that has phytochromes present. If you got the correct answer, well done! If you got an incorrect answer, where did the process break down? Did you remember that phytochromes play an indirect role in photosynthesis? Did you have trouble weighing the merits of the presence of phytochromes to determine the correct answer? 75) D
Version 1
67
Clarify Question • What is the key concept addressed by the question? o This question is asking about seasonal fluctuations in the levels of cytokinins. • What type of thinking is required? o This question is asking you to evaluate-the changes that occur in cytokinin levels in plants as the seasons change in a temperate climate. • What key words does the question contain and what do they mean? o Temperate, which refers to climates that have moderate temperature changes between seasons. o Cytokinin, which are plant hormones that promote cell division in plants. Gather Content • What do you already know about cytokinin levels and seasonality? How does it relate to the question? o Highest growth rates for temperate plants occur in warmer months. o In the winter, cytokinin levels drop in temperate plants. This places buds into a state of dormancy during the cold and dark winter. This will protect the delicate tissues until the spring when cytokinin levels will increase again and buds will break dormancy and grow. Consider Possibilities • What other information is related to the question? Which information is most useful? o During warmer months, biochemical activities and plant growth would occur at a higher rate than they would in colder months. Choose Answer
Version 1
68
• Given what you now know, what information and/or problemsolving approach is most likely to produce the correct answer? o To answer this question, you must connect the ideas of cold weather and the inhibition of metabolic activity. This is adaptive because plant growth at this time would leave the tissues vulnerable to damage due to freezing. Reflect on Process • Did your problem-solving process lead you to the correct answer? If not, where did the process break down or lead you astray? How can you revise your approach to produce a more desirable result? o This question asked you to evaluate the importance of seasonal changes on cytokinin activity. Answering this question correctly depended on your ability to evaluate how living tissues react to temperature changes. If you got the correct answer, well done! If you got an incorrect answer, where did the process break down? Did you remember that freezing temperatures destroy living tissue? Did you have trouble weighing the merits of plant growth to environmental extremes to determine the correct answer? 76) B Auxin (Section 31.8) has many roles in a plant, two of which are to increase the plasticity (flexibility) of plant cell walls and stimulate stem elongation. 77) C Charles and Francis Darwin's experiments on phototropism are discussed in Section 31.8. 78) C See Section 31.8. 79) B
Version 1
69
Please refer to the section on ABA (Section 31.8) and the physiologic control of the stomata (Section 31.4).
Version 1
70
CHAPTER 32 TRUE/FALSE - Write 'T' if the statement is true and 'F' if the statement is false. 1) The axon of a neuron can only communicate with other neuronal cells. ⊚ ⊚
true false
MULTIPLE CHOICE - Choose the one alternative that best completes the statement or answers the question. 2) To diagnose and treat a disease, a doctor must evaluate the patient’s body at multiple levels of organization. Identify the level of organization relevant to each stage of the process described below. (1) As part of Fred’s annual physical, the doctor examined his skin, fingernails, and hair for signs of disease. Noticing an unusual lesion on his neck, the doctor referred Fred to a dermatologist. (2) Upon observing the lesion, the dermatologist said, “Yes, this looks like it could be a squamous cell carcinoma, a tumor in the epidermis of the skin.” (3) The dermatologist carefully examined all of Fred’s skin for signs of additional tumors, but did not find any. (4) The dermatologist removed the lesion and sent it for a biopsy. Under the microscope, the pathologist saw some cells with abnormal morphology and confirmed a diagnosis of squamous cell carcinoma. However, he noted that the margins of the lesion were clear of abnormal cells, and the tumor had been successfully removed. A) organism, cell,tissue, organ B) organ system,tissue, organ, cell C) organ system,organ, tissue, cell D) organism, cell,organ system, tissue
3)
Which of the following is not a level of organization in the vertebrate body?
Version 1
1
A) Cells B) Bones C) Tissues D) Organs E) Organ systems
4) It’s the ninth inning, and the bases are loaded. As the pitcher winds up to throw the ball, how does each tissue in his arm contribute to this critical pitch? The __________ of his fingers grips the ball. His __________ sends instructions that trigger his __________ to contract. His __________ provides stability and transmits the force produced. A) ectoderm;ectoderm; mesoderm; endoderm B) connective tissue; nerve tissue; muscle tissue; epithelial tissue C) epithelium; ectoderm; mesoderm; endoderm D) epithelial tissue; nerve tissue; muscle tissue; connective tissue
5)
If at the last minute the pitcher decides to throw a curveball, which tissue responds first? A) Epithelial tissue B) Ectoderm C) Nerve tissue D) Muscle tissue
6)
Which of the following are the fundamental embryonic tissues called the "germ layers"? A) Tissues, organs, and organ systems B) Mesoderm, muscle, and tissue C) Ectoderm, endoderm, and exoskeleton D) Mesoderm, endoderm, and ectoderm E) Mesoderm, exoderm, and hydroderm
Version 1
2
7) In 1822, the Canadian fur trader Alexis St. Martin was accidentally shot with a musket. The shot entered his left chest, fractured a rib, damaged the lungs, lacerated the diaphragm, perforated the stomach, and finally exited his body. Amazingly he survived, but the hole in his belly never completely healed. This allowed Dr. William Beaumont direct access to his stomach for experiments, and led to his discovery of the importance of stomach acid in digestion. Which body cavities did the musket shot pass through? A) The shot enteredthe dorsal cavity and exited through the ventral cavity. B) The shot enteredthe thoracic cavity and exited through the abdominopelvic cavity. C) The shot enteredthe thoracic cavity and exited through the pleural cavity. D) The shot enteredthe pericardial cavity and exited through the pleural cavity.
8) In mammals and some reptiles, the coelom is divided into smaller cavities that enclose the lungs, heart, and gut. What are these cavities called? A) Pleural, pericardial, and peritoneal cavities B) Thoracic, pleural, and pericardial cavities C) Pleural, pericardial, and cardial cavities D) Perithoracic, peritoneal, and pericardial cavities
9)
What are epithelial cells with a flat, irregular shape and tapered edges called? A) Columnar B) Pseudostratified C) Squamous D) Cuboidal E) Stratified
10) Goblet cell hyperplasia is associated with asthma. Based on their function, what is the likely outcome of an overabundance of goblet cells?
Version 1
3
A) Decreased mucus production B) Excess mucus production C) Excess O 2 absorption D) Decreased CO 2 absorption
11)
The glands of vertebrates are derived from __________ epithelium. A) invaginated B) stratified C) endocrine D) scarified E) keratinized
12) If the connection between a gland and the epithelial membrane is maintained as a duct, the gland is called a(n) __________ gland. A) exocrine B) endocrine C) lymph D) sebaceous E) adrenal
13)
Which of the following would not provide protection to the epidermis? A) Keratinized epithelium B) Simple squamous epithelium C) Stratified squamous epithelium D) Layers of dead squamous cells
14)
Which of the following epithelial tissues secretes mucus and is dense with cilia?
Version 1
4
A) Cuboidal B) Squamous C) Stratified D) Columnar
15) A friend asks you how exocrine glands differ from endocrine glands. She knows that both form from invaginated epithelium. How do you respond? A) Exocrine and endocrine glands are essentially the same, both produce products that are secreted to the skin's surface. B) Endocrine glands produce hormones that enter the blood stream, while exocrine glands secrete sweat and oil directly to the surface of the skin. C) Exocrine glands are found in the thoracic cavity while endocrine glands are located in the pleural cavity. D) The liver is anendocrine gland that secretes directly into epithelial tissues. The pituitarygland is an exocrine gland that secretes into the blood stream.
16) Connective tissue includes adipose tissue, tendons, bone, cartilage, and blood. What do all these tissues have in common? A) They all consistof few cells sparsely distributed in large amounts of extracellularmaterial. B) They all providestructural support to the body. C) They are all hardtissues. D) They allphysically bind other tissues together.
17) The most abundant protein in mammals is collagen. This strong, ropelike molecule is particularly abundant in which tissue?
Version 1
5
A) Heart muscle B) Spinal cord C) Ligaments D) Skeletal muscle
18) When collagen is hydrolyzed it forms gelatin, the main ingredient of a jiggly dessert adored by children. What parts of a cow provide a rich source of collagen? A) Heart and other muscles B) Brain C) Glandular organs D) Skin, tendons, and cartilage
19) You are researching a family with a heritable disease whose symptoms include hyperflexible joints, pliable skin, stretchy ligaments, and fragile muscles that tear easily. You plan to use genetic testing to determine the cause of the disease. Which gene do you plan to study first? A) Actin B) Epidermal growth factor C) Myosin II heavy chain D) Collagen
20) All connective tissues share which of the following structural features? I Contain cells II Contain a matrix filled with extracellular material III Contain sarcomere units
Version 1
6
A) I and III B) II only C) II and III D) I, II, and II E) I and II
21)
In humans, which of these structures is not made of cartilage? A) Tip of the nose B) Outer ear C) Intervertebral disks of the backbone D) Knee cap E) Larynx
22)
Which of these is not part of loose connective tissue? A) Collagen B) Elastin C) Reticulin D) Chondroitin E) Adipose cells
23)
What tissue type produces the tough coverings that package organs? A) Loose connective tissue B) Dense irregular connective tissue C) Collagen D) Dense regular connective tissue E) Adipose
24)
Which connective tissue has a fluid matrix?
Version 1
7
A) Adipose B) Blood C) Cartilage D) Dense connective tissue E) Loose connective tissue
25)
Which of the following statements about bone and its structure is not correct?
A) Bone contains anextracellular matrix of collagen fibers and inorganic calcium phosphatecrystals. B) Bone tissue is notliving because it contains inorganic calcium phosphate crystals. C) Bone tissue is aspecial form of connective tissue. D) Bone tissue servesas a reservoir for calcium and phosphate ions. E) Bone marrow is areservoir for blood stem cells
26)
What is cartilage composed of? A) Chondroitin and calcium phosphate B) Chondroitin and collagen fibers C) Collagen fibers and osteocytes D) Osteocytes and glycoproteins E) Osteocytes and calcium phosphate
27) Which is another name for bone cells that remain alive even after the extracellular matrix becomes hardened with calcium phosphate crystals? A) Osteocytes B) Erythrocytes C) Leukocytes D) Fibrocytes E) Chondrocytes
Version 1
8
28) A patient has a disease with symptoms in the outer ear, nose, larynx, and spine. Which cells are most likely to have a problem? A) Leukocytes B) Erythrocytes C) Chondrocytes D) Osteocytes E) Fibroblasts
29)
Which is not a cell or fiber type found in loose connective tissues? A) Macrophages B) Mast cells C) Adipose cells D) Collagen fibers E) Chondrocytes
30) The characteristic that makes the muscle cells unique is the relative abundance and organization of A) collagen fibers. B) gelatinous mass. C) actin and myosin fibers. D) keratin fibers. E) fibroblast cells.
31) unit?
What feature of cardiac muscle helps the cells coordinate their contraction as a single
Version 1
9
A) Fused cells with multiple nuclei B) Covalent bonds between actin and myosin molecules C) Intercalated disks rich in gap junctions D) Large numbers of motor neurons
32) A scientist is trying to coax stem cells to differentiate into muscle to use as a therapy for muscular dystrophy. He looks at his Petri dish and is excited to see the cells contracting! The cells appear striated and multinucleate. What type of muscle are they? A) Cardiac muscle B) Smooth muscle C) Involuntary muscle D) Skeletal muscle
33) Which term refers to muscles that are attached to bones and cause bones to move at their joints when they contract? A) Smooth muscle B) Skeletal muscle C) Cardiac muscle D) Ligament muscle E) Involuntary muscle
34)
Which of the following types of muscles are striated? A) Skeletal and smooth B) Cardiac only C) Skeletal and cardiac D) Skeletal, smooth, and cardiac
35)
Which muscle contraction is involuntary?
Version 1
10
A) Cardiac only B) Smooth only C) Cardiac and smooth D) Skeletal only E) Cardiac, smooth, and skeletal
36) The cell body of a motor neuron resides in the spinal cord, but it may control muscles a long distance away. In a giraffe, the __________ of a leg motor neuron may be six feet long! A) dendrite B) soma C) axon D) node ofRanvier
37) What are the cells that are specialized to produce and conduct electrochemical impulses called? A) Neurons B) Dendrites C) Axons D) Fibroblasts E) Astrocytes
38)
Where is the nucleus of a neuron found? A) Axon branches B) Companion cell C) Cell body D) Dendrites E) Myelin sheath
Version 1
11
39) In your Ph.D. research, you use thin slices of brain tissue to study neuronal signaling. You attach an electrode to a single neuron, but cannot detect any activity. What is the most likely explanation? A) When you sliced the brain, you disconnected the cell from its dendrites. B) Neurons are not electrically active; you should be recording neuroglia instead. C) When you sliced the brain, you disconnected the cell from all of its axons. D) Excessivemyelination is blocking inputs to the cell.
40)
What structure do myelin sheaths form along? A) Axons B) Dendrites C) Neuroglia D) Cell bodies E) Ganglia
41) In addition to the senses of vision, taste, smell, hearing, and touch, we also have a sense called proprioception. It tells us the position of our bodies, based in part on stretch receptors in muscles. Proprioceptors are part of the __________ nervous system. A) peripheral B) central C) neuroglial D) motor
42) A recovering stroke patient complained about strange smells in his room, but no cause could be found. The doctor diagnosed this as phantosmia, or olfactory hallucination, and said it was likely due to stroke-related damage to which tissue?
Version 1
12
A) Central nervous system B) Olfactory receptors C) Peripheral nervous system D) Nodes of Ranvier
43) Controversy surrounds the use of zinc-containing nasal sprays as a cold remedy, since they have been associated with anosmia (loss of smell). If true, this is likely due to the sprays damaging which tissue? A) Central nervous system B) Osmic nerve C) Peripheral nervous system D) Neuroglial system
44) When we watch Olympic athletes perform, we are admiring the expert control of which systems working together? A) Skeletal, endocrine, and circulatory systems B) Nervous, circulatory, and skeletal systems C) Muscular, integumentary, and circulatory systems D) Nervous, muscular, and skeletal systems
45)
The mammalian jaw is an example of what kind of moveable joint? A) Hinge joint B) Combination joint C) Gliding joint D) Ball-and-socket joint
Version 1
13
46) As the Baby Boomer generation ages, more and more joint replacement surgeries take place each year. Hip replacement surgery is the gold standard of all joint replacement surgeries, with many patients able to walk just a few hours after surgery and return home the very next day. In contrast, knee replacement surgery usually requires hospital stays of several days, followed by weeks to months of physical therapy. What best explains why the recovery after hip replacement surgery is so much faster than after knee replacement surgery? A) The hip joint is more superficial and more accessible to surgeons than theknee joint. B) The knee joint is stabilized by many more structures (many of which have tobe cut) than the hip joint. C) The hip only moves in one direction, and recovery of one motion is easierthan recovering movement in many directions. D) The upper end ofthe thigh bone has less overall surface contact with the hip bone than itslower end has in the knee with bones of the lower leg. E) The knee is a type of combination joint, which are difficult to replace.
47) Shown below is an electron micrograph of part of a myofibril from a relaxed muscle fiber. Which of the labeled regions will not shorten when the muscle contracts?
A) A band B) H band C) I band D) sarcomere
Version 1
14
48) In a stimulated muscle fiber, the calcium ions released by the sarcoplasmic reticulum bind to the protein __________. A) troponin B) tropomyosin C) actin D) myosin
49) You decide to do an experiment with isolated myofibrils to explore the effects of ATP hydrolysis on myofibril function. You do this by bathing the myofibrils in a solution that contains a modified form of ATP, which cannot be split into ADP and P i. At what step will myofibril function be arrested by using this compound instead of unmodified ATP? A) There will be no block in the cycle, since ATP hydrolysis is not requiredfor this process. B) Myosin molecules will be unable to assemble into thick filaments. C) Myosin heads will be unable to bind to the actin filaments. D) Myosin heads willbind to the actin filaments, but the power stroke will not occur and actinfilaments will not slide over the myosin filaments. E) The myosin heads will remain attached to the actin filaments after the powerstroke.
Version 1
15
50) The sliding filament model for muscle contraction can be studied using an isolated skeletal muscle that is fixed at each end, while a machine records the tension that is generated when the muscle is stimulated to contract. In one particular muscle tested, the length of the thick filaments was 1.6 μm, and the length of the thin filaments that project in from each Z line toward the center of the sarcomere was 1.0 μm. A summary of the results comparing sarcomere length to the degree of tension produced during contraction is shown below. What most likely explains the difference between segment II and segment III of the graph?
A) There is anincreasing overlap of the free ends of the thin filaments in segment III butnot in segment II. B) The muscle cellsused up all the ATP by the end of segment II. C) The distancebetween the Z lines is constant in segment II but rapidly increasing in segmentIII. D) Fewer myosincross-bridges are forming in segment II than in segment III. E) The length of thethick filaments is decreasing in segment III but not in segment II.
51) The structure enabling depolarization to travel deep into the muscle fiber is the __________.
Version 1
16
A) T tubule B) sarcoplasmic reticulum C) motor unit D) sarcomere
52) The neurotransmitter released from the nerve terminus at the neuromuscular junction is __________. A) troponin B) Ca 2+ C) tropomyosin D) acetylcholine
53)
Which of the following regarding bone growth is false?
A) Cartilage ofgrowth plates actively grow in lengthwise direction. B) Increase in bone thickness results from the adding of bone layers on top of the periosteum. C) Some cartilage remains after it is replaced by bone. D) Cartilage calcification encroaches on the cartilaginous growth plate.
54)
Which of the following regarding the haversian system is false? A) It keeps osteocytes alive. B) Haversian canals may contain nerve fibers. C) Haversian canals run parallel to the length of long bone. D) Haversian canalsare created by osteoclasts that tunnel through the bone matrix.
55) You witness a demonstration of a martial arts master breaking a large concrete block with his hand. Besides mastering the technique, what is another benefit of the repetitive practice of breaking concrete? Version 1
17
A) Repetitive stress on the long bones will solidify cartilage. B) The Haversiancanal system will supply more blood and strengthen the bone. C) The repetitivestress will lead to remodeling and strengthening of the long bones. D) The marrow cavity will solidify and strengthen the bone.
56)
Which of the following is the smallest unit in terms of size? A) Quadriceps B) Myofibril C) Muscle fiber D) Sarcomere E) Myofilaments
57)
Which of the following is required to detach myosin from actin? A) Calcium B) ATP C) Phosphate D) Troponin E) ADP
58)
In arthropods, locomotion is accomplished by muscles that work against A) each other. B) the exoskeleton. C) a hydrostatic skeleton. D) mineralized bones. E) the pseudocoel.
59) Insects such as mosquitoes and beetles can beat their wings at tremendous speeds. This can be accomplished because Version 1
18
A) their wing muscle contractions causes another set of muscles to stretch, which in turn produces another contraction without waiting for the arrival of a nerve impulse. B) their wing muscle contractions cause another set of muscles to relax, which in turn produces another wing movement. C) their wing muscle contractions are stimulated by the continuous nerve signal impulses, which produce a continuous contraction of their wing muscles. D) their wing muscle contractions are capable of producing more ATP which allows these insects to have much faster and sustained wing movements. E) their wing muscle contractions produce nerve impulses, which lead to more contraction of the wing muscles which in turn permits faster motion of the wings.
60)
Select the incorrect statement related to sarcomere structure and function A) Thin filaments are not present in the H band of a sarcomere. B) Two Zlinesform the opposite bordersof one sarcomere. C) Stacks of thickfilaments produce dark bands, called the A band. D) I bands consistofboth thin and thick filaments. E) Sarcomeres are the smallest subunits of muscle contraction.
61)
Which of the following joint types is not correctly matched with its example? A) Ball-and-socket joint—femur and pelvis B) Gliding joint—vertebral column C) Immovable joint—skull sutures D) Hinge joint—finger joint E) Combination joint—knee joint
62)
In order for an earthworm to lengthen several contiguous segments of its body, it must
Version 1
19
A) contract itscircular muscles, which pulls up the chaetae and moves the body fluid parallelto the long axis of the worm. B) contract itslongitudinal muscles, which pulls up the chaetae and moves the body fluidperpendicular to the long axis of the worm. C) contract itscircular muscles, which extendsthe chaetae into the surroundingsoiland moves the body fluid parallel to the long axis of the worm. D) contract itslongitudinal muscles, which extendsthe chaetae into the surroundingsoiland moves the body fluid parallel to the long axis of the worm. E) contract itscircular muscles, which extendsthe chaetae into the surroundingsoiland moves the body fluid perpendicular to the long axis of theworm.
63)
All of the following statements about exoskeletons are correct except A) exoskeletons are made of chitin. B) muscles are attached to the exoskeleton. C) exoskeletons donot limit growth because they can stretch. D) exoskeletons are shed by many arthropods in a process called molting. E) chitin is a polysaccharide.
64) You have just read an advertisement for a new drug that claims people who take itcan get muscular bodieswithout spending months lifting weightsin the gym. The drug stimulates muscle growth in the same way that weight lifting does, which is to A) stimulate growth inthe size of existing muscle fibers. B) stimulate existingmuscle fibers to divide. C) stimulate existingmuscle fibers to fuse with one another. D) turn surroundingconnective tissue into muscle tissue. E) stimulate an increase in the proportion of slow-twitch muscle fibers.
65) In order for a muscle to contract, calcium ions released fromthe sarcoplasmic reticulum must bind to
Version 1
20
A) troponin. B) tropomyosin. C) myosin. D) actin. E) ATP.
66) Impressive was a champion Quarter Horse stallion with a magnificent muscular physique, a trait that he passed on to dozens of offspring. However, many of Impressive's offspring developed periodic muscular twitching that sometimes left the horses temporarily unable to move. Some horses even died. As it turns out, Impressive's award-winning physique was due to a mutation in a single, protein-encoding gene, resultingin hyperexcitable muscle (which contract even in theabsence of a stimulus from its motor neuron). Which is not a plausible explanation for the identity of theabnormal muscle protein in Impressive and his offspring? A) Acetylcholine receptors that function in the absence of acetylcholine B) Leaky Na + channels C) Overactive Ca 2+ pumps in the membrane of the sarcoplasmic reticulum D) Tropomyosin that is unable to bind to actin E) Overactive troponin molecules
67) What is the association between tropomyosin and troponin molecules in resting striated muscle fibers? A) Tropomyosin holds the troponin in place on the myosin. B) Troponin holds the tropomyosin in place on the myosin. C) Tropomyosin holds the troponin in place on the actin. D) Troponin holds the tropomyosin in place on the actin. E) There is no known association.
68)
Impulses conducted along the transverse tubules of the muscle fiber stimulate
Version 1
21
A) the release ofATP. B) the binding ofactin to the Z lines. C) tropomyosin and troponin binding. D) the release ofcalcium ions. E) the release ofacetylcholine.
69) A muscle fiber contracts by a sequence of events initiated by the binding of acetylcholine to its receptor. Which of the following is the correct sequence? I - The calcium-troponin complex pulls tropomyosin away from the myosin-binding sites of actin. II - The freed-sites of actin bind to myosin to form cross-bridges. III - Calcium ion binds to the troponin. A) I, III, and II B) III, II, and I C) II, III, and I D) I, II, and III E) III, I, and II
70) A newborn baby dies soon after birth because her diaphragm and rib cage muscles failed to contract and she was unable to breathe on her own. Examination of the baby’s DNA revealed that she had a genetic mutation in a gene that encoded a calcium-binding protein. Which protein was most likely defective in the baby? A) Actin B) Myosin C) Acetylcholine D) Tropomyosin E) Troponin
71) The movement of actin and myosin fibers in a muscle contraction containsall of the following steps except
Version 1
22
A) the myosin headattaches to the actin filament. B) myosin rolls up from one end, forming a ball. C) the myosin filament advances relative to the actin filament. D) the globular myosin head detaches from the actin filament. E) the myosin head returns to its original conformation.
72) Myofibril contractile subunits, the sarcomeres, are bounded on each end by a disc of protein called the A) H band. B) I band. C) A band. D) Z line. E) M line.
73)
A cross bridge is the binding of which two proteins? A) Actin and myosin B) Actin and troponin C) Myosin and troponin D) Collagen and troponin E) Myosin and collagen
74) In bone, crystals of calcium phosphate impregnate the extracellular matrix containing fibers of A) actin. B) chitin. C) collagen. D) myosin. E) troponin.
Version 1
23
75)
The human skeleton is a typical vertebrate A) endoskeleton. B) exoskeleton. C) hydrostaticskeleton. D) axial skeleton only. E) appendicular skeleton only.
76) If you rub the scaled skin of a shark, it feels rough when you move your hand from tail to head but smooth when you move from head to tail. What is a major advantage of the scales being oriented this way? A) To counteract friction as the shark swims through the water B) To keep the shark afloat in the water C) To betterprotect the shark from predators D) To better prevent water loss from the body tissues E) To enable the shark to orient itself in space and detect prey
77) Sarin gas is an inhibitor of an enzyme that functions in the neuromuscular junction to break down acetylcholine. What is the most likely description of how a person would die after inhaling this poisonous gas? A) By a heart attack B) Paralysis of skeletal muscles C) Suffocation via tetanus, or constant stimulation, of the respiratory muscles D) Suffocation via paralysis, or a block in stimulation, of the respiratory muscles E) Epileptic convulsions caused by disorganized electrical activity in the brain
78) The masseter muscles originate on your cheekbones and insert into the posterior corners of the mandible (the lower jaw bone) underneath your ears. Will contraction of the masseter muscles open or close your jaw?
Version 1
24
A) Close B) Open
79) The figure below represents a joint in the hind leg of a grasshopper. The exoskeleton of the leg is indicated by the two pairs of parallel black lines. A muscle that moves this leg is fixed to the skeleton at the end marked with a blue star. When the muscle contracts, it will pull on the tendon at its opposite end. Which configuration shows the most likely arrangement of the exoskeleton and a muscle that increases the flexion of this leg?
A) A B) B C) C D) D
Version 1
25
80)
Based on the above diagram and other information from your text, what is the most likely (e.g., most parsimonious) explanation for the evolution of flight in vertebrates? A) Wings evolved once in vertebrates (e.g., the common ancestor of all terrestrial vertebrates had wings), and wings were subsequently lost in all but three lineages: bats, pterosaurs, and birds. B) The common ancestor of all terrestrial vertebrates was wingless, and wings evolved independently three different times: in bats, in pterosaurs, and in birds. C) The common ancestor of all terrestrial vertebrates was wingless, and wings evolved independently two different times: once in bats and once in the common ancestor of birds, pterosaurs, and dinosaurs. D) The common ancestor of all terrestrial vertebrates was wingless, and wings evolved independently three different times: in bats, in pterosaurs and in the common ancestor of birds and dinosaurs. E) The common ancestor of all terrestrial vertebrates was wingless, and wings evolved independently three different times: in bats, in birds and in the common ancestor of pterosaurs and dinosaurs.
81) Imagine an earthworm that has no chaetae. How would the lack of chaetae affect movement of the earthworm?
Version 1
26
A) The earthworm wouldnot be able to move at all. B) The earthworm wouldonly be able to extend its body segments. C) The earthworm wouldonly be able to shortenits body segments. D) The earthwormwould have difficultychanging locations, since it cannot anchor itself inthe soil. E) The earthworm would be much flatterthan normalsince it has no hydrostatic skeleton.
82) If the skin formed as a simple epithelium during development, what result would you expect? A) This is the normal situation. B) The skin would have increased transfer and improved water and gashomeostasis. C) This would make the skin very fragile and impair its protectivefunction. D) This would mildly decrease its protective function.
83)
How would you expect a disease of cartilage formation to affect bone development?
A) Cartilage is used for other tissues, not bone, so it will not affect bonedevelopment. B) Cartilage defects will affect bone only via the effect on tendons andligaments. C) It would greatly disrupt bone development, since bones are originally formed of cartilage that becomes replaced. D) The epiphysis is formed of cartilage so that will be disrupted, butotherwise the bone will be normal.
Version 1
27
Answer Key Test name: Chap 32_3e 1) FALSE Motor neurons communicate directly with muscle. 2) B
Version 1
28
Clarify Question • What is the key concept addressed by the question? The concept addressed here is the categorization of vertebrate structure into one of four levels of organization. • What type of thinking is required? This question is asking you to identify key factors that qualify something as a cell, tissue, organ, or organ system. Gather Content • What do you already know about biological levels of organization? What other information is related to the question? o Biologists divide the vertebrate body into four levels of organization. o The levels of organization are cells, tissues, organs, and organ systems. o Cells are small units of life that in eukaryotes (including vertebrates) possess a nucleus and organelles. o Tissues are composed of cells with similar structure and function. o Vertebrates have four tissue types: epithelial, connective, muscular, and nervous tissue. o Organs are functional units that are composed of multiple tissue types. o Organ systems are groups of organs that function together. Choose Answer • Given what you now know, what information is most likely to produce the correct answer? o In scenario 1, the doctor is examining organs such as the fingernails, nails, and hair within the larger organ system, which in this case is the integumentary system. This scenario is an examination of an organ system. Version 1
29
o Scenario 2 involves an examination of a possible squamous cell carcinoma, which are tissue-level with the vertebrate organizational scheme. o The doctor is examining Fred’s skin in Scenario 3. Skin is an organ. o In scenario 4, the pathologist is using a microscope to examine cells. Reflect on Process • Did your problem-solving process lead you to the correct answer? If not, where did the process break down or lead you astray? How can you revise your approach to produce a more desirable result? o Answering this question correctly depended not only on distinguishing between the levels of organization in the vertebrate body, but on your ability to break down, or analyze, specific examples of each level. If you got an incorrect answer, did you remember that the skin is an organ, or that the skin, along with the nails and hair, fall into the category of organ system? Did you have trouble breaking down the levels of organization to determine the correct answer? 3) B 4) D 5) C
Version 1
30
Clarify Question • What is the key concept addressed by the question? The concept addressed here is the identification of the tissue that would facilitate a last-minute response. • What type of thinking is required? This question is asking you to identify key factors that qualify something as one of the tissues of the vertebrate body. Gather Content • What do you already know about the tissues of the vertebrate body? What other information is related to the question? o Vertebrates have four tissue types: epithelial, connective, muscular, and nervous tissue. o Epithelial tissue covers or lines. For example, the skin is epithelial is epithelial tissue, as is the lining of the gut and the lining of the blood vessels. o Connective tissue includes blood, fat, bone, and cartilage. o Muscle tissue includes smooth muscle, skeletal muscle, and cardiac muscle. o Nervous tissue includes the brain, spinal cord, and nerves. Choose Answer • Given what you now know, what information is most likely to produce the correct answer? o Ectoderm is a germ layer, not a tissue. o Epithelial tissue is exemplified here by the skin, but the skin is not involved in generating the last-minute idea. o Muscle tissue is also involved here, but it is involved in carrying out the response, not in the initial formulation of the idea. o The tissue that would respond first would be that tissue that originates Version 1
31
the action. The thought would have originated in the brain, which is nervous tissue. Reflect on Process • Did your problem-solving process lead you to the correct answer? If not, where did the process break down or lead you astray? How can you revise your approach to produce a more desirable result? o Answering this question correctly depended not only on distinguishing between germ layers and tissues, but on your ability to break down, or analyze, the individual tissue types. If you got an incorrect answer, did you remember that components of a response include nervous stimulation for the action to be initiated or that the brain is nervous tissue? Did you have trouble breaking down the types of tissue to determine the correct answer? 6) D 7) B
Version 1
32
Clarify Question • What is the key concept addressed by the question? The concept addressed here is the location of the body cavities relative to the organs contained within. • What type of thinking is required? This question is asking you to identify key factors that qualify something as an occupant of one of the recognized cavities of the body. Gather Content • What do you already know about the body cavities? What other information is related to the question? o The vertebrate body contains two main cavities; the dorsal cavity and the ventral cavity. o The dorsal cavity can be divided into the cranial cavity and the vertebral cavity. o The ventral cavity can be divided into the thoracic cavity, which contains the pericardial and pleural cavities, and the abdominopelvic cavity, which contains the abdominal and pelvic cavities. o The pericardial cavity contains the heart. o The pleural cavities contain the lungs. o The abdominopelvic cavity contains the stomach and the organs below the diaphragm. Choose Answer • Given what you now know, what information is most likely to produce the correct answer? o The shot never affected the lungs, so the pleural cavities can be ruled out. o As the shot never affected the brain or spinal cord, the dorsal cavity can be disregarded. Version 1
33
o Since the shot is described as entering the chest, the associated cavity is the thoracic cavity. o The shot is described as going through the diaphragm and into the stomach, which means it entered the abdominopelvic cavity before it left the body. Reflect on Process • Did your problem-solving process lead you to the correct answer? If not, where did the process break down or lead you astray? o Answering this question correctly depended not only on distinguishing between the dorsal and ventral cavities, but on your ability to break down, or analyze, the subdivisions of those cavities. If you got an incorrect answer, did you remember that the dorsal cavity contains the brain and spinal cord or that the thoracic cavity is the chest cavity? Did you have trouble breaking down the body cavities to determine the correct answer? 8) A 9) C 10) B
Version 1
34
Clarify Question • What is the key concept addressed by the question? This question is asking about the consequences of a greater than normal number of goblet cells. • What type of thinking is required? You are being asked to take what you already know and use, or apply, it to the excessive production of goblet cells. Gather Content • What do you already know about goblet cells? What other information is related to the question? o Goblet cells are named in accordance with their goblet-like shape. o Goblet cell locations include mucous membranes. o Goblet cells are known to produce mucus. Choose Answer • Given what you now know, what information is most likely to produce the correct answer? o If a greater than expected number of goblet cells are present, it stands to reason that an excessive amount of their secretion, mucus, would also be present. Reflect on Process • Did your problem-solving process lead you to the correct answer? If not, where did the process break down or lead you astray? How can you revise your approach to produce a more desirable result? o Answering this question correctly depended on your ability to use goblet cells in a new situation. If you got an incorrect answer, did you remember that goblet cells produce mucus? Did you have trouble extending goblet cells to determine the correct answer? Version 1
35
11) A 12) A 13) B 14) D A simple columnar epithelium lines the airways of the respiratory tract and the inside of most of the gastrointestinal tract. Interspersed among the columnar epithelial cells of mucous membranes are numerous goblet cells, which are specialized to secrete mucus. The columnar epithelial cells of the respiratory airways contain cilia on their apical surface, which move mucus and dust particles towardthe throat. 15) B 16) A 17) C 18) D
Version 1
36
Clarify Question • What is the key concept addressed by the question? This question is asking about the locations in which collagen is found in the body of a cow. • What type of thinking is required? You are being asked to take what you already know and use, or apply, it to relating collagen to specific cow parts. Gather Content • What do you already know about collagen? What other information is related to the question? o Collagen is a protein that can be very strong, especially when fibers are concentrated in one location. o Collagen is found in dense connective tissue in the vertebrate body. o Collagen functions as a component of the skin, organ coverings, as the covering nerves, muscle, and bone, and it is found within cartilage. Choose Answer • Given what you now know, what information is most likely to produce the correct answer? o Collagen is not found in abundance within muscle (including heart muscle), glandular organs, and the brain. o Since collagen is found in connective tissue, it would be abundant in tendons and cartilage, which are examples of connective tissue. o The skin also contains collagen, which provides strength to the skin. Reflect on Process • Did your problem-solving process lead you to the correct answer? If Version 1
37
not, where did the process break down or lead you astray? How can you revise your approach to produce a more desirable result? o Answering this question correctly depended on your ability to use collagen in a new situation. If you got an incorrect answer, did you remember that collagen is present in the skin and in a variety of connective tissues? Did you have trouble extending collagen to determine the correct answer? 19) D
Version 1
38
Clarify Question • What is the key concept addressed by the question? The concept addressed here is the identification of a gene product that reinforces tissue. • What type of thinking is required? This question is asking you to weigh and judge, or evaluate, the gene that is most likely to produce a tissue-strengthening product. Gather Content • What do you already know about the strengthening of tissues? What other information is related to the question? o A major strengthening component of a variety of different tissues is collagen. o Collagen is the most common protein in mammals. o In muscle tissue, although collagen is not especially abundant, it is found in the endomysium that surrounds individual muscle cells. Choose Answer • Given what you now know, what information is most likely to produce the correct answer? o Epidermal growth factor does not strengthen tissue; it encourages the growth of tissue. o Actin and myosin serve in the contraction of muscles. While actin is a component of the cytoskeleton, it does not play a role in strengthening tissue. o Collagen is the logical choice here, as it is the most abundant protein in mammals and it is known to strengthen tissue. o This example is similar to the disease known as Ehlers-Danlos Syndrome, which may be caused by the mutation of one of several Version 1
39
collagen genes, or a few related genes. Reflect on Process • Did your problem-solving process lead you to the correct answer? If not, where did the process break down or lead you astray? How can you revise your approach to produce a more desirable result? o Answering this question correctly depended on your ability to weigh and judge, or evaluate, genes and their products as they relate to the strengthening of tissue. If you got an incorrect answer, did you remember that collagen is a protein that strengthens tissue or that collagen is a component of many different tissues? Did you have trouble weighing the merits of collagen to determine the correct answer? 20) E 21) D 22) D 23) B 24) B 25) B 26) B 27) A 28) C 29) E 30) C 31) C 32) D
Version 1
40
Clarify Question • What is the key concept addressed by the question? This question is asking you to identify which specific type of muscle is present based on the characteristics of the muscle. • What type of thinking is required? You are being asked to take what you already know and use, or apply, it to the specific identification of muscle tissue. Gather Content • What do you already know about muscle tissue? What other information is related to the question? o Muscle tissue can be divided into three subdivisions: cardiac, skeletal, and smooth muscle. o Cardiac muscle is striated, involuntary, and tends to have a single nucleus per cell. o Smooth muscle is not striated, it is involuntary, and it tends to have one nucleus per cell. o Skeletal muscle is striated, voluntary, and each cell has more than one nucleus. Choose Answer • Given what you now know, what information is most likely to produce the correct answer? o Involuntary muscle includes cardiac and smooth muscle. o Neither smooth muscle nor cardiac muscle is multinucleate. o Skeletal muscle is the only striated, multinucleate muscle tissue. Reflect on Process
Version 1
41
• Did your problem-solving process lead you to the correct answer? If not, where did the process break down or lead you astray? How can you revise your approach to produce a more desirable result? o Answering this question correctly depended on your ability to use the types of muscle tissue in a new situation. If you got an incorrect answer, did you remember that skeletal muscle is striated, or that skeletal muscle cells are multinucleate? Did you have trouble extending muscle tissue to determine the correct answer? 33) B 34) C 35) C 36) C 37) A 38) C 39) A
Version 1
42
Clarify Question • What is the key concept addressed by the question? This question is asking about the nature of the detectable signaling of the brain. • What type of thinking is required? You are being asked to take what you already know and use, or apply, it to the passage of signals within the brain. Gather Content • What do you already know about neuronal signaling? What other information is related to the question? o Neurons are the basic units of nervous tissue. o A neuron receives messages through its dendrites, passes the message along the cell body, and passes the message along through its single axon. Choose Answer • Given what you now know, what information is most likely to produce the correct answer? o Neuroglia are support cells for neurons; they do not pass messages along. o Myelination insulates the axon; it does not block the passage of a signal. o Since dendrites are the input for a signal, the only correct answer from these choices is that the dendrites were cut, thereby stopping the neuronal signal by severing the inputs to the nerve cell. Reflect on Process
Version 1
43
• Did your problem-solving process lead you to the correct answer? If not, where did the process break down or lead you astray? How can you revise your approach to produce a more desirable result? o Answering this question correctly depended on your ability to use neuronal signaling in a new situation. If you got an incorrect answer, did you remember that dendrites receive neuronal signals or that axons pass signals along? Did you have trouble extending neuronal signaling to determine the correct answer? 40) A 41) A 42) A
Version 1
44
Clarify Question • What is the key concept addressed by the question? This question is asking about the tissue that is impacted by a stroke. • What type of thinking is required? You are being asked to take what you already know and use, or apply, it to the effects of a stroke. Gather Content • What do you already know about olfaction? What other information is related to the question? o Olfaction, or the sense of smell, is a component of the nervous system. o Dendrites in the nasal mucosa receive signals in the form of molecules. o Once a molecule binds with a receptor on a dendrite, a signal is sent to the central nervous system so the molecule can be recognized. Choose Answer • Given what you now know, what information is most likely to produce the correct answer? o Damage to olfactory receptors or to peripheral nerves would likely result in lack of olfaction, not abnormal olfaction. o Since nodes of Ranvier are involved in passing signals along, if they were damaged, it is most likely that olfaction would not occur. o A stroke is damage to the brain caused by loss of blood flow. o The recognition of a scent occurs in the brain, so damage to the brain could alter the perception of smell. Reflect on Process
Version 1
45
• Did your problem-solving process lead you to the correct answer? If not, where did the process break down or lead you astray? How can you revise your approach to produce a more desirable result? o Answering this question correctly depended on your ability to use olfaction in a new situation. If you got an incorrect answer, did you remember that scents are received in the nose but are perceived in the brain? Did you have trouble extending olfaction to determine the correct answer? 43) C
Version 1
46
Clarify Question • What is the key concept addressed by the question? This question is asking about the difference between receiving olfactory stimuli and perceiving them. • What type of thinking is required? Choose from the following samples to streamline your writing: o You are being asked to take what you already know and use, or apply, it to olfaction. Gather Content • What do you already know about olfaction? What other information is related to the question? o Olfaction, or the sense of smell, is a component of the nervous system. o Dendrites in the nasal mucosa receive signals in the form of molecules. o Once a molecule binds with a receptor on a dendrite, a signal is sent to the central nervous system so the molecule can be recognized. Choose Answer • Given what you now know, what information is most likely to produce the correct answer? o The recognition of a scent occurs in the brain, so damage to the central nervous system could alter the perception of smell, but it would not stop the passage of a signal from the nose. o The neuroglia are support cells for neurons, but they do not receive or perceive sensory input. o Damage to olfactory receptors or to peripheral nerves would likely result in lack of olfaction. Version 1
47
Reflect on Process • Did your problem-solving process lead you to the correct answer? If not, where did the process break down or lead you astray? How can you revise your approach to produce a more desirable result? o Answering this question correctly depended on your ability to use olfaction in a new situation. If you got an incorrect answer, did you remember that scents are received in the nose but are perceived in the brain? Did you have trouble extending olfaction to determine the correct answer? 44) D
Version 1
48
Clarify Question • What is the key concept addressed by the question? This question is asking about the integration of body systems. • What type of thinking is required? You are being asked to take what you already know and use, or apply, it the systems that work together to cause movement. Gather Content • What do you already know about movement? What other information is related to the question? o To move, a plan must be formulated in the central nervous system. o Once the plan is formulated, the message must be passed along through nerves to the muscles. o The move is accomplished by the contractions of muscles, which move the bones of the limbs or trunk. Choose Answer • Given what you now know, what information is most likely to produce the correct answer? o While the integumentary, endocrine, and circulatory systems support the health of the body, they do not play a direct role in movement. o The actions that are required to successfully complete a movement are planned in the brain and are carried by the nerves to the muscles, which move components of the skeletal system. Reflect on Process • Did your problem-solving process lead you to the correct answer? If not, where did the process break down or lead you astray? How can you revise your approach to produce a more desirable result? Version 1
49
o Answering this question correctly depended on your ability to use movement in a new situation. If you got an incorrect answer, did you remember that movement is planned in the central nervous system or that the muscles contract to move the skeletal system? Did you have trouble extending movement to determine the correct answer? 45) B 46) B
Version 1
50
Clarify Question • What is the key concept addressed by the question? o The question asks about the structure of the knee and hip. • What type of thinking is required? o You are being asked to apply your knowledge about the structure of the knee and hip. Gather Content • What do you know about the structure of the knee and hip? What other information is related to the question? o The hip has a wide range of motion, able to rotate in three dimensions. In contrast, the knee, just moves forward and backwards. The hip has a few tendons and muscles to stabilize it and hold the ball in the socket. The knee has multiple tendons and muscles that attach to the joint and stabilize it. Choose Answer • Given what you now know, what information is most likely to produce the correct answer? o The knee joint is stabilized by many more structures (many of which have to be cut) than the hip joint. Because of this, it takes longer for a patient to heal from the damage done by a knee replacement than a hip replacement. Reflect on Process • Did your problem-solving process lead you to the correct answer? If not, where did the process break down or lead you astray? How can you revise your approach to produce a more desirable result? Version 1
51
o This question asked you to apply your knowledge about the structure of the knee and hip. If you got the correct answer, great job! If you got an incorrect answer, where did the process break down? Did you think that the hip had less surface area in contact than the knee? Did you think that the hip has a smaller range of motion than the knee? 47) A
Version 1
52
Clarify Question • What is the key concept addressed by the question? o The question asks about the structure of a myofibril. • What type of thinking is required? o You are being asked to apply your knowledge about the structure of a myofibril to identify, which will not change in shape during contraction. Gather Content • What do you know about the structure of a myofibril? What other information is related to the question? o The A band is determined by the length of the thick filaments made up of myosin molecules in a sarcomere: this does not change length during a contraction. The Z lines form the borders of each sarcomere. The thin filaments are within the I bands and extend into the A bands interdigitated with thick filaments. The H band is the lighter-appearing central region of the A band containing only thick filaments. In a contracted muscle, the Z lines have moved closer together, with the I bands and H bands becoming shorter. The A band does not change in size as it contains the thick filaments, which do not change in length. Choose Answer • Given what you now know, what information is most likely to produce the correct answer? o The I band, H band, and sarcomere become shorter during a muscle contraction as the actin and myosin filaments slide past each other. The A band does not change in length as it is defined by the myosin thick filament. Reflect on Process • Did your problem-solving process lead you to the correct answer? If not, where did the process break down or lead you astray? How can you revise your approach to produce a more desirable result? o This question asked you to apply your knowledge about the structure of a myofibril to identify, which will not change in shape during contraction. If you got the correct answer, great job! If you got an incorrect answer, where did the process break down? Did you think that the myosin molecules in the A chain shorten during a contraction? Did you think that actin and myosin sliding across each other would not affect the I band, H band, or sarcomere length?
48) A 49) C
Version 1
53
Clarify Question • What is the key concept addressed by the question? o The question asks about the role of ATP hydrolysis in myofibril function. • What type of thinking is required? o You are being asked to analyze statements to explain the role of ATP hydrolysis in myofibril function. Gather Content • What do you know about the role of ATP hydrolysis in myofibril function? What other information is related to the question? o The cross-bridge cycle involves hydrolysis of ATP by myosin causing a conformational change that moves the head into an energized state. The ADP and Pi remain bound to the myosin head, which can bind to actin. In the second step, myosin binds to actin, forming a cross-bridge. In the third step, the power stroke, myosin returns to its original conformation, releasing ADP and Pi. In the last step, ATP binds to the myosin head breaking the cross-bridge. ATP hydrolysis returns the myosin head to its energized conformation, allowing the cycle to begin again. Choose Answer • Given what you now know, what information is most likely to produce the correct answer? o Without ATP hydrolysis, myosin heads will be unable to bind to the actin filaments. Reflect on Process
Version 1
54
• Did your problem-solving process lead you to the correct answer? If not, where did the process break down or lead you astray? How can you revise your approach to produce a more desirable result? o This question asked you to analyze statements to explain the role of ATP hydrolysis in myofibril function. If you got the correct answer, great job! If you got an incorrect answer, where did the process break down? Did you think that ATP hydrolysis was necessary for breaking the bridge between actin and myosin? Did you think that ATP hydrolysis was necessary for the power stroke? 50) A
Version 1
55
Clarify Question • What is the key concept addressed by the question? o The question asks about the correlation between sarcomere length and tension. • What type of thinking is required? o You are being asked to analyze statements about the correlation between sarcomere length and tension. Gather Content • What do you know about the correlation between sarcomere length and tension? What other information is related to the question? o As the sarcomere contracts, the free ends of the thin filaments approach each other as seen in segment I. Once the ends reach each other, any further contraction in sarcomere length does not increase tension (section II). As the thin filaments overlap each other, fewer actin and myosin bridges are formed and the tension of the sarcomere decreases (segment III). Choose Answer • Given what you now know, what information is most likely to produce the correct answer? o There is an increasing overlap of the free ends of the thin filaments in segment III but not in segment II. Reflect on Process • Did your problem-solving process lead you to the correct answer? If not, where did the process break down or lead you astray? How can you revise your approach to produce a more desirable result? o This question asked you to analyze statements about the correlation between sarcomere length and tension. If you got the correct answer, great job! If you got an incorrect answer, where did the process break down? Did you think that the distance between Z lines in segment III are longer than in segment II? Did you think that the length of the thin filaments change during a contraction?
51) A 52) D 53) B 54) D 55) C Version 1
56
Clarify Question • What is the key concept addressed by the question? o The question asks about the effects of stress on bone strength. • What type of thinking is required? o You are being asked to apply your knowledge about the effects of stress on bone strength. Gather Content • What do you know about the effects of stress on bone strength? What other information is related to the question? o Unlike chitin, both cartilage and bone are living tissues. Bone, particularly, can have high metabolic activity especially if bone cells are present throughout the matrix, a common condition. Bone, and to some extent cartilage, can change and remodel itself in response to injury or to physical stresses. Under a mild compressive load, the bone does not bend. If the load is large enough, and the bone is not sufficiently thick, the bone will bend. Osteoblasts are signaled by the stresses in the bending section to produce additional bone. As the bone becomes thicker, the degree of bending is reduced. Choose Answer • Given what you now know, what information is most likely to produce the correct answer? o As a bone is stressed, more bone is made through remodeling, strengthening the bone. Reflect on Process • Did your problem-solving process lead you to the correct answer? Version 1
57
If not, where did the process break down or lead you astray? How can you revise your approach to produce a more desirable result? o This question asked you to apply your knowledge about the effects of stress on bone strength. If you got the correct answer, great job! If you got an incorrect answer, where did the process break down? Did you think that the bone marrow in the center of the bone made it stronger? Did you think that cartilage that connects bones makes them stronger? 56) E 57) B 58) B 59) A 60) D 61) E 62) A 63) C 64) A
Version 1
58
Clarify Question • What is the key concept addressed by the question? o The question asks about how weight lifting increases muscle strength. • What type of thinking is required? o You are being asked to apply your knowledge about how weight lifting increases muscle strength. Gather Content • What do you know about how weight lifting increases muscle strength? What other information is related to the question? o Endurance training does not increase muscle size and increases the proportion of slow twitch muscle fibers. Muscle enlargement is produced only by frequent periods of high-intensity exercise in which muscles work against high resistance, as in weight lifting. Resistance training increases the thickness of fast-twitch muscle fibers, causing skeletal muscles to grow by hypertrophy (increased cell size) rather than by cell division and an increased number of cells. Choose Answer • Given what you now know, what information is most likely to produce the correct answer? o Weight lifting, stimulates the growth in the size of existing muscle fibers, not the number of muscle fibers. So if the drug works like weight lifting the size of existing muscle fibers will increase. The number of cells and proportion of slow-twitch muscle fibers will not increase. Reflect on Process • Did your problem-solving process lead you to the correct answer? Version 1
59
If not, where did the process break down or lead you astray? How can you revise your approach to produce a more desirable result? o This question asked you to apply your knowledge about how weight lifting increases muscle strength. If you got the correct answer, great job! If you got an incorrect answer, where did the process break down? Did you think that the number of muscle fibers would increase? Did you think that the proportion of slow-twitch muscle fibers would increase? 65) A 66) C
Version 1
60
Clarify Question • What is the key concept addressed by the question? o The question asks about excitation of muscles. • What type of thinking is required? o You are being asked to analyze statements about what could cause hyperexcitable muscles. Gather Content • What do you know about excitation of muscles? What other information is related to the question? o When a nerve signal reaches a muscle, the neurotransmitter acetylcholine is released at a neuromuscular junction and then binds to acetylcholine receptors that, in turn, open Na+ channels. This causes the muscle cell membrane to depolarize. This depolarization is conducted along the muscle cell membrane and down the transverse tubules to stimulate the release of Ca2+ from the sarcoplasmic reticulum. Ca2+ diffuses through the cytoplasm to myofibrils, causing contraction. When the muscle is at rest, a long filament of the protein tropomyosin blocks the myosin-binding sites on the actin molecule. Because myosin is unable to form cross-bridges with actin at these sites, muscle contraction cannot occur. When Ca2+ binds to another protein, troponin, the Ca2+– troponin complex displaces tropomyosin and exposes the myosinbinding sites on actin, permitting cross-bridges to form and contraction to occur. o When impulses from the motor neuron cease, it stops releasing acetylcholine, in turn stopping the production of impulses in the muscle fiber. Another membrane protein in the sarcoplasmic reticulum then uses energy from ATP hydrolysis to pump Ca2+ back into the sarcoplasmic reticulum by active transport. Troponin is no longer bound to Ca2+, so Version 1
61
tropomyosin returns to its inhibitory position, allowing the muscle to relax. Choose Answer • Given what you now know, what information is most likely to produce the correct answer? o A hyperexcitable muscle would be excited even in the absence of a stimulus. So if Na+ ions can cross the muscle cell membrane through a leaky Na+ channel, or if acetylcholine receptors can function in the absence of acetylcholine, this could allow the muscle to be excited in the absence of a signal. Also inactive tropomyosin or overactive troponin could lead to hyperexcitability.This is because tropomyosin binds to actin preventing myosin binding in the absence of Ca2+. Alternatively when Ca2+ is present, troponin displaces tropomyosin allowing actin and myosin to bind and contraction to occur. o The correct answer is that overactive Ca2+ pumps in the membrane of the sarcoplasmic reticulum would not lead to hyperexcitability as these pumps would remove Ca2+ from the muscle cell cytoplasm and decrease excitability. So an overactive Ca2+ pump would decrease muscle contractions. Reflect on Process • Did your problem-solving process lead you to the correct answer? If not, where did the process break down or lead you astray? How can you revise your approach to produce a more desirable result? o This question asked you to analyze statements about what could cause hyperexcitable muscles. If you got the correct answer, great job! If you got an incorrect answer, where did the process break down? Did you think that blocking the troponin/tropomyosin reaction would lead to Version 1
62
hyperexcitability? Did you think that decreased Na+ movement across the membrane would lead to hyperexcitability? 67) D 68) D 69) E 70) E
Version 1
63
Clarify Question • What is the key concept addressed by the question? o The question asks about the proteins the bind to calcium in a muscle contraction. • What type of thinking is required? o You are being asked to apply your knowledge about that proteins the bind to calcium in a muscle contraction. Gather Content • What do you know about the proteins the bind to calcium in a muscle contraction? What other information is related to the question? o When a nerve signal reaches a muscle, the neurotransmitter acetylcholine is released at a neuromuscular junction and then binds to acetylcholine receptors that, in turn, open Na+ channels. This causes the muscle cell membrane to depolarize. This depolarization is conducted along the muscle cell membrane and down the transverse tubules to stimulate the release of Ca2+ from the sarcoplasmic reticulum. Ca2+ diffuses through the cytoplasm to myofibrils, causing contraction. When the muscle is at rest, a long filament of the protein tropomyosin blocks the myosin-binding sites on the actin molecule. Because myosin is unable to form cross-bridges with actin at these sites, muscle contraction cannot occur. When Ca2+ binds to another protein, troponin, the Ca2+– troponin complex displaces tropomyosin and exposes the myosinbinding sites on actin, permitting cross-bridges to form and contraction to occur. Choose Answer • Given what you now know, what information is most likely to Version 1
64
produce the correct answer? o When troponin binds to Ca2+, it can displace tropomyosin, allowing actin and myosin to bind and form cross bridges. Actin, myosin, tropomyosin, and acetylcholine do not bind calcium so the mutation cannot be in these proteins. Reflect on Process • Did your problem-solving process lead you to the correct answer? If not, where did the process break down or lead you astray? How can you revise your approach to produce a more desirable result? o This question asked you to apply your knowledge about the proteins that bind to calcium in a muscle contraction. If you got the correct answer, great job! If you got an incorrect answer, where did the process break down? Did you think that actin, myosin, tropomyosin, or acetylcholine bind calcium? Did you think that a lack of troponin binding to calcium would not block muscle contraction? 71) B 72) D 73) A 74) C 75) A 76) A
Version 1
65
Clarify Question • What is the key concept addressed by the question? o The question asks about the role of friction in movement. • What type of thinking is required? o You are being asked to apply your knowledge about the role of friction in movement to explain the orientation of scales on a shark. Gather Content • What do you know about the role of friction in movement? What other information is related to the question? o For swimming animals, the buoyancy of water reduces the effect of gravity. As a result, the primary force retarding forward movement is frictional drag, so body shape is important in reducing the force needed to push through the water. In addition to shape, the orientation of scales is important. If the scales overlap like shingles on a roof from the head to the tail, water will move over them smoothly, reducing friction. Water moving back from tail to head would encounter a rough surface, producing more friction that would slow down the shark. Choose Answer • Given what you now know, what information is most likely to produce the correct answer? o The best answer is that the scales feel smooth from head to tail on the shark as that is the direction water will flow as the shark swims. This reduces friction and makes movement through the water more efficient. The orientation of the scales will not affect the rate of water loss or buoyancy. While the skin of some sharks can be used to detect prey and provide defense, there is no evidence in this question that the orientation of the scales will make a difference. Version 1
66
Reflect on Process • Did your problem-solving process lead you to the correct answer? If not, where did the process break down or lead you astray? How can you revise your approach to produce a more desirable result? o This question asked you to apply your knowledge about the role of friction in movement to explain the orientation of scales on a shark. If you got the correct answer, great job! If you got an incorrect answer, where did the process break down? Did you think that buoyancy or water loss would be affected by the orientation of the shark’s scales? Did you think that the orientation of the shark’s scales would affect detection of prey or protection from predators? 77) C
Version 1
67
Clarify Question • What is the key concept addressed by the question? o The question asks about the effect of inhibition of acetylcholinesterase. • What type of thinking is required? o You are being asked to analyze statements about the effect of inhibition of acetylcholinesterase to predict how sarin gas would kill a person. Gather Content • What do you know about the effect of inhibition of acetylcholinesterase? What other information is related to the question? o The motor neuron releases the neurotransmitter acetylcholine into the neuromuscular junction. Acetylcholine binds to receptors in the muscle cell membrane, to open Na+ channels. The influx of Na+ ions depolarizes the muscle cell membrane causing the muscle to contract. For the muscle to relax, the enzyme acetylcholinesterase breaks down the acetylcholine, allowing the muscle cell to repolarize. If a signal cannot reach a muscle and cause contraction, this is called paralysis. If a muscle remains contracted and cannot relax, this is called tetanus. Choose Answer • Given what you now know, what information is most likely to produce the correct answer? o Sarin gas, inhibits acetylcholinesterase, which means that acetylcholine cannot be broken down. After a nerve stimulates a muscle, the muscle will contract and will not be able to relax again. This is a state called tetanus. If this happens in the respiratory muscles after Version 1
68
inhaling sarin gas then the patient will no longer be able to breathe and will suffocate. Reflect on Process • Did your problem-solving process lead you to the correct answer? If not, where did the process break down or lead you astray? How can you revise your approach to produce a more desirable result? o This question asked you to analyze statements about the effect of inhibition of acetylcholinesterase to predict how sarin gas would kill a person. If you got the correct answer, great job! If you got an incorrect answer, where did the process break down? Did you think that the muscle cells would not be able to contract if exposed to sarin gas, causing paralysis? Did you think that sarin gas would affect the nervous system instead of muscle contractions? 78) A
Version 1
69
Clarify Question • What is the key concept addressed by the question? o The question asks about how muscles move a hinge joint like the jaw. • What type of thinking is required? o You are being asked to apply your knowledge about how muscles move a hinge joint like the jaw. Gather Content • What do you know about how muscles move a hinge joint like the jaw? What other information is related to the question? o Skeletal muscles produce movement of the skeleton when they contract. Usually, the two ends of a skeletal muscle are attached to different bones, although some may be attached to other structures, such as skin. The masseter is connected to two bones that are joined together by a hinge joint, the cheek bone and jaw bone. When the masseter contracts, it generates force by pulling the two bones closer together. This causes bending to occur at the joint, thereby closing the jaw. Choose Answer • Given what you now know, what information is most likely to produce the correct answer? o When the masseter contracts, the jaw will close. You can feel the masseter contract when you close your jaw and clench your molars. Reflect on Process • Did your problem-solving process lead you to the correct answer? If not, where did the process break down or lead you astray? How can you revise your approach to produce a more desirable result? Version 1
70
o This question asked you to apply your knowledge about how muscles move a hinge joint like the jaw. If you got the correct answer, great job! If you got an incorrect answer, where did the process break down? Did you think that when muscles contract they expand? Did you think that the muscles were oriented so they would push the two bones apart? 79) C
Version 1
71
Clarify Question • What is the key concept addressed by the question? o The question asks about the orientation of a muscle in a joint. • What type of thinking is required? o You are being asked to analyze diagrams of a joint to predict how the muscles will be oriented. Gather Content • What do you know about the orientation of a muscle in a joint? What other information is related to the question? o Exoskeletal muscles produce movement of the exoskeleton when they contract. In the diagram, the exoskeleton of the leg is indicated by the two pairs of parallel black lines. The muscle that moves this leg is fixed to the skeleton at the end marked with a blue star with tendon at its opposite end. For the leg to flex or bend when the muscle contracts, it will pull the two ends of the exoskeleton closer to each other as shown in A and B. The muscle is inside of the grasshopper’s exoskeleton as shown in A and C, just like your muscles are under your skin. Choose Answer • Given what you now know, what information is most likely to produce the correct answer? o The best answer is C because it has the muscles inside of the exoskeleton and the connections to the joint are oriented so that when the muscle contracts the two ends of the exoskeleton will be pulled toward each other causing the joint to flex. Reflect on Process • Did your problem-solving process lead you to the correct answer? If not, where did the process break down or lead you astray? How can you revise your approach to produce a more desirable result? o This question asked you to analyze diagrams of a joint to predict how the muscles will be oriented. If you got the correct answer, great job! If you got an incorrect answer, where did the process break down? Did you think that the muscles could be outside of the exoskeleton? Did you think that the muscles would extend to push the joint causing it to flex?
80) B
Version 1
72
Clarify Question • What is the key concept addressed by the question? o The question asks about the evolution of flight in vertebrates. • What type of thinking is required? o You are being asked to evaluate statements about the evolution of flight in vertebrates. Gather Content • What do you know about the evolution of flight in vertebrates? What other information is related to the question? o The evolution of flight is a classic example of convergent evolution, having occurred independently four times, once in insects and three times among vertebrates. All three vertebrate fliers modified the forelimb into a wing structure, but they did so in different ways, illustrating how natural selection can sometimes build similar structures through different evolutionary pathways. Based on the evolutionary tree, you can see that the vertebrates with and without wings are mixed, showing that the common ancestor to vertebrates did not have wings and that wings arose three different times in bats, in pterosaurs and in birds. Choose Answer • Given what you now know, what information is most likely to produce the correct answer? o The common ancestor of all terrestrial vertebrates was wingless, and wings evolved independently three different times: in bats, in pterosaurs, and in birds. If the common ancestor to dinosaurs and either pterosaurs or birds had wings that were lost in dinosaurs, then the fossils of those species would have evidence of wings. This has not been seen. Reflect on Process • Did your problem-solving process lead you to the correct answer? If not, where did the process break down or lead you astray? How can you revise your approach to produce a more desirable result? o This question asked you to evaluate statements about the evolution of flight in vertebrates. If you got the correct answer, great job! If you got an incorrect answer, where did the process break down? Did you think that the common ancestor to dinosaurs and either pterosaurs or birds had wings and then it was lost in dinosaurs? Did you think that the common ancestor to all vertebrates had wings?
81) D
Version 1
73
Clarify Question • What is the key concept addressed by the question? o The question asks about the function of chaetae. • What type of thinking is required? o You are being asked to apply your knowledge about the function of chaetae. Gather Content • What do you know about the function of chaetae? What other information is related to the question? o On the underside of a worm’s body are short, bristlelike structures called chaetae. When circular muscles act, the chaetae of that region are pulled up close to the body and lose contact with the ground. Circularmuscle activity is passed backward, segment by segment, to create a backward wave of contraction. As this wave continues, the anterior circular muscles now relax, and the longitudinal muscles take over, thickening the front end of the worm and allowing the chaetae to protrude and regain contact with the ground. The chaetae now prevent that body section from slipping backward. This locomotion process involves waves of circular-muscle contractionfollowed by waves of longitudinal-muscle effects. Choose Answer • Given what you now know, what information is most likely to produce the correct answer? o Without chaetae, the earthworm would have difficulty changing locations, since it cannot anchor itself in the soil. The chaetae are on the outside of the earthworm, while the muscles that extend and shorten body segments are inside of the earthworm. Version 1
74
Reflect on Process • Did your problem-solving process lead you to the correct answer? If not, where did the process break down or lead you astray? How can you revise your approach to produce a more desirable result? o This question asked you to apply your knowledge about the function of chaetae. If you got the correct answer, great job! If you got an incorrect answer, where did the process break down? Did you think that the chaetae controlled muscle contractions? Did you think that chaetae regulated the skeleton? 82) C
Version 1
75
Clarify Question • What is the key concept addressed by the question? o The question is asking about skin formation and simple epithelium cells. • What type of thinking is required? o You are being asked to take what you already know and use, or apply, it to determine what would happen if simple epithelium was the origin of skin. Gather Content •What do you already know about skin formation? What other information is related to the question? o Skin is normally stratified epithelium. Stratified epithelial membranes are two to several layers thick, which is part of the protective feature of skin. o Simple epithelium is one cell thick. These cells are thin to allow the passage of molecules rapidly (an example is the diffusion of gases). Choose Answer • Given what you now know, what information is most likely to produce the correct answer? o A very thin layer would not protect the organism very well so the answer that makes the most sense is that skin would be fragile and not be as protective. Reflect on Process • Did your problem-solving process lead you to the correct answer? If not, where did the process break down or lead you astray? How can Version 1
76
you revise your approach to produce a more desirable result? o Answering this question correctly depended on your ability to apply epithelial cells types in a new situation. If you got an incorrect answer, did you remember that simple epithelial is only one layer of cells thick? 83) C
Version 1
77
Clarify Question • What is the key concept addressed by the question? o The question is asking about bone formation and cartilage. • What type of thinking is required? o You are being asked to take what you already know and use, or apply, it to determine what would happen if cartilage formation was adversely affected by a disease. Gather Content •What do you already know about bone formation? What other information is related to the question? o Cartilage is a specialized connective tissue in which the ground substance forms from a characteristic type of glycoprotein, called chondroitin, and collagen fibers laid down along lines of stress in long, parallel arrays. o In the course of fetal development, the bones of vertebrate fins, arms, and legs, among other appendages, are first “modeled” in cartilage. The cartilage matrix then calcifies at particular locations, so that the chondrocytes are no longer able to obtain oxygen and nutrients by diffusion through the matrix. Living bone replaces the dying and degenerating cartilage. Choose Answer • Given what you now know, what information is most likely to produce the correct answer? o Bone originates from cartilage so it would greatly affect the formation of bones. o Cartilage is found in the nose, ears, etc. but it still does affect bone Version 1
78
formation. Reflect on Process • Did your problem-solving process lead you to the correct answer? If not, where did the process break down or lead you astray? How can you revise your approach to produce a more desirable result? o Answering this question correctly depended on your ability to apply the importance of cartilage in bone formation in a new situation. If you got an incorrect answer, did you remember that cartilage is a precursor to bone formation?
Version 1
79
CHAPTER 33 TRUE/FALSE - Write 'T' if the statement is true and 'F' if the statement is false. 1) When a neuron is not firing, the sodium-potassium pump is inactive. ⊚ true ⊚ false
2)
There is only a single type of pain receptor. ⊚ ⊚
3) CO
true false
When the respiratory rate is low, the concentration of plasma CO 2 increases. Increased 2 is converted to carbonic acid, which is associated with a fall in the blood pH. ⊚ ⊚
true false
CHECK ALL THE APPLY. Choose all options that best completes the statement or answers the question. 4) "Synapses are connections between the axon of one neuron and the dendrite of another neuron, where neurotransmitters transmit a chemical signal." How would you modify this statement to make it more complete? (Check all that apply.) A) The statementneeds no modification. B) Synapses can also form between an axonand a cell soma. C) Neuromuscularsynapses form between the axon of a neuron and a muscle. D) Synapses oftenform among the parts of a single cell. E) Synapses can beelectrical as well as chemical.
5) Choose the two primary factors that contribute to the resting membrane potential of a neuron.
Version 1
1
A) K + leak channels make the cell membrane more permeable to K +. B) The sodium-potassium pump exports 3 K+ from the cell for every 2 Na+ it imports. C) Na +leak channels make the cell membrane more permeable to Na +. D) K +leak channels cause a higher concentration of K + outside thecell. E) The sodium-potassium pump exports 3 Na+ from the cell for every 2 K+ it imports.
6) When an action potential begins and sodium channels open, why does sodium rush into the cell? (Check all that apply.) A) The inside of thecell is negatively charged. B) The inside of thecell is positively charged. C) Chloride ions arewaiting to bind to the sodium inside the cell. D) The diffusion gradient drives sodiuminto the cell. E) Channels useenergy to move ions across the membrane whenever they are open.
7)
How do somatic neurons and autonomic neurons differ? (Check all that apply.)
A) Somatic neuronsinnervate the body surface, while autonomic neurons innervate internalstructures. B) Somatic neuronscontrol skeletal muscle, while autonomic neurons control smooth and cardiacmuscle. C) Somatic neuronscontrol exocrine glands, while autonomic neurons do not. D) Somatic neurons control their effectors directly, while autonomic neurons use a series of neurons. E) Somatic neurons are always excitatory,while autonomic neurons may be either excitatory or inhibitory.
8)
Why are mammalian reflexes so fast? (Check all that apply.)
Version 1
2
A) They use neurons with extra wide axons. B) They complete thesensory input-to-motor output circuit in only 2-3 neurons. C) They do not sendsignals to the brain and back. D) They activate alarge number of sensory neurons, which speeds things up afterthe motorneuron performs spatial summation. E) There was a selective advantage toevolve rapid responses to certain types of stimuli.
9)
Which of the following are tactile receptors? (Check all that apply.) A) Merkle cell B) Node of Ranvier C) Meissner corpuscle D) Ruffini corpuscle E) Rod cell F) Pacinian corpuscle G) Free nerve ending H) Ganglion cell I) Cone cell
10)
How is the lateral line of a fish similar to the ears of a human? (Check all that apply.) A) Cilia embedded in a gelatinous matrix B) Cochlea C) Hair cells D) Large external structures for capturing pressure waves E) Line of cells extends along length of the body F) Organ of Corti G) Pressure waves H) Sensory neurons I) Several kinocilia and one stereocilium J) Several stereocilia and one kinocilium
Version 1
3
MULTIPLE CHOICE - Choose the one alternative that best completes the statement or answers the question. 11) In vertebrates, the __________ nervous system is composed of the brain and the spinal cord. A) peripheral B) central C) somatic D) autonomic E) sympathetic
12)
What are the branched extensions of a neuron that receive signals called? A) Axons B) Cell bodies C) Dendrites D) Oligodendrocytes E) Synapses
13)
What is the elongated extension of a neuron that nerve impulses travel along? A) Axon B) Cell body C) Dendrite D) Oligodendrocyte E) Synapse
14)
What are examples of cells that support neurons both structurally and functionally?
Version 1
4
A) Dendrites and nodes of Ranvier B) Neuroglia and nodes of Ranvier C) Schwann cells and dendrites D) Dendrites, nodes of Ranvier, and neuroglia E) Schwann cells, oligodendrocytes, and other neuroglia
15) Rapid inward diffusion of Na What is this event called?
+
produces a dramatic change in membrane potential.
A) Hyperpolarization B) Excitatory postsynaptic potential C) Saltatory conduction D) Monosynaptic reflex E) Depolarization
16)
Nerve impulses are electrical signals produced by which structure? A) Lipid sheath of Schwann cell B) Plasma membrane of neuron C) Synapse D) Neuromuscular junction
17)
The membrane of a resting neuron is most permeable to which of the following ions? A) Cl – B) Ca 2+ C) Na + D) K + E) H +
Version 1
5
18) When neurons are not producing electrical signals, there is still a voltage difference across their membranes. What is this voltage called? A) Channel-gate bias B) Electrolyte voltage C) Equilibrium potential D) Resting potential E) Ion current
19) Which of the following characteristics are true for an action potential? I A threshold potential must be exceeded for an action potential to occur. II A stimulus either produces a full action potential or none at all. III During the refractory period it is less likely that stimuli can produce another action potential. A) I only B) I and II C) II and III D) I and III E) I, II, and III
20)
What is the process in which impulses jump from node to node? A) Facilitated transmission B) Refractory action C) Incomplete transfer D) Repolarization E) Saltatory conduction
Version 1
6
21) Which diagram correctly illustrates the distribution of ions in a resting neuron? (Note: A larger symbol indicates a higher concentration of that ion.)
A) A B) B C) C D) D
22) Which figure correctly illustrates the mechanism of action of the sodium-potassium pump?
Version 1
7
A) A B) B C) C D) D
23) What determines the direction of the voltage change that occurs at the postsynaptic membrane? A) The type of neurotransmitter B) The type of second messenger system used C) The type(s) of ions that travel through opened channels D) The number of ion channels on the postsynaptic membrane
24) At myelinated areas of the axon (i.e. between nodes of Ranvier), which of the following statements is true? A) There isanincreased number of voltage-gated sodium channels. B) There is increasedresistance across the membrane. C) There is increasedcapacitance across the membrane. D) There is increasedtransport of potassium across the membrane.
25)
Which ion channel is primarily responsible for the action potential? A) The voltage-gated sodium channel B) The voltage-gated potassium channel C) The ligand-gated sodium channel D) The ligand-gated potassium channel E) The non-gated sodium channel
26) What diffuses across the narrow synaptic cleft between the presynaptic axon and the postsynaptic cell to transmit a nerve impulse? Version 1
8
A) Neurotransmitter B) Electrical impulse C) Na + ion D) Vesicle E) Action potential
27)
Which neurotransmitter is released from a motor neuron at the neuromuscular junction? A) Acetylcholine B) GABA C) Ephinephrine D) Dopamine E) Serotonin
28) When acetylcholine stimulates the opening of ligand-gated ion channels on a postsynaptic cell, what is this event called? A) Neuromuscular junction B) Temporal summation C) Inhibitory postsynaptic potential (IPSP) D) Excitatory postsynaptic potential (EPSP) E) Synaptic integration
29)
What is the name for the gap into which neurotransmitters are released? A) Gap junction B) Synaptic cleft C) Postsynaptic membrane D) Presynaptic membrane E) Impulse channel
Version 1
9
30) Place the following events that occur at the vertebrate neuromuscular junction in the correct order. I Depolarization of the muscle fiber membrane. II Generation of an action potential in the neuron. III Stimulation of Ca 2+ entry into the neuron. IV Release of acetylcholine into the synapse. A) I, II, III, IV B) III, II, IV, I C) I, III, IV, II D) IV, II,III, I E) II, III, IV, I
31)
Which of the following are examples of biogenic amines? A) Epinephrine and endorphins B) GABA and serotonin C) Epinephrine, dopamine, and GABA D) Epinephrine, dopamine, and serotonin E) Epinephrine, norepinephrine, dopamine, and serotonin
32) The spinal cord is enclosed by the vertebral column and layers of protective membranes. What are these membranes called? A) Neural net B) White matter C) Dorsal root D) Meninges E) Viscera
33) Damage to dopamine-producing neurons can produce the muscle tremors characteristic of what disease?
Version 1
10
A) Epilepsy B) Parkinson disease C) Seizures D) Alzheimer disease E) Insomnia
34) In early vertebrates, which component of the brain was devoted largely to coordinating motor reflexes? A) Thalamus B) Forebrain C) Midbrain D) Hindbrain E) Medulla oblongata
35) Where did information processing become increasingly centered in terrestrial vertebrates? A) Thalamus B) Forebrain C) Midbrain D) Hindbrain E) Medulla oblongata
36)
Which region of the CNS listed below is not correctly paired with its function? A) Somatosensory cortex—receives sensory input from all parts of the body B) Hypothalamus—regulates body temperature, appetite, thirst, and emotions C) Cerebellum—relaystation for ascending and descending tracts D) Corpus callosum—connects the two brain hemispheres E) Auditory cortex—interprets sound information from the ears
Version 1
11
37)
Which major regions compose the contemporary vertebrate brain? A) Rhombencephalon and prosencephalon only B) Mesencephalon and rhombencephalon only C) Prosencephalon and metencephalon only D) Metencephalon, rhombencephalon, and prosencephalon E) Rhombencephalon, mesencephalon, and prosencephalon
38)
Which of the following structures is not a component of the hindbrain? A) Pons B) Medulla oblongata C) Hypothalamus D) Cerebellum
39)
What is the primary function of the hindbrain in vertebrates? A) Temperature control B) Integration of sensory data C) Memory storage D) Control of the heartbeat E) Coordination of motor reflexes
40)
Where do correlation, association, and learning occur in the brain? A) Cerebellum B) Corpus callosum C) Medulla D) Cerebrum E) Pons
Version 1
12
41)
What is thesite of primary sensory integration in the brain? A) Medulla oblongata B) Cerebrum C) Cerebellum D) Thalamus E) Corpus callosum
42)
What region of the brain integrates visceral activities, body temperature, and heartbeat? A) Medulla oblongata B) Cerebrum C) Hypothalamus D) Thalamus E) Corpus callosum
43) Which division of the nervous system regulates the activity of smooth muscle, cardiac muscle, and glands? A) Antagonistic nervous system B) Autonomic nervous system C) Sympathetic nervous system D) Parasympathetic nervous system E) Limbic nervous system
44)
Which of the following is not part of the peripheral nervous system?
Version 1
13
A) Sympathetic division of the autonomic nervous system B) Spinal cord C) Parasympathetic division of the autonomic nervous system D) Somatic nervous system E) Sensory pathways
45)
Which of the following targets is not regulated by the autonomic nervous system? A) Skeletal muscles B) Cardiac muscles C) Glands of the body D) Smooth muscles E) None of these
46) For your birthday, you get a robot-building kit with three parts: (1) a set of gadgets for detecting sound, touch, and light, (2) a set of motors that can move mechanical parts, and (3) a microprocessor that can be programmed to respond to inputs according to predetermined rules. You build a robot car, and find it surprisingly lifelike as it navigates around the room. The three parts of the kit are analogous to which parts of the human body? A) (1) autonomic nervous system, (2) somatic nervous system, (3) hypothalamus B) (1) sympathetic pathways, (2) parasympathetic pathways, (3) CNS C) (1) sensory pathways, (2) motor pathways and muscles, (3) CNS D) (1) motor pathways and muscles, (2) sensory pathways, (3) CNS
47) Dr. Robinson tapped on a patient's knee with a reflexhammer, but failed to see the usual kick of the patellar reflex. Since this reflex only involves a few neurons, what part of the nervous system might the doctor most likely be concerned about?
Version 1
14
A) Parasympathetic nervous system B) Central nervous system C) Sympathetic nervous system D) Peripheral nervous system
48) Your roommate is on an ultralow fat diet to try to lose weight. Given what you know about structure of the nervous system, what advice would you give him? A) "You shoulddrink large quantities of water, because the brain is mostly water." B) "Good foryou! Losing weight by any means necessary can only help your brain." C) "You shouldeat some healthy unsaturated fats, like those found in fish, olive oil, andnuts, because your brain needs fatty acids to build the myelin sheaths aroundyour axons." D) "You shouldeat trans fats, from donuts, because 'trans' is short forneurotransmission."
49) What branched structure allows a neuron to take in information from more than one source? A) Node of Ranvier B) Axon C) Myelin D) Dendrite
50) What part of the neuron is responsible for most of the "house-keeping" tasks of the cell, including protein synthesis? A) Myelin B) Axon C) Dendrite D) Soma E) Synapse
Version 1
15
51) To process information, postsynaptic neurons often add up simultaneous signals from several presynaptic neurons. Which structure and process allow them to do so? A) Axons; temporal summation B) Axons; spatial summation C) Dendrites; temporal summation D) Dendrites; spatial summation
52) A neuron that engages in spatial summation acquires a mutation that causes dendrite overgrowth. What is the likely outcome?
A) Decreased number of synapses B) Summation will not occur C) Increased number of synapses D) Action potentials with stronger depolarization
53)
What important resource do Schwann cells and oligodendrocytes provide to neurons? A) Oxygen B) Nutrients C) Physical strength from microtubules D) Ca 2+ currents E) Insulating layers of membrane
54) A patient suffers from a disease that slows fatty acid synthesis. What is a likely impact of this disease on nervous system function? A) Improved functioning due to a healthier weight B) Insufficient myelination and slow axon conduction rate C) Improved functioning due to better blood flow D) Weaker action potentials with smaller depolarization
Version 1
16
55) Is the resting membrane potential of a neuron closer to the equilibrium potential for K + or Na ?
+
A) K + B) Na + C) It is an averageof the two.
56) Three-quarters of the substantial amount of energy consumed by the brain is used to run a single molecular machine that maintains the resting potential of neurons. What is this machine called? A) K +leak channel B) Sodium-potassium pump C) Acetylcholine receptor D) Voltage-gated sodium channel E) Voltage-gated calcium channel
57) Why is the inside of a resting neuron negative (-70mV) relative to the outside of the neuron?
A) The sodium-potassium pump moves K + and Na + to the outside of the cell and negative ions to the inside of the cell, and the positive and negative ions cannot move back across the membrane by diffusion. B) The concentration gradient created by the sodium-potassium pump and the permeability of the membrane to K + causes more K + to diffuse out of the cell than is pulled back into the cells by electrical attraction. C) The membrane is more permeable to negative ions than to K+ or Na+, such that more negative ions than positive ions diffuse into the cell, while the sodium-potassium pump continues to move positively charged ions to the outside of the cell. D) A concentration gradient drives the diffusion of K+ out of the cell and Na+ into the cell, but Na+ ions diffuse back out of the cell due to the attraction of negatively charged proteins and ions outside the cell.
Version 1
17
58) Sodium ions are single atoms. Why can't they simply diffuse through the cell membrane's lipid bilayer? A) Since they arecharged, ions can't cross the hydrophobic center of the membrane without achannel or pump. B) Sodium ions arecovalently attached in groups of three, which are too large to diffuseeasily. C) Ions do traversethe membrane easily, but the sodium-potassium pump moves them back across. D) Chelator proteinscapture any ions that start to cross the bilayer.
59) If K + can exit the cell through leak channels, why don't they flow out until there are equal concentrations on both sides of the membrane? A) K + canact as an enzymatic cofactor, and most K + ions are bound to cellularenzymes. B) The leak channelsallow a very slow leak that cannot keep pace with active import + ofK ions. C) The membrane is more permeable to Na+ than to K+, and the flow of Na+ into the cell reduces the concentration gradient of K+ across the cell membrane. D) While a concentration gradient tends to drive K+ out of the cell, some K+ are pulled by a negative electric potential back into the cell.
60) The outside of a neuron is positively charged at resting potential. Why do K the cell when K + channels open during an action potential?
+
ions leave
A) The diffusiongradient is the only factor that affects ion movement. B) K +ions are actively pumped out by the sodium-potassium pump acting inreverse. C) Voltage-gated K+ channels open when membrane potential reaches +50 mV during the depolarization phase. The diffusion gradient and electrical force cooperate to drive K+ ions out of the cell. D) Negativelycharged proteins leave the cell at the onset of the action potential.
Version 1
18
61) The outside of a neuron is positively charged at resting potential. Why don't negative ions exit from the cell to normalize charge across the membrane? A) The positivecharge outside the cell repels negative ions. B) There are nonegative ions inside the cell. C) All of thechloride ions are already outside the cell. D) The negativecharge inside the cell is carried by large proteins that cannot diffuse throughthe membrane.
62)
What does the Nernst equation allow us to do? A) Predict the function of a specific type of neuron B) Calculate the equilibrium potential for an ion C) Use a computer program to model axonal transmission D) Calculate the magnitude of depolarization during an action potential
63)
What limits the frequency of the action potentials? A) Chloride ions B) Spike amplitude C) Nodes of Ranvier D) Refractory period
64)
How do temporal and spatial summation improve the processing power of the brain?
Version 1
19
A) Temporal summationallows circadian rhythms to regulate neuronal activity. Spatial summationallows latitude to regulate neuronal activity. B) Temporal summationallows a neuron to fire prolonged action potentials, and spatial summationallows a neuron to synapse onto new targets. C) Temporal andspatial summation both make the neuron more likely to fire actionpotentials. D) Temporal summationallows a neuron to recognize a persistent signal from a single presynapticcell, and spatial summation allows a neuron to recognize a broad signal frommultiple presynaptic cells.
65) All sensory input arrives at the central nervous system in the same form, as __________ propagated by afferent neurons. Perception of the type and intensity of a stimulus depends on which part of the brain the impulse projects to and the firing frequency of the sensory neuron. A) action potentials B) receptor potentials C) reflexes D) frequency localizations E) thresholds
66) The simplest sensory receptors are __________ that respond to mechanical distortion, changes in temperature, or specific chemicals. A) nociceptors B) free nerve endings C) gated channels D) photoreceptors E) ganglia
67)
What sensory receptors sense stimuli that arise from within the body?
Version 1
20
A) Gustatory receptors B) Photoreceptors C) Interoreceptors D) Exteroreceptors
68) When membrane proteins on an olfactory neuron are stimulated by the binding of a specific chemical, what is the immediate result? A) Membrane depolarization B) Membrane repolarization C) Active ion transport D) Membrane hyperpolarization E) Neurotransmitter release
69)
Which includes all of the components that make up the Organ of Corti? A) The basilar membrane only B) The tectorial and basilar membranes C) Hair cells and associated neurons only D) Hair cells and the tectorial membrane E) Hair cells and the basilar and tectorial membranes
70) In the lateral line system of ______, cilia on sensory hair cells are deflected by pressure waves, allowing the animal to sense vibrations and movement.
A) adultamphibians B) reptiles C) birds D) mammals E) fish
Version 1
21
71)
Which of the following receptors does not trigger depolarization of a sensory neuron? A) Mechanoreceptors B) G-protein receptors C) Chemoreceptors D) Photoreceptors E) Exteroceptors and interoceptors
72) Which correctly represents the sequence of steps involved to convey sensory information to the CNS? A) Stimulation and ion flow through the synaptic cleft. B) Neurotransmitter release and interpretation. C) Neurotransmitter release and ion flow through the synaptic cleft. D) Stimulation, transduction, transmission, and interpretation. E) Stimulation, transmission, and neurotransmitter release.
73)
Which of the following stimuli do vertebrates not have a specialized receptor for? A) Hearing B) Taste C) Smell D) Humidity E) Vision
74) Which is not a stimulus that the simplest sensory receptors, free nerve endings, will respond to?
Version 1
22
A) Bending of the sensory neuron membrane B) Changes in temperature C) Changes in the oxygen content of the extracellular fluid D) Release of odorants from freshly baked bread E) Stretching of the sensory neuron membrane
75)
Which is not a stimulus that exteroceptors can sense? A) Gravity B) Smell C) Light D) Muscle tension E) Sound
76)
Which is not a stimulus that interoceptors can sense? A) Limb position B) Pain C) Gravity D) Body temperature E) Muscle length
77)
Which of the following is not an example of a mechanoreceptor? A) Meissner'scorpuscles B) Ruffiniendings C) Merkel cells D) Red corpuscles E) Pacinian corpuscles
Version 1
23
78) Cutaneous receptors are classified as interoceptors. These specialized skin receptors can respond to which of the following stimuli? A) Heat, cold, and light B) Pain, pressure, and sound C) Chemicals, light, and sound D) Heat, cold, pain, touch, and pressure E) Heat, cold, pain, pressure, light, and sound
79) What are the receptors called that transmit impulses that are perceived by the brain as pain? A) Baroreceptors B) Chemoreceptors C) Nociceptors D) Thermoreceptors E) Propioceptors
80)
Which of the following stimuli will not activate a nociceptor? A) Extremes in temperature B) Very intense mechanical stimulation C) Specific chemicals in the extracellular fluid D) Chemicals released by injured cells E) External odorant molecules
81) What are the sensory structures that enable invertebrates to determine the orientation of the body with respect to gravity called?
Version 1
24
A) Carotid bodies B) Statocysts C) Ruffini endings D) Baroreceptors E) Interoceptors
82)
Which of the following do humans not have taste buds specialized to sense? A) Salty B) Sour C) Sweet D) Vitamins E) Bitter
83) Which of the following are components of vertebrate gravity receptors? I Utricle and saccule chambers of a membranous labyrinth II Hair cells with stereocilia and kinocilium III Hairlike processes embedded with otolith membrane, a gelatinous membrane containing calcium carbonate crystals IV Peripheral chemoreceptors A) I and II only B) II, III, and IV C) I and IV only D) I, II, and IV E) I, II, and III
84)
Which of the following constitute the vestibular apparatus?
Version 1
25
A) Saccule and utricle only B) Semicircular canals, saccule, and utricle C) Ampullae of Lorenzini D) Saccule, utricle, and Organ of Corti E) Saccule, utricle, and ventricle
85)
What does the middle ear consist of? A) Three ossicle bones B) The ear canal C) The ear drum D) The vestibular canal E) The tympanic canal
86) Which sequence of events is required for the brain to interpret sound? I. Vibration of the basilar membrane relative to the tectorial membrane causes cilia on hair cells to bend and triggers depolarization. II. Hair cells stimulate the production of action potentials in sensory neurons that project to the brain. III. Ganglion cell axons transmit action potentials to the occipital lobe. A) II then I B) III then I then II C) III then I D) I then II E) I then II then III
87)
In the vertebrate eye, what structure is light focused onto?
Version 1
26
A) Pupil B) Lens C) Retina D) Sclera E) Iris
88)
Which is not a part of the vertebrate eye? A) Cornea B) Retina C) Iris D) Cochlea E) Pupil
89)
In the vertebrate eye, what structure controls the amount of light that reaches the retina? A) Fovea B) Cornea C) Iris D) Lens E) Sclera
90)
What is the photopigment in rod cells called? A) Photopsin B) Rhodopsin C) Opsin D) Carotene E) Melanin
91)
Which of the following cells are not found in the retina?
Version 1
27
A) Rods B) Cones C) Bipolar cells D) Hair cells E) Ganglion cells
92)
What are the short retinal photoreceptors responsible for color vision called? A) Rods B) Cones C) Bipolar cells D) Glial cells E) Ganglion cells
93) What sequence of events leads to the perception of a visual scene? I. In response to light, retinal dissociates from rod and cone photopigments, hyperpolarizing the photoreceptor cells and leading to activation of bipolar neurons. II. Action potentials are propagated through ganglion cells, whose axons project to the occipital lobe of the brain. III. In response to light, rod and cone photoreceptors depolarize, causing increased release of neurotransmitter and activation of bipolar neurons. A) I then III B) III then II C) III then I then II D) I then II then III E) I then II
94)
What sensory equipment is required for an animal to have binocular vision?
Version 1
28
A) Eyes located to the sides of the head B) Eyes located on the front of the head, facing forward C) A camera-type eye with multiple lenses D) Infrared vision E) Ultraviolet vision
95)
Which is not an example of a stimulus sensed by an interoceptor? A) Temperature B) Vibration C) Pain D) Magnetism E) Touch
96) In what order will you encounter the following structures if you examine the human ear from the outside inward? A) tympanic membrane → malleus →incus → stapes → oval window → cochlea B) tympanic membrane → incus →malleus → stapes → oval window → cochlea C) tympanic membrane → stapes →malleus → incus → oval window → cochlea D) tympanic membrane → malleus →incus → stapes → cochlea →oval window E) tympanic membrane → oval window → malleus →incus → stapes → cochlea
97) Each type of cone cell is stimulated by a different color of light. In humans, a colorful scene is perceived as a combination of signals from three cone types. Which colors correspond to these three cones? I Red II Yellow III Orange IV Blue V Green VI Purple
Version 1
29
A) I, II, and III B) I, III, and IV C) I, IV, and V D) I, V, and VI E) II, IV, and VI
98)
Which of the following statements about hair cells is not true? A) They are found inthe mammalian ear. B) They are found inthe fish lateral line system. C) They can berepaired if they rupture. D) Bending of thecells triggers ion flow across the cell membrane. E) Their structureincludesstereocilia and a kinocilium.
99)
Which of the following structures is found in a human eye, but not in an octopus eye? A) Retina B) Lens C) Optic nerve D) Blind spot E) Cones
100)
What effect do high frequency sounds tend to have on the ear? A) Displace the whole length of the basilar membrane with a maximum near the apex B) Move only the basal portion of the basilar membrane C) Move only the apex portion of the basilar membrane D) Move only the portion of the basilar membrane between the basal portion and the
apex E) Only move the tectorial membrane
Version 1
30
101) The statoliths of a lobster are replaced with iron filings, and a strong magnetic field is produced at the top of its tank. How will the lobster respond? A) Its dorsal surfacewill stick to the top of the tank and it will be unable to move. B) It will turn upsidedown and grip the underside of the tank lid. C) It will turn upsidedown and remain at the bottom of the tank. D) It will not respondin any way. E) It willcontinuously spin in circles.
102)
When a cone cell is stimulated by light, what will occur at the cell membrane? A) Excitation B) Inhibition C) An action potential D) Hyperpolarization E) Depolarization
103)
Which statement related to baroreceptor function is accurate?
A) Blood pressure is monitored by blood vessels in the brain. B) Blood pressure is monitored by the heart itself. C) When blood pressure decreases, the frequency of impulses produced by baroreceptors decreases. D) When blood pressure decreases, the frequency of impulses produced by baroreceptors increases.
104)
Which of the following statements regarding vision is not true? A) Vertebrate rods and cones hyperpolarize in response to light. B) Insects and birds can often see into the ultraviolet range. C) Bright light will often produce action potentials in both rods and cones. D) Rhodopsin isfound in the membrane of rods.
Version 1
31
105)
Visual acuity is highest when an image is focused on what part of the retina? A) Rods outside the fovea B) Cones outside the fovea C) Rods within the fovea D) Cones within the fovea E) Anywhere, as long as there is ample light
106) What difference in axonal signaling determines whether we experience a mild or a strong sensation? A) Amplitude of action potentials B) Frequency of action potentials C) Action potential threshold D) Conduction speed E) Amount of neurotransmitter released per vesicle
107) What information would best help you distinguish what type of sensation a sensory neuron transmits? A) The rate of action potential firing B) The proteins it expresses most abundantly C) The part of the brain it transmits signals to D) The amplitude of action potentials E) The diameter of the axon
108) Pain receptors are activated by tissue damage, but pain perception requires intact, functional neurons. How does this work?
Version 1
32
A) Nociceptor neuronshave special membrane repair mechanisms. B) The nociceptor neuron only sends action potentials as it is dying. C) The axon continues to fire action potentials despite damage to the cell body. D) The neuron itselfdoes not need to be damaged; it responds to signals released by nearby damagedcells.
109) In "The Disembodied Lady," neurologist Oliver Sacks tells the story of a patient who suffered a catastrophic loss of sensory function. She said, "I feel my body is blind and deaf to itself. ... It has no sense of itself." The patient's nerve damage was strikingly specific. Of the following choices, what type of neuron most likely suffered damage?
A) gustatoryreceptors B) proprioceptors C) olfactoryreceptors D) nociceptors E) rods and cones
110) Nociceptors can respond to nearby tissue damage by sensing cytosolic factors released by damaged cells. Such "algogenic agents" can cause pain sensation even in the absence of tissue damage. What might be a practical use for such an agent? A) A nonlethal riot control measure B) An anesthetic drug C) A treatment for damaged tissue D) A tasty food additive
111) The discovery that ATP is a pain-signaling factor immediately suggested what possible therapeutic approach to alleviate chronic pain?
Version 1
33
A) create largeintracellular aggregates of ATP B) treat with an ATPreceptor antagonist C) block the synthesis of ATP inside cells D) use electric stimulation on pain centers of the brain
112) Mutations in the DFNA20/26 locus, which codes for the actin gamma 1 protein, are associated with congenital deafness. An irate senator complains that government money is being wasted on studies of this mutant gene—in yeast! "I am no biologist," says the senator, "but I am pretty sure yeast don't have ears." How do you respond? A) "You are right, Senator—yeast do not have ears. We should revoke the funding immediately." B) "We can learnvaluable information about the structure of the mutant protein by expressing itin yeast. It is difficult to collect large amounts of actin from tiny haircells, and unethical to collect it from human patients." C) "We can learnvaluable information about the transcription of the mutant gene in yeast. It ismuch more difficult to study transcription in human cells." D) "We can learnvaluable information about sound transmission through the mutant yeast. Byplacing a speaker against a yeast vial and recording sound from the other side,we can infer how the mutant actin affects sound waves."
113)
What are the categories of taste sensation? A) Sweet, salty, sour, creamy, umami B) Sweet, sour, salty, bitter, umami C) Sweet, salty, bitter, chocolate, coffee D) Sweet, sour, salty, creamy, savory
114) You make a knockout of a gene involved in neurological development, and find that the mice have malformed semicircular canals. Instead of being orthogonal (at right angles) to each other, they are parallel. What aberrant behavior might you see in these mice?
Version 1
34
A) Overeating B) Falling over C) Not startling to sudden noise D) Unable to respond to a call to locate another mouse E) Low muscle tone
115) In 1908, chemist Kikunae Ikeda identified a new taste modality he called "umami," or "delicious" in Japanese. It gives a savory flavor to meat, seaweed, and tomatoes. What molecule is responsible for umami? A) Na +ions B) H +ions C) glutamate D) Mg +2ions E) Fe +2ions
116) Beta-N-Methylamino-L-Alanine, or BMAA, is a glutamate receptor agonist produced by cyanobacteria. Since glutamate is an excitatory neurotransmitter, it is not surprising that BMAA is associated with neurological damage. Buildup of BMAA in fruit bats was hypothesized to cause neurodegenerative disease in islanders that fed on the bats. A recent study found high levels of BMAA in sharks, suggesting that consumption of shark fin soup is also dangerous. The researchers suggested that a secondary effect of BMAA may actually make these hazardous foods surprisingly desirable. What might this be? A) Since BMAA is aGlu receptor agonist, it promotes incorporation of Glu into proteins, and hashealth-promoting effects. B) Since BMAA is derived from cyanobacteria, it provides unique nutrients not found in the animal or plant kingdom. C) Since BMAA is aGlu receptor agonist, it imparts a strong umami flavor that makes food moredelicious. D) Since BMAAcontains alanine, it improves protein synthesis by providing this rarenutrient.
Version 1
35
117) Researchers studying olfactory and taste receptors have stumbled upon an unusual phenomenon. These receptor proteins are sometimes expressed in other animal tissues in addition to the nose or tongue. For example, the sweet taste receptor T1R2 is expressed in the pancreas. Suggest the best possible explanation for these findings. A) Damaged receptorsare recycled by the body and end up in waste-disposal organs. B) Control ofprotein expression is imperfect. As long as a receptor doesn't interfere withorgan function, expression will not be selected against. C) Chemoreceptorsbind molecules that help the body sense and respond to internal phenomena, suchas sugar levels in the blood. D) Techniques used to visualize protein expression are imperfect and may lead to false positive results.
118)
What statement best summarizes the evolutionary history of eyes?
A) Developmentoflight-sensing organs evolved once, but several eye structures evolvedseparately. B) Eyes evolved once and were slightly modified in different phyla. C) Eyes evolved separately in each phylum from an ancestor with no ability to detect light. D) Light-detection organs can develop with very little developmental constraint.
119)
Why are carrots said to be good for you?
A) That is just anold wive's tale. B) Beta-carotene,the orange pigment in carrots, is a precursor to Vitamin A and retinal, thecofactor in photopigments. C) Beta-carotene, the orange pigment incarrots, is necessary to form lens proteins. D) Carrots have large amounts ofglutamate, a neurotransmitter that activates sensory neurons.
120) A research strain ofmice has a mutation that causes voltage-gated sodium channelsto take longer to return to the active state after a spike. What is the likely consequence for transduction of sensory stimuli? Version 1
36
A) There will be noeffect on sensory perception, because sensory information relies onstimulus-gated ion channels and not voltage-gated ion channels. B) Sensory informationwill be incorrectly perceived as high intensity stimuli, due to the increasedfrequency of action potentials. C) The mutation willinterfere with the ability of stimuli to create receptor potentials,andthus the neuronwill fire with slow, broad action potentials,giving the animal enhanced sensory perception. D) Due to a longer refractory period, the neuron will be unable to conduct highfrequency action potentials and thus will not effectively transmit information about strong stimuli.
SECTION BREAK. Answer all the part questions. 121) Nodes of Ranvier are the spaces between sections of axon enwrapped with myelin. Because myelin creates an insulating layer that prevents ion flow, the progress of the action potential "jumps" from node to node, thus speeding up transmission down the axon.
121.1) You are studying a gene that, when mutant, causes oligodendrocytes to enwrap sections of axon so that nodes of Ranvier are more closely spaced. What is the likely effect on nerve impulses? A) They will travelfaster. B) They will travel slower. C) They will increasein amplitude. D) They will decreasein amplitude. E) They will travellaterally across connected axons.
121.2)
Version 1
What other cells should you examine for effects from this mutant gene?
37
A) Prefrontal neurons B) Gustatory neurons C) Cholinergic neurons D) Schwann cells
122) Charcot-Marie-Tooth is the most commonly inherited peripheral neuropathy, and has no known cure. Patients suffer damage to motor and sensory neurons, leading to degeneration of muscles and gradual loss of motor control.
122.1) In Charcot-Marie-Tooth neuropathy, bothsensory and motor neurons are damaged. What do these two sets of neurons comprise? A) Peripheral nervous system B) Autonomic nervous system C) Somatic nervous system D) Parasympathetic nervous system E) Central nervous system
122.2) Genetic studies have found that Charcot-Marie-Tooth may be caused by a mutation in the gene Mitofusin-2 (MFN2), which causes mitochondria to form large clumps. In what area of a neuron are these clumps most likely to cause a problem? A) Axon B) Synapse C) Soma D) Dendrite E) Myelin
122.3) Mutation of the peripheral myelin protein 22 (PMP22) gene, which causes demyelination of peripheral neurons, is a more common cause of Charcot-Marie-Tooth disease. In which cells is the function of the PMP22 gene important for normal function?
Version 1
38
A) Schwann cells B) Fibroblasts C) Peripheral neurons D) Oligodendrocytes E) Astrocytes
123) The word hyponatremia derives from the words hypo ("under" or "less") and natrium ("sodium"). The word natrium is the basis for the abbreviation Na for sodium. Hyponatremia is a rare condition, but has become of more concern with the rise in amateur runners participating in marathons. Since they run slower, they spend longer on the course and have more opportunity to drink water and sweat out additional sodium.
123.1) Marathon runners are encouraged to stop at water stations along the course to stay hydrated. However, occasionally a runner drinks too much water and suffers from hyponatremia, or "water intoxication". While water itself is not toxic, drinking too much can upset ion balance in the extracellular space. Based on what you know about ions and nerve signaling, the levels of which important ion are most likely to be disrupted in hyponatremia? A) Hydrogen B) Potassium C) Calcium D) Sodium E) Oxygen
124) A typical AA battery has a potential of 1.5 V. One can think of the potential in a neuron as having similarities to that of a battery.
124.1) If you were to compare the resting potential of a neuron to a battery, which way is the battery oriented?
Version 1
39
A) Positive terminal outside of the cell, and negative terminal inside B) Positive terminal inside of the cell, and negative terminal outside C) Positive terminal pointing toward the axon, negative terminal toward the dendrite D) Positive terminal pointing toward the dendrite, negative terminal toward the axon E) Both terminals inside the cell soma
124.2) How does the resting potential of a typical neuron compare to that of a 1.5 V AA alkaline battery? A) A typical neuron has a resting potential of –70mV, or about 5% the voltage of the battery. B) A typical neuron has a resting potential of –70V, or about 50 times the voltage of the battery. C) A typical neuronhas a resting potential of +70mV, or about 5% the voltage of the battery. D) A typical neuron has a resting potential of +70V, or about 50 times the voltage of the battery.
125) Palytoxin is incredibly toxic, perhaps the second most dangerous nonpeptide toxin. It targets the sodium-potassium pump, locking it open and forming a channel through which the ions can passively flow. Palytoxin is produced by zoanthid cnidarians (soft coral), and there have been incidents of poisoning from zoanthids within marine aquaria tanks in people's homes. It is also thought to be the source of the legend of the Limu make o hana ("Seaweed of Death from Hana").
125.1) A Hawaiian legend tells of a village that angered a shark god. In revenge, a "seaweed" began to grow in the tidepools that killed anyone who touched it. In fact, this "limu — make — o — Hana" — a zoanthid cnidarian — contains a palytoxin that locks the sodium — potassium pump open, allowing free flow of ions. If a villager suffers palytoxin poisoning, which way will the ions in his neurons flow?
Version 1
40
A) All ions will flow out B) Potassium in, sodium out C) Chloride and sodium out, potassium in D) Sodium in, potassium out
125.2)
What is the likely impact of this toxin on the resting potential of a neuron?
A) The resting potential will require more energy to maintain. B) The restingpotential will be more negative. C) It will destroythe ion gradients necessary for the resting potential. D) The restingpotential will promote action potentials with a larger depolarization.
125.3)
What is the likely impact of this toxin on action potentials?
A) They will occurmore frequently. B) They will bestronger and more unpredictable. C) Without iongradients, action potentials are not possible. D) They will havelonger refractory periods.
126) Honeybees have an incredible ability to distinguish thousands of different odorant molecules, using their large repertoire of olfactory receptors. A wide variety of different types of chemicals can be detected as odorants; they simply require olfactory neurons with areceptor that can bind the chemical and transduce that stimulus into a receptor potential.
126.1) Because small quantities of powerful explosives can be hidden easily, a sensor that could detect trace amounts of illicit chemicals would be a useful tool. What animal sensory modality is exquisitely sensitive to minute quantities of airborne chemicals?
Version 1
41
A) Olfaction B) Taste C) Vision D) Hearing E) Touch
126.2) Honeybees can detect a variety of odorants through olfactory receptors on their antennae. You genetically engineer a receptor to be gated by the explosive chemical TNT and use a transgene to express the receptor in honeybee olfactory neurons. What would this provide to the bee? A) The ability to taste TNT B) It would have no effect C) The ability to smell TNT D) The ability to collect TNT molecules E) The ability to see TNT
126.3) If genetically engineered bees could express a receptor for TNT in their olfactory neurons, what type of receptor would the TNT receptor be? A) Chemoreceptor B) Mechanoreceptor C) Electromagnetic receptor D) A novel type of receptor
127) Hair cells are delicate sensors of sound, movement, and gravity. They are stimulated by movement of the stereocilia, which are actually microvilli, and the single kinocilium, which is a true cilium.
127.1) One patient at your audiology clinic has an actin defect, and another patient has a microtubule defect. How might these defects affect the function of their hair cells in hearing? Version 1
42
A) Neither actin normicrotubule mutations will disrupt hair cell cilia function. B) Only the actin mutation will affect hair cell cilia function. C) Only the microtubule mutation will affect hair cell cilia function. D) Both can createproblems in hair cell cilia function.
127.2) One patient at your audiology clinic has an actin defect, and another patient has a microtubule defect. What sense other than hearing may be affected in these patients? A) Smell B) Taste C) Balance D) Touch E) Proprioception
128) Unlike sweet, salty, sour, or umami, the "bitter" category of tastants includes wide structural variety of detectedmolecules. Multiple receptors are required to sense the differentbitter tastants. What do they have in common? Bitter taste is thought to have evolved as a warning system for toxic compounds.
128.1) The bitter receptor hTAS2R38 is activated by glucosinolates found in plants such as turnips and broccoli. The gene variant a person has determines how sensitive their neurons are to these compounds. What would you predict for people who have the highsensitivity variant of hTAS2R38? A) They will noteasily digest turnips and broccoli. B) They will have to eat more turnips and broccoli to get the same flavor. C) They will disliketurnips and broccoli. D) They will become allergic to turnips and broccoli.
Version 1
43
128.2) The bitter receptor hTAS2R38 is activated by glucosinolates found in plants such as turnips and broccoli. Glucosinolates can block synthesis and transport of iodine. Before the advent of iodized salt, this was a particular problem in areas of the world that are naturally low in iodine. Why might the human population include different variants of the hTAS2R38 gene? A) Areas of highiodine are generally coastal, and turnips and broccoli are not easily grownthere. B) Iodine created mutations in the hTAS2R38 gene. C) Where iodine was low, eating plants rich in glucosinolates helped people metabolize iodine, but where iodine was high, they avoided such foods. D) Where iodine waslow, it was important to avoid foods with glucosinolates, but where iodine wasplentiful, palatability of a wider range of plants was beneficial.
129) Pax6 is a transcription factor noted for its unique role in specifying eye development across the animal kingdom. A homologous gene known as eyeless specifies eyes in Drosophila, and misexpression of mouse Pax6 can cause ectopic eyes in flies! This creates a quandary -- how can we explain the universality of the master regulatory gene for eye development, with the striking variability of eye anatomy in different phyla?
129.1) Misexpression of the Pax6 gene was shown to promote eye development in both vertebrates and invertebrates. The authors of this study suggested that this was evidence that eyes evolved only once. What other evidence contradicts this view? A) Invertebratesnormally have only simple eyespots. B) Many different transcription factors can initiate eye development when misexpressed. C) While the anatomy of all animal eyes is the same, lens proteins vary greatly. D) The anatomicalstructure of insect, mollusk, and vertebrate eyes are quite different.
Version 1
44
Answer Key Test name: Chap 33_3e 1) FALSE Even when a neuron is not firing, the pump is maintaining the resting membrane potential. 2) FALSE Activation of multiple types of receptors can be interpreted as "pain" especially with unusually high degrees of activation. 3) TRUE 4) [B, C, E]
Version 1
45
Clarify Question • What is the key concept addressed by the question? o Types of synaptic connections. • What type of thinking is required? o This is an analyze question because you have to break synapses into their component pieces to understand the cell types and parts of cells that are involved. Gather Content/Choose Answer • What do you already know about synapses? What other information is related to the question? o To solve this problem, you’ll need to understand what a synapse is and where they can be found. Synapses are connections between the axon of one neuron and the dendrite of another neuron. By definition, synapses are connections between two cells, so the answer where synapses form on parts of a single cell cannot be correct. The question states that this is where neurotransmitters transmit a chemical signal, but are neurotransmitters the only way to send a signal from one neuron to another?No, electrical synapses allow ion flow across gap junctions from one neuron to another without the use of neurotransmitters. Can synapses occur between an axon and a muscle cell (neuromuscular junction) or the soma of another neuron? Yes, they can. Reflect on Process • Did your problem-solving process lead you to the correct answer? If not, where did the process break down or lead you astray? How can you revise your approach to produce a more desirable result? o If you figured out the correct answer, good job! If not, where did you go wrong? Answering this question correctly depended not only on distinguishing between chemical synapses and electrical synapses but on Version 1
46
your ability to break down different categories of synaptic connections. If you got an incorrect answer, did you remember that a neuromuscular junction is a synapse, or remember that neurons can also have electrical synapses? While synapses commonly occur between axons and dendrites, did you recall that they can also occur at the soma? 5) [A, E] 6) [A, D] 7) [B, D, E] 8) [B, C, E]
Version 1
47
Clarify Question • What is the key concept addressed by the question? o Cause of the speed of reflexes. • What type of thinking is required? o Answering this question correctly depended not only on distinguishing between the different components of a reflex arc, but also analyzing why this neural set-up promotes speed of signal transmission. Gather Content/Choose Answer • What do you already know about reflexes? What other information is related to the question? o To solve this problem, you’ll need to know that a reflex arc is composed of only two or three neurons that connect in the spinal cord.One is a sensory neuron that has its cell body in the dorsal root ganglia outside of the spinal cord. Sometimes these neurons synapse directly to a motor neuron that exits the spinal cord through the ventral roots. Often, the connections between these two types of neurons are modulated by interneurons, which are commonly inhibitory. All of these connections occur in the spinal cord and not the brain, which makes the distances that these signals travel much shorter and faster. These faster responses have evolved to help vertebrates respond rapidly to stimuli that could be dangerous. Reflect on Process • Did your problem-solving process lead you to the correct answer? If not, where did the process break down or lead you astray? How can you revise your approach to produce a more desirable result? o If you figured out the correct answer, great! If not, where did you go wrong?Did you think that neurons in reflexes are so fast because they Version 1
48
are so large in diameter? It is true that action potentials are conducted faster down larger axons than smaller ones, but many of the axons involved in reflexes are not large (for example, simple pain receptors). Did you think that large numbers of sensory neurons would speed a reflex up? Probably not. If anything, it might slow things down. Spatial summation involves input from multiple neurons, but a reflex could only involve two neurones, so the answer beginning with 'They activate a large number of sensory neurons' cannot be correct. 9) [A, C, D, F, G] 10) [A, C, G, H, J]
Version 1
49
Clarify Question • What is the key concept addressed by the question? Similarities between the ear and the lateral line. • What type of thinking is required? This is an analyze question because you have to break the inner ear and the lateral line system into its separate components in order to look for commonalities. Gather Content • What do you already know about the lateral line and inner ear? What other information is related to the question? o You already know that the inner ear of humans detect pressure waves of sound in the air and the lateral line of fishes detect pressure waves in the water. Both systems have sensory hair cells that convert mechanical movement into a neural signal. In both systems the mechanical movement is transferred from the surrounding environment via a gelatinous membrane (the cupula in the lateral line and the tectorial membrane in the cochlea). Sensory hair cells have one longer kinocilium and multiple, shorter stereocilia. When these cilia bend, it opens up ion channels that depolarize the cell. This signal is then sent to the sensory neurons of the auditory or lateral line nerves and then processed by the brain. Choose Answer • Given what you now know, what information is most likely to produce the correct answer? o Both the lateral line and inner ear have sensory hair cell stereocilia that are embedded in a gelatinous matrix (the cupula in the lateral line and the tectorial membrane in the inner ear). These hair cells have one kinocilia and multiple stereocilia. These hair cells are innervated by Version 1
50
sensory neurons in both systems. While mammals have a cochlea and an organ of Corti in the middle of it, fish do not. Similarly, while mammals have large external structures for capturing pressure waves (i.e., like pinna of the external ear), fish do not have external ears. Fish, on the other hand, have a lateral line system along the length of their body to detect vibrations and pressure waves in the water, while mammals have no need for such a system since they detect pressure waves in the air which vibrate the tympanum. Reflect on Process • Did your problem-solving process lead you to the correct answer? If not, where did the process break down or lead you astray? How can you revise your approach to produce a more desirable result? o Answering this question correctly depended upon your ability to pick out similarities and differences between the inner ears of humans and the lateral line of fishes. If you answered incorrectly, what went wrong? Did you remember that both systems have sensory hair cells that are mechanotransducers that convert a vibratory signal into a neural one? Did you think that since the cochlea and organ of Corti are part of the inner ear, and you knew that fish had inner ears, that fish also have these structures? Did you realize that both the cupula of the lateral line system and the tectorial membrane of the cochlea are gelatinous membranes? 11) B 12) C 13) A 14) E 15) E 16) B 17) D 18) D
Version 1
51
19) E 20) E 21) B Clarify Question • What is the key concept addressed by the question? o The question is asking about the relative distribution of key ions (sodium, potassium, and chloride) across the membrane of a neuron. • What type of thinking is required? o This question is an analyze question because you have to understand the differential distribution (shown by the size of the icons) of ions in a neuron during its resting state. Gather Content/Choose Answer • What do you already know about ions in cells? What other information is related to the question? o To solve this problem, you’ll need to apply your knowledge of the differential concentration of ions across all cell membranes, which causes a membrane potential. At rest, this potential is referred to as the resting membrane potential, as opposed to an action potential, which is caused by a rapid, but transient, shift away from the resting membrane potential. Resting membrane potential is largely controlled by the distribution of K+ ions, with equilibrium being reached with much greater K+ inside the cell compared to outside the cell. Action potentials are produced by an influx of Na+ ions going down their concentration gradient. Thus, at rest the concentration of Na+ must be greater outside the cell than inside. Remember that this unequal distribution of Na+ and K+ is produced by sodium-potassium pumps that transport 2 potassium ions inside and 3 sodium ions outside of the cell. Reflect on Process • Did your problem-solving process lead you to the correct answer? If not, where did the process break down or lead you astray? How can you revise your approach to produce a more desirable result? o If you figured out the correct answer, great! If not, where in your thought process did things go wrong? If you arrived at an incorrect answer, were you thinking that a neuron at rest should have its ions in equilibrium and did you wrongly assume that electrochemical equilibrium meant that concentrations of ions are the same on both sides of the membrane? o Answering this question correctly depended upon on distinguishing between the differing distribution patterns of Na+, K+, and Cl– ions while at resting membrane potential.
22) C Version 1
52
23) C 24) B 25) A 26) A 27) A 28) D 29) B 30) E 31) E 32) D 33) B 34) D 35) B 36) C 37) E 38) C 39) E 40) D 41) D 42) C 43) B 44) B 45) A 46) C
Version 1
53
Clarify Question • What is the key concept addressed by the question? o The roles of components of the nervous system. • What type of thinking is required? o This question is asking you to analyze the features of a robot and find analogies between the functions of the components of the robot with components of the nervous system. Gather Content/Choose Answer • What do you already know about the components of the nervous system? How does it relate to the question? o To solve this problem, you’ll need to first understand the components of the nervous system and their functional roles. The sensory systems (vision, touch, smell and taste, hearing, pain) receive stimuli from the external environment and send this information to sensory centers in the brain. The robot car gadgets that can detect sound, touch, and light are acting like a sensory system. The brain has motor centers that control movement of skeletal muscles (somatic nervous system). The brain is similar to the robot car microprocessor that can allow you to program its responses. The robot car motor that moves the car is similar to the somatic nervous system. Reflect on Process • Did your problem-solving process lead you to the correct answer? If not, where did the process break down or lead you astray? How can you revise your approach to produce a more desirable result? o If you figured out the correct answer, excellent! If not, where did you go wrong? Did you think that sympathetic and parasympathetic pathways were one of the answers? Since these pathways control Version 1
54
autonomic physiological responses and the robot does not have something analogous to physiology (it does not have a heart or respiratory rate to control, for example). In addition, the hypothalamus mentioned in an answer would be similar to a battery needing to be recharged which is not mentioned in the question. The answer that first lists motor pathways and muscles before sensory pathways and the CNS has many of the correct components, but they are not in a sensible order. 47) D
Version 1
55
Clarify Question • What is the key concept addressed by the question? o The question is asking you to think about the components of a kneejerk reflex and understand which part of the nervous system is involved. • What type of thinking is required? o This question is asking you to identify key components of a reflex arc and then evaluate which components belong to which part of the nervous system to predict which system the doctor would be concerned about potential damage to. Gather Content/Choose Answer • What do you already know about components of the nervous system? What other information is related to the question? o To solve this problem, you’ll need to apply your knowledge of the function and categorization of the nervous system and a reflex arc.The reflexhammer stretches the leg muscle and activates stretch receptors in the muscle that sends a neural signal through sensory neurons that travel to the dorsal root ganglion and into the spinal cord. Within the spinal cord, the sensory neurons synapse directly to motor neurons that travel back down to the leg muscle and cause it to contract, raising the lower leg. The sensory and motor neurons involved are part of the peripheral nervous system, although they synapse in the spinal cord, which is a part of the central nervous system. If the patellar reflex is nonfunctional, then it is likely the result of damage to peripheral nerves (either sensory or motor). Reflect on Process • Did your problem-solving process lead you to the correct answer? If not, where did the process break down or lead you astray? How can Version 1
56
you revise your approach to produce a more desirable result? o If you figured out the correct answer, great! If not, where in your thought process did things go wrong? Did you remember that the sympathetic and parasympathetic systems are divisions of the autonomic nervous system that is controlled by the medulla oblongata in the brain (which is part of the central nervous system)? You may have thought that since the sensory and motor neurons synapse in the spinal cord, which is part of the central nervous system, that lack of a reflex response could be the result of damage to the spinal cord. This is a possibility, but if there was a serious spinal injury that may have paralyzed the patient’s legs, it is unlikely that the doctor would rely on a simple knee-jerk reflex to test the paralysis. Thus, you can assume the doctor is concerned about the main components of the reflex arc, the sensory and motor neurons, which are part of the peripheral nervous system. 48) C
Version 1
57
Clarify Question • What is the key concept addressed by the question? o The question is asking you to give diet advice to your friend trying to lose weight. • What type of thinking is required? o This question is asking you to evaluate the potential negative consequences of an ultralow fat diet on the nervous system. Gather Content/Choose Answer • What do you already know about components of the nervous system? What other information is related to the question? o To solve this problem, you’ll need to apply your knowledge of the importance of lipids in the nervous system. Myelin is the lipid-rich layer wrapped around axons to insulate them. This insulation speeds up conduction of action potentials. Thus, lipids are vital for the nervous system to operate properly. Axon tracts in the brain and spinal cord are called white matter because this lipid-rich material has a shiny, white appearance and makes up a considerable portion of the nervous system. Thus, a diet with some lipids is necessary for a healthy nervous system. Drinking sufficient water is also important to one’s health, but the question addresses fat and not water. Trans fats is not short for neurotransmission so you can eliminate that answer. Losing weight by any extreme means can actually be very dangerous for your health, but eating moderate levels of healthy, unsaturated fats can provide the lipids needed to myelinate your axons. Reflect on Process • Did your problem-solving process lead you to the correct answer? If not, where did the process break down or lead you astray? How can Version 1
58
you revise your approach to produce a more desirable result? o If you figured out the correct answer, good job! If not, where did you go wrong? Did you think that the answer concerned water intake because our brain, as well as most of our cells, are primarily water? This is true but the diet your roommate is on is an ultralow fat diet, and water was not mentioned. Did you get fooled into thinking that trans fats really is short for neurotransmitter fats? 49) D 50) D 51) D 52) C 53) E 54) B
Version 1
59
Clarify Question • What is the key concept addressed by the question? o Importance of fatty acids on the nervous system. • What type of thinking is required? o This is an analyze question because you have to extend your knowledge of the importance of fatty acids to a potential nervous system disease. Gather Content/Choose Answer • What do you already know about fatty acids? What other information is related to the question? o To solve this problem, you’ll need to understand why fatty acids are important to the nervous system.Recall that myelin, which is wrapped around axons to improve action potential conduction, is lipid rich. Thus, a disease that slows fatty acid synthesis may also slow myelination, which would presumably slow axon conduction rate. Reflect on Process • Did your problem-solving process lead you to the correct answer? If not, where did the process break down or lead you astray? How can you revise your approach to produce a more desirable result? o If you figured out the correct answer, good job! If not, where did you go wrong? Answering this question correctly depended on extending your knowledge of the role of fatty acid synthesis in the nervous system. If you got an incorrect answer, did you remember that action potentials do not get weaker or stronger, but either decrease or increase in frequency? This would have eliminated an answer. Lipids, in excess and depending upon what type, can promote plaques in blood vessels which can block their flow, not improve their flow, so another answer is not Version 1
60
correct. That leaves two possible choices. While losing weight can benefit one’s health and even the functioning of the brain, there is no definitive information given stating that the patient actually lost weight, which leaves insufficient myelination and slow axon conduction rate. 55) A 56) B 57) B 58) A 59) D 60) C 61) D 62) B 63) D 64) D
Version 1
61
Clarify Question • What is the key concept addressed by the question? o Similarities and differences between spatial and temporal summation. • What type of thinking is required? o Answering this question correctly depended not only on distinguishing between temporal and spatial summation, but on your ability to break down, or analyze, the advantages or benefits of each. Gather Content/Choose Answer • What do you already know about summation? What other information is related to the question? o To solve this problem, you’ll need to remember that that in spatial summation, graded potentials, from different presynaptic neurons that occur at the same time, add together to potentially reach an abovethreshold voltage and thus initiate an action potential. In temporal summation, a single neuroncan produce sufficient depolarization, to produce an action potential, if it produces EPSPs that are close enough in time to sum to an above-threshold voltage. Reflect on Process • Did your problem-solving process lead you to the correct answer? If not, where did the process break down or lead you astray? How can you revise your approach to produce a more desirable result? o If you figured out the correct answer, great! If not, where did you go wrong?Did you think that temporal summation and circadian rhythms were linked because they both have to do with time? It is true that both concepts relate to time, but the question is asking about the processing power of the brain. Temporal summation does not allow for prolonged action potentials, as they are an all-or-none response, but both temporal Version 1
62
and spatial summation can regulate whether an action potential will be elicited by producing above-threshold voltages. Summation does not necessarily make a neuron more likely to fire, because it depends upon whether the signals that are summed are excitatory or inhibitory. 65) A 66) B 67) C 68) A 69) E 70) E 71) D 72) D 73) D 74) D 75) D 76) C 77) D 78) D 79) C 80) E 81) B 82) D 83) E 84) B 85) A 86) D 87) C 88) D 89) C 90) B Version 1
63
91) D 92) B 93) E 94) B 95) D 96) A 97) C 98) C 99) D 100) B 101) B
Version 1
64
Clarify Question • What is the key concept addressed by the question? The role of statoliths. • What type of thinking is required? This question is asking you to evaluate the effect of manipulating the lobster’s statocyst organ with iron filings. Gather Content • What do you already know about statoliths? What other information is related to the question? o Statoliths are calcium carbonate crystals that are embedded at the top of invertebrate statocyst organs to act as weight-bearing structures that are acted upon by gravity. Statocysts are composed of sensory hair cells embedded in a gelatinous mass. The statoliths sit on the gelatinous mass. When the lobster tilts, the statoliths tilt with it and this deflects the cilia of the hair cells. If the statoliths were replaced with iron filings, then those iron filings would be attracted to a magnetic field. If that magnetic field was placed at the top of the tank, the filings would be attracted to the direction of the top of the tank. Choose Answer • Given what you now know, what information is most likely to produce the correct answer? o Normally, with the lobster flat on the bottom of its tank, the weight of the statoliths would be pulled directly down by gravity on the hair cells of the statocyst, neither deflecting them to the left or right. If a magnetic field was produced from the top of the tank, it would pull the filings upward away from the hair cells.This would provide the sensation that lobster was upside down. If the lobster gripped the underside of the top of the tank, the iron filings would then put pressure on the hair cells as if the statoliths were pushing down on the hair cells in the lobster’s normal Version 1
65
position flat on the bottom of the tank. Thus, the answer would be the lobster will turn upside down and grip the underside of the tank lid. Reflect on Process • Did your problem-solving process lead you to the correct answer? If not, where did the process break down or lead you astray? How can you revise your approach to produce a more desirable result? o Answering this question correctly depended on your ability to weigh and judge, or evaluate, which different potential orientations of the lobster in response to a magnet reversing the normal gravitation pull on the statocyst organ. If you got an incorrect answer, did you remember that gravity normally pulls down on statoliths, pushing them against the hair cells of the statocyst organ and that replacing statoliths with iron filings and applying a magnetic field above would produce the opposite force (taking pressure off the hair cells). If you chose the dorsal surface would stick to the top of the tank, you probably thought that the iron filings would be attracted to the magnetic field. You are correct in that the iron filings would be attracted to the magnetic field, but the small amount of iron filings that one could fit into a statocyst organ would not be great enough to lift the weight of a whole lobster. If you chose that it would turn upside down and remain at the bottom of the tank, you realized that the pressure on the hair cells would be opposite and so the lobster’s orientation would also be opposite, which is correct, but the lobster would be drawn to the source of the magnetic field, which is at the top of the tank and not the bottom. 102) D 103) C 104) C 105) D 106) B
Version 1
66
Clarify Question • What is the key concept addressed by the question? How is strength of axonal signaling coded in the nervous system? • What type of thinking is required? This is an analyze question because you have to break neural signaling into different stages and understand which aspects of axonal signaling vary with signal strength. Gather Content • What do you already know about axonal signaling? What other information is related to the question? o You already know that signals are transmitteddown axons via action potentials. But what makes one signal stronger than another one? You know that action potentials are an “all-or-none” response. Aftermembrane potential reaches a certain threshold,voltagegated ion channels open up and an action potential occurs. The amplitude of the action potential does not change with increased stimulation, but the rate at which action potentials fire does change, with higher frequency of action potentials resulting from stronger stimuli. Once action potentials reach the end of the axon, neurotransmitters are released that bind to postsynaptic receptors, which can change the membrane potential and initiate an action potential in the post-synaptic neuron. Choose Answer • Given what you now know, what information is most likely to produce the correct answer? o Since action potentials are all-or-none responses that do not vary in amplitude or threshold, you can eliminate amplitude and threshold as correct responses. Similarly, conduction speed and the amount of neurotransmitter released per vesicle is constant for an individual Version 1
67
neuron. That leaves frequency of action potentials as the correct answer. Greater stimulation leads to a higher frequency of action potentials. Reflect on Process • Did your problem-solving process lead you to the correct answer? If not, where did the process break down or lead you astray? How can you revise your approach to produce a more desirable result? o Answering this question correctly depended not only on distinguishing between different properties of neurons and action potentials, but on your ability to analyze which characteristics are constant and which ones would shift with intensity of stimulation. If you got an incorrect answer, did you remember that action potentials are an all-or-none event that either happens or it does not, with the threshold and amplitude being constant for a particular neuron? Did you forget that axon conduction speed is dependent upon axon width and whether it is myelinated or not? Did you realize that the amount of neurotransmitter in each vesicle is constant? 107) C
Version 1
68
Clarify Question • What is the key concept addressed by the question? How to determine sensory modality. • What type of thinking is required? This question is asking you to weight and judge, or evaluate, traits that might vary by sensory modality. Gather Content • What do you already know about sensory neurons? What other information is related to the question? o You already know that sensory neurons either detect stimuli or are connected to sensory receptors that do. Axons from sensory neurons then enter the dorsal roots of the spinal cord before sending their signals on to the brain. The area of the brain that the signals get sent to depends upon which sensory system is involved. Visual information gets sent to the visual cortex in the occipital lobe. Somatosensory information gets processed in the parietal cortex. Hearing is processed in the temporal lobe. Commonalities of all sensory neurons are that they all fire action potentials and the frequency of action potentials codes for intensity of sensory stimulation. Choose Answer • Given what you now know, what information is most likely to produce the correct answer? o Let us evaluate each answer individually. First, the rate of action potential firing encodes the strength of sensory stimulation in all sensory modalities, so it would not be a good means of distinguishing between different types of sensation. Second, it is true that different sensory systems utilize different neurotransmitters and thus may express proteins for different receptors, but there is also an overlap between Version 1
69
neurotransmitters utilized, and proteins expressed, in different sensory systems so you could not distinguish between sensory modalities using this alone. The part of the brain that a sensory neuron sends signals to is unique for each sensory system, so this is the correct answer. The amplitude of action potentials do not vary by sensory system. It is true that some sensory systems have sensory neurons that differ in morphology. For example, some pain fibers are thin and unmyelinated while other somatosensory neurons are myelinated and have a thicker diameter. Thus, it would be hard to determine if a sensory neuron belonged to the somatosensory system by the diameter of the axon alone. Reflect on Process • Did your problem-solving process lead you to the correct answer? If not, where did the process break down or lead you astray? How can you revise your approach to produce a more desirable result? o Answering this question correctly depended on your ability to weigh and judge, or evaluate, which different characteristics of sensory neurons. If you got an incorrect answer, did you remember that different regions of the brain process different sensory information or that all sensory systems use frequency coding? Did you have trouble weighing the merits of differential protein expression relative to unique parts of the brain for different sensory systems to determine the correct answer? 108) D
Version 1
70
Clarify Question • What is the key concept addressed by the question? How do pain receptors work if they sense damage? • What type of thinking is required? This is an analyze question because you have to break down the process of pain detection by nociceptors. Gather Content • What do you already know about pain receptors? What other information is related to the question? o You already know that pain reception uses nociceptors that detect tissue damage, but how do they work? When tissue is damaged, such as a cut in the skin, an inflammatory response occurs in the tissue that releases molecules detected by nociceptors. These moleculescan diffuse to surrounding tissue that is not damaged and has fully functioning neurons. The molecules can affect ion movement in and out of the free nerve endings used to detect pain. Choose Answer • Given what you now know, what information is most likely to produce the correct answer? o Nociceptor neurons do not have special membrane repair mechanisms, so that answer is not correct. Nociceptors need to be alive and healthy in order to send action potentials, so the answer that says the neurons only send action potentials as it is dying does not make sense. If the cell membrane of the neuron’s cell body is ruptured, then action potentials would not be possible as the ion gradient across the membrane would be lost, thus that answer is not likely. That leaves only one answer, which is that the neuron itself does not need to be damaged, but it responds to molecular signals released by nearby damaged cells. Version 1
71
Reflect on Process • Did your problem-solving process lead you to the correct answer? If not, where did the process break down or lead you astray? How can you revise your approach to produce a more desirable result? o Answering this question correctly depended not only on understanding the steps of pain perception, but on your ability to discriminate among different potential means of how nociceptors work without being damaged themselves. If you got an incorrect answer, did you think that the pain signal associated with tissue damage only came from damaged neurons? Did you realize the molecular signals from damaged tissue could diffuse to adjacent areas to affect healthy nociceptive neurons? 109) B
Version 1
72
Clarify Question • What is the key concept addressed by the question? What neurons help give the body a sense of itself? • What type of thinking is required? This is an evaluate question because you have to use logic to evaluate which potential sensory receptors might produce the symptoms that the patient experienced. Gather Content • What do you already know about sensory receptors? What other information is related to the question? o You already know that sensory receptors send information from the environment to the spinal cord and then the brain to be processed. The body's sense of self and position in space comes from sensory receptors in the vestibular apparatus of the ear and from proprioceptors in muscle spindles. Choose Answer • Given what you now know, what information is most likely to produce the correct answer? o The patient did not say she was blind or deaf, but that her body was blind and deaf to itself and had no sense of itself. Thus rods and cones, which deal with vision, is not the correct answer. Gustatory and olfactory receptors are responsible for taste and smell, respectively, and the patient did not exhibit losses in these senses. Nociceptors detect pain, not body position, and respond to extreme mechanical, chemical, or thermal stimulation. Proprioceptors provide information from within the environment of the body itself about relative position or movement of the body’s parts. If one was to lose these specific receptors, one might feel "disembodied," not being able to detect oneself moving. Reflect on Process • Did your problem-solving process lead you to the correct answer? Version 1
73
If not, where did the process break down or lead you astray? How can you revise your approach to produce a more desirable result? o Answering this question correctly depended on your ability to weigh and judge, or evaluate, which sensory receptors would cause the patient's symptoms if they were lost or damaged and unable to function properly. Were you thrown off because she said her body was blind and deaf to itself, thinking it had to do with vision or hearing? Did you remember that proprioceptors provide information about the position and movement of the limbs and body parts? 110) A
Version 1
74
Clarify Question • What is the key concept addressed by the question? What is a use for algogenic agents? • What type of thinking is required? This is an analyze question because you have to identify key factors that qualify something as a nociceptor. Gather Content • What do you already know about nociceptors? What other information is related to the question? o You already know that nociceptors are pain receptors. They are usually free nerve endings that are sensitive to noxious substances as well as tissue damage since specific cytostolic factors are released by damaged cells. Such factors, called algogenic agents, can cause a pain sensation even in the absence of tissue damage. Choose Answer • Given what you now know, what information is most likely to produce the correct answer? o What might be the practical use of an agent that would cause pain? An anesthetic drug blocks pain, so that cannot be the answer. A tasty food additive does not make much sense although capsaicin in spicy foods can produce a heat and pain sensation. There is evidence that factors that are released during damage produce vasodilation and attract white blood cells to the area and promote healing, but how could someone practically inject such agents into a wound? Since algogenic agents produce pain without injury, it makes sense that they could be used as a nonlethal riot control measure, and in fact, they are. Reflect on Process
Version 1
75
• Did your problem-solving process lead you to the correct answer? If not, where did the process break down or lead you astray? How can you revise your approach to produce a more desirable result? o Answering this question correctly depended not only on understanding the function of nociceptors, but on your ability to analyze potential uses of agents that would activate them. If you got an incorrect answer, did you remember that capsaicin can produce a painful sensation, and knowing that is a component of peppers, think that a tasty food additive was the correct answer? Did you think that because cytostolic factors are released during tissue damage that they would a good treatment for tissue damage? Did you forget that anesthetics block pain instead of produce it? 111) B
Version 1
76
Clarify Question • What is the key concept addressed by the question? Possible therapeutic approaches related to ATP. • What type of thinking is required? This is an evaluate question because you have to use logic to evaluate which potential therapeutic approach might alleviate chronic pain. Gather Content • What do you already know about pain? What other information is related to the question? o You already know that the sensation of pain is elicited by activating nociceptors. If ATP is a pain-signaling factor, then this means that ATP must bind to a nociceptive receptor to open up ion channels. It also means that ATP antagonists could be used to block those same receptors and potentially block pain. You also know that ATP is the power source for the cell, so any manipulation of ATP itself inside the cell could be very dangerous. Choose Answer • Given what you now know, what information is most likely to produce the correct answer? o Creating large intracellular aggregates of ATP should not have an effect of nociceptors since these receptors would be on the outside of cells. Blocking the synthesis of ATP would be harmful to the cells since ATP is their source of energy, without which, they would die. Using electrical stimulation on pain centers in the brain may or may not alleviate chronic pain, but has nothing to do with ATP being a painsignaling factor. Treating with an ATP receptor antagonist, which would prevent ATP from binding to it and sending a pain signal, is the most logical answer. Version 1
77
Reflect on Process • Did your problem-solving process lead you to the correct answer? If not, where did the process break down or lead you astray? How can you revise your approach to produce a more desirable result? o Answering this question correctly depended on your ability to weigh and judge, or evaluate, which the potential therapeutic approaches to alleviate chronic pain. If you got it wrong, did you think that by blocking the synthesis of ATP you would prevent ATP-associated nociceptors from being bound by ATP and thus block pain? The problem with that is that nociceptors are located at the cell surface, not inside the cell, and blocking ATP production would kill the cell. 112) B
Version 1
78
Clarify Question • What is the key concept addressed by the question? Why study mutant actin in yeast? • What type of thinking is required? This is an evaluate question because you have to weigh the logic and content of different responses to see which would be best to answer the senator. Gather Content • What do you already know about mutant genes? What other information is related to the question? o You already know that mutations are changes in the sequence of DNA that encodes a protein. Sometimes a mutation causes a significant change in the structure of a protein, leaving it nonfunctional and potentially causing disease. We know that actin is an important component of the stereocilia of hair cells that are in the inner ear, which is why a mutation in actin gamma protein 1 is associated with congenital deafness. Because yeast and human are both eukaryotes and contain similar transcription and translation machinery, human genes can be added to yeast, which will then result in the yeast translating the protein of interest. In addition, there is considerable homology between yeast and human genes. As yeast can be grown rapidly and inexpensively, this is a much easier means of collecting large enough amounts of the protein to study in the lab than in mammalian systems. Choose Answer • Given what you now know, what information is most likely to produce the correct answer? o The researcher is probably not interested in studying transcription per se (transcription is similar between yeast and human), but is likely Version 1
79
interested in the function, or lack thereof, of the mutated protein. Studies sound transmission through the yeast does not make sense at all and has nothing to do with human deafness (as yeast do not have ears), but having yeast express the protein of interest is a good way to collect large amounts of the protein that would be very difficult to obtain from mammalian hair cells and unethical to obtain from human patients. Reflect on Process • Did your problem-solving process lead you to the correct answer? If not, where did the process break down or lead you astray? How can you revise your approach to produce a more desirable result? o Answering this question correctly depended upon your ability to weigh or judge the validity of certain arguments for using yeast as a model organism for a human deafness gene. If you answered incorrectly, which answer did you choose? Did you think that it is more difficult to study transcription in human cells than yeast cells? Did you think that because the gene was involved with deafness that the answer had to be related to sound transmission? 113) B 114) B
Version 1
80
Clarify Question • What is the key concept addressed by the question? What are the results of semicircular canal defects? • What type of thinking is required? This is an evaluate question because you have to weigh and judge the different symptoms and behaviors to match expected outcomes of malformed semicircular canals. Gather Content • What do you already know about semicircular canals? What other information is related to the question? o You already know that semicircular canals are part of the inner ear, linked to the cochlea. While the cochlea allows us to hear, the semicircular canals provide us with a sense of balance. The three fluidfilled tubes that make up the semicircular canals are orthogonal to one another. That is, they are oriented at 90 degrees from each other so that motion in any orientation can be detected. If this system did not work, you would not be able to maintain your balance and would fall over easily. Choose Answer • Given what you now know, what information is most likely to produce the correct answer? o Malformed semicircular canals would lead to problems maintaining balance. Thus a behavior you might see in mice with this defect is that they would fall often. The other behavioral problems are not related to semicircular canals. Reflect on Process • Did your problem-solving process lead you to the correct answer? If Version 1
81
not, where did the process break down or lead you astray? How can you revise your approach to produce a more desirable result? o Answering this question correctly depended upon your ability to weigh or judge the validity of certain symptoms of malformed semicircular canals. If you answered incorrectly, which answer did you choose? Did you forget that semicircular canals are involved in maintaining balance? 115) C 116) C
Version 1
82
Clarify Question • What is the key concept addressed by the question? What are two effects of glutamate receptor agonists? • What type of thinking is required? This is an evaluate question because you have to weigh and judge the different potential benefits of BMAA despite the fact that it is associated with neurological damage. Gather Content • What do you already know about glutamate? What other information is related to the question? o You already know that that glutamate is an important excitatory neurotransmitter. Too much glutamate can lead to excitotoxicity and neuron death, which is why people that eat animals with too much BMAA can have neurological problems. You also know that glutamate is derived from the amino acid glutamic acid and that amino acids are what make up proteins. This is why glutamic acid is often used as a food additive and flavor enhancer in the form of its salt, monosodium glutamate. Since BMAA can bind to glutamate receptors, it can bind to umami taste receptors that bind amino acids like glutamate, and thus provide a savory flavor. Choose Answer • Given what you now know, what information is most likely to produce the correct answer? o Since BMAA is a glutamate receptor agonist, potentially positive side effects could be imparting a strong umami flavor that makes food more delicious. It would not promote the incorporation of glutamate into proteins since the agonist has nothing to do with the translation of proteins. Just because it is derived from cyanobacteria, does not ensure Version 1
83
that it has unique nutrients. Alanine is a nonessential amino acid, meaning that it can be manufactured by the human body and is not rare. Thus it would not need to come from a diet of cyanobacteria, fruit bats, shark fins, or any other diet. Reflect on Process • Did your problem-solving process lead you to the correct answer? If not, where did the process break down or lead you astray? How can you revise your approach to produce a more desirable result? o Answering this question correctly depended upon your ability to weigh or judge the validity of certain potential benefits of BMAA. If you answered incorrectly, which answer did you choose? Did you think that cyanobacteria might have some potential health benefits? Did you know that alanine is a nonessential amino acid? 117) C
Version 1
84
Clarify Question • What is the key concept addressed by the question? Why are sensory receptors sometimes internal? • What type of thinking is required? This is an evaluate question because you have to weigh and judge the different reasons that taste receptors could be found internally in the body. Gather Content • What do you already know about taste receptors? What other information is related to the question? o You already know that that taste receptors are specific to different types of chemical that elicit different flavors (sweet, salty, umami, bitter). You know that sweet receptors like T1R2 are activated by sugars such as sucrose and fructose and artificial sweeteners like saccharin. We know that the pancreas cannot taste in the same sense that our tongue can, so why would it have taste receptors? The pancreas is an organ in the digestive system that produces important hormones such insulin and glucagon, which are vital for controlling blood sugar. Since sugars bind to T1R2 receptors, these receptors could be involved in monitoring and regulating blood sugar, which is vitally important as individuals with diabetes can attest. Choose Answer • Given what you now know, what information is most likely to produce the correct answer? o The pancreas is both an endocrine organ, producing hormones, and a digestive organ, producing enzymes that help to digest food. Thus referring to it as a waste-disposal organ is not very accurate. It is true that protein expression is not always perfect, but cells will avoid wasting Version 1
85
energy producing receptors that they do not need. Thus it is unlikely that cells will produce unneeded receptors without any negative consequences. Techniques used to visualize protein expression may lead to background fluorescence or staining, but that is why controls are necessary to avoid false positives. The most logical answer is that chemoreceptors like T1R2 in the pancreas bind to molecules to help monitor blood sugar levels. Reflect on Process • Did your problem-solving process lead you to the correct answer? If not, where did the process break down or lead you astray? How can you revise your approach to produce a more desirable result? o Answering this question correctly depended upon your ability to weigh or judge the potential reasons that T1R2 receptors are expressed in the pancreas. If you answered incorrectly, which answer did you choose? Did you think that damaged receptors would be recycled by the pancreas? Did you forget that T1R2 receptors are taste receptors that detect sugars? 118) A
Version 1
86
Clarify Question • What is the key concept addressed by the question? How did eyes evolve? • What type of thinking is required? This is an evaluate question because you have to weigh and judge the different means by which eyes could have evolved. Gather Content • What do you already know about eyes? What other information is related to the question? o You already know that the structure of animal eyes differ by group. For example, insects have eyes with multiple lenses while vertebrates only have one. You also know that all eyes have to be able to detect light by means of photoreceptors, and that even the simplest eyes, like the eyespots of some invertebrates like flatworms, contain photoreceptor cells. Choose Answer • Given what you now know, what information is most likely to produce the correct answer? o The statement that the development of light-sensing organs evolved once, but several eye structures evolved separately makes the most sense in terms of evolution and what you know about the features of eyes in different animal taxa. The statement that eyes were only slightly modified in different taxa is false since eyes can be very different in flatworms, flies, and vertebrates. The statement that modern animal eyes evolved from an ancestor that had no ability to detect light is not true as even simple organisms like flatworms can detect light, although some organisms have secondarily lost the ability to see (such as cavefishes). Reflect on Process Version 1
87
• Did your problem-solving process lead you to the correct answer? If not, where did the process break down or lead you astray? How can you revise your approach to produce a more desirable result? o Answering this question correctly depended upon your ability to weigh or judge the proper process of evolution of the eye. If you chose an incorrect answer, where did you go wrong? Did you think that light detection organs can develop with very little developmental constraints? Some genes that control eye development are similar across taxa (like Pax6), showing that development is constrained by similar processes across taxa. Also there are physical constraints on how an eye can develop in order to produce a focused image upon a set of photoreceptor cells. Did you think that eyes only evolved once? It is true that it is thought that light-detection evolved once, but that different types of eyes evolved separately. 119) B
Version 1
88
Clarify Question • What is the key concept addressed by the question? Why are carrots good for you? • What type of thinking is required? This is an analyze question because you have to make a link between something in carrots to an effect on health. Gather Content • What do you already know about carrots? What other information is related to the question? o You already know that carrots are orange and full of vitamins. What makes a carrot orange? It is called beta-carotene, an orange pigment. You may have heard that carrots are good for you vision, but why? Betacarotene is a precursor to Vitamin A and retinal, which is important in the production of photopigments that are needed for vision. Choose Answer • Given what you now know, what information is most likely to produce the correct answer? o The statement that beta-carotene is necessary to form lens proteins is incorrect. Also, carrots do not have large amounts of glutamate (which would be found in foods with protein). Since beta-carotene is a precursor to Vitamin A and retinal, which are cofactors in photopigments, eating carrots rich in this compound can be good for your eyes and vision. Reflect on Process • Did your problem-solving process lead you to the correct answer? If not, where did the process break down or lead you astray? How can you revise your approach to produce a more desirable result? o Answering this question correctly depended not only on Version 1
89
distinguishing between compounds that could be found in carrots, but also understanding the functional role of the proper compound (betacarotene) and how this might benefit vision. If you selected a wrong answer, which one? Did you think that since the lens is a vital part of the eye, and you knew that carrots help vision, that it was the correct answer? Did you think this was just an old wive’s tale your mother told you so you would eat your vegetables? 120) D
Version 1
90
Clarify Question • What is the key concept addressed by the question? What is the effect of the delay of a neuron'sreturn to a resting state following an action potential? • What type of thinking is required? This is an analyze question because you have to think about the whole process of an action potential to understand how slowing down part of that process can affect subsequent action potentials. Gather Content • What do you already know about action potentials? What other information is related to the question? o You already know that action potentials are produced by the opening and closing of sodium and potassium channels. When a stimulus is applied, if it is strong enough to bring the neuron membrane potential above threshold, sodium channels are opened and the cell is depolarized. Shortly thereafter, the sodium channels close and potassium channels open, repolarizing the cell back toward the resting membrane potential. The time it takes to repolarize the cell is called the refractory period. If a mutation causes the sodium channels to stay open longer, then it will take much longer for the cell to be repolarized. A neuron needs to be repolarized before a subsequent action potential can fire. Thus, the longer the refractory period, the longer the time possible between action potentials, and thus a decrease in action potential frequency. Choose Answer • Given what you now know, what information is most likely to produce the correct answer? o The mutation slowing voltage-gated sodium channels would produce a longer refractory period, thus preventing high-frequency action Version 1
91
potentials. As high-frequency action potentials represent stronger stimuli, the nervous system would not be able to transmit information about such stimuli. Although receptor potentials may not be affected as significantly as action potentials, receptor potentials lead to action potentials once they reach threshold. This, of course, would affect sensory perception. Reflect on Process • Did your problem-solving process lead you to the correct answer? If not, where did the process break down or lead you astray? How can you revise your approach to produce a more desirable result? o Answering this question correctly depended not only on knowing the steps of an action potential including the refractory period, and knowing the difference between a receptor and action potential. If you got it wrong, which answer did you choose? Did you think that this mutation might affect both receptor and action potentials? Did you get the link between refractory period and frequency of action potentials (that is, the shorter the refractory period, the higher the potential frequency)? 121) Section Break 121.1) B
Version 1
92
Clarify Question • What is the key concept addressed by the question? o The question is asking about the function of nodes of Ranvier in action potential conduction. • What type of thinking is required? o This question is asking you to take your knowledge concerning nodes of Ranvier and to analyze the effects of changes to these nodes. • What key words does the question contain? o Nodes of Ranvier, spaced, nerve impulses, oligodendrocytes. How are all these terms related? Gather Content/Choose Answer • What do you already know about nodes of Ranvier? How do they relate to nerve impulses? o To solve this problem, you’ll need to apply your knowledge of the form and function of nodes of Ranvier, which are unmyelinated regions of axons. Remember that myelin insulates axons by preventing ions from leaking out. Since ions only flow into the axon at nodes of Ranvier, this is where action potentials are generated. Following an action potential at one node, sodium rapidly diffuses to the next node, producing an action potential there in a saltatory manner. Consider Alternatives • What other information is related to the question? o The answers are all related to how node of Ranvier spacing could influence nerve impulses (action potentials). You should recall that action potentials are all-or-none responses that do not increase in Version 1
93
amplitude with increased stimulation and that action potentials move along axons to dendrites of another neuron, not laterally across adjacent axons. Choose Answer • Given what you now know, what information and/or problemsolving approach is most likely to produce the correct answer? o At this point, you should have everything you need to know to answer the question. Since you know that action potentials do not increase or decrease in amplitude and you also know that action potentials do not travel from axon to axon, but from axon to dendrite, you can eliminate most of the answer options. So the question is whether tighter spacing would increase or decrease action potential conduction speed. Since myelination speeds up action potentials by preventing ion leaking, and the more nodes of Ranvier that there are, the less overall myelination of the axon, you can deduce that more nodes would decrease the speed of nerve impulses. Therefore, travelling slower is the correct answer. Reflect on Process • Did your problem-solving process lead you to the correct answer? If not, where did the process break down or lead you astray? How can you revise your approach to produce a more desirable result? o If you figured out the correct answer, wonderful! If not, where in your thought process did things go wrong? If you arrived at an incorrect answer, did you remember that nerve impulses are all-ornone events that do not change amplitude? Did you remember that myelination speeds up action potential conduction and that action potentials only occur at nodes of Ranvier? o Was it clear to you that the type of thinking needed to answer this Version 1
94
question is to analyze your knowledge about nerve impulses to predict the effects of a shift away from normal conditions? Consider where in the problem solving process your thinking went astray. 121.2) D
Version 1
95
Clarify Question • What is the key concept addressed by the question? o The question is asking about the effects of a specific mutation on types of neural cells. • What type of thinking is required? o This question is asking you to take your knowledge concerning myelination and to evaluate which cell types will most likely be affected. • What key words does the question contain? o Cells, effects, mutant gene, oligodendrocytes (from previous question).How are all these terms related to the previous question? Gather Content/Choose Answer • What do you already know about this mutation? Which cell type do we know it affects? o To solve this problem, you’ll need to apply your knowledge of the form and function of nodes of Ranvier, which are unmyelinated regions of axons. The mutation causes oligodendrocytes to enwrap shorter sections of axons with myelin. Remember that oligodendrocytes enwrap axons with myelin in the central nervous system, and Schwann cells perform the same function in the peripheral nervous system. Consider Alternatives • What other information is related to the question? o The answers are all types of neural cells. The first three are types of neurons, and the last answer, Schwann cells, is a type of neuroglia. We learned in the previous question that oligodendrocytes were affected by the mutation? What type of cell Version 1
96
is an oligodendrocyte? It is a glial cell. Choose Answer • Given what you now know, what information and/or problemsolving approach is most likely to produce the correct answer? o At this point, you should have everything you need to know to answer the question. Since you know that the mutation affects oligodendrocytes and not neurons, you can eliminate all the answers other than Schwann cells. In further support of Schwann cells as the answer, you know that oligodendrocytes and Schwann cells are both glial cells that myelinate axons. Reflect on Process • Did your problem-solving process lead you to the correct answer? If not, where did the process break down or lead you astray? How can you revise your approach to produce a more desirable result? o If you figured out the correct answer, great! If not, where in your thought process did things go wrong? If you arrived at an incorrect answer, were you thinking that one of the neuron types was the answer because the mutation affects nerve impulses?Did you remember that both oligodendrocytes and Schwann cells perform the same myelination function? o Was it clear to you that the type of thinking needed to answer this question is to evaluate different neural cell types and find one that myelinated axons as oligodendrocytes do? Consider where in the problem solving process your thinking went astray. 122) Section Break 122.1) A 122.2) A Version 1
97
Clarify Question • What is the key concept addressed by the question? o The question is asking you to evaluate which part of an neuron would be prone to disfunction because of mitochondrial clumping. • What type of thinking is required? o This question is asking you to judge which part of an neuron would be the most vulnerable to blockage of neural transmission. Gather Content/Choose Answer • What do you already know about components of the nervous system? What other information is related to the question? o To solve this problem, you’ll need to apply your knowledge of the morphology and physiology of a neuron. The long thin structure of the axon makes it vulnerable to “traffic jams” from clumps of proteins or organelles. This can block fast axonal transport of neurotransmitters, and their precursors to the synaptic terminal. Without neurotransmitters chemical synapses cannot function. Reflect on Process • Did your problem-solving process lead you to the correct answer? If not, where did the process break down or lead you astray? How can you revise your approach to produce a more desirable result? o If you figured out the correct answer, wonderful! If not, where in your thought process did things go wrong?Did you think that dendrites are thin and could be blocked by clumped proteins? That is a possibility, but dendrites are usually much smaller and shorter than axons.Also, transport of neurotransmitters and their precursors occur along axons, not dendrites, so blocking axons would be more Version 1
98
detrimental to the working of the neuron than the blocking of a few of many dendrites. Mitochondria do come from the soma, but the soma is so large that mitochondrial clumping would probably not inhibit action potentials. Myelin is an important part of the axon, but it occurs outside the neuron and so does not have contact with mitochondria (with its potential clumping) from the soma. 122.3) A 123) Section Break 123.1) D
Version 1
99
Clarify Question • What is the key concept addressed by the question? o The question is asking you to solve which ion is responsible for hyponatremia. • What type of thinking is required? o This question is asking you to analyze the potential negative consequences of drinking too much water on cellular function. Gather Content/Choose Answer • What do you already know about components of the nervous system? What other information is related to the question? o To solve this problem, you’ll need to remember what the relative concentrations of important ions are across the cell membrane. Most of the cell is composed of hydrogen and oxygen in the form of water or other molecules, but these are not the ions responsible for neural signaling. Remember that the concentration of sodium is much greater outside the cell than inside, and the opposite is true for potassium. Since the question states that drinking too much can upset the ion balance in the extracellular space, you can surmise that the extra water will dilute the ions in the extracellular space, decreasing the concentration of ions outside the cell. Since sodium is usually greatest outside the cell, diluting it would disrupt the gradient that usually results in sodium rushing into the cell, which is the basis of action potentials. If there is no gradient due to hyponatremia, no action potentials! Also, diluted extracellular fluid can lead to water coming into your cells (via osmosis) and causing cells to swell. Reflect on Process • Did your problem-solving process lead you to the correct Version 1
100
answer? If not, where did the process break down or lead you astray? How can you revise your approach to produce a more desirable result? o If you figured out the correct answer, good job! If not, where did you go wrong? Did you think it was potassium? Too little potassium can result in muscle cramps in runners, but the question mentioned the ion balance in the extracellular space and the ion of greatest concentration there is sodium. While the concentration of calcium ions is also greater outside the cell than inside the cell, it is much smaller than that of sodium. 124) Section Break 124.1) A 124.2) A 125) Section Break 125.1) D
Version 1
101
Clarify Question • What is the key concept addressed by the question? o Sodium and potassium ion flow. • What type of thinking is required? o This is an analyze question because you have to consider the normal distribution of ions across a cell membrane, and then predict the result of free movement of these ions. Gather Content/Choose Answer • What do you already know about sodium and potassium? What other information is related to the question? o To solve this problem, you’ll need to understand that sodium normally has a greater concentration outside the cell and potassium has a greater concentration inside the cell. If a channel is locked open, this would allow free flow of sodium and potassium. Sodium will rush into the cell, and potassium will rush out of the cell, down their concentration gradients. This would reduce the ionic gradients and prevent the formation of action potentials leading to paralysis. Reflect on Process • Did your problem-solving process lead you to the correct answer? If not, where did the process break down or lead you astray? How can you revise your approach to produce a more desirable result? o If you figured out the correct answer, good job! If not, where did you go wrong? Did you remember that the distribution of sodium and potassium across the cell membrane are opposite of each other and that sodium has a greater concentration outside of the cell, whereas potassium has a greater concentration inside the cell. Did Version 1
102
you recall that ions always flow down their electrochemical gradients? 125.2) C
Version 1
103
Clarify Question • What is the key concept addressed by the question? o Sodium and potassium ion flow and equilibrium. • What type of thinking is required? o This is an analyze question because you have to consider the normal distribution of ions across a cell membrane, and then predict the result of free movement of these ions. Gather Content/Choose Answer • What do you already know about sodium and potassium? What other information is related to the question? o To solve this problem, you’ll need to understand that sodium normally has a greater concentration outside the cell and potassium has a greater concentration inside the cell. If a channel is locked open, this would allow free flow of sodium and potassium. Sodium will rush into the cell, and potassium will rush out of the cell, down their concentration gradients. This would reduce the ionic gradients and prevent the formation of action potentials (caused by sodium rushing into the cell) leading to paralysis as well as prevent the maintenance of the resting membrane potential (which is largely caused by the equilibrium potential of potassium). Resting membrane potential is negative (usually about –60 to –70 mV). Although leaky potassium channels allow a small amount of potassium to flow out of the cell continually, having sodiumpotassium channels locked open would allow potassium to flow out rapidly, which would increase the resting membrane potential (i.e., make it less negative). Reflect on Process
Version 1
104
• Did your problem-solving process lead you to the correct answer? If not, where did the process break down or lead you astray? How can you revise your approach to produce a more desirable result? o If you figured out the correct answer, good job! If not, where did you go wrong? Did you remember that the distribution of sodium and potassium across the cell membrane are opposite of each other and that sodium has a greater concentration outside of the cell, whereas potassium has a greater concentration inside the cell. Did you recall that ions always flow down their electrochemical gradients? Did you recall that action potentials are “all-or-none” responses such that larger depolarizations would not be possible? Also, did your remember that the sodium-potassium pump requires energy in the form of ATP? If the pump is locked open, it will not be utilizing ATP and so the resting potential would require more energy to maintain, but with the channels locked open it is not able to. 125.3) C
Version 1
105
Clarify Question • What is the key concept addressed by the question? o Sodium and potassium ion flow and equilibrium. • What type of thinking is required? o This is an analyze question because you have to consider the normal distribution of ions across a cell membrane, and then predict the result of free movement of these ions. Gather Content/Choose Answer • What do you already know about sodium and potassium? What other information is related to the question? o To solve this problem, you’ll need to understand that sodium normally has a greater concentration outside the cell and potassium has a greater concentration inside the cell. If a channel is locked open, this would allow free flow of sodium and potassium. Sodium will rush into the cell, and potassium will rush out of the cell, down their concentration gradients. This would reduce the ionic gradients and prevent the formation of action potentials (caused by sodium rushing into the cell) leading to paralysis as well as prevent the maintenance of the resting membrane potential (which is largely caused by the equilibrium potential of potassium). Also remember that action potentials are “all-or-none” events that do not increase in amplitude (strength), but vary in frequency. Reflect on Process • Did your problem-solving process lead you to the correct answer? If not, where did the process break down or lead you astray? How can you revise your approach to produce a more desirable result? Version 1
106
o If you figured out the correct answer, good job! If not, where did you go wrong? Did you remember that the distribution of sodium and potassium across the cell membrane are opposite of each other and that sodium has a greater concentration outside of the cell, whereas potassium has a greater concentration inside the cell. Did you recall that ions always flow down their electrochemical gradients? Did you recall that action potentials are “all-or-none” responses such that larger depolarizations would not be possible? Remember that ion gradients are necessary for the production of action potentials. If sodium can flow freely down its concentration gradient, then its gradient will be gone and there will be no force attracting it into the cell during an action potential (and thus no action potential will occur). 126) Section Break 126.1) A 126.2) C
Version 1
107
Clarify Question • What is the key concept addressed by the question? What would a transgenic receptor in antennae provide? • What type of thinking is required? This is an analyze question because you have to break sensory (olfaction) signaling into different stages and understand the effect of a novel transgenic receptor on olfaction sensation in honeybees. Gather Content • What do you already know about olfaction? What other information is related to the question? o You already know that olfaction detects odorants when odorants bind to specific olfactory receptors and opening up ion channels, producing receptor potentials. What would happen if you express a novel olfactory receptor for TNT on honeybee antennae? The honeybee would be able to sense or smell the TNT. Choose Answer • Given what you now know, what information is most likely to produce the correct answer? o Since the TNT receptor that you have genetically engineered in the honeybee antennae is an olfactory receptor, and olfaction is the sense of smell, then the honeybee will have the ability to smell the TNT, not taste or see TNT. These receptors would bind to TNT, not collect TNT. Reflect on Process • Did your problem-solving process lead you to the correct answer? If not, where did the process break down or lead you astray? How can you revise your approach to produce a more Version 1
108
desirable result? o Answering this question correctly depended not only on understanding the steps of the sensory pathway of olfaction, but on your ability to understand how a change in this system by genetic engineering can change the perception of the honeybee. If you got an incorrect answer, did you get confused with the definition of olfaction thinking that it was referring to taste, which also uses a similar receptor system? Did you think that TNT does not have a smell? Although we might not be able to detect TNT readily, many animals can. Dogs, for example, can be trained to detect things like explosives via smell. 126.3) A
Version 1
109
Clarify Question • What is the key concept addressed by the question? What type of receptor would a TNT receptor be? • What type of thinking is required? This is an analyze question because you have to break sensory (olfaction) signaling into different stages and understand what activates the receptors involved. Gather Content • What do you already know about olfaction? What other information is related to the question? o You already know that olfaction detects odorants when odorants bind to specific olfactory receptors and opening up ion channels, producing receptor potentials. What would happen if you express a novel olfactory receptor for TNT on honeybee antennae? The honeybee would be able to sense or smell the TNT since TNT, which is a chemical, binds to the receptor on an olfactory neuron. Thus the receptor would be a chemoreceptor. Mechanoreceptors are activated in response to mechanical stresses, and electromagnetic receptors detect electric currents. Choose Answer • Given what you now know, what information is most likely to produce the correct answer? o Since the TNT receptor that you have genetically engineered in the honeybee antennae is an olfactory receptor, and TNT is a chemical that binds to and activates the receptor, it is a chemoreceptor. Mechanoreceptors are activated in response to mechanical stress and electromagnetic receptors respond to electric currents. While this receptor would be a novel receptor for the Version 1
110
honeybee, it is not a novel type of receptor since chemoreceptors are common in the olfactory and gustatory sensory systems. Reflect on Process • Did your problem-solving process lead you to the correct answer? If not, where did the process break down or lead you astray? How can you revise your approach to produce a more desirable result? o Answering this question correctly depended not only on understanding the steps of the sensory pathway of olfaction, but on your ability to understand how the change in this system by genetic engineering can provide an additional olfactory receptor for the honeybee and distinguishing what type of receptor this is. If you got an incorrect answer, did you get confused with the definition of chemoreceptor? Did you realize TNT is a chemical? Did you think that because TNT does not normally have an olfactory receptor in honeybees that its receptor would have to be an entirely novel type? 127) Section Break 127.1) D
Version 1
111
Clarify Question • What is the key concept addressed by the question? Would actin or microtubule defects affect hair cells? • What type of thinking is required? This is an analyze question because you have to break down the components of a sensory hair cell and what proteins are involved in order to determine how defects in these proteins would effect hearing. Gather Content • What do you already know about hair cells? What other information is related to the question? o You already know that hair cells are found in the organ of Corti in the inner ear. These hair cells have one long kinocilium that is a true cilium that contains microtubules and many smaller stereocilia that are microvilli containing actin fibers. If there is a defect in either microtubules or actin, this may hinder hair cell and thus hearing function. Choose Answer • Given what you now know, what information is most likely to produce the correct answer? o As hair cells contain both microtubules (in kinocilia) and actin (in stereocilia), and since both are important in the development and function of hair cells, then a defect in either or both of these types of proteins would likely create problems for hair cell function. Reflect on Process • Did your problem-solving process lead you to the correct answer? If not, where did the process break down or lead you Version 1
112
astray? How can you revise your approach to produce a more desirable result? o Answering this question correctly depended upon your knowledge of the localization of actin and microtubules in sensory hair cells and understanding the function of the stereocilia and kinocilium. If you answered incorrectly, which answer did you choose? Did you think that since there is only one kinocilium per hair cell that the hair cell might still function properly without it? 127.2) C
Version 1
113
Clarify Question • What is the key concept addressed by the question? What sense would actin or microtubule defects affect? • What type of thinking is required? This is an analyze question because you have to break down the components of a sensory hair cell and what proteins are involved in order to determine how defects in these proteins would affect hearing and other senses. Gather Content • What do you already know about hair cells? What other information is related to the question? o You already know that hair cells are found in the organ of Corti in the inner ear. These hair cells have one long kinocilium that is a true cilium that contains microtubules and many smaller stereocilia that are microvilli containing actin fibers. If there is a defect in either microtubules or actin, this may hinder hair cell and thus hearing function. Hair cells are also associated with the semicircular canals of the ear that are used for sensing the orientation of the body and balance. Choose Answer • Given what you now know, what information is most likely to produce the correct answer? o The patient is at an audiology clinic, which usually works with patients with hearing problems. You know that hair cells in the ear have actin and microtubules that are necessary for their proper function for hearing, but hearing is not one of the answers to choose from. So what other sense requires hair cells? Smell, taste, touch, and proprioception do not. That leaves balance. The sense of Version 1
114
balance is provided by hair cells that are located at each of three semicircular canals. Reflect on Process • Did your problem-solving process lead you to the correct answer? If not, where did the process break down or lead you astray? How can you revise your approach to produce a more desirable result? o Answering this question correctly depended upon your knowledge of the localization of actin and microtubules in sensory hair cells and understanding the function of the stereocilia and kinocilium. If you answered incorrectly, which answer did you choose? Did you think that actin and microtubules are important to other sensory systems besides hearing and balance? Actin and microtubules are important parts of the cytoskeleton of all cells, so that is a good thought, but the key is that the patient is at an audiology clinic, which studies diseases of the ear. Balance is provided by semicircular canals and hair cells in the ear, so it is the only logical choice. 128) Section Break 128.1) C
Version 1
115
Clarify Question • What is the key concept addressed by the question? What is the effect of high-sensitivity bitter receptors? • What type of thinking is required? This is an analyze question because you have to break down the components of taste receptors and understand how changes in receptor sensitivity would change the perception of tastants and result in behavior shifts. Gather Content • What do you already know about taste receptors? What other information is related to the question? o You already know that in order to detect a taste, tastants must first bind to specific receptors on the tongue and inside the mouth. Bitter tastants detect a variety of molecules and arethought to have evolved as a warning system for toxic compounds. If a receptor has high sensitivity for bitter tastants, then bitter foods will be perceived to be extra bitter and unpalatable and people will avoid those foods. Choose Answer • Given what you now know, what information is most likely to produce the correct answer? o Individuals with the high-sensitivity variant of hTAS2R38 will be extra sensitive to bitter-tasting foods such as turnips and broccoli and will dislike them and avoid them. They will have to eat less of turnips and broccoli to get the same bitter taste, not more of them. Taste receptors have nothing to do with digestibility or allergies of foods. Reflect on Process
Version 1
116
• Did your problem-solving process lead you to the correct answer? If not, where did the process break down or lead you astray? How can you revise your approach to produce a more desirable result? o Answering this question correctly depended upon your knowledge of the role of taste receptors and how changing their sensitivity would change how tastes are perceived. If you answered incorrectly, what went wrong? Did you think that high-sensitivity receptors would cause individuals to eat more of bitter foods than less? 128.2) D
Version 1
117
Clarify Question • What is the key concept addressed by the question? Why are there variants in human bitter receptors? • What type of thinking is required? This is an evaluate question because you have to weigh and judge the different reasons that variability in a human bitter taste receptor exist. Gather Content • What do you already know about taste receptors? What other information is related to the question? o You already know that taste receptors are specific to different types of chemicals that elicit different flavors (sweet, salty, umami, bitter). You know that it is hypothesized that bitter receptors evolved to detect compounds that are toxic. Glucosinolates are found in plants like turnips and broccoli, which activate hTAS2R38 receptors. Although eating such vegetables has nutritional benefits, the glucosinolates contained in them can also block synthesis and transport of iodine. If an individual’s diet is low in iodine, this could become a problem as blocking what little iodine the individual has access to can lead to diseases such as goiter. If an individual’s diet is high in iodine, then glucosinolates may not cause a problem. Thus natural selection on receptors for glucosinolates would vary by geographical region and food supply. Choose Answer • Given what you now know, what information is most likely to produce the correct answer? o The case study presented here does not present any evidence that iodine produces mutations in genes. As glucosinolates block the Version 1
118
synthesis and transport of iodine, it could not help people metabolize iodine no matter what the iodine levels were in their food. It is true that areas of high iodine are coastal, whereas mountainous regions have had much of the iodine washed from the soil, but turnips and broccoli can grow well in coastal areas. Thus the answer is that in areas were iodine levels are low, it was important to avoid foods with glucosinolates so as to not block what little iodine was available to the body, whereas in areas of high iodine, eating a wider range of plants in the diet could be beneficial by providing additional fiber and nutrients. Reflect on Process • Did your problem-solving process lead you to the correct answer? If not, where did the process break down or lead you astray? How can you revise your approach to produce a more desirable result? o Answering this question correctly depended upon your ability to weigh or judge the potential benefits and costs of having a diet high in glucosinolates. If you answered incorrectly, which answer did you choose? Did you misunderstand that glucosinates could help people metabolize iodine instead of blocking its synthesis and transport? Did you think that plants high in glucosinolates, like turnips and broccoli, could not be grown in coastal areas? 129) Section Break 129.1) D
Version 1
119
Clarify Question • What is the key concept addressed by the question? How did eye development evolve? • What type of thinking is required? This is an analyze question because you have to break down the role of Pax6 in the process of eye development and understand the process of convergent evolution. Gather Content • What do you already know about eyes? What other information is related to the question? o You already know that Pax6 is involved in eye development and that it has been referred to as the master regulatory gene for eye development. You also know, from the chapter reading, that eyes have evolved independently in several distinct taxa of animals, with distinct features. For example, flies have eyes with hundreds of different lenses while mollusks and chordates have a single lens. This is an example of convergent evolution in which analogous structures (not homologous ones) have evolved the same function of vision in different taxa. But something you may not know is that Pax6 regulates the development of a number of sensory organs and other neural tissues, not just eyes. Also, Pax6 may be an important regulatory gene, but it works in conjunction with a number of other developmental genes. Choose Answer • Given what you now know, what information is most likely to produce the correct answer? o The statement that invertebrates normally have only simple eyespots is simply not true for all invertebrates. Mollusks like squid Version 1
120
and octopi have well-developed eyes with a single large lens similar to those of vertebrates. Many different transcription factors can aid in the process of eye development, but there is no evidence of eye development being initiated by the misexpression of genes other than Pax6. The statement that all animal eyes have the same anatomy is also not true, as insects, mollusks, and vertebrate eyes are different in morphology. Reflect on Process • Did your problem-solving process lead you to the correct answer? If not, where did the process break down or lead you astray? How can you revise your approach to produce a more desirable result? o Answering this question correctly depended not only on distinguishing between true and false statements about the anatomy of eyes from different taxa, but also understanding that Pax6 is the major, although not only, gene involved in eye development. Since Pax6 affects the development of sensory tissues in general, it makes sense that it could affect the development of eyes in different taxa even if these structures are analogous (evolved independently) instead of homologous (from the same ancestor).
Version 1
121
CHAPTER 34 TRUE/FALSE - Write 'T' if the statement is true and 'F' if the statement is false. 1) Animals with closed circulatory systems are more successful than animals with open circulatory systems. ⊚ ⊚
true false
MULTIPLE CHOICE - Choose the one alternative that best completes the statement or answers the question. 2) James has been having trouble digesting lean steaks and other protein-rich foods, and these foods seem to stay in his stomach longer than usual. James is likely lacking the enzyme __________. A) pepsin B) glucagon C) bile D) lipase E) amylase
3) A baby is born without the ability to make parietal cells. This baby would not be able to __________. A) emulsify fats B) digestlactose C) absorb glucoseand galactose D) digest fats E) denature (unfold)milk proteins
4) Because of the hepatic portal vein, the __________ is the first organ to receive most of the products of digestion.
Version 1
1
A) liver B) kidney C) pancreas D) brain E) heart
5) The products of fat digestion in the small intestine are reassembled into triglycerides by the intestinal epithelium before combining them with proteins to form __________. A) chylomicrons B) chyme C) vesicles D) bile E) cholesterol
6)
The pancreas does not secrete __________. A) bile B) bicarbonate C) chymotrypsin D) lipase E) glucagon F) amylase
7)
The liver produces __________ that functions in emulsification of fats. A) trypsin B) glucagon C) insulin D) bile E) secretin
Version 1
2
8) Jane and Tom are worried about their newborn baby who has just begun to nurse, but chokes and turns blue whenever she is fed. Their baby most likely has a defect in her __________. A) tonguemuscles B) soft palate C) muscle in theupper-third of her esophagus D) epiglottis E) muscle in thelower two-thirds of her esophagus
9) Saliva contains the hydrolytic enzyme, salivary amylase, which initiates the breakdown of the polysaccharide __________ into the disaccharide, maltose. A) cellulose B) sucrose C) glucose D) starch E) fructose
10) Most digestion in humans occurs in the first 25 centimeters of the 6-meter length of the small intestine called the __________. A) cecum B) jejunum C) duodenum D) omasum E) colon
Version 1
3
11) Scientists in Switzerland compared the total surface area of the small intestine in different species of bats. When the calculations were done as if the inner mucosal surface was smooth, the average inner surface area of the intestinal tube was 9.1 cm2 in one species. The mucosalsurface area increased to 48 cm2 when the villi were factored in and reached a total of 0.13 m2 after adding in the membranesurface area of themicrovilli. Approximately what percent of the total surface area of the small intestine in these bats can be attributed to the membrane surface area of the microvilli? A) 12% B) 28% C) 55% D) 78% E) 96%
12)
The vertebrate digestive system consists of A) a one-way tube with a separate mouth and anus and accessory organs. B) a two-way tube with a separate mouth and anus. C) a tube with a single opening that serves as mouth and anus. D) a two-way tube with a separate mouth and anus and accessory organs. E) a one-way tube with a separate mouth and anus and no accessory organs.
13)
The gastrointestinal tract does not include the __________. A) mouth B) larynx C) esophagus D) stomach E) small intestine
14) When diet and exercise has failed to produce any significant weight loss, some patients who are seriously overweight elect to have stomach surgery. A procedure galled gastric sleeve surgery removes more than half of the stomach, leaving a tube (or sleeve) about the size of a banana. Following gastric sleeve surgery, patients would have the least trouble__________.
Version 1
4
A) eating largeamounts of food at one time B) eatingcarbohydrate-rich foods C) eatingprotein-rich foods D) eating fatty foods
15)
The villi of the small intestine are invaginations of the __________. A) serosa only B) muscularis only C) mucosa, submucosa, and serosa D) mucosa only E) submucosa andmucosa F) serosa, muscularis, submucosa, and mucosa
16)
All of the following teeth are found in adult vertebrates except A) canines. B) premolars. C) molars. D) precanines. E) incisors.
17)
The gizzard is to a bird as the __________ is/are to a mammal. A) teeth B) stomach C) pancreas D) liver E) cecum
Version 1
5
18) Dorothy suffered a serious stroke that destroyed neurons in the swallowing center of her brain. This damage to her brain would prevent __________. A) peristalsis B) bolus formationby her tongue C) elevation of thesoft palate D) mastication E) secretion ofsaliva
19) The enzyme _______has a pH optimum most similar to peak A in the above graph.
A) trypsin B) pepsin C) lipase D) salivaryamylase E) chymotrypsin F) pancreatic amylase
20) Place theprocesses occurring in the digestive tract in the correct order. I Peristaltic waves of contraction propel the food along the esophagus. II Gastric juices are secreted with the arrival of food into the duodenum. III The acidic chyme is transferred through the pyloric sphincter. IV HCl and pepsin begin digestion of proteins into shorter polypeptides.
Version 1
6
A) IV, III, and II B) II, III, and IV C) I, II, and III D) I, II, and IV E) I, IV, andIII
21) A baby is born without the ability to make chief cells. What would be the best type of diet for this baby? A) A low-fat diet B) A diet low in simple sugars such as glucose and fructose C) A diet low in protein D) A diet low in complex carbohydrates such as starch E) A diet low in minerals and vitamins
22)
In humans, the surface area of the small intestine is A) 3 squarecentimeters. B) 3 squaremeters. C) 300 square meters. D) 3,000 squaremeters. E) 300,000squaremeters.
23) The pancreas is an important accessory organ of the digestive system because it adds all of the following substances to the small intestine except A) lactase. B) trypsin. C) amylase D) lipase. E) bicarbonateions.
Version 1
7
24) Deficiencies of brush border enzymes in the small intestine would not affect the complete digestion of __________. A) fats B) starch C) DNA and RNA D) sucrose E) proteins
25)
Bicarbonate that neutralizes stomach acid is produced by the A) duodenum. B) small intestine. C) pancreas. D) large intestine. E) liver.
26) Three-year old Ben’s parents are ecstatic because he is now using the potty each time he needs to defecate. What muscle has Ben learned to control? A) The cardiac sphincter made of striated muscle B) The internal anal sphincter made of smooth muscle C) The pyloric sphincter made of smooth muscle D) The external anal sphincter made of striated muscle E) The Sphincter of oddi made of smooth muscle
27) An ambulance is rushing a man to the emergency room with a knife wound. The man wasstabbed in the upper-right side of his abdomen and has a large amount of internal bleeding.What vital organ has most likely been damaged by the knife wound?
Version 1
8
A) Stomach B) Pancreas C) Heart D) Liver E) Appendix F) Gall bladder
28) The number of chylomicrons in lymphatic capillaries of the intestine would be increased by eating __________. A) lactose B) triglycerides C) sucrose D) proteins E) starch
29) Out of the estimated total of 9 liters of material passing through the small intestine, how much of it is absorbed in the small intestine? A) about 50milliliters B) about 350milliliters C) about 4.5 liters D) about 8.5liters E) all of it
Version 1
9
30) The above diagramshows four different types of transport proteins in the membranes of epithelial cells that line the small intestine. What protein most likelycarries out coupled transport ofbothNa + and glucose in the overall process of glucose absorption? A) 1 (purple) B) 2(yellow) C) 3 (blue) D) 4 (green) E) None of these answers are correct.
31) The arrival of fat-rich chyme to the duodenum stimulates the gall bladder to contract and release __________ into the small intestine.
Version 1
10
A) cholecystokinin B) bile C) chylomicrons D) lipase E) secretin
32) The liver regulates the concentration in the blood of all of the following except__________. A) alcohol anddrugs B) ammonia C) steroidhormones D) plasmaproteins E) chylomicrons
33)
What is not true about the human large intestine? A) About 4% of fluidabsorption takes place in it. B) It is longer thanthe small intestine. C) It contains threerelatively straight segments with no villi. D) Sodium, vitamin K, and other bacterial metabolic products are absorbed across its
wall. E) It compacts andstores undigested materials until theirexcretion as feces.
34)
The secretion of pepsinogen and HCl is controlled by the hormone A) gastrin. B) pepsin. C) insulin. D) cholecystokinin. E) secretin.
Version 1
11
35) Many animalshave a mutually-beneficial, symbiotic relationship with bacteria that reside in their guts. To study the effects of these relationships in more detail, research organisms such as fish and mice have been raised under germ-free conditions. What important process do you predict is compromised in germ-free mice? A) Vitamin C synthesis B) Vitamin D synthesis C) Synthesis of essential amino acids D) Vitamin K synthesis E) Leptin synthesis
36) A patient with advanced colon cancer just had most of his large intestine removed. What process will be most reduced in this patient as compared to a healthy person? A) Digestion of cellulose B) Detoxification of harmful substances C) Absorption of fatty acids and glycerol into lymph capillaries D) Absorption of sugars and amino acids into the blood E) Absorption of water into the blood
37) Which of the following represents the correct order of adult human teeth, beginning at the front and moving right along the bottom jaw? A) Incisors, molars, canines, premolars B) Incisors, canines, molars, premolars C) Incisors, canines, premolars, molars D) Incisors, premolars, molars, canines E) Canines, incisors, premolars, molars
38) In the human digestive system, the stomach produces about 2 liters of HCl and other gastric secretions every day. Acid secretion by the stomach
Version 1
12
A) is stimulated bygastrin. B) promotes a high pH. C) inhibits pepsin activity. D) begins the process of starch digestion. E) inhibitsperistalsis.
39) Gluten-free food options are becoming more and more common place due to the increased incidence and awareness of celiac disease in the U.S population. If a person with this disease eats foods containing gluten, the immune system responds by attacking and damaging or destroying his/her own villi. What digestive symptom would not be consistent with greatlyreduced numbers of villi? A) Abdominal bloating and pain B) Chronic diarrhea C) Fatty stools D) Weight loss E) Peptic ulcer
40)
A person has his/her gall bladder removed and as a result __________. A) will die without agallbladder B) can now eat as much fat as desired and not absorb it C) can only eat small amounts of fat D) can only eat very small meals E) can only eat liquid meals
41)
After consuming an antacid like Rolaids, ________. A) the stomach wouldabsorb water B) HCl productionwould decrease C) the pH of thestomach fluid would decrease D) the pH of thestomach fluid would increase
Version 1
13
42) A drop to pH 4.0 in the duodenum would primarily stimulate the release of ________________. A) cholecystokinin B) gastrin C) insulin D) pepsin E) secretin
43)
Why doesn't the stomach acid just digest right through the wall of the stomach? A) It does, we just rebuild the layers faster than they can be digested. B) Neutralizers areimbedded in the stomach lining. C) The pH of thestomach acid is not low enough to be able to digest the lining. D) Food prevents the acid from digesting the wall of the stomach. E) The lining of the stomach is protected by mucus.
44)
Which molecule is not able to be absorbed across the stomach wall? A) Water B) Alcohol C) Aspirin D) Glucose
45)
If a person is diagnosed with lactose intolerance, which of the following is likely? A) Eating fatty foods would be lethal. B) Eating dairy foods would be lethal. C) Digesting sugars is no longer possible. D) Eating dairy products could cause gastrointestinal pain. E) This person could only drink milk.
Version 1
14
46)
Which process in the diagram would occur the most after eating a fattymeal of french fries and ice cream? A) A B) B C) C D) A and B equally E) A and C equally F) B and C equally
Version 1
15
47)
Which process in the diagram causes the greatest insulin release? A) A B) B C) C D) A and B equally E) A and C equally F) B and C equally
48)
What digestive hormone stimulates gall bladder contraction? A) Gastrin B) Cholecystokinin C) Gastric inhibitory peptide D) Secretin E) Insulin
49) The luminal contents ofthe same organ that secretes the hormoneinto the blood inhibits the further release of ______ via a negative feedback loop.
Version 1
16
A) gastrin B) cholecystokinin C) gastric inhibitorypeptide D) secretin E) insulin
50)
Pancreatic secretion of bicarbonate is stimulated by _____. A) gastrin B) cholecystokinin C) gastric inhibitory peptide D) secretin E) glucagon
51)
Fish and clamsare similar in that both use _____________ for gas exchange. A) lungs B) tracheal tubes C) the mantle D) gills E) the moist surfaceof their skin
52) The large number of alveoli in mammalian lungs, the numerous lamellae in fish gills, the papulae of echinoderms, and the extensive tracheal system of insects are all adaptations that optimize what factor in the formula for Fick’s law of diffusion? A) D B) Δp C) d D) A
53)
What step in the respiratory process of mammals requires the most energy?
Version 1
17
A) Inhalation B) Exhalation C) Transport of oxygen across the walls of the alveoli D) Movement of carbon dioxide into red blood cells E) Binding of oxygen to hemoglobin F) Activity of carbonic anhydrase in the lungs
54)
Each breath is initiated by neurons in the "respiratory control center" located in the A) spinal cord. B) larynx. C) cerebellum. D) medullaoblongata. E) carotid body.
55) In tissue capillaries, __________ combines with water to form carbonic acid, which after being transported to the lungs, dissociates back to its constituents. A) CO 2 B) O 2 C) H 2CO 3 D) hemoglobin E) HCO 3 –
56) Chemoreceptors located in the aortic and carotid bodies stimulate the respiratory control center in the brain when the blood __________ decreases. A) P O2 B) P CO2 C) hemoglobinconcentration D) HCO 3 – concentration E) pH
Version 1
18
57)
In the above graph, curve _______ is most likely the oxygen-binding curve for normal adult human hemoglobin, curve _______ is most likely that of llama hemoglobin (from a llama population that has lived in the mountains of Peru for thousands of years), and curve _______ is most likely that of myoglobin. A) A, B, C B) A, C, B C) B, A, C D) B, C, A E) C, B, A
Version 1
19
58) Ciliated gills in clams, sharks continuously swimming with their mouths partly open, countercurrent flow of blood and water in fish gills, and the use of air sacs to effect unidirectional flow of air through bird lungs are all adaptations that optimize what factor in the formula for Fick’s law of diffusion? A) D B) Δp C) d D) A
59) Imagine that you have been transported light years away to another planet with waterfilled oceans. This distant planet has a barometric pressure of 1,000 mmHg at sea level and air that contains 15 percent O2. Would you take in much more oxygen, much less oxygen, or about the same amount of oxygen per breath on this planet as you would on Earth?
A) Much less oxygen on the distant planet B) Much more oxygen on the distant planet C) About the same amount of oxygen as on Earth
60) The general term that describes the uptake of oxygen from the environment and the disposal of carbon dioxide at the body system level is A) circulation. B) inspiration. C) exhalation. D) respiration. E) secretion.
61)
The exchange of gases across a respiratory surface always occurs by
Version 1
20
A) facilitated transport. B) active transport. C) diffusion. D) bulk flow. E) osmosis.
62) Tim, who is an avid cyclist, is struck by a car during an organized bike ride. Two of his ribs are broken, puncturing his chest wall on the right side. What will most likely happen to his lungs? A) Both lungs willcollapse. B) Both lungs willoverinflate. C) His left lung will beokay, but his right lung will collapse. D) His left lung willbe okay, but his right lung will overinflate. E) His right lung willoverinflate and his left lung will collapse. F) His right lung willcollapse and his left lung will overinflate.
Version 1
21
63) What would a graph of internal lung pressure compared to atmospheric pressure look like during one inhalation/exhalation cycle in a frog?
A) A B) B C) C D) D E) E
64) The most efficient lungs with respect to oxygen uptake are found in this group of vertebrates.
Version 1
22
A) Amphibians B) Birds C) Fish D) Mammals E) Reptiles
65)
The oxygen content of dry air in the atmosphere A) is about78.1%. B) is about 21.0%. C) is about 0.03%. D) varies depending on the altitude. E) is inverselyproportional to the partial pressure of oxygen.
66) Fick's law of diffusion states that the rate of diffusion of a gas across a membrane depends on all of the following except A) the membranesurface area. B) the concentrationdifference of the gas on the twosides of the membrane. C) the width of themembrane. D) the size of thegas molecule. E) which membranetransport proteins are present.
67) The external gills of an adult axolotl are not as efficient as fish gills because___________. A) water flows overthe axolotl gills at many different angles B) blood flowsthough the axolotl gills more slowly than through fish gills C) axolotl gills arenot highly branched D) axolotl gills are covered by thick layers of cells E) axolotls are slow swimmers and all fish swim fast to ventilate theirgills
Version 1
23
68) The principal reason(s) that gills were replaced with different types of respiratory organs in terrestrial animals is (are) I Tissues and organsare less buoyant in air than in water. II Countercurrent exchange is not very efficient at maximizing blood oxygenation levels. III Water diffuses into air through evaporation. A) I, II, and III. B) just II. C) just I. D) I and III. E) II and III.
69) The habitat of different vertebrate species has dramatically shaped the evolution of respiratory adaptations, particularly since the oxygen concentration in air is approximately 20 times higher than the oxygen concentration in water. Per unit area of gas exchange surface, which group of vertebrates can extract the greatest total amount of oxygen from the external medium? A) Amphibians B) Birds C) Fish D) Mammals E) Reptiles
70) Four-year old Sammy is having a temper-tantrum. In a final attempt to get his way, he yells to his mom, “I'm gonna hold my breath ‘til I die!” What is the primary reason that Sammy’s mom is unconcerned by this statement?
Version 1
24
A) Sammy is too young to hold his breath for more than 30 seconds. B) It is impossible forSammy to control the contraction of his diaphragm and intercostal muscles sincethey are made up of smooth muscle. C) As Sammy holds hisbreath, the decreasing levels of oxygen in his blood will lead to stimulationofthe respiratory controlcenter in his brain. D) As Sammy holds hisbreath, the increasing levels of carbon dioxide in his blood will lead tostimulation ofthe respiratory control center in his brain.
71) When is the PO2 an especiallystrong stimulant of the respiratory control center in the brain of healthy individuals? A) While hyperventilating B) While hypoventilating C) While running a road racealong the shoreline of Vancouver D) While climbing to the summit of Mt. Everest E) While free-diving to a coral reef 60 feet below the surface of the sea
72)
Which blood vessels carry oxygenated blood? A) Pulmonary arteries B) Pulmonary veins C) Systemic veins (such as the femoral vein in your leg) D) Systemic capillaries
73) What is the likely outcome if you were able to force water through fish gills in the opposite direction to how it normally flows over the gills?
Version 1
25
A) The blood's oxygenconcentration would not be able to exceed 50%. B) Gas exchange wouldbe inhibited and movement of oxygen into the blood would cease. C) The direction ofblood flow would reverse through the gills to compensate for the reverseddirection of water flow. D) The movement ofcarbon dioxide from the blood into the water would increase. E) There would be noeffect, and maximal oxygenation of blood would still occur.
74) What change would increase the rate of oxygen diffusing from lung alveoli into the blood? A) An increase in the hemoglobin concentration of the red blood cells B) A decrease in the partial pressure of oxygen in the alveoli C) An increase in the partial pressure of oxygen in the blood entering thelungs D) A decrease in the pH of the blood entering the lungs E) An increase in the humidity of air in the lungs
75) In the countercurrent exchange in a fish's gills, the oxygen concentration is always __________ in the water than the blood along the entire pathway of each lamella. A) higher B) lower C) the same
76)
Which of these animals uses positive-pressure breathing? A) Amphibians B) Reptiles C) Birds D) Mammals
Version 1
26
77) Which group of animals has one-way flow of air through its lungs, whereas all the others have tidal flow? A) Birds B) Amphibians C) Mammals D) Reptiles
78) The gas exchange surfaces in a mammalian lung are organized into structuresthat look like clusters of hollow grapes. The hollow, grape-like structures are the __________. A) parabronchi B) lamellae C) alveoli D) opercula E) bronchi
79)
The respiratory systems of fish and birds are similar in that both __________.
A) employ countercurrentexchange. B) utilize air sacs. C) have unidirectionalflow of the environmental medium over the gas exchange surfaces. D) have similar partialpressures of oxygen in the environmental medium.
80) A major genetic cause of emphysema is a deficiency of the protein alpha-1 anti-trypsin (AAT). AAT is an enzyme that inhibits the activity of another protein. What protein is most likely inhibited by AAT?
Version 1
27
A) carbonic anhydrase B) collagen C) elastin-specific protease D) hemocyanin E) hemoglobin F) myoglobin
81) Tom is choking on a piece of steak that is stuck in his trachea, and he signals for his young son to call 911. It takes the paramedics four minutes to reach his house. Will this be in time to save Tom (and why), or will Tom already be dead (and why)? A) Tom will be alive since the myoglobin in his muscles will release oxygeninto the blood after he stops breathing. B) Tom will be dead due to the lowered pH of his blood. C) Tom will be alive since he had a reserve of about three-quarters of hishemoglobin still bound to oxygen when he started choking. D) Tom will be dead since all the hemoglobin in his arterial blood will unloadits oxygen during the first pass through the tissues after his last breath.
82)
Which dissolved gas cannot be transported by hemoglobin? A) Nitric acid (NO) B) Oxygen (O 2) C) Carbon dioxide (CO 2) D) Carbon monoxide (CO) E) Nitrogen (N 2)
83) The primary advantage of gills versus the body surface for gas exchange in aquatic animals is that __________.
Version 1
28
A) gills are located near the mouth B) all gills containa countercurrent exchange system C) gills have a much greater diffusion surface area D) gills decreaseresistance to movement
84)
Humans cannot survive long at altitudes greater than 6000 m because __________. A) alveoli becomecompressed at the lower atmospheric pressures B) there is much moreCO 2 in the atmosphere at thehigher altitudes than at sealevel C) the partial pressure of O 2 becomes less than half that at sea level D) the diaphragm cannot be contracted enough to produce a full breath
85) Major evolutionary changes have resulted in anatomical and physiological adaptationsthat optimize the rate of gas exchangein animals by each of these factors except __________. A) increasing thesurface area for diffusion B) increasing the concentration difference driving diffusion C) decreasing the distance for diffusion D) increasing thediffusion constant for each gas
86) Most phyla of aquatic invertebrates have improved the rate of gas exchangeby creating a water current over their respiratory surfaces, thereby __________. A) decreasing the distance for diffusion B) increasing the surface area for diffusion C) increasing the concentration difference D) changing the diffusion constant
87)
The large number of alveoli in lungs improve gas exchangeby __________.
Version 1
29
A) decreasing the distance for diffusion B) increasing the surface area for diffusion C) increasing the concentration difference D) changing the diffusion constant
88)
The many lamellae in the gills of fish improve respiratory efficiency by __________. A) decreasing thedistance for diffusion B) increasing the surface area for diffusion C) increasing the concentration difference D) changing the diffusion constant
89) The close proximity of capillaries to the alveoli in lungs improve respiratory efficiency by __________. A) decreasing the distance for diffusion B) increasing the surface area for diffusion C) increasing the concentration difference D) changing the diffusion constant
90) The North American pronghorn antelope can run almost as fast as a cheetah, but unlike the cheetah, can also sustain speeds of 30–40 mph for miles. The pronghorn is about the size of a large, domestic goat. You would predict the antelope to have a __________ than the goat. A) smaller trachea B) higherconcentration of hemoglobin C) smaller lungvolume D) lower concentrationof myoglobin E) lower concentrationof bicarbonate
Version 1
30
91) If each human lung contains about 300 million alveoli with asummed total surface area for gas exchange of 40 square meters, what is the approximate surface area of a single alveolus? A) 0.133 square micrometers B) 1.33 square micrometers C) 133 squaremillimeters D) 7.5 squaremicrometers E) 7.5 squaremillimeters F) 750 squaremicrometers
92) What is the order of structures or spaces that a surgeon will pass through to operateon a human lung? A) skin ->intercostal muscles-> parietal pleural membrane -> pleural cavity-> visceral pleural membrane -> lung tissue B) skin ->intercostal muscles -> pleural cavity -> parietal pleural membrane-> visceral pleural membrane -> lung tissue C) skin ->visceral pleural membrane -> intercostal muscles -> pleural cavity-> parietal pleural membrane -> lung tissue D) skin ->intercostal muscles -> visceral pleural membrane -> pleural cavity-> parietal pleural membrane -> lung tissue E) skin -> pleural cavity ->intercostal muscles -> visceral pleural membrane -> parietal pleuralmembrane -> lung tissue
93)
During exhalation, the diaphragm __________. A) contracts andmoves down B) relaxes and movesup C) relaxes and movesdown D) contracts and moves up
94)
Which part of a hemoglobin molecule binds to a molecule ofO
Version 1
2?
31
A) The surface of an α globin chain B) The surface of a β globin chain C) The iron atom in the center of a heme group D) The copper atom wedged between an α and a β globin chain E) The hydrophobic pocket between two α chains
95)
In which PO2environment would hemoglobin most easily release its O2? A) 40 mm Hg B) 65 mm Hg C) 75 mm Hg D) 90 mm Hg E) 100 mm Hg
96) During exercise, the blood returning to a person's heart contains hemoglobin that is about __________ saturated. A) 3% B) 22% C) 35% D) 75% E) 97%
97) Every winter, many individuals tragically die in their sleep due to faulty heaters that leak carbon monoxide (CO) during the night. How does CO poisoning lead to death? A) CO binds more tightly to carbonic anhydrase than does carbon dioxide. B) CO binds to oxygen, and the CO-oxygen complex cannot bind to hemoglobin. C) CO coats the surfaces of the alveoli, preventing diffusion of oxygen intothe blood. D) CO inhibits the chemosensitive neurons in the aortic and carotid bodies. E) CO binds more tightly to hemoglobin than doesoxygen.
Version 1
32
98)
The axolotl, an aquatic amphibian, primarily uses its __________ for gas exchange. A) gills protectedby an operculum B) external gills C) lungs D) trachealtubes E) parabronchi
99)
Ram ventilation is used by __________ during respiration. A) frogs B) tuna C) whales D) penguins E) salamanders
100)
Air enters human lungs from the mouth by passing through the __________. A) trachea B) nasal cavities C) diaphragm D) posterior air sacs E) esophagus
101)
Why is an open circulatory system less efficient than a closed circulatory system?
Version 1
33
A) It is moredifficult to quickly deliver O 2 to specifictissues when needed with an open circulatory system. B) There is no heartto pump the blood in an open circulatory system. C) Open circulatory systems lack respiratory pigments that bind to oxygen. D) Metabolic wastescannot be removed from tissues in an open circulatory system. E) Blood cannot be used to transportnutrientsin an open circulatory system.
102) In 2012, Lance Armstrong was stripped of his record-setting, seven Tour de France titles and banned for life from competitive cycling because of his alleged use of performanceenhancing drugs (PEDs). One of the PEDs Armstrong was accused of taking is erythropoietin (EPO). Why would systematic use of this PED give Armstrong an unfair advantage in a bike race? A) EPO stimulates formation of new capillary beds in skeletal and heart muscle. B) EPO stimulates red blood cell production by the bone marrow. C) EPO stimulates growth of cardiac and skeletal muscle tissue. D) EPO increases cardiac output by stimulating the ventricles to contract more strongly. E) EPO stimulates the synthesis of myoglobin by muscle cells. F) EPO stimulates the formation of new alveoli in the lungs.
103) The graph below can be used to understand the forces underlying the movement of fluid between the blood and the interstitial fluid. What is a true statement concerning these processes?
Version 1
34
A) Sodium ions contribute significantly to the movement of fluid in regionB. B) In people suffering from severe liver disease or protein starvation, thefluid flow in region B is increased. C) Edema results from reduction of fluid flow in region A. D) Fluid leaves theblood in region A and returns to the blood in region B. E) Normally, the net amount of fluid flowin region A is less than the net amount of fluid flow in region B.
104) The liquid portion of blood that contains metabolites, wastes, hormones, ions, and a variety of proteins is called the __________. A) eosinophil B) lymph C) edema D) plasma E) platelet
105) Leukocytes (white blood cells) account for about __________ of the cells in human blood. A) 0.1% B) 1% C) 10% D) 60%
106) Blood is pumped from the heart into the arterial system, which branches from arteries into fine __________. A) arterioles B) venules C) capillaries D) veins
Version 1
35
107) As compared to the circulatory system of most fishes, the circulatory system of adult amphibians has the advantage of A) a four-chambered heart. B) a single circuit. C) partially separate circuits for gas exchangeand systemic delivery of blood. D) a pocket of the gut that serves as an air bladder. E) complete separation between oxygenated and deoxygenated blood.
108)
The benefit provided by all hearts with two atria is A) complete separation of oxygenated and deoxygenated blood. B) that blood is pumped into body tissues at high pressure. C) there is an extra place to store blood when the animal is at rest. D) not all the blood has to pass through both sides of the heart. E) that there are two muscular regions for pumping blood out of the heart to different
organs.
109) Mammalian, bird, and crocodile hearts have four chambers with two separate atria and ventricles to ensure __________ circuit(s) for blood flow. A) two B) three C) four D) one
110) Jim is suffering from congestive heart failure, in which blood flow is backed up in his lungs and excess fluid is escaping the blood and accumulating in his lung tissue. If untreated, increasing pulmonary edema will lead to death by suffocation. What is the most likely cause of Jim’s congestive heart failure?
Version 1
36
A) Contraction of his right atrium is not keeping up with contraction of his left atrium. B) Jim’s pulmonary semilunar valve is not opening properly. C) Jim’s right atrioventricular valve (tricuspid valve) is not closing properly. D) Jim’s SA node is malfunctioning. E) Contraction of his left ventricle is not keeping up with contraction of his right ventricle.
111) A higher protein concentration within the distal portion of capillaries draws water into them through the process of A) active transport. B) facilitated diffusion. C) diffusion. D) filtration. E) osmosis.
112) In birds and mammals, a vestige of the sinus venosus of the fish heart remains as the sinoatrial node, which serves as a(n) A) diastoledetector. B) baroreceptor. C) site oferythropoiesis. D) inhibitorynode. E) pacemaker.
113)
All of the following are functions of the circulatory system except A) oxygen, nutrient, and waste transport. B) creation of redand white blood cells. C) blood clotting and immune defense. D) temperature regulation. E) hormone transport.
Version 1
37
114) A bone marrow sample from Patient X has abnormally low numbers of megakaryocytes. What body process would you expect to be impaired in Patient X? A) Transport of oxygen B) Defense against foreign microorganisms C) Blood clotting following injury to a blood vessel D) Temperature regulation by vasoconstriction or vasodilation of superficial blood vessels E) Maintaining normal osmotic balance between the blood and the interstitial fluids
115)
Which of the following solutes is not normally found in the plasma of mammals? A) Metabolites, wastes, and hormones B) Ions C) Albumin D) Hemoglobin E) Globulins
116)
Which blood cell type is correctly matched to its function? A) Platelets—hematopoiesis B) Leukocytes—blood clotting C) Erythrocytes—immunity D) Leukocytes—oxygen transport E) Monocytes—defense against microorganisms
117) You are taking a lab practical exam. One station has a microscope slide of a cross section through an artery and vein. Which vessel is the artery?
Version 1
38
A) The vessel with the wider lumen (i.e., interior space that contains the blood) B) The vessel with more muscle tissue and elastic fibersin its wall C) The vessel lined only with endothelium D) The vessel with valves to prevent backflow E) The vessel with a greater overall diameter
118)
Exchange of gases and metabolites between the blood and tissues occurs in the A) arteries. B) arterioles. C) capillaries. D) venules. E) veins.
119) When you are standing upright, what is the primary mechanism that propels blood in your feet back to your inferior vena cava? A) The valves in the leg veins alternating between open and closed B) Contraction of smooth muscle in the walls of the leg veins C) Recoil of elastic fibers in the walls of the leg veins D) Contraction of the left ventricle of the heart E) Contraction of skeletal muscles in your legs
Version 1
39
120)
Refer to the diagram below of the mammalian circulatory system.
If blood is going from the lungs to space3, what is the name of space 3? A) Left ventricle B) Left atrium C) Right ventricle D) Right atrium E) Pulmonary vein F) Aorta
121) Elephantiasis is an infectious disease in which blockage of the circulatory system by microscopic, parasitic roundworms results in a dramatic swelling of the tissues in the lower half of the body. What region of the circulatory system is most likely blocked by these small, threadlike worms? A) The left atrium of the heart B) A pulmonary artery C) Lymphatic vessels and/or lymph nodes D) A carotid artery E) The superior vena cava
122)
What istrue about the lymphatic system?
Version 1
40
A) Fluid movesthrough the lymphatic system under high pressure. B) It collects fluidthat seeps into the tissues and returns it tothe blood. C) It is bidirectional. D) It exhibits peristaltic contractions. E) It carries allabsorbed materialsfrom the stomach andintestines to the liver.
123)
The cells that normally lack a nucleus and have a relatively short life span in humans are A) neutrophils. B) eosinophils. C) basophils. D) erythrocytes. E) lymphocytes.
124) Under what circumstance would your body experience an increase in erythropoietin secretion? A) If you just moved from New York City to start training for a marathon in the "MileHigh" city of Denver, Colorado B) If your hematocrit is elevated above normal levels C) If you are suffering from dehydration D) After drinking 5 liters of water in just 2 hours E) If you are suffering from hypertension
125)
Scientists believe that the heart of the common (chordate) ancestor of all vertebrates A) was little morethan a heavily-muscled regionof the ventral artery. B) had twochambers. C) had threechambers. D) had fourchambers. E) consisted of asingle ventricle.
Version 1
41
126)
Blood returning tothe left atrium of a frog heart has just come from__________. A) the lungs only B) the skin only C) the brain only D) the lungs or theskin E) the lungs or thebrain F) the brain or the skin
127)
The left ventricle in mammals is more muscular than the right ventricle because
A) the right ventricleonly pumps blood into theright atrium. B) the left ventriclepumps more total blood than the right ventricle. C) the left ventriclepumps blood with a higher hematocrit than the blood that leaves the right ventricle. D) the left ventriclehas to pump blood throughout the body at high pressure. E) the left ventricleis always contracting to maintain a systemic blood pressure.
128)
What would happen if your SA node and AV node fired at same time? A) The QRS wave on an ECG would disappear, as it would be masked by the P wave. B) Your stroke volume would decrease. C) Blood would be forced back into the atria from the ventricles. D) Your blood pressure would increase. E) The production of nitric oxide by your endothelial cells would increase.
129) The blood pressure in the brachial artery—the major artery in your upper arm—is at its lowest during
Version 1
42
A) exercise. B) the time the aortais maximally stretched. C) the systolic period. D) the diastolic period. E) ventricular contraction.
130)
What animal group is mismatched with its circulation system type (open or closed)? A) Oyster—open B) Cricket—closed C) Earthworm—closed D) Lobster—open E) Killer whale—closed
131)
What is true about about mammalian red blood cells? A) Red blood cells are also called leukocytes. B) A mature redblood cell is shaped like abiconcave disc because it hasnonucleus. C) Red blood cellscontain albumin. D) Red blood cellsdevelop from lymphoid stem cells. E) Aged red bloodcells are removed from the blood by the kidneys.
132) In the blood clotting cascade mechanism, what is the first event that occurs following injury to a blood vessel? A) Secretion of fibrinogen into the blood B) Secretion of prothrombin into the blood C) Fragmentation of a megakaryocyte to form platelets D) Conversion of fibrinogen to fibrin E) Aggregation of fibrin threads
Version 1
43
133)
One mechanism that some animalsuse to limit heat loss in cold environments is A) vasodilation ofarteries in your arms and legs. B) vasoconstrictionof arteries leading to your brain. C) vasoconstrictionof the precapillary sphincters in the skin. D) vasoconstrictionof yourveins. E) increasing thevolume of blood that leaves the ventricles with each beat of the heart.
134) Edema is a result of increased interstitial fluid in various tissues of the body. All of the following are causes of edema except A) pregnancy. B) liver disease. C) injury to a limb. D) protein malnutrition. E) abnormally lowblood pressure.
135)
The greatest metabolic advantage of a fish's two-chambered heart is
A) its ability topumpblood under high pressure to all areas of the fish's body. B) its ability topump blood to the gills for gas exchange before the blood is pumped to theremaining areas of the body. C) its ability to stay full of blood even as it is emptying into the conus arteriosus. D) its ability tokeepoxygen-rich blood in one chamber separate from oxygen-poor blood inthe other chamber. E) its ability tomaintain a constant blood pressure in the fish.
136) When lungs evolved in vertebrates, the evolution of _____also occurred in the circulation of blood within those vertebrates.
Version 1
44
A) doublecirculation B) red blood cells C) a lymphaticsystem D) a muscularheart E) more plasma
137)
The depolarization of the heart is initiated by the A) left and rightbundle branches. B) sinoatrial node. C) atrioventricular node. D) Purkinjefibers. E) bundle of HIS.
Version 1
45
138)
Which animal is likely to have the circulatory pattern shown in the figure?
A) Frog B) Crocodile C) Bird D) Mammal E) Fish
139)
What factor most limits a person’s maximal increase in cardiac output during exercise?
Version 1
46
A) The time available to fill the heart chambers with blood as the heart rate increases B) The time it takes for electrical impulses to reach all parts of the heart C) The rate at which cardiac muscle contracts D) The rate at which oxygen is delivered to the cardiac muscle cells E) The total amount of blood in an individual
140) The loudest sounds a nursehears while listening to a patient'sheart through a stethoscope are caused by A) contraction of theventricles. B) contraction of theatria. C) closing of the AVvalves. D) opening of thesemilunar valves. E) depolarization ofthe ventricles.
141) In a vertebrate with a four-chambered heart, the structure(s) containing blood with the highest O 2 concentration is/are the __________. A) pulmonaryarteries B) pulmonary veins C) right atrium D) inferior vena cava E) rightventricle
142) During your annual physical,a nurse takes your blood pressure by wrapping a sphygmomanometer cuff around your upper arm, which modulates theflow of blood through your brachial artery.The nursethen enters a reading of 115/70 in your medical chart. These numbers mean that the
Version 1
47
A) pressure in your brachial artery during ventricular contraction is 115, while the pressure during ventricular relaxation is 70. B) pressure in your brachial artery during ventricular contraction is 115 while the pressure during atrial contraction is 70. C) nurse heard sounds through the stethoscope only when the pressure in your brachial artery was exactly 115 and again when it was exactly 70. D) pressure in the ventricles of the heart is 115 when the ventricles are contracting and 70 when the ventricles are relaxing. E) diastolic pressure is 115 and the systolic pressure is 70.
143)
The P wave of the ECG represents A) the contractionof the atria. B) the relaxation of the atria. C) the depolarization of the atria. D) the repolarization of the atria. E) the contraction of the ventricle.
144) If your blood has a higher than normal concentration of albumin, what is also likely to betrue? A) The osmotic pressure of your interstitial fluidis higher than normal. B) Six hourspreviously, you ate a very large steak, which was rich in muscle proteins. C) Your hematocrit isabnormally high. D) The netreabsorption of interstitial fluid in the capillaries is higher thannormal. E) Proteins areescaping into the interstitial fluid, causing a reduction in blood volume.
145)
Myeloid stem cells give rise to all formed elements in the blood except
Version 1
48
A) erythrocytes. B) lymphocytes. C) platelets. D) monocytes. E) basophils.
146) You are taking a lab practical exam, and one station has pins in various internal structures of a bisected mammalian heart. You can orient yourself without picking up and examining the exterior of the heart because A) the width of the external wall of the right ventricle is thicker than all theother chamber walls. B) the width of the external wall of the left atrium is thicker than all theother chamber walls. C) the width of the external wall of the left ventricle is thicker than all theother chamber walls. D) the width of the external wall of the right atrium is thicker than all theother chamber walls. E) the ascending aorta exits from the right ventricle.
147) If an arteriolehas a radius of64 micrometers (μm) and a capillary in the same organ has a radius of 8 μm, what is the difference in resistance in blood flow between the two? A) The resistance inthe arterioleis 6 times the resistance in the capillary. B) The resistance inthe capillary is 6 times the resistance in the arteriole. C) The resistance inthe arterioleis 48 times the resistance in the capillary. D) The resistance inthe capillary is 48 times the resistance in the arteriole. E) The resistance inthe arteriole is 96 times the resistance in the capillary. F) The resistance inthe capillary is 96 times the resistance in the arteriole.
148) Susie just squished a cockroach and its pale yellow hemolymph squirted out on the floor. If Susie analyzed the hemolymph, she would expect to find all of the following except
Version 1
49
A) Amino acids. B) Sugars. C) Erythrocytes. D) Water. E) Ions. F) Cells that protect against foreign microbes.
149) When submerged, frogs receive most of their O 2 from capillaries within the skin. What structure would contain blood with the highest O 2 concentration in a submerged frog, which is not able to breathe using its lungs? A) Left atrium B) Right atrium C) Aorta D) Ventricle E) Pulmonary vein
Version 1
50
Answer Key Test name: Chap 34_3e 1) FALSE
Version 1
51
Clarify Question • What is the key concept addressed by the question? o This question is asking about the relationship between closed circulatory systems the success of a species. • What type of thinking is required? o You are being asked to take what you already know and use, or apply, it to the idea of success as a species. Gather Content • What do you already know evolutionary success? What other information is related to the question? o The evolutionary success of a species is related to its current existence, the duration of its history of a species, and the abundance of the species over time. o Open circulatory systems have a beating heart and some blood vessels, but the blood eventually enters an open space in which it bathes the tissue directly. o In a closed circulatory system, the red blood cells do not leave the blood vessels under normal circumstances. o In a closed circulatory system the blood is moved directly to individual tissues, so the efficiency of oxygen delivery is increased. o Since the vascularization in a closed circulatory system is more extensive than that which occurs in an open circulatory system, a closed circulatory system can be considered more complex. Choose Answer • Given what you now know, what information is most likely to produce the correct answer? Version 1
52
o While the increased efficiency of a closed circulatory system does make larger body size possible, greater body size does not indicate success. o The most numerous animals on our planet today, the insects, have open circulatory systems. Reflect on Process • Did your problem-solving process lead you to the correct answer? If not, where did the process break down or lead you astray? How can you revise your approach to produce a more desirable result? o Answering this question correctly depended on your ability to use the idea of biological success in a new situation. If you got an incorrect answer, did you remember that success is not necessarily related to complexity or that animals with open circulatory systems are both abundant and diverse on our planet? Did you have trouble extending the idea of evolutionary success to determine the correct answer? 2) A
Version 1
53
Clarify Question • What is the key concept addressed by the question? o The question asks about enzymes that digest proteins. • What type of thinking is required? o You are being asked to apply your knowledge about enzymes that digest proteins. Gather Content • What do you know about enzymes that digest proteins? What other information is related to the question? o Your body makes many digestive enzymes, including pepsin, trypsin and chymotrypsin, which digest proteins; amylase, which digests starch; and lipase, which digests fat. Choose Answer • Given what you now know, what information is most likely to produce the correct answer? o Pepsin is an enzyme that digests proteins, so if James lacks this enzyme he would have digesting protein-rich foods. Amylase is used to digest carbohydrates and lipase digests lipids. Glucagon is a hormone and bile is a fluid that contains bile acids to solubilize fats. Reflect on Process • Did your problem-solving process lead you to the correct answer? If not, where did the process break down or lead you astray? How can you revise your approach to produce a more desirable result? o This question asked you to apply your knowledge about enzymes that digest proteins. If you got the correct answer, great job! If you got an Version 1
54
incorrect answer, where did the process break down? Did you think that glucagon or bile were enzymes? Did you think that amylase or lipase digested proteins? 3) E
Version 1
55
Clarify Question • What is the key concept addressed by the question? o The question asks about the function of parietal cells. • What type of thinking is required? o You are being asked to apply your knowledge about the function of parietal cells. Gather Content • What do you know about the function of parietal cells? What other information is related to the question? o The stomach contains a third layer of smooth muscle for churning food and mixing it with gastric juice, an acidic secretion of the tubular gastric glands of the mucosa. These exocrine glands contain three kinds of secretory cells: mucus-secreting cells, parietal cells, which secrete hydrochloric acid (HCl), and chief cells, which secrete pepsinogen, the inactive form of the protease (protein-digesting enzyme) pepsin. Choose Answer • Given what you now know, what information is most likely to produce the correct answer? o Parietal cells produce HCl and this acid helps to denature proteins in the diet. If a baby lacked parietal cells, they could not produce HCl to be secreted in the stomach, and proteins in milk that the baby consumed would not be denatured or unfolded. This would make it more difficult for the baby to digest the milk proteins. Reflect on Process • Did your problem-solving process lead you to the correct answer? If Version 1
56
not, where did the process break down or lead you astray? How can you revise your approach to produce a more desirable result? o This question asked you to apply your knowledge about the function of parietal cells. If you got the correct answer, great job! If you got an incorrect answer, where did the process break down? Did you think parietal cells produce bile acids that help emulsify fats?Did you think that parietal cells produce digestive enzymes? 4) A 5) A 6) A 7) D 8) D
Version 1
57
Clarify Question • What is the key concept addressed by the question? o The question asks about the anatomical structure that malfunctions in choking. • What type of thinking is required? o You are being asked to analyze explanations for choking in a baby. Gather Content • What do you know about the anatomical structure that malfunctions in choking? What other information is related to the question? o Neurons within the walls of the pharynx send impulses to the swallowing center in the brain. In response, electrical impulses in motor neurons stimulate muscles to contract and raise the larynx (voice box). This pushes the glottis, the opening from the larynx into the trachea (windpipe), against a flap of tissue called the epiglottis. These actions keep food out of the respiratory tract, directing it instead into the esophagus. Choose Answer • Given what you now know, what information is most likely to produce the correct answer? o If the epiglottis does not close properly when the baby is nursing, then milk will enter the lungs causing a choking reflex as the baby tries to cough the milk out of their lungs. The esophagus has muscles that move food by peristalsis, and a lack of this function could cause reflux or regurgitation, but probably not choking. The tongue and palate are used in the mouth to mechanically break down food and would not contribute to choking, especially in digesting a liquid food like milk.
Version 1
58
Reflect on Process • Did your problem-solving process lead you to the correct answer? If not, where did the process break down or lead you astray? How can you revise your approach to produce a more desirable result? o This question asked you to analyze explanations for choking in a baby. If you got the correct answer, great job! If you got an incorrect answer, where did the process break down? Did you think that the tongue and palate were necessary to digest a liquid food like milk? Did you think that the esophagus would be necessary to keep food out of the lungs? 9) D 10) C 11) E % MV surface area = [ 0.13 m2 - 48 cm2]/0.13 m2 Since 1 m = 100 cm, 1 m2 = 10,000 cm2. Equation then becomes [ 1300 cm2 - 48 cm2]/1300 cm2 =1252/1300 = ~96%.
Version 1
59
Clarify Question • What is the key concept addressed by the question? o The question asks about the relative surface area of microvilli. • What type of thinking is required? o You are being asked to analyze data to calculate the relative surface area of microvilli of the intestines. Gather Content • What do you know about microvilli? What other information is related to the question? o The epithelial wall of the small intestine is covered with tiny, fingerlike projections called villi. In turn, each epithelial cell lining the villi is covered by many folds of the plasma membrane that form cytoplasmic extensions called microvilli. The microvilli greatly increase the surface area of the intestines to allow for greater nutrient absorption. There are 100 cm in 1 m, so 1 cm = 0.01 m and 1 cm2 = .0001 m2. Choose Answer • Given what you now know, what information is most likely to produce the correct answer? o The inner surface of the intestines including villi is 48 c m2 = 0.0048 m2. Including microvilli gives asurface area of 0.13 m2. Tocompare the total area includingmicrovilli to the total area including villi, you need to subtract the total area including villi from the total area including microvilli (0.13 – 0.0048 = 0.1252 m2). Nowdivide this area by the total area including microvilli to get the percent of the area attributed to the microvilli (0.1252 m2/0.130 m2 = 0.96 = 96%). So, 96% of the area of the intestines can be attributed to the microvilli. Reflect on Process Version 1
60
• Did your problem-solving process lead you to the correct answer? If not, where did the process break down or lead you astray? How can you revise your approach to produce a more desirable result? o This question asked you to analyze data to calculate the relative surface area of microvilli of the intestines. If you got the correct answer, great job! If you got an incorrect answer, where did the process break down? Did you think that you should compare the area of the total intestines to microvilli? Did you think there were 10 cm per m? 12) A 13) B 14) B
Version 1
61
Clarify Question • What is the key concept addressed by the question? o The question asks about digestion in the stomach. • What type of thinking is required? o You are being asked to apply your knowledge about digestion in the stomach to predict what would happen after gastric sleeve surgery. Gather Content • What do you know about digestion in the stomach? What other information is related to the question? o Digestion of carbohydrates begins in the mouth where amylase is released from salivary glands. Digestion of proteins begins in the stomach with acid and proteases like pepsin. Digestion of lipids begins in the small intestines with bile acids and lipase.Mechanical digestion occurs through peristalsis in the stomach and small intestines. With a smaller stomach smaller meals need to be eaten. Choose Answer • Given what you now know, what information is most likely to produce the correct answer? o Carbohydrate digestion begins in the mouth and will not be affected as much by a smaller stomach. Proteins are digested in the stomach. With a smaller stomach, you would need to eat smaller portions. Reflect on Process • Did your problem-solving process lead you to the correct answer? If not, where did the process break down or lead you astray? How can you revise your approach to produce a more desirable result? Version 1
62
o This question asked you to apply your knowledge about digestion in the stomach to predict what would happen after gastric sleeve surgery. If you got the correct answer, great job! If you got an incorrect answer, where did the process break down? Did you think that proteins were not digested in the stomach? Did you think that the size of the stomach would not affect the portions of meals that could be consumed? 15) D 16) D 17) A 18) A
Version 1
63
Clarify Question • What is the key concept addressed by the question? o The question asks about swallowing. • What type of thinking is required? o You are being asked to apply your knowledge about swallowing. Gather Content • What do you know about swallowing? What other information is related to the question? o Many vertebrates have teeth (Figure 47.6), used for chewing, or mastication, that break up food into small particles and mix it with fluid secretions like saliva. When food is ready to be swallowed, the tongue moves it to the back of the mouth. In mammals, the process of swallowing begins when the soft palate elevates, pushing against the back wall of the pharynx (Figure 47.7). Elevation of the soft palate seals off the nasal cavity and prevents food from entering it. Pressure against the pharynx triggers an automatic, involuntary response, the swallowing reflex. Because it is a reflex, swallowing cannot be stopped once it is initiated. Neurons within the walls of the pharynx send impulses to the swallowing center in the brain. In response, electrical impulses in motor neurons stimulate muscles to contract and raise the larynx (voice box). This pushes the glottis, the opening from the larynx into the trachea (windpipe), against a flap of tissue called the epiglottis. These actions keep food out of the respiratory tract, directing it instead into the esophagus. Swallowed food enters a muscular tube called the esophagus, which connects the pharynx to the stomach. The esophagus actively moves a processed lump of food, called a bolus, through the action of muscles by peristalsis. Choose Answer Version 1
64
• Given what you now know, what information is most likely to produce the correct answer? o Mastication is chewing and bolus formation by the tongue are voluntary motions. Saliva secretion responds to food being placed in the mouth.Elevation of the soft palate prevents food from entering the lungs. The correct answer is that the swallowing center controls peristalsis, which moves food through the esophagus Reflect on Process • Did your problem-solving process lead you to the correct answer? If not, where did the process break down or lead you astray? How can you revise your approach to produce a more desirable result? o This question asked you to apply. If you got the correct answer, great job! If you got an incorrect answer, where did the process break down? Did you think that chewing was regulated by the swallowing center? Did you think that secretion of saliva was controlled by the brain? 19) B
Version 1
65
Clarify Question • What is the key concept addressed by the question? o The question asks about enzymes that work at pH 2. • What type of thinking is required? o You are being asked to analyze the optimal pH for an enzyme. Gather Content • What do you know about optimal pH for an enzyme? What other information is related to the question? o Enzymes have an optimal pH that is typically matched to the environment in which they are active. Saliva has a pH of 7.4, and salivary amylase would have an optimal pH in this range, as seen in curve B. The pancreas secretes many digestive enzymes including trypsin and chymotrypsin, which digest proteins; pancreatic amylase, which digests starch; and lipase, which digests fat. Pancreatic fluid also contains bicarbonate, which neutralizes the HCl from the stomach and gives the chyme in the duodenum a slightly alkaline pH as seen in curve C. The stomach has a pH of 2, and the enzyme pepsin will have an optimal pH as seen in curve A. Choose Answer • Given what you now know, what information is most likely to produce the correct answer? o Pepsin is active in the acidic environment of the pH with an optimal pH of 2. Reflect on Process • Did your problem-solving process lead you to the correct answer? If not, where did the process break down or lead you astray? How can you revise your approach to produce a more desirable result? o This question asked you to analyze the optimal pH for an enzyme. If you got the correct answer, great job! If you got an incorrect answer, where did the process break down? Did you think that the lower pH was an alkaline environment? Did you think that pancreatic enzymes were more active at an acidic than an alkaline environment?
20) E 21) C
Version 1
66
Clarify Question • What is the key concept addressed by the question? o The question asks about the function of chief cells. • What type of thinking is required? o You are being asked to apply your knowledge about the function of chief cells. Gather Content • What do you know about the function of chief cells? What other information is related to the question? o The stomach contains a third layer of smooth muscle for churning food and mixing it with gastric juice, an acidic secretion of the tubular gastric glands of the mucosa. These exocrine glands contain three kinds of secretory cells: mucus-secreting cells, parietal cells, which secrete hydrochloric acid (HCl), and chief cells, which secrete pepsinogen, the inactive form of the protease (protein-digesting enzyme) pepsin. Choose Answer • Given what you now know, what information is most likely to produce the correct answer? o Pepsin is used to digest proteins. If a baby cannot make chief cells, then they will not be able to secrete pepsin into their stomach and will not be able to digest proteins. So they should be placed on a low-protein diet.Pepsin is not used to digest carbohydrates or lipids. Reflect on Process • Did your problem-solving process lead you to the correct answer? If not, where did the process break down or lead you astray? How can you revise your approach to produce a more desirable result? Version 1
67
o This question asked you to apply your knowledge about the function of chief cells. If you got the correct answer, great job! If you got an incorrect answer, where did the process break down? Did you think that pepsin is used to digest carbohydrates or lipids? Did you think that chief cells were involved in nutrient absorption? 22) C 23) A 24) A
Version 1
68
Clarify Question • What is the key concept addressed by the question? o The question asks about enzymes found on the brush border. • What type of thinking is required? o You are being asked to apply your knowledge about enzymes found on the brush border. Gather Content • What do you know about enzymes found on the brush border? What other information is related to the question? o The microvilli also participate in digestion because a number of digestive enzymes are embedded within the epithelial cells’ plasma membranes, with their active sites exposed to the chyme. These brush border enzymes include those that hydrolyze the disaccharides lactose and sucrose, and nucleases.Inactive proteases secreted by the pancreas are first activated by a brush border enzyme of the intestine. Choose Answer • Given what you now know, what information is most likely to produce the correct answer? o Starch, sucrose, and nucleic acids like DNA and RNA can be digested directly by brush border enzymes. Proteins are digested indirectly, by brush border enzymes activating trypsinogen into trypsin and chymotrypsinogen into chymotrypsin, which then digest the proteins. The only molecules on the list that would not be digested by brush border cell enzymes are fats, as these cells do not release lipases. Reflect on Process
Version 1
69
• Did your problem-solving process lead you to the correct answer? If not, where did the process break down or lead you astray? How can you revise your approach to produce a more desirable result? o This question asked you to apply your knowledge about enzymes found on the brush border. If you got the correct answer, great job! If you got an incorrect answer, where did the process break down? Did you think that brush border cells produced lipase? Did you realize that brush border cell enzymes would activate proteases? 25) C 26) D
Version 1
70
Clarify Question • What is the key concept addressed by the question? o The question asks about the voluntary control of defecation. • What type of thinking is required? o You are being asked to apply your knowledge about the voluntary control of defecation. Gather Content • What do you know about the voluntary control of defecation? What other information is related to the question? o Compacted feces, driven by peristaltic contractions of the large intestine, pass from the large intestine into a short tube called the rectum. From the rectum, the feces exit the body through the anus. Two sphincters control passage through the anus. The first is composed of smooth muscle and opens involuntarily in response to pressure inside the rectum. The second, composed of striated muscle, can be controlled voluntarily by the brain, thus permitting a conscious decision to delay defecation. Choose Answer • Given what you now know, what information is most likely to produce the correct answer? o Feces pass through two anal sphincters is it passes out of the body. The inner sphincter is not under voluntary control and contains smooth muscles. The outer sphincter is under voluntary control and contains striated muscles. The pyloric sphincter controls movement of food out of the stomach. There is not a cardiac sphincter; the reverse flow of blood is regulated by valves. Reflect on Process Version 1
71
• Did your problem-solving process lead you to the correct answer? If not, where did the process break down or lead you astray? How can you revise your approach to produce a more desirable result? o This question asked you to apply your knowledge about the voluntary control of defecation. If you got the correct answer, great job! If you got an incorrect answer, where did the process break down? Did you think that the inner anal sphincter was under voluntary control?Did you think that the pyloric sphincter was in the rectum? 27) D
Version 1
72
Clarify Question • What is the key concept addressed by the question? o The question asks about which organ is in the upper right side of the abdomen and has a lot of blood flow. • What type of thinking is required? o You are being asked to apply your knowledge about which organ is in the upper right side of the abdomen and has a lot of blood flow. Gather Content • What do you know about which organ is in the upper-right side of the abdomen and has a lot of blood flow? What other information is related to the question? o The liver is a large organ in the upper-right side of the abdomen. To maintain blood homeostasis, the liver regulates lipid and carbohydrate levels among other metabolites. To accomplish this, there is a high level of blood flow through the liver. The gall bladder, pancreas, stomach and appendix are all in the abdomen, but do not receive as much blood flow and are not in the upper-right side of the abdomen. The heart is in the thoracic cavity. Choose Answer • Given what you now know, what information is most likely to produce the correct answer? o The liver is a large organ in the upper-right corner of the abdomen with a large amount of blood flow. If the man had a knife wound in this region, the most likely organ involved is the liver. Reflect on Process • Did your problem-solving process lead you to the correct answer? If not, where did the process break down or lead you astray? How can Version 1
73
you revise your approach to produce a more desirable result? o This question asked you to apply your knowledge about which organ is in the uupper-right side of the abdomen and has a lot of blood flow. If you got the correct answer, great job! If you got an incorrect answer, where did the process break down? Did you remember that the heart was in the thoracic cavity? Did you think that the gall bladder, pancreas, stomach or appendix were in the upper-right corner of the adbomen? 28) B
Version 1
74
Clarify Question • What is the key concept addressed by the question? o The question asks about what chylomicrons contain. • What type of thinking is required? o You are being asked to apply your knowledge about what chylomicrons contain. Gather Content • What do you know about what chylomicrons contain? What other information is related to the question? o Fats (triglycerides) are hydrolyzed into fatty acids and monoglycerides by digestion. These fatty acids and monoglycerides are nonpolar and can thus enter epithelial cells, by simple diffusion. Once inside the intestinal epithelial cells they are reassembled into triglycerides. The triglycerides then combine with proteins to form small particles called chylomicrons, which are too bulky to enter blood capillaries in the intestine. Instead of entering the hepatic portal circulation, the chylomicrons are absorbed into lymphatic capillaries, which empty their contents into the blood in veins near the neck. Choose Answer • Given what you now know, what information is most likely to produce the correct answer? o Chylomicrons transport dietary fats absorbed from the intestines. If you eat more triglycerides you will produce more chylomicrons that will be transported through lymphatic capillaries. Reflect on Process
Version 1
75
• Did your problem-solving process lead you to the correct answer? If not, where did the process break down or lead you astray? How can you revise your approach to produce a more desirable result? o This question asked you to apply. If you got the correct answer, great job! If you got an incorrect answer, where did the process break down? Did you think that proteins were transported on chylomicrons? Did you think that sugars were transported on chylomicrons? 29) D 30) A 31) B 32) E 33) B 34) A 35) D
Version 1
76
Clarify Question • What is the key concept addressed by the question? o The question asks about essential nutrients can be made by bacteria in the intestines. • What type of thinking is required? o You are being asked to apply your knowledge about essential nutrients can be made by bacteria in the intestines. Gather Content • What do you know about essential nutrients can be made by bacteria in the intestines? What other information is related to the question? o All mammals rely on intestinal bacteria to synthesize vitamin K, which is necessary for the clotting of blood. Vitamin C must be consumed, while vitamin D can be consumed or made by the skin. Essential amino acids by definition cannot be made in sufficient levels and need to be eaten. Leptin is a hormone that is made in adipose tissue. Choose Answer • Given what you now know, what information is most likely to produce the correct answer? o The best answer is that all mammals rely on intestinal bacteria to synthesize vitamin K. The other answers are either not essential, or must be consumed and are not made by intestinal bacteria. Reflect on Process • Did your problem-solving process lead you to the correct answer? If not, where did the process break down or lead you astray? How can you revise your approach to produce a more desirable result? Version 1
77
o This question asked you to apply your knowledge about essential nutrients can be made by bacteria in the intestines. If you got the correct answer, great job! If you got an incorrect answer, where did the process break down? Did you think that leptin could not be made by animals? Did you think that essential amino acids could be made by an animal? 36) E
Version 1
78
Clarify Question • What is the key concept addressed by the question? o The question asks about the function of the large intestine. • What type of thinking is required? o You are being asked to apply your knowledge about the function of the large intestine. Gather Content • What do you know about the function of the large intestine? What other information is related to the question? o After food enters the stomach, the food passes to the small intestine, where a battery of digestive enzymes continues the digestive process. The products of digestion, together with minerals and water, are absorbed across the wall of the small intestine into the bloodstream. What remains is emptied into the large intestine, where some of the remaining water and minerals are absorbed. Choose Answer • Given what you now know, what information is most likely to produce the correct answer? o The primary function of the large intestines is to absorb water. If the large intestines were removed the patient would become dehydrated as not enough water would return to the blood. Reflect on Process • Did your problem-solving process lead you to the correct answer? If not, where did the process break down or lead you astray? How can you revise your approach to produce a more desirable result? Version 1
79
o This question asked you to apply your knowledge about the function of the large intestine. If you got the correct answer, great job! If you got an incorrect answer, where did the process break down? Did you think that sugars or fatty acids were absorbed in the large intestines?Did you think that cellulose was digested in the large intestines? 37) C 38) A 39) E
Version 1
80
Clarify Question • What is the key concept addressed by the question? o The question asks about the function of villi. • What type of thinking is required? o You are being asked to analyze symptoms that would occur in a patient with damaged villi due to celiac disease. Gather Content • What do you know about the function of villi? What other information is related to the question? o The epithelial wall of the small intestine is covered with tiny, fingerlike projections called villi. In turn, each epithelial cell lining the villi is covered by many folds of the plasma membrane that form cytoplasmic extensions called microvilli. Absorption of fats and sugars occur through these epithelial cells. Damage of the epithelial cells on the villi would decrease absorption of fats and sugars leading to weight loss. The fats would leave the body in the feces, resulting in fatty stools. The sugars would draw water into the intestines through osmosis, leading to diarrhea, abdominal bloating and pain. Choose Answer • Given what you now know, what information is most likely to produce the correct answer? o Peptic ulcers are caused by Heliobacter pylori infections and not by celiac disease. Reflect on Process • Did your problem-solving process lead you to the correct answer? If Version 1
81
not, where did the process break down or lead you astray? How can you revise your approach to produce a more desirable result? o This question asked you to analyze symptoms that would occur in a patient with damaged villi due to celiac disease. If you got the correct answer, great job! If you got an incorrect answer, where did the process break down? Did you recall that peptic ulcers are caused by bacteria? 40) C
Version 1
82
Clarify Question • What is the key concept addressed by the question? o The question asks about the function of the gall bladder. • What type of thinking is required? o You are being asked to apply your knowledge about the function of the gall bladder. Gather Content • What do you know about the function of the gall bladder? What other information is related to the question? o The gall bladder contains bile acids made by the liver. The bile acids are released into the small intestines after a fatty meal. The bile acids help to emulsify and break down fats so they can be digested and absorbed more readily. If a person without a gall bladder eats too much fat, it will cause a fatty stool, which is similar to diarrhea. Choose Answer • Given what you now know, what information is most likely to produce the correct answer? o Without a gall bladder a patient cannot secrete bile acids into the small intestine to help break down and absorb fats. As a result, they would be put on a low fat diet. Reflect on Process • Did your problem-solving process lead you to the correct answer? If not, where did the process break down or lead you astray? How can you revise your approach to produce a more desirable result? o This question asked you to apply your knowledge about the function Version 1
83
of the gall bladder. If you got the correct answer, great job! If you got an incorrect answer, where did the process break down? Did you think that a person could eat as much fat as they wanted to because it would not be absorbed? Did you think that the gall bladder was associated with the size of the stomach that would affect the portion size of meals the patient could consume? 41) D 42) E
Version 1
84
Clarify Question • What is the key concept addressed by the question? o The question asks about a hormone secreted in response to acid. • What type of thinking is required? o You are being asked to apply your knowledge about a hormone secreted in response to acid. Gather Content • What do you know about a hormone secreted in response to acid? What other information is related to the question? o The major enterogastrones include cholecystokinin, secretin, and gastric inhibitory peptide. Chyme with high fat content is the strongest stimulus for CCK and GIP secretions, whereas increasing chyme acidity primarily influences the release of secretin. Pepsin is a digestive enzyme secreted into the stomach. Insulin is a blood hormone made by the pancreas. Gastrin and pepsin are made by the stomach. Choose Answer • Given what you now know, what information is most likely to produce the correct answer? o Chyme is acidic and lowers the pH of the duodenum to 4. This stimulates the release of secretin. Gastrin and pepsin are not made in the duodenum; they are made in the intestine. Insulin is made in the pancreas. Cholecystokinin is released in the duodenum but in response to high fat content. Reflect on Process • Did your problem-solving process lead you to the correct answer? If not, where did the process break down or lead you astray? How can Version 1
85
you revise your approach to produce a more desirable result? o This question asked you to apply your knowledge about a hormone secreted in response to acid. If you got the correct answer, great job! If you got an incorrect answer, where did the process break down? Did you think that insulin, gastrin, or pepsin were released into the duodenum? Did you think that cholecystokinin was released in response to low pH? 43) E 44) D 45) D 46) C
Version 1
86
Clarify Question • What is the key concept addressed by the question? o The question asks about the digestion of fats. • What type of thinking is required? o You are being asked to apply your knowledge about the digestion of fats. Gather Content • What do you know about the digestion of fats? What other information is related to the question? o French fries and ice cream both contain lots of fat (panel C). French fries also contain starch (panel B). Panel A is the digestion of proteins into amino acids that are absorbed into the blood stream. Panel B is the digestion of starch into glucose, which is also absorbed into the blood stream. Panel C is the digestion of fats, which are packaged into chylomicrons and secreted into the intestinal lymph. Choose Answer • Given what you now know, what information is most likely to produce the correct answer? o Given that both French fries and ice cream contain fat and panel C shows the digestion of fats, the best answer is panel C. Reflect on Process • Did your problem-solving process lead you to the correct answer? If not, where did the process break down or lead you astray? How can you revise your approach to produce a more desirable result? o This question asked you to apply your knowledge about the digestion of fats. If you got the correct answer, great job! If you got an incorrect answer, where did the process break down? Did you think that ice cream and French fries both contain a lot of protein or starch? Did you think that panel C was the digestion of protein or starch?
47) B
Version 1
87
Clarify Question • What is the key concept addressed by the question? o The question is asking which type of food most greatly affects the release of insulin. • What type of thinking is required? o You are being asked to evaluate three different scenarios as to their impact on insulin release. Gather Content • What do you know about the release of insulin? What other information is related to the question? o Insulin is released in the body in response to an increase in blood glucose levels. Eating a meal higher in carbohydrates would trigger more insulin to be released into the bloodstream. Choose Answer • Given what you now know, what information is most likely to produce the correct answer? o Panel I represents a meal higher in proteins (with the blue balls representing amino acids), and so this meal would be less likely to cause the body to release insulin. Likewise, panel II represents a meal high in fatty acids(with the larger yellow globules representing fatty acids), and so this meal would be less likely to cause the body to release insulin. The hexagon structures in panel III represent a string of glucose molecules. A meal high in these carbohydrates would trigger the greatest release of insulin in the body. Reflect on Process • Did your problem-solving process lead you to the correct answer? If not, where did the process break down or lead you astray? How can you revise your approach to produce a more desirable result? o This question asked you to evaluate three different meals and predict which would trigger the greatest release of insulin. If you got the correct answer, great job! If you got an incorrect answer, where did the process break down? Did you incorrectly identify the contents of the different meals in the panels? Did you forget that insulin is associated with the digestion of glucose molecules, and so the greatest amount of insulin would be release in response to meals higher in carbohydrates (glucose molecules)?
48) B 49) A 50) D Version 1
88
51) D 52) D
Version 1
89
Clarify Question What is the key concept addressed by the question? • This question is asking you to identify the components of Fick’s law of diffusion. What type of thinking is required? • Apply level: o You are being asked to take what you already know and use, or apply, it to the understanding of how vertebrate structures factor into Fick’s law of diffusion. Gather Content What do you already know about gas exchange? What other information is related to the question? • Alveoli, the sites of gas exchange in mammalian lungs, are sacs that are surrounded by capillaries. • Lamellae are extensions in fish gills where gas exchange occurs. • Papulae, or skin gills, in echinoderms are extensions of the body that allow gas exchange to occur. • The tracheal system of insects is a series of tubes that facilitate gas exchange. Choose Answer Given what you now know, what information is most likely to produce the correct answer? • Alveoli, lamellae, paulae, and tracheal systems all provide extra surface area over which gas exchange can occur. • The area over which gas exchange occurs is represented by “A” in the formula for Fick’s law of diffusion. Reflect on Process Version 1
90
Did your problem-solving process lead you to the correct answer? If not, where did the process break down or lead you astray? How can you revise your approach to produce a more desirable result? • Apply level: o Answering this question correctly depended on your ability to use vertebrate structures in a new situation. If you got an incorrect answer, did you remember that each of the mentioned structures increases surface area, or that surface area is a component of Fick’s law of diffusion? Did you have trouble extending the nature of the vertebrate structures to determine the correct answer? 53) A 54) D 55) A 56) E 57) E
Version 1
91
Clarify Question What is the key concept addressed by the question? o This question is asking you to interpret oxygen dissociation curves. What type of thinking is required? o This question is asking you to identify key factors that qualify something as hemoglobin or myoglobin. Gather Content What do you already know about myoglobin and hemoglobin? What other information is related to the question? • As altitude increases in mountainous regions, available oxygen levels decrease. • Myoglobin is found in muscle cells. • The affinity of myoglobin for oxygen is much higher than that of hemoglobin. • When hemoglobin has been depleted of oxygen, in the same circumstance myoglobin will still have oxygen. Choose Answer Given what you now know, what information is most likely to produce the correct answer? • This graph shows oxygen saturation increasing on the Y axis, and available oxygen available increasing on the X axis. • When comparing the three curves, example A has the highest oxygen saturation (60%) at PO2 of 20 mmHg. This is characteristic of myoglobin. • Example B has a moderate amount of oxygen saturation (approximately 25%)at PO2 of 20 mmHg. This is characteristic of hemoglobin, and since this is a moderate value for these three curves, it would represent the llamas in a mountainous environment. Organisms such as the llamas in this example have less oxygen available, so they may have increased RBC counts to deliver more oxygen to the tissues. • Example C has the lowest oxygen saturation (approximately 10%) at PO2 of 20 mmHg. This is characteristic of hemoglobin in an organism does not live at an extreme altitude. Reflect on Process Did your problem-solving process lead you to the correct answer? If not, where did the process break down or lead you astray? How can you revise your approach to produce a more desirable result? o Answering this question correctly depended not only on distinguishing between hemoglobin and myoglobin, but also on your ability to break down, or analyze, oxygen dissociation curves. If you got an incorrect answer, did you remember that components of hemoglobin include a lower affinity for oxygen than myoglobin, or that atmospheric pressure can affect oxygen saturation? Did you have trouble breaking down the differences between myoglobin and hemoglobin to Version 1
92
determine the correct answer?
58) B
Version 1
93
Clarify Question What is the key concept addressed by the question? o This question is asking you to identify the components of Fick’s law of diffusion. What type of thinking is required? o You are being asked to take what you already know and use, or apply, it to the understanding of how vertebrate structures factor into Fick’s law of diffusion What type of thinking is required? Gather Content What do you already know about diffusion? What other information is related to the question? • Diffusion is the movement of a substance from high concentration to low concentration. • The greater the difference between the area of high concentration and low concentration, the faster diffusion occurs. • In the gas exchange that occurs to sustain life, oxygen diffuses from an area of high concentration in the environment to an area of low concentration in the organism. Choose Answer Given what you now know, what information is most likely to produce the correct answer? • Ciliated gills in clams place blood near water, where oxygen can be taken up. • The continual swimming with an open mouth exhibited by sharks ensures that a steady stream of water with a relatively high oxygen content will flow past the gills, which contain blood with a lower oxygen Version 1
94
content. • These adaptations would maximize delta p, which is the difference in pressure between the environment and the location of gas exchange. • As delta p increases, gas exchange occurs more rapidly. Reflect on Process Did your problem-solving process lead you to the correct answer? If not, where did the process break down or lead you astray? How can you revise your approach to produce a more desirable result? o Answering this question correctly depended on your ability to use vertebrate structures in a new situation. If you got an incorrect answer, did you remember that each of the mentioned structures maximize the difference between blood oxygen levels and the oxygen levels of the environment, or that the difference in these levels is a component of Fick’s law of diffusion? Did you have trouble extending the nature of the vertebrate structures to determine the correct answer? 59) C
Version 1
95
Clarify Question What is the key concept addressed by the question? o This question is asking you to recognize the atmospheric conditions on Earth. What type of thinking is required? o This is an analyze question because you have to break atmospheric gas composition and atmospheric pressure into their component pieces to understand how they function. Gather Content What do you already know about earth’s atmosphere? What other information is related to the question? • Dry air on Earth is 20.95% oxygen. • Barometric pressure at sea level on Earth is 760 mmHg. Choose Answer Given what you now know, what information is most likely to produce the correct answer? • The oxygen content and atmospheric pressure of the planet in this question are not identical to the conditions found on Earth, but they are similar. Reflect on Process Did your problem-solving process lead you to the correct answer? If not, where did the process break down or lead you astray? How can you revise your approach to produce a more desirable result? o Answering this question correctly depended not only on distinguishing between oxygen content and atmospheric pressure, but on your ability to analyze these conditions in a novel setting. If you got an Version 1
96
incorrect answer, did you remember that components of the oxygen content of Earth’s atmosphere is 20.95% or that atmospheric pressure at sea level is 760 mmHg? Did you have trouble breaking down the conditions on Earth to determine the correct answer? 60) D 61) C 62) C
Version 1
97
Clarify Question What is the key concept addressed by the question? o This question is asking you to extrapolate the effects of an injury to the chest. What type of thinking is required? o You are being asked to take what you already know and apply it to predict the consequences of an injury. Gather Content What do you already know about the respiratory passages? What other information is related to the question? • Air travels from the nose and/or mouth down the pharynx and larynx, and into the trachea. The trachea splits into the left and the right bronchus, which carry air separately into the lungs. • Broken ribs can result in the puncture of a lung if the ribs are pushed inward during trauma. Choose Answer Given what you now know, what information is most likely to produce the correct answer? • If Tim’s right chest wall has been compromised by the accident, collapse of his right lung could occur. Unless the trauma went through to the left side of the chest, his left lung would be unaffected. Overinflation of the lungs would not occur. Reflect on Process Did your problem-solving process lead you to the correct answer? If not, where did the process break down or lead you astray? How can you revise your approach to produce a more desirable result? o Answering this question correctly depended on your ability to use the Version 1
98
anatomy of the lungs in a new situation. If you got an incorrect answer, did you remember that the function of one lung is not dependent on the function of the other lung or that an injury to one lung can result in the collapse of that lung? Did you have trouble extending the anatomy of the lungs to determine the correct answer? 63) C
Version 1
99
Clarify Question What is the key concept addressed by the question? o This question is asking you to identify the specifics of pressure changes in frog respiration. What type of thinking is required? o This is an analyze question because you have to break the mechanics of amphibian respiration and corresponding pressure changes into their component pieces to understand how they function. Gather Content What do you already know about respiration in a frog? What other information is related to the question? • Frogs breathe by positive pressure breathing. • Positive pressure breathing involves forcing air into the lungs by pushing air from the mouth into the lungs. • Elastic recoil of the lung tissue pushes air back out of lungs, and the pressure drops as air leaves the lungs. • This breathing can be seen externally as the underside of the frog’s mouth and throat moves with each breath. Choose Answer Given what you now know, what information is most likely to produce the correct answer? • To initiate a breath, a frog increases pressure to force air into the lungs. • The lungs expand as air is forced in. • Pressure decreases as the lungs release air. • Air pressure in the lungs thus increases steadily, then decreases steadily. The pressure is never negative. Reflect on Process Did your problem-solving process lead you to the correct answer? If not, where did the process break down or lead you astray? How can you revise your approach to produce a more desirable result? o Answering this question correctly depended not only on distinguishing between frog respiration and the respiration of other animals, but on your ability to break down, or analyze, the mechanics of frog respiration. If you got an incorrect answer, did you remember that components of frog respiration include positive pressure changes or that frog respiratory pressure changes are not negative? Did you have trouble breaking down the mechanics of frog breathing to determine the correct answer?
64) B Version 1
100
65) B 66) E 67) A
Version 1
101
Clarify Question What is the key concept addressed by the question? o This question is asking you to compare the gills of an axolotl to the gills of a fish. What type of thinking is required? o You are being asked to take what you already know and apply it to the efficiency of axolotl gills. Gather Content What do you already know about axolotl gills? What other information is related to the question? • Axolotl gills are located around the neck of the animal. • These gills must be moved constantly to ensure that gas exchange will occur. • Axolotl gills are highly branched, and this external branching creates drag on the animal and slows its locomotion. • The branching of gills creates multiple directions of water flow over the gills. Choose Answer Given what you now know, what information is most likely to produce the correct answer? • While axolotls do move relatively slowly, some fish swim slowly as well. • Axolotl gills are highly branched, but they are not covered with a thick layer of cells. • Blood flow through axolotl gills is not slower than blood flow through fish gills. • Axolotl gill efficiency is lowered by the lack of consistent flow of water over the gill structures. Version 1
102
Reflect on Process Did your problem-solving process lead you to the correct answer? If not, where did the process break down or lead you astray? How can you revise your approach to produce a more desirable result? o Answering this question correctly depended on your ability to use axolotl gill structure in a new situation. If you got an incorrect answer, did you remember that axolotl gills are external or that axolotls must move water over their gills for gas exchange to occur? Did you have trouble extending the anatomy of axolotl gills to determine the correct answer? 68) D 69) B 70) D
Version 1
103
Clarify Question What is the key concept addressed by the question? • This question is asking you to explain the regulation of breathing. What type of thinking is required? Choose from the following samples to streamline your writing: o You are being asked to take what you already know and apply it to the involuntary initiation of breathing. Gather Content What do you already know about respiration? What other information is related to the question? • A normal child that voluntarily holds his or her breath does not suffocate, as breathing resumes within a few seconds. • The respiratory control center stimulates an individual to take a breath. • As carbon dioxide accumulates in the blood, the pH of the blood decreases. • A decrease in the pH of the blood is detected by chemoreceptors, which in turn stimulate the respiratory control center to initiate a breath. Choose Answer Given what you now know, what information is most likely to produce the correct answer? • The diaphragm and intercostal muscles are skeletal muscles, not smooth muscles, and are therefore under voluntary control. • It is changing carbon dioxide levels, not changing oxygen levels, that stimulate the respiratory control center. • As Sammy holds his breath, carbon dioxide would build up, and pH would decrease. Decreased pH would stimulate the respiratory control center and Sammy would take a breath. Version 1
104
Reflect on Process Did your problem-solving process lead you to the correct answer? If not, where did the process break down or lead you astray? How can you revise your approach to produce a more desirable result? o Answering this question correctly depended on your ability to use the regulation of breathing in a new situation. If you got an incorrect answer, did you remember that breathing is stimulated by the respiratory control center or that increased carbon dioxide lowers pH and stimulates the respiratory control center? Did you have trouble extending the regulation of breathing to determine the correct answer? 71) D 72) B 73) A
Version 1
105
Clarify Question What is the key concept addressed by the question? • The question is asking you to apply what you know about fish respiration a new situation. What type of thinking is required? o You are being asked to take what you already know apply, it to the reversal of water flow over fish gills. Gather Content What do you already know about fish gills? What other information is related to the question? • Water flows from the gill arch over the gill filaments. • The direction of water is the opposite of the direction of blood flow. • The countercurrent nature of the water and blood flow allows efficient gas exchange to occur, since equilibrium of oxygen levels cannot be reached. Choose Answer Given what you now know, what information is most likely to produce the correct answer? • If water flow was reversed so that water and blood ran in the same direction in a fish gill, diffusion of oxygen would stop when the water and blood reached equal oxygen levels, which would be 50% oxygen in each. Once that point is reached, not further diffusion of oxygen would occur. Reflect on Process Did your problem-solving process lead you to the correct answer? If Version 1
106
not, where did the process break down or lead you astray? How can you revise your approach to produce a more desirable result? o Answering this question correctly depended on your ability to use the principles of fish respiration in a new situation. If you got an incorrect answer, did you remember that the opposing nature of water flow in fish gills is necessary to maximize gas exchange or that changing the direction of water flow would remove this advantage? Did you have trouble extending the principles of fish respiration to determine the correct answer? 74) A
Version 1
107
Clarify Question What is the key concept addressed by the question? • This question is asking you to identify the variable that affects oxygen diffusion. What type of thinking is required? o This is an analyze question because you have to break diffusion and the composition of blood into their component pieces to understand how they function. Gather Content What do you already know about diffusion? What other information is related to the question? • Diffusion is a passive process that is driven by difference in concentration between two regions. • The greater the difference in concentrations between the two regions, the faster diffusion will occur. • Diffusion is the movement of a substance from high to low concentration. • Oxygen diffuses from the alveoli of the lungs into the blood. • Hemoglobin binds with oxygen and carries it through the circulatory system to the tissues. Choose Answer Given what you now know, what information is most likely to produce the correct answer? • Increasing hemoglobin would cause an increased uptake of oxygen in the alveoli. • Lower concentrations of oxygen in the blood would speed up the rate of diffusion from the alveoli. Version 1
108
Reflect on Process Did your problem-solving process lead you to the correct answer? If not, where did the process break down or lead you astray? How can you revise your approach to produce a more desirable result? o Answering this question correctly depended not only on distinguishing between X and Y, but on your ability to break down, or analyze, the factors that affect diffusion. If you got an incorrect answer, did you remember that hemoglobin has an affinity for oxygen or that the binding of oxygen to hemoglobin would lower the concentration of oxygen in the alveoli? Did you have trouble breaking down the factors that affect diffusion to determine the correct answer? 75) A 76) A 77) A 78) C 79) C 80) C
Version 1
109
Clarify Question What is the key concept addressed by the question? • This question is asking about the conditions that result in emphysema. What type of thinking is required? o You are being asked to take what you already know and apply it to the genetic causes of emphysema. Gather Content What do you already know about emphysema? What other information is related to the question? • Breathing requires expansion and recoil of breathing structures such as the lungs and the alveoli. • Emphysema is a disease that makes breathing very difficult due to the breakdown of the walls of alveoli. • Emphysema can be caused by a genetic mutation but is most often caused by cigarette smoking. • Emphysema results in a greater number of larger alveoli. • The lungs of someone with emphysema become less elastic, and moving air becomes much more difficult. Choose Answer Given what you now know, what information is most likely to produce the correct answer? • Hemocyanin is similar to hemoglobin but it is characteristic of mollusks. • Myoglobin and hemoglobin bind with oxygen but are not involved in the physical mechanics of breathing. • Similarly, carbonic anhydrase converts carbon dioxide and water into bicarbonate, but is not involvedin the mechanics of breathing. Version 1
110
• Collagen is a critical protein for the structure and function of the lungs. • Proteases are enzymes that break down protein, so elastin-specific protease would break down elastic proteins and would contribute to emphysema. This enzyme would need to be inhibited to prevent these changes from occurring. Reflect on Process Did your problem-solving process lead you to the correct answer? If not, where did the process break down or lead you astray? How can you revise your approach to produce a more desirable result? o Answering this question correctly depended on your ability to use the nature of normal lung structure in a new situation. If you got an incorrect answer, did you remember that the structure of lungs is dynamic or that emphysema results in a loss of this elasticity? Did you have trouble extending the nature of lung structure to determine the correct answer? 81) C
Version 1
111
Clarify Question What is the key concept addressed by the question? • This question is asking about whether humans can survive the lack of oxygen for any period of time. What type of thinking is required? o You are being asked to take what you already know and use, or apply, it to a situation in which oxygen deprivation occurs for a short time. Gather Content What do you already know about oxygen transport? What other information is related to the question? • Blockage of the trachea prevents oxygen from reaching the lungs. • As the final electron acceptor in cellular respiration, oxygen is critical for survival. • Hemoglobin carries oxygen to the tissues from the lungs. • At rest, most of the oxygen that is bound to hemoglobin remains in the blood. Choose Answer Given what you now know, what information is most likely to produce the correct answer? • Myoglobin binds with oxygen and stores it muscle tissue. This oxygen is used within muscle but is not released back into the blood. • Although increased carbon dioxide in the blood does lower pH, the effects of this change would take longer to become fatal than lack of oxygen. • The majority of oxygen does remain bound to hemoglobin, which creates a limited reserve if breathing is interrupted. Version 1
112
Reflect on Process Did your problem-solving process lead you to the correct answer? If not, where did the process break down or lead you astray? How can you revise your approach to produce a more desirable result? o Answering this question correctly depended on your ability to use the nature of hemoglobin in a new situation. If you got an incorrect answer, did you remember that hemoglobin carries oxygen in the blood or that hemoglobin does not immediately release all of the oxygen that it carries? Did you have trouble extending the nature of hemoglobin to determine the correct answer? 82) E 83) C 84) C 85) D 86) C 87) B 88) B 89) A 90) B
Version 1
113
Clarify Question What is the key concept addressed by the question? • This question is asking you to identify the physiological basis for sustained running. What type of thinking is required? o You are being asked to take what you already know and use, or apply, it to explain the sustained running speed of a large mammal. Gather Content What do you already know about oxygen use? What other information is related to the question? • Cellular respiration provides the energy that is necessary for all activities, including running. • Oxygen is necessary for cellular respiration to occur. • Oxygen binds to hemoglobin and is transported to cells through the circulatory system. • Myoglobin binds to oxygen and stores in muscle tissue. Choose Answer Given what you now know, what information is most likely to produce the correct answer? • A smaller lung volume and a smaller trachea would reduce the available oxygen and subsequently the potential for more cellular respiration. • A lower concentration of myoglobin would also reduce available oxygen. • Bicarbonate is a molecule that transports carbon dioxide; it is not involved in making oxygen available for cellular respiration. • A higher concentration of hemoglobin would increase the amount of Version 1
114
oxygen that could be transported to cells and thereby the amount of energy available for activity. Reflect on Process Did your problem-solving process lead you to the correct answer? If not, where did the process break down or lead you astray? How can you revise your approach to produce a more desirable result? o Answering this question correctly depended on your ability to use the nature of energy that is available for use in a new situation. If you got an incorrect answer, did you remember that oxygen is required to produce usable cellular energy or that hemoglobin carries oxygen? Did you have trouble extending the requirements for cellular respiration to determine the correct answer? 91) A 1 meter (m) = 1,000 millimeters (mm). Thus, 1 m 2 = 1 x 10 6 mm 2. Substitution of square millimeters for square meters results in 40 x 10 6 mm 2 ÷ 300 x 10 6 alveoli = 0.133 mm 2/alveolus.
Version 1
115
Clarify Question What is the key concept addressed by the question? • This question is asking you to calculate the surface area of an alveolus. What type of thinking is required? o This is an analyze question because you have to break alveoli and surface area into their component pieces to understand how they function. Gather Content What do you already know about alveoli? What other information is related to the question? • Alveoli are small sacs within the lungs that are surrounded by capillaries. • Alveoli are surrounded by capillaries, and provide the surface area over which gas exchange occurs. Choose Answer Given what you now know, what information is most likely to produce the correct answer? • If the total surface area is 40 m^2, and the number of alveoli is 300,000,000, the surface area of one alveolus is 40 m^2/300,000,00 alveoli, which is 0.000000133 m^2/alveolus. • 0.000000133 m^2 is equal to 0.133 square micrometers. Reflect on Process Did your problem-solving process lead you to the correct answer? If not, where did the process break down or lead you astray? How can you revise your approach to produce a more desirable result? Version 1
116
o Answering this question correctly depended not only on distinguishing between alveoli and lungs, but on your ability to break down, or analyze, surface area. If you got an incorrect answer, did you remember that components of alveoli include their role as small units of function within the lung or that their surface area is related to the surface area of the lungs? Did you have trouble breaking down the idea of surface area to determine the correct answer? 92) A
Version 1
117
Clarify Question What is the key concept addressed by the question? • The question is asking you to describe, from most superficial to most deep, the layers of the thoracic cavity and lung. What type of thinking is required? o You are being asked to take what you already know and use, or apply, it to describe the structure of the lung and the surround chest cavity. Gather Content What do you already know about the lungs? What other information is related to the question? • The intercostal muscles are found in association with the ribcage. • Parietal refers to membrane that lines a cavity. • Lungs sit within the pleural cavity. • Visceral membrane is membrane that directly covers an organ. Choose Answer Given what you now know, what information is most likely to produce the correct answer? • A surgeon would first have to cut through the skin. • Recall that the lungs are protected by the ribs, so the intercostal muscles, would be the next layer that would be encountered. • The lungs are housed in the pleural cavity, and the outer boundary of that cavity is the parietal pleural membrane. • The inner layer of the pleural cavity, which is also the layer that touches the lung, is the visceral pleural membrane. Reflect on Process Did your problem-solving process lead you to the correct answer? If Version 1
118
not, where did the process break down or lead you astray? How can you revise your approach to produce a more desirable result? o Answering this question correctly depended on your ability to use the anatomy of the lungs and the surrounding area in a new situation. If you got an incorrect answer, did you remember that the lungs are surround by the ribcage and intercostal muscles or that the pleural cavity has parietal pleura as its outer boundary and visceral pleural as its inner membrane? Did you have trouble extending the anatomical features of the lungs and their surrounding tissue to determine the correct answer? 93) B 94) C 95) A
Version 1
119
Clarify Question What is the key concept addressed by the question? • This question is asking about the environmental influence on the release of oxygen by hemoglobin. What type of thinking is required? o You are being asked to take what you already know and use, or apply, it to hemoglobin and its tendency to release oxygen. Gather Content What do you already know about oxygen levels? What other information is related to the question? • Diffusion is the movement of a substance from high to low concentration. • As the difference between high and low concentration increases, the rate of diffusion increases. • When blood leaves the alveoli, the PO2 is 100mmHg. • The PO2 of the tissues of the body is 40mmHg. Choose Answer Given what you now know, what information is most likely to produce the correct answer? • Since 40 mmHg is the lowest value, it creates the largest concentration gradient, which would facilitate the greatest movement of oxygen. Reflect on Process Did your problem-solving process lead you to the correct answer? If not, where did the process break down or lead you astray? How can you revise your approach to produce a more desirable result? o Answering this question correctly depended on your ability to use the nature of hemoglobin in a new situation. If you got an incorrect answer, Version 1
120
did you remember that the PO2 of blood leaving the alveoli is 100 mmHg or that typical cells have a PO2 of 40 mmHg? Did you have trouble extending the nature of hemoglobin to determine the correct answer? 96) C 97) E 98) B 99) B 100) A 101) A
Version 1
121
Clarify Question What is the key concept addressed by the question? • This question is asking about the inefficiency of an open circulatory system. What type of thinking is required? o You are being asked to take what you already know and use, or apply, it to the lack of efficiency of an open circulatory system. Gather Content What do you already know about open circulatory systems? What other information is related to the question? • An open circulatory system consists of a beating heart and blood vessels. • Rather than entering capillaries, blood in an open circulatory system leaves the vessels and bathes the tissues generally. • Blood and the extracellular fluid mix in an open circulatory system and are together referred to as hemolymph. Choose Answer Given what you now know, what information is most likely to produce the correct answer? • Open circulatory systems do have hemolymph that has respiratory pigments, so that answer can be dismissed. • Some of the functions of blood in any circulatory system, including an open system, is to transport nutrients and to remove waste. • A heart is present in an open circulatory system. • A characteristic of an open circulatory system that does lead to inefficiency is that blood cannot be delivered to specific tissues; rather it is delivered to sinuses, where it bathes multiple tissues at once. Version 1
122
Reflect on Process Did your problem-solving process lead you to the correct answer? If not, where did the process break down or lead you astray? How can you revise your approach to produce a more desirable result? o Answering this question correctly depended on your ability to use the characteristics of an open circulatory system in a new situation. If you got an incorrect answer, did you remember that the hart drives theblood into vessels and then into open sinuses or that the blood is still used for transport and waste removal in open systems? Did you have trouble extending the features of an open circulatory system to determine the correct answer? 102) B
Version 1
123
Clarify Question What is the key concept addressed by the question? • This question is asking about the function of erythopoietin. What type of thinking is required? o You are being asked to take what you already know and use, or apply, it to the role of erythropoietin in athletic ability. Gather Content What do you already know about erythropoietin? What other information is related to the question? • Red blood cells are known as erythrocytes. • Erythrocytes carry oxygen to the tissues of the body. • Oxygen is used in the last stage of cellular respiration. • Cellular respiration makes energy available for activity by converting the stored energy in nutrients into a usable form. • A plasma protein is converted into erythropoietin in the kidneys. • Erythropoietin stimulates the red bone marrow to make erythrocytes. • Erythrocytes are made from stem cells. Choose Answer Given what you now know, what information is most likely to produce the correct answer? • The only function of erythropoietin is to stimulate the production of new red blood cells. • This would increase the amount of oxygen that can be carried to cells and would potentially increase cellular respiration and available energy. • Erythropoietin does not act outside of the production of blood, so although the other answers might accelerate oxygen delivery to cells, the only correct response is that erythropoietin stimulates the production of Version 1
124
red blood cells. Reflect on Process Did your problem-solving process lead you to the correct answer? If not, where did the process break down or lead you astray? How can you revise your approach to produce a more desirable result? o Answering this question correctly depended on your ability to use the function of erythropoietin in a new situation. If you got an incorrect answer, did you remember that erythropoietin stimulates the redbone marrow or that the red bone marrow produces erythrocytes? Did you have trouble extending the function of erythropoietin to determine the correct answer? 103) D
Version 1
125
Clarify Question What is the key concept addressed by the question? • This question is asking you to interpret a graph that shows the movement of materials in and out of the blood. What type of thinking is required? o This is an analyze question because you have to break pressure and blood flow into their component pieces to understand how they function. Gather Content What do you already know about the movement of materials in and out of the circulatory system? What other information is related to the question? • A solution is made of dissolved particles (solute) and a liquid component (solvent). • Where there is more solute, there proportionally is less solvent. • Osmosis is the movement of a liquid (solvent) from high to low concentration of that liquid. • Osmotic pressure is the tendency of a solution to allow water to flow in water via osmosis. • Osmotic pressure increases as solute concentration increases and solvent concentration decreases. • Through osmosis, fluid leaves the circulatory system near the arteriole ends of capillaries and returns to the blood near the venular side of capillaries. Choose Answer Given what you now know, what information is most likely to produce the correct answer? • In region A, the osmotic pressure is lower than the blood pressure. Where osmotic pressure is lower, solute concentration is lower, and there is a higher concentration of fluid. Fluid moves from an area of high concentration to low concentration. Therefore, fluid flows out of the arterioles, where fluid concentration is higher, toward the tissues. • In region B, the opposite is true. Osmotic pressure is higher than the blood pressure in the circulatory system, which means there is proportionally less fluid. Fluid flows form high concentration to low concentration, so the net movement would be into the blood. Reflect on Process Did your problem-solving process lead you to the correct answer? If not, where did the process break down or lead you astray? How can you revise your approach to produce a more desirable result? o Answering this question correctly depended not only on distinguishing between osmotic pressure and solvent concentration but on your ability to break down, or analyze, osmosis. If you got an incorrect answer, did you remember that components of osmosis include the movement of Version 1
126
fluid from high concentration to low concentration, or that as osmotic pressure increases, fluid concentration decreases? Did you have trouble breaking down the osmotic pressure to determine the correct answer?
104) D 105) B 106) A 107) C 108) B 109) A 110) E
Version 1
127
Clarify Question What is the key concept addressed by the question? • This question is asking about the flow of blood between the lungs and the heart. What type of thinking is required? o You are being asked to take what you already know and use, or apply, it to the cause of congestive heart failure. Gather Content What do you already know about the path of blood between the heart and the lungs? What other information is related to the question? • Deoxygenated blood returns to the right atrium of the heart. • The right atrium forces blood into the right ventricle. • The right ventricle pumps deoxygenated blood to the lungs, where it can receive oxygen. • Oxygenated blood returns to the left atrium of the heart. • The left atrium pumps blood into the left ventricle. • The left ventricle pumps oxygenated blood to the tissues of the body. Choose Answer Given what you now know, what information is most likely to produce the correct answer? • The right side of Jim’s heart, including the tricuspid valve, the right ventricle, and the pulmonary semilunar valve are working normally since there is an excess of blood in the lungs. • Since an irregular heartbeat was not noted here, there is not reason to suspect the SA node is not functioning as it should. • Jim’s blood flow is backing up in his lungs, which means the heart is not pushing blood to the body fast enough. Since the left ventricle is Version 1
128
responsible for this, it may not be functioning as it should in Jim’s case. Reflect on Process Did your problem-solving process lead you to the correct answer? If not, where did the process break down or lead you astray? How can you revise your approach to produce a more desirable result? o Answering this question correctly depended on your ability to use the path of blood between the heart and lungs in a new situation. If you got an incorrect answer, did you remember that the right side of the heart is responsible for sending blood to the lungs or that the left side of the heart sends blood to the body once it has returned from the lungs? Did you have trouble extending the normal path of blood to determine the correct answer? 111) E 112) E 113) B 114) C
Version 1
129
Clarify Question What is the key concept addressed by the question? • The question is asking about the consequences of a decreased megakaryocyte count. What type of thinking is required? o You are being asked to take what you already know and use, or apply, it to the shortage of megakaryocytes. Gather Content What do you already know about megakaryocytes? What other information is related to the question? • Blood cells are produced from stem cells in the red marrow of bones. • Blood cells go through different developmental stages. • A committed cell is a cell that is on a definite, specific developmental path. • Megakaryocytes are committed cells in the bone marrow. • Megakaryocytes give rise to platelets. • Platelets play a vital role in clot formation after injury. Choose Answer Given what you now know, what information is most likely to produce the correct answer? • It is white blood cells, not megakaryocytes, that defend the body against foreign invaders. • Red blood cells, not megakaryocytes, transport oxygen. • Temperature regulation and osmotic balance are not functions of megakaryocytes. • Megakaryocytes are large cells that give rise to platelets. Since platelets are required for a blood clot, a low number of megakaryocytes Version 1
130
would result in low platelet numbers, which would impair the clotting response. Reflect on Process Did your problem-solving process lead you to the correct answer? If not, where did the process break down or lead you astray? How can you revise your approach to produce a more desirable result? o Answering this question correctly depended on your ability to use the function of megakaryocytes in a new situation. If you got an incorrect answer, did you remember that megakaryocytes produce platelets or that platelets are required for blood clotting? Did you have trouble extending the function of platelets to determine the correct answer? 115) D 116) E 117) B 118) C 119) E 120) B
Version 1
131
Clarify Question What is the key concept addressed by the question? • This question is asking about the path of blood between the heart and the lungs. What type of thinking is required? o You are being asked to take what you already know and use, or apply, it to the flow of blood between the heart and the lungs. Gather Content What do you already know about the flow of blood between the heart and the lungs? What other information is related to the question? • Deoxygenated blood returns to the right atrium of the heart. • The right atrium forces blood into the right ventricle. • The right ventricle pumps deoxygenated blood to the lungs, where it can receive oxygen. • Oxygenated blood returns to the left atrium of the heart. • The left atrium pumps blood into the left ventricle. • The left ventricle pumps oxygenated blood to the tissues of the body. Choose Answer Given what you now know, what information is most likely to produce the correct answer? • The correct path from the lungs to the heart has blood entering the heart in the left atrium. Reflect on Process Did your problem-solving process lead you to the correct answer? If not, where did the process break down or lead you astray? How can you revise your approach to produce a more desirable result? o Answering this question correctly depended on your ability to use the opath of blood between the heart and the lungs in a new situation. If you got an incorrect answer, did you remember that the lungs send blood back to the left atrium? Did you have trouble extending the path of blood between the heart and lungs to determine the correct answer?
121) C 122) B 123) D 124) A
Version 1
132
Clarify Question What is the key concept addressed by the question? • This question is asking about the function of erythripoietin. What type of thinking is required? o You are being asked to take what you already know and use, or apply, it to the need for erythropoietin. Gather Content What do you already know about erythropoietin? What other information is related to the question? • As altitude increases, the oxygen that is available to animals decreases. • Red blood cells are known as erythrocytes. • Erythrocytes carry oxygen to the tissues of the body. • A plasma protein is converted into erythropoietin in the kidneys. • Erythropoietin stimulates the red bone marrow to make erythrocytes. • Erythrocytes are made from stem cells. Choose Answer Given what you now know, what information is most likely to produce the correct answer? • The only function of erythropoietin is to stimulate the production of new red blood cells. • This would increase the amount of oxygen that can be carried to cells, which would be important at higher altitudes where oxygen is less available. Reflect on Process Did your problem-solving process lead you to the correct answer? If Version 1
133
not, where did the process break down or lead you astray? How can you revise your approach to produce a more desirable result? o Answering this question correctly depended on your ability to use the function of erythropoietin in a new situation. If you got an incorrect answer, did you remember that erythropoietin stimulates the redbone marrow or that the red bone marrow produces erythrocytes? Did you have trouble extending the function of erythropoietin to determine the correct answer? 125) A 126) D 127) D
Version 1
134
Clarify Question What is the key concept addressed by the question? • This question is asking about the asymmetry of the ventricles of a mammalian heart. What type of thinking is required? o You are being asked to take what you already know and use, or apply, it to the size difference of the right ventricle and left ventricle of a mammalian heart. Gather Content What do you already know about the mammalian heart? What other information is related to the question? • Deoxygenated blood returns to the right atrium of the heart. • The right atrium forces blood into the right ventricle. • The right ventricle pumps deoxygenated blood to the lungs, where it can receive oxygen. • Oxygenated blood returns to the left atrium of the heart. • The left atrium pumps blood into the left ventricle. • The left ventricle pumps oxygenated blood to the tissues of the body. Choose Answer Given what you now know, what information is most likely to produce the correct answer? • The force that is required to pump blood to the lungs is much less than the force needed to pump blood to the extremities of the body. • The left ventricle has a much thicker muscle layer because it pumps blood a much greater distance. Reflect on Process Did your problem-solving process lead you to the correct answer? If Version 1
135
not, where did the process break down or lead you astray? How can you revise your approach to produce a more desirable result? o Answering this question correctly depended on your ability to use the mammalian heart in a new situation. If you got an incorrect answer, did you remember that the right side of the heart pumps blood to the lungs or that the left side of the heart pumps blood much further, to the extremities of the body? Did you have trouble extending the mammalian heart to determine the correct answer? 128) B
Version 1
136
Clarify Question What is the key concept addressed by the question? • The question is asking about the effects of firing the SA and AV nodes. What type of thinking is required? o This is an analyze question because you have to break the musculature of the left and right ventricles into their component pieces to understand how they function. Gather Content What do you already know about the SA and AV nodes? What other information is related to the question? • An electrocardiogram (ECG) records the electrical activity of the heart. • The P wave shows the depolarization/contraction of the atria. • The depolarization/contraction of the ventricles is shown in the QRS wave. The repolarization of the atria is masked by this wave. • The T wave represents the repolarization of the ventricles. • A normal heartbeat requires the flow of an impulse throughout the heart. • The initiation of a heartbeat occurs in the sinoatrial (SA) node located in the right atrium. • The SA node is the pacemaker of the heart. • The SA node initiates the depolarization of the atria. • The action potential from the SA node travels via the internodal pathway to the atrioventricular node. • The impulse travels from the AV node to the bundle branches, and then to the Purkinje fibers where it is dispersed through the ventricles. • To beat efficiently, the atria contract to fill the ventricles. Version 1
137
• The ventricles are relaxed as the atria contract, which allows a maximum amount of blood to enter the ventricles. Choose Answer Given what you now know, what information is most likely to produce the correct answer? • The P wave is smaller than the QRS wave, so the QRS wave would not be masked. • As long as the atrioventricular valves are working properly, blood will not flow from the ventricle back into the atria. • If the SA and AV nodes fire at the same time, the atria and ventricles would depolarize at the same time. Since the atria normally fill the ventricles by contracting when the ventricles are relaxed, this simultaneous firing of the SA and AV nodes would result in less full ventricles and lower blood pressure. Since less blood would be present in the ventricles, stroke volume would also be lower in this case. • Nitric oxide acts to relax the walls of blood vessels when blood pressure is high, which would not be needed in this situation. Reflect on Process Did your problem-solving process lead you to the correct answer? If not, where did the process break down or lead you astray? How can you revise your approach to produce a more desirable result? o Answering this question correctly depended not only on distinguishing between the actions of the SA and AV nodes, but on your ability to break down, or analyze, the consequences of their simultaneous activity. If you got an incorrect answer, did you remember that components of firing the SA node include the depolarization of the atria, or that the firing of the AV node would likely affect cause blood to leave the ventricles before they were full? Did you have trouble breaking Version 1
138
down the actions of the SA and AV nodes to determine the correct answer? 129) D 130) B 131) B 132) B 133) C 134) E 135) B
Version 1
139
Clarify Question What is the key concept addressed by the question? • This question is asking about the nature of the heart of a fish. What type of thinking is required? o You are being asked to take what you already know and use, or apply, it to determine the metabolic advantage that is conferred by the two-chambered heart of a fish. Gather Content What do you already know about the effect of exercise on heart rate? What other information is related to the question? • The ventricle of the fish heart pumps deoxygenated blood to the gills, where it picks up oxygen. • Oxygenated blood moves from the gills through the body. • Oxygen is a required component of aerobic respiration, which occurs in the cells of the body. Choose Answer Given what you now know, what information is most likely to produce the correct answer? • The circulatory system of a fish is subject to fluctuating blood pressures in different regions, so the idea of maintaining high pressure is an answer that can be discarded. • Oxygenated and deoxygenated blood are not kept separate by the chambers of the fish heart. • Since the ventricle moves blood to the gills, it becomes oxygen-rich before it moves throughout the body. The high oxygen content allows high metabolism.
Version 1
140
Reflect on Process Did your problem-solving process lead you to the correct answer? If not, where did the process break down or lead you astray? How can you revise your approach to produce a more desirable result? o Answering this question correctly depended on your ability to use the circulatory pattern of a fish in a new situation. If you got an incorrect answer, did you remember that the ventricle of a fish heart pumps blood to the gills or that the gills allow oxygen to move into the blood? Did you have trouble extending the nature of fish blood circulation to determine the correct answer? 136) A 137) B 138) A 139) A
Version 1
141
Clarify Question What is the key concept addressed by the question? • The question is asking about the greatest limiting factor that impacts cardiac output during exercise. What type of thinking is required? o You are being asked to take what you already know and use, or apply, it to the limitations of cardiac output. Gather Content What do you already know about the limiting factors on cardiac output? What other information is related to the question? • Cardiac output is equal to the stroke volume multiplied by beats per minute. • Stroke volume is the volume of blood that is moved by a ventricle with one contraction. • With exercise, heart rate increases. • The ventricles receive blood from the atria, which contract to fill relaxed ventricles. Choose Answer Given what you now know, what information is most likely to produce the correct answer? • As heart rate increases, there is less time to fill ventricles. • As ventricles fill less, stroke volume and cardiac output decrease. Reflect on Process Did your problem-solving process lead you to the correct answer? If not, where did the process break down or lead you astray? How can you revise your approach to produce a more desirable result? Version 1
142
o Answering this question correctly depended on your ability to use cardiac output in a new situation. If you got an incorrect answer, did you remember that cardiac output is directly proportional to stroke volume or that stroke volume is the volume of blood that is moved by a ventricle? Did you have trouble extending cardiac output to determine the correct answer? 140) C 141) B 142) A 143) C 144) D 145) B 146) C
Version 1
143
Clarify Question What is the key concept addressed by the question? • This question is asking about the cause of the asymmetry of the heart. What type of thinking is required? o You are being asked to take what you already know and use, or apply, it to the external appearance of the heart. Gather Content What do you already know about the structure of the mammalian heart? What other information is related to the question? • The heart of a mammal has three layers. • The most variable layer is the middle layer, which composed of muscle. • The heart of a mammal has four chambers. • The upper chambers are the atria. • The atria push blood into the ventricles. • The lower chambers are the ventricles. • The right side of the heart pumps blood to the lungs and back. • The left side of the heart pumps blood to the extremities and back. Choose Answer Given what you now know, what information is most likely to produce the correct answer? • Since the atria on the left and right side of the heart do the same thing, their size is similar. • The aorta exits from the left ventricle, not the right ventricle. • The left ventricle is much thicker than the right ventricle, which can be seen externally and can be used to identify the sides of the heart.
Version 1
144
Reflect on Process Did your problem-solving process lead you to the correct answer? If not, where did the process break down or lead you astray? How can you revise your approach to produce a more desirable result? o Answering this question correctly depended on your ability to use the structure of a mammalian heart in a new situation. If you got an incorrect answer, did you remember that the left side of the heart sends blood to the tissues of the body or that doing so requires a thicker layer of muscle? Did you have trouble extending the structure of the mammalian heart to determine the correct answer? 147) D
Version 1
145
Clarify Question • What is the key concept addressed by the question? o The question is asking about blood flow through capillaries and arterioles. • What type of thinking is required? o You are being asked to take what you already know and use, or apply, it to the change in blood flow when there is a change in the radius of the vessel. Gather Content • What do you already know about blood flow through capillaries and arterioles? What other information is related to the question? o The rate of blood flow through vessels is governed by hydrodynamics. The smaller the cross-sectional area of a vessel, the faster the fluid moves through it. Given this, flow in the capillaries would be expected to be the fastest in the system. This would not be ideal for diffusion and is actually not the case. Although each capillary is very narrow, there are so many of them that the capillaries have the greatest total crosssectional area of any other type of vessel. o Blood loses pressure and velocity as it moves through the arterioles and capillaries, but as the cross-sectional area decreases in the venous side, velocity increases. o The narrower the vessel, the greater the frictional resistance to flow. In fact, a vessel that is half the diameter of another has 16 times the frictional resistance. o Resistance to blood flow is inversely proportional to the fourth power of the radius of the vessel. Therefore, within the arterial tree, the small arteries and arterioles provide the greatest resistance to blood flow.
Version 1
146
Choose Answer • Given what you now know, what information is most likely to produce the correct answer? o There will be greater resistance in the capillary than the arteriole. o Each time the radius halves there is a 16-fold increase in resistance. Starting with 64 micrometers, half is 32, half again is 16, and half again is 8. So there were three times that an increase of 16-fold should be counted. (16× 3 = 48) Reflect on Process • Did your problem-solving process lead you to the correct answer? If not, where did the process break down or lead you astray? How can you revise your approach to produce a more desirable result? o Answering this question correctly depended on your ability to use information about the circulation system in a new situation. If you got an incorrect answer, did you remember that blood flow in smaller vessels has greater resistance? 148) C
Version 1
147
Clarify Question • What is the key concept addressed by the question? o This question is asking about the composition of hemoymph. • What type of thinking is required? o You are being asked to take what you already know and use, or apply, it to the nature of hemolymph. Gather Content • What do you already know about hemolymph? What other information is related to the question? o Hemolymph is a combination of fluid that circulates and extracellular fluid. o Extracellular fluid contains proteins, ions, various macromolecules, and water. o Insects have an open circulatory system that moves hemolymph to sinuses that allow the hemolymph and its contents to directly contact tissue. Choose Answer • Given what you now know, what information is most likely to produce the correct answer? o Ions, sugar, water, and amino acids are materials that are found in and around cells. They are present in hemolymph. o Insects do have immune cells in their hemolymph. o Insects do not have red blood cells (or respiratory pigments), as they use their tracheal system (and not their open circulatory system) to transport oxygen and carbon dioxide directly to and from cells, respectively. o The absence of respiratory pigments results in the pale yellow color of the hemolymph. Version 1
148
Reflect on Process • Did your problem-solving process lead you to the correct answer? If not, where did the process break down or lead you astray? How can you revise your approach to produce a more desirable result? o Answering this question correctly depended on your ability to use the composition of hemolymph in a new situation. If you got an incorrect answer, did you remember that hemolymph is not used to transport oxygen or that hemolymph contains the macromolecules that are characteristic of extracellular fluid? Did you have trouble extending hemolymph to determine the correct answer? 149) B
Version 1
149
Clarify Question • What is the key concept addressed by the question? o The question is asking about the location of oxygenated blood in a frog that is not using lungs to breathe. • What type of thinking is required? o You are being asked to take what you already know and use, or apply, it to the location of the highest level of oxygenation in the blood of a frog. Gather Content • What do you already know about the path of blood through an amphibian? What other information is related to the question? o The heart of an amphibian has three chambers; two atria and one ventricle. o The right atrium receives deoxygenated blood from the body. o The left atrium receives oxygenated blood from the lungs. o The ventricle pumps deoxygenated blood to the lungs and oxygenated blood to the body. o Frogs can also breathe cutaneously. They take in oxygen and release carbon dioxide through capillaries that are near their thin, permeable skin. Choose Answer • Given what you now know, what information is most likely to produce the correct answer? o If oxygen enters the systemic circulation through the skin, it would follow the established systemic circuit. o The only difference in the normal circulatory pattern would have to do with the oxygenation of the blood. Normally when blood returns from the body, it has been depleted of oxygen, but if cutaneous breathing Version 1
150
occurs, the blood returning to the heart would be oxygenated. o In the systemic circuit, blood returns from the body and enters the right atrium of the heart. Reflect on Process • Did your problem-solving process lead you to the correct answer? If not, where did the process break down or lead you astray? How can you revise your approach to produce a more desirable result? o Answering this question correctly depended on your ability to use the circulatory pattern of an amphibian in a new situation. If you got an incorrect answer, did you remember that blood return to the right atrium of the heart or that if the frog is breathing cutaneously, the blood returning to the heart from the body would be oxygenated? Did you have trouble extending the circulatory pattern of a frog to determine the correct answer?
Version 1
151
CHAPTER 35 TRUE/FALSE - Write 'T' if the statement is true and 'F' if the statement is false. 1) Hormones must be present at high concentrations to elicit an effect. ⊚ ⊚
2)
true false
A molecule can act as both a neurotransmitter and a hormone. ⊚ ⊚
true false
3) The brain controls organ system function only through direct contact with sympathetic or parasympathetic nerves. ⊚ ⊚
true false
4) If Ca +2 levels in the blood are too low, hormones from the parathyroid gland can induce bones to dissolve and release calcium. ⊚ ⊚
true false
MULTIPLE CHOICE - Choose the one alternative that best completes the statement or answers the question. 5) A(n) __________ is a regulatory molecule that is secreted into the blood by an endocrine gland. A) antibody B) hormone C) enzyme D) G protein E) neurotransmitter
Version 1
1
6) Imagine that you are a professor supervising first-year medical students. One student is examining the adrenal gland of his cadaver. "Wow!" he exclaims, "This person must have had an endocrine problem because their adrenal gland doesn't have a duct. Their adrenal hormones would have no way to get into the bloodstream." How do you respond? A) "Looks like they had an abnormal adrenal gland that was more similar to an exocrine organ." B) "Their abnormal adrenal gland probably led to adrenal insufficiency symptoms such as low body fat stores and muscle degeneration." C) "That's normal. Endocrine glands secrete hormones into the extracellular fluid where they diffuse into surrounding blood capillaries. D) "Look carefully and you will see the nerves that transport the hormones into the bloodstream."
7) The __________ lobe of the pituitary gland appears fibrous because it contains axons originating from hypothalamic neurons. A) anterior B) posterior C) superior D) inferior E) medial
8) There is a molecule in black licorice that inhibits an enzyme which normally limits activation of mineralocorticoid receptors. Thus, overconsumption of licorice can mimic the effect of excess aldosterone. What would be the result? A) Infertility B) Hypertension due to insufficient potassium in the blood C) Excessive body hair D) Low blood pressure due to excessive potassium in the blood
9) A novel disease causes destruction of the adrenal glands. If a patient with this disease failed to receive aldosterone replacement therapy, what would result? Version 1
2
A) Death B) Diminished cognitive function C) Infertility in both men and women D) Poor blood sugar regulation
10)
The islets of Langerhans located in the __________ secrete both insulin and glucagon. A) liver B) pituitary C) hypothalamus D) kidney E) pancreas
11) Oxytocin is a peptide hormone that causes uterine contractions in childbirth and stimulates milk secretion from nipples. Who produces natural oxytocin? A) Female mammals B) Female and male mammals C) Female invertebrates D) Female and male invertebrates E) Female vertebrates
12) Women often receive synthetic oxytocin to induce childbirth or augment contractions. What is an important consideration about the use of synthetic oxytocin? A) Synthetic oxytocin does not perfectly mimic the chemical structure of natural oxytocin, so it may act on other receptors and cause side effects. B) Synthetic oxytocin does not travel along motor neurons as quickly as natural oxytocin, so it is less effective than the natural form. C) It can bedifficultto get the levels just right, so uterine contractions can be toostrong. D) Synthetic G protein must also be given or the synthetic form of oxytocin will be ineffective.
Version 1
3
13) A couple visits a fertility clinic to determine the basis of their difficulty in conceiving a child. They are surprised to hear they will both be tested for levels of two of the same hormones, which play roles in the ovarian cycle and sperm production. Which hormones are they? A) Adrenocorticotropic hormone (ACTH) and corticotropin-releasing hormone (CRH) B) Insulin and glucagon C) Oxytocin and luteinizing hormone (LH) D) Follicle-stimulating hormone (FSH) and luteinizing hormone (LH)
14) The use of bovine growth hormone (BGH) to increase milk yield in dairy cows is highly controversial. What other hormone is regularly added to milk, yet causes little controversy? A) Vitamin A B) Vitamin D C) MSG D) Preservatives E) Estrogen
15)
What is the primary function of the hormone ADH? A) Regulates biological rhythms B) Mediates adaptation to long term stress C) Stimulates bone breakdown D) Conserves water by stimulating its reabsorption from urine E) Inhibits loss of calcium from bone
16)
What is a primary function of growth hormone?
Version 1
4
A) Initiates stress response B) Raises blood glucose level C) Lowers blood glucose level D) Stimulates bone growth E) Inhibits loss of calcium from bone
17)
What is a primary function of the hormone calcitonin? A) Initiates stress response B) Raises blood glucose level C) Lowers blood glucose level D) Stimulates intestinal absorption of calcium E) Stimulates uptake of calcium from blood into bones
18) Lampreys completely lack parathyroid glands. Why night these glands may be important for other vertebrates, but not for jawless fish? A) Because they are parasitic, lampreys get plenty of calcium from their hosts. B) Living in the ocean, they are surrounded by large amounts of sodium, chloride, and magnesium ions. C) Jawless fishemploy a different system than other vertebrates to regulate blood sugar. D) Since they have acartilaginous skeleton but not true bone, they have less need to regulatecalcium levels.
19)
What hormone helps with adaptation to long term stress? A) Glucocorticoids B) Ephinephrine C) Insulin D) Glucagon E) Melatonin
Version 1
5
20)
What hormone lowers blood glucose levels? A) Glucocorticoids B) Ephinephrine C) Insulin D) Glucagon E) Melatonin
21)
What hormone raises blood glucose levels? A) Glucocorticoids B) Ephinephrine C) Insulin D) Glucagon E) Melatonin
22) At the breakfast table, your roommate says, "Oops! I forgot to take my melatonin last night. I'd better take it now." How should you respond? A) "Good idea—you want to keep your bones strong." B) "Be sure notto take too much, or your blood sugar could get dangerously low." C) "Don't take it now - it regulates biological rhythms, and is normally high at night and low during the day." D) "Don't takeit after a meal - it promotes the 'fight-or-flight' response, not'rest-anddigest'."
23)
What hormone sets the body's basal metabolic rate?
Version 1
6
A) Estrogen B) Insulin C) Norepinephrine D) Cortisol E) Thyroxine
24) The hormones epinephrine and norepinephrine constitute the "alarm" response of the body to stress. What organ secretes these hormones? A) Pancreas B) Thyroid gland C) Adrenal medulla D) Pineal gland E) Anterior pituitary gland
25)
What is an example of a molecule that can act as both a neurotransmitter and a hormone? A) Estrogen B) Progesterone C) Thyroxine D) Epinephrine E) Insulin
26) Which of the following hormones exerts its action by interacting with a receptor at the cell surface? A) Insulin B) Cortisol C) Testosterone D) Estrogen E) Progesterone
Version 1
7
27)
What hormone is not produced by the anterior pituitary gland? A) Prolactin B) Luteinizing hormone C) Calcitonin D) Growth hormone E) Corticotropin-releasing hormone
28)
Why does consumption of alcohol stimulate urination? A) It stimulates ADHsecretion. B) It inhibits ADHsecretion. C) It stimulatesvasopressin. D) It stimulatesoxytocin. E) It inhibitsaldosterone.
29)
What hormonestimulates the milk-ejection reflex? A) Antidiuretic hormone B) Oxytocin C) Vasopressin D) Parathyroid hormone E) Aldosterone
30)
How are hormones of the posterior pituitary glandtransported to the hypothalamus? A) In portal veins B) In the cerebrospinal fluid C) Via axons D) Via thelymphatic system E) Packaged in lipoproteins
Version 1
8
31)
What hormone stimulates milk production in mammals? A) Aldosterone B) Melatonin C) Prolactin D) Luteinizing hormone E) Growth hormone
32)
What hormones are also known as thegonadotropins? A) Prolactin and oxytocin B) Follicle-stimulating hormone and oxytocin C) Luteinizing hormone and estrogen D) Prolactin and estrogen E) Follicle-stimulating hormone and luteinizing hormone
33) Nonpolar hormones can pass through the intestinal cell membrane without being digested, and therefore can be taken orally. What hormones might therefore be available as pills? A) Insulin and growth hormone B) Thyroid hormones and steroid hormones C) Calcitonin and glucagon D) Melatonin and prolactin E) Oxytocin and antidiuretic hormone
34)
What is not an effect of the hormonal products of the adrenal medulla? A) Accelerate heartbeat B) Stimulate synthesis of glycogen C) Increase blood pressure D) Reduce blood flow to the skin and digestive organs E) Increase blood sugar
Version 1
9
35) Starving children in war-torn countries have stick-thin limbs, as their muscle proteins are being broken down and the resultantamino acids are beingused by other cells for energy. What hormone(s) would you expect to be especially elevated in these children? A) Aldosterone B) Glucocorticoids C) Glucagon D) Insulin E) Prolactin
36)
What hormone is not produced by the adrenal gland? A) Epinephrine B) Cortisol C) Prolactin D) Aldosterone E) Norepinephrine
37)
What two hormones interact to maintain normal levels of blood glucose? A) Cortisone and testosterone B) Epinephrine and norepinephrine C) Glucagon and insulin D) LH and FSH E) Oxytocin and vasopressin
38)
When the body's blood glucose level rises above normal, what hormone is produced?
Version 1
10
A) Glucagon B) Epinephrine C) Insulin D) Norepinephrine E) Prolactin
39)
What condition is not associated with diabetes mellitus? A) Elevated levels of blood glucose B) Lowered production of insulin by the pancreas C) Reduced body cell sensitivity to insulin D) Defects in pituitary gland function E) Elevated glucose in the urine
40)
What molecule is not an example of a sex steroid hormone? A) Estrogen B) Progesterone C) Prolactin D) Testosterone E) Estradiol
41)
What hormone regulates the metamorphosis of tadpoles into frogs? A) Melatonin B) Thyroid hormone C) Estrogen D) Somatotropin E) Cortisol
42)
Choose the hormone that is not correctly matched with its chemical category.
Version 1
11
A) Peptide—antidiuretic hormone B) Steroid—norepinephrine C) Amino acid derivative—melatonin D) Glycoprotein—lutenizing hormone
43) it.
Choose the hormone that is not correctly matched with the endocrine gland that secretes
A) Adrenal cortex—aldosterone B) Pancreas—insulin C) Pineal gland—melatonin D) Adrenal medulla—cortisol E) Posterior lobe of pituitary gland—oxytocin
44) Large or polar hormones cannot enter a cell directly. What molecule can act as a second messenger for such hormones? A) Catecholamine B) Cyclic AMP C) Epinephrine D) Norepinephrine E) Calcitonin
45)
What statement does not correctly describe hormones that can directly enter a cell?
Version 1
12
A) Steroid hormonescan enter directly into cells because of their hydrophobic makeup. B) Water-solublehormones can enter directly into cells because of their nonpolar makeup. C) Steroid hormonesdissolve poorly in the plasma and are transported to target cells via specialprotein carriers. D) Once thesehormones enter a cell, they bind to a cytoplasmic receptor and travel as acomplex to the nucleus, or they may bind to the receptor after entering thenucleus.
46)
What statement does not correctly describe antidiuretic hormone? A) It binds tomembrane receptors ofcells in themammalian kidneys. B) ADH secretion isstimulated by high blood osmolarity. C) It ultimately acts to reabsorb more water. D) Damage to theposterior pituitary will decrease the production of ADH. E) It inhibits thesynthesis of aquaporins.
47)
What hormone is secreted by alpha cells of the pancreas in response to hypoglycemia? A) Glucagon B) Insulin C) Glucose D) Cortisol E) Glycogen
48) What mammalian hormone is most similar in amino acid structure to arginine vasotocin in chickens? A) Antidiuretic hormone B) Aldosterone C) Angiotensin II D) Growth hormone E) Oxytocin
Version 1
13
49)
What would be the likely effect of drinking three alcoholic beverages? A) Increased urination due to alcohol activating antiduretic hormone production B) Decreased urination due to alcohol activating aldosterone production C) Increased urination due to alcohol inhibiting antidiuretic hormone production D) Decreased urination due to alcohol inhibiting antidiuretic hormone production E) Increased urination due to alcohol inhibiting aldosterone production
50)
What is not affected by the secretion ofoxytocin? A) Pair bonding B) Milk-ejection reflex C) Milk production D) Uterine contractions
51)
Which of the following hormones has the fewest targets? A) Growth hormone B) Melatonin C) Oxytocin D) Thyroid-stimulating hormone E) Testosterone
52)
What is not a function of parathyroid hormone? A) It increasescalcium production in the liver. B) It increasescalcium absorption across the intestine. C) It increasesreabsorption of Ca 2+ in the kidney. D) It stimulates osteoclasts to dissolve calcium phosphate crystals in bone.
53)
What feature distinguishes all hormones from neurotransmitters?
Version 1
14
A) Hormones require large concentrations to have an effect, while neurotransmitters require only low concentrations. B) A hormone affects reproductive functions, while neurotransmitters control all other processes. C) Hormones travel viathe blood to a target tissue, while neurotransmitters cross the synaptic cleftto a postsynaptic cell. D) Hormones are steroids, while neurotransmitters are amino acids.
54)
What is the difference between the endocrine system and the autonomic nervous system?
A) The endocrinesystem employs chemical signals only, while the autonomic nervous systeminvolves a combination of electrical and chemical signals. B) The autonomicnervous system promotes "fight or flight" functions, while theendocrine system promotes "rest and digest" functions. C) Norepinephrine is produced by the autonomic nervous system, while epinephrine is produced by the endocrine system. D) The axons of neurons of the endocrine system travel a shorter distance than those of the autonomic nervous system.
55) If blood transports hormones throughout the body, how do they communicate with specific targets? A) Special gatewayvalves in the blood vessels direct hormones to their target tissues. B) Special carrierproteins "walk" along microtubule tracts to deliver the hormones totheir targets. C) Axonal pathfinding mediates the delivery of hormones to their specific targets. D) Only targettissues have receptors that allow them to receive the signal.
56)
Why don't hormones build up in the body over time?
Version 1
15
A) They are secretedfrom the body by exocrine glands. B) They are filtered out of the blood by lymph tissues. C) They do, but target tissues become desensitized over time. D) They aredeactivated or degraded by target cells after hormone-receptor binding. E) Hormone synthesis is subject topositive feedback regulation.
57) You are studying a disease caused by insufficient levels of a specific peptide hormone. You have identified mutations in several genes that can lead to the disease, but your startup company can only afford to pursue one gene product as a possible therapy. Which would you choose? A) Gene A—a transcription factor. B) Gene B—an enzyme that activates the hormone by cleaving its inactive precursor. C) Gene C—the hormone itself. D) Gene D—a kinase that acts downstream of the hormone.
58) You are researching an "orphan disease" with no available treatment. Your lab develops a drug that mimics a lipophilic hormone thought to be lacking in the disease, and you are looking for a pharmaceutical company to partner with to run a clinical trial. Based on the reaction from each team, which would you choose to work with? A) Company A: "Your lipophilic hormone mimic is a good start, but we'll need to develop a delivery system to get the drug across the cell membrane." B) Company B:"Your new drug seems very promising. Do you know which G-protein coupledreceptor it binds to?" C) Company C:"We're interested in developing this drug - it could really help people. Doyou know which genes the hormone regulates?" D) Company D: "The drugyou want to test shows strong potential, but patients might also need to take a blood thinner to increase its solubility in the blood."
59) You are an attending physician supervising a new resident. Together, you decide to treat a patient with a steroid drug. The resident administers the drug and watches the patient anxiously. "It's not working," he mutters. How do you respond? Version 1
16
A) "You're right,it doesn't seem to be working. Let's increase the dose, since we need toachieve high steroid levels in the blood." B) "Why don't yougo on your rounds and come back? Steroid hormones need several hours to work,since they regulate gene expression. Transcription and translation taketime." C) "We forgot toadminister the carrier protein to get the drug across the cell membrane. We canadd that now." D) "Let's grab acup of coffee and then check again. Activating the G-protein coupled receptorand its downstream kinase cascade can take several minutes."
60) ACTH stands for adrenocorticotropic hormone, but it is produced in the anterior pituitary. What is the basis of this name? A) It is expressed most highly in tropical climates. B) It stimulates the adrenal cortex. C) It signals to the kidneys to reabsorb water. D) It stimulates appetite by acting on the cerebral cortex.
61)
What stimulates the production of adrenocorticotropic hormone (ACTH)? A) GHRH B) CRH C) CIH D) ADH
62) Your doctor returns with your lab results and looks grave. “I’m afraid,” she says, “you seem to have a problem with your hypophysis.” What do you say?
Version 1
17
A) “That makes sense because I've been feeling fatigued, gaining weight, and am oftencold." B) “Well, that’s only responsible for one or two hormones, so it should be easy to treat.” C) “I suppose that means I am going to need insulin shots.” D) “Is it theposterior? Neurological problems run in my family.”
63) A middle-aged female patient complains of changes to her face. Her cheekbones and eyebrows have become more prominent, and her jaw is growing. You suspect acromegaly, a disorder caused by excessive growth hormone. She had nogrowth abnormalities in her youth.What do you suspect is the most likely cause of her problem? A) A tumour of the pineal gland B) Congenital mutation of the GH regulatory region C) A tumour of the anterior pituitary D) A tumour of the posterior pituitary E) Menopause
64) Your friend is concerned because although she has never been pregnant, her breasts are producing milk. When she went to the doctor, he sent her to the MRI lab for a brain scan. She was too embarrassed to ask the doctor why. What do you tell her? A) "They wereactually scanning your thyroid gland, not your brain. Are you getting enoughiodine in your diet?" B) "Prolactin from the hypothalamus stimulates milk production. The doctor probably wants to rule out a tumor in the hypothalamus." C) "LH and FSH from the posterior pituitary stimulate milk production. The doctor wants to rule out excess levels of estrogen in the pituitary stalk." D) "Prolactin from the anterior pituitary stimulates milk production. The doctor probably wants to rule out a pituitary tumor."
65) Blood glucose levels must be carefully maintained within a narrow window. What endocrine organ is responsible for this task?
Version 1
18
A) Pancreas B) Pineal gland C) Pituitary gland D) Parathyroid gland
66)
What strategy does the body use for blood glucose homeostasis? A) Release of hormones by the brain B) Energy stores in muscle allow for quick glucose release C) Glucose usage by the brain is shut down when necessary D) The action of two antagonistic effectors
67)
When a diabetic injects insulin, what does this accomplish? A) Promotes glucose release from cells, raising blood glucose B) Promotes uptake of glucose into cells, lowering blood glucose C) Promotes hydrolysis of glycogen, raising blood glucose D) Improves the ability of the body to sense blood glucose levels
68) The residents of a small town are concerned about possible pollutants being released by a nearby chemical plant. The cause of their concern is the recent discoveryof dozens of small tadpoles with legs in ponds near the chemical plant. What is a possible explanation for these legged tadpoles? A) The chemicalplant is releasing chemicals that are similar in structure to steroidhormones. B) The chemicalplant is releasing chemicals that are similar in structure to cyclic AMP. C) The chemicalplant is releasing chemicals that are similar in structure to thyroidhormones. D) The chemicalplant is releasing chemicals that are similar in structure to G proteins. E) The chemical plant is releasingchemicals that are similar in structure to aldosterone.
Version 1
19
69) Beta-blockers are a class of drugs that target beta-adrenergic receptors, blocking the action of epinephrine and norepinephrine. These drugs are used to treat heart conditions, hypertension, glaucoma, and migraineheadaches and are also used off-label to alleviate stage fright in performers. Since the beta-adrenergic receptor is a GPCR, what intracellular event is prevented by the use of a beta-blocker? A) A protein kinase cascade, initiated by the receptor's intracellular kinase domain B) Release of an activatedG protein subunit bound to GTP C) Transport of the receptor-hormone complex into the cell and the nucleus, where it would regulate transcription D) A protein phosphatase cascade, initiated by the receptor's intracellular phosphatasedomain
70) What type of molecule typically conveys the signal from a peptide hormone to the effectors in the cell? A) Nucleotide B) NO (nitric oxide) C) H 2O D) Phosphate group E) Amino acid
71) The regulatory systems of the animal body maintain a relatively stable internal condition through a process called __________. A) homeostasis B) equilibrium C) osmoreception D) countercurrentexchange E) absorption
72) The organ that performs filtration of blood, reabsorption, and secretion in vertebrates is the __________.
Version 1
20
A) kidney B) liver C) pancreas D) small intestine E) large intestine
73) About 180 liters of blood plasma is filtered into human nephrons per day and, of that amount, about 2-3 liters leaves the body as urine. Therefore, the percentage of plasma filtrate that leaves as urine each day is about __________. A) 0.1–0.2 percent B) 0.5–1.0 percent C) 1–2 percent D) 5–10 percent E) 10–20 percent F) 20–50 percent
74) In vertebrates, most of the fluid that ultimately exits the body as urine first entersthe nephron tubules by the process of __________. A) reabsorption B) absorption C) secretion D) filtration E) excretion
75) This molecule or ion never usesactive transport as its motive force for reabsorption into blood capillaries in the kidney.
Version 1
21
A) Na + ions B) Glucose C) Amino acids D) Water E) HCO 3 – ions F) Cl – ions
76) In mammalian kidneys, blood in the vasa recta and fluid in the loop of Henle flow in opposite directions to create a(n) __________ multiplier system. A) homeostatic B) reversible C) countercurrent D) stepped E) exponential
77) Each human kidney contains about a million __________, which are the tubular functional units of the kidney. A) ureters B) nephrons C) renal medullae D) nephridia E) flame cells
78) Water and small solutes are filtered under pressure out of the blood plasma into nephrons through the porous walls of __________, tufts of capillaries in the renal cortex.
Version 1
22
A) vasa rectae B) loops of Henle C) glomeruli D) collectingducts E) juxtaglomerularapparati
79) The presence of a loop of Henle in nephrons is an evolutionary adaptation that enables mammals and birds to reabsorb water efficiently and produce a(n) __________ urine. A) hypertonic B) hypotonic C) osmolar D) isotonic
80)
The diffusion of water across a plasma membrane is referred to as A) Osmosis. B) Active transport. C) Facilitated diffusion. D) Countercurrent flow. E) Bulk flow.
81) Jill is lost in the desert and ran out of water six hours ago. What physiological effect or response would most likely occur in this situation? A) Decreased release of ADH from the posterior pituitary B) Decrease in blood osmolarity C) Increased permeability of the collecting ducts in the kidney D) Decreased thirst sensation E) Increased release of atrial natriuretic hormone
Version 1
23
82)
What protein is most likely inhibited bynegative feedbackwhen blood volume rises? A) Renin B) Aldosterone C) Angiotensin II D) Angiotensinogen E) Angiotensin I
83) You wish to determine which organs function to remove waste products from the hemolymph (blood) of an insect. You do so by injecting a suspension of particulate dye particles into the abdominal cavity of an anesthetized cricket. Where will you find these dye particles 45 minutes later, after you euthanize and dissect the cricket? A) In the nephridia B) In the Malpighian tubules C) In the nephrons of the kidneys D) In the protonephridia E) In the antennal glands F) In the urinary bladder
84) You put a solution of sucrose into a bag of dialysis tubing, which is permeable to water but not to sucrose. You then put this bag into a beaker of water. An hourlater, you observe that the bag has swelled due to A) Active transport of water molecules. B) The osmotic pressure of the sucrose solution. C) The pressure exerted on the bag by the surrounding water. D) Passive transport of sucrose molecules. E) The high osmolarity of the surrounding water.
85) All of the following are animal mechanisms to cope with the problem of water balance except
Version 1
24
A) Coupling of water/salt removal with metabolic waste removal. B) Passage of water and waste across a layer of cellsinto a tubular structure. C) Kidneys used as filtering organs. D) Secretion of regulatory hormones into the tubule. E) Evolution of osmoconformity
86) What is true of normal urine production? I Blood pressure is used to push nitrogenous waste, water, and small moleculesthrough a filter, forming the filtrate, while cells and most proteins are retained in the blood. II Most water is reabsorbed as the filtrate is passing through a long tube. III All the urea that enters a kidney nephronis excreted. A) Just I B) Just II C) II and III D) I, II, and III E) I and II
87) What is the route a red blood cellwould take to circulate through a kidney, starting from the aorta? A) Renal artery -> afferent arteriole -> vasa recta -> efferent arteriole -> glomerulus -> renal vein B) Renal artery -> glomerulus -> afferent arteriole -> efferent arteriole -> vasa recta -> renal vein C) Renal vein -> efferent arteriole -> glomerulus -> afferent arteriole -> vasa recta -> renal artery D) Renal artery -> afferent arteriole -> glomerulus -> efferent arteriole -> vasa recta -> renal vein E) Renal vein -> vasa recta -> afferent arteriole -> glomerulus -> efferent arteriole -> renal artery F) Renal vein -> glomerulus -> efferent arteriole -> afferent arteriole -> vasa recta -> renal artery G) Renal artery -> vasa recta -> afferent arteriole -> efferent arteriole -> glomerulus -> renal vein
Version 1
25
88) Claire lost the ability to completely empty her bladder following a stroke, and a nurse inserts a catheter into the open end of her urinary tract each day to remove the urine. Into what structure does the nurse insert the catheter? A) Ureter B) Bladder C) Urethra D) Nephron E) Collecting duct F) Renal pelvis
89)
All of the following are reabsorbed by the nephron except __________. A) Glucose B) Amino acids C) Water D) Vitamins E) Albumin F) Na + and Cl – ions
90) Which solutemust be administered in high amounts because of its very efficient secretion and removal by the kidney tubules? A) Penicillin B) HCO 3 – ions C) Amino acids D) Glucose E) Vitamin K
91)
Substances not usually found in normal human urine are
Version 1
26
A) urea and uric acid. B) glucosemolecules. C) watermolecules. D) K +, H +, and other ions. E) certaindrugs.
92) The presence of the loop of Henle allows which animals to reabsorb water and produce a hypertonic urine? A) Birds B) Marine fish C) Mammals D) Birds and mammals E) Freshwater fish and mammals F) Birds and reptiles
93) Gary has a kidney stone lodged in his ureter. Where will the radiologist find the kidney stone on an x-ray? A) Between the end of a nephron and the renal pelvis B) Between the bladder and the outside of the body C) Between a kidney and the bladder D) Between a Bowman’s capsule and a loop of Henle
94) How does the loop of Henle enable the mammalian kidney to produce hypertonic urine? I The ascending limb of the loop actively extrudes Na +. II The descending limb interacts with the ascending limb to raise the osmolarity of the extracellular fluid in the renal medulla. III The concentration is further enhanced by the presence of urea, which draws water out of the collecting ducts by osmosis.
Version 1
27
A) Just I B) II and III C) Just III D) I and II E) I, II, and III
95) The kidneys are important homeostatic organs, contributing to maintenance of all of the following except A) Blood volume. B) Degradation of old red blood cells. C) Blood pressure. D) Ion concentration of extracellular fluids. E) Blood pH.
96)
What response would correct for a drop in glomerular blood pressure? A) Angiotensin II-mediated inhibition of the adrenal cortex B) Inhibition of osmoreceptors in the brain, leading to decreased thirst C) Dilation of the afferent renal arterioles by angiotensin II D) Inhibition of antidiuretic hormone release by the posterior pituitary gland E) Stimulation of sodium secretion by the distal convoluted tubules
97)
Where are the receptors that respond to elevated blood osmolarity? A) In the adrenal cortex B) In the adrenal medulla C) In the hypothalamus D) In the kidneys E) In the heart
98)
The adrenal hormone, aldosterone, regulates kidney function by
Version 1
28
A) Reducing the blood flow to the kidney. B) Stimulating active reabsorption of sodium, decreasing its excretion. C) Increasing the permeability of the collecting duct to urea. D) Increasing the permeability of the collecting duct to salt. E) Triggering a sensation of thirst.
99)
An increase in the production of ADH affects the kidney tubules by
A) Inhibiting reabsorption of sodium ion in the collecting duct. B) Decreasing the permeability of the glomerulus to water. C) Causing the walls of the collecting duct to become more permeable to water. D) Inhibiting the reabsorption of water in the glomerulus. E) Decreasing the insertion of aquaporins into the membranes of cells that line thecollecting ducts.
100) Most vertebrates maintain homeostasis in regard to the total solute concentrations of their extracellular fluids and theconcentrations of specific inorganic ions. The kidneys participate in regulatingthe extracellular fluid concentration of all ions below except A) Fe 2+ ions. B) Ca 2+ ions. C) K + ions. D) Na + ions. E) Cl – ions.
101)
Most marine invertebrates are osmoconformers. This means that
Version 1
29
A) That their body fluidscontain the exact same solutes in the same concentration as their cells. B) The osmolarity of their body fluids is the same as their seawater environment. C) That their body fluidscontain the exact same solutes in the same concentration as their surroundings. D) The osmolarity of their body fluids is equal to that of the osmoregulators who inhabit the same environments. E) The osmolarity of their body fluids fluctuates between hypertonic to hypotonic depending on the season and the tides.
102)
Select the incorrectly-matched osmoregulatory organ and the organism that uses it. A) Nephridia—annelids B) Protonephridia—flatworms C) Malpighiantubules—insects D) Kidney—mammals E) Nephrons—sea urchins
103) No matter how much water a human drinks, the kidneys are always regulating various aspects of the blood, including __________. I The pH of the blood II The plasma concentration of iron ions III The plasma concentration of sodium ions IV The plasma concentration of potassium ions V The temperature of the blood A) I, II, and III B) I, II, III, and IV C) I, III, and IV D) I, III, IV, and V E) I, II, III, IV, and V
Version 1
30
104) Margaret had an unusual viral infection that damaged her posterior pituitary. Ever since, she has been excreting a large volume of dilute urine.The doctor tells Margaret thatshe has the disorder known as __________. A) atherosclerosis B) diabetesmellitus C) hypertension D) antidiuresis E) diabetesinsipidus
105) Each nephron of the kidney consists of a long tubule and associated small blood vessels. Blood cells and plasma proteins are too large to enter with the glomerular filtrate. What is the correct path for the filtrate to follow through the nephron in the production of urine? A) Bowman's capsule → distal convoluted tubule → loop of Henle → proximal convoluted tubule → collecting duct B) Bowman's capsule→ proximal convoluted tubule → loop of Henle → distal convoluted tubule →collecting duct C) Bowman's capsule → loop of Henle → proximal convoluted tubule → distal convoluted tubule → collecting duct D) Bowman's capsule → collecting duct → proximal convoluted tubule → loop of Henle → distal convoluted tubule E) collecting duct → proximal convoluted tubule → loop of Henle → distal convoluted tubule→ Bowman's capsule
106)
Freshwater fish are __________ with respect to osmoregulation. A) hypotonicregulators B) hypotonicconformers C) hypertonicregulators D) hypertonicconformers E) isotonicconformers
107)
What animal cannot make hypertonic urine?
Version 1
31
A) Blue jay B) Dog C) Human D) Grasshopper E) American toad
Version 1
32
108)
According to the above figure, which letter corresponds to the lowest tubular fluid osmolarity under normal conditions?
Version 1
33
A) A B) B C) C D) D E) E
Version 1
34
109)
According to the above figure, which letter corresponds to the area where water is absolutely not reabsorbed?
Version 1
35
A) A B) B C) C D) D E) E
Version 1
36
110)
According to the above figure, which letter corresponds to the area where most, if not all, amino acids are reabsorbed?
Version 1
37
A) A B) B C) C D) D E) E
Version 1
38
111)
Refer to the figure. What is the most likely concentration of thefluid in regionE during moderate-volume urine production?
Version 1
39
A) 300 mOsm B) 700 mOsm C) 75 mOsm D) 5 mOsm E) 1400 mOsm
112) Connor decides to celebrate his 21st birthday by drinking a few beers at a local sports bar. After a while, Connor notices that he is making many more trips to the men’s room than the week before when he drank similar amounts of carbonated water. Why is that? A) Alcohol stimulatesfiltration of the blood plasma by the glomeruli. B) Alcohol stimulatessecretion of ions into the nephron and water follows by osmosis. C) Alcohol inhibitsreabsorption of <span></span>sugars, amino acids and other smallmetabolites, which decreases reabsorption of water. D) Alcohol inhibits the secretionof ADH by the posterior pituitary gland, thereby decreasing waterreabsorption. E) Alcohol stimulates theinsertion of aquaporins into the plasma membranes of cells that line thecollecting ducts, thereby decreasing water reabsorption.
113) The kidneys are both protected and held in place by cushions of fat. In cases of rapid weight loss, the kidneys may drop to a lower position, possibly causing kinks in a nearby tubular organ. If this happens, urine will back up into the kidneys, severely damaging them. What organ(s) is/are most likely kinked in this situation? A) Urethra B) Ureters C) Renal arteries D) Nephrons E) Renal veins
114)
What cells arephagocytes that can also present antigens to T
Version 1
H cells?
40
A) B cells B) Macrophages C) Plasma cells D) T cells E) Monocytes
115) What cells directly stimulate the activity of other cells in the generation of an immune response? A) B cells B) Natural killer cells C) Plasma cells D) T cells E) Monocytes
116)
What are the source of antibody-producing cells? A) B cells B) Macrophages C) Plasma cells D) T cells E) Monocytes
117)
What cells are the precursors of macrophages? A) B cells B) Macrophages C) Plasma cells D) T cells E) Monocytes
Version 1
41
118)
Which cells produce specific antibodies? A) B cells B) Macrophages C) Plasma cells D) T cells E) Monocytes
119)
What markers of "self" are on the surface of most vertebrate cells? A) Major histocompatibility complex proteins B) T-cell receptors C) Antigens D) Immunoglobulins
120) A patient with melanoma receives three types of therapy: surgery, radiation, and interferon. Which of these represents something the immune system uses against cancer as well? A) Radiation—a class of lymphocytes uses powerful electromagnetic radiation to attack "altered self" cells. B) Interferon—T H cells and NK cells secrete interferon in response to tumor cells. C) Interferon—NK cells insert interferon into the membranes of tumor cells, creating large pores that burst the cells. D) Interferon—In complexes bound with MHC proteins, the interferons indicate the presence of altered tissue to B cells.
121)
What category of cellsincludes both phagocytes and lymphocytes? A) Leukocytes B) Erythrocytes C) Platelets D) Target cells
Version 1
42
122)
What molecules are released by activated helper T cells? A) Antigens B) Immunoglobulins C) Cytokines D) Antibodies
123) Lymphocyte receptors are encoded by genes that are assembled by rearrangement and mutation of what molecules? A) pre-mRNA B) mRNA C) tRNA D) DNA
124)
What is a necessary component of a powerful immune system? A) An ability to distinguish self from nonself B) Strict temperature homeostasis C) Thousands of different immunoglobulin genes D) A large thyroid gland E) Different classes of plasma cells
125) The human immunodeficiency virus mounts a direct attack on TH cells by recognizing the __________ proteins in the plasma membrane ofthese cells. A) CD8 B) CD4 C) Toll-likereceptor D) cytokinereceptor E) major histocompatibility class I
Version 1
43
126) At Thanksgiving dinner, you sit next to Uncle Ed, an immunologist. "I'm a big eater," he says, "just like the cells I study." What type of cells does Uncle Ed most likely study? A) Monocytes B) Erythrocytes C) Macrophages D) Bacteriophages E) Antibodies
127)
What is the name for moleculesthat provoke a specific immune response? A) Antigens B) Lymphocytes C) Antibodies D) Lysozymes
128) In the humoral response, some B cells differentiate intoplasma cells. What do plasma cells produce in large quantities? A) Agglutinations that are specific for foreign antibodies B) Interferons specific for foreign antigens C) Immunoglobulins specific for foreign antigens D) Antigens specific for foreign antibodies E) Macrophages specific for foreign antibodies
129) The human immunodeficiency virus (HIV) cripples the functioningof the adaptive immune system because it selectively infects and destroys a class of cellswhich are required for animmune response to pathogens andcancer cells. What cell is the most likely targetof the HIV virus?
Version 1
44
A) Suppressor T cells B) Cytotoxic T cells C) Natural killer cells D) Neutrophils E) Helper T cells
130)
MHC class II cell surface proteins are found on what types of cells? A) Macrophages, B cells, and dendritic cells B) B cells and macrophages C) Natural killer cells, neutrophils, and B cells only D) Neutrophils only E) Natural killer cells and dendritic cells
131)
The cell-mediated immune response is brought about by which cells? A) B cells B) T cells C) Erythrocytes D) Neutrophils E) Monocytes
132)
What do cytotoxic T cells interact with? A) MHC class Iproteins only B) MHC class IIproteins only C) Viral or cancer cell peptidesonly D) MHC class I andMHC class II proteins E) MHC class Iproteins and viral or cancer cell peptides F) MHC class II proteins and viral orcancer cell peptides
133)
Which part of the antibody molecule determines what epitope it binds to?
Version 1
45
A) The two arms of the Y B) Disulfide chains C) The heavy chains D) The stem of the Y E) The Fc portion
134) The ability of a vertebrate to generate millions of different lymphocytes is a result of its DNA undergoing what process? A) DNArearrangement B) DNAreestablishment C) Clonal variability D) Instructional variation E) Gene cloning
135) Some viral pathogens evade detection by the vertebrate immune system by a mechanism that most likely involves what process? A) Becoming a symbiont B) Becoming nonvirulent C) Frequent changing of surface antigens D) Becoming metabolically inactive E) Becoming a spore
136) When a "new" antigen first challenges the immune system, if the primary immune response produces B cells, what are the fates of those B cells? I Some of the B cells become plasma cells that secrete antibodies. II Some of the B cells become memory cells and can produce a swifter response if the body encounters that particular antigen again. III Some of the B cells secrete chemicals called interleukins, which travel to the brain to inducea fever.
Version 1
46
A) just II B) just I C) II and III D) I and II E) I, II, and III
137)
Select the correct statement regarding the temperature response of the immune system.
A) When neutrophilsencounter invading cells, they release immunoglobulin-1, which is carried tothe brain by the circulatory system. B) After binding toa pathogen, macrophages release interleukin-1, which stimulates neurons in thehypothalamus to raise the body's temperature, producing a fever. C) Fever contributesto the body's defenses by stimulating the maturation of B cells into plasmacells. D) Fevers between99° and 100°F are often fatal. E) Fever contributes to the body'sdefenses by stimulating the liver and spleen to store glucose.
138) A friend tells you: "My aunt just received a heart transplant. Her doctors warned us that her body might reject the tissue. I don't understand how that could happen as human hearts are all made of the same types of tissue." How do you respond? A) "Yes, all human hearts are made of cardiac muscle so the chance of rejection is almost zero." B) "Yes, all human hearts are made of cardiac muscle, but the ratio of the different kinds of muscle fibers varies from person to person. This variation can lead to tissue rejection by the recipient." C) "Yes, all human hearts are made of cardiac muscle. However, your aunt might have different antibodies than did the donor, which could lead to tissue rejection." D) "Yes, all human hearts are made of cardiac muscle. Your aunt was probably given a drug after surgery that can cause tissue rejection." E) "Yes, all human hearts are made of cardiac muscle, but everyone has different MHC proteins on their tissues. Your aunt's immune system recognizes her ‘self' tissues but can reject "nonself" tissue that has different MHC proteins."
Version 1
47
139)
What is true regarding antigens?
A) A large antigen is likely to have many different epitopes, each of which can stimulate a distinct immune response. B) An antigen is amolecule that promotes a general immune response. C) Bacteria do not contain antigens. D) Antigens can onlybe recognized by their lipid moieties. E) The most effective antigens are small simple amino acid complexes.
140)
What do we call a lymphocyte that has never encountered an antigen? A) T cell B) B cell C) Foreign lymphocyte D) Stem leukocyte E) Naive lymphocyte
141)
Which of the following cells is matched to anincorrect function? A) Natural killer cells—kill virus-infected and tumor cells B) Plasma cells—secrete immunoglobulins C) Macrophage—phagocytic cell D) Neutrophil—important antigen-presenting cell E) Monocyte—precursor of macrophage
142)
What is true regarding cytokines?
Version 1
48
A) Cytokines are secreted by T H cells and bind to receptors. B) Cytokines are immune cells involved in producing antibodies. C) Cytokines aresecreted by pathogens during infection. D) Cytokines bind to intracellular receptors. E) Cytokines protect in the attack of pathogens by phagocytosis.
143) What is the most fundamental difference between the innate immune system and the adaptive immune system? A) The way in which pathogens are recognized B) The way in which pathogens are destroyed C) The types of cells that participate in each response D) The speed
144) Imagine that a Toll-like receptor (TLR) gene has duplicated and mutated, creating a leucine-rich region binding pocket that can now bind to the hemagglutinin protein in the envelope of influenza virus. What do you predict will happen to this novel gene? A) It will beselected for and over evolutionary time will become an even better receptor forHA protein. B) It will spreadfrom species to species. C) It will undergoVDJ rearrangement. D) It will continue to mutate, perhapschanging to bind other proteins or becoming deleted.
145) Until renamed by Paul Ehrlich in the 1890s, the heat-labile, nonspecific part of the immune system that enhances inflammation was known as "alexin." We now know that various proteins of this system have a variety of functions, including bursting pathogens by insertion of a membrane-attack complex. What did Erhlich rename this specific part of the innate immune system?
Version 1
49
A) The complement system B) Immunoglobulins C) Toll-likereceptors D) Major histocompatibility proteins E) Mannose-binding lectins
146) Imagine you are a doctor and a patient has come to you for a second opinion. "I have these mysterious lumps in my right armpit," he says, "The other doctor I saw says they need to be removed. What do you think?" You notice red scratches on the patient's right arm. "You must have a new kitten who has transmitted cat scratch disease, from the bacterium Bartonella ," you say, "And we certainly won't remove those lumps, they are really important! Those are your..." A) "thymusglands, where naive B and T cells encounter antigens and becomeactivated." B) "lymph nodes,where T-cell receptor genes are rearranged as T cells mature." C) "lymph nodes,where naive B and T cells encounter antigens and become activated." D) "spleen, where hematopoietic stemcells produce lymphoid and myeloid progenitors."
147) T cells binding to MHC-peptide complexes are responsible for acceptance or rejection of transplanted organs. To facilitate acceptance of the transplant, the ideal donor will have a close genetic relationship with the recipient—even then, immune suppression drugs are often necessary. In a strange variation of this phenomenon, the endangered Tasmanian devil of Australia suffers from a devastating disease in which tumorous tissue is transmitted from animal to animal. What is the likely explanation for the lack of rejection of this transmissible tumor? A) MHC genes are unique to humans. In marsupials such as the Tasmanian devil, the T cells do not distinguish self from non-self. B) Marsupials lack an adaptive immune system and rely solely on their innate immune response to fight pathogens and tumors. C) Low genetic diversity has resulted in invariant MHC genes, so the transmitted tumor tissue is accepted as self. D) Tumors are not targeted by T cells unless they are caused by a viral infection.
Version 1
50
148) You are studying a trafficking protein important for secretion in helper T cells. What is the likely outcome of a defect in this protein? A) Impaired secretion ofimmunoglobulins B) Impaired secretion ofdegraded antigens C) Impaired secretion of membrane attack complex D) Impaired secretion ofcytokines
149) What would happen to immunoglobulin genes if alternative mRNA splicing was nonfunctional? A) Transcripts would contain a single V region only. B) Transcripts wouldcontain all possible V, D, J, and constant regions. C) Transcripts wouldhave extra V, D, and J segments attached to a single constant region. D) Transcripts would have single V and Dsegments, but extra J segments and constant regions.
150) Two proteins, Rag-1 and Rag-2, catalyze VDJ recombination. What is the likelyoutcome of a deletion of the Rag genes? A) Mild susceptibility to infection due to T cell defects B) Increased antibody diversity from B cells C) Severe combined immunodeficiency (SCID) of both B and T cells D) Increased resistance to infection due to enhanced B and T cell function
151) Two proteins, Rag-1 and Rag-2, catalyze VDJ recombination. Where would you expect these proteins to be expressed?
Version 1
51
A) Thyroid and parathyroid B) Bone marrow only C) Bone marrow and thymus D) Thyroid only E) Blood serum
152) Some scientists are trying to develop a vaccine that would prevent cancer. What is an important consideration? A) Antigen selection would be critical, due to the risk of the immune system attacking the patient's ownhealthy tissue. B) It is not possible—vaccines can only protect against viruses or bacteria. C) The natural killer (NK) cells already attack cancer cells and a vaccine is unlikely to perform better than the body's natural immune system. D) Many cancer cells have MHC molecules that mark them as "self", so they cannot be attacked by the immune system.
153) B cells can generate antibodies with over 10 10 different antigen-binding sites. How do so many different antibodies fit onto a single B cell? A) B cells are unusually large lymphocytes. B) B cells have invaginated plasma membranes that result in an extremely large surface area. C) B cells have longflagella-like extensions to provide more surface area. D) They don't—each B cell makes only a single variety of antibody.
154) A pathogenic virus has a new mutation in one of the coat proteins, that just happens to resemble the structure of a normal human protein present in glial cells of the CNS. What do you predict may be an outcome of infection by this virus?
Version 1
52
A) The initialresponse to the virus will be strong, but will then be shut down byimmunological tolerance. B) An autoimmunedisease of the brain may develop. C) The innate immunesystem will ignore the pathogen, but the acquired immune system will attackit. D) The membrane attack complex will beunable to burst the viral membrane.
155) When developing a monoclonal antibody-based test kit, great care must be taken to choose an antibody with the right binding properties. Imagine that you are developing an antibody assay kit to specifically detect E. coli 0157:H7, a pathogenic serotype of E. coli that causes food poisoning. Choose the hybridoma line best suited for use in your test kit. hybridoma line
E. coli 0157:H7
E. coli K-12 MG1655
E. coli 0150:H5
E. coli 0152:H28
1
+++
-
+++
-
2
-
++
++
+
3
++
-
-
-
4
+++
++
++
++
5
+
+++
-
-
Version 1
53
A) 1 B) 2 C) 3 D) 4 E) 5
156)
Why must T cell activation take place outside the thymus?
A) An activated Tcell could destroy the other maturing T cells. B) An activated T cell could destroy the thymus gland itself. C) Many T cells areself-reactive and must be eliminated in the thymus before any possibility ofactivation. D) Other cell typesmight be accidentally activated in the thymus.
157) In the old days, milkmaids were exposed to cowpox from milking the cows, and this fortuitously gave them some protection from smallpox. What was happening in the milkmaids? A) They werereceiving a dose of generic IgG from the sick cows. B) They wereeffectively receiving a vaccination from a related virus. C) They wereheightening their overall immune response due to exposure to the cowpox andother cow parasites. D) They had a small immune response from touching and drinking the cow's milk, which resulted in cow B cells circulating through their blood.
158) Homeostasis is maintained using negative feedback loops. Which part of a negative feedback loop detects conditions? A) Response B) Effector C) Sensor D) Integrating center E) Stimulus
Version 1
54
159) Low body temperature triggers adaptive responses to conserve heat. Once the integrating center detects that body temperature has risen, it will decrease these responses. What is this an example of? A) Adaptation B) Negative feedback C) Positive feedback D) Acclimatization
160) Due to a mutation in a smooth muscle gene, the blood vessels in the skin of a certain strain of mouse become dilated when temperature falls below a certain point. Based on the likely outcome of this mutation, what precautions should the researchers take to maintain the health of their mice? A) The mouse roomshould be maintained at a cooler than normal temperature. B) The mouse roomshould be consistently maintained at a warm temperature. C) The mice willhave weak muscles, and should be provided with exercise wheels. D) The mice will besusceptible to high blood pressure, and should not be provided with exercisewheels.
161) Which statement describes a home heating and cooling system that functions in a homeostatic way? A) The thermostat is always on and the temperature inside the house is kept constant. B) The heater constantly blows air, but the temperature of the air it produces changes depending upon the temperature inside the house. C) The heater is on at a constant rate resulting in a wide range of temperature inside the house. D) The thermostat turns on and off and keeps the temperature in the house within a narrow range.
162) Mechanisms that help generate heat and those that allow release of heat are an example of __________. Collectively, these mechanisms increase the __________ of body temperature. Version 1
55
A) endothermicregulation; flexibility B) poikilothermicregulation; stability C) ectothermicregulation; flexibility D) antagonisteffectors; stability
163) Some animals allow their body temperature to conform to the environment. What is the old term for these animals, and what is the newer preferred term? A) Heterotherms; homeotherms B) Homeotherms; endotherms C) Isotherms; poikilotherms D) Poikilotherms; ectotherms E) Endotherms; homeostats
164)
What is nota method used by ectotherms to affect body temperature? A) Countercurrent heat exchange B) Basking in the sun C) Shivering D) Dilation or contraction of blood vessels in skin E) Increasing metabolic rate
165) A scout troop sets out on a backpacking trip. Soon, an unexpected storm gets them drenched, and the troop leaders discover that few boys have packed warm clothes. Several scouts start shivering uncontrollably, a sign of hypothermia. The leaders call off the trip and the group heads back to their cars. This shivering response was A) an attempt to produce heat by burning large amounts of ATP. B) a maladaptive response that wastes energy. C) an attempt to produce heat by creating friction with their clothing. D) a social signal that evolved to alert adults to take action.
Version 1
56
166) If the boys progressed into full-blown hypothermia, this would be a failure of which regulatory system? A) Endodermic regulation B) Temperature homeostasis C) Ectothermic regulation D) Glucose homeostasis
167) Dogs use panting to dissipate excess heat. What part of their brain detects the need for this response and sends the signal to pant? A) Hippocampus B) Prefrontal cortex C) Amygdala D) Hypothalamus
168) When a dog's brain detects the need to dissipate excess heat and the dog begins to pant, the brain acts as the __________, and the panting behavior acts as the __________. A) stimulator;effector B) integratingcenter; sensor C) integratingcenter; effector D) sensor;integrating center
169) After yet another patron dripped sweat on her desk, the clerk at the gym complained, “It’s gross working at a gym, with everyone sweating all the time.” “Maybe,” the manager replied, “and while it probably makes our __________ bill higher, I’ll bet we’re saving money on our __________ bill.”
Version 1
57
A) Towel; heating B) Towel; treadmill C) Water cooler; air conditioning D) Air conditioning; water cooler
170) If an animal suffered damage to the temperature sensor in its brain, what would be the result? A) The integratingcenter would not receive inputs to compare to the set point. Body temperaturecould rise or fall to dangerous levels. B) Only theintegumentary system is required to sense temperature changes. Body temperaturewould remain stable. C) The animal wouldhave to rely on shivering instead of vasoconstriction to maintain bodyheat. D) The animal wouldfall back onto a more primitive ectothermic mode of regulation.
171) You are an engineer at a power plant that creates large volumes of hot waste water, which disrupts the ecosystem of the river it drains into. Another part of the plant requires warm oil, and the plant uses large amounts of energy to heat it. Inspired by countercurrent exchange in whales, you save energy and benefit the environment by doing what? A) Running the oil pipes under the river after dumping in the hot waste water B) Running the oil pipes around the waste water pipes, to allow heat transfer from water to oil C) Running the waste water and oil through a single pipe D) Adding a sensor, integrating center and effectors to control temperature in both pipes
172) Thermoregulation depends on monitoring of blood temperature. What structure acts as the mammalian temperature sensor?
Version 1
58
A) Hypothalamus B) Peripheral blood vessels C) Adrenal medulla D) Anterior pituitary gland E) Thyroid
173) If the set point of the hypothalamus were to suddenly rise to 40°C (as in the induction phase of a fever), what would the initial response of the body be? A) Glands release sweat B) Muscles contract to cause shivering C) Body temperature drops D) Blood vessels dilate
174)
Which of these factors would have the least effect on heat transfer processes in animals? A) Surface area of the body B) Difference between body temperature and ambient temperature C) Internal insulation such as blubber D) External insulation such as feathers or fur E) Bone density
175)
What thermoregulation process can be sustained for the longest period of time? A) Shivering B) Sweating C) Contraction of blood vessels in skin D) Hibernation E) Fever
176)
What animalhas the highest metabolic rate per unit body mass?
Version 1
59
A) Mouse B) Horse C) Cat D) Human E) Elephant
177) Diana has been fasting for a few days and decides to go for a short run before breaking her fast. The glucose she needs for her run will comefrom______________________. A) Breaking starch down to small molecules B) Glycolysis C) Insulin D) Breaking glycogen down to small molecules E) Cellular respiration
178)
The liver removes excess glucose from the blood and converts it into __________. A) cellulose B) maltose and otherdisaccharides C) starch D) glycogen E) bile
179)
After not eating for three days, __________ levels will be high. A) Insulin B) Glycogen C) Glucagon D) Blood glucose E) Pancreatic enzymes
Version 1
60
180) Something is terribly wrong with Beth and Steve's newborn baby girl. A biopsy of her kidneys reveals that the kidney nephrons all have extremely short Loops of Henle. What will be the ultimate fate of the baby? A) She may diebecause her kidneys cannot reabsorb all the filtered sugars and aminoacids. B) She may diebecause her kidneys cannot filter urea out of the blood. C) She may diebecause her kidneys cannot reabsorb enough filtered water. D) She may diebecause her kidneys cannot secrete K + and H + ions into the nephron tubule. E) She will live because the kidneys offish, amphibians and reptiles function perfectly well without any Loops ofHenle.
181) A mountain climber was rescued after spending 36 hoursexposed to the elementson a slopeof Mount Everest. Miraculously, the only permanent damage was to his toes, all of which had to be removed. What was the most likely cause of the damage to his toes? A) Vasoconstriction of the arteries to his legs andfeet B) Vasodilation of the arteries to his legs and feets C) The inability of his white blood cells to destroy infectious bacteria D) Metabolic (nitrogenous) wastes pooling in his feet E) Increased levels of carbon dioxide in his feet
SECTION BREAK. Answer all the part questions. 182) Sometimes, the most practical way to do an experiment is not to perform it in an animal, but to look at cell responses in culture. Cells from mice, humans, and other mammals have been used to establish cell culture lines that have been very important for research. You are studying a novel water-soluble mouse hormone. You know cell culture can be a practical model to reveal protein function, so you apply the hormone to yeast cells, but nothing happens.
182.1)
Version 1
What is a likely explanation for why nothing happened in your experiment?
61
A) Yeast have a cellwall, so the molecule cannot pass through the cell membrane as it would inmice. B) Water-solublehormones are lipids, not proteins. C) You need to applythe hormone in large amounts to see an effect. D) Yeast may lack thereceptor required for the hormone to act.
182.2)
How could you improve your experiment from the previous question?
A) Apply the hormone to a mouse cell culture instead of a yeast cell culture. B) Increase the concentration of the hormone by 0.5 M at each application until you see an effect. C) Hydrolyze thehormone before application. D) Add ATP as acofactor.
183) The basic categories of hormones are peptide or proteins, steroids, and amino acid derivatives. The chemical properties of each hormone type determine some of its functional properties and how it must interact with receptors to signal to the cell.
183.1) A new hormone is discovered. Tests reveal that it is small, hydrophilic, and degraded by a protease. What type of hormone is it? A) Peptide B) Amino acid derivative C) Steroid D) Glucocorticoid E) Prostaglandin
184) Cushing's syndrome causes weight gain, excessive sweating, and muscle weakness. It can result from either the production of abnormally high levels of cortisol by the body or overuse of glucocorticosteroid drugs. You are an endocrinologist working with a Cushing's syndrome patient. She is on hormonal birth control pills and takes an occasional ibuprofen.
Version 1
62
184.1)
What lab tests would you order to find the source of the problem?
A) Cortisol, LH, and CRH B) None—the birth control pills include corticosteroid hormones. She should switch to another form of birth control. C) Cortisol, ACTH, and CRH D) Cortisol, FSH, and ACTH
184.2) Results from lab tests reveal abnormally high endogenous hormone levels, and you suspect a possible tumor. What regions will you ask the MRI technician to focus on? A) Hypothalamus, anterior pituitary, and adrenal cortex B) Hypothalamus and adrenal cortex C) Hypothalamus and posterior pituitary D) Anterior and posterior pituitary
185) Two kinds of custom antibodies are available—monoclonal or polyclonal. Polyclonal antibodies are created in responseto a mixture of epitopes oncomplex injected antigenic material; monoclonal antibodies are produced bya hybridoma strain, which is created from fusion of an immortal tumor cell line with a single B lymphocyte, and thus respond to a single specific epitope.
185.1) You are designing a fast assay system for detecting flu virus in patients. You hope to send these tests to clinics in far-flung regions of the globe so they can conduct surveillance on influenza epidemics. The kits will need to remain functional for several years. What type of antibody will you choose? A) Polyclonal B) Monoclonal
Version 1
63
185.2) Your initial version of the influenza assay kit was a great success. Now you are making a new version that will detect one specific strain of influenza that is particularly virulent, and distinguish it from other typical strains. What type of antibody would be best in this case? A) Polyclonal B) Monoclonal
186) Different viruses have different properties that affect how the immune system responds to them and consequently how vaccines must be developed.
186.1) Every year a new version of the flu vaccine is made using a mixture of strains. The strains are chosen based on surveillance and analysis by experts from the FDA, WHO, and CDC. For other diseases however, such as varicella (chickenpox), the same vaccine is used continually and repeated vaccinations are not necessary. What do you conclude is the important difference between the influenza virus and the varicella zoster virus with regards to vaccinations? A) Influenza virus istransmitted through aerosol droplets in the air, while varicella can betransmitted via skin secretions. B) Varicella isattacked by the innate immune system, while influenza is attacked by theacquired immune system. C) Influenza virus hasa much higher mutation rate than varicella, leading to frequent antigen changesin the coat proteins. D) Influenza envelope proteins are boundby IgM, while varicella envelope proteins are bound by IgG.
186.2) What is a consequence of the fact that the chickenpox virus has a much lower mutation rate than the influenza virus?
Version 1
64
A) Chickenpox viruscan "hide out" as latent virus in nerve cell bodies, but influenzacannot. B) Chickenpox createsskin lesions, while influenza leads to body aches. C) Chickenpox viruscan reemerge decades later with the symptoms of shingles, while influenza istransient. D) A chickenpox infection generally givesimmunity against a repeat infection, but people can catch the flu everyyear.
187) Match the letter of each phase from childbirth to the appropriate component of a positive feedback loop. A. Pituitary gland secretes oxytocin, whichacts on the uterus B. Uterine contractions increase C. Fetus is pushed against uterine opening D. Receptors detect stretch E. Brain receives and compares stretch information
187.1)
Effector
187.2)
Integrating Center
187.3)
Response
187.4)
Stimulus
188) Adult giraffes can be up to 20 ft tall, and thus require special adaptations to maintain appropriateblood pressure. One adaptation is the rete mirabile, a group of blood vessels that form a web or net-like structure to slow blood flow.
Version 1
65
188.1) The giraffe, as the tallest living land animal, has several adaptations for its height. To prevent blood from pooling in the head when it bends down to drink, the jugular veins of a giraffe have multiple one-way valves. Furthermore, a web of arteries and veins in the neck called the rete mirabile (“wonderful net”) slows the flow of blood into the brain. These specializations of the __________ act as mechanisms to maintain __________. A) Nervous system; homeothermy B) Nervous system; temperature homeostasis C) Circulatory system; blood pressure homeostasis D) Circulatory system; pH homeostasis
188.2) Although especially useful to the giraffe, the rete mirabile is similar to a countercurrent heat exchange system in other related mammals. Thus a mechanism for __________ homeostasis is likely to have evolved from a mechanism previously used for __________ homeostasis. A) blood pressure; pH B) temperature; blood pressure C) blood pressure; temperature D) pH; blood pressure
Version 1
66
Answer Key Test name: Chap 35_3e 1) FALSE Hormone concentrations in the blood can be as low as 10
-10
M.
2) TRUE Examples of molecules that can act as both are epinephrine and norepinephrine. 3) FALSE Brain regions such as the hypothalamus, posterior pituitary, and pineal gland are endocrine organs that coordinate body functions via bloodtransported hormones. 4) TRUE 5) B 6) C
Version 1
67
Clarify Question • What is the key concept addressed by the question? How do adrenal glands release their hormones? • What type of thinking is required? This is an evaluate question because you have to judge different possibilities for an explanation for the lack of a duct in anadrenal gland. Gather Content • What do you already know about adrenal gland? What other information is related to the question? o You already know that the adrenal gland is part of the endocrine system. What you may not know is that the adrenal gland has one of the greatest networks of arteries and veins of any tissue of the body. Thus, the adrenal gland does not need a duct;the hormones can diffuse directly into the capillaries. Choose Answer • Given what you now know, what information is most likely to produce the correct answer? o Exocrine glands release substances via ducts. As adrenal glands don’t have ducts, then they are not exocrine organs. The adrenal gland is a “ductless gland” that diffuses hormones into nearby capillaries. Thus, the gland in this cadaver is normal and will not result in adrenal insufficiency. Nerves do not transport hormones, so that answer cannot be correct. Hormones are transported via the bloodstream. Reflect on Process • Did your problem-solving process lead you to the correct answer? If not, where did the process break down or lead you astray? How can you revise your approach to produce a more desirable result? Version 1
68
o Answering this question correctly depended on understanding that adrenal glands do not have ducts, but that hormones diffuse directly through a bed of capillaries. If you got the right answer, great! If not, which answer did you choose? Did you think that the adrenal gland is an exocrine organ since it “excretes” hormones to the surrounding blood vessels? 7) B 8) B
Version 1
69
Clarify Question • What is the key concept addressed by the question? The effects of overconsumption of licorice. • What type of thinking is required? This is an evaluate question because you have to access various symptoms to see which of these systems would be produced by excess aldosterone. Gather Content • What do you already know about the aldosterone? What other information is related to the question? o Black licorice inhibits an enzyme which normally limits activation of mineralocorticoid receptors. Since aldosterone is a mineralocorticoid, this mimics the effect of excess aldosterone. Aldosterone regulates blood pressure by regulating mineral balance, specifically by stimulating the kidneys to reabsorb Na+ ions and secreting K+ ions in the urine. As a result, aldosterone increases water retention and increases blood pressure. Choose Answer • Given what you now know, what information is most likely to produce the correct answer? o Applying what you know, you can discount the answers concerning sexual characteristics such at infertility or excessive body hair, because aldosterone is a mineralocorticoid involved in the regulation of mineral balance and not a sex steroid hormone like testosterone. The remaining answers deal with blood pressure. So how does aldosterone affect blood pressure? It increases it by increasing blood Na+ and decreasing K+. Reflect on Process
Version 1
70
• Did your problem-solving process lead you to the correct answer? If not, where did the process break down or lead you astray? How can you revise your approach to produce a more desirable result? o Answering this question correctly depended upon understanding the role of aldosterone on blood pressure and the mechanism of this regulation. Then you had to evaluate each answer to see if excess aldosterone would fit the symptom. If you selected the correct answer, great! If not, what went wrong? Did you confuse mineralocortocoids with steroid hormones? Did you think that aldosterone decreased blood pressure instead of increasing it? 9) A
Version 1
71
Clarify Question • What is the key concept addressed by the question? What would loss of the adrenal glands cause? • What type of thinking is required? This is an evaluate question because you have to access various symptoms to see which of these systems would be produced by the loss of aldosterone. Gather Content • What do you already know about aldosterone? What other information is related to the question? o The adrenal glands produce aldosterone, so loss of these glands would result in severe loss of mineral balance within the body. Aldosterone regulates blood pressure by regulating mineral balance, specifically by stimulating the kidneys to reabsorb Na+ ions and secreting K+ ions in the urine. As a result, aldosterone increases water retention and increases blood pressure. Without aldosterone, blood pressure would drop and the brain and the rest of the body tissues would not get enough oxygen. In addition, the loss of Na+ would result in problems sending neural signals as action potentials are based on Na+ rushing into neurons. Choose Answer • Given what you now know, what information is most likely to produce the correct answer? o Applying what you know, you can discount the answer concerning sexual characteristics such at infertility, because aldosterone is a mineralocorticoid involved in the regulation of mineral balance and not a sex steroid hormone like testosterone. Insulin and glucagon regulate blood sugar, not aldosterone, so that is not the correct answer. Aldosterone does control blood pressure and a drop in blood pressure, Version 1
72
although might initially produce diminished cognitive function, will ultimately result in death. Reflect on Process • Did your problem-solving process lead you to the correct answer? If not, where did the process break down or lead you astray? How can you revise your approach to produce a more desirable result? o Answering this question correctly depended upon understanding the role of aldosterone on blood pressure and the mechanism of this regulation. Then you had to evaluate each answer to see if a lack of aldosterone would fit the symptom. If you selected the correct answer, great! If not, what went wrong? Did you confuse mineralocortocoids with steroid sex hormones? Did you think that aldosterone decreased blood pressure instead of increasing it? Did you realize that a severe drop in blood pressure would result in death since oxygen would not be delivered to the brain? 10) E 11) B 12) C
Version 1
73
Clarify Question What is the key concept addressed by the question? • What is a special consideration for the injection of synthetic oxytocin? What type of thinking is required? o This is an evaluate question because you have to consider the role of oxytocin in childbirth and what precautions might need to be taken while inducing labor. Gather Content What do you already know about oxytocin? What other information is related to the question? • Oxytocin induces childbirth by stimulating uterine contractions. Synthetic oxytocins, such as Pitocin, mimic the chemical structure of natural oxytocin, and thus bind to oxytocin receptors, which are Gprotein coupled receptors. Within short periods of time, the amount of oxytocin receptors and G-proteins remains stable so that shifts in hormone levels result in physiological changes. Too much oxytocin can result in uterine contractions being too strong. This could potentially harm the baby during delivery. Choose Answer Given what you now know, what information is most likely to produce the correct answer? • You know that oxytocin is associated with childbirth and nursing, and that synthetic versions of it can induce labor and delivery via stimulating uterine contractions. You also know that oxytocin receptors are Gprotein mediated receptors and are only activated by oxytocin or analogs of oxytocin, but that G-proteins are associated with cell membranes and not transported around the bloodstream the way that hormones are. Version 1
74
Thus, synthetic G-proteins do not need to be given along with synthetic oxytocin. If oxytocin can induce uterine contractions, then too much oxytocin could cause contractions that are strong enough to harm the baby. Thus, doctors must take great care with using synthetic oxytocin to induce labor. Reflect on Process Did your problem-solving process lead you to the correct answer? If not, where did the process break down or lead you astray? How can you revise your approach to produce a more desirable result? o Answering this question correctly depended upon understanding the roles of oxytocin, and determining what might go wrong with it’s administration. If you got the answer correct, great! If not, where did you go wrong? Did you think that synthetic oxytocin would not be as effective as natural oxytocin? Did you remember that oxytocin receptors are G-protein mediated, and so thought that synthetic G-proteins might be needed along with the synthetic oxytocin? 13) D
Version 1
75
Clarify Question • What is the key concept addressed by the question? Which hormones play roles in both the ovarian cycle and sperm production? • What type of thinking is required? This is an analyze question because you have to consider the roles of a number of hormones and match those roles to the proper pair of hormones. Gather Content • What do you already know about sex hormones? What other information is related to the question? o The couple is being tested for the same hormones related to conceiving a child. Luteinizing hormone (LH) stimulates ovulation and corpus leteum formation in females and stimulates the secretion of testosterone in males. Follicle-stimulated hormone (FSH) stimulates spermatogenesis in males and the development of ovarian follicles in females. Choose Answer • Given what you now know, what information is most likely to produce the correct answer? o You know that FSH is involved in both the production of sperm and eggs and that LH stimulates ovulation and testosterone secretion. Both are necessary for conceiving a child, and so this is most likely the correct answer. But to make sure, let us consider the other alternatives. Insulin and glucagon are involved in the regulation of blood sugar levels. Adrenocotricotropic hormone and corticotropin-releasing hormone stimulate the release of adrenal cortical hormones such as cortisol, so it does not play a direct role in reproduction. Oxytocin does Version 1
76
play a role in reproduction as it promotes uterine contraction in childbirth and milk let-down in nursing, and also promotes pair bonding and sexual arousal between mates. Although oxytocin and LH are both involved in reproduction, FSH and LH are the hormones that would most likely cause problems for conception if they were not regulated properly in both the male and female. Reflect on Process • Did your problem-solving process lead you to the correct answer? If not, where did the process break down or lead you astray? How can you revise your approach to produce a more desirable result? o Answering this question correctly depended upon understanding the roles of various hormones, and determining which ones might cause a problem for conception. If you got the answer correct, great! If not, where did you go wrong? Did you think that ACTH and CRH were sex hormones? Did you remember that oxytocin is involved with reproduction and can affect both male and females, and so thought that these two hormones would affect fertility? Did you forget that insulin and glucagon are involved with the regulation of blood sugar? 14) B
Version 1
77
Clarify Question • What is the key concept addressed by the question? What hormone is added to milk? • What type of thinking is required? This is an analyze question because you have to consider what is generally added to milk as well as the definition of a hormone. Gather Content • What do you already know about hormones? What other information is related to the question? o Bovine growth hormone (BGH) has been given to cows to increase milk production and some of this hormone can end up in the milk. Vitamin D is also commonly added to milk since Vitamin D helps the intestinal absorption of calcium found in milk. Vitamin D becomes a hormone after the addition of two hydroxyl groups. It is this hormone form of Vitamin D that promotes the absorption of calcium and other minerals from the intestines. Choose Answer • Given what you now know, what information is most likely to produce the correct answer? o Both Vitamin A and D are added to milk. Vitamin A is added to replace loss when low-fat and reduced-fat milk is processed. Vitamin D is found naturally in milk, but milk is often fortified with additional Vitamin D to assist in the absorption of calcium. MSG (mono-sodium glutamate) isnot added to milk, butcan be found naturally in many dairy products. Preservatives are not put in milk sold for public consumption. Instead, potential pathogens are controlled by a combination of heat treatment (pasteurization), refrigeration, and protection from light. Estrogen is not added to milk. So if Vitamin A and Vitamin D are the Version 1
78
only answers that are actually added to milk, which one is a hormone (or a prehormone)? Vitamin D. Reflect on Process • Did your problem-solving process lead you to the correct answer? If not, where did the process break down or lead you astray? How can you revise your approach to produce a more desirable result? o Answering this question correctly depended upon understanding which molecules are hormones, and which molecules are added to milk. If you got the answer correct, great! If not, where did you go wrong? Did you know that Vitamin A is added to milk so think that it was the correct answer? 15) D 16) D 17) E 18) D
Version 1
79
Clarify Question • What is the key concept addressed by the question? Why do lampreys not need a parathyroid gland? • What type of thinking is required? This is an evaluate question because you have to consider different potential reasons why a parathyroid gland is not needed in lampreys. Gather Content • What do you already know about the parathyroid gland? What other information is related to the question? o The parathyroid gland releases parathyroid hormone, which increases blood calcium levels by promoting the dissolution of calcium phosphate crystals of the bone matrix, the increased absorption of calcium by the intestines, and the reabsorption of calcium by the kidneys. Choose Answer • Given what you now know, what information is most likely to produce the correct answer? o Why would lampreys not need a parathyroid gland while other vertebrates do? The parathyroid hormone regulates calcium and a large portion of calcium is usually stored in bones. Bones are something the cartilaginous lampreys do not have, thus they have less need to regulate calcium levels as vertebrates with bones. Lampreys are parasitic and feed on other fishes, but their teeth in their circular mouths are not designed to crunch bones of their hosts, but to bite out little pieces of muscle, so they do not get considerable amounts of calcium from feeding. Reflect on Process • Did your problem-solving process lead you to the correct answer? If Version 1
80
not, where did the process break down or lead you astray? How can you revise your approach to produce a more desirable result? o Answering this question correctly depended upon understanding the function of the parathyroid gland and thinking about how lampreys are different from other vertebrates. If you got the answer correct, great! If not, where did you go wrong? Did you know that lampreys are parasites and think that they received plenty of calcium from their hosts? 19) A 20) C 21) D 22) C
Version 1
81
Clarify Question • What is the key concept addressed by the question? How does melatonin function? • What type of thinking is required? This is an evaluate question because you have to consider different reasons why the roommate should or should not take melatonin after thinking about the role of melatonin. Gather Content • What do you already know about melatonin? What other information is related to the question? o Melatonin is produced by the pineal gland deep within the brain. It helps to regulate circadian rhythms and is activated in the dark. Thus melatonin levels are high at night and promote sleep. Thus it would be unwise to take melatonin in the morning because it would make you sleepy. Choose Answer • Given what you now know, what information is most likely to produce the correct answer? o Each answer describes the effects of a different hormone. The answer referring to brittle bones may be referring to calcitonin, which inhibits loss of calcium from bone, or parathyroid hormone, which also regulates calcium by removing calcium from bone. The answer mentioning blood sugar dropping is referring to insulin, which promotes the uptake of glucose from the blood to the cells. The answer mentioning the “flightor-flight” response is referring to epinephrine or norepinephrine. The answer mentioning biological rhythms must be the correct answer since melatonin regulates circadian rhythms such as the sleep-wake cycle. Reflect on Process Version 1
82
• Did your problem-solving process lead you to the correct answer? If not, where did the process break down or lead you astray? How can you revise your approach to produce a more desirable result? o Answering this question correctly depended upon understanding the function of melatonin and thinking about the effects of taking melatonin in the morning. If you got the answer correct, great! If not, where did you go wrong? Did you remember that melatonin affects biological rhythms such as the sleep-wake cycle? Did you see how the other responses fit into the roles of other hormones? 23) E 24) C 25) D 26) A
Version 1
83
Clarify Question • What is the key concept addressed by the question? Which hormone needs a cell surface receptor? • What type of thinking is required? This is an analyze question because you have to consider how these hormones differ from each other and how they interact with receptors differently. Gather Content • What do you already know about hormones? What other information is related to the question? o Hormones are signaling molecules that travel via the bloodstream to target tissues where they bind to receptors to initiate cellular responses. There is two types of receptor binding in hormones, binding that occurs at receptors on the cell surface and binding the occurs on intracellular receptors. Hydrophobic (or lipophilic) molecules can diffuse across the phospholipid bilayer of cell membranes. Steroid hormones are cholesterol based and can diffuse directly through cell membranes. Protein hormones such as insulin cannot; they must bind to receptors on the cell surface. Choose Answer • Given what you now know, what information is most likely to produce the correct answer? o Cortisol, testosterone, estrogen, and progesterone are all steroid hormones. Thus these hormones do not need cell-surface receptors. Insulin is a small polypeptide that cannot diffuse across the cell membrane and thus needs cell-surface receptors to bind to. Reflect on Process
Version 1
84
• Did your problem-solving process lead you to the correct answer? If not, where did the process break down or lead you astray? How can you revise your approach to produce a more desirable result? o Answering this question correctly depended upon understanding the function of hormone-receptor binding and realizing that some hormones can cross the cell membrane while others cannot. If you got the answer correct, great! If not, where did you go wrong? Did you forget that cortisol is a steroid hormone? Did you forget that steroid hormones can diffuse across the cell membrane and thus do not need a cell-surface receptor? 27) C 28) B 29) B 30) C 31) C 32) E 33) B 34) B 35) B 36) C 37) C 38) C 39) D 40) C 41) B 42) B 43) D 44) B 45) B 46) E Version 1
85
47) A 48) E 49) C 50) C 51) D
Version 1
86
Clarify Question • What is the key concept addressed by the question? Which hormone has the fewest targets? • What type of thinking is required? This is an analyze question because you have to separately consider each hormone and list how many target tissues each one has. Gather Content • What do you already know about hormones? What other information is related to the question? o Growth hormone (GH) has many targets. In fact, most cell types have GH receptors. Oxytocin primarily targets the uterus and mammary glands, but can also affect males without these structures. Melatonin targets the gonads, brain, and pigment cells. Testosterone targets male reproductive structures and many organs that effect the growth of secondary sex characteristics and puberty. Thyroid-stimulating hormone, as the name suggests, only stimulates the thyroid gland. Choose Answer • Given what you now know, what information is most likely to produce the correct answer? o Since all of the hormones affect multiple target tissues except thyroidstimulating hormone (TSH), then TSH must be the hormone with the fewest targets. Reflect on Process • Did your problem-solving process lead you to the correct answer? If not, where did the process break down or lead you astray? How can you revise your approach to produce a more desirable result? o Answering this question correctly depended upon understanding the Version 1
87
targets for each of the given hormones. If you got the answer correct, wonderful! If not, which answer did you choose? Were you unclear on the targets of some of these hormones? Did you think that testosterone only affects the gonads in males or that oxytocin only affects the uterus and breasts of females? 52) A 53) C 54) A 55) D
Version 1
88
Clarify Question • What is the key concept addressed by the question? How do hormones target part of the body? • What type of thinking is required? This is an analyze question because you have to consider each answer for accuracy in describing how hormones deliver their signals to the appropriate target tissues. Gather Content • What do you already know about hormones? What other information is related to the question? o Hormones deliver their signals through the bloodstream. As blood travels to all tissues, why don’t the hormones equally affect all tissues? It is because only tissues that express the receptors for the hormone will allow the hormone to bind and transmit the signal inside the cell. Choose Answer • Given what you now know, what information is most likely to produce the correct answer? o There are no special gateway valves in blood vessels to direct hormones to their target tissues. This would not work since multiple hormones are transported across all the blood, so it would be impossible to shunt blood with one hormone one direction and another hormone to another tissue. As blood flows rather quickly, there is no need for carrier proteins to move along microtubule tracks to deliver hormones. Axonal pathfinding and transport only deals with neural tissue and hormones can affect all tissue types. Thus, the answer must be that only target tissues have receptors that allow them to receive the signal. Reflect on Process
Version 1
89
• Did your problem-solving process lead you to the correct answer? If not, where did the process break down or lead you astray? How can you revise your approach to produce a more desirable result? o Answering this question correctly depended upon understanding that differential expression of receptors is how hormones communicate with specific targets. If you got the answer correct, wonderful! If not, which answer did you choose? Did you know that hormones bind to receptors but not realize that receptors for specific hormones are only expressed in specific tissues? 56) D 57) C
Version 1
90
Clarify Question • What is the key concept addressed by the question? What gene is best for drug targeting? • What type of thinking is required? This is an evaluate question because you have to consider each potential gene to pursue as a potential therapy for hormonal deficiency and consider its benefits and drawbacks. Gather Content • What do you already know about hormones? What other information is related to the question? o Hormones deliver their signals through the bloodstream. As blood travels to all tissues, they bind to either cell-surface or intracellular receptors to initiate changes in the cell.Hormones can be regulated at many levels. Genes for hormones can be regulated by transcription factors. Precursor hormone molecules are modified by enzymes to convert them into hormones. In addition, regulation can occur downstream of the hormone in a cellular pathway. While modifying all these regulatory methods could help with hormone deficiencies, supplying patients with more of the hormone itself is the most efficient means of therapy. Choose Answer • Given what you now know, what information is most likely to produce the correct answer? o Peptide hormones are an attractive drug target since they are easily synthesized and become bioavailable as soon as they enter the bloodstream. Transcription factors are generally difficult to target with drugs. The enzyme is likely to be large and intracellular. The kinase is intracellular and likely functions in many other tissues, so targeting it Version 1
91
may produce negative side effects. Reflect on Process • Did your problem-solving process lead you to the correct answer? If not, where did the process break down or lead you astray? How can you revise your approach to produce a more desirable result? o Answering this question correctly depended upon considering different levels of regulation of hormones. If you got the answer correct, great! If not, which answer did you choose? Did you choose transcription factor since you knew that transcription is the first step in turning a peptide hormone gene into a protein? That is true, but, given a limited budget, the most direct and effective method would be to simply supply the patient with the hormone itself. 58) C
Version 1
92
Clarify Question • What is the key concept addressed by the question? Which company is the best partner for hormone drug development? • What type of thinking is required? This is an evaluate question because you have to consider each potential company’s comments to assess their understanding of lipophilic hormones. Gather Content • What do you already know about hormones? What other information is related to the question? o Hormones deliver their signals through the bloodstream. Since the drug that your lab developed mimics a lipophilic hormone, it will not beblood soluble and musttravel through the blood with the help of transport proteins. Hormones bind to either cell-surface receptors or intracellular receptors. Lipophilic hormones can travel through the cell membrane while hydrophilic hormones such as proteins cannot cross the cell membrane and so must bind to cell-surface receptors. Lipophilic hormonesthen directly cross the cell membrane and interact with DNA to regulate genes. Because Company A and B mentioned needed a drug delivery system to get the drug across the cell membrane and G-protein coupled receptors that are associated with cell-surface receptors, they were showing their ignorance as to how lipophilic hormones work. Company D referred to the drug being soluble in the blood, which shows a lack of understanding of the differences between lipophilic and hydrophilic hormones. Choose Answer • Given what you now know, what information is most likely to produce the correct answer? Version 1
93
o Because Company A and B mentioned needing a drug delivery system to get the drug across the cell membrane and G-protein coupled receptors that are associated with cell-surface receptors, they were showing their ignorance as to how lipophilic hormones work. Company D referred to the drug being soluble in the blood, which shows a lack of understanding of the differences between lipophilic and hydrophilic hormones. Thus, you would want to select Company C. Reflect on Process • Did your problem-solving process lead you to the correct answer? If not, where did the process break down or lead you astray? How can you revise your approach to produce a more desirable result? o Answering this question correctly depended upon understanding how lipophilic hormones work. If you got the answer correct, great! If not, which answer did you choose? Did you think that lipophilic hormones would be associated with G-proteins or need some other means to get across the cell membrane? Lipophilic (or hydrophobic) molecules can readily cross the cell membrane, so such mechanisms are not needed. 59) B
Version 1
94
Clarify Question • What is the key concept addressed by the question? Response time of steroid drugs. • What type of thinking is required? This is an evaluate question because you have to consider each response to assess its accuracy in understanding the mode of action of lipophilic hormones. Gather Content • What do you already know about hormones? What other information is related to the question? o Hormones deliver their signals through the bloodstream. As blood travels to all tissues, they bind to either cell-surface receptors or intracellular receptors. Lipophilic hormones, like steroids, can travel through the cell membrane while hydrophilic hormones such as proteins cannot cross the cell membrane and so must bind to cell-surface receptors. Since the treatment drug is a steroid, it will directly cross the cell membrane and interact with DNA to regulate genes. The process of transcription and translation take time, so the effect of the drug will not be immediate. Choose Answer • Given what you now know, what information is most likely to produce the correct answer? o The drug that is being administered is a steroid. Steroid hormones cross the cell membrane to interact with DNA to regulate gene expression. No carrier protein is needed so that answer is incorrect. Similarly, no G-protein coupled receptor or downstream kinase cascade is needed, so that answer is also incorrect. Gene expression involves transcription of the DNA into mRNA and translation of the mRNA into Version 1
95
protein. As this process takes some time, you should not expect the drug to have an immediate effect. Increasing the dose without giving the original dose time to have an effect could be dangerous. Reflect on Process • Did your problem-solving process lead you to the correct answer? If not, where did the process break down or lead you astray? How can you revise your approach to produce a more desirable result? o Answering this question correctly depended upon understanding how lipophilic hormones work. If you got the answer correct, great! If not, which answer did you choose? Did you think that lipophilic hormones would be associated with G-proteins or need some other means (like a carrier protein) to get across the cell membrane? Lipophilic (or hydrophobic) molecules can readily cross the cell membrane, so such mechanisms are not needed. Did you think that increasing the dosage would be useful? High levels of steroids in the blood are not needed for them to be effective. 60) B 61) B 62) D
Version 1
96
Clarify Question • What is the key concept addressed by the question? What could problems with your pituitary gland cause? • What type of thinking is required? This is an analyze question because you have to carefully examine each response to assess its accuracy in terms of describing the hypophysis. Gather Content • What do you already know about the pituitary gland? What other information is related to the question? o The hypophysis is another name for the pituitary gland. The pituitary gland hangs from a stalk from the hypothalamus at the base of the brain. This gland has two parts, the anterior pituitary, or adenohypophyis, and the posterior pituitary gland, or the neurohypophysis. These two portions have different embryonic origins and secrete different hormones. Developmentally, the posterior pituitary is derived from the hypothalamus of the brain, while the anterior pituitary is derived from a pouch of epithelia tissue from the roof of the embryo’s mouth. Hormones released by the posterior pituitary gland include antidiuretic hormone and oxytocin, while the anterior pituitary gland produces adrenocorticotropic hormone, melanocyte-stimulating hormone, growth hormone, prolactin, thyroid-stimulating hormone, luteinizing hormone, and follicle-stimulating hormone. Choose Answer • Given what you now know, what information is most likely to produce the correct answer? o The individual responses are like multiple True/False questions. From the reading you should know that the pituitary is a compound endocrine gland composed of the anterior and posterior pituitary. The posterior Version 1
97
portion is derived from the brain and the anterior pituitary is not, thus the response linking neurological problems to the posterior is accurate. Weight gain, fatigue, and feeling cold are symptoms of hypothyroidism, which is different from the hypophysis. The posterior pituitary produces at least two hormones and the anterior produces at least seven hormones, so the pituitary is responsible for producing at least nine major hormones, not just one or two. As the pancreas produces insulin, and not the hypophysis, the patient should not need insulin shots. Reflect on Process • Did your problem-solving process lead you to the correct answer? If not, where did the process break down or lead you astray? How can you revise your approach to produce a more desirable result? o Answering this question correctly depended upon assessing each statement as to its accuracy in describing the pituitary gland. If you got the answers correct, great! If not, which answers did you choose? Did you think that the hypophysis only produce one or two hormones? It produces at least nine. Did you confuse the work hypothesis with hypophysis? Did you think that the hypophysis, or pituitary gland, produced insulin? Insulin is produced by the pancreas. 63) C
Version 1
98
Clarify Question • What is the key concept addressed by the question? What is the likely cause of acromegaly? • What type of thinking is required? This is an evaluate question because you have to consider each potential cause of acromegaly and decide whether it would lead to increased growth hormone. Gather Content • What do you already know about growth hormone? What other information is related to the question? o Growth hormone (GH) is produced by the anterior pituitary gland and regulates the growth of muscles, bones, and other tissues. The hypothalamus produces two hormones that regulate the production of GH, growth hormone-releasing hormone (GHRH) and growth hormoneinhibiting hormone (GHIH). As the names suggest, GHRH increases GH production and GHIH decreases it. Since the patient had no growth abnormalities in her youth, her problem is something that has changed recently. Choose Answer • Given what you now know, what information is most likely to produce the correct answer? o While the patient is middle-aged and may be starting to experience menopause, hormonal shifts due to menopause do not significantly affect growth hormone or cause acromegaly. The pineal gland affects circadian rhythms in the body, not growth hormone. The anterior pituitary gland produces GH, not the posterior pituitary, so a tumor in the anterior pituitary gland could lead to an enlarged gland that produces more GH and lead to acromegaly. The patient was normal in her youth, Version 1
99
which suggests that the problem is something recent, not a congenital mutation in the GH regulatory region that she was born with. Reflect on Process • Did your problem-solving process lead you to the correct answer? If not, where did the process break down or lead you astray? How can you revise your approach to produce a more desirable result? o Answering this question correctly depended upon evaluating each potential cause as to its appropriateness in finding the cause of increased growth hormone and thus acromegaly. If you got the answer correct, great! If not, which answer did you choose? Did you think that GH was produced in the posterior pituitary instead of the anterior pituitary? Did you think that because the woman is middle-aged that it had something to do with menopause? 64) D
Version 1
100
Clarify Question • What is the key concept addressed by the question? What could cause milk production not associated with pregnancy? • What type of thinking is required? This is an evaluate question because you have to consider each potential cause of nonpregnancy-related milk production. Gather Content • What do you already know about the pituitary gland? What other information is related to the question? o The pituitary gland produces prolactin, which stimulates milk production in mammals. Estrogen and progesterone are also involved in the regulation of milk production, but they do so by blocking the release of prolactin. The doctor sent the patient for a brain scan, and since estrogen and progesterone are produced in the ovaries, the doctor must not be concerned about any problems with the ovaries in producing this milk production. Choose Answer • Given what you now know, what information is most likely to produce the correct answer? o One of the potential responses discussed scanning the thyroid gland, potentially because the patient is not getting enough iodine. Lack of iodine leads to goiter, or an enlarged thyroid gland, which is not the symptom of the patient. LH and FSH do not stimulate milk production but regulates ovulation in females. Thus, the most likely reason for the MRI brain scan is to rule out a pituitary tumor since prolactin, which stimulates milk production, is produced in the anterior pituitary gland. Reflect on Process Version 1
101
• Did your problem-solving process lead you to the correct answer? If not, where did the process break down or lead you astray? How can you revise your approach to produce a more desirable result? o Answering this question correctly depended upon evaluating each potential response concerning your friend’s MRI brain scan. If you got the answer correct, great! If not, which answers did you choose? Did you think that since LH and FSH regulate ovulation that they are female sex hormones that would affect milk production as well? Did you think the technician should have done a thoracic scan to look at the ovaries since they produce estrogen and progesterone, which is also involved in milk production? Remember that estrogen and progesterone, which are high during pregnancy, inhibit the release of prolactin and thus would prevent mild production, not stimulate it. 65) A 66) D 67) B 68) C 69) B
Version 1
102
Clarify Question • What is the key concept addressed by the question? What intracellular event is prevented by beta-blockers? • What type of thinking is required? This is an analyze question because you have to discriminate between different potential results of using a beta-blocker. Gather Content • What do you already know about G protein-coupled receptors? What other information is related to the question? ● G protein-coupled receptors are found in the cell membrane and ligands outside of the cell bind to the receptors to activate G proteins located inside the cell. ● When activated, G proteins bind to GTP and are released from the receptor. These G proteins activate second-messenger producing enzymes to elicit changes within the cell. ● Such receptors are important for hydrophilic molecules such as protein hormones that cannot cross the phospholipid bilayer of the cell membrane. Choose Answer • Given what you now know, what information is most likely to produce the correct answer? ● One of the answers suggests that the drug would need to get into the cell’s nucleus where it would regulate transcription. While hydrophobic molecules like steroid hormones can diffuse across the cell membrane and into the cell’s nucleus to elicit changes in gene expression, protein hormones cannot do this and so must use receptors located on the outside of the cell. ● G protein-coupled receptors bind to a variety of unique ligands, so Version 1
103
can be involved in a number of different cellular pathways. G proteins are indeed proteins. They are enzymes that bind to GTP when active and hydrolyze it to GDP. ● Activated G protein activates channels or enzymes that produce second messengers like cAMP or inositol triphosphate. Receptors that activate intracellular kinase domains act on the amino acids serine, threonine, and tyrosine. ● Phosphatases remove phosphates instead of adding them like kinases. Phosphatases are usually not associated with receptors. Reflect on Process • Did your problem-solving process lead you to the correct answer? If not, where did the process break down or lead you astray? How can you revise your approach to produce a more desirable result? o Answering this question correctly depended upon analyzing each potential answer for accuracy in terms of describing the role of G protein-coupled receptors. If you got the answers correct, great! If not, which answers did you choose? Did you think that G-proteins cross the nuclear membrane? Because proteins are usually hydrophilic, they do not readily cross membranes. G proteindoeshydrolyze GTP to GDPand is a protein. If you selected one of the answers referring to intracellular kinase domains and phosphatase domains, they were referring to different types of receptors/enzymes and not G protein-coupled receptors. 70) D
Version 1
104
Clarify Question • What is the key concept addressed by the question? What intracellular molecule transmits peptide hormone signals? • What type of thinking is required? This is an analyze question because you have to discriminate between different potential signaling molecules. Gather Content • What do you already know about peptide receptors? What other information is related to the question? o Peptide hormone receptors can either be a G protein-coupled receptor or a receptor kinase. In both types of receptors, the intracellular cascade often involves one or a series of kinases each transferring a phosphate group onto their targets. Choose Answer • Given what you now know, what information is most likely to produce the correct answer? ● Nucleotides are found in DNA and RNA. Since peptide hormones do not enter the nucleus like steroid hormones do, they would not regulate gene expression to produce an effect. ● Water is a ubiquitous molecule both intracellularly and extracellularly (most of the cell is composed of water). Thus, it would not make a good signaling molecule. ● Nitric oxide is known to act as a hormone and neurotransmitter, but it is a gas, not a peptide hormone. ● Amino acids are what proteins are made of, but peptide hormones do not disintegrate into amino acids to elicit their effects; the amino acids are needed to maintain the three-dimensional structure of the protein so it will fit its appropriate receptors. Version 1
105
● That leaves phosphate groups. In both types of peptide hormone receptors, G protein-coupled receptors and receptor kinases, phosphorylation, or the addition of phosphate groups to activate proteins, is involved. Reflect on Process • Did your problem-solving process lead you to the correct answer? If not, where did the process break down or lead you astray? How can you revise your approach to produce a more desirable result? o Answering this question correctly depended upon analyzing each potential signaling moleculefor hydrophilic (peptide) hormones. If you got the answers correct, great! If not, which answer did you choose? Did you think that because peptide hormones are made of amino acids, that amino acid would be the answer? That is true, but amino acids do not break off of peptides to become signaling molecules. Since hormone regulation often leads to shifts in gene expression, did you think that nucleotide was the answer? Hormones that directly regulate DNA are hydrophobic (steroid) ones, not peptides. 71) A 72) A 73) C (2-3)/180 x 100 = 1.1-1.67% (or 1-2% rounded down and up, respectively)
Version 1
106
Clarify Question • What is the key concept addressed by the question? • This question is asking about the efficiency of the kidneys. • What type of thinking is required? o This question is asking you to identify the percentage of filtrate out of the total blood plasma that is filtered that is released as urine per day. Gather Content • What do you already know about filtrate? What other information is related to the question? o Human kidneys produce urine that is much more concentrated than blood plasma. o Concentration of urine occurs in the nephrons as water is reabsorbed. Choose Answer • Given what you now know, what information is most likely to produce the correct answer? o Solving this question is accomplished by dividing the urine volume that forms by the volume of the blood plasma that is filtered, and multiplying that number by 100 to obtain a percent value. o 2/180 = 0.011 (100) = 1.1% Reflect on Process • Did your problem-solving process lead you to the correct answer? If not, where did the process break down or lead you astray? How can you revise your approach to produce a more desirable result? o Answering this question correctly depended your ability to recognize that the amount of urine that is formed can be expressed as a percent of the total plasma filtered. If you got an incorrect answer, did you remember that components of the urine formed in this case was 2-3 liters Version 1
107
and the amount of plasma that was filtered was 180 liters? Did you have trouble breaking down the amount of filtrate that becomes urine to determine the correct answer? 74) D 75) D 76) C 77) B 78) C 79) A 80) A 81) C
Version 1
108
Clarify Question • What is the key concept addressed by the question? This question is asking about the effects of dehydration. • What type of thinking is required? You are being asked to take what you already know and use, or apply, it to the physiological changes that occur with dehydration. Gather Content • What do you already know about dehydration? What other information is related to the question? o The hot, dry conditions of the desert can be very dehydrating in short period of time. o Humans have some water-conserving physiological traits, but they still must consume large amounts of water in very dry climates. o In the kidney, water is reabsorbed in the loop of Henle and in the collecting duct. Choose Answer • Given what you now know, what information is most likely to produce the correct answer? o Atrial natriuretic hormone causes water to be excreted, so increased presence of this hormone would be dangerous in the desert. o Blood osmolarity is something that must stay in homeostasis. Little deviations are not unusual, but a measureable decrease would not be compatible with life. o The sensation of thirst is an attempt to counteract the effects of dehydration, so decreasing this would not be advantageous in the desert. o Antidiuretic hormone caused the reabsorption of water. Decreasing this hormone would not help dehydration. o Since water can be reabsorbed through aquaporins in the collecting Version 1
109
duct, an increase in their permeability would aid water reabsorption and lessen the effects of dehydration. Reflect on Process • Did your problem-solving process lead you to the correct answer? If not, where did the process break down or lead you astray? How can you revise your approach to produce a more desirable result? o Answering this question correctly depended on your ability to use dehydration in a new situation. If you got an incorrect answer, did you remember that dehydration is hastened by a hot dry climate and by lack of water? Did you have trouble extending dehydration to determine the correct answer? 82) A 83) B
Version 1
110
Clarify Question • What is the key concept addressed by the question? This question is asking about the pathway that removes waste from the body of an insect. • What type of thinking is required? You are being asked to take what you already know and use, or apply, it to the waste-removal mechanism of an insect. Gather Content • What do you already know about insect physiology? What other information is related to the question? o Insects do not have blood as mammals do; they have hemolymph. o Hemolymph is a combination of circulatory fluid and extracellular fluid. Choose Answer • Given what you now know, what information is most likely to produce the correct answer? o Insects do not have kidneys, nor do they have nephrons or protonephridia. o Antennal glands are present in some freshwater arthropods, but their function is to excrete excess water. o Insects do not have a bladder, as their nitrogenous waste is excreted through the alimentary canal. o Insects do utilize Malphigan tubules for the secretion of their waste products, so the dye particles should be expected to appear in the Malphigian tubules as they are processed out of the body. Reflect on Process • Did your problem-solving process lead you to the correct answer? Version 1
111
If not, where did the process break down or lead you astray? How can you revise your approach to produce a more desirable result? o Answering this question correctly depended on your ability to use waste removal in the insect body in a new situation. If you got an incorrect answer, did you remember that insects lack kidneys and secrete their wastes into Malphigian tubules? Did you have trouble extending insect physiology to determine the correct answer? 84) B
Version 1
112
Clarify Question • What is the key concept addressed by the question? This question is asking about the movement of water in an experimental setting. • What type of thinking is required? You are being asked to take what you already know and use, or apply, it to the cause of the uptake of water by a model cell. Gather Content • What do you already know about the movement of water? What other information is related to the question? o Solvents such as water move from high solvent concentration to low concentration solvent concentration via osmosis. o Concentration differences between two regions, often across a membrane, are driven by solute concentrations. o As solute concentration increases, osmotic pressure increases, and water concentration decreases. o Solutions move from areas of low osmotic pressure to areas of high osmotic pressure via osmosis. Choose Answer • Given what you now know, what information is most likely to produce the correct answer? o The dialysis tubing is impermeable to sucrose, so those molecules cannot diffuse through the membrane. o If active transport was involved, the water would move in the opposite direction than it would via osmosis, that is, from low water concentration to high water concentration. This would shrink, rather than fill the dialysis tubing. o The surrounding water does not have high osmolarity, as no solute is Version 1
113
mentioned. o The osmotic pressure of the sucrose solution would cause the water to move into the dialysis tubing until that osmotic pressure reaches equilibrium with the surrounding solution. Reflect on Process •Did your problem-solving process lead you to the correct answer? If not, where did the process break down or lead you astray? How can you revise your approach to produce a more desirable result? o Answering this question correctly depended on your ability to use the movement of water in a new situation. If you got an incorrect answer, did you remember that osmosis is the movement of water from high water concentration to low water concentration or that solute lowers water concentration in a solution? Did you have trouble extending the movement of water to determine the correct answer? 85) E 86) E 87) D 88) C 89) E 90) A 91) B 92) D 93) C 94) E 95) B 96) C
Version 1
114
Clarify Question • What is the key concept addressed by the question? This question is asking about the response that is necessary to counteract a drop in glomerular blood pressure. • What type of thinking is required? You are being asked to take what you already know and use, or apply, it to a needed response when blood pressure drops in the glomerulus. Gather Content • What do you already know about glomeruli? What other information is related to the question? o Afferent renal arterioles supply blood to glomeruli. o Glomeruli are groups of capillaries that occur around tubules in nephrons. o Waste absorption occurs in the glomeruli. Choose Answer • Given what you now know, what information is most likely to produce the correct answer? o The action of antidiuretic hormone occurs after the glomerulus, so even though its inhibition could increase blood pressure, its effects would be muted. o Distal convoluted tubules also occur after the glomerulus, so action here would not be effective. o Secretion of angiotensin II would be the most immediate and effective way to counteract a drop in blood pressure in the glomerulus, as it would act to constrict the afferent renal arterioles and increase blood pressure. Reflect on Process • Did your problem-solving process lead you to the correct answer? Version 1
115
If not, where did the process break down or lead you astray? How can you revise your approach to produce a more desirable result? o Answering this question correctly depended on your ability to use glomeruli in a new situation. If you got an incorrect answer, did you remember that glomeruli receive blood from afferent arterioles, or that angiotensin II dilates arterioles which would bring more blood into the glomeruli? Did you have trouble extending the workings of the glomeruli to determine the correct answer? 97) C 98) B 99) C 100) A 101) B 102) E 103) C 104) E 105) B 106) C 107) E 108) D
Version 1
116
Clarify Question • What is the key concept addressed by the question? This question is asking where osmolarity is lowest in a nephron. • What type of thinking is required? You are being asked to take what you already know and use, or apply, it to the osmolarity of fluid in different regions of the nephron. Gather Content • What do you already know about osmolarity? What other information is related to the question? o Osmolarity is the solute concentration of a solution. o Osmolarity changes as fluid moves through the kidney. o Solutes are reabsorbed from fluid in the kidney in the loop of Henle. o Water is reabsorbed in the loop of Henle and in the collecting duct. Choose Answer • Given what you now know, what information is most likely to produce the correct answer? o Sections A, B, and C in the diagram show the nephron structure before and in the loop of Henle. o Since solutes are still present at higher concentrations in before and as fluid moves through the loop of Henle, osmolarity is relatively high here. o Section D is a region where osmolarity would be lowest, as solutes have been removed, but not all water has been reabsorbed yet. o Section E, the collecting duct, would have increased osmolarity due to the reabsorption of water in this region. Reflect on Process • Did your problem-solving process lead you to the correct answer? If not, where did the process break down or lead you astray? How can you revise your approach to produce a more desirable result? o Answering this question correctly depended on your ability to use the changes that occur in tubular fluid in a new situation. If you got an incorrect answer, did you remember that solutes are reabsorbed in the loop of Henle or that water is reabsorbed in the loop of Henle and also in the collecting duct? Did you have trouble extending the nature of the tubular fluid to determine the correct answer?
109) C 110) A 111) B Version 1
117
112) D
Version 1
118
Clarify Question • What is the key concept addressed by the question? • This question is asking about the effects of alcohol on urinary output. • What type of thinking is required? o You are being asked to take what you already know and use, or apply, it to deduce the effects of alcohol on urine volume. Gather Content • What do you already know about the production of urine? What other information is related to the question? o Antidiuretic hormone normally causes the reabsorption of water, which concentrates the urine and prevents dehydration. Choose Answer • Given what you now know, what information is most likely to produce the correct answer? o Increasing the number of aquaporins in the collecting ducts would increase water reabsorption and lessen the volume of urine. o Alcohol does not directly influence ion secretion, absorption of metabolites, or glomerular filtration. o Since antidiuretic hormone is inhibited by the use of alcohol, the consequences of ingestion of alcohol include lack of water reabsorption and excessive urination. Reflect on Process Did your problem-solving process lead you to the correct answer? If not, where did the process break down or lead you astray? How can you revise your approach to produce a more desirable result? o Answering this question correctly depended on your ability to use reabsorption of water during urine production in a new situation. If you Version 1
119
got an incorrect answer, did you remember that antidiuretic hormone cause reabsorption of water or that alcohol inhibits the action of antidiuretic hormone? Did you have trouble extending the reabsorption of water during urine production to determine the correct answer? 113) B
Version 1
120
Clarify Question • What is the key concept addressed by the question? This question is asking about the tubular structure that carries urine out of the kidneys. • What type of thinking is required? You are being asked to take what you already know and use, or apply, it to the change in position of a kidney and the resulting aftermath. Gather Content • What do you already know about urine flow from the kidney? What other information is related to the question? o When urine is produced in the kidney, it flows out through the renal pelvis. o The renal pelvis opens into the ureter, which carries urine to the bladder. o As the bladder empties, urine is carried through the urethra out of the body. Choose Answer • Given what you now know, what information is most likely to produce the correct answer? o Renal arteries and veins carry blood to and from the kidney, but do not carry urine. o Nephrons filter blood within the kidney, but do not leave the kidney. o The urethra is a passage that is distal to the bladder. It carries urine out of the body. o Impeded urinary flow from the kidney is most immediately caused by a blockage in the ureter. Reflect on Process
Version 1
121
• Did your problem-solving process lead you to the correct answer? If not, where did the process break down or lead you astray? How can you revise your approach to produce a more desirable result? o Answering this question correctly depended on your ability to use the anatomy of the urinary tract in a new situation. If you got an incorrect answer, did you remember that the ureter carries urine away from the kidney or that the renal artery and vein only move blood to and from the kidney? Did you have trouble extending the path of urine flow to determine the correct answer? 114) B 115) D 116) A 117) E 118) C 119) A 120) B
Version 1
122
Clarify Question • What is the key concept addressed by the question? This question is asking you to compare the action of the immune system to the action of chemotherapy. • What type of thinking is required? This question is asking you to identify key factors that qualify something as a function of the immune system or a function of chemotherapeutic action. Gather Content • What do you already know about the actions of the immune system? What other information is related to the question? o Interferons are induced by innate defenses. o Two classes of interferons are known. o Type I interferon is produced when viral infections occur. They signal the infection to neighboring cells. o Type II interferon is produced by T cells and natural killer cells. They protect against infection and cancer. o Type II interferon helps to initiate the cellular response, which includes the stimulation of macrophages. Choose Answer • Given what you now know, what information is most likely to produce the correct answer? o As interesting as this sounds, lymphocytes do not possess electromagnetic radiation that can be used against other cells. o Interferon is not inserted into the cell membrane of tumor cells by NK cells, nor does it bind with MHC proteins. o Interferon is secreted by T cells and NK cells in response to the presence of tumor cells. Version 1
123
Reflect on Process • Did your problem-solving process lead you to the correct answer? If not, where did the process break down or lead you astray? How can you revise your approach to produce a more desirable result? o Answering this question correctly depended not only on distinguishing between the actions of chemotherapeutic agents and the actions of the immune system, but on your ability to break down, or analyze, similar outcomes of the two. If you got an incorrect answer, did you remember that components of chemotherapy include interferon or that interferon is secreted by T cells and natural killer cells in the immune system? Did you have trouble breaking down the characteristics of chemotherapy and immune function to determine the correct answer? 121) A 122) C 123) D 124) A 125) B 126) C "Macrophage" means "big eater." 127) A 128) C 129) E 130) A 131) B 132) E 133) A 134) A 135) C 136) D Version 1
124
137) B Macrophages release interleukin-1, not immunoglobulin-1. Fever contributes to the body's defenses by stimulating phagocytosis, not B cell maturation.In response to fever, the liver and spleen store iron, which bacteria need to multiply. Fevers above 105° F are often fatal. 138) E
Version 1
125
Clarify Question • What is the key concept addressed by the question? This question is asking about the underlying cause of transplant rejection. • What type of thinking is required? You are being asked to take what you already know and use, or apply, it to understand the cause of transplant rejection. Gather Content • What do you already know about tissues and transplants? What other information is related to the question? o An organ in one person is made of the same type of tissue in another person. For instance, heart tissue includes cardiac muscle in all people. o In addition to the foundational tissues that do not differ between people, within a species, there are molecular markers that include species, individual, and tissue-level identity. These include MHC markers. o The immune system of one individual recognizes self-markers and does not attack tissue with those markers. o Anything that lack self-markers is attacked by the immune system. Choose Answer • Given what you now know, what information is most likely to produce the correct answer? o Although organs are made of the same general tissue types, individual MHC markers differ between individuals. o Transplant rejection is not caused by differing antibodies or pharmaceuticals. o If non-self-markers are encountered, the immune system attacks the foreign tissue, which causes transplant rejection. Version 1
126
Reflect on Process • Did your problem-solving process lead you to the correct answer? If not, where did the process break down or lead you astray? How can you revise your approach to produce a more desirable result? o Answering this question correctly depended on your ability to use immune function in a new situation. If you got an incorrect answer, did you remember that tissues have MHC markers on them or that if the immune system does not recognize the markers, that the tissue is attacked? Did you have trouble extending immune function to determine the correct answer? 139) A 140) E 141) D 142) A 143) A 144) D
Version 1
127
Clarify Question • What is the key concept addressed by the question? This question is asking about the fate of a Toll-like receptor gene that has mutated. • What type of thinking is required? This is an analyze question because you have to break TLRs and mutations that occur in them into their component pieces to understand how they function. Gather Content • What do you already know about TLRs? What other information is related to the question? o 11 TLRs have been identified in humans. o Human TLRs bind to specific antigen types that are components of pathogens. o Leucine-rich pockets on TLRs allow greater flexibility for binding with pathogens. o Activation of TLRs stimulates innate and adaptive immune responses. o Viruses can evade this response by mutating quickly. o The influenza virus is notorious for the rapid mutation rate of its HA and NA envelope proteins, which usually makes them poor targets for TLRs. Choose Answer • Given what you now know, what information is most likely to produce the correct answer? o A mutation within a human species for a TLR would not be spread to another species. o With amutation for a leucine-rich pocket, TLRs would bebetter able to recognize specific components of the influenza virus. Version 1
128
o This recognition would lead to a much faster immune response whenthe influenza virus is present. Reflect on Process • Did your problem-solving process lead you to the correct answer? If not, where did the process break down or lead you astray? How can you revise your approach to produce a more desirable result? o Answering this question correctly depended not only on distinguishing between normal and mutated TLRs, but on your ability to break down, or analyze, the effects of mutation. If you got an incorrect answer, did you remember that TLRs are not usually effective against influenza, but thata leucine-rich pocketwould increase their responsiveness? Did you have trouble breaking down the nature of TLRs to determine the correct answer? 145) A 146) C
Version 1
129
Clarify Question • What is the key concept addressed by the question? This question is asking you to identify the nature of inflamed structures in the armpit. • What type of thinking is required? This question is asking you to weigh and judge, or evaluate, the presence of lumps in the armpit. Gather Content • What do you already know about the composition of tissue in the region of the armpit? What other information is related to the question? o The armpit is one of the regions of the body in which lymph nodes are concentrated. o Lymph nodes become inflamed when infection is present nearby. o Lymph nodes are sites where lymph fluid is filtered. o When pathogens are filtered in lymph nodes, the immune response is perpetuated. Choose Answer • Given what you now know, what information is most likely to produce the correct answer? o The thymus is located in the chest, so this answer can be excluded immediately. o The spleen is located in the upper left abdominal cavity, under the ribcage, so this answer can also be excluded. o When pathogens are filtered in lymph nodes, B and T cells become activated and the immune response continues. Reflect on Process
Version 1
130
• Did your problem-solving process lead you to the correct answer? If not, where did the process break down or lead you astray? How can you revise your approach to produce a more desirable result? o Answering this question correctly depended on your ability to weight and judge, or evaluate, the presence of a bacterial infection and concurrent swellings in the armpit region. If you got an incorrect answer, did you remember that the armpit is one of the areas in which the lymph nodes are concentrated or the lymph nodes become inflamed when pathogens are filtered within? Did you have trouble weighing the merits of the immune response to determine the correct answer? 147) C
Version 1
131
Clarify Question • What is the key concept addressed by the question? This question is asking about the cause of the lack of tissue rejection in Tasmanian devils. • What type of thinking is required? This question is asking you to compare and contrast human tissue transplants with tissue transplants in Tasmanian devils, making it an evaluate-level question. Gather Content • What do you already know about tissue transplants? What other information is related to the question? o The immune system of ahuman recognizes self-markers such as MHCs and does not attack tissue with those markers. o Anything that lack self-markers is attacked by the immune system. o Within small groups of individuals, such as endangered species, genetic variability may be quite low. Choose Answer • Given what you now know, what information is most likely to produce the correct answer? o Other organisms dohave adaptive immunity and MHC markers, especially mammals such as marsupials. o T cells do in fact target tumors, so this answer can be disregarded. o If there is insufficient genetic variability, it is possible that tissue markers do not differ between individuals. This would result in lack of rejection of the transmitted tumor tissue. Reflect on Process • Did your problem-solving process lead you to the correct answer? Version 1
132
If not, where did the process break down or lead you astray? How can you revise your approach to produce a more desirable result? o Answering this question correctly depended on your ability to weigh and judge, or evaluate, tissue transplants in abundant versus endangered species. If you got an incorrect answer, did you remember that transplant rejection occurs because of a lack of similarity in tissue markers or that endangered species may have little genetic variability, including in the genetic variability that leads to different tissue markers? 148) D 149) D
Version 1
133
Clarify Question • What is the key concept addressed by the question? This question is asking about the consequences to immunoglobulins of alternative splicing of mRNA. • What type of thinking is required? This is an analyze question because you have to break alternative splicing of mRNA and the nature of immunoglobulins into their component pieces to understand how they function. Gather Content • What do you already know about mRNA splicing? What other information is related to the question? o Immunoglobulins, or antibodies, have variable regions that recognize specific antigens, as well as constant regions that are similar for different antibodies. o Many different combinations are possible for the variable regions. o The DNA sequence that codes for the heavy chains of immunoglobulins has approximately 50 V segments, 30 smaller D segments, and 6 smaller J segments. o Each segment codes for a different series of amino acids. o In the synthesis of a heavy chain of an immunoglobulin, a V, D, and J region are randomly selected and joined together. o In the synthesis of immunoglobulins, heavy-chain-encoding premRNA transcripts start at the rearranged VDJ and continue through exons encoding μ and δ constant regions. o Light chains are synthesized in a similar, way except they lack a D segment. Choose Answer • Given what you now know, what information is most likely to Version 1
134
produce the correct answer? o Alternative splicing of tRNA transcripts removes any extra J segments that remain 3′ to the rearranged VDJ, as well as either δ or μ sequences, resulting in transcripts that all encode the same variable region but either μ or δ constant region exons, respectively. o Translation results in a μ or δ heavy-chain polypeptide, which associates with a light-chain polypeptide in the rough endoplasmic reticulum. o Thus, the mature naive B cell expresses both IgM and IgD on its surface, both having the same antigen-binding specificity.. Reflect on Process • Did your problem-solving process lead you to the correct answer? If not, where did the process break down or lead you astray? How can you revise your approach to produce a more desirable result? o Answering this question correctly depended not only on distinguishing between the constant and variable regions of an antibody, but on your ability to break down, or analyze, alternative splicing. If you got an incorrect answer, did you remember that components of the variable region include many different possible combinations or that alternative splicing of mRNA can accommodate this variation? Did you have trouble breaking down the nature of antibodies to determine the correct answer? 150) C
Version 1
135
Clarify Question • What is the key concept addressed by the question? The question is asking about the consequences of the loss of Rag genes. • What type of thinking is required? You are being asked to take what you already know and use, or apply, it to the effects of the loss of VDJ recombination on immunoglobulin structure and function. Gather Content • What do you already know about the VDJ segments? What other information is related to the question? o Immunoglobulins, or antibodies, have variable regions that recognize specific antigens, as well as constant regions that are similar for different antibodies. o Many different combinations are possible for the variable regions. o The DNA sequence that codes for the heavy chains of immunoglobulins has approximately 50 V segments, 30 smaller D segments, and 6 smaller J segments. o Each segment codes for a different series of amino acids. o In the synthesis of a heavy chain of an immunoglobulin, a V, D, and J region are randomly selected and joined together. o VDJ recombination leads to thousands of different heavy chain region sequences. o Light chains are synthesized in a similar, way except they lack a D segment. Choose Answer • Given what you now know, what information is most likely to produce the correct answer? o T cells are not the products of antibodies, so they would be unaffected Version 1
136
by this deletion. o Antibody diversity would decrease, not increase without these genes, and resistance to infection would decrease as wll. o Since VDJ recombination leads to thousands of different heavy chain region sequences, without the recombination, the heavy chains would be quite similar. o This would result in immunodeficiency due to lack of antibody diversity. Reflect on Process • Did your problem-solving process lead you to the correct answer? If not, where did the process break down or lead you astray? How can you revise your approach to produce a more desirable result? o Answering this question correctly depended on your ability to use heavy chain formation in a new situation. If you got an incorrect answer, did you remember that heavy chains have three variable components or that lack of recombination would lead to fewer matches between antibodies and antigens? Did you have trouble extending heavy chain formation to determine the correct answer? 151) C
Version 1
137
Clarify Question • What is the key concept addressed by the question? This question is asking about the location of antibody production. • What type of thinking is required? You are being asked to take what you already know and use, or apply, it to the specifics of antibody production. Gather Content • What do you already know about antibodies? What other information is related to the question? o VDJ segments are components of antibodies. o Antibodies are proteins that are produced by B cells. o B cells are a subtype of lymphocytes. Choose Answer • Given what you now know, what information is most likely to produce the correct answer? o The thyroid and parathyroid do not play a role in the production of antibodies. o The blood serum house antibodies, but does not play a role in their production. o Cells that give rise to B cells are found in the bone marrow. o Lymphocytes then migrate to the thymus and other sites to mature. Reflect on Process • Did your problem-solving process lead you to the correct answer? If not, where did the process break down or lead you astray? How can you revise your approach to produce a more desirable result? o Answering this question correctly depended on your ability to use antibody production in a new situation. If you got an incorrect answer, Version 1
138
did you remember that VDJ segments are components of antibodies or that antibodies are produced by B cells, which are a type of lymphocytes? Did you have trouble extending antibody production to determine the correct answer? 152) A 153) D 154) B
Version 1
139
Clarify Question • What is the key concept addressed by the question? This question is asking about the effects of a viral infection if the virus has coat proteins that are similar to markers of the host. • What type of thinking is required? This question is asking you to weight and judge, or evaluate, the effects of a viral infection in which the virus has a coat protein that resembles those of glial cells. Gather Content • What do you already know about viral infections? What other information is related to the question? o Viruses are able to infect host cells that have recognizable markers on the cell membrane. o Once inside the host cell, the virus causes the host cell to make copies of the virus, which can spread throughout the host. o Antibodies are the body’s specific response to pathogens, including viruses. o An autoimmune disease occurs when the immune system does not recognize tissue as self, and correspondingly attacks that tissue. Choose Answer • Given what you now know, what information is most likely to produce the correct answer? o Since the coat protein of the virus is similar to that of a cell in the CNS, it stands to reason that an antibody that would defeat the virus could also attack cells of the CNS. o This attack could escalate to an autoimmune disease. o In fact, this type of "molecular mimicry" has been proposed as a mechanism accounting for neurological complications in AIDS patients Version 1
140
becausean HIV envelope protein has a 12 amino acid sequence in common with a protein on astrocytes. Reflect on Process • Did your problem-solving process lead you to the correct answer? If not, where did the process break down or lead you astray? How can you revise your approach to produce a more desirable result? o Answering this question correctly depended on your ability to weigh and judge, or evaluate, the immune response to a viral infection. If you got an incorrect answer, did you remember that antibodies are made in response to markers on pathogens or the criteria to compare a virus and a cell of the body as targets for antibodies includes similarity of markers on both? Did you have trouble weighing the merits of viral infection and immune response to determine the correct answer? 155) C
Version 1
141
Clarify Question • What is the key concept addressed by the question? This question is asking about different hybridoma lines and their use in detecting E.coli 0157:H7 • What type of thinking is required? This question is asking you to weight and judge, or evaluate, the best hybridoma line to identify a specific pathogen. Gather Content • What do you already know about hybridomas? What other information is related to the question? o A hybridoma is made from the fusion of a lymphocyte and a tumor cell. o Hybridomas are used to produce antibodies. o The best test would indicate the specific pathogen with no ambiguity. Choose Answer • Given what you now know, what information is most likely to produce the correct answer? o The only hybridoma line that identifies E coli 0:157:H7 specifically, without indicating any other bacterium, is hybridoma 3. Reflect on Process • Did your problem-solving process lead you to the correct answer? If not, where did the process break down or lead you astray? How can you revise your approach to produce a more desirable result? o Answering this question correctly depended on your ability to weight and judge, or evaluate, various hybridomas. If you got an incorrect answer, did you remember that hybrodoma 3 is the only hybridomas that specifically identifies E.coli 0157:H7 and nothing else? Did you have Version 1
142
trouble weighing the merits of the different hybrodomas to determine the correct answer? 156) C Most of the time, the peptides bound to MHC proteins are derived from self-proteins from the individual’s own cells. For this reason, it is important that T cells undergo selection in the thymus so that those that bind too strongly to peptides of self-proteins on self-MHC are eliminated. In this way, T cells normally are activated only outside the primary lymphoid organs in which they mature, when they encounter peptides of foreign proteins on self-MHC—for example, in the case of viral infection or cancer. 157) B Milkmaids who had caught a much milder form of “the pox” called cowpox (presumably from cows) rarely experienced smallpox. Jenner set out to test the idea that cowpox could confer protection against smallpox. He inoculated a healthy child with fluid from a cowpox vesicle and later deliberately infected him with fluid from a smallpox vesicle; as he had predicted, the child did not become ill. (Jenner’s experiment would be considered unethical today.) Subsequently, many people were protected from smallpox by immunization with fluid from cowpox vesicles, a much less risky proposition. We now know that smallpox and cowpox are caused by two different viruses that have similar surfaces. Jenner's patients who were injected with the cowpox virus mounted a defense that was also effective against a later infection of the smallpox virus. 158) C 159) B 160) B
Version 1
143
In the mutant mice, body temperature will fall rapidly due to positive feedback from heat loss when blood vessels are dilated, so cold temperatures might lead to hypothermia and must be avoided. 161) D 162) D 163) D 164) E 165) A Shivering is used by mammals to increase body heat when the hypothalamus detects a body temperature below the set point. Although the behavior involves muscles, the outcome is not movement but "wasted" energy in the form of heat. This is an example of nature reusinga body structure for an additional purpose. 166) B 167) D 168) C 169) C 170) A 171) B 172) A 173) B 174) E 175) D 176) A 177) D 178) D 179) C 180) C 181) A Version 1
144
Clarify Question • What is the key concept addressed by the question? o The question is asking about frostbite and severe damage to tissues. • What type of thinking is required? o You are being asked to take what you already know and use, or apply, it to the possible causes of damage to tissues resulting from extreme temperatures. Gather Content • What do you already know about the circulation system? What other information is related to the question? ● Frostbite occurs when skin is exposed to very low temperatures.Severe frostbite can result in losing fingers,toes, orother body parts. ● The cardiovascular system participates in temperature regulation. Contraction and dilation of blood vessels near the surface of the body, beneath the epidermis, helps to conserve or to dissipate heat as needed. ● Contraction of the smooth muscle layer of the arterioles results invasoconstriction,which greatly increases resistance and decreases flow. Shunting blood away from the skin is effective when an animal needs to minimize heat exchange, as might happen in cold weather. ● Relaxation of the smooth muscle layer results in vasodilation,decreasing resistance and increasing blood flow to an organ. This can increase the rate of heat exchange, which is beneficial for gaining or losing heat. Choose Answer • Given what you now know, what information is most likely to produce the correct answer? ● The answer that makes the most sense is that vasoconstriction has Version 1
145
occurred, which leads to a reduction in blood flow. ● Vasodilation wouldincreaseblood flow to the legs and feet, which would not be helpful severecold weather. ● If carbon dioxide levels increased, that would eventually lead to hyperventilation, which doesn’t relate to losing toes. ● Infectious bacteria causing a loss of toes in 36 hours as a result of being in the cold doesn’t make sense. Reflect on Process • Did your problem-solving process lead you to the correct answer? If not, where did the process break down or lead you astray? How can you revise your approach to produce a more desirable result? o Answering this question correctly depended on your ability to use information about the circulation system in a new situation. If you got an incorrect answer, did you remember that blood flow dissipates throughout the body and can be reduced or increased by vasocontraction or vasodilation? Did you realize that crystallization can occur which would rupture cells? 182) Section Break 182.1) D
Version 1
146
Clarify Question • What is the key concept addressed by the question? What is the cause of species specific reactivity of hormones? • What type of thinking is required? This is an analyze question because you have to understand differences between water soluble and lipid hormones and then seek logical explanations for why mouse hormones may not affect yeast cells. Gather Content • What do you already know about hormones? What other information is related to the question? o The question already tells you that you are dealing with a cell culture of yeast that you are applying a water-soluble mouse hormone to. You know that hormones can be either lipid-based or water-soluble. Lipid hormones like steroids can pass through the cell membrane while water-soluble ones cannot, they must first bind to a receptor on the outside of target cells before signal changes can be changed within the cells. Yeast receptors and mouse receptors may not be the same. Choose Answer • Given what you now know, what information is most likely to produce the correct answer? o Yeast do not have cell walls like plant cells or bacteria do, so that cannot be the reason. Water-soluble hormones are not lipids, as lipids are hydrophobic and water-soluble molecules are hydrophilic. Hormones do not need to be applied in large amounts before an effect is seen, but receptors for that specific hormone is needed to see effects and yeast likely do have the proper receptor for a Version 1
147
mammalian hormone. Reflect on Process • Did your problem-solving process lead you to the correct answer? If not, where did the process break down or lead you astray? How can you revise your approach to produce a more desirable result? o Answering this question correctly depended on realizing that the water-soluble hormone being applied cannot diffuse across the cell membrane and thus need a proper receptor, and figuring out that yeast may not have proper receptors for mouse hormones. Did you select the correct answer? If so, great! If not, which concept did you get confused on? Did you think that water-soluble hormones are lipids that can diffuse across the cell membrane? Did you think that yeast have cell walls like bacteria? 182.2) A
Version 1
148
Clarify Question • What is the key concept addressed by the question? How can you improve the experiment? • What type of thinking is required? This is an evaluate question because you have to distinguish between different experimental procedures that might improve the experiment. Gather Content • What do you already know about hormones? What other information is related to the question? o From the previous question you should have already determined that yeast may not have receptors specific to mouse hormones. Thus, you could solve this problem by applying the mouse hormone to a mouse cell culture instead of a yeast one. Hydrolyzing the hormone before application would destroy the three dimensional structure of the hormone so that it would not bind to its receptor properly. Applying the hormone at greater concentrations would not help if there is no receptor to bind any of the hormone. Choose Answer • Given what you now know, what information is most likely to produce the correct answer? o Applying the hormone to a mouse cell culture makes sense if the hormone does not work with yeast cells as mouse cells should have the correct receptors for mouse hormones. Hydrolyzing the hormone before application would destroy the three dimensional structure of the hormone so that it would not bind to its receptor properly. Applying the hormone at greater concentrations would not help if there is no receptor to bind any of the hormone. Version 1
149
Reflect on Process • Did your problem-solving process lead you to the correct answer? If not, where did the process break down or lead you astray? How can you revise your approach to produce a more desirable result? o Answering this question correctly depended on realizing that mouse cells should have receptors for mouse hormones. Did you get it right? If so, great! If not, did you think greater concentrations of hormone would help? No matter how much hormone is added, it will not affect cells that do not have receptors for it. Did you realize that hydrolyzing the hormone would destroy its structure leaving it unable to bind with receptors? 183) Section Break 183.1) A 184) Section Break 184.1) C
Version 1
150
Clarify Question • What is the key concept addressed by the question? What lab tests would you perform for Cushing’s syndrome? • What type of thinking is required? This is an analyze question because you have to examine each set of lab tests to see if it would be appropriate to find the source of the symptoms. Gather Content • What do you already know about cortisol? What other information is related to the question? o As the question states, Cushing’s syndrome causes weight gain, excessive sweating, and muscle weakness and can be caused by high levels of cortisol. Adrenocorticotropic hormone (ACTH), produced by the anterior pituitary, stimulates the adrenal cortex to produce corticosteroid hormones including cortisol. These hormones are important in regulating glucose homeostasis and the body’s response to stress. ACTH is stimulated by corticotropinreleasing hormone (CRH) produced by the hypothalamus. Choose Answer • Given what you now know, what information is most likely to produce the correct answer? o As birth control pills usually contain analogs of estrogen and progesterone to regulate ovulation, they should not produce the effects of Cushing’s syndrome. Similarly, LH and FSH are sex hormones that should not significantly influence Cushing’s syndrome. This syndrome is caused by excessive cortisol, so cortisol blood levels should be the first lab test to be performed. As cortisol levels are regulated by ACTH and ACTH is stimulated via Version 1
151
CRH, all three of these molecules would be appropriate to test to find the cause of the symptoms. Reflect on Process • Did your problem-solving process lead you to the correct answer? If not, where did the process break down or lead you astray? How can you revise your approach to produce a more desirable result? o Answering this question correctly depended upon evaluating each group of molecules as to its appropriateness in finding the cause of Cushing’s syndrome. If you got the answer correct, great! If not, which answer did you choose? Did you think that birth control pills contained corticosteroid hormones? They do include hormones (estrogen and progesterone), but not corticosteroids. Did you forget that CRH releases ACTH? Since both CRH and ACTH regulate cortisol, then testing all three of these molecules would be needed to find the source of the problem. 184.2) A
Version 1
152
Clarify Question • What is the key concept addressed by the question? What regions cause too much cortisol? • What type of thinking is required? This is an analyze question because you have to examine each set of regions for MRI tests to see if it would be appropriate to find the potential tumor. Gather Content • What do you already know about cortisol? What other information is related to the question? o As the question states, Cushing’s syndrome causes weight gain, excessive sweating, and muscle weakness and can be caused by high levels of cortisol. Adrenocorticotropic hormone (ACTH), produced by the anterior pituitary, stimulates the adrenal cortex to produce corticosteroid hormones including cortisol. These hormones are important in regulating glucose homeostasis and the body’s response to stress. ACTH is stimulated by corticotropinreleasing hormone (CRH) produced by the hypothalamus. Choose Answer • Given what you now know, what information is most likely to produce the correct answer? o Cushing’s syndrome is produced by excess cortisol, which is produced by the adrenal cortex. The adrenal cortex is stimulated by ACTH, which is produced in the anterior pituitary. This hormone is stimulated by CRH, which is produced by the hypothalamus. If cortisol levels are extremely high, it could mean that there is a tumor any of these areas: the hypothalamus, anterior pituitary, or adrenal cortex. Version 1
153
Reflect on Process • Did your problem-solving process lead you to the correct answer? If not, where did the process break down or lead you astray? How can you revise your approach to produce a more desirable result? o Answering this question correctly depended upon evaluating each region as to its appropriateness in finding the cause of Cushing’s syndrome- potentially caused by a tumor. If you got the answer correct, great! If not, which answer did you choose? Did you get confused as to which regions produce cortisol, ACTH, and CRH. Did you forget that CRH releases ACTH? Since both CRH and ACTH regulate cortisol, then testing all three of the regions that produce them would be needed to find the potential tumor. 185) Section Break 185.1) A
Version 1
154
Clarify Question • What is the key concept addressed by the question? This question is asking about the nature of the influenza virus. • What type of thinking is required? This question is asking you to weight and judge, or evaluate, the most efficient way to identify strains of the flu. Gather Content • What do you already know about monoclonal and polyclonal antibodies? What other information is related to the question? o An epitope is a portion of an antigen to which an antibody attaches. o Monoclonal antibodies respond to one specific epitope. o Polyclonal antibodies can respond to multiple types of epitopes. o The influenza virus is subject to high mutation rates and high rates of genetic recombination. Choose Answer • Given what you now know, what information is most likely to produce the correct answer? o Since influenza virus is notorious for rapid mutation of envelope proteins, the best chance to detect a wide variety of possible viruses would be to develop polyclonal antibodies against a mix of subtypes—thus responding to multiple antigens. Reflect on Process • Did your problem-solving process lead you to the correct answer? If not, where did the process break down or lead you astray? How can you revise your approach to produce a more desirable result? Version 1
155
o Answering this question correctly depended on your ability to weight and judge, or evaluate, monoclonal and polyclonal antibodies. If you got an incorrect answer, did you remember that monoclonal antibodies differ from polyclonal antibodies in that monoclonal antibodies recognize only one specific epitope or that the influenza virus is constantly changing? Did you have trouble weighing the merits of monoclonal versus polyclonal antibodies to determine the correct answer? 185.2) B
Version 1
156
Clarify Question • What is the key concept addressed by the question? This question is asking about the type of antibody that would be most likely to indicate one specific strain of influenza. • What type of thinking is required? This question is asking you to weight and judge, or evaluate, the use of monoclonal versus polyclonal antibodies to detect influenza. Gather Content • What do you already know about monoclonal and polyclonal antibodies? What other information is related to the question? o An epitope is a portion of an antigen to which an antibody attaches. o Monoclonal antibodies respond to one specific epitope. o Polyclonal antibodies can respond to multiple types of epitopes. o The influenza virus is subject to high mutation rates and high rates of genetic recombination. Choose Answer • Given what you now know, what information is most likely to produce the correct answer? o The monoclonal antibody is the best choice since it should be highly specific, and would effectively identify one specific strain of influenza. Reflect on Process • Did your problem-solving process lead you to the correct answer? If not, where did the process break down or lead you astray? How can you revise your approach to produce a more desirable result? Version 1
157
o Answering this question correctly depended on your ability to weight and judge, or evaluate, monoclonal and polyclonal antibodies. If you got an incorrect answer, did you remember that monoclonal antibodies differ from polyclonal antibodies in that monoclonal antibodies recognize only one specific epitope or that the question is asking for the identification of one specific type of the influenza virus? Did you have trouble weighing the merits of monoclonal versus polyclonal antibodies to determine the correct answer? 186) Section Break 186.1) C
Version 1
158
Clarify Question • What is the key concept addressed by the question? This question is asking you to differentiate between the influenza and varicella zoster vaccinations. • What type of thinking is required? You are being asked to take what you already know and use, or apply, it to determine the reason for having different vaccination schedules for different viruses. Gather Content • What do you already know about varicella zoster and influenza? What other information is related to the question? o Vaccinations are designed to stimulate antibodies against specific antigens. o If antigens change through mutation, antibodies that were produced by a previous vaccination may not recognize the newly mutated virus. o The mutation rate of varicella zoster is relatively low. o The mutation rate of influenza is high. Choose Answer • Given what you now know, what information is most likely to produce the correct answer? o Difference is transmission of a virus would not influence the internal efficacy of a vaccine. o Both viruses are attacked by antibodies that are the result of adaptive immunity. o Influenza has a much higher mutation rate, and consequently new vaccines must be developed every year. Reflect on Process Version 1
159
• Did your problem-solving process lead you to the correct answer? If not, where did the process break down or lead you astray? How can you revise your approach to produce a more desirable result? • Apply level: o Answering this question correctly depended on your ability to use varicella zoster and influenza viruses in a new situation. If you got an incorrect answer, did you remember that influenza mutates reapidly or that varicella zoster has a much lower mutation rate? Did you have trouble extending the nature of these viruses to determine the correct answer? 186.2) D
Version 1
160
Clarify Question • What is the key concept addressed by the question? This question is asking about the consequences of a lower mutation rate of a virus in terms of vaccination schedule. • What type of thinking is required? You are being asked to take what you already know and use, or apply, it to understand the effect of a low mutation rate of a virus. Gather Content • What do you already know about viral mutation rates? What other information is related to the question? o Chickenpox is caused by the virus varicella zoster. o Varicella zoster is a virus that has a low mutation rate compared to other viruses. o A vaccine is the intentional injection of epitopes of a pathogen. This will not cause disease, but will stimulate antibody production in the recipient. o The efficacy of antibodies that are stimulated by the receipt of a vaccine relies on the recognition by the antibodies of epitopes on the antigen of the virus. o If a virus does not mutate quickly, its epitopes remain unchanged and can be recognized by the antibodies. Choose Answer • Given what you now know, what information is most likely to produce the correct answer? o While chickenpox can become latent for decades and reappear as shingles later, this is not a consequence of the low mutation rate. o Differences in symptoms between influenza and chickenpox are also not a consequence of mutation rate. Version 1
161
o Because of influenza's high mutation rate, immunity against last year's version of the virus may not protect against this year's virus. o Chickenpox immunity is much longer lasting because the virus does not change significantly and can still be recognized by the immune system of the host. Reflect on Process • Did your problem-solving process lead you to the correct answer? If not, where did the process break down or lead you astray? How can you revise your approach to produce a more desirable result? o Answering this question correctly depended on your ability to use the mutation orate of viruses in a new situation. If you got an incorrect answer, did you remember that mutations can change viruses so they are no longer recognizable by the immune system of the host? Did you have trouble extending the mutation rate of viruses to determine the correct answer? 187) Section Break 187.1) A 187.2) E 187.3) B 187.4) C 188) Section Break 188.1) C 188.2) C
Version 1
162
CHAPTER 36 CHECK ALL THE APPLY. Choose all options that best completes the statement or answers the question. 1) You are having lunch with a friend and notice that she throws out her pack birth control pills even though there is a whole week of pills left. When you ask her about this, she explains that oral contraceptives usually have three weeks of active pills (containing hormones) and one week of sugar pills (containing no hormones). She skips the sugar pill week and instead keeps taking active pills from a new pack. Why do you think she does this? Check all that apply. A) To suppress the withdrawal bleeding that occurs during the sugar pill week B) To keep her risk of getting pregnant as low as possible C) To keep her progesterone and estrogen levels steady D) To allow her progesterone and estrogen levels to rise gradually
2)
What are attributes of oral contraceptives? (Check all that apply.) A) They contain aprogesterone and sometimes an estrogen analogue. B) The progesteroneis in the 21-day pills, and the estrogen is in the 7-day pills. C) They help protectagainst ovarian and endometrial cancer. D) They areappropriate for use by smokers over age 35. E) They have a low failure rate of only 1% to 5% a year. F) They act by inhibiting LH and FSHsecretion, inhibiting ovulation.
3)
Which are structures formed from the neural crest? (Check all that apply.) A) A gill chamber of primitive chordates B) Blastopores C) Sensory neurons of the dorsal root ganglia D) Schwann cells E) Olfactory and hearing organs
4)
Which of the following are fetal membranes? (Check all that apply.)
Version 1
1
A) Chorion B) Endometrium C) Allantois D) Amnion E) Yolk sac
5) Which developmental events are accurately placed in the following timeline? (Check all that apply.) A) After 6 days—implantation B) After 1 week—gastrulation C) After 2 weeks—neurulation D) After 3 months—organogenesis E) 266 days—birth
6) Which tissues are correctly matched to their progenitor germ layer? (Check all that apply.) A) Ectoderm—lining of the digestive tract B) Ectoderm—epidermis C) Endoderm—lining of the respiratory tract D) Endoderm—muscles E) Mesoderm—blood vessels F) Mesoderm—gonads
7)
Which of the following result from sperm penetration? (Check all that apply.)
Version 1
2
A) Eggs complete meiosis B) Haploid nucleus of egg undergoes final mitotic division C) Movement of egg cytoplasm D) Outer pigmented cap of egg cytoplasm rotates toward point of entry E) Sharp increase in protein synthesis
8)
Which of the following are derivedfrom mesoderm? A) Blood B) Epidermis of skin C) Heart D) Kidney E) Lining of the digestive tract F) Lining of the respiratory tract G) Muscles H) Nervous system I) Skeleton J) Somites
MULTIPLE CHOICE - Choose the one alternative that best completes the statement or answers the question. 9) Imagine that you are an ecologist and are studying an endangered species of salamander. Over the course of several years, you tag and (gently) measure hundreds of salamander larvae, juveniles, and adults. You notice that the ratio of tail length to body length is remarkably similar between all the stages. The same is true of the head length to body length ratio. What can you say about postnatal growth in this species of salamander?
Version 1
3
A) Postnatal growth is allometric because larvae and juveniles have different proportions than adults. B) Postnatal growth is allometric because larvae and juveniles have very similar proportions to adults. C) Postnatal growth is isometric because larvae and juveniles have different proportions than adults. D) Postnatal growth is isometric because larvae and juveniles have very similar proportions to adults.
10) Working in an experimental fertility clinic, a researcher removes one cell from an 8-cell stage chimpanzee embryo and implantsthe cell and the 7-cell embryo into two different chimpanzee surrogates. What is the likely result? A) Both the cell and the 7-cell embryo will die. B) The cell and the 7-cell embryo will develop normally into identical twins. C) The cell will develop into a placenta, but will not form an embryo; the 7-cell embryo will develop into a normal chimp. D) The cell will not develop. The 7-cell embryo will develop but will likely have abnormalities.
11) Imagine you are a researcher working on the treatment ofthyroid disorders. Specifically, you are using iPS cells (induced pluripotent stem cells)to grow newthyroid glandcells that can be transferred to patients with thyroid disorders. What is the best termto describe your research? A) Reproductive cloning B) Epigentic therapy C) Therapeutic cloning D) Nuclear reprogramming
Version 1
4
12) There are three main types of uteri in mammals: 1. Duplex—two entirely separate uteri and cervices with one or more vaginas 2.Bicornuate—oneuterine body and two uterine horns; the size of the uterine body and the degree of separation betweenthe horns varies greatly among species 3.Simplex—the entire uterus is fused into a single chamber What type of uterus would you expect to find in a koala bear? A) Bicornuate B) Duplex C) Simplex D) None of these; koalas are such unique mammals that they have an entirely different type of uterus.
13) A cell that was capable of forming any type found in the organisms body but was not capable for forming the amniotic membrane would be __________. A) totipotent B) pluripotent C) multipotent D) unipotent
14) Which researcher would be most accurately described as engaging in reproductive cloning? A) Roger works in a lab that is trying to induce the formation of pancreatic cells. B) Conseula's research is focued in generating new bone tissue from iPS cells. C) Peter is trying to clone particularly large bull. D) Shellywants to induce cells to form new neural tissue.
15) Terrestrial vertebrates and some fish evolved a form of fertilization in which the male gametes are introduced into the female reproductive tract. What is this called?
Version 1
5
A) Synchronous fertilization B) External fertilization C) Internal fertilization D) Asexual fertilization
16)
Internally fertilized organisms can either be born alive, or emerge from what structures? A) Follicles B) Ducts C) Capsules D) Eggs
17) Some internally fertilized eggs develop and hatch outside the mother's body. What is this developmental strategy called? A) Oviparity B) Placental development C) Marsupial development D) Vivipary
18)
What was themost important adaptation of the reptiles and birds to life on land? A) The shelled egg B) The amniotic egg C) The external egg D) The impermeable egg
19)
What kind of oocyte is released from the ovary at ovulation?
Version 1
6
A) Primary B) Germinal C) Secondary D) Ovum
20)
In males, sterilization involves a vasectomy, which removes a portion of what structure? A) Vas deferens B) Epididymis C) Seminiferous tubules D) Sertoli cells
21)
What term describes avoidance of reproduction without giving up sexual intercourse? A) Abstinence B) The rhythm method C) Asexual reproduction D) Contraception
22)
What stimulates ovulation? A) Estradiol B) FSH C) LH D) Testosterone E) Progesterone
23)
What stimulates spermatogenesis?
Version 1
7
A) Estradiol B) FSH C) LH D) Testosterone E) Progesterone
24)
Whatstimulates secretion of testosterone by Leydig cells? A) Estradiol B) FSH C) LH D) Testosterone E) Progesterone
25)
What stimulates growth of ovarian follicles? A) Estradiol B) FSH C) LH D) Progesterone E) Testosterone
26)
What completes preparation of uterus for pregnancy? A) FSH B) LH C) Testosterone D) Progesterone E) Prolactin
27)
What prompts monthly preparation of uterus for pregnancy?
Version 1
8
A) Estradiol B) Testosterone C) Progesterone D) Prolactin E) Oxytocin
28) An adult male produces sperm continuously by meiotic division of cells lining what structure? A) Malpighian tubules B) Fallopiantubules C) Seminiferous tubules D) Epididymis tubules E) Vas deferens tubules
29)
What process is absent in mammals with an estrous cycle? A) Oogenesis B) Homeostasis C) Ovulation D) Menstruation E) Placenta formation
30)
What hormone is tested for in pregnancy tests? A) Estrogen hormone B) Luteinizing hormone C) Follicle stimulation hormone D) Progesterone hormone E) hCG hormone
Version 1
9
31) What caused intense selective pressure for terrestrial vertebrates to evolve internal fertilization? A) The eggs are toosmall for the males to find and fertilize. B) The gametes couldnot be released next to each other, as they would soon dry up. C) There is nomechanism for eggs and sperm to find one another. D) Terrestrialorganisms only produce small quantities of gametes. E) Gametes andzygotes are vulnerable to predators.
32) What is the term for organisms in which internally-fertilized eggs are laid to develop and hatch outside the mother's body? A) Placental B) Viviparous C) Oviparous D) Marsupial E) Ovoviviparous
33) The scrotum is a sac-like structure outside the abdomen in which the testes are suspended. Why is this structureessential? A) Sperm must becloser to the penis to be ejaculated. B) There is not enoughspace in the abdomen to grow properly. C) Lower temperatureis required for normal development of sperm. D) There is too muchhormonal influence if they remain in the abdomen.
34)
The sperm head encloses a compact nucleus and is capped by a vesicle called what?
Version 1
10
A) Sertoli cell B) flagellum C) Acrosome D) Vas deferens E) Epididymis
35) The uterine lining proliferates during the female cycle and then sloughs off in what process? A) Implantation B) Fertilization C) Menstruation D) Ovulation E) Placentation
36)
Where does normal fertilization of an egg by a sperm in a human female occur? A) At the ovary upon ovulation B) In the uterus after the egg has implanted C) In the uterus before the egg has implanted D) Near the cervix before the egg leaves the female E) In the Fallopian tube
37)
Select the correct statement regarding the human female reproductive system. A) The ovarianfimbria is the functional unit of the ovary. B) The granulosacells secrete testosterone, which triggers menarche. C) The Fallopian tubes transport ova from the ovary to the uterus. D) The cervixcontains corpora cavernosa tissue, which is a type of erectile tissue. E) The uterus is lined with granulosa cells.
Version 1
11
38)
How do birth control pills prevent fertilization? A) They alter theuterine environment so that sperm cannot survive. B) They preventfollicle development and ovulation. C) The provide hCG,indicating that fertilization has already occurred. D) They inhibitmenstruation. E) They preventimplantation.
39)
What is the most commonly employed form of birth control in the United States? A) Abstinence B) Prevention of ovulation by hormonal inhibition C) Sperm blockage with condoms D) Sterilization E) Prevention of embryo implantation with IUD
40)
What is an important benefit of using condoms? A) They preventprostate cancer. B) They have a failurerate of only 1%. C) They can helpprevent osteoporosis. D) They help protectagainst STDs.
41) Vertebrates that practice internal fertilization employ three strategies for embryonic and fetal development. In onestrategy, the female retains the eggs within her body and the young are fully developed when they hatch inside of her. What is this called? A) Oviparity B) Ovoviviparity C) Viviparity D) Protogyny E) Protandry
Version 1
12
42)
Even if utilized properly, what is the least effective means of birth control? A) Oral contraceptive B) Condom C) Foam D) IUD E) Implant
43) At a party, one of your friends has just revealed that he and his wife are trying to conceive. What do you advise him to avoid at the party? A) The loud music B) The raw vegetable appetizers C) The hot tub D) The melted cheese fondue
44) What would be the result for a patient if all hisspermatogonia went through meiosis and proceeded to develop into spermatids? A) He would producetwice as many sperm and would be protected against infertility from othercauses. B) He would producetwice as many sperm, which would crowd them during later development and leadto morphological defects. C) He would beentirely normal, since this is what usually happens. D) After one round of sperm production, he would no longer be able to produce sperm and would be infertile.
45) You are doing veterinary research on dog hormones. In your set of blood samples, you have one male dog with abnormally high levels of follicle-stimulating hormone and luteinizing hormone. What do you suspect may be the explanation for this outlier?
Version 1
13
A) The dog had beenneutered. B) The dog was afemale, not a male. C) The dog had itspituitary gland removed. D) The dog had a hypothalamic problem causing low levels of gonadotropin-releasing hormone (GnRH).
46) What could be the result if somehow two follicles made it to the late tertiary stage and burst from the ovary? A) Nothing unusual—this describes typical ovulation B) Excessively high estrogen levels, causing menstruation C) Permanent infertility D) Twins
47) A 38-year old woman is going toovulate in about a week. For how long has the primary oocyte in her Graafian follicle been arrested at prophase I of meiosis? A) About a day B) 1 week C) 3 weeks D) 38 years
48) A patient at a fertility clinic has been diagnosed with “luteal phase defect” (LPD). Her progesterone levels are low and her cycles are short. What step of reproduction are these symptoms disrupting? A) Follicle development B) Ovulation C) Fertilization of the egg D) Implantation
Version 1
14
49) A Women's Studies professor wants her students to learn more about their bodies, so she asks the female students to measure their basal body temperature daily. A small rise in temperature indicates that ovulation is occurring. However, some students do not record a change in temperature—which ones? A) Students withIUDs. B) Students usingoral contraceptives, hormonal implants, or injections. C) Students who havenever had intercourse. D) Students using anyform of birth control.
50)
What cause of infertility happens to nearly every woman at some point? A) Idiopathic infertility B) Pelvic inflammatory disease (PID) C) Endometriosis D) Luteal phase defect E) Aging
51)
What event initiates development? A) Fertilization B) Cleavage C) Gastrulation D) Organogenesis
52)
What factor has the strongest influence on the cleavage pattern of an embryo? A) Viscosity of albumin B) Amount of yolk C) Maturity of cytoplasm D) Oxygen in air
Version 1
15
53) In mammals, embryonic cells form an inner cell mass that will become the body of the embryo and a layer of surrounding cells that will become the chorionic membrane and placenta. What are these surrounding cells called? A) Blastocoel B) Yolk C) Blastodisc D) Trophoblast
54)
What structure is formed from both fetal and maternal tissue? A) Chorion B) Placenta C) Yolk sac D) Amnion
55) A couple conceives at the beginning of January. During which month will their child be undergoing organogenesis? A) January B) February C) March D) April
56) A couple conceives a child on January 1st. Which day falls within the period of somitogenesis? A) January3 rd B) January7 th C) January17 th D) January31 st
Version 1
16
57) New mothers may benefit from the aid of lactation consultants when learning to breastfeed. These consultants give advice ontechnique and help the mother to feel relaxed and confident. What hormone is associated with calm, warm emotions and is also critical for the milk let-down reflex? A) Prolactin B) Estradiol C) Oxytocin D) Mammotropin
58)
What would be the result of a mutation that reduces the amount of acrosomal enzymes? A) Male infertility due to inability of the sperm to tunnel through the blastula B) Male infertility due to inability of the sperm to tunnel through the zona pellucida C) Female infertility due to blockage of sperm from tunneling through the allantois D) Female infertility due to blockage of sperm from tunneling through the archenteron
59) Certain groups of cells move inward from the surface of the blastula in a carefully orchestrated migration called what? A) Blastulation B) Archenteron formation C) Morula formation D) Gastrulation E) Cleavage
60) What developmental event was key tothe evolution of the chordates by forming the notochord and dorsal nerve cord?
Version 1
17
A) Morphogenesis B) Organogenesis C) Gastrulation D) Neural crest formation E) Neurulation
61)
What factors stimulate uterine contractions and birth? A) FSH andprostaglandins B) LH andprostaglandins C) Oxytocin and prostaglandins D) Prolactin and prostaglandins E) hCG andprostaglandins
62) size?
What is the period of rapid cell division in which the embryo does not increase in overall
A) Syngamy B) Cleavage C) Neurulation D) Meiosis E) Crest formation
63) What is the term for segmented blocks of tissue that form on either side of the notochord and are added sequentially? A) Blastopores B) Gastropores C) Somites D) Amniotic cells E) Sensory neurons
Version 1
18
64) The developing embryo of reptiles, birds, and mammals is encased in the fluid-filled __________ membrane. A) amniotic B) embryonic C) chorionic D) yolk sac E) allantois
65) The human placenta is formed by the interaction of uterine tissue and what extraembryonic tissue? A) Amnion B) Embrion C) Chorion D) Yolk sac E) Allantois
66)
What stimulates milk production in mammary alveoli? A) Oxytocin B) Prolactin C) Chorionic gonadotropin D) Progesterone E) Estrogen
67)
How is brain growth in humans different from other mammals?
Version 1
19
A) In humans, brain growth is complete by the third trimester. B) In humans, brain growth stops at birth. C) Human growth is notallometric. D) In humans, brain growth continues after birth. E) Only humans developa cerebrum.
68) The head of a sperm is capped with an organelle that contains glycoprotein-digesting enzymes. What is this organelle called? A) Zona pellucida B) Acrosome C) Granulosa D) Blastula E) Neural cap
69)
Select the correct sequence of vertebrate development beginning with fertilization.
A) fertilization → cleavage → gastrulation → neurulation → organogenesis → neural crest formation B) fertilization → gastrulation → cleavage → neurulation → neural crest formation → organogenesis C) fertilization → neurulation → gastrulation → organogenesis → neural crest formation → cleavage D) fertilization → gastrulation → neurulation → neural crest formation → cleavage → organogenesis E) fertilization → cleavage → gastrulation → neurulation → neural crest formation → organogenesis
70)
What is the first milk that is produced after birth called?
Version 1
20
A) Prolactase B) Colostrum C) hCG D) Chorionic lactase E) Allantoicase
71)
Which of the following activates the egg? A) Entrance of sperm into the zona pellucida B) Contact of sperm with the outside of the zona pellucida C) Contact of sperm with the vitelline envelope D) Fusion of sperm and egg membranes E) Fusion of sperm and egg nuclei
72)
What actually begins the uterine contractions which leads to giving birth? A) Oxytocin B) Prolactin C) Estrogen D) Prostaglandins E) The movement of the fetus
73) A research group claims that they have discovered microscopic fossilized embryos, including some at the blastula stage, but other researchers are skeptical. Finally, one student develops a technique for splitting the embryos in half to reveal the substructure. Cutting open the first blastocyst embryo, the student is very excited. “It is a blastula!" she shouts,"Look, you can see the __________ inside!” A) granulosacells B) hollowblastocoel C) archenteron D) neural tube
Version 1
21
74) What would result if an error inside gastrulation caused too few cells inside the archenteron? A) Too little endoderm, resulting in missing gut and organs B) Too little ectoderm, resulting in missing nervous system or skin C) Problems in muscle development D) Too little protoderm, resulting in missing organs
75) What would result if a defect in gastrulation caused too little mesoderm inside the embryo? A) Too little endoderm, resulting in missing gut and organs B) Too little ectoderm, resulting in missing nervous system or skin C) Problems in muscle development D) Too little protoderm, resulting in missing organs
76) What would result if too many cells left the surface of the embryo to move inside during gastruation? A) Too little endoderm, resulting in missing gut and organs B) Too little ectoderm, resulting in missing nervous system or skin C) Problems in muscle development D) Too little protoderm, resulting in missing organs
77)
What human organ is formed from two genetically different tissues? A) Neural crest B) Neural tube C) Placenta D) Uterus
Version 1
22
SECTION BREAK. Answer all the part questions.
78)
78.1)
According to the figure, what is occurring during day 2?
A) Ovulation B) Release of corpus luteum C) Menstruation D) Luteal regression E) Copulation
Version 1
23
78.2)
According to the figure, what is a likely day of implantation?
A) Day 3 B) Day 27 C) Day 13 D) Day 21 E) Day 7
78.3)
According to the figure, what hormone doesn't peak during ovulation?
A) Progesterone B) FSH C) LH D) Estrogen E) All female reproductive hormones peak at ovulation
Version 1
24
Answer Key Test name: Chap 36_3e 1) [A, C]
Version 1
25
Clarify Question What is the key concept addressed by the question? ● The question asks about the result of continuously taking active birth control pills. What type of thinking is required? ● This is an analyze question because you have to identify the roles of various hormones in the menstrual cycle and understand how oral contraceptives act to mimic certain phases of the cycle. Gather Content What do you already know about oral contraceptives? What other information is related to the question? ● The menstrual cycle has two phases: follicular and luteal. ● During the luteal phase, high levels of progesterone and estradiol inhibit the secretion of FSH and LH by negative feedback, thus preventing follicle development and ovulation. ● Oral contraceptives contain analogs of progesterone and estrogen at levels designed to mimic this stage of the luteal phase, blocking ovulation. ● Blank (or sugar) pills cause a sudden drop in progesterone and estrogen analogs, which leads to withdrawal bleeding (similar to but distinct from true menstruation). Choose Answer Given what you now know, what information is most likely to produce the correct answer? ● Skipping the sugar pills and continually taking active pills is called continuous dosing. Itis a completely safe way to suppress ovulation and withdrawal bleeding. ● Without the drop in progesterone and estrogen that occurs during the sugar pill week, your friend can suppress withdrawal bleeding. ● The practice also allows your friend to keep her estrogen and Version 1
26
progesterone levels steady; the answer about allowing these hormones to gradually rise can be discounted. ● Assuming correct use of the pills, skipping the sugar pills neither raises nor lowers a woman’s risk of getting pregnant. Reflect on Process Did your problem-solving process lead you to the correct answer? If not, where did the process break down or lead you astray? How can you revise your approach to produce a more desirable result? ● Answering this question correctly depended on your ability to analyze how levels of progesterone and estrogen/estradiol affect ovulation and the shedding of the endometrium. If you got an incorrect answer, did you remember that oral contraceptives block ovulation by mimicking hormone levels at the start of the luteal phase? Or that a sudden drop in hormones is what triggers menstruation or withdrawal bleeding? Did you realize that continually taking active pills would keep hormone levels steady? 2) [A, C, E, F] 3) [A, C, D, E] 4) [A, C, D, E] 5) [A, B, C, E] 6) [B, C, E, F] 7) [A, C, D, E] 8) [A, C, D, G, I, J] 9) D
Version 1
27
Clarify Question What is the key concept addressed by the question? ● The question asks about postnatal growth in a hypothetical species of salamader What type of thinking is required? ● This is an analyze question because it is asking you to identify key factors that qualify a growth pattern as allometric or isometric. Gather Content What do you already know about postnatal growth patterns? What other information is related to the question? ● The body proportions of infants are different from those of adults. ● Ex: The head is disproportionately large in newborns, but after birth it grows more slowly than the rest of the body. ● Such a pattern of growth, in which different components grow at different rates, is referred to as allometric growth. Choose Answer Given what you now know, what information is most likely to produce the correct answer? ● The question describes a salamander species in which the proportions of the babies, juveniles, and adults are the same. ● A species that has very similar proportions at all ages does not exhibit allometric growth, so those two answers are incorrect. ● Isometric growth is exhibited when all body parts grow at approximately the same rate. ● A species with very similar infant and adult proportions exhibits isometric growth. Reflect on Process Did your problem-solving process lead you to the correct answer? If not, where did the process break down or lead you astray? How can you revise your approach to produce a more desirable result? Version 1
28
● Answering this question correctly depended on your ability to analyze postnatal growth patterns. If you got an incorrect answer, did you remember that in allometric growth infants and adults have different proportions? Did you deduce that in isometric growth all body parts grow at approximately the same rate? Did you realize that an isometric growth pattern would produce infants and adults with very similar proportions? 10) B
Version 1
29
Clarify Question • What is the key concept addressed by the question? This question is asking about the effects of the removal of a blastomere from an early-stage mammalian embryo. • What type of thinking is required? You are being asked to take what you already know and use, or apply, it to understand the fate of a mammalian blastomere. Gather Content • What do you already know about early development? What other information is related to the question? o Early cells that form from the divisions of a fertilized egg are called blastomeres. o In mammals, regulative development is present. o With regulative development, blastomeres are not predetermined to be specific tissue. After removal of a blastomere from an 8-cell embryo, both the blastomere and the 7-cell embryo could form all of the necessary germ layers and resulting structures. Choose Answer • Given what you now know, what information is most likely to produce the correct answer? o In a mammal, early-stage embryonic cells are not yet committed to a developmental pathway (such as becoming a placental cell). o Since mammals exhibit regulative development, the removed Version 1
30
blastomere and the 7-cell embryocould survive and develop normally in surrogate mothers. o Early-stage mammalian embryosthat are split in half develop into identical twins.
Reflect on Process • Did your problem-solving process lead you to the correct answer? If not, where did the process break down or lead you astray? How can you revise your approach to produce a more desirable result? o Answering this question correctly depended on your ability to use early development in a new situation. If you got an incorrect answer, did you remember that mammal development is regulative or that regulative development means that early cells are not predetermined to be a specific type of tissue? 11) C
Version 1
31
Clarify Question What is the key concept addressed by the question? ● The question describes a hypothetical line of research and asks you to pick the best term for it. What type of thinking is required? ● Evaluate: This question is asking you to weight and judge, or evaluate, the best term for a hypothetical line of research. Gather Content What do you already know about stem cell research? What other information is related to the question? ● Induced pluripotent stem cells (iPS cells) are made from somatic cells in culture that have been reprogrammed by the introduction of specific transcription factors. ● Nuclear reprogramming involves using a nucleus from a differentiated cell and reversing epigenetic changes to allow the nucleus to direct development. ● Reproductive cloning refers to using somatic cell nuclear transfer (SCNT) to create an animal that is genetically identical to another animal. ● Therapeutic cloning aims to create new cells or tissue types. It can address the problem of immune rejection that must be solved for stem cells to be used to repair tissues damaged by injury or disease. Choose Answer Given what you now know, what information is most likely to produce the correct answer? ● The hypothetical research attempts to create new thyroid cells in order to treat patients with thyroid disorders. This is a form of therapeutic cloning. ● The research uses iPS cells, which are created from differentiated cells that have had specific transcription factors introduced into them. Version 1
32
Therefore, nuclear reprogramming is part of the research, but is not the best term for it overall. ● Although the research involves reversing epigenetic changes, epigenetic therapy is not a term used to describe research, so this answer can be discounted. ● Reproductive cloning attempts to create a fully functional clone of an animal, so this answer is incorrect. Reflect on Process Did your problem-solving process lead you to the correct answer? If not, where did the process break down or lead you astray? How can you revise your approach to produce a more desirable result? ● Answering this question correctly depended on your ability to weight and judge, or evaluate, different types of stem cell research techniques. If you got an incorrect answer, did you remember that iPS cells are created from somatic cells that have had specific transcription factors introduced into them? Or that reproductive cloning aims to create a fully functional animal? Did you realize that the goal of therapeutic cloning is the formation of new cells and tissues? 12) B
Version 1
33
Clarify Question What is the key concept addressed by the question? ● The question asks about the type of uterus found in a koala bear. What type of thinking is required? ● This is an apply question because you have to take what you already know about mammalian reproductive tracts and apply it to a specific taxon. Gather Content What do you already know about mammalian reproductive tracts? What other information is related to the question? ● In mammals, the Fallopian tubes (or oviducts) transport ova from the ovaries to the uterus. ● In humans, the uterus is a muscular, pear-shaped organ that narrows to the cervix, which leads to the vagina. ● Mammals other than primates have more complex female reproductive tracts, in which part of the uterus divides to form uterine “horns”. Choose Answer Given what you now know, what information is most likely to produce the correct answer? ● Marsupials have two unconnected uterine horns, two cervices, and two vaginas. This corresponds to the description of a duplex uterus given in the question stem. ● Koala bears are marsupials, so they most likely have a duplex uterus. Reflect on Process Did your problem-solving process lead you to the correct answer? If not, where did the process break down or lead you astray? How can you revise your approach to produce a more desirable result? ● Answering this question correctly depended on your ability to apply Version 1
34
your knowledge of mammalian reproductive tracts to a specific taxon. If you got an incorrect answer, did you remember that marsupials have two unconnected uterine horns, two cervices, and two vaginas? Did you realize that koala bears are marsupials? 13) B 14) C 15) C 16) D 17) A 18) B 19) C 20) A 21) D 22) C 23) B 24) C 25) B 26) D 27) A 28) C 29) D 30) E 31) B 32) C 33) C 34) C 35) C 36) E 37) C Version 1
35
38) B 39) C 40) D 41) B 42) C 43) C
Version 1
36
Clarify Question • What is the key concept addressed by the question? This question is asking about environmental factors that may affect fertility. • What type of thinking is required? You are being asked to take what you already know and use, or apply, it to improve fertility based on environmental variables. Gather Content • What do you already know about spermatogenesis? What other information is related to the question? o Sperm develop at an optimal temperature that is a few degrees cooler than the typical human body temperature. o Failure to maintain a lower temperature during spermatogenesis leads to impaired sperm. Choose Answer • Given what you now know, what information is most likely to produce the correct answer? o Loud music may have long-term effects on hearing, but its impact on fertility has not been noted. o If contaminated with bacterial, raw vegetable appetizers may be dangerous to pregnant women, but would not pose a threat to the fertility of a man. o Dairy products are not known to impact fertility. Version 1
37
o Since sperm develop at an optimal temperature of 34°C, exposure to hotter temperatures can result in disrupted sperm for months. Reflect on Process • Did your problem-solving process lead you to the correct answer? If not, where did the process break down or lead you astray? How can you revise your approach to produce a more desirable result? o Answering this question correctly depended on your ability to use spermatogenesis in a new situation. If you got an incorrect answer, did you remember that spermatogenesis must occur in temperatures that are lower than human body temperature or that warmer temperatures cause defective sperm? Did you have trouble extending the concept of spermatogenesis to determine the correct answer? 44) D
Version 1
38
Clarify Question What is the key concept addressed by the question? This question is asking about the effects of an alteration of the normal course of spermatogenesis. What type of thinking is required? You are being asked to take what you already know and use, or apply, it to understand the fate of spermatogonia if they were to undergo meiosis. Gather Content What do you already know about spermatogonia? What other information is related to the question? o Spermatogonia are diploid germ cells. They are the cells from which sperm cells are derived. o Spermatogonia produce one primary spermatocyte and one spermatogonium, which are both also diploid cells. o Primary spermatocytes go through meiosis to produce secondary spermatocytes, which are haploid but still have joined sister chromatids for each chromosome. o Secondary spermatocytes complete meiosis to form spermatids, which mature into sperm. Choose Answer Given what you now know, what information is most likely to produce the correct answer? Version 1
39
o This is an alteration of the normal events of spermatogenesis, since spermatogonia do not undergo meiosis during spermatogenesis. o The sperm count would not necessary increase, as it is the type of cell, not the number of cells, that differ in this example. o If spermatogonia were to undergo meiosis instead of mitosis, they would lose their ability to replicate further. The result of meiosis is haploid gametes, which cannot replicate further. o Men's lifelong ability to produce sperm is due to the continued renewal of spermatagonia by one of the mitotic progeny remaining a spermatogonium, and not undergoing meiosis. Reflect on Process Did your problem-solving process lead you to the correct answer? If not, where did the process break down or lead you astray? How can you revise your approach to produce a more desirable result? o Answering this question correctly depended on your ability to use spermatogenesis in a new situation. If you got an incorrect answer, did you remember that spermatogonia do not undergo meiosis or that once a cell undergoes meiosis it cannot replicate further? Did you have trouble extending the concept of spermatogenesis to determine the correct answer? 45) A
Version 1
40
Clarify Question • What is the key concept addressed by the question? The concept addressed in this question involves the source of FSH and LH. • What type of thinking is required? This is an analyze question because you have to dissect unexpected levels of FSH and LH into their component pieces to understand how they function. Gather Content • What do you already know about FSH and LH? What other information is related to the question? o The same hormones may influence both male and female fertility in vertebrates. o Follicle-stimulating hormone (FSH) stimulates spermatogenesis in males, and follicle development in females. o Luteinizing hormone (LH) stimulates the release of testosterone in males, and ovulation in females. o In males, negative feedback acts when testosterone and another hormone, inhibin, are present. In this case, testosterone inhibits FSH and LH production. Choose Answer • Given what you now know, what information is most likely to produce the correct answer? Version 1
41
o If the pituitary gland had been removed, the dog’s life would have ended. Additionally, the removal of the pituitary gland would lower the levels of FSH and LH. o Low levels of GnRH would result in low levels of FSH and LH, not elevated levels. o Negative feedback from testes-produced testosterone and inhibin act to inhibit FSH and LH production. In a neutered animal, this negative feedback is absent and FSH and LH levels will be increased. Reflect on Process • Did your problem-solving process lead you to the correct answer? If not, where did the process break down or lead you astray? How can you revise your approach to produce a more desirable result? o Answering this question correctly depended not only on distinguishing between testosterone and other hormones, but on your ability to break down, or analyze, the role of negative feedback in regulating these. If you got an incorrect answer, did you remember that components of negative feedback include the inhibition of FSH and LH by testosterone or that lack of testosterone would raise the levels of FSH and LH in the body? Did you have trouble breaking down the relationships between these hormones to determine the correct answer? 46) D
Version 1
42
Clarify Question • What is the key concept addressed by the question? This question is asking about what may occur if two follicles burst. • What type of thinking is required? You are being asked to take what you already know and use, or apply, it to understand the rupture of two follicles. Gather Content • What do you already know about follicles? What other information is related to the question? o During each menstrual cycle, usually one dominant follicle emerges. o Follicles contain eggs. o When the follicle ruptures, an egg is released into the Fallopian tube. o Fertilization may occur in the Fallopian tube. Choose Answer • Given what you now know, what information is most likely to produce the correct answer? o High estrogen levels occur when menstruation does not occur, so that answer may be disregarded. o Typically one follicle bursts per cycle, releasing one egg that can be fertilized. o If two follicles burst, two eggs are released, and both could be fertilized. The result of the fertilization of two eggs in one cycle is fraternal twins. Version 1
43
Reflect on Process • Did your problem-solving process lead you to the correct answer? If not, where did the process break down or lead you astray? How can you revise your approach to produce a more desirable result? o Answering this question correctly depended on your ability to use the role of follicles in a new situation. If you got an incorrect answer, did you remember that a follicle releases an egg or that if more than one follicle ruptures, more than one egg is released? Did you have trouble extending the function of follicles to determine the correct answer? 47) D
Version 1
44
Clarify Question • What is the key concept addressed by the question? This question is asking about the age of an oocyte relative to the age of the mother. • What type of thinking is required? You are being asked to take what you already know and use, or apply, it to understand the relationship between maternal age and the age of oocytes. Gather Content • What do you already know about eggs? What other information is related to the question? o By the time a baby girl is born, her lifetime's supply of oocytes is already formed and arrested in prophase I. o Primary oocytes finish meiosis I in the follicular phase of the menstrual cycle, and meiosis II begins. o Meiosis II arrests in metaphase II. This is the stage that is released from the ovary. o Meiosis II will not be completed until the oocyte is fertilized. Choose Answer • Given what you now know, what information is most likely to produce the correct answer? o Since all of the oocytes that a female has are present before her birth, the age of her oocytes is the same as her age. Version 1
45
Reflect on Process • Did your problem-solving process lead you to the correct answer? If not, where did the process break down or lead you astray? How can you revise your approach to produce a more desirable result? o Answering this question correctly depended on your ability to use oocytes in a new situation. If you got an incorrect answer, did you remember that oocytes are present at the birth of a female or oocytes mature as part of a woman’s monthly cycle? Did you have trouble extending the concept of oocytes to determine the correct answer? 48) D
Version 1
46
Clarify Question • What is the key concept addressed by the question? The concept that is addressed by this question is the role that progesterone and cycle length play in reproduction. • What type of thinking is required? This question is asking you to weigh and judge, or evaluate, the effects of low progesterone and short menstrual cycles. Gather Content • What do you already know about progesterone? What other information is related to the question? o Progesterone is produced by the corpus luteum. o Progesterone and estradiol cause the thickening of the uterine lining. o A thickened uterine lining is necessary for successful implantation. Choose Answer • Given what you now know, what information is most likely to produce the correct answer? o Progesterone is relatively low in a normal cycle before ovulation occurs, so low progesterone does not interfere with the development of a follicle or ovulation. o Since ovulation is not impacted by low progesterone, fertilization will not be impacted. o If progesterone is too low, this will cause early menstruation and the uterine lining will be shed before an embryo has a chance to implant. Version 1
47
o The main function of the luteal phase of a woman's cycle is to grow and maintain the endometrial lining of the uterus so it is prepared for implantation of a fertilized egg. Reflect on Process • Did your problem-solving process lead you to the correct answer? If not, where did the process break down or lead you astray? How can you revise your approach to produce a more desirable result? o Answering this question correctly depended on your ability to weigh and judge, or evaluate, the effects of low progesterone and short cycle length. If you got an incorrect answer, did you remember that progesterone causes the uterine lining to thicken or that low progesterone can prevent implantation? Did you have trouble weighing the merits of low progesterone relative to fertility to determine the correct answer? 49) B
Version 1
48
Clarify Question • What is the key concept addressed by the question? This question is asking why some women do not show a rise in their basal body temperature that would indicate the ovulation occurred. • What type of thinking is required? This is an analyze question because you have to consider the differences between female students that do and do not exhibit raised basal body temperature to understand how they function. Gather Content • What do you already know about ovulation? What other information is related to the question? o As estradiol rises during the follicular phase of a woman’s cycle, the pituitary gland suddenly releases luteinizing hormone (LH). o LH causes a mature follicle in the ovary to rupture and release an oocyte. o Ovulation is marked by a small but measureable increase in basal body temperature. Choose Answer • Given what you now know, what information is most likely to produce the correct answer? o IUDs do not stop ovulation, so they would not interfere with the temperature spike that marks the time of ovulation. o Ovulation occurs with or without intercourse; this is not a factor in the Version 1
49
observed temperature spike. o Some with control methods contain hormones; others do not. The general idea of birth control does not necessarily interfere with ovulation. o Hormonal birth control methods block pregnancy by preventing ovulation, and thus there would be no associated change in body temperature. Reflect on Process • Did your problem-solving process lead you to the correct answer? If not, where did the process break down or lead you astray? How can you revise your approach to produce a more desirable result? o Answering this question correctly depended not only on distinguishing between different forms of birth control, but on your ability to break down, or analyze, their effects on ovulation. If you got an incorrect answer, did you remember that components of oral contraceptives include hormones or that those hormones stop ovulation? Did you have trouble breaking down the forms of birth control to determine the correct answer? 50) E The most common cause of infertility is the eventual failure of ovulation and reproductive cycling around the time of menopause. 51) A 52) B 53) D 54) B 55) B
Version 1
50
Clarify Question • What is the key concept addressed by the question? This question is asking you to identify the period of organogenesis. • What type of thinking is required? You are being asked to take what you already know and use, or apply, it to the identification of the time period in which organogenesis occurs. Gather Content • What do you already know about organogenesis? What other information is related to the question? o Organogenesis is the formation of organs in their specified locations. o Organogenesis begins before gastrulation ends, which in humans also marks the initiation of the process of neurulation. o In humans, organogenesis begins the 4th week after fertilization. Choose Answer • Given what you now know, what information is most likely to produce the correct answer? o Since organogenesis begins the 4th week after fertilization, and fertilization occurred in early January, organogenesis in this example would occur in February. Reflect on Process
Version 1
51
• Did your problem-solving process lead you to the correct answer? If not, where did the process break down or lead you astray? How can you revise your approach to produce a more desirable result? o Answering this question correctly depended on your ability to use organogenesis in a new situation. If you got an incorrect answer, did you remember that organogenesis starts in the 4th week after fertilization or that fertilization took place in early January? 56) C
Version 1
52
Clarify Question • What is the key concept addressed by the question? This question is asking about the timing of somitogenesis. • What type of thinking is required? You are being asked to take what you already know and use, or apply, it to the determination of the timing of the appearance of somites. Gather Content • What do you already know about somitogenesis? What other information is related to the question? o The mesoderm gives rise to somitomeres, which give rise to somites. o The timing of somitogenesis overlaps that of neurulatiom. The first somites appear in the 3rd week after fertilization. Choose Answer • Given what you now know, what information is most likely to produce the correct answer? o Since somitogenesis occurs in the 3rd week after fertilization, January 3 and 7 are too early, and January 31 is too late. o January 17 does coincide with the 3rd week of the pregnancy in this example. Reflect on Process
Version 1
53
• Did your problem-solving process lead you to the correct answer? If not, where did the process break down or lead you astray? How can you revise your approach to produce a more desirable result? o Answering this question correctly depended on your ability to use somitogenesis in a new situation. If you got an incorrect answer, did you remember that somitogenesis occurs in the 3rd week of a human pregnancy or that the pregnancy in this example was intiated on January 1? 57) C 58) B
Version 1
54
Clarify Question • What is the key concept addressed by the question? This question is asking you to identify the function of the acrosomal region of the sperm. • What type of thinking is required? You are being asked to take what you already know and use, or apply, it to understand the importance of the acrosomal region of the sperm. Gather Content • What do you already know about the acrosomal region? What other information is related to the question? o The acrosomal region of the sperm occurs between the nucleus and the front end of the sperm. o The acrosomal region contains digestive enzymes that allow the sperm to penetrate the outer layers of the egg. Choose Answer • Given what you now know, what information is most likely to produce the correct answer? o The blastula is a stage of growth that occurs after fertilization. Sperm have already reached the egg if a blastula is present. o The allantois is a membrane that is found after implantation has occurred and the pregnancy is well underway. o The archenteron is a feature that marks gastrulation, which also occurs after fertilization. Version 1
55
o Without the necessary digestive enzymes, a sperm cell cannot penetrate the outer layers of an egg, which include the zona pellucida. Reflect on Process • Did your problem-solving process lead you to the correct answer? If not, where did the process break down or lead you astray? How can you revise your approach to produce a more desirable result? o Answering this question correctly depended on your ability to use the acrosomal region of sperm in a new situation. If you got an incorrect answer, did you remember that the acrosomal region aids fertilization or that without it fertilization could not occur? 59) D 60) E 61) C 62) B 63) C 64) A 65) C 66) B 67) D 68) B 69) E 70) B 71) D 72) D 73) B
Version 1
56
Clarify Question What is the key concept addressed by the question? • The concept addressed here is the recognition of the developmental stage known as the blastula. What type of thinking is required? o You are being asked to take what you already know and use, or apply, it to identify the diagnostic features of a blastula. Gather Content What do you already know about a blastula? What other information is related to the question? • A fertilized egg is known as a zygote. • As a zygote divides, it does not increase in size. Rather it subdivides, so each generation of cells is smaller than the previous one. • Eventually the new cells form a hollow ball of cells. These cells surround a central cavity called a blastocoel. Choose Answer Given what you now know, what information is most likely to produce the correct answer? • The archenteron and neural tube form after the blastula. If either of these structures are present, the blastula stage has been left behind. • Granulosa cells are part of an unfertilized egg. If these cells are present it would be too early for a blastula to have formed. • Seeing a blastocoel would be indicative that a blastula is present, as a Version 1
57
blastocoel is a characteristic of a blastula. Reflect on Process Did your problem-solving process lead you to the correct answer? If not, where did the process break down or lead you astray? How can you revise your approach to produce a more desirable result? o Answering this question correctly depended on your ability to use the nature of a blastula in a new situation. If you got an incorrect answer, did you remember that blastula is a hollow ball of cells or that in the middle of the hollow ball of cells is a space known as the blastocoel? Did you have trouble recognizing the diagnostic features of a blastula to determine the correct answer? 74) A
Version 1
58
Clarify Question • What is the key concept addressed by the question? This question is asking about the consequences of a deficiency of cells inside of the archenteron. • What type of thinking is required? You are being asked to take what you already know and use, or apply, it to understand the fate of an embryo with too few cells in the archenteron. Gather Content • What do you already know about the fate of the archenteron? What other information is related to the question? o Gastrulation creates the three primary germ layers: endoderm, ectoderm, and mesoderm. o The cells in each germ layer have very different developmental fates. o The cells that move into the embryo to form the tube of the primitive gut are endoderm; they give rise to the lining of the gut and its derivatives (pancreas, lungs, liver, etc.). o The cells that remain on the exterior are ectoderm, and their derivatives include the epidermis on the outside of the body and the nervous system. o The cells that move into the space between the endoderm and ectoderm are mesoderm; they eventually form the notochord, bones, blood vessels, connective tissues, muscles and internal organs such as the kidneys and gonads.
Version 1
59
Choose Answer • Given what you now know, what information is most likely to produce the correct answer? o Since the cells inside the archenteron give rise to the endoderm, lack of cells here could cause a deficiency in the lining of the gut and its derivatives. Reflect on Process • Did your problem-solving process lead you to the correct answer? If not, where did the process break down or lead you astray? How can you revise your approach to produce a more desirable result? o Answering this question correctly depended on your ability to use the germ layers in a new situation. If you got an incorrect answer, did you remember that the endoderm is derived from the inside of the archenteron or that the lining of the gut and its derivatives are derived from the endoderm? 75) C
Version 1
60
Clarify Question • What is the key concept addressed by the question? This question is asking about the consequences of a deficiency of mesoderm cells. • What type of thinking is required? You are being asked to take what you already know and use, or apply, it to understand the fate of an embryo with too few mesoderm cells. Gather Content • What do you already know about the fate of the mesoderm? What other information is related to the question? o Gastrulation creates the three primary germ layers: endoderm, ectoderm, and mesoderm. o The cells in each germ layer have very different developmental fates. o The cells that move into the embryo to form the tube of the primitive gut are endoderm; they give rise to the lining of the gut and its derivatives (pancreas, lungs, liver, etc.). o The cells that remain on the exterior are ectoderm, and their derivatives include the epidermis on the outside of the body and the nervous system. o The cells that move into the space between the endoderm and ectoderm are mesoderm; they eventually form the notochord, bones, blood vessels, connective tissues, muscles and internal organs such as the kidneys and gonads. Choose Answer Version 1
61
• Given what you now know, what information is most likely to produce the correct answer? o A deficiency of cells in the mesoderm would not necessarily impact the endoderm or the ectoderm. o Since the mesoderm cells give rise to the muscles and other structures, lack of cells here could cause an issue with muscle development. Reflect on Process • Did your problem-solving process lead you to the correct answer? If not, where did the process break down or lead you astray? How can you revise your approach to produce a more desirable result? o Answering this question correctly depended on your ability to use the germ layers in a new situation. If you got an incorrect answer, did you remember that the mesoderm is the middle germ layer that gives rise to many structures or that the muscles are one type of tissue that is derived from the mesoderm? 76) B
Version 1
62
Clarify Question • What is the key concept addressed by the question? This question is asking about the consequences of a deficiency of surface cells during gastrulation. • What type of thinking is required? You are being asked to take what you already know and use, or apply, it to understand the fate of an embryo with too few surface cells. Gather Content • What do you already know about the fate of the surface cells? What other information is related to the question? o Gastrulation creates the three primary germ layers: endoderm, ectoderm, and mesoderm. o The cells in each germ layer have very different developmental fates. o The cells that move into the embryo to form the tube of the primitive gut are endoderm; they give rise to the lining of the gut and its derivatives (pancreas, lungs, liver, etc.). o The cells that remain on the exterior are ectoderm, and their derivatives include the epidermis on the outside of the body and the nervous system. o The cells that move into the space between the endoderm and ectoderm are mesoderm; they eventually form the notochord, bones, blood vessels, connective tissues, muscles and internal organs such as the kidneys and gonads. Choose Answer Version 1
63
• Given what you now know, what information is most likely to produce the correct answer? o The endoderm is derived from the inside of the archenteron, so a lack of surface cells would not affect its development. o Muscles are derived from the mesoderm, so lack of surface cells, which would give rise to the ectoderm, would not affect muscle development. o Since the surface cells give rise to the ectoderm, lack of cells here could cause an issue with deficiencies in the skin or nervous system. Reflect on Process • Did your problem-solving process lead you to the correct answer? If not, where did the process break down or lead you astray? How can you revise your approach to produce a more desirable result? o Answering this question correctly depended on your ability to use the germ layers in a new situation. If you got an incorrect answer, did you remember that the ectoderm is derived from the outer layer during gastrulation or that the nervous system and skin are derived from the ectoderm? 77) C The placenta is formed from a combination of maternal and fetal tissues, which have only 50% genetic identity. 78) Section Break 78.1) C 78.2) D
Version 1
64
Clarify Question • What is the key concept addressed by the question? This question is asking you to identify the hormonal levels that characterize implantation. • What type of thinking is required? You are being asked to take what you already know and use, or apply, it to the conditions that occur with implantation. Gather Content • What do you already know about implantation? What other information is related to the question? o FSH stimulation causes a follicle to finish the maturation process in the early portion of the cycle. o Increasing estradiol promotes a sudden spike in LH secretion, which in turn promotes ovulation during the mid point, of the cycle. At this point the egg can be fertilized. o The corpus luteum, which secretes progesterone and estradiol, finishes maturation at midcycle. o Higher levels of estradiol and progesterone from the corpus luteum inhibit further secretion of FSH and LH. o Estradiol and progesterone are secreted in the last half of a human female’s menstrual cycle to prepare the lining of the uterus for implantation. o For humans, implantation occurs approximately 5-10 days after fertilization. o A decline in estradiol and progesterone triggers menstruation.
Version 1
65
Choose Answer • Given what you now know, what information is most likely to produce the correct answer? o Days 3, 7, and 13 are too early for implantation. On day 3, the shedding of the uterine lining is still underway. Day 7 lining would not be thick enough for implantation, nor would it be on day 13. Additionally, day 13, as is true for days 3 and 7, is before ovulation, so implantation would not be possible. o Day 27 is immediately before menstruation, and the endometrium is not thick enough for implantation. o On day 21, the lining is at its thickest, ovulation has occurred a week or so previously, and implantation would be very possible. Reflect on Process • Did your problem-solving process lead you to the correct answer? If not, where did the process break down or lead you astray? How can you revise your approach to produce a more desirable result? o Answering this question correctly depended on your ability to use implantation in a new situation. If you got an incorrect answer, did you remember that implantation occurs 5-10 days after fertilization or that the endometrium must be thick enough or implantation cannot occur? Did you have trouble extending the concept of implantation to determine the correct answer? 78.3) A
Version 1
66
CHAPTER 37 TRUE/FALSE - Write 'T' if the statement is true and 'F' if the statement is false. 1) Haplodiploidy is necessary for eusociality. ⊚ ⊚
true false
2) In the dunnock, females that mate with a subordinate male are guaranteed to have his help raising her young. ⊚ ⊚
true false
CHECK ALL THE APPLY. Choose all options that best completes the statement or answers the question. 3) Which of the following are used by species for navigation? (Check all that apply.) A) Landmarks B) Magnetic fields C) Phases of the moon D) Stars E) The sun
4) Foraging behaviors are those having to do with which of the following? (Check all that apply.) A) How an animal avoids becoming the prey of a larger predator B) How an animal finds its food C) How much food an animal eats D) What an animal eats E) When an animal eats
5)
Which of the following are eusocial (true social) insects? (Check all that apply.)
Version 1
1
A) Ants B) Fruit flies C) Honeybees D) Termites E) Wasps
6) Which of the following statements comparing the naked mole rat social system with social insect societies are true? (Check all that apply.) A) As with socialinsect societies, colony members are kin. B) As with social insect societies, it is based on haplodiploidy. C) As with social insect societies, there is a division of labor within thecolony. D) As with socialinsect societies, there is one queen and several reproductive males percolony. E) Unlike social insect societies, all colony members are diploid.
7)
What three features define eusociality? (Check three.) A) Colony resides in an enclosed hive or nest B) Cooperative brood care C) Haplodiploidy D) Insect species only E) Overlap of generations F) Reproductive division of labor G) Reproduction during a mating flight
8)
What would be included under the umbrella of "inclusive fitness"? (Check all that apply.)
Version 1
2
A) Reproduction by an another individual that practices reciprocal altruism B) Reproduction of the individual C) Reproduction by nieces and nephews D) Reproduction by unrelated colony members E) Reproduction by siblings
9) Which of the studies below provide evidence linkinggenes and behavior? (Check all that apply) A) Fruit flies that are mutant for the genes dunce or rutabaga have impaired learning. B) Mice that are lacking either the ephrinB3 or the EphA4 gene cannot walk normally, but instead hop like a kangaroo. C) Two lines of fruit flies were created by repeatedly selecting for high orlow aggressive behavior. After multiple generations, the high line was reliablymore aggressive than the low line. D) Twin studiescomparing monozygotic and dizygotic twins have found a high heritability forheight, although environmental influences like nutrition also play a role.
10) Male fruit flies perform a complex mating dance that includes following the female, vibrating a wing to produce a species-specific song, and picking up chemosensory cues by tapping with the legs and licking with the tongue. What physiological features are likely important for causing this behavior? (Check all that apply) A) Adaptive value of successful mating B) Brain circuits C) Hormones D) Natural selection E) Performance of phylogenetically-related species F) Sensory neurons on the legs and tongue G) Sperm competition H) Threats of predation during the performance I) Visual system J) Wings and wing muscles
11)
In which of the following examples has communication occurred? (Check all that apply)
Version 1
3
A) A bird gives analarm call when a hawk approaches, but no other birds are in the area. B) A bird sings fromthe top of a tree in its territory attracting a female of the same species toit. C) A bird sings fromthe top of a tree in its territory keeping other birds of the same species fromtrespassing. D) An ant lays down atrail of pheromones that other ants follow. E) A waggle dancing bee feeds some of thenectar she is carrying to bees attending her dance; they now know what type offlowers she has visited.
12) What sensory modalities are used by the message receivers in honeybee waggle dance communication? (Check all that apply.) A) Auditory B) Autonomic C) Chemosensory D) Proprioceptive E) Visual
13) Which are examples of advantages that different species might have due to living in a group? (Check all that apply.) A) Wolves in a packcan take down larger prey than those alone. B) Zebras in a herdhave added protection from predators. C) Because of theirsocial nature, diseases like rabies can spread quickly through a batcolony. D) A colony of termites can build mounds over 20 feet high, with elaboratetunnels and ventilation systems. E) A flamingo in a large flock may havetrouble competing for food.
14)
Which features of naked mole rats are similar to eusocial insects? (Check all that apply.)
Version 1
4
A) A single queen is responsible for breeding B) Colony has a large number of individuals C) Division of labor D) Haplodiploidy E) Higher relatedness between siblings than between parent and child
15) Based on the animal cognition experiments you have learned about, if you wanted to design a test ofanimal cognition, what might you include in your experiment? (Check all that apply.) A) Count the numberof individuals in a colony. B) Create a problemfor the animal to solve, like getting a treat out of a box. C) Determine ahead oftime what parameters to record as data, such as how long it takes the animal toretrieve the treat. D) Determine thehormonal basis of caste differences. E) Expose the animalto the same scenario again later. F) Focus on carefulquantitative analysis of fixed action patterns. G) Include a negativecontrol. H) Include multiplesubjects. I) Measure thefrequency of response to sign stimuli. J) Provide tools forthe animal to use.
Version 1
5
16)
Which frog species exhibit altricial care?
A) Ameerega trivittata B) Dendrobates leucomelas C) Oophaga granulifera D) Oophaga pumillo E) Oophaga speciosa
MULTIPLE CHOICE - Choose the one alternative that best completes the statement or answers the question. 17) What is the probability that identical twins will share the same genetype for genes that influence human behavior? A) 25% B) 50% C) 100% D) It depends on which genes are being studied.
Version 1
6
18) Tinbergen is credited with founding the field of behavioral __________, the study of how natural selection shapes behavior. A) genetics B) ecology C) symptomatology D) demographics
19) Animals tend to feed on prey that maximize their net energy intake. This is called the __________ theory. A) competitive exclusion B) maximal consumption C) optimal foraging D) optimization
20) Animals that acquire energy efficiently during foraging will increase their fitness by having more energy available for __________. A) competition B) defenses C) respiration D) reproduction
21) Learning is possible only within the boundaries set by an innate predisposition called __________. A) instinct B) free will C) consciousness D) operance
Version 1
7
22)
Alarm calling seems to be an example of __________, that is, it favors relatives. A) reciprocal altruism B) eusociality C) natural selection D) kin selection
23) Chemical signals that mediate interactions between two or more members of a given species are called __________. A) pheromones B) alarm signals C) hormones D) competitive exclusions
24)
Artificial selection and hybridization can demonstrate the __________ basis of behavior. A) cognitive B) operant C) genetic D) associational E) endogenous
25) As an animal matures, it forms attachments to other individuals and develops preferences. This process is called A) instinct. B) imprinting. C) associational learning. D) habituation. E) sensitization.
Version 1
8
26) Current research in behavioral ecology focuses on the overall contribution that behavior makes to an animal's A) fitness. B) learning. C) competitive strategies. D) foraging efficiency. E) length of life.
27) The processing of information and response in a manner that suggests thinking in the animal is called A) endogenous behavior. B) environmental induced behavior. C) instinctive behavior. D) associative behavior. E) cognitive behavior.
28) In many species, the behavior of a male triggers a specific behavior in a female that eventually results in mating. This is termed a(n) __________. A) conditioned reflex. B) fixed action pattern. C) reinforcing stimulus. D) imprinting mechanism. E) stimulus-response chain.
29) In birds, if the offspring require extensive care, usually involving both the male and female, they are called
Version 1
9
A) semelparous species. B) altricial species. C) precocial species. D) iteroparous species. E) sympatric species.
30)
Who conducted the famous "imprinting" experiment on geese? A) Mendel B) Russel C) Lorenz D) Darwin E) Raven
31) A behavior that has evolved to aid relatives, although at personal risk, and thus increases the chance of your genes being passed on to the next generation is known as A) reciprocal altrusim B) innate behavior. C) kin selection. D) cognitive behavior. E) adaptive behavior.
32)
Social insect colonies are composed of highly specialized groups called what? A) Clines B) Ecotypes C) Castes D) Species E) Filial relatives
Version 1
10
33) Insect workers propagate more of their own alleles by helping their mother reproduce rather than by reproducing themselves; thus, they share a larger fraction of their __________ with the next generation. A) food resources B) time C) genome D) defensive abilities E) home
34)
Courtship is often dependent on what? A) Social releasers B) A stimulus/response chain C) An alarm call D) Taxis E) Size and colors
35) Foraging bees communicate with other bees in a waggle dance to give information about the food. The important cue in the dance that conveys information about the required flight direction to the food source relative to the hive-sun direction is the A) angle between the food source and the hive in reference to the sun. B) angle between the waggle run and the vertical axis within the hive. C) angle between the waggle run in reference to the hive-sun angle. D) angle between the waggle run in reference to the direction north from the hive. E) speed of the waggle dance.
36) Konrad Lorenz demonstrated that newly hatched birds would direct their social behavior toward him if they saw him first after they hatched from their eggs. What is this called?
Version 1
11
A) Foraging behavior B) Cognitive behavior C) Behavioral genetics D) Stimulus-response chain E) Imprinting behavior
37) Recently the focus of research in behavioral ecology has been on the contribution by behavior to an animal's reproductive success. What is this called? A) Average number of mates B) Fitness C) Foraging efficiency D) Longevity
38) A friend of yours is wondering about the differences between the words "orientation" and "navigation" when these two terms are used in descriptions of bird migrations. Since you have studied and know something about animal behavior, what do you tell your friend? A) Orientation isadjusting a bearing, while navigation is actually following a bearing. B) Orientation andnavigation are used interchangeably when referring to avian migrations. C) Orientation isfollowing a bearing, while navigation is setting or adjusting a bearing. D) Orientation isfollowing the sun in the day as starling do, while navigation is following abearing with a small amount of magnetite, which is found in the heads of somemigratory birds. E) Orientation is theability to find true east, navigation is the ability to find a bearing whilecrossing water.
39) The area over which an animal moves in the course of daily activity, but which it does not necessarily defend against other animals, is called what?
Version 1
12
A) Family home B) Foraging space C) Home range D) Nesting site E) Territory
40) In species where the young are precocial, meaning requiring little parental care, males may be more likely to be what? A) Monogamous B) Polygynous C) Polyandrous D) Altricial
41)
Living as a member of a group may have all of the following advantages except what? A) Being more resistant to disease and parasites B) Increased feeding rate C) Greater protection from predators D) Members learn about new food sources from other members E) More individuals scan the environment for dangers
42) Haldane pointed out that if he received a certain allele, the chance that one of his brothers would receive that allele would be A) 100% B) 50% C) 10% D) Random E) 25%
Version 1
13
43)
What is the term for selection that favors altruism toward relatives? A) Kin selection B) Mate choice C) Group selection D) Reciprocal altruism
44) Hamilton explained the origin of altruism in selected insect societies with his kin selection model of sex determination. Since males are haploid and females are diploid, the workers share a very high proportion of alleles, theoretically as high as 75%. What is this called? A) Diploidy B) Haploidy C) Haplodiploidy D) Parthenogenesis E) Altruism
45) Your study buddy asks,"Would you please explain the difference between home range and territory?" What isthe best response? A) Territory is the entire area that an animal can utilize for its resources, such as shelter, food, and mates. Home range is near its nest or den. B) Territory is the area that an animal can utilize for its resources, such as shelter, food and mates and will defend against other members of its species. Home range is near its nest or den. C) Territory is the area that an animal can utilize for its resources, such as shelter, food and mates, and will defend against other members of its species. Home range is the area that an animal may roam on a daily basis. D) Territory is the area that an animal can utilize for its resources, such as shelter, food and mates, and will defend against others members of its species. Home range is a smaller area within the territory that the animal is found in when it is resting or hiding from predators.
46) What is the term for learning that occurs only during a brief period early in life and results in a behavior that is difficult to modify. Version 1
14
A) Imprinting B) Filial imprinting C) Innate behavior D) Instinct E) Operant conditioning
47)
Which of the following statements best defines an altruistic act? A) Benefits the performer and another individual B) Benefits another individual at some cost to the performer C) Benefits another, related individual at some cost to the performer D) Imposes a cost on the performer and another individual E) Imposes a cost on the performer without benefiting another individual
48) Research on starlings indicates that young birds migrate by __________, but older birds use true __________. A) orientation;navigation B) navigation;migration C) migration;orientation D) navigation;orientation
49) Which statement best sums up our understanding of how animals navigate during migration?
Version 1
15
A) The mechanisms of orientation and navigation have beenworked out in great detail, from the genes involved, to the brain structures, to the resulting behavior. B) For most migrating species, we have a thorough understanding of orientation, but not of navigation. C) We have some understanding of the information different animals rely on to guide their migration, but the exact mechanisms are still unknown. D) For most migrating species, we have a thorough understanding of navigation, but not of orientation.
50)
If a behavior has evolved by natural selection, it must have __________. A) benefits for bothmales and females B) reduced the death rate for individuals that practice it C) evolved in an ancestral species D) a genetic basis
51) A certain species has offspring that require a very large amount of parental care. Predict their mating system. A) Monogamous B) Polyandrous C) Polygynous D) Monotonous
52) Due to reproductive isolation, a new species of weaver bird is evolving. Is it likely to use a social or solitary nesting strategy? A) It depends on whether it lives in the forest or the open savanna. B) It depends on whether it lives in near water or near the desert. C) It depends on how many similar species it is competing with. D) It depends on how many eggs it lays.
Version 1
16
53)
Which of the following is an appropriate interpretation for these graphs?
Maternal care (as measured by minutes crouching over offspring and proportion of pups retrieved) in female mice that have the fosB allele is A) less than the maternal care given by female mice without the fosB allele. B) greater than the maternal care given by female mice without the fosB allele. C) the same as the maternal care given by female mice without the fosB allele. D) less than the maternal care given by female mice without the fosB allele; however, the graphs depict only minor differences, which are most likely not significant. E) not possible to determine from the data.
Version 1
17
54) Which of the following is the best interpretation of the graph below of mussel size (Xaxis) versus energy gain (line graph, left Y-axis), and number of mussels eaten per day (histogram, right Y-axis)?
A) Mussels are selected as food sources by crabs. B) Crabs select mussels in a way that maximizes their energy gain. C) Mussel size does not seem to be a good predictor as to which mussel hungry crabs will select. D) Crabs tend to consume most of the largest mussels. E) Crabs prefer the mussels with the smallest length for their food resources.
55) You are on the committee to update the design of the hospital NICU (neonatal intensive care unit), which currently has incubators for the babies but not much else.Based on what you have learned about Harlow's experiments with rhesus monkeys, what would you suggest adding to the NICU to improve the babies' health and well-being? A) Comfortable chairs for parents to hold their babies B) Softer lighting C) White-noise devices to mask the sound of beeping machines D) More frequent feedings for babies
Version 1
18
56) A zoo is hand-rearing an endangered species of songbird. What is the best approachto helppreparethe males toattract mates? A) Expose the males to a mentor male bird singing their species' song during the critical period. B) Expose the males to female pheromones from their species to initiate development of song circuits duringthe critical period. C) Expose the males to each other during the critical period to initiate development of song circuits duringthe critical period. D) Expose the males toa variety of birdsongsduring the critical period.
57) A bird sanctuary sometimes takes over the care ofeggs or chicks of anendangered duck species if the parents are killed. They are sometimes able to use mothers of farm ducks to cross– foster the chicks, but it doesn't always work.Looking at the following data table, what is the critical period for imprinting in this species? (Note: DAH stands for "days after hatching".) Number of chicks
Age of chicks when paired with foster mother
Successful imprinting? (assessed at 14 DAH)
13
eggs (0 DAH)
yes
7
1 DAH
yes
11
3 DAH
yes
5
8 DAH
no
9
11 DAH
no
A) The critical period extends from 1–3 DAH. B) The critical period extends from 3–8 DAH. C) The critical period ends sometime between 1–3 DAH. We can not determine from this data when the critical period begins. D) The critical period ends sometime between 3–8 DAH. We can not determine from this data when the critical period begins.
58) At a research conference, you meet a graduate studentwho studies the hormonal underpinnings of mating behaviorintwo species of squirrel. "It turns out the difference between my two squirrel species is similar to the difference between the prairie and the montane voles," he says. You nod knowingly, and reply ...
Version 1
19
A) "Oh, so thepromiscuous species has a lot ofvasopressin and oxytocin receptors in thenucleus accumbus, blocking pair bonding, and the monogamous speciesdoesn't?" B) "Oh, so themonogamous species has a lot ofvasopressin and oxytocin receptors in thenucleus accumbus, promoting pair bonding, and the promiscuous speciesdoesn't?" C) "Oh, so thepolyandrous species has a lot ofserotonin and dopamine receptors in thenucleus accumbus, promoting pair bonding, and the promiscuous speciesdoesn't?" D) "Oh, so themonogamous species has a lot ofserotonin and dopamine receptors in thenucleus accumbus, promoting pair bonding, and the promiscuous speciesdoesn't?"
59) Female fire beetles, Melanophilia sp., must seek out freshly burned trees to lay their eggs in so the larvae can feed on the burned wood. What specialized neural structures might you expect these beetles to have to facilitate this behavior? A) Extra large axons for rapid axonal transduction B) Special, enlarged synapses for rapid synaptic transmission C) Giant brain structures for processing auditory information D) Special sensory structures that can detect signs of fire or burned wood
60) Which one of the following examples describes a communication mode that would be best for signaling over the greatest distance in a dark, densely forested environment? A) Display of plumage of a male bird to attract females B) Waggle dance of a honeybee in a colony within the forest C) Pheromones released by a female moth D) Territorial song of a male bird to repel other males E) Flashing of a male firefly to attract conspecific females
61) One spring, your friend living in Minnesota eagerly sets up her new hummingbird feeders and is thrilled to watch hummingbirds feedingon the nectar. However, she is disappointed when after justa week or twothe hummingbirds disappear. What advice would you give her?
Version 1
20
A) Set up a bell(conditioned stimulus) near the feeders so the hummingbirds make an associationwith the food. B) Set up flashing red lights that mimic the male neck feathers—that should attract more malesto the area. C) Set up the feedersagain in the fall. The hummingbirds may have briefly traveled through duringtheir annual spring migration. D) Wait till next year—hummingbirds are a high-metabolism, semelparous species and have already reproduced and died.
62)
In regards to altruism, what way are humans like vampire bats? A) Their altruism isbased on haplodiploidy. B) They are bothChiroptera. C) Their altruism isbased on kin selection. D) They engage inreciprocal altruism.
63) Correctly rephrase the following famous quote from J. B. S. Haldane. "I would lay down my life in order to save two brothers or __________." A) one son or daughter B) four cousins C) three uncles D) two nephews and one son or daughter E) two grandsons or granddaughters
64) It is 6:00 PM, and the sun is due west of the hive. A honeybee has just returned to its hive from a patch of flowers that are due north of the hive. Which of the following best describes the type of waggle dance that this bee should perform? Note: The waggle run is the straight part of the dance.
Version 1
21
A) The waggle run should be straight down the vertical axis. B) The waggle run should be straight up the vertical axis. C) The waggle run should be 90° to the left of the vertical axis. D) The waggle run should be 90° to the right of the vertical axis. E) The waggle run should be 45° to the right of the vertical axis.
65) Some bighorn sheep are bolder than others, and when mountain lion predation is common, the bolder animals have higher survival. What prediction would you make about a population of bighorn sheep experiencing a relatively constant rate of mountain lion predation? A) The proportion of bold sheep in the population would gradually increase over time. B) The proportion of bold sheep in the population would gradually decrease over time. C) The proportion of bold sheep in the population would remain relatively constant over time. D) The proportion of bold sheep in the population would rapidly decrease until the genes for bold behavior were bred out.
66) In a population of side-blotched lizards, what would you expect to happen if the proportion of sedentarymales increased? A) The proportion of sneaker males would decrease. B) The proportion of sneaker males would also increase. C) The proportion of dominant males would decrease. D) The proportion of dominant and sneaker males would arrive at equilibrium.
Version 1
22
SECTION BREAK. Answer all the part questions. 67) Tinbergen observed that after gull nestlings hatch, the parents remove the eggshells from the nest. To understand why (ultimate causation), he painted chicken eggs to resemble gull eggs, which had camouflage coloration to allow them to be inconspicuous against the natural background. He distributedthemthroughouttheareainwhich thegulls were nesting, placing broken eggshells with their prominent white interiors next to some of the eggs. As a control, he left other camouflaged eggs alone without eggshells. He then noted which eggs werefound moreeasilybycrows.Because the crows could use the white interior of a broken eggshell as a cue, theyate more of thecamouflaged eggsthat werenear eggshells. Tinbergen concluded that eggshell-removal behavior is adaptive: itreduces predationand thusincreasestheoffspring’s chances of survival.
67.1) What legitimate factors might lead a scientist to do an imperfect version of an experiment, such as using chicken eggs as a stand-in for gull eggs? (Check all that apply.) A) Because the scientist doesn't care about proper procedure. B) The perfect experiment is too expensive. C) The perfect experiment would harm the study species. D) To quickly get preliminary data for a grant. E) Ignorance. F) The perfect experiment would take too much time.
68) Lepidopterans (moths and butterflies) sometimeshave specialized flying behaviors. Some moths have the ability to do sudden vertical drops when they detect bat sonar, to avoid being eaten. Other species use special flying techniques to attract a mate. When biologists talk about the "cause" of a behavior, they may be referring to two different things. The proximate cause is the sequence of physiological events that lead to the behavior in an individual. The ultimate cause is the adaptive benefit to the animal that allowed natural selection to sculpt the behavior over evolutionary time. Understanding both the proximate and ultimate basis of behavior is important, and different types of biologists may focus on each.
68.1) A moth species is known for its unusual flyingbehavior, which includes sudden vertical drops and horizontal zigzags. Not much is known about the cause of this behavior.What could you study to better understand the proximate basis of this behavior? (Check all that apply.)
Version 1
23
A) Brain structure B) Flying behavior in related species C) Hormones D) Mating behavior E) Physiology F) Predation G) Sensory organs H) Wing muscle development
68.2) What could you study to better understand the ultimate basis of the moth's flying behavior? (Check all that apply) A) Brain structure B) Flying behavior in related species C) Hormones D) Mating behavior E) Physiology F) Predation G) Sensory organs H) Wing muscle development
Version 1
24
Answer Key Test name: Chap 37_3e 1) FALSE Not necessarily. Termites and naked mole rats have complex societies, but do not have haplodiploidy. However, haplodiploidy does seem to favor the evolution of eusociality, because it increases the relatedness of female siblings (75%)compared to mothers and progeny (50%). 2) FALSE Females must mate with a subordinate male a sufficient number of times to receive his help raising her young. A single occurrence of copulation would not guarantee that the male would provide parental care. 3) [A, B, D, E] 4) [B, C, D, E] 5) [A, C, D, E] 6) [A, C, D, E] 7) [B, E, F] 8) [B, C, E] 9) [A, B, C]
Version 1
25
Clarify Question • What is the key concept addressed by the question? o Identifying evidence for the connections between a behavior and the genes involved. • What type of thinking is required? o This is an analyze question. You are being presenting the four different studies and you need to select the ones that are linking genes and behavior. You need to read each choice and see what the genes are doing and then determine if the answer choice is an example of a gene modification to an animal’s behavior. There is more than one correct answer. Gather Content • What do you already know about the genetics that underlies a behavior? What other information is related to the question? o This question comes from the section of the textbook that discusses scientific studies that have shown the links between animal behavior and the genes that underlie the behavior. Learning, locomotion, and aggression are all examples of behavior. The physical size of an organism is not a behavior. Choose Answer • Given what you now know, what information is most likely to produce the correct answer? o All are examples of behaviors except for the twin study that demonstrates the heritability of height. Reflect on Process • Did your problem-solving process lead you to the correct answer? If not, where did the process break down or lead you astray? How can Version 1
26
you revise your approach to produce a more desirable result? o Were you able to select all of the correct choices? You had to look over all the choices and pick out all of the ones that demonstrated the link between genes and behavior. 10) [B, C, F, I, J]
Version 1
27
Clarify Question • What is the key concept addressed by the question? o The question asks about the underlying factors affecting various behaviors in fruit flies. • What type of thinking is required? o This is an analyze question. You have to analyze the various factors that might play a role in the mating dance of male fruit flies. Physiological factors are going to allow the fly to perform the dance. Evolutionary factors help to determine the adaptive value of the dance. This is a check-all-that-apply question; all of the choices that are directly involved in performing the behaviors are correct choices. Gather Content • What do you already know about physiological and evolutionary factors underlying a behavior? What other information is related to the question? o In this question, you need to select all of the physiological factors from the choices given. The physiological factors are the parts of the fly that allow it to perform the dance. Evolutionary forces certainly play a role in selecting for certain behaviors, but they are not physiological factors that allow for the performance of the specific behavior. Choose Answer • Given what you now know, what information is most likely to produce the correct answer? o Brain circuits, hormones, sensory neurons, visual system, and wings are all involved in the physiological factors that are involved in the male mating dance. Reflect on Process
Version 1
28
• Did your problem-solving process lead you to the correct answer? If not, where did the process break down or lead you astray? How can you revise your approach to produce a more desirable result? o Did you get the question right? You had to look over all the choices and pick out all that are physiological factors that influence behavior. 11) [B, C, D, E]
Version 1
29
Clarify Question • What is the key concept addressed by the question? o The question is asking about situations that involve communication. • What type of thinking is required? o This is an analyze question. You need to look at each choice presented, break it down into its parts, and see if it matches the criteria to count as communication. This is a check all that apply question, which means that there can be more than one correct answer. Examine each choice and see if it fulfills the definition of communication. Gather Content • What do you already know about what behavior qualifies as communication? What other information is related to the question? o This question is from the section of the textbook that discusses what defines animal communication and different modes of animal communication. All of the choices are examples of behaviors; you need to select the behaviors that are example of communication. Choose Answer • Given what you now know, what information is most likely to produce the correct answer? o There are five choices presented, and all but one are examples, of animal communication. In the first example a bird gives an alarm call but no other birds hears the call, thus no communication took place. Reflect on Process • Did your problem-solving process lead you to the correct answer? If not, where did the process break down or lead you astray? How can you revise your approach to produce a more desirable result? Version 1
30
o Were you able to select all of the correct choices? You needed to extract the information from the graph and pick the correct choice. The key to this question is the realization that communication needs at least one individual to receive the message. 12) [A, C, E]
Version 1
31
Clarify Question • What is the key concept addressed by the question? o The waggle dance behavior in bees and what sensory inputs are used. • What type of thinking is required? o This is an analyze question; you are given several different sensory information modalities used by bees and are being asked to choose the ones that will allow the bee to get information from the waggle dance. Gather Content • What do you already know about the waggle dance in bees? What other information is related to the question? o This question comes from the section of the textbook that discusses the role of the waggle dance in honeybee communication. All of the choices are sensory modalities, but not all of them are used to gather information from the dance. Choose Answer • Given what you now know, what information is most likely to produce the correct answer? o The dance produces signals via sight, sounds, and chemical information. Reflect on Process • Did your problem-solving process lead you to the correct answer? If not, where did the process break down or lead you astray? How can you revise your approach to produce a more desirable result? o Did you get the question right? The key to this question is to figure out how the dance provides information to the other bees. 13) [A, B, D] Version 1
32
Clarify Question • What is the key concept addressed by the question? o The advantages of living in a group and which examples exhibit those. • What type of thinking is required? o This is an analyze question, so you will need to break the choices down into their components to get the correct answer. This question gives you several scenarios and you are being asked to identify which ones represent an advantage to group living. This is a check-all-thatapply question; there are multiple correct answers. Gather Content • What do you already know about the advantages of group living? What other information is related to the question? o Everything is a trade-off. Group living has advantages and disadvantages. You need to look at each choice and determine if it represents an advantage or a disadvantage. Disease spreads much quicker in large groups of individuals and large groups of individuals will need more food and other resources. Choose Answer • Given what you now know, what information is most likely to produce the correct answer? o More efficient predation, more protection from predators, and the construction of large scale structures are certainly advantages. Reflect on Process • Did your problem-solving process lead you to the correct answer? If not, where did the process break down or lead you astray? How can you revise your approach to produce a more desirable result? Version 1
33
o Did you select all of the correct answers? Understanding that living in groups is a trade-off is the key to this question. 14) [A, B, C]
Version 1
34
Clarify Question • What is the key concept addressed by the question? o Comparing group living in naked mole rats and insects. • What type of thinking is required? o This is an analyze question; you will need to break the choices down into their components to get the correct answer. In this question, you are being asked to select all of the similarities between naked mole rats and social insects. There are multiple correct answers. Gather Content • What do you already know about group living in different animal groups? What other information is related to the question? o These 2 types of societies have lots of similarities, but they aren’t identical. The organizations are quite similar, but the underlying genetics are quite different. Choose Answer • Given what you now know, what information is most likely to produce the correct answer? o The features responsible for the organization of the colony are the similarities. Reflect on Process • Did your problem-solving process lead you to the correct answer? If not, where did the process break down or lead you astray? How can you revise your approach to produce a more desirable result? o Did you select all of the correct answers? The key is to see the difference between how the organization of the colony and the genes that produced those behaviors are different. Version 1
35
15) [B, C, E, G, H, J]
Version 1
36
Clarify Question • What is the key concept addressed by the question? o What would be the requirements for designing a experiment studying animal cognition. • What type of thinking is required? o This is an apply question. You need to take what you know about animal cognition and apply it to this situation. This is a check-all-thatapply type of question; there are multiple correct answers. All of the choices are parts of an experiment. You need to select all of the choices that would be useful in designing an experiment that test animal cognition. Gather Content • What do you already know about animal cognition experiments? What other information is related to the question? o Counting individuals, determining hormones levels, analyzing fixed action patterns, and measuring the response to key stimuli are all valid experimental objectives, but they do not provide any insight into an animal’s cognitive ability. All of the other choices are good experimental design characteristics and will help assess the level of cognitive ability in the research animals. Choose Answer • Given what you now know, what information is most likely to produce the correct answer? o All of the other choices are good experimental design characteristics and will help assess the level of cognitive ability in the research animals. Reflect on Process • Did your problem-solving process lead you to the correct answer? If Version 1
37
not, where did the process break down or lead you astray? How can you revise your approach to produce a more desirable result? o Were you able to select all of the correct choices? The key to this apply question is to look at the choices are see if the information they provide will help measure an animal's cognitive ability. 16) [C, D] 17) C Identical twins have exactly the same genome 18) B 19) C 20) D 21) A 22) D 23) A 24) C 25) B 26) A 27) E 28) E 29) B 30) C 31) C 32) C 33) C 34) B 35) B 36) E 37) B 38) C 39) C Version 1
38
40) B 41) A 42) B 43) A 44) C 45) C 46) A 47) B 48) A 49) C 50) D Natural selection can only select for behaviors that have some genetic basis. The special display behaviorof a male peacock is an example of a behavior that was naturally selected, but is only present in the male probably increases rather than reduces predation, and is not seen in other related species. 51) A 52) A
Version 1
39
African weaver birds, which construct nests from vegetation, provide an excellent example of the relationship between ecology and social organization. Their roughly 90 species can be divided according to the type of social group they form. One group of species lives in the forest and builds camouflaged, solitary nests. Males and females are monogamous; they forage for insects to feed their young. The second group of species nests in colonies in trees on the savanna. They are polygynous and feed in flocks on seeds. The feeding and nesting habits of these two groups of species are correlated with their mating systems. In the forest, insects are hard to find, and both parents must cooperate in feeding the young. The camouflaged nests do not call the attention of predators to their brood. On the open savanna, building a hidden nest is not an option. Rather, savanna-dwelling weaver birds protect their young from predators by nesting in trees, which are not very abundant. This shortage of safe nest sites means that birds must nest together in colonies. Because seeds occur abundantly, a female can acquire all the food needed to rear young without a male’s help. The male, free from the duties of parenting, spends his time courting many females—a polygynous mating system. 53) B
Version 1
40
Clarify Question • What is the key concept addressed by the question? o This question requires the interpretation of a graph. The data on the graph show the relationship between intensity of material behavior and the presence/absence of a particular allele. • What type of thinking is required? o This is an evaluation question, you are being asked to look at the choices being presented and pick the best one. Gather Content • What do you already know about the relationship between maternal behavior and a particular allele? What other information is related to the question? o What do you know about this situation? Genes can control behaviors. Providing maternal care to offspring can increase their survivability, increasing their fitness. Alleles are forms of a gene. Which group has the highest level of maternal behaviors? Choose Answer • Given what you now know, what information is most likely to produce the correct answer? o Mice with fosB allele spend much longer crouching over their infants and retrieving a great proportion of their pups when they are separated from them. These behaviors demonstrate maternal care provided to the infants. The choice that says “greater than the maternal care given by female mice without the fosB allele” is the correct answer. Reflect on Process • Did your problem-solving process lead you to the correct answer? If not, where did the process break down or lead you astray? How can you revise your approach to produce a more desirable result? o Were you able to select the correct answer? You were being asked to look at a graph, evaluate a series of statements and pick the choice that best fit the graph. This question is linked to the section in the textbook that describes the relationship between genes and behavior. In this example the presence of one allele was strongly associated with an increase in the intensity of maternal care, providing evidence for a link between the observed behavior (maternal care) and the genes that underlie that behavior.
54) B
Version 1
41
Clarify Question • What is the key concept addressed by the question? o This graph show three sets of data: size of mussels, energy content relative to the size of the mussel, and the number and size of mussels that a crab consumes per day. • What type of thinking is required? o This is an evaluate question. You are being asked to break down the information into individual components and then pick the best interpretation of the data. Gather Content • What do you already know about optimal foraging behaviors? What other information is related to the question? o This question is linked to the section of the textbook that discusses optimal foraging behaviors. Optimal foraging essentially means getting the most bang for your buck. This data set certainly shows that crabs eat muscles, but which is the best interpretation of the graph? Is there a pattern in the size that is selected or are the crabs eating mussels randomly? Do the largest mussels provide the most nutrition to the crab? Choose Answer • Given what you now know, what information is most likely to produce the correct answer? o The peak of the histogram and the line graph match pretty well. That shows that the crabs are not just randomly eating mussels, but are preferentially selecting the mussels that provide the highest energy gain. A crab that preferentially eats the most energy-rich mussels will have to spend less time eating and will have optimized their foraging. Reflect on Process • Did your problem-solving process lead you to the correct answer? If not, where did the process break down or lead you astray? How can you revise your approach to produce a more desirable result? o Were you able to pick the correct answer? You needed to extract the information from the graph, and pick the best interpretation of the data from the choices presented. The concept here is optimal foraging; the crab was trying to get the most reward for the least amount of effort.
55) A
Version 1
42
Clarify Question • What is the key concept addressed by the question? o Looking at what humans need in light of the Harlow experiment with rhesus monkeys. • What type of thinking is required? o This is an evaluate question, you are making a judgement as to which choice is the best possible answer. You will need to pick the choice that takes the results of Harlow’s experiments with rhesus monkeys and apply it to human babies. Gather Content • What do you already know about Harlow's experimental results? What other information is related to the question? o Harlow’s experiment with baby rhesus monkeys demonstrated that baby monkeys need to have physical contact with their mothers in order to develop normally. Most of these choices would provide a more comforting environment for an infant, but only one of these would make it easier for mothers to provide their infants with more opportunities for physical contact with their mothers. Choose Answer • Given what you now know, what information is most likely to produce the correct answer? o Comfortable chairs would allow for more tactile stimulation between mothers and infants. Reflect on Process • Did your problem-solving process lead you to the correct answer? If not, where did the process break down or lead you astray? How can Version 1
43
you revise your approach to produce a more desirable result? o Did you get the question right? The key to this evaluate question was understand the implications of Harlow’s research on the influence of tactile stimulation and the development of the infant. Then the challenge was to evaluate the choices and determine which one would be the best for promoting increased contact between mother and infant. 56) A
Version 1
44
Clarify Question • What is the key concept addressed by the question? o Knowing what environment will be important for the males to learn mating behaviors. • What type of thinking is required? o This is an evaluate question. You will need to make a judgement as to which of the choices is the best possible answer. You are being presented with four experimental designs. You need to examine all four choices and pick the best experimental design. Gather Content • What do you already know about mating behaviors in male birds? What other information is related to the question? o You need to use what you know about the interaction of instinct and learning in songbirds to help you pick the correct experimental design. Male songbirds have an innate ability to produce songs, but they cannot sing their mating song well without listening to the correct song. All of the approaches are valid but only one of them is going to be helpful in making the young males perfect their mating songs. Choose Answer • Given what you now know, what information is most likely to produce the correct answer? o Male birds usually need to hear their songs during development, and so it will be important to expose the males to a mentor male bird singing their species' song during the critical period.. Reflect on Process • Did your problem-solving process lead you to the correct answer? If not, where did the process break down or lead you astray? How can Version 1
45
you revise your approach to produce a more desirable result? o Were you able to select the correct answer? The key is to understand that that although male song birds have some innate ability to perform the mating song, they need to be exposed to the song during the critical period in order for them to perfect it. 57) D
Version 1
46
Clarify Question • What is the key concept addressed by the question? o The goal of the question is to determine when the critical period for imprinting occurs. • What type of thinking is required? o This is an analyze question. You need to break each of the choices down into its components to see if it matches the data from the table. The data from the table shows the age of the chick and when imprinting was observed. Gather Content • What do you already know about imprinting? What other information is related to the question? o Imprinting occurs during an early critical period, in the life of an animal. If the imprinting does not occur within the critical period then the animals will never be able to imprint on its mother. Paring a chick with a foster mother between 0–3 days after hatching led to successful imprinting whereas chicks that were paired with foster mothers 8–11 days after hatching did not successfully imprint on the foster mother. Choose Answer • Given what you now know, what information is most likely to produce the correct answer? o The data show that the critical period ends somewhere between days 3–8, the data does not give any information about when the imprint period begins. It may begin within minutes after hatching or later during the first day. Reflect on Process
Version 1
47
• Did your problem-solving process lead you to the correct answer? If not, where did the process break down or lead you astray? How can you revise your approach to produce a more desirable result? o Did you get the questions right? The key to this question is to interpret the data presented on the table and see what choice best matches the data. 58) B
Version 1
48
Clarify Question • What is the key concept addressed by the question? o Understanding the mating behaviors of prairie and montane voles and how they relate to different species. • What type of thinking is required? o This is an analyze question. You need to break down the four statements into the components and determine which one is correct. Gather Content • What do you already know about the hormones involved in mating in the species of voles? What other information is related to the question? o This question comes from the section of the textbook that discusses the role of genes in pair bonding. You need to identify the functions of the hormones and their receptors. Do vasopressin and oxytocin receptors increase monogamy or promiscuity? Choose Answer • Given what you now know, what information is most likely to produce the correct answer? o Vasopressin and oxytocin receptors are associated with monogamy. Reflect on Process • Did your problem-solving process lead you to the correct answer? If not, where did the process break down or lead you astray? How can you revise your approach to produce a more desirable result? o Did you come up with the correct answer? You had to look over all the choice and pick out all of the ones that demonstrated the link between the presence of hormone receptors and the associated Version 1
49
behaviors. 59) D
Version 1
50
Clarify Question • What is the key concept addressed by the question? o The sensory structures that would help the beetle find the most advantageous spot to lay its eggs. • What type of thinking is required? o This is an apply question. You need to look at the four choices, break them down into their constituents, and see which one would be the most useful for seeking out freshly burned wood. Gather Content • What do you already know about sensory systems involved with behavior? What other information is related to the question? o This question comes from the section of the textbook that deals with the structure of neural circuits and how they are linked to animal behavior. The choices deal with the speed of signals, the processing of signals, and the sensory systems. Choose Answer • Given what you now know, what information is most likely to produce the correct answer? o Sensory structures are what will allow the beetle to detect burned wood. The beetle must be able to detect the burned wood before laying eggs on it. Reflect on Process • Did your problem-solving process lead you to the correct answer? If not, where did the process break down or lead you astray? How can you revise your approach to produce a more desirable result? o Were you able to select the correct answer choice? You had to look over all the choices and pick out all of the ones that are involved in the Version 1
51
detection of burned wood. 60) D
Version 1
52
Clarify Question • What is the key concept addressed by the question? o The question asks about modes of communication and which works best over distances in a dark, dense forest. • What type of thinking is required? o This is an analyze question. You need to break down each of the answer choices into its components and see which mode of communication would be the most effective. Gather Content • What do you already know about the different modes of communication? What other information is related to the question? o The animal is trying to communicate over long distance in a dark, dense forest. One fair assumption is that the forest is full of trees and other types of vegetation. Given the limitations of such a place, what is best way to communicate? Due to the density of the vegetation and the lack of light, animals are probably unlikely to be able to see each other over long distances. Watching a dance from a long distance away is also unlikely to be possible. Pheromones can travel through air, but all of the vegetation in a forest would reduce the distance that a chemical could travel. Light flashes would also be hard to see over long distance given all of the vegetation. This leaves sound. Sound waves can travel over long distances and provided that they are distinct from other sounds, they can convey information. Choose Answer • Given what you now know, what information is most likely to produce the correct answer? o Given the choices, sound based communication is the best option. Version 1
53
Reflect on Process • Did your problem-solving process lead you to the correct answer? If not, where did the process break down or lead you astray? How can you revise your approach to produce a more desirable result? o Did you get it right? You needed to consider the limitations of the environment and what constraints it would place on effective long distance communication. 61) C
Version 1
54
Clarify Question • What is the key concept addressed by the question? o Understanding what can explain the feeding behavior of hummingbirds, that are migratory birds. • What type of thinking is required? o In this question, you are being asked to analyze the situation of the disappearing hummingbirds and come up with a recommendation for your friend. Gather Content • What do you already know about feeding behaviors in birds? What other information is related to the question? o This question comes from the section of textbook that focuses on migratory behavior. Hummingbirds do migrate during the year. Only one of the choices address migrations, the other options might be able to attract hummingbirds to a feeder if the hummingbirds remain in the area. None of the other strategies would work if the hummingbirds have already migrated away. Choose Answer • Given what you now know, what information is most likely to produce the correct answer? o Migrations are long distance two-way movements. Typical bird migrations in North America involve flying south during the autumn and then return north in the spring. If a bunch of hummingbirds visit the feeders in the spring and then disappear, then they will probably move back through the area in the fall during the reverse leg of the migration Reflect on Process
Version 1
55
• Did your problem-solving process lead you to the correct answer? If not, where did the process break down or lead you astray? How can you revise your approach to produce a more desirable result? o Were you able to pick the correct choice? The key to this apply question is to understand that migrations are two-way, long distance mass movements of species. 62) D
Version 1
56
Clarify Question • What is the key concept addressed by the question? o Comparing altruism in humans versus vampire bats. • What type of thinking is required? o This is an analyze question; you will need to break the answers down into their components. You are being asked to examine the choices and pick the one that shows a commonality between humans and vampire bats. Gather Content • What do you already know about altruism? What other information is related to the question? o Humans and vampire bats are both mammals but are very different from each other. Both do live in large groups and both practice altruistic behaviors. Altruistic behaviors are behaviors that increase another individual’s fitness at the cost of one's own fitness. Vampire bats go out at night and feed on blood, they have been observed to feed other bats that are not related to them. They will feed other bats, but the behavior is a give and take. Bats only feed other bats that have fed them. Haplodiploidy is not relevant here; humans and bats are diploid. Chiroptera is the group that contains the bats, humans are primates. The feeding is not based on kinship; it is based on concept of reciprocity. If you scratch my back, I’ll scratch yours. Choose Answer • Given what you now know, what information is most likely to produce the correct answer? o Humans and bats show reciprocal altruism. Reflect on Process Version 1
57
• Did your problem-solving process lead you to the correct answer? If not, where did the process break down or lead you astray? How can you revise your approach to produce a more desirable result? o Were you able to select the correct answer? The key here is to recognize the different explanations for altruistic behavior. 63) D
Version 1
58
Clarify Question • What is the key concept addressed by the question? o This question is about family relationships and kin selection. • What type of thinking is required? o This is an apply question. You are being presented with a situation. You need to apply what you know about inclusive fitness to determine which of the choices is correct. Gather Content • What do you already know about kin selection? What other information is related to the question? o This question comes from the section of the textbook that discusses kin selection and inclusive fitness. Inclusive fitness is the concept that copies of your genes reside with yourself and your close relatives. A full sibling has on average 50% of the genes that you have. This is why you look like your brother or sister. If you were killed before reproducing, then 0% of your genes would be passed on. However, if your death saved the lives of two brothers and they reproduced, then 100% of your genes would be passed on (50% from each brother). A niece or nephew is a mixture of your full sibling (50% of your genes) and an unrelated individual (0% of your genes). That means that a niece or nephew shares 25% of your genes. A son or daughter is a mixture of your genes (100%) and an unrelated individual (0%); thus a son or daughter shares 50% of your genes. Uncles share 25% (half of the genes from one of your parents), grandchildren share 25% (half of your child’s half), and cousins share 12.5% (half of your aunt or uncle and an unrelated individual) Choose Answer Version 1
59
• Given what you now know, what information is most likely to produce the correct answer? o Knowing this, the question becomes a math problem 1 son or daughter = 50% 4 cousins = 50% 3 uncles = 75% 2 nephews (50%) plus 1 son or daughter (50%) = 100% 2 grandchildren = 50% Reflect on Process • Did your problem-solving process lead you to the correct answer? If not, where did the process break down or lead you astray? How can you revise your approach to produce a more desirable result? o Were you able to select the correct choice? You needed to take what you know about inclusive fitness and calculate the different percentages for each combination. The one that equals 100% accounts for all of your genes. 64) D
Version 1
60
Clarify Question • What is the key concept addressed by the question? o The question is asking about the waggle dance exhibited by bees. • What type of thinking is required? o This is an apply level question. You are being presented with a situation. Using what you know about the bee communication you can explain what information the bee is try to covey via the dance. Gather Content • What do you already know about the waggle dance in bees? What other information is related to the question? o This question comes from the section of the textbook that discusses the details of bee communication. The bee dance conveys the information about the source of the pollen relative to the position of the sun. If a bee dances along the vertical axis it means the flowers are towards the sun. Dances to the right signal a deflection to the right relative to the sun and dances to the left mean a left deflection. If the sun is due West and the flowers are due north, then what is difference in degrees from west to north? Choose Answer • Given what you now know, what information is most likely to produce the correct answer? o If you faced west, due north would be 90 degrees to your right. Reflect on Process • Did your problem-solving process lead you to the correct answer? If not, where did the process break down or lead you astray? How can you revise your approach to produce a more desirable result? Version 1
61
o Did you get the question right? You needed to take what you know about bee dances and apply it to this situation to come up with the answer. You also needed to figure out what information is relevant and what is not. 65) A 66) A 67) Section Break 67.1) [B, C, D, F]
Version 1
62
Clarify Question • What is the key concept addressed by the question? o Need to understand the process of designing an experiment and what variations the design can take without impacting the results. • What type of thinking is required? o This is an evaluate question. You are being presented with several potential justifications for an experimental design that does not perfectly replicate the natural situation. You need to judge whether each choice is or is not a valid justification. Gather Content • What do you already know about experimental design? What other information is related to the question? o This question comes from the section of the textbook that discusses behavioral ecology. There is more than one correct answer. This question is not directly about behavioral ecology, rather, it concerns experimental design. The ideal animal behavior experiment would occur in the wild and would replicate natural conditions as closely as possible. This is always the most desirable because the data the experiment generates would be much more valid. The question is asking you to pick out a valid reason why a researcher would conduct an experiment that doesn’t exactly replicate the natural situation (ex. using chicken eggs instead of sea gull eggs when testing egg predation rates). Choose Answer • Given what you now know, what information is most likely to produce the correct answer? o The only incorrect answer is the choice “because the scientist Version 1
63
doesn’t care about proper procedure”, this is clearly wrong. Getting an experiment to perfectly match natural conditions is always the best option, but sometimes that just isn’t possible. Reflect on Process • Did your problem-solving process lead you to the correct answer? If not, where did the process break down or lead you astray? How can you revise your approach to produce a more desirable result? o Were you able to select all of the correct justifications? The key to this evaluate question is to see that perfection in experimental design is always ideal but not always possible. 68) Section Break 68.1) [A, C, E, G, H]
Version 1
64
Clarify Question • What is the key concept addressed by the question? o The question is asking about the proximate basis of an unusual behavior in a species of moth. • What type of thinking is required? o This is an analyze question. You need to read the description of the moth behavior, break them down into their components, and figure out which of the choices would be involved in the proximate cause of behaviors. This is a check-all-that-apply question; there are multiple correct answers. Gather Content • What do you already know about proximate causes of a behavior? What other information is related to the question? o Proximate causes are physiological series of events that produce the particular behavior in question. Which of the choices are involved in the physiological causes of the moth’s diving behavior? Choose Answer • Given what you now know, what information is most likely to produce the correct answer? o Brain structure, hormones, physiology, sensory organs, and wing muscles are involved in producing the behavior. The other choices are unrelated or are involved in the ultimate causes of the behavior. Reflect on Process • Did your problem-solving process lead you to the correct answer? If not, where did the process break down or lead you astray? How can you revise your approach to produce a more desirable result? Version 1
65
o Were you able to determine the correct answer? You needed to consider the how each of the choices influences the diving behavior and determine if it is part of the proximate cause or the ultimate cause of the behavior. 68.2) [B, D, F]
Version 1
66
Clarify Question • What is the key concept addressed by the question? o The question asks about an ultimate cause of an unusual behavior in a species of moth. • What type of thinking is required? o This is an analyze question; You need to read the description of the moth behavior and figure out which of the choices would be involved in ultimate cause of behaviors. This is a check-all-thatapply question, there are multiple correct answers. Gather Content • What do you already know about ultimate causes of a behavior? What other information is related to the question? o Ultimate causes are the adaptive benefits that allowed natural selection of favor the particular behavior. Which of the choices are involved in the ultimate causes of the moth’s diving behavior? Choose Answer • Given what you now know, what information is most likely to produce the correct answer? o Brain structure, hormones, physiology, sensory organs, and wing muscles are involved in producing the behavior. Flying behavior in related species, mating behavior, and predation are all involved in the ultimate cause of the moth’s diving behavior. Reflect on Process • Did your problem-solving process lead you to the correct answer? If not, where did the process break down or lead you astray? How can you revise your approach to produce a more desirable result? Version 1
67
o Did you get the question right? You needed to consider the how each of the choices influences the diving behavior and determine if it is part of the proximate cause or the ultimate cause of the behavior.
Version 1
68
CHAPTER 38 TRUE/FALSE - Write 'T' if the statement is true and 'F' if the statement is false. 1) An r-selected species is more likely to become an invasive pest than a K-selected species. ⊚ true ⊚ false
CHECK ALL THE APPLY. Choose all options that best completes the statement or answers the question. 2) Two populations have very similar birth rates and death rates, but the change in the growth of the populations is quite different. What other factors might account for the difference? (Check all that apply.) A) Age structure B) Primary food source C) Generation time D) Immigration E) Sex ratio
3)
Which of the following describe density-dependent factors? A) They act toregulate population growth. B) They are especially important in K-selected populations. C) They can affect growth rates but not population size. D) They can affect birth rates or deathrates. E) They do notinvolve biological interactions. F) One example would be intraspecific competition for resources.
4)
What are characteristics ofr-selected populations? (Check all that apply.)
Version 1
1
A) Early age of first reproduction B) Late age of first reproduction C) Small brood size D) Large brood size E) Little or no parental care F) Extensive parental care G) Short generation time H) Long generation time I) Type I survivorship curves J) Type III survivorship curves
5) Which of the following apply to the demography of human populations today? (Check all that apply.) A) Our ecologicalfootprint is decreasing. B) We are using distributed resources disproportionately. C) We have plenty of resources still untouched. D) The birth rate has remained unchanged over the past 300 years. E) The fraction of the world’s population that lives in industrialized countries is increasing.
6) Which of the following are environmental factors that determine where an organism can live? (Check all that apply.) A) Intraspecific competition B) Predation C) Prey availability D) Soil type E) Sunlight F) Temperature G) Water
7)
Which of the following describe source-sink metapopulations?
Version 1
2
A) Sink populationsnear source populations are less likely to go extinct. B) Sink populationswithout access to immigrants from source populations are less likely to goextinct. C) Source populationsusually occupy better habitats. D) Source populationsare less likely to go extinct. E) The emigration rate from sourcepopulations exceeds that from sink populations.
8) An underwater volcano has erupted, and as the lava cools a new island has formed. Which types of animal species are likely to be among the first to colonize the island? (Check all that apply) A) Annelids B) Bats C) Birds D) Insects E) Snails F) Snakes G) Tapirs H) Tortoises
9) A population of squirrels is in a phase of exponential growth. Which events would act to slow this growth? (Check all that apply) A) A forest fire that destroys habitat B) Logging of nut trees C) Growth of the hawk population D) Heavy rainfall, resulting in larger seed and nut harvest E) Increased population that is outstripping food supply F) Less competition from a diminished population of ground squirrels
10) A developing nation with a rapidly increasing population is reaching its carrying capacity.What change(s) could allow it to increase its carrying capacity? (Check all that apply)
Version 1
3
A) A cultural shift to eating less meat B) Education for women C) Greater yield from agricultural crops D) Increased availability of birth control E) Water treatment plants
MULTIPLE CHOICE - Choose the one alternative that best completes the statement or answers the question. 11) What age group on a population pyramid would you examine to predict the population in 25 years? A) 0–5 B) 10–15 C) 20–25 D) 25–30
12) Life history adaptations of __________ populations are characterized by an early age of first reproduction and short maturation time and life span. A) r-selected B) K-selected
13) Species that have a delayed reproductive stage, are competing for limited resources and have smaller numbers of slowly maturing large offspring show __________ adaptations. A) K-selected B) r-selected
14) The statistical study of populations including sex ratio, age structure, and predicting growth rates is called __________.
Version 1
4
A) ethology B) demography C) population genetics D) biometrics
15)
A group of population members all of the same age is called a __________. A) deme B) species C) cohort D) tribe
16) What is the life history adaptation called where organisms produce offspring several times over many seasons? A) Semelparity B) Iteroparity C) Biparity D) Polyparity E) Alloparity
17) Effects that are dependent on the size of the population and regulate the growth of populations are called __________ effects. A) K-related B) density-independent C) environmental resistance D) density-dependent E) demographic
Version 1
5
18) The trade-off between investments in current reproduction and in growth that promotes future reproduction is referred to as the total cost of what? A) Adaptation B) Selection C) Reproduction D) Alloctaion E) Fitness
19) A small group of mice are released on an island without mice but with abundant food for mice and no predators. Initially, the growth of the mouse population will be limited mainly by what? A) The carrying capacity B) The biotic potential C) Only density-dependent factors D) Only density-independent factors
20)
Demographic studies include all of the following except what? A) Age structure B) Growth rates C) Mortality and survivorship curves D) Sex ratio E) Measurements of interspecific competition
21) Patches of populations that undergo local periodic extinction and recolonization are called what?
Version 1
6
A) Randomly spaced populations B) Uniformly spaced populations C) Metapopulations D) Oversized populations E) Endangered populations
22) The biotic potential representing growth without limits at its maximal rate is indicated bywhat symbol? A) r i B) N C) K D) dN/dt E) N/K
23) In the sigmoid growth curve, the number of individuals at any one time is indicated bywhat symbol? A) r i B) N C) K D) dN/dt E) N/K
24) In the sigmoid growth curve, the carrying capacity of the environment is indicated by what symbol? A) r i B) N C) K D) dN/dt E) N/K
Version 1
7
25)
The range of a population A) is very fluid and changes frequently in a random fashion. B) is stable and almost never changes. C) only changes after a disaster has wiped out a former range. D) changes over time due to external events. E) only changes due to iteroparity.
26)
Which is an example of a density-independent factor on a population? A) Weather B) Nesting sites C) Intraspecific competition D) Amount of prey E) Predation
27)
What do we call populations of organisms that are usually near the carrying capacity? A) K-selected B) r-selected C) Predators D) Parasites E) Perennials
28)
Which of the following is an example of a species with r-selected adaptations? A) House fly B) Blue whale C) Gorilla D) Horse E) Cockroaches
Version 1
8
29)
Which is an example of a species with K-selected adaptations? A) Dandelion B) Fruit fly C) Herring D) Bobcat
30) A species colonizes an island, goes through a phase of exponential growth, and reaches the carrying capacity of the island. At what stage is the growth rate slowest? A) Initial colonization B) Exponential phase C) Approaching K D) After reaching K
31) In the logistic growth model, as the number of individuals in the population (N) approaches the carrying capacity (K), what happens to the intrinsic rate of growth (r i)? A) It will beaffected by an increased birth rate. B) It will beaffected by a decreased death rate from predation. C) It will beaffected by increased competition with other species. D) It will beaffected by increased competition within the species. E) It will notchange.
32) There are three aspects of entire populations that are important and often studied. Select the best choice from the ones listed.
Version 1
9
A) a population's range, the dispersal of individuals within the range, and the size that the population attains B) a population's range, the amount of food available within the range, and the size that the population attains C) a population's range, the parental care received by each offspring within the population, and the size that the population attains D) a population's range, the size home range of an individual in the population, and the parental care expended for each offspring
33) Populations have three basic types of dispersal patterns—clumped, random, and uniform. What situation supportsrandom distributions? A) When individuals of the populations do not interact strongly with one another B) When individuals of the population are in competition for resources C) When there is aneven distribution of resources D) When individuals of the population display social interactions
34)
Which is anexample of aclumped distribution of a population?
A) A tree species releases a toxin to defend the soil around its roots, resulting in trees spaced evenly like a grid. B) Antelope travel in herds. C) Two male hummingbirds have a confrontation on the border between their territories. D) Multiple species join each other at the waterhole to drink.
35) Many times species are composed of networks of distinct populations called metapopulations. When do metapopulations occur?
Version 1
10
A) When apopulation is large and uniformly distributed B) When apopulation in poor habitat continually sends out dispersers to bolster populations in better habitats C) When a population in a better habitat does not send out colonizers into less suitable habitats D) When suitable habitat is patchily distributed and separated by areas of unsuitable habitat
36)
Which factor increases the likelihood of population extinction? A) Large population size B) Isolation of a population from density-independent effects C) Isolation of a population from sources of immigrants D) High resource availability E) High genetic diversity
37)
What are population pyramids used to show? A) Death rates B) Birth rates C) Competition D) Age composition of a population E) The carrying capacity
38) What kind of population spacing would you expect for a species that is strongly territorial? A) Uniform B) Clumped C) Random D) Uniform or clumped depending on whetherit is a source or sink metapopulation E) Uniform or clumped depending on the life history strategy of the species
Version 1
11
39) You construct a life table for a plant species and find that in all cases about the same proportion of the cohort survive to the beginning of the next time interval. What would you call this? A) A Type I survivorship curve B) A Type II survivorship curve C) A Type III survivorship curve D) A semelparous life history adaptation E) Populations regulated by density-independent events
40) Parental care of the young is usually associated with species with a type __________ survivorship curve. A) I B) II C) III D) I or II (depending on the sex ratio) E) I or III (depending on whether the reproductive strategy is iteroparous or semelparous)
41) Which one of the following expressions from the logistic equation (dN/dt) = rN ((K – N)/K) represents the proportion of unused resources remaining for use by the population? A) The carrying capacity ( K) B) The population size ( N) C) The biotic potential ( rN) D) ( K – N) /K E) The growth rate ( dN/dt)
42)
Under which condition will a population experience growth?
Version 1
12
A) When N equals K B) When N is less than K C) When dN/ dt equals zero D) When r equals zero E) When N equals zero
43) What is the most reasonable conclusion that can be made from data graphed in the figure?
Version 1
13
A) After high mortality early in life, survivorship becomes constant for the rest of life. B) Survivorship isconstant throughout life. C) After lowmortality early in life, mortality increases to a constant rate for the rest oflife. D) After following aconstant rate early inlife, mortality becomes low late in life.
44) Select the description of a population pyramid for a population that has the largest potential for exponential growth. A) Broad at the base, narrow at the top, with more females than males B) Broad at the base, narrow at the top, with more males than females C) Uniform at all age classes, with more females than males D) Broad at the top, narrow at the base, with more males than females E) Broad at the top, narrow at the base, with more females than males
45) You work for a successful company that sells maternity clothes. They have asked you to evaluate whether Australia or Singapore would be a better country to expand into with new stores. What keyword will help you find the data you need to make this evaluation? A) Population size B) Carrying capacity C) Metapopulation D) Demographics
46) Within a genus that normally produces large numbers of offspring, one particular species evolves maternal care behavior. What do you predict will happen to the survivorship curve? A) No change islikely. B) It may change fromtype I to type III. C) It may change fromtype III to type I. D) It may change fromtype I to type II.
Version 1
14
47) A graduate student is studying the feeding behavior of a small octopus that must be laboriously collected from the shores of a remote island. Her intern wishes to do experiments to look at the octopus’s mating behavior. Why might the graduate student veto this idea? A) The octopus has semelparous reproduction and will eat its mate. B) The octopus has iteroparous reproduction and will eat its mate. C) The octopus has semelparous reproduction and will die after laying one batch of eggs. D) The octopus has iteroparous reproduction and will die after laying one batch of eggs.
48)
What would be the best approach to curbing excessive human population growth? A) Lowering birth rates through education and family planning B) A return to historical health care practices C) Encouraging women to postpone children till after age 35 D) Embracing traditional agricultural practices
49) A group of international students is having a heated discussion in the dining hall. “Your country needs to get its population under control!” asserts a student from the United States. “Perhaps,” replies the Indian student, “but your country __________.” A) has the highest per capita resource consumption B) has its own overpopulation problem C) has a much higher birth rate D) has the lowest infant mortality rate E) has a declining ecological footprint
50) A small group of mice were released on an island. The island previously had no mice on it, but had abundant food and no predators. After several years of growth, the size of the new island population stabilizes. However, at this point a hurricane drastically reduces the population. How would you describe the situation?
Version 1
15
A) The bioticpotential of the population has been reduced. B) The new population size is a result of density-dependent regulation. C) The new population size is a result of density-independent regulation. D) The island mouse population can now act as a sink metapopulation. E) The island mouse population can now act as a source metapopulation.
51) During the late 1800s, cattle egrets arrived in South America from Africa and began to colonize. Their range has expanded dramatically over the years. Why were they able to do this? A) The habitats that they left in Africa were not suitable for any further colonization; thus, they were forced to emigrate. B) The habitats thatthey encountered in South America were suitable to them and unoccupied. C) There were abundant cattle for the birds to gather around in South America; furthermore, various animals that the egrets had lived around in Africa had become quite scarce because of overhunting and poaching, causing the birds to extend their range. D) The food resourcesin South America were far superior to those in Africa, allowing the egrets moreopportunity to grow and reproduce and ultimately expand their range.
52)
What environmental challenge is posed to young plants by an old growth forest? A) Low humidity B) High temperatures C) Low light D) Thick leaf litter
53) You are earning your PhD in marine biology by surveying microbial ocean communities eachyear.This year, you are shocked to discover that one of your study sites has become contaminated due to an industrial waste water pipe. This dumping is drastically raising the temperature and lowering the salinity of the water. What do you expect to find in your survey of the ecosystem, and what might you find in a survey five years from now?
Version 1
16
A) Many species will be lost from the study site this year; in five years even fewer species will remain. B) Some individuals will acquire genes to adapt to the environmental changes, and the community composition will end up largely the same this year and in five years. C) Many species will decrease or be lost from the study site this year; in five yearssome species may increase by acquiring adaptive mutations and some new species may appear. D) Many species will be lost from the study site this year; in five years the community will be composed of all new species.
54) Kettle ponds derive from a hole left by a retreating glacier that is subsequently filled with rainwater. A certain region has a series of disconnected kettle ponds, but occasionally heavy rains flood them and create streams. Which type ofanimal is most likely to be present in this area as a metapopulation? A) A fish B) An aquatic bird C) A mosquito D) A dragonfly
55) A farmer’s land includes a wilderness area that is home to an endangered species of bird. He wants to sell the land to a developer, and you have been hired to evaluate the environmental concerns. “This little patch of land is not crucial for the bird species!” the farmer says, “In fact, my land is surrounded by several other wilderness patches that also have that bird.” You reply: “It is possible that your land acts as the __________ and the other areas are __________ . If that's the case, development would be devastating to the population." A) sink; sources B) source; sinks C) population;metapopulation D) metapopulation;population
56)
Which organism is most likely to have a type III survivorship curve?
Version 1
17
A) An oak tree B) A domestic dog C) Corn in a farmer's field D) Paramecium
57) An unscrupulous dog breeder has been selecting for dogs that have very large litters, with the goal of maximizing profits. Why might this strategy be counterproductive? A) It will cost toomuch money to feed all those puppies. B) Litter size is notinfluenced by genetics. C) The puppies fromlarge litters are likely to be malnourished, and the mother will have fewresources to devote to future litters. D) Intrauterinecannibalism will reduce the number of puppies.
58) A rancher is suspected of shooting wolves near Yellowstone National Park, and you have been enlisted to go have a chat with him. The rancher doesn’t see any benefit to having the wolves there. What do you say? A) "If there areno predators like wolves, just one pair of jackrabbits can quickly create apopulation with explosive, logisticgrowth. Wolves help reduce thefecundity rate,keeping the population at a reasonable level." B) "If there areno predators like wolves, just one pair of jackrabbits can quickly create apopulation with explosive, exponential growth. Wolves help reduce the carryingcapacity, keeping the population at a reasonable level." C) "If there areno predators like wolves, just one pair of jackrabbits can quickly create ametapopulation. Wolves help reduce the biotic potential, keeping the populationat a reasonable level." D) "If there areno predators like wolves, just one pair of jackrabbits can quickly create apopulation with explosive, density-dependent growth. Wolves help reduce the agestructure, keeping the population at a reasonable level."
Version 1
18
59) You are working in an aquarium that has been breeding an endangered species of fish for return into the wild. You have a batch of progeny from the first generation of breeding. The staff are discussing whether to return those fish to an area that has been completely depleted or add them to a partially depleted population. Considering the Allee effect, what do you say? A) “They may survive better in a large school, and will have an easier timefinding mates.” B) "They may dobetter in the empty area without any competition for food." C) "They may do better in the empty area without any competition for territory." D) "The genetic diversity may declineif they join the partially depleted population."
60) This summer, you are returning to the research station in Costa Rica to follow up on the population of butterflies you have been studying. You are disappointed to find that there are fewer this year than last. In fact, at dinner your friends studying frogs and birds are complaining about the same thing. What type of influence do you suspect? A) A density-independent effect like an introduced predator B) A density-dependent effect like environmental disruption C) A density-independent effect like environmental disruption D) A density-dependent effect like an introduced predator
61)
Which factor allows bacteria species to adapt to environmental changes faster than fish? A) Shortergeneration time B) Smaller size C) Smaller nutrient needs D) Lack of a nuclear membrane
Version 1
19
62) The popular "locavore" movement encourages people to seek out food grown locally, rather than shipped thousands of miles from its site of production. Surprisingly, careful analysis reveals that local food production does little to reduce one's ecological footprint. More effective—but less popular—actions to reduce one's ecological footprint include reducing consumption of milk products and beef, and living in high-density urban housing. To perform an "ecological footprint" analysis, the costs of each choice must be carefully analyzed and comparedusing what units? A) Dollar percentage of GDP B) Damage from pollutants in parts per million C) Hours of labor D) Acres of land
63) Your friend is designing fantastic creatures for a new science fiction movie set on a distant planet. You point out that the ears on the creatures from the cold environment are longer than those from the warm environment, which is different than the general rule on Earth. “Oh, that’s all arbitrary,” he says, “Who knows what the rules would be on a different planet!” What is your best response? A) “Yes, that's true—the features of animals here are mostly a reflection of arbitrary outcomes of their genetic history.” B) “Short ears helpanimals conserve heat, and long ears help animals disperse heat due to laws ofphysics that are likely to be the same even on another planet.” C) "If the animals on another planet were mammals, then the rule is likely to apply since mammalian ears can develop only in certain ways." D) "The gene for ear development is probably linked to genes on the same chromosome that regulate temperature. So you're right—that probably wouldn't be the case for independently evolved animals."
64) When a particular species mates, often the female eats the male. She then expends all her energy producing and defending her large clutch of eggs. When they hatch, she dies. Which reproductive strategy is this species using?
Version 1
20
A) Iteroparity B) Semelparity C) Annoparity
SECTION BREAK. Answer all the part questions. 65) Feedback:
65.1)
Version 1
Which of the following statements about the graph is true?
21
A) Oysters live longer than Hydra. B) Hydra and humans have parallel life spans. C) Humans and oysters have similar life spans. D) Humans have lowmortality rates early in life. E) Oysters have highmortality rates late in life.
65.2)
What often describes organisms with a type III survivorship curve?
A) K-selected B) r-selected C) Iteroparous D) At their carrying capacity E) Subject to low predation rates
65.3)
What organism would be most likely to exhibit a type III survivorship curve?
A) Whale B) Coconut palm C) Lion D) Gorilla E) Pine tree
Version 1
22
Answer Key Test name: Chap 38_3e 1) TRUE 2) [A, C, D, E] 3) [A, B, D, F] 4) [A, D, E, G, J] 5) [B, D] 6) [D, E, F, G] 7) [A, C, D, E] 8) [B, C, D]
Version 1
23
Clarify Question • What is the key concept addressed by the question? o The key concept is to identify the types of animals that would be able to populate a newly formed island faster. • What type of thinking is required? o This is an application question. You will need to apply your knowledge of population dynamics to this situation. This is a check-allthat-apply type of question; there is more than once correct answer. Gather Content • What do you already know about colonizing a new environment? What other information is related to the question? o This is a newly formed island that rose out of the ocean. It clearly was never connected to the mainland. Of all the choices presented, some will have a distinct advantage in terms of colonization speed. Choose Answer • Given what you now know, what information is most likely to produce the correct answer? o Eventually all of the types of organisms listed may colonize the island, but the ones with the ability to fly will probably be first. These species will have the easiest time crossing the open water. Reflect on Process • Did your problem-solving process lead you to the correct answer? If not, where did the process break down or lead you astray? How can you revise your approach to produce a more desirable result? o Were you to select all the correct answers? Animals that are able to fly would give them an advantage in populating a new and remote island. These situations actually happen, in 1883, the volcano Krakatoa Version 1
24
blew itself up. The entire island was blasted in the air. In the 1920s a new island broke the surface and started to grow. The island is called Anak Krakatoa (son of Krakatoa) and has mature forests on it. 9) [A, B, C, E]
Version 1
25
Clarify Question • What is the key concept addressed by the question? o You are being asked to determine which factors would check the growth of a squirrel population. • What type of thinking is required? o This is an analysis question; you will need to break the question down into its components in order to get to the answer. This is a check-allthat-apply question, there is more than one correct answer. Gather Content • What do you already know about factors that could slow down the growth of a squirrel population? What other information is related to the question? o Populations can grow exponentially when they are getting optimal amounts of resources and have very few forces acting to reduce the population size. In nature these conditions can persist for a short time, but many factors can prevent this kind of growth from lasting. Choose Answer • Given what you now know, what information is most likely to produce the correct answer? o Habitat loss, increased predation, loss of a major food source, and famine can all act to slow down population growth and reduce the population size. Reflect on Process • Did your problem-solving process lead you to the correct answer? If not, where did the process break down or lead you astray? How can you revise your approach to produce a more desirable result? o Were you able to select all the correct answers? The key was to break Version 1
26
down the choices and determine which ones would negatively affect the population growth rate. 10) [A, C, E]
Version 1
27
Clarify Question • What is the key concept addressed by the question? o You are being asked to determine some factors that could increase the carrying capacity for a human population in a developing country. • What type of thinking is required? o This is an analysis question; you will need to break the question down into its components in order to get to the answer. This is a check-allthat-apply question, there is more than one correct answer. Gather Content • What do you already know about the carrying capacity of a population? What other information is related to the question? o Carrying capacity is the size of the population that can be supported by the environment. All individuals in the population consume some amount of resources. Anything that increases the amount of resources available or reduces the amount of resources an individual would use will increase the carrying capacity. Choose Answer • Given what you now know, what information is most likely to produce the correct answer? o The carrying capacity is the maximum number of individuals that a given environment can support. Anything that increases the food or water resources available to support people would increase carrying capacity, such as improved crop yields, a shift to vegetarian diets (which require far less land per calorie of food produced), and water treatment plants to produce clean water. Reflect on Process • Did your problem-solving process lead you to the correct answer? If Version 1
28
not, where did the process break down or lead you astray? How can you revise your approach to produce a more desirable result? o Were you able to select all the correct answers? You needed to break down each of the choices and see if they could possibly increase the carrying capacity of the environment. Increasing the carrying capacity involves being able to support more individuals in the population, not limiting the size of the population. 11) A 12) A 13) A 14) B 15) C 16) B 17) D 18) C 19) B 20) E 21) C 22) A 23) B 24) C 25) D 26) A 27) A
Version 1
29
When resources are limited, the cost of reproduction often will be very high. Consequently, selection will favor individuals that can compete effectively and utilize resources efficiently. Such adaptations often come at the cost of lowered reproductive rates. Such populations are termed K-selected because they are adapted to thrive when the population is near its carrying capacity ( K). Examples of K-selected species include coconut palms, whooping cranes, whales, and humans. By contrast, in populations far below the carrying capacity, resources may be abundant. Costs of reproduction are low, and selection favors those individuals that can produce the maximum number of offspring. Selection here favors individuals with the highest reproductive rates; such populations are termed r-selected. Examples of organisms displaying r-selected life history adaptations include dandelions, aphids, mice, and cockroaches. Most natural populations show life history adaptations that exist along a continuum ranging from completely r-selected traits to completely Kselected traits. 28) A 29) D 30) D 31) E 32) A 33) A 34) B Social interactions between members of a species can lead to clumped distributions. Many species live and move around in large groups (such as flocks, herds, or prides). These groupings can provide many advantages, including defense against predators, decreased energy cost of moving through air and water, and access to the knowledge of all group members.
Version 1
30
35) D 36) C 37) D 38) A 39) B 40) A 41) D 42) B 43) C 44) A 45) D 46) C 47) C 48) A 49) A 50) C
Version 1
31
Clarify Question • What is the key concept addressed by the question? o The key concept is the impact of a natural event on a population of mice. • What type of thinking is required? o This is an analyze question; you will need to break the question down into its components to determine the answer. You will need to determine how a hurricane effects a population of mice on an island. Gather Content • What do you already know about natural event affecting a population? What other information is related to the question? o Metapopulations are a group of small semi-isolated populations spread across an area. Density-dependent effects are changed by the population size whereas density-independent processes are not affected by the population size. Choose Answer • Given what you now know, what information is most likely to produce the correct answer? o The hurricane is independent of the size of the mouse population; it is an example of a density-independent process. Reflect on Process • Did your problem-solving process lead you to the correct answer? If not, where did the process break down or lead you astray? How can you revise your approach to produce a more desirable result? o You need to break down the question into all its components to determine the correct answer. Were you able to analyze each situation and determine what type of impact the hurricane had on the mouse Version 1
32
population, knowing that the population size had no influence on the results of this natural disaster? 51) B
Version 1
33
Clarify Question • What is the key concept addressed by the question? o You will need to figure out how egrets that were adapted to life in Africa were able to successfully colonize South America. • What type of thinking is required? o This is an analyze question; you will need to break the question down into its components to determine the answer. Gather Content • What do you already know about what impacts the colonization of a new habitat? What other information is related to the question? o If a species is going to successfully colonize a new area, it must be able to survive and reproduce in the new area. If a species is adapted to survive in its ancestral area, what must be true about the new area that is being colonized? Choose Answer • Given what you now know, what information is most likely to produce the correct answer? o The range of a population may expand either because environmental changes improve the suitability of a new region or because a chance event allows a species to cross a barrier into a new suitable region. The relative quality of the old range has no bearing on whether the population will expand into the new region. In the example of the cattle egret, some chance crossing of the Atlantic allowed the species to arrive in South America in the 1880s, and the population has gradually expanded both northward and southward into all suitable regions. Reflect on Process • Did your problem-solving process lead you to the correct answer? If Version 1
34
not, where did the process break down or lead you astray? How can you revise your approach to produce a more desirable result? o Were you able to analyze each scenario in terms of how it impacts the ability of the egret population to expand its range and what factors would be important and what factors wouldn't provide any advantages? 52) C
Version 1
35
Clarify Question • What is the key concept addressed by the question? o The key concept is about identifying challenges a young plant would face in a mature developed forest. • What type of thinking is required? o This is an analyze question; you will need to break the question down into its components to determine the answer. Gather Content • What do you already know about the conditions that exist in a mature forest environment? What other information is related to the question? o A young plant needs light, nutrients, and water in order to grow. In an old-growth forest with mature trees there will be plenty of water and nutrients. The abundance of tall trees presents a problem for the plant. The mature trees, are full of leaves and absorb almost all of the available sunlight. Without enough light the young plants will not be able to do photosynthesis. Choose Answer • Given what you now know, what information is most likely to produce the correct answer? o Light is the biggest problem for the young plant. The forest canopy absorbs 95-plus percernt of the light, that is why it is so dark in a mature forest. Reflect on Process • Did your problem-solving process lead you to the correct answer? If not, where did the process break down or lead you astray? How can Version 1
36
you revise your approach to produce a more desirable result? o Were you able to make the correct choice? You needed to break down the question and figure out which components of the mature forest habitat would provide the biggest challenge for a young plant. 53) C
Version 1
37
Clarify Question • What is the key concept addressed by the question? o The key concepts is predicting how an environment will be affected by pollution. • What type of thinking is required? o This is an analyze level question; you will need to break down the question into its components to get to the answer. Gather Content • What do you already know about how an ecosystem reacts to pollution? What other information is related to the question? o The environment has experienced a drastic change. Initially, most species will not be adapted to this different environment, but if the change is persistent over a long span of time, what will happen? There will be a new set of selection pressures on the community. Choose Answer • Given what you now know, what information is most likely to produce the correct answer? o There will be aninitial decrease in some species and others will go locally extinct. However,some species may acquire mutations that allow them to adapt. and otherspecies may be better suited to live in the new environment. Some species may have been held to low levels because of ecological interaction with other species, but with those competing species gone, their population may begin to expand. There may also be new species that immigrate and can occupy empty ecological niches. Reflect on Process • Did your problem-solving process lead you to the correct answer? If Version 1
38
not, where did the process break down or lead you astray? How can you revise your approach to produce a more desirable result? o Were you able to make the correct choice? Changes to an environment are often initially detrimental but given the short generations of microbial species, they can quickly respond to new sets of selection pressures. 54) A
Version 1
39
Clarify Question • What is the key concept addressed by the question? o The key concept is to identify the type of organism that will form a metapopulation under these conditions. • What type of thinking is required? o This is an analyze question; you will need to break down the question into its components to get to the answer. Gather Content • What do you already know about metapopulations? What other information is related to the question? o A metapopulation is a series of small populations that on their own would not be large enough to form a stable single population. The series of populations are intermittently connected to each other, and the whole group of subpopulations forms a metapopulation. Choose Answer • Given what you now know, what information is most likely to produce the correct answer? o Birds, mosquitos, and dragonflies can all freely move across the area. They would not be considered a metapopulation because they are not confined to separate subpopulations. Small populations of fish in the kettle ponds would be a metapopulation. The populations would be separate for most of the year, only joining when floods connected the separate kettle ponds. Reflect on Process • Did your problem-solving process lead you to the correct answer? If not, where did the process break down or lead you astray? How can you revise your approach to produce a more desirable result? Version 1
40
o Were you able to make the correct choice? You needed to break the question down and see which of the answer choices matched the definition of a metapopulation. 55) B
Version 1
41
Clarify Question • What is the key concept addressed by the question? o The key concept is to identify how the different wilderness patches relate to each other in affecting this endangered species of birds. • What type of thinking is required? o This is an evaluate question; you will need to make a judgment between all of the possible answers choices. There are two terms that you must select to make the statement correct. Gather Content • What do you already know about metapopulations? What other information is related to the question? o A metapopulation is a series of small populations that on their own would not be large enough to form a stable single population. The series of populations are intermittently connected to each other and the whole group of subpopulations forms a metapopulation. In a metapopulation, the sources are the larger more stable populations and the sinks are the small populations that are not sustainable. The sink areas are constantly being replenished by the source areas. Choose Answer • Given what you now know, what information is most likely to produce the correct answer? o If the farmer’s land were the source area for the metapopulation and the other areas were the sinks, then if the farmer sold his land to the developer, the source population would be destroyed. If that happened, it would only be a matter of time before the smaller sink populations became depleted. Reflect on Process Version 1
42
• Did your problem-solving process lead you to the correct answer? If not, where did the process break down or lead you astray? How can you revise your approach to produce a more desirable result? o Were you able to make the correct choice? You needed to weigh each of the choices and decide which one made the statement about the farmer’s land correct. Metapopulations are dynamic and changing populations but without the source subpopulations the meta population cannot persist. 56) A
Version 1
43
Clarify Question • What is the key concept addressed by the question? o Identify an organism that exhibits the characteristics of a type III survivorship curve. • What type of thinking is required? o This is an analysis question; you will need to break the question down into its components in order to get to the answer. You will need to figure out which species has a type III survivorship curve. Gather Content • What do you already know about the characteristics of a type III survivorship curve? What other information is related to the question? o Paramecia reproduce by mitosis, which only produces two new paramecia. The dog and the corn produce a smaller number of offspring. Choose Answer • Given what you now know, what information is most likely to produce the correct answer? o One single oak tree produces thousands of acorns, sometimes up to 10,000 acorns if it is a particularly good year. There is a huge attrition rate; only a tiny fraction of the acorns successfully germinate and only a tiny fraction of the young trees survive to be large mature oaks. The oaks obviously follow at type II curve with very high juvenile mortality. Reflect on Process • Did your problem-solving process lead you to the correct answer? If not, where did the process break down or lead you astray? How can you revise your approach to produce a more desirable result? o Were you able to select the correct answer? You needed to to break Version 1
44
down each choice and figure out what would be the reproductive strategy for each species, thereby being able to identify the one that exhibits a type III curve. 57) C
Version 1
45
Clarify Question • What is the key concept addressed by the question? o In this question you will need to figure out why produce a large litter of puppies may be counterproductive to the goal of having lots of puppies to sell. • What type of thinking is required? o This is an analysis question; you will need to break the question down into its components in order to get to the answer. Gather Content • What do you already know about the risks versus benefits of producing a large little of puppies? What other information is related to the question? o What would happen if the dog breeder only wanted huge litters? Dogs are iteroparous, meaning that they can breed multiple times during their lifetime. If they produce one huge litter of puppies, those puppies may be of low quality and it might compromise the mother's future breeding potential. Choose Answer • Given what you now know, what information is most likely to produce the correct answer? o Instead of producing one large litter of low-quality puppies, the strategy of procuring fewer higher-quality litters over a longer period of time would probably produce the highest number of good-quality puppies. Reflect on Process • Did your problem-solving process lead you to the correct answer? If Version 1
46
not, where did the process break down or lead you astray? How can you revise your approach to produce a more desirable result? o Were you to select all the correct answers? The key here was to break down the question and see the trade-off between quality and quantity of offspring. 58) B
Version 1
47
Clarify Question • What is the key concept addressed by the question? o In this question you are being asked to explain the benefits of having wolves. • What type of thinking is required? o This is an analysis question; you will need to break the question down into its components in order to get to the answer. Gather Content • What do you already know about the benefits of wolves in a community? What other information is related to the question? o When breaking down the answer choices, the difference between all the choices is exactly what effect the wolves are having on the rabbits. Do the wolves reduce the fecundity rate, carrying capacity, biotic potential, or the age structure? Wolves eat rabbits, but this does not reduce the number of offspring a rabbit could potentially have. Wolves are most likely going to eat younger rabbits who are less experienced at avoiding wolves, thus they will not reduce the age structure of the population. Choose Answer • Given what you now know, what information is most likely to produce the correct answer? o The answer is that wolves reduce the carrying capacity of the rabbit. Without wolves acting as a predator, the population size of the rabbit would be higher. The wolves act as a break on the growth of rabbits. There are many other factors that play a role in determining the carrying capacity for the rabbits. The wolves are just one of many factors that can act to reduce it. Version 1
48
Reflect on Process • Did your problem-solving process lead you to the correct answer? If not, where did the process break down or lead you astray? How can you revise your approach to produce a more desirable result? o Were you able to select the correct answer? The key was to break down the choices and see what effect was the best choice. In the case, the wolves serve to control the population size of another animal that could be more detrimental to the rancher. 59) A
Version 1
49
Clarify Question • What is the key concept addressed by the question? o You are being asked to consider how the Allee effect would modify your thinking when it comes to introducing an endangered fish into the wild. • What type of thinking is required? o This is an analysis question; you will need to break the question down into its components in order to get to the answer. . Gather Content • What do you already know about the Allee effect? What other information is related to the question? o The Allee effect is a correlation between individual fitness and the population size. As the population size increases, the individual fitness of each individual also increases. Living in a larger groupprovides higher genetic diversity, more opportunities for finding mates and a decreasedrisk of predation fro a given individual. Choose Answer • Given what you now know, what information is most likely to produce the correct answer? o Considering the Allee effect, introducing the fish into an existing school would give them the best chance of surviving and finding mates. Reflect on Process • Did your problem-solving process lead you to the correct answer? If not, where did the process break down or lead you astray? How can you revise your approach to produce a more desirable result? o Were you able to select the correct answer? You needed to break Version 1
50
down the answer choices and determine which one would give the young fish the best chance of surviving. 60) C
Version 1
51
Clarify Question • What is the key concept addressed by the question? o You are being asked to explain the cause of a large drop in populations of several unrelated species. • What type of thinking is required? o This is an analysis question; you will need to break the question down into its components in order to get to the answer. Gather Content • What do you already know about factors that affect many different populations of organisms? What other information is related to the question? o Density-dependent effects change as the population size changes. Diseases are a good example. A disease will have an easier time spreading in a large population especially if the individuals live close together. Density-independent effects are not affected by population size. Hurricanes and other natural disasters are good example of densityindependent effects. Choose Answer • Given what you now know, what information is most likely to produce the correct answer? o A density-dependent environmental disruption is the most likely explanation for a drop in the population size of several unrelated species in one particular area. Reflect on Process • Did your problem-solving process lead you to the correct answer? If not, where did the process break down or lead you astray? How can you revise your approach to produce a more desirable result? Version 1
52
o Were you to select all the correct answers? You needed to break down the answer choices to figure out which one would best explain the drop in populations. Also you needed to figure out that an environmental disturbance was a density-independent effect. 61) A
Version 1
53
Clarify Question • What is the key concept addressed by the question? o The key concept is to identify the characteristic that of bacteria that makes them better able to adjust to changing environmental conditions. • What type of thinking is required? o This is an application question; you will need to apply what you know about selection to the situation being presented. Gather Content • What do you already know about the characteristics of bacteria that might allow them to respond to changes in their environment? What other information is related to the question? o Selection acts to favor the best adapted individuals; these individuals reproduce and pass their genes to the next generation. Bacteria can reproduce every 20 minutes given enough resources. Fish cannot reproduce nearly as fast. Choose Answer • Given what you now know, what information is most likely to produce the correct answer? o The shorter generation time in bacteria is going to allow them faster adaptation to a changing environment. Reflect on Process • Did your problem-solving process lead you to the correct answer? If not, where did the process break down or lead you astray? How can you revise your approach to produce a more desirable result? o Were you able to identify the shorter generation time as being the factor that will give bacteria the advantage in adapting to a changing Version 1
54
environment? 62) D
Version 1
55
Clarify question: What is the key concept addressed by the question? The key concept is to analyze factors that contribute to an ecological footprint. What type of thinking is required? This is an analyze level question, you will need to break the question down into its components to figure out the answer. Gather Content: What do you already know about ecological footprints? What other information is related to the question? The key is to be able to compare different individuals and see how their choices affect their impact on the environment. In order to make meaningful comparisons all of an individual’s choices have to be converted into a common unit. Choose Answer: Given what you now know, what information is most likely to produce the correct answer? An ecological footprint is the amount of productive land required to support an individual through the course of his or her life. This figure estimates the acreage used for the production of food (both plant and animal), forest products, and housing, as well as the area of forest required to absorb carbon dioxide produced by the combustion of fossil fuels. Reflection on Process: Did your problem-solving process lead you to the correct answer? If Version 1
56
not, where did the process break down or lead you astray? How can you revise your approach to produce a more desirable result? Were you able to break down the question and come up with the parameter that relates to calculating one's ecological footprint to come up with the correct answer? 63) B
Version 1
57
Clarify question: What is the key concept addressed by the question? The question is about adaptations that arise in response to environmental conditions. What type of thinking is required? This is an analyze level question, you will need to look at all the choices and break them down into their components in order to figure out if there would be a similar pattern of ear size and climate on a different planet. Gather Content: What do you already know about the impact of environmental conditions on animal adaptations? What other information is related to the question? This question comes from the section of the text that deals with environmental changes and the adaptations that are needed to cope with the changes. Large ears present a large surface area that is sticking out of the head of an animal. This large area of exposed tissue would have a much larger interaction with the environment. Organisms are a reflection of their genes, but the environment acts as a force of selection on those genes. Genetically unrelated animals that live in similar environments often have very similar adaptations to survive in them. Choose Answer: Given what you now know, what information is most likely to produce the correct answer? This applies to all organisms, not just mammals. Genetic organization of organisms on different planets might be different from Earth’s organisms. The laws of physics are universal and apply across the Version 1
58
universe. Reflection on Process: Did your problem-solving process lead you to the correct answer? If not, where did the process break down or lead you astray? How can you revise your approach to produce a more desirable result? Answering this question correctly depended on being able to break down the answer choices and realize that the one thing Earth and another planet would have in common is the physics laws of nature. Heat flows from hot to cold regardless of what planet it is on. Were you able to figure out the correct answer? 64) B
Version 1
59
Clarify question: What is the key concept addressed by the question? The key concept is the reproductive strategy used by a particular species. What type of thinking is required? This question will require you to breakdown the question into all of its components to figure out the answer. Examine each part of this reproductive strategy and figure out which overall strategy is being used. Gather Content: What do you already know about different reproductive strategies? What other information is related to the question? The male is eaten after mating; obviously he only gets to mate once. The female literally kills herself in the process of producing and defending a huge clutch of eggs (eggs costs energy to produce). Clearly both parents are only going to reproduce once. Choose Answer: Given what you now know, what information is most likely to produce the correct answer? Semelparity is the reproductive strategy that involves putting all of one’s reproductive resources into one large reproductive event? Reflection on Process: Did your problem-solving process lead you to the correct answer? If not, where did the process break down or lead you astray? How can you revise your approach to produce a more desirable result? Version 1
60
To answer the question, it was necessary to break the question down to identify the component adaptations that affect the life history of the species. Were you able to identify the different adaptations? Did you know which adaptations were associated with the different life histories? 65) Section Break 65.1) D
Version 1
61
Clarify Question • What is the key concept addressed by the question? o The key concept is to analyze the graph and determine how the survivorship curves relate to each other. • What type of thinking is required? o This is an analyze question; you will need to analyze the graph to extract the data you need to answer the question. Gather Content • What do you already know about the characteristics of the three survivorship curves? What other information is related to the question? o How does the type III survivorship curve differ from the type I or the type II curves? Where does the maximal rate of mortality occur? Choose Answer • Given what you now know, what information is most likely to produce the correct answer? o In a type III curve, there is very high initial mortality followed by a sharp decrease in mortality rate once the organism survives to adulthood. The strategy of producing lots of offspring is the best strategy to use here, which is the r-selected strategy. Reflect on Process • Did your problem-solving process lead you to the correct answer? If not, where did the process break down or lead you astray? How can you revise your approach to produce a more desirable result? o You needed to extract the information from the graph and decide Version 1
62
which strategy was best. Were you able to determine from the graphs how each survivorship curve related to mortality rates and life spans for members of the population? 65.2) B Review the characteristicsof species that exhibit each of the three types of survivorship curves. 65.3) E
Version 1
63
Clarify Question • What is the key concept addressed by the question? o The key concept is to identify which organism exhibits the characteristics of a type III survivorship curve. • What type of thinking is required? o This is an analyze question; you will need to break down each of the answer choices and figure out which one most likely follows a type III survivorship curve. Gather Content • What do you already know about a type III survivorship curve? What other information is related to the question? o Due to the extremely high mortality rates for juveniles, investing lots of energy in each offspring is not a good use of energy for organisms that follow a type III survivorship curve. Choose Answer • Given what you now know, what information is most likely to produce the correct answer? o Whales, lions, and gorillas all invest a lot of energy into each of their offspring. A coconut palm produces a few very large seeds (coconuts) that are loaded with energy for the developing plant. Pine trees produce thousands of seeds at a time. Reflect on Process • Did your problem-solving process lead you to the correct answer? If not, where did the process break down or lead you astray? How can you revise your approach to produce a more desirable result? Version 1
64
o Were you able to make the correct choice? You needed to break down the type III survivorship curves and apply it to all of the choices to pick out which organism was the best example.
Version 1
65
CHAPTER 39 TRUE/FALSE - Write 'T' if the statement is true and 'F' if the statement is false. 1) In general, communities in early succession will be dominated by fast-growing species with r-selected life histories. ⊚ ⊚
true false
2) Resource partitioning would be most likely to occur between allopatric populations of species with similar ecological niches. ⊚ ⊚
3)
Solar energy is the same thing as heat energy. ⊚ ⊚
4)
true false
true false
Interspecific competition can affect the phenotypic characteristics of organisms. ⊚ ⊚
true false
5) Geographical allopatry between species is definitive evidence of interspecific competition. ⊚ ⊚
true false
6) When there are high levels of disturbance in a community, the number of K-selected species should increase along with the overall diversity of the community. ⊚ ⊚
Version 1
true false
1
7)
Mimicry is a form of camouflage. ⊚ ⊚
true false
CHECK ALL THE APPLY. Choose all options that best completes the statement or answers the question. 8) Which of the following are types of symbioses? Check all that apply. A) Batesian mimicry B) Commensalism C) Predation D) Parasitism E) Mutualism F) Müllerianmimicry
9) In studies of two species of barnacles in the marine intertidal zone, it was observed that Chthamalus can live in the upper intertidal zone and the lower intertidal zone if Semibalanus is absent, and Semibalanus can only live in the lower zone because it is more subject to dehydration. Based on this, which of the following statements are true? Check all that apply. A) The realized niches of the two species differ. B) The fundamental niche of Chthamalus is larger than its realized niche. C) The fundamental niche of Chthamalus is larger than the fundamental niche of Semibalanus. D) The fundamental and the realized niches of Chthamalus are the same. E) The fundamental and the realized niches of Semibalanus are the same.
10) Which of the following statements about the fundamental and realized niche are true? Check all that apply.
Version 1
2
A) A species' realized niche could be the same size as its fundamental niche. B) A species' realized niche could be smaller than its fundamental niche. C) A species' fundamental niche can be smaller than its realized niche. D) The extent of the realized niche is determined, in part, by interspecific competition. E) The extent of thefundamental niche is determined, in part, by conditions of the physicalenvironment.
11) Which of the following would be an expected long-term change during primary succession on glacial moraines? Check all that apply. A) Increase in species richness B) Decrease in soil depth C) Increase in soil salinity D) Increase in soil nitrogen E) Alder replaced by spruce
12) Which of the following statements about an ecosystem receiving a moderate level of continued disturbance are true? Check all that apply. A) A moderate level of continued disturbance predicts that most communities eventually reach an end-state or climax community. B) A moderate level of continued disturbance should lead to increases in species richness. C) A moderate level of continued disturbance tends to lead to the dominance of Kselected species. D) An example of a moderate level of continued disturbance would be a tree fall in a mature rain forest. E) A moderate level of continued disturbance should lead to all successional stages being present in the community at the same time.
13) Which of the following statements about symbiotic relationships are true? Check all that apply.
Version 1
3
A) Plant-pollinator interaction isclassified as a good example of mutualism. B) Both mutualisms and commensalisms result from the process of coevolution. C) It is sometimes difficult to determine if a specific relationship is mutualism, commensalism, or parasitism. D) Parasites can sometimes influence the behavior of their hosts to facilitate completion of their life cycles. E) Mutualists sometimes become parasites.
14)
Which of the following statements about mimicry are true? Check all that apply.
A) Generally, the number of mimics in a Müllerian mimicry system must be less than the number of models. B) Only Batesian mimicry depends on the ability of the predator to learn. C) In Müllerian mimicry, the participants are both mimics and models. D) In Batesian mimicry, the participants are either mimics or models. E) AMüllerian mimic is "a sheep in wolf's clothing."
15)
Which of the following statements about predation are true? Check all that apply. A) Predation reduces competition in a community. B) Removal of a predator often leads to increased community stability. C) Animal predation on plants is called herbivory. D) Predation can induce coevolutionary changes in prey species. E) Batesian mimicry is not a coevolutionary adaptation to predation.
16) In which of the following ways does species richness affect ecosystem function? Check all that apply. A) Low species richness can contribute to ecosystem stability. B) High species richness can increase ecosystem resistance to drought. C) Low species richness can make an ecosystem more susceptible to invasive species. D) High species richness reduces overall ecosystem productivity.
Version 1
4
17)
Which of the following are true about ecosystems? Check all that apply. A) They contain both living and nonliving components. B) There is a transfer of energy through the system and some energy is lost as heat
energy. C) Nutrients repeatedly cycle through the ecosystem. D) They represent the lowest level of biological organization. E) The ultimate source of energy for most ecosystems is the sun.
18)
Which of the following are true statements about the carbon cycle? Check all that apply.
A) Carbon and nitrogen fixation are both faciliated by microbes. B) Burning of gasoline is a contributor to the current imbalance in the carbon cycle. C) Replacing rainforest with farms decreases atmospheric CO 2 levels. D) In water, inorganic carbon exists in two different forms, both of which can be used by autotrophs. E) Carbon cycles locally.
19) Which of the following observations were an outcome of studies by David Tilman and coworkers at Cedar Creek, Minnesota on ecosystem stability? Check all that apply. A) Plots with more species showed less year-to-year variation in biomass. B) Species-rich plots were less affected by drought than species-poor plots. C) Over-yielding was observed in more recent work at Cedar Creek. D) Nitrogen uptake and total biomass produced were negatively correlated to species richness. E) More diverse plots were less susceptible to invasion by new species.
20) Which of the following examples would be undergoing secondary succession? Check all that apply.
Version 1
5
A) An abandoned potato farm B) A coastal grassland damaged by a hurricane C) An abandoned landfill D) Rocks in a national park covered by mosses and lichens E) A garden bed that has been weeded
21) Which of the following factors could be important in determining a species' fundamental niche? Check all that apply. A) Salinity B) Presence of a commensalist C) Soil type D) Presence of a predator/parasite E) Seasonal temperatures
22) Following their respective breeding seasons, several species of hummingbirds occur at the same locations in North America and several hummingbird flowers bloom simultaneously in these habitats. These flowers seem to have converged to a common morphology and color.Birds are most visually sensitive to the color red. Hummingbird flowers are pollinated by hummingbirds. Which of the following features of these flowers would indicate they have coevolved with hummingbirds? (Check all that apply) A) Long tubular flowers B) Flowers are colored violet and blue C) Blooming time of the flowers coincides with the arrival of the hummingbirds D) Flowers produce a high volume of nectar and sugar E) Flowers are low to the ground
23) Forests, like other ecological systems, are subject to the species succession process. Which of the following would be characteristics of pioneer plant species facilitating the early stages of forest succession? (Check all that apply)
Version 1
6
A) Slow-growing B) Pollinated by animals C) Seedling growth requires direct sunlight D) Seeds germinate in the shade E) Plants produce great quantity of seeds
24) Communities with a high level of bacterial diversity are found in many habitats including soil and aquatic environments. Recent studies indicate this diversity is crucial to the functioning of these communities. The same studies have also found that phage abundance plays a critical role in the maintenance of the bacterial diversity within these communities. What is the probable role of phages in this community? (Check all that apply.) A) they act as facilitators B) they have a mutualistic relationship with the bacteria C) they parasitize the bacteria D) they have a commensal relationship with the bacteria E) they compete with the bacteria for space and nutrients F) they keep any one species of bacteria from becoming too abundant in the community
25) Phosphorus deficiency often appears early in plant growth and results in stunting. Which of the following are reasons why a deficiency in phosphorus would cause growth abnormalities in plants? (Check all that apply.) A) Phosphorus is required for the storage and transfer of energy. B) Phosphorus is required for the production of amino acids that form proteins. C) Phosphorus is required for DNA replication. D) Phosphorus is necessary for building moleculesin the cell membrane. E) Without phosphorus, cellular replication is impeded.
MULTIPLE CHOICE - Choose the one alternative that best completes the statement or answers the question. 26) In a pond ecosystem, a crayfish feeds on three species of algae. It isin turn preyed upon by one species of fish and a heron. The heron also preys upon the fish. What is the minimum number of food chains in this food web?
Version 1
7
A) 2 B) 3 C) 4 D) 5 E) 6
27) What structures evolved in the Acacia in order to develop a mutualistic relationship with the Pseudomyrmex ant? A) Nectaries and Beltian bodies B) Nectaries and tough fibrous leaves C) Hollow thorns and a fibrous root system D) Secondary compounds and Beltian bodies E) Secondary compounds and tough fibrous leaves
Version 1
8
28) Two of Darwin's finches display a character displacement when they occur as sympatric species. Which of the statements correctly interprets the graph?
A) Both species have the same size beak on Santa Maria Island. B) Both species have the same size beaks on Daphne Major. C) Both species have the same size beaks on Los Hermanos Island. D) The two species have different beak sizes when they occur on the same island. E) The two species feed on different food resources; one feeds on seeds while the other feeds on insects.
29)
All organisms living together in a place are called a A) community. B) population. C) species. D) ecosystem.
Version 1
9
30)
Competition shapes communities because there are usually limited A) species. B) resources. C) interactions. D) nutrients.
31)
The attempt of two organisms trying to utilize the same resource is called A) synergy. B) parasitism. C) competition. D) interference.
32) Two species of warblers colonize the same island habitat. Both species nest in similar locations and eat the same type of prey. Eventually, competition will probably cause one species to A) migrate. B) grow exponentially. C) speciate. D) be eliminated.
33) Warning coloration, serves to protect an animal or plant by sending a warning signal to potential ________. A) competitors B) predators C) mates D) rivals
Version 1
10
34) An unprotected species resembling others that are distasteful is an example of which type of mimicry? A) Müllerian B) Competitive C) Batesian D) Aposematic
35) Two or more unrelated but protected species resembling each other and achieving a kind of group defense is an example of what type of mimicry? A) Batesian B) Disruptive C) Müllerian D) Cooperative
36) What type of relationship is it when two or more kinds of organisms live together in often elaborate and more or less permanent association? A) Symbiotic B) Parasitic C) Mutualistic D) Synergistic
37)
The way in which an organism utilizes its environment may be called A) resource partitioning. B) its habitat. C) competitive exclusion. D) intraspecific competition. E) its niche.
38)
A relationship in which both members benefit is called
Version 1
11
A) predation. B) parasitism. C) mutualism. D) commensalism. E) sympatric.
39)
Communities evolve to have greater biomass and species richness in a process called A) sympatric interactions. B) adaptive modifications. C) succession. D) symbiotic relationships. E) competitive exclusion.
40) its
The actual niche the organism is able to occupy in the presence of competitors is called
A) fundamental niche. B) realized niche. C) interference niche. D) intraspecific niche. E) exploitative niche.
41)
Competition between species is called __________ . A) interspecific B) exploitative C) interference D) fundamental E) intraspecific
Version 1
12
42) Resource partitioning can often be seen in similar species that occupy the same geographic area. These species avoid competition by living in different portions of the habitat or by utilizing different food or other resources. They are referred to as being __________ . A) sympatric B) allopatric C) competitive D) fundamental E) exploitative
43) Chemicals that play the dominant role in protecting plants from being eaten by herbivores or predators are called A) primary compounds. B) secondary compounds. C) poisons. D) oils. E) amino acids.
44) Cardiac glycosides, molecules causing a drastic effect on vertebrate heart function, are produced as defensive chemicals by plants belonging to A) the milkweed and dogbane families. B) the mustard family. C) the grassfamily. D) the poison ivy,oak, and sumac families. E) the bean family.
45)
Chemical defenses are found in all of the following except
Version 1
13
A) marine animals. B) insects. C) plants. D) snakes, spiders, and fishes. E) Batesian mimics.
46)
Insects that lay eggs on living hosts are called A) ectoparasites. B) endoparasites. C) brood parasites. D) parasitoids. E) predators.
47) Which statement is an accurate interpretation of the outcome in an ecosystem when a major predator is removed? A) The remaining predators fill the empty niche and diversity remains approximately stable. B) The diversity of the ecosystem increases because more species can thrive without the risk of predation. C) The diversity of the ecosystem decreases due to an increase in competition. D) The diversity of the ecosystem decreases because parasites become more of a problem. E) The diversity of the ecosystem decreases because new herbivores move in.
48) Alligators excavate holes in the bottom of bodies of water. During times of severe drought these holes act as refugia for various aquatic organisms that might perish if there were no water available. Thus, alligators in this system can be classified as
Version 1
14
A) an ecosystem engineer. B) a symbiotic species. C) a commensal species. D) an allopatric species. E) a sympatric species.
49) Succession happens because species in the habitat alter that habitat in ways that assist other species. There are three dynamic concepts that are of critical importance for succession to take place. They are A) establishment, facilitation, and inhibition. B) symbiotic relationships, facilitation, and aposematic coloration. C) establishment, coevolution, and competitive exclusion. D) competition, climax communities, and tolerance. E) competition, inhibition, and coevolution.
50) A species that plays a critical role in maintaining the structure of an ecological community and helps to determine the types and numbers of various other species in the community is known as A) a predator. B) a keystone species. C) the primary species. D) the dominant species. E) the alpha species.
51)
The difference fundemental niche and realized niche is that
Version 1
15
A) the fundamental niche is the actual niche that a species occupies while the realized niche is the potential area that the species is capable of inhabiting. B) the fundamental niche is the entire niche that a species is capable of using while the realized niche is just what is being occupied. C) the fundamental niche is smaller than the realized niche. D) the realized niche is theoretical while the fundamental niche is the entire niche that an organism can use.
52) During the mid 1930s, G. F. Gause studied competition among three species of Paramecium. Through his experiments, he formulated a principle of A) niche overlap. B) exploitative competition. C) metapopulation fluctuation. D) competitive exclusion. E) interspecific competition.
Version 1
16
53) Which statement correctly interprets the graph? (Note, Paramecium caudatum is in red and Paramecium aurelia is in mustard color.)
A) Paramecium caudatum drives Paramecium aurelia to near extinction. B) Paramecium aurelia drives Paramecium caudatum to near extinction. C) Paramecium caudatum and Paramecium aurelia are able to compete for the same resource, and their population densities are not affected. D) Paramecium caudatum and Paramecium aurelia are unable to exist, and both populations go extinct after 24 days.
54) Which of the following statements accurately reflects the differences between Batesian mimicry and Müllerian mimicry?
Version 1
17
A) In Batesian mimicry the model must behave differently than the mimic; in Müllerian mimicry they behave the same. B) In Batesian mimicry the model must be more dangerous than the mimic; in Müllerian mimicry they are both dangerous. C) Batesian mimicry does not differ from Müllerian mimicry. Two different scientists discovered these two types at the same time, and they disagreed on what to call it. D) Batesian mimicry differs from Müllerian mimicry in that they occur on different continents—Batesian on North America and Müllerian on Europe. E) Batesian mimicry involves invertebrates; Müllerian mimicry involves vertebrates.
55) The cyclic path that involves both biological and chemical processes of an ecosystem is called a __________. A) biogeochemical pathway B) complete pathway C) hydrological pathway D) complementary pathway
56) A permeable underground layer of rock, sand, and gravel saturated with groundwater is called a(n) ________. A) natural spring B) aquifer C) hot spring D) submergence
57) The upper, unconfined portion of the groundwater which flows into streams and is partly accessible to plants is called the ____. A) aquifer B) hydrozone C) water table D) watershed
Version 1
18
58) The combustion of fossil fuels is altering the balance between photosynthesis and __________. A) respiration B) sunlight C) growth D) decomposition
59)
Heterotrophs must obtain organic molecules that have been synthesized by A) biogeochemical cycles. B) bacteria. C) decomposers. D) autotrophs.
60)
A wolf that feeds on moose and elk would be classified within which trophic level? A) Decomposer B) Primary producer C) Primary carnivore D) Secondary carnivore E) Detritivore
61) The amount of sunlight that falls on plant leaves and is captured by photosynthesis is approximately __________. A) 1% B) 10% C) 25% D) 50%
Version 1
19
62) Over tropical rainforest areas, approximately 90% of the water that reaches the atmosphere comes from A) animal metabolism. B) animal perspiration. C) plant transpiration. D) plant absorption. E) plant metabolism.
63)
The carbon cycle is based on carbon dioxide. CO
2 makes up about
A) 30% of the atmosphere. B) 3% of the atmosphere. C) 0.3% of the atmosphere. D) 0.03% of the atmosphere. E) 0.003% of the atmosphere.
64) Even though nitrogen constitutes 78% of Earth's atmosphere, the amount available for living things nearly all comes from A) a few groups of nitrogen-fixing bacteria. B) the breakdown of minerals. C) a few genera of detritivores. D) the breakdown of dead plants. E) nitrogen-containing fertilizers.
65)
With almost no exceptions, the nitrogen fixers are
Version 1
20
A) beans. B) green plants. C) insects. D) mammals. E) prokaryotes.
66)
The form of nitrogen most likely to be absorbed from the soil by plants is A) nitrogenous wastes. B) nucleic acids. C) nitrogen gas. D) nitrous oxide gas. E) nitrite/nitrate ions.
67)
Which of the following biogeochemical cycles contain a mineral component? A) Phosphorus B) Carbon C) Nitrogen D) Oxygen E) Water
68)
The scarcity of which two elements limits plant growth? A) oxygen, nitrogen B) nitrogen, phosphorus C) oxygen, phosphorus D) carbon, nitrogen E) carbon, phosphorus
69)
Which of the following groups do not contain any species that are autotrophs?
Version 1
21
A) Bacteria B) Protista C) Plants D) Archaea E) Fungi
70)
Energy enters most ecosystems by the process of A) biosynthesis. B) respiration. C) biogeochemical weathering. D) photosynthesis. E) fermentation.
71)
The net weight of all of the organisms living in an ecosystem is its A) productivity. B) biomass. C) vital force. D) photosynthesis minus metabolism. E) combined output of metabolism and photosynthesis.
72) In an ecosystem, the amount of organic matter produced in a given time that is available for heterotrophs is known as A) daily yield. B) gross primary productivity. C) net primary productivity. D) effective vitality. E) net yield.
Version 1
22
73) A good average value for the amount of energy that reaches the next trophic level is about A) 20%. B) 10%. C) 5%. D) 2%. E) 1%.
74)
The rate of production of new biomass by heterotrophs is called A) secondary productivity. B) primary productivity. C) consumer productivity. D) gross productivity. E) net productivity.
75)
In an ecosystem A) both energy and nutrients cycle. B) energy flows and nutrients cycle. C) energy cycles and nutrients flow. D) both energy and nutrients flow. E) energy is contained with nutrient cycles.
76) Which one of the following ecosystems is most likely to show an inverted biomass pyramid but a normal energy pyramid?
Version 1
23
A) Grassland B) Open ocean C) Intertidal D) Tropical forest E) Desert
77) Which one of the following quantities best represents the respiratory heat lost by plants in an ecosystem? A) Gross primary productivity B) Net primary productivity C) Gross primary productivity – net primary productivity D) Gross primary productivity – standing crop biomass E) Gross primary productivity – secondary productivity
78) In a marine ecosystem with an inverted biomass pyramid, which trophic level should have the smallest number of organisms? A) Phytoplankton B) Zooplankton C) Primary carnivores (sardines) D) Secondary carnivores (tuna) E) Detritivores
79)
What feature do the carbon and nitrogen cycles have in common? A) A large atmospheric component B) Involvement in global climate change C) Often a limiting nutrient D) Mostly local cycling E) Many microbes involved in cycling
Version 1
24
80)
Which statement is the correct interpretation of the graph?
A) The population of Didinium goes extinct with the addition of Paramecium on day 4. B) The population of Didinium continues to increase and remains high even after the extinction of the Paramecium. C) The population of Paramecium goes extinct with the addition of Didinium on day 8. D) The population of Didinium increased but then went extinct after the population of Paramecium went extinct. E) The population of Didinium is able to increase at the expense of the Paramecium population. After a brief period both populations are able to coexist.
Version 1
25
81) Based on the following graph, what is the most likely relationship between Paramecium and Didinium?
A) Paramecium prey on Didinium. B) Paramecium have a commensal relationship with Didinium. C) Didinium and Paramecium have a mutualistic relationship. D) Didinium prey on Paramecium. E) Paramecium is a parasite that feeds on Didinium.
82)
Which of the following is an example of commensalism? A) A tapeworm living in the intestines of a mule deer B) Barnacles hitching a ride on the skin of a whale C) A female mosquito sucking blood from a musk oxen D) Wood-digesting flagellates living in the gut of termites E) Acacia trees and their ants
Version 1
26
83) In India, golden jackals that have been expelled from their pack have been known to attach themselves to a particular tiger, trailing it at a safe distance in order to feed on the big cat's kills.What type of relationship exists between the jackal and the tiger? A) Mutualism B) Predation C) Parasitism D) Competition E) Commensalism
84) You have been studying the relationship between jackals and tigers in India. Until recently, the only relationship you have observed is that jackals will associate themselves with a particular tiger and follow it at a safe distance in order to feed on the big cat's kills. However, recently you observeda jackal alerting a tiger to a potential kill with a loud cry. If you continue to observe this alerting behavior, you might change the categorization of the jackal/tiger relationship from __________ to __________. A) mutualism; parasitism B) mutualism; commensalism C) commensalism; mutualism D) parasitism; mutualism E) competition; predation
85) Cattle egrets follow African ungulates such as African buffalo around and catch insects that the buffalo flush out. Oxpeckers perch on the backs of buffalo and feed on ectoparasites that infest the buffalo. Which one of the following shows the ecological interaction that the buffalo has with each bird? A) Cattle egret: mutualism; oxpecker: commensalism B) Cattle egret: commensalism; oxpecker: mutualism C) Cattle egret: competition; oxpecker: mutualism D) Cattle egret: mutualism; oxpecker: mutualism E) Cattle egret: commensalism; oxpecker: commensalism
Version 1
27
86) Communities with a high level of bacterial diversity are found in many habitats including soil and aquatic environments. Recent studies indicate this diversity is crucial to the functioning of these communities. The same studies have also found that phage abundance plays a critical role in the maintenance of the bacterial diversity within these communities. Are phages a density-dependent or a density-independent regulating factor? A) Density-dependent B) Density-independent
87)
Which of the following is a producer? A) Pine tree B) Cricket C) Mushroom D) Rabbit E) Red-tailed hawk F) Earthworm
88) NPP is the difference between plant photosynthesis and respiration. Therefore, NPP quantifies the net absorption of what element(s) by living plants? A) Carbon B) Phosphorus C) Nitrogen D) Oxygen E) Nitrogen, phosphorus, and potassium
89)
A wolf that feeds on moose and elk would be classified within which trophic level? A) Herbivore B) Primary producer C) Primary carnivore D) Secondary carnivore E) Detritivore
Version 1
28
90) The honey possum (Tarsipes rostratus) is a tiny Australian marsupial that feeds entirely on nectar. Which of the following terms describes the role of a honey possum in its ecosystem? A) Herbivore B) Primary producer C) Primary carnivore D) Secondary carnivore E) Detritivore
91) Vampire squid (Vampyroteuthisinfernalis) are deep-sea cephalopods with features similar to both octopus and squid. Extensive video recordings show the diet of the vampire squid consists of items like crustacean remains and fecal pellets. Which of the following terms describes the position of the vampire squid in the deep-sea trophic pyramid? A) Herbivore B) Primary producer C) Primary carnivore D) Secondary carnivore E) Detritivore
92) Which one of the following factors would not influence the productivity of an ocean community? A) The presence of nitrogen-fixing cyanobacteria B) The season of the year C) Levels of autotrophs D) Fertilizer runoff E) The amount of rainfall
93) In the food chain below, what does the blue jay represent? crabgrass—cricket—garden spider—blue jay—red-tailed hawk
Version 1
29
A) Herbivore B) Primary producer C) Primary carnivore D) Secondary carnivore E) Detritivore
94) In the food chain below, what does the garden spider represent? crabgrass—cricket—garden spider—blue jay—red-tailed hawk A) Herbivore B) Primary producer C) Primary carnivore D) Secondary carnivore E) Detritivore
95) A Viceroy caterpillar feeds on a leaf. One day's feeding consumes 1000 calories worth of leaves. 50% of the calories are lost in its feces and 33% of the calories are used in cellular respiration.How many calories of the original intake are available to add to the caterpillar’s biomass each day? A) 830 calories B) 670 calories C) 500 calories D) 330 calories E) 170 calories
96) In a food chain with four trophic levels, what would be the energy in the top trophic level if the energy contained in the primary producer level is 40,000 kcal/m 2/yr? Assume a trophic level transfer efficiency of 10%.
Version 1
30
A) 4,000 kcal/m 2/yr B) 400 kcal/m 2/yr C) 40 kcal/m 2/yr D) 4 kcal/m 2/yr E) 0.4 kcal/m 2/yr
97) In a food chain consisting of four trophic levels, if the energy present in the top level is 48 kcal/m2/yr, how much energy do you expect in herbivores? (Use an energy transfer rate of 10%.)
A) 4.8 kcal/m 2/yr B) 48 kcal/m 2/yr C) 480 kcal/m2/yr D) 4,800 kcal/m2/yr E) 48,000 kcal/m2/yr
98) The tropics are known to have greater species diversity than the temperate zones. Which explanation for this phenomenon is the least convincing? A) Seasonal variability is less in the tropics. B) Niches are narrower in the tropics. C) Fewer predators in the tropics allow more prey species to flourish. D) Greater spatial heterogeneity in the tropics creates more niches. E) The tropics have greater plant production, which supports greater overall species richness.
99) In an aquatic ecosystem, a secondary carnivore is temporarily excluded and then, after a period of time, is returned. Which one of the following is not an expected top-down effect after it is returned?
Version 1
31
A) Primary carnivores become less abundant. B) Primary productivity increases. C) Algae decrease. D) Herbivores become more abundant.
100) Garlic mustard is an invasive plant species in North American temperate forests. Garlic mustard secretes an allelochemical that interferes with relationships between native tree roots and their mycorrhizal fungi. This would be an example of A) interference competition. B) exploitative competition.
Version 1
32
Answer Key Test name: Chap 39_3e 1) TRUE R-selected species are species that grow and disperse quickly and reproduce quickly and in large numbers. Therefore, they would be the first species to dominate an area in early succession (Section 39.4). 2) FALSE Resource partitioning (Section 39.1) occurs when species with similar niches inhabit the same area. Species that inhabit the same area are sympatric. 3) FALSE Both solar energy and heat are forms of energy, but solar energy is organized and can be captured and converted and used to drive cellular work. Heat energy is a by-product of energy conversions and is a disorganized form of energy that cannot be used to power cellular processes. 4) TRUE
Version 1
33
Clarify Question • What is the key concept addressed by the question? o The question requires an understanding of interspecific competition and how it might affect organisms involved in the competitive relationship. • What type of thinking is required? o This is an application question; you need to apply your knowledge of character displacement and interspecific competition to determine if the statement is true or false. Gather Content • What do you already know about interspecific competition? What other information is related to the question? o Character displacement occurs when a species’ range overlaps with a competitor. Often in areas of sympatry, competing species tend to diverge from each other in order to reduce level of competition. Choose Answer • Given what you now know, what information is most likely to produce the correct answer? o Where two species occur together, they often tend to exhibit character displacement. Character displacement is a result of natural selection. Since evolution acts on genes, it can alter the genotypes and therefore phenotypes of those species in competition Reflect on Process • Did your problem-solving process lead you to the correct answer? If not, where did the process break down or lead you astray? How can you revise your approach to produce a more desirable result? Version 1
34
o You needed to apply your knowledge of character displacement and interspecific competition to decide if the statement is true or false. Were you able to select the correct answer? 5) FALSE
Version 1
35
Clarify Question • What is the key concept addressed by the question? o The question requires an understanding of interspecific competition and what factors contribute to this type of competition. • What type of thinking is required? o This is an application question; you need to apply your knowledge of allopatric populations and interspecific competition to determine if the statement is true or false. Gather Content • What do you already know about interspecific competition and the contribution that geographic allopatry might have in establishing the competition? What other information is related to the question? o Allopatric populations are geographically isolated from each other. Is competition the only reason why 2 populations may live in completely different areas with no overlap? Choose Answer • Given what you now know, what information is most likely to produce the correct answer? o When two species do not overlap in space, this could be the result of ecological interactions, but it could also be the result of evolutionary forces. Reflect on Process • Did your problem-solving process lead you to the correct answer? If not, where did the process break down or lead you astray? How can you revise your approach to produce a more desirable result? o You need to apply you knowledge of allopatric populations in order to Version 1
36
figure out if the statement is true or false. Although geographic allopatric populations could be the result of competition, other explanations are also possible. 6) FALSE
Version 1
37
Clarify Question • What is the key concept addressed by the question? o This question requires an understanding of K-selected species and the conditions are associated with those species. • What type of thinking is required? o This is an application question; you need to apply your knowledge of disturbance and reproductive strategies in order to determine if the statement is true or false. Gather Content • What do you already know about K-selected species? What other information is related to the question? o A habitat that experiences a high level of disturbance is not going to be a very stable environment. Are r- or K-selected species going to do better in an unstable environment? r-selected species are typically shortlived, have lots of offspring, and provide little or no parental care. Kselected species are typically long-lived organisms that produce a very small number of offspring that receive lots of parental investment. Choose Answer • Given what you now know, what information is most likely to produce the correct answer? o r-selected species are species that grow and disperse quickly and reproduce quickly and in large numbers. Therefore, they would be the first species to dominate an area in early succession. Reflect on Process • Did your problem-solving process lead you to the correct answer? If not, where did the process break down or lead you astray? How can you revise your approach to produce a more desirable result? Version 1
38
o You needed to apply your knowledge of reproductive strategies to the situation involving a high disturbance environment. Were you able to apply your knowledge about species that are able to adjust to rapidly changing environmental condition and how that relates to r- versus Kselected species? 7) TRUE
Version 1
39
Clarify Question • What is the key concept addressed by the question? o This question requires an understanding of mimicry and camouflage. • What type of thinking is required? o This is an evaluate question; you need to examine all of the choices and make a judgement if mimicry is a form of camouflage. Gather Content • What do you already know about mimicry and whether it could be involved with camouflaging? What other information is related to the question? o The purpose of camouflage is to hide from a predator. This is achieved by looking like something other than yourself. Typically camouflage is achieved by looking like the surrounding environment. A mimic is a species that looks like a different organism. Choose Answer • Given what you now know, what information is most likely to produce the correct answer? o Camouflage is a method of concealment that allows otherwise visible animals to remain unnoticed by blending with their environment or by resembling something else. In this way, organisms that can mimic their environment can remain camouflaged. Reflect on Process • Did your problem-solving process lead you to the correct answer? If not, where did the process break down or lead you astray? How can you revise your approach to produce a more desirable result? o You need to use you knowledge of mimicry and camouflage to decide Version 1
40
if mimicry is a form of camouflage. Were you able to select the correct answer? 8) [B, D, E] 9) [A, B, C, E] 10) [A, B, D, E] 11) [A, D, E] 12) [B, D, E] 13) [A, C, D, E] 14) [C, D] 15) [A, C, D] 16) [B, C] 17) [A, B, C, E] 18) [A, B, D] 19) [A, B, C, E] 20) [A, B, C, E]
Version 1
41
Clarify Question • What is the key concept addressed by the question? o The key concepts is identifying examples of secondary succession. • What type of thinking is required? o This is an application question. You will need to apply your knowledge of ecological succession to determine which scenarios represent secondary succession. This is a check-all-that-apply question; there is more than one correct answer. Gather Content •What do you already know about secondary succession? What other information is related to the question? o Ecological succession is the gradual shift in a habitat from one that is colonized with pioneer species to a climax community. When the process is undisturbed, it is termed primary succession. Any major disturbance to the habitat stops the process. Once the process restarts, it is termed secondary succession. Choose Answer • Given what you now know, what information is most likely to produce the correct answer? o Farms, landfills, and garden beds have all been drastically altered from a natural state, and the costal grasslands damaged by a hurricane has also been altered. In all of these cases, the habitats were not taken to a condition of primary succession. They are examples of secondary succession. Rocks in a national park are undisturbed with pioneering species beginning the succession process. Reflect on Process Version 1
42
• Did your problem-solving process lead you to the correct answer? If not, where did the process break down or lead you astray? How can you revise your approach to produce a more desirable result? o You need to apply your knowledge of succession to determine which choices presented phases of secondary succession. Were you able to select the correct answer? 21) [A, C, E]
Version 1
43
Clarify Question • What is the key concept addressed by the question? o The question requires knowledge about fundamental niches and the factors that affect the fundamental niche. • What type of thinking is required? o This is an application question. You will need to apply your knowledge of ecological niches to select the correct answers. In this question, you need to select all of the factors that could influence the fundamental niche of a particular species. This is a check-all-that-apply question; there is more than 1 correct answer. Gather Content • What do you already know about fundamental niches and what factors influence the niche? What other information is related to the question? o The niche is a portion of a particular habitat that is suitable for the species being considered. The fundamental niche represents all of the possible habitats that could be occupied by the species in question without taking other species into account. The realized niche is the actual portion of the habitat where the species is found. The realized niche is always smaller because biotic factors (interactions with other living organisms) restrict the size of the realized niche. Choose Answer • Given what you now know, what information is most likely to produce the correct answer? o Salinity, soil type, and temperature are all example of factors that influence the fundamental niche. Parasites, predators and commensals would all be factors that could potentially influence the realized niche Version 1
44
but not the fundamental niche. Reflect on Process • Did your problem-solving process lead you to the correct answer? If not, where did the process break down or lead you astray? How can you revise your approach to produce a more desirable result? o In this question you needed to apply your knowledge of ecological niches to determine which factors would most likely influence the fundamental niche. Did you understand that some factors might affect a realized niche but not the fundamental niche? 22) [A, C, D]
Version 1
45
Clarify Question • What is the key concept addressed by the question? o The question requires an understanding of coevolution and what type of environmental conditions can contribute to coevolution in the flowers and the hummingbirds that visit them. • What type of thinking is required? o This is an application question; you need to apply your knowledge of coevolution to determine which features represent evidence of coevolution between the hummingbirds and the flowers. This is a checkall-that-apply question; there is more than one correct answer. Gather Content • What do you already know about coevolution? What other information is related to the question? o Coevolution occurs when two species evolve in lockstep with each other. Coevolution can lead to an “arms race” type of interaction when it occurs between competitors or predator/prey pairings, but it can also be mutually beneficial if the interaction species are in a mutualistic relationship to begin with. Choose Answer • Given what you now know, what information is most likely to produce the correct answer? o Hummingbird flowers have evolved to attract hummingbirds. They have long tubular flowers that are generally red in color. The flowers, which are generally on stalks, produce a high volume of sugar and nectar since hummingbirds have a high energy requirement. Reflect on Process • Did your problem-solving process lead you to the correct answer? If Version 1
46
not, where did the process break down or lead you astray? How can you revise your approach to produce a more desirable result? o You needed to apply your knowledge of coevolution to the situation of the hummingbirds and the flowers they are feeding on. Were you able to identify the characteristics that were similar in the different species flowers and how those characteristics relate to attracting hummingbirds? 23) [C, E]
Version 1
47
Clarify Question • What is the key concept addressed by the question? o This question requires an understanding of succession and the characteristics of organisms that are better adapted for an envirnment undergoing succession. • What type of thinking is required? o This is an evaluate question; you need to examine all of the choices and make a judgement about the suitability of a characteristic to a pioneer species. This is check-all-that-apply question; there is more than one correct answer. Gather Content • What do you already know about succession? What other information is related to the question? o An established forest habitat would have large mature trees, low levels of light at the forest floor, and very little room for a new species to get started. In a very early succession stage, it would look very different. The habitat would have many smaller plants, high levels of light, and lots of new plant species would be growing and trying to get established. Choose Answer • Given what you now know, what information is most likely to produce the correct answer? o Pioneer plants would have many characteristics consistent with rselected life history strategies. They would be fast-growing and reproduce in great numbers. They would also be able to germinate and grow in direct sunlight since there would be few other plants to produce shade. They would also likely be wind pollinated as there would not be Version 1
48
many animals available for pollination. Reflect on Process • Did your problem-solving process lead you to the correct answer? If not, where did the process break down or lead you astray? How can you revise your approach to produce a more desirable result? o You need to use your knowledge of ecological succession to judge which characteristics would be most beneficial to a pioneer species that is try to establish itself. Did you remember that r-selected species are better able to survive in conditions of succession, like those seen in an establishing forest in the early stages of succession? Knowing that were you able to identify characteristics of r-selected species in a forest succession environment? 24) [C, F]
Version 1
49
Clarify Question • What is the key concept addressed by the question? o This question requires an understanding of how bacterial diversity is maintained in a community and how bacterial phages may play a role in this diversity. • What type of thinking is required? o This is an analyze question; you need to break down the question into its components in order to determine the role of phages in communities of bacteria. This is a check-all-that-apply question; there may be more than one correct answer. Gather Content • What do you already know about maintaining bacterial diversity in a community? What other information is related to the question? o How could an abundance of viruses be beneficial to maintaining a high level of diversity in a bacterial community? When phages infect bacteria, they usually kill the bacteria, which would seem counter to the argument that the phage help maintain bacterial diversity. Choose Answer • Given what you now know, what information is most likely to produce the correct answer? o Phages are viruses that parasitize bacterial cells. In this situation, the parasites keep any one species of bacteria from becoming dominant, which maintains bacterial diversity in the community. Reflect on Process • Did your problem-solving process lead you to the correct answer? If not, where did the process break down or lead you astray? How can you revise your approach to produce a more desirable result? Version 1
50
o You need to analyze the question by breaking it down and trying to figure out how having a high level of viruses in an environment help to maintain the bacterial diversity of a community. Did you remember that without predatory pressure on a population, that population can grow unchecked? In that situation, one or two species of bacteria could overtake others and reduce diversity. The phages act as parasites to give many bacterial species a fighting chance and acts to maintain bacterial diversity. 25) [A, C, D, E]
Version 1
51
Clarify Question • What is the key concept addressed by the question? o This question addresses the phosphorus cycle. • What type of thinking is required? o This question is asking you to take what you already know and apply it to this unfamiliar situation. Gather Content • What do you already know about the phosphorus cycle? o Phosphorus is found in ATP, the energy carrier of the cell. o Phosphorus is found in the phosphate backbone of DNA and RNA. o Phosphorus is found in phospholipids of the cell membrane. o However, phosphorus is not found in amino acids or proteins. Choose Answer • Given what you now know, what information and/or problem solving approach is most likely to produce the correct answer? o Phosphorus is an essential plant nutrient for many reasons. Phosphorus is necessary to the metabolism of a plant because it is part of ATP, which stores and transfers energy within cells. Phosphorus is found in phospholipids that comprise the majority of the cellular membranes and in the bonds that hold nucleotides together in a DNA molecule. Without phosphorus, cell division would be negatively affected because division requires DNA synthesis and membrane formation. Reflect on Process • Did your problem-solving process lead you to the correct answer? If Version 1
52
not, where did the process break down or lead you astray? How can you revise your approach to produce a more desirable result? o The question required you to take what you already know and apply it to this unfamiliar situation. o Did you recognize that phosphorus is important for plants because it is in ATP, DNA, and phospholipids? 26) E 27) A 28) D 29) A 30) B 31) C 32) D The competitive exclusion principle (Section 39.1) predicts that one species would be eliminated from that community. 33) B 34) C 35) C 36) A 37) E 38) C 39) C 40) B 41) A 42) A 43) B 44) A 45) E 46) D 47) C Version 1
53
48) A 49) A 50) B 51) B 52) D 53) A 54) B 55) A 56) B 57) C 58) A 59) D 60) C Wolves generally eat herbivores and are classified as primary carnivores (Section 39.6). 61) A 62) C 63) D 64) A 65) E 66) E 67) A 68) B 69) E 70) D 71) B 72) C 73) B 74) A 75) B Version 1
54
76) B 77) C 78) A 79) A 80) D Clarify Question • What is the key concept addressed by the question? o The key concept is being able to interpret the interaction of two species. • What type of thinking is required? o This is an analysis question. You will need to extract the information from the graph in order to determine the correct answer. Gather Content • What do you already know about the relationship between the two species? What other information is related to the question? o In the situation presented in the question, the Paramecium is the prey and Didinium is the predator. Paramecium is introduced first, and the Didinium is introduced later. Choose Answer • Given what you now know, what information is most likely to produce the correct answer? o The Didinium increased at the expense of the Paramecium (they were eating them). Once the Paramecium population crashed, the Didinium died of starvation. Reflect on Process • Did your problem-solving process lead you to the correct answer? If not, where did the process break down or lead you astray? How can you revise your approach to produce a more desirable result? o You need to extract information from the graph in order to select the correct answer. Were you able to select the correct answer?
81) D
Version 1
55
Clarify Question • What is the key concept addressed by the question? o The key concept is being able to interpret the interaction of two species. • What type of thinking is required? o This is an analysis question. You will need to extract the information from the graph in order to determine the correct answer. Gather Content • What do you already know about relationship between the two species? What other information is related to the question? o In the situation presented in the question, the population of Paramecium initially increases; once the Didinium is introduced, the Paramecium population begins to decline as the Didinium population grows. Once the Paramecium disappears, that Didinium disappears shortly afterwards. Choose Answer • Given what you now know, what information is most likely to produce the correct answer? o Once the Didinium is introduced, the Paramecium population begins to decline. This could suggest competition between the two protists for some resource. Once the paramecia are gone, the population of Didinium starts to decline. The Didinium were eating the paramecia. Once the paramecia were gone, the Didinium starved. Reflect on Process • Did your problem-solving process lead you to the correct answer? If not, where did the process break down or lead you astray? How can you revise your approach to produce a more desirable result? o You need to extract information from the graph in order to select the correct answer. Were you able to select the correct answer?
82) B
Version 1
56
Clarify Question • What is the key concept addressed by the question? o In this question, you are being presented with five situations, and you need to determine which one is a commensalistic relationship. • What type of thinking is required? o This is an application question. You will need to apply your knowledge of symbiotic relationships in order to determine the correct answer. Gather Content • What do you already know about commensalism? What other information is related to the question? o Commensalism relationships are where one organism benefits and the other is not affected in any way. Choose Answer • Given what you now know, what information is most likely to produce the correct answer? o Barnacles hitching a ride on the skin of whales is an example of commensalism. By attaching to the whale skin, the barnacles, which are otherwise sedentary as adults, are able to locate habitats with a better food supply. The whale is not affected by the barnacle. Reflect on Process • Did your problem-solving process lead you to the correct answer? If not, where did the process break down or lead you astray? How can you revise your approach to produce a more desirable result? o You needed to apply your knowledge of symbiotic relationships. Parasitic relationships are often the easiest to identify, and the Version 1
57
commensalism is less clear because it is sometime difficult to determine that the second organism is not affected by the relationship. Were you able to select the correct answer? 83) E
Version 1
58
Clarify Question • What is the key concept addressed by the question? o The question describes a relationship between a golden jackal and a tiger. • What type of thinking is required? o This is an application question. You will need to apply your knowledge of symbiotic relationships in order to determine the correct answer. Gather Content • What do you already know about symbiotic relationships? What other information is related to the question? o Both partners benefit from a mutualism, whereas one partner benefits and the expense of the other in a parasitism. In commensalistic relationships one partner benefits and the other is not affected. Choose Answer • Given what you now know, what information is most likely to produce the correct answer? o In this relationship, the jackal benefits from the tiger but the tiger neither benefits nor is harmed by the relationship. This relationship is an example of commensalism. Reflect on Process • Did your problem-solving process lead you to the correct answer? If not, where did the process break down or lead you astray? How can you revise your approach to produce a more desirable result? o You needed to apply your knowledge of symbiotic relationships to figure out what type of relationships existed between the jackal and the Version 1
59
tiger. Were you able to select the correct answer? 84) C
Version 1
60
Clarify Question • What is the key concept addressed by the question? o The question requires a knowledge of different types of symbiotic relationships and in which type of relationship these animals are engaged. • What type of thinking is required? o This is an application question. You will need to apply your knowledge of symbiotic relationships in order to determine the correct answer. Gather Content • What do you already know about symbiotic relationships how the different behaviors of the animals relate to each type of symbiosis? What other information is related to the question? o Both partners benefit from a mutualistic relationship, whereas one partner benefits and the expense of the other in a parasitic relationship. In commensalistic relationships one partner benefits and the other is not affected. Choose Answer • Given what you now know, what information is most likely to produce the correct answer? o Initially, this was a commensal relationship. The jackal benefited from its relationship with the tiger, but the tiger was unaffected by the jackal. If the jackal assists the tiger in making a kill, both animals benefit. The relationship would then be classified as mutualism. Reflect on Process • Did your problem-solving process lead you to the correct answer? If not, where did the process break down or lead you astray? How can Version 1
61
you revise your approach to produce a more desirable result? o You needed to apply your knowledge of symbiotic relationships to figure out which type of relationships existed between the jackal and the tiger. Were you able to correctly identify the previous relationship as commensalistic because the tiger wasn't benefiting or hurt by the relationship with the jackal? Did you recognize that the jackal was in fact helping the tiger, which makes it a mutualistic relationship? 85) B
Version 1
62
Clarify Question • What is the key concept addressed by the question? o The question requires an understanding of different types of symbiotic relationships and identification of relationships based on a descriptions of the associations between different animals. • What type of thinking is required? o This is an application question; you need to apply your knowledge of symbiotic relationships to situations with cattle egrets and oxpeckers. In this question, you need to match the bird (oxpecker or egret) with the type of symbiotic relationship it has with larger grazers. Gather Content • What do you already know about symbiotic relationships and the qualifications for each? What other information is related to the question? o Commensalisms are relationships where one benefits and the other is not affected in any way. In a mutualistic relationship both partners benefit from the relationship.Parasitic relationships are where one benefits and the other is harmed by the relationship. Choose Answer • Given what you now know, what information is most likely to produce the correct answer? o The egret has a commensalistic relationship; it gets food but the buffalo receives no benefit from the association. The oxpecker has a mutualistic relationship; the bird gets food and the buffalo gets ectoparasites (ex. ticks) removed. Reflect on Process
Version 1
63
• Did your problem-solving process lead you to the correct answer? If not, where did the process break down or lead you astray? How can you revise your approach to produce a more desirable result? o You need to apply your knowledge of symbiosis to the situation. Were you able to recognize if the buffalo benefits in either, both, or neither relationship? In both cases, the birds benefit and so the correct answers depend on being able to determine if the buffalo benefits or not or if it is harmed. 86) A
Version 1
64
Clarify Question • What is the key concept addressed by the question? o This question requires an understanding of environmental pressures and whether the presence of phages is a density-dependent or densityindependent factor the help to regulate the bacterial community. • What type of thinking is required? o This is an evaluate question; you need to evaluate the impact of phages on the diversity of the bacterial community. Gather Content • What do you already know about density-dependent versus densityindependent factors affecting a community? What other information is related to the question? o Phages are viruses that parasitize bacterial cells. In this situation, the parasites keep any one species of bacteria from becoming dominant, which maintains diversity within the community. The effect of the phages on any particular species of bacteria can vary with size of the population of the particular bacteria. At a low population size, the virus will spread slowly and have a small effect on the bacteria. If the population of bacteria is particularly high, the virus (phage) can spread quickly through the population and have a huge effect. Choose Answer • Given what you now know, what information is most likely to produce the correct answer? o Phages are parasites and parasites move between members of a species faster when population numbers are high. This makes them density-dependent regulators. Parasites play an important role in maintaining bacterial community diversity because they prevent any one Version 1
65
species of bacteria from becoming dominant. Reflect on Process • Did your problem-solving process lead you to the correct answer? If not, where did the process break down or lead you astray? How can you revise your approach to produce a more desirable result? o To answer this question you had to ignore certain information in the question to focus on the issue of density-dependent versus densityindependent control of populations. The fact that phages help to maintain bacterial diversity is important, but the question if the phage infection is density-dependent or density-independent relates more to whether the size of the bacterial population affect phage infections. Larger populations of bacteria allow phage infections to spread faster, making phages a density-dependent factor. 87) A
Version 1
66
Clarify Question • What is the key concept addressed by the question? o This question addresses trophic levels. • What type of thinking is required? o This question is asking you to take what you already know and apply it to this unfamiliar situation. Gather Content • What do you already know about trophic levels? o The first trophic level in an ecosystem, called the primary producers, consists of all the autotrophs in the system. These are usually photosynthetic organisms–plants, algae and cyanobacteria. At deep sea vents, however, the primary producers are chemosynthetic microbes. o The other trophic levels consist of the heterotrophs—the consumers. All the heterotrophs that feed directly on the primary producers are placed together in a trophic level called the herbivores. o In turn, the heterotrophs that feed on the herbivores (eating them or being parasitic on them) are collectively termed primary carnivores, and those that feed on the primary carnivores are called secondary carnivores. o An additional consumer level is the detritivore trophic level. Detritivores feed on the remains of already dead organisms. Choose Answer • Given what you now know, what information and/or problem solving approach is most likely to produce the correct answer? o A producer in an ecosystem is nearly always photosynthetic (except for a few chemoautotrophic prokaryotes such as those near deep sea vents). Version 1
67
o The only photosynthetic organism as an option was the pine tree. Any plant is a primary producer. Reflect on Process • Did your problem-solving process lead you to the correct answer? If not, where did the process break down or lead you astray? How can you revise your approach to produce a more desirable result? o The question required you to take what you already know and apply it to this unfamiliar situation. o Did you recognize that producers are nearly always plants, algae, or cyanobacteria? 88) A
Version 1
68
Clarify Question • What is the key concept addressed by the question? o This question addresses productivity. • What type of thinking is required? o This question is asking you to take what you already know and apply it to this unfamiliar situation. Gather Content • What do you already know about productivity? o The productivity of a trophic level is the rate at which the organisms in the trophic level collectively synthesize new organic matter in their tissues. o Primary productivity is the productivity of the primary producers. An important complexity in analyzing the primary producers is that not only do they synthesize new organic matter by photosynthesis, but they also break down some of the organic matter to release energy by means of aerobic cellular respiration. The respiration of the primary producers, in this context, is the rate at which they break down organic compounds. o Gross primary productivity (GPP) is simply the raw rate at which the primary producers synthesize new organic matter; net primary productivity (NPP) is the GPP minus the respiration of the primary producers. The NPP represents the organic matter available for herbivores to use as food. o The productivity of a heterotroph trophic level is termed secondary productivity. For instance, the rate that new organic matter is made by means of individual growth and reproduction in all the herbivores in an ecosystem is the secondary productivity of the herbivore trophic level. Choose Answer Version 1
69
• Given what you now know, what information and/or problem solving approach is most likely to produce the correct answer? o NPP quantifies the net carbon absorption rate by living plants. NPP is the difference between plant photosynthesis, which fixes atmospheric CO2 into sugars and other organic molecules, and respiration, which combusts sugar and releases the carbon back into the atmosphere. Reflect on Process • Did your problem-solving process lead you to the correct answer? If not, where did the process break down or lead you astray? How can you revise your approach to produce a more desirable result? o The question required you to take what you already know and apply it to this unfamiliar situation. o Did you recognize that NPP is net primary productivity–the net absorption of carbon by plants? 89) C
Version 1
70
Clarify Question • What is the key concept addressed by the question? o This question addresses trophic levels. • What type of thinking is required? o This question is asking you to take what you already know and apply it to this unfamiliar situation. Gather Content • What do you already know about trophic levels? o The first trophic level in an ecosystem, called the primary producers, consists of all the autotrophs in the system. These are usually photosynthetic organisms—plants, algae and cyanobacteria. At deep sea vents, however, the primary producers are chemosynthetic microbes. o The other trophic levels consist of the heterotrophs—the consumers. All the heterotrophs that feed directly on the primary producers are placed together in a trophic level called the herbivores. o In turn, the heterotrophs that feed on the herbivores (eating them or being parasitic on them) are collectively termed primary carnivores, and those that feed on the primary carnivores are called secondary carnivores. o An additional consumer level is the detritivore trophic level. Detritivores feed on the remains of already dead organisms. Choose Answer • Given what you now know, what information and/or problem solving approach is most likely to produce the correct answer? o The wolf is not a plant, so it is not a producer. o The wolf does not eat plants, so it is not an herbivore. o The wolf does not eat things that are already dead, so it is not a Version 1
71
detritivore. o The wolf eats mostly herbivories, not carnivores. That makes it a primary carnivore. Reflect on Process • Did your problem-solving process lead you to the correct answer? If not, where did the process break down or lead you astray? How can you revise your approach to produce a more desirable result? o The question required you to take what you already know and apply it to this unfamiliar situation. o Did you recognize that a wolf is an example of a primary carnivore? 90) A
Version 1
72
Clarify Question • What is the key concept addressed by the question? o This question addresses trophic levels. • What type of thinking is required? o This question is asking you to take what you already know and apply it to this unfamiliar situation. Gather Content • What do you already know about trophic levels? o The first trophic level in an ecosystem, called the primary producers, consists of all the autotrophs in the system. These are usually photosynthetic organisms—plants, algae and cyanobacteria. At deep sea vents, however, the primary producers are chemosynthetic microbes. o The other trophic levels consist of the heterotrophs—the consumers. All the heterotrophs that feed directly on the primary producers are placed together in a trophic level called the herbivores. o In turn, the heterotrophs that feed on the herbivores (eating them or being parasitic on them) are collectively termed primary carnivores, and those that feed on the primary carnivores are called secondary carnivores. o An additional consumer level is the detritivore trophic level. Detritivores feed on the remains of already dead organisms. Choose Answer • Given what you now know, what information and/or problem solving approach is most likely to produce the correct answer? o Since the honey possum eats nectar, which is the sweet liquid produced in flowers, it is a plant eater or herbivore. Reflect on Process Version 1
73
• Did your problem-solving process lead you to the correct answer? If not, where did the process break down or lead you astray? How can you revise your approach to produce a more desirable result? o The question required you to take what you already know and apply it to this unfamiliar situation. o Did you recognize that some herbivories don’t eat leaves, but eat other parts of a plant? 91) E
Version 1
74
Clarify Question • What is the key concept addressed by the question? o This question addresses trophic levels. • What type of thinking is required? o This question is asking you to take what you already know and apply it to this unfamiliar situation. Gather Content • What do you already know about trophic levels? o The first trophic level in an ecosystem, called the primary producers, consists of all the autotrophs in the system. These are usually photosynthetic organisms—plants, algae and cyanobacteria. At deep sea vents, however, the primary producers are chemosynthetic microbes. o The other trophic levels consist of the heterotrophs—the consumers. All the heterotrophs that feed directly on the primary producers are placed together in a trophic level called the herbivores. o In turn, the heterotrophs that feed on the herbivores (eating them or being parasitic on them) are collectively termed primary carnivores, and those that feed on the primary carnivores are called secondary carnivores. o An additional consumer level is the detritivore trophic level. Detritivores feed on the remains of already dead organisms. Choose Answer • Given what you now know, what information and/or problem solving approach is most likely to produce the correct answer? o Since the vampire squid feed on bits of dead organic matter, that makes them detritivores. Reflect on Process Version 1
75
• Did your problem-solving process lead you to the correct answer? If not, where did the process break down or lead you astray? How can you revise your approach to produce a more desirable result? o The question required you to take what you already know and apply it to this unfamiliar situation. o Did you recognize that detritivores are organisms that feed on dead organic matter? 92) E
Version 1
76
Clarify Question • What is the key concept addressed by the question? o This question addresses productivity. • What type of thinking is required? o This question is asking you to analyze the information given, using logic, to dissect the problem and determine the answer. Gather Content • What do you already know about productivity? o The productivity of a trophic level is the rate at which the organisms in the trophic level collectively synthesize new organic matter in their tissues. o Primary productivity is the productivity of the primary producers. An important complexity in analyzing the primary producers is that not only do they synthesize new organic matter by photosynthesis, but they also break down some of the organic matter to release energy by means of aerobic cellular respiration. The respiration of the primary producers, in this context, is the rate at which they break down organic compounds. o Gross primary productivity (GPP) is simply the raw rate at which the primary producers synthesize new organic matter; net primary productivity (NPP) is the GPP minus the respiration of the primary producers. The NPP represents the organic matter available for herbivores to use as food. o The productivity of a heterotroph trophic level is termed secondary productivity. For instance, the rate that new organic matter is made by means of individual growth and reproduction in all the herbivores in an ecosystem is the secondary productivity of the herbivore trophic level. Choose Answer Version 1
77
• Given what you now know, what information and/or problem solving approach is most likely to produce the correct answer? o Since production is the amount of carbon fixed by the photosynthetic activity of plants, algae, and cyanobacteria, factors that increase their success increase production. For instance, nutrient availability from fertilizer or from nitrogen-fixing bacteria, or increased sunlight during the summer, will increase production. o On land, rainfall can also promote production. But in the ocean, this factor is likely to have little effect. Reflect on Process • Did your problem-solving process lead you to the correct answer? If not, where did the process break down or lead you astray? How can you revise your approach to produce a more desirable result? o The question required you to analyze the information given, using logic, to dissect the problem and determine the answer. o Did you recognize that many factors can influence the productivity of an ecosystem? 93) D
Version 1
78
Clarify Question • What is the key concept addressed by the question? o This question addresses trophic levels. • What type of thinking is required? o This question is asking you to take what you already know and apply it to this unfamiliar situation. Gather Content • What do you already know about trophic levels? o The first trophic level in an ecosystem, called the primary producers, consists of all the autotrophs in the system. These are usually photosynthetic organisms—plants, algae and cyanobacteria. At deep sea vents, however, the primary producers are chemosynthetic microbes. o The other trophic levels consist of the heterotrophs—the consumers. All the heterotrophs that feed directly on the primary producers are placed together in a trophic level called the herbivores. o In turn, the heterotrophs that feed on the herbivores (eating them or being parasitic on them) are collectively termed primary carnivores, and those that feed on the primary carnivores are called secondary carnivores. o An additional consumer level is the detritivore trophic level. Detritivores feed on the remains of already dead organisms. Choose Answer • Given what you now know, what information and/or problem solving approach is most likely to produce the correct answer? o In this food chain, the blue jay feeds mainly on spiders, the primary carnivore. Therefore the jays are the secondary carnivore. Reflect on Process Version 1
79
• Did your problem-solving process lead you to the correct answer? If not, where did the process break down or lead you astray? How can you revise your approach to produce a more desirable result? o The question required you to take what you already know and apply it to this unfamiliar situation. o Did you recognize that there are often multiple levels of carnivores in a food chain? 94) C
Version 1
80
Clarify Question • What is the key concept addressed by the question? o This question addresses trophic levels. • What type of thinking is required? o This question is asking you to take what you already know and apply it to this unfamiliar situation. Gather Content • What do you already know about trophic levels? o The first trophic level in an ecosystem, called the primary producers, consists of all the autotrophs in the system. These are usually photosynthetic organisms—plants, algae and cyanobacteria. At deep sea vents, however, the primary producers are chemosynthetic microbes. o The other trophic levels consist of the heterotrophs—the consumers. All the heterotrophs that feed directly on the primary producers are placed together in a trophic level called the herbivores. o In turn, the heterotrophs that feed on the herbivores (eating them or being parasitic on them) are collectively termed primary carnivores, and those that feed on the primary carnivores are called secondary carnivores. o An additional consumer level is the detritivore trophic level. Detritivores feed on the remains of already dead organisms. Choose Answer • Given what you now know, what information and/or problem solving approach is most likely to produce the correct answer? o Since the spiders are feeding on crickets, which are herbivores, the spiders are the primary carnivore.
Version 1
81
Reflect on Process • Did your problem-solving process lead you to the correct answer? If not, where did the process break down or lead you astray? How can you revise your approach to produce a more desirable result? o The question required you to take what you already know and apply it to this unfamiliar situation. o Did you recognize that invertebrates that eat meat are also called carnivores? 95) E
Version 1
82
Clarify Question • What is the key concept addressed by the question? o This question addresses energy flow. • What type of thinking is required? o This question is asking you to analyze the information given, using logic, to dissect the problem and determine the answer. Gather Content • What do you already know about energy flow? o Energy flow between trophic levels is inefficient. Whenever organisms use chemical-bond or light energy some of it is converted to heat. Consistent with the Second Law of Thermodynamics, a partial conversion to heat is inevitable. Put another way, animals and plants require chemical-bond energy and light to stay alive, but as they use these forms of energy, they convert them to heat, which they cannot use to stay alive and which they cannot cycle back into the original forms. Fortunately for organisms, the Earth functions as an open system for energy because light arrives every day from the Sun. Choose Answer • Given what you now know, what information and/or problem solving approach is most likely to produce the correct answer? o One day's feeding consumes 1000 calories worth of leaves. o 50% of the calories are lost in its feces, so 500 calories are lost just due to inefficient digestion. o 33% of the calories are used in cellular respiration, so 330 calories are used to keep the caterpillar alive. o 1000 – 500 – 330 = 170 calories o So only 170 calories are available to add to the caterpillar’s biomass Version 1
83
each day. Reflect on Process • Did your problem-solving process lead you to the correct answer? If not, where did the process break down or lead you astray? How can you revise your approach to produce a more desirable result? o The question required you to analyze the information given, using logic, to dissect the problem and determine the answer. o Did you recognize that most of the energy from food is lost or used to support respiration? 96) C
Version 1
84
Clarify Question • What is the key concept addressed by the question? o This question addresses energy flow. • What type of thinking is required? o This question is asking you to analyze the information given, using logic, to dissect the problem and determine the answer. Gather Content • What do you already know about energy flow? o Energy flow between trophic levels is inefficient. Whenever organisms use chemical-bond or light energy some of it is converted to heat. Consistent with the Second Law of Thermodynamics, a partial conversion to heat is inevitable. Put another way, animals and plants require chemical-bond energy and light to stay alive, but as they use these forms of energy, they convert them to heat, which they cannot use to stay alive and which they cannot cycle back into the original forms. Fortunately for organisms, the Earth functions as an open system for energy because light arrives every day from the Sun. Choose Answer • Given what you now know, what information and/or problem solving approach is most likely to produce the correct answer? o The food chain has four trophic levels: primary producer, herbivore, primary carnivore, and secondary carnivore. o The primary producer level (level 1) has 40,000 kcal/m2/yr. o If the transfer efficiency is 10%, then the herbivore level (level 2) will have 10% of 40,000 kcal/m^2/yr, or 4,000 kcal/m2/yr. o The primary carnivore level (level 3) will have 10% of 4,000 kcal/m^2/yr, or 400 kcal/m2/yr. Version 1
85
o The secondary carnivore level (level 4) will have 10% of 400 kcal/m^2/yr, or 40 kcal/m2/yr. Reflect on Process • Did your problem-solving process lead you to the correct answer? If not, where did the process break down or lead you astray? How can you revise your approach to produce a more desirable result? o The question required you to analyze the information given, using logic, to dissect the problem and determine the answer. o Did you recognize that energy is lost between each trophic level of a food chain? 97) D
Version 1
86
Clarify Question • What is the key concept addressed by the question? o This question addresses energy flow. • What type of thinking is required? o This question is asking you to analyze the information given, using logic, to dissect the problem and determine the answer. Gather Content • What do you already know about energy flow? o Energy flow between trophic levels is inefficient. Whenever organisms use chemical-bond or light energy, some of it is converted to heat. Consistent with the Second Law of Thermodynamics, a partial conversion to heat is inevitable. Put another way, animals and plants require chemical-bond energy and light to stay alive, but as they use these forms of energy, they convert them to heat, which they cannot use to stay alive and which they cannot cycle back into the original forms. Fortunately for organisms, the Earth functions as an open system for energy, because light arrives every day from the Sun. Choose Answer • Given what you now know, what information and/or problem solving approach is most likely to produce the correct answer? o The food chain has four trophic levels: primary producer, herbivore, primary carnivore, and secondary carnivore. o We are told that the top level (level 4, secondary carnivores) has 48 kcal/m2/yr. o If the energy transfer rate is 10%, then the primary carnivores level (level 3) will have 10 times the energy of the next level up. The primary carnivore level (level 3) therefore should have 480 kcal/m2/yr. Version 1
87
o The herbivores level (level 2) will then have 10 times that, or 4,800 kcal/m2/yr. Reflect on Process • Did your problem-solving process lead you to the correct answer? If not, where did the process break down or lead you astray? How can you revise your approach to produce a more desirable result? o The question required you to analyze the information given, using logic, to dissect the problem and determine the answer. o Did you recognize that energy is lost between each trophic level of a food chain? 98) C
Version 1
88
Clarify Question • What is the key concept addressed by the question? o This question addresses species richness. • What type of thinking is required? o This question is asking you to weigh and judge evidence, or evaluate, to choose the best of the possible answers. Gather Content • What do you already know about species richness? o Species richness is the number of species present in a community. Species richness is influenced by ecosystem characteristics, including primary productivity, habitat heterogeneity, and climate factors. o Tropical regions have the highest diversity, although the reasons are unclear. Reasonable explanations include the following: o The tropics have existed over long, uninterrupted periods of evolutionary time, whereas temperate regions have been subject to repeated glaciations. The greater age of tropical communities would have allowed complex population interactions to coevolve within them, fostering a greater variety of plants and animals. o The tropics receive more solar radiation than do temperate regions. The argument is that more solar energy, coupled to a year-round growing season, greatly increases the overall photosynthetic activity of tropical plants. o Seasonal variation, though it does exist in the tropics, is generally substantially less than in temperate areas. This reduced seasonality might encourage specialization, with niches subdivided to partition resources and so avoid competition. The expected result would be a larger number of more specialized species in the tropics. o Predation is more intense in the tropics. In theory, more intense Version 1
89
predation could reduce the importance of competition, permitting greater niche overlap and thus promoting greater species richness. o Spatial heterogeneity promotes species richness. Tropical forests, by virtue of their complexity, create a variety of microhabitats and so may foster larger numbers of species. Choose Answer • Given what you now know, what information and/or problem solving approach is most likely to produce the correct answer? o Is seasonal variability less in the tropics? Yes. This may foster species diversity. o Are niches narrower in the tropics? Yes. This may foster species diversity. o Could greater spatial heterogeneity in the tropics create more niches? o Do the tropics have greater plant production, which supports greater overall species richness? Yes. o Are there fewer predators in the tropics, allowing more prey species to flourish? No. In fact, predation is thought to be more intense in the tropics. So this is the worst explanation. Reflect on Process • Did your problem-solving process lead you to the correct answer? If not, where did the process break down or lead you astray? How can you revise your approach to produce a more desirable result? o The question required you to weigh and judge evidence, or evaluate, to choose the best of the possible answers. o Did you recognize that the tropics have high species diversity for several reasons? These include high primary productivity, less extreme seasonal changes, and a large number of specific niches. 99) B Version 1
90
Clarify Question • What is the key concept addressed by the question? o This question addresses trophic cascades. • What type of thinking is required? o This question is asking you to analyze the information given, using logic, to dissect the problem and determine the answer. Gather Content • What do you already know about trophic cascades? o Because of food chain interactions, species in any one trophic level may have effects on more than one trophic level. o Primary carnivoresmay have effects not only on the animals they eat, but also, indirectly, on the plants or algae eaten by their prey. The process by which effects exerted at an upper trophic level flow down to influence two or more lower levels is termed a trophic cascade. The effects themselves are called top-down effects. o Conversely, increases in primary productivity may provide more food not just to herbivores, but also, indirectly, to carnivores. When an effect flows up through a trophic chain, such as from primary producers to higher trophic levels, it is termed a bottom-up effect. Choose Answer • Given what you now know, what information and/or problem solving approach is most likely to produce the correct answer? o After returning, a secondary carnivore to an ecosystem, would you expect primary carnivores to become less abundant? Yes. This is a reasonable prediction because the secondary carnivores would eat the primary carnivores. o After returning, a secondary carnivore to an ecosystem, would you Version 1
91
expect that herbivores would become more abundant.? Yes. This is a reasonable prediction because more secondary carnivores would mean fewer primary carnivores, which would mean more herbivores. o After returning, a secondary carnivore to an ecosystem, would you expect algae to decrease? Yes. This is a reasonable prediction because more secondary carnivores would mean fewer primary carnivores, which would mean more herbivores, which would eat the algae. o After returning, a secondary carnivore to an ecosystem, would you expect primary productivity to increase? No. With more secondary carnivores, fewer primary carnivores, more herbivores eating plants and algae, you would expect productivity to decrease, not increase. Reflect on Process • Did your problem-solving process lead you to the correct answer? If not, where did the process break down or lead you astray? How can you revise your approach to produce a more desirable result? o The question required you to analyze the information given, using logic, to dissect the problem and determine the answer. o Did you recognize that changing the numbers of individuals at a high trophic level can have top-down effects all the way down the food chain? o How might this phenomenon influence conservation efforts? 100) A The garlic mustard is releasing a chemical compound that disrupts the mutualistic relationship between native trees and mycorrhizal fungi. This would interfere with the trees' ability to uptake phosphorus. This is an example of interference competition.
Version 1
92
CHAPTER 40 TRUE/FALSE - Write 'T' if the statement is true and 'F' if the statement is false. 1) If the tilt of Earth's axis relative to its plane of orbit was increased to 30 degrees, the seasons in Boston would become less distinct. ⊚ ⊚
true false
CHECK ALL THE APPLY. Choose all options that best completes the statement or answers the question. 2) Which of the following are characteristics of biomes? Check all that apply. A) A biome is a large geographical area with distinctive plant and animal groups. B) Biomes are only located at certain longitudes and latitudes. C) The climate and geography of a region determines what type of biome can exist in that region. D) Each biome consists of only one type of ecosystem. E) Temperate evergreen forest is one of the major biomes.
3) The distribution of biomes is dependent upon which of the following? Check all that apply. A) Available moisture B) Temperature C) The presence of mountains D) Distance from the oceans and elevation E) A thermocline
4)
Which statements about acid precipitation are accurate? Check all that apply.
Version 1
1
A) Acid rain is intermediate between point-source and diffuse pollution. B) Mercury is a pollutant that often accompanies acid precipitation. C) Acid precipitation is a severe problem in areas with soils having high buffering capacity. D) Acid precipitation can affect aquatic ecosystems. E) Acid precipitation can affect forests, weakening the trees and making them more susceptible to disease.
5)
Which of the following statements about ozone depletion are true? Check all that apply. A) Concentrations of ozone-depleting chemicals continue to rise in the atmosphere. B) Chlorofluorocarbons (CFCs) are the major cause of ozone depletion. C) Free chlorine in the upper atmosphere catalyzes the conversion of O 3 into O 2. D) Weather conditions also influence the size of the ozone hole in the stratosphere. E) CFCs are acted on by conditions in the stratosphere to form diatomic chlorine (Cl 2).
6) Which of following statements about the causes and effects of global warming are accurate? Check all that apply. A) The affects of global warming will be uniform throughout the world. B) In addition to carbon dioxide, other greenhouse gases include methane and nitrous oxide. C) Melting of the Artic permafrost will slow the process of global warming because of new plant growth in those areas. D) The effects of global warming can be measured today. E) Carbon dioxide absorbs short wavelength radiant energy better than long wavelength radiant energy.
7) Which of the following statements about the consequences of global warming are true? Check all that apply.
Version 1
2
A) The rate of global warming may be too great for populations to be able to adapt through natural selection. B) Rising sea levels will be a consequence of global warming. C) Global warming will cause a decrease in the incidence of diseases caused by mosquitoes. D) The frequency ofEl Niño events may increase as a consequence of global warming. E) Agricultural production will increase.
8) Which of the following statements about the effects of deforestation on terrestrial ecosystems are true? Check all that apply. A) At present rates of deforestation, it is predicted that all of the world's tropical rain forests will be degraded or gone within 10 years. B) Today, deforestation is one of the greatest problems facing terrestrial ecosystems. C) Deforestation usually leads to the overaccumulation of nutrients such as nitrates/nitrites in the soil. D) Deforestation can lead to loss of topsoil. E) Deforestation can lead to eutrophication of aquatic ecosystems.
9) Your biology professor takes the class on a field trip to a local lake. You spend an hour studying the lake and writing down all observations. You hypothesize that the lake is oligotrophic. Which observations would support this hypothesis? Check all that apply. A) The shoreline is mostly rock and sand with very little vegetative matter. B) The lake has extensive aquatic plant beds. C) The lake has little planktonic growth. D) The bottom of the lake is easily visible. E) An oxygen probe reveals low oxygen content in the water.
10) Which of the following human activities could contribute to the eutrophication of a lake? Check all that apply.
Version 1
3
A) Dumping piles of leaves near a lake B) Planting trees around the periphery of the lake C) Washing clothes with soap in the lake D) Feeding the ducks in the lake E) Dumping plastic trash into the lake F) Fertilizing lawns near the lake
11)
Which of the following are examples of diffuse pollution? Check all that apply. A) Chemicals released from a factory waste pipe B) Wind-borne debris blowing out to sea C) Pet waste D) Oil leaking from a cargo ship E) Oil that washes off of roads
MULTIPLE CHOICE - Choose the one alternative that best completes the statement or answers the question. 12) If a 2200 kg whale were to die, how much of it would be consumed before it reaches the organisms that live at the deep sea bottom? A) 1200 kg B) 2178 kg C) 420 kg D) 220 kg E) 22 kg
13) You are standing on top of a mountain in Colorado in front of a cannon. You want to fire a cannonball and hit a flag pole that's on the top of another mountain, many miles away but directly North of you. Which direction should you point the cannon before firing?
Version 1
4
A) Northwest B) North C) Northeast D) South E) Southeast F) Southwest
14) Which of the following statements best explains why the waters over continental shelves have more nutrients than the open ocean? A) The waters over continental shelves have more aquatic plants that provide nutrients. B) The water over the continental shelves are shallower so the nutrients remain in place. C) The open ocean has a greater concentration of consumers that deplete the water of nutrients. D) About 99% of seafood harvested by humans comes from the open ocean, thus depleting it of its nutrients. E) Sunlight penetration is greater over continenntal shelves, making it more nutreint rich.
15) Temperate grasslands and temperate deciduous forests are two different biomes that are located in the same latitudes. Which one of the following abiotic conditions is most likely to account for the differences in their community composition? A) Temperature B) Precipitation C) Sunlight D) Soil nutrients E) A rain shadow
16) As the global concentration of carbon dioxide increases, there is great potential for direct impact on the world's
Version 1
5
A) climate. B) biodiversity. C) biogeochemistry. D) ozone layer.
17) CO 2 and other gases, which absorb the longer wavelengths of infrared light and radiated heat from Earth, are responsible for causing the A) ozone layer depletion. B) greenhouse effect. C) decreased biodiversity. D) acid rain.
18) Drifting freely in the upper, well-illuminated waters of the ocean, a diverse biological community exists, primarily consisting of microscopic organisms called A) phytoplankton. B) archaeans. C) detritus. D) benthos. E) bacteria.
19)
Most of the ultraviolet radiation reaching the upper atmosphere is absorbed by A) CFCs. B) ozone. C) nitrogen. D) carbon dioxide. E) water vapor.
20)
Photosynthetic organisms of the open oceans are confined to the upper
Version 1
6
A) two hundred millimeters. B) two hundred centimeters. C) two hundred meters. D) two hundred kilometers. E) two thousand kilometers.
21) The relatively shallow part of the ocean extending from the low tide mark to the edge of the continental shelf is called the A) surface zone. B) abyssal zone. C) neritic zone. D) littoral zone. E) limnetic zone.
22)
Destruction of which biome would result in the greatest loss of biodiversity? A) Desert B) Grassland C) Savanna D) Taiga forest E) Tropical rain forest
23)
Tropical climates are warmer than temperate climates because
Version 1
7
A) temperate climates have more mountains and thus more snow causing the landmasses to warm slower. B) tropical climates have more vegetation and thus can absorb more heat. C) temperate climates are closer to the poles and the polar surfaces cause a reduction in heat gain. D) tropical climates receive the sun's rays at an almost perpendicular angle; since Earth is a sphere, the temperate climates receive the sun's rays at a much steeper incidence thus spreading the radiation over a broader area. E) tropical climates receive the sun's rays at a much steeper incidence thus spreading the radiation over a broader area and causing the tropics to be warmer while the temperate climates receive the sun's rays at an almost perpendicular angle, which causes some areas to be warm and other areas to be cold.
24) Which of the following choices best describes the relationship that exists between moisture and air temperature? A) The moisture-holding capacity of air decreases when it is warm and increases when it is cool. B) The moisture-holding capacity of air remains the same (warm or cool) and is not related to its temperature. C) The moisture-holding capacity of air increases when it is warm and decreases when it is cool. D) The moisture-holding capacity of air decreases as the higher altitude winds of the polar regions descend onto the continents.
25)
Which statement best describes the relationship between elevation and latitude?
A) Elevation and latitude are related because they are both measurements of distance. B) Elevation and latitude are related because as one increases the other increases. C) Elevation and latitude are related because they have similar effects on temperature. As the elevation increases, the temperature of the air increases, which is likewise with latitude. D) Elevation and latitude are related because they have similar effects on temperature. As the elevation increases, the temperature of the air decreases, which is likewise with latitude.
Version 1
8
26) If you plot mean annual precipitation against mean annual temperature and look at the distribution of major biomes relative to these two variables, which biome would be in the central region of the graph? A) Savanna B) Hot desert C) Tropical rain forest D) Temperate deciduous forest E) Taiga
27) Which one of the following marine ecosystems would be expected to have the lowest species diversity and primary production per unit area? A) Oligotrophic oceans B) Upwelling regions of ocean C) Coral reefs D) Estuaries E) Intertidal zone
28) A mountain range along the coast of a continent is located perpendicular to the prevailing sea breeze. Which one of the following describes an area where the rain shadow is located? A) On the leeward side of the mountain where cool, dry air is descending B) On the leeward side of the mountain where warm, moist air is descending C) On the windward side of the mountain where warm, moist air is rising D) On the windward side of the mountain where cool, dry air is rising E) On the leeward side, far inland from the mountain
29)
Which of the following adversely affects the ozone layer?
Version 1
9
A) Chlorofluorocarbons B) Greenhouse gases C) Oil D) Coal and natural gas E) Plastics
30)
Which group of organisms is responsible for primary production in a freshwater lake? A) Phytoplankton B) Aquatic grasses C) Submerged plants D) Photosynthetic bacteria E) Moss
31) A decrease in the concentration of which of the following components would have the greatest impact upon a freshwater community? A) Nitrogen B) Calcium C) Oxygen D) Carbon E) Sodium
32)
What is the primary source of energy for organisms that live in the deep sea? A) Sunlight B) Chemical energy C) Cellular respiration D) Inorganic metabolism
33)
The changes in the average temperature of the Earth's surface is known as
Version 1
10
A) climate change B) greenhouse gases C) global warming D) ozone depletion
34)
What is predicted to be the increase in the Earth's temperature by the end of the century? A) 1.1–6.4 degrees Celsius B) 2.0–5.5 degrees Celsius C) 0.6–3.5 degrees Celsius D) 4.4–7.2 degrees Celsius
35) Biomagnification is a significant problem in aquatic communities. Levels of chemicals magnify (increase or accumulate) at each new link in a food chain. Use the following information to estimate the level of chemical X in a large-mouth bass. One alga can accumulate 1 unit of chemical X. Each copepod eats 15 algae. A minnow consumes 10 copepods, each largemouth bass consumes 20 minnows. A) 15 units B) 20 units C) 30 units D) 150 units E) 300 units F) 3,000 units G) 20,000 units
Version 1
11
36) Biomes are broad regional areas with defined flora and fauna. Which one of the following choices correctly identifies the biomes shown plotted in the graph of precipitation versus temperature?
A) Tropical rain forest = 1; Desert = 2; Tundra = 3; Grassland = 4 B) Tropical rain forest = 1; Desert = 3; Tundra = 4; Grassland = 2 C) Tropical rain forest = 3; Desert = 4; Tundra = 1; Grassland = 2 D) Tropical rain forest = 3; Desert = 1; Tundra = 2; Grassland = 3
37) Researchers in Alaska were interested in increasing the sockeye salmon (Oncorhynchus nerka) population in three coastal lakes. To accomplish this, the ponds were treated with fertilizers. As a result, zooplankton biomass increased between 40% and 700% in the three lakes. The substantial growth in zooplankton would suggest that prior to these experiments, these lakes were __________. A) eutrophic B) stratified C) subject to low light levels D) oligotrophic
38) While visiting the South Island of New Zealand, you notice that conditions vary sharply across the island. The West Coast is very wet and receives between 600 and 1600 mm of rainfall per year, while the East Coast only receives about 640 mm of rain per year and is very dry. What best explains this regional difference in precipitation? Version 1
12
A) Proximity to Antarctica B) The absence of inland lakes C) The presence of a large river along the West Coast D) The presence of mountains along the West Coast E) Latitudinal differences between the east and west coasts
39) Rhizosphaera needle cast is a fungal disease that affects spruce trees. Which of the following biomes would be most severely affected by an outbreak of this disease? A) Tundra B) Temperate deciduous forest C) Prairie D) Taiga E) Tropical rain forest
40)
Biological magnification of persistent toxins would be greatest in a A) deer. B) fox. C) polar bear. D) mushroom. E) oak tree.
41)
Which two countries are both likely to have prevailing winds blowing from west to east? A) the United States and Argentina B) the United States and Panama C) Canada and Brazil D) Panama and Brazil
Version 1
13
42) Oxygen levels in the photic zone fluctuate over the course of a 24-hour period. If you were measuring dissolved oxygen over the course of the day, when would you expect to see the lowest levels of oxygen and why? A) Mid-morning because fish are most active and their respiration rates are highest. B) Early afternoon when the surface water warms up and sinks to the bottom. C) Mid-afternoon because the activity of the Calvin cycle (the dark reactions) of photosynthesis are at their peak. D) Early evening when animals come to feed and disturb the surface waters. E) During the night because the light reactions of photosynthesis are inactive.
SECTION BREAK. Answer all the part questions. 43) For the following questions, choose the letter from the list below that best matches the description (please note the answer should be a single upper case letter): A. Desert B. Savanna C. Taiga D. Temperate grassland E. Tropical rain forest
43.1) Very high solar energy input; temperature and moisture not limiting; high biomass, high total productivity; many species.
43.2) Southern hemisphere; grassland with scattered trees; seasonally dry; nutrient poor soils; large grazing mammals.
43.3) Restricted by water availability; 20 degrees to 30 degrees latitude; hot sunny days, cold nights; annuals; shrubs with long roots; succulents.
43.4) Central region of most continents; long, cold winters; moderate amount of rainfall; grazing mammals and burrowing rodents; fires.
Version 1
14
43.5) Northern part of North America and Eurasia, large percentage of Earth's landmass; brief summer; marshes, lakes, and ponds.
Version 1
15
Answer Key Test name: Chap 40_3e 1) FALSE
Version 1
16
Clarify Question • What is the key concept addressed by the question? o This question addresses solar radiation. • What type of thinking is required? o This question is asking you to analyze the information given, using logic, to dissect the problem and determine the answer. Gather Content • What do you already know about solar radiation? o A major reason for differences in solar radiation from place to place is the fact that Earth is a sphere, or nearly so. The tropics are particularly warm because the Sun’s rays arrive almost perpendicular to the surface of the Earth in regions near the equator. Closer to the poles, the angle at which the Sun’s rays strike, called the angle of incidence, spreads the solar energy out over more of the Earth’s surface, providing less energy per unit of surface area. o The Earth’s annual orbit around the Sun is also important. The axis of rotation of the Earth is not perpendicular to the plane in which the earth orbits the Sun. Because the axis is tilted by approximately 23.5°, a progression of seasons occurs on all parts of the Earth, especially at latitudes far from the equator. The northern hemisphere tilts toward the Sun during some months but away during others, giving rise to summer and winter; the farther away from the equator, the greater the difference between summer and winter. Choose Answer • Given what you now know, what information and/or problem solving approach is most likely to produce the correct answer? o If the tilt of the Earth increased, it would place Boston (and the Version 1
17
Northern Hemisphere) further from the sun in the Winter and closer to the sun in Summer. This would result in more extreme seasons, not less. Reflect on Process • Did your problem-solving process lead you to the correct answer? If not, where did the process break down or lead you astray? How can you revise your approach to produce a more desirable result? o The question required you to analyze the information given, using logic, to dissect the problem and determine the answer. o Did you recognize that the seasons are caused by the tilt of the Earth’s axis? 2) [A, C, E] 3) [A, B, C, D] 4) [A, B, D, E] 5) [B, C, D, E] 6) [B, D] 7) [A, B, D] 8) [B, D, E] 9) [A, C, D]
Version 1
18
Clarify Question • What is the key concept addressed by the question? o This question addresses eutrophic versus. oligotrophic lakes. • What type of thinking is required? o This question is asking youto weigh and judge evidence, or evaluate, to choose the best of the possible answers. Gather Content • What do you already know about eutrophic vs. oligotrophic lakes? o Lakes and rivers that are low in algal nutrients (such as nitrate or phosphate) and low in the amount of algal material per unit of volume are termed oligotrophic. Such waters are often crystal clear. Oligotrophic lakes and ponds tend to be high in dissolved oxygen at all depths all year because they also have a low rate of oxygen use. o Eutrophic bodies of water are high in algal nutrients and often populated densely with algae. They are more likely to be low in dissolved oxygen, especially in summer. In a eutrophic body of water, decay microbes often place high demands on the oxygen available because when thick populations of algae die, large amounts of organic matter are made available for decomposition. o Human activities have often transformed oligotrophic lakes into eutrophic ones. For example, when people overfertilize their lawns or fields, nitrate and phosphate from the fertilizers wash off into local water systems. Lakes that receive these nutrients become more eutrophic. Choose Answer • Given what you now know, what information and/or problem solving approach is most likely to produce the correct answer? o An oligotrophic lake has low nutrient levels, and consequently little Version 1
19
plant growth and clear water, with high oxygen content. o So these three observations fit an oligotrophic lake: o The shoreline is mostly rock and sand with very little vegetative matter. o The lake has little planktonic growth. o The bottom of the lake is easily visible. Reflect on Process • Did your problem-solving process lead you to the correct answer? If not, where did the process break down or lead you astray? How can you revise your approach to produce a more desirable result? o The question required you to weigh and judge evidence, or evaluate, to choose the best of the possible answers. o Did you recognize that clear water and little plant growth is the sign of an oligotrophic lake? 10) [A, C, D, F]
Version 1
20
Clarify Question • What is the key concept addressed by the question? o This question addresses eutrophication. • What type of thinking is required? o This question is asking you to analyze the information given, using logic, to dissect the problem and determine the answer. Gather Content • What do you already know about eutrophication? o Eutrophic bodies of water are high in algal nutrients and often populated densely with algae. They are more likely to be low in dissolved oxygen, especially in summer. In a eutrophic body of water, decay microbes often place high demands on the oxygen available because when thick populations of algae die, large amounts of organic matter are made available for decomposition. Moreover, light does not penetrate eutrophic waters well because of all the organic matter in the water; photosynthetic oxygen addition is therefore limited to just a relatively thin layer of water at the top. o Human activities have often transformed oligotrophic lakes into eutrophic ones. For example, when people overfertilize their lawns or fields, nitrate and phosphate from the fertilizers wash off into local water systems. Lakes that receive these nutrients become more eutrophic. When excessive nitrates and phosphates enter rivers and lakes, the character of the bodies of water is changed for the worse; the concentration of dissolved oxygen declines, and fish species such as carp take the place of more desirable species. The problem is exacerbated when rivers empty into the ocean. The eutrophication caused by the accumulation of chemicals can lead to enormous areas of water with no oxygen, causing massive die-offs of fish and other animals. Version 1
21
Choose Answer • Given what you now know, what information and/or problem solving approach is most likely to produce the correct answer? o Any activity that increases nutrient input into the lake causes eutrophication. Planting trees would likely slow runoff into the lake reducing nutrient input. Plastic would not affect nutrient levels. o Dumping piles of leaves near a lake would add the nutrients that were in the leaves. o Washing clothes with soap in the lake would add phosphorus because detergents have phosphates. o Feeding the ducks in the lake would add nutrients from the food. o Fertilizing lawns near the lake would add nutrients from the fertilizer (usually N, P, and K). Reflect on Process • Did your problem-solving process lead you to the correct answer? If not, where did the process break down or lead you astray? How can you revise your approach to produce a more desirable result? o The question required you to analyze the information given, using logic, to dissect the problem and determine the answer. o Did you recognize that anything that increase nutrient levels will contribute to eutrophication? 11) [B, C, E]
Version 1
22
Clarify Question • What is the key concept addressed by the question? o This question addresses pollution. • What type of thinking is required? o This question is asking you to take what you already know and apply it to this unfamiliar situation. Gather Content • What do you already know about pollution? o Point-source pollution comes from an identifiable location—such as easily identified factories or other facilities that add pollutants at defined locations, such as an outfall pipe. Examples include sewage-treatment plants, which discharge treated effluents at specific spots on rivers. o Diffuse pollution is harder to identify and target for remediation. An example is eutrophication caused by excessive runoff of nitrates and phosphates from lawn and agricultural field fertilization. o Solutions to diffuse pollution often depend on public education and political action. Choose Answer • Given what you now know, what information and/or problem solving approach is most likely to produce the correct answer? o Diffuse pollution does not have a single clear source. Instead, little bits of pollution are contributed from multiple people and areas. o Examples of diffuse pollution are pet waste, oil runoff from roads, and bits of trash that blow out to sea. Reflect on Process
Version 1
23
• Did your problem-solving process lead you to the correct answer? If not, where did the process break down or lead you astray? How can you revise your approach to produce a more desirable result? o The question required you to take what you already know and apply it to this unfamiliar situation. o Did you recognize that diffuse pollution is caused by numerous small sources of contamination? 12) B
Version 1
24
Clarify Question • What is the key concept addressed by the question? o The question is asking about predation and decomposition of a baleen whale carcass in the deep sea. • What type of thinking is required? o You are being asked to take what you already know about food and energy transfer in the deep sea and apply it to a new situation. Gather Content • What do you already know about the source of food for bottomdwelling, deep sea organisms? What other information is related to the question? o Nearly all food originates from photosynthesis that takes place in the photic zone. o Carcasses and waste products drift from the surface of the deep sea to the bottom over a period of weeks or months. o During this time, approximately 99% of the food is consumed or decomposes. Therefore, about 1% reaches the bottom of the ocean. Choose Answer • Given what you now know, what information is most likely to produce the correct answer? o You know that about 1% of the whale carcass will be left after a month, so you need to find 1% of its initial biomass. Reflect on Process • Did your problem-solving process lead you to the correct answer? If not, where did the process break down or lead you astray? How can Version 1
25
you revise your approach to produce a more desirable result? o Answering this question correctly depended upon your ability to apply your knowledge of energy transfer in the deep sea to a new situation. o If you answered incorrectly, did you remember that most of the whale carcass would be consumed or decomposed after a month? o Were you able to correctly find 1% of its initial biomass? 13) A
Version 1
26
Clarify Question • What is the key concept addressed by the question? o The question is asking about the trajectory of a cannonball shot from a mountain in Colorado. • What type of thinking is required? o You are being asked to analyze the situation and decide which direction to aim the cannon in order to hit a target. Gather Content • What do you already know about the source of food for bottomdwelling, deep sea organisms? What other information is related to the question? o If the Earth were not rotating, you would want to point your cannon due north to hit the flagpole. o However, the Earth’s rotation means that the equator travels much farther than the poles during each rotation. This leads to a deflection in the trajectories of objects, termed the Coriolis effect. o The Earth is a rotating sphere, which means that the direction of its rotation, and the deflection from the Coriolis effect appears opposite when viewed from the two poles. o In the northern hemisphere, wind, water, and objects always curve to the right of their direction of motion; in the southern hemisphere, they always curve to the left. o There is no Coriolis force exactly at the equator. Choose Answer • Given what you now know, what information is most likely to produce the correct answer? o You know your location is Colorado; what hemisphere you are in? Version 1
27
o You know that that in the northern hemisphere, objects deflect to the right. How could you compensate for this deflection in order to hit the flagpole? o What direction is to the left when you are facing north? Reflect on Process • Did your problem-solving process lead you to the correct answer? If not, where did the process break down or lead you astray? How can you revise your approach to produce a more desirable result? o Answering this question correctly depended upon your ability to analyze the situation and decide how to hit the target while accounting for the Coriolis effect. o If you answered incorrectly, did you remember that Colorado is in northern hemisphere, where objects are always deflected to the right? o Did you figure out that you would need to compensate for this by pointing the cannon to the left of north, or northwest? 14) B 15) B 16) A 17) B 18) A 19) B 20) C 21) C 22) E 23) D 24) C 25) D 26) D 27) A
Version 1
28
28) A 29) A 30) A 31) C 32) A 33) C Climate change is the bigger picture of what is happening. Climate change is more than just changes in the average temperature of the Earth. 34) A 35) F
Version 1
29
Clarify Question • What is the key concept addressed by the question? o This question addresses biomagnification. • What type of thinking is required? o This question is asking you to analyze the information given, using logic, to dissect the problem and determine the answer. Gather Content • What do you already know about biomagnification? o Environmental pollutants can reach high levels in some organisms due to biomagnification. o For instance, DDT is an insecticide that was sprayed widely in the decades following World War II, often on wetlands to control mosquitoes. During the years of heavy DDT use, populations of ospreys, bald eagles, and brown pelicans—all birds that catch large fish— plummeted. Ultimately, the use of DDT was connected with the demise of these birds. o Scientists established that DDT and its metabolic products became more and more concentrated in the tissues of animals as the compounds were passed along food chains. Animals at the bottom of food chains accumulated relatively low concentrations in their fatty tissues. But the primary carnivores that preyed on them accumulated higher concentrations from eating great numbers, and the secondary carnivores accumulated higher concentrations yet. Top-level carnivores, such as the birds that eat large fish, were dramatically affected by the DDT. In these birds, scientists found that metabolic products of DDT disrupted the formation of eggshells. The birds laid eggs with such thin shells that they often cracked before the young could hatch. o Decades after DDT was banned, populations of ospreys, eagles, and Version 1
30
pelicans are rebounding dramatically. Choose Answer • Given what you now know, what information and/or problem solving approach is most likely to produce the correct answer? o Calculating the amount of the chemical requires multiplication. o One alga can accumulate 1 unit of chemical X. Each copepod eats 15 algae. So each copepod accumulates 15 units. o A minnow consumes 10 copepods, so each minnow accumulates 10 x 15 = 150 units of the chemical. o Each large-mouth bass consumes 20 minnows. So each bass accumulates 20 x 150 = 3000 units of the chemical. Reflect on Process • Did your problem-solving process lead you to the correct answer? If not, where did the process break down or lead you astray? How can you revise your approach to produce a more desirable result? o The question required you to analyze the information given, using logic, to dissect the problem and determine the answer. o Did you recognize that bio magnification causes chemicals to reach high levels in top predators, because they accumulate all the chemical from the food chain below? 36) A
Version 1
31
Clarify Question • What is the key concept addressed by the question? o This question addresses biomes. • What type of thinking is required? o This question is asking you to take what you already know and apply it to this unfamiliar situation. Gather Content • What do you already know about biomes? o Temperature and moisture often determine biomes. Average annual temperature and rainfall, as well as the range of seasonal variation, determine different biomes. Eight major types of biomes are recognized: o Tropical rainforests are highly productive equatorial systems. o Savannas are tropical grasslands with seasonal rainfall. o Deserts are regions with little rainfall. o Temperate grasslands have rich soils. o Temperate deciduous forests are adapted to seasonal change. o Temperate evergreen forests are coastal. o Taiga is the northern forest where winters are harsh. o Tundra is a largely frozen treeless area with a short growing season. Choose Answer • Given what you now know, what information and/or problem solving approach is most likely to produce the correct answer? o The graph shows temperature on the vertical axis and precipitation (rainfall) on the horizontal axis. o So the biome graphed at the top right (1) would have high temperature and high precipitation. Which biome is hot and wet? The tropical rainforest. o The biome graphed at the top left (2) would have high temperature and low precipitation. Which biome is hot and dry? The desert. o The biome graphed at the bottom left (3) would have low temperature and low precipitation. Which biome is cold and dry? The tundra. o The biome graphed at the bottom right (4) would have moderate temperature and moderate precipitation. Which biome is warm with moderate rainfall? The grassland. o So the correct order is: 1. Tropical rain forest; 2. Desert; 3. Tundra; 4. Grassland Reflect on Process
Version 1
32
• Did your problem-solving process lead you to the correct answer? If not, where did the process break down or lead you astray? How can you revise your approach to produce a more desirable result? o The question required you to take what you already know and apply it to this unfamiliar situation. o Did you recognize that each biome has a unique combination of temperature and rainfall?
37) D
Version 1
33
Clarify Question • What is the key concept addressed by the question? o This question addresses eutrophic versus oligotrophic lakes. • What type of thinking is required? o This question is asking you to take what you already know and apply it to this unfamiliar situation. Gather Content • What do you already know about eutrophic vs. oligotrophic lakes? o Lakes and rivers that are low in algal nutrients (such as nitrate or phosphate) and low in the amount of algal material per unit of volume are termed oligotrophic. Such waters are often crystal clear. Oligotrophic lakes and ponds tend to be high in dissolved oxygen at all depths all year because they also have a low rate of oxygen use. o Eutrophic bodies of water are high in algal nutrients and often populated densely with algae. They are more likely to be low in dissolved oxygen, especially in summer. In a eutrophic body of water, decay microbes often place high demands on the oxygen available because when thick populations of algae die, large amounts of organic matter are made available for decomposition. o Human activities have often transformed oligotrophic lakes into eutrophic ones. For example, when people overfertilize their lawns or fields, nitrate and phosphate from the fertilizers wash off into local water systems. Lakes that receive these nutrients become more eutrophic. Choose Answer • Given what you now know, what information and/or problem solving approach is most likely to produce the correct answer? o Oligotrophic bodies of water are naturally low in nutrients. As a Version 1
34
result, the growth of algae is limited. Limited algal growth keeps zooplankton levels low. The addition of the fertilizer created an opportunity for the algae populations to grow rapidly and this in turn supported more zooplankton. Once the zooplankton populations increased, more salmon could be supported in the lakes. Reflect on Process • Did your problem-solving process lead you to the correct answer? If not, where did the process break down or lead you astray? How can you revise your approach to produce a more desirable result? o The question required you to take what you already know and apply it to this unfamiliar situation. o Did you recognize that some lakes have low levels of algae, and thus zooplankton, due to low nutrient levels? 38) D
Version 1
35
Clarify Question • What is the key concept addressed by the question? o This question addresses precipitation. • What type of thinking is required? o This question is asking you to take what you already know and apply it to this unfamiliar situation. Gather Content • What do you already know about precipitation? o Regional differences in precipitation can occur because of rain shadows. o Deserts on land sometimes occur because mountain ranges intercept moisture-laden winds from the sea. When air flowing landward from the oceans encounters a mountain range, the air rises, and its moistureholding capacity decreases because it becomes cooler at higher altitude, causing precipitation to fall on the mountain slopes facing the sea. o As the air—stripped of much of its moisture—then descends on the other side of the mountain range, it remains dry even as it is warmed, and as it is warmed, its moisture-holding capacity increases, meaning it can readily take up moisture from soils and plants. o One consequence is that the two slopes of a mountain range often differ dramatically in how moist they are, and a desert may develop on the dry side.The mountains are said to produce a rain shadow. Choose Answer • Given what you now know, what information and/or problem solving approach is most likely to produce the correct answer? o In New Zealand, the Southern Alps along the West Coast intercept most of the moisture coming off the Tasman Sea, and as a result, the Version 1
36
West Coast receives a lot of rain while the East Coast is extremely dry. This is like the rain shadow effect seen in the Sierra Nevada of California. Reflect on Process • Did your problem-solving process lead you to the correct answer? If not, where did the process break down or lead you astray? How can you revise your approach to produce a more desirable result? o The question required you to take what you already know and apply it to this unfamiliar situation. o Did you recognize that deserts often form on the continental side of mountain ranges due to the rain shadow effect? 39) D
Version 1
37
Clarify Question • What is the key concept addressed by the question? o This question addresses biomes. • What type of thinking is required? o This question is asking you to take what you already know and apply it to this unfamiliar situation. Gather Content • What do you already know about biomes? o Temperature and moisture often determine biomes.Average annual temperature and rainfall, as well as the range of seasonal variation, determine different biomes. Eight major types of biomes are recognized: o Tropical rainforests are highly productive equatorial systems. o Savannas are tropical grasslands with seasonal rainfall. o Deserts are regions with little rainfall. o Temperate grasslands have rich soils. o Temperate deciduous forests are adapted to seasonal change. o Temperate evergreen forests are coastal. o Taiga is the northern forest where winters are harsh. o Tundra is a largely frozen treeless area with a short growing season. Choose Answer • Given what you now know, what information and/or problem solving approach is most likely to produce the correct answer? o Would the tundra have a large number of spruce trees? No, the tundra is too cold for trees. o Would the prairie have a large number of spruce trees? No, the prairie is dominated by grass. o Would the tropical rain forest have a large number of spruce trees? Version 1
38
No, spruce are poorly adapted to the warm, humid rain forest. o Would the temperate deciduous forest have a large number of spruce trees? It may have some, but not as many as colder areas. o Would the taiga have a large number of spruce trees? Yes! Spruce trees and other conifers are well-adapted to the cold. Reflect on Process • Did your problem-solving process lead you to the correct answer? If not, where did the process break down or lead you astray? How can you revise your approach to produce a more desirable result? o The question required you to take what you already know and apply it to this unfamiliar situation. o Did you recognize that spruce trees are well-adapted to the cold weather of the taiga? 40) C
Version 1
39
Clarify Question • What is the key concept addressed by the question? o This question addresses biomagnification. • What type of thinking is required? o This question is asking you to take what you already know and apply it to this unfamiliar situation. Gather Content • What do you already know about biomagnification? o Environmental pollutants can reach high levels in some organisms due to biomagnification. o For instance, DDT is an insecticide that was sprayed widely in the decades following World War II, often on wetlands to control mosquitoes. During the years of heavy DDT use, populations of ospreys, bald eagles, and brown pelicans—all birds that catch large fish— plummeted. Ultimately, the use of DDT was connected with the demise of these birds. o Scientists established that DDT and its metabolic products became more and more concentrated in the tissues of animals as the compounds were passed along food chains. Animals at the bottom of food chains accumulated relatively low concentrations in their fatty tissues. But the primary carnivores that preyed on them accumulated higher concentrations from eating great numbers, and the secondary carnivores accumulated higher concentrations yet. Top-level carnivores, such as the birds that eat large fish, were dramatically affected by the DDT. In these birds, scientists found that metabolic products of DDT disrupted the formation of eggshells. The birds laid eggs with such thin shells that they often cracked before the young could hatch. o Decades after DDT was banned, populations of ospreys, eagles, and Version 1
40
pelicans are rebounding dramatically. Choose Answer • Given what you now know, what information and/or problem solving approach is most likely to produce the correct answer? o Toxic chemicals like DDT accumulate in the largest quantities in higher level carnivores like a polar bear. o A fox is also a predator, but a fox typically eats herbivores, whereas a polar bear eats other predators (seals), so it is higher up the food chain. Reflect on Process • Did your problem-solving process lead you to the correct answer? If not, where did the process break down or lead you astray? How can you revise your approach to produce a more desirable result? o The question required you to take what you already know and apply it to this unfamiliar situation. o Did you recognize that top predators are most vulnerable to the effects of biomagnification? 41) A
Version 1
41
Clarify Question • What is the key concept addressed by the question? o This question addresses the Coriolis effect and winds. • What type of thinking is required? o This question is asking you to analyze the information given, using logic, to dissect the problem and determine the answer. Gather Content • What do you already know about the Coriolis effect and winds? o If Earth did not rotate on its axis, global air movements would follow simple patterns. But the Earth rotates under the winds, so the winds move in curved paths across the surface, rather than straight paths. The curvature of the paths of the winds due to Earth’s rotation is termed the Coriolis effect. o If you were standing on the North Pole, the Earth would appear to be rotating counterclockwise on its axis, but if you were at the South Pole, the Earth would appear to be rotating clockwise. This is why the direction of the Coriolis effect is opposite in the two hemispheres. In the northern hemisphere, winds always curve to the right of their direction of motion; in the southern hemisphere, they always curve to the left. o The reason for these wind patterns is that the circumference of the Earth changes with latitude. It is zero at the poles and 38,000 km at the equator. Thus, land surface speed changes from about 0 to 1500 km per hour going from the poles to the equator. Air descending at 30° north latitude may be going roughly the same speed as the land surface below it. As it moves toward the equator, however, it is moving more slowly than the surface below it, so it is deflected to its right in the northern hemisphere and to its left in the southern hemisphere. In other words, in both the northern and southern hemispheres, the winds blow westward Version 1
42
as well as toward the equator. The result is that winds on both sides of the equator—called the Trade Winds—blow out of the east and toward the west. o Conversely, air masses moving north from 30° are moving more rapidly than underlying land surfaces and thus are deflected again to their right, which in this case is eastward. Similarly, in the southern hemisphere, air masses between 30° and 60° are deflected eastward, to the left. In both hemispheres, therefore, winds between 30° and 60° blow out of the west and toward the east; these winds are called Westerlies. Choose Answer • Given what you now know, what information and/or problem solving approach is most likely to produce the correct answer? o In North America, the Westerly winds blow from west to east. On both sides of the equator, the Trade Winds blow from east to west. Further south in the southern hemisphere, though, countries like Argentina have winds from west to east. So the pair of countries with west-to-east winds is the United States and Argentina. Reflect on Process • Did your problem-solving process lead you to the correct answer? If not, where did the process break down or lead you astray? How can you revise your approach to produce a more desirable result? o The question required you to analyze the information given, using logic, to dissect the problem and determine the answer. o Did you recognize that in the United States the prevailing winds are the Westerlies that blow from west to east? 42) E
Version 1
43
Clarify Question • What is the key concept addressed by the question? o This question addresses oxygen levels. • What type of thinking is required? o This question is asking you to analyze the information given, using logic, to dissect the problem and determine the answer. Gather Content • What do you already know about oxygen levels? o The light reactions of photosynthesis create oxygen, but only during the daytime when light is available, and only in the photic zone. o Water absorbs light passing through it, and the intensity of sunlight available for photosynthesis decreases sharply with increasing depth. In deep lakes, only water relatively near the surface receives enough light for phytoplankton to exhibit a positive net primary productivity. Those waters are described as the photic zone. o The supply of dissolved oxygen to the deep waters of a lake can be a problem because oxygen enters aquatic systems near their surface, where gas exchange occurs with the air and photosynthesis occurs in aquatic organisms. In the still waters of a lake, mixing between the surface and deeper layers may not occur except occasionally as a result of thermal stratification; consequently, oxygen only rarely enters the deep waters from the surface waters. Choose Answer • Given what you now know, what information and/or problem solving approach is most likely to produce the correct answer? o Oxygen levels would be lowest at night when the light reactions of photosynthesis are quiet. The light reactions, not the Calvin cycle, Version 1
44
produce oxygen. Reflect on Process • Did your problem-solving process lead you to the correct answer? If not, where did the process break down or lead you astray? How can you revise your approach to produce a more desirable result? o The question required you to analyze the information given, using logic, to dissect the problem and determine the answer. o Did you recognize that photosynthesis produces oxygen only while the sun is up? 43) Section Break 43.1) E 43.2) B 43.3) A 43.4) D 43.5) C
Version 1
45